Está en la página 1de 746

FISICA 1

Autor: Hugo Medina Guzmán


Profesor de la Pontificia Universidad Católica del Perú
Agosto 2009
PRESENTACIÓN

Me agradó saber que Hugo Medina Guzmán estaba por publicar un texto sobre Física. Había
dos razones suficientes para este sentimiento. Por un lado, tenía curiosidad de saber lo que
podría aportar un texto más de Física sobre los otros ya disponibles. Por otro lado, conozco de
la larga carrera de Hugo Medina como cultor de la enseñanza de [a Física, y tenía curiosidad
de ver cómo este compromiso como docente y experiencia se manifestarían en su texto. Tuve
la suerte de conocer al Ing. José Castro Mendívil en su taller, donde desplegó una destacada
labor en el diseño y construcción de equipo de laboratorio para la enseñanza de la Física.
Considero que Hugo es un digno discípulo del Ing. Castro Mendívil e igualmente ha dedicado
una fracción considerable de su tiempo a la docencia, y al diseño y construcción de equipo de
laboratorio para resaltar los conceptos básicos de la Física.
He revisado el contenido de este texto y veo con gran satisfacción que su autor utiliza un
enfoque muy acertado. Toma como punto de partida una observación experimental y a partir
de allí desarrolla los conceptos físicos que permiten interpretar esta observación utilizando la
formulación matemática más sencilla. Todo esto lo hace con el detalle suficiente de manera
que el lector pueda seguir el argumento lógico con facilidad. Considero que éste es un gran
aporte de este texto. Este enfoque contrasta con textos que enfatizan la formulación
matemática y dejan al alumno huérfano de una orientación para aplicarla a una realidad física
concreta.
El contenido de temas de la Física General que son desarrollados en este texto se ajusta al
programa de estudios de la PUCP. El desarrollo de cada tema incluye ejemplos bien
seleccionados que son desarrollados con un detalle muy esmerado. Al final de cada capítulo
se incluye un conjunto de preguntas y problemas propuestos; se incluye las respuestas.
Algunos problemas plantean configuraciones complejas pero que contienen ciertas
propiedades de simetría que permiten su reducción a configuraciones sencillas. Al final del
texto encontramos un listado de referencias bibliográficas a un buen número de textos de
Física General que han servido de consulta al autor.
En general, considero que este texto constituye una representación gráfica de la obra cotidiana
que Hugo ha venido desarrollando durante su carrera docente y, por lo tanto, es un aporte muy
valioso para la comunidad académica y público en general.

Lima, julio de 2007


PRÓLOGO

Los estudiantes a menudo se preguntan por qué llevan un curso de Física. La mejor razón por la que se
estudia Física es porque proporciona un método coherente y lógico para comprender el mundo que nos
rodea; una persona que comprende lo que sucede a su alrededor, es capaz de convivir en su entorno de
manera racional y efectiva. Sin embargo, en ocasiones los estudiantes ignoran el potencial que tiene la
Física para explicar el entorno en términos fáciles de entender;
Este libro tiene por objeto brindar a los estudiantes de la Física General una ayuda para dominar los
principios físicos que son la base de la tecnología moderna. En éste libro se asume que los estudiantes
tienen una base de álgebra, geometría, y trigonometría. Es mucho más compacto que los libros de
texto tradicionales, proporciona muchos ejemplos trabajados y pide resolver problemas
Este libro será útil también como texto para una persona que repasa o que consolida su conocimiento
de la Física.
La discusión y las explicaciones narrativas son suficientemente claras y completas para poder utilizar
el libro o como texto, o como suplemento a un texto más amplio.
La forma de aprender la física es trabajar realmente con problemas. Al usar este libro, el estudiante
debe ser activo. Debe intentar trabajar cada uno de los problemas y los ejemplos. Debe mirar las
soluciones solamente si no logra dar con el camino a su solución.
Los ejemplos en este libro están trabajados exhaustivamente, de modo que puedan servir como
modelos para el propio trabajo de los estudiantes. En este sentido se considera que los estudiantes se
benefician al observar los cálculos realizados en más de una manera, por lo que se han incluido varios
métodos para efectuar los cálculos.
Además, se tuvo especial cuidado en incluir problemas y preguntas que combinan el material del
capítulo en cuestión, con material de capítulos anteriores. Tales problemas y preguntas destacan el
hecho importante de que diversas áreas de la Física se manifiestan de manera simultánea en el mundo
real. Además, este método de temas múltiples proporciona una manera para que los estudiantes
repasen lo estudiado y ayuda a mejorar la habilidad para resolver problemas.
El diseño gráfico es de gran importancia, y para mejorar su función se ha intentado enfocar solamente
una idea principal en cada figura en lo posible. Por consiguiente, las figuras del libro a menudo se
dividen en dos o más partes, para evitar la confusión de mezclar varias ideas en la misma figura.
Los profesores conocen la importancia de los diagramas de cuerpo libre cuando utilizan la segunda ley
de movimiento de Newton, y todos los estudiantes aprenden de ellos a medida que estudian Física.
Tales diagramas se utilizan en todo el libro, no solamente en los primeros capítulos en los que se
presenta y aplica la segunda ley de Newton. Por ejemplo, cuando se analiza la relación en las
oscilaciones, también entre la presión y profundidad en un fluido, el análisis se simplifica
considerablemente por medio de un diagrama de cuerpo libre. De manera semejante, cuando se deduce
la expresión para la rapidez de una onda transversal en una cuerda, un diagrama de cuerpo libre es
muy útil.
Cifras significativas. A lo largo de todo el libro se siguen los procedimientos normales para las cifras
significativas.
Se espera que el esfuerzo en la elaboración de este libro sea de utilidad tanto para los estudiantes como
para los profesores. Toda opinión al respecto será bienvenida.

Hugo Medina Guzmán


Lima Perú
AGRADECIMIENTOS

El autor agradece primeramente a los estudiantes, quienes han contribuido bastante en la


elaboración de este libro a través de su influencia en el establecimiento de las técnicas y
principios de enseñanza y a los profesores que con sus sugerencias y revisiones a las
separatas de los capítulos hicieron notar puntos que necesitaban una mayor aclaración.

Hugo Medina Guzmán


CONTENIDO
CAPÍTULO 1. Unidades, magnitudes físicas y vectores
Introducción al curso. Magnitudes físicas: escalares y vectores. Unidades. Sistema
internacional de unidades.
Precisión y cifras significativas.
CAPÍTULO 2. Movimiento rectilíneo
Definición de partícula. Concepto de movimiento de traslación y rotación. Sistemas de
referencia. Posición y desplazamiento. Movimiento en una dimensión. Velocidad.
Aceleración. Movimiento con aceleración constante. Movimiento vertical con aceleración de
la gravedad. Gráficos en cinemática: obtención de la velocidad y de la aceleración por
derivación de la función posición versus tiempo, obtención de la velocidad y de la posición
por integración de la función aceleración versus tiempo.
CAPÍTULO 3. Movimiento en un plano y en el espacio
Sistemas de referencia y el sistema de coordenadas cartesianas en dos dimensiones.
Componentes de los vectores y vectores unitarios en coordenadas cartesianas. Adición
vectorial. Movimiento en un plano. Vector posición, desplazamiento y trayectoria. Velocidad.
Rapidez. Aceleración. Movimiento parabólico. Movimiento circular: descripción horaria
(posición, velocidad y aceleración angular) y descripción vectorial cartesiana.
Componentes normal y tangencial de la aceleración. Velocidad y aceleración relativas.
Generalización del movimiento a tres dimensiones en coordenadas cartesianas.
CAPÍTULO 4. Dinámica de una partícula
Leyes de Newton del movimiento. Sistemas de referencia inerciales. Masa y fuerza. Masa y
peso. Fuerzas de contacto y a distancia (Ley de gravitación universal). Diagrama de cuerpo
libre. Aplicaciones de las leyes de Newton: partículas en equilibrio (Estática) y en
movimiento acelerado (Dinámica), fuerzas de fricción.
Dinámica del movimiento circular. Dinámica en sistemas de referencia no inerciales.
CAPÍTULO 5. Trabajo y energía
Producto escalar de vectores. Trabajo de una fuerza. Energía cinética. Trabajo y energía
cinética. Fuerzas conservativas y no conservativas. Energía potencial gravitacional y elástica.
Energía mecánica.
Generalización de la ley de conservación de la energía mecánica. Potencia.
CAPÍTULO 6. Sistema de partículas
Centro de masa. Posición, velocidad y aceleración del centro de masa. Cantidad de
movimiento lineal de una partícula y de un sistema de partículas. Impulso de una fuerza.
Segunda ley de Newton y la conservación de la cantidad de movimiento lineal para un
sistema de partículas. Energía cinética de un sistema de partículas.
Colisión elástica e inelástica.
CAPÍTULO 7. Cuerpo rígido
Producto vectorial. Torque. Segunda condición de equilibrio (Estática del cuerpo rígido).
Cantidad de movimiento angular. Momento de inercia. Rotación alrededor de un eje fijo.
Conservación de la cantidad de movimiento angular. Energía en el movimiento de rotación.
Energía cinética de rotación. Rodadura.
CAPITULO 1

INTRODUCCIÓN AL
CURSO

¿QUE ES LA FISICA? 1
METODOLOGIA DE LA FISICA 1
PARTES DE LA FISICA 1
MAGNITUDES FÍSICAS: ESCALARES Y VECTORES. 1
UNIDADES. SISTEMA INTERNACIONAL DE UNIDADES. 2
MEDICIÓN. 2
UNIDADES. 2
Unidades fundamentales 2
Unidades derivadas 3
Prefijos comúnmente encontrados. 3
CONVERSION DE UNIDADES 3
Factores de Conversión 3
ANALISIS DIMENSIONAL 4
a) Verificación de una fórmula específica. 4
b) Desarrollo de ecuaciones. 4
c) Convertir un sistema de unidades a otro. 4
CIFRAS S1GNIFICATIVAS 5
Regla 1: Redondeo de un número 6
Regla 2: Suma y Resta 6
Regla 3: Multiplicación y División 6
ERRORES 6
Error absoluto 7
Error relativo 7
Porcentaje de error 7
Clasificación de errores. 7
a) Error inherente 7
b) Error de truncado 7
c) Error de redondeo 7
d) Error de interpolación 7
e) Error de aproximación 7
PROPAGACION ERRORES 8
a) Suma de dos o más variables. 9
b) Diferencia de dos variables. 9
c) Producto de dos o más variables. 9
d) Potencias y raíces. 10
e) Cocientes. 10
PRECISIÓN Y EXACTITUD 11
RANGO DE ERROR O INCERTIDUMBRE 11
ESTIMADOS Y CÁLCULOS DEL ORDEN DE MAGNITUD 12
MODELOS IDEALIZADOS 13
¿COMO ESTUDIAR FISICA? 13
PREGUNTAS Y PROBLEMAS 14
CAPITULO 2

Movimiento rectilíneo

DEFINICIÓN DE PARTÍCULA 1
CONCEPTO DE MOVIMIENTO DE TRASLACIÓN Y ROTACIÓN 1
CONCEPTO DE MOVIMIENTO 1
CLASIFICACIÓN DEL MOVIMIENTO 1
SISTEMAS DE REFERENCIA. POSICIÓN Y DESPLAZAMIENTO 1
Sistemas de referencia 1
Vector Posición 2
Desplazamiento 2
Trayectoria y Ecuación Horaria del Movimiento 2
VELOCIDAD Y RAPIDEZ 3
Rapidez 3
Derivadas de algunas funciones 4
Velocidad 4
Velocidad instantánea 5
ACELERACIÓN 6
Aceleración Media 6
Aceleración Instantánea o simplemente aceleración 7
MOVIMIENTO RECTILÍNEO UNIFORME 8
MOVIMIENTO RECTILÍNEO UNIFORMEMENTE VARIADO 8
La Ecuación de Torricelli 9
MOVIMIENTO VERTICAL CON ACELERACIÓN DE LA GRAVEDAD. 11
a) Caída libre 12
b) Lanzamiento hacia arriba 12
c) Lanzamiento hacia abajo 12
PROBLEMA INVERSO - CÁLCULO INTEGRAL 18
Pequeña Tabla de Integrales 19
CINEMÁTICA DE PARTÍCULAS LIGADAS. MOVIMIENTOS DEPENDIENTES. 21
PREGUNTAS Y PROBLEMAS 23
CAPITULO 3

Movimiento en un plano y en el
espacio

MOVIMIENTO CIRCULAR 1
Posición angular 1
Velocidad angular 1
Aceleración angular 1
RELACIÓN ENTRE LAS MAGNITUDES ANGULARES Y LINEALES 1
Hallar el desplazamiento angular a partir de la velocidad angular. 2
Hallar el cambio de velocidad angular a partir de la aceleración angular. 2
MOVIMIENTO CIRCULAR UNIFORME 2
MOVIMIENTO CIRCULAR UNIFORMEMENTE ACELERADO 2
COMPONENTES NORMAL Y TANGENCIAL DE LA ACELERACIÓN 2
Velocidad. 2
Aceleración. 2
MOVIMIENTO CURVILÍNEO 7
El radio de curvatura 7
MOVIMIENTO PARABÓLICO 10
Ecuación de la trayectoria 10
Tiempo de vuelo 11
El alcance horizontal 11
La altura máxima 11
VELOCIDAD Y ACELERACIÓN RELATIVAS 18
Movimiento Relativo de Traslación Uniforme. La Relatividad de Galileo 18
PREGUNTAS Y PROBLEMAS 26
CAPÍTULO 4

Dinámica de una partícula

INTRODUCC1ON 1
EL ORIGEN DEL MOVIMIENTO 1
PRIMERA LEY DE NEWTON DEL MOVIMIENTO 1
¿QUÉ ES FUERZA? 1
CAMBIO DE VELOCIDAD 2
SEGUNDA LEY DE NEWÍON DEL MOVIMIENTO 3
UNIDADES DE FUERZA Y MASA 3
PESO DE UN CUERPO 4
ACCION Y REACCIÓN 3
TERCERA LEY DE NEWTON DEL MOVIMIENTO 4
APLICACIONES DE LAS LEYES DE NEWTON 4
ESTÁTICA DE LAS MASAS PUNTUALES. 4
DINÁMICA CON FRICCIÓN DESPRECIABLE. 7
FRICCIÓN 11
Algunos valores típicos de coeficientes de fricción 13
DINÁMICA DEL MOVIMIENTO CIRCULAR 27
FUERZA CENTRÍPETA 27
CURVAS EN LAS PISTAS 32
MOVIMIENTO EN MARCOS DE REFERENCIA NO INERCIALES 34
MARCO CON MOVIMIENTO DE TRASLACION NO UNIFORME 34
MARCO DE ROTACIÓN 37
FUERZA CENTRÍFUGA 38
FUERZA DE CORIOLIS 39
PREGUNTAS Y PROBLEMAS 40
CAPITULO 5

TRABAJO Y ENERGÍA

INTRODUCCION 1
TRABAJO 1
ENERGIA CINETICA 4
SISTEMAS CONSERVATIVOS Y NO CONSERVATIVOS 6
LA FUNCION ENERGÍA POTENCIAL 8
CONSERVACION DE LA ENERGÍA 9
Observadores en movimiento relativo 13
SISTEMAS NO CONSERVATIVOS 15
LA CONSERVACIÓN DE LA ENERGÍA Y LA FRICCIÓN 16
POTENCIA 16
MAQUINAS 18
PREGUNTAS Y PROBLEMAS 19
CAPÍTULO 6

SISTEMA DE PARTÍCULAS

INTRODUCCION 1
SISTEMA DE PARTICULAS 1
SEGUNDA LEY DE NEWTON APLICADA A UN SISTEMA DE PARTICULAS 1
CENTRO DE MASA 2
MOVIMIENTO DEL CENTRO DE MASA. 2
IMPULSO Y CANTIDAD DE MOVIMIENTO 4
CONSERVACIÓN DE LA CANTIDAD DE MOVIMIENTO 6
SISTEMA DE REFERENCIA CENTRO DE MASA 9
CHOQUES 9
CASOS DE CHOQUE 11
El péndulo balístico 18
MOVIMIENTO CON MASA VARIABLE - PROPULSIÓN POR REACCIÓN 20
CANTIDAD DE MOVIMIENTO ANGULAR Y TORQUE 22
MOMENTO DE INERCIA 23
MOMENT0 DE UNA FUERZA o TORQUE 23
CONSERVACION DE LA CANTIDAD DE MOVIMIENTO ANGULAR 24
CONSERVACION DE LA CANTIDAD DE MOVIMIENTO ANGULAR DE UN 26
SISTEMA DE PARTICULAS.
PREGUNTAS Y PROBLEMAS 30
CAPÍTULO 7
CUERPO RÍGIDO

INTRODUCCION 1
CUERPO RIGIDO 1
MOVIMIENTO DE UN CUERPO RÍGIDO 1
TRASLACION 1
ROTACIÓN 1
CANT1DAD DE MOVIMIENTO ANGULAR DE UN CUERPO RÍGIDO 2
MOMENTO DE INERCIA DEL CUERPO RÍGIDO. 2
El teorema de Steiner o de los ejes paralelos. 2
El teorema de la figura plana 2
SEGUNDA LEY DE NEWTON PARA ROTACION 5
Maquina de atwood tomando en cuenta la polea 7
EQUILIBRIO ESTÁTICO 11
TRABAJO Y ENERGIA EN ROTACIÓN 15
POTENCIA 16
TRASLACIONES Y ROTACIONES COMBINADAS 24
CONSERVACION DE LA CANTIDAD DE MOVIMIENTO ANGULAR 35
GIROSCOPOS Y TROMPOS - MOVIMIENTO DE PRECESION 43
PREGUNTAS Y PROBLEMAS 44

BIBLIOGRAFÍA
THEORETICAL PHYSICS, Mechanics of particles, rigid and elastic bodies, fluids and
heat flow. F: Woobridge Constant. Trinity College. Addison – Wesley Publishing Company (1959)
THEORETICAL PHYSICS,Thermodinamics, electromagnetism,waves, and particles. F:
Woobridge Constant. Trinity College. Addison – Wesley Publishing Company (1959)
The Feynman LECTURES ON PHYSICS. Volumenes I, II y III. Richard P.Feynman, Robert B. Leighton.
California Institute of Technology, Matthew Sands, Stanford University. Addison – Wesley Publishing
Company (1964)
CORRIENTES, CAMPOS Y PARTÍCULAS. Francis Bitter. Massachussets Institute of Technology. Editorial
Reverté S. A. (1964).
INTRODUCCIÓN AL ESTUDIO DE LA MECÁNICA, MATERIA Y ONDAS. Uno
Ingard, William L. Kraushaar. Editorial Reverté. (1966).
FUNDAMENTOS DE ELECTRICIDAD Y MAGNETISMO. Arthur F. Kip. University of
California. Mc Graw – Hill Book Company (1967)
CIENCIA FÍSICA Orígenes y principios Robert T. Langeman, Universidad Vanderbilt. UTEHA, (1968)
PROBLEMS IN ELEMENTARY PHYSICS. B. Bukhotsev, V: Krivchenkov, G.
Myakishev, V.Shalnov. Mir Publishers. Moscow (1971)
PROBLEMES DE PHYSIQUE COMMENTES. Tomos I y II Hubert Lumbroso. Mason et Cie, París. (1971)
ELECTRICIDAD Y MAGNETISMO PARA ESTUDIANTES DE CIENCIAS E
INGENIERÍA. Luis L. Cantú. Instituto Tecnológico y de Estudios Superiores de Monterrey.
Editorial Limusa Mexico (1973)
FÍSICA PARA LAS CIENCIAS DE LA VIDA Y LA SALUD. Simon G. G. MacDonald / Desmond M.
Burns University of Dundee. Fondo educativo interamericano. (1975)
MECÁNICA NEWTONIANA, MIT Physics course. A. P. French. Editorial Reverté. (1974).
FÍSICA I y II. Solomon Gartenhaus. Purdue University. INTERAMERICANA. (1977)
TEACHING TIPS. A guidebook for the beginning College Teacher. Wilbert J. McKeachie (University of
Michigan). Seventh edition D. C. Heath and Company (1978)
FÍSICA PARA LAS CIENCIAS DE LA VIDA. Alan H. Cromer. Northeastern University. Editorial Reverté.
(1978)
GENERAL PHYSICS WITH BIOSCIENCE ESSAYS. Jerry B. Marion. University of Maryland. John
Wiley & Sons Inc. (1979)
Física general II: Teoría Hugo Medina Guzmán, Miguel Piaggio H. QC 21 M19 (Biblioteca PUCP) (1979)
Física general II: Problemas resueltos Hugo Medina Guzmán, Miguel Piaggio H. FIS 111 M364 (Biblioteca
PUCP) (1979)
Física general I: problemas resueltos Hugo Medina Guzmán, Miguel Piaggio H. FIS 104 M364 (Biblioteca
PUCP) (1981)
FÍSICA PARA ESTUDIANTES DE CIENCIAS E INGENIERÍA. 1 y 2. John P.
McKelvey, Clemson University – Howard Grotch, Pennsilvania State University. HARLA.
Mexico. (1981)
Física 3: electricidad y magnetismo para estudiantes de ciencias e ingeniería
Hugo Medina Guzmán, FIS 141 M36 (Biblioteca PUCP) (1982)
EXPLORING PHYSICS Concepts and applications. Roger W. Redding North Texas State University, Stuart
Kenter, Wadsworth Publishing Company (1984)
PROBLEMAS DE FISICA. J. Aguilar Peris, Universidad Complutense de Madrid - J. Casanova Colas,
Facultad de Ciencias de Valladolid. Alambra (1985)
PROBLEMAS DE FISICA. dirigido por S. Kósel. Editorial Mir Moscú. (1986)
PROBLEMAS DE FISICA Y COMO RESOLVERLOS. Clarence E. Benett Maine University. CECSA
(1986)
PHYSICS for Engineering and Science. Michael E. Browne, Ph. D. (professor of Physics University of Idaho.
Schaum’s outline series Mcgraw-Hill (1988)
FÍSICA: VOLUMEN 1. Mecánica, ondas y termodinámica. Duane E. Roller, Ronald Blum. Editorial
Reverté. (1990).
FÍSICA: VOLUMEN 2. Electricidad, magnetismo y óptica. Duane E. Roller, Ronald Blum. Editorial
Reverté. (1990).
PROBLEMAS DE FISICA. dirigido por O. Ya. Sávchenko. Editorial Mir Moscú. (1989)
MECÁNICA. Berkeley physics course – volumen 1. Charles Kittel, Walter D. Knight,
Malvin A. Ruderman. Editorial Reverté SA. (1992).
ELECTRICIDAD Y MAGNETISMO. Berkeley physics course – volumen 2. Edward
M.Purcell. Editorial Reverté SA. (1992).
FÍSICA. Tomos I y II Tercera edición revisada (Segunda edición en español), Raymond S: Serway, James
Madison University, Mcgraw-Hill, (1993)
PROBLEMAS DE FISICA Santiago Burbano de Ercilla, enrique Burbano de Ercilla, Carlos Gracia Muñoz,
XXVI edición, Zaragoza, MIRA editores (1994)
ONDAS. Berkeley physics course – volumen 3. Frank S. Crawford, Jr. Editorial Reverté SA.
(1994).
FÍSICA Para las ciencias de la vida, David Jou Mirabent Universidad autónoma de Barcelona, Joseph Enric
Llebot Rabagliati, Universidad de Girona, Carlos Pérez garcía, Universidad de Navarra. Mcgraw-Hill, (1994)
Física uno Hugo Medina Guzmán, FIS 104 M365 (Biblioteca PUCP) (1995)
APPLIED PHYSICS. Arthur Beiser, Ph. D. Schaum’s outline series Mcgraw-Hill (1995)
TEACHING INTRODUCTORY PHISICS A Sourcebook. Clifford E: Swartz (State University of New York,
Stony Brook) and Thomas Miner (Associate Editor The Physics Teacher 1972 – 1988). ATP Press – Springer.
(1996)
TEACHING INTRODUCTORY PHYSICS Arnold Arons University of Washington JOHN WILEY &
SONS, INC. (1997)
FÍSICA John Cutnell / Kenneth W. Johnson. Southern Illinois University. LIMUSA (1998)
FÍSICA EN LA CIENCIA Y EN LA INDUSTRIA. A . Cromer. Northeastern University. Editorial Reverté.
(2000)
FÍSICA CONTEMPORANEA Edwin Jones.– Richard Childers, University of South Carolina. Mcgraw-
Hill, (2001)
PROBLEMAS Y CUESTIONES DE FISICA. Atanasio Lleó, Begoña Betete, Javier Galeano, Lourdes Lleó,
Ildefonso Ruiz – Tapiador. Universidad Politécnica de Madrid. Ediciones Mundi – prensa (2002)
The PHYSICS of every day phenomena. A conceptual introduction to Physics. W. Thomas Griffith, Pacific
University. Mcgraw-Hill, (2004)
FÍSICA UNIVERSITARIA. Francis W.Sears, Mark W. Zemansky, Hugh D. Young (Carnegie Mellon
University) y Roger A. Freedman (University of California. Santa Barbara) Volumen 1, Volumen 2. Undecima
edición. Pearson - Addison Wesley (2004)
FIVE EASY LESSONS Strategies for successful Physics teaching. Randall D. Knight California Polytechnic
State University, San Luis Obispo. Addison Wesley (2004)
FUNDAMENTALS OF PHYSICS. David Halliday (Univ. of Pittsburgh), Robert Resnick (Rensselaer
Polytechnic Institute), Jearl Walker (Cleveland State Univ.). 7th Edition (2005)
INTRODUCCIÓN AL CURSO Hugo Medina Guzmán

Capitulo 1. INTRODUCCIÓN AL CURSO


¿QUE ES LA FÍSICA? Actualmente la física se divide en dos clases: Física
La física es una ciencia dedicada a la comprensión de Clásica y Física Moderna.
los fenómenos naturales que ocurren en el universo. La física clásica se ocupa de los fenómenos y las
El objetivo principal del estudio científico es leyes que se conocían hasta la final del siglo XIX. La
desarrollar teorías físicas basadas en leyes física moderna se ocupa de los descubrimientos
fundamentales que permitan predecir los resultados hechos desde entonces.
de algunos experimentos. Las leyes de la física tratan La física clásica se subdivide en cierto número de
de describir los resultados de observaciones ramas que originalmente se consideraban autónomas:
experimentales y de mediciones cuantitativas de los la mecánica, el electromagnetismo, la óptica, la
procesos naturales. acústica y la termodinámica.
La física es la ciencia más simple porque estudia los La mecánica se ocupa del estudio del movimiento
sistemas más simples. La física es la base de todas las efectos físicos que pueden influir sobre este.
demás ciencias. El electromagnetismo se ocupa del estudio de los
La relación entre la física y la ingeniería es más fenómenos eléctricos y magnéticos y las relaciones
directa que la que existe entre la física y cualquier entre ellos.
otra ciencia. En la ingeniería se trabaja con sistemas a La óptica se ocupa de los efectos físicos que se
los que se aplica inmediatamente los principios de la asocian a la luz visible.
física. Cualquiera sea la rama de la ingeniería o de la La acústica al estudio de los efectos físicos
ciencia a la que uno se dedique, va a encontrar a cada relacionados con los sonidos audibles.
paso la aplicación de las nociones que aprendió en la La termodinámica se ocupa de la generación, el
física. Siempre se encontrarán útiles los conceptos transporte y la disipación del calor.
específicos de la física, las técnicas que se emplean Estas disciplinas que originalmente se desarrollaron
para resolver los problemas, la forma de pensar que independientemente, están enlazadas por medio de la
se adquiere en el estudio de la física. mecánica y el electromagnetismo.
La física moderna se inició a fines del siglo XIX, con
METODOLOGIA DE LA FISICA el descubrimiento de cierto número de fenómenos
La metodología que se usa tiene tres formas físicos que entraban en conflicto con algunos
características. conceptos de la física clásica.
La primera forma es el análisis de un sistema físico Básicamente, esas alteraciones conceptuales fueron
que se realiza en base a las propiedades de sistemas de dos tipos. Una de ellas estableció el límite superior
más sencillos, estos sistemas están relacionados de para las velocidades de las partículas a las que se
algún modo importante con el sistema original, pero aplicaban las leyes de la física clásica, esto se asocia
poseen un número menor de factores en su a la Teoría de la Relatividad de Einstein. El segundo
comportamiento. Siendo estos más sencillos se se puede considerar como el establecimiento de un
pueden investigar hasta entender bien sus límite inferior para las dimensiones lineales y de
propiedades, una vez que se obtenga el conocimiento masa de los sistemas físicos, para los que son válidas
de cada sistema se puede hacer una reconstrucción las leyes clásicas, esto se asocia a la Teoría de la
hasta lograr entender las propiedades del sistema Mecánica Cuántica. Para poder comprender estas dos
original. teorías modernas y los fenómenos de que se ocupan,
La segunda forma parte del principio de que la física es necesario estudiar primeramente las leyes de la
se fundamenta necesariamente en la experimentación. física clásica.
A veces la teoría sugiere el experimento, pero más
frecuentemente un experimentador realiza el trabajo MAGNITUDES FÍSICAS: ESCALARES Y
inicial en un área particular de la física y luego el VECTORES.
físico teórico sintetiza los resultados de los En la descripción y estudio de los fenómenos físicos
experimentos y perfecciona el entendimiento de su se han desarrollado (y se desarrollan) conceptos
significado. abstractos muy especiales llamados magnitudes
La tercera se refiere al uso frecuente de las físicas. Estas magnitudes se definen por medio de un
matemáticas. La física estudia las interacciones entre conjunto de operaciones experimentales que permiten
objetos. Los objetos interaccionan de acuerdo a obtener un número como medida de la magnitud en
ciertas leyes, sean estas conocidas o no. Como las cualquier situación.
leyes físicas son casi siempre cuantitativas, es Esta definición comprende dos pasos esenciales:
esencial poder establecer relaciones lógicas 1) La elección de una unidad de medida con
cuantitativas al estudiar los sistemas físicos. Las múltiplos y submúltiplos y
reglas que gobiernan todas estas relaciones son objeto 2) un proceso para comparar la magnitud a medir con
de las matemáticas. Por eso se dice que la matemática la unidad de medida y establecer un número (entero o
es el lenguaje de la física. fraccionario) como medida de la magnitud. Son
ejemplos de magnitudes físicas: la longitud, el área, el
PARTES DE LA FISICA volumen, el tiempo, la masa, la energía, la

1
INTRODUCCIÓN AL CURSO Hugo Medina Guzmán

temperatura, la fuerza, la potencia, la velocidad, la medir, que por definición tiene 1m de largo. Este
aceleración, etc. estándar define una unidad de la cantidad. El metro es
Llamamos magnitud física a aquella propiedad de un una unidad de distancia, y el segundo, de tiempo. Al
cuerpo que puede ser medida. La masa, la longitud, la describir una cantidad física con un número, siempre
velocidad o la temperatura son todas magnitudes debemos especificar la unidad empleada; describir
físicas. El aroma o la simpatía, puesto que no pueden una distancia como "4,29" no significa nada.
medirse, no son magnitudes físicas. Las medidas de Las mediciones exactas y fiables exigen unidades
las magnitudes se realizan mediante las unidades de inmutables que los observadores puedan duplicar en
medida, establecidas por la Unión Internacional de distintos lugares. El sistema de unidades empleado
Pesas y Medidas (UIPM), que forman el Sistema por los científicos e ingenieros se denomina
Internacional de unidades (S. I.), aunque existen otras comúnmente "sistema métrico", pero desde 1960 su
unidades que se siguen usando por tradición (como el nombre oficial es Sistema Internacional, o SI.
kilate, que se emplea para medir la masa de las Las definiciones de las unidades básicas del sistema
piedras preciosas). métrico han evolucionado con los años. Cuando la
Academia Francesa de Ciencias estableció el sistema
Magnitud escalar. Para muchas magnitudes físicas métrico en 1791, el metro se definió como una
basta con indicar su valor para que estén diezmillonésima parte de la distancia entre el Polo
perfectamente definidas. Así, por ejemplo, si decimos Norte y el Ecuador (ver figura). El segundo se definió
que José Antonio tiene una temperatura de 38 ºC, como el tiempo que tarda un péndulo de 1m de largo
sabemos perfectamente que tiene fiebre y si Rosa en oscilar de un lado a otro. Estas definiciones eran
mide 165 cm de altura y su masa es de 35 kg, está poco prácticas y difíciles de duplicar con precisión,
claro que es sumamente delgada. Cuando una por lo que se han sustituido por otras más refinadas y
magnitud queda definida por su valor recibe el por acuerdo internacional.
nombre de magnitud escalar.

Magnitudes vectoriales. Otras magnitudes, con su


valor numérico, no nos suministran toda la
información. Si nos dicen que Daniel corría a 20
km/h apenas sabemos algo más que al principio.
Deberían informarnos también desde dónde corría y
hacia qué lugar se dirigía. Estas magnitudes que,
además de su valor precisan una dirección se llaman
magnitudes vectoriales, ya que se representan
mediante vectores. En este tema estudiaremos los
vectores y sus propiedades. Unidades fundamentales
Las fuerzas, velocidades, presiones, energías, en
UNIDADES. SISTEMA INTERNACIONAL DE realidad todas las propiedades mecánicas, pueden
UNIDADES. expresarse en términos de tres cantidades básicas:
masa, longitud y tiempo. En el sistema SI, las
MEDICIÓN. La física es una ciencia experimental. unidades correspondientes son:
Los experimentos requieren mediciones cuyos Masa Kilogramo
resultados suelen describirse con números. Cualquier Longitud Metro
número empleado para describir cuantitativamente un Tiempo Segundo
fenómeno físico se denomina cantidad física. Dos Estas unidades se conocen como unidades
cantidades físicas que describen a una persona son su fundamentales.
peso y su altura. Algunas cantidades físicas son tan
básicas que sólo podemos definirlas describiendo la TIEMPO
forma de medirlas, es decir, con una definición Desde 1889 a 1967, la unidad de tiempo se definió
operativa. Ejemplos de esto son medir una distancia como una cierta fracción del día solar medio (el
con una regla, o un intervalo de tiempo con un tiempo medio entre llegadas sucesivas del Sol al
cronómetro. En otros casos definimos una cantidad cenit). El estándar actual, adoptado en 1967, es
física describiendo la forma de calcularla a partir de mucho más preciso; se basa en un reloj atómico que
otras cantidades medibles. Así, podríamos definir la usa la diferencia de energía entre los dos estados
velocidad media de un objeto como la distancia energéticos más bajos del átomo de cesio. Cuando se
recorrida (medida con una regla) dividida por el bombardea con microondas de una determinada
tiempo de recorrido (medido con un cronómetro). frecuencia, los átomos de cesio sufren una transición
entre dichos estados. Se define un segundo como el
UNIDADES. Al medir una cantidad, siempre la tiempo requerido por 9 192 631 770 ciclos de esta
comparamos con un estándar de referencia. Si radiación.
decimos que un automóvil mide 4,29 m, queremos
decir que es 4,29 veces más largo que una regla de LONGITUD

2
INTRODUCCIÓN AL CURSO Hugo Medina Guzmán

En 1960 se estableció también un estándar atómico


para el metro, usando la longitud de onda de la luz 1 nanosegundo = 1 ns =10-9 s (tiempo en el que la luz
naranja emitida por átomos de kriptón (86Kr) en un viaja 30 m)
tubo de descarga de luz. En noviembre de 1983 el 1 microsegundo = 1 μ s = 10-6 s (tiempo en el que
estándar se modificó de nuevo, esta vez de forma más una bala del rifle viaja 1 μ m)
radical. Se definió que la velocidad de la luz en el
1 milisegundo = 1 ms = 10-3 s (cerca de 14 ms entre
vacío es exactamente 299 792 458 m/s. Por
los latidos del corazón)
definición, el metro es consecuente con este número y
con la definición anterior del segundo. Así, la nueva
CONVERSION DE UNIDADES
definición de metro es la distancia que recorre la luz
Algunas veces encontramos los datos dados en
en el vacío en 1/299 792458 s. Éste es un estándar de
unidades distintas al sistema SI. En este caso
longitud mucho más preciso que el basado en una
debemos convertir las unidades al sistema SI usando
longitud de onda de la luz.
los factores conocidos de conversión.
La tabla siguiente muestra tales factores.
MASA
El estándar de masa, el kilogramo, se define como la
Factores de Conversión
masa de un determinado cilindro de aleación platino-
Longitud
iridio que se guarda en la Oficina Internacional de
1 pulgada (in) = 2,54 centímetros (cm)
Pesos y Medidas en Sevres, cerca de París. Un
1 pie (ft) = 0,3048 metro (m)
estándar atómico de masa, sería más fundamental,
1 milla (mi) = 5280 ft = 1,609 kilómetros (km)
pero aún no podemos medir masas a escala atómica
1 m = 3,281 ft
con tanta exactitud como a escala macroscópica.
1 km= 0,6214mi
Unidades derivadas ⎛o⎞
1 ángstrom ⎜ A ⎟ = 10-10 m
Las cantidades que interesan a los científicos no se ⎝ ⎠
limitan a masa, longitud y tiempo. A menudo el
1 año luz = 9,461 x 1015 m
comportamiento de objetos se describe en términos
1 unidad astronómica (AU) = 1,496 x 1011m
de sus velocidades; hay que identificar las fuerzas que
1 pársec (pc) 3,09 x 1016 m
actúan sobre los cuerpos; se paga por la energía que
Masa
consumen los aparatos domésticos y nos interesa la
1 slug = 14,59 kilogramos (kg)
potencia que pueda desarrollar un motor; la presión
1 kg = 1000 gramos = 6,852 x 10-2 slug
atmosférica es un indicador útil de las condiciones del
1 unidad de masa atómica (amu) = 1,6605 x 10-27 kg
tiempo. Todas las anteriores propiedades,
(1 kg tiene un peso de 2,205 lb donde la aceleración
aparentemente dispares, que se miden en metros por
de la gravedad es 32,174 ft/s2)
segundo (velocidad), newton (fuerza), joules
Tiempo
(energía), watts (potencia) y pascales (presión),
1 dia =24 h= 1,44 x 103 min = 8,64 x 104 s
finalmente se pueden expresar como productos de
1 año = 365,24 días = 3,156 x 107s
potencias de masa, longitud y tiempo. Esas unidades,
1 hora (h) =60min =3600s
por tanto, se conocen como unidades derivadas, para
Velocidad
distinguirlas de las tres unidades fundamentales.
1 mi/h = 1,609 km/h = 1,467 ft/s 0,4470 m/s
1 km/h = 0,6214 mi/h = 0.2778 m/s 0,9113 ft/s
Prefijos comúnmente encontrados. Utilizamos con
Volumen
frecuencia prefijos para obtener unidades de un
1 litro (L) = 10 m3 = 1000 cm3 = 0,353 1 ft3
tamaño más conveniente. Ejemplos de prefijos
1 ft3 = 0,02832 m3 = 7,481 U.S. galones (gal)
comúnmente encontrados:
1 U.S. gal = 3,785 x 10 m3 = 0,1337 ft3
1 manómetro = 1 nm = 10-9 m (un poco más grande
Fuerza
que el diámetro del átomo)
1 pound (lb) = 4,448 Newton (N)
1 micrómetro = 1 μ m =10-6 m (una célula de sangre
1 N = 10 Dinas = 0,2248 lb
humana es aproximadamente de 7 μ m) Trabajo y Energía
1 milímetro = 1 mm =10-3 m (el carbón del lápiz es 1 joule (J) = 0,7376 ft.lb = 107 ergios
aproximadamente de 0,5 milímetros en diámetro) 1 kilogramo-caloría (kcal) = 4186 J
1 centímetro = 1 cm =10-2 m (el diámetro de un 1 Btu (60°F) = 1055 J
bolígrafo) 1 kilowatt-hora (kWh) = 3,600 x 106 J
1 kilómetro = 1 km = (1000 m) 1 electron volt (eV) = 1,602 x 10-19 J
Angulo
1 microgramo = 1 μ g =10-6 g = 1-9 kg (masa de una 1 radian (rad) = 57,30°
partícula pequeña de polvo) 1° = 0,0 1745 rad
1 miligramo = 1 mg = 10-3 g = 10-6 kg (una gota de Presión
agua es aproximadamente 2 mg) 1 pascal (Pa) 1 N/m2 = 1,450 x 104 lb/in2
1 gramo = l g = 10-3 kg (la masa de un clip para papel 1 lb/in2 = 6.895 x 10-5 Pa
es de aproximadamente 1 g)

3
INTRODUCCIÓN AL CURSO Hugo Medina Guzmán

l atmósfera (atm)= 1,013 x 10 Pa= 1,013 bar = 14,70 x ∝ m a g bt c


lb/in2 = 760 torr
Donde a, b y c son exponentes que deben ser
Potencia
determinados y el símbolo ∝ indica
1 horsepower (hp) = 550 ft.lb/s = 745,7 W
1 watt (W) = 0,7376 ft.lb/s proporcionalidad. Esta ecuación es correcta
únicamente si las dimensiones de ambos lados son
iguales, como la dimensión de x es de longitud, la
ANALISIS DIMENSIONAL
La especificación numérica de una cantidad física dimensión, del lado izquierdo también debe ser de
longitud.
[m ]
depende de las unidades que se empleen. Por
ejemplo, aunque una distancia se mida en unidades de
a
g bt c = L
metros o pies o millas siempre será una distancia. Se b
⎛ L ⎞
dice que su dimensión es de longitud, la M ⎜ 2 ⎟ Tc = L
a
denominación no depende del sistema de unidades ⎝T ⎠
empleado.
Los símbolos usados para especificar la 1ongitud, la
a b c - 2b
M LT =L
masa y el tiempo son L, M y T, respectivamente. Igualando exponentes en ambos miembros
Para denotar las dimensiones de una cantidad se usan obtendremos
[]
corchetes, por ejemplo de distancia l = L, de
a = 0, b =1, c-2b = 0
De aquí a = 0, b = 1 y c = 2
[]
velocidad v = L/T, de área A = L2. [ ] Por lo tanto la expresión debe tener la forma
Entre sus aplicaciones tenemos: x ∝ gt 2 o x = kgt 2
El análisis dimensional puede describir la forma de la
a) Verificación de una fórmula específica. El ecuación pero no indica el valor de la constante k.
análisis dimensional utiliza el hecho de que las
dimensiones se pueden tratar como cantidades Ejemplo 2. Mediante el análisis dimensional
algebraicas (se pueden sumar y restar sólo si se tienen determinar la expresión para la aceleración centrípeta
las mismas dimensiones). de una partícula que describe un movimiento circular
Si una ecuación se lee uniforme.
A=B+C Solución.
Los términos A, B, y C deben tener las mismas Supongamos que la aceleración centrípeta depende de
dimensiones. la velocidad, del radio de curvatura y el peso
Ejemplo 1. Verificar la fórmula siguiente ac = kv a R bW c

x = x 0 + vt +
1 2
at , donde x y x0 representan aceleración centrípeta [ac ] = L
2 T2
[v] = L
distancias, v es velocidad, a es aceleración y t es un
intervalo de tiempo. velocidad
Solución. T
Como []
radio v = L

[x] = [x0 ] + [vt ] + ⎡⎢ 1 at 2 ⎤⎥ = L peso [W ] = 2


ML
⎣2 ⎦ T
Y las dimensiones de la velocidad son L/T y de la Reemplazando
aceleración L/T2, tenemos: L ⎛L⎞
a c
b ⎛ ML ⎞
= ⎜ ⎟ (L ) ⎜ 2 ⎟
[vt ] = ⎛⎜ L ⎞⎟(T ) = L T 2
⎝T⎠ ⎝ T ⎠
⎝T⎠ a +b +c
⇒ LT -2 = L T − a −2c M c
⎡1 2 ⎤ ⎛ L ⎞ 2
( )
⎢⎣ 2 at ⎥⎦ = ⎜⎝ T 2 ⎟⎠ T = L
Igualando exponentes para L: 1 = a + b + c
para T: − 2 = − a − 2c
Podemos ver que esta fórmula es correcta porque para M: 0 = c
todos los términos tienen la dimensión de longitud.
de donde obtenemos a = 2 , b = −1 y c = 0
por lo tanto
b) Desarrollo de ecuaciones. Esto lo podemos ver en
el ejemplo de encontrar la distancia recorrida por un v2
cuerpo en caída libre. ac = kv 2 R −1 = k
R
Pongamos que esta caída puede depender de la masa,
la aceleración de la gravedad y del tiempo. c) Convertir un sistema de unidades a otro. Si
x = f (m, g , t ) tenemos una fórmula en un sistema de unidades
El procedimiento para el análisis dimensional es podemos convertirlo a una fórmula en otro sistema de
poner la expresión en la forma unidades. Sean L1, M1, T1 y L2, M2, T2 sus unidades.

4
INTRODUCCIÓN AL CURSO Hugo Medina Guzmán

Si la cantidad G de una ecuación tiene dimensiones G M 12 M 22


= La Mb Tc. Se mide g1 con la unidad G1, y mide g2 g1 = g 2
con la unidad G2, la relación es: L21T12 L22 T22
G1 ⎛ M1 ⎞
2
g1G1 = g 2 G2 ⇒ g 2 = g 1 ⎜⎜ ⎟⎟
G2
a b c ⇒ g 2 = g1 ⎝ M2 ⎠
⎛L ⎞ ⎛ M1 ⎞ ⎛ T1 ⎞ 2 2
g 2 = g1 ⎜⎜ 1 ⎟⎟ ⎜⎜ ⎟⎟ ⎜⎜ ⎟⎟ ⎛ L1 ⎞ ⎛ T1 ⎞
⎜⎜ ⎟⎟ ⎜⎜ ⎟⎟
⎝ L2 ⎠ ⎝ M2 ⎠ ⎝ T2 ⎠
⎝ L2 ⎠ ⎝ T2 ⎠
Ejemplo 3. Si en el sistema MKS la fórmula para el
⇒ g2 = 5
(2,2 )
2
= 2,25
cálculo de la variable R de unidades kg/ms aparece
1 (3,28)2 (1)2
⎛ 5p ⎞ 2 Luego en el Sistema Inglés la ecuación
como R = ⎜⎜ ⎟⎟ correspondiente es
⎝ 1,782 A + p ⎠ 1
Donde. p tiene unidades de m/s y A de km/m3. ⎛ 2,25 p ⎞ 2
Hallar la fórmula en el Sistema Inglés. R = ⎜⎜ ⎟⎟
1 kg = 2,2 1b l m = 3,28 pie ⎝ 95,75 A + p ⎠
Solución. Para comprobar esta expresión evaluemos
Sean en el sistema MKS, L1, M1, T1, y en el sistema m kg
Inglés, L2, M2, T2. R1 para p1 = 1 , A1 = 1 3 y R2 para
s m
Las relaciones entre estos sistemas son;
pie
M1 L T p 2 = 3,28 ,
= 2,2 , 1 = 3,28 , 1 = 1 s
M2 L2 T2
2,2 lb lb
1 A2 = = 6,23 × 10 − 2
⎛ 5p ⎞2 (3,28 pie ) 3
pie 3
En la ecuación R = ⎜⎜ ⎟⎟
⎝ 1,782 A + p ⎠
Operando en las ecuaciones respectivas obtenemos
kg lb
R1 = 1,34 y R2 = 0,899
[R] = M , [ p] = L , [A] = M3 m.s pie.s
LT T L Realizando la conversión de unidades R1 encontramos
que es equivalente a R2.
La cantidad l,782 A tiene las mismas unidades que p

[1,782 A] = [1,782][A] = [1,782 ] M3 = L CIFRAS SIGNIFICATIVAS


Cuando e realizan mediciones, los valores medidos se
L T
Las unidades de 1,782 son conocen únicamente dentro de los límites de la
incertidumbre experimental, 1o datos medidos
L4
[1,782 ] = inherentemente no son exactos y si se registran en
MT notación decimal consisten de un conjunto finito de
Observando la ecuación de R, concluimos que las dígitos llamados cifras significativas, la última de las
unidades de 5 son las correspondientes a (R)2. cuales es conocida como cifra dudosa.
Cuando se mide una longitud mediante una regla se
M2
[5 ] = observa la lectura de un instrumento en el cual hay
L2 T 2 una escala, el punto de observación para la lectura
Para obtener el valor correspondiente a 1,7132 en el llega a una posición como la que se indica en la
sistema Inglés figura siguiente.
4
⎛ L1 ⎞
4 4
⎜⎜ ⎟⎟
⇒ g 2 = g1 ⎝ 2 ⎠
L1 L2 L
g1 = g2
M1T1 M 2 T2 ⎛ M1 ⎞⎛ T1 ⎞
⎜⎜ ⎟⎟⎜⎜ ⎟⎟
⎝ M 2 ⎠⎝ T2 ⎠ Se puede leer exactamente hasta 11 y apreciar un
⇒ g 2 = 1,7132
(3,28)4 = 95,75 dígito más, este último depende de cada persona
(2,2)(1) puede ser 11,6 , 11,5 ó 11,7.
Si suponemos que nuestros instrumentos están
Para obtener el valor correspondiente a 5 en el adecuadamente construidos, entonces las lecturas que
sistema Inglés tomemos tendrán significado y serán reproducibles,
excepto el último digito, el de los décimos de la

5
INTRODUCCIÓN AL CURSO Hugo Medina Guzmán

división más pequeña, será aunque con significado un Regla 3: Multiplicación y División
poco incierto. El número de cifras significativas del producto
Por lo que no hay objeto en añadir una segunda cifra cociente será redondeado a un número de
incierta. Una cifra significativa es cualquier dígito Significativas igual a aquel componente de
que denota la magnitud de la cantidad según el lugar aproximación como se muestra en los ejemplos:
que ocupa en un número. Por ejemplo si escribimos 3,14159 x 21,13 = 66,38179 = 66,38
S/. 10,52, todas las cifras son significativas, el uno 3,14159 / 21,13 = 0,14868 = 0,1487
representa el número de decenas en soles, el 0 Esto es porque 21,13 tiene sólo cuatro cifras
representa que no hay unidad de sol y es significativo significativas, el resultado se redondea a cuatro cifras
y finalmente sabemos que tenemos 52 céntimos. En significativas
la expresión 0,01052 gr. el primer cero de la
izquierda sirve para llamar la atención hacia la coma, Regla 4. Potencias y raíces
el segundo cero muestra que el 1 ocupa el segundo La potencia o raíz de un número de n cifras
lugar después de la coma. Estos ceros no son significativas se redondea a n cifras significativas.
significativos, sin embargo el 0 entre 1 y 5 es como se muestra en los ejemplos:
significativo. 2,14 2 = 4,5796 = 4,58 2,14 3 = 9,800344 = 9,80
10,52 tiene cuatro cifras significativas (1, 0, 5 y 2)
0,01052 tiene cuatro cifras significativas (1, 0, 5 y 2) 2,14 = 1,46287 = 1,46 3
2,14 = 1,288658 = 1,29
La incertidumbre más pequeña posible con cualquier
aparato de medición es mitad del límite de la lectura. Ejemplo 4. ¿Cuáles son los resultados en las cifras
Sin embargo, la mayoría de las investigaciones correctas de las siguientes operaciones indicadas?
generan una incertidumbre mayor que esto. La tabla a) 2,5 x 10-2 x 20
siguiente enumera la incertidumbre de algunos b) 3,32 x 103 + 3,2 x 10
equipos comunes del laboratorio. c) 4,52 x 108 + - 4,2 x 103
d) 2,801 x 4 x 10-3
Regla de metro ± 0,05 cm e) 6,2 x 104 / 3,0 x 10
Calibrador vernier ± 0,005 cm Solución.
Micrómetro ± 0,005 mm Aquí todos los números están expresados en notación
científica.
Reloj de segundos ± 0,5 s
Por ejemplo:
Cronómetro ± 0,0005 s
0,025 = 2,5 x10-2 = 2,5(-02), tiene 2 cifras
Dinamómetro ± 0,1 N significativas
20 = 2 x 10 = 2(+1), tiene una cifra significativa.
Cuando se anotan y se manipulan números obtenidos
por medidas, serán de mucha ayuda las siguientes a) 2,5 x 10-2 x 20 = 5 x 10-1
reglas: b) 3,32 x 103 + 3,2 x 10 = 3,35 x 103
c) 4,52 x 108 - 4,2 x 103 = 4,52 x 108
Regla 1: Redondeo de un número - d) 2,801 x 4 x 10-3 = 11 x 10-3
En el proceso de rechazo de uno o varios de los e) 6,2 x 104 / 3, 0 x 10 = 2,1 x 103
últimos dígitos. La última cifra retenida se
incrementará en 1 si la cifra rechazada es 5 o mayor. Ejemplo 5. Para determinar la densidad de un líquido
Ejemplo. se toman 10 cm3 de éste. La masa del líquido medida
Número Redondeo a en una balanza es 15,38g. ¿Cuál es la expresión
dado Cuatro Tres Dos correcta de la densidad?
cifras cifras cifras Solución.
62,578 62,58 62,6 63 La densidad del líquido es
10 232 10 230 10 200 10 000 m 15,38 g
329 350 329 400 329 000 330 000 ρ= = = 1,538 3
V 10 cm
Regla 2: Suma y Resta Siendo 10 el número con menos cifras significativas
El número de cifras significativas de la suma o (2), el resultado se redondea a 2 cifras significativas.
diferencia será redondeado desechando todas las La expresión correcta de la densidad es
cifras a la derecha del lugar ocupado por la cifra g
incierta en cualquiera de las cantidades que esté más ρ = 1,5
cm 3
hacia la izquierda, como se muestra en el ejemplo:
ERRORES
Como hemos indicado las mediciones físicas
involucran incertidumbre. El valor exacto de una
magnitud medida es algo a lo cual intentamos
aproximarnos pero que nunca conocemos. Un número
de lecturas cuando se promedia se considera como el

6
INTRODUCCIÓN AL CURSO Hugo Medina Guzmán

mejor acercamiento al verdadero valor de una lectura, Si se usan los dos primeros términos.
⎛π ⎞
3
y la diferencia entre una lectura y la verdadera lectura
o lectura exacta se llama error. Aquí la palabra error ⎜ ⎟
π
et = N - N = 1,00000 − + ⎝ ⎠ = +0,07516
2
no significa equivocación sino una incertidumbre.
2 3!
Error absoluto es la diferencia entre el valor (+7,5%)
aceptado N (asumimos conocido) y el valor Si se usan los tres primeros términos.
⎛π ⎞ ⎛π ⎞
3 5
aproximado N , obtenido por mediciones o cálculos. ⎜ ⎟ ⎜ ⎟
e= N-N π ⎝2⎠ ⎝2⎠
et = N - N = 1,00000 − + −
2 3! 5!
Error relativo es la relación entre el error absoluto e = -0,00453 (-0,5%)
y el valor aceptado N Si se usan los cuatro primeros términos.
e N et = 0,00015 , el error de truncado ya es
e= = 1− insignificante.
N N
c) Error de redondeo (er ) , es el error introducido
Porcentaje de error es el número de partes por cada por redondeo de un decimal. Por ejemplo.
100 en que un número está errado Si π = 3,14159
⎛ N⎞ Si redondeamos a π = 3,14, entonces:
e% = (100e )% = ⎜⎜1 − ⎟⎟% er = 3,14159 - 3,14 = 0,00159 y
⎝ N⎠
0,00159
er = × 100 = 0,05%
Cuando calcule el porcentaje de error en física 3,14159
d) Error de interpolación (e p ) , es el error
elemental no use más de dos cifras significativas.
Por ejemplo si una pista para carreras de 3500 metros
tiene 17 metros más. introducido por la aproximación de un valor por su
El error absoluto o simplemente error es equivalente interpolado. Por ejemplo:
e = 17 m Si conocemos la circunferencia de un círculo de l0
El error relativo es metros de diámetro y de otro circulo de 11 metros.
17 C10 = 10π = 31,42 m y
e=
3500 C11 = 11π = 34,56 m
El porcentaje de error es Por interpolación lineal la circunferencia de un
17 círculo de 10,6 metros es:
e% = × 100% = 0,49%
3500 C10,6 = C10 + (C11 − C10 ) × 0,6 = 33,30 m
Pero el valor exacto es
Clasificación de errores. C10,6 = 10,6 × π = 33,31 m
En los cálculos numéricos pueden ocurrir cinco tipos De aquí
de errores básicos. e p = 33,31 − 33,30 = 0,01 m
a) Error inherente (ei ) . Es el error en los datos
0,01
iniciales debido a mediciones, observaciones o o ep % = × 100 = 0,03%
registros inexactos. 33,31
b) Error de truncado et . Es el error creado por e) Error de aproximación (ea ) , es el error
representar una función con sólo unos cuantos introducido por la aproximación de una constante o
términos de una serie. Por ejemplo: una función por un valor elegido. Por ejemplo:
π La aceleración debido a la gravedad g = 9,80665
El valor correcto de N = sen = 1,000 m/s2 puede aproximarse por:
2 51 m
El valor aproximado de N computado por expansión g= × 10 = 9,80769 2 ⇒ ea % = 0,01%
52 s
de series es: mejor por
⎛π ⎞ ⎛π ⎞ ⎛π ⎞
3 5 7
507 m
⎜ ⎟ ⎜ ⎟ ⎜ ⎟ g= × 10 = 9,80658 2 ⇒
π ⎝2⎠ ⎝2⎠ ⎝2⎠ 517 s
N= − + − ...
2 3! 5! 7! ea % = 0,00%
Si se usa solo el primer término. (El error aparece en el cuarto decimal)
π
et = N - N = 1,00000 − = -0,57080 (-57%)
2

7
INTRODUCCIÓN AL CURSO Hugo Medina Guzmán

Error cuadrático medio o desviación normal o mm el error absoluto o incertidumbre de la medida es


estándar Δl = 0,05 mm.
En general cuando se realiza una medición cualquiera
siempre se comete error, cuando repetimos las Ejemplo 6. Un estudiante realiza varias mediciones
mediciones varias veces, encontramos casi siempre de la masa de un cuerpo, obteniendo los siguientes
resultados diferentes para cada una, aunque resultados: 35,73 g , 35,76 g , 35,80 g, 35,76 g, 35,70
empleemos el mismo método y el mismo aparato. g
Las mediciones sucesivas de un objeto determinado ¿Cuál es el mejor valor estimado de la masa del
presentan discrepancias debido a los errores al azar o cuerpo?
aleatorios de las medidas. Si la longitud verdadera de Solución.
una varilla es l 0 la media aritmética de un gran La masa media es:
35,73 + 35,76 + 35,80 + 35,76 + 35,70
número de medidas sucesivas será un número que mm =
representa la longitud media l m . Una medida 5
Individual cualquiera tendrá una desviación de la = 35,75 g
media e = l − l m , cantidad que puede ser positiva o La desviación de la media de cada medición es:
negativa según l sea mayor o menor que l m , es m1 − mm = 35,73 − 35,75 = −0,02
decir m2 − mm = 35,76 − 35,75 = 0,01
l = lm ± e m3 − mm = 35,80 − 35,75 = 0,05
Si elevamos al cuadrado cada uno de los valores de e
m4 − mm = 35,76 − 35,75 = 0,01
y tomamos la media de todos los e 2 , obtenemos
m5 − mm = 35,70 − 35,75 = - 0,05
em2 que es la varianza de las medidas.
La varianza de las medidas es:
(− 0,02 )2 + (0,01)2 + (0,05)2 + (0,01)2 + (− 0,05)2
n

∑e 2
i em2 =
5
em2 = 1=1
= 0,0112
n La desviación normal
A la raíz cuadrada de esta medía se la conoce como el
error cuadrático medio o desviación normal o σ = em2 = 0,0112 = 0,0334
estándar σ . La incertidumbre o error estándar de la medida es:
σ= e 2

σ
m
0,0334
Cuanto mayor sea el número n de medidas, menor Δm = = = 0,01496 = 0,02
será la diferencia entre su media l m y la longitud n 5
El mejor valor estimado es:
verdadera l 0 , es decir el error estándar de la media,
m = mm ± Δm = 35,75 ± 0,02
σ
, será menor. Por esto el mejor valor estimado m = (35,75 ± 0,02 ) g
n Si hubiéramos realizado una sola medición con una
de l 0 es: balanza cuya menor división es de 0,1 g la
incertidumbre seria 0,05 y el resultado de la medición
σ
l = lm ± = l m ± Δl podría expresarse así:
n m = (35,75 ± 0,05) g
En donde Δl es la incertidumbre o error absoluto Observemos que en ambos casos la incertidumbre
determinado a partir de n mediciones. En el caso de corresponde al segundo orden decimal (0,02 y 0,05
verdaderos errores aleatorios, la media l m cae en un respectivamente) incidiendo por lo tanto en la cifra 5,
que es la cifra dudosa.
68 por ciento de las veces dentro de una distancia Δl
del valor verdadero pero desconocido l 0 . PROPAGACIÓN ERRORES
De esta forma podemos presentar el resultado final de La determinación experimental de algunas cantidades
un experimento en el cual se mide varias veces una físicas tales como densidad o volumen se obtienen
magnitud. Sin embargo, muchas veces realizamos por medición directa. Generalmente, la cantidad a
sólo una medición de la magnitud. En este caso se determinar se re1aciona de alguna manera conocida a
considera generalmente que la incertidumbre o error una o más cantidades medibles. El procedimiento es
absoluto es igual a la mitad de la división menor de la medir estas cantidades y con estas calcular por medio
escala del instrumento. Por ejemplo: si para medir de relaciones conocidas la cantidad original. Por
longitudes se usa una regla cuya división minina es 1 ejemplo el volumen de un cilindro puede conocerse si
tenemos su longitud y Su diámetro. Estas pueden
medirse directamente, cada una con su intervalo de

8
INTRODUCCIÓN AL CURSO Hugo Medina Guzmán

error asociada, Estos intervalos de error determinan el magnitudes la incertidumbre en el resultado es la raíz
Intervalo de error de la cantidad calculada. Es cuadrada de la suma en cuadratura de las
importante saber como hacer esta determinación de la incertidumbres en las magnitudes.
propagación de errores.
A continuación determinemos los errores para Ejemplo 7. Medimos la masa de un tomillo y
diferentes situaciones. obtenemos m1 ± Δm1 = (253 ± 5) g , luego
a) Suma de dos o más variables.
medimos también la masa de una tuerca,
Consideremos z = x + y .
m2 ± Δm2 = (48 ± 5) g . ¿Cuánto vale la masa M
z ± Δz = ( x ± Δx ) + ( y ± Δy ) del tornillo y la tuerca juntos?
Puesto que x e y tienen las incertidumbres Δx y Solución.
Δy , ¿cuál es la incertidumbre Δz en z? Evidentemente, la masa M es
Los mayores valores posibles para x e y son x + Δx M = m1 + m2 == 253 + 48 = 301 g
e y + Δy , respectivamente, dando un valor superior La Incertidumbre en la suma es
de Δz = Δx + Δy . ΔM 2 = Δm12 + Δm22 = 50 = 7 g
Los menores valores posibles para x e y son x − Δx y el resultado final es
e y − Δy , respectivamente, dando un valor inferior M = (301 ± 7 ) g
de Δz = −(Δx + Δy ) .
Ejemplo 8. ¿Cuál es la diferencia M’ entre las masas
Es decir, los valores límites para z son
z = ( x + y ) ± (Δx + Δy )
m1 y m2 del tornillo y la tuerca respectivamente?
Solución.
Sin embargo, no utilizamos los (Δx + Δy ) como la Evidentemente, la masa M’ es
incertidumbre. M ' = m1 − m2 == 253 − 48 = 205 g
La razón es que para que z realmente valga La Incertidumbre en la diferencia también es
z = ( x + y ) ± (Δx + Δy ) se necesita que la
incertidumbre en la medición, tanto de x como de y,
ΔM ' 2 = Δm12 + Δm22 = 50 = 7 g
sea tal que los dos resultados experimentales sean y el resultado final es
subestimaciones. M ' = (205 ± 7 ) g
Más probable es que uno de los resultados sea un
poco bajo y el otro un poco alto. Si éste es el caso, la c) Producto de dos o más variables.
incertidumbre en una de las mediciones puede Supongamos z = xy
compensar, en parte, la incertidumbre en la otra.
Para tomar en cuenta esta posibilidad, lo que hacemos z ± Δz = ( x ± Δx )( y ± Δy )
no es sumar las incertidumbres, sino que calculamos = xy ± yΔx ± xΔy + ΔxΔy
Δz = Δx 2 + Δy 2 el error de z es Δz = yΔx + xΔy
Esta manera de combinar las incertidumbres, considerando el mayor valor posible y no tomando en
sumándolas elevadas al cuadrado, se llama suma en cuenta ΔxΔy por se el producto de dos cantidades
cuadratura. pequeñas.
La incertidumbre Δz calculada de esta manera es El significado de esto se más claramente en el error
siempre mayor que las a Δx y Δy por separado, relativo.
pero menor que la suma Δx + Δy . La diferencia Δz yΔx + xΔy Δx Δy
= = +
entre simplemente sumar las incertidumbres y z xy x y
sumarlas en cuadratura es que la suma simple da la
Ejemplo 9. ¿Cuál es el producto de (2,6 ± 0,5) cm
incertidumbre máxima en el resultado, mientras que
la suma en cuadratura da la incertidumbre más
probable. y (2,8 ± 0,5) cm?
b) Diferencia de dos variables Solución.
Consideremos z = x − y . Primero, determinamos el producto de 2,6cm x 2,8cm
z ± Δz = ( x ± Δx ) − ( y ± Δy ) = 7,28 cm2
La incertidumbre que queremos es la incertidumbre 0,5
Error relativo 1 = = 0,192
más probable, que viene a ser la raíz cuadrada de la 2,6
suma en cuadratura de las incertidumbres
0,5
Δz = Δx 2 + Δy 2 Error relativo 2 =
2,8
=0,179
Por lo tanto, tenemos una regla para la propagación Suma de los error relativos = 0,371 o 37,1 %
de incertidumbres Cuando sumamos o restamos dos

9
INTRODUCCIÓN AL CURSO Hugo Medina Guzmán

Error absoluto = 0,37l x 7,28 cm2 o 3,71 % x 7,28 Ejemplo 12. Encontrar el error en el cálculo de
cm2 = 2,70cm2 1
Los errores son expresados con una cifra significativa z= = x −3
= 3 cm2 x3
El producto es igual a 7,3 ± 3 cm2 Solución.
Δx
d) Potencias y raíces.
Δz = −3x −3−1 Δx = −3 x − 4 Δx = −3
x4
Sea z = x
n
Como los errores son indeterminados debemos elegir
Donde n es el número entero o fracción positivo o el signo de tal manera que éste sea el máximo, por
negativo. esto:
z ± Δz = ( x ± Δx ) Δx
n
Δz = 3
Esto se puede escribir x4
y el error relativo es
⎛ Δx ⎞
n

z ± Δz = x ⎜1 ± ⎟
n
Δx
⎝ x ⎠ Δz
3 4
Δx
= x =3
⎛ Δx ⎞
n
z 1 x
Haciendo la expansión binomial de ⎜1 + ⎟
⎝ x ⎠ x 3

e) Cocientes.
⎛ Δx ⎞ =
n

⎜1 + ⎟ x
⎝ x ⎠ Supongamos z =
Δx n(n − 1) ⎛ Δx ⎞ n(n − 1)(n − 2) ⎛ Δx ⎞
2 3 y
1+ n
x
+ ⎜ ⎟ +
2! ⎝ x ⎠ 3!
⎜ ⎟ + ...
⎝ x ⎠
z ± Δz =
(x ± Δx )
ignorando las potencias mayores que 1 de Δx ( y ± Δy )
⎛ Δx ⎞ Δx
n
Esto se puede escribir como:
⎜1 + ⎟ = 1+ n z ± Δz = ( x ± Δx )( y ± Δy )
−1
⎝ x ⎠ x
−1
De aquí ⎛ Δx ⎞ 1 ⎛ Δy ⎞
Δx ⎞ = x⎜1 ± ⎟ ⎜1 ± ⎟

z ± Δz = x n ⎜1 ± n ⎟ ⎝ x ⎠ y ⎜⎝ y ⎟⎠
⎝ x ⎠
x ⎛ Δx ⎞⎛ Δy ⎞
n −1
El error de z es Δz = nx Δx ≈ ⎜1 ± ⎟⎜1 m ⎟
Y el error relativo es
y⎝ x ⎠⎜⎝ y ⎟⎠
Δz Δx x ⎛ Δx Δy Δx Δy ⎞
=n ≈ ⎜1 ± ± + ⎟
z x y ⎜⎝ x y x y ⎟⎠
Ignorando el último término por se muy pequeño y
Ejemplo 10. Encontrar el error en el cálculo de tomando el valor máximo para Δz .
z = x2 El error de z es:
Solución. x ⎛ Δx Δy ⎞ yΔx + xΔy
Δz = 2 x 2−1 Δx = 2 xΔx Δz = ⎜ + ⎟=
E error relativo es
y ⎜⎝ x y ⎟⎠ y2
Δz Δx El error relativo es:
=2
z x yΔx + xΔy
Ejemplo 11. Encontrar el error en el cálculo de Δz y2 yΔx + xΔy Δx Δy
= = = +
z = x = x1 2 z x xy x y
Solución y
1 Δx
1
1 2 −1
Δz = x Δx = Ejemplo 13. Supongamos que queremos calcular la
2 2 x densidad ρ de un cilindro de metal habiendo
E error relativo es
medido su masa M, su longitud L y su diámetro D. Al
Δz 1 Δx
= mismo tiempo queremos calcular el error relativo
z 2 x resultante de los errores en las cantidades medidas.
Sabemos que la densidad está dada por la ecuación

10
INTRODUCCIÓN AL CURSO Hugo Medina Guzmán

M 4M
ρ= =
π (D 2 ) L
Ejemplo 16. La medida de los lados de un rectángulo
2
πD 2 L son (1,53 ± 0,06) cm, y (10,2 ± 0,1) cm,
Solución. respectivamente. Hallar el área del rectángulo y el
4M 4
ρ= = MD − 2 L−1 error de la medida indirecta.
πD L π
2 Solución.
Como 4 y π son cantidades exactas no tienen error. El área es A = 1,53 × 10,2 = 15,606 cm2
ΔM Como debe de tener solamente 3 cifras significativas
El error relativo de M es
M A = 15,6 cm 2
2ΔD El error relativo del área
El error relativo de D es 2 2
D ΔA ⎛ 0,06 ⎞ ⎛ 0,1 ⎞
= ⎜ ⎟ +⎜ ⎟ = 0,0404422504
ΔL A ⎝ 1,53 ⎠ ⎝ 10,2 ⎠
El error relativo de L es
L El error absoluto del área
De aquí ΔA = 0,0404422504(1,53 × 10,2 ) = 0,63083
El error relativo de ρ es
El error absoluto con una sola cifra significativa es
Δρ ΔM 2ΔD ΔL 0,6.
= + +
ρ M D L La medida del área junto con el error y la unidad se
escribirá como
Ejemplo 14. El volumen de un cilindro de base A = (15,6 ± 0,6 ) cm 2
circular es V = π R L . ¿Cuánto vale la
2

incertidumbre o error en el volumen en términos de Ejemplo 17. Se mide x con una incertidumbre Δx
las incertidumbres ΔR y ΔL ? y se calcula y = ln x . ¿Cuánto vale Δy ?
Solución. Solución.
Como π es cantidad exacta no tienen error. y + Δy = ln ( x + Δx )
2ΔR En este caso podemos usar aproximaciones para
El error relativo de R es
R cantidades pequeñas, cuando x << 1 , tales como:
ΔL
El error relativo de L es (1 ± x )n
≈ 1 ± nx , e x ≈ 1 + x , ln (1 + x ) ≈ x ,
L
De aquí senx ≈ x , cos x ≈ 1 , tan x ≈ x
En nuestro caso
⎛ Δx ⎞
y + Δy = ln( x + Δx ) = ln x⎜1 +
El error relativo de V es
ΔV ΔR ΔL ⎟
=2 + ⎝ x ⎠
V R L ⎛ Δx ⎞ Δx
Y el error absoluto: = ln x + ln⎜1 + ⎟ ≈ ln x +
⎛ ΔR ΔL ⎞ ⎛ R ⎞ ⎝ x ⎠ x
ΔV = ⎜ 2 + ⎟V = πR⎜ 2ΔR + ΔL ⎟ Δx
⎝ R L ⎠ ⎝ L ⎠ Como << 1 podemos aplicar
x
Ejemplo 15. Supongamos que queremos medir el ⎛ Δx ⎞ Δx
periodo T de un oscilador, es decir, el tiempo que ln⎜1 + ⎟≈ , luego:
tarda en efectuar una oscilación completa, y
⎝ x ⎠ x
disponemos de un cronómetro que aprecia las ⎛ Δx ⎞ Δx
décimas de segundo, 0,1 s. Medimos el tiempo que y + Δy = ln x + ln⎜1 + ⎟ ≈ ln x +
tarda en hacer 10 oscilaciones, por ejemplo 4,6 s,
⎝ x ⎠ x
dividiendo este tiempo entre 10 resulta t =0,46 s, Siendo y = ln x :
¿cómo se expresa la medida? Δx
Solución. Δy =
x
t Δt
T= , ΔT =
10 10 PRECISIÓN Y EXACTITUD
0,1 Los términos "PRECISION " y "ACCURACY" del
Obtenemos para el error ΔT = = 0,01 s . Por idioma inglés no son sinónimos, para efectos de
10 lenguaje estadístico traduciremos "Precision" como
tanto, la medida la podemos expresar como precisión y "Accuracy" como exactitud, estableciendo
T = (0,46 ± 0,01) s diferencias claras entre las dos palabras.

11
INTRODUCCIÓN AL CURSO Hugo Medina Guzmán

La precisión es una indicación de la concordancia


entre un número de medidas hechas de la manera
indicada por el error absoluto. Un experimento de
gran precisión tiene un bajo error al azar.
La exactitud es una indicación de cuan cercana está
una medida al valor aceptado indicado por el error
relativo o del porcentaje de error en la medida. Un
experimento de gran exactitud tiene un error
sistemático bajo. En la dirección vertical, dibujamos una línea arriba y
Así como la obtención de una serie de medidas con abajo para que cada punto muestre la gama de
las unidades correctas, se requiere una indicación del incertidumbre del valor de la fuerza. Entonces
error experimental o el grado de incertidumbre en las ponemos una pequeña línea marcadora horizontal en
medidas y la solución. Cuanto mayor es la exactitud el límite del extremo incierto para el punto.
y la precisión en nuestras investigaciones, más bajo En la dirección horizontal, dibujamos una línea a la
es el grado de incertidumbre. izquierda y a la derecha para que cada punto muestre
Las cuatro figuras a continuación ilustran la la gama de incertidumbre del valor de la extensión.
diferencia: Entonces ponemos una pequeña línea marcadora línea
vertical en el límite del extremo incierto para el
punto.
Cuando todos los puntos de la tabla se trazan en un
gráfico, la línea del mejor ajuste con las barras
apropiadas de error se muestra en la figura siguiente y
se puede ver que la línea del mejor ajuste cae dentro
del rango de la incertidumbre de la barra del error.

RANGO DE ERROR O INCERTIDUMBRE


Cuando una respuesta se expresa como valor con
incertidumbre tal como 2,3 ± 0,1 cm, entonces la
gama de la incertidumbre es evidente. ¿El valor cae
entre 2,4 (2,3 + 0,1) y 2,2 (2,3 - 0,1) cm. En la
ESTIMADOS Y CÁLCULOS DEL ORDEN DE
física, determinamos a menudo la relación que existe
MAGNITUD
entre las variables. Para visión la relación, podemos Hasta donde hemos visto, es importante cuidar el
realizar una investigación y trazar un gráfico del eje seguimiento de las incertidumbres en la medición
dependiente) contra la variable independiente (eje x). cuando se calculan las respuestas a los problemas. En
Considere un resorte que tenga varios pesos, unido a algunas ocasiones, tanto en la vida cotidiana como en
él. A mayor peso se une a un resorte, el resorte el quehacer científico, es necesario resolver un
extiende más lejos de su posición del equilibrio. La problema del que no tenemos información suficiente
tabla siguiente muestra algunos valores para esta para obtener una respuesta precisa. A menudo
investigación de Fuerza/alargamiento. podemos obtener una respuesta útil mediante la
estimación de los valores de las magnitudes
Fuerza ± 5 N 100 150 200 250 300 apropiadas. Estas estimaciones, realizadas
Alargamiento 3,0 4,4 6,2 7,5 9,1 generalmente a la potencia de diez más cercana, se
± 0,2 cm denominan estimaciones del orden de magnitud. El
Cuando se traza un gráfico de la fuerza contra el cálculo resultante del orden de magnitud no es
alargamiento, la línea del mejor ajuste no pasa por exacto, pero generalmente es correcto con un factor
cada punto. Una barra del error se puede utilizar para de diez. El conocimiento justo del orden de magnitud
dar una indicación del rango de la incertidumbre para de las cantidades físicas con frecuencia nos
cada punto según se muestra en la figura a proporciona información suficiente para obtener una
continuación Fuerza/alargamiento. comprensión útil de la situación física y la capacidad
para formarnos un juicio y hacer cálculos para la
construcción de modelos.
Realizar estimaciones de magnitud con frecuencia es
sencillo. Por ejemplo, imagine que va a la escuela por

12
INTRODUCCIÓN AL CURSO Hugo Medina Guzmán

primera vez y que quiere estimar cuánto dinero Así, el número aproximado de caramelos que hay en
necesitara para comprar libros. Usted conoce que la el frasco es:
carga habitual para la mayor parte de los estudiantes 800cm3
es de cinco materias, y que en cada una se necesita un Número de caramelos ≈ ≈ 240 .
libro de texto. 27 3
cm
Con estos datos puede estimar el costo de un solo 8
libro con el razonamiento siguiente. Sabe por Un conteo realizado de los caramelos que llenan un
experiencia que S/. 1 es demasiado bajo y que S/. 100 frasco de un cuarto (0,95 litros) dio 255 caramelos.
es demasiado alto. Incluso S/. 10 es bajo. Una
estimación razonable puede ser S/. 50. Así, el costo MODELOS IDEALIZADOS
estimado de los libros para un semestre es de 5 x S/. Ordinariamente usamos la palabra "modelo" para
50 = S/. 250. Aunque el resultado no es exacto, está referimos a una réplica en menor escala (digamos, de
dentro del orden de magnitud correcto y proporciona un ferrocarril) o a una persona que exhibe ropa (o se
una estimación razonable a un problema real. El exhibe sin ropa). En física, un modelo es una versión
siguiente ejemplo ilustra la aplicación de las simplificada de un sistema físico que sería demasiado
estimaciones del orden de magnitud. complejo si se analizase de forma detallada. Por
ejemplo, supongamos que nos interesa analizar el
Cuando hacemos cálculos de este tipo con frecuencia movimiento de una pelota de béisbol lanzada en el
también efectuamos otras aproximaciones. Al aire. ¿Qué tan complicado es el problema? La pelota
remplazar π por 3 o remplazar 2 por 3/2 no es perfectamente esférica ni perfectamente rígida:
hacemos pocas diferencias en el orden de magnitud, tiene costuras, está girando y se mueve en el aire. El
pero hacerlo simplifica mucho los cálculos. Los viento y la resistencia del aire afectan su movimiento,
ejemplos siguientes ilustran esta técnica. la Tierra gira, el peso de la pelota varía un poco al
cambiar su distancia respecto al centro de la Tierra,
Ejemplo 18. Una tienda ofrece un premio al cliente etc. Si tratamos de incluir todos estos factores, la
que adivine con la mayor aproximación el número de complejidad del análisis nos abrumará. En vez de
caramelos de goma que llenan un frasco de un litro ello, inventamos una versión simplificada del
exhibido en un mostrador de la tienda. (Un litro es problema. Omitimos el tamaño y la forma de la pelota
igual a 1000 cm3.) Estime cual será el número. representándola como objeto puntual, o partícula.
Despreciamos la resistencia del aire haciendo que la
pelota se mueva en el vacío, nos olvidamos de la
rotación terrestre y suponemos un peso constante.
Ahora tenemos un problema sencillo de tratar.
Para crear un modelo idealizado del sistema debemos
pasar por alto muchos efectos menores y
concentramos en las características más importantes.
Claro que hay que ser cuidadosos para no despreciar
demasiadas cosas. Si ignoramos totalmente los
efectos de la gravedad, nuestro modelo predecirá que
Solución. si lanzamos la pelota hacia arriba ésta se moverá en
Una revisión cuidadosa del frasco (véase la figura) línea recta y desaparecerá en el espacio. Necesitamos
revela varias cosas. Los caramelos de goma pueden algún criterio y creatividad para crear un modelo que
aproximarse vagamente a pequeños cilindros de casi simplifique lo suficiente un problema sin omitir sus
2 cm de largo por aproximadamente 1,5 cm de características esenciales.
diámetro. Además, los caramelos no están apretados Al usar un modelo para predecir el comportamiento
en el frasco; posiblemente tan só1o se ha llenado 80% de un sistema, la validez de las predicciones está
de éste. Podemos hacer uso de estas observaciones limitada por la validez del modelo. La predicción de
para estimar el número de caramelos que hay en el Galileo respecto a la caída de los cuerpos corresponde
frasco. a un modelo idealizado que no incluye la resistencia
Volumen ocupado del frasco del aire. El modelo funciona bien para una bala de
Número de caramelos =
Volumen de un caramelo cañón, pero no para una pluma.
El concepto de modelos idealizados es muy
EI volumen ocupado del frasco = 0,8 x 1000 = 800 importante en física y en tecnología. Al aplicar
cm3, principios físicos a sistemas complejos siempre
Volumen de un caramelo = usamos modelos idealizados, y debemos tener
2 presentes las suposiciones que hacemos. De hecho,
⎛3 ⎞
⎛d ⎞
2 ⎜ cm ⎟ 27 3
los principios mismos se expresan en términos de
hπ ⎜ ⎟ ≈ 2cm × 3⎜ 2 ⎟ ≈ cm modelos idealizados; hablamos de masas puntuales,
⎝2⎠ ⎜⎜ 2 ⎟⎟ 8 cuerpos rígidos, aislantes ideales, etc. Estos modelos
⎝ ⎠ desempeñan un papel crucial en este libro. Trate de

13
INTRODUCCIÓN AL CURSO Hugo Medina Guzmán

distinguirlos al estudiar las teorías físicas y sus habilidad para resolver problemas puede ser la
aplicaciones a problemas específicos. principal prueba de los conocimientos. Es esencial
que se comprendan los principios y conceptos básicos
¿COMO ESTUDIAR FISICA? antes de intentar resolver problemas.
Para estudiar física es necesario dar atención especial En física general los exámenes se componen
a los significados específicos de las palabras para principalmente de problemas a resolver, es muy
poder entender el material, deben estudiarse importante que se entiendan y recuerden las hipótesis
detenidamente los gráficos, dibujos, tablas y que sirven de base a una teoría o formalismo en
fotografías incluidos para entender claramente los particular.
principios físicos involucrados. Para la resolución de problemas se incluyen cinco
Gran parte de lo que se aprenderá será en las clases. etapas básicas:
Deberán aprender a tomar apuntes exclusivamente de a) Dibuje un diagrama con ejes coordenados si son
las partes significativas de cada lección y necesarios y ponga las notaciones identificatorias,
concentrarse por completo en lo que el profesor está con esto podemos eliminar errores de signo.
diciendo, estos apuntes son necesariamente breves y b) Identifique el principio básico, incógnitas, listando
carentes de relación. Por lo tanto, es recomendable los datos y las incógnitas.
tener un cuaderno ordenado con las notas de clase c) Seleccione una relación básica o encuentre una
completando con apuntes tomados del estudio de los ecuación que se pueda utilizar para determinar la
libros. Hagan esto tan pronto como sea posible incógnita y resuélvala simbólicamente. En esta forma
después de clase, esto permitirá tener un conjunto de se evitan errores y ayuda a pensar en términos físicos
notas claras e inteligibles para repaso; ayudará a el problema.
detectar las áreas débiles de conocimiento. d) Sustituya los valores dados con las unidades
La parte más importante de los apuntes son los apropiadas dentro de la ecuación y obtenga el valor
problemas resueltos. Resuélvanse todos los ejemplos numérico de la incógnita.
vistos en clase y los dejados como tarea. e) Verificación y revisión del resultado por medio de
Richard Feynman premio Nóbel en física dijo: "usted las siguientes preguntas:
no sabe nada sobre algo hasta que lo ha practicado". ¿Las unidades coinciden?
La habilidad para resolver problemas no es sólo una ¿Es razonable el resultado?
prueba del dominio que cada cual posee de la ciencia, ¿Es apropiado el signo? ¿Tiene significado?
sino también un índice del crecimiento de nuestra Una vez que el estudiante ha desarrollado un sistema
propia capacidad como herramienta en las futuras organizado para examinar problemas y extraer la
tareas del intelecto. información relevante, tendrá confianza y seguridad
Se recomienda desarrollar las habilidades necesarias cuando tenga que resolverlos.
para resolver un amplio rango de problemas. La

PREGUNTAS Y PROBLEMAS

1. Suponga que está planeando un viaje en automóvil


a otra ciudad y estima el tiempo que se requiere para 7 ¿Qué modelo describe en la forma más sencilla las
ir allá. Demuestre cómo esta estimación depende de observaciones siguientes?
un modelo. ¿Cómo se ha descrito en el texto y qué a) Una pelota colocada en cualquier lugar sobre el
tan confiable es? piso permanece en reposo.
b) Una pelota colocada en cualquier lugar sobre el
2. Dé un ejemplo personal del uso de un modelo para piso empieza a rodar.
el análisis de los datos medidos. c) Dé otros modelos más sencillos para estas
observaciones.
3. Explique la idea básica detrás de la conversión de Respuesta.
unidades. a) Bola esférica uniforme sobre un piso horizontal.
b) Bola esférica uniforme sobre un piso inclinado.
4. Explique la diferencia en significado de las tres c) Para a) la bola tiene una parte plana o no es
cantidades 10 m, 10.0 m y 10.00 m. uniforme y para b) la bola es asimétrica y empieza a
5. ¿Cuál de los números siguientes se da con tres rodar hacia su lado más pesado.
cifras significativas: 0,003 m, 0,32 cm, 0,320 cm,
3,21 mm o 3,213 mm? 8. Se lanza un dado muchas veces con los resultados
siguientes para el número que aparece en su cara
6. Un estudiante mide un rectángulo con una regla superior: 1, 63 veces; 2, 58 veces; 3, 62 veces; 4, 63
cuya medida varía ± 1 mm. Encuentra que la altura veces; 5, 75 veces y 6, 61 veces. ¿Qué modelo puede
es 37 mm y el acho 46 mm. ¿Por qué debe informar hacer para el dado?
que el área del rectángulo Respuesta.
1700 mm2 en lugar de 1702 mm2? El dado es más pesado hacia el punto 2.

14
INTRODUCCIÓN AL CURSO Hugo Medina Guzmán

19. ¿Cuál es el área en centímetros cuadrados de un


9. Un cubo de metal flota en un líquido. ¿Cuál es el pedazo de papel de 8 pulg x 14 pulg?
modelo más sencillo del cubo y del líquido? ¿Hay Respuesta.
otros modelos? 1.25 768 cm2
Respuesta.
El cubo tal vez sea hueco si flota en el agua. 20. Los listones de madera en una cerca están
Alternativamente, el cubo es sólido pero flota en un espaciados 6,0 pulgadas, de centro a centro.
líquido que es más denso que él. ¿Cuántos listones están contenidos en un metro de
valla?
10. Un litro (L) es un volumen de 10 cm3. ¿Cuántos Respuesta.
centímetros cúbicos hay en 2,5 mililitros? 6,6
Respuesta.
2,5 cm3 21. La Luna gira sobre su eje cada 271/3 días de
modo que la misma cara está siempre hacia la Tierra.
11. ¿Qué tan lejos viaja la luz en un vacío en 1,0 ¿A cuántos grados rotará la Luna respecto a su
nanosegundos (Velocidad de la luz = 3,0 x l08 m/s.) propio eje en una hora?
Respuesta Respuesta.
30cm 0,549°

12. Los granos negros en algunos tipos de películas 22. ¿Cuántas revoluciones hace el segundero de un
fotográfica son de aproximadamente 0,8 μ m de reloj en tres años? Suponga que no hay año bisiesto
sección. Asuma que los granos tienen una sección en el intervalo.
transversal cuadrada y que todos quedan en un solo Respuesta.
plano de la película. ¿Cuántos granos se requieren 1,58 x 106 revoluciones
para oscurecer completamente 1 cm2 de película?
Respuesta. 23. La Tierra tiene una masa de 5.98 x 1024 kg y un
1,6 x 108 radio de 6,38 x 106 m. a) ¿Cuál es la masa por unidad
de volumen de la Tierra en kg/m3? b) ¿Cuál es la
13. Una fórmula se lee y = ½ at2, donde y está en masa por unidad de volumen de un núcleo de oro que
metros y t en segundos. ¿Cuáles son las dimensiones tiene una masa de 3,27 x 1025 kg y un radio de 6,98 x
de a? 10-15 m? c) ¿Cuál sería el radio de la Tierra si su
Respuesta. masa no cambiara, pero tuviera la misma masa, por
m/s2 unidad de volumen, que el núcleo de oro?
Respuesta.
14. ¿Cuál es la altura en centímetros de una persona a) 5,50 x 103 kg/m3, b) 2,30 x 1017 kg/m3, c) 184 m
cuya estatura es 5’l1’’? 24. Calcule el volumen de la tabla rectangular con
Respuesta. altura de 17,5 mm, ancho de 29,4cm y longitud 115,4
180cm cm. Recuerde la regla que se refiere a las cifras
significativas.
15. ¿Cómo es 40,2 mi expresado en kilómetros?
Respuesta
64,7 km

16. Exprese 130 km/h en términos de millas por


hora.
Respuesta. Respuesta.
80,8 mi/h 5,94 x 103 cm3

17 Una tienda anuncia un tapete que cuesta US


$18,95 por yarda cuadrada. ¿Cuánto cuesta el tapete 25. Si usted mide los lados de un cuadrado y son de
por metro cuadrado? diez centímetros con una exactitud de ±1 %, ¿cuál es
Respuesta. el área del cuadrado y cuál es la incertidumbre?
22,66 dólares/m2 Respuesta.
(100 ± 2) cm2
18. Cuando la gasolina se vende a US $1,609 por
galón, ¿cuál es el precio en dólares por litro? (1 gal = 26. Sume los números siguientes: 3,57 x 102, 2,43 x
3,l7853 L) 103 y 4,865 x 102.
Respuesta. Respuesta.
0,282 dólares/L 3,27 x 103

15
INTRODUCCIÓN AL CURSO Hugo Medina Guzmán

27. Un legajo de papel copia tiene 5,08 cm de 32. En algunos países el consumo de gasolina de un
espesor. ¿Cuál es el espesor de una sola hoja del automóvil se expresa en litros consumidos por 100
papel? Exprese su respuesta en m y mm. km de viaje. Si un automóvil logra 27 millas/galón,
Respuesta. cuál es el consumo de combustible en litros por 100
1,02 x 10-4 m o 0,102 mm km? (1 gal = 3,7853 L)
Respuesta.
28. El piso rectangular de un gimnasio tiene lados de 8,7 L/100 km
longitud de x ± Δ x por y ± Δ y donde Δ x y Δ y son
las incertidumbres estimadas en las mediciones y son 33. La velocidad del sonido a la temperatura
pequeñas comparadas con x e y. Demuestre por ambiente es 340 m/s. Exprese la velocidad del sonido
cálculo directo que el área del piso y la en unidades de millas por hora.
incertidumbre en esa área están dadas por Respuesta.
⎛ Δx Δy ⎞ cuando se ignoran términos 761 mi/h
A = xy ± xy⎜⎜ + ⎟
⎝ x y ⎟⎠
34. a) ¿Cuántos milisegundos hay en un minuto?
muy pequeños, del orden de ( Δ x)2. (En la mayor ¿Cuántos gigasegundos hay en un siglo?
parte de los casos, este resultado sobrestima la Respuesta.
incertidumbre en el área, porque no toma en a) 1 min = 60000 ms, b) 1 siglo = 3,16 Gs
consideración que las incertidumbres en las
longitudes, Δ x y Δ y, provienen de una serie de 35. a) Calcule la altura de un cilindro de radio R que
medidas, que tienen una dispersión natural en sus tiene el mismo volumen de una esfera de radio R. b)
valores.) Demuestre que el cilindro tiene un área superficial
mayor que la esfera.
29. Estime el espesor de las páginas de un libro. Dé Respuesta.
su resultado en milímetros. 4
Respuesta. h= R
Aproximadamente 0,06 mm 3

30. Alrededor de cuántos ladrillos se requieren para 36. Considere una esfera que se ajusta exactamente
construir una pared de altura hasta el hombro de 100 dentro de un cubo. ¿Cuál es la relación del volumen
pies de largo? Los ladrillos estándar tienen 8 pulg de de la esfera al volumen del cubo?
largo por 2 1/4 pulg de alto y están separados por 3/8 Respuesta.
de pulgada de mortero. π /6
Respuesta.
3,3 x 103 ladrillos 37. Un vaso cilíndrico para malteada tiene un radio
interior medido de r ± Δ r y una altura de h ± Δ h.
31. ¿Cuál es el volumen en milímetros cúbicos de un Demuestre que el volumen del vaso es
cubo de 1,00 pulg por lado?
V = πr 2 h ± 2πhΔr ± πr 2 Δh si se ignoran los
Respuesta.
términos muy pequeños del orden (Δr )
2
1,64 x 104 mm3

16
Movimiento rectilíneo Hugo Medina Guzmán

CAPITULO 2. Movimiento rectilíneo


DEFINICIÓN DE PARTÍCULA. Según la velocidad: Uniforme y uniformemente
El Punto Material variado.
Es una idealización de los cuerpos que existen en la Movimiento uniforme: La velocidad de movimiento
naturaleza y que llamamos punto material. Es un es constante
cuerpo cuyas dimensiones son despreciables al Movimiento uniformemente variado: La aceleración
compararlas con las otras dimensiones que es constante, como es el caso de los cuerpos en caída
intervienen en el movimiento. libre sometidos a la aceleración de de la gravedad.
La Mecánica comienza con el estudio de los puntos
materiales y después extiende estos estudios a los SISTEMAS DE REFERENCIA. POSICIÓN Y
sistemas de puntos materiales, incluyendo cuerpos DESPLAZAMIENTO.
rígidos y deformables.
El punto material, a diferencia de un punto El movimiento es una noción esencialmente relativa.
geométrico, está asociado a una masa inercial; esta Así resulta que el movimiento como el reposo son
propiedad está íntimamente ligada al movimiento de hechos relativos, no se puede decir que algo se
los cuerpos, como podemos ver cuando tratamos de mueve o que está en reposo sin añadir respecto a
entender cómo se mueven los cuerpos. qué. En consecuencia necesitamos un sistema de
referencia para descubrir el movimiento.
CONCEPTO DE MOVIMIENTO
El movimiento es un fenómeno físico que se define Sistemas de referencia. Desde el punto de vista
como todo cambio de posición que experimentan los estrictamente matemático, un sistema de referencia
cuerpos en el espacio, con respecto al tiempo y a un en un espacio vectorial de dimensión n está formado
punto de referencia, variando la distancia de dicho por n vectores linealmente independientes,
cuerpo con respecto a ese punto o sistema de formando una base del espacio, y por un punto,
referencia, describiendo una trayectoria. Para definido por n coordenadas, que suele llamarse
producir movimiento es necesaria una intensidad de origen del sistema de referencia.
interacción o intercambio de energía que sobrepase En el dominio de la física, el espacio suele ser la
un determinado umbral. base más habitual la llamada ortonormal ( iˆ , ĵ ,
La parte de la física que se encarga del estudio del
movimiento es la cinemática. k̂ ), y el origen se sitúa a conveniencia del
observador. Los vectores de la base son
iˆ = (1,0,0), ĵ = (0,1,0) y k̂ = (0,0,1).
CLASIFICACIÓN DEL MOVIMIENTO
Según se mueva un punto o un sólido pueden
distinguirse distintos tipos de movimiento: Atendiendo a su posible estado de reposo o
movimiento, los sistemas de referencia pueden ser
Según la trayectoria del punto: Rectilíneo y clasificados siempre y cuando hablemos de su
curvilíneo relación respecto a otro sistema de referencia que
Movimiento rectilíneo: La trayectoria que describe arbitrariamente supongamos inmóvil. En efecto,
el punto es una línea recta. debe tenerse en cuenta que cualquier sistema de
Movimiento curvilíneo: El punto describe una curva referencia está moviéndose respecto a otro (este
cambiando su dirección a medida que se desplaza. papel gira y se traslada con la Tierra alrededor del
Casos particulares del movimiento curvilíneo son la Sol, el cual a su vez se desplaza en la galaxia, que a
rotación describiendo un círculo en torno a un punto su vez se expande en el Universo...), por lo que no
fijo, y las trayectorias elípticas y parabólicas. cabe hablar de un sistema de referencia absoluto.
De acuerdo con lo anterior, un sistema de referencia
Según la trayectoria del sólido: Traslación y puede estar:
rotación. a) en reposo respecto a otro

Traslación: Todos los puntos del sólido describen
trayectorias iguales, no necesariamente rectas. b) moviéndose con velocidad constante v respecto
Rotación: Todos los puntos del sólido describen al supuestamente fijo
trayectorias circulares concéntricas. c) con una aceleración respecto al fijo.

Según la dirección del movimiento: Un buen ejemplo del primer caso podemos
Alternativo y pendular. encontrarlo en un sistema de referencia como la
Alternativo: Si la dirección del movimiento cambia, pizarra, que se encuentra en reposo relativo respecto
el movimiento descrito se denomina alternativo si es a las paredes del aula (en condiciones normales).
sobre una trayectoria rectilínea o pendular. Un ejemplo de sistema de referencia inercial
Pendular: Si lo es sobre una trayectoria circular (un podemos encontrarlo en un tren que se mueve en un
arco de circunferencia). tramo de vía rectilíneo con una velocidad
sensiblemente constante.

1
Movimiento rectilíneo Hugo Medina Guzmán

Y por último, la propia Tierra constituye un sistema una recta, circunferencia, espiral, parábola o curvas
de referencia no inercial, ya que gira con una tan complicadas como se nos ocurra.
aceleración normal, que si bien es pequeña, en La trayectoria no define el movimiento, pues no
ciertos fenómenos se observa con claridad. sabemos en que instante de tiempo ocupó cada
punto. Sabemos dónde estuvo, pero no cuando y si
Vector Posición.- Para fijar la posición de un punto estuvo varias veces en cada punto o no. Hace falta la
en el espacio respecto a un origen de coordenadas ecuación horaria.
bastan tres números que pueden ser las proyecciones Para encontrar la ecuación horaria debemos medir
sobre los ejes de un sistema cartesiano ortogonal. las distancias en función del tiempo.

En la figura P0 es un origen fijo sobre la curva (C)


que porta la trayectoria.
El vector posición del punto P es: Sea P la posición de la partícula en el instante t
→ → sobre la trayectoria definida por el arco
OP = r ∩
El movimiento quedará especificado si conocemos P0 P = S
el vector posición para cada instante, es decir: La ecuación horaria del movimiento de la partícula P
→ →
r = r (t ) es
Esto se conoce como ley de movimiento.
S = S (t )
El vector posición puede ser expresado a través de
las ecuaciones paramétricas de sus componentes en Ejemplo experimental. Estudio del movimiento de
función del tiempo: la caída libre de un cuerpo.
x = x(t ) , y = y (t ) , z = z (t ) Solución.
Si dejamos caer un objeto, obtenemos que la

trayectoria sea una recta vertical.
r = x(t )iˆ + y (t ) ˆj + z (t )kˆ Para encontrar la ley del movimiento podemos
intentar medir a partir de dónde la dejamos caer,
Desplazamiento. distancias sucesivas para diferentes tiempos.
La figura muestra una partícula que se está Una forma experimental es usando una película
moviendo a lo largo de la trayectoria curvilínea C. fotográfica y una flash electrónico que se encienda
por ejemplo cada 1/30 de segundo. En una
habitación oscura dispondremos el cuerpo, la
película y un disparador que deje caer el cuerpo y
simultáneamente accione el flash. Paralelamente a la
trayectoria a seguir por el objeto se fija una regla.

Sean P1 y P2 las posiciones de la partícula en los


instantes t1 y t 2 = t1 + Δt . Los vectores posición
correspondientes son
→ → →
OP 1 y OP 2 = r 2 = r 1 + Δ r .

Siendo Δ r el vector desplazamiento y describe el
desplazamiento de la partícula de la posición P1 a la
posición P2.

Trayectoria y Ecuación Horaria del Movimiento.-


Se llama trayectoria de una partícula en movimiento
al lugar geométrico de las posiciones efectivamente
ocupadas por la partícula en el transcurso del
tiempo. De acuerdo al tipo de movimiento podrá ser

2
Movimiento rectilíneo Hugo Medina Guzmán


r = −kt 2 kˆ
Las ecuaciones paramétricas son
x = 0 , y = 0 y z = kt 2
En esencia para cualquier movimiento debemos
ingeniarnos para obtener la ecuación horaria y
conocida su trayectoria, queda determinado el
movimiento.

VELOCIDAD Y RAPIDEZ
La fotografía mostrada permite conocer las cotas de
la foto en los diferentes instantes bien determinados. Rapidez. La rapidez (que en el lenguaje común se
La tabla muestra los resultados de la fotografía: denomina simplemente velocidad) se define como el
cociente entre la distancia recorrida y el tiempo
Tiempo Cota(m) transcurrido. La distancia s recorrida a lo largo de
0,2480 una trayectoria es una magnitud escalar,
t0 independiente de la dirección. Como el tiempo
0,3250 también es un escalar, la rapidez es también un
t1
escalar.
t2 0,4130 La rapidez se designa mediante el símbolo v y sus
0,5130 dimensiones son:
[v] = LT -1
t3
t4 0,6235
La unidad en el sistema SI es el metro por segundo
t5 0,7450 (m/s).
La figura muestra una partícula que se está
t6 0,8875 moviendo a lo largo de la trayectoria curva C. En el
t7 1,0215 instante t1 esta en P1, a una distancia S1 de un

t8 1,1760 punto P0 de referencia. En el instante t 2 está en P2 a


1,3405 una distancia S 2 del punto de referencia.
t1
t 10 1,5155

Tracemos la curva representativa del la función


z = f (t )
En el tiempo que transcurre entre t1 y t 2 ,
Δt = t 2 − t1 , la partícula ha recorrido una distancia
ΔS es la diferencia entre S 2 y S1 , esto es
ΔS = S 2 − S1 .
Se define como rapidez media dentro de este
intervalo
S 2 − S 1 ΔS
vm = =
Esta curva corresponde a una parábola y su t 2 − t1 Δt
expresión matemática es El símbolo Δ (delta) significa un incremento de una
z = kt 2 magnitud física.
⎧ z está en segundos Si la rapidez de la partícula varía a lo largo de la
⎪ trayectoria, para conocer con mejor precisión el
⎪ m movimiento debemos hacer los intervalos ΔS más
Donde ⎨k = 4,9 2
⎪ s pequeños y tomar la rapidez media de cada uno de
ellos. La figura a continuación nos muestra el
⎪⎩t está en segundos gráfico distancia recorrida versus tiempo, observen
Luego la ecuación horaria es que cuando t 2 tiende a t1 , Δ t tiende a cero.
s = kt 2 Mediante este proceso llamamos a la rapidez
Si fijamos el origen del movimiento en z = 0, la ley instantánea v en el instante t. Este proceso se
del movimiento es expresa matemáticamente como

3
Movimiento rectilíneo Hugo Medina Guzmán

ΔS dS S (t ) = Asen (ωt )
v = lim =
Δt → 0 Δt dt Solución.
En el intervalo de tiempo de t hasta t + Δt la
partícula que se mueve:
ΔS = S (t + Δt ) − S (t )
= Asenω (t + Δt ) − Asenωt
= Asenωt cos(ωΔt ) + Acosωtsen(ωΔt )
- Asenωt
La rapidez en un instante t cualquiera es
dS
La cantidad se llama “derivada de S con ΔS
dt v = lim
Δt → 0 Δt
respecto a t ” y el proceso de encontrarla se llama
= lim Asen ωt cos (ωΔt ) + Acosωtsen (ωΔt ) − Asenωt
derivación o diferenciación. La notación dS , dt , Δt → 0 Δt
expresa incrementos infinitesimalmente pequeños v = Aω cos ωt
que se conocen como diferenciales.
El proceso desarrollado en los dos ejemplos
Ejemplo 1. anteriores se hace simple con la práctica.
a) Hallar una expresión para la rapidez de una Hay muchas reglas o fórmulas para derivar
partícula que se mueve de acuerdo a la ley horaria diferentes tipos de funciones. Estas pueden
S = At 2 memorizarse o encontrarse en tablas. La tabla
2
b) Si A = 1,4 m/s , hallar la distancia a la que se siguiente es una pequeña muestra de estas.
encuentra la partícula y su rapidez para 10 segundos
después de iniciado su movimiento. Derivadas de algunas funciones
Solución.
Función Derivada
a) Si en el tiempo t está en S(t ) :
S =t n
dS
= nt n −1
S(t ) = At 2 dt
Transcurrido un tiempo Δt , la partícula estará en S =c dS
S(t + Δt ) =0
dt
S(t + Δt ) = A(t + Δt ) S = cu
2
dS du
=c
At 2 + 2 At Δt + A(Δt ) ,
2 dt dt
=
S =u+v dS du dv
Como = +
ΔS = S(t + Δt ) − St dt dt dt
S = uv dS du dv
= At 2 + 2 At Δt + A(Δt ) − At 2 =v +u
dt dt dt
= 2 At Δt + A(Δt )
2
S = Asenωt dS
La rapidez en el instante t es: = Aωcosωt
dt
ΔS
v(t ) = lim S = Acosωt dS
Δt → 0 Δt = − Aωsenωt
2 AtΔt + A(Δt )
2 dt
= lim = 2 At
Δt →0 Δt
b) Para t = 10 es Ejemplo 3. Hallar una expresión para la rapidez de
una partícula que se mueve de acuerdo a la ley
⎛ m⎞
S(10 ) = ⎜1,4 2 ⎟(10s ) = 140 m
2
horaria S = At , usando fórmulas de la tabla
2

⎝ s ⎠ anterior.
y su rapidez es Solución.
⎛ m⎞ m Tenemos que:
v(10 ) = 2⎜1,4 2 ⎟(10s ) = 28 2
( )
2

⎝ s ⎠ s dS d At 2 dt 2
v= = =A = 2 At
dt dt dt
Ejemplo 2. Hallar una expresión para la rapidez de
una partícula que se mueve según la ecuación Ejemplo 4. Hallar una expresión para la rapidez de
horaria una partícula que se mueve de acuerdo a la ley

4
Movimiento rectilíneo Hugo Medina Guzmán

horaria S (t ) = Asen (ωt ) , usando fórmulas de la →


La magnitud del vector velocidad instantánea, v ,
tabla anterior. r
Solución. es decir v o simplemente v es igual a la rapidez
Tenemos que instantánea en ese punto.
dS d ( Asenωt ) dsenωt La velocidad es la pendiente del gráfico de x versus
v= = =A = Aω cos ωt
dt dt dt t, como se muestra en la figura.

Velocidad. La velocidad (que más apropiadamente


sería vector velocidad), a diferencia de la rapidez
debemos incluir el concepto de dirección en nuestro
estudio; para esto debemos emplear vectores.
La figura muestra una partícula que se está
moviendo a lo largo de la trayectoria curvilínea C.

Cuando la pendiente es positiva, el objeto se está


moviendo a la derecha.
Cuando la pendiente es negativa, el objeto se está
moviendo a la izquierda.
Cuando la pendiente es cero, el objeto se detiene.

Sean P1 y P2 las posiciones de la partícula en los Ejemplo 5. Entre dos observadores hay una
instantes t1 y t 2 = t1 + Δt . Los vectores posición distancia de 1050 m, uno de ellos dispara un arma de
→ fuego y el otro cuenta el tiempo que transcurre desde
correspondientes son OP1 = r 1 y que ve el fogonazo hasta que oye el sonido,
→ → → → obteniendo un valor de 3 s. Despreciando el tiempo
OP 2 = r 2 = r 1 + Δ r . Siendo Δ r el vector empleado por la luz en hacer tal recorrido, calcular
desplazamiento y describe el desplazamiento de la la velocidad de propagación del sonido.
partícula de la posición P 1 a la posición P 2 .

Velocidad media. El cociente entre el vector



desplazamiento Δ r y el intervalo de tiempo Δt es
el vector velocidad media. Solución.
→ La velocidad es:
→ Δr c = s/t = 1050/3 = 350 m/s
vm =
Δt
Como el desplazamiento es un vector y el tiempo es Ejemplo 6. Nos encontramos en una batalla naval,
un escalar positivo, la velocidad es una magnitud en un buque situado entre el enemigo y los
vectorial que tiene la misma dirección y sentido que acantilados de la costa. A los 3 s de ver un fogonazo
el desplazamiento. Esto significa que si una oímos el disparo del cañón, y a los 11 s del fogonazo
partícula sufre un desplazamiento negativo, su percibimos el eco. Calcular la distancia a que están
velocidad será también negativa. de nosotros el enemigo y la costa. Velocidad del
sonido, 340 m/s.
Velocidad instantánea. Como en el caso de la

rapidez obtendremos la velocidad instantánea v
tomando la velocidad media en un intervalo de
tiempo cada vez menor Δt medido desde un cierto
tiempo t1 . En el límite, cuando Δt tiende a cero:
→ → → → Solución.

r 2− r1 Δr d r
v (t1 ) = lim
Despreciando el tiempo empleado por la luz en su
= lim = recorrido, la distancia a que se encuentra el enemigo
t 2 →t1 Δt Δt →0 Δt dt
es:
S = 340 x 3 = 1020 m
La dirección de este vector es la dirección límite del
El sonido emplea para ir y volver a la costa, desde
vector cuando Δt → 0 de la figura anterior. Es nuestra posición, un tiempo que es:

t = 11 - 3 = 8 s ⇒ 2S’= 340 x 8 ⇒ S’ = 1360 m
evidente que en este límite la dirección de Δ r es la
de la tangente la trayectoria en P1. La costa está a 1020 + 1360 = 2380m.

5
Movimiento rectilíneo Hugo Medina Guzmán

Δx dx
Ejemplo 7. La posición de una partícula en b) de la ecuación v x = lim = , la velocidad
Δt →0 Δt dt
coordenadas cartesianas está dada por la ecuación
→ instantánea en función del tiempo es
r (t ) = x(t )iˆ + y (t ) ˆj + z (t )kˆ v x = 2bt − 3ct 2 = (4,80)t − (0,360)t 2
donde tal que
x(t ) = 5 + 6t 2 , y (t ) = 3t , z (t ) = 6 i) v x (0) = 0,
t en segundos, x, y, z en metros.
ii) v x (5) = (4,80 )(5) − (0,360)(5) = 15,0 y
2
a) Determinar el desplazamiento entre t = 0 y t = 1 s.
b) Determinar la velocidad media iii) v x (10) = ( 4,80)(10) − (0,360)(10) = 12,0
2

c) Determinar la velocidad y la rapidez para t = 1 s.


Solución. c) el auto está en reposo cuando v x = 0 .
a) para t = 0 s, x = 5m, y = 0m, z = 6m
Por consiguiente ( 4,80)t − (0,360)t = 0 .
2

r 0 = 5iˆ + 6kˆ El único tiempo después de t = 0 en que el auto se
Para t = 1s, x = 11m, y =3m, z = 6m 4,8
→ encuentra en reposo es t = = 13,3 s
r 1 = 11iˆ + 3 ˆj + 6kˆ 0,360
El desplazamiento es
→ → → Ejemplo 9. Un ciclista marcha por una región
Δ r = r 1 − r 0 = (11 − 5)iˆ + (3 − 0) ˆj + (6 − 6 )kˆ donde hay muchas subidas y bajadas
En las cuestas arriba lleva una rapidez constante de 5
= 6iˆ + 3 ˆj
km/h y en las cuestas abajo 20 km/h. Calcular:
b) la velocidad media es a) ¿Cuál es su rapidez media si las subidas y bajadas

tienen la misma longitud?
→ Δ r 6iˆ + 3 ˆj
vm = = = 6iˆ + 3 ˆj b) ¿Cuál es su rapidez media si emplea el mismo
Δt 1− 0 tiempo en las subidas que en las bajadas?
c) la velocidad instantánea es c) ¿Cuál es su rapidez media si emplea doble tiempo

[( ) ]
→ en las subidas que en las bajadas?
→ d r d 5 + 6t 2 iˆ + 3tˆj + 6kˆ Solución.
v= =
dt dt stotal ssubida + sbajada
a) vm = =
= 12ti + 3 j
ˆ ˆ ttotal ttotal
La magnitud de v es 2v1v 2
2s
v = 122 + 32 = 153 = = = 8 km / h
s s v1 + v 2
= 12,4 m/s +
Valor que corresponde a la rapidez instantánea para
v1 v 2
t = 1s. v t + v2t v1 + v2
b) vm = 1 =
2t 2
Ejemplo 8. Un auto está parado ante un semáforo.
= 12,5 km/h
Después viaja en línea recta y su distancia respecto
al semáforo está dada por x(t) = bt2 - ct3 , donde b = v1 2t + v2t 2v1 + v2 2 × 5 + 20
c) vm = = =
2,40 m/s2 y c = 0,120 m/s3. 3t 3 3
a) Calcule la velocidad media del auto entre t = 0 y = 10 km/h
t = 10,0 s. (Obsérvese que la rapidez media es la media
b) Calcule la velocidad instantánea en aritmética de las rapideces uniformes únicamente en
i) t = 0; ii) t = 5,0 s; iii) t = 10,0 s. el caso de que el tiempo que duran los distintos
c) ¿Cuánto tiempo después de arrancar vuelve a recorridos sea el mismo).
estar parado el auto?
Solución. ACELERACIÓN
a) En t1 = 0, x1 = 0 , tal que la ecuación En el lenguaje ordinario el término aceleración se
refiere sólo a incrementos del módulo de la
x 2 − x1 Δx
vm = = ⇒ velocidad (rapidez), pero en Física se utiliza con un
t 2 − t1 Δt sentido más amplio para designar un cambio del
vector velocidad. En Física se dice que un cuerpo
x2 (2,4)(10) − (0,120 )(10)
2 3
está siendo acelerado no sólo cuando aumenta su
vm = =
t2 (10) velocidad sino también cuando disminuye o cambia
de dirección.
= 12,0 m/s
Se llama aceleración al cambio de la velocidad
(vector velocidad) en el tiempo.

6
Movimiento rectilíneo Hugo Medina Guzmán

Observe que la aceleración negativa no significa


Aceleración Media. necesariamente “bajar la velocidad”.
Cuando la velocidad y la aceleración ambas tienen el
mismo signo, el objeto aumenta su velocidad.
Cuando la velocidad y la aceleración tienen signos
opuestos, el objeto disminuye su velocidad.
Los gráficos de la figura siguiente ilustran el
desplazamiento, la velocidad, y la aceleración para
un objeto en movimiento.

La razón en la cual la velocidad cambia se mide por


la aceleración. Así si un objeto tiene la velocidad
→ →
v 1 en el t1 del tiempo y velocidad v 2 en el t 2 , su
aceleración media es
→ → →
→ v 2 − v 1 Δ v Δv ˆ
am = = = i
t 2 − t1 Δt Δt
Supongamos que una partícula que se mueve en la
trayectoria C de la figura anterior en el instante t1
está en P1 con una velocidad v1 y en el instante
t 2 = t1 + Δt está en P2 con una velocidad v 2 . Por
definición el vector aceleración media de la partícula Ejemplo 10. Una partícula se mueve a lo largo de
una línea curva
entre los instantes es t1 y t 2 es
( ) ( )

→ → → r (t ) = t 2 + t iˆ + (2t − 1) ˆj + t 3 − 2t 2 kˆ
→ v 2 − v1 Δ v
am = = Encontrar:
t2 − t1 Δt a) La velocidad para t = 1 s y para t = 3 s .
[]
Las dimensiones de la aceleración son a = LT
−2 b) La aceleración media entre t = 1 s y para t = 3 s .
c) La aceleración y su magnitud para t = 1 s .
La unidad de la aceleración en el sistema SI está en Solución.
metros / segundo por segundo: a) Las ecuaciones paramétricas son:
ms m
= 2 x(t ) = t 2 + 1 , y (t ) = 2t − 1 , z (t ) = t 3 − 2t 2
s s Las componentes de la velocidad son:
dx dy
Aceleración Instantánea o simplemente vx = = 2t + 1 , v y = = 2.
aceleración. Cuando t 2 → t1 o Δt → 0 dt dt
llegaremos al valor de la aceleración en el instante dz
vz = = 3t 2 + 4t
t1 . Este proceso para el límite se expresa dt
matemáticamente como La velocidad es:

( )

v (t ) = (2t + 1)iˆ + 2 ˆj + 3t 2 − 4t kˆ
→ → → →
→ v 2 − v1 Δv d v
a (t1 ) = lim = lim = →
t 2 →t1 t − t Δt →0 Δt
2 1 dt Para t = 1 s : v (1) = 3iˆ + 2 ˆj − kˆ



Como v =
dr
, tenemos: Para t = 3 s . v (3 ) = 7iˆ + 2 ˆj + 15kˆ
dt b) La aceleración media entre t = 1 s y t = 3 s .

⎛ →⎞ → → → →
→ d v d ⎜d r ⎟ d2 r → Δ v v (3) − v (1)
a= = ⎜ = am = = =
dt dt ⎜ dt ⎟⎟ dt 2 Δt 3 −1
⎝ ⎠
Es mejor evitar el uso de la palabra común (7 − 3)iˆ + (2 − 2) ˆj + (15 + 1)kˆ
“desaceleración.” 2
Describa la aceleración simplemente como positiva →
o negativa. a m = 2iˆ + 8kˆ

7
Movimiento rectilíneo Hugo Medina Guzmán

c) la aceleración instantánea es →

→ Δ r Δx ˆ x − x0 ˆ
v = viˆ = = i=

a (t ) =
dv d
dt
=
dt
[ (
(2t + 1)iˆ + 2 ˆj + 3t 2 − 4t kˆ )] Δt Δt t − t0
i

Δx x − x0
= 2iˆ + (6t− 4 )kˆ v= = = tan α
Δt t − t0
para t = 1s

a (1) = 2iˆ + 2kˆ Diagrama velocidad-tiempo
la magnitud de la aceleración es
a(1) = 2 2 + 2 2 = 2 2 m s 2

Ejemplo 11. Una persona que se asoma por la


ventana de un edificio alto de oficinas observa lo
que sospecha es un ovni. La persona registra la
posición del objeto en función del tiempo y El gráfico velocidad-tiempo del movimiento
uniforme es una recta paralela al eje del tiempo.
determina que está dada por
( )

r (t ) = −5,0t iˆ + 10,0t ˆj + 7,0t − 3,0t 2 kˆ x − x0
a) Obtenga los vectores de: desplazamiento, De v = ⇒ x = x 0 + v(t − t 0 )
velocidad y aceleración del objeto en t = 5,0 s.
t − t0
b) ¿Hay algún tiempo en que la velocidad del objeto Si el instante inicial t 0 = 0 , tenemos
sea cero?
c) ¿La aceleración del objeto es constante o cambia x = x0 + vt
con el tiempo?
Solución. Diagrama espacio-tiempo
a) El vector desplazamiento es:
( )

r (t ) = −5,0t iˆ + 10,0t ˆj + 7,0t − 3,0t 2 kˆ
El vector velocidad es la derivada del vector
desplazamiento:

d r (t )
= −5,0iˆ + 10,0 ˆj + [7,0 − 2(3,0)t ]kˆ
dt
y el vector aceleración es la derivada del vector El gráfico indica las posiciones instantáneas del
velocidad: móvil en cada instante

d 2 r (t )
= −6,0 kˆ
dt 2 MOVIMIENTO RECTILÍNEO
en t = 5,0 s: UNIFORMEMENTE VARIADO.

[
r (5 ) = −5,0(5)iˆ + 10,0(5) ˆj + 7,0(5) − 3,0(5) kˆ
2
] Para que un movimiento sea rectilíneo
= − 25,0iˆ + 50,0 ˆj − 40,0kˆ uniformemente variado su aceleración debe ser
→ constante y diferente de cero.
d 2 r (5)
= −6,0 kˆ
dt 2 Estudio del Movimiento
→ →
b) la velocidad en ambas direcciones x e y es Como la aceleración es constante, a m =a
constante y diferente de cero, luego la velocidad dv Δv
nunca puede ser cero a= = = constante
c) La aceleración del objeto es constante, ya que t no dt Δt

aparece en el vector aceleración. → Δ v Δv ˆ
a = aiˆ = = i
MOVIMIENTO RECTILÍNEO UNIFORME. Δt Δt
Para que un movimiento sea rectilíneo uniforme su Δv v − v0
velocidad debe ser constante, es decir, que la a= =
aceleración sea siempre igual a cero. Δt t − t 0
v − v 0 = a(t − t 0 )
Estudio del Movimiento
→ → Si el tiempo inicial t 0 =0
Como el movimiento es uniforme v m = v , y
considerando que su trayectoria está en el eje x
v = v0 + at

8
Movimiento rectilíneo Hugo Medina Guzmán

Diagrama velocidad-tiempo

Δv v − v0
a= = = tan α Ejemplo 12. Demostrar que el área encerrada bajo la
Δt t − t 0 curva de la velocidad del diagrama velocidad-tiempo
La velocidad media: es igual al módulo del desplazamiento

Si la posición en t 0 es r 0 = x 0 iˆ y la posición en t
Δx = x − x 0 .

es r = xiˆ , la velocidad media en este intervalo es
Δx x − x0
vm = =
Δt t − t0

La posición.
De lo anterior:
x − x0 = vm (t − t0 )
y x = x0 + vm (t − t0 ) Solución.
El área encerrada es igual al área de un trapecio
Por otra parte como la velocidad es una función
cuyas bases son b1 = v y b2 = v 0 con altura
lineal, la velocidad media v m es
h = (t − t 0 ) .
v + v0
vm =
2 (b1 + b2 )
Area del trapecio = h
y como v = v 0 + a(t − t 0 ) 2
(v + v0 )
resulta = (t − t 0 )
v 0 + [v 0 + a (t − t 0 )] a(t − t 0 ) 2
vm = = v0 +
1
2 2 = v 0 (t − t 0 ) + (v − v 0 )(t − t 0 )
finalmente 2
⎡ a(t − t 0 ) ⎤ (v − v0 )
x = x 0 + ⎢v 0 + (t − t 0 ) Pero como a = tan α =
⎣ 2 ⎥⎦ (t − t 0 )
1 ⇒ (v − v0 ) = a(t − t 0 )
⇒ x = x 0 + v0 (t − t 0 ) + a(t − t 0 )
2

2 Luego
1
Area del trapecio = v0 (t − t 0 ) + a(t − t 0 )
2
Si el tiempo inicial t 0 =0 2
1 2 Valor que precisamente corresponde al
x = x0 + v0 t + at desplazamiento Δx = x − x 0 .
2
Diagrama espacio -tiempo
LA ECUACIÓN DE TORRICELLI.
Podemos obtener una relación muy útil eliminando
el tiempo como variable en la ecuación
1
x = x 0 + v0 (t − t 0 ) + a(t − t 0 )
2

2
(v − v0 ) (v − v0 )
Como a = ⇒ (t − t 0 ) =
(t − t 0 ) a

9
Movimiento rectilíneo Hugo Medina Guzmán

Sustituyendo
(v − v0 ) 1 (v − v0 )
2

x = x0 + v0 + a
a 2 a2
De donde se puede despejar: Los diagramas aceleración-tiempo, velocidad-
tiempo y espacio-tiempo correspondientes son los
v = v + 2a( x − x 0 )
2 2
0 siguientes:
Conocida como la ecuación de Torricheli.

Descripción del movimiento de una partícula con


aceleración constante.

Consideramos una aceleración constante a > 0 en a 0 = constante


el sentido positivo de la trayectoria.

1er Caso:
La partícula tiene una velocidad inicial v0 ≥ 0.
La partícula se desplaza de P 0 al infinito con un
sentido constante y aumentando su velocidad.

v = v 0 + at

Los diagramas aceleración-tiempo, velocidad-


tiempo y espacio-tiempo correspondientes son los
siguientes:

1
x = x0 + v0 t + a0 t 2
a 0 = constante 2
Ejemplo 13. Una tortuga camina en línea recta
sobre lo que llamaremos eje x con la dirección
positiva hacia la derecha. La ecuación de la posición
de la tortuga en función del tiempo es
x(t) = 50,0 cm + (2,00 cm/s)t - (0,0625 cm/s2)t2 .
v = v 0 + at a) Determine la velocidad inicial, posición inicial y
aceleración inicial de la tortuga.
b) ¿En qué instante t la tortuga tiene velocidad cero?
c) ¿Cuánto tiempo después de ponerse en marcha
regresa la tortuga al punto de partida?
d) ¿En qué instantes t la tortuga está a una distancia
de 10,0 m de su punto de partida? ¿Que velocidad
(magnitud y dirección) tiene la tortuga en cada uno
1 de esos instantes?
x = x0 + v0 t + a0 t 2 e) Dibuje las gráficas: x-t, vx-t y ax-t para el intervalo
2 de t = 0 a t = 40,0 s.
2do. Caso: Solución.
La partícula tiene una velocidad inicial v 0 < 0. dx
vx = = 2,00 cm s − (0,125 cm s 2 )t
La partícula se desplaza de P 0 en sentido negativo dt
con movimiento retardado (desacelerado) hasta dv
ax = x = −0,125 cm s 2
detenerse en P 1 y cambia de sentido. A partir de ese dt
instante la velocidad aumenta constantemente a) En t = 0, x = 50,0 cm, v x = 2,00 cm s ,
(acelerado) y se desplaza al infinito con un sentido
constante. a x = −0,125 cm s 2 .
b) Hagamos vx = 0 y resolvamos para t:
t = 16,0 s

10
Movimiento rectilíneo Hugo Medina Guzmán

c) Hagamos x = 50,0 cm y resolvamos para t. ⇒ t2 +10t -11 = 0 ⇒ t = 1 s


Esto da: t = 0 y t = 32,0 s . v D = vC + at = 50 + 10 = 60 cm/s
La tortuga regresa al punto de partida después de
32,0 s. b) v B = 2aAB
d) La tortuga está a 10,0 cm del punto de partida vB2 400
cuando x = 60,0 cm o x = 40,0 cm. ⇒ AB = = = 20cm
Hagamos x = 60,0 cm y resolvamos para t: 2a 20
t = 6,20 s y t = 25,8 s c)
En t = 6,20 s, vx = + 1,23 cm/s. v B = at ⎫ 20
En t = 25,8 s, vx = - 1,23 cm/s. ⎬ ⇒ t= =2 s
Hagamos x = 40,0 cm y resolvamos para t :
20 = 10t ⎭ 10
d) Será la suma de los tiempos parciales:
t = 36,4 s t = 2 + 3 +1 = 6 s
(la otra raíz de la ecuación cuadrática es negativa y
por lo tanto sin significado físico). MOVIMIENTO VERTICAL CON
En t = 36,4 s, vx = - 2,55 cm/s. ACELERACIÓN DE LA GRAVEDAD.
e)
La variación de la magnitud de la aceleración g φ
debido a la gravedad en la superficie de la tierra con
la latitud está dada por la fórmula internacional de la
gravedad adoptada en 1930 por el Congreso
Geofísico Internacional:
g φ = 978,049000 (1 + 0,0052884 sen 2 φ
- 0,0000059 sen 2 φ )
2

g en cm/s ,
2
φ en grados
Donde φ es la latitud de la tierra medida en el
ecuador
Para φ = 0º (ecuador), g 0 = 978,0490
Ejemplo 14. Un móvil parte del reposo y de un
punto A, con movimiento rectilíneo y Para φ = 90º (polos), g 90 = 983,2213
uniformemente acelerado (a =10 cm/s2); tarda en
recorrer una distancia BC = 105 cm un tiempo de 3 La variación de la aceleración gravitacional con la
s, y, finalmente, llega al punto D (CD = 55 cm). altura sobre el nivel del mar es aproximadamente
Calcular:
a) La velocidad del móvil en los puntos B, C y D. g = g φ − 0,000002860h
b) La distancia AB. h en metros y g φ en m/s
2
c) El tiempo invertido en el recorrido AB y en el
CD. Donde h ≤ 40 000 m
d) El tiempo total en el recorrido AD.
Cerca de la superficie de la tierra la magnitud de la
aceleración debido a la gravedad varía muy poco
con la altura y en los cálculos técnicos ordinarios se
toma g = 9,81 m/s2 (dirigido verticalmente hacia
abajo).
Solución.
a) Un cuerpo que se deja caer está sometido a la
1 2 ⎫ aceleración de la gravedad y su movimiento
BC = v B t + at ⎪⎪ corresponde a un movimiento rectilíneo
2
⎬ ⇒ vB =20 cm/s uniformemente variado en el eje vertical
1 2⎪ perpendicular a la tierra,
105 = v B 3 + 10 × 3 ⎪
2 ⎭ 1 2
y = y 0 + v0 t + at
vC = v B + at = 20 + 30 = 50 cm/s 2
v = v 0 + at
1 ⎫
CD = vC t + at 2 ⎪ a = −g
2 ⎪

1 2⎪ a) Caída libre
55 = 50t + 10t ⎪
2 ⎭

11
Movimiento rectilíneo Hugo Medina Guzmán

1 2v
h = h + v0 t + − gt 2 ⇒ t P = 0 y por
2 g
supuesto t P = 0 , que corresponde al tiempo inicial.
Observamos que t P = 2t m
La velocidad es
⎛ 2v ⎞
v P = v0 − g ⎜⎜ 0 ⎟⎟ = v0 − 2v0 = −v0
Si se deja caer un cuerpo desde una altura h sobre el ⎝ g ⎠
nivel del piso y consideramos despreciable la Finalmente toca piso cuando y = 0
resistencia del aire.
En este caso y 0 = h , v 0 = 0 , luego: 1 2v 2h
h + v 0 t − gt 2 = 0 ⇒ t 2 − 0 t − =0
1 2 2 g g
y =h− gt cuya solución es
2
v = − gt v0 v02 + 2h
a = −g t= ±
g g
El cuerpo toca tierra cuando y = 0 toca el piso al tiempo
1 2 2h v02 + 2h
Luego h − gt = 0 ⇒ t = v0
2 g t= +
g g
y la velocidad es v = 2 gh con una velocidad
v = − v02 + 2 gh
b) Lanzamiento hacia arriba
Si el mismo cuerpo desde la misma altura h se lanza
c) Lanzamiento hacia abajo
hacia arriba con velocidad v 0 , se mueve con un Si el mismo cuerpo desde la misma altura h se lanza
movimiento rectilíneo uniformemente retardado hacia abajo con una velocidad v 0 , el cuerpo se
(desacelerado).
mueve en un movimiento rectilíneo uniformemente
acelerado.

1
y = h + v 0 t + − gt 2
2
1 2
v = v0 − gt y = h − v0 t − gt
a = −g 2
El cuerpo sube hasta que alcanza la altura máxima v = −v0 − gt
y m . Esta corresponde a cuando la velocidad a = −g
disminuye a cero. y = 0.
El cuerpo alcanza el piso cuando
v0 1 2v 2h
v0 − gt = 0 ⇒ t m = h − v0 t − gt 2 = 0 ⇒ t 2 + 0 t − =0
g 2 g g
2 cuya solución es
⎛v ⎞ 1 ⎛v ⎞
De aquí y m = h + v0 ⎜⎜ 0 ⎟⎟ + + g ⎜⎜ 0 ⎟⎟ v02 + 2h
⎝g ⎠ 2 ⎝g ⎠ v0
t=− ±
v02 g g
= h+ toca el piso al tiempo
2
Cuando el cuerpo pasa por el punto de lanzamiento v0 v02 + 2h
y=h t=− +
g g
con una velocidad

12
Movimiento rectilíneo Hugo Medina Guzmán

y2 = h - 30(t - 2) - 5(t - 2)2 = 0


v = − v02 + 2 gh De aquí t = 4 s;
h = 80m
Ejemplo 15. Desde lo alto de un edificio, se lanza
verticalmente hacia arriba una pelota con una Ejemplo 19. Desde el piso, se lanza hacia arriba una
rapidez de 12,5 m/s. La pelota llega a tierra 4,25 s, pelota con una rapidez de 40 m/s. Calcule:
después. Determine: a) El tiempo transcurrido entre los dos instantes en
a) La altura que alcanzó la pelota respecto del que su velocidad tiene una magnitud de 2,5 m/s.
edificio. b) La distancia respecto al piso que se encuentra la
b) La rapidez de la pelota al llegar al suelo. pelota en ese instante.
Solución. Solución.
La altura en función del tiempo será 1 2
1 y = v0t − gt (1)
y = h + v0 t − gt 2 2
2 v y = v0 − gt ( 2)
Con g = 10m/s2, v0 = 12,5 m/s
y = h + 12,5t - 5t2
a) Al tiempo t = 4,25 s, y = 0, luego: a) De la ecuación (2):
h + 12,5(4,25) - 5(4,25)2 = 0, v y = v0 − gt1 = 2,5
⇒ h = 37,19 m
b) vy = 12,5 - 10t = 12,5 - 10(4,25) v y = v0 − gt 2 = −2,5
= -30,0 m/s Restando obtenemos:
5
Ejemplo 16. Se deja caer un cuerpo desde una altura Δt = t2 − t1 = = 0,5s
de y0 = 33 m, y simultáneamente se lanza hacia g
abajo otro cuerpo con una rapidez inicial de 1 m/s. b) De la ecuación (2):
Encontrar el instante en que la distancia entre ellos v y = v0 − gt1 = 2,5
es de 18 m.
Solución. 40 − gt1 = 2,5
y1 = 33 - 5t2 37,5
y2 = 33 - t - 5t2 ⇒ t1 = = 3,83 s.
y1 - y2 = t 9,8
Entonces la distancia entre ellos es 18m a los 18 s Con t1 en (1):
1
h = 40(3,83) − g (3,83) = 81,41 m.
2
Ejemplo 17. Un cuerpo que cae, recorre en el último
segundo 68,3 m. Encontrar La altura desde donde
2
Con t2 se obtiene la misma altura, porque es cuando
cae.
la pelota está de bajada.
Solución. Suponiendo que se soltó del reposo
y = h - 5t2
Ejemplo 20. Una roca cae libremente recorriendo la
h segunda mitad de la distancia de caída en 3(s).
El tiempo en que llega al suelo es t= Encuentre
5
La distancia recorrida en el último segundo será a) la altura desde la cual se soltó.
b) El tiempo total de caída.
⎛ h ⎞ ⎛ h⎞
y⎜⎜ − 1⎟⎟ − y⎜⎜ ⎟ = Solución.
⎟ 1 2
⎝ 5 ⎠ ⎝ 5⎠ y =h− gt
2 2 2
⎛ h⎞ ⎛ h ⎞
5⎜⎜ ⎟ − 5⎜
⎟ ⎜ 5 − 1 ⎟ = 68,2
⎟ h
⎝ 5 ⎠ ⎝ ⎠ El tiempo en que alcanza h/2 es t1 = y el
g
⇒ h = 268,6 m
2h
Ejemplo 18. Desde lo alto de un acantilado, se deja
tiempo en que h = 0 es t2 =
g
caer una piedra, desde la misma altura se lanza una
segunda piedra 2 s más tarde con una rapidez de 30 a) por lo tanto el tiempo empleado en la segunda
m/s. Si ambas golpean el piso simultáneamente. parte de recorrido es
Encuentre: La altura del acantilado. 2h h
Solución. − = 3 ⇒ h = 524,6 m
g g
y1 = h - 5t2
y2 = h - 30(t - 2) - 5(t - 2)2 2h 524,6
Siendo al mismo tiempo b) t= = = 10,2 s
y1 = h - 5t2 = 0
g 5

13
Movimiento rectilíneo Hugo Medina Guzmán

Ejemplo 21. Se deja caer una piedra desde un globo


que asciende con una velocidad de 3 m/s; si llega al
suelo a los 3 s, calcular:
a) Altura a que se encontraba el globo cuando se
soltó la piedra.
b) Distancia globo-piedra a los 2 s del lanzamiento.

Solución.
Primer método:
En el instante en que empieza a caer el cuerpo el
ascensor lleva una velocidad vertical hacia arriba v.

Solución. Tomaremos el origen de coordenadas en El espacio vertical y hacia abajo que debe recorrer la
el punto en que se suelta la piedra. Magnitudes lámpara es:
positivas son las que tienen direcci6n hacia arriba. ⎛ 1 ⎞
a) h − ⎜ vt + at 2 ⎟
⎝ 2 ⎠
v0 = 3m/s ⎫ (h = altura del ascensor) y (vt + at2/2) ascenso del
⎪ 1
g ≈ 10m/s 2 ⎬ y = h + 3t − 10t 2 suelo de éste. La lámpara al desprenderse lleva una
⎪ 2 velocidad inicial hacia arriba v. Aplicando la
t = 3s ⎭ ecuación:
Cuando la piedra toca suelo, y = 0 1 2
Luego s = vt + at
2
1
h = 3(3) − 10(3) Siendo positivas las magnitudes hacia arriba y
2

2 negativas las descendentes, tendremos:


= 36 m 1 2 1
b) − h + vt + at = vt − gt 2 ⇒
2 2
t’ = 2 s.
2h 2×3
t= = = 0,74 s
h1: distancia al origen del globo en t'. g+a 9,8 + 1
h2: distancia al origen de la piedra en t'.
Segundo método:
h1 = v 0 t ' = 3 × 2 = 6m ⎫
⎪ La aceleración de la lámpara respecto al ascensor,
1 1 ⎬ considerando magnitudes positivas hacia abajo, es:
h2 = v 0 t '+ gt ' 2 = 3 × 2 − 10 × 40 − 14m ⎪
2 2 ⎭ aBA = aB - aA = 9,8 – (-1) = 10, 8 m/s2
⇒ d = 6 + 14 1
= 20 m h= a BA t 2 ⇒
2
Ejemplo 22. La cabina de un ascensor de altura 3 m 2h 2×3
asciende con una aceleración de 1 m/s2. Cuando el t= = = 0,74 s
a BA 10,8
ascensor se encuentra a una cierta altura del suelo, se
desprende la lámpara del techo. Calcular el tiempo
que tarda la lámpara en chocar con el suelo del Ejemplo 23. Una bola es lanzada verticalmente
ascensor. hacia arriba con una velocidad de 20 m/s de la parte
alta de una torre que tiene una altura de 50 m. En su
vuelta pasa rozando la torre y finalmente toca la
tierra.
a) ¿Qué tiempo t1 transcurre a partir del instante en
que la bola fue lanzada hasta que pasa por el borde
de la torre? ¿Qué velocidad v1 tiene en este tiempo?
b) ¿Qué tiempo total t 2 se requiere para que la bola
llegue al piso? ¿Cuál es la velocidad v 2 , con la que
toca el piso?

14
Movimiento rectilíneo Hugo Medina Guzmán

c) ¿Cuál es la máxima altura sobre el suelo Ejemplo 24. Una maceta con flores cae del borde de
alcanzada por la bola? una ventana y pasa frente a la ventana de abajo. Se
d) Los puntos P1 y P2 están a 15 y 30 m, puede despreciar la resistencia del aire. La maceta
respectivamente, por debajo del techo de la torre. tarda 0,420 s en pasar por esta ventana, cuya altura
¿Qué tiempo se requiere para que la bola viaje de P1 es de 1,90 m. ¿A qué distancia debajo del punto
a P2? desde el cual cayó la maceta está el borde superior
e) ¿Se desea que después de pasar el borde, la bola de la ventana de abajo?
alcance la tierra en 3s, ¿con qué velocidad se debe Solución.
lanzar hacia arriba de la azotea? Si la velocidad de la maceta en la parte superior de
la ventana es v 0 , podemos encontrarla en función
de la altura h de la ventana y el tiempo que tarda en
pasarla::
1 2 2h − gt 2
h = v0 t + gt ⇒ v 0 =
2 2t
2(1,90 ) − (9,8)(0,42)
2
m
Luego: v 0 = = 2,47
2(0,42) s
La distancia y desde la azotea al borde superior de
la ventana es:
Solución. v02 2,47 2
y= = = 0,311 m
2 g 2(9,8)
a) Para el sistema de coordenadas mostrado en la
1 2
figura, y = v 0 t + at . Otra forma de encontrar la distancia es: como
2 t = 0,420 s es la diferencia entre los tiempos
Pero en el borde del techo y = 0, luego tomados en caer la las alturas ( y + h ) e y ,
1 2 tenemos
0 = v0 t1 + at1 ,
2 2( y + h) 2y
De la cual t1 = 0, indica el instante en el cual la bola t= − ⇒
es lanzada, y también t1 = 4,08 s, la cual es el g g
tiempo en que la bola retorna al borde.
gt 2
Luego, de v = v 0 + at + y= y+h
2
v1 = 20 + (− 9,8)(4,08) = −20m / s , que es el Elevando al cuadrado:
negativo de la velocidad inicial.
gt 2
1 + 2 gyt 2 + y = y + h
b) − 50 = 20t2 + (− 9,8)t22 ⇒ t2 = 5,8 s 2
2
v2 = 20 + (− 9,8)(5,8) = −37 m / s gt 2
⇒ + 2 gyt 2 = h
c) Máxima altura sobre tierra: h = y max + 50 .
2
Resolviendo para y :
De v 0 + 2ay max = 0 , ⇒
2
2
1 ⎛ 2h − gt 2 ⎞
− (20)
2 y= ⎜ ⎟⎟
ymax = = 20,4 m 2 g ⎜⎝ 2t ⎠
− 2(9,8) Con los datos
Luego, h = 70,4 m.
1 ⎡ 2(1,9) − (9,8)(0,42) ⎤
2 2
d) Si t1 y t2 son los tiempos para alcanzar P1 y P2,
y= ⎢ ⎥ = 0,311 m
respectivamente, 2(9,8) ⎣ 2(0,42) ⎦
− 15 = 20t1 − 4,9t12 y − 30 = 20t 2 − 4,9t 22
Resolviendo, t1 = 4,723 s, t2 = 5,248 s, y el tiempo Ejemplo 25. Malabarismo. Un malabarista actúa
de P1 a P2 es (t2 - tl) = 0,525 s. en un recinto cuyo cielorraso está 3,0 m arriba del
e) Si v0 es la velocidad inicial deseada, entonces –v0 nivel de las manos. Lanza una pelota hacia arriba de
es la velocidad cuando pasa el borde. Luego modo que apenas llega al techo.
1 2 a) ¿Qué velocidad inicial tiene la pelota?
aplicando y = v 0 t + at al viaje hacia abajo de b) ¿Cuánto tiempo tarda la pelota en llegar al techo?
2
En el instante en que la primera pelota está en el
la torre, encontramos:
cielorraso, el malabarista lanza una segunda pelota
-50 = (- v0)(3) – 4,9(3)2, ⇒ v0 = 1,96 m/s.
hacia arriba con dos terceras parte de la velocidad
inicial de la primera.

15
Movimiento rectilíneo Hugo Medina Guzmán

c) ¿Cuánto tiempo después de lanzada la segunda


pelota se cruzan las dos pelotas en el aire? d) ¿A qué
altura sobre la mano del malabarista se cruzan las
dos pelotas
Solución.
a) Tomemos el sentido positivo hacia arriba.
Tenemos que v y = v 0 y − 2 g ( y − y 0 )
2 2

En el cielorraso, v y = 0 , y − y 0 = 3,0 m .
Luego: 0 = v 0 y − 2(9,8)(3) ⇒ v 0 y = 7,7 m s .
2

b) También tenemos:
v y = v0 y − gt = 0 = 7,7 − 9,8t
⇒ t = 0,78 s .
c) Tomemos el sentido positivo hacia abajo.
La primera bola viaja hacia abajo una distancia d Ejemplo 27. En el salto vertical, un atleta se
agazapa y salta hacia arriba tratando de alcanzar la
en el tiempo t . Como comienza desde su máxima
mayor altura posible. Ni los campeones pasan
altura, v0 y = 0. mucho más de 1,00 s en el aire (“tiempo de
suspensión”). Trate al atleta como partícula y sea
d = v0 y t + 12 gt 2 ⇒ d = (4,9 m s 2 )t 2
y máx su altura máxima sobre el suelo. Para explicar
La segunda bola tiene
por qué parece estar suspendido en el aire, calcule la
v' 0 y = 13 (7,7 m s) = 5,1 m s .
razón del tiempo que está sobre y máx / 2 al tiempo
En el tiempo t habrá viajado hacia arriba
que tarda en llegar del suelo a esa altura. Desprecie
(3,0 m − d ) y estará en el mismo lugar que la la resistencia del aire.
primera bola. (3 − d ) = v' 0 y t − 12 gt
2 Solución.
El tiempo al caer para alcanzar y máx es:
(3 − d ) = 5,1t − 4,9t 2
Tenemos dos ecuaciones con dos incógnitas. 2 y máx
t1 = =1 s .
Resolviéndolas obtenemos: g
t = 0,59 s y d = 1,7 m.
El tiempo al caer para alcanzar y máx / 2 es:
d) 3,0 m − d = 1,3 m
2 y máx / 2 y máx t 1
t2 = = = 1 = s.
Ejemplo 26. Una manzana cae libremente de un g g 2 2
árbol, estando originalmente en reposo a una altura
H sobre un césped crecido cuyas hojas miden h. 1
El tiempo debajo de y máx / 2 es 1 − , de tal
Cuando la manzana llega al césped, se frena con 2
razón constante de modo que su rapidez es 0 al manera que la razón entre el tiempo que está sobre la
llegar al suelo, mitad de la altura máxima y el tiempo que está por
a) Obtenga la rapidez de la manzana justo antes de debajo de la altura máxima es.
tocar el césped.
b) Obtenga la aceleración de la manzana ya dentro 1/ 2 1
= = 2,4.
del césped. 1 − 1/ 2 2 −1
c) Dibuje las gráficas: v-t y a-t para el movimiento
de la manzana. Esto explica porque el atleta parece estar suspendido
Solución. en el aire.
a) La rapidez de un objeto que cae una distancia
Ejemplo 28. Un excursionista despierto ve caer un
H en caída libre una distancia H − h es:
peñasco desde un risco lejano y observa que tarda
v = 2 g ( H − h). 1,30 s en caer el último tercio de la distancia. Puede
b) La aceleración para llevar a un objeto desde la despreciarse la resistencia del aire.
rapidez v al reposo sobre una distancia h es: a) ¿Qué altura (en m) tiene el risco?
b) Si en (a) obtiene dos soluciones de una ecuación
v2 2 g ( H − h) ⎛H ⎞ cuadrática y usa una para su respuesta, ¿qué
a= = = g ⎜ − 1⎟.
2h 2h ⎝h ⎠ representa la otra?
Solución.
c)
a) Sea h la altura y toma un tiempo t en caer:
h = 12 gt 2

16
Movimiento rectilíneo Hugo Medina Guzmán

Si tarda 1,30 s en caer el último tercio h :


Ejemplo 30. Una piedra que cae libremente pasa a
2
3 h = 12 g (t − 1,3) 2 las 10 horas frente a un observador situado a 300 m
Eliminando h de estas dos ecuaciones obtenemos: sobre el suelo, y a las 10 horas 2 segundos frente a
1
3 gt 2 = 12 g (t − 1,3) 2 un observador situado a 200 m sobre el suelo. Se
pide calcular:
t − 7,8t + 5,07 = 0
2
a) La altura desde la que cae.
b) En qué momento llegará al suelo.
⎧t = 7,08s
Resolviendo t = 3,9 ± 3,18 ⎨ 1 c) La velocidad con que llegará al suelo.
⎩t 2 = 0,73s
La primera es la solución correcta porque es mayor
que 1,30 s,
h= 1
2
(9,8)(7,08)2 = 245,6 m
b) Con la segunda solución para t encontramos h =
2,6 m. Esto correspondería a un objeto que estaba
inicialmente cerca del fondo de este "acantilado" que
era lanzado hacia arriba y tomando 1,30 s la subida a
la cima y la caída al fondo. Aunque físicamente es
posible, las condiciones del problema imposibilitan
esta respuesta.

Ejemplo 29. Desde la cornisa de un edificio de 60 m


de alto se lanza verticalmente hacia abajo un Solución.
proyectil con una velocidad de 10 m/s. Calcular:
a) Velocidad con que llega al suelo. h1 = 300m
b) Tiempo que tarda en llegar al suelo.
h2 = 200m t1 = 2 s
c) Velocidad cuando se encuentra en la mitad de su
recorrido. g ≈ 10m/s 2
d) Tiempo que tarda en alcanzar la velocidad del
h3 = 100m
apartado c). a)
v2 = v1 + gt1 ⇒ v2 = v1 + 10 × 2
1 2 1
h3 = v1t1 + gt1 ⇒ 100 = 2v1 + 10 × 4
2 2
2 2
v v
h4 = 2 ⇒ h4 = 2
2g 2 × 10
H = h2 + h4
⎧v1 = 40m/s

Solución. De aquí se obtiene ⎨v2 = 60m/s ,
Tomamos corno origen de coordenadas el punto de ⎪h = 180m
lanzamiento y como sentido positivo el del eje ⎩ 4
vertical descendente. Las ecuaciones de este Finalmente H = 200 + 180 = 380 m
movimiento serán:
v = v 0 + gt b) Llamando t2 al tiempo que tarda en recorrer hl:
v0 = 10 m/s
1 2 1 2
s = v0 t + gt g ≈ 10m/s 2 h1 = v1t 2 + gt 2
2 2
a) y b) h = 60 m 1
⇒ 300 = 40t 2 + 10t 22
v = 10 + 10t 2
t = 2,6 s
1 ⇒ ⇒ t 2 = 5s
60 = 10t + 10t 2
v = 36m/s Luego llega al suelo a las 10 horas 5 segundos
2
c) y d) h’ = 30 m c) v = 2 gH = 2 × 10 × 380
v' = 10 + 10t ' = 87 m/s
t ' = 1,65 s
1 ⇒
30 = 10t '+ 10t ' 2
v' = 26,5m/s
2

17
Movimiento rectilíneo Hugo Medina Guzmán

PROBLEMA INVERSO - CÁLCULO El desplazamiento total para el intervalo (t − t 0 ) es


INTEGRAL
la suma de todas las áreas de todos los rectángulos
Conociendo la ley del movimiento x = x (t ) es de tal modo que:
posible sin mayores dificultades calcular v(t ) y Δx = ∑ v m (t i )Δt
a (t ) tal como fue mostrado i

La regla para los tiempos es que t i +1 = t i + Δt .


dx(t ) dv(t ) d 2 x(t )
x(t ) ⇒ v(t ) = ⇒ a(t ) = = La distancia que obtenemos con este método no será
dt dt dt 2 la correcta porque la velocidad cambia durante el
Como hemos visto, el cálculo diferencial tiempo del intervalo Δt .
proporciona la herramienta para determinar la Si tomamos los intervalos muy pequeños la suma
velocidad y aceleración en cualquier instante del tiene mayor precisión. Así es que los hacemos tan
tiempo. pequeños a fin de tener una buena aproximación.
En esta sección veremos cómo el cálculo integral, Obtendremos la distancia real en el límite:
que es el inverso del cálculo diferencial, puede
utilizarse para deducir las fórmulas que ya hemos Δx = lim ∑ v(t i )Δt
Δt → 0
visto. Por ejemplo, hallar la posición de una i

partícula en un instante cualquiera, dado su Obsérvese que hemos reemplazado la velocidad


velocidad inicial y su aceleración conocida. promedio v m por la velocidad instantánea v ,
porque en el límite esta aproximación es válida.
Ya hemos demostramos que el área encerrada bajo Los matemáticos han inventado un símbolo para este
la curva de la velocidad del diagrama velocidad- límite, análogo al símbolo para la diferencial. El
símbolo Δ se convierte en d , v(t i ) se llama v(t )
tiempo es igual al desplazamiento.
1
Area del trapecio = v0 (t − t 0 ) + a(t − t 0 )
2
y el símbolo sumatoria ∑ se escribe como una "s”
2 grande ∫ la cual se conoce el signo integral Luego
1 escribimos
x − x 0 = v0 (t − t 0 ) + a(t − t 0 )
2

Δx = ∫ v(t )dt
t
2
En el caso de un movimiento con velocidad t0

constante el desplazamiento entre los tiempos t y El proceso de integración es el inverso del proceso
t 0 es de derivación. Con un diferencial obtenemos una
fórmula integral si la invertimos.
x − x 0 = v0 (t − t 0 )
o Δx = v 0 (t − t 0 )
Ejemplo 31. Encontrar la velocidad de un móvil a
partir de la aceleración.
Para un movimiento cualquiera con aceleración Solución.
variable el diagrama velocidad-tiempo será el dv
mostrado en la figura siguiente a= ⇒ dv = adt
dt
v t t
⇒ ∫
v0
dv = ∫ adt = a ∫ dt
t0 t0
Integrando obtenemos
v − v0 = a(t − t0 ) ⇒ v = v0 + a(t − t0 )
Para encontrar la posición
dx
v= ⇒ dx = vdt
dt
x t

Si descomponemos el tiempo total desde t 0 hasta t


⇒ ∫
x0
dx = ∫ vdt
t0

dx = ∫ [v0 + a(t − t0 )]dt


x t
en segmentos pequeños Δt , entonces cada tramo ⇒ ∫
x0 t0
vertical que baja desde la curva de velocidades hasta
Integrando obtenemos
el eje de absisas tiene un área
1
ΔA = v m Δt x − x0 = v0 (t − t0 ) + a(t − t0 )
2

2
Donde v m es la velocidad media del intervalo. Esta
1
⇒ x = x0 + v0 (t − t0 ) + a(t − t0 )
2
área corresponde al desplazamiento en ese intervalo
que como se puede observar el área faltante se 2
complementa con el excedente del otro lado.

18
Movimiento rectilíneo Hugo Medina Guzmán

También se puede encontrar la ecuación del x = ∫ [v0 + a(t − t 0 )]dt + C 2


movimiento expresando la integral de la siguiente
manera: Integrando:
v = ∫ adt + C1 , x = ∫ vdt + C2 1 2
x = v0 t + at − at 0 t + C 2
2
Los valores de C1 y C2 dependen de las
Como para t = t 0 se tiene x = x 0 , tenemos
condiciones iniciales del movimiento.
1
x0 = v0 t 0 + at 02 − at 02 + C 2
2
Pequeña Tabla de Integrales
1
⇒ C 2 = x0 − v0 t 0 + at 02
2
∫ dx = x Reemplazando el valor de C 2 obtenemos
x n +1
∫ x dx =
n
(n ≠ −1) 1 ⎛ 1 ⎞
x = v0 t + at 2 − at 0 t + ⎜ x0 − v0 t 0 + at 02 ⎟
n +1 2 2
⎝ ⎠
dx
∫ x
= lnx 1
⇒ x = x0 + v0 (t − t0 ) + a(t − t0 )
2

2
e ax
∫ e dx = a
ax
Ejemplo 33. La aceleración de una motocicleta está
cos(ax ) dada por a (t ) = 1,5t − 0,12t , con t en s m/s3. La
2

∫ sen(ax ) = a moto está en reposo en el origen en t = 0.


a) Obtenga su posición y velocidad en función de t.
∫ (u + v )dx = ∫ udx + ∫ vdx b) Calcule la velocidad máxima que alcanza.
Solución.
a) Para encontrar v(t ) .

⇒ dv = adt = (1,5t − 0,12t 2 )dt


Ejemplo 32. Encontrar las ecuaciones del dv
movimiento para una partícula que se mueve con a=

dt
aceleración constante a = aiˆ y que para el tiempo Integrando con v0 = 0 y t 0 = 0 :

( )
→ t
inicial t 0 se encontraba en r 0 = x 0 iˆ y tenía una v = ∫ 1,5t − 0,12t 2 dt = 0,75t 2 − 0,40t 3
0

Para encontrar x(t ) .



velocidad inicial v 0 = v 0 iˆ .
Solución. dx
v= = 0,75t 2 − 0,40t 3
El movimiento es en el eje x . dt
La aceleración es a=
dv ⇒ dx = (0,75t 2 − 0,40t 3 )dt
dt Integrando con x0 = 0 y t 0 = 0 :
La velocidad se puede encontrar en términos de una
integral como t
( )
x = ∫ 0,75t 2 − 0,40t 3 dt = 0,25t 3 − 0,10t 4
v = ∫ adt + C1 ⇒ v = at + C1 0
b) Para que la velocidad sea máxima la aceleración
Como para t = t 0 se tiene v = v0 , tenemos debe ser cero,

v0 = at 0 + C1 ⇒ C1 = v0 − at 0 ⎧t = 0

a(t ) = 1,5t − 0,12t = 0 ⇒ ⎨
2
1,5
Reemplazando el valor de C1 obtendremos la
⎪t = 0,12 = 12,5s
ecuación de la velocidad: ⎩
v = v0 + a(t − t 0 ) Para t = 0 la velocidad es mínima
Para t = 12,5 la velocidad
v = 0,75(12,5) − 0,40(12,5) = 39,1 m/s
2 3
Ahora consideremos la definición de la velocidad
dx
v= Ejemplo 34. Salto volador de la pulga. Una
dt película tomada a alta velocidad por M. Rothschild,
También se puede escribir en forma integral Y. Schlein. K. Parker, C. Neville y S. Sternberg
x = ∫ vdt + C2 (3500 cuadros por segundo, “The Flying Leap of the
Flea”, en el ScientificAmerican de noviembre de
Reemplazando el valor de v : 1973) de una pulga saltarina de 210 μg produjo los

19
Movimiento rectilíneo Hugo Medina Guzmán

datos que se usaron para dibujar la gráfica de la 1


figura. La pulga tenía una longitud aproximada de 2 = (1,3 ×10-3 )(133) + (0,2 × 10-3 )(133)
2
mm y saltó con un ángulo de despegue casi vertical.
= 0,11 cm
Use la gráfica para contestar estas preguntas.
a) ¿La aceleración de la pulga es cero en algún
momento? Si lo es, ¿cuándo? Justifique su respuesta. Ejemplo 35. La gráfica de la figura describe, en
b) Calcule la altura máxima que la pulga alcanzó en función del tiempo, la aceleración de una piedra que
los primeros 2,5 ms. baja rodando por una ladera, habiendo partido del
c) Determine la aceleración de la pulga a los: 0,5 ms, reposo.
1,0 ms y 1,5 ms. a) Determine el cambio de velocidad de la piedra
d) Calcule la altura de la pulga a los: 0,5 ms, 1,0 ms entre t = 2,5 s y t = 7,5 s.
y 1,5 ms. b) Dibuje una gráfica de la velocidad de la piedra en
función del tiempo.

Solución.
Solución.
a) Pendiente de a = 0 para t ≥ 1,3 ms dv
a) a= ⇒ dv = adt
b) La altura máxima corresponde al recorrido hasta dt
cuando la aceleración se hace cero y llega al tiempo Como a (t ) es la ecuación de la recta:
t = 2,5 ms, y es el área bajo la curva v versus t.
a−2 8−4
(Dibujado aproximándolo a Un triángulo y un = = 0,8 ⇒ a = 0,8t + 2
rectángulo). t − 0 7,5 − 2,5
dv = (0,8t + 2 )dt
dv = ∫ (0,8t + 2)dt
v t
Integrando: ∫v0 t0

⇒ v − v0 = 0,4(t − t 02 ) + 2(t − t 0 )
2

Con t 0 = 2,5s , t = 7,5s , y Δv = v − v0 :


hmax = área bajo (v − t )
≈ ATriángulo + ARectángulo Δv = 0,4(7,52 − 2,52 ) + 2(7,5 − 2,5)
cm
1 = 30
≈ [(1,3)(133) + (2,5 − 1,3)(133)]10 −3 s
2 Otra manera de encontrar el cambio de velocidad es
≈ 0,25 cm encontrando el área bajo la curva a versus t, entre las
c) a = pendiente del gráfico v– t. líneas en t = 2,5 s y t = 7,5 s. El área es:
a (0,5 ms) ≈ a (1,0 ms) cm
133
1
2 (4 + 8)(7,5 − 2,5) = 30
≈ = 1,0 ×105 cm s
2
s
1,3 ×10 -3
Como la aceleración es positiva, el cambio de
a (1,5 ms) = 0 porque la pendiente es cero. velocidad es positivo.
b)
d) h = área bajo el gráfico v– t.
1
h (0,5) ≈ ATriángulo = (0,5 × 10 -3 )(33)
2
= 8,3 × 10 −3 cm
1
h (1,0) ≈ ATriángulo = (1,0 × 10 -3 )(100)
2
= 5,0 × 10 −2 cm Ejemplo 36. La velocidad de un punto que se mueve
h (1,5) ≈ ATriángulo + ARectángulo en trayectoria recta queda expresada, en el SI por la
ecuación: v = 40 - 8t. Para t = 2 s, el punto dista del
origen 80 m. Determinar:

20
Movimiento rectilíneo Hugo Medina Guzmán

a) La expresión general de la distancia al origen.


b) El espacio inicial.
c) La aceleración.
d) ¿En qué instante tiene el móvil velocidad nula?
e) ¿Cuánto dista del origen en tal instante?
f) Distancia al origen y espacio recorrido sobre la
trayectoria a partir de t = 0, cuando t = 7 s, t = 10 s y
t = 15 s.
Solución.
a) s = ∫ vdt = ∫ (40 − 8t )dt = 40t − 4t 2 + C
⇒ s = s 0 + 40t − 4t 2
b) 80 = s0 + 80 - 16 ⇒ s0 = 16
dv m
c) a = = −8 2 Representación gráfica de la velocidad origen en
dt s función del tiempo
d) 0 = 40 - 8t ⇒ t =5 s
e) s5 =16 + 40x5 - 4x52 = 116 m
f) s7 =16 + 40x7 - 4x72 = 100 m
sl0 =16 + 40x10 - 4x102 = 16 m
s15 = 16 + 40x15 - 4x152 = -284 m
Cálculo de caminos sobre la trayectoria a partir de
t = 0:
El móvil cambia el sentido de su velocidad para
t = 5s
El recorrido en los 5 primeros segundos es:
C5 = s – s0 = 116 - 16 = 100 m En la gráfica de la velocidad frente al tiempo, el área
A ellos hay que sumar el recorrido en los segundos limitada por el eje de abscisas y la gráfica entre dos
restantes que se obtienen de la integral de la instantes coincide numéricamente con el camino
ecuación general de la velocidad, en valor absoluto, recorrido por el móvil entre esos dos instantes.
entre los limites t = 5 s y t = instante final.

∫ (40 − 8t )dt = 116m


7 Ejemplo 37. El vector velocidad del movimiento de
C7 = 100 + una partícula viene dado por
5

v = (3t - 2)iˆ + (6t 2 - 5) ˆj m/s. Si la posición del

móvil en el instante t =1 s es r = 3iˆ − 2 ˆj m.
Calcular

a) El vector posición del móvil en cualquier instante.

∫ (40 − 8t )dt = 200m


10
C10 = 100 + b) El vector aceleración.
5

c) Las componentes tangencial y normal de la


aceleración en el instante t = 2 s. Dibujar el vector
velocidad, el vector aceleración y las componentes
tangencial y normal en dicho instante.

Solución.

∫ (40 − 8t )dt = 500m


15
C15 = 100 + a) Para el movimiento horizontal
5

dvx m
vx = 3t - 2 ⇒ ax = =3 2
dt s

Representación gráfica de la distancia al origen en dx


función del tiempo
Como vx = ⇒ dx = v x dt , integrando
dt

21
Movimiento rectilíneo Hugo Medina Guzmán

t
dx = ∫ (3t − 2 )dt
t
an = asen (θ − ϕ ) = 2m/s 2
∫3 1

⎛3 7⎞ CINEMÁTICA DE PARTÍCULAS LIGADAS.


⇒ x = ⎜ t 2 − 2t + ⎟m MOVIMIENTOS DEPENDIENTES.
⎝2 2⎠
Observemos los sistemas físicos de la figura.
Podríamos decir que estos sistemas se componen de
Para el movimiento vertical varias partículas ligadas (conectadas).

dv y m
v y = 6t 2 - 5 ⇒ ay = = 12t
dt s2

dy
Como vy = ⇒ dy = v y dt , integrando
dt

∫−2
t t
(
dy = ∫ 6t 2 − 5 dt ⇒
1
) Las partículas podrían ser las poleas y los cuerpos a
desplazar (bloques, baldes). La ligadura la tienen a
y = (2t 3 − 5t + 1) m través de las cuerdas. Es decir, cuando el hombre
desplaza el extremo de la cuerda con una aceleración
a, la aceleración de las poleas y los cuerpos a
⎛3 7⎞
( )
→ desplazar (bloques, baldes) tendrán una dependencia
r = ⎜ t 2 − 2t + ⎟î - 2t 3 − 5t + 1 ĵ de a. Lo mismo se cumplirá para las otras variables
⎝2 2⎠ cinemáticas (desplazamiento y velocidad).

→ Ejemplo 38. Análisis del montaje de la figura


b) a = 3iˆ + 12tˆj siguiente.

c) Para t = 2 s

vx = 4 m/s, vy = 19 m/s

ax = 3 m/s2, ay = 24 m/s2
a = ax2 + a y2 = 24,2m / s 2

vy 19 Para analizar las relaciones que hay entre las


tan ϕ = = = 4,75 ⇒ ϕ = 78 o variables cinemáticas del bloque m1 , del balde m 2
vx 4
y de la polea móvil, debemos primero saber cuáles
son sus posiciones. Para ello elegimos un sistema de
ay 24 coordenadas. En nuestro caso elegimos el eje y
tan θ = = = 3 ⇒ θ = 83o apuntando hacia abajo y con el origen en el techo.
ax 3 Para el sistema de coordenadas escogido las
posiciones del bloque, del balde y de la polea son
respectivamente: y1 , y 2 , y p . Estas se representan
en la figura siguiente.

at = a cos(θ − ϕ ) = 24,1m/s 2

22
Movimiento rectilíneo Hugo Medina Guzmán

La longitud de la cuerda debe permanecer constante Análogamente podríamos hacer un análisis para las
en todo instante. Por tanto debe ser siempre válida la aceleraciones, y concluiríamos que:
siguiente relación: 1
Longitud de la cuerda = constante a p = − a1
AB + arco BC + CD +arco DE +EF = constante 2
De la figura podemos concluir que las siguientes Es decir, si el bloque por ejemplo, baja con una
relaciones son válidas: aceleración igual a 2,0 m/s2 , la polea subirá con una
aceleración igual a 1,0 m/s2 .
AB = y p
CD = y p − c2 De esta figura también se deduce la siguiente
relación entre la posición del balde y la posición de
EF = y1 − c2 la polea móvil:
Por tanto, y 2 = y p + c1 (3)
y p + arcoBC + ( y p − c2 ) + arcoDE + y1 = constante
Si el balde se desplaza una cantidad Δy 2 , y la polea
Como los arcos BC y DE permanecen constantes
podremos escribir la relación anterior así: se desplaza una cantidad Δy p .
2 y p + y1 = k (1) El balde pasa a ocupar la posición: y 2 + Δy 2 ,
Siendo k una constante. La polea pasa a ocupar la posición y p + Δy p .
Esta ecuación relaciona las variables cinemáticas de
la polea móvil y del bloque. Sin embargo, la relación anterior se debe seguir
cumpliéndose.
Si el bloque se desplaza una cantidad Δy1 y la polea
( )
( y 2 + Δy 2 ) = y p + Δy p + c1 (4)
en una cantidad Δy p .
Restando (3) y (4) obtenemos,
La nueva posición de la polea: Δy 2 = Δy p
y p + Δy p ,
Los desplazamientos de la polea y el balde son
La nueva posición del bloque: y1 + Δy1 . iguales.
Sin embargo, la relación anterior debe seguir Si dividimos la ecuación anterior por el intervalo de
cumpliéndose: tiempo Δt obtenemos como se relacionan las
( )
2 y p + Δy p + ( y1 + Δy1 ) = k (2) velocidades: v2 = v p .
Restando (1) de (2), obtenemos: Las velocidades de la polea y del balde son iguales.
2Δy p + Δy1 = 0 Lo mismo podremos concluir para las aceleraciones:
a2 = a p
Δy
Δy p = − 1 En definitiva si el bloque baja con una aceleración
2 igual a 4 m/s2, el balde y la polea móvil subirán con
Por ejemplo, si el bloque baja 1,0 m, la polea solo una aceleración igual a 2 m/s2.
sube 0,50 m. La polea solo se desplaza la mitad de lo
que se desplaza el bloque.

PREGUNTAS Y PROBLEMAS

1. Un acelerador atómico emite partículas que se 3. Una cucaracha se desplaza en línea recta y su
desplazan con una rapidez de 2,8x108 m/s. ¿cuánto posición con respecto al tiempo se expresa de
demoran estas partículas en recorrer una distancia de acuerdo al siguiente gráfico. De acuerdo a la
5,6mm? información dada se pide calcular.
Respuesta a) distancia recorrida entre 4s y 9 s
2x10-11 s. b) distancia recorrida entre 9 s y 14s
c) distancia recorrida entre 0 y 16s.
2. Se desea calcular cuál es la profundidad de un d) velocidad media entre 0s y 16s.
lago, para tal efecto se usa un instrumento conocido e) velocidad media entre 9s y 16s.
como sonar que mide el tiempo que tarda un pulso
sonoro en ir y volver desde la superficie del agua. Si
se sabe que la rapidez del sonido en el agua es de
1450m/s y el instrumento marcó 0,042s cuando se
hizo la medición, calcule la profundidad del lago.
Respuesta. 30,45m

23
Movimiento rectilíneo Hugo Medina Guzmán

a) v = 2t 2 − t 4 ; b) x = 2 + 2t 3 / 3 − t 5 / 5
9. El movimiento de una partícula se define
mediante la relación x = t / 3 − 3t + 8t + 2 ,
3 2

donde x se expresa en metros y t en segundos.


Determinar
a) el momento en que la velocidad es nula;
b) la posición y la distancia total recorrida cuando la
Respuesta aceleración es nula.
a) 4m b) 8m c) 22m d) 5/8 m/s e) 0 Respuesta
→ a) 2s, 4s; b) 8m, 7,33m
4. Un hombre camina con una velocidad v
constante pasa bajo un farol que cuelga a una altura 10. El movimiento de una partícula está dado por la
ecuación horaria x = t + 4t + 5 x sobre el eje x,
H sobre el suelo. Encontrar la velocidad con la que 3 2

el borde de la sombra de la cabeza del hombre se x en metros t en segundos.


mueve sobre la tierra. El alto del hombre es h. a) Calcular la velocidad y la aceleración de la
Respuesta partícula en el instante t.

b) Encontrar la posición, la velocidad y la
Hv
aceleración de la partícula para t 0 = 2s y t 1 = 3s.
H −h
c) ¿Cuáles son la velocidad media y la aceleración
5. Un tren arranca en una estación y acelera media de la partícula entre t 0 y t1 ?
2 Respuesta.
uniformemente a razón de 0,6 m/s hasta alcanzar
2 2
una velocidad de 24 m/s. Determinar el tiempo a) v = (3t + 8t)m/s , a = ( 6t + 8 ) m/s
empleado y la distancia recorrida en ese período si la b) x0 = 29m, v0 = 27 m/s, a 0 = 20 m/s 2
velocidad media fue: a) 16 m/s, b) 22m/s.
Respuesta x1 = 68 m v1 = 51 m/s, a1 = 26 m/s 2
a) 60s, 960m, b) 240s, 5280m 2
c) v m = 39 m/s , a m = 23 m/s
6. Un ciclista recorre 100 km en 2 horas. El viaje de
vuelta dos días más tarde lo realiza en el tiempo 11. La posición de una partícula que se mueve en el
usual de 6 horas. eje x está dada por 8 t + 5, x es la distancia a origen
a) ¿Cuál es su rapidez media a la ida? en metros y t es el tiempo en segundos.
b) ¿Cuál es su rapidez media al regreso? a) Para t = 2, encontrar la posición, velocidad y
c) ¿Su rapidez media en e¡ viaje completo? aceleración
d) ¿Su velocidad media en e} viaje entero? b) Grafique x versus t
Respuesta. a) 50 km/h , b) 16,7 km/h c) Encuentre la ley horaria, la ley del movimiento y
c) 25 km/h d) 0 la trayectoria.
d) Analizar el movimiento.
7. Un automóvil que viaja con una velocidad de 50 Respuesta. a) x = -3, v = 0 , a = 4

( − 8t + 5)iˆ
km/h hacia el oeste repentinamente empieza a perder →
b) s = 2t − 8t + 5 , r = 2t
2 2
velocidad a un ritmo constante y 3 segundos más
tarde su velocidad es de 25 km/h hacia el oeste. Trayectoria rectilínea en el eje x.
a) ¿Cuánto tiempo tardará en detenerse el auto,
contando a partir del momento en que empezó a 12. Un automóvil se encuentra detenido frente a un
desacelerar? semáforo, le dan luz verde y arranca de modo que a
b) ¿Cuál es la distancia total que recorrerá antes de los 4s su rapidez es de 72 km/hora. Si se movió en
detenerse? trayectoria rectilínea, con aceleración constante,
c) ¿Cuál sería el tiempo necesario para detenerse y la I.- Determine:
distancia recorrida el) la frenada con la misma a) La rapidez inicial en metros por segundo.
aceleración, pero con una velocidad inicial de 100 b) El módulo de la aceleración en ese tramo.
km/h? c) La rapidez que lleva a los 3s.
Respuesta. a) t = 6s ; b) 41,7m ; c) 125; 125m d) La distancia que recorre en los tres primeros
segundos
8. La aceleración de una partícula está dada por: e) La distancia que recorre entre t = 2s y t = 4s.
a = 4t − 4t 3 , t ≥ 0 . II.- Haga un gráfico representativo de posición
a) Hallar la velocidad de la partícula en función del versus tiempo y de la rapidez versus tiempo.
tiempo.
b) Hallar su posición en función del tiempo. Respuesta. a) 20m/s b) 5 m/s2 c) 15m/s
Respuesta d) 22.45m e) 30m

24
Movimiento rectilíneo Hugo Medina Guzmán

movimiento para retornar hacia A con una


13. Una partícula A, se mueve en el eje X, de aceleración constante de módulo 6(m/s2). Calcular:
acuerdo a la siguiente gráfica. a) La distancia total cubierta hasta que la partícula
Determinar a partir del gráfico de la partícula: retorne al punto A.
b) El tiempo total para el recorrido completo hasta
a) Velocidad media entre t = 0 y t = 4 s volver a dicho punto A.
b) Velocidad instantánea en t = 2 s c) El intervalo de tiempo que transcurre entre los
c) Aceleración media entre t = 0 y t = 4 s pasos de la partícula por el punto situado a 1/3 de
d) Intervalos de tiempo en que se acerca al AB, medido desde A.
origen
e) Intervalos de tiempo en que se aleja del 17. Desde una altura de 45m se deja caer un objeto
origen A. simultáneamente se lanza un objeto B
f) Ecuación Itinerario de la partícula A verticalmente desde una altura de 5m. Calcular:
g) ¿Qué tipo de movimiento tiene esta partícula? a) la velocidad inicial de B para que los objetos se
crucen a una altura de 20m.
b) la distancia que separa a los objetos cuando B
alcanza su altura máxima.

18. Sobre un mismo eje x se mueven dos partículas


A y B. En t = 0 la partícula A parte desde P con
aceleración constate de 15iˆ (m/s2). Un segundo
después, B pasa por Q con una velocidad de − 20iˆ
(m/s). Encuentre las retardaciones constantes que
Respuesta. a) ( -8;0)m/s b) (-8;0)m/s c) 0 deben aplicar A y B a partir de este último instante
para que ambas partículas se detengan
d) (0-3)s e)(3-....) f) x(t ) = 24 − 8t simultáneamente antes de chocar.
g) Movimiento rectilíneo uniforme.

14. Un vehículo se mueve en el eje x de acuerdo con


la siguiente ecuación de itinerario:
x(t ) = 20 − 36t + 6t 2 . Con x medido en metros y
t en segundos.
a) Identifique a posición inicial, la velocidad inicial
y la aceleración. 19. Una partícula se mueve a lo largo del eje x
b) Determine la ecuación que entregue la velocidad con aceleración constante. En t = 0 pasa por la

para cualquier instante.
c) Determine el instante en que cambia de sentido posición x 0 = −10iˆ m con una velocidad

d) La velocidad de la partícula en t = 2 s y en t = 4
s
v 0 = −20iˆ m/s y en t =3s su posición es

e) Posición de la partícula en t = 6 segundos
x = −52iˆ m. Calcule:
f) Gráfico x versus t. Describa la curva
a) La ecuación itineraria de la partícula
g) Gráfico v x versus t. Describa la curva b) La distancia recorrida en el intervalo (3-6) s.
h) Gráfico a versus t. Describa la curva c) La velocidad media en el intervalo (4-7) s.
Respuesta. a) (20,0)m (-36,0)m/s d) Intervalos de tiempo en que la partícula se aleja
(12,0)m/s2 b) v(t ) = −36 + 12t c)3s del origen del sistema.
d) (-12,0)m/s (12,0)m/s e) (20,0)m

15. Se lanza un cuerpo hacia arriba con una rapidez


de 16m/s, 20. Sobre el eje x de un sistema de coordenadas se
a) ¿Qué altura alcanza a subir? mueven dos partículas A y B. El gráfico (a) es una
b) ¿Qué tiempo demora en volver al punto de parábola cuadrática que muestra la variación de
partida? la componente x de la posición en función del
Respuesta. a) 3,2m b) 6,4s tiempo de la partícula A. El gráfico (b) muestra la
variación de la componente v x de la velocidad en
16. Una partícula se mueve sobre una recta
horizontal; parte hacia la derecha desde un punto A función del tiempo de la partícula B. Si en t = 0,
con una rapidez de 28 (m/s) y una retardación ambas partículas tienen la misma posición,
constante de módulo 12(m/s2). En el punto B, es determinar:
donde se anula su rapidez, invierte el sentido de a) Ecuación horaria de las partículas A y B.

25
Movimiento rectilíneo Hugo Medina Guzmán

b) Posición de B cuando A cambia de sentido de


movimiento. 23. Un cuerpo que se ha dejado caer desde cierta
c) Instante en que se encuentran. altura, recorre 72 m en el último segundo de su
d) Distancia recorrida por A y B entre 3 y 9 s. movimiento.
Calcule la altura desde la cual cayó el cuerpo y el
tiempo que empleó en llegar al suelo.

24. Un hombre parado en el techo de un edificio tira


un cuerpo verticalmente hacia arriba con una rapidez
de 14m/s. El cuerpo llega al suelo 4,7s más tarde.
a) Cuál es la máxima altura alcanzada por el
cuerpo?
b) Qué altura tiene el edificio?
c) Con qué rapidez llegará el cuerpo al suelo?

21. En el gráfico de la figura están representadas la


componente v x del vector velocidad de dos
partículas, A y B, que se mueven a lo largo del eje
x Calcular:
a) La aceleración de B.
b) Camino recorrido por A y B cuando B alcanza la 25. Un malabarista mantiene cinco bolas continuamente
→ en el aire, lanzando cada una de ellas hasta una altura de
velocidad v B = 30iˆ m/s. 3m.
a) ¿Cuál es el tiempo que debe transcurrir entre
c) Desplazamiento de B en el intervalo (0-10)s.
lanzamientos sucesivos?
d) Ecuación horaria de A si en t0 = 0 su posición es b) ¿Cuáles son las alturas de las otras pelotas en el

momento en que una de ellas vuelve a su mano?
x 0 = 8iˆ m. Respuesta. a) 0,31s ;
b) 1,91; 2,87; 2,87 y 1,91 m.

26. Dos cuerpos son lanzados uno después de otro con las
mismas velocidades v0 desde una torre alta. El primer
cuerpo es lanzado verticalmente hacia arriba, y el segundo
verticalmente hacia abajo después del tiempo τ .
Determinar las velocidades de los cuerpos una con
respecto al otro y las distancias entre ellos en el instante
t >τ .
Respuesta. La velocidad del primer cuerpo relativa al

22. Dos partículas A y B se mueven sobre el mismo segundo es: v1 − v 2 = 2v0 − gτ .


eje x. En t = 0, B pasa por Q con 1 2
→ La distancia es S = 2v0 t − v0τ − gtτ + gτ
m/s v B (0 ) = (− 5,0 ) m/s y 2s después A pasa 2
por P a 6iˆ m/s. Encuentre las retardaciones
constantes que deben aplicar A y B a partir de este
último instante para que ambas partículas se
detengan simultáneamente justo antes de chocar.
Determine la ecuación itinerario de A y B (diga cuál
es su origen).

26
Movimiento en un plano y en el espacio Hugo Medina Guzmán

CAPITULO 3. Movimiento en un plano y en el espacio

MOVIMIENTO CIRCULAR ω1 . La velocidad angular del móvil ha cambiado


es
Se define movimiento circular como aquél cuya
trayectoria es una circunferencia. Una vez situado el Δω = ω1 − ω 0  en el intervalo de tiempo
origen O de ángulos describimos el movimiento Δt = t1 − t 0 comprendido entre t 0 y t1 .
circular mediante las siguientes magnitudes.

Posición angular, θ
En el instante t el móvil se encuentra en el punto P.
Su posición angular viene dada por el ángulo θ , que
hace el punto P, el centro de la circunferencia C y el
origen de ángulos O.
El ángulo θ , es el cociente entre la longitud del arco
S y el radio de la circunferencia r, θ = S / r . La
Se denomina aceleración angular media al cociente
posición angular es el cociente entre dos longitudes
entre el cambio de velocidad angular y el intervalo
y por tanto, no tiene dimensiones.
de tiempo que tarda en efectuar dicho cambio.
Δω
αm =
Δt
La aceleración angular en un instante, se obtiene
calculando la aceleración angular media en un
intervalo de tiempo que tiende a cero.
Δω dω
α = lim =
Δt →0 Δt dt
RELACIÓN ENTRE LAS MAGNITUDES
ANGULARES Y LINEALES
Velocidad angular, ω De la definición de radián (unidad natural de medida
de ángulos) obtenemos la relación entre el arco y el
En el instante t1 el móvil se encontrará en la
radio. Como vemos en la figura, el ángulo se obtiene
posición P1 dada por el ángulo θ 1 . El móvil se habrá dividiendo la longitud del arco entre su radio
desplazado Δθ = θ1 − θ 0 en el intervalo de tiempo s s'
θ= =
Δt = t1 − t 0 comprendido entre t 0 y t1 . r r'

Derivando s = rθ respecto del tiempo obtenemos la


Se denomina velocidad angular media al cociente relación entre la velocidad lineal y la velocidad
entre le desplazamiento y el tiempo. angular
Δθ ds dθ
ωm = , con las unidades en el SI de rad/s. =r ⇒ v = rω
Δt dt dt
Como ya se explicó en el movimiento rectilíneo, la La dirección de la velocidad es tangente a la
0H

velocidad angular en un instante se obtiene trayectoria circular, es decir, perpendicular a la


calculando la velocidad angular media en un dirección radial
intervalo de tiempo que tiende a cero.
Δθ dθ Aceleración tangencial
ω = lim = Derivando esta última relación con respecto del
Δt →0 Δt dt tiempo obtenemos la relación entre la aceleración
tangencial a t y la aceleración angular.
Aceleración angular, α
Si en el instante t la velocidad angular del móvil es dv dω
=r ⇒ at = rα
ω y en el instante t1 la velocidad angular del móvil dt dt

1
Movimiento en un plano y en el espacio Hugo Medina Guzmán

Existe aceleración tangencial, siempre que el dθ


módulo de la velocidad cambie con el tiempo, es Siendo ω= ⇒ dθ = ωdt , integrando
decir, en un movimiento circular no uniforme dt
obtenemos el desplazamiento θ − θ 0 del móvil
Hallar el desplazamiento angular a partir de la
entre los instantes t 0 y t :
velocidad angular.

∫θ dθ = ∫ [ω
θ
+ α (t − t0 )] dt ⇒
Si conocemos un registro de la velocidad angular del t

móvil podemos calcular su desplazamiento θ − θ 0


0
0 t0

1
entre los instantes t 0 y t , mediante la integral θ = θ 0 + ω0 (t − t0 ) + α (t − t0 )2
definida. 2
t Habitualmente, el instante inicial t 0 se toma como
θ − θ 0 = ∫ ωdt
t0 cero. Las fórmulas del movimiento circular
uniformemente acelerado son análogas a las del
Hallar el cambio de velocidad angular a partir de 2H movimiento rectilíneo uniformemente acelerado.
la aceleración angular. α = constante , ω = ω 0 + α t ,
Del mismo modo que hemos calculado el
desplazamiento angular del móvil entre los instantes 1
θ = θ 0 + ω0 t + α t 2
t 0 y t , a partir de un registro de la velocidad 2
angular ω en función del tiempo t . Despejando el tiempo t en la segunda ecuación y
sustituyéndola en la tercera, relacionamos la
dθ dω
ω= α= velocidad angular ω con el desplazamiento θ − θ 0 .
dt dt
t t
θ − θ 0 = ∫ ω dt ω − ω 0 = ∫ α dt ω 2 = ω 02 + 2α (θ − θ 0 )
t0 t0

MOVIMIENTO CIRCULAR UNIFORME COMPONENTES NORMAL Y TANGENCIAL


Un movimiento circular uniforme es aquél cuya DE LA ACELERACIÓN.
velocidad angular ω es constante, por tanto, la Cuando el sistema de referencia se sitúa sobre la
aceleración angular es cero. partícula tal como se indica en la figura, pero no de
dθ cualquier modo. Uno de los ejes siempre está
ω= ⇒ dθ = ωdt perpendicular a su trayectoria, y el otro siempre es
dt tangente a la misma. Así pues,
La posición angular θ del móvil en el instante t
podemos calcularla integrando
θ t
∫θ 0
dθ = ∫ ωdt
t0

θ − θ 0 = ω (t − t 0 )
O gráficamente, en la representación de ω en
función de t.
Habitualmente, el instante inicial t 0 se toma como
cero. Las ecuaciones del movimiento circular
uniforme son análogas a las del movimiento
1H
El primero siempre pasará por el centro de la
rectilíneo uniforme circunferencia. Al primer eje se le denomina eje
α = 0 ω = constante θ = θ 0 + ω t normal, con vector unitario (rˆ = nˆ ) y al segundo eje

MOVIMIENTO CIRCULAR
()
tangencial, con vector unitario tˆ . Debemos
estudiar ahora que componentes tienen la velocidad
UNIFORMEMENTE ACELERADO
y la aceleración en este sistema de referencia.
Un movimiento circular uniformemente acelerado es
aquél cuya aceleración α es constante.
Velocidad.
Dada la aceleración angular podemos obtener el Con anterioridad se ha deducido que el vector
cambio de velocidad angular ω − ω 0 entre los velocidad siempre es tangente a la trayectoria
instantes t 0 y t , mediante integración de la descrita. Por tanto es fácil afirmar que en este

ω en función del tiempo
velocidad angular movimiento la velocidad será de la forma v = vtˆ
ω = ω0 + α (t − t0 ) .
Aceleración.

2
Movimiento en un plano y en el espacio Hugo Medina Guzmán

No es tan obvio que la aceleración tenga una sola dtˆ


componente, de manera que adoptará la expresión paréntesis es efectivamente n̂ , por lo que
→ dt
general a = at tˆ + an nˆ dtˆ v
quedará como = −ω nˆ = − nˆ .
Sabemos por la definición de aceleración que dt R

2
→ dv → dv v
a= , luego. Finalmente: a = tˆ − nˆ
dt dt R
→ Así, en esta expresión, se denomina aceleración
→ d v dvtˆ dv ˆ dtˆ dv
a= = = t +v tangencial (at ) al término at = y aceleración
dt dt dt dt dt
dtˆ v2
Estudiemos el último término de esta expresión normal (a n ) a la ecuación an = −
dt R
Si se define el ángulo θ , como el ángulo formado De esta expresión para la aceleración pueden
por el eje normal con el eje de abscisas (eje x), tal concluirse cosas sustancialmente importantes:
como se muestra en la figura. Existen dos componentes: Una tangente a la
trayectoria y una perpendicular y orientada hacia el
centro de la circunferencia.
La aceleración tangencial sólo se dará en aquellos
movimientos en los que el módulo de la velocidad
varíe con el tiempo. Por tanto, en el caso particular
del MCU, su aceleración tangencial será nula.
La aceleración normal siempre existirá, salvo que el
No es difícil darse cuenta que el vector tˆ desde el radio de curvatura fuera muy grande, con lo cual
sistema de referencia situado en el centro de la tendería a cero, que es el caso extremo de los
circunferencia tendrá la forma movimientos rectilíneos.
tˆ = −senθ iˆ + cosθ ˆj , mientras que n̂ al ser Concluyendo pues, en un MCU, la aceleración
perpendicular a este adoptará la expresión → v2
tendrá la expresión a = − nˆ es decir sólo
nˆ = cosθ iˆ + senθ ˆj R
Derivando tˆ ⇒ presentará aceleración normal.
dtˆ dθ dθ
= −cosθ iˆ − senθ ˆj Un objeto puede experimentar la aceleración normal
dt dt dt o centrípeta y la aceleración tangencial. En las
dtˆ dθ figuras siguientes se muestran algunas
= (− cosθ iˆ − senθˆj ) combinaciones posibles para v y a para un auto en
dt dt movimiento. Para entender la aceleración,
descompóngala en las componentes paralela y
perpendicular a v . Para decir si el auto está dando
vuelta a la derecha o a la izquierda, imagínese que
usted es el conductor que se sienta con el vector de
la velocidad dirigido hacia adelante de usted. Un
Ahora bien, si tomamos un desplazamiento diminuto componente de la aceleración hacia adelante
significa que la velocidad está aumentando.
sobre la circunferencia, al que denominamos ds ,
teniendo en cuenta que arco = ángulo x radio, del
esquema adjunto se deduce que ds = Rdθ , y
además el módulo de la velocidad instantánea lo
ds
podemos expresar como v = , utilizando estos
dt
dθ v
dos últimos llegamos a =ω = ,
dt R
ˆ
dt
reemplazando en :
dt
dtˆ v
= − (cosθ iˆ + senθ ˆj ) , si observamos
dt R
detenidamente esta ecuación, comprobaremos que el

3
Movimiento en un plano y en el espacio Hugo Medina Guzmán

rapidez lineal de un pasajero en el borde es constante


e igual a 7,00 m/s. ¿Qué magnitud y dirección tiene
la aceleración del pasajero al pasar
a) por el punto más bajo de su movimiento circular?
b) por el punto más alto?
c) ¿Cuánto tarda una revolución de la rueda?

Ejemplo 1. Un avión a chorro militar de combate


volando a 180 m/s sale de una picada vertical dando
la vuelta hacia arriba a lo largo de una trayectoria
circular de 860 m de radio ¿cuál es la aceleración del
avión? Exprese la aceleración como múltiplo de g.
Solución.
v 2 180 2 m
a= = = 37,7 2
r 860 s Solución.
g v 2 7,00 2 m
a = 37,7 = 3,8 g a) a = = = 3,50 2 . La aceleración el
9,8 R 14,0 s
punto más bajo del círculo es hacia el centro, hacia
Ejemplo 2. Una rueda de 75 cm de diámetro gira arriba.
alrededor de un eje fijo con una velocidad angular de b) a = 3,50 m/s , dirigida hacia abajo., hacia el
2
1 rev/s. La aceleración es de 1,5 rev/s2.
a) Calcúlese la velocidad angular al cabo de 6 centro.
segundos. 2πR
c) Como v = ⇒
b) ¿Cuánto habrá girado la rueda en ese tiempo? T
c) ¿Cuál es la velocidad tangencial en un punto de la
2πR 2π (14,0)
periferia de la rueda en t = 6 s? T= = = 12,6 s
d) ¿Cuál es la aceleración resultante de un punto de v 7,00
la de la periferia para t = 6 s?
Solución. Ejemplo 4. La rueda de la figura del problema
rad rad anterior, que gira en sentido antihorario, se acaba de
R = 37,5 cm , ω 0 = 2π , α = 3π 2 poner en movimiento. En un instante dado, un
s s pasajero en el borde de la rueda que está pasando
a) ω (t ) = ω 0 + α t ⇒ por el punto más bajo de su movimiento circular
rad tiene una rapidez de 3,00 m/s, la cual está
ω (6 ) = 2π + 3π (6) = 20π aumentando a razón de 0,500 m/s2.
s Calcule la magnitud y la dirección de la aceleración
1 del pasajero en este instante.
b) θ (t ) = ω 0 t + α t 2 ⇒ Solución.
2
v 2 3,00 2 m m
θ (6 ) = 2π (6) + (3π )(6 2 ) = 66π rad
1 ac = = = 0,643 2 , y at = 0,5 2
2 R 14,0 s s
66π Luego:
Habrá girado = 33 vueltas. →
2π a = a c nˆ + at tˆ = − 0,643 ˆj + 0,5iˆ
c) v(t ) = Rω(t ) ⇒ m
a = a c2 + at2 = 0,643 2 + 0,5 2 = 0,814
cm s2
v(6 ) = 37,5(20π ) = 750π
s 0,5
θ = tan −1 = 37,9º
d) an = ω R ⇒
2
(6 ) 0,643
an = (20π ) (37,5) = 147894 cm/s2.
2

at = α R ⇒ an = (3π )(37,5) = 353,25 cm/s2.


a = an2 + at2 = 147894,42 cm/s2.

Ejemplo 3. Una rueda de la fortuna de 14,0 m de


radio gira sobre un eje horizontal en el centro. La

4
Movimiento en un plano y en el espacio Hugo Medina Guzmán

2π 2π
Como ωH = ,ω M = donde TH = 12 h y
TH TM
TM = 1 h y bajo la condición que estos formen un
π
ángulo de 90º, es decir, θ M −θH =
2
De (2) - (1), con θ 0 H
= θ 0M = 0 ,
θ M − θ H = (ω M − ω H )t
Ejemplo 5. Una partícula se mueve sobre una
Se encuentra para t:
circunferencia de radio R con aceleración angular
constante partiendo del reposo. Si la partícula realiza π 3
t= = h,
n vueltas completas a la circunferencia en el primer 2(ωM − ωH ) 11
segundo, determine la aceleración angular de la
Es decir, en t = 16,36 min.
partícula. Determine además el número de vueltas
Por lo tanto forman 90º a las 12:16:22 h.
que realiza la partícula durante el siguiente segundo
del movimiento.
Ejemplo 7. Dos partículas describen movimientos
Solución.
circulares de radio R = 1m, como lo muestra la
1 figura. El primero (1) parte de O con rapidez angular
Aquí θ = αt 2
2 ω = 10 rad/s constante en sentido antihorario y el
1 segundo (2) parte del reposo del mismo punto en
Entonces 2πn = ⇒ α = 4πn sentido horario con aceleración tangencial constante
2α 2
de 2 m/s . Determine cuando y donde se cruzan
1
Como θ = (4πn )t = 2πnt ,
2 2
ambas partículas.
2
Número de vueltas para t = 1
θ (1)
n(1) =
2


Número de vueltas para t = 2
θ (2 )
n(2) =
2


Durante el siguiente segundo (dos) realiza Solución.
θ (2 ) − θ (1)
= n(2 2 − 12 ) = 3n
Como el cuerpo (1) se mueve con M.C.U., la
vueltas. posición angular de este será:

θ1 = 0 + ω1 t = 10 t . (1)
Ejemplo 6. En un reloj análogo el horario y el El cuerpo (2) posee una aceleración tangencial
minutero coinciden a las 12:00:00 horas. constante y por lo tanto, se trata de un M.C.U.A.
¿A qué hora minutero y horario formarán un ángulo Debido que at = α R = 2 m/s ,α = 2rad/s . Por
2 2
de 90º?
Solución. otro lado, como parte del reposo, ω0 = 0.
1
θ 2 = − 2 t 2 = −t 2
2
El recorrido se muestra en la figura siguiente:

Como los movimientos del horario y minutero son


circulares uniformes, encontramos para la posición
angular del horario:
θ H = θ 0 H + ω H t . (1) El encuentro se produce cuando:
Análogamente para el minutero se tiene: θ1 + θ 2 = 2π ⇒
θ M = θ 0 M + ω M t . (2) 10t + t 2 = 2π

5
Movimiento en un plano y en el espacio Hugo Medina Guzmán

⎧t = 0,59 s ⎧ π
t 2 + 10t − 2π = 0 ⎨ 1 ⎪α 2 = − rad / s
2

⎩t 2 = −10,59 s ⎪ 6
La solución significativa es: t = 0,59 s ⎪ ⎛ π ⎞
Reemplazando este valor de t en ecuación (1), se ⎨ω 2 = ⎜ 4π − t ⎟rad / s
⎪ ⎝ 6 ⎠
obtiene para el ángulo de encuentro:
⎪ ⎛π π 2⎞
θ encuentro = 5,9 rad = 338,04º . ⎪θ 2 = ⎜ + 4π t − t ⎟rad
⎩ ⎝2 12 ⎠
Ejemplo 8. Dos vehículos describen la misma
trayectoria circular de radio 0,75 m. El primero está
animado de un movimiento uniforme cuya velocidad Los móviles se encontrarán cuando θ1 = θ 2
angular es de 60 rpm. y sale de la posición A cuando
se empieza a contar el tiempo. El segundo móvil está π π
animado de un movimiento uniformemente 2π t = + 4π t − t2 ⇒
acelerado cuya aceleración angular vale - π/6 rad/s2, 2 12
pasa por B dos segundos más tarde llevando una π π
velocidad angular de 120 rpm. t 2 − 2π t − =0
12 2
a) Escribir las ecuaciones del movimiento de cada
uno de los móviles. Hallar el instante y la posición
de encuentro por primera vez de ambos móviles. ⇒ t 2 − 24t − 6 = 0
b) La velocidad lineal, la velocidad angular, las
componentes tangencial y normal de la aceleración
de cada uno de los móviles en el instante de ⎧t = −0,25 s
Resolviendo ⎨
encuentro. ⎩t = 24,25 s
c) Realícese un esquema en el que se especifique los
vectores velocidad, aceleración, en dicho instante de
encuentro. La solución es 24,25 s.

El punto de encuentro es

θ1 = 2π (24,25) = 48,5π rad

π
θ2 = 0,5π + 4π (24,25) − (24,25)2 = 48,5π rad
12

Los valores son iguales, tal como esperábamos.

Solución. Como θ 1 = θ 2 = 48,5π rad , equivalente a 24


vueltas mas 1/4 de vuelta, el encuentro es en punto B.
a) Para t = 2 s el móvil 1 como su velocidad angular b) La velocidad lineal, la velocidad angular, las
es 2π rad/s estará en el punto A, y podemos componentes tangencial y normal de la aceleración
considerar ese instante como tiempo inicial, con lo de cada uno de los móviles en el instante de
que: encuentro.
Móvil 1
Móvil 1: ⎧ω1 = 2π rad / s
⎪v = ω r = 1,5π m/s
⎪ 1 1
⎧α 1 = 0 ⎨
⎪ ⎪α 1 = 0 → at1 = α 1 r = 0
⎨ω1 = 2π rad / s ⎪a = ω 2 r = 3π 2 m / s
⎪θ = 2π t rad ⎩ n1 1
⎩ 1 Móvil 2

Móvil 2:

6
Movimiento en un plano y en el espacio Hugo Medina Guzmán

⎧ 24,25
⎪ω 2 = 4π − π 6 = −0,04π rad / s
⎪⎪
⎨v 2 = ω 2 r = −0,03π m/s
⎪at 2 = α 2 r = −0,125π m/s 2

⎪⎩a n 2 = ω 22 r = 0,0012π 2 m/s 2
El móvil 2 tiene velocidad negativa, porque a l
tiempo t = 24 s su velocidad se hizo cero e inicia el
retorno, al tiempo t = 24,25 s se produce el
encuentro.
Como at = a cos θ m/s y a n = asenθ m/s ,
2 2
c) Esquema especificando los vectores velocidad,
aceleración, en el instante de encuentro.
En el instante del encuentro el esquema sería el
siguiente: La aceleración tangencial en cualquier instante, se
obtiene a partir del producto escalar del vector
→ →
aceleración a y el vector velocidad v .

→ →
v ⋅ a = va cos θ = vat

→ →
v⋅ a vx ax + v y a y
at = =
v v x2 + v y2

MOVIMIENTO CURVILÍNEO La aceleración normal, se obtiene a partir del


módulo de la aceleración a y de la aceleración
El movimiento curvilíneo es aquel en el que pueden tangencial a t .
combinarse tramos rectos y/o curvos. La extensión
de las ecuaciones en el sistema intrínseco es
inmediata sufriendo sólo una ligera modificación a 2 = a x2 + a y2 = a n2 + a t2
respecto a la aceleración. Esta adopta la expresión
→ dv ˆ v 2
a = t + nˆ donde ρ es el denominado radio ⇒ a n2 = a x2 + a y2 − at2
dt ρ
de curvatura y corresponde al radio de una hipotética
2
circunferencia en cada uno de los puntos de la ⎛v a +v a ⎞
trayectoria. Es evidente que en el caso del a n2 = a x2 + a y2 − ⎜⎜ ⎟
x x y y
movimiento circular éste no varía ya que coincide ⎜ v x + v y2
2 ⎟⎟
con el radio de la circunferencia en cada uno de esos ⎝ ⎠
dv v2
puntos. at = y an = v y ax − vx a y
dt ρ Finalmente a n =
v x2 + v y2

El radio de curvatura

v2 v2
an = ⇒ ρ=
ρ an

Ejemplo 9. El vector velocidad del movimiento de


una partícula viene dado por

( )
La figura siguiente muestra la velocidad y la →
aceleración con las coordenadas x e y para un v = (3t − 2)iˆ + 6t 2 − 5 ˆj m/s. Calcular las
determinado instante.

7
Movimiento en un plano y en el espacio Hugo Medina Guzmán

componentes tangencial y normal de la aceleración y b) las componentes normal y tangencial de la


el radio de curvatura en el instante t =2 s. velocidad y aceleración.
c) la ecuación de la trayectoria en coordenadas
Solución. cartesianas.
Solución.
dv x = 3t , y = 2t − 5t 2
v x = (3t − 2) m/s ⇒ a x = x = 3 m/s 2
dt a) vx = 3; vy = 2 - 10t; ax = 0; ay = -10;

dv y
v y = (6t 2 − 5) m/s ⇒ a y =

= 12t m/s 2 v 3iˆ + (2 − 10t ) ˆj
dt b) tˆ = = ,
v 9 + (2 − 10t )
2

En el instante t = 2 s − 3 ˆj + (2 − 10t )iˆ


⎧⎪v x = 4 m/s a x = 3 m/s 2 nˆ = tˆ × kˆ = entonces
9 + (2 − 10t )
2

⎪⎩v y = 19 m/s a y = 24 m/s
2

vt = v ⋅ tˆ = v = 9 + (2 − 10t )
2

vn = 0
v = 4 2 + 19 2 = 19,49 m/s
→ − 10t (2 − 10t )
a = 3 2 + 24 2 = 24,19 m/s 2 aT = a .tˆ =
9 + (2 − 10t )
2

La aceleración tangencial es:



− 10t (2 − 10t )
aT = a .tˆ =
9 + (2 − 10t )
→ →
v ⋅ a v x a x + v y a y 4(3) + 19(24 )
2
at = = = = 24 m/s 2
v vx + v y
2 2 19 , 49 → 30
a n = a ⋅ nˆ =
9 + (2 − 10t )
2

La aceleración normal es:


v y a x − v x a y 19(3) − 4(24) 2 5
an = = = −2 m/s 2 c) y = x − x2
vx + v y
2 2 19,49 3 9

Ejemplo 11. Una partícula se mueve en el plano xy


El radio de curvatura de acuerdo con la ley ax = 0, ay = 4cos(2t) m/s2. En
el instante t = 0, el móvil se encontraba en x = 0, y =
-1 m, y tenía la velocidad vx = 2, vy = 0 m/s.
v2 v2
an = ⇒ ρ=
ρ an a) Hallar las expresiones de r(t) y v(t).

b) Dibujar y calcular las componentes tangencial y


a = 3 2 + 24 2 = 24,19 m/s 2 normal de la aceleración en el instante t = π/6 s.
v = 4 2 + 19 2 = 19,49 m/s
Solución.

v 2 = 377 , an = −2 m/s 2 a) En t = 0

v 2 377 m
ρ= = = 188,5 m ax = 0 , vx = 2 , x=0
an 2 s

m
a y = 4 cos(2t )
Ejemplo 10. Una partícula se mueve de modo que
sus coordenadas cartesianas están dadas como , v y = 0 , y = −1 m
s2
funciones del tiempo
x = 3t , y = 2t − 5t 2 m
Determine En el eje x el movimiento es uniforme v x = 2 ,
a) las componentes cartesianas de la velocidad y de s
la aceleración. x = 2t m

8
Movimiento en un plano y en el espacio Hugo Medina Guzmán

Para encontrar el movimiento en y hay que integrar ⎧v y = 0


Para t = 2 s ⎨
m ⎩v x = 4
v y = ∫ 4 cos(2t )dt ⇒ v y = 2sen (2t )
vy t
∫0 0 s

dy = ∫ 2sen (2t )dt ⇒ y − (− 1) = 1 − cos(2t )


y t
∫−1 0

⇒ y = − cos(2t ) m

b) Las componentes tangencial y normal de la


aceleración en el instante t = π/6 s.

vx = 2 , ax = 0

m m
vy = 3 , ay = 2 at = ax = 2 an = a y = 10
s2 s2

v2 v 2 42
an = ⇒ ρ= = = 1,6m
ρ an 10

Ejemplo 13. El vector velocidad del movimiento


de una partícula viene dado por

( )

v = (3t − 2)iˆ + 6t 2 − 5 ˆj m/s.

Si la posición del móvil en el instante t = 1 s es



m r = 3iˆ − 2 ˆj m. Calcular
at = 2 cos θ = 1,31 2 ,
s
m v 2 a) El vector posición del móvil en cualquier instante.
a n = 2senθ = 1,51 2 , tan θ = x = ⇒
s vy 3 b) El vector aceleración.
θ = 49,1º
c) Las componentes tangencial y normal de la
aceleración en el instante t = 2 s.
Ejemplo 12. Un móvil se mueve en el plano xy
con las siguientes aceleraciones: ax=2 m/s2, ay =10
m/s2. Si en el instante inicial parte del origen con Dibujar el vector velocidad, el vector aceleración y
velocidad inicial vx = 0 y vy =20 m/s. las componentes tangencial y normal en dicho
instante.
Calcular las componentes tangencial y normal de la
aceleración, y el radio de curvatura en el instante t = Solución.
2 s.
a) Para el movimiento horizontal
Solución.
dv x m
m vx = 3t - 2 ⇒ ax = =3 2
a y = −10 2 v y = 20 + (− 10 )t dt s
s
dx
m Como v x = ⇒ dx = v x dt , integrando
ax = 2 v x = 2t dt
s2

9
Movimiento en un plano y en el espacio Hugo Medina Guzmán

Considere un objeto que se desplaza en el aire sin


dx = ∫ (3t − 2 )dt ⇒
t t
∫3 1 ninguna fuerza con excepción de la gravedad y de la
resistencia del aire. La fuerza de la gravedad
⎛3 7⎞
x = ⎜ t 2 − 2t + ⎟m produce una aceleración constante hacia abajo de
⎝2 2⎠ magnitud 9,80 m/s2. Como primera aproximación,
no tomemos los efectos del aire y de variaciones en
Para el movimiento vertical g . Asumiremos que la tierra es plana para el rango
horizontal de los proyectiles. A pesar de estas
simplificaciones, podemos aún obtener una
dv y m descripción bastante buena del movimiento del
v y = 6t 2 - 5 ⇒ ay = = 12t
dt s2 proyectil. El recorrido de un proyectil se llama su
trayectoria.
Si se desprecia la resistencia del aire, no hay
dy entonces aceleración en la dirección horizontal, y
Como v y = ⇒ dy = v y dt , integrando
dt a x = 0 . La aceleración en la dirección de y es
debido a la gravedad. Es constante y dirigida hacia

∫−2
t

1
t
( )
dy = ∫ 6t − 5 dt ⇒ y = 2t − 5t + 1 m
2
( 3
) abajo, así que a y = − g . Es conveniente elegir
x0 = 0 y y 0 = 0 (es decir, poner el origen en el
punto donde el proyectil comienza su movimiento).
⎛3 7⎞
( )

r = ⎜ t 2 − 2t + ⎟iˆ - 2t 3 − 5t + 1 ˆj Además, nos referimos típicamente a v 0 como la
⎝2 2⎠
rapidez inicial del proyectil. Si el proyectil es
lanzado con un ángulo θ sobre la horizontal, la

b) a = 3iˆ + 12tˆj velocidad inicial en la dirección x y la velocidad
inicial en la dirección y se pueden expresar en
términos de g y de y θ usando la trigonometría.
c) Para t = 2 s

vx = 4 m/s, vy = 19 m/s

ax = 3 m/s2, ay = 24 m/s2
a = ax2 + a y2 = 24,2m / s 2

vy 19 v0 x = v 0 cosθ , v0 y = v0 senθ
tan ϕ = = = 4,75 ⇒ ϕ = 78 o
vx 4 ax = 0 , a y = −g
Con esto:
ay 24 v x = v0 cosθ = constante , v y = v0 senθ − gt
tan θ = = = 3 ⇒ θ = 83o
ax 3 1 2
x = (v0 cos θ )t , y = (v0 senθ )t − gt
2
Ecuación de la trayectoria.
x
De la ecuación para x obtenemos t = .
v0 cos θ
Sustituyendo en la ecuación para y

⎛ g ⎞ 2
y = (tan θ )x − ⎜⎜ 2 ⎟x

⎝ 2v 0 cos θ
2

at = a cos(θ − ϕ ) = 24,1m / s 2 Corresponde a la ecuación de una parábola que pasa
por el origen. Una característica dominante del
movimiento del proyectil es que el movimiento
an = a(senθ − ϕ ) = 2m horizontal es independiente del movimiento vertical.
Así un proyectil se mueve a una velocidad constante
MOVIMIENTO PARABÓLICO.

10
Movimiento en un plano y en el espacio Hugo Medina Guzmán

en la dirección horizontal, independiente de su v02sen (2θ )


movimiento vertical. Esto se ilustra en la figura. =
g

La altura máxima que alcanza un proyectil se


obtiene con v y = 0 .

v y = v0 senθ − gt = 0 , despejando t.

Podemos entender mejor el significado de la v0senθ


1 2 t= , como vemos es igual a la mitad del
ecuación y = (v 0 senθ )t − gt viendo el g
2 tiempo de vuelo.
movimiento del proyectil de esta manera: Primero, si
no hubiera fuerza de la gravedad y aceleración hacia
1 2
abajo, en el tiempo t el proyectil movería una y máx = (v0 senθ )t − gt
distancia v 0 t en una línea inclinada recta. Si ahora 2
imaginamos con la gravedad el efecto sería hacer 2
que el proyectil se aleje de la trayectoria recta por = (v0senθ )⎛⎜ v0senθ ⎞ 1 ⎛ v0senθ
⎟⎟ − g ⎜⎜

⎟⎟
una distancia ½ gt2. De la superposición de estos ⎜ g
⎝ ⎠ 2 ⎝ g ⎠
dos efectos resulta la trayectoria parabólica como se
muestra en la figura.
Finalmente:

v02 sen 2θ
y máx =
2g

Su valor máximo se obtiene para el ángulo de


disparo θ = 90º.
Tiempo de vuelo. Poniendo y = 0
Ejemplo 14. UN BLANCO EN CAÍDA LIBRE
1
y = (v 0 senθ )t − gt 2 = 0 , despejando t, (Tiro al mono)
2 Se deja caer una botella desde el reposo en el
instante en que una piedra es lanzada desde el origen.
Determinar los valores del ángulo y de la velocidad
2v0senθ de disparo para que la piedra rompa la botella.
t2 − t=0
g (Tómese g = 9,8 m/s2)

Resolviendo obtenemos dos soluciones t = 0, que


corresponde al disparo del proyectil y

2v0senθ
t=
g

El valor máximo de t se obtiene para θ = 90º.


Cuando el proyectil se lanza verticalmente hacia
arriba, describiendo una trayectoria rectilínea a lo
largo del eje y. Solución.
Movimiento de la piedra: El movimiento
El alcance horizontal de cada uno de los curvilíneo de la piedra se realiza bajo la aceleración
3H

proyectiles se obtiene para y = 0. constante de la gravedad, es decir, es la composición


de dos movimientos
- Uniforme a lo largo del eje horizontal
⎛ 2v senθ ⎞
xmáx = (v0 cosθ )t = (v0 cosθ )⎜⎜ 0 ⎟⎟
⎝ g ⎠

11
Movimiento en un plano y en el espacio Hugo Medina Guzmán

⎧a px = 0 De estas ecuaciones, obtenemos:



Horizontal ⎨v px = v0 cos θ 2x 2y g
⎪ =− ⇒ y = − x Ecuación de una
ax g ax
⎩ x p = v 0 cos θ t
- Uniformemente acelerado a lo largo del eje vertical. línea recta.
b) En tierra, y = −124 , tal que
⎧a px = − g
⎪⎪ 2(− 124)
Vertical ⎨v px = v 0 senθ − gt t2 = − ⇒ t = 5,03 s
9,8

⎪⎩ y p = v 0 senθ t − gt / 2 c) v y = v0 y − gt = 0 − (9,8)(5,03)
2

m
Movimiento de la botella: La botella se mueve = − 49,3
verticalmente bajo la aceleración constante de la s
vx = v0 x + axt = 0 + (1,10)(5,03)
4H

gravedad.
abx = − g m
= 5,53
vbx = − gt s
y b = H − gt 2 / 2 v = vx2 + v y2 = (5,53)2 + (− 49,3)2
Choque de la piedra con la botella: Cuando se
m
produce el choque, la posición de la piedra y de la = 49,6
botella coincide. s
A = v0 cos θ t Ejemplo 16. Disparamos un proyectil desde el
origen y éste describe una trayectoria parabólica
H − gt 2 / 2 = v0 senθ t − gt 2 / 2 como la de la figura. Despreciamos la resistencia del
aire.
⇒ H = v0 senθ t Dibuja en las posiciones A, B, C, D y E el vector
Dividimos la segunda ecuación entre la primera. velocidad, el vector aceleración y las componentes
H normal y tangencial de la aceleración. (No se trata
tan θ = de dar el valor numérico de ninguna de las variables,
A sólo la dirección y el sentido de las mismas)
Para romper la botella debemos de apuntarla
¿Qué efecto producen an y at sobre la velocidad?
directamente y en el instante en el que se deja caer,
se debe lanzar la piedra. La velocidad debe tener un
valor mínimo para hacer el recorrido A, mientras la
botella esté en el aire.
2H
Esto sucede para el tiempo t = , y el
g
recorrido horizontal de la piedra debe cumplir:
⎛ 2H ⎞
v0 cos θ ⎜⎜ ⎟ ≥ A ⇒ v0 ≥ A g
⎟ cos θ Solución.
⎝ g ⎠ 2H

Ejemplo 15. Una bolsa de arena cae del reposo de v es tangente a la trayectoria
un globo de aire caliente desde una altura de 124 m
está soplando un viento horizontal, y el viento da a
bolsa de arena una aceleración horizontal constante
de 1,10 m/s2.
a) Demuestre que la trayectoria de la bolsa de arena
es una línea recta.
b) ¿Cuanto tiempo toma para llegar la tierra?
c) ¿Con qué velocidad llega a la tierra?
Solución.
1 2x
a) x = axt 2 ⇒ t 2 =
2 ax Cuando sube
1 2y
y = − gt 2 ⇒ t 2 = −
2 g

12
Movimiento en un plano y en el espacio Hugo Medina Guzmán

→ y = 150 + 180(sen π/6)t - 5t2


at y v tienen sentidos opuestos. a) Punto de caída
150 + 180(sen π/6)t - 5t2 = 0,
Cuando baja t = 19,5 s
x = 180(cos π/6)(19,5) = 3039,8m

b) Tiempo para la altura máxima
at y v tienen el mismo sentido 180(sen π/6) - 10t = 0, t = 9,0 s
entonces
ymax = 150 + 180(sen π/6)(9) - 5(9)2
at modifica el módulo de la velocidad con el = 555,0m
tiempo. El vector unitario tangente es

v π π
→ tˆ = = iˆ cos + ˆjsen
a n modifica la dirección de v v 6 6

a = −10 ˆj
Ejemplo 17. Una bala del rifle se dispara con una
Entonces
velocidad de 280 m/s hacia arriba de una superficie
plana inclinada 30° sobre la horizontal. La bala se
→ π
at = a⋅ tˆ = −10sen = −5 m/s 2
dispara con un ángulo de elevación inicial de 45° 6
sobre la horizontal (es decir, 15° sobre la superficie
plana). ¿Cuál es el alcance de la bala sobre el plano? an = a 2 − an2 = 100 − 25 = 8,66 m/s 2
Solución.
La ecuación del plano inclinado es Ejemplo 19. Un cañón de artillería lanza
y x proyectiles con una rapidez de 300 (m/s). El artillero
= tan 30° y = debe darle a un blanco que se encuentra a 8640 (m)
x 3 detrás de un cerro, cuya altura es de 1000 (m)
La ecuación de la trayectoria parabólica. ubicado a 1200 (m) del cañón. Demuestre que es
⎛ g ⎞ 2 posible darle al blanco y determine el ángulo de
y = (tan θ )x − ⎜⎜ 2 ⎟x
⎟ elevación para cumplir el objetivo.
⎝ 2v 0 cos θ
2
⎠ Solución.
La intersección de la parábola y la línea recta ocurre Supondremos que damos en el blanco entonces
cuando
gx 2
x g y = x tan α − 2 =0
= (tan θ )x − 2 x2 2v0 cos 2 α
3 2v0 cos 2 θ
5(8649)
2
v2 ⎛ 1 ⎞ 8649 tan α − =0
Para θ = 45° : x = 0 ⎜1 −
g ⎝
⎟ (300)2 cos 2 α
3⎠
Tiene dos raíces reales
Para un triángulo 30°, 60°, 90° vemos que α1 = 53,03º
3 α2 = 36,97º
x = S cos 30° = S. Debemos verificar que el disparo pasa sobre el cerro,
2
para ello evaluamos en ambos ángulos y(1200)
De aquí S =
2
3
( )v2 v2
3 − 1 0 = 0,49 0 , arriba del
g g
y1 (1200) = 1373,0 m
y2 (1200) = 777,95 m
La altura del cerro es excedida en el primer caso.
plano.
Con y 0 = 280 m/s, S = 3,90 km. Ejemplo 20. Se dispara un proyectil de modo que
su alcance horizontal es igual al triple de la altura
Ejemplo 18. Se dispara un proyectil desde la cima máxima. Encuentre el ángulo de lanzamiento.
de una colina de 150 (m) de altura con una rapidez Solución.
de 180 (m/s) y formando un ángulo de 30º con la Sabemos que
horizontal. Calcule: v02sen 2α
(a) La distancia horizontal entre el punto de xmax =
lanzamiento y el punto de caída del proyectil. g
(b) La altura máxima del proyectil con respecto al
v02sen 2α
suelo. ymax =
(c) Las componentes normal y tangencial de la 2g
aceleración al salir en el punto de disparo. Entonces
Solución:
x = 180(cos π/6)t

13
Movimiento en un plano y en el espacio Hugo Medina Guzmán

v02sen 2α v 2sen 2α
=3 0
g 2g
⇒ 2 cos α = 3senα
2
⇒ tan α = ⇒ α =33,69º
3
Ejemplo 21. Un lanza granadas tiene un alcance
máximo de 300 m. Para dar en un blanco que se
encuentra a una distancia de 400 m del lanza Solución.
granadas. Determine: a) y b)
a) La altura mínima que debe subirse el lanza
granadas.
b) La rapidez de lanzamiento.
c) El ángulo de lanzamiento,
Solución.
La ecuación de la parábola de seguridad es
v02 gx 2
y = h+ −
2 g 2v02
Sabemos también que para h = 0 la distancia
máxima alcanzable es
v02
x (0 ) = = 300 ax = 0 v x = 45 cos α
g
y para una altura h la distancia horizontal máxima a y = −10 v y = 45senα − 10t
será x = 45 cos α .t
x(h ) = (v2
0 + 2hg ) vg = 400m
0
1
y = 45senα .t − 10t 2
2
de la primera
b) Punto de impacto x = 45 , y = −45 3

v0 = 3000 = 54,77
m 45 = 45 cos α .t ⎫
s 2⎬

− 45 3 = 45senα .t − 5t ⎭
y de (54,77 )2 + 2h(10) 54,77 = 400 1 1
10 − 45 3 = 45senα . −5
a) cos α cos 2 α
h = 116,701m tan 2 α − 9 tan α + 1 − 9 3 = 0 ⇒
c) El ángulo de lanzamiento cuando el blanco está
sobre el límite de la parábola de seguridad es α 1 = 84,5 o t1 = 10,45s
v 02 α 2 = −54,5 o t 2 = 1,72 s
tan α = entonces α = 36,87o
gx t1 ⎧v x = 4,31 ax = 0
c) Para t = ⎨
2 ⎩v y = −7,46 a y = −10
Ejemplo 22. Un patinador desciende por una pista
helada, alcanzando al finalizar la pista una velocidad vx
de 45 m/s. En una competición de salto, debería tan θ = ⇒ θ = 30 o
alcanzar 90 m a lo largo de una pista inclinada 60º vy
respecto de la horizontal.
a) ¿Cuál será el ángulo (o los ángulos) α que debe
at = g cos 30 o = 5 3 m s 2
formar su vector velocidad inicial con la horizontal?. a n = gsen30 o = 5 m s 2
b) ¿Cuánto tiempo tarda en aterrizar?
c) Calcular y dibujar las componentes tangencial y
normal de la aceleración en el instante t/2. Siendo t
el tiempo de vuelo. Tomar g =10 m/s2

14
Movimiento en un plano y en el espacio Hugo Medina Guzmán

Punto de encuentro
20015 cosθ .t
1 1
30 − 9,8t 2 = 15senθ t − 9,8t 2
2 2
⎧ 30 θ = 56,3 o

⎨tan θ =
⎩ 20 y = 1,69m

b)

Ejemplo 23. Se deja caer una botella desde el


reposo en la posición x =20 m e y =30 m. Al mismo
tiempo se lanza desde el origen una piedra con una
velocidad de 15 m/s.

a) Determinar el ángulo con el que tenemos que


lanzar la piedra para que rompa la botella, calcular la
altura a la que ha ocurrido el choque.

b) Dibujar en la misma gráfica la trayectoria de la


piedra y de la botella. (Tomar g = 9,8 m/s2). Ejemplo 24. Desde un cañón que está sobre un
plano inclinado un ángulo α con la horizontal se
Solución: dispara un proyectil. Este sale con una velocidad v 0
formando un ángulo θ con el plano horizontal.
a) Encontrar.
a) El punto más alto al que llega el proyectil.
b) El alcance del proyectil.
Solución.

a) v 0 x = v 0 cos θ v 0 y = v 0 senθ
Movimiento de la botella
v y = v0 senθ − gt
x = 20 La altura máxima se produce cuando v y = 0
ax = 0 vx = 0
1 v02 sen 2θ
a y = −9,8 v y = −9,8t y = 30 − 9,8t 2
2 y máx =
2g
Con ese valor,
Movimiento de la piedra
⎛ v senθ ⎞ v 02
x = v0 cos θ ⎜⎜ 0 ⎟⎟ = sen 2θ
ax = 0 vx = 15 cosθ ⎝ g ⎠ 2g
a y = −9,8 v y = 15senθ − 9,8t v2
y = x tan α = 0 sen 2θ tan α
x = 15 cosθ t 2g
v2
h = y máx − y = 0 (sen 2θ − sen 2θ tan α )
1
y = 15 senθ t − 9,8t 2
2 2g
b) El alcance máximo S .
x = v0 cos θ t y = v 0 senθ t − 12 gt 2
Ecuación del plano en función de t y = x tan α
Dividiendo y x :

15
Movimiento en un plano y en el espacio Hugo Medina Guzmán

c) El punto de caída ocurre para z = 0 y la distancia


y v0 senθ t − 12 gt
2
= = tan α ⇒ v02⎡1 + cos 2θ ⎤
x v 0 cos θ t vale x(θ ) = ⎢ tan α + sen 2θ ⎥
gsenα⎣ ⎦
v0 senθ t − 12 gt 2 = tan α v 0 cos θ t
dx(θ )
Resolviendo encontramos el tiempo para el que el La distancia máxima ocurre para = 0.
proyectil toca tierra: dθ
2v0
t= (senθ − cos θ tan α )
g
El valor de x cuando el proyectil toca tierra es:
Ejemplo 26. Se lanza una pelota verticalmente
2v02
x = v0 cosθ t = cosθ (senθ − cosθ tan α ) hacia arriba con una velocidad de 20 m/s desde la
azotea de un edificio de 50 m de altura. La pelota
g
además es empujada por el viento, produciendo un
Y el alcance S es:
movimiento horizontal con aceleración de 2 m/s2.
x 2v02 cos θ
S= = (senθ − cos θ tan α ) Calcular:
cos α g cos α
a) La distancia horizontal entre el punto de
Ejemplo 25. La figura muestra una colina inclinada lanzamiento y de impacto.
un ángulo α respecto a la vertical y la trayectoria
de un proyectil. El proyectil se lanza desde el origen b) La altura máxima
O con una velocidad inicial de módulo v 0 y que
c) El valor de las componentes tangencial y normal
forma un ángulo θ con el eje z (perpendicular al de la aceleración cuando la pelota se encuentra a 60
plano). El eje x se toma tangente al plano apuntando m de altura sobre el suelo.
hacia abajo.
a) Tome el sistema de referencia indicado en la
Tómese g =10 m/s2.
figura y halle las componentes de los vectores
aceleración, velocidad y posición del proyectil en
función del tiempo.
b) Halle la máxima separación entre el proyectil y la
colina.
c) Halle la distancia entre el origen y el punto de
caída del proyectil sobre la colina. Demuestre que
esa distancia es máxima si θ = α / 2 .

Solución.

1 2
a x = 2 , v x = 2t , x = 2t
2

a y = −10 , v y = 20 + (− 10 )t ,
Solución. 1
a) a x = g cos α , v x = g cos α t + v 0 senθ , y = 20t + (− 10)t 2
2
1
x= g cos α t 2 + v0 senθ t a) Punto de impacto
2
a z = − gsenα , v z = − gsenα t + v0 cosθ ,
y = -50 ⇒ t = 5,74 s ⇒ x = 32,97 m
1
z = − gsenα t 2 + v 0 cosθ t
2 b) altura máxima
b) La máxima separación ocurre para v z = 0 y vale
v02 cos 2 θ v y = 0 ⇒ t = 2 s ⇒ y = 20 m
z=
2 g 2senα
hmáxima = 70 m sobre el suelo.

16
Movimiento en un plano y en el espacio Hugo Medina Guzmán

c) h = 60 ⇒ y = 10 m ⇒ t1 = 0,59 s t2 = 3,41 s ϕ = ϕ2 − ϕ1 = 14,5o


⎧⎪a n = a. cos ϕ = 9,87 m, /s 2

⎪⎩at = a.senϕ = 2,55 m/s 2

Ejemplo 27. Nos encontramos en la antigua


Suiza, donde Guillermo Tell va a intentar ensartar
con una flecha una manzana dispuesta en la cabeza
de su hijo a cierta distancia d del punto de disparo
(la manzana está 5 m por debajo del punto de
lanzamiento de la flecha). La flecha sale con una
velocidad inicial de 50 m/s haciendo una inclinación
de 30º con la horizontal y el viento produce una
aceleración horizontal opuesta a su velocidad de 2
⎧v x = 1,17 ⎧a x = 2 m/s2.
t1 = 0,59 s ⎨ ⎨
⎩v y = 14,14 ⎩a y = −10 a) Calcular la distancia horizontal d a la que deberá
a = 2 2 + 10 2 estar el hijo para que pueda ensartar la manzana.

b) Hállese la altura máxima que alcanza la flecha


vx 1,17
tan ϕ1 = = = 0,08 ⇒ ϕ1 = 4,7 o medida desde el punto de lanzamiento. (g = 9,8
v y 14,14 m/s2)
ay 10
tan ϕ 2 = = = 5 ⇒ ϕ 2 = 78,7 o
ax 2

ϕ = ϕ 2 − ϕ 1 = 73o
⎧⎪a n = a. cos ϕ = 2,81 m/s 2

⎪⎩at = a.senϕ = 9,80 m/s 2
Solución.

⎧v x = 6,83 m/s ⎧a x = 2
t 2 = 3,41s ⎨ ⎨
⎩v y = −14,14 m/s ⎩a y = −10 ax = −2 , v x = 50 cos 30 o − 2t ,
a = 2 2 + 10 2 1
x = 50 cos 30 o − 2t 2
2
vy 14,14
tan ϕ1 = = = 2,07 ⇒ ϕ 1 = 64,2 o
vx 6,83 a y = −9,8 , v y = 50sen30 o − 9,8t ,
ay 1
10 y = 50sen30o − 9,8t 2
tan ϕ 2 = = = 5 ⇒ ϕ 2 = 78,7 o 2
ax 2
Punto de impacto x = d, y = -5

-5 =25 t -4,9 t2 ⇒ t = 5,29 s ⇒ x = 201,23 m

17
Movimiento en un plano y en el espacio Hugo Medina Guzmán

Máxima altura vy = 0

50sen30º - 9,8t = 0 ⇒ t = 2,55 s ⇒ y = 31,89 m

Ejemplo 28. Un paraguas abierto mojado se


sostiene hacia arriba como se muestra en la figura y
se gira sobre la manija a razón uniforme de 21
revoluciones en 44 s. Si el borde del paraguas es un
círculo 1 m de diámetro, y la altura del borde sobre Sea un objeto P determinado por un observador en el
el piso es 1,5 m, hallar donde las gotas del agua al →
hacer girar del borde tocan el piso. sistema S por r = xiˆ + yˆj + zkˆ y por un
observador en el sistema S' por

r ' = x' iˆ + y ' ˆj + z ' kˆ como se muestra en la figura.

Las ecuaciones de transformación de Galileo que


relacionan las observaciones desde los sistemas S y
S' son
x = x'+ Vt , y = y ' , z = z ' t = t '
Aquí se supone que puede establecerse una escala de
tiempo absoluta aplicable a ambos marcos de
referencia de manera que t = t ' . Esto sucedería si la
Solución. velocidad de la luz fuera infinita (Debemos
La velocidad angular del paraguas es reconocer que las escalas de tiempo asociadas a dos
21 × 2π rad marcos de referencia no son los mismos si existe
ω= = 3rad / s movimiento relativo entre ellos es uno de los
44s principios fundamentales de la teoría especial de la
La velocidad tangencial de las gotas de agua que relatividad propuesta por Einstein en 1905).
salen del borde del paraguas es Vectorialmente podemos representar la
v0 = rω = (0,5)(3) = 1,5m / s transformación de Galileo como
→ → →
Para calcular el tiempo en que la gota llega al piso
r = r '+ V t .
1 2
usamos h = gt Derivando las relaciones anteriores podemos obtener
2 la relación de la velocidad.
2h 2(1,5) dx dx'
= + V ⇒ v x = v' x ' + V
⇒ t= = = 0,553m dt dt
g 9,8
El alcance horizontal de la gota es dy dy '
= ⇒ v y = v' y '
x = v0t = (1,5)(0,55) = 0,83 m; dt dt
y el locus de las gotas es un círculo de radio dz dz '
= ⇒ v z = v' z '
R= (0,5)2 + (0,83)2 = 0,97 m. dt dt
→ → →
VELOCIDAD Y ACELERACIÓN RELATIVAS. Vectorialmente v = v '+ V
Derivando nuevamente obtenemos la relación de la
Movimiento Relativo de Traslación Uniforme. aceleración
La Relatividad de Galileo dv x dv' x ' dV dV
= + ⇒ a x = a' x ' +
Consideramos dos sistemas de referencia S y S', S' dt dt dt dt
tiene un movimiento de traslación rectilíneo dv y dv' y '
uniforme con respecto a S; S' se aleja de S con una = ⇒ a y = a' y '

dt dt
velocidad V = viˆ

18
Movimiento en un plano y en el espacio Hugo Medina Guzmán

dv z dv' z '
= ⇒ a z = a' z '
dt dt

Si la velocidad V del sistema S' es constante,

dV → →
= 0 y a = a'
dt
Estas relaciones encontradas son de aplicación
general si S y S' están animadas por un movimiento
relativo cualquiera, como se muestra en la figura b) Si se observa el mismo proyectil desde un sistema
siguiente de referencia situado en el suelo S con un origen en
el lugar de lanzamiento (para t 0 = 0 , O = O'),
entonces las posiciones, las velocidades y las
aceleraciones respecto de O estarán dadas por la
transformación de Galileo. En este caso la velocidad
inicial v 0 vista desde el suelo será
r
v 0 = Viˆ + v y 0 ˆj v0 = V 2 + v y20
Las ecuaciones son: v y0
→ → → → → →
θ 0 = tan −1
V
r = r '+ V t , r ' = r − V t La trayectoria será una parábola tal como se ve en la
→ → →
v = v'+ V figura siguiente
→ →
a = a'
Ejemplo 29. Desde la plataforma de un camión en
r
movimiento horizontal V constante se lanza un
proyectil directamente hacia arriba con una
r
velocidad v 0 . ¿Cómo será visto el movimiento del
proyectil por: La componente horizontal del movimiento del
a) un observador situado en el camión (sistema S')? proyectil es igual al movimiento del cañón, de modo
b) un observador situado en el suelo (sistema S)? que cuando cae el proyectil coincidirá con el cañón.
Solución.
a) El tiempo se mide desde el momento del Ejemplo 30. El observador O suelta una piedra del
lanzamiento t 0 = 0 , cuando el proyectil se eleva trigésimo piso de un rascacielos. El observador O’,
con velocidad v 0 . La componente horizontal de la descendiendo en un ascensor a velocidad constante
de V = 5,0 m/s, pasa el trigésimo piso justo cuando
velocidad coincide con la velocidad V del camión. se suelta la piedra. Al tiempo t = 3,0 s después de
El observador O' en el camión verá únicamente la que se suelta la piedra, hallar:
componente vertical v ' y '0 , la componente horizontal a) La posición, la velocidad, y la aceleración de la
piedra relativa a O.
será v ' x '0 = 0 . b) La posición, la velocidad, y la aceleración de la
Para un instante t cualquiera piedra relativa a O’.
Solución.
x' = 0 y ' = v' y '0 t − ½ gt 2 a) Para O, la posición de la piedra está dada por:
v' x ' = 0 v' y ' = v' y '0 − gt 1 2
x = x0 + v0 t + at
a' x ' = 0 a' y = − g 2
Donde x = 0 en el trigésimo piso con la dirección
hacia abajo como la dirección positiva de x. Así, en
t = 3,0 s,
1
x = 0+0+ (9,8)(3,0)2 = + 44 m/s
2
También, v = v0 + at da
v = 0 + 9,8 m/s2 x 3,0 s = +29 m/s.

19
Movimiento en un plano y en el espacio Hugo Medina Guzmán

La aceleración de un cuerpo en caída libre, según el Sea W la rapidez del río y u la rapidez de los botes
observador O que está inmóvil con respecto a la respecto al agua, (igual en ambos), entonces
tierra, se sabe que la aceleración gravitacional es V1 = u -W
constante. V2 = u + W
(De hecho, esto es la base de la validez de los dos de modo que
cálculos anteriores.) V2 − V1
Así tenemos: W =
a = +g = +9,8 m/s2. 2

b) O’ mide la posición x', relativa a x por medio de Ejemplo 34. Un bote cruza un río que mide de
la ecuación x' = x - Vt. ancho a en el cual la corriente fluye con una rapidez
Luego, después de 3,0 s, uniforme de u. El botero mantiene una orientación
x' = 44 m – 5,0 m/s x 3,0 s = +29 m. (es decir, la dirección en la cual apunta el bote)
Es decir, la piedra se localiza 29 m debajo del perpendicular al río y al motor fijo para dar una
observador O’ después de 3,0 s. rapidez constante de v m/s con respecto al agua. De
La velocidad de la piedra relativa a O' es v' = v -V; acuerdo a los datos
de aquí, en t =3,0s, (a) ¿Cuál es la velocidad del bote respecto a un
v' = 29 m/s – 5,0 m/s = +24 m/s observador detenido en la orilla?
Puesto que V es constante, a' = a, y a'= +g = +9,8 (b) ¿Hasta dónde estará el bote, medido corriente
m/s2. abajo paralelamente al río, desde la posición inicial
El observador O’ ve la piedra con la misma hasta cuando alcance la orilla opuesta?
aceleración vista por O. (en general, las Solución.
aceleraciones son iguales en todos los sistemas a)

inerciales.)
V = uiˆ + vˆj
Ejemplo 31. Un automovilista viaja hacia el oeste b) La componente de la velocidad absoluta
sobre la Ruta Interestatal 80 a 80 km/h y es seguido perpendicular al río determine el tiempo de cruce de
por un auto patrulla que viaja a 95 km/h. a
a) ¿Cuál es la velocidad del automovilista respecto acuerdo a t =
al auto patrulla?
v
Por lo tanto el bote avanza paralelamente al río una
b) ¿Cuál es la velocidad del auto patrulla respecto al
distancia
automovilista?
u
Solución. d = ut = a
Si el Oeste indica el sentido positivo entonces v
a) 80 - 95 = -15 km/h
b) 95 - 80 = 15 km/h Ejemplo 35. Un comprador que está en una tienda
puede caminar sobre una escalera mecánica en 30 s
Ejemplo 32. Un río tiene una rapidez uniforme de cuando está detenida. Cuando la escalera mecánica,
0,5 m/s. Un estudiante nada corriente arriba una funciona normalmente, puede llevar al comprador
distancia de 1 km y regresa al punto de partida. Si el sin caminar al siguiente piso en 20 s. ¿Cuánto
estudiante puede nadar con una rapidez de 1,2 m/s tiempo le tomaría al comprador al subir caminando
en agua tranquila, ¿cuánto dura el recorrido? con la escalera mecánica en movimiento? Suponga
Compare este resultado con el tiempo que duraría el que el comprador hace el mismo esfuerzo al caminar
recorrido si el agua estuviera tranquila. sobre la escalera mecánica en movimiento o cuando
Solución. está parada.
La rapidez absoluta (respecto a la ribera) cuando Solución.
nada corriente arriba es 1,2 – 0,5 = 0,7 y cuando Sea L el largo de la escalera. Entonces la velocidad
nada corriente abajo es 1,2 + 0,5 = 1,7 entonces el de la persona respecto a la escalera es
tiempo de ida y vuelta será
L
1000 1000 v' = .
t= + = 2016,81 s = 0,56 h 30
0,7 1,7 Sea ve la velocidad de la escalera. Ella corresponde a
la de la persona cuando no camina, es decir
Ejemplo 33. Dos remeros en idénticas canoas L
ejercen el mismo esfuerzo remando en un río, uno ve =
corriente arriba (y se mueve corriente arriba), 20
mientras que el otro rema directamente corriente Si la escalera funciona y la persona camina, entonces
abajo. Un observador en reposo sobre la orilla del L L L
río determina sus rapideces que resultan ser de V1 y v = ve + v' = + =
20 30 t
V2 respectivamente. Determine en términos de los
de donde el tiempo será
datos la rapidez de las aguas del río.
t = 12 s
Solución.

20
Movimiento en un plano y en el espacio Hugo Medina Guzmán

Ejemplo 36. El piloto de un avión observa que la


brújula indica que va dirigiéndose hacia el oeste. La
rapidez del avión respecto al aire es de 150 km/h. Si
existiera un viento de 30 km/h hacia el norte, calcule
la velocidad del avión respecto a la Tierra.

a) La dirección en la que debe dirigirse el avión está


dada por el ángulo θ.
vv 50
Solución. cosθ = = = 0,25 ⇒ θ = 75,5º
La velocidad del viento es vv = 30 km/h y la rapidez v 200
del avión respecto al aire es v’ = 150 km/h. Debe dirigirse 75,5º dirección N-O.
Pero b) Su velocidad respecto a la Tierra es:
→ → → →
v = vˆj = 30iˆ + v' v = v' − 50iˆ
→ Y su rapidez respecto a tierra es:
De donde v' = vˆj − 30iˆ
y si tomamos magnitudes v = v'2 −502 = 200 2 − 502
= 193,6 km/h
150 = v 2 + 302 ⇒
v = 146,969 km/h Ejemplo 38. Un niño en peligro de ahogarse en un
río está siendo llevado corriente abajo por una
Ejemplo 37. El piloto de un avión desea volar corriente que fluye uniformemente con una rapidez
hacia el oeste en presencia de un viento que sopla de 2,5 km/h. El niño está a 0,6 km de la orilla y a 0,8
hacia el sur a 50 km/h. Si la rapidez del avión km corriente arriba de un embarcadero cuando un
cuando no sopla el viento es de 200 km/h, bote de rescate se pone en camino.
a) ¿en qué dirección debe dirigirse el avión? a) si el bote procede a su rapidez máxima de 20
b) ¿cuál debe ser su rapidez respecto a la Tierra? km/h con respecto al agua, ¿cuál es la dirección,
relativa a la orilla, que deberá tomar el conductor del
bote?
b) ¿Cuál es el ángulo que hace la velocidad, v, del
bote con respecto a la orilla?
c) ¿Cuánto tiempo le tomará al bote para alcanzar al
niño?
Solución.
a) Considerando al bote y al niño dentro del río se
encuentran en un sistema inercial S’.
En este sistema el niño esta en reposo y el bote se
Solución. mueve con su velocidad, para poder alcanzar en el
La velocidad del viento es vv = 50 km/h hacia el sur menor tiempo el bote de enfilar con un ángulo
y la rapidez del avión respecto al aire es v’ = 200 relativo a la orilla dado por:
km/h.
0,6
Para poder volar directamente hacia el oeste con tan θ = = 1,5 ⇒ θ = 37 º
respecto a tierra debe compensar el arrastre 0,8
producido por el viento, tal como se muestra en la
figura siguiente.

b) La velocidad del bote v, con respecto a la orilla


vx = −20 cos 37 º +2,5 = −13,5 (1)

21
Movimiento en un plano y en el espacio Hugo Medina Guzmán

v y = 20sen37º = 12 (2) V 2 cot 2 α


h=
Dividiendo (2) : (1) 2g
vx 12
= tan φ = = −0,89
vy − 13,5 Ejemplo 41. La brújula de un avión indica que se
está dirigiendo hacia el este con una velocidad de
⇒ φ = −41º 400 km/h. La información de tierra indica que el
viento sopla hacia el norte con una velocidad de 300
km/h. ¿cuál es la velocidad del avión con respecto a
tierra?
Solución.
En este caso tenemos dos sistemas, el sistema tierra
(S) y el sistema aire (S') que se mueve con una
velocidad de 300 km/h respecto a tierra.
c) El tiempo que le tomará al bote para alcanzar al
niño:
d
d = vt ⇒ t =
v
Siendo v = 20 km/h y
d = 0,82 + 0,62 = 1,0 km
1
t= = 0,05 h = 3 min
20

Ejemplo 39. Desde el techo del carro de un tren que V = 300 ˆj
está acelerando hacia el norte a una razón de 2,5 →
m/s2 se suelta y cae un perno. ¿Cuál es la aceleración v' = −400iˆ
del perno con respecto a: → →
a) el carro del tren? R =V t
b) la estación? → →
Solución: r ' = v' t
Si y es la vertical hacia arriba y x es la dirección de La posición del avión visto desde O es
la aceleración del tren, entonces → → → → →
a) r = R+ r ' = V t + r '
→ La velocidad es
a ' = −2,5iˆ − 9,8 ˆj . →
b)
→ dr → →

v= = V + v'
dt
a = −9,8 ˆj →
Luego v = 300 ˆj + 400iˆ
Ejemplo 40. Un estudiante de la Facultad de Su magnitud
Ingeniería pasea sobre el vagón de un tren que viaja
km
a lo largo de una vía horizontal recta a una rapidez v = 300 2 + 400 2 = 500
constante de V m/s. El estudiante lanza una pelota al h
aire a lo largo de una trayectoria que inicialmente 300
forma un ángulo de α° con la horizontal y está en α = tan -1 = 37 o
línea con la vía. El profesor del estudiante, que está
400
parado cerca sobre la tierra, observa que la pelota El avión se dirige hacia el NE formando un ángulo
sale verticalmente. ¿Qué altura subirá la pelota? de 37° con la dirección este, el módulo de la
Solución. velocidad es 500 km/h.
Si V’ es la rapidez inicial de lanzamiento relativa al
tren, entonces en la dirección x tenemos: Ejemplo 42. Un nadador recorre una piscina de 100
m en 2 min. Va a nadar en un río observando antes
Vx = V’ cos α V = 0
de lanzarse e al agua, que un trozo de madera que
Porque el profesor observa que sale verticalmente.
flota en ella recorre 20 m en 1 minuto. Calcular el
V tiempo que tardará el nadador en recorrer 100 m en
V '=
cos α el río, según vaya a favor o en contra de la corriente.
Luego Solución.
Vy = V’y = V’sinα= V cot α
Subirá una altura h dada por

22
Movimiento en un plano y en el espacio Hugo Medina Guzmán

La velocidad del nadador es:


s 100 m
vn = = = 50
t 2 min
m
La velocidad del agua del río es: vr = 20
min
La velocidad nadando a favor de la corriente es:
v1 = v n + v r = 50 + 20 = 70 m/min Si en la figura y es el ancho del río y x el avance
Y el que tarda en recorrer 100 m es: producido por la corriente, el camino recorrido por
s 100 la lancha es s.
t1 = = = 1 min 26 s
v1 70 s = x 2 + y 2 = 60 2 + 50 2
La velocidad nadando en contra de la corriente es: = 78,1 m
v 2 = v n − v r = 50 - 20 = 30 m/min
Ejemplo 45. La velocidad que provocan unos
Y el que tarda en recorrer 100 m es:
remeros a una barca es de 8 km/h, La velocidad del
s 100 agua de un río es 6 km/h, y el ancho de tal río 100 m.
t2 = = = 3 min 20 s
v2 30 a) Suponiendo la posición de la proa perpendicular a
las orillas, calcular el tiempo que tarda la barca en
cruzar el río y la distancia a que es arrastrada, aguas
Ejemplo 43. Un acorazado navega con rumbo NE a
abajo, por la corriente.
una velocidad de 50,56 km/h. Suena zafarrancho de
combate y uno de los tripulantes marcha corriendo
de babor a estribor para ocupar su puesto, a una
velocidad de 10 km /h. Calcular el valor de la
velocidad resultante y su dirección.
Solución.

b) ¿En qué dirección debe colocarse la proa de la


barca para alcanzar el punto de la orilla opuesta
situado enfrente del de partida? (punto de partida y
llegada en la perpendicular común a las orillas),

km km
v A = 55,56 , vT = 10
h h
V = 55,56 + 10 = 56,45 km/h
2 2

c) ¿Qué velocidad, respecto a la tierra, lleva la barca


55,56 en los dos casos estudiados?
tan α = ⇒ α = 79,8
10 d) ¿Cuánto tarda en atravesar el río?.
ϕ = 79,8 − 45 = 34,8 = 34º 47’ 49’’ Solución.
a) vx = vr = 6 km/h, vy = vb = 8 km/h
La dirección será 90º - ϕ = 55º 12’ 11’’
y 0,1
y = vyt ⇒ t = = h = 45 s
Ejemplo 44. Una pequeña lancha atraviesa un río de vy 8
50 m de. Anchura, al mismo tiempo la corriente lo La distancia a que es arrastrada por la corriente:
arrastra 60 m aguas abajo. ¿Qué camino ha 0,1
recorrido? x = vxt = 6 × Km = 75 m
Solución. 8
b) Para que la barca vaya en la dirección de v2 la
componente horizontal de vb ha de ser igual a 6 km/h.
6
vb senϕ = vr ⇒ senϕ =
8
⇒ ϕ = 48o 35’
c) En el primer caso

23
Movimiento en un plano y en el espacio Hugo Medina Guzmán

b y c) La velocidad del bote con respecto a la orilla


v1 = v x2 + v y2 = 6 2 + 8 2 → → →
= 10 km/h es v neta = v B + v R .
En el segundo caso: → →
v2 = vbcos ϕ = 8cos 48º 35’ Como v B y v R son perpendiculares, tenemos
= 5,3 km/h v neta = v B2 + v R2
d) En el primer caso son 45 s ya calculados.
En el segundo caso: = 3 + 4 = 5m / s .
2 2

y 0,1 El ángulo ϕ mostrado en la figura se determina por


t= = h = 68 s
v2 5,3 vR
tan ϕ = .
vB
Ejemplo 46. Una canoa de 2,5 m de larga está junto
Para las velocidades dadas encontramos ϕ = 53,1 .
o
a la orilla de un río y perpendicularmente a ella. Se
pone en marcha con una velocidad de 5 m/s y al El bote se mueve a lo largo de una línea dirigida
llegar a la orilla opuesta ha avanzado en el sentido 53,1º río abajo.
de la corriente 23,4 m.
a) Calcular la velocidad del agua sabiendo que el río
tiene una anchura de 100 m.
b) Si la canoa marcha a lo largo del río, determinar
el camino recorrido en 1 minuto según vaya en el
sentido de la corriente o en sentido contrario.
Solución .
a) La proa de la canoa debe recorrer un espacio en
dirección perpendicular al río: d) Haciendo D = distancia río abajo, tenemos
y = 100 – 2,5 = 97,5 m D v 4
siendo y = vc t = 97,5 m = R = , tal que D = 133 m.
el río arrastra a la canoa x = 23,4 m = vr t 100 v B 3
dividiendo las dos anteriores
97,5 5 Ejemplo 48. Un submarino de propulsión
= ⇒ vr = 1,2m / s convencional (Diesel) sufrió un incendio en el
23,4 v r Atlántico norte después de salir de Inglaterra.
b) v1 = v c + v r = 5 + 1,2 = 6,2 m/s Debido a un huracán no era posible enviar barcos ni
aviones para ayudar al submarino diesel. La marina
⇒ x1 = 6,2 x 60 =372 m decidió enviar un submarino de propulsión nuclear
v 2 = v c − v r = 5 - 1,2 = 3,8 m/s para ayudar al de propulsión Diesel. El submarino
⇒ x2 = 3,8 x 60 =228 m diesel se encuentra al Sur a 500 km de distancia del
submarino nuclear (ver figura). La rapidez del
Ejemplo 47. Un bote de remos se dirige submarino nuclear respecto al agua es de 54 km/h.
perpendicular a la orilla de un río. Los remos Además, hay una corriente marina de 36 km/h que
pueden propulsar el bote con una velocidad de 3,0 se mueve al NE formando un ángulo de 30° respecto
m/s con respecto al agua. El río tiene una corriente al norte. (Asuma que el eje x es el eje DE, y el eje y
de 4,0 m/s. es el NS).
(a) Construya un diagrama en el cual las dos a) Si V es el módulo de la velocidad del submarino
velocidades se representen como vectores. nuclear visto desde tierra, escriba en forma vectorial,
(b) Encuentre el vector que representa la velocidad usando el sistema de coordenadas x -y, la velocidad
del bote con respecto a la orilla. del submarino nuclear respecto a tierra para que
(c) ¿Qué ángulo forma este vector con la dirección llegue al submarino diesel y la velocidad de la
en la cual el bote está señalando? corriente marina con respecto a tierra.
(d) Si el río tiene 100 m de ancho, determínese cuan b) Halle la velocidad del submarino con respecto a la
lejos río abajo del punto del lanzamiento el bote corriente de agua.
llega al orilla opuesta. c) Calcule el módulo de la velocidad V.
Solución. d) Halle el tiempo en el cual los marineros son
Solución: rescatados.
a) Diagrama.

24
Movimiento en un plano y en el espacio Hugo Medina Guzmán

a) ¿A qué distancia del poste contada a lo largo de la


vía, y a qué distancia de esta chocará el cuerpo con
el suelo?

b) Realícese un esquema de la trayectoria seguida


por el cuerpo

Dato: la altura inicial del objeto sobre el suelo es de


2,45 m

Solución.
a) Si V es el módulo de la velocidad del submarino
nuclear visto desde tierra, escriba en forma vectorial,
usando el sistema de coordenadas x-y, la velocidad
del submarino nuclear respecto a tierra para que
llegue al submarino diesel y la velocidad de la Solución.
corriente marina con respecto a tierra.
km m
Velocidad del tren v y = 108 = 30 ,
h s

km m
Velocidad de la piedra vx = 36 = 10
h s
m
g ≈ 10
s2

b) Halle la velocidad del submarino con respecto a la a) El movimiento de la piedra lanzada está dada por
corriente de agua. las ecuaciones:

V sR = − 54 sen α iˆ − 54 cos α ˆj ,
1
→ x = 10t , y = 30t , z = 2,45 − 10t 2
v c = 36sen30º iˆ + 36 cos 30º ˆj = 18iˆ + 31,18 ˆj 2
18 1
− 54senα + 18 = 0 ⇒ senα = = Cuando la piedra llega al suelo z = 0
54 3
⇒ cos α = 0,94
1
→ z = 0 = 2,45 − 10t 2 ⇒ t = 0,7 s
V sT = (− 54 cos α + 31,18) ˆj 2
= (− 50,76 + 31,18) ĵ
Distancia del poste medida desde la vía:
= 19,18 ĵ y = 30t = 30(0,7) = 21m
c) Calcule el módulo de la velocidad V.
19,18 km/hora
d) Halle el tiempo en el cual los marineros son Distancia de la vía al punto de caída:
rescatados. x = 10t = 10(0,7) = 7m
d 500
t= = b)
V 19,18
= 26 horas

Ejemplo 49. Desde el interior de un tren que viaja a


108 km/h, un niño lanza un objeto por una ventana
con una velocidad de 36 km/h, horizontalmente y
perpendicularmente a la marcha del tren, justo en el
momento en que pasa en frente de un poste
indicador.

25
Movimiento en un plano y en el espacio Hugo Medina Guzmán

PREGUNTAS Y PROBLEMAS

1. La velocidad de la corriente de un río aumenta en → →


proporción a la distancia de la orilla y alcanza su b) r 1 = r 2 = 840iˆ + 36 ˆj , t = 12
valor máximo v 0 en el medio. Cerca de la orilla la
6. Las posiciones de dos partículas P1 y P2 están
velocidad es cero. Un bote que navega en el río tiene
( )

una velocidad u relativa al agua, constante y dadas por r 1 = 5 + 3t + 2t iˆ ,
2
perpendicular a la corriente.
r 2 = (t + 5t 2 )iˆ .

a) Encontrar la distancia que fue arrastrando el bote
al cruzar el río de ancho C.
b) Determinar la trayectoria del bote a) ¿En qué instante chocarán las dos partículas?
b) ¿Cuál es la diferencia de velocidades en ese
v0
Respuesta. a) d = C instante?
2u Respuesta: a) t = 2 b) 8

2. Un automovilista entra en una curva de 150 m de 7. El movimiento de una partícula está definido por
radio, una velocidad de 72 km/h. Accionando los el vector posición
frenos hace disminuir su velocidad de modo →
2 r = Rsenb t iˆ + Ct ˆj + Rcosb t kˆ . Determinar.
uniforme a razón de 1,5 m/s .
Determinar el módulo de la aceleración del a) La velocidad y aceleración de la partícula.
automóvil cuando su velocidad es de 63 km/h. b) La trayectoria de la partícula.
2 c) El radio de curvatura.
Respuesta: 2,53 m/s
Respuesta. a) v = C 2 + R 2 b 2 , a = Rb 2 ,
3. Las ecuaciones paramétricas del movimiento de
C2
una partícula son x = R cos ω t , y = Rsenω t , b) Helicoide, c) ρ = R +
z = vt . R, ω, v son constantes. Rb 2
Probar que se trata de un movimiento uniforme,
8. El movimiento de una partícula está definido por
dibujar la trayectoria.
el vector posición

Respuesta: Movimiento helicoidal con velocidad r = 0,1senπ t iˆ + 0,25cos2π t ˆj , r en metros y t
angular ω y subiendo con velocidad v.
en segundos:
a) Determinar la velocidad y aceleración para t = l s.
4. Dadas las ecuaciones paramétricas de un
b) Demostrar que la trayectoria de la partícula es una
movimiento x = Asenω t , y = A cos ω t , parábola.
a) Escribir la ecuación del movimiento. → →
b) La ley horaria Respuesta. a) v = −0,1π iˆ m/s , a = 0
c) La trayectoria
b) y = 0,025 − 5 x
2

Respuesta. a) r = Asenω t iˆ + Acosω t ˆj , b)
9. La aceleración de un cuerpo es:
s = ωAt , c) x 2 + y 2 = A 2 →
( )
a = 3iˆ + 2 ˆj + kˆ cm/s 2
5. Dos objetos se mueven en el plano xy de acuerdo a) Si el cuerpo parte del reposo ¿Cuál es su
( )
→ velocidad después de 3 segundos?
a r 1 = 4t + 3t + 228 iˆ + (2t + 12 ) ˆj y
2
b) ¿Cuál es su posición después de 10 segundos?

r 2 = (8t 2 + 11t − 444 )iˆ + (5t − 24) ˆj


→ c) ¿Cuál es su rapidez media durante los primeros 10
segundos?
respectivamente. (
Respuesta. a) 9iˆ + 6 ˆj + 3kˆ cm/s )
( )
a) ¿Cuales son la velocidad y aceleración de cada
objeto? b) 150iˆ + 100 ˆj + 50kˆ cm
b) ¿Dónde y cuando chocan? c) 18,71 cm/s
Respuesta.
→ →
a) v 1 = (8t + 3)iˆ + 2 ˆj , a 1 = 8iˆ
10. Si una partícula que se mueve sobre una
trayectoria curva tiene una aceleración total en un
→ →
v 2 = (16t + 11)iˆ + 5 ˆj , a 1 = 16iˆ ( )

momento dado a = 3tˆ + 2nˆ cm/s . Hallar:
2

26
Movimiento en un plano y en el espacio Hugo Medina Guzmán

a) La aceleración tangencial. →
b) La aceleración centrípeta. partícula 1 con aceleración constante a = aˆj , y la
c) El módulo de la aceleración total. partícula 2 con aceleración angular constante α , en
d) El ángulo ϕ que la aceleración total forma con la sentido contrario al movimiento de las agujas de un
tangente a la curva. reloj, describiendo una circunferencia de radio R,
Respuesta: a) at = 3 cm/s
2 como se muestra en la figura. Determine en función
de a y R:
b) a c = −4 cm/s
2
a) El tiempo que tardan en encontrarse, suponiendo
que lo hacen sobre el eje de las ordenadas, antes que
c) a = 5cm/s
2
la partícula 2 complete una vuelta completa.
d) ϕ = 53,1o Encuentre el valor de α que hace esto posible.
b) Halle los vectores velocidad y aceleración de las
11. Dos cuerpos se lanzan simultáneamente desde un dos partículas para el instante del encuentro.
mismo punto con la misma rapidez inicial pero en
distintas direcciones, uno verticalmente hacia arriba
y el otro formando un ángulo θ = 60° con la
horizontal. Conociendo que la rapidez inicial de
ambos cuerpos es v 0 = 25 m/s, ¿a qué distancia se
encontrarán cuando hayan pasado 1,7 s?

12. Una partícula se mueve en un plano de tal suerte 16. Un niño hace girar uniformemente una piedra en
que su radio vector con respecto a un punto fijo un círculo horizontal por medio de una cuerda de 1
barre ángulos iguales en tiempos iguales mientras m de longitud. El niño se encuentra sobre un
que la distancia al punto fijo es variable con el montículo de tal forma que el plano del movimiento
tiempo. Escriba las componentes radial y tangencial se encuentra a 5 m de altura sobre el suelo. La
de la velocidad y la aceleración de la partícula cuerda se rompe y la piedra sale disparada
mostrando explícitamente cualquier cantidad que se horizontalmente, golpeando el suelo a 3 m de
mantenga constante durante el movimiento. distancia. ¿Cuál fue la aceleración centrípeta de la
piedra mientras estaba en movimiento circular?
13. Un tren pasa por una estación con una velocidad
de 30 km/h. En el instante en que la locomotora pasa
junto al guardagujas este lanza una bolsa a uno de
los ingenieros de maquinas. Sabiendo que la rapidez
inicial con que el guardagujas lanzó la bolsa fue de
45 km/h
a) ¿Cuál tendrá que ser el ángulo de lanzamiento
para lograr el objetivo?.
b) Describa la trayectoria de la bolsa en el sistema
de referencia del maquinista.
17. Desde un sistema de referencia situado en el
14. Un arquero está en una colina cuya pendiente
suelo, con eje horizontal x y vertical y, se observa el
forma un ángulo α con la horizontal. Si el arquero movimiento de un objeto sometido a una aceleración
dispara la flecha según una dirección β respecto a
a = −2iˆ − 6 ˆj (m/s). Si en el instante inicial el
la colina y con velocidad v 0 , encontrar la distancia, objeto se encontraba en el punto P = (-3, 2) (m),
medida a lo largo de la colina, a la cual caerá la
moviéndose con una velocidad v (t = 0 ) = 3 ˆj (m/s):
flecha.
a) Obtenga la ecuación explícita de la trayectoria del
objeto.
b) Determine el instante en el que la velocidad y la
aceleración son perpendiculares.
c) Calcule las coordenadas del punto más alto de la
trayectoria.
d) Calcule el tiempo que tardó el móvil desde que
salió del punto P hasta que llegó al suelo.

18. La figura muestra una cuenta p que desliza por


un alambre plano en forma de parábola. La ecuación
15. Dos partículas se encuentran inicialmente en
de la parábola es y = x2/b, donde b es una constante
reposo en las posiciones que muestra la figura.
positiva con dimensiones de longitud. Llamaremos a
Ambas comienzan a moverse al mismo tiempo, la

27
Movimiento en un plano y en el espacio Hugo Medina Guzmán

al ángulo entre la tangente a la curva y el eje x. en el Respuesta.


punto donde se encuentra la cuenta. Llamaremos Ê y N̂ a los vectores unitarios en
a) Halle tan α en función de la coordenada x de P. dirección Este y Norte respectivamente.
b) Suponga que la cuenta tiene rapidez v y se mueve
v = (300 Eˆ + 60 Nˆ ) km/h, v = 60 26 km/h.

hacia la derecha. Halle las componentes x e y de la
velocidad de la cuenta en función de y y de la
coordenada x de P. 21. Un hombre guía su automóvil bajo lluvia a una
Ayuda: recuerde que el vector velocidad es tangente velocidad constante respecto a Tierra de módulo y
a la trayectoria. dirección. Mientras conduce el hombre observa que
la trayectoria de cada gota es una línea recta que se
aparta un ángulo α de la vertical y al detenerse
observa que la lluvia cae verticalmente y
prácticamente con velocidad constante. Halle el
vector velocidad de las gotas de lluvia respecto al
auto en movimiento y respecto a Tierra (tome
vertical hacia arriba).
Respuesta.
Respuesta. → v
v gota ,Tierra = − ˆj ,
2x tan α
a) tan α =
b → v
v gota , Auto =− ˆj − viˆ
bv 2 xv tan α
b) v x = , vy =
b + 4x
2 2
b + 4x 2
2
22. Un vagón de ferrocarril motorizado va cuesta
abajo sobre un plano inclinado un ángulo
19. Un ascensor parte del reposo y desciende con
aceleración constante de 1 m/s2 respecto a Tierra.
α . La distancia entre el techo y el piso del vagón es
Dos segundos después de iniciarse el descenso se H y su aceleración respecto a Tierra es constante y

cae la lámpara del techo del ascensor. La distancia
vale a = aiˆ , ver figura. Un pasajero del vagón
del techo al piso del ascensor es de 2 m. Definimos
observa que una lámpara, situada en el centro del
el referencial del ascensor como aquél con origen en
techo del vagón, se desprende y choca con el piso en
su techo y dirección y positiva apuntando hacia
el punto O (en el extremo inferior del vagón).
abajo.
a) Halle la aceleración de la lámpara respecto a
a) Halle los vectores aceleración, velocidad y
Tierra y respecto al pasajero del vagón. Exprese sus
posición de la lámpara respecto al ascensor.
b) Determine el tiempo que tarda la lámpara en caer. resultados en términos de los vectores unitarios iˆ y
c) Encuentre la distancia recorrida por el ascensor ĵ .
mientras cae la lámpara. b) Escriba las componentes cartesianas de la
Respuesta. velocidad y posición de la lámpara según el pasajero.
Todas las unidades están expresadas en el sistema Torne el origen en el punto o solidario al vagón y
MKS. L indica lámpara, A ascensor y T Tierra. llame L a la longitud del vagón.
a) Tomaremos como t = 0 el instante para el cual se c) Halle el tiempo que tarda la lámpara en caer y la
desprende la lámpara. longitud L del vagón.
→ → → →
d) Determine la ecuación de la trayectoria de la
a LA = a LT − a AT = 9 ˆj , v LA = 9tˆj ,
lámpara, y = y ( x ) , según el pasajero. ¿Qué clase

9
r LA = t 2 ˆj de curva es la trayectoria de la lámpara vista por el
2 pasajero y vista desde Tierra?
9 2 2
b) y LA = t = 2 ⇒ t =
2 3
14
c) D =
9
20. Los instrumentos de un aeroplano en vuelo
horizontal indican que se dirige hacia el Este con
una rapidez de 300 km/h respecto al aire. En Tierra
se observa que el aeroplano se encuentra en medio Respuesta.
de una corriente de aire que sopla hacia el Norte con Los subíndices L, P y T hacen referencia a la
rapidez de 60 km/h. Halle la velocidad y rapidez del lámpara, al pasajero y al referencial inercial de
avión respecto a Tierra. Tierra respectivamente.

28
Movimiento en un plano y en el espacio Hugo Medina Guzmán

→ 24. El aro de la figura tiene radio R y rueda sobre


(
a) a LT = g senθ iˆ − cos θ ˆj , ) una superficie horizontal fija a Tierra.
→ El aro gira en sentido horario mientras su centro e se
a LP = (gsenθ − a )iˆ − g cos θ ĵ mueve hacia la derecha con rapidez
b) v x = (gsenθ − a )t , v y = − g cos θ t V respecto a la superficie. Considere un observador
con origen en C (se traslada con el aro)
x=
1
(gsenθ − a ) t 2 − L , y que no rota respecto a Tierra. Suponga que todos
los plintos del aro tienen rapidez V respecto al
2 2 observador (se dice entonces que el aro rueda sin
1 deslizar).
y = − g cos θ t 2 + H
2 En la figura se han marcado cuatro puntos para un
2 H (gsenθ − a )
cierto instante. El punto A es el punto más alto del
2H
c) t = , L= aro, el B el más bajo, el D el punto del extremo
g cos θ g cos θ izquierdo y el E con un radio vector que forma un
d) Vista por el pasajero la trayectoria es una línea ángulo θ con la vertical.
recta de ecuación a) Halle la velocidad angular w del aro.
g cos θ b) Halle los vectores velocidad de los puntos A, B y
y=− x D respecto a la superficie.
gsenθ − a c) Halle el vector velocidad del punto E respecto a la
Vista desde Tierra la trayectoria es una parábola. superficie y diga para qué ángulo θ su módulo es
igual a V.
23. La corriente de un río fluye de Este a Oeste con
rapidez constante v = 2 m/s respecto a Tierra. Un
bote atraviesa el río y de acuerdo a sus instrumentos
de a bordo se mueve respecto al río dirigiéndose al
Norte con rapidez constante = 10 m/s. Respecto al
bote un pasajero se desplaza sobre la cubierta en
línea recta desde el punto A hasta el punto G con
una rapidez constante v1 = 10 m/s. Suponga que BA
= 4 m y apunta hacia el Norte y BC = 3 m y apunta
hacia el Este.
Respuesta.
a) Halle el vector unitario û que apunta de A a C y a) La rapidez de cualquier punto del aro respecto a C
las velocidades del bote y del pasajero respecto a
es V = Rω , luego ω = V / R .
Tierra. r r r
b) Halle el tiempo que tarda el pasajero en ir de A b) V A = 2Viˆ , V B = 0 , V D = Viˆ + Vˆj
r r
hasta C. ¿Qué distancia recorre el bote en ese tiempo c) V E = V (1 − cos θ ) iˆ + Vsenθ ˆj , V E = V
según un observador en Tierra?
⇒ θ = ±60°
25. Para conocer la rapidez de un avión es necesario
determinar cuanto tiempo toma volar en un rizo
cerrado de longitud conocida. ¿Cuánto tiempo
tomará al avión volar alrededor de un cuadrado de
lado a, con el viento soplando con una velocidad u?,
en dos casos:
a) la dirección del viento coincide con uno de los
lados del cuadrado;
b) la dirección del viento coincide con la diagonal
Respuesta. del cuadrado?
Las letras b, p y T designarán respectivamente el Sin viento la rapidez del avión es v, mayor que u.
bote, pasajero y Tierra.

( )
Respuesta,
3iˆ + 4 ˆj →
a) uˆ = ( )
, v b ,T = − 2iˆ − 10 ˆj m/s,
a) t1 =
2a v + v 2 − u 2


(
v2 − u 2 )
, b)

v p ,T = (4iˆ − 2 ˆj ) m/s. 4a v 2 − u 2 / 2
t2 =
1
b) t = s , d = 26m
(
v2 − u 2 )
2
26. Un hombre que viaja en un camión intenta
golpear un poste con una piedra, y cuando pasa

29
Movimiento en un plano y en el espacio Hugo Medina Guzmán

frente a él arroja la piedra con una velocidad


horizontal de 20 m/s respecto al camión. Sabiendo
que la velocidad del camión es de 40 km/h, Calcular:
a) la dirección en que debe lanzar la piedra.
b) la velocidad horizontal de la piedra respecto al
suelo.
Respuesta. a) 56,3º con relación a la dirección
trasera del camión
b) 16,63 m/s

27. El piloto A está volando con un avión con una


velocidad de 150 km/h, sobrevolando al piloto B, 30. Un automóvil viaja hacia el Este con una
cuyo avión vuela a 135 km/h, 300 m por debajo Con rapidez de 50 km/h. Está lloviendo verticalmente
el mismo rumbo. El piloto A para mandar un con respecto a la Tierra. Las marcas de la lluvia
mensaje a B lo sujeta a una piedra y la arroja a la sobre las ventanas laterales del automóvil forman un
cabina de B. Sin tomar en cuenta la resistencia del ángulo de 60° con la vertical, calcule la velocidad de
aire. la lluvia con respecto a:
a) ¿Con qué velocidad deberá lanzarla respecto a su a) el automóvil y
avión cuando B está directamente debajo de él? b) la Tierra.
b) ¿Cuándo B está todavía a 300 metros delante de l?
Respuesta, a) v = 15 km/h hacia atrás; b) v = 128 31. La distancia de A a B es l . Un aeroplano vuela
km/h hacía adelante. desde A hasta B y vuelve otra vez con una velocidad
constante V relativa al aire. Calcular el tiempo, total
28. Una partícula describe una circunferencia de que empleará en realizar el recorrido si el viento
radio R = 0,5 m con una frecuencia de sopla con una velocidad v en las siguientes
10 r pm. Si en t0= 0 la partícula está en la posición A direcciones:
moviéndose en el sentido horario, calcular: a) Sobre la línea que une A y B.
a) El período T y la rapidez del movimiento b) Perpendicular a esta línea.
b) La velocidad media y aceleración media en el c) Formando un ángulo θ con esta línea.
intervalo (0; 0,75T). Demostrar que la duración del trayecto siempre
c) La aceleración en t =T / 2 aumenta con la existencia del viento.
Respuesta.
2l
Poniendo T0 = , los resultados son:
V
T0
a)
⎛ v2 ⎞
⎜⎜1 − 2 ⎟⎟
⎝ V ⎠
T0
b) 12
⎛ v2 ⎞
29. Una partícula P se mueve con aceleración
⎜⎜1 − 2 ⎟⎟
angular constante sobre una circunferencia de radio ⎝ V ⎠
1
R =3m. Parte desde el reposo del punto A y
completa la primera vuelta en un tiempo t = 2s. ⎡ ⎛ vsenθ ⎞ 2
⎤ 2

Calcular: ⎢1 − ⎜ ⎟ ⎥
a) El módulo de la aceleración angular ⎢⎣ ⎝ V ⎠ ⎥⎦
c) T0
→ → ⎛ v2 ⎞
b) La ecuación r = r(t ) . ⎜⎜1 − 2 ⎟⎟
c) El tiempo que emplea para llegar a la posición ⎝ V ⎠
definida por θ = 3π/2 .
d) La velocidad lineal en θ = π 32. El bloque deslizante A se mueve hacia la
izquierda a una velocidad constante de 0,3iˆ m/s,
Determinar:
a) La velocidad del bloque B;
b) las velocidades de los tramos de cable C y D;
e) la velocidad relativa de A respecto a D;
d) La velocidad relativa del tramo de cable C
respecto al tramo D.

30
Movimiento en un plano y en el espacio Hugo Medina Guzmán

Respuesta.
a) − 0,2iˆ m/s, b) − 0,2iˆ m/s, − 0,4iˆ m/s,
c) − 0,1iˆ m/s, d) 2iˆ m/s,

31
Dinámica de una partícula Hugo Medina Guzmán

CAPÍTULO 4. Dinámica de una partícula


INTRODUCCIÓN
En el capítulo anterior estudiamos el movimiento de
una partícula con respecto a un sistema de referencia
sin preguntarnos sobre la causa del movimiento. Lo
describimos simplemente en términos de los vectores
→ → →
r , v y a.
Nuestra discusión fue geométrica, en este capítulo
discutiremos la causa del movimiento. Seguiremos
tratando a los cuerpos como partículas simples. En ausencia de una fuerza resultante, el objeto se
Posteriormente trataremos sobre sistemas de mantiene en movimiento con velocidad uniforme o
partículas y cuerpos rígidos. permanece en reposo. Esta es la PRIMERA LEY
DE NEWTON DEL MOVIMIENTO
EL ORIGEN DEL MOVIMIENTO Ahora podemos pensar acerca de la situación cuando
¿Qué origina el movimiento? ¿Qué detiene el un objeto era empujado sobre un plano. Cuando la
movimiento? ¿Se necesita causa para mover las fuerza era pequeña no había movimiento, pero una
cosas? ¿Por qué un objeto al que se le da un empujón fuerza debería causar movimiento; la conclusión es
pronto se detiene? ¿Por qué los planetas mantienen su que debe haber otra fuerza actuando sobre el cuerpo
movimiento alrededor del sol? la cual anula justamente el efecto de la fuerza que
Aristóteles joven filósofo griego (siglo IV a.c.) decía aplicamos. Al incrementar nuestra fuerza, la fuerza
que un cuerpo permaneciera en movimiento era opuesta también se incrementa, hasta que en algún
necesario ejercer alguna acción sobre él ya que el valor particular la fuerza opuesta termina de
estado natural es el reposo. Esto parece ser razonable, incrementarse y comienza el movimiento porque hay
cuando dejamos de empujar un cuerpo, este pronto una fuerza resultante actuando sobre el objeto. La
alcanza el reposo. Parece ser necesaria una acción fuerza opuesta es la fuerza de Fricción
exterior o fuerza aplicada al cuerpo para mantener el
movimiento. Sin embargo, observemos esta situación ¿QUÉ ES FUERZA? En la vida cotidiana se
con mayor detenimiento. La figura siguiente muestra considera fuerza a una sensación común asociada con
un bloque de madera sobre un plano. la dificultad para mover o levantar un cuerpo. En
Física se identifica una fuerza por el efecto que
produce. Uno de los efectos de una fuerza es cambiar
el estado de reposo o de movimiento del cuerpo, más
concretamente, una fuerza cambia la velocidad de un
objeto, es decir produce una aceleración. Cuando se
aplica una fuerza sobre un cuerpo y no se produce
Aplicamos una fuerza pequeña al bloque, no pasa movimiento, entonces puede cambiar su forma, aún si
nada. Incrementamos la fuerza y a un valor particular el cuerpo es muy rígido. La deformación puede o no
el bloque se mueve. Si seguimos incrementando la ser permanente. Entonces los efectos de la fuerza neta
fuerza empujando o jalando más, el objeto se mueve son dos: cambiar el estado de movimiento de un
con mayor rapidez, Cuando dejamos de empujar el cuerpo o producir una deformación, o ambas cosas.
cuerpo rápidamente vuelve al reposo. Sin embargo si Normalmente sobre un cuerpo pueden actuar varias
ponemos ruedas al bloque el resultado es diferente, fuerzas, entonces el cuerpo acelerará cuando el efecto
una fuerza muy pequeña causa el movimiento. La de la fuerza neta que actúa sobre él no es cero.
diferencia son las ruedas debido a la fricción. Se llama fuerza neta o fuerza resultante a la suma de
Para hacer un estudio libre de la fricción busquemos todas las fuerzas que actúan sobre un cuerpo. Si la
llegar cercanamente a esta condición, una forma de fuerza neta es cero, la aceleración es cero, el
lograr esto es con una mesa neumática, se sopla aire movimiento es con velocidad igual a cero (cuerpo
sopla hacia arriba a través de pequeños agujeros detenido) o con velocidad constante. Cuando un
manteniendo un disco suspendido sobre un colchón cuerpo está en reposo o se mueve con velocidad
de aire. ¿Qué pasa cuando empujamos un objeto en constante, se dice que está en equilibrio.
ausencia de fricción? Este se mantiene en Se pueden distinguir dos grandes clases de fuerzas:
movimiento a velocidad constante. fuerzas de contacto, representan el resultado del
contacto físico entre el cuerpo y sus alrededores, por
ejemplo mover un carro o estirar un resorte; y fuerzas
de acción a distancia que actúan a través del espacio
sin que haya contacto físico entre el cuerpo y sus
alrededores, por ejemplo la fuerza con que la Tierra
atrae a los cuerpos que caen en caída libre. Todas las
diferentes formas de fuerzas se encuentran dentro de
esas dos grandes clasificaciones.

1
Dinámica de una partícula Hugo Medina Guzmán

Para describir el mundo, la física contemporánea si se estira 2,5 unidades, entonces la fuerza aplicada
recurre a cuatro interacciones o fuerzas es 2,5 veces la unidad de fuerza.
fundamentales, que actúan sobre las partículas de Este procedimiento es válido para pequeños
materia (y sobre las antipartículas), son vehiculadas alargamientos del resorte, ya que si la fuerza es muy
por unas partículas llamadas vectores de interacción, intensa, se puede deformar y no volver a su forma
que son: fotón (interacción electromagnética), bosón original.
(interacción débil), gluón (interacción fuerte) y
gravitón (interacción gravitacional). CAMBIO DE VELOCIDAD
1) Fuerzas electromagnéticas de atracción o repulsión Nuestro siguiente problema es encontrar una relación
entre partículas cargadas en reposo o en movimiento, entre la fuerza y el cambio en el movimiento
explica la cohesión de los átomos, es mucho más producido por ésta.
intensa que la fuerza gravitacional. Para esto necesitamos lo siguiente:
2) Fuerzas nucleares intensas entre partículas 1. Un carro muy ligero que pueda moverse sin
subatómicas, responsable de la existencia del núcleo fricción sobre una superficie horizontal.
atómico asegura la cohesión interna de los 2. Una fuerza constante. Esta podernos obtenerla
constituyentes del núcleo atómico, protones y mediante un resorte (Si mantenemos un resorte
neutrones, y es responsable de un gran número de estirado una misma longitud, la fuerza que la estira es
reacciones y de desintegraciones; es la de mayor constante).
magnitud (102 - 103 veces la fuerza
electromagnética).
3) Fuerzas nucleares débiles de corto alcance, rige
algunos procesos radiactivos, establece la estabilidad
de algunos núcleos, es varios órdenes de magnitud
(1012) menor que la fuerza electromagnética.
4) Fuerza de atracción gravitacional entre cuerpos 3. Un registrador de tiempo. El movimiento del carro
debido a sus masas, entre otras cosas hace que caigan puede estudiarse si una cinta de papel atada a éste
las manzanas y que suba la marea, es la fuerza de pasa a través del registrador que produce marcas en la
menor magnitud comparada con las otras. cinta a intervalos de tiempo regulares.
Para que el concepto de fuerza sea exacto se debe
establecer un método para medirla. Una fuerza se
puede medir por el efecto que produce. Por ejemplo
se puede usar la deformación que una fuerza produce
en un resorte, como en la figura. Si se aplica una
fuerza verticalmente a un resorte y se estira una
unidad, le asignamos a la fuerza una magnitud
unitaria F. Se aplica ahora otra fuerza al mismo
resorte horizontalmente, produciéndole un
estiramiento de dos unidades, la magnitud de la
fuerza será de 2F. Si se aplican simultáneamente las
dos fuerzas, el resorte se inclina, y se estira 5
veces. La fuerza equivalente que produce ese
estiramiento del resorte es la suma vectorial de F y
2F. Es decir, la fuerza es un vector.

La figura siguiente muestra la cinta de papel


producida por una fuerza constante.

El instrumento para medir fuerzas se llama


dinamómetro, es un resorte que se estira sobre una
escala. Si se aplica una fuerza de una unidad sobre el
dinamómetro, el resorte se estira hasta que ejerce una
fuerza igual y contraria a la aplicada. En la escala se Con los datos obtenidos en esta experiencia se realiza
mide el alargamiento del resorte y se le asigna una el gráfico distancia - tiempo y se obtiene una curva.
unidad de fuerza. De esa manera se calibra el Con los datos también se puede obtener la velocidad
dinamómetro y se usa para medir fuerzas, por media en cada intervalo de tiempo. El gráfico
ejemplo se aplica una fuerza sobre el dinamómetro y velocidad - tiempo es una línea recta que indica que

2
Dinámica de una partícula Hugo Medina Guzmán

el movimiento es con aceleración constante. De aquí internacional (S.I.) la unidad de aceleración es m/s.
podemos concluir que una fuerza constante produce ¿Cuales son las unidades de fuerza y de masa? Como
una aceleración constante. son dos cantidades que se relacionan sólo tenernos
que especificar un estándar para una de ellas.
El sistema internacional adopta corno unidad una
pieza de material llamado KILOGRAMO, cuyo
símbolo es kg. El kilogramo es la masa un prototipo
de platino iridiado sancionado por la Conferencia
General de Pesas y Medidas realizada en París en
1889 y depositado en el pabellón de Breleuil en
Sevres.
Si duplicamos la fuerza usando dos resortes iguales La unidad de fuerza es el newton, cuyo símbolo es N
estirados la misma longitud, como se muestra en la y se define así:
figura. El newton la fuerza que produce una aceleración de
un metro por segundo al cuadrado a una masa de un
kilogramo.
kgm
N=
Duplica la fuerza y produce el doble de aceleración. s2
Si triplicamos la fuerza se obtiene una aceleración de Otros sistemas:
valor triple. MKS: igual al S.I.
Concluimos que la aceleración a del cuerpo es CGS: Masa → gramo (g), l g = 10-3 kg
directamente proporcional a la fuerza. Aceleración → cm/s2
a∝F Fuerza → dina = g.cm/s2
Podemos escribir esto como F = ma , donde m es la Inglés técnico: En este sistema la unidad fundamental
constante de proporcionalidad. A esta constante la es la unidad de fuerza.
llamaremos MASA. Fuerza → libra (lb), 1 lb = 4,45 N
Para una determinada fuerza a mayor constante m la Aceleración → pie/s2
aceleración es menor. A mayor valor de la constante Masa → slug = lb58
es más difícil acelerar el cuerpo. s2/pie
Para conocer qué factores cambian esta constante
realicemos el siguiente experimento: en lugar de usar PESO DE UN CUERPO. El peso de un cuerpo es la
un solo carro jalado por el resorte estirado usemos fuerza de atracción que ejerce la Tierra sobre el
dos carros uno sobre otro y luego tres carros como se cuerpo. Un cuerpo de masa m sometido a cierta
muestra en la figura fuerza cae con la aceleración de la gravedad g, el
peso P de este cuerpo es
→ →
P = mg

La aceleración que se obtiene con los carros es igual a


la mitad y con tres es igual a un tercio. Como el valor
de F es igual en todos los casos, quiere decir que la
constante con dos carros es igual a 2m y con tres
carros es 3m.
Como la aceleración es una cantidad vectorial la
fuerza también lo es y tiene la misma dirección que la
aceleración, pero un módulo m veces mayor, de modo
que la relación anterior puede escribirse en la forma Su dirección es hacia abajo (hacia el centro de la
→ → Tierra). Como el peso es una fuerza debe medirse en
F = ma Newtons.
Fuerza = masa x aceleración. Debido a que la aceleración de la gravedad varía de
Esta expresión constituye la SEGUNDA LEY DE un lugar a otro de la Tierra, el peso de un cuerpo es
NEWTON DEL MOVIMIENTO. diferente en lugares distintos, sin embargo la masa de
La fuerza que actúa sobre un cuerpo es igual al un cuerpo es la cantidad fija que no depende del lugar
producto de la masa del cuerpo por la aceleración que donde está situado el cuerpo,
le imprime. Aunque el peso de un objeto varía de un sitio a otro,
esta variación es demasiado pequeña para ser
UNIDADES DE FUERZA Y MASA observada en la mayor parte de las aplicaciones
La relación F = ma nos da una relación entre prácticas, por esto, el peso de un cuerpo parece ser
fuerza, masa y aceleración. En el sistema una característica constante al igual que su masa. Este

3
Dinámica de una partícula Hugo Medina Guzmán

hecho ha conducido al empleo ordinario de otras dos aplicación de las leyes de Newton.
medidas:
KILOGRAMO FUERZA, es el peso de un Kilogramo APLICACIONES DE LAS LEYES DE NEWTON
masa. Cuando estudiamos Cinemática, encontrarnos las
1 kgf = 9,8 N relaciones entre desplazamiento, aceleración y
LIBRA MASA, es la masa de un cuerpo que pesa una →
libra. tiempo. Por ejemplo, conociendo la aceleración a
1 libra masa = 0,454 kg. las condiciones tales como posición inicial, velocidad
Estas unidades son prácticas pero incorrectas y no inicial, es decir la posición y la velocidad en el
deben ser usadas en Física. tiempo que llamamos inicial (t = 0), podemos conocer
la velocidad y posición para cualquier tiempo. Las
ACCION Y REACCION. condiciones iniciales las tenemos pero la aceleración,
Hagamos una observación más detallada cuando → →
jalamos el carro con un resorte estirado una ¿de dónde? Para esto tenemos F = m a , todo lo que
determinada longitud. tenemos que hacer es conocer las fuerzas sobre el

cuerpo y su masa, y entonces podremos encontrar a .
La mejor forma de estar seguros que comprendemos
→ →
el significado de F = m a , es hacerlo con algunos
problemas que involucran las leyes de Newton. Para
Para que el resorte esté estirado es necesario jalarlo resolver un problema sugerimos cuatro pasos a
por los dos lados. Se necesitan fuerzas en sentidos seguir:
opuestas y en cada extremo del resorte. 1. Dibujar un esquema del sistema
Cuando jalamos el carro, una fuerza actúa sobre el 2. Identificar el cuerpo a cuyo movimiento se refiere
carro y una fuerza en sentido opuesto actúa sobre el problema.
nuestra mano. ¿Cuáles son las magnitudes de estas 3. Dibujar otra figura con solamente el objeto en
fuerzas? particular manteniendo el marco de referencia poner
todas las fuerzas que actúan sobre el objeto mediante
flechas. Esto se conoce como DIAGRAMA DEL
CUERPO LIBRE (DCL). Si se comete una
equivocación todo lo demás fallará, por eso es
conveniente hacerlo bien. Una mejor forma de
comenzar es poner la fuerza de gravedad primero y
Con el objeto de dar respuesta a esta pregunta luego preguntarse:
pongamos dos resortes iguales al primero y jalemos “¿Qué toca al cuerpo?”, la acción de tos resortes,
de tal manera que el carro adquiera la misma cuerdas, manos y otros objetos, todos deben ser
aceleración que antes, esto quiere decir, por la considerados. Así como también las fuerzas que
segunda ley de newton que siendo la misma masa m actúan sin tocar el cuerpo, como la fuerza eléctrica,
estamos aplicando la misma fuerza (F = ma) que magnética de las cuales no nos preocupamos en este
antes y observamos que los resortes estiran la misma curso.
longitud, lo que quiere decir que la fuerza sobre la 4. Finalmente, aplicar la segunda ley de Newton a
mano es igual a la fuerza sobre el carro. cada componente de fuerza y aceleración.
→ →
F =ma
∑ Fx = ma x , ∑F y = ma y , ∑ Fz = ma z .
y ahora resolver para la aceleración.
En algunos de los problemas que se presentan más
Esto constituye la TERCERA LEY DE NEWTON frecuentemente, las acciones se producen por fuerzas
DEL MOVIMIENTO. sin contacto; en otros se usan cuerdas y varillas como
Si un cuerpo ejerce una fuerza sobre un segundo, éste medios de conexión. Cuando las masas de estos
ejerce una fuerza igual y opuesta sobre el primero. La medios de conexión son despreciables su único efecto
fuerza ejercida por el primer cuerpo sobre el segundo es el de transmitir
es la ACCIÓN, la fuerza igual y opuesta actuando
sobre el primero es la REACCIÓN, ESTÁTICA DE LAS MASAS PUNTUALES.
Expresado en símbolos, es: Los sistemas en los cuales todas sus partes satisfacen
→ → la primera ley son llamados sistemas estáticos, es
F sobre 2 debido a 1 = F sobre 1 debido a 2 decir si la suma vectorial de todas las fuerzas que
Fuerza de contacto de un cuerpo a otro con un cambio actúan es nula, el cuerpo esta en equilibrio y
de dirección o sin él permanece en reposo, o si está en movimiento, se
A continuación presentarnos algunos casos tipo de la mantiene con velocidad constante

4
Dinámica de una partícula Hugo Medina Guzmán

La condición de este equilibrio es



∑F = 0
y en componentes cartesianas:
∑F x = 0, ∑F y = 0 , ∑ Fz = 0 .
Las fuerzas son ejercidas sobre el objeto o sistemas
por. Medios exteriores al sistema.

Ejemplo 1. La Fuerza gravitacional Dado que la


aceleración de un cuerpo en caída libre en la tierra es
g, ¿cuál es la fuerza de la gravedad?
Solución.
Como este movimiento es en una sola dimensión,
consideramos que este se realiza en el eje z, tal que

a = − gkˆ
Según la Segunda Ley de Newton
→ → Diagrama del cuerpo libre (DCL)
F = m a = −mgkˆ Aplicando la condición de equilibrio de la masa m
T1 − mg = 0
∑ Fx = 0 , ∑ Fy = 0 , ∑ Fz = −mg . Luego ⇒ T1 = mg
Siendo esta la respuesta que ya conocíamos.
Si despreciamos la masa del dinamómetro, tenemos
Ejemplo 2. El dinamómetro. El dinamómetro es un que:
instrumento que se utiliza para medir las fuerzas. T1 − T2 = 0 y T1 = T2
Consta de un resorte con una escala que indica su El dinamómetro indica en la escala la fuerza
estiramiento, la cual está graduada en Newtons.
Cuando lo utilizamos para pesar se dispone como lo
T2 = mg
muestra la figura.
Ejemplo 3. Se tiene los dispositivos mostrados en la
figura. ¿Cuánto indica el dinamómetro de la figura (a)
y cuánto el dinamómetro de la figura (b)?

Se suspende la masa m, el resorte del dinamómetro se


estira hasta que alcanza el equilibrio estático.

Solución.
a) El diagrama de cuerpo libre de la figura (a) es

5
Dinámica de una partícula Hugo Medina Guzmán

En la masa
T1 − mg = 0 ⇒ T1 = mg
En la polea
T1 = T2
En el dinamómetro
T3 = T2 = T1 = mg
El dinamómetro es tensionado por la fuerza T1 y su
indicación será
Empezando por la derecha
T1 − mg = 0 ⇒ T1 = mg T1 = mg
Como se puede ver esta situación es completamente
La figura siguiente muestra la polea
análoga a la anterior, sólo que hemos sustituido una
de las poleas por la pared.

Ejemplo 4. Un cuerpo de masa m se sostiene por


medio de cuerdas como se muestra en la figura.
Encontrar las tensiones T1, T2 en las tres cuerdas.

Para que el trozo de cuerda este en equilibrio



∑F = 0
Descomponiendo las fuerzas sobre el trozo de cuerda
en los ejes x e y.
Como la cuerda se considera sin masa la suma de
fuerzas a lo largo del eje x es
T1 cos θ − T2 cos θ = 0 ⇒ T1 = T2 Solución.
En el dinamómetro, considerándolo de masa
despreciable.

∑F = 0
T2 − T3 = 0 ⇒ T2 = T3
En la polea de la izquierda
T4 = T3
En la masa de La izquierda Tomando un sistema de ejes horizontal y vertical
→ como el mostrado en la figura tenemos:
∑F = 0 →
T1 = −mgˆj
T4 − mg = 0 ⇒ T4 = mg →
Como conclusión todas las tensiones son iguales a T2 = T2 cos θ iˆ + T2 senθ ˆj
mg →

T4 = T3 = T2 = T1 = mg T3 = −T3 cos α iˆ + T3senα ˆj



El dinamómetro es tensionado por la fuerza
indicación será:
T1 , y su Con ∑F = 0
→ → →
T1 = mg T1 + T2 + T3 = 0
b) El diagrama de cuerpo libre de la figura siguiente Obtenemos:
es
∑F x = T2 cos θ − T3 cos α = 0
∑F y = T2 senθ + T3senα − mg = 0
Resolviendo estas dos ecuaciones
mg cos α mg cos θ
T2 = , T3 =
sen (θ + α ) sen (θ + α )

Ejemplo 5. Un bloque de 50N de peso se ubica sobre


un plano inclinado en un ángulo α de 30º con la

6
Dinámica de una partícula Hugo Medina Guzmán

horizontal. El bloque se sujeta con una cuerda ideal Solución.


que se encuentra fija en la parte superior del plano El D. C. L. del cuerpo:
inclinado, como en la figura. Estudiar el
comportamiento mecánico del bloque.

Del diagrama de cuerpo libre se obtiene:


∑F x : mgsenα = ma x

Solución.
∑F y : N − mgcosα = ma y = 0
El D. C. L. del cuerpo: De estas ecuaciones se obtiene:
a x = gsenα y N = mg cos α
Se concluye que la aceleración del bloque en
dirección del plano inclinado es la componente de g
en esa dirección. Estudiando ahora el movimiento del
bloque, considerando que parte del reposo y se
desliza una distancia D, se puede calcular la rapidez
con que llega a la base del plano. Si se considera que
el movimiento del bloque comienza desde el reposo,
se puede usar:
Fuerza de atracción de la Tierra, que es su peso mg.
Fuerza de la cuerda que lo sostiene, que es la tensión v 2 = v 02 + 2a x Δx ⇒ v 2 = 2( gsenα )D
T
y v= 2gDsenα
Fuerza que el plano ejerce sobre el cuerpo, que es la
normal N
Como el sistema está en equilibrio, se aplica la Ejemplo 7. Para el siguiente sistema mecánico,
primera Ley de Newton: calcular la aceleración de las masas y la tensión de la
cuerda.

Del diagrama de cuerpo libre se obtiene:


∑F x : − T + mgsenα = 0
∑F y : N − mgcosα = 0
Despejando T y N, y reemplazando los valores
numéricos, se obtiene: Solución.
T = mgsenα = 50sen30° = 25 N Como no se conoce la dirección del movimiento,
N = mgcosα = 50cos30° = 43,2 N supongamos que el cuerpo de masa M sube por el
plano inclinado, lo que determina el sentido de la
aceleración, entonces aplicando la segunda Ley de
DINÁMICA CON FRICCIÓN Newton se aplica cada masa:
DESPRECIABLE. El D. C. L. del cuerpo M:
Los sistemas en los cuales todas sus partes satisfacen
la primera ley son llamados sistemas estáticos, es
decir si la suma vectorial de todas las fuerzas que
actúan no es nula y la fricción se considera
despreciable,
Ejemplo 6. Si un bloque de masa m se ubica sobre un
plano sin roce, inclinado un ángulo α con la
horizontal, resbalará una distancia D a lo largo del
plano. Describir su movimiento. Del diagrama de cuerpo libre se obtiene:
∑F xT − Mgsenα = Ma ⇒
:
T = Mgsenα + Ma
∑ Fy : N − Mgcosα = 0
De estas ecuaciones se obtiene:
El D. C. L. del cuerpo m:

7
Dinámica de una partícula Hugo Medina Guzmán

Del diagrama de cuerpo libre se obtiene:


∑F y : T − mg = −ma ⇒ T = mg − ma
De estas ecuaciones se obtiene
Mgsenα + Ma = mg − ma
(m − Msenα ) g a) Sumando (1), (2) y (3):
a= F1 − (m1 + m2 + m3 )g = (m1 + m2 + m3 )a
(m + M )
Se observa que el signo de a depende del término F1
y a= −g
(m - M sen α). (m1 + m2 + m3 )
Ahora se calcula el valor de la tensión reemplazando
560
el valor de a en T: a= − 9,8 = 8,87 m/s2
⎛ m − Msenα ⎞ (20 + 8 + 2)
T = mg − m⎜ ⎟g
⎝ m+M ⎠ b) De (3) FB = m2 ( g + a )
mM FB = 8(9,8 + 8,87 ) = 149,4 N
T= (1 + senα )g De (1) FA = F1 − m1 ( g + a )
(m + M )
FA = 560 − 20(9,8 + 8,87 ) = 186,6 N
Ejemplo 8. Dos bloques de masas m1 = 20 kg y
m2 = 8 kg, están unidos mediante una cuerda Ejemplo 9. La máquina de ATWOOD. Es un aparato
homogénea inextensible que pesa 2 kg. Se aplica al que se utiliza para determinar con exactitud la
conjunto una fuerza vertical hacia arriba de 560 N.
Calcular: gravedad y consiste de dos masas m1 y m 2 ,
a) La aceleración del conjunto; ( m1 > m 2 ), que están unidas mediante una cuerda
b) Las fuerzas que actúan en los extremos de la que pasa sobre una polea. Considerar la cuerda
cuerda. inextensible y sin masa. Asimismo, no tornar en
cuenta la fricción y la masa de la polea. Describir el
movimiento y calcular la tensión en la cuerda.

Solución.
En el D. C. L. de m1:
F1 − FA − m1 g = m1 a (1) Solución.
En el D. C. L. de la cuerda de masa m3: Siendo m1 mayor que m 2 , la masa m1 se moverá
FA − FB − m3 g = m3 a (2) hacia abajo con una aceleración a y la masa m 2 se
En el D. C. L. de m2: moverá hacia arriba con la misma aceleración a .
FB − m2 g = m2 a (3) La figura siguiente muestra los diagramas de cuerpo
libre de cada una de las partes del sistema.

8
Dinámica de una partícula Hugo Medina Guzmán

Ahora la reacción del piso es R ' .


Aplicando la Segunda Ley de Newton al movimiento
de la persona
R'− mg = ma ⇒ R ' = m( g + a )
Si el ascensor sube el pasajero se siente más pesado,
como si fuera empujado contra el piso. Si el ascensor
desciende con esta aceleración,
R '− mg = −ma ⇒ R ' = m( g − a ) , el pasajero se
siente más liviano.

La polea cumple la función de cambiar la dirección Ejemplo 11. La figura muestra a un hombre
T1 Considerando el sentido de la aceleración o como elevándose mediante una fuerza vertical que aplica él
positiva. mismo a la cuerda que tiene en las manos. Si el
hombre y la silla juntos tienen una masa de 100 kg.
Aplicando la Segunda Ley de Newton a la masa m1 Se pregunta:
m1 g − T1 = m1 a a) ¿Con qué fuerza debe jalar para, subir con una
Aplicando la Segunda Ley de Newton para la masa velocidad constante?
b) ¿Con qué fuerza debe jalar para subir con una
m2 : aceleración de l m/s2 (considerar g = 10 m/s2?
T1 − m2 g = m2 a
De estas dos ecuaciones obtenemos:
(m1 − m2 ) 2m1 m2
a= g y T1 = g
(m1 + m2 ) (m1 + m2 )
Si las masas m1 y m 2 fueran casi iguales, el valor de
la aceleración sería pequeña y podría determinarse
midiendo el tiempo en que una de las masas sube o
baja una distancia determinada.
La razón (m1 − m2 ) se determina pesando los cuerpos.
(m1 + m2 )
Finalmente, la magnitud de g se obtiene a partir de
estas cantidades mediante la ecuación Solución.
(m1 + m2 ) a) La figura siguiente muestra los diagramas de
g= a
(m1 − m2 ) cuerpo libre de cada una de las partes del sistema.

Ejemplo 10. El peso de un pasajero en ascensor.


Consideremos un pasajero de peso mg en un ascensor
este peso es equilibrado por la reacción que el piso
ejerce sobre él, si el ascensor estuviera parado
R = mg .
Si el ascensor sube con aceleración a. ¿Cuál es el
peso de la persona?
Solución.
La figura muestra el ascensor subiendo con una
aceleración a

Como se considera la cuerda con masa despreciable


en el D.C.L. del trozo de cuerda
T=F
La polea solo cambia la dirección de la tensión T .
En el D.C.L .del hombre-silla
T + F − W = 0 ⇒ 2F = W
W
y F =
2
Como W = 100 × 10 = 1000 N

9
Dinámica de una partícula Hugo Medina Guzmán

1000
F= = 500 N
2
b) Ahora como el hombre debe subir con una
aceleración de l m/s2 tenemos:

Aplicando la Segunda Ley de Newton


T1 − m1 g = m1 a ⇒ T1 = m1 (a + g )
T1 = 1100(2 + 9,8)
W W = 12980 N
T + F −W = a ⇒ 2F = W + a
g g b) Consideremos el D.C.L. de la masa m 2 :
W⎛ a⎞
y F = ⎜⎜1 + ⎟⎟
2⎝ g⎠
Como W = 1000 N , a = 1 m/s 2 y = 1 m/s 2
1000 ⎛ 1⎞
F= ⎜1 + ⎟ = 550 N
2 ⎝ 10 ⎠

Ejemplo 12. La figura muestra un ascensor. Este


consiste de la caja con masa m1 = 1100 kg , el Aplicando La Segunda Ley de Newton
contrapeso con masa m 2 = 1000 kg . El cable y m1 g − T2 = m2 a ⇒ T2 = m2 ( g − a )
poleas con masa y fricción despreciables. T2 = 1000(9,8 − 2)
Cuando el ascensor tiene una aceleración hacia arriba = 7800 N
de 2 m/s2, el contrapeso tiene igual aceleración pero c) En el motor Fuerza ejercida por el motor (T1 y T2
hacia abajo. pueden considerarse colineales)
a) ¿Cuál es el valor de la tensión T1 ?
b) ¿Cuál es el valor de la tensión T2 ?
c) ¿Cuál es la fuerza ejercida por el motor sobre el
cable?

FM = T1 − T2 = 12980 – 7800
= 5180 N

Ejemplo 13. Demostración de la tercera ley de


Newton mediante el uso de la segunda ley.
Se tienen dos cuerpos de masas m1 y m 2 los cuales
son empujados sobre un plano sin fricción por una
fuerza de magnitud P . Demostrar que aquí se
Solución. cumple la tercera ley de Newton.
a) Consideremos el D.C.L de la masa m1 :

Solución.

10
Dinámica de una partícula Hugo Medina Guzmán

Asumiremos que no hay fricción entre las superficies a) el valor y sentido de la velocidad del carrito,
de contacto de m1 y m 2 . b) el lugar, donde encontrará
c) el desplazamiento del carrito
La figura muestra los D.C.L. para los bloques 1, 2 y
d) el recorrido total del carrito.
para el sistema.
(Usar g = 9,8 m/s2)

Solución.
N 1 y N 2 son las fuerzas ejercidas por el plano.
F21 es la fuerza que el bloque 2 ejerce sobre el
bloque 1.
F12 es la fuerza que el bloque 1 ejerce sobre el
bloque 2.
La fuerza P solo actúa sobre el bloque 1, ya que
solo está en contacto con él.
Como asumimos que no hay fricción entre los
bloques, las fuerzas son normales a la superficie de
contacto.
Para el bloque 1 tenemos:
P − F21 = m1 a1x y N 1 − m1 g = 0
Para la masa M:
Similarmente para el bloque 2 − T = Ma (1)
F12 = m2 a 2 x y N 2 − m2 g = 0 Para la masa m:
Para el sistema T − mg = ma (2)
P = (m1 + m2 )a x y Sumando (1) y (2)
N 1 + N 2 − (m1 + m2 )g = 0 − mg = (M + m )a ⇒
m 0,2
En este caso no nos interesan las ecuaciones en y pero
a=− g =− (9,8) = - 2,8 m/s2
si las ecuaciones en x.
Como los bloques se mueven juntos:
(M + m ) 0,7
La aceleración es en sentido contrario al indicado en
a1x = a 2 x = a x la figura.
Sumamos la ecuación para el bloque 1 con la a) La velocidad inicial del carrito es v0 = 7 m/s y su
ecuación para el bloque 2. aceleración es a = - 2,8m/s2.
P − F21 + F12 = m1 a1x + m2 a 2 x = (m1 + m2 )a x De las ecuaciones de cinemática
Comparando con la ecuación para el sistema 1 2
x = v0 t + at , v = v0 + at ,
tenemos: 2
P − F21 + F12 = P Hallamos:
Esto dice que la magnitud de la fuerza de 1 sobre 2 es x = 7t − 1,4t 2 , v = 7 − 2,8t
igual a la fuerza de2 sobre 1. Como ellas son opuestas Dentro de 5 s el carrito tendrá una velocidad
resulta ser precisamente la tercera ley de Newton. v = - 7 m/s (dirigida a la izquierda).
F21 = F12 , Acción y reacción. b) x = 7(5) − 1,4(5) = 35 − 35 = 0
2

El carrito se encontrará en la posición inicial.


Ejemplo 14.. Un carrito de masa M = 500 gramos c) El desplazamiento es cero.
está unido a una carga de masa m = 200 gramos
mediante una cuerda. En el momento inicial el carrito d) El carrito se detiene cuando v = 0 e inicia el
tenia la velocidad inicial v0 = 7 m/s y se movía a la camino de vuelta.
derecha por un plano horizontal. Determinar para t =
5 s:

11
Dinámica de una partícula Hugo Medina Guzmán

0 = 7 − 2,8t ⇒ t =
7
= 2,5 s
(F − F )fk
a=
2,8 m
[
Recorrido total s = 2 7(2,5) − 1,4(2,5)
2
] Si incrementamos la fuerza F, punto C, la fuerza neta
sobre el bloque F − F fk se incrementa y también se
= 17,5 m
Recorrerá un trayecto igual a 17,5 m. incrementa la aceleración.

FRICCIÓN Observación. Encontramos que con fuerzas menores


Cuando un cuerpo sobre una superficie se empuja o que 10 N no se produce movimiento.
se jala éste puede permanecer inmóvil, esto sucede Con 10 N el bloque comienza a moverse.
porque la fuerza aplicada no ha sido suficiente para Para fuerzas mayores a 10 N el bloque se acelera.
vencer la fuerza de fricción. Cuando lograrnos que el Si medimos la aceleración podemos conocer la fuerza
cuerpo deslice sobre la superficie es necesario aplicar resultante sobre el bloque aplicando la segunda ley de
una fuerza para que éste continúe en movimiento. Newton, F = ma .
Cuando el dinamómetro indica 12 N la fuerza
Comportamiento de un cuerpo que descansa sobre resultante a partir de la aceleración medida es 4 N,
un plano horizontal esto significa que se necesita 12 N – 4 N = 8 N, para
Supongamos que jalamos un bloque con un vencer la fuerza de fricción Si aplicamos 10 N al
dinamómetro, como se muestra en la figura. bloque para que inicie el movimiento, después de esto
es posible reducir la fuerza a 8 N y aún mantener el
Comportamiento de un cuerpo que descansa sobre bloque en movimiento.
un plano horizontal En resumen:
Una fuerza de 10 N inicia el movimiento del bloque.
Una fuerza de 8 N mantiene el movimiento del
bloque.

El origen de este fenómeno se debe a la existencia de


fuerzas entre las moléculas del cuerpo y la superficie;
Dibujemos una gráfica de la fuerza F aplicada sobre si la superficie de contacto del cuerpo con la
el bloque versus el tiempo t . superficie fuera perfectamente plana, la fuerza de
atracción podría ser considerable, como es el caso de
dos placas de vidrio perfectamente limpias que una
vez puestas en contacto, difícilmente pueden ser
separadas.
Las superficies nunca son perfectamente lisas y las
imperfecciones constituyen verdaderos obstáculos al
desplazamiento como se muestra en la figura. Es
preciso vencer estos obstáculos para iniciar el
movimiento y también para mantenerlo.

1. Desde el origen hasta el punto A la fuerza F


aplicada sobre el bloque no es suficientemente grande
como para moverlo. Estamos en una situación de
equilibrio estático
F = Ffs = μ s N
En el punto A, la fuerza de rozamiento F fs alcanza
su máximo valor μ smáx N A esta fuerza se le conoce como FUERZA DE
F = Ffsmáx = μ smáx N FRICCION O ROZAMIENTO F f . ( )
2. Si la fuerza F aplicada se incrementa un poquito Con la finalidad de conocer la dependencia de esta
más, el bloque comienza a moverse. La fuerza de fuerza de rozamiento realicemos la siguiente
rozamiento disminuye rápidamente a un valor menor experiencia.
e igual a la fuerza de rozamiento dinámico, Supongamos un plano inclinado con un bloque de
F = F fk = μ k N masa ni descansando sobre él.
Si la fuerza F no cambia, punto B, y permanece igual
a F fsmáx , el bloque comienza moviéndose con una
aceleración

12
Dinámica de una partícula Hugo Medina Guzmán

intermoleculares son tanto mayores, cuanto mayor es


la superficie de contacto. En realidad se debía esperar
que F f fuera proporcional a la superficie, lo que
suceder es que si el cuerpo pesa muy poco,
prácticamente no hay puntos de contacto entre las dos
superficies (el área de contacto es despreciable).
Cuando N aumenta, la superficie aumenta y F f
Encontramos que el bloque empieza a resbalar para también, por lo tanto F f = μN donde se está
un determinado ángulo θ . Si colocamos dos bloques
juntos, el ángulo con el cual inician el movimiento incluyendo ya el aumento de superficie. Es decir, la
sigue siendo θ , lo mismo ocurre con tres bloques. La fuerza de fricción F f es proporcional a la fuerza
fuerza que jala al cuerpo es la componente del peso normal N porque la verdadera superficie de contacto
mgsenθ , paralela al plano. La otra componente es es proporcional a la fuerza normal.
perpendicular al plano mg cos θ . Esta es la fuerza
Ejemplo 15. ¿Cuál es la fuerza mínima F necesaria
que sostiene al bloque sobre la superficie (Fuerza para mover la masa m , siendo μ el coeficiente de
Normal). Si duplicarnos el peso mg a 2mg,
rozamiento estático entre el piso y el bloque en cada
duplicamos la fuerza que jale al bloque y la fuerza
uno de los casos siguientes?
normal tal que:
Fuerza que inicia el movimiento
= Constante
Fuerza normal
O
mg senθ
= tan θ = μ s = Constante
mg cos θ Solución.
Ff a) La figura muestra el D.C.L.
= μs
N
A esta constante μs se le llama coeficiente de
fricción estática.
Si se toman los datos con el bloque en movimiento, el
ángulo para que el movimiento continúe es
generalmente menor y obtenemos
Fuerza para continuar el movimiento
= μk
Fuerza normal
A esta constante se le llama coeficiente de fricción ∑F y : N − mg = 0 ⇒ N = mg
cinética μ k . ∑F x : F − μN = 0 ⇒ F = μN
μ es una constante que depende de la superficie y se Luego:
puede escribir simplemente. F = μmg
F f = μN . b) La figura muestra el D.C.L.
Algunos valores típicos de coeficientes de fricción.

Material Sobre μs μk
material
Acero Acero 0,78 0,42
Cuero Cuero 0,64 0,56
Cuero Roble 0,60 0,50
Bronce Hierro 0,40 0,30
Aluminio Aluminio 1,05 1,40
Vidrio Vidrio 0,92 0,40
Caucho Asfalto 0,60 0,40 ∑F y : N + Fsenθ − mg = 0
Caucho Concreto 0,80 0,70
⇒ N = mg − Fsenθ
Caucho Hielo 0,02 0,005
Piedra Piedra 0,65 0,60 ∑ Fx : F cosθ − μN = 0
El hecho que la fuerza de fricción es independiente
⇒ F cos θ = μN
del área de contacto parece absurdo ya que las fuerzas De estas dos ecuaciones obtenemos:

13
Dinámica de una partícula Hugo Medina Guzmán

μmg une los bloques m1 y m2. El coeficiente de rozamiento


F= entre los bloques y el plano inclinado es μ .
cos θ + μsenθ
c) La figura muestra el D.C.L.

Solución.
∑F y : N − Fsenθ − mg = 0
⇒ N = mg + Fsenθ
∑ Fx : F cosθ − μN = 0
⇒ F cos θ = μN
De estas dos ecuaciones obtenemos:
μmg
F=
cos θ − μsenθ

Ejemplo 16. ¿Cuál es el valor mínimo de F para


Para m0 : {m0 g − T1 = m0 a
sostener el bloque de masa m sobre una pared ⎧T1 − T2 − μN 2 = m2 a
vertical, como se muestra en la figura, μ es el Para m 2 : ⎨
coeficiente de fricción estático entre la pared y el ⎩ N 2 − m2 g = 0
bloque? ⎧T2 − μN 1 = m1 a
Para m1 : ⎨
⎩ N 1 − m1 g = 0
De estas ecuaciones obtenemos:
N 2 = m2 g , N 1 = m1 g
y m0 g − μ (m1 + m2 )g = (m0 + m1 + m2 )a
De aquí:
Solución. a=
[m0 − μ (m1 + m2 )] g
La figura siguiente muestra el D.C.L. (m0 + m1 + m2 )
La tensión del cable que une los bloques m1 y m2:
m1 m0
T2 = m1 (a + μg ) = (1 + μ )g
(m0 + m1 + m2 )
Ejemplo 18. Se tiene una masa m 2 sobre una masa
m1 sobre un piso horizontal, tal como muestra la
∑ Fy : N − F = 0 ⇒ N = F figura. Se aplica una fuerza horizontal F sobre la
mg masa m1 . La masa carece de fricción. ¿Cuál es el
∑F x : μN − mg = 0 ⇒ N =
μ valor máximo de F para que la masa m1 no resbale
Por consiguiente sobre m 2 . ¿Cuál es la aceleración resultante de los
mg bloques?
F=
μ
Ejemplo 17. En el esquema de la figura las masas de
la polea y del cable son despreciables y no hay
rozamiento entre el cable y la polea. Hallar la
aceleración del bloque m0 y la tensión del cable que

14
Dinámica de una partícula Hugo Medina Guzmán

Solución.
La figura muestra el D.C.L. de las masas m1 y m 2 .

Solución.
La figura muestra el D.C.L. para este caso

Aplicando la Segunda Ley de Newton a la masa m 2 ,


la que suponemos se mueve con aceleración a 2 .

∑F y : N 2 − m2 g = 0
Las ecuaciones para la masa m 2 son
∑F : μN 2 = m2 a 2
∑F
x
y : N 2 − m2 g = 0
Aplicando la Segunda Ley de Newton a la masa m1 ,
la que suponemos se mueve con aceleración a1 . ∑F x : F − μN 2 = m2 a 2
Las ecuaciones para la masa m1 son.
∑F y : N 1 − N 2 − m1 g = 0
∑F x : F − μN 2 = m1 a1 ∑F y : N 1 − N 2 − m1 g = 0
Trabajando con estas ecuaciones encontramos que ∑F x : μN 2 = m1 a1
F = m1 a1 + m2 a 2 Trabajando con estas ecuaciones encontramos que
La aceleración de la masa m 2 es: F = m1 a1 + m2 a 2
μN 2 μm2 g La aceleración de la masa m1 es:
a2 = = = μg μN 2 μm2 g m
m2 m2 a1 = = = μg 2
Como el valor de μ varía desde 0 hasta el valor m1 m1 m1
máximo μ máx : Como el valor de μ varía desde 0 hasta el valor
máximo μ máx :
a 2 = μ máx g o simplemente a 2 = μg .
Pero como queremos encontrar el valor máximo m2
posible de F para que las masas vayan juntas, es
a1 = μ máx g
m1
decir, para que m1 no se quede, se tiene como
Como la condición de que las masas m1 y m 2 vayan
condición que;
juntas es,
a1 = a 2 = μg
a1 = a 2
Luego: Fmáx = (m1 + m 2 )μ máx g
Luego el valor máximo de F pera que m1 y m 2
Si aplicamos una fuerza mayor el bloque m1 vayan juntas es,
avanzará dejando atrás al bloque m 2 . (m1 + m2 )m2
Fmáx = μ máx g
m1
Ejemplo 19. Usando el dispositivo del ejemplo
anterior discuta el caso en ci que la fuerza F se aplica
Ejemplo 20. En el dispositivo de la figura
a la masa m 2 . encontramos el valor mínimo de F para sacar la masa
m1 .
El coeficiente de fricción entre m1 y la mesa es μ1 y
el coeficiente de fricción entre m1 y m 2 es μ2 .

15
Dinámica de una partícula Hugo Medina Guzmán

Solución.
Solución.
La figura muestra los D.C.L. de las masas m1 y m 2
La figura muestra el D.C.L.de las masas m1 y m 2

Considerando que el equilibrio es la condición


Considerando que el equilibrio es la condición mínima de inicio del movimiento.
mínima de inicio de movimiento Aplicando la segunda ley de Newton a la masa m 2 :
Aplicando la Segunda ley de Newton para la masa
m2 .
∑F y : N 2 − m2 g = 0

∑F y : N 2 − m2 g = 0 ∑F x : μ2 N2 − T = 0
Aplicando la segunda ley de Newton para la masa
∑F x : μ2 N2 − T = 0 m1 :
Aplicando la Segunda Ley de Newton para la masa
m1
∑F y : N 2 − N 1 + m1 g = 0

∑F y : N 2 − N 1 + m1 g = 0 ∑F x : F − μ1 N1 − μ 2 N 2 − T = 0
Resolviendo estas ecuaciones
∑F x : F − μ1 N 1 − μ 2 N 2 = 0 N 2 = m2 g
Resolviendo estas ecuaciones T = μ 2 N 2 = μ 2 m2 g
N 2 = m2 g
N 1 = N 2 + m1 g = (m1 + m2 )g
T = μ 2 N 2 = μ 2 m2 g
F = μ1 N1 + μ 2 N 2 + T
N 1 = N 2 + m1 g = (m1 + m2 )g
= μ1 (m1 + m2 )g + μ 2 m2 g
F = μ1 N 1 + μ 2 N 2
= [μ1 m1 + m2 (μ1 + μ 2 )]g
= μ1 (m1 + m 2 )g + μ 2 m2 g Siendo este valor de F el mínimo para iniciar el
Siendo este valor de F el mínimo para iniciar el movimiento.
movimiento de la masa m1 .
Ejemplo 22. Los bloques m1 y m 2 de 20 y 60 kg,
Ejemplo 21. En el dispositivo de la figura, encontrar respectivamente, están unidos por una cuerda de masa
el valor mínimo de F para sacar la masa m1 . El despreciable que pasa por una polea sin rozamiento.
El coeficiente de rozamiento cinético entre las masas
coeficiente de fricción entre m1 y la mesa es μ1 , el y la superficie es 0,3. Determinar la velocidad del
coeficiente de fricción entre m1 y m 2 es μ2 . sistema 4 segundos después de partir del reposo.

16
Dinámica de una partícula Hugo Medina Guzmán

Solución. Ejemplo 23. En una mesa un plato descansa sobre el


La figura muestra el D.C.L. de la masa m1 . mantel, cuyo centro está a 0,25m del borde de la
mesa. El mantel se jala súbitamente en forma
Consideremos que el movimiento es de izquierda a
horizontal con una aceleración constante de 10 m/s2.
derecha con aceleración a
El coeficiente de fricción cinético entre el mantel y el
plato es μ k = 0,75 . Asumiendo que el mantel llega
justo al borde de la mesa.
Cuando el extremo del mantel pasa bajo el centro del
plato, encontrar:
a) La aceleración del plato
b) La velocidad de! plato
c) La distancia del plato al borde de la mesa.
Solución.
a) Aplicando la segunda ley de Newton para el plato,
∑F y : N 1 − m1 g cos 30º = 0 la masa del plato es m y su aceleración a p .

∑F x : T − F f 1 − m1 gsen30º = m1 a
De estas ecuaciones
3
N 1 = m1 g cos 30º = 20 × 10 × = 173 N
2
F f 1 = μN 1 = 0,3 × 173 = 51,9 N
1
y T − 51,9 − 20 × 10 × = 20a
2
⇒ T = 151,9 + 20a ∑F V = 0 ⇒ mg − N = 0
La figura muestra D.C.L. de la masa m 2 . ∑F H = ma p ⇒ F f = ma p
De aquí obtenemos:
N = mg y μ k mg = ma p
De donde:
a p = μ k g = 0,75 x 9,8 = 7,35 m/s2
El plato resbala ya que a p es menor que 10 m/s2
b) En el instante en que el extremo del mantel
coincide con el centro del plato están a la misma

∑F y : N 2 − m2 g cos 60º = 0 distancia del borde de la mesa

∑F x : m2 gsen 60º − F f 2 − T = m2 a
De estas ecuaciones
1
N 2 = m2 g cos 60º = 20 × 10 × = 150 N
2
Ff 2 = μN 2 = 0,3 × 150 = 45 N
3 x p = xm
y 30 × 10 ×− 45 − T = 30a
2
⇒ T = 214,5 − 30a 1 1
Igualando los valores de T: x p = 0,25 + a p t 2 = 0,25 + 7,35t 2
2 2
m
151,9 + 20a = 214,5 − 30a ⇒ a = 1,25 1 1
s2 x m = a m t 2 = 10t 2
2 2
Como v = v0 + at , Igualando
Siendo v0 = 0 ⇒ v = 1,25t 2
1 1
0,25 + 7,35t 2 = 10t 2
m 2 2
Para t = 4 s ⇒ v = 1,25 × 4 = 5 Resolviendo:
s
t = 0,58 s y

17
Dinámica de una partícula Hugo Medina Guzmán

v p = v0 + a p t = 0 + 7,35 × 0,58 horizontal con coeficiente de fricción µ. La polea por


donde cuelga otro bloque de masa M no tiene roce y
= 4,26 m/s. la cuerda se considera inextensible y de masa
1 despreciable. Calcular la aceleración y la tensión de
c) x p = 0,25 + a pt 2 la cuerda.
2
1
= 0,25 + 7,35 × 0,58 = 1,49 m
2

2
Ejemplo 24. El plano inclinado mostrado en la figura
tiene una aceleración a hacia la derecha. Si el
coeficiente de fricción estático entre el plano y el
bloque es μ , encontrar la condición para que el
bloque resbale.
Solución.
Se hacen los DCL y se aplica la segunda ley de
Newton, suponiendo que el cuerpo de masa M
desciende y tira a m hacia la derecha, lo que define el
sentido de la aceleración.
Para m

Solución.
Consideremos que el bloque tiene masa m , la figura
a continuación muestra su DCL.

∑F V = 0 ⇒ N + F senα − mg = 0
⇒ N = mg − F senα (1)
y ∑ FH = ma
⇒ T − F cos α − F f = ma (2)
Para M
Para que el bloque no resbale debe tener la misma
aceleración a .
Aplicando la segunda ley de Newton
∑ F = 0 ⇒ N cos α + μN senα − mg = 0
V

y ∑ F = ma ⇒ − N senα + μN cos α = ma
H
De estas ecuaciones
mg
N= y
cos α + μ senα
mg
(− senα + μ cos α ) = ma ∑F = − Ma
((cos α + μ senα )) V

Finalmente ⇒ T − Mg = − Ma (3)

a=
(μ cos α − senα ) g Además: F f = μN
(cos α + μ senα ) De la ecuación (1):
Este es el va1or crítico de a para que no resbale; el F f = μ (mg − Fsenα ) (4)
bloque resbalará para valores menores que el
De (3) se despeja T:
indicado.
T = Mg − Ma (5)
Ejemplo 25. En el siguiente sistema mecánico, se
aplica una fuerza F inclinada un ángulo α sobre el Ahora 4) y (5) se reemplazan en (2), lo que permite
cuerpo de masa m, ubicado sobre la superficie despejar la aceleración

18
Dinámica de una partícula Hugo Medina Guzmán

Mg − Ma − F cos α − μ (mg − Fsenα ) = ma deslizarse sobre la viga? ¿Dentro do cuánto tiempo el

a=
(M − μm )g − F (cos α − μsenα ) cuerpo caerá de la viga? La longitud do la viga es l .

M +m
y la tensión T

T = Mg − M
(M − μm )g − F (cos α − μsenα )
M +m
Solución.
Ejemplo 26. Una viga de masa M está situada en un
plano horizontal. Sobre la viga se encuentra un
cuerpo do masa m. El coeficiente de rozamiento entre
el cuerpo y la viga, así como entre la viga y el plano
es μ k . Analizar el movimiento para diferentes
valores do la fuerza F. Las ecuaciones del movimiento de la viga y del
cuerpo tienen la siguiente forma:
F fm = ma m , (1)
F − μ k mg = Ma M (2)
Donde F fm es la fuerza do rozamiento, am y aM son
las aceleraciones.
Solución. Supongamos que no hay deslizamiento, entonces
Si F ≤ μ k (m + M )g , no hay movimiento. am = aM
F > μ k (m + M )g . Analicemos
De las ecuaciones del movimiento podemos
Supongamos que
determinar la aceleración y la fuerza de rozamiento.
el caso de ausencia de deslizamiento del cuerpo por la mF
viga. Las ecuaciones del movimiento, en este caso, La fuerza de rozamiento es F fm =
tendrían la siguiente forma: (m + M )
F fm = ma , Para que no haya deslizamiento la fuerza de
rozamiento debe satisfacer la siguiente desigualdad:
Ma = F − F fm − F fM = F − F fm − μ k (m + M )g ; F
F fm ≤ μ k mg , es decir, ≤μ g.
F fm ≤ μ k mg (m + M ) k
de donde Si F > μk (M + m) g, entonces surge el deslizamiento.
F Las ecuaciones (1) y (2) en este caso deben escribirse
a= −μ g,
(m + M ) k en la siguiente forma:
ma m = μ k mg , Ma M = F − μ k mg
mF
F fm = − μ mg ≤ μ k mg De estas ecuaciones obtenemos am y aM:
(m + M ) k (F − μ k mg )
que es posible, si am = μ k g , aM = .
k (m + M) g < F < 2k (m + M) g. M
Si F > 2μk(m + M)g, entonces el cuerpo deslizará por Es evidente que aM > am.
la barra. En este caso las ecuaciones del movimiento
tendrán la siguiente forma:
ma m = μ k mg ,
Ma M = F − μ k mg − μ k (M + m )g
1 1
de donde xm = am t 2 , xM = aM t 2
F (2m + M ) g 2 2
am = μ k g , aM = − μk 1 1
M M xM − xm = l = a M t 2 − am t 2
2 2
Que es fácilmente verificar en el caso de a M > a m
2l 2l
⇒ t= =
Ejemplo 27. Una viga do masa M está sobre un plano aM − am (F − μ k mg )
horizontal liso, por el cual puede moverse sin − μk g
M
fricción. Sobre la viga hay un cuerpo do masa m. El
coeficiente de rozamiento entre el cuerpo y la viga es 2lM
=
μ k . ¿Con qué valor de la fuerza F que actúa sobre la F − μ k g (M + m )
viga en dirección horizontal, el cuerpo comienza a

19
Dinámica de una partícula Hugo Medina Guzmán

Ejemplo 28. En la figura, encontrar la aceleración


del carro requerida para evitar que caiga el bloque B.
El coeficiente de rozamiento estático entre el bloque
y el carro es μ k .

Cuerpo 1: ∑F verticales = m1 g − T = m1 a
Cuerpo 2: ∑ F horizontales = T = m2 a
Solución. ⎧⎪∑ F verticales = N 3 − N 2 − T − m3 g = 0
Mesa: ⎨
Si el bloque no cae, la fuerza de fricción, Ff, debe ⎪⎩∑ F horizontales = T − Ff 3 = 0
balancear el peso del bloque:
Donde N3 y Ff3 (fricción) las componentes verticales
Ff = mg. y horizontales de la fuerza ejercida por el piso sobre
Pero el movimiento horizontal del bloque está dado la mesa.
por y N = ma. (Asumimos que las patas de la izquierda y de la
derecha comparten la carga igualmente. Esto no
Luego, afecta nuestro análisis)
Ff g g De las primeras dos ecuaciones,
= ⇒ a= m1 g
N a Ff a=
N (m1 + m2 )
Ff m1 m2 g
Como el valor máximo de es μ s , debemos Luego, F f 3 = T = m 2 a =
N (m1 + m2 )
g Finalmente,
tener a ≥ si el bloque no cae.
μs N 3 = T + m2 g + m3 g
⎡ m1m2 ⎤
Ejemplo 29. Dos cuerpos, de las masas m1 y m2, se = ⎢ + m2 + m3 ⎥
liberan de la posición mostrada en la figura. Si la ⎣ (m1 + m2 ) ⎦
masa de la mesa de superficie lisa (sin fricción) es
m3, encuentre la reacción del piso sobre la mesa Ejemplo 30. Se tiene un bloque de 20 kg sobre un
mientras los dos cuerpos están en movimiento. plano inclinado que está sujeto a una cuerda (ver
Asuma que la mesa permanence inmóvil. figura). Las superficies de contacto entre el bloque y
el plano inclinado son rugosas con coeficiente de
fricción cinética μk = 0,5 y el de fricción estática
μs = 0,7.
a) Si la tensión de la cuerda es de 150 N, determine la
magnitud y sentido de la fuerza de rozamiento.
b) Si por un accidente se corta la cuerda, determine
la aceleración del bloque.

Solución. La figura muestra los diagramas de cuerpo


libre de cada uno de los elementos.

Solución.
a)

20
Dinámica de una partícula Hugo Medina Guzmán

T − mgsen30º − F f = 0 ⇒
F f = T − mgse30º = 150 − 100 = 50 N
La segunda ley de Newton para m1 es
en el sentido indicado en la figura (hacia abajo).
b) T − m1 a = 0 , N 1 − m1 g = 0
De aquí ⇒ T = m1 a (2)
La segunda ley de Newton para m2 es
N 2 − m2 a = 0 , T − m2 g = 0
De aquí ⇒ T = m2 g (3)
De (2) y (3) se tiene
m2
⇒ a= g (4)
Cuando se rompe la cuerda para iniciar el m1
movimiento debe vencerse a la máxima fuerza de Sustituyendo (4) en (1) se obtiene la fuerza aplicada a
fricción estática: M
⎛ 3⎞ m2
F fs = μ s mg cos 30º = 0,7⎜⎜ 20 g ⎟ = 173 N
⎟ F= (M + m1 + m2 )g
⎝ 2 ⎠ m1
Como 20g sen 30º = 100 N
100 N < 173 N, el movimiento no se inicia , por lo Ejemplo 32. Determinar la aceleración mínima con
tanto la aceleración del bloque es cero. que debe desplazarse el bloque de masa M en sentido
horizontal para que los bloques de masas m1 y m2 no
Ejemplo 31. Determinar la fuerza F aplicada al se muevan respecto de M, siendo μ el coeficiente de
bloque de masa M de la figura adjunta, para que los rozamiento entre los bloques. La polea y el cable
bloques de masas m1 y m2 apoyados en M, no se tienen masa despreciable.
muevan respecto de M. Todas las superficies son
lisas, la polea y el cable tienen masa despreciable.

Solución.
Consideremos un sistema de referencia fijo en el
suelo con el eje x paralelo a la fuerza aplicada
Solución. →
Consideremos un sistema de referencia fijo en el F.
suelo con el eje x paralelo a la fuerza aplicada De la segunda ley de Newton aplicada al conjunto se

tiene:
F. → →
De la primera ley de Newton aplicada al conjunto se F = (M + m1 + m2 ) a (1)
tiene:

→ →
F = (M + m1 + m2 ) a (1) Siendo a la aceleración del conjunto.
Las masas m1 y m2 están en reposo sobre el bloque M,

luego en la referencia O su aceleración es del
Siendo a la aceleración del conjunto. Las masas m1
conjunto.
y m2 están en reposo sobre el bloque M, luego en la
La fuerza que ejerce el cable sobre m1 y la que ejerce
referencia O su aceleración es del conjunto. La fuerza
sobre m2 tiene el mismo módulo T.
que ejerce el cable sobre m1 y la que ejerce sobre m2
tiene el mismo módulo T.

21
Dinámica de una partícula Hugo Medina Guzmán

Diagrama del cuerpo libre masas separadas

La segunda ley de Newton para m1 es


T − m1 a − F f 1 = 0 , N1 − m1 g = 0
F f 1 = μN 1 = μm1 g
⇒ T = m1 a + μm1 g (2) a) Consideremos un sistema de referencia fijo en el

La segunda ley de Newton para m2 es
suelo con el eje x paralelo a la fuerza aplicada F .
N 2 − m2 a = o , T + F f 2 − m2 g = 0 Sea τ el instante en que m empieza a deslizar sobre
F f 2 = μN 2 = μm2 a M. Hasta dicho instante t ≤ τ , el conjunto se mueve

⇒ T = m2 g − μm2 g (3) con una aceleración común a .
De (2) y (3) se tiene ⇒ La segunda ley de Newton aplicada al conjunto en el
instante t = τ es
m1 a + μm1 g = m2 g − μm2 g
kτ = (M + m )a(τ ) , N 2 − (M + m )g = 0
(m − μm1 )
a= 2 g (4) k
(m1 + μm2 ) ⇒ a (τ ) = τ (1)
Sustituyendo (4) en (1) se obtiene la fuerza aplicada a
(M + m )
M La segunda ley de Newton aplicada a la masa m en el
instante t = τ es, ( la fuerza de rozamiento sobre m
m2
F= (M + m1 + m2 )g tiene, en ese instante, su valor máximo Ff = μ m g )
m1 F f = μN 1 = ma(τ ) , N 1 = mg
μmg
Ejemplo 33. Un bloque de masa m se encuentra ⇒ a(τ ) = = μg (2)
sobre otro bloque de masa M que está apoyado sobre m
una superficie horizontal lisa. El coeficiente de
De (1) y (2) queda ⇒ τ=
(M + m ) μg
rozamiento entre los dos bloques es μ. Al bloque M s
se le aplica una fuerza horizontal dirigida hacia la k
derecha que depende del tiempo según la ley F = k t.
Determinar: b) De (1) se tiene que la aceleración del conjunto para
a) El instante τ en que m empieza a deslizar sobre M. t < τ es
b) La aceleración de cada uno de los bloques. k
⇒ a1(t ) = a(t ) = t
(M + m )
Para t > τ . Las fuerzas que actúan sobre m son
constantes, luego la aceleración de m es
a1 = a(τ ) = μg
La segunda ley de Newton aplicada a la masa M es
kt − F f = kt − μN 1 = Ma 2 (t ) , como N 1 = mg
Solución. ⇒ kt − μmg = Ma 2(t ) y
Diagrama del cuerpo libre del conjunto
m k m
a2(t ) = − μg + t 2
M M s

Gráfica de las aceleraciones en función del tiempo

22
Dinámica de una partícula Hugo Medina Guzmán

El movimiento de B es hacia arriba, luego


⇒ mB g < T
El movimiento de A es hacia abajo, luego
⇒ T + μm A cos θ < m A gsenθ
El movimiento de los bloques es el indicado si
⇒ m B g < m A gsenθ − μm A cos θ
mB
Ejemplo 34. Dos bloques A y B de masas mA y mB ⇒ < senθ − μ cos θ
están unidos mediante un cable que pasa a través de mA
una polea tal como se muestra en la figura adjunta. El Los bloques no se mueven si
coeficiente de rozamiento entre el bloque A y el plano
inclinado es μ. Determinar el sentido del movimiento
mB
⇒ senθ − μ cos θ < < senθ + μ cos θ
cuando se dejan en libertad a partir del reposo. El mA
cable es inextensible y las masas del cable y la polea
Ejemplo 35. Dos bloques A y B de masas m A = 10
despreciables.
kg y m B = 7 kg, están unidos mediante un cable que
pasa a través de las poleas tal como se muestra en la
figura adjunta. El coeficiente de rozamiento entre el
bloque A y el plano inclinado es μ = 0,10 y θ =
30º. El cable es inextensible y las masas del cable y
las poleas son despreciables. Determinar:
a) Las aceleraciones de los bloques;
b) La tensión del cable.
Solución.
Supongamos que el bloque A sube sobre el plano
inclinado. Sea T la fuerza que ejercen los extremos
del cable sobre los bloques dirigida, en ambos
bloques, tal como se indica.

Solución.

El movimiento de B es hacia abajo, luego


⇒ mB g > T
El movimiento de A es hacia arriba, luego
⇒ T > m A gsenθ + μm A cos θ
Supongamos que el movimiento de A es hacia abajo,
El movimiento de los bloques es el indicado si
luego:
⇒ m B g > m A gsenθ + μm A cos θ
T + μm A g cos θ < m A gsenθ
m
⇒ B > senθ + μ cos θ ⇒ T < m A gsenθ − μm A g cos θ
mA El movimiento de B es hacia arriba, luego:
m B g < 2T
Supongamos que el bloque A desciende sobre el
De ambas expresiones queda
plano inclinado.
1
m B g < m A gsenθ − μm A g cos θ
2
1
(7 ) < (10)sen30º −0,10(10) cos 30º
2
Con los valores ⇒ 3,5 < 4,13
Desigualdad que se cumple, luego el movimiento es
el previsto.

23
Dinámica de una partícula Hugo Medina Guzmán

a) Consideremos un sistema de referencia con el eje x


horizontal. Las posiciones de los bloques están
relacionadas por la condición de ligadura
s A + 2 y B = constante ,
Luego sus aceleraciones cumplen
1
a A + 2a B = 0 ⇒ a B = − a A = a (1) Supongamos que el bloque A asciende por el plano
2 inclinado. Consideremos un sistema de referencia con
Fuerzas sobre los bloques
el eje x horizontal.
La segunda ley de Newton aplicada al bloque A es
Las posiciones, por una parte, del bloque A y de la
m A a A = m A gsenθ − T − μN A , polea móvil, están relacionadas por las condiciones
N A − m A g cos θ = 0 de ligadura
De estas dos obtenemos: s A + h − y p = constante
T = m A g (senθ − μ cos θ ) − m A a A (2) Las posiciones de la polea y el bloque B, están
La segunda ley de Newton aplicada al bloque B es relacionadas por las condiciones de ligadura
2T − m B g = m B a B 2 y p − y B = constante
1 De estas dos ecuaciones obtenemos:
⇒ T = m B (a B + g ) (3) 2 s A + 2h − y B = constante
2
Igualando las ecuaciones (2) y (3), Las componentes de las aceleraciones de los bloques
satisfacen la condición
m B (a B + g ) = 2m A g (senθ − μ cos θ ) − 2m A a A
Teniendo en cuenta la ecuación (1) y los valores:
2a A = a B (1)
7(a + 9,8) = 2(10 )(9,8)(0,5 − 0,1 × 0,87 ) − 20(2a )
Resolviendo:
a = 0,26 m/s2
Las aceleraciones de los bloques son :
a A = 0,26 m/s 2 para arriba.
a B = 0,52 m/s 2 para abajo.
b) La magnitud de la tensión del cable es el valor de
la fuerza que el cable ejerce sobre los bloques. De la
Sean T A y TB las fuerzas que los cables ejercen
ecuación (3) se tiene
1 sobre los respectivos bloques. Fuerzas sobre los
T= (7 )(0,26 + 9,8) = 35,2 N bloques y sobre la polea móvil.
2 Como la polea superior tiene masa despreciable solo
cambia el sentido de la fuerza.
Ejemplo 36. Dos bloques A y B de masas m A y La masa de la polea móvil es cero, luego
m B están unidos mediante un cable que pasa a La tensión en ambos lados son iguales (TB ) y
través de las poleas tal como se muestra en la figura T A = 2TB (2)
adjunta. El coeficiente de rozamiento entre el bloque De la segunda ley de Newton aplicada al bloque A se
A y el plano inclinado es μ . El cable es inextensible tiene:
y las masas del cable y la polea son despreciables. TA − mA gsenθ − μN A = mAa A
Estudiar el sentido del movimiento de los bloques.
N A − m A g cos θ = 0
De estas ecuaciones obtenemos:
T A = m A g (senθ + μ cos θ ) + m A a A (3)
De la segunda ley de Newton aplicada al bloque B se
tiene
m B g − TB = m B a B
⇒ TB = m B ( g − a B ) (4)
Solución. De las ecuaciones (1), (2), (3) y (4) obtenemos:
m A g (senθ + μ cos θ ) + m A a A = 2m B ( g − 2a A )
2m B g − m A g (senθ + μ cos θ )
aA =
m A + 4m B

24
Dinámica de una partícula Hugo Medina Guzmán

El movimiento es el indicado, si se cumple: Las fuerzas exteriores que actúan sobre la m1 son la
2m B
> (senθ + μ cos θ ) tensión del cable T y el peso m1 g , y sobre m 2 son
mA la tensión del cable T y el peso m2 g .
El movimiento es de sentido opuesto, si se cumple: De la ecuación fundamental de la dinámica en la
2 mB
< (senθ − μ cos θ ) referencia no inercial se tiene
mA g
El signo menos es porque en este caso el peso de la
m1 a '1 = T − m1 g − m1
2
masa A es el que mueve al sistema y la fuerza de
3
rozamiento está en sentido contrario a éste. ⇒ m1 a'1 = T − m1 g (1)
2
Ejemplo 37. A los extremos de un hilo que pasa a g
través de una polea fija al techo de la cabina de un m1 a ' 2 = T − m2 g − m2
2
ascensor se atan los cuerpos de masa m1 y m 2
3
(m1 < m2 ) . La cabina comienza a subir con una ⇒ m1 a' 2 = T − m2 g (2)
2
aceleración constante g / 2. Despreciando la masa de
De la condición de ligadura para los bloques se tiene
la polea y la del hilo, así como el rozamiento,
calcular: a '1 +a ' 2 = 0 ⇒ a '1 = − a ' 2 = a ' (3)
a) La aceleración de m1 y m 2 respecto de la cabina De las ecuaciones (1), (2) y (3) se obtiene
y con relación al foso del ascensor. 3 3
m1 a' = T − m1 g y m1 a' = −T + m2 g
b) La fuerza con la cual la polea actúa sobre el techo 2 2
de la cabina. Sumando estas ecuaciones:

(m2 + m1 )a' = 3 (m2 − m1 )g


2
Despejando a '
3 (m2 − m1 )
a' = g
2 (m2 + m1 )
Finalmente:
→ →
a'1 = a ' ˆj y a' 2 = −a' ˆj

En la referencia fija, las aceleraciones de m1 y de


m 2 se obtienen de sumar a las anteriores la
Solución. aceleración del ascensor
a) El ascensor constituye una referencia no inercial en
traslación que se mueve con una aceleración g (2m2 − m1 )
a1 = + a' = g y
constante en sentido ascendente respecto de una 2 (m2 + m1 )
referencia fija.
Seleccionemos una referencia con origen O′ en un g (2m1 − m2 )
a 2 = − a' = g
punto del ascensor. La aceleración del origen 2 (m2 + m1 )
O′ respecto de la referencia fija O es la aceleración b)
1
del ascensor gˆj . Sean a '1 ˆj la aceleración de m1
2
y a ' 2 ˆj la aceleración de m 2 en la referencia O’.

La fuerza que la polea ejerce sobre el techo de la


cabina es
F − 2T = 0 ⇒ F = 2T
De la ecuación (1) y (3) se tiene
⎛ 3 ⎞ 3m1m2
T = m1 ⎜ a '1 + g ⎟ = g
⎝ 2 ⎠ (m2 + m1 )

25
Dinámica de una partícula Hugo Medina Guzmán

Luego
6m1 m2
∑F x = F f − ma cos 30º = 0 (1)
F = 2T =
(m2 + m1 ) ∑F y = N − mg + masen 30º = 0 (2)
g⎛ 3⎞
de (1) ⇒ F f = 45 ⎜ ⎟ = 191N
Ejemplo 38. Un niño de masa m = 45 kg se pesa en 2 ⎜⎝ 2 ⎟⎠
una báscula de resorte situada sobre una plataforma
especial que se desplaza por un plano inclinado de de (2) ⇒
ángulo θ = 30º como muestra la figura (no hay ⎛ g⎞
rozamiento entre la plataforma y el plano inclinado). N = mg − masen30º = 45⎜ g − ⎟
¿Cuál será la lectura de la báscula en estas ⎝ 4⎠
condiciones? = 33,45 Kg.
Siendo N la cantidad que marca la báscula.

Solución en una referencia no inercial .


Seleccionemos una referencia con origen O′ (x’,y’) en
un punto de la plataforma. El niño está en reposo
sobre la plataforma.

Solución.
Sea M la masa del conjunto niño - cuña., y a la
aceleración con la que desliza hacia abajo el
conjunto.

Aplicando la segunda ley de Newton al DCL del


niño.
∑F x = F f = ma cos 30º (1)
∑F y = N − mg = − masen 30º (2)
g⎛ 3⎞
Aplicando la segunda ley de Newton al conjunto niño de (1) ⇒ F f = 45 ⎜ ⎟ = 191N
- cuña. 2 ⎜⎝ 2 ⎟⎠
∑F // = Ma ⇒ Mgsen30º = Ma ⇒ de (2) ⇒
g ⎛ g⎞
a = gsen 30º = N = mg − masen30º = 45⎜ g − ⎟
2 ⎝ 4⎠
1 = 33,45 kg
La aceleración del conjunto es a = g Siendo N la cantidad que marca la báscula.
2
Solución en una referencia inercial. Ejemplo 39. Un ascensor de masa total 3M es
Sobre el niño actúan: su peso mg y la reacción Ff en levantado bajo la acción de una fuerza F. El piso del
el apoyo. La indicación de la báscula el valor de la
ascensor está inclinado un ángulo θ , con respecto a
normal.
la horizontal. Además, un bloque de masa M se apoya
sobre el centro del piso rugoso del ascensor (con
coeficiente de fricción estática μ ).
a) Hallar la aceleración del ascensor.
b) Haga el diagrama de cuerpo libre de la masa M.
c) ¿Cuál es el valor máximo de F para que el bloque
dentro del ascensor no resbale respecto del piso del
ascensor?
d) Si el ascensor pierde contacto con la fuerza F y
empieza a caer libremente, calcule el valor de la
fuerza normal entre el bloque y el piso del ascensor, y
Aplicando la segunda ley de Newton al DCL del la fuerza de fricción sobre el bloque.
niño. Solución.
a) Para hallar la aceleración del ascensor.

26
Dinámica de una partícula Hugo Medina Guzmán

Por ser proporcional a la aceleración centrípeta, la


fuerza Fc se llama fuerza centrípeta. Su efecto es
cambiar la dirección de la velocidad de un cuerpo. Se
puede sentir esta fuerza cuando se hace girar a un
objeto atado a una cuerda, ya que se nota el tirón del
objeto. Las fuerzas centrípetas no son diferentes de
otras fuerzas ya conocidas, su nombre se debe a que
apunta hacia el centro de una trayectoria
circunferencial. Cualquiera de las fuerzas ya conocida
F − 3Mg − Mg = (3M + M )a pueden actuar como fuerza centrípeta si producen el
efecto correspondiente, como ser la tensión de una
F − 4Mg F cuerda, una fuerza de roce, alguna componente de la
⇒ a= = −g normal, la fuerza gravitacional en el caso de
4M 4M
b) Diagrama de cuerpo libre de la masa M. movimientos de planetas y satélites, etc.

Ejemplo 40. Un cuerpo de masa m, sujeto al extremo


de una cuerda de longitud L, que describe una
trayectoria circular en el plano horizontal, genera una
superficie cónica, por lo que se llama péndulo cónico.
Determinar la rapidez y el período de revolución de la
c) Para que el bloque dentro del ascensor no resbale masa.
respecto del piso del ascensor se debe cumplir

M ( g + a )senθ ≤ μM ( g + a ) cos θ
⇒ μ ≥ tan θ .

Como a depende de F, y a esta en miembros de la


igualdad, el que el bloque resbale dentro del ascensor
solamente depende del coeficiente de fricción.
d) Si el ascensor pierde contacto con la fuerza F y Solución.
empieza a caer libremente, La partícula está sometida a una aceleración
centrípeta, y la fuerza centrípeta correspondiente está
N = 0, por lo tanto Ff = 0 dada por la componente de la tensión de la cuerda en
dirección radial hacia el centro de la circunferencia.
DINÁMICA DEL MOVIMIENTO CIRCULAR El D. C. L. de la masa m.
La primera ley de Newton dice que un objeto
permanecerá en movimiento uniforme en línea recta
con velocidad constante o en reposo si no actúa una
tuerza sobre él. Entonces cuando un objeto se mueve
en trayectoria circular, debe haber una fuerza sobre él
cambiándole la trayectoria recta. Esta fuerza puede
ser proporcionada por la tensión en una cuerda, para
un objeto que se hace girar en una circunferencia
horizontal al extremo de una cuerda; por la fuerza de
la gravedad para un satélite orbitando la tierra.
Los objetos en movimiento circular no están en Aplicando la segunda ley de Newton:
equilibrio, debe haber una fuerza resultante, de otro
modo sólo habría un movimiento en línea recta. ∑F y =0
⇒ T cosα − mg = 0
FUERZA CENTRÍPETA.
⇒ Tcosα = mg (1)
Una partícula que se mueve sobre una trayectoria
circular de radio R con rapidez constante, se y ∑ Fx = ma
encuentra sometida a una aceleración radial de
magnitud v2/R. Por la segunda ley de Newton, sobre v2
⇒ Tsenα = ma = m (2)
la partícula actúa una fuerza en la dirección de hacia r
el centro de la circunferencia, cuya magnitud es: Dividiendo (2) entre (1):
v2 v2
Fc = mac = m tanα = ⇒ v 2 = rg tan α
R rg

27
Dinámica de una partícula Hugo Medina Guzmán

De la geometría de la figura, r = Lsenα ,


reemplazando se obtiene la rapidez de m:
v 2 = (Lsenα )g tan α
⇒ v = Lg tan α senα

Para calcular el periodo T, esto es el tiempo que


demora en dar una vuelta. Solución. La figura muestra el D.C.L.
Se sabe que
Δx = vΔt , con Δx = 2πr , entonces:
2πr 2πL senα
Δt = =
v Lg tan α senα
L cos α
= 2π
g
Aplicando la segunda ley de Newton.
L cos α
⇒ T = 2π ∑F n = mac ⇒ − T − mgsenθ = − mRω 2
g
∑F t = mat ⇒ − mg cos θ = mRα
Ejemplo 41. Una bola de masa m, atada al extremo La tensión en la cuerda es
de una cuerda se hace ir en un plano horizontal T = mRω 2 − mgsenθ
formando una circunferencia de radio R. Si tiene una
La fuerza tangencial es − mg cos θ y la aceleración
velocidad angular ω , ¿cuál es la tensión en la
cuerda? angular es
g
α =− cos θ
R
d 2θ
Como α = , obtenemos la ecuación
dt
diferencial:
d 2θ g
= − cos θ
Solución. dt R
La figura muestra el D.C.L. cuya solución esta fuera del alcance de este curso.
Pero podríamos encontrar la tensión y fuerza
tangencial para posiciones determinadas, es decir
para valores dados de θ .
⎧T = mRω 2
Para θ = 0º ⎨ ,
⎩ Ft = − mg
⎧T = mRω 2 − mg
Aplicando la segunda ley de Newton a la masa m . Para θ = 90º ⎨ ,
⎩ Ft = 0
∑F n = mac ⇒ − T = −mRω 2
⎧T = mRω 2
∑F t = mat ⇒ 0 = mRα Para θ = 180º ⎨
⎩ Ft = mg
,

La tensión en la cuerda es T = mRω . La fuerza


2

tangencial es cero y la aceleración tangencial α ⎧T = mRω 2 + mg


Para θ = 270º ⎨ ,
también es cero, ya que la velocidad angular es
⎩ Ft = 0
constante.
⎧T = mRω 2
Ejemplo 42. Resolver el problema anterior pero en Para θ = 360º ⎨
el caso que el giro sea en el plano vertical. ⎩ Ft = − mg

28
Dinámica de una partícula Hugo Medina Guzmán

b) Cuando el bloque está en A se dirige a B, su


velocidad es en el sentido antihorario y su aceleración
en el sentido horario. Luego su rapidez disminuye.
c) Si en "B" su velocidad es nula, ¿cuál es la
trayectoria que seguirá la masa m?

Ejemplo 43. Un pequeño bloque de masa m se


v2 v2
desliza sobre una superficie lisa circular de radio R N + mg = ma c = m ⇒ N = − mg + m
como se muestra en la figura. (La pista está sobre un R R
plano vertical y g = aceleración de la gravedad) Si v = 0, el valor de N es negativo, lo que no permite
a) Trace el diagrama de cuerpo libre del bloque al bloque sostenerse sobre la circunferencia, por
cuando se encuentra en "A" y muestre (dibujando los consiguiente el bloque caerá verticalmente.
vectores) la dirección de la fuerza resultante y su
aceleración.
d) Si en "B" su velocidad es gR , ¿qué trayectoria
b) Cuando está en "A", ¿su rapidez aumenta o seguirá la masa m?
disminuye? (Justifique) v2 gR
c) Si en "B" su velocidad es nula, ¿cuál es la N = − mg + m ⇒ N = − mg + m = 0,
trayectoria que seguirá la masa m? R R
el bloque tiene suficiente velocidad para seguir en la
d) Si en "B" su velocidad es gR , ¿qué trayectoria trayectoria circular.
seguirá la masa m?
Ejemplo 44. Un avión describe un rizo (un camino
circular en un plano vertical) de 150 m de radio. La
cabeza del piloto siempre apunta al centro del rizo. La
rapidez del avión no es constante; es mínima en el
cenit del rizo y máxima en el nadir.
a) En el cenit el piloto experimenta ingravidez. ¿Qué
rapidez tiene el avión en ese punto?
b) En el nadir, la rapidez del avión es de 280 km/h.
¿Qué peso aparente tiene el piloto aquí? Su peso real
Solución. es de 700 N.
a) Trace el diagrama de cuerpo libre del bloque Solución.
cuando se encuentra en "A" y muestre (dibujando los a) Sí el piloto siente ingravidez, está en caída libre, y
vectores) la dirección de la fuerza resultante y su v2
aceleración. a=g= , luego
R
v = Rg = (150 )(9,80) = 38,3 m s , o
138 km h .
b) El peso aparente es la suma de la fuerza neta hacia
adentro (arriba) y el peso del piloto, o
v2
P' = P + ma = P + m
R
Aquí:
P = 700 N
P 700
m= = = 71,43 kg
g 9,8
v = 280 km/h = 77,6 m/s
R = 150 m
Luego:
⎛ 77,6 2 ⎞
P' = 700 + 71,43⎜⎜ ⎟⎟ = 3579 N
⎝ 150 ⎠

29
Dinámica de una partícula Hugo Medina Guzmán


Ejemplo 45. Una partícula de masa m que está unida a = a t tˆ + a n nˆ
al extremo de un cable de longitud l , cuyo otro at = gsenθ ,
extremo está fijo, se mueve en un plano vertical, a
partir de un punto A tal que el cable forma con la T − mg cos θ = ma n = 2mg (cos θ 0 − cos θ )
vertical un ángulo θ 0 , iniciando el movimiento con ⇒ a n = 2 g (cos θ 0 − cos θ )
velocidad cero. Determinar: Ejemplo 46. Una partícula de masa m se encuentra en
a) La velocidad de v de la esfera en función de θ. el polo de una semiesfera de radio R, la cual está
b) La tensión del cable en función de θ . apoyada sobre una superficie horizontal. Desplazada
ligeramente de su posición de equilibrio, la partícula
c) La aceleración a en función de θ .
desliza sobre la superficie, la cual se supone lisa.
Determinar:
a) La velocidad v de la partícula en función del
ángulo θ que forma su radio posición con el radio
inicial.
b) El valor de la normal N en función de θ.
c) El valor de θ, en el instante en que la partícula se
despega de la superficie.
Solución.

Solución.
En la referencia de origen O, la esfera recorre una
circunferencia de radio l con velocidad variable v(t).
Las componentes intrínsecas la aceleración son:
dv v2 En la referencia de origen O, la partícula m tiene un
at = , an = movimiento circular no uniforme de radio R. Las
dt l componentes de la aceleración son:
Sobre la masa m actúan la tensión del cable T y su dv v2
peso mg . at = , an =
dt R
De la segunda ley de Newton en componentes n̂ y tˆ Sobre la masa m actúan el peso mg y la reacción en
se tiene:
el apoyo N.
∑F t = mat ⇒ mgsenθ = mat Aplicando la segunda ley de Newton:

∑F n = ma n ⇒ T − mg cos θ = ma n ∑F t = mat ⇒ mgsenθ = mat


a) Para la componente tangencial se tiene: ∑F n = ma n ⇒ N − mgcosθ = − ma n
dv
mgsenθ = m a) De la componente tangencial se tiene:
dt dv ds dv
ds dv mgsenθ = m ⇒ = gsenθ
⇒ = gsenθ dt dt ds
dt ds ⇒ vdv = gsenθ ds = Rgsenθ dθ
⇒ vdv = gsenθ ds = gsenθ ldθ Integrando y teniendo en cuenta las condiciones
Integrando y teniendo en cuenta las condiciones iniciales queda
iniciales queda v 2 = 2 Rg (1 − cos θ )
v = 2 gl(cos θ 0 − cos θ )
2
Finalmente:
v = 2 gl(cos θ 0 − cos θ ) v = 2 Rg (1 − cos θ )
b) Para la componente normal: b) De la componente normal se tiene:
v 2 N = mgcosθ − ma n =
T − mg cos θ = m = 2mg (cos θ 0 − cos θ )
l v2
La tensión del cable es mgcosθ − m = mg cos θ − 2mg (1 − cos θ )
R
T = mg (2 cos θ 0 − 3 cos θ ) La normal es N = mg (3 cos θ − 2 )
c) De las ecuaciones anteriores se tiene la c) La masa m deja de estar en contacto con la
aceleración: superficie cuando N = 0

30
Dinámica de una partícula Hugo Medina Guzmán

N = mg (3 cos θ − 2 ) = 0 m 2 . Los bloques están a la distancia R del eje de


2 rotación. El coeficiente de rozamiento estático entre
⇒ cos θ = ⇒ θ = 48,19º las masas y entre m 2 y la tornamesa es μ
3
Considerando el rozamiento y la masa de la polea
Ejemplo 47. En un parque de diversiones hay un despreciables, encontrar la velocidad angular de la
cilindro grande vertical, de radio R que rota alrededor tornamesa para la cual los bloques justamente
de su eje, con velocidad angular constante ω . comienzan a resbalar.
Explicar cómo es posible que las personas que están
dentro, al retirárseles el piso permanezcan “pegadas”
a la pared interior del cilindro.

Solución.
Solución.
En este problema todo depende de tomar
correctamente la dirección de la fuerza de fricción
entre m1 y m 2 . Consideremos m 2 > m1 , por lo
tanto m 2 tenderá a moverse hacia afuera, jalando a
m1 hacia adentro. La fuerza de fricción actuará en
oposición a su movimiento relativo.
La figura muestra los D.C.L. de los componentes del
La figura muestra el D.C.L del hombre. sistema.

Aplicando La segunda ley de Newton:


Como el hombre no cae, radialmente está en reposo
(R = constante) Aplicando la segunda Ley de Newton
∑ Fr = mac ⇒ − N = −mω 2 R ∑F z = ma z , ∑Fr = ma r y ∑F t = mat
∑F z = 0 ⇒ mg − μN = 0 A la masa m1 :
De estas ecuaciones: mg − μmω R N 1 − m1 g = 0 , − T + F1 = − m1ω 2 R , Ft = 0
2

g A la masa m 2 :
y ω=
μR N 2 − N1 − m2 g = 0 ,
Esto quiere decir que para que suceda el efecto de − T − F1 − F2 = − m2ω 2 R , Ft = 0
suspensión de las personas, la velocidad angular ω
De las ecuaciones obtenemos:
tiene que tener un valor relacionado con el radio R y
el coeficiente de fricción μ . N 1 = m1 , N 2 = (m1 + m2 )g
F1 ≤ μm1 g , F2 ≤ μ (m1 + m2 )g
y 2 F1 + F2 = (m2 − m1 )ω R
Ejemplo 48. En la tornamesa mostrada en la figura el 2
bloque de masa m1 descansa sobre el bloque de masa

31
Dinámica de una partícula Hugo Medina Guzmán

Corno ω puede incrementarse hasta que F1 y F2 Curvas sin peraltar


En estos casos la fuerza de rozamiento es la que nos
alcancen sus valores máximos
proporciona toda la componente normal que servirá
2μm1 g + μ (m1 + m2 )g = (m2 − m1 )ω 2 R para tomar la curva. Siempre que tengamos que ésta
Finalmente es mayor que la aceleración normal el automóvil será
μ (3m1 + m2 ) capaz de tomar la curva, es decir, el caso límite se
ω= alcanza cuando
R(m2 − m1 ) v2
Fr = ma c = m
Ejemplo 49. ¿Cómo afectará la rotación de la tierra
R
al peso aparente de un cuerpo en el ecuador?
Ejemplo 50. ¿Cuál es la velocidad a que puede ir un
Solución.
automóvil por una curva sin peralte, de radio R, sin
La figura muestra la situación de un cuerpo situado
derrapar?, el coeficiente de rozamiento entre las
en la línea ecuatorial
ruedas y el suelo vale μ.
Solución.

Aplicando la segunda ley de Newton


∑F z = maz ⇒ Fz = 0
v2
∑F r = mar ⇒ N − mg = − mω R 2 ∑F h = mac ∑F V = 0 ac =
R
∑F t = mat ⇒ Ft = 0
F f = μN = μmg = m
v2
⇒ v = μgR
El peso de la masa es representado por la reacción N R
N = mg − mω 2 R
Ejemplo 51. El ciclista tiene que inclinarse al
Para tener una idea de cuánto afecta la rotación de la
desplazarse por una pista circular (o para pasar por
tierra es necesario hacer el cálculo numérico para esta
una curva), Encontrar la relación de la velocidad con
consideración:
el radio de curvatura, el ángulo de inclinación y μ
El radio de la tierra en el ecuador: R = 6,378 x l06m
La velocidad angular de la tierra coeficiente de fricción.
2π rad −5 rad
ω= = 7,27 × 10
24 × 3600 s s
La aceleración de la gravedad en el
Ecuador: g = 9,780490 m/s2
ω2R
Porcentaje = × 100 = 0,34%
g

CURVAS EN LAS PISTAS.


Para un cuerpo como un vehículo o un vagón de tren
que se mueven describiendo una trayectoria curva de
radio r, sobre el vehículo debe actuar una fuerza
centrípeta para evitar que continúe moviéndose en Solución.
línea recta y se salga de la pista; esta es la fuerza para
hacer que el vehículo gire por la pista curva.
La fuerza centrípeta necesaria la da el roce de las
llantas o las pestañas de las ruedas del tren.

La figura muestra el D.C.L.

32
Dinámica de una partícula Hugo Medina Guzmán

Ejemplo 52. ¿Cuál es la velocidad a que puede ir un


automóvil por una curva con peralte, de radio R, sin
derrapar, el peralte es de θ grados?
Solución.

Aplicando la segunda ley de Newton:


∑F z = maz ⇒ N − mg = 0
v2
v2
∑ F⊥ = 0 ac =
∑F r = mar ⇒ μN = m
R
R
v2
De las ecuaciones obtenemos ∑ F// = mac cos θ ⇒ mgsenθ = m R
v2
N = mg y μmg = m ⇒ v = gR tan θ
R
Finalmente v = μgR Curvas peraltadas con rozamiento
Del D.C.L. también obtenemos: Este es un caso bastante más complejo de analizar.
μN
tan θ = =μ Ejemplo 53. ¿Cuál es la velocidad a la que puede ir
N un automóvil por una curva con peralte, de radio R,
Esto quiere decir que si el motociclista al realizar una para que no se deslice hacia el exterior, el coeficiente
curva no se reclina y el piso no es lo suficientemente de rozamiento entre las ruedas y el suelo vale μ., el
áspero (fricción), éste caerá. peralte es de θ grados?
Solución.
Curvas peraltadas sin rozamiento

Para no tener que confiar en el roce o reducir el


desgaste de los rieles y pestañas, la carretera o la vía
pueden inclinarse, como en la figura. En este caso la
componente de la normal dirigida hacia el centro de
curvatura proporciona la fuerza necesaria para
mantener al móvil en la pista. A la inclinación de la
pista o vía se le llama ángulo de peralte, θ . v2
F f = μN , a c =
R
∑F // = ma c cos θ
v2
⇒ mgsenθ + μN = m cosθ
R
v2
∑F ⊥ = 0 ⇒ N − mg cosθ = m
R
senθ

v2
⇒ N = mg cosθ + m senθ
R
⎛ v2 ⎞ v2
mgsenθ + μ ⎜⎜ mg cos θ + m senθ ⎟⎟ = m cos θ
⎝ R ⎠ R
En estos casos se toma la proyección de la normal
2
sobre la horizontal como causante de la fuerza v v2
mgsenθ + μmg cos θ + μm senθ = m cos θ
centrípeta. Este caso se tiene, que: R R
v2 v2
m mg (senθ + μ cos θ ) = m (cos θ − μsenθ )
R v2 R
tan θ = =
mg Rg
v = gR
(senθ + μcosθ )
Siendo θ , la inclinación de la carretera. (cosθ − μsenθ )

33
Dinámica de una partícula Hugo Medina Guzmán

⇒ v = gR
(tan θ + μ ) Para que no se vaya
conoce como MARCOS DE REFERENCIA
INERCIALES.
(1 − μ tan θ ) En los problemas trabajados hasta esta parte el primer
paso era dibujar un sistema de coordenadas. Elegimos
Ejemplo 54. ¿Cuál es la velocidad a laque puede ir un sistema fijo a tierra, pero no pusimos atención al
un automóvil por una curva con peralte, de radio R, hecho que la tierra no es un marco inercial debido a
para que no se deslice hacia el interior, el coeficiente que la tierra al viajar en su orbita casi circular
de rozamiento entre las ruedas y el suelo vale μ., el alrededor del sol experimenta una aceleración
peralte es de θ grados? centrípeta hacia el centro de la tierra. Sin embargo,
Solución. estas aceleraciones son pequeñas comparadas con la
aceleración de la gravedad y a menudo se pueden
despreciar. En la mayoría de los casos se supondrá
que la tierra es un marco inercial.
Ahora veremos cómo cambian los resultados cuando
se trabaja en un MARCO DE REFERENCIA NO
INERCIAL, que es el nombre que se da a un marco
de referencia acelerado.

MARCO CON MOVIMIENTO DE


v2
F f = μN , a c = TRASLACION NO UNIFORME.
R Consideremos los sistemas S y S’ tal corno se
muestra en la Figura siguiente. El sistema S es
∑ F// = mac cos θ ⇒ inercial y el sistema S’ se mueve con respecto a S con

v2 aceleración constante A = Aiˆ , tal que
mgsenθ − μN = m cosθ
R 1 2
v2 D= At .
∑ F⊥ = 0 ⇒ N − mg cosθ = m R senθ 2

v2
⇒ N = mg cos θ + m senθ
R
⎛ v2 ⎞ v2
mgsenθ − μ ⎜⎜ mg cos θ + m senθ ⎟⎟ = m cos θ
⎝ R ⎠ R
2 2
v v De la figura obtenemos que la posición de la partícula
mgsenθ − μmg cos θ − μm senθ = m cos θ P es:
R R
v2 1 2
mg (senθ − μ cosθ ) = m (cosθ + μsenθ ) x = x'+ At , y = y ' , z = z '
R 2
→ → 1
v = gR
(senθ − μcosθ ) r = r '+ At 2 iˆ
2
(cosθ + μsenθ ) Derivando con respecto al tiempo encontramos

⇒ v= gR
(tan θ − μ ) Para que no se caiga
v x = v' x ' + At , v y = v' y , v z = v' z '
(1 + μ tan θ ) → →
v = v'+ Atiˆ
Derivando nuevamente encontramos
La velocidad debe de estar entre esos valores para
permanecer en la carretera. a x = a' x + A , a y = a' y ' , a z = a' z '

gR
(tan θ + μ ) ≥ v ≥ gR
(tan θ − μ ) → → →
a = a'+ Aiˆ o a = a'+ A
→ →

(1 − μ tan θ ) (1 + μ tan θ ) Si la partícula P tiene una masa m y aplicarnos la


segunda ley de Newton del movimiento en el sistema
MOVIMIENTO EN MARCOS DE inercial S obtenemos
REFERENCIA NO INERCIALES → →
Hasta este momento nuestro estudio de mecánica F = ma
clásica lo hemos realizado en sistemas de referencia Donde P es la suma de todas las fuerzas de
que están en reposo o con movimiento con velocidad interacción que actúan sobre las partículas.
constante con respecto a un sistema considerado en Para relacionar con el sistema no inercial S’
reposo. A este conjunto de marcos de referencia se le

34
Dinámica de una partícula Hugo Medina Guzmán


⎛→ ⎞ → → →
F = m⎜ a '+ Aiˆ ⎟ o m a' = F − m A
⎝ ⎠
Aquí vemos que para que el observador según S’
pueda aplicar la segunda ley de Newton debemos

introducir una fuerza extra FA a la llamaremos El observador ve que el resorte se estira Δl . La
fuerza es
fuerza de arrastre y debemos incluirla en los
diagramas de fuerzas: F = kΔl
→ → Aplicando la segunda ley de Newton:
FA = − m A ∑F x = ma x ⇒ kΔl = mA
→ → →
m a ' = F + FA mA
⇒ Δl =
De este modo, en el sistema S’: F
→ Observador en el vagón:
Donde F ' es la suma de las fuerzas reales más la de La figura a continuación muestra el D.C.L. de la masa
arrastre m que no se mueve para el observador en el vagón.
→ → → Como es sistema no inercial tenemos que aplicar la
F ' = F + FA fuerza ficticia − mA .

Recalquemos el carácter ficticio de FA . Para aplicar
una fuerza real sobre un cuerpo debemos ponerlo en
interacción con otro, de manera que, según la tercera

ley de Newton, si A ejerce una fuerza sobre B, FAB , Aplicando la segunda ley de Newton

a su vez B ejercerá una fuerza sobre A, FBA , tal que
∑F x' = 0 ⇒ − mA = kΔl
mA

FAB = − FBA .

⇒ Δl =
F
Ahora, ¿es la reacción de la fuerza de arrastre?, ¿cuál
es el otro cuerpo que está ejerciendo la fuerza ?. No Ejemplo 56. Analizar el caso de masa m colgada
existe tal cuerpo, la fuerza no tiene reacción, es una mediante un hilo del techo de un vagón, que se
fuerza ficticia que agrega un observador ubicado en mueve con una aceleración A.
un sistema acelerado (respecto a uno inercial) para a) Desde el punto de vista de un observador en tierra
justificar los fenómenos que observa. (S).
b) para un observador dentro del vagón (S’).
Ejemplo 55. La fuerza para estirar o comprimir un
resorte es proporcional a su deformación lineal,
F = − kΔl , donde k es la constante del resorte y el
signo menos significa que la fuerza es en oposición a
la deformación. Si sobre una mesa sin fricción que se
encuentra en un vagón se coloca una masa. m sujeta
a un resorte de constante k y largo l , como se Solución.
muestra en la figura. El tren arranca con una a) Para un observador en S:
aceleración A que se mantiene constante en la El D.C.L. de la masa m
dirección x. Calcular la deformación del resorte desde
el punto de vista del observador en tierra y desde el
punto de vista del observador en el vagón.

Aplicando la segunda ley de Newton:

Solución.
∑F x = ma x ⇒ Tsenθ = mA (1)
Observador en tierra: ∑F y = 0 ⇒ T cos θ − mg = 0
La figura muestra el D. C. L. de la masa m. ⇒ T cos θ = mg (2)
Dividiendo (1) : (2)

35
Dinámica de una partícula Hugo Medina Guzmán

A g 2
tan θ = tan θ = = ⇒ θ = 33,7°
g 3 3
g
b) Para un observador en S’ 2
El D.C.L..de la masa m
Ejemplo 58. Resolver el caso del peso del hombre
en un ascensor cuando asciende con una aceleración
constante A, desde el punto de vista del hombre en el
ascensor.
Solución.

Aplicando la segunda ley de Newton


∑F x' = 0 ⇒ Tsenθ − mA = 0
⇒ Tsenθ = mA = 0 (1)
∑ Fy ' = 0 ⇒ T cosθ − mg = 0
⇒ T cos θ = mg (2)
Dividiendo (1) : (2) obtenemos:
A
tan θ = Aplicamos la segunda ley de Newton,
g
Ejemplo 57. Desde el techo de un carrito de juguete
∑F y' = ma y ' ⇒ N − mg − ma = 0
cuelga una masa m unida al cielorraso mediante una ⇒ N = m( g + a )
cuerda ideal. El carrito se encuentra en el piso de un El peso del hombre será la reacción N
ascensor que sube con aceleración g/2. A su vez el En caso de subir con aceleración a:
carrito tiene una aceleración horizontal de magnitud g
N = m( g + a )
respecto al ascensor. Encuentre el ángulo que forma
la cuerda con la vertical, resuelva para un observador En caso de bajar con aceleración a:
situado dentro del ascensor. N = m( g − a )

Ejemplo 59. El pasajero de un tren deja caer una


piedra en diversos estados de movimiento del tren.
Hallar la trayectoria de dicha piedra que ve el
pasajero y la trayectoria vista por un observador en
tierra.
a) El tren acelera con aceleración A constante.
b) El tren frena con aceleración A constante.
Solución.
Solución.
Para un observador en el ascensor.
El tiempo en que la piedra esta en movimiento, es el
El D.C.L..de la masa m
mismo para todo sistema puesto que el movimiento
vertical es independiente del horizontal.
1 2
y = y' = h − gt , para y = 0 la piedra lega al
2
piso:
1 2 2h
h− gt = 0 ⇒ t =
Aplicando la segunda ley de Newton 2 g
∑F x' = ma x '
a) Cuando el tren va con aceleración A, deja caer una
⇒ Tsenθ = mg (1)
piedra.
⎛ g⎞ Considerando que en el momento que suelta la piedra
∑F = 0 ⇒ T cos θ − m⎜ g + ⎟ = 0
y'
⎝ 2⎠ el tren tiene una velocidad v 0 .
3
⇒ T cos θ = m g (2)
2
Dividiendo (1) / (2)

36
Dinámica de una partícula Hugo Medina Guzmán

1 2
xtren = v0 t − At
2
La piedra cae a una distancia
1 2
Δx = xtren − x piedra = At , detrás del punto de
2
plomada.
Observador en tierra Observador en el tren
Las ecuaciones del movimiento en el sistema S. La ecuación del movimiento en el sistema S’
Movimiento de la piedra Movimiento de la piedra
x piedra = v0 t
Movimiento del tren
1 2
xtren = v0 t + At
2
La piedra cae a una distancia
1 2 1 2
Δx = xtren − x piedra = At , detrás del punto de x piedra = At
2 2
plomada. 1 2
La piedra cae a una distancia Δx = At , detrás
Observador en el tren 2
La ecuación del movimiento en el sistema S’ del punto de plomada.
Movimiento de la piedra El gráfico siguiente muestra el moviendo visto por un
observador en el sistema S y en el sistema S’.

1 2
x piedra = − At
2
1 2
La piedra cae a una distancia Δx = At , detrás MARCO DE ROTACIÓN
2
Veamos el caso de un marco de referencia que está
del punto de plomada.
rotando con velocidad angular ω con respecto a otro
El gráfico siguiente muestra el moviendo visto por un
observador en el sistema S y en el sistema S’. marco de referencia. Supongamos que tenemos un
objeto moviéndose alrededor de un punto arbitrario;
este es un caso específico, sin embargo tiene todos los
efectos en él.
La posición de la partícula con respecto a un sistema

inercial está determinada por un vector r .
Consideremos un nuevo sistema de coordenadas tal
que siga al objeto, el nuevo origen está determinado
b) Cuando el tren desacelera con aceleración A, deja → →
caer una piedra. por R contenido en r tal que la posición de la

Considerando que en el momento que suelta la piedra
partícula en este nuevo sistema está ciada por r ' .
el tren tiene una velocidad v 0 .

Observador en tierra
Las ecuaciones del movimiento en el sistema S.
Movimiento de la piedra De la figura tenemos.
→ → →
x piedra = v0 t r = R + r ' = Rrˆ + r ' rˆ = (R + r ')rˆ
Movimiento del tren Derivando:

37
Dinámica de una partícula Hugo Medina Guzmán

→ Si la partícula tiene una masa m y aplicamos la


dr d d ( R + r ') drˆ
= (R + r ')rˆ = rˆ + (R + r ') segunda ley de Newton en el sistema inercial
dt dt dt dt → →
F = ma
drˆ
Como = ωtˆ →
dt donde F es la suma de todas las fuerzas de
→ interacción que actúan sobre la partícula.
d r dR dr ' Para relacionar con el sistema inercia!
= rˆ + rˆ + (R + r ')ωtˆ
dt dt dt →
⎛→ →⎞ → → →


F = m⎜ a '+ A ⎟ o m a' = F − m A
dr → ⎝ ⎠
= v es la velocidad de la partícula vista en el Para que el observador pueda aplicar la segunda ley
dt de Newton debemos introducir aquí también una
→ →
d r' → fuerza extra FA y debemos incluirla en los
sistema inercial y = v' es la velocidad de la
dt diagramas de fuerzas
partícula vista en el sistema no inercial. → →
Tal que FA = − m A
→ dR → → → →
v= rˆ + v'+ (R + r ')ωtˆ FA = FAr rˆ + FAt tˆ
dt d 2 ( R + r ') d (R + r ') drˆ d (R + r ') dω dtˆ
Para encontrar la aceleración es necesario derivar rˆ + + ωtˆ + (R + r ') tˆ + (R + r ')ω
dt 2 dt dt dt dt dt
nuevamente: → ⎡ d 2 ( R + r ') ⎤

FAr = −m ⎢ − (R + r ')ω 2 ⎥ rˆ
d ⎡ d ( R + r ')
2 2
d r d ⎤ 2
= 2 (R + r ')rˆ = ⎢ r + (R + r ')ωtˆ⎥ ⎣ dt ⎦
dt 2
y FAt = 2m ⎡ d (R + r ') ω + (R + r ')α ⎤tˆ
dt dt ⎣ dt ⎦ →

drˆ dtˆ ⎢⎣ dt ⎥⎦
Como = ωtˆ y = −ωrˆ
dt dt De este modo, en el sistema no inercial
→ → → → →
d 2 r d 2 ( R + r ') d ( R + r ') ˆ F ' = m a ' = F + FA
2
= 2
rˆ + ωt
dt dt dt →
d ( R + r ') ˆ dω Recalquemos el carácter ficticio de FA Con el objeto
+ ωt + (R + r ') tˆ − (R + r ')ω 2 rˆ
dt dt de clarificar esta idea veamos dos casos especiales:

d 2 (R + r ') d ( R + r ') ˆ a) El origen O’ rota con velocidad angular constante


= 2
rˆ + 2 ωt ω a una distancia constante b, tal
dt dt
R + r ' = b , R y r’ son constantes.
+ (R + r ')αtˆ − (R + r ')ω 2 rˆ
d ( R + r ') d 2 ( R + r ')
⎡ d 2 ( R + r ') ⎤ ⎡ d ( R + r ') ⎤ =0 y =0
=⎢ − (R + r ')ω 2 ⎥ rˆ + 2⎢ ω + (R + r ')α ⎥tˆ
⎣ dt
2
⎦ ⎣ dt ⎦ dt dt 2

d2 r


ω = constante, α = =0
donde a = es la aceleración de la partícula dt
dt 2 Sólo nos queda
vista en el sistema inercial y →
→ FAr = m(R + r ')ω 2 rˆ = mbω 2 rˆ

d 2 r' Que es la fuerza ficticia del centro hacia afuera y se le
a' = es la aceleración de la partícula vista en
dt 2 da el nombre de FUERZA CENTRÍFUGA, debemos
e1 sistema no inercial. insistir que solo aparece en el marco no inercial.
Llamando a
b) El origen O’ rota con velocidad angular constante
→ ⎡ d 2 ( R + r ') ⎤ ω y también se está alejando del origen fijo en O
Ar = ⎢ − (R + r ')ω 2 ⎥ rˆ
d ( R + r ')
2
⎣ dt ⎦
con una velocidad constante V = .

⎡ d ( R + r ') ⎤
y At = 2
⎢ ω + (R + r ')α ⎥tˆ dt
⎣ dt ⎦ dω

Con esto, α= =0
Tenemos: A = Ar rˆ + At tˆ
dt
y nos queda
→ → → →
Tal que: a = a '+ A FAr = m(R + r ')ω 2 rˆ

38
Dinámica de una partícula Hugo Medina Guzmán


y FAt = −2mVωtˆ ∑ F' z' = ma' z ' , ∑ F' r' = ma' r ' ,
Esta última fuerza ficticia, cuya dirección es ∑ F' t' = ma' t '
transversal, se conoce como FUERZA DE
CORIOLIS. Como a ' z ' = 0 , a ' r ' = 0 , ∑ F' t' = ma' t '
Tenemos
Ejemplo 60. Un cuerpo de masa de masa m unido a N − mg = 0 , − T + mω 2 (l + Δl ) = 0 , Ft = 0
un resorte de constante k y longitud l que gira con
Como T = kΔl
ve1ocidad angular ω constante en un plano
horizontal sin fricción. Se quiere calcular el − kΔl + mω 2 (l + Δl ) = 0
estiramiento Δl del resorte. mω 2 l
y Δl =
k − mω 2
Visto por un observador no inercial colocado sobre la
misma masa Este caso es idéntico al caso anterior.

Ejemplo 61. Se tiene una plataforma circular de


radio R a la cual se le ha pintado un radio y gira con
velocidad angular constante ω . Un hombre camina
Solución. de afuera hacia adentro de la plataforma siguiendo la
Visto por el observador inercial. línea con una velocidad de módulo constante v .
La figura muestra el D.C. L. de la masa
¿Cuál es la fuerza que la plataforma ejerce sobre el
hombre, en función de su posición?

Aplicando la segunda ley de Newton, el resorte estira


Δl , luego su longitud es (l + Δl )
∑F z = ma z , ∑F
r = ma r , ∑F t = mat Solución.
La figura muestra el D.C.L. del hombre
Como: a z = 0 , a r = −ω
2
(l + Δl ) , at =0
Tenemos
N − mg = 0 , − T = −mω 2 (l + Δl ) , Ft = 0
De aquí obtenemos:
N = mg y T = mω 2 (l + Δl )
Como T = kΔl
kΔl = mω 2 (l + Δl )
mω 2 l Aplicando la segunda ley de Newton:
y Δl =
k − mω 2 ∑F r = ma r ⇒ − Rr = ma r − mrω 2
∑F t = mat ⇒ Rt = m(− 2vω + αr )
Visto por un observador no inercial colocado en el
centro de rotación y girando con la misma velocidad Como: a r = 0 y α = 0:
angular.
Rr = mrω y 2
Rt = −2mvω
Rt es debido a la aceleración de coriolis.
Rr es el sentido indicado en la figura y Rt en el
sentido contrario.

Aplicando la segunda ley de Newton:

39
Dinámica de una partícula Hugo Medina Guzmán

PREGUNTAS Y PROBLEMAS

1. Sobre una partícula de masa m que parte del Respuesta. 20N opuesta a la velocidad.
reposo en origen de coordenadas. actúa una fuerza → →
5. ¿Qué fuerza en adición a F1 = 4iˆ N y F2 = 2 ˆj
( )

F = 2iˆ + 3 ˆj Después de l0s la posición de la N debe aplicarse al cuerpo en la figura, tal que:
partícula viene dada por las coordenadas (3m; 4,5 m). a) no acelere?
¿Cuál es su masa? b) tenga una aceleración − 4iˆ m/s2
Respuesta. m = 33,3 kg.

2 Hallar las fuerzas que actúan sobre cada una de las


seis barras rígidas de peso despreciable. Si están
unidas mediante pivotes lisos y cada una de las barras
cortas tiene una longitud l .

( )

Respuesta. a) F = − 4iˆ − 2 ˆj N, b)

( )

F = − 16iˆ − 2 ˆj N

6. ¿Cuál es la mínima aceleración con la que puede


deslizarse hacia abajo un hombre de 75 kg por una
cuerda que solo soporta una tensión de 490N, ¿Cuál
Respuesta. AD = DB = mg ; CB = CA = mg/2, BC = será la velocidad de la persona después de deslizarse
2mg; CD = 0. la distancia de 20m?
CD se puede retirar y no pasa nada. Respuesta. a = 3,27 m/s2 ; v = 11,4 m/s

3. Dos cubos de masa m están unidos mediante una 7. El libro de Física I, está apoyado en el extremo
cuerda y uno de ellos está sujeto al techo mediante superior de un resorte vertical, que a su vez esta
otra cuerda igual. ‘parado’ sobre una mesa. Para cada componente del
a) Si en el cubo inferior se hace presión suavemente sistema libro-resorte-mesa-tierra:
hacia abajo. ¿Cuál de las cuerdas se romperá antes? a) dibujar el diagrama de cuerpo libre,
¿porqué? b) identificar todos los pares de fuerzas de acción y
b) Si la masa interior se golpea hacia abajo con un reacción.
martillo, se rompe la cuerda de abajo ¿porqué?
8. De acuerdo con la leyenda, un caballo aprendió las
leyes de Newton. Cuando se le dijo que tirara una
carreta, se negó argumentando que si él tiraba la
carreta hacia delante, de acuerdo con la tercera ley de
Newton habría una fuerza igual hacia atrás. De esta
manera, las fuerzas estarían balanceadas y de acuerdo
con la segunda ley de Newton, la carreta no
aceleraría. ¿Cómo podría usted razonar con este
misterioso caballo?
Respuesta. a) La cuerda superior debido a que la
9. Dos alumnos de forestal ubicados en los bordes
tensión es mayor.
opuestos de un camino recto tiran a un carro por el
b) La tuerza de reacción inercial de la masa superior
camino, con fuerzas de 160 N y 200 N, que forman
aumenta la resistencia frente a una aceleración rápida.
un ángulo de 30º y 60º respectivamente, con la
dirección del camino.
4. Una caja de 40 kg que está resbalando en el piso
Calcular la magnitud de la fuerza resultante y la
disminuye su velocidad de 5 m/s a 2 m/s. Asumiendo
dirección en la que se moverá el carro.
que la fuerza sobre la caja es constante, encontrar su
Respuesta. 256,1N, -21,3º
magnitud y dirección relativa a la velocidad de la
caja.

40
Dinámica de una partícula Hugo Medina Guzmán

10. Una masa de 5kg cuelga de una cuerda de 1m de


longitud que se encuentra sujeta a un techo. Calcular
la fuerza horizontal que aplicada a la masa la desvíe
30 cm de la vertical y la mantenga en esa posición.
Respuesta. 15,7 N.

( )

11. Tres fuerzas F1 = − 2iˆ + 2 ˆj N,
Respuesta: a) a A = a B = 0,738 m/s2, b) 5,68 N
( ) = (− 45iˆ ) N que actúan
→ →
F2 = 5iˆ − 3 ˆj N y F3
sobre un objeto le producen una aceleración de valor 16. Un bloque A de 100 kg está unido a un contrapeo
3 m/s2. 8 de 25 kg mediante un cable dispuesto como muestra
a) ¿Cuál es la dirección de la aceleración? la figura. Si el sistema se abandona en reposo,
b) ¿Cuál es la masa del objeto? determinar:
c) Si el objeto esta inicialmente en reposo, calcular su a) la tensión en el cable.
velocidad después de 10s? b) la velocidad de B transcurridos 3 s,
c) la velocidad de A cuando ha recorrido 1,2 m.
12. Una mano ejerce una fuerza horizontal de 5 N
para mover hacia la derecha a dos bloques en
contacto entre sí uno al lado del otro, sobre una
superficie horizontal sin roce. El bloque de la
izquierda tiene una masa de 2 kg y el de la derecha de
1 kg.
a) Dibujar el diagrama de cuerpo libre para cada
bloque.
b) Calcular la aceleración del sistema,
c) Calcular la aceleración y fuerza sobre el bloque de
1 kg,
d) Calcular la fuerza neta actuando sobre cada Respuesta. a) 302 N, b) 6,79 ĵ m/s, c) -1,346 ĵ m/s
cuerpo.
Respuesta. b) 5/3 m/s2, c) 5/3 m/s2, 5N, d) 5 N. 17. Determinar la aceleración de cada uno de los
bloques de la figura, ¿Que bloque llega primero al
13. Una fuerza F aplicada a un objeto de masa m1 suelo?
produce una aceleración de 3 m/s2. La misma fuerza mA=5kg, mB = 15 kg, mC = 10kg
aplicada a una masa m2 produce una aceleración 1
m/s2.
a) ¿Cuál es el valor de la proporción m1/m2?
b) Si se combinan m1 y m2, encuentre su aceleración
bajo la acción de F.
Respuesta. a) 1/3, b) 0,75 m/s2.

14. Dos bloques de masas M y 3M ubicado a la


derecha de M, que están sobre una mesa horizontal
lisa se unen entre sí con una varilla de alambre
horizontal, de masa despreciable. Una fuerza
horizontal de magnitud 2Mg se aplica sobre M hacia
la izquierda.
a) Hacer los diagrama de cuerpo libre.
b) Calcular la aceleración del sistema. →
c) Calcular la tensión del alambre. Respuesta. a A == 4,04 ˆj m/s2,
Respuesta. b) 5 m/s2, c) 15Mg N. → →
a B == −0,577 ˆj m/s2, a C == −2,89 ˆj m/s2
15. Dos paquetes se colocan sobre un plano inclinado C llega primero.
como muestra la figura. El coeficiente de rozamiento
entre el plano y el paquete A es 0,25 y entre el plano
y el paquete B es 0,15. Sabiendo que los paquetes
18. En la figura μ = 0,45 , 5 kg . m A = 5 kg, m B =
están en contacto cuando se dejan libres, determinar: 20 kg mC = 15 Kg. determinar la aceleración de cada
a) la aceleración de cada paquete, bloque.
b) la fuerza ejercida por el paquete A sobre el B.
c) Resolver el problema invirtiendo las posiciones de
los paquetes.

41
Dinámica de una partícula Hugo Medina Guzmán

b) 28 N y 37 N

21. Pepe anda esquiando, cuando en algún momento


sube 5 m deslizándose por la pendiente de un cerrito
nevado en sus esquíes, saliendo desde la cima
ubicada a 3 m de altura respecto a la horizontal, con
una rapidez de 10 m/s. El coeficiente de roce entre la
nieve y los esquíes es 0,1.
a) Calcular la rapidez con la cual el esquiador
comienza a subir la pendiente.
b) Determine la distancia horizontal que vuela Pepe
→ →
cuando sale de la punta del cerro.
Respuesta. a A = 4,91 ˆj m/s2, a B = −2,45 ˆj m/s2, Respuesta. a) 13 m/s, b) 16,6 m.

aC = 0 22. El bloque de masa m de la figura parte del
reposo, deslizándose desde la parte superior del plano
inclinado 30º con la horizontal. El coeficiente de roce
19. Determinar la aceleración del cilindro B de la cinético es 0,3.
figura, si a) T = 1500 N, b) T = 4000 N. a) Calcular la aceleración del bloque mientras se
mA=250 kg, mB = 100 kg, mueve sobre el plano.
b) Calcular la longitud del plano si el bloque sale con
una rapidez de 5 m/s.
c) Si el bloque cae al suelo a una distancia horizontal
de 3 m desde el borde del plano, determine el tiempo
total del movimiento.

Respuesta. a) 2,4 m/s2, b) 5,2 m, c) 2,8 s.

23. En el sistema de la figura, se aplica una fuerza F


Respuesta. a) -3,11 ĵ N b) -9,81 ĵ N sobre m. El coeficiente de roce es μ entre cada cuerpo
y los planos. Deducir la expresión de la magnitud de
20. Se tiene un sistema formado por tres bloques y F para que el sistema se mueva:
una polea sin fricción. El bloque A tiene una masa de a) con rapidez constante,
6,0 kilogramos y está en una superficie áspera (μ = b) con aceleración a constante.
0,40). El bloque C tiene una masa de 4,0 kilogramos.
Una fuerza externa P = 80 N, se aplica verticalmente
al bloque A, la que mantiene el sistema en equilibrio
estático según como se muestra.

Respuesta. b)
Mg (μ cos α + senα ) + μmg + a(m + M ) .

24. En el sistema de la figura, la fuerza F paralela al


plano inclinado empuja al bloque de masa m
haciéndolo subir una distancia D sobre el plano, de
a) ¿Cuál es la masa del bloque B? ¿Cuál es la fuerza coeficiente de roce μ. Calcular en función de m, F, g,
de fricción sobre el bloque A? D, μ y α, la aceleración del bloque.
b) se quita la fuerza externa de 8,0 N. Las masas de
los bloques B y C se ajustan, de modo el sistema siga
en reposo tal como se muestra, pero están justo por
iniciar el movimiento. La masa del bloque A no se
cambia. Las tensiones en las dos cuerdas verticales
son:
Respuesta.
a) 3,1 kg 25.2 N

42
Dinámica de una partícula Hugo Medina Guzmán

25. Una fuerza F se aplica a un pequeño bloque de


masa m para hacerlo moverse a lo largo de la parte 30. Sobre el planeta X un objeto pesa 12 N. En el
superior de un bloque de masa M y largo L. El planeta Y, donde la magnitud de la aceleración de
coeficiente de roce es μ entre los bloques. El bloque caída libre es 1,6g, el objeto pesa 27 N. ¿Cuál es la
M desliza sin roce en la superficie horizontal. Los masa del objeto y cuál es la aceleración de caída libre
bloques parten del reposo con el pequeño en un en el planeta X?
extremo del grande, como se ve en la figura. Respuesta. 1,7 kg, 7m/s2.
a) Calcular la aceleración de cada bloque relativa a la
superficie horizontal. 31. Dos bloques de 1 y 2 kg, ubicados sobre planos
b) Calcular el tiempo que el bloque m demora en lisos inclinados en 30º, se conectan por una cuerda
llegar al otro extremo de M, en función de L y las ligera que pasa por una polea sin roce, como se
aceleraciones. muestra en la figura. Calcular:
a) la aceleración de cada bloque,
b) la tensión en la cuerda.
c) si la aceleración cuando los planos son rugosos
fuera ½ de la calculada en ese problema, calcular: el
coeficiente de roce y la tensión en la cuerda.
Respuesta. a) (F- µmg)/m, µmg/(m+M),
b) [2L/(a1-a2)]1/2.

26. Un bloque de masa M se ubica sobre un pequeño


plano inclinado un ángulo α sin roce, que tiene su
extremo inferior fijo a un eje vertical que puede girar.
En algún momento el eje gira con el plano con
rapidez constante.
Demostrar que si la masa asciende desde la base del
plano, su rapidez cuando ha subido una distancia L es 32. Un trineo de 50 kg de masa se empuja a lo largo
v = gLsenα . de una superficie plana cubierta de nieve. El
coeficiente de rozamiento estático es 0,3, y el
coeficiente de rozamiento cinético es 0,1.
27. Una fuerza dependiente del tiempo, a) ¿Cuál es el peso del trineo?
( )

b) ¿Qué fuerza se requiere para que el trineo
F = 8iˆ − 4tˆj N (donde t está en segundos), se
comience a moverse?
aplica a un objeto de 2 kg inicialmente en reposo. c) ¿Qué fuerza se requiere para que el trineo se
a) ¿En qué tiempo el objeto se moverá con una mueva con velocidad constante?
velocidad de 15 m/s? d) Una vez en movimiento, ¿qué fuerza total debe
b) ¿A qué distancia está de su posición inicial cuando aplicársele al trineo para acelerarlo a 3 m/s2?
su velocidad es 15 m/s?
c) ¿Cuál es la posición del objeto en este tiempo? 33. La masa m1 sobre una mesa horizontal sin fricción
Respuesta. a) 3s, b) 20,1m, c) (18iˆ − 9 ˆj ) m se conecta a la masa m2 por medio de una polea móvil
y una polea fija sin masas. Si a1 y a2 son magnitudes
28. Una araña de 2 x 10-4 kg está suspendida de una de las aceleraciones de m1 y m2, respectivamente.
hebra delgada de telaraña. La tensión máxima que Determinar:
soporta la hebra antes de romperse es 2,1 x 10-3 N. a) una relación entre estas aceleraciones.
¿Cuál es la aceleración máxima con la cual la araña b) las tensiones en las cuerdas, y
puede subir por la hebra con toda seguridad? c) las aceleraciones a1 y a2 en función de m1, m2 y g.
Respuesta. 0,5m/s2.

29. Los instrumentos de un globo meteorológico


tienen una masa de 1 kg.
a) El globo se suelta y ejerce una fuerza hacia arriba
de 5 N sobre los instrumentos. ¿Cuál es la aceleración
del globo y de los instrumentos?
b) Después de que el globo ha acelerado durante 10 34. Calcular la fuerza F que debe aplicarse sobre un
segundos, los instrumentos se sueltan. ¿Cuál es bloque A de 20 kg para evitar que el bloque B de 2 kg
velocidad de los instrumentos en el momento en que caiga. El coeficiente de fricción estático entre los
se sueltan? bloques A y B es 0,5, y la superficie horizontal no
c) ¿cuál es la fuerza neta que actúa sobre los presenta fricción.
instrumentos después de que se sueltan?
d) ¿En qué momento la dirección de su velocidad
comienza a ser hacia abajo?

43
Dinámica de una partícula Hugo Medina Guzmán

b) Calcular la tensión de la cuerda.


c) Si el sistema da una vuelta en 30 s, determinar
El ángulo que forma la cuerda con la vertical.

35. Una bola de masa m se suelta sin velocidad inicial


desde un punto A y oscila en un plano vertical al
extremo de una cuerda de longitud L. Determinar:
a) la componente tangencial de la aceleración en el
punto B. Respuesta. a) v = g (l + Lsenα ) tan α ,
b) la velocidad en el punto B. b) mg/cos α.
c) la tensión en la cuerda cuando la bola para por el
punto mas bajo. 39. Una bola pequeña da vueltas con una rapidez y
d) el valor de si la tensión en la cuerda es 2 mg recorriendo una circunferencia horizontal en el
cuando la bola pasa por el punto C interior de un cono recto de base circular. Expresar la
rapidez y en función de la altura y de la trayectoria
sobre el vértice del cono.

Respuesta. a) gsenθ , b) 2 gL(cos θ − cos θ 0 ) ,


c) mg (3 − 2 cos θ 0 ) , d) 60°.

Respuesta. v = gy
36. Tres automóviles circulan a la velocidad de 80
km/h por la carretera representada en la figura.
Sabiendo que el coeficiente de rozamiento entre las 40. ¿Cuál es el mínimo radio que un motociclista con
llantas y la carretera es 0,60, determinar la velocidad de 21 m/s puede hacer en una pista que
desaceleración tangencial de cada automóvil sí sus tiene un coeficiente de fricción con las llantas igual a
respectivos frenos sen repentinamente accionados y 0,3? ¿Cuál es el ángulo que hará la motocicleta con la
las ruedas deslizan. horizontal?
Respuesta: 147 m; 73° 20’

41. Un estudiante hace girar un balde que contiene 2


kg de agua en una circunferencia vertical de l,2m de
radio, considerar
a) ¿Cuál es la máxima velocidad para que el agua
Respuesta. a A =3,91 m/s2, a B = 7,86 m/s2, aC = permanezca en el balde?
b) ¿Cuál es la fuerza ejercida por el balde sobre el
5.89 m/s2.
agua en el punto inferior de la circunferencia?
c) ¿a la altura de los hombros?
37. ¿Con qué ángulo debe peraltarse una carretera en
d) Si el balde pesa 10k, hallar cada una de las fuerzas
una curva de 50 m de radio, para que un vehículo
que actúan sobre el balde en el punto inferior de la
pueda tomar la curva a 72 km/h, con un coeficiente
circunferencia.
de rozamiento 0,30?
Respuesta: 22,5° ≤ θ ≤ 55,9° r
Respuesta. a) 2π , b) 2mg , c) 2mg
g
38. En el sistema de la figura, el brazo del péndulo es d) 10 N debido a la tierra, 40 N debido al agua, 100 N
de longitud l y la cuerda de largo L. debido al hombre.
a) Calcular la rapidez tangencial para que el sistema
gire en torno al eje de rotación que pasa por la barra 42. Una mesa giratoria horizontal tiene una
vertical, de modo que la cuerda que sostiene a la aceleración angular de α = 3 rad/s2. En el instante en
masa m forme un ángulo de α º con la vertical. que la velocidad angular vale 2,4 rad/s, una partícula

44
Dinámica de una partícula Hugo Medina Guzmán

de masa 1,8 kg descansa sin deslizar sobre la mesa, c) a = 5 cm/s2, d) φ = 53,1°; e) Ft = 15 x 10-5 N,
con tal que esté situada a una distancia inferior a 50
cm del eje vertical de rotación de la mesa, f) Fr = 20 x l0-5 N, g) F = 25 x 10-5 N.
a) ¿Cuál es el valor de la tuerza de rozamiento?
b) Hallar el coeficiente de rozamiento estático entre el 44. Describir e interpretar las fuerzas que realmente
objeto y la mesa. se apreciarían si nos encontráramos con los ojos
Respuesta: a) Ff = 7,9 N b) μ s = 0,45 vendados y:
a) de pie sobre una plataforma elevada.
b) cayendo libremente en el aire.
43. Se tiene una partícula de masa 5g que se mueve
c) estando sentado en el suelo de una plataforma en
sobre una trayectoria curva y su aceleración en un

rotación, como la de un carrusel a una cierta distancia
momento dado vale a = (3tˆ + 4nˆ ) cm/s2. Hallar: de su centro.
Respuesta. a) Una fuerza de reacción de la
a) la aceleración tangencial, plataforma hacia arriba.
b) la aceleración centrípeta, b) Ninguna fuerza.
c) el módulo de la aceleración total, c) Una fuerza de reacción de la plataforma y una
d) el ángulo φ que la aceleración total forma con la fuerza hacia afuera (radial).
tangente a la curva,
e) la componente tangencial de la fuerza aceleradora, 45. Calcular el ángulo de peralte de una carretera en
f) la componente centrípeta de la fuerza aceleradora, una curva de radio 150 m, para que un camión de 15
g) la fuerza aceleradora total. toneladas pueda girar con una rapidez de 70 km/hr,
Respuesta. a) a t = 3 cm/s2 , b) a t = - 4 cm/s2 ; sobre un pavimento cubierto de escarcha.
Respuesta. 14º

45
TRABAJO Y ENERGÍA Hugo Medina Guzmán

CAPÍTULO 5. TRABAJO Y ENERGÍA

INTRODUCCIÓN Sin embargo, es importante notar que los conceptos


Con lo que hemos visto hasta el momento estamos en de Trabajo y Energía se fundamentan en las leyes de
condiciones de analizar un movimiento en situaciones Newton y por lo tanto no requieren ningún principio
en que la fuerza es constante. Una vez aplicada La nuevo.
segunda ley de Newton, determinamos la aceleración
a = F / m . De aquí podemos determinar la TRABAJO
velocidad y la posición. Pero en el caso en que la El término “trabajo” que se usa en la vida cotidiana es
fuerza no es constante, por ejemplo cuando se jala para definir una actividad de algún tipo que incluye
una masa situada en un extremo de un resorte, el un esfuerzo físico o mental y cuya finalidad sea el
problema se complica. alcance de algún objetivo definido y bien establecido.
En el estudio de la mecánica tiene un significado más
restringido, por ejemplo si subimos cierta altura h con
una masa m decimos que hemos realizado un trabajo
W, si subimos la misma altura h pero con una masa
2m, se habrá realizado un trabajo 2W, igual a que si
se hubiese transportado una masa m una altura 2h, o
si se hubiese transportado dos veces la masa m, la
altura h. Estas observaciones sugieren que el trabajo
es una magnitud física proporcional a la fuerza y a la
distancia, pero que puede sumarse como un escalar.
Cuando una fuerza constante Fx mueve un cuerpo
La figura muestra un cuerpo de masa m sobre una realizando un desplazamiento Δx que tiene la misma
superficie horizontal lisa, conectado a un resorte dirección que la fuerza, se define la cantidad de
helicoidal. Si el resorte se estira o se comprime una trabajo realizado por esta fuerza como:
longitud pequeña desde su posición no deformada o W = Fx Δx
de equilibrio, el resorte ejercerá una fuerza sobre el
cuerpo F = − kx , donde x es el desplazamiento del
cuerpo desde la posición de equilibrio ( x = 0 ) , k es
la constante del resorte, el signo negativo (-) significa
que la fuerza es en sentido opuesto al sentido del
desplazamiento. Esta ley de fuerza se conoce como la
ley de Hooke, de la cual nos ocuparemos en el
Capítulo de Elasticidad

Apliquemos la segunda ley de Newton: Ahora consideremos que sobre la misma masa m
∑ F = ma actúa una fuerza vertical Fy , menor que el peso mg
dv d 2 x del bloque, como tal no dará origen a ningún
Con F = − kx y a = = , movimiento vertical y por lo tanto no estará
dt dt 2 realizando trabajo.
Obtenemos:
d 2x
− kx = m 2
dt
2
d x k
⇒ + x=0
dt 2 m
A pesar de ser una ecuación simple esta última,
todavía no tenernos el conocimiento matemático para
resolverla. Es decir, estamos en condiciones de
Si ahora aplicamos al mismo tiempo las dos fuerzas,
plantear las ecuaciones del movimiento, pero no
la fuerza aplicada es:
sabemos resolverlas.

Veremos aquí que se puede tomar un atajo y resolver F = Fx iˆ + Fy ˆj
de otra forma el problema. En este capitulo se verán
los conceptos de Trabajo y Energía que se pueden Si el desplazamiento del bloque es únicamente en la
aplicar a la dinámica de un sistema mecánico sin dirección x,
recurrir a las leyes de Newton.

1
TRABAJO Y ENERGÍA Hugo Medina Guzmán

→ → →
Δ r = xiˆ dW = F d r cos θ

dW = F ds cos θ
dW = Ft ds
Es el trabajo realizado por la componente tangencial
de la fuerza Ft .
El trabajo de la componente normal Fn es nulo.

El trabajo realizado es el producto escalar de la tuerza Para evaluar el trabajo realizado para ir desde el
por el desplazamiento es: punto P1(x1, y1, z1) a un punto P2(x2, y2, z2) tenemos

ΔW = F ⋅ Δ r = (Fx iˆ + Fy ˆj ) ⋅ Δxiˆ = Fx Δx
→ → que integrar el trabajo diferencial.
P2 P2 → →
WP1P2 = ∫ dW = ∫ F ⋅ d r
O ΔW = FΔx cos θ P1 P1
Donde →

F = F +F 2 2
y
Para esto tenemos que conocer como varía F
x y →

θ es el ángulo formado entre la fuerza aplicada y el F = Fxiˆ + Fy ˆj + Fz kˆ


desplazamiento. →
→ Siendo d r = dxiˆ + dyˆj + dzkˆ
Consideremos el caso general de una fuerza F → →
cualquiera que mueve a una partícula sobre una Tenemos: F ⋅ d r = Fx dx + Fy dy + Fz dz
trayectoria curva como se muestra en la siguiente x2 y2 z2
figura. Luego: WP1 P2 = ∫x1
Fx dx + ∫ Fy dy + ∫ Fz dz
y1 z1
La unidad de trabajo es una unidad derivada de las
unidades de fuerza y de longitud.
[W ] = FL = ML2T −2
En el sistema Internacional la unidad de trabajo es el
Joule (J).
1 Joule = (1 Newton)(1 metro)

Ejemplo 1. Un hombre levanta una masa m con una


fuerza tal que la coloca a una altura h sobre el piso a
Sea P la posición de la partícula en un instante t , la velocidad constante.
posición con respecto al origen de coordenadas O está a) ¿Cuánto trabajo realiza la gravedad?
dada por b) ¿Cuál es la magnitud de la fuerza que ejerce el
→ → hombre?
OP = r Solución.
La partícula en el tiempo Δt describe la trayectoria a)

PP' , si esta es suficientemente pequeña se puede

asimilar como la cuerda PP' , el desplazamiento de la
→ →
partícula en el tiempo Δt es PP' = Δr
Cuando P’ tiende a P (Δt → 0 ) .

La dirección de la cuerda
→ →
PP' es el de la tangente PT
Wgravedad = ∫
y =h →

y =0
h →
(
F ⋅ d r = ∫ − mgkˆ ⋅ dykˆ
0
)
en P, Δ r es d r , la fuerza es constante en dirección h
y sentido.

= − mg ∫ 0
dy = − mgh
El trabajo de la fuerza F para el desplazamiento b) Podríamos hacerlo directamente por la ley de
→ Newton, pero lo haremos con los conceptos de
d r es un trabajo diferencial. trabajo. Como la masa se mueve con velocidad
→ → constante, el trabajo realizado es cero.
dW = F ⋅ d r Whombre + Wgravedad = 0

2
TRABAJO Y ENERGÍA Hugo Medina Guzmán

⇒ Whombre = −Wgravedad = mgh Calcule el trabajo cuando el cuerpo se mueve desde x


= 0 hasta x = 8 m.
También tenemos:
y=h → → h
Whombre = ∫ Fhombre ⋅ d r = ∫ Fkˆ ⋅ dykˆ
y =0 0
h
= F ∫
0
dy = Fh
Luego: Fh = mgh ⇒ F = mg

Y F = mgkˆ Solución.
El trabajo realizado por la fuerza es exactamente
Ejemplo 2. Se arrastra una caja de masa m sobre un igual al área bajo la curva desde x = 0 hasta x = 8.
piso horizontal, el coeficiente de fricción cinético
entre la caja el piso es μ , mediante una fuerza que
W =
1
(5N )(2 − 0)m + (5N )(5 − 2)m + 1 (5N )(8 − 5)m
2 2
forma un ángulo θ con la horizontal, la caja se = (5 + 15 + 7,5) Nm
desplaza un distancia s hacía la derecha, = 27,5 J
a) Calcule el trabajo realizado por la fuerza
b) Calcule el trabajo efectuado por La fuerza de Ejemplo 4. Trabajo realizado por un resorte.
fricción.
e) Determine el trabajo neto efectuado sobre la caja
por todas las fuerzas que actúan sobre ella.

El resorte de la figura, cuando se deforma o estira


hasta una cierta posición x, ejercerá una fuerza
Solución.
→ restauradora F = − kx .
a) El trabajo efectuado por F es: Solución.
x=s → → Supongamos que el objeto se empuja hacia la
WF = ∫ F ⋅d r izquierda una distancia x respecto a la posición de
x =0
equilibrio y se deja libre.
→ →
Como F = F cos θ iˆ + Fsenθ ˆj y d r = dxiˆ El trabajo realizado desde x1 = − x hasta x 2 = 0

(F cosθ iˆ + Fsenθ ˆj )⋅ dxiˆ


x=s por la fuerza del resorte a medida que el objeto se
WF = ∫ mueve es
x =0
x2 = 0
= Fs cos θ = (− kx )dx = kx 2
1
0
Fx s W =∫ Fx dx = ∫
x1 = − x −x 2

La componente vertical de F no realiza trabajo. Y si consideramos el trabajo realizado por el resorte a
→ medida que se estira de x1 = 0 a x2 = x el trabajo
b) Como F f = − μNiˆ
1 2
Y N = mg − Fsenθ es W = − kx
2

Obtenemos F f = − μ (mg − Fsenθ )iˆ
Este resultado podemos obtenerlo también de La
gráfica F versus x, como se muestra en la figura
→ siguiente.
El trabajo efectuado por F f es
s → →
W f = ∫ F f ⋅ d r = ∫ − μ (mg − Fsenθ )iˆ ⋅ dxiˆ
s

0 0

= − μ (mg − Fsenθ )s
c) El trabajo neto sobre la caja es la suma de los
resultados obtenidos en a) y b).
Wneto = WF + W f = F cos θs − μ (mg − Fsenθ )s
[
= F cos θ − μ (mg − Fsenθ ) s ]
Ejemplo 5. La posición de una partícula en el plano

Ejemplo 3. Una fuerza que actúa sobre un cuerpo está dada por r = 3tiˆ − 2t ˆj (t en segundos, r en
2
varía con respecto a x como se muestra en la figura.

3
TRABAJO Y ENERGÍA Hugo Medina Guzmán

metros), la fuerza ejercida sobre la misma es P2 v2 ⎛ 1 ⎞


→ W1→2 = ∫ dW = ∫ d ⎜ mv 2 ⎟
F = 4iˆ − 5 ˆj (en Newton). P1 v1
⎝2 ⎠
¿Qué trabajo se realiza sobre la partícula en el 1 2 1 2
intervalo de t = l s a t = 3 s? = mv 2 − mv1
Solución.
2 2
→ →
Aquí tenemos una medida para el trabajo realizado
r = 3tiˆ − 2t 2 ˆj ⇒ d r = 3dtiˆ − 4tdtˆj sobre la partícula expresada en función de la
Luego ⎛1 2⎞
variación de la magnitud ⎜ mv ⎟ .
( )( )
→ →
dW = F ⋅ d r = 4iˆ − 5 ˆj ⋅ 3dtiˆ − 4tdtˆj ⎝2 ⎠
Esta magnitud se define como la ENERGIA
= 12dt + 20tdt CINETICA K de la partícula.
El trabajo W realizado sobre la partícula entre t = 1 y 1 2
t = 3. Entonces: K = mv
t =3 2
dW = ∫ (12 + 20t )dt
3
W =∫ La energía cinética es una propiedad general del
xt =1 1
3
movimiento de la partícula es la ENERGIA DEL
⎡ 1 ⎤ MOVIMIENTO. Sus dimensiones son las de trabajo.
= ⎢12t + 20t 2 ⎥ = 126 − 22 = 104 J
⎣ 2 ⎦1 [K ] = ML2 T -2
El trabajo realizado sobre la partícula es 104 Joules. Su unidad es la misma que la del trabajo.

Resulta conveniente escribir:


ENERGIA CINETICA W1→2 = K 2 − K 1 = ΔK
Consideremos una partícula de masa m bajo la acción El trabajo realizado por la fuerza al desplazar una

partícula es igual al cambio de energía cinética de la
de la fuerza F .
partícula.
La segunda ley de Newton afirma que:

→ →
dv Ejemplo 6. Encontrar la variación de la energía
F = ma = m cinética de un proyectil en función de su altura. Se
dt lanza un proyectil de masa m desde el punto P0 (x0,
→ → →
También sabemos que d r = v dt . y0) con una velocidad inicial v 0 = v 0 x iˆ + v 0 y ˆj .
Multiplicando escalarmente:
→ Solución.
→ → dv → → →
F⋅ d r = m ⋅ v dt = m v ⋅ d v
dt
→ →
Como F ⋅ d r es el trabajo diferencial dW y
d ⎛ 1 2 ⎞ 1 d ⎛→ →⎞
⎜ mv ⎟ = m ⎜ v ⋅ d v ⎟
dt ⎝ 2 ⎠ 2 dt ⎝ ⎠
→ →
1 → dv 1 dv →
= m v⋅ + m ⋅v Para un proyectil la posición en función del tiempo
2 dt 2 dt es;

dv → 1 2
= m v⋅ x = x0 + v0 x t , y = y 0 + v0 y t − gt
dt 2
De aquí: Y la velocidad
⎛1 ⎞ → → v x = v0 x , v y = v0 y − gt
d ⎜ mv 2 ⎟ = m v ⋅ d v
⎝2 ⎠ La energía cinética en P0 es
Reemplazando obtenemos: K0 =
1 2 1
(
mv0 = m v 02x + v 02y )
⎛1 ⎞ 2 2
dW = d ⎜ mv 2 ⎟ La energía cinética en P es
⎝2 ⎠
El trabajo para ir de P1 donde la velocidad es v1 al K=
1 2 1
2
(
mv = m v x2 + v y2
2
)
punto P2 donde la velocidad es v 2 será: 1
2
2
(
= m v0 x + v0 y − 2v0 y gt + g t
2 2 2
)

4
TRABAJO Y ENERGÍA Hugo Medina Guzmán

La variación de energía entre P y P0 es: dirigida hacia abajo y la bola se mueve lo más
1 1 rápidamente.
ΔK = K − K 0 = mv 2 − mv02
2 2 mv 2
fuerza radial =
1
(
= m − 2v 0 y gt + g t
2
2 2
) mv 2
r
1 2
T − mg = y mhg = mv
⎛ 1 2⎞ r 2
= − mg ⎜ v 0 y t − gt ⎟
⎝ 2 ⎠ h = r − r cos θ T − mg = 2mg (1 − cos θ )
1 2 T − mg 120 − 80
Como y − y 0 = v 0 y t − gt cos θ = 1 − = 1− = 0,75
2 2mg 2(80 )
⇒ θ = 41,4º
Resulta ΔK = mv − mv0 = − mg ( y − y 0 )
1 2 1 2
2 2
Ejemplo 8. Se arrastra una caja de masa m sobre un
Ejemplo 7. En una demostración experimental para piso horizontal, el coeficiente de fricción cinético
ilustrar la conservación de la energía por medio del entre la caja el piso es μ , mediante una fuerza que
dispositivo siguiente. Se ata una bola del bowling a forma un ángulo θ con la horizontal. Si se empieza a
un extremo de una cuerda, y se sujeta el otro extremo jalar desde el reposo y considerando que ya se inició
al techo de la sala de conferencias. Se sostiene la el movimiento ¿Cuál es la velocidad del bloque
bola parado en una escala tijeras alta, Para la después que recorre una distancia s?
demostración se suelta del reposo en el extremo de la
nariz, la bola volverá de la oscilación más arriba y
golpeará violentamente la cara, (intente esto alguna
vez si usted desea experimentar un juego para
asustar)
La demostración impresiona a la clase, pero no por la
razón esperada. Aunque la cuerda es bastante fuerte
para sostener la bola cuando está inmóvil, cuando la Solución.
dejé ir, la cuerda se rompió en el fondo del arco y la En este caso como la fuerza F es constante, por la ley
bola fue despedida alrededor del salón "Boing boing, de Newton podríamos encontrar la aceleración, que es
boing" y dispersó a los presentes en todas las constante, pero vamos a hacerlo por conceptos de
direcciones. Energía Cinética y Trabajo.
Una bola de bowling realmente rebota en el concreto. Encontramos que
Suponga que la bola pesa 80 N y la cuerda tenía 4,0 WNeto = [F cosθ − μ (mg − Fsenθ )]s
m de largo y tenía una resistencia a ruptura de 120 N. Sabemos que
¿Cuál es el máximo ángulo con la vertical con el que
1 2 1 2
se habría podido lanzar la bola sin tener la rotura de WNeto = K 2 − K1 = mv2 − mv1
la cuerda? 2 2
Como: v1 = 0 y v2 = v
Finalmente:

v=
2
[F cosθ − μ (mg − Fsenθ )]s
m

Ejemplo 9. Para el caso de la masa m atada a un


resorte con constante de rigidez k . ¿Cuál es la
velocidad cuando pasa por la posición de equilibrio
Solución. después de estirarlo una longitud L y soltarlo?
Solución.

La cuerda debe proporcionar suficiente fuerza


ascendente para balancear el peso más la fuerza radial
necesaria para que la bola haga la curva hacia arriba.
La tensión en la cuerda será así la mayor en el punto El trabajo realizado desde x = L a x = 0 por la fuerza
más bajo del arco, donde la fuerza de la gravedad está restauradora del resorte F = - kx Es:

5
TRABAJO Y ENERGÍA Hugo Medina Guzmán

1 2
WR = kL
2
1 2 1 2
También WR = K 2 − K1 = mv2 − mv1
2 2
Siendo v2 = v0 y v1 = 0 Si en un sistema conservativo el trabajo efectuado por
1 2 1 2 la fuerza para desplazar la partícula de A a B es
Tenemos kL = mv0 − 0 independiente del camino entre A y B, se puede
2 2 escribir:
k WAB = −WBA
⇒ v0 = ± L
m En un circuito cerrado
Para el caso que mostramos la respuesta correcta es la WAA = WAB + WBA
negativa. Como WAB = −WBA ⇒ WAA = WAB − WAB = 0
Ejemplo 10. Un objeto de masa m se mueve en el eje El trabajo total efectuado por una fuerza conservativa
→ sobre una partícula es cero cuando la partícula se
A ˆ
x sujeto a la fuerza F = m i donde A es una mueve alrededor de cualquier trayectoria cerrada y
x2 regresa a su posición inicial.
constante y x es la distancia desde el origen. Naturalmente la definición de un sistema no
a) ¿Cuánto trabajo realiza esta fuerza si el objeto se conservativo es aquel que no satisface las condiciones
mueve de x = a a x = b? anteriores.
b) ¿Si la masa tenía una velocidad v en la dirección
positiva de x, Cuál es su velocidad en b? Ejemplo 11. Sistema no Conservativo. - La fuerza
Solución. de fricción. Supongamos que un bloque se mueve del
a) El trabajo que realiza la fuerza para mover la masa punto P1 (x1, y1) al punto P2 (x2, y1), siguiendo Las
desde x = a a x = b es: trayectorias mostradas en las figuras siguientes, el
x =b → → → A ˆ → coeficiente de fricción entre el bloque y la superficie
Wab = ∫ F ⋅d r , F = m i , d r = dxiˆ es μ . Calcular el trabajo realizado por la fricción en
x=a x2 ambos casos.
b Aˆ b dx
Luego Wab = ∫ m 2
i ⋅ dxiˆ = mA∫ 2 =
a x a x
b
⎡ 1⎤ ⎛1 1⎞
mA⎢− ⎥ = mA⎜ − ⎟
⎣ x ⎦a ⎝a b⎠

1 2 1 2 Solución.
b) Como Wab = K b − K a = mvb − mva Por la trayectoria (a)
2 2 x2 → →
WP1 P2 = ∫ F f ⋅ d r
va = v0
x1
Siendo
→ →

⎛1 1⎞ 1 2 1 2 Aquí F f = − μNiˆ , d r = dxiˆ


Tenemos mA⎜ − ⎟ = mvb − mv0
⎝a b⎠ 2 2 Luego
(− μN )dx = μN (x2 − x1 )
x2

⎛1 1⎞ WP1 P2 = ∫
vb = 2 A⎜ − ⎟ + v02 x1

⎝a b⎠
Por la trayectoria (b)
y2 → → x2 → → y1 → →
SISTEMAS CONSERVATIVOS Y NO WP1P2 = ∫ F f 1 ⋅ d r1 + ∫ F f 2 ⋅ d r2 + ∫ F f 3 ⋅ d r3
CONSERVATIVOS y1 x1 y2

Un sistema conservativo es aquel en el que el trabajo Aquí


realizado por las fuerzas del sistema es independiente → →
de la trayectoria seguida por el móvil desde una F f 1 = − μNˆj , d r1 = dyˆj
posición a otra, no existen fuerzas de rozamiento, ni → →
dispositivos que puedan producir pérdida de la F f 2 = − μNiˆ , d r2 = dxiˆ
energía cinética. → →
F f 3 = μNˆj , d r3 = dyˆj
Luego

6
TRABAJO Y ENERGÍA Hugo Medina Guzmán

(− μN )dy + ∫x (− μN )dx + ∫y (μN )dy r2 → → r2 → →


y2 x2 y1
WP1 P2 = ∫ WP1P2 = ∫ Fg ⋅ d r + ∫ Fg ⋅ d r
y1 1 2

= − μN ( y2 − y1 ) − μN ( x2 − x1 ) + μN ( y1 − y2 )
r1 r3
Aquí
→ →

= − μN ( x2 − x1 ) − 2 μN ( y2 − y1 ) Fg = −mgˆj , d r = dxiˆ + dyˆj


→ →
Fg ⋅ d r = (− mgˆj )⋅ (dxiˆ + dyˆj ) = (− mg )dy
Obviamente el trabajo realizado por la fuerza de
fricción por las dos trayectorias a) y b) no son
iguales, por consiguiente cuando hay fuerza de Luego
fricción el sistema no es conservativo. (La fricción no
es conservativa).
(− mg )dy + ∫y (− mg )dy
y3 y2
WP1P2 = ∫
y1 3
Ejemplo 12. Sistema Conservativo. La fuerza de
la gravedad Supongamos que un bloque de masa m
se mueve del punto P1(x1 ,y2) al punto P2(x2 ,y2) donde = − mg ( y3 − y1 ) − mg ( y2 − y3 )
y es la dirección vertical. Calcular el trabajo realizado
= − mg ( y 2 − y1 )
por la fuerza gravitacional con los tres casos
mostrados en la figura.
Resultado igual que en a) y b)
Luego la fuerza de la gravedad es una fuerza
conservativa.

Trabajo en una trayectoria cerrada.


Si completamos la trayectoria volviendo al punto
inicial, tenemos una trayectoria cerrada y el trabajo es
cero.

El trabajo para ir de 1 a 2 es
x2 →
(F iˆ + F ˆj )⋅ (dxiˆ + dyˆj )
→ x2
Wr1r2 = ∫ F ⋅ d r1 = ∫ x y
x1 x1
Solución.
x2 y2
∫ Fx dx + ∫ Fy dy
Por la trayectoria a)
=
y2 → → x1 y1
WP1P2 = ∫ Fg ⋅ d r Como
y1

Aquí
→ →
F = −mgˆj : Fy = − mg . Fx = 0
Fg = −mgˆj , d r = dyˆj Wr1r2 = 0 − mg ( y2 − y1 )
= − mg ( y2 − y1 )
Luego
WP1 P2 = ∫ − mgdy = −mg ( y2 − y1 )
y2

y1
El trabajo para ir 2 a 1 es
x1 → → x1 y1
Por la trayectoria b) Wr2 r1 = ∫ F ⋅ d r = ∫ Fx dx + ∫ Fy dy
x2 → → y2 → → x1 → → x2 x2 y2
WP1P2 = ∫ Fg ⋅ d r1 + ∫ Fg ⋅ d r2 + ∫ Fg ⋅ d r3
x1 y1 x2 = 0 − mg ( y1 − y2 )
= − mg ( y1 − y2 )
Aquí
→ → → →
Fg = −mgˆj , d r1 = dxiˆ , d r2 = dyˆj , d r3 = dxiˆ
El trabajo total es
Luego
Wr1r1 = Wr1r2 + Wr2 r1
(− mg )dy + 0 = − mg ( y2 − y1 )
y2
WP1P2 = 0 + ∫
y1 = − mg ( y2 − y1 ) − mg ( y1 − y2 )
Igual que en a) =0

Por la trayectoria c). Esto no sucedería en el caso de una fuerza no


WP1P2 = WP1P3 + WP3 P2 conservativa, como la fuerza de fricción.

7
TRABAJO Y ENERGÍA Hugo Medina Guzmán

LA FUNCION ENERGÍA POTENCIAL Si consideramos la energía potencial igual a cero en


El trabajo realizado por la fuerza el nivel de referencia y = 0, la energía potencial a
→ cualquier altura con respecto a y = 0 es:
F = Fxiˆ + Fy ˆj + Fz kˆ U ( y ) = mgy
Para mover una partícula de También podíamos haber determinado esta función a
P1(x1. y1, z1) a P2(x2, y2, z2) es igual a: partir de:

∫ (F dx + F dy + F dz ) dU = − Fg dy ⇒ dU = −(− mg )dy = mgdy


P2 → → P2
W12 = ∫ F ⋅ d r = x y z
P1 P1
Integrando
Para un sistema conservativo el trabajo es
independiente de la trayectoria seguida.
∫ dU = ∫ mgdy + C
Su integral debe ser un diferencial exacto, U ( y ) = mgy + C
digamos - dU, tal que integrándolo, solamente los Donde C es una constante relacionada con las
límites determinan el valor de la integral. condiciones de cada caso, por ejemplo aquí
Esto es: consideramos para y = 0
W12 = ∫
P2
(− dU ) = (− U )PP 2
= −(U 2 − U1 ) = − ΔU ⇒ U (0 ) = 0 .
P1 1

Aquí llamamos a U, energía potencial, cuyas La constante es C = 0


unidades son las mismas que las de trabajo. ⇒ U ( y ) = mgy
Como comprobación, a partir de esta energía
Hemos determinado la función energía potencial a potencial podemos encontrar la fuerza.
partir de una fuerza dada.
∂U ∂mgy
Fx = − =− =0
Consideremos ahora el problema inverso, a partir de ∂x ∂x
una función energía potencial determinar la fuerza ∂U ∂mgy
→ → Fy = − =− = −mg
dU = − F ⋅ d r = − Fx dx − Fy dy − Fz dz ∂y ∂y
Como U es función de x, y y z, podemos escribir esta ∂U ∂mgy
Fz = − =− =0
derivada en función de sus derivadas parciales: ∂z ∂z
∂U ∂U ∂U →
dU ( x , y , z ) = dx + dy + dz Luego: F = − mgˆj
∂x ∂y ∂z

Relacionando con los componentes de la fuerza Ejemplo 14. Determinar la función energía potencial
obtenemos asociada a un resorte de constante de rigidez k.
Solución.
∂U ∂U ∂U
Fx = − , Fy = − , Fz = − Consideremos que el resorte está en el eje x, y se
∂x ∂y ∂z estira en esa dirección.

→ ⎛ ∂U ˆ ∂U ˆ ∂U ˆ ⎞ F = −kxiˆ
F = −⎜⎜ i+ j+ k ⎟⎟
⎝ ∂x ∂y ∂ z ⎠ Tenemos que:
dU = − Fx dx = −(− kx )dx = kxdx
Ejemplo 13. La fuerza de la gravedad es un ejemplo Integrando
de fuerza conservativa. 1 2
Solución. U= kx + C
Tomemos la vertical a la tierra como el eje y, tal que: 2
→ Si para la posición de equilibrio x = 0, la energía
Fg = Fg ˆj = −mgˆj potencial es cero, C es igual a cero y
1 2
El trabajo realizado por la gravedad cuando la U= kx
partícula se desplaza desde el punto y1 al punto y2 es: 2
y2 y2
W12 = ∫ − mgˆj ⋅ dyˆj = − mg ∫ dy Ahora realicemos el problema inverso:
y1 y1 1 2
Dado U =
= − mg ( y2 − y1 )
kx encontrar la fuerza
2
Como W12 − ΔU : correspondiente:
− mg ( y2 − y1 ) = −ΔU = U ( y1 ) − U ( y 2 ) ∂U ∂ ⎛1 ⎞
Fx = − = − ⎜ kx 2 ⎟ = −kx
∂x ∂x ⎝ 2 ⎠
O ΔU = U ( y 2 ) − U ( y1 ) = mgy2 − mgy1

8
TRABAJO Y ENERGÍA Hugo Medina Guzmán

∂U Colocando las energías iniciales a un lado y las


Fy = − =0 finales al otro tenemos:
∂y
K1 + U1 = K 2 + U 2
∂U
Fz = − =0 Esta ecuación es la forma matemática de “El
∂z principio de conservación de la energía mecánica”.
→ Si definimos la energía mecánica total del sistema E
Luego F = − kxiˆ como la suma de la energía cinética y potencial se
puede expresar la conservación de la energía
Ejemplo 15. Energía potencial gravitatoria cerca mecánica como:
de la tierra. Por la ley de Newton de la gravitación E = K + U = Constante
universal, la fuerza de atracción de dos masas es
directamente proporcional al producto de estas e Ejemplo 16. Fuerza de la gravedad: Se suelta una
inversamente proporcional al cuadrado de la partícula de masa m desde la altura h sobre el suelo.
distancia. Cuando la partícula está a una altura y del suelo, su
→ mM velocidad es v.
F = −G rˆ Su energía potencial es U = mgy
r2
Donde m es la masa e un cuerpo, M la masa de la 1 2
tierra, r la distancia entre las masas, G es la constante Su energía cinética es K = mv
gravitatoria universal. 2
Si r = R (radio de la tierra), la masa m está sobre la La energía mecánica total es:
superficie de la tierra y 1 2

E = K +U = mv + mgy
mM 2
F = −G rˆ = − mgrˆ
R2 Para y = h , v = 0
La energía potencial es E = 0 + mgh = mgh
→ →
mM Para y = 0 , v = v 0
dU = − F ⋅ d r = G dr
r2 1 1
mM E = mv02 + 0 = mv02
U = ∫G dr + C 2 2
r2 Para cualquier instante
mM 1 2
U = −G +C E= mv + mgy = mgh
r 2
De aquí v = 2 g (h − y ) ⇒
Para evaluar la constante C consideremos que el 2
potencial U es cero para r infinito, de aquí C es igual
a cero, v = 2 g (h − y )
Luego
El gráfico de la variación de energía potencial y
mM
U ( r ) = −G cinética es:
r

Ejemplo 17. Una masa pequeña m se suelta desde el


CONSERVACION DE LA ENERGÍA reposo de la parte más alta de una superficie esférica
Hasta esta parte tenemos dos formas de encontrar el de radio R, sin fricción. ¿A qué ángulo con vertical
trabajo realizado sobre un objeto por una fuerza, la dejará el contacto con la esfera?
primera válida para todo caso ya sea fuerza
conservativa o no conservativa
1 2 1 2
W12 = mv2 − mv1 = K 2 − K1 = ΔK .
2 2
Y la segunda para el caso de fuerzas conservativas
W12 = U1 − U 2 = − ΔU Solución.
Cuando la masa está a una altura h su energía es igual
Luego podemos escribir a cuando está en el punto más alto.
W12 = K 2 − K1 = U1 − U 2

9
TRABAJO Y ENERGÍA Hugo Medina Guzmán

1 2 1
E= mv0 + 0 = mv02
2 2
Para cualquier instante
1 2 1 2 1 2
1 E= mv + kx = kL
mgR + 0 = mgh + mv 2 2 2 2

Con h = R cos θ
2
De aquí: v =
2 k
m
(
L2 − x 2 )
1
mgR = mgR cosθ + mv 2
2 ⇒ v= (L − x 2 )
k 2

⇒ v = gR(2 − 2 cosθ )
2 m
El gráfico de la variación de la energía potencial y
La segunda ecuación de Newton cuando la masa esta cinética es:
en la posición del ángulo θ:

v2
Con ac = :
R
Ejemplo 19. Calcular la velocidad necesaria para
v2
N − mg cosθ = m que una partícula pueda escapar de la atracción de la
R tierra. La energía total E de una partícula de masa m
La masa deja la superficie esférica cuando: que está a una distancia r del centro de la tierra y que
N =0 tiene una velocidad v es:

mv 2 mgR(2 − 2 cosθ )
1 2
E = K + U , donde K = mv y
mg cos θ = = 2
R R
mM
v 2
U = −G
⇒ cosθ = = 2 − 2 cosθ r
Rg 1 2 mM
2 Luego: E = mv − G = Constante
⇒ cos θ = 2 r
3 Si la partícula escapa de la atracción de la tierra y se
⇒ θ = 48,2º sitúa a una distancia infinita de ésta su potencial es
cero.
Ejemplo 18. Fuerza de un resorte: Se jala una masa mM
a sujeta a un resorte de constante k sobre una
r → ∞ , U ∞ = −G →0
r
superficie sin fricción, desde la posición de equilibrio En esta región con la velocidad menor posible
x = 0 hasta una distancia L y se suelta.
A una distancia x de la posición de equilibrio la v∞ = 0 Tenemos K ∞ = 0
velocidad de la masa es v. Luego: E = K + U = 0
1 2 Como E es constante ⇒ E = 0
Su energía potencial es U = kx
2 La energía E de la partícula en la superficie de la
1 2
Su energía cinética es K = mv tierra con la velocidad ve para que pueda escapar:
2
1 2 mM
Su energía mecánica total es: E= mve − G =0
1 2 1 2 2 R
E = K +U = mv + kx
2 2 2GM
⇒ ve =
Para x = L , v = 0 R
1 1
E = 0 + kL2 = kL2 Como en la superficie de la tierra
2 2
mM GM
Para x = 0 , v = v 0 F = −G 2
= −mg ⇒ g = 2
R R
Tenemos: v e = 2 gR

10
TRABAJO Y ENERGÍA Hugo Medina Guzmán

Siendo
m
g = 9,81 2
y R = 6,4 × 10 m
6

s
4 m
Obtenemos; ve = 1,12 × 10
s
Ejemplo 20. Se tiene un resorte de longitud L y Solución.
constante k conectado a la base de un bloque de masa Velocidad de la partícula:
m, Se suelta el bloque desde la altura H. ¿Cuál será la Tenemos que
distancia mas cercana al piso que alcanzará el bloque 1 2
antes de rebotar?
ETotal = K + U = mv + U ( x )
2
Solución.
1 2
⇒ mv = E − U ( x ) ⇒
2

v=
2
(E − U ( x ) )
m

La energía cinética:
- Es igual a cero en x1 y x5.
- Tiene su valor máximo donde U(x) es mínimo, el
punto x2
En el instante inicial la energía es solamente la
potencia1 gravitatoria es U = mgH , la energía La partícula se mueve entre x1 y x5, fuera de estos
cinética es cero, tal que la energía total es valores la velocidad sería imaginaria.
E = mgH .
En el instante final: La energía potencial es la dU ( x )
correspondiente a la masa a una altura y, más la del Como Fx = − , la pendiente del gráfico de
resorte comprimido una longitud (L − y ) , es decir:
dt
U(x) en determinado punto corresponde a La fuerza
U = U g + U r = mgy + k (L − y )
1 2 actuante, tal que la fuerza se hace cero donde la
pendiente es cero, como en x2, x3 y x4.
2
Como en ese instante ha cesado el movimiento, la La fuerza es positiva entre x1 y x2.entre x3 y x4. La
energía cinética es cero, fuerza es negativa entre x2 y x3, entre x4 y x5.
La energía total es: Los puntos en que U es mínimo, son posiciones de
E = mgy + k (L − y )
1 2 equilibrio estable, como son x2 y x4.
2
Ejemplo 22. En la figura, un auto de juguete de
Por la conservación de la energía masa m se libera del reposo en la pista circular. ¿Si
se suelta a una altura 2R sobre el piso, ¿cuán arriba
mgH = mgy + k (L − y )
1 2
sobre el piso estará cuando sale de la pista, desprecie
2 la fricción?

La solución de esta ecuación es:


⎞ mg ⎡ 2k
⎛ mg
y = −⎜ − L⎟ ± ⎢1+ (H − L )⎤⎥
⎝ R ⎠ k ⎣ mg ⎦

Siendo el valor positivo de y la solución significativa.

Ejemplo 21. El gráfico de la figura muestra la Solución.


función potencial y la energía total de un En la figura de arriba:
movimiento. ¿Qué podemos decir acerca del h = R (1 + senθ )
movimiento? Despreciando las pérdidas por fricción la energía total
es constante, de tal manera que:
Siendo v la velocidad del auto a la altura h.

11
TRABAJO Y ENERGÍA Hugo Medina Guzmán

Solución.
mg (2 R ) = mg (h ) + mv 2 ⇒
1
a) Siendo v la velocidad de la masa en la parte
2 superior del rizo.
mg (2 R ) = mgR(1 + senθ ) + mv 2 ⇒
1 Por conservación de la energía:

mg (H ) = mg (2 R ) + mv 2 ⇒
2 1
1 2
gR = gRsenθ + v 2 (1)
2 1 2
gH = 2 gR + v (1)
Aplicando la segunda ley de Newton en la altura h: 2
Aplicando la segunda ley de Newton en ese punto:

mgsenθ − N = mac
N = 0, condición de caída.
mg − N = mac
v2
ac = N = 0, condición de caída.
R
v2
Luego: ac =
v2 R
mgsenθ = m ⇒ Luego:
R
v2
v = gRsenθ (2)
2
mg = m ⇒
Reemplazando (2) en (1):
R
1 v 2 = gR (2)
gR = gR sen θ + gR sen θ ⇒ Reemplazando (2) en (1):
2 1
3 2 gH = 2 gR + gR ⇒
gR = gRsenθ ⇒ senθ = 2
2 3 5
Finalmente: H= R ⇒ H = 2,5 R
2
⎛ `2 ⎞
h = R (1 + senθ ) = R⎜1 + ⎟
⎝ 3⎠ b) Sea v la velocidad en el punto A su altura es
5 h = R(1 + cos α )
= R = 1,67 R
3
Ejemplo 23. Una masa pequeña resbala sobre una
superficie inclinada pasando por un rizo de radio R.
a) ¿Cuál es la altura mínima H de la que debe soltarse
a fin de que el cuerpo no deje la superficie interior del
rizo al dar la vuelta? Por conservación de la energía:

mg (H ) = mgR(h ) + mv 2 ⇒
b) ¿Con que velocidad llega la masa al punto A? 1
c) ¿Cuál es el valor del ángulo α , con el que se 2

puede retirar el segmento AB de la circunferencia mg (2,5R ) = mgR(1 + cos α ) + mv 2 ⇒
1
de tal modo que la masa que sale de A alcance el 2
v 2 = 2 g (2,5R ) − 2 gR(1 + cos α ) ⇒
punto B después de viajar una cierta distancia en el
aire.
v 2 = 3gR − 2 gR cos α ⇒
v = gR(3 − 2 cos α )

c) La masa sale del punto A, como un proyectil con



velocidad inicial v = vxiˆ + v y ˆj

12
TRABAJO Y ENERGÍA Hugo Medina Guzmán

Solución.
F 220
a) k = = = 344 N/m
x 0,64
b) U 1 + K 1 = U 2 + K 2
1 2 1 k
kx + 0 = 0 + mv 2 ⇒ v = x
En el tiempo t de su recorrido vertical debe alcanzar 2 2 m
al punto B.
Recorrido vertical: v=
344
(0,64) = 76,6 m/s
1 2 0,024
y = vsenα t − gt
2 Ejemplo 24. Puenting. Un saltador que pesa 800 N
Cuando llega a B, y = 0: se ata con una cuerda elástica al tobillo y se salta de
1 2 2v
0 = vsenα t − gt ⇒ t = senα una torre alta. La cuerda tiene una longitud si estirar
2 g de 30 m, y un extremo se une al punto donde el salto
comienza. ¿La constante del resorte de la cuerda
Su recorrido horizontal es
elástica es 200 N/m. ¿Cuánto recorrerá el saltador
x = vxt = v cos α t antes de que la cuerda detenga su descenso?
2v
Para t = senα debe de estar en B, luego:
g
⎛ 2v ⎞
2 Rsenα = v cos α ⎜⎜ senα ⎟⎟ ⇒
⎝ g ⎠
gR
v2 =
cos α
Igualando esta expresión de la velocidad con la
encontrada anteriormente: Solución.
gR(3 − 2 cos α ) =
gR Sea el punto más bajo del salto h = 0. La energía

cos α cinética inicial y la energía cinética en el punto más
bajo son ambas igual a cero.
3 1
cos 2 α − cos α + = 0 Tal que por la conservación de la energía:
2 2 1 2
Resolviendo: mgh = 0 + kx , donde x = h − 30 .
2
⎧1 Sustituyendo
cos α = ⎨ mg = 800 N y k = 200 N/m, y resolviendo:
⎩1 2
En nuestro caso tomamos la solución ½, con la que h 2 − 68h + 900 = 0 ⇒
obtenemos α = 60º h = 68 ± (68)2 − 4(900) = 50 m, o 18 m.
La solución correcta es h = 50 m. La solución h = 18
Ejemplo 24. Un arco del tiro al arco ejerce la fuerza
m corresponde al rebote que comprime la cuerda
kx de la ley de Hooke en una flecha cuando la cuerda
“amortiguador auxiliar”, pero una cuerda no se
se jala una distancia x. Se supone que un arquero
comprime como un resorte.
ejerce una fuerza de 220 N jalando a la flecha una
distancia de 64 cm.
Ejemplo 25. En la figura mostrada, el hombre y la
a) ¿Cuál es la constante del resorte del arco?
plataforma tienen una masa m, el hombre se eleva una
b) ¿Cuál es la velocidad de una flecha de masa 24 g
distancia h tirando la cuerda del lado derecho.
cuando deja el arco?
a) ¿En cuánto aumenta su energía potencial
gravitatoria?
b) ¿Qué fuerza debe ejercer para elevarse?
c) ¿Qué longitud de cuerda debe tirar para llegar a la
posición superior?
d) ¿Despreciando el rozamiento ¿Qué trabajo habrá
realizado?

13
TRABAJO Y ENERGÍA Hugo Medina Guzmán

W = Fd
F
La aceleración de la caja es a =
m
Como la caja parte del reposo su velocidad en la
posición final es:
2 Fd
v2 = 2ad =
m
El observador determina que el cambio de energía:
Solución.
1 2 1 2
a) La energía potencial gravitatoria es ΔK = K 2 − K1 = mv2 − mv1
U ( y ) = mgy + C 2 2
Para la posición inicial 2 Fd
Como v1 = 0 y v2 =
U1 = mgy1 + C m
Para la posición final 1 ⎛ 2 Fd ⎞
U 2 = mgy2 + C ΔK = m⎜ ⎟ = Fd
2 ⎝ m ⎠
El aumento de la energía potencial gravitatoria es:
U = U 2 − U1 = mg ( y2 − y1 ) = mgh
El observador sobre la plataforma concluye que:
1 ⎛ 2 Fd ⎞
b) La fuerza para elevar el sistema, siendo esta ΔK = m⎜ ⎟ = Fd
conservativa, 2 ⎝ m ⎠
∂U W = ΔK
F =− = −mg
∂y b) Observador situado en tierra:

Como la polea divide en dos, la fuerza Fh que debe


mg
ejercer el hombre es: Fh = .
2
c) Para llegar a la posición superior la cuerda debe ser
tirada en una longitud dos veces h El observador en tierra ve que la caja se mueve bajo
d = 2h. la acción de la fuerza F, en este caso la caja se mueve
d) EL trabajo realizado por el hombre es: la distancia d ' = Vt´+ d ,. Siendo t el tiempo que
demora el recorrido de la distancia d sobre la
⎛ mg ⎞
Wh = Fh d = ⎜ ⎟(2h ) = mgh plataforma,
⎝ 2 ⎠ 2d 2dm 2dm
Justamente igual al cambio de energía. t= = , luego d ' = V +d
a F F
Ejemplo 26. Observadores en movimiento El trabajo es:
relativo. Sobre una plataforma en movimiento ⎛ 2dm ⎞
horizontal con una velocidad constante V. un hombre W ' = Fd ' = F ⎜⎜V + d ⎟⎟
empuja una caja de masa m con una fuerza F una ⎝ F ⎠
distancia d partiendo del reposo. Demostrar la validez
de la conservación de la energía desde los puntos de W ' = Fd + V 2 Fdm
vista de observadores en marcos inerciales diferentes. El observador ve que la caja tiene una velocidad
Solución. inicial
Las leyes de Newton se cumplen sólo en marcos de v'1 = V
referencia inerciales. Si se cumplen en uno en y una velocidad final
particular entonces se cumplen en todos los marcos
2 Fd
de referencia que se muevan a velocidad constante en v'2 = V + v2 = V +
relación a este mareo. m
a) Observador en la plataforma.
El observador en tierra determina que el cambio de
energía es:
1 1
ΔK ' = K '2 − K '1 = mv'22 − mv'12
2 2
2
1 ⎛ 2 Fd ⎞ 1
El observador en la plataforma ve que la caja, de ΔK ' = m⎜⎜V + ⎟ − mV 2
masa m, se mueve bajo la acción de la fuerza F. El 2 ⎝ m ⎟⎠ 2
trabajo realizado para mover la distancia d es:

14
TRABAJO Y ENERGÍA Hugo Medina Guzmán

ΔK ' = Fd + V 2 Fdm La energía inicial es solo la energía cinética de la


masa:
1 2
Aquí se cumple también la conservación de la Ei = mv0
energía: 2
W ' = ΔK ' La energía final es solo la energía potencial del
resorte:
SISTEMAS NO CONSERVATIVOS. 1 2
Supongamos que también intervienen fuerzas no Ef = kx
conservativas, como la fricción. 2
El trabajo hecho por la fricción
x2
El trabajo total para mover la partícula de r1 a r2 es W f = ∫ F f dx , x1 = 0 , x 2 = L + x ,
x1
1 1
W12 = mv22 − mv12 = K 2 − K1 F f = − μN = − μmg
2 2
Luego:
L+ x
Este trabajo es también igual a la suma del trabajo W12 = ∫ (− μmg )dx = − μmg (L + x )
realizado por las fuerzas conservativas y del trabajo 0

realizado por las fuerzas no conservativas, es decir: Como en un Sistema no Conservativo.


W12 NO CONSERVATIVAS = E2 − E1
W12 = W12 CONSERVATIVAS + W12 NO CONSERVATIVAS
− μmg (L + x ) =
1 2 1 2
kx − mv0
Como: 2 2
W12 CONSERVATIVAS = U 1 − U 2 Ecuación de segundo grado cuya solución es:

W12 = U 1 − U 2 + W12 NO CONSERVATIVAS


μmg μ 2m2 g 2
x=−
k
±
k 2

m
k
(
2μgL − v02 )
De las expresiones de trabajo total tenemos:
K 2 − K1 = U 1 − U 2 + W12 NO CONSERVATIVAS
Ejemplo 28. Un cuerpo de masa 10 kilogramos cae
⇒ (K 2 − U 2 ) − (K1 − U 1 ) = W12 NO CONSERVATIVAS desde una altura de 15 metros y alcanza el suelo en 2
segundos. Considerando constante la fuerza de
E 2 − E1 = W12 NO CONSERVATIVAS resistencia del aire.
a) ¿Cuál era la magnitud de la fuerza de resistencia?
b) ¿Cuánta energía mecánica se ha perdido?
A diferencia que en un Sistema conservativo, no es
c) ¿Qué velocidad tenía el cuerpo inmediatamente
igual a cero.
antes de chocar Contra el suelo?
Esta última expresión nos permite calcular el trabajo
Solución.
de fuerzas no conservativas, fuerzas que en general
a) Siendo el peso y la fuerza de resistencia del aire las
son complicadas y que en principio deberíamos de
fuerzas que intervienen y siendo ambas constantes
calcular resolviendo integrales curvilíneas.
tenemos que la aceleración a del cuerpo es constante.
1 2
Ejemplo 27. A un bloque de masa m se le da un Como h = at
empujón tal que adquiere la velocidad v 0 a lo largo 2
del eje x. Después de resbalar distancia L golpea un 2h 2(15) m
La aceleración es a = 2
= 2 = 7,5 2
resorte de constante k. Si el coeficiente de fricción t 2 s
entre el bloque y la masa es μ . ¿Cuánto se Aplicando la segunda ley de Newton:
comprime el resorte? mg − Fg = ma
Solución.
Fg = m( g − a ) = 10(9,8 − 7,5) = 23 N
b) La energía que se ha perdido es el trabajo realizado
por las fuerzas no conservativas.
W NO CONSERVATIVAS = Fg d = (23)(15) = 345 J
c) Como W NO CONSERVATIVAS = E 2 − E1
Siendo
E1 = K1 + U1 = 0 + mgh = (10)(9,8)(15)
Sea x La longitud que se comprime el resorte. = 1470 J
La distancia recorrida por la masa es 1 2
(L + x). E2 = K 2 + U 21 = mv2 + 0 = 5v22
2

15
TRABAJO Y ENERGÍA Hugo Medina Guzmán

Tenemos: v2
1470 − 345 s=
5v22 − 1470 = 345 ⇒ v22 = = 225 2 g (senθ + μ cos θ )
5
m (40)2
Finalmente: v2 = 15 =
2(9,8)(sen30º +0,5 cos 30º )
= 87,5 m
s
Una manera directa de llegar al mismo resultado es
considerar que la aceleración efectiva de salida es POTENCIA
m Tan importante como saber cual es el trabajo
a = 7,5 , la velocidad después de 2 segundos es: realizado es conocer también la rapidez con la cual se
s2 realiza. Para proporcionar una medida cuantitativa de
⎛ m⎞ m este concepto que incluye tanto el trabajo como el
v2 = at = ⎜ 7,5 2 ⎟ = 15 tiempo necesario para realizarlo se tiene a la
⎝ s ⎠ s Potencia.
La potencia mide la rapidez con la que el trabajo se
LA CONSERVACIÓN DE LA ENERGÍA Y LA está realizando.
FRICCIÓN
La ley de la conservación de la energía se puede Si se realiza un trabajo W en un intervalo de tiempo
aplicar a los sistemas donde las fuerzas no (de t1 a t2) la Potencia media es:
conservativas como actúan las fuerzas de la fricción. W12 ΔW
Si un sistema trabaja contra la fricción, la energía Pm = =
mecánica del sistema disminuirá. t2 − t1 Δt
Así si Wf es el trabajo hecho contra la fricción, Cuando t 2 → t1 , Δt → 0 , tendremos
entonces energía inicial - la energía perdida por la
La Potencia instantánea en el instante t.
fricción
ΔW dW
E1 − W f = E 2 P = lim =
Δt → 0 Δt dt
1 2 1
U1 + mv1 − W f = U 2 + mv 22 También como
2 2 → →
dW = F ⋅ d r
Ejemplo 29. Cerca de Lewiston, Idaho, hay una Tenemos

carretera muy inclinada donde circulan camiones
dW → d r
cargados con madera. Han ocurrido varios accidentes P= = F⋅
serios cuando los carros perdieron sus frenos yendo dt dt
para abajo de la colina a gran velocidad. Se han → →
construido rampas de contención que se espera P = F⋅ v
puedan detener a los vehículos sin frenos. Suponga
que un carro que viaja a 40 m/s encuentra una rampa El análisis dimensional
inclinada para arriba 30º sobre horizontal. La grava
floja en la rampa proporciona una fuerza friccional
[P] = [F ][L][T ]−1 = [M ][L]2 [T ]−1
para ayudar a detener al carro mientras sube la rampa.
La grava tiene un coeficiente eficaz de fricción de Su unidad en el sistema internacional es J/s llamado
0,50. ¿Cuán lejos a lo largo de la rampa el carro Watt ó Vatio cuyo símbolo es W.
viajaría antes de detenerse? Un múltiplo muy usado es el kilowatt (kW)
Solución. 1 kW = 103 W
Existe una unidad de energía o trabajo en términos de
N = mg cos θ
la unidad de potencia el kilowatt-hora (kwh), es la
F f = μN = μmg cosθ energía convertida o consumida en una hora a una
U1 + K1 − W f = U 2 + K 2 razón constante de 1 kW.
1 kWh (103W)(3600s) = 3,6 x l06 .J

Para tener una idea de cuanto es 1 Watt, imaginemos


que tenemos que levantar una masa de 50 kg. a una
altura de 1 metro, cada 5 minutos y realizar este
trabajo durante una jornada de 8 horas. Si levanta
cada 5 minutos, serán 12 veces por hora, siendo 8
1 2 horas por día, hará un total de 12 x 8 = 96 veces al
0+ mv − F f s = mgh + 0 h = s senθ día.
2 El trabajo realizado es:
mv − (μmg cos θ )s = mgs senθ
1 2 W = 96mgh
2 = 96(50Kg )(9,8m / s )(1m ) = 47040 J

16
TRABAJO Y ENERGÍA Hugo Medina Guzmán

Para determinar la potencia tenemos que dividirlo por Luego: P = (mg senθ )( g senθ t )
el número de segundos en un día.
47040J = mg 2 sen 2θ t
P=
2(h − y )
= 1,63 W
8 × 3600 s Como t=
Comparemos esta potencia con la potencia de un gsen 2θ
motor pequeño de 1 hp (horse power).
El hp es la unidad de potencia en el sistema inglés Obtenemos:
1 hp = 746 W 2(h − y )
P = mg 2 sen 2θ
gsen 2θ
Ejemplo 30. Si un objeto que parte del reposo se
desliza por un piso liso inclinado un ángulo θ con = mgsenθ 2 g (h − y )
respecto a la horizontal de altura h, hallar la potencia
P gastada por la gravedad en función de la posición y Ejemplo 31. El flujo de agua de un río es de 50 m3
del objeto con respecto a la parte inferior plano por segundo, se tiene un desnivel de 200 metros y se
inclinado. quiere aprovechar construyendo una hidroeléctrica
Solución. a) Si la energía del agua que cae se utilizase
totalmente ¿Que potencia se podría obtener?
b) Si toda la energía procedente de la caída del río se
convirtiese en energía eléctrica y se vendiese a un sol
el kilowatt-hora ¿Cuánto dinero se cobraría en un
día?
Solución.
a) El trabajo realizado por una masa m que cae desde
La potencia es:
una altura h es:
dW
P= , siendo W = Fd W = mgh
dt Como m = ρV,
Con Donde ρ es la densidad del agua. V es el volumen.
1 2 1 W = ρVgh
F = mg senθ y d = at = g senθ t 2
2 2
Tenemos La potencia que se obtiene al pie de la salida es
⎛1 ⎞ dW d
W = (mg senθ )⎜ g senθ t 2 ⎟ P= = ρVgh
dt dt
⎝2 ⎠ De estas cantidades la que varía con el tiempo es V.
1
= mg sen θ t y m3
2 2 2
dV
2 = 50
dt s
dW d ⎛1 2⎞
P= ⎜ mg sen θ t ⎟ Luego
2 2
=
dt dt ⎝ 2 ⎠ dV
P = ρgh
= mg sen θ
2 2
dt
Como ha recorrido la distancia s: Como

s=
(h − y ) = 1 gsenθ t 2 ρ = 1000
kg m
, g = 9,8 2 , h = 200m
3
senθ 2 m s
Obtenemos: Obtenemos
2(h − y ) ⎛ kg ⎞⎛ m ⎞ ⎛ m3 ⎞
P = ⎜1000 3 ⎟⎜ 9,8 2 ⎟(200m )⎜⎜ 50 ⎟
t=
gsen 2θ ⎝ m ⎠⎝ s ⎠ ⎝ s ⎟⎠
Luego = 9,8 x 107 W
2(h − y )
P = mg 2 sen 2θ b) Si tenemos una potencia P = 9,8 x 107 = 9,8 x 104
gsen 2θ kW y consideramos que se consume las 24 horas del
= mgsenθ 2 g (h − y )
día. La energía obtenida es igual a todo el trabajo
realizado.
dW = Pdt
Otra manera de obtener es considerar que:
W = P ∫ dt = P(t 2 − t1 ) = PΔt
t2
P = Fv t1

W = (9,8 × 104 kW )(24h ) = 235,2 x 104 kW-h


Donde
F = mg senθ y v = at = gsenθ t

17
TRABAJO Y ENERGÍA Hugo Medina Guzmán

si el precio de cada kW-h es 1 sol, cada día se MÁQUINAS


obtendrán 2,352 millones de soles. Una máquina simple es un dispositivo usado para
magnificar una fuerza o para cambiar una
Ejemplo 32. En la figura, un bloque de masa m desplazamiento pequeño en grande. Las máquinas
descansa sobre una faja que se mueve con velocidad comunes son la palanca, el plano inclinado, el gato
constante v. El coeficiente de fricción entre el bloque hidráulico, o una combinación de engranajes.
y la faja es μk. El trabajo se hace típicamente en la máquina (el
Tomando como tiempo inicial t = 0, una fuerza trabajo W1 de entrada), y entonces la máquina
horizontal F aplicada al bloque le produce una alternadamente hace un cierto trabajo W2 de salida.
aceleración constante a. El estado de la energía de la máquina no cambia
a) Determinar la fuerza F y la potencia disipada en apreciable durante este proceso, así que si la fricción
fricción como función del tiempo. es insignificante, W1 = W2, basado en la idea de la
b) Si la fuerza F es ejercida por un hombre que se conservación de energía. Muy a menudo las fuerzas
encuentra sobre la faja. Determinar la potencia que de entrada y de salida son constantes, en las cuales el
este libera en función del tiempo. caso W1 = W2, lo que lleva a:
e) Si la fuerza F es ejercida por un hombre que d1
camina sobre el piso al costado de la faja. Determinar F1 d1 = F2 d 2 ⇒ F2 = F1
la potencia que este libera en función del tiempo. d2
Aquí F1 actúa sobre una distancia d1 y F2 actúa sobre
una distancia d2. La ventaja mecánica de la máquina
se define como
F2
VM =
F1
Solución.
a) Aplicando la segunda ley de Newton a la masa m Ejemplo 33. La palanca de barra es un dispositivo
en la figura usado para levantar objetos pesados (por ejemplo, un
piano o una pieza grande de maquinaria). Consiste en
una barra larga que se apoya en un fulcro una
distancia corta del extremo de levantar de la barra.
Suponga que el fulcro de una barra de la palanca está
a 3 centímetros de la carga, y el punto donde usted
empuja hacia abajo en el otro extremo está a 1,50 m
F − F f = ma del fulcro.
Como F f = μ k N = μ k mg , obtenemos: ¿Qué fuerza mínima tendría que ejercer para levantar
una carga de 2000 N?
F = ma + μ k mg ¿Si mueve el extremo de la barra 4 centímetros hacia
y la potencia disipada en fricción es abajo, cuánto levantará la carga?
P = F f v0 = (μ k mg )v0 , siendo v0 = at Solución.

P = μ k mgat

b) La fuerza que hace el hombre sobre la faja es


F = ma + μ k mg
Su velocidad en función del tiempo es
v' = v + v0 = v + at Si la barra rota con un ángulo pequeño Δθ, entonces
y la potencia que debe dar el hombre es d1 = L1 Δθ y d 2 = L2 Δθ
P = Fv' = (ma + μk mg )at F1 L1 Δθ = F2 L2 Δθ
L
c) La tuerza que hará el hombre sobre el piso es igual F1 = 2 F2 ⇒
al caso anterior: L1
F ' = ma + μ k mg F1 =
(0,03) (2000)
La velocidad del hombre en función del tiempo en 1,50
este caso es: = 40 N
v' = v + at
Luego la potencia que debe dar el hombre es: Para triángulos semejantes
P' = F ' v' = (ma + μ k mg )(v + at ) d1 L1 L
= d 2 = 2 d1 ⇒
d 2 L2 L1

18
TRABAJO Y ENERGÍA Hugo Medina Guzmán

d2 =
(0,03) (0,04) Así el trabajo de la salida es W2 = F(L).
L(1,50)
Despreciando la fricción.
W1 = W2 o F1(18 L) = F2(L)
= 0,008 m = 8 mm.
Observe que una fuerza pequeña de entrada da lugar a La ventaja mecánica del polipasto es VM = 18.
una fuerza grande de salida, pero el precio que se
paga es que un desplazamiento grande de la entrada Ejemplo 35. Un trailer está equipado de un sistema
produce solamente un desplazamiento pequeño de para sacar barcos del agua. Consiste en una manija
salida. larga de 30 centímetros unido al eje de un engranaje
pequeño con 12 dientes. Este engranaje pequeño
Ejemplo 34. Se bosqueja aquí un polipasto endienta con un engranaje más grande con 36 dientes.
diferenciado de la clase usada para levantar un motor Se une a este engranaje grande un tambor del radio 2
de auto. Las poleas tienen dientes que engranan con centímetros en el cual se enrolla la línea atada al
una cadena continua. Las poleas están soldadas barco (la línea es una cuerda.)
juntas, hay 18 dientes en la polea externa y 16 dientes ¿Qué tensión se puede aplicar a la línea cuando la
en la polea interna. Así cuando la polea hace una manivela se empuja con una fuerza de 80 N?
revolución, 18 acoplamientos de la cadena se So1ución.
levantan y 16 acoplamientos bajan, dando por Considere que pasa cuando la manivela hace una
resultado la elevación de la carga. ¿Cuál es la ventaja revolución. La mano mueve una distancia d1 = 2πR1.
mecánica de esta máquina? El engranaje grande mueve 12/36 = 1/3 revoluciones.
La línea es jalada una distancia d2 = 2πR2/3.

F1 d1 = F2 d 2 ⇒
d 2πR1 R
F2 = 1 F1 = F1 = 3 1 F1
d2 2πR2 / 3 R2
⎛ 30 ⎞
F2 = 3⎜ ⎟(80 ) = 3600 N.
Solución.
Considere qué pasa cuando la polea superior hace una
revolución, es decir, cuando el trabajador jala 18 ⎝ 2⎠
eslabones de la cadena hacia él con fuerza F1. La ventaja mecánica:
Sea L = longitud de un eslabón. 3600
El trabajo de la entrada es W1 = F1(18 L). VM = = 45
80
El lazo de la cadena que va bajo de la carga es
La ventaja mecánica del torno (despreciando la
acortado así por 18 eslabones y alargado por 16
fricción) es 45.
eslabones, con un acortamiento neto de 18L - 16L =
2L que acorta al lazo 2L y levanta la carga L (intente
esto con un pedazo de cuerda para convencerse de
esta característica).

PREGUNTAS Y PROBLEMAS

1 Defina primero en palabras y luego en una


expresión matemática.
a) El trabajo realizado por una fuerza cualquiera.
b) La energía cinética de una partícula.

2 Una partícula P en el plano xy está sometida a la



acción de la fuerza F = y iˆ − x ˆj . Calcular el
2 2

trabajo efectuado por la fuerza para desplazar P sin


Respuesta. W=
ab
(a + b )
3
fricción desde B (0,.b) a A (a, 0). 3. Un depósito cilíndrico de altura H tiene una masa
m de agua que lo llena hasta la mitad, que ha de
bombearse en su totalidad por encima del borde del
mimo. ¿Cuánto trabajo ha de realizar la bomba?

19
TRABAJO Y ENERGÍA Hugo Medina Guzmán

Respuesta. ΔW =
3
mgH cos θ = 1 − x 2 . El
fuerza también varía tal que
4 objeto se mueve entre − 2 < x < 2 .
¿Cuál es el trabajo realizado cuando el objeto se
4. ¿Qué fuerza horizontal, constante debe aplicarse a mueve de x = 0 a x = a?
un carro de masa 500 kg que viaja en una carretera
a2 a4
horizontal a 36 km/h para que se detenga en 30 Respuesta. Aa + B −B
metros? ¿Quién proporciona la fuerza? 2 4
Respuesta. 2500 N, proporcionada por la carretera.
9. Un bloque que se mueve a lo largo del eje x
5. Un resorte está unido en A a un plano vertical fijo comienza del reposo en x = A y se mueve a x = B
y a un bloque B que resbala sobre una varilla lisa luego vuelve a x = A donde queda en reposo
horizontal Ox. La longitud del resorte no estirado es nuevamente. Si una de las fuerzas actuante sobre el
45 cm y la constante del resorte es k = I000 N/m. bloque es opuesta en dirección y proporcional a la
¿Cuál es el trabajo realizado por el resorte sobre B → →
cuando se mueve 60 cm desde O por efecto de la magnitud de la velocidad, tal que Fv = −b v con b
fuerza F? Constante. Demostrar que el trabajo realizado por
esta fuerza no es cero para una trayectoria cerrada.


10. La fuerza F = x yiˆ + xy ˆj actúa sobre la
2 2

partícula .P (x,y) que se mueve en el plano xy.


a) Demostrar que F no es una fuerza conservativa.
b) Determinar el trabajo de F cuando se mueve de A
a C, a lo largo de los caminos ABC, ADC y AC.
Respuesta: 99,38 J

6. Un resorte de masa despreciable y constante k


cuelga del cielorraso de un ascensor y lleva
suspendido una masa in. Cuando el ascensor se
mueve hacia arriba durante t segundos con una
1
aceleración uniforme a= g . la reacción inercial
2 ∂Fx ∂Fy a4
hace que el resorte se alargue. Respuesta. a) Si ≠ , b) WABC = ,
a) ¿Cuánto trabajo realiza el ascensor sobre el sistema ∂y ∂x 3
resorte-masa? a4 a4
b) ¿Cuánto trabajo realiza sobre el resorte? WADC = . WAC =
3 2
1 1 m2 g 2
Respuesta. a) mg 2 t 2 b)
4 8 k 11. El tubo de la figura se halla en un plano
horizontal, su resorte comprimido inicialmente 10
7. En la figura se mueve el cuerpo A a lo largo de un cm.. y al dispararse una bolita entra en una canaleta
plano horizontal liso por medio de la fuerza constante circular de radio R, la fricción es constante igual a 1
F = 5º N aplicada al extremo de una cuerda unida a A Newton. ¿Cuántas vueltas dará la bolita antes de
y que pasa por una pequeña polea sin rozamiento en detenerse?
B. Calcular el trabajo realizado sobre A por la cuerda R= 50 cm k = 62 N/m
mientras A se desplaza 3 m,

Respuesta. W = 120 J
Respuesta. Una vuelta.
8. Una fuerza cuya magnitud varía con x de acuerdo a
F = A + Bx actúa sobre objeto que puede moverse 12. Se aplica una fuerza de 1 N a una partícula de 50
solamente en el eje x. El ángulo con el que actúa la g que está inicialmente en reposo sobre una
superficie.

20
TRABAJO Y ENERGÍA Hugo Medina Guzmán

a) ¿Cuánto trabajo realiza sobre la partícula en l0 s si zy


la superficie es lisa y la fuerza es horizontal? U (r ) = 3 x 2 y + − y2
b) El mismo caso de a) pero la fuerza hace un ángulo x
de 60º con la horizontal. Respuesta.
c) El caso b) pero con rozamiento entre la partícula y →
⎛ z ⎞ ⎛ z⎞ˆ y ˆ
la superficie 0,25 y ¿Cuánto trabajo se consume en F = y⎜ 2 − 6 x ⎟iˆ + ⎜ 2 y − 3x 2 − ⎟j− k
vencer el rozamiento? ⎝x ⎠ ⎝ x⎠ x
Respuesta. a) ΔW = 1000 J , b) ΔW = 2505 J ,
17. Una partícula de masa 4y penetra en una región
c) ΔW = 143 J , ΔW = 46 J en la cual su energía potencial es la indicada en la
figura y pasa valores grandes de x, a los cuales su
13. Encontrar la función energía potencial de un energía potencial es cero, tiene una energía cinética
resorte si el origen se coloca en la pared y la longitud de 16 x 10-7 J .
del resorte sin estirar es L. a) ¿Cuál es su energía cinética en los puntos A, B y
C?
b) Estando en el punto A, la partícula pierde
bruscamente la mitad de su energía total. (la gráfica
de la energía potencial no se altera). Describe
cualitativamente el movimiento subsiguiente, dando
el dominio de valores de x en el cual puede moverse
1 2
Respuesta. U(x) = kx − kLx + C la partícula.
2
1 2
Si C= kL ⇒
2

2
( 1
2
)
U ( x ) = k x 2 − 2 Lx + L2 = k ( x − L )
1 2

14. Una partícula que se mueve a lo largo del eje x


está sometida a la acción de una fuerza en un sistema
conservativo a la que le corresponde la siguiente Respuesta.E A = 8 × 10 -7 J , EB = 12 × 10-7 J ,
función energía potencial. EC = 6 × 10-7 J
U ( x ) = a + bx 2 − cx 4
Determinar los coeficientes a. b y c, si se sabe que el 18. Un bloque de masa m es lanzado hacia arriba en
potencial se anula en el origen, que x = 2 m en una un plano inclinado con una velocidad de magnitud
posición de equilibrio y que una partícula de 5 kg con v0 . El ángulo del plano es θ y el coeficiente de
una velocidad en el origen de 2 m/s queda en reposo
en x = l m. fricción del bloque y el plano es μ . Si el bloque
Respuesta. a = 0, b = 80/7 J/m2 , c = 10/7 J/m4 viaja una distancia L hasta detenerse y comienza a
bajar volviendo a su posición original. Calcular,
15. La energía potencial entre dos moléculas vecinas a) El trabajo realizado por la fuerza normal durante el
viene dada por: movimiento.
A B b) El trabajo realizado por la fuerza de fricción
U (r ) = + durante el movimiento.
r 6 r12 c) El trabajo realizado por la fuerza de gravedad
siendo r la separación entre las moléculas. durante el movimiento.
a) ¿Cuál es la fuerza entre ellas en función de r?
d) Encontrar L en función de v0 , y θ .
b) ¿Cuál es la posición de equilibrio de las dos
moléculas? e) ¿Cuál es la velocidad del bloque cuando vuelve al
c) ¿Qué energía seria necesaria para alejarlas de su punto inicial?
posición de equilibrio indefinidamente? Respuesta. a) 0, b) − 2 μmgL cosθ , c) 0,
A B v02
Respuesta. a) F(r ) = −6 + 12 13 , b) d) L = ,
2 g (μ cosθ + senθ )
7
r r
1
⎛ 2B ⎞ 6 A2 e) v = v02 − 4 μgL cosθ
r =⎜ ⎟ , c) ΔE =
⎝ A ⎠ 4B
19. Se dispara un proyectil con una velocidad inicial
de magnitud v0 y formando un ángulo θ con la
16. Hallar la fuerza conservativa que da origen a la
función energía potencial.

21
TRABAJO Y ENERGÍA Hugo Medina Guzmán

horizontal. Usando la conservación de la energía


encontrar.
a) La altura máxima alcanzada.
b) La magnitud de la velocidad cuando el proyectil
está a la mitad de su máxima altura.
v02sen 2θ
Respuesta. a) h = ,
2g Respuesta. a) 0,5 mg , b) 0.3 mg , c) 2,5 mg d) 1,1

(1 + cos θ )
mg , e) 5,4 mg
2
b) v = v0 24. Un bloque pequeño de masa m resbala partiendo
2 de la parte superior de una esfera sin fricción de radio
R. ¿Cuál es el ángulo en el que el bloque pierde
20. Una fuerza F = 8t (t en segundos, F en Newton), contacto con la esfera.
actúa la partícula P de masa m = 4kg durante un
tiempo t = 6 s.
Sí parte del reposo a partir del origen.
a) Calcular el trabajo efectuado.
b) Calcular la energía cinética al instante t.
Respuesta. a) W = 2592 J, b) K = 2t4 J.

21. Un resorte de longitud l y constante k se sujeta a


un bloque de masa m y al piso. Si el bloque se levanta
a una altura 3 l y soltado desde el reposo.
2
a) ¿Cuál será la velocidad del bloque cuando esté a Respuesta. a) cos θ =
una altura 2 l ? 3
b) ¿Cuál será la máxima compresión del resorte?
25. 1n saco se empuja suavemente por el borde de
k 2
Respuesta. a) v = 3 l + 2 gl , una pared en A y oscila en un plano vertical colgado
m del extremo de una cuerda de 4m que puede soportar
b) una tensión máxima igual a dos veces el peso del

( )
saco.
kl − mg ⎛ kl − mg ⎞
2
3kl 2 + 6mgl a) Determinar la altura a la que se rompe la cuerda.
y= + ⎜ ⎟ + b) ¿A qué distancia de la pared vertical caerá al .suelo
k ⎝ k ⎠ k
el saco?
22. Dos placas cuyas masas son m1 y m2,
respectivamente, están conectadas por un resorte.
¿Qué fuerza deberá aplicarse a la placa superior para
elevar la placa inferior después que se retira la
presión? No tomar en cuenta la masa del resorte.

Respuesta. a) y = 1,33 m
Respuesta. a) F > (m1 + m 2 )g
26. Una bola pequeña de masa m = l g desliza hacia el
23. Una bolita de masa m desliza a partir del reposo fondo de un valle moviéndose sin rozamiento como
hacia abajo por un carril doblado como se muestra en se indica en la figura. Partiendo del reposo, la bola
la figura, el rozamiento es despreciable, hallar: cae desde una altura h = 2m y abandona el fondo del
a) La reacción normal del carril en A. valle formando un ángulo θ con la horizontal. En el
b) La energía cinética de la bolita en B. punto más elevado de su trayectoria la bola choca con
c) La reacción normal del carril en 8. un resorte montado sobre una pared y lo comprime 2
d) La energía cinética de la bolita en C. cm. La constante del resorte es k = 49 N/m.
e) La reacción normal del carril en C. a) ¿A qué altura y está el resorte? b) ¿Cual es el
ángulo θ ?

22
TRABAJO Y ENERGÍA Hugo Medina Guzmán

a) Considerando que la fuerza de rozamiento es


independiente de la velocidad, calcule su valor
medio.
b) ¿Qué potencia debe consumirse para mantener el
automóvil en movimiento con una velocidad de 48
km/h?
Respuesta. a) Ff = 110 N b), P = 2 hp

Respuesta. a) y =1 m, b) θ = 45º 33. Un automóvil de 1500 kg se desplaza 200 m


mientras es acelerado uniformemente desde 50 hasta
73 km/h. Durante todo el movimiento el automóvil se
27. Una bola de acero de masa 1 kg está unida a un desplaza sobre una carretera horizontal, y la
extremo de un alambre de 1m de largo y gira resistencia al movimiento es igual al 2 por ciento del
alrededor del otro extremo con una velocidad angular peso del automóvil. Determinar:
de 120 rpm. ¿Cuál es la energía cinética de la bola? a) La máxima potencia requerida.
Respuesta. 78,88 J b) La potencia requerida para mantener la velocidad
constante de 75 km/h.
28. La faja transportadora de la figura se mueve con Respuesta. a) 25 kW , b) 6,13 kW
una velocidad constante v 0 y descarga los paquetes
34. Un peso D y el contrapeso C tienen cada uno una
sobre la rampa AB. El coeficiente de rozamiento masa de 350 kg. Determinar la potencia requerida
entre los paquetes y la rampa es 0,30. Sabiendo que cuando el peso:
los paquetes deben alcanzar el punto B con una a) Se mueve hacia arriba con velocidad constante de
velocidad de 4 m/s, determinar la velocidad v 0 4m/s.
requerida en la faja transportadora. b) Tiene una velocidad instantánea de 4m/s hacia
arriba y una aceleración hacia arriba de 0,9 rn/s2.

Respuesta. 3,02 m/s

29. Una locomotora ejerce un tiro constante en la


barra de tracción de 160000 N mientras aumenta la
velocidad de 48 a 72 km/h. ¿Cuál es la potencia que
desarrolla la locomotora:
a) al comienzo del periodo?
b) al final del periodo?
c) ¿Cuáles la potencia .media durante el periodo?
Respuesta. a) 2859 hp , b) 4290 hp c) 3574 hp
Respuesta. 6,86 kW , 8.44 kW
30. Una grúa industrial puede levantar su máxima
permitida de 25 toneladas a la velocidad de 20mm/s. 35. Un bloque de 0,50 kilogramos es sujetado contra
Sabiendo que la grúa es movida por un motor de 10 el resorte por una fuerza externa horizontal de 36 N.
kW. Determinar su rendimiento. Se quita la fuerza externa, y el bloque se proyecta con
Respuesta. 49% una velocidad v1 = 1,2 m/s a partir de la separación
del resorte. El bloque desciende una rampa y tiene
31. ¿Cuál es la velocidad máxima la que un motor una velocidad v2 = 1,8 m/s en la base. La pista es sin
capaz de suministrar 10 kW puede elevar un ascensor fricción entre los puntos A y B. El bloque ingresa a
de masa 500kg, sin tomar en cuenta las fuerzas de una sección rugosa en B, extendiendo hasta E. El
rozamiento? coeficiente de fricción cinética es 0,30. La velocidad
Respuesta. v = 2,0 m/s del bloque es v3 = 1,4 m/s en C. El bloque se mueve
hasta C donde se detiene.
32. Si a un automóvil de masa 1000 kg que se mueve
sobre una carretera horizontal con una velocidad de
48 km/h se le apaga al motor, este recorre aún 0,8 km
antes de detenerse. carga

23
TRABAJO Y ENERGÍA Hugo Medina Guzmán

38. Un cajón de 100 kilogramos está en una


superficie áspera inclinada 30º. Una fuerza externa
constante P de 800 N se aplica horizontalmente al
cajón. La fuerza empuja el cajón una distancia de 3,0
m arriba de la pendiente, en un intervalo del tiempo
de 2,0 s, y la velocidad cambia de v1 = 0,8 m/s a
v2 = 2,2 m/s.
a) La constante del resorte es:
b) La compresión inicial del resorte en cm es:
c) La altura h de la rampa en cm es:
d) El trabajo realizado por la fricción entre los puntos
B y C es:
e) La distancia s que el bloque viaja entre los puntos
B y D es: a) El trabajo realizado por el peso es:
Respuesta. b) El trabajo realizado por la fuerza de fricción es:
a) 1800 N/m, b) 2,0, c) 9, d) -0.32 J e) 0,55 m c) El trabajo realizado por la fuerza normal es:
d) La potencia media producida por la fuerza externa
36. Una barra sin masa de 1,5 m se fija libremente a P durante los 2,0 segundos es:
un pivote sin fricción en O. Una bola de 3,0 Respuesta.
kilogramos se une al otro extremo de la barra. La a) -1500 J, b) - 400 J c) Cero , d) 1050 W
bola se sostiene en A, donde la barra hace un ángulo
30º sobre el horizontal, y se lanza. El montaje de la 39. Una muchacha lanza una piedra de un puente.
bola-barra puede girar libremente en un círculo Considere las maneras siguientes que ella puede
vertical entre A y B lanzar la piedra. La velocidad de la piedra con la que
lanza es igual en cada caso.
Caso A: Lanzada derecho para arriba.
Caso B: Lanzada derecho para abajo.
Caso C: Lanzada con ángulo de 45º sobre horizontal.
Caso D: lanzada horizontalmente.
¿En qué caso la velocidad de la piedra será mayor
cuando llega al agua?
Respuesta. la rapidez es la misma en todos los
casos.

40. Para hacer el trabajo sobre un objeto,


A) es necesario que haya fricción.
a) La bola pasa a través de C, donde la barra forma un B) es necesario que no haya fricción.
ángulo de 30º debajo de la horizontal. La rapidez de C) el objeto debe moverse.
la bola cuando pasa por C es: D) la fuerza que hace el trabajo debe estar dirigida
b) la tensión en la barra cuando la bola pasa por el perpendicularmente al movimiento del objeto.
punto más bajo D es: E) la fuerza aplicada debe ser mayor que la fuerza de
Respuesta. a) 5,4 m/ s, b) 120 N la reacción del objeto.
Respuesta. C) el objeto debe moverse.
37. Una fuerza externa constante P =120 N se aplica
a una caja de 20 kilogramos, que está en una 41. Un bloque de 8,0 kilogramos se lanza del reposo,
superficie horizontal áspera. La fuerza empuja la caja vl = 0 m/s, en una pendiente rugosa. El bloque se
una distancia de 8,0 m, en un intervalo del tiempo de mueve una distancia de 1,6 m abajo de la pendiente,
4,0 s, y la velocidad cambia de v1 = 0,5 m/s a v2 = 3,5 en un tiempo de 0,80 s, y adquiere una velocidad de
m/s. v2 = 4,0 m/s.

a) El trabajo realizado por la fuerza externa es:


b) El trabajo realizado por la fricción es: a)) El trabajo realizado por el peso es:
c) La razón de cambio promedio de la energía b) La razón promedio a la cual la fuerza de fricción
cinética de la caja, en los 4,0 segundos es: realiza trabajo en el intervalo de tiempo de 0,80 s es:
Respuesta. a) 830 J, b) -700 J, c) 30W

24
TRABAJO Y ENERGÍA Hugo Medina Guzmán

c) La razón promedio a la cual la fuerza normal


realiza trabajo en el intervalo de tiempo de 0,80 s es:
d) La razón promedio a la cual el bloque gana energía
cinética durante el intervalo de tiempo de 0,80 s es:
Respuesta.
a)) + 80 J, b) - 20 W, c) Cero, d) 80 W

42. Una persona de 60 kilogramo cae desde el


reposo uno distancia 1,20 m sobre una plataforma de
masa insignificante apoyada sobre un resorte duro.
La plataforma baja 6 cm antes de que persona vuelva a) El resorte 2 estira 0,06 m. La constante de fuerza
al reposo. ¿Cuál es la constante del resorte? del resorte 2 es:
Respuesta. 4,12 x 105 N/m b) La constante de fuerza del resorte 1 es igual a 30
N/ m. La longitud sin estirar del resorte 1 es:
43. Un objeto está sujeto a una fuerza restauradora F Respuesta. a) 20 N/m, b) 0,53 m
= 6x3, donde x es el desplazamiento del objeto desde
su posición de equilibrio. ¿Qué trabajo debe 46. Una barra ligera de 0,80 m se fija libremente a un
realizarse para mover al objeto desde x = 0 x = 0,15 eje vertical en A. Un disco de 2,0 kilogramos se une a
m? la barra en B. Un resorte se une a la masa en B y a la
Respuesta. 7,59 x 10-4 J manga en el eje en C. A La manga es sin fricción,
permitiendo que se baje y suba libremente, de modo
44. Dos resortes idénticos tienen longitudes sin que el resorte sea siempre horizontal cuando esté
estirar de 0,25 m y las constantes de la fuerza de 200 estirado. La longitud del resorte sin estirar es 0,45 m
N/m. Los resortes se unen a un bloque pequeño y se y la constante es 210 N/m.
estiran a una longitud de 0,30 m como en la figura A.
Una fuerza externa P tira del bloque 0,02 m a la
derecha y lo sostiene allí. (Véase La Figura B)

a) El eje está girando y el resorte estirado tiene una


longitud de 0,48 m. La aceleración radial del disco es:
a) El trabajo requerido para ensamblar los resortes y b) El eje está girando y la varilla forma un ángulo de
el bloque (figura A) es : 40º con el eje. El resorte está estirado y horizontal.
b) La fuerza externa P, que mantiene al bloque en su La aceleración radial del disco es:
lugar (figura B) es: c) El eje está girando y el resorte tiene una longitud
c) El trabajo realizado por la fuerza externa P en jalar de 0,45 m. La aceleración radial del disco es:
el bloque 0,02 m es: Respuesta.
Respuesta. a) 10,5 m/s2 b) 15,0 m/s2, c) 6,7 m/ s2
a) 0,50 J, b) E) 8 N, c) 0,08 W
47. Cierto coche que viaja 20 resbalones del mph a
45. El bloque A (0,40 kg) y el bloque B (0,30 kg) una parada en 20 metros del punto donde los frenos
están sobre una mesa sin fricción. El resorte 1 conecta fueron aplicados. ¿En qué distancia el coche pararía
al bloque A a una varilla sin frición O y el resorte 2 aproximadamente la tenía que va 40 mph?
conecta el bloque Ay el bloque B. Los bloques están Respuesta. 80 metros
en movimiento circular uniforme alrededor de o, y los
resortes tienen longitudes de 0,60 m y 0,40 m, como 48. Un motor de la arena en una mina levanta 2.000
se muestra. La velocidad lineal del bloque B es 2.0 kilogramos de la arena por minuto una distancia
m/s. vertical de 12 metros. La arena está inicialmente en
el resto y se descarga en la tapa del motor de la arena
con la velocidad 5 m/s en un canal inclinado de
cargamento. ¿En qué tarifa mínima se debe la
energía proveer a esta máquina?
Respuesta. 4,34 kW

49. La constante de un resorte es 500 N/m y su


longitud sin estirar es 0,60 m. Un bloque de 4,0
kilogramos se suspende del resorte. Una fuerza

25
TRABAJO Y ENERGÍA Hugo Medina Guzmán

externa tira hacia abajo lentamente el bloque, hasta


que el resorte se ha estirado a una longitud de 0,72 m.
se quita y el bloque sube.
a) La fuerza externa sobre el bloque es:
b) Cuando el resorte se ha contraído una longitud de
0.60 m, la velocidad del bloque hacia arriba es: :
c) Cuando el resorte se ha contraído una longitud de
0.66 m, la aceleración del bloque incluyendo su
dirección es:
Respuesta. a) 20 N, b) 0,4 m/ s, c) 2 m/s2, hacía a) La compresión del resorte en la figura b es:
abajo b) La fuerza externa P en la figura c es:
c) La energía potencial elástica del resorte en la
50. la constante de un resorte es 200 N/m y su figura c es:
longitud sin estirar es 10 centímetros. El resorte se d) La aceleración inicial del disco cuando la fuerza
pone dentro de un tubo liso de 10 centímetros de alto externa es removida es:
(la figura a). Un disco de 0,40 kilogramos se coloca e) La velocidad v del disco cuando emerge del tubo
sobre el resorte (figura b). Una fuerza externa P en la figura d es:
empuja el disco hacia abajo, hasta que el resorte tiene Respuesta. a) 2,0 , b) 8N, c) 0,36 J, d) 20 m/s2,
4 centímetros de largo (la figura c). Se quita la fuerza e) 0,80m/s
externa, el disco se proyecta hacia arriba y emerge del
tubo (figura d).

26
Sistema de partículas Hugo Medina Guzmán

CAPÍTULO 6. SISTEMA DE PARTÍCULAS

La selección del contorno de un sistema es similar a


INTRODUCCIÓN seleccionar un sistema de coordenadas.

Hasta ahora hemos estado estudiando el movimiento SEGUNDA LEY DE NEWTON APLICADA A
de los objetos cualquiera que sea sin considerar su UN SISTEMA DE PARTICULAS
estructura. Ahora demostraremos que lo estuvimos
haciendo bien considerando al objeto sin tomar en La figura siguiente muestra un sistema de n partículas
cuenta las fuerzas que actúan sobre sus partes. de masas m1, m2, …..mn, con posiciones especificadas
Introduciremos el concepto de centro de masa de un → → →
sistema de partículas, también se introducirá el por r 1 , r 2 , …………. r n ,, respectivamente.
concepto de cantidad de movimiento y se demostrará
que este se conserva cuando el sistema se encuentra
aislado de los alrededores,

SISTEMA DE PARTICULAS

La segunda ley de Newton para la partícula mi es:


→ → → →
F i = m a i = F iexter + F i int

Donde:


La figura muestra un sistema de partículas compuesto F i int = suma de las fuerzas internas sobre mi
de tres masas. En el sistema existen dos tipos de
fuerzas,

F i ext = suma de las fuerzas externas sobre mi
a) Las fuerzas externas como la atracción
gravitacional de la tierra por ejemplo.
La suma de las fuerzas internas sobre la masa mi es:
b) Las fuerzas internas que las partículas ejercen unas
→ → → → n →
sobre otras (estas fuerzas pueden ser gravitacionales,
F 1int = F 12 + F 13 + ............ F 12 = ∑ Fij
e1éctricas, etc.)
( j ≠i )

En general para la partícula i es:

→ n →
F i int = ∑ Fij
( j ≠i )

La fuerza total para el sistema es:

i =n → i =n → i =n → n n →
∑ Fi = ∑ m a i = ∑ F i ext + ∑∑ Fij
i =1 i =1 i =1 i =1 ( j ≠i )

En la figura hemos cambiado el contorno del sistema, Por la tercera ley de Newton cada una de las fuerzas
excluyendo la masa m3. Como Una Consecuencia de
esto las fuerzas internas Sobre m1 y m2 debido a m3 ya
→ →
no son internas, se han sumado a las fuerzas externas
Fij tiene un F ji igual, pero de sentido contrario
previas, produciendo una nueva fuerza resultante.

1
Sistema de partículas Hugo Medina Guzmán

→ → n
1 1
Fij = − F ji xCM = lim
Δmi →0 M
∑ x Δm
i =1
i i =
M∫
xdm

n n →
De modo que ∑ ( ∑) F
i = 0 j ≠ i =1
ij =0 De igual forma se obtiene:

1 n 1
Consecuentemente solo queda y CM = lim
Δmi →0 M
∑i =1
y i Δmi =
M∫
ydm ,

1 n 1
n →
∑ mi ai = ∑ Fi ext o
n →
d2 n
∑ ii ∑
m

r =
n
F

i ext
z CM = lim
Δmi →0 M

i =1
z i Δmi =
M∫
zdm y
i =1 i =1 dt 2 i =1 i =1 →
1 →
M∫
rCM = r dm
CENTRO DE MASA

Frecuentemente es muy práctico reemplazar un MOVIMIENTO DEL CENTRO DE MASA.


sistema de muchas partículas con una partícula
simple equivalente de masa igual. La pregunta es d2 n → n →

2 ∑ i i ∑
donde colocar esta partícula simple con respecto al Si en la ecuación: m r = Fiext
origen de x e y. dt i = 0 i =1

Definamos el vector posición del centro de masa por n → →


la ecuación: Sustituimos ∑m
i =1
i ri = M rCM
n →

→ ∑m i ri Obtendremos la ecuación del movimiento del centro


rCM = i =1
n
de masa

∑m i
d2 → n → → n →
i =1

dt 2
M rCM = ∑
i =1
Fiext ⇒ M a CM = ∑
i =1
Fiext

n
Llamando a ∑m
i =1
i = M (masa total de las n →
El punto indicado por rCM , vector posición del
partículas).
centro de masa, se mueve se mueve como si en el
estuviera concentrada toda la masa y las fuerzas
n →

→ ∑m i ri externas del sistema.

rCM = i =1
Ejemplo 1. Centro de masa de tres masas
M
puntuales.

Como rCM = xCM iˆ + y CM ˆj + z CM kˆ

n
1
Tenemos que: xCM =
M
∑m x ,
i =1
i i

n n
1 1
yCM =
M
∑ mi yi , zCM =
i =1 M
∑m z
i =1
i i

El centro de masa esta dado por:


Si hacemos que el número de elementos n, se
aproximen al infinito, la sumatoria se reemplaza por n
1
una integral y m por el elemento diferencial dm. xCM =
M
∑m x
i =1
i i

Luego.
m(1) + 2m(1) + 3m(2)
=
m + 2m + 3m
2
Sistema de partículas Hugo Medina Guzmán

9m 3 Sustituyendo:
= =
x (a − x )dx = 2 ∫ x(a − x )dx
2 a b 2 a
6m 2 xCM =
ab ∫0 a a 0
a
⎡ x2 x3 ⎤
∫( )
n
1
∑ mi yi 2 2
a
yCM = = 2 ax − x dx = 2
2
⎢a + ⎥
M i =1 a 0 a ⎣ 2 3 ⎦0

m(1) + 2m(3) + 3m(2 ) ⎛ a3 a3 ⎞ a


= 2
m + 2m + 3m = ⎜⎜ + ⎟⎟ =
a2 ⎝ 2 3 ⎠ 3
13m 13
= = Realizando cálculos similares encontramos:
6m 6 b
y CM =
→ 3
3 ˆ 13 ˆ
rCM = i+ j
2 6 → aˆ b ˆ
Finalmente: rCM = i+ j
Ejemplo 2. Centro de masa de un triángulo. 3 3

Ejemplo 3. Centro de masa de un arco


semicircular.

1
xCM =
M ∫ xdm
Por el sistema de coordenadas escogido, xCM = 0 ,
Para evaluar porque por cada elemento de masa a la derecha (+),
existe otro elemento igual a la izquierda (-). Sin
masa total embargo para y CM es diferente.
dm = × área de la lámina
área total
1
M 2M
y CM =
M ∫ ydm , en este caso dm = λdl
= ydx = ydx
1 ab
ab M
2 Donde λ= y dl = Rdθ
πR
1 1 ⎛ 2M ⎞
Luego: xCM =
M ∫ xdm = M ∫ x⎜⎝ ⎟ ydx
ab ⎠
Como y = Rsenθ , tenemos:

π λR 2 π
∫0 (Rsenθ )λRdθ
1
2 a y CM = =
∫ senθdθ
ab ∫0
= xydx M M 0

λR 2 R2 ⎛ M ⎞
Para poder integrar tenemos que expresar la variable = [− cosθ ]0 π
= ⎜ ⎟(2 )
y en función de x. M M ⎝ πR ⎠
Por semejanza de triángulos:
2R
= = 0,64 R
y a−x π
⇒ y = (a − x )
b
=
b a a

3
Sistema de partículas Hugo Medina Guzmán

El centro de masa no se encuentra dentro del cuerpo.


Las figuras siguientes muestran como localizar
experimentalmente el centro de masa primero
colgándolo de la parte superior y luego de otro punto
cualquiera.

Después que P1 y P2 se separan, las velocidades


→ → → →
respectivas son v '1 y v ' 2 diferentes de v1 , y v 2 .

Ejemplo 4. Explosión de una granada

Ahora nos preguntamos. ¿Qué pasa durante el choque?

El tiempo de contacto total Δt es muy pequeño,


quizás solo de aproximadamente 0,001 segundos. La
fuerza de contacto inicialmente es cero, aumenta
Una granada lanzada al aire que explota en varios hasta un valor muy grande y. finalmente disminuye
fragmentos. La única fuerza externa sobre la granada hasta cero, cuando dejan de estar en contacto. La
es la fuerza de la gravedad, entonces la granada sigue figura siguiente muestra una variación típica de la
una trayectoria parabólica. Si la granada no estallara fuerza en el tiempo de contacto.
continuaría moviéndose a lo largo de la trayectoria
parabó1ica indicada en la figura. Como las fuerzas de
la explosión son internas, no afectan al movimiento
del centro de masa. Entonces. Después de La
explosión el centro de masa de los fragmentos sigue
la misma trayectoria que tendría la granada s! no
hubiera habido explosión.

IMPULSO Y CANTIDAD DE MOVIMIENTO

Supongamos el caso de dos partículas esféricas P1 y


Sea t f − t i = Δt el tiempo que dura el choque,
P2 de masas m1 y m2 con trayectorias contenidas en la
misma recta, se aproximan una a otra con velocidades aplicando la segunda ley de Newton a las partículas
→ → P1 y P2.
v1 , y v 2 respectivamente.

→ → dv
F12 = m1 a1 = m1 1 y
dt

→ → d v2
F21 = m2 a 2 = m2
dt

→ → → →
Cuando P1 y P2 entran en contacto, P1 ejerce sobre P2
la fuerza F12 y P2 ejerce sobre P1 la fuerza F21. De O F12 dt = m1 d v1 y F21 dt = m2 d v 2
→ →
acuerdo con la tercera ley de Newton F12 = − F21 . Integrando las dos relaciones durante el choque,

4
Sistema de partículas Hugo Medina Guzmán

tf → v '1 → La partícula P1 ha sufrido en el intervalo


∫ti
F12 dt = m1 ∫ d v1 y
v1 t f − t i = Δt , un cambio de la cantidad de
→ →
tf v '2 →
⎛ → →⎞ → →
∫ F21 dt = m2 ∫ d v2
tf

ti v2
movimiento ∫ ti
F12 dt = m1 ⎜ v'1 − v1 ⎟ = p f − pi
⎝ ⎠
Finalmente →
y esta cantidad es también igual al impulso J
tf →
⎛ →
⎞ → recibido en ese instante por la partícula
∫ti
F12 dt = m1 ⎜ v'1 − v1 ⎟ y
⎝ ⎠ →
→ →
tf →
⎛ → →

∫ti
F21 dt = m2 ⎜ v' 2 − v 2 ⎟
⎝ ⎠
J = p f − pi

Luego: “El cambio de la cantidad de movimiento


Trabajando con el primer miembro es igual al impulso”.

tf
∫ Fdt corresponde al área bajo la curva mostrada Ejemplo 5. Una pelota de 100 gramos está en reposo
ti sobre el piso, cuando recibe un puntapié que la lanza
en la figura anterior, a ésta cantidad la llamaremos con una velocidad de 30 m/s.

a) ¿Qué impulso se dio a la pelota?


⎛→⎞
IMPULSO ⎜ J ⎟
⎝ ⎠ b) Si el tiempo que el pie está en contacto con la
pelota es 10-3 segundos. ¿Cuál es la magnitud
→ tf → aproximada de la fuerza impulsiva?
J = ∫ F(t ) dt
ti
Solución.
Sus dimensiones son: [F] [T] = [M][L][T]-1
a) El impulso es igual al cambio de la cantidad de
movimiento:
En el sistema internacional sus unidades son:
→ → → → →
Newton.segundo (N.s) J = p f − p i = m v f − m vi

Trabajando con el segundo miembro


En este caso

⎛ → →⎞ ⎛ → →⎞ → →
m1 ⎜ v'1 − v1 ⎟ y m2 ⎜ v' 2 − v 2 ⎟ m = 0,1 kg, vi = 0 , v f = 30iˆ m/s
⎝ ⎠ ⎝ ⎠

→ → →
J = (0,1)(30&i&) − 0 = 3iˆ
kg m
Llamaremos a la cantidad mv = p,
s
CANTIDAD DE MOVIMIENTO LINEAL o b) Se puede obtener un estimado de la fuerza que
Momentum lineal de la partícula (lo designaremos →
en la práctica simplemente como cantidad de actúa sobre la pelota, dividiendo e1 impulso J por el
movimiento), cuyas dimensiones son:
tiempo Δt = t f − t i en que actúa la fuerza :

[M] [[L]] = [M] [L] [T]-1 → J
F=
En el sistema internacional sus unidades son: Δt

kg.m.s-1 → kg m
Como J = 3iˆ y Δt = 0,001 s
s

5
Sistema de partículas Hugo Medina Guzmán

→ 3iˆ Usando la expresión de centro de masa


F= = 3000iˆ N
0,001 n → →
∑m v
i =1
i i = M v CMi
Ejemplo 6. Se deja caer una pelota de masa m de una
altura h sobre el nivel del suelo y rebota hasta una
altura h1 → n → →
De aquí p total = ∑ mi vi = M v CMi
i =1
a) ¿Cuál es la velocidad vi inmediatamente antes de
chocar con el suelo?
La cantidad de movimiento total de un sistema es
igual a la cantidad de movimiento de la masa total
b) ¿Cuál es la velocidad v f inmediatamente después concentrada en el centro de masa del sistema.
de chocar con el suelo?
Derivando nuevamente la expresión anterior:

c) ¿Cuál es el impulso J que se le da a la pelota en d → d → → →
el impacto con el suelo? p total = M v i CM = M a iCM = F i ext
dt dt
Solución.
Esta cantidad es muy importante, ya que si no hay
fuerza externa,

a) Como v 0 = 0 , x = 0 , y = h0
→ d →
F i ext = 0 ⇒ p total = 0
→ dt
vi = 2 gh0 ˆj

⇒ p total = CONSTANTE
b) Como después de chocar y = h1 , la velocidad

v f después de chocar es: Esto es la conservación de la cantidad de
movimiento. Si no hay fuerzas externas sobre un
sistema. La cantidad de movimiento total del
→ sistema es constante.
v f = 2 gh1 ˆj
Ejemplo 7. Tres partículas de masas 2 kg, 1 kg y 3 kg
c) El impulso de la pelota es: respectivamente con vectores posición

→ → →
J = m v f − m vi = m 2 g ( )
h1 − h0 ˆj

[ ]
r1 = 5tiˆ − 5t 2 ˆj + (3t − 2)kˆ cm ,
r = [(2t − 3)iˆ − (12 − 5t ) ˆj + (4 + 6t − 3t )kˆ ]cm

2 3
2

y r = [(12t − 1)iˆ − (t + 2 ) ˆj − t kˆ ]cm


CONSERVACIÓN DE LA CANTIDAD DE

MOVIMIENTO 2 3
3

La cantidad de movimiento de una partícula de masa


→ Donde t es el tiempo en segundos.
m y velocidad v es:
Encontrar: a) La velocidad del centro de masa en t =
→ → 1 s y t = 2 s.
p = mv
b) La cantidad de movimiento lineal total del sistema
La cantidad de movimiento de n partículas es la suma en t = 1 s y t = 2 s.
de las cantidades de movimiento individuales,
c) Analizar si el sistema de tres partículas es sistema
→ n → n → aislado
p total = ∑ p i = ∑ mi vi
i =1 i =1 Solución.

6
Sistema de partículas Hugo Medina Guzmán

a) La posición del centro de masa esta dada por la ayudante ahogándose se encontrará el pescador
expresión: cuando alcance el extremo del bote?

→ → → Solución.
→ m r + m2 r2 + m3 r3
rCM = 1 1
m1 + m2 + m3 Consideremos aislado el sistema bote, pescador,
ayudante, por lo tanto su cantidad de movimiento es
constante.
Reemplazando valores, obtenemos:

[ ( ) (
rCM = (3t − 1)iˆ + − t 2 + 3 ˆj + − t 3 + 2t kˆ cm )] → →
p = M v cm = CONSTANTE
La velocidad del centro de masa es
Como en inicio el sistema está en reposo:

vCM =
d →
dt
[ (
rCM = 3iˆ − 2tˆj + − 3t 2 + 2 kˆ
cm
s
)] → →
p = 0 ⇒ v cm = 0

Para t = l s →
→ d r cm
= =0
[ ]

Como v cm
cm dt
v1M = 3iˆ − 2 ˆj + − kˆ →
s
r cm = CONSTANTE , la posición del centro de
masa permanece constante
Para t = 2 s
En éste problema que es en una sola dimensión:

v2 M [
= 3iˆ − 4 ˆj + −8kˆ
cm
s
] xcm = CONSTANTE

Tomemos como punto de referencia la posición del


b) La cantidad de movimiento del sistema es: ayudante en el extremo del bote, al soltarse seguirá en
la misma posición.
→ → → → →
p = + m1 v1 + m2 v 2 + m3 v3 = M vCM Analicemos la posición inicial.


[ (
p = 6 3iˆ − 2tˆj + − 3t 2 + 2 kˆ
kg cm
s
)]
Para t = l s


[
p1 = 6 3iˆ − 2 ˆj − kˆ
kg cm
s
]
El centro de masa del sistema pescador-bote está en:

mb xb + m p (2,5)
Para t = 2 s
xcm =

[
p2 = 6 3iˆ − 4 ˆj − 8kˆ
kg cm
s
] mb + m p

Analicemos la posición final.


→ → →
c) Como , p1 ≠ p 2 , p no es constante, luego el
sistema no es aislado.

Ejemplo 8. Un pescador de masa 70 kg está en un


bote estacionario de masa 200 kg, cuando su
ayudante que no sabe nadar y está en el agua cogido
del extremo opuesto, se suelta. El pescador corre 2,5
m hasta alcanzar este extremo. ¿A que distancia del
El centro de masa esta en:

7
Sistema de partículas Hugo Medina Guzmán

mb ( x b + x ) + m p x EI muchacho sale con una velocidad de módulo


xcm =
mb + m p
m2
v1 = v dirigida hacia el oeste,
m1
Como la posición del centro de masa del sistema es
invariante, se tiene:
Ejemplo 10. Dos personas de masa m cada una, se
mb xb + m p (2,5) mb ( xb + x ) + m p x encuentran paradas en los extremos opuestos de un
= bote de longitud d y masa 3m que se encuentra en
mb + m p mb + m p reposo sobre un líquido sin fricción, tal como se
muestra en la figura. Las personas caminan una hacia
⇒ (mb + m p )x = m p (2,5) la otra con rapidez constante y se encuentran a d/4 del
extremo izquierdo del bote.
a) Si la persona de la izquierda se mueve con
Reemplazando valores:
velocidad v 0 respecto al bote, ¿cuál es la velocidad

70(2,5) que tiene la otra persona, respecto al bote?


x= = 0,65m b) ¿Cuál es la velocidad del bote, respecto a tierra,
(200 + 70) durante el movimiento de ambas personas?
c) ¿Cuánto avanzo el bote hasta el momento del
La posición del pescador estará a 0,65 metros del encuentro?
ayudante.

Ejemplo 9. Un muchacho de masa m1 y una


muchacha de masa m2 , ambos con patines, se
encuentran en reposo uno en frente del otro, El
muchacho empuja a la muchacha, mandándola hacia
→ Solución.
el este con una velocidad v . Describa el movimiento
del muchacho.

Solución.

Siendo un sistema cerrado la cantidad de movimiento


se conserva, a)
El tiempo empleado para encontrarse es el mismo
→ → para las dos personas
p antes = p después = 0 , d 3d
4= 4 ⇒ v1 = 3v 0 Hacia la izquierda
→ →
v0 v1
Si v 1 y v 2 son las velocidades del muchacho y la b) Por conservación de la cantidad de movimiento
muchacha después del empujón, respectivamente: → →
p antes = p después
→ → →
m1 v 1 + m2 v 2 = 0 p antes = 0

Considerando el movimiento en el eje x, y la p después = m(v 0 + vb )iˆ + m(− 3v0 + vb )iˆ + 3mvb iˆ = 0
dirección al este como sentido positivo
→ 2 ˆ
→ → ⇒ vb = v0 i
v 2 = v = viˆ 5
c)
De aquí d
El tiempo de caminata de las personas es t = ,
4v0
→ → m2
m1 v 1 + m2 viˆ = 0 ⇒ v 1 = − viˆ luego el bote se habrá movido
m1

8
Sistema de partículas Hugo Medina Guzmán

→ →
⎛ 2 ⎞⎛ d ⎞ d → m v + m2 v 2
x = vb t = ⎜ v0 ⎟⎜⎜ ⎟⎟ = Como vCM = 1 1
⎝ 5 ⎠⎝ 4v 0 ⎠ 10 m1 + m2

SISTEMA DE REFERENCIA CENTRO DE → →


MASA → →m v + m2 v 2
u1 = v1 − 1 1
Cuando la fuerza externa resultante que actúa sobre
m1 + m2
un sistema es cero, la cantidad de movimiento total es
constante. Muchas veces es conveniente escoger un m2 ⎛→ →⎞
sistema de coordenada., con el origen situado en el = ⎜ v1 − v 2 ⎟
centro de masa. Este sistema se denomina m1 + m2 ⎝ ⎠
“SISTEMA DE REFERENCIA CENTRO DE
MASA” → →
→ m v + m2 v2

y u 2 = v2 − 1 1
Con respecto a este sistema la velocidad del centro de m1 + m2
masa por supuesto es cero y la cantidad de
movimiento total es cero.
m1 ⎛→ →⎞
= − ⎜ v1 − v 2 ⎟
El análisis de la mayor parte de los choques es más
m1 + m2 ⎝ ⎠
sencillo en el sistema de referencia centro de masa.

La transformación de un sistema de referencia Como comprobación, calculemos la cantidad de


cualquiera a un sistema centro de masa no es difícil. movimiento total con respecto al centro de masa, el
Consideremos un sistema do dos partículas m1 y m2 cual debe ser igual a cero.
→ →
con velocidades v1 y v 2 respectivamente cuyo → → →
→ p = m1 u 1 + m2 u 2
centro de masa se mueve con velocidad vCM , como
se muestra en la figura. → ⎡ m2 ⎛ → → ⎞⎤
= p = m1 ⎢ ⎜ v1 − v 2 ⎟⎥
⎣ m1 + m2 ⎝ ⎠⎦

⎡ m1 ⎛ → → ⎞⎤
= m2 ⎢ − ⎜ v1 − v2 ⎟⎥ = 0
⎣ m1 + m2 ⎝ ⎠⎦

La cantidad de movimiento es: En la sección siguiente veremos ejemplos de


aplicación usando el sistema de referencia centro de
→ → → →
p = m1 v 1 + m2 v 2 = (m1 + m2 ) v CM
masa.

CHOQUES
Para transformar esta expresión al sistema Centro de
masa, las velocidades de las partículas con respecto al Se llama choque o colisión entre dos cuerpos a un
centro de masa son como se muestra en la figura fenómeno en el que los cuerpos Participantes son
siguiente. libres antes y después de la interacción, sobre los que
no actúan fuerzas resultantes.

La interacción dura un tiempo muy corto, durante el


cual los cuerpos ejercen entre si fuerzas de cierta
intensidad.

Por lo general en los choques só1o participan dos


Las velocidades relativas al centro de masa son:
cuerpos, aunque esto no es estrictamente necesario.
→ → → → → →
u1 = v1 − vCM y u 2 = v 2 − vCM

9
Sistema de partículas Hugo Medina Guzmán

Sean dos cuerpos de masas m1 y m2 con velocidades Para la masa m 2 :


→ → → →
t0 → → → → →


v1 y v 2 antes del choque y velocidades v'1 y v'2 F2 dt = m2 v0 − m2 v 2 = J 2 D = − J 1D
después del choque respectivamente. ti

En todo choque entre dos cuerpos se conserva la → →


cantidad de movimiento, esto es: Resolviendo para v1 y v 2 .

→ → → → → → → →
p = p' ⇒ p1 + p 2 = p'1 + p' 2 → J → → J →
v1 = − 1D + v 0 , v 2 = 1D + v0
m1 m2
→ → → →
m1 v1 + m2 v 2 = m1 v'1 + m2 v' 2
La diferencia de estas velocidades es:
Ahora nos introduciremos en el proceso complejo que
→ → → ⎛ 1 1 ⎞ → ⎛m +m ⎞
acompaña al choque, el instante Δt = t f − t i , en el
v 2 − v1 = J 1D ⎜⎜ + ⎟⎟ = J 1D ⎜⎜ 1 2
⎟⎟
que aparece la fuerza de interacción, este periodo ⎝ 1
m m 2 ⎠ ⎝ 1 2
m m ⎠
vamos a dividirlo en dos partes, los periodos de
deformación y restitución. La figura muestra el
Ahora aplicaremos la ecuación Impulso-cantidad de
gráfico de la fuerza de interacción en función del movimiento por el periodo de restitución (R).
tiempo entre las masas m1 y m2.
t0 → t f .

Para la masa m1 :

t0 → → → →

∫ ti
F1 dt = m1 v'1 − m1 v0 = J R

Para la masa m 2 :
t0 → → → → →

∫ti
F2 dt = m2 v' 2 − m2 v 0 = J 2 R = − J 1R

→ →
Resolviendo para v '1 y v ' 2 .
E1 tiempo t 0 es el instante de máxima deformación
en el que empieza la restitución y las dos masas → →
→ J → → J →
poseen la misma velocidad v'1 = 1R + v0 , v' 2 = − 1R + v 0
m1 m2
→ → →
v01 = v02 = v0
La diferencia de estas velocidades es:

Vamos a aplicar la ecuación impulso - cantidad de


→ → → ⎛ 1 1 ⎞
movimiento para el periodo de deformación (D),
v' 2 − v'1 = − J 1R ⎜⎜ + ⎟⎟
ti → t 0 : ⎝ m1 m2 ⎠

Para la masa m1 : → ⎛ m1 + m2 ⎞
= − J 1R ⎜⎜ ⎟⎟
t0 → → → →
⎝ m1 m2 ⎠
∫ti
F1 dt = m1 v 0 − m1 v1 = J D
De lo visto encontramos la relación entre el impulso
de restitución y el impulso de deformación.

10
Sistema de partículas Hugo Medina Guzmán

→ a) Choque perfectamente elástico. En este caso no


J 1R (v' −v' )
=− 2 1 =ε
hay pérdida en la energía mecánica asociada al

J 1D (v 2 − v1 ) impacto, la energía cinética permanece constante.

K 1 + K 2 = K '1 + K ' 2
A esta relación se le conoce como coeficiente de
restitución (ε ) .
1 1 1 1
m1v12 + m2 v 2 = m1v'12 + m2 v' 22
Esta relación fue propuesta por Newton y tiene
2 2 2 2

( ) ( )
validez solamente aproximada.
m1 v'12 −v12 = m2 v' 22 −v 22
EI valor de esta relación depende de muchos factores
tales como la geometría, las propiedades de los Por conservación de la cantidad de movimiento
materiales, la velocidad, por ello debemos tenemos:
contentarnos con una determinación experimental.
→ → → →
Ejemplo 11. Una pelota de béisbol de 0,15 kg de p1 + p 2 = p'1 + p' 2 ,
masa se está moviendo con una velocidad de 40 m/s → → → →
cuando es golpeada por un bate que invierte su m1 v1 + m2 v 2 = m1 v'1 + m2 v' 2
dirección adquiriendo una velocidad de 60 m/s, ¿qué
fuerza promedio ejerció el bate sobre la pelota si
estuvo en contacto con ella 5 ms?. ⎛→ → ⎞ ⎛ → →⎞
m1 ⎜ v1 − v'1 ⎟ = m2 ⎜ v' 2 − v 2 ⎟
Solución. ⎝ ⎠ ⎝ ⎠
Datos: m = 0,15 kg
vi = 40 m/s Asumiendo que el movimiento es en una sola
vf = - 60 m/s (el signo es negativo ya que cambia el dirección
sentido)
t = 5 ms = 0,005 s m1 (v1 − v'1 ) = m2 (v' 2 −v 2 )
Δp = J
pf - pi = J ⇒ mvf - mvi = F t Dividiendo entre si las expresiones halladas por
energía y por cantidad de movimiento obtenemos.
⇒F = m(vf - vi)/t

v1 + v'1 = v' 2 +v 2 ⇒ (v' 2 −v'1 ) = −(v 2 − v1 )


F = 0,15 kg.(- 60 m/s - 40 m/s)/0,005 s
= 0,15 kg.(- 100 m/s)/0,005 s
= - 3000 N
(v' 2 −v'1 )
⇒ =1
CASOS DE CHOQUE − (v 2 − v1 )

Perfectamente elástico El cual es por supuesto el coeficiente de restitución


de un choque perfectamente elástico ε = 1 .
ε = 1 , (v' 2 −v'1 ) = −(v 2 − v1 )
b) Choque perfectamente plástico. En un choque
Inelástico ε <1 perfectamente Plástico, después del choque las masas
quedan juntas, es decir tienen la misma velocidad, tal
que
El coeficiente de restitución y tiene un valor entre 0 y
1.
→ →
v' 2 = v'1 , por lo tanto:
Perfectamente plástico
→ →
ε = 0 , (v' 2 −v'1 ) = 0 v' 2 − v'1 = 0 y ε = 0

Explosivo ε >1 Ejemplo 13. Medición del coeficiente de


restitución ε.
Ejemplo 12.

11
Sistema de partículas Hugo Medina Guzmán

Si se quiere medir el coeficiente de restitución de 1os sentido contrario. Luego del choque ambos cuerpos
materiales, se realiza mediante una bola hecha con continúan juntos con una velocidad final común a
uno de los materiales y una superficie plana hecha ambos.
con el otro material, la que se coloca sobre el suelo.
Se suelta verticalmente la bola sobre la superficie La velocidad final será:
desde una altura h1 .
m1v1i + m2v2i = m1v1f + m2v2f
Conocemos la velocidad de la bola al momento del
choque Como v1f y v2f son iguales porque ambos cuerpos
siguen juntos:
v1 = 2gh1
v1f = v2f = vf

La bola rebota verticalmente hasta una altura h2 , tal m1v1i + m2v2i = (m1 + m2)vf
que la velocidad v '1 de la bola después del choque es:
⇒ vf =
(m1v1i + m2v2i )
v'1 = − 2 gh2 (m1 + m2 )
b) Velocidades de igual dirección y sentido
Como la superficie no tiene velocidad inicial ni
contrario.
velocidad final v 2 = 0 y v ' 2 = 0 .

Encontramos que:

(v' 2 −v'1 ) v'1


ε =− =−
(v2 − v1 ) v1

Reemplazando valores:

En este caso los cuerpos poseían velocidades de igual


− 2 gh2 h2
ε =− = dirección pero de sentido contrario antes del choque,
2 gh1 h1 como en el caso anterior luego del impacto continúan
juntos, con una velocidad final que estará dada por la
diferencia de las cantidades de movimiento. La
Ejemplo 14. Choque plástico o inelástico velocidad final será:

a) Velocidades de igual dirección y sentido. m1v1i - m2v2i = m1v1f + m2v2f

Igualmente:

v1f = v2f = vf

m1v1i - m2v2i = (m1 + m2)vf

⇒ vf =
(m1v1i − m2v2i )
(m1 + m2 )
Supongamos un cuerpo 1 de masa m1 y velocidad v1 La velocidad final mantendrá la misma dirección pero
que se dirige a hacia el cuerpo 2 de masa m2 y tendrá el sentido de la velocidad del cuerpo que antes
velocidad v2, siendo ambas velocidades de igual del choque tenía mayor cantidad de movimiento.
dirección y sentido. Sobre cada cuerpo actuó en el
momento del choque, el impulso que le provocó el Ejemplo 15. Choque elástico
otro cuerpo, entonces hay dos acciones de igual
intensidad y sentido contrario, en consecuencia a) Velocidades de igual sentido
ambas cantidades de movimiento serán iguales y de

12
Sistema de partículas Hugo Medina Guzmán

Ejemplo 16. Choque plástico. Las dos partículas


quedan en contacto después del choque.

Estudiar desde dos puntos de vista:

a) Observado desde tierra, sistema laboratorio y

b) Observado desde el centro de masa.

Solución.
Durante el choque cada cuerpo recibe una cantidad de
movimiento que es igual a la velocidad perdida por el
otro. Al recuperar su forma inicial, cada uno pierde o a) Sistema laboratorio.
gana respectivamente, la cantidad de movimiento
ganada o perdida en el momento del choque, la La figura muestra las dos partículas antes y después
velocidad final de cada uno será: del choque.

v1 f =
m2
(v2i − v2 f ) + v1i
m1

Si las masas son iguales

v1 f = v2i − v2 f + v1i

b) Velocidades de distinto sentido Por conservación de la cantidad de movimiento

→ → →
m1 v1 + m2 v2 = (m1 + m2 )v' y

→ →
→ m v + m2 v2 (m1v1 + m2v2 ) ˆ
v' = 1 1 = i
(m1 + m2 ) (m1 + m2 )
La energía mecánica antes del choque es:
En este caso los cuerpos literalmente rebotan, y la
velocidad final de cada uno será: 1 1
K = K1 + K 2 = m1v12 + m2 v 22
2 2
v1 f =
m2
m1
(v2i + v2 f ) − v1i La energía mecánica después del choque es:

(m1 + m2 )v'2 = 1 (m1v1 + m2 v 2 )


2
Si las masas son iguales 1
K '=
2 2 (m1 + m2 )
v1 f = v2i + v2 f − v1i
La relación de la energía es:
El principio de conservación del impulso es el mismo
que el de conservación de la cantidad de movimiento. 2
⎛ m ⎞
⎜⎜ v1 + 2 v 2 ⎟⎟
Cabe aclarar que en la práctica podemos aplicar el K'
=
m1 ⎝ m1 ⎠
principio de conservación de la cantidad de
K (m1 + m2 ) ⎛ 2 m2 2 ⎞
movimiento durante los choques, siempre que el ⎜⎜ v1 + v2 ⎟
tiempo que el tiempo de duración del impacto sea ⎝ m1 ⎟⎠
muy pequeño.

13
Sistema de partículas Hugo Medina Guzmán

Por ejemplo la caída de un meteorito a la tierra, la que La figura muestra las dos partículas antes y después
suponemos inmóvil (v2 = 0) y m1 << m2, obtenemos del choque.
K'
K’ = 0 y = 0 , éste es un choque perfectamente
K
plástico. Si K fuera diferente de cero, la totalidad de
la energía se transformaría en calor.

b) Sistema centro de masa.

La figura muestra las dos partículas antes y después


del choque.

Por conservación de la cantidad de movimiento:

→ → →
m1 v1 = m1 v'1 + m2 v' 2 (1)

En sus componentes:
En éste caso:
m1v1 = m1v'1 cos θ1 + m2 v' 2 cos θ 2
→ m2 ⎛ → → ⎞
u1 = ⎜ v1 − v 2 ⎟ y 0 = m1v'1 senθ1 − m2 v' 2 senθ 2
m1 + m2 ⎝ ⎠
→ m1 ⎛ → → ⎞
u2 = − ⎜ v1 − v 2 ⎟ Como es un choque elástico la energía mecánica se
m1 + m2 ⎝ ⎠ conserva:

→ 1 1 1
Con v 2 = 0 , m1v12 = m1v'12 + m2 v' 22 (2)
2 2 2
Obtenemos: En las ecuaciones (1) y (2) conocidas las masas m1 y
m2, tenemos como incógnitas v1, v’1, v’2, θ1 y θ2.
→ m2 → → m1 → Contamos con tres ecuaciones. Para resolver
u1 = v1 y u 2 = − v1 necesitamos conocer al menos dos de las cantidades
m1 + m2 m1 + m2 anteriores.

Después del choque m1 y m2 entran en contacto En el caso particular en que m1 = m2, podemos llegar
constituyendo una sola partícula de masa (m1 + m2) a la relación;
que está en reposo en el sistema centro de masa,
→ → →
u’1 = u’2 = 0. v1 = v'1 + v' 2

Aquí también K’ = 0, ε = 0. Elevándola al cuadrado:

→ →
Ejemplo 17. Choque elástico. Consideremos dos

v12 = v'12 +v' 22 + 2v'1 ⋅ v' 2
partículas, una con masa m1 y velocidad v1 , la
→ Por la conservación de la energía:
segunda con masa m 2 y velocidad v 2 = 0
v12 = v'12 +v' 22
Solución.
Luego, obtenemos:
a) Sistema laboratorio.

14
Sistema de partículas Hugo Medina Guzmán

→ →
m1u1 = m1u '1 ⇒ u1 = u '1 y
v'1⋅ v'2 = 0

→ → m2 u 2 = m2 u ' 2 ⇒ u 2 = u ' 2
Las velocidades v '1 y v '2 son ortogonales, esto nos
dice que las trayectorias de las partículas después del Para un choque elástico ε = 1 , como se espera.
choque son perpendiculares entre sí, tal que:
θ1 + θ 2 = π 2 Ejemplo 18. Reflexión de partícula sobre un plano.
Consideremos dos partículas, una con masa m1 , que
b) Sistema centro de masa.
incide sobre una masa m 2 de superficie plana como

La figura muestra las dos partículas antes y después se muestra en la figura. La masa m1 tiene
del choque. → →
velocidades v1 y v '1 antes y después del choque, la

superficie inicialmente está inmóvil v 2 = 0 y tiene

una velocidad v' 2 después del choque.

Solución.
Por conservación de la cantidad de movimiento:
Por conservación de la cantidad de movimiento:
→ → → →
m1 u1 + m2 u 2 = m1 u '1 + m2 u ' 2 = 0 → → →
m1 v1 = m1 v'1 + m2 v' 2
De aquí:
Para la energía tenemos que tomar en cuenta si el
2 2 choque es elástico o no.
⎛m ⎞ ⎛m ⎞
u = ⎜⎜ 1 u1 ⎟⎟ , u '22 = ⎜⎜ 1 u '1 ⎟⎟
2
2
⎝ m2 ⎠ ⎝ m2 ⎠ a) Choque elástico.

En éste caso la energía mecánica se conserva


Como es un choque elástico la energía mecánica se
conserva: K = K'

1 1 1
1 1 1 1
m1u12 + m2 u 22 = m1u '12 + m2 u ' 22 m1v12 = m1v'12 + m2v'22
2 2 2 2 2 2 2

Reemplazando u 2 y u ' 2 en función de u1 y u '1 m2 2


De aquí obtenemos: v '1 −v1 =
2 2
v' 2
respectivamente. m1

⎛ 1 1 ⎞ ⎛ 1 ⎞ Expresión que podemos escribir como;


(m1u1 )2 ⎜⎜ + ⎟⎟ = (m1u '1 )2 ⎜⎜ +
1
⎟⎟
⎝ 2m1 2m2 ⎠ ⎝ 2m1 2m2 ⎠ ⎛ → → ⎞ ⎛ → → ⎞ m2 → →
⎜ v1 + v'1 ⎟ ⋅ ⎜ v1 − v'1 ⎟ = v 2 ⋅ v' 2
⎝ ⎠ ⎝ ⎠ m1
De aquí se deduce:
De la conservación de la cantidad de movimiento

15
Sistema de partículas Hugo Medina Guzmán


⎛→ → ⎞ →
⎛→ → ⎞ m →
m1 ⎜ v1 − v'1 ⎟ = m2 v' 2 ⇒ ⎜ v1 − v'1 ⎟ = 2 v' 2 el otro con velocidad v 2 de tal modo que chocan.
⎝ ⎠ ⎝ ⎠ m1 →
Después del choque el auto 1 sale con velocidad v '1
Reemplazando ésta expresión en la de la energía, cuya dirección forma un ángulo β , tal como se
obtenemos: indica en la figura.

→ → →
a) Hallar la velocidad del auto 1 luego del impacto.
v1 + v'1 = v' 2
b) Determinar la posición del centro de masa y las
Como ecuaciones paramétricas del mismo.


c) Determinar si el choque es elástico o no.
v1 = v1senθ1iˆ − v1 cos θ1 ˆj ,

v'1 = v'1 senθ '1 iˆ + v'1 cos θ '1 ˆj , m1 = m2 = 200 kg , v01 = 3 m/s ,
v02 = 1 m/s , v'1 = 2 m/s , α = 53º , β = 37º ,

d = 3m
v' 2 = −v' 2 ˆj

Reemplazando obtenemos:

v1senθ1iˆ − v1 cos θ 1 ˆj + v'1 senθ '1 iˆ


+ v'1 cos θ '1 ˆj = −v'2 ˆj

De aquí:

v1senθ 1 + v'1 senθ '1 = 0 y


v1 cos θ1 − v'1 cos θ '1 = v' 2 Solución.

En el caso en que v ' 2 = 0 (la superficie no se mueve) a) por conservación de la cantidad de movimiento

→ →
θ 1 = θ '1 y v1 = v '1 p antes = p después ⇒
→ → → →
b) Choque inelástico. m1 v1 + m2 v 2 = m1 v'1 + m2 v' 2

En éste caso K > K ' Aquí

1 1 1 →
m1v12 > m1v'12 + m1v' 22 v1 = 3iˆ ,
2 2 2

Para encontrar la relación de K y K’ podemos usar el v 2 = (1) cos 53º iˆ + (1)sen53º ˆj
coeficiente de restitución ε .

= 0,6iˆ + 0,8 ˆj ,
K'
ε= , siendo 0 ≤ ε ≤ 1
K →
v'1 = (2)cos 37º iˆ + (2)sen37º ˆj
Ejemplo 19. En un parque de diversiones dos amigos
juegan con los autitos “chocones”. En cierto
momento las direcciones de ambos vehículos forman = 1,6iˆ + 1,2 ˆj

un ángulo α . Un auto se dirige con velocidad v1 y

16
Sistema de partículas Hugo Medina Guzmán

Reemplazando: Como m1 = m 2 = 200kg



3iˆ + 0,6iˆ + 0,8 ˆj = 1,6iˆ + 1,2 ˆj + v'2


xCM =
1
(x1 + x 2 ) , yCM = 1 ( y1 + y 2 )
2 2
⇒ v'2 = 2iˆ − 0,4 ˆj
Antes del choque:
v'2 = 2 + 0,4 = 4,16
2 2

x1 = −3 + 3t , y1 = 0
= 2,04 m/s
x 2 = −0,6 + 0,6t , y 2 = −0,8 + 0,8t
2
tan γ = = −5 ⇒ γ = −79º Luego
− 0,4
xCM = −1,8 + 1,8t , y CM = −0,4 + 0,4t

Después del choque:

x1 = 1,6(t − 1) , y1 = 1,2(t − 1)

x2 = 2(t − 1) , y 2 = −0,4(t − 1)

Luego

xCM = 1,8(t − 1) , yCM = 0,4(t − 1)

c) Para saber si es elástico o no, tenemos que analizar


b) Para determinar la posición del centro de masa es si la energía se conserva o no.
necesario conocer la posición inicial de la masa m 2 .
La energía cinética antes del choque es:
Como m1 y m 2 emplean el mismo tiempo desde el
inicio hasta el choque: 1 1
K= m1v12 + m2 v 22
2 2
d 3m
t= = = 1s
v1 3 m s
=
1
(200)(3)2 + 1 (200)(1)2
2 2
La posición inicial de m 2 es:
= 900 +100 = 1000 J
x20 = −v2 xt = −0,6(1) = −0,6 m ,
y20 = −v2 yt = −0,8(1) = −0,8 m
La energía cinética después del choque es:

1 1
K'= m1v'12 + m2 v' 22
Siendo la posición inicial de m1 2 2

x10 = −3m , y10 = 0


=
1
(200)(2)2 + 1 (200)(2,04)2
2 2
El centro de masa está dado por:
= 400 + 416 = 816 J
x1 m1 + x 2 m2 y m + y 2 m2
xCM = , y CM = 1 1
m1 + m2 m1 + m2 Hay una disminución de la energía cinética:

17
Sistema de partículas Hugo Medina Guzmán

ΔK = 816 – 1000 = - 184 J


V = 2 gy = 2(9,80 )(0,45 × 10−2 )
Luego el choque es inelástico.
= 0,297 m/s
Ejemplo 20. El péndulo balístico. Este es el caso de La rapidez final v de la bala es
un choque perfectamente plástico, se utiliza para
medir la velocidad de un proyectil. Un proyectil de p mv0 − MV M
masa m y velocidad v se incrusta en el bloque de v= = = v0 − V
madera de masa M. m m m
1,00
= 450 − (0,297)
5,00 × 10− 3
= 390,6 m/s .

Ejemplo 22. Un satélite artificial en vuelo explota en


tres partes iguales. Una parte continúa a lo largo de su
línea original de vuelo y las otras dos van en
direcciones cada una inclinada 60º a la trayectoria
original. La energía liberada en la explosión es dos
veces más que la energía que tenía el saté1ite en el
Aplicando la conservación de la cantidad de momento de la explosión. Determinar la energía
movimiento. cinética de cada fragmento Inmediatamente después
de la explosión.
→ →
m v = (m + M )V Solución.

La energía cinética después del choque es: La figura muestra el satélite antes y después de la
explosión.

K'=
1
(m + M )V 2 , ésta se convierte en energía
2
potencial U = (m + M )gh

Luego (m + M )V = (m + M )gh
2

⇒ V = 2 gh

La velocidad del proyectil es:


Por conservación de la cantidad de movimiento.

v=
(m + M ) V =
(m + M ) 2 gh → →
m m p antes = p después
Ejemplo 21. Una bala de 5,00 g se dispara contra un
bloque de madera de 1,00 kg suspendido de un hilo La cantidad de movimiento debe conservarse en las
de 2,000 m, atravesándolo. El centro de masa del tres dimensiones x, y, z , independientemente, de allí
bloque se eleva 0,45 cm. Calcule la rapidez de la bala que v1, v2, v3 y V deben ser coplanares.
al salir del bloque si su rapidez inicial es de 450 m/s.
Así obtenemos:
Solución. M M M
Mv = v1 + v 2 cos 60º + v3 cos 60º
3 3 3
La rapidez del bloque de madera después de que la
bala ha atravesado (pero antes de que el bloque M M
comience a elevarse; esto asume una gran fuerza y v 2 sen 60º − v3sen 60º = 0
aplicada por un tiempo corto, una situación 3 3
característica de las balas) es
De estas dos ecuaciones encontramos que:

18
Sistema de partículas Hugo Medina Guzmán

v 2 = v3 (1) b) W f = μmgd
Wf 0,5
3v = v1 + v 2 (2) ⇒ μ= = = 0,41
mgd 2,5 × 9,8 × 0,05
1 2 1 2
1 c) kx − Wf = mv
La energía inicial es: Ei = Mv 2 2 2
2 1
(1040)(0,05)2 − 0,5 = 1 (2,5)v 2
La energía final es: 2 2
1 1 3 ⇒ v = 0,80 m/s
Ef = Mv 2 + 2 Mv 2 = Mv 2
2 2 2 Ejemplo 24. Una pelota de masa m = 100 g se deja
caer desde una altura h = 2m. La pelota rebota
Esta energía es igual a la suma de las energías de los verticalmente hasta ¾ h después de golpear el suelo.
tres fragmentos. a) Calcular la cantidad de movimiento de la pelota
antes y después de golpear el suelo,
b) si la duración del golpe fue de 0,01 s, calcular la
3 1M 2 1M 2 1M 2
Mv 2 = v1 + v2 + v3 fuerza media ejercida por el piso sobre la pelota.
2 2 3 2 3 2 3 Solución.
En la figura se muestra el esquema de la situación.
9v 2 = v12 + v 22 + v32 (3)

De las ecuaciones (1), (2) y (3) obtenemos:

v1 = v , v 2 = 2v , v3 = 2v

La energía cinética de cada uno de los fragmentos


inmediatamente después de la explosión es:
a) Cantidad de movimiento inicial:

1 2 2 pi = − mvi ˆj
K1 = Mv 2 , K 2 = Mv 2 , K 3 = Mv 2
6 3 3 Cantidad de movimiento final:

Ejemplo 23. Un bloque de 2,5 kg, se desliza sobre p f = mv f ˆj
una superficie rugosa, cuando contacta con el resorte Los valores de las velocidades inicial y final se
tiene una velocidad de 1,2 m/s. el bloque se detiene pueden calcular usando el principio de conservación
momentáneamente cuando el resorte se ha de la energía.
comprimido 5,0 cm. El trabajo realizado por la Inicial:
fricción, desde el instante en que el bloque hace
1 2
contacto con el resorte hasta el instante en que hace el 0 + mghi = mvi + 0 ⇒ vi = 2 ghi
alto es 0,50 J. 2
a) ¿Cuál es la constante del resorte (k)? Final:
b) ¿Cuál es el coeficiente de fricción? 1 2
c) Después de la compresión del resorte, el bloque se mv f + 0 = 0 + mgh f
aleja de él, ¿cual es la velocidad del bloque, después 2
de separarse del resorte?
⎛3 ⎞ 3
⇒ v f = 2 gh f = 2 g ⎜ hi ⎟ = ghi
⎝4 ⎠ 2

Por lo tanto, las cantidades de movimiento inicial y


final son:
Solución. → →
3
a) Energía antes = Energía después pi = −m 2 ghi ˆj , p f = m ghi ˆj
2
1
(2,5)(1,2)2 = 1 k (0,05)2 + 0,5 Reemplazando los valores, se tiene:
2 2 pi = −0,63 kgm/s , p f = −0,54 kgm/s
⇒ k = 1040N/m

19
Sistema de partículas Hugo Medina Guzmán

b) Usando la aproximación del impulso:


→ → →
→ → →
p − pf
J = Fm Δt = Δ p ⇒ Fm = Δ p = f

Δt Δt

Fm =
(
0,54 ˆj − − 0,63 ˆj )
0,01
= 118 ˆj N

Ejemplo 25. Una bala de la masa m y con velocidad


v pasa a través de un péndulo de masa M. La bala
emerge con una velocidad de v/2. El péndulo está
suspendido por una barra rígida de longitud l y masa
insignificante. ¿Cuál es el valor mínimo de v tal que
la masa del péndulo gire un círculo completo?
Solución.
En esta colisión, se conserva la cantidad de T + Mg = Mac
movimiento pero la energía no. Condición mínima para hacer movimiento circular
Este es un ejemplo de una colisión inelástica que no T =0
es perfectamente inelástica. Luego
Para la colisión:
V '2
Mg = M ⇒ V ' 2 = gl (3)
l
Reemplazando (3) en (2):

MV 2 = Mgl + Mg (2l ) ⇒
1 1
2 2
1 5
MV 2 = Mgl ⇒ V = 5 gl
2 2
Reemplazando el valor de V en (1):
2M
→ → v v= 5 gl
p antes = p después ⇒ mv = m + MV m
2
De aquí:
2M MOVIMIENTO CON MASA VARIABLE -
v= V (1) PROPULSIÓN POR REACCIÓN
m
Después de la colisión se conserva la energía para el
péndulo (la conservación de la energía para la bala Por la conservación de la cantidad de movimiento si
después de la colisión no es útil desde que su energía un cuerpo en reposo puede expulsar una parte de su
no cambia). Este tratamiento nos da la velocidad del masa en cierta dirección, el resto de la masa se
péndulo el momento después de la colisión: moverá en sentido opuesto, con igual cantidad de
movimiento. Si este proceso puede mantenerse
MV 2 = MV ' 2 + Mg (2l ) (2)
1 1 durante un tiempo, el resto de la masa, como es el
2 2 caso de un cohete, aparecerá para un observador
Condición para que pueda dar la vuelta externo en reposo. Como si se estuviese acelerando.
Esto se expresa mediante la forma más general de la
segunda ley de Newton.

Como:


→ dp → →
F= , p = mv
dt

Siendo la masa variable

20
Sistema de partículas Hugo Medina Guzmán

→ → →
→ dm v d v → dm Como el infinitésimo dmd v es de segundo orden,
F= = m +v podemos despreciar éste término; luego
dt dt dt
→ →
Expresión que nos permite determinar el movimiento
md v − dm u = 0
de un cuerpo cuya masa cambia durante su
movimiento.

d v → dm
Cuando aplicamos al caso de un cohete aparecen los Dividiendo por dt se obtiene: m −u =0
problemas, evidentemente m es la masa del cohete dt dt
que va cambiando. ¿Cuál es la velocidad de escape
→ →
del combustible? No es igual a v , la velocidad del → dv
→ Como v es la velocidad del cohete, es la
cohete. Si no existe fuerza externa, ¿ F debe ser cero? dt

Entonces no se moverá el cohete. Analicemos el
aceleración a .
problema desde el punto de vista de la conservación
de la cantidad de movimiento.
De éste modo:
Sea el cohete mostrado en la figura siguiente, en el


→ → dm
instante t, tiene una masa m y una velocidad v = viˆ , m
dv
= ma = u
→ dt dt
con una cantidad de movimiento lineal m v .
Por la segunda ley de Newton se puede identificar la
→ dm
cantidad u como una fuerza, tal que la fuerza
dt
de empuje es:
En la figura siguiente se muestra el cohete en el
→ →
instante t + dt , a expulsado una masa dm que sale F =u
dm
→ dt
con una velocidad u = −uiˆ relativa al cohete.
Ahora la masa del cohete es (m − dm ) y su → dm
Como u es negativa y por se pérdida de mas
⎛ → →

velocidad es ⎜ v + d v ⎟ . dt
⎝ ⎠ →
también es negativa, F es positiva, como
esperábamos. Esta es una fuerza externa que produce
aceleración al cohete, al que ahora consideraremos
como un sistema aislado sobre el que hay una fuerza
externa.

Velocidad del cohete.


La cantidad de movimiento lineal total es:
→ →
De la expresión md v − dm u = 0 , obtenemos:
(m − dm )⎛⎜ v + d v ⎞⎟ + dm⎛⎜ v − u ⎞⎟
→ → → →

→ →
⎝ ⎠ ⎝ ⎠ dv=u
dm
m
Por conservación de la cantidad de movimiento lineal
→ → → → dm → m → m
= u [ln ]m0 = u ln
m
tenemos: p antes = p después v (t ) = u ∫
m0 m m0

⎛→ →
⎞ ⎛→ →⎞
m v = (m − dm )⎜ v + d v ⎟ + dm⎜ v − u ⎟ → m
⎝ ⎠ ⎝ ⎠ Como u es negativa y m < m0 , ln es negativa.
m0
→ → → → → → →
m v = m v + md v − dm v + dmd v + dm v − dm u Luego la velocidad es positiva.

21
Sistema de partículas Hugo Medina Guzmán

Velocidad límite del cohete. (m0 − Ct )


v(t ) = − gt − uln
Una vez que se haya terminado el combustible la
m0
masa se reduce a m1 , y llegamos a la velocidad
b) Antes de encontrar la altura que alcanza en el
límite.
momento que se acaba el combustible, encontraremos
la altura para el tiempo t.
→ → m1
v m = u ln
m0 dz
Como v (t ) = , tenemos:
dt
Una vez alcanzada ésta velocidad, ésta permanecerá
constante.
(m0 − Ct )
dz = − gtdt − uln dt
Ejemplo 26. Un cohete y su combustible tienen una m0
masa inicial m0 . El combustible se quema a una
Integrando
dm
razón constante = C .Los gases son expulsados
m z t t (m0 − Ct )

con una velocidad constante u respecto al cohete. ∫0
dz = − ∫ gtdt − ∫ uln
0 0 m0
dt

⎡ (m − Ct ) ⎤
a) Despreciando la resistencia del aire, hallar la
z = − gt 2 − (m0 − Ct )⎢ln 0
1 u
velocidad del cohete en el instante t, después de − 1⎥
despegar de la tierra. 2 C ⎣ m0 ⎦
b) ¿Cuál es la altura que alcanza en el momento que
se acaba el combustible? El tiempo t1 en que se acaba el combustible es
cuando m = m1 . Como m = m1 − Ct , obtenemos:
Solución.
m0 − m1
a) Las fuerzas que actúan sobre el cohete son la t1 =
fuerza de empuje y la fuerza de la gravedad. C
Aplicando la segunda ley de Newton.
Reemplazando t1 en la expresión anterior
→ →dm → → →
∑ F = m a ⇒ u dt + m g = m a encontramos la altura que alcanza en el momento en
que el combustible se acaba.

→ → → CANTIDAD DE MOVIMIENTO ANGULAR Y



Donde u = −ukˆ , g = − gkˆ , a = d v = dv kˆ MOMENTO DE UNA FUERZA O TORQUE
dt dt
Consideremos una partícula P de masa m, su posición
dm con respecto al origen O en un instante dado está
y = −C ⇒ m = m0 − Ct →
dt determinada por el vector r .

Reemplazando uC − (m0 − Ct )g = (m0 − Ct )


dv →

dt La partícula tiene una velocidad v y su cantidad de


→ →
movimiento lineal es p = mv .
dv uC
o = −g +
dt (m0 − Ct )
v t t uC
Integrando ∫
0
dv = ∫ − gdt + ∫
0 0 (m0 − Ct )
dt

La velocidad en el instante t es:

22
Sistema de partículas Hugo Medina Guzmán

→ → →
dL d → → dr → → d p
= r× p = × p+ r ×
dt dt dt dt

Como


dr → → →
= v y p = mv ⇒
dt

LA CANTIDAD DE MOVIMIENTO ANGULAR
dr → → →

× p = v× m v = 0
O MOMENTUM ANGULAR L dt
→ →
Se define como el producto vectorial de r y p ,
Luego

→ → →
→ →
L = r× p dL → d p
= r×
dt dt

La dirección de L es perpendicular al plano definido
→ → →

por r y p , su sentido lo da la regla de la mano Por otra parte si F es la fuerza que produce el
movimiento de la partícula, por la Segunda Ley de
derecha, su módulo es:
Newton tenemos:

L = rp senθ = rmv senθ → →


→ → dv dp
F = ma = m =
Como vsenθ es la velocidad tangencial ( vt ) y dt dt
vt = ωr , siendo ω la velocidad angular. Podemos →
escribir: dL → →
Luego = r× F
dt
L = mr ω 2

A esta cantidad que produce un cambio en la cantidad


MOMENTO DE INERCIA (I ) . de movimiento angular se le conoce como
2
Llamando Momento de Inercia al producto mr ,
⎛→⎞
MOMENT0 DE UNA FUERZA o TORQUE ⎜ τ ⎟
Tenemos: ⎝ ⎠
→ → →
L = I ω, vectorialmente L = I ω → dL → →
τ = = r× F
dt
Las dimensiones de la cantidad de movimiento
angular son:
Tiene como módulo τ = rF senθ

[L] = [M ][L]2 [T ]−1 Su sentido está dado por la regla de la mano derecha.

Sus unidades en el sistema internacional: Ejemplo 27. Una partícula de masa m se mueve en el

2 plano xy con una velocidad v a lo largo de una línea
Kg m recta. ¿Cuál es la magnitud y dirección de su cantidad
o Js
s de movimiento angular con respecto al origen O?


Derivando la cantidad de movimiento angular L con
respecto al tiempo:

23
Sistema de partículas Hugo Medina Guzmán

Ejemplo 29. Un cilindro sólido Puede girar alrededor


de un eje sin fricción como se ve en la figura. Una
cuerda enrollada alrededor del radio exterior R1 ejerce
una fuerza F1 hacia la derecha. Una segunda cuerda
enrollada alrededor de la otra sección cuyo radio es
R2 ejerce una fuerza F2 hacia abajo. ¿Cuál es el torque
que actúa sobre el cilindro alrededor del eje z que
pasa por O?

Solución.


La posici6n de la partícula es r .


La velocidad de la partícula es v .

Su cantidad de movimiento lineal es


→ →
p = mv
Solución.
→ → →
Su cantidad de movimiento angular es L = r × p Sobre el cilindro actúan:

→ → →
→ → →
L = r × m v = rmvsenθ − kˆ ( ) F 1 = F1iˆ en r 1 = R1 ˆj y F 2 = − F2 ˆj en

r 2 = − R2 iˆ
La magnitud es: L = rmvsenθ = mvd , donde
d = rsenθ El torque neto sobre el cilindro es:


Luego L = −(mvd )kˆ
→ → →
τ = τ 1+ τ 2
Podemos ver que la cantidad de movimiento angular → → → → →
con respecto a O' es cero. τ = r 1 × F 1 + r 2 × F 2 = − R1 F1 kˆ + R2 F2 kˆ

= (R2 F2 − R1 F1 )kˆ
Ejemplo 28. En determinado instante, la Posición de
una partícula con respecto a un origen O de

coordenadas está dada por el vector r = 3iˆ + 4 ˆj Si F1 = 10 N, R1 = 2 m y F2 = 5 n, R2 = 1 m:
(en metros) . En ella actúa una fuerza
→ →
F = 16iˆ + 32 ˆj (en Newton) . Encontrar el torque τ = [(5 N )(1m ) − (10 N )(2m )]kˆ = −15kˆ N m

originado por la fuerza F que actúa sobre la
CONSERVACION DE LA CANTIDAD DE
partícula. Con referencia al origen O.
MOVIMIENTO ANGULAR

Solución.
En el caso de una partícula come en la sección
anterior, si el torque aplicado con relación a un punto
→ → →
El torque es: τ = r × F = (3iˆ + 4 ˆj ) × (16iˆ + 32 ˆj ) dado de referencia es cero, tenemos que:


= (3)(32)kˆ + (4)(16)(− kˆ ) = 96kˆ − 64kˆ dL →
= 0 , por consiguiente: L = CONSTANTE
dt
= 32k̂ Nm
La cantidad de movimiento angular con respecto al
punto de referencia es constante.

24
Sistema de partículas Hugo Medina Guzmán

Ejemplo 30. Una partícula de, masa M en el extremo Ejemplo 31. Cálculo de la desviación de un cuerpo
de un hilo gira con velocidad v1 cuando el radio es r1, situado en la línea ecuatorial y que cae desde una
si disminuimos el radio de r1 a r2, ¿qué sucede con la altura h .
velocidad?
Solución.
Solución.

En la figura, sea una partícula de masa m a una altura


h sobre la superficie de la Tierra en un punto que,
para simplificar, consideramos que se encuentra sobre
el ecuador. Para los casos de interés físico, la altura h
será por lo común muy pequeña en comparación con
el radio, R de la Tierra. Si se supone que la partícula
La figura indica la forma como se puede realizar esta parte del reposo en relación a un punto de la
experiencia, para disminuir el radio basta jalar el hilo. superficie de la Tierra verticalmente por debajo de él
entonces, inicialmente, el componente radial de la
velocidad vr de la partícula desaparece y su
Aplicando la conservación de la cantidad de
componente tangencial vθ será ω (R + h), en donde ω
movimiento angular:
es la velocidad angular de la Tierra. Al soltarse,
debido a la atracción gravitacional de la Tierra, la
Lantes = Ldespués partícula comienza a caer verticalmente hacía abajo y,
por ende, su distancia radial r del centro de la Tierra
r1 comienza a disminuir. De L = mrvθ se deduce que
r1 Mv1 = r2 Mv 2 ⇒ v 2 = v1 el componente tangencial de su velocidad vθ debe
r2
aumentar este proceso y de modo tal que haga que el
producto rvθ sea constante. En términos más
También podemos hallar el trabajo realizado para cuantitativos, esto quiere decir que durante su
acortar el radio. descenso hacia el suelo, la distancia radial r y la
velocidad tangencial vθ se deben relacionar por
1 medio de
K antes = Mv12 ,
2 mrvθ = mω ( R + h) 2 (1)
2 2
1 1 ⎛ r ⎞ 1 ⎛r ⎞ 2 Puesto que, inicialmente, la velocidad de la partícula
K después = Mv 22 = M ⎜⎜ v1 1 ⎟⎟ = M ⎜⎜ 1 ⎟⎟ v1 es ω(R + h), de tal modo que su cantidad de
2 2 ⎝ r2 ⎠ 2 ⎝ r2 ⎠ movimiento angular L en relación al centro de la
Tierra es mω(R + h)2 Anticipándonos al hecho de que
El trabajo realizado es igual al cambio de energía la desviación hacia el este será muy pequeña,
cinética. podemos escribir para la distancia radial r del cuerpo
al centro de la Tierra en cualquier instante t,
2
1 ⎛r ⎞ 2 1
W = ΔK = M ⎜⎜ 1 ⎟⎟ v1 − Mv12 r = R+h−
1 2
gt (2)
2 ⎝ r2 ⎠ 2 2

⎡ 2

1 2 ⎛ r ⎞ Y al sustituir esto en (1) obtenemos:
= Mv1 ⎢⎜⎜ 1 ⎟⎟ − 1⎥
2 ⎢⎣⎝ r2 ⎠ ⎥⎦

25
Sistema de partículas Hugo Medina Guzmán

ω ( R + h) 2 ω ( Ro + h ) (3) CONSERVACION DE LA CANTIDAD DE


vθ = = MOVIMIENTO ANGULAR DE UN SISTEMA
1
R + h − gt 2 ⎡ 1 2 ⎤
⎢1 − 2 gt /( R + h)⎥ DE PARTICULAS.
2 ⎣ ⎦
Vamos a considerar un sistema de dos partículas,
Para calcular la magnitud de la desviación hacia el como se muestra en la figura.
este, sea vy en el instante t, la velocidad del cuerpo
que cae en la dirección hacia el este, tal y como lo ve
un observador fijo con respecto a la superficie de la
Tierra. Entonces
v y = v o − rω
ω ( Ro + h ) 1 2
= − (R + h − gt )ω
⎡ 1 2 ⎤ 2
⎢⎣1 − 2 gt /( R + h)⎥⎦
⎡ 1 2⎤
⎢ 2 gt ⎥ 1 2
= ( R + h)ω ⎢1 + ⎥ − ( R + h)ω + gt ω
⎢ R + h⎥ 2

⎢⎣ ⎥⎦ → d L1
Para la partícula 1: τ 1 = , donde
= gt 2ω dt
τ 1 = r 1 × F 1 = r 1 × ⎛⎜ F 12 + F 1ext ⎞⎟
→ → → → → →
Donde la segunda igualdad se obtiene de la
utilización de (2) y (3) y la tercera, a continuación, ⎝ ⎠
mediante el hecho de que gt2 <<(R + h) y el empleo
del teorema binomial. Al integrar esta fórmula para vy →
se obtiene, para la desviación total hacia el este y en F1 Es la fuerza total sobre la partícula 1.
el instante t,
1 3
y= gt ω . →
3 F12 Es la fuerza ejercida por la partícula 2 y

Finalmente, puesto que el tiempo que necesita la →


1 F1 ext Es la suma de las fuerzas externas sobre la
partícula para caer la distancia h es de ( 2hg ) 2 , la
partícula 1.
deflexión total hacia el este d, se puede expresar
como sigue

→ d L1 → → → → → →
2 2h τ1 = = r1 × F12 + r1 × F1 ext = τ 12 + τ 1 ext
d = ωh (4) dt
3 g
Por ejemplo, si se deja caer una partícula desde una Similarmente para partícula 2.
altura de 100 metros, su desviación hacia el este,
según esta fórmula, se descubre que es (al sustituir los →
valores h = 100 metros y ω = 7,2 x 10-5rad/s) de 2,2 → d L2 → → → → → →
cm. Esta desviación es muy pequeña y sólo se puede τ2 = = r2 × F21 + r2 × F2 ext = τ 21 + τ 2 ext
observar en condiciones controladas cuidadosamente.
dt

Es importante recordar la base física para la deflexión → → →


→ →d L1 d L2 d Ltotal
pronosticada en (4). Conforme la partícula desciende Sumando τ 1 + τ 2 = + =
hacia la superficie de la Tierra su velocidad dt dt dt
tangencial vθ debe aumentar para que el producto rvθ
sea constante. Por consiguiente, de esto se desprende Para los Torques internos tenemos:
que su velocidad tangencial debe sobrepasar a la del
punto de la superficie que se encontraba inicial e
→ → → → → →
inmediatamente por debajo, y, en esta forma, se τ 12 + τ 21 = r1× F12 + r2 × F21
desvía hacia el este.

→ →
Como F21 = − F12

26
Sistema de partículas Hugo Medina Guzmán

τ 12 + τ 21 = r1× F12 − r2 × F12 = ⎛⎜ r1 − r2 ⎞⎟ × F12


→ → → → → → → → →

⎝ ⎠

Al estar sujetos los dos hombres a la cuerda su


movimiento es circular y si consideramos que el piso
está en el plano xy,
→ → → → → →
De la figura: r1 + r12 = r2 ⇒ r1 − r2 = r12 Tenemos:

Reemplazando → l ˆ
L inicial = 2 pk = lpkˆ ,
→ → → →
2
τ 12 + τ 21 = − r12 × F12 →
L final
l
= 2 p' kˆ =
lp ' ˆ
k
4 2
→ → → →
Como r12 y F12 son paralelos: r12 × F12 = 0 , y → → → →
Como L inicial = L final ⇒ p ' = 2 p
→ →
τ 12 + τ 21 = 0 La cantidad de movimiento lineal final de cada
hombre es el doble de la cantidad de movimiento
→ → lineal inicial.
→ → d Ltotal → d Ltotal
De aquí: τ 1 ext + τ 2 ext = y τ ext =
dt dt CONSTANTE, Independiente, del tiempo

→ → Ejemplo 33. Una partícula de masa m1 se desplaza


Vemos si τ ext = 0 ⇒ Ltotal = CONSTANTE , →
sobre un plano horizontal con velocidad v 1 . Dos
independiente del tiempo. partículas de masas m2 y m3 unidas por una varilla de

Ejemplo 32. Dos hombres se encuentran en una pista masa despreciable se mueven con velocidad v 2 ,
de patinaje, ambos sostienen una cuerda de longitud como se indica en la figura.
l . ¿Qué pasa con la cantidad de movimiento lineal

p de cada uno de ellos, si ambos jalan la cuerda y
acortan la distancia entre ellos a l / 2 ? Asumir que
se mueven en círculo y que la magnitud de sus
cantidades de movimiento son iguales.

Solución.

Las únicas fuerzas externas al sistema son la fuerza


de la gravedad y la reacción normal del piso, estas Suponiendo un choque totalmente plástico entre m1 y
fuerzas se cancelan. Las únicas fuerzas que, m2
intervienen en el sistema son las internas, por lo tanto
la cantidad de movimiento angular del sistema se → mˆ → m
conserva. m1 = m2 = m3 = 1 kg, v 1 = 20 i , v 2 = −10 iˆ ,
s s
l = 1m

27
Sistema de partículas Hugo Medina Guzmán

Calcular: b) Como parten del reposo, la cantidad de


movimiento total del sistema es cero.
a) La posición del centro de masa respecto a la masa
m2 en el momento del choque. → →
P total = M v cm = 0
b) La ley del movimiento del centro de masa.

d → →

c.) La velocidad angular de rotación alrededor del


Como v cm = r cm ∴ r cm = constante.
dt
centro de masa después del choque.
El centro de masa permanece en la misma posición.
Solución.
c) Consideremos la cantidad de movimiento angular
a) En el momento del choque, tomando como con respecto al centro de masa antes y después del
referencia la posición de m2, el centro de masa está en: choque.

Posición en el instante partida (t = 0). Antes del choque.

Posición en el instante de encuentro.


→ → → → → → →
L = r 1cm × p 1 + r 2cm × p 2 + r 3cm × p 3

→ 2d 1 → →
r 1cm = − iˆ − ˆj , p1 = m1 v 1 = 20iˆ
3 3

→ → →
r 2 cm = (d − v2t )iˆ −
1
ˆj , p 2 = m2 v 2 = −10iˆ
3

→ → →
r 3cm = (d − v2t )iˆ +
2
ˆj , p 3 = m3 v 2 = 20iˆ
3
⎛ 2d ⎞ ⎛ 2d ⎞ ⎛ 2d ⎞
m1 ⎜ ⎟ + m2 ⎜ ⎟ + m3 ⎜ ⎟ → → →

= ⎝ ⎠ ⎝ 3 ⎠ ⎝ 3 ⎠
3 Nota: En los vectores posición ( r 1cm , r 2 cm , r 3cm )
xCM
m1 + m2 + m3 solo ponemos la posición en el eje y, porque la
posición en x se va a anular con el producto vectorial.

m(2d ) 2d
= = Reemplazando:
3m 3 →
⎛ 1 ⎞ ⎛ 1 ⎞ ⎛2 ⎞
L = ⎜ − ˆj ⎟ × (20iˆ ) + ⎜ − ˆj ⎟ × (− 10iˆ ) + ⎜ ˆj ⎟ × (− 10iˆ )
⎝ 3 ⎠ ⎝ 3 ⎠ ⎝3 ⎠
m1 (0) + m2 (0) + m3 (l )
yCM = →
m1 + m2 + m3 L=
20 ˆ 10 ˆ 20 ˆ
k− k+ k = 10kˆ
3 3 3
l 1
= = m Después del choque:
3 3

28
Sistema de partículas Hugo Medina Guzmán

Como después del choque el sistema gira alrededor



del centro de masa con velocidad angular ω ,
podemos expresar la cantidad de movimiento angular
en función del momento de inercia.

→ →
L' = I ω , I = I 1 + I 2 + I 3 ⇒
Solución.
2 2 2
⎛l⎞ ⎛l⎞ ⎛ 2l ⎞ 2
I = m1 ⎜ ⎟ + m2 ⎜ ⎟ + m3 ⎜ ⎟ = a) La cantidad de movimiento con que se acerca el
⎝ 3⎠ ⎝ 3⎠ ⎝ 3⎠ 3 muchacho es:

→ 2→ → → →
luego L' = ω L = r × p = rmv senθkˆ
3

Por conservación de la cantidad de movimiento
→ →
Como r senθ = l 0 ⇒ L = ml 0 vkˆ
angular L' = L
b) Cuando el muchacho se coge del poste, las fuerzas
2 → →
rad de reacción centrípeta e impulsiva deben pasar por el
10kˆ = ω ⇒ ω = 15kˆ poste por lo tanto no ejercen ningún torque sobre el
3 s muchacho y la cantidad movimiento angular se
conserva.


L = ml 0 vkˆ = constante

La energía cinética después de cogerse del poste es


4
K '= K.
5

K es la energía cinética antes de cogerse,


1 2
Ejemplo 34. Un muchacho va corriendo por la acera K= mv
con una velocidad constante v con sus brazos 2
estirados perpendicularmente a su recorrido. La
distancia entre los extremos de los dedos de sus Luego:
manos es 2l 0 . Cuando al correr pasa junto a un
poste, se coge al mismo con la mano izquierda, 4 4⎛1 ⎞ 2
levanta los pies del suelo, y gira por aire alrededor del K'= K = ⎜ mv 2 ⎟ = mv 2 (1)
5 5⎝2 ⎠ 5
poste.

1 2 →
a) Si su masa es, M. ¿Cuál es el valor de su cantidad
También K ' = Iω , como L = Iωkˆ ⇒
de movimiento angular respecto al poste cuando corre 2
por la acera?
L
ω=
b) Si la fuerza de reacción del poste no só1o lo hace I
girar, sino que además proporciona una fuerza
impulsiva que hace frenar ligeramente su movimiento Reemplazando éste valor de ω en K’ :
hacia adelante, de modo que su energía cinética se

L2 (ml 0 v )
reduce a los cuatro quintos de su valor original. ¿Cuál 2
es su momento de inercia respecto al poste? K'= = (2)
2I 2I

Igualando (1) y (2)

29
Sistema de partículas Hugo Medina Guzmán

2 2 (ml 0 v )
2
5 2
mv = Luego su momento de inercia es I = ml 0
5 2I 4

PREGUNTAS Y PROBLEMAS

1. Una masa m1 se sitúa en (x1, y1, z1) y otra masa m2 5. Suponga que la fuerza que actúa sobre una pelota
en (x2, y2, z2). de tenis (m = 0,060 kg) en función del tiempo está
a) Hallar 1a distancia r0 entre m1 y m2. dada por la gráfica de la figura. Usando métodos
b) Hallar la distancia r1 entre m1 y el centro de masa. gráficos estime:
c) Hallar la distancia r2 entre m2 y et centro de masa. a) El impulso total dado a la bola.
Respuesta. a) b) La velocidad de ésta después de haber sido
r0 = (x 2 − x1 )2 + ( y 2 − y1 )2 + (z 2 − z1 )2 golpeada, suponiendo que estaba en reposo en el
momento de ser golpeada.
m2 r0 m1 r0
b) r1 = , c) r2 =
(m1 + m2 ) (m1 + m2 )
2. Dos partículas de masas m1 = 1 kg y m2 = 3 kg se
mueven por el espacio, sus vectores posición Son:
→ →
r1 = 3iˆ + tˆj − t kˆ , r2 = sen t 2 iˆ + kˆ
a) Hallar el centro de masa.
b) ¿Cuál es su aceleración?
c) Hallar su aceleración vista por un observador que
→ Respuesta. a) 4,5 Ns b) 75 m/s
se mueve con velocidad constante v = ˆj + 3kˆ .
Respuesta. a)

[( ( )]
6. Un flujo de partículas idénticas de masa m y
)

1
r CM = 3 + 3sen t 2 iˆ + tˆj + 3t − t kˆ →
4 velocidad uniforme v , inciden sobre un plano fijo de
área A, la dirección forma un ángulo θ con la normal.
( )

1 1 ˆ
b) a CM = cos t + 2t sen t iˆ +
2 2 2
tk Después del choque las partículas tienen una
2 16 → →
c) igual que b) velocidad v ' , la dirección es simétrica a la de v .
→ →
3. Encontrar el centro de masa de una lámina delgada También v = v ' .
mostrada en la figura a) Calcular el Impulso que se ejerce sobre cada
partícula en el momento del choque.
b) Calcular el valor de la fuerza comunicada a la
superficie por unidad de tiempo. Siendo n el numero
de partículas por unidad de volumen de chorro
incidente.

Respuesta. a) J = (2v cos θ )nˆ ,
b) F = 2nAmv cos
2 2
θ

Respuesta. y = 0,983m encima del centro del 7. Un nadador de 70 kg se lanza al agua desde el
orificio podio de una piscina con una velocidad de 3m/s en la
dirección de la figura. Calcular la fuerza ejercida
→ sobre el podio durante los 0,8s que el nadador ejerce
4. Una fuerza F = tiˆ + t ˆj + t kˆ actúa sobre un
2 3 el esfuerzo sobre el mismo para impulsarse en el salto.
cuerpo en el intervalo de 0 ≤ t ≤ 6 s . Hallar el
impulso sobre el cuerpo.
Respuesta. 181 + 72j + 324l 18iˆ + 72 ˆj + 324kˆ

30
Sistema de partículas Hugo Medina Guzmán

( )
→ →
Respuesta. F = − 227iˆ − 818 ˆj N Encontrar la velocidad v del obús.
d) Deducir del cálculo anterior el alcance R del obús.
8. Un recipiente de 0,25 kg con capacidad para 5 kg
de agua se llena de un caño en 5 s. En el instante en
que el recipiente está medio lleno, la balanza lee 3,0
kg. Si no se salpica el agua, ¿Cuál es la velocidad del
agua que cae en dicho instante.

Respuesta. 2,45 m/s


M V
Respuesta. a) v = − ,
9. Una bala de fusil de masa m y de velocidad m cos α
constante v 0 , penetra en un bloque de madera fijo; la ⎛ M⎞ V
b) v ' = −⎜1 + ⎟ ,
bala se detiene después de recorrer una distancia d ⎝ m ⎠ cos β
con un movimiento uniformemente retardado. m
e) Calcular la desaceleración a de la bala, deducir la c) u − v = v' cos β ,
fuerza de desaceleración. m+M

v = (u + Mv' cos β )iˆ + v' senβˆj
b) Calcular el tiempo de desaceleración.
c) ¿Cuál es el impulso comunicado a la bala por el
bloque? Comparar con la cantidad de movimiento de
2v 2 ⎛ Mv 2 ⎞
la bala antes del choque. d) R = senβ ⎜⎜ u + cos β ⎟⎟
Realizar la aplicación numérica para v 0 = 600 m/s, d g ⎝ M +m ⎠
=30 cm, m = 40 g
11. Desde la plataforma de un tren que se mueve con
v2 5 m
Respuesta. a) a = 0 = 6 × 10 2 , una velocidad de 10 m/s se arroja un paquete de 25
2d s kg, Este es cogido en el aire por una persona que está
F = ma = 24 × 10 N ,
7 junto a la vía. Desde el tren se observa que esta
persona retrocede con una velocidad de 7,5 m/s.
v −3
b) t = 0 = 10 s
¿Cuál es la masa de la persona?
a Respuesta. 75 kg.
c) J = F t = 24 N, J = mv0
12. Un muchacho está en medio de un lago
10. Un cañón fijo sobre un vagón que se puede congelado sin fricción de tal manera que no puede
desplazar si fricción sobre una vía rectilínea moverse. Para poder salir él lanza su sombrero de
horizontal con una masa total M. El cañón forma un masa 0,5 kg hacia el norte con una velocidad de 12
ángulo α con la horizontal. m/s a 53’ con la horizontal. Si la masa del muchacho
a) Si el vagón está en reposo el cañón dispara un obús es 60 kg y el radio del lago es 400 metros. ¿Qué
de masa m, determinar la relación entre las pasa7
velocidades v y V del obús y del cañón. Respuesta. El muchacho resbala hacia el sur y llega a
b) Si la velocidad del obús relativa al cañón es v’ la orilla 1 h 51 min después.
(forma un ángulo θ con V ), determinar la relación
13. Un paquete de 10 kg se descarga de una cinta
entre v ' y V . transportadora con una velocidad de 3 m/s y cae en
c) El vagón se desplaza con una velocidad rectilínea una vagoneta de 25 kg. ¿Si la vagoneta está
constante u sobre la vía antes del disparo. El obús inicialmente en reposo y puede rodar libremente,
tiene una velocidad v relativa al cañón en movimiento Cuál es su velocidad final?
a la velocidad V después del disparo:
Calcular la variación de la velocidad (u - v) del
vagón

31
Sistema de partículas Hugo Medina Guzmán

Respuesta. a) xCM = - 3 1/3 m,


→ → →
b) p1 = 25 iˆ kg.m/s, p 2 = 0 , pCM = 25 iˆ kg.m/s

c) v = 1 2/3 iˆ m/s

Respuesta. v = 0,732 iˆ m/s 17. Dos bolas P1 y P2 de masas m1 y m2 están


suspendidas del cielorraso por dos hilos inextensibles
14. Un hombre de 75 kg se lanza al agua desde la de la misma longitud l ; P1 y P2 están en contacto sin
proa de su bote de 50 kg. La componente horizontal presión con los hilos verticales.
de su movimiento es 1 m/s respecto al bote. Hallar las Se saca P1 de la posición de equilibrio a un ángulo
velocidades del hombre y del bote respecto a un
observador en el muelle.
θ 0 manteniendo el hilo tenso, luego se suelta sobre
a) Si el bote está inicialmente en reposo P2.
b) Si el bote se movía inicialmente hacia adelante con Calcular:
una velocidad de 2 m/s. No considerar pérdidas de a) La velocidad de P1 justo antes del choque.
energía debido al agua b) Las velocidades v’1 y v’2 de P1 y P2
Respuesta. a) v1 = 0,4 m/s , v2 = 0,6 m/s inmediatamente después del choque perfectamente
b) v1 = 2,4 m/s , v2 = 1,4 m/s elástico. Discutir este resultado para valores relativos
de las masas m1 y m2.
c) Las alturas de las posiciones limites de P1 y P2
15. Un cañón dispara un obús de 2,4 kg hacia arriba. después del choque.
A1canza su máxima altura, 313,6 m y se parte en dos, Aplicación numérica; l = l m . θ 0 = 60º, m2 = m1/2
0,8 kg y 1,6kg. Las dos partes llegan a tierra
simultáneamente. La pieza de 1,6 kg toca tierra a 480 Respuesta. a) v1 = 2 gl cos θ 0 , v1 = 3,13 m / s ,
m de la explosión (medida a lo largo del eje x).
m1 − m2
a) ¿Cuánto tiempo tomaría al obús volver a tierra si b) v '1 = v1 , v'1 = 1,05 m / s
no se hubiera partido? m1 + m2
b) ¿Cuál es la velocidad de cada una de las piezas
justamente después de la explosión?
Para m1 > m 2 v1 y v'1 tienen el mismo sentido.
c) Encontrar la cantidad de movimiento de cada pieza
justamente antes de tocar tierra. Para m 2 > m1 v'1 tiene sentido contrario de v1 .
Respuesta. a) 8 segundos
2m1v1
→ → v' 2 = , v '2 = 4,22 m / s
b) v (1, 6 ) = 60iˆ m/s (16), v (0,8 ) = −120iˆ m/s m1 + m2

c) p (1, 6 ) = 96iˆ − 125,44 ˆj kg.m/s ,
v' 2 en todo caso tiene el mismo sentido que v1
→ v'12 v '2
c) h1 = = 0,056 m , h2 = 2 = 0,91 m
p (0,8 ) = −96iˆ − 62,72 ˆj kg.m/s 2g 2g
(El movimiento es en el plano xy; g = 9,8 m/s2)
18. En un Juego de billar, la bola A está moviéndose
16. Un bloque de masa 10 kg está en reposo en el →
origen segundo con masa 5 kg se mueve a lo largo del con 1a velocidad v 0 = 31 m/s cuando choca con las
eje x con velocidad de magnitud v 0 = 5 m/s. Los bolas B y C que están juntas en reposo. Tras el
bloques choca quedan unidos. y se mueven en el eje x. choque, se observan la tres bolas moviéndose en las
La superficie tiene fricción despreciable. direcciones que muestra 1a figura, con θ = 30º.
a) ¿Cuando el bloque de 5 kg está en x = -10 donde Suponiendo Superficies lisas y choques
está centro de masa? perfectamente elásticos, hallar los módulos de la
b) Encontrar la cantidad de movimiento de la masa de → → →
5 kg, de la masa de 10kg y del centro de masa antes velocidades, v A , v B y vC .
del choque.
c) ¿Cuál es la velocidad del sistema combinado?

32
Sistema de partículas Hugo Medina Guzmán


Respuesta. v A = 1,5 m/s , v B = 1,29 m/s , vC 2,25 Respuesta. a) 0,943 m, b) 0,711 m, 0,37 m
m/s
23. Un objeto de 5 kg que se mueve con una
velocidad de 1,2 m/s choca directamente con un
19. Se dispara una bala de 39 g con una velocidad de
objeto de 20 kg que está en reposo. Se observa que el
500 m/ contra un bloque A de 5 kg de El coeficiente
objeto menor rebota con una velocidad de 0,6 m/s
de rozamiento entre el bloque A y la plataforma es
a) ¿Cuál es la pérdida de energía cinética por el
0,5. Si la masa de la plataforma es 4 kg y puede rodar
impacto?
libremente, hallar:
b) ¿Cuál es el coeficiente de restitución?
a) La velocidad final de la plataforma y e1 bloque.
Respuesta: a) ΔK = - 0,675 J, b) ε = 0,875
b) La posición final del bloque sobre la plataforma.
24. Una bola choca contra un plano liso formando un
ángulo φ1 con la normal del mismo y rebota con un
ángulo φ 2 . Encontrar La expresión correspondiente
al coeficiente de restitución

Respuesta. a) 2,16 m/s b) El bloque se detiene a


0,33 m de B.

20. La figura muestra dos masas sobre una


superficie con rozamiento despreciable. El coeficiente
de restitución entre Las dos masas es 0,73; determinar:
a) Sus velocidades después del choque.
b) La pérdida de energía durante el choque.
tan φ1
Respuesta. ε=
tan φ 2

25. Ira partícula de masa m1 tiene un choque frontal


perfectamente elástico con una partícula de masa m 2
→ →
Respuesta: a) v A = - 0,563 iˆ m/s, v B = 6,94 iˆ inicialmente en reposo. ¿Cuál es la pérdida relativa de
energía cinética correspondiente a la partícula . m1
m/s
b) ΔK = 41 J
ΔK m1
Respuesta. =4 2
K ⎛ m ⎞
21. Se deja caer una pelota al suelo desde una altura m2 ⎜⎜1 + 1 ⎟⎟
y. Si el coeficiente de restitución es ε, escribir ⎝ m2 ⎠
expresiones para el tiempo total que tardará la pelota
en dejar de dar bote y la distancia total que recorrerá 26. Una masa m1 se mueve a lo largo del eje x con
en este tiempo. una velocidad v 0 a lo largo de una mesa sin fricción.

Respuesta. t =
2 y (1 + ε )
, s= y
(
1+ ε 2 ) Choca con otra nasa, la cual está inicialmente en
g (1 − ε ) (
1− ε 2 ) reposo. La masa m2 sale a lo largo del eje y. Si se
pierde la mitad de la energía cinética original en el
choque.
22. Una bola cae desde una altura h = 0,900 m sobre ¿Cual es el módulo de la velocidad y con que ángulo
una superficie lisa. Si la altura del primer rebote es h1 sale después de la colisión?
= 0,800 m y la distancia d1 = 0,400 m, calcular:
a) El coeficiente de restitución.
b) La altura y longitud del segundo rebote.

33
Sistema de partículas Hugo Medina Guzmán

3 v ' B −v ' A v ' −v '


Respuesta. v 2 = m1v 0 = 0,9 ⇒ B A = 0,9 ⇒
(
2 m2 + m1 m2
2
,
) vA 3,43
1 v' B −v' A = 0,9(3,43) = 3,09 (2)
⎛mv ⎞ ⎡ 2 (m1 + m2 )⎤ 2
θ = cos −1 ⎜⎜ 1 0 ⎟⎟ = cos −1 ⎢ ⎥
Sumando (1) y (2):
⎝ m2 v2 ⎠ ⎣3 m2 ⎦ v' B +1,6v' B = 3,09 + 3,43
La velocidad de B inmediatamente después del
27. Se deja en libertad el bloque A cuando θΑ = 90 y choque es
desliza sin rozamiento, hasta chocar con la bola B. Si v' B = 2,51 m/s
el coeficiente de restitución es 0,90, calcular b) El diagrama del cuerpo libre de B, inmediatamente
a) La velocidad de B inmediatamente después del después del choque
choque.
b) La máxima tracción que soporta el hilo que
sostiene a B
c) La altura máxima a la que se eleva B.

v' 2B
∑ Fr = mB ac ⇒ T − mB g = mB 0,9

⎛ v' 2 ⎞
T = m B ⎜⎜ B + g ⎟⎟
⎝ 0,9 ⎠
⎛ 2,51 2

= 2⎜⎜ + 9,8 ⎟⎟ = 33,6 N
⎝ 0,9 ⎠
Solución. c) Por conservación de energía encontramos la altura
a) Por conservación de energía encontraremos v A . máxima a la que se eleva B.

.
1
m A gr = m A v A2 ⇒
2 1
m B v' 2B = m B gh ⇒
v A = 2 gr = 2(9,8)(0,6 ) 2
= 3,43 m/s v' 2 2,512
h= B = = 0,321 m
2 g 2(9,8)
Por conservación de la cantidad de movimiento
encontraremos v ' B
28. Un bloque de masa M está en reposo sobre una
masa sin fricción. Lo podemos golpear con un bloque
que se quede adherido o con un bloque muy duro con
el que se producirá un choque perfectamente elástico.
Ambos bloques tienen masa m y pueden ser lanzados
can velocidad V0 ¿En cuál de los casos el bloque M se
v A = 3,43 m/s v B = 0 moverá más rápidamente? (considerar el movimiento
m A v A + m B v B = m A v' A + m B v' B ⇒ en una sola dimensión).
(1,25)(3,43) = 1,25v' A +2v B ⇒ Respuesta. a) v ' 2 =
m
V0 ,
v' A +1,6v' B = 3,43 (1)
m+M
2m
El coeficiente de restitución es 0,90 b) v ' 2 = V0
(v' 2 −v'1 ) v' B −v' A m+M
ε =− = = 0,9
(v 2 − v1 )
En el segundo caso es el doble que en el primero.
vA

34
Sistema de partículas Hugo Medina Guzmán

29. Un cilindro A cae sobre otro B apoyado sobre un potencial del resorte comprimido es de 60 J y el
resorte de constante k = 3000 N/m desde una altura de →
2m. Si el choque es perfectamente plástico, calcular: conjunto posee la velocidad inicial v 0 Si se rompe la
a) El desplazamiento máximo de B. cuerda cuando θ = 30º , hallar la velocidad
b) La pérdida de energía en el choque. resultante de cada partícula

Respuesta. a) 3,47 cm , b) 8,18 J



30. Los parachoques se diseñan de tal manera que un Respuesta. v A = 9iˆ + 5,2 ˆj m/s y
automóvil de 1600 g que golpea una pared rígida a la →
velocidad de 12 km/h no sufra daño. Suponiendo que v B = 4iˆ − 3,5 ˆj m/s
ese choque es perfectamente plástico. Calcular:
a) La energía absorbida por el parachoques durante el 34. Un depósito suelta arena a una banda
impacto. transportadora razón de 75 kg/s. Si la banda se mueve
b) La velocidad a la que el automóvil puede golpear a con una rapidez constate v = 2,2 m/s. ¿Qué fuerza se
otro de iguales características, que está en reposo sin necesita para mantenerla en movimiento? No tomar
dañarse. en cuenta la fricción
Respuesta. a) 8890 J b) 24 km/h

31. Se dispara una bala de 25g en dirección


horizontal. La bala atraviesa el bloque A y queda
alojada dentro de bloque B. Por dicha causa los
bloques A y B comienzan a moverse con velocidades
iniciales de 2,4 y 1.8 m/s. respectivamente.
Hallar: Respuesta. 165 N
a) La velocidad inicial v 0 de la bala. 35. Un trineo lleno de arena se desliza sin fricción
b) La velocidad de la bala en el trayecto entre el por una pendiente de 30º. La arena se escapa por un
bloque A y el B. agujero en el trineo a un ritmo de 2 kg/s. Si el trineo
parte del reposo con una masa inicial de 40 kg.
¿Cuánto tardó en recorrer 120m a lo largo de la
pendiente?
Respuesta. 7 segundos

36. Un cohete que consume combustible a un ritmo


constante k se encuentra sometido a la acción de una
fuerza externa constante de valor F además de la
fuerza de reacción de los gases. La masa inicial del
cohete más combustible es m0 . La configuración de
Respuesta. a) 470iˆ m s b) 3261 m/s la tobera de escape es de tal manera que la velocidad
relativa de los gases es igual al negativo de la
32. Una explosión rompe un objeto en dos piezas una velocidad v del cohete.
de las cuales tiene 1,5 veces la masa de la otra. Si se a) Escribir la ecuación del movimiento.
liberan 4500 J en la explosión. ¿Cuánta energía b) Obtener v (t ) .
cinética adquiere cada pedazo?
Respuesta. a) F = (m0 − kt )
Respuesta. 1800 J, 2700 J dv
− kv ,
dt
33. Cuando se rompe la cuerda que une las partículas
A y B, el resorte comprimido las obliga a separarse
(el resorte no está unido a las partículas). La energía

35
Sistema de partículas Hugo Medina Guzmán

Ft b) Las velocidades de A y B cuando la varilla ha


b) v (t ) = girado 90º.
m0 c) Las velocidades de A y B cuando la varilla ha
girado 180º.
37. Un cohete experimental se proyecta de forma que
pueda mantenerse inmóvil sobre el suelo. El cuerpo
del cohete tiene una masa de 1200 kg y la carga de
combustible inicial es de 3600 kg,. e1 combustible se
quema y se expulsa con una velocidad de 2500 m/s.
Hallar la velocidad de consumo de combustible
necesario.
a) en el momento de encender el cohete.
b) cuando se consume la última partícula de
combustible.

→ → 3
Respuesta: a) p = mv 0 iˆ , L = mlv 0 kˆ
4
→ 1 3 → 1 1
b) v A = v 0 iˆ + v 0 ˆj , v B = v 0 iˆ − v 0 ˆj
4 4 4 4
→ 1 ˆ → 1 ˆ
c) v A = − v 0 i , v B = v 0 i
2 2
Respuesta. a) 18,84 kg/s . b) 4.71 kg/s 4l. Se tiene una varilla rígida de masa despreciable
sujeta a un eje sin rozamiento de tal manera que la
38. Una bala de masa m se dispara con una velocidad varilla pueda rotar libremente. Al otro extremo de la
→ varilla hay un bloque de masa M. Si se dispara una
v B = −v B iˆ , si para x = x 0 , y = a (permanece bala de masa m con una velocidad v 0 tal como se
constante) ¿Cuál es su cantidad de movimiento muestra en la figura. ¿Si la bala se incrusta en el
angular con respecto al origen en función de x? bloque, cuál será la velocidad angular del bloque

Respuesta. L = mv0 akˆ alrededor del eje7

39. Un obús de masa m se dispara de un cañón en el


origen, El obús se mueve en el plano y con una
velocidad inicial de magnitud v 0 y un ángulo θ con
el eje x.
a) ¿Cuál es el torque sobre el obús, con respecto al
origen en función del tiempo?
b) ¿Cuál es la cantidad de movimiento angular del,
obús con respecto al origen en función del tiempo?
→ mv0
→ Respuesta. ω=
c) Comprobar que τ =
dL
(m + M )a
dt

Respuesta. a) τ = −v0 mgt cos θ iˆ 42. Una barra de longitud b está pivotada en su
→ 1 centro de tal manera que puede rotar en el plano
b) L = − v 0 mgt cos θ iˆ
2
horizontal. Dos niños están sobre la barra en las
2 posiciones mostradas en la figura 7.59. a cual está
rotando con una velocidad angular en el sentido
40. Dos esferas pequeñas A y B están unidas por una
antihorario visto desde arriba. Si el niño de masa m1
varilla rígida de longitud l y masa despreciable. Las empieza a moverse hacia el centro tal que su distancia
dos masas reposan sobre una superficie lisa
a el es b 4 − at , ¿Cuál debe ser el movimiento del
2
horizontal cuando se comunica repentinamente a A la
→ niño de masa m2 para que la velocidad angular de la
velocidad v 0 = v 0 iˆ . Hallar: a) La cantidad de barra permanezca constante? (La masa de la barra es
movimiento lineal y la cantidad de movimiento despreciable),
angular del sistema respecto al centro de masa.

36
Sistema de partículas Hugo Medina Guzmán

49. Un cuerpo de masa m1 = 2 kg se desliza sobre


una mesa horizontal sin fricción con una velocidad
inicial v1i = 10 m/s, frente a él moviéndose en la
misma dirección y sentido se encuentre el cuerpo de
masa m2 = 5 kg cuya velocidad inicial es v2i = 3 m/s,
éste tiene adosado un resorte en su parte posterior,
cuya constante elástica es k = 1120 N/m, ¿cuál será la
máxima compresión del resorte cuando los cuerpos
Respuesta. Debe cambiar su distancia al centro de choquen?.
Respuesta. Δx = 0,28 m
b 2 m1 2 ⎛ b ⎞
acuerdo a la ecuación + at ⎜ − at 2 ⎟
16 m2 ⎝2 ⎠ 50. Un bloque de 3,0 kilogramos, moviéndose sobre
una superficie sin fricción con una velocidad de 1,2
m/s, tiene una colisión perfectamente elástica con un
43. Un taco golpea a una bola de billar ejerciendo bloque de la masa M en el reposo. Después de la
una fuerza promedio de 50 N durante un tiempo de colisión el bloque de 3,0 kilogramos retrocede con
0,01 s, si la bola tiene una masa de 0,2 kg, ¿qué una velocidad de 0,4 m/s.
velocidad adquirió la bola luego del impacto?.
Respuesta. vf = 2,5 m/s

44. Una fuerza actúa sobre un objeto de 10 kg


aumentando uniformemente desde 0 hasta 50 N en 4
s. ¿Cuál es la velocidad final del objeto si partió del a) La masa M es:
reposo?. b) La velocidad del bloque de masa M después de la
colisión es:
Respuesta. vf = 10 m/s
c) Los bloques están en el contacto para 0,20 s. La
fuerza media en el bloque de 3,0 kilogramos,
45. Se rocía una pared con agua empleando una mientras los dos bloques están en contacto, es:
manguera, la velocidad del chorro de agua es de 5 Respuesta. a) 6,0 kg, b) 0,8 m/ s, c) 24 N
m/s, su caudal es de 300 cm3/s, si la densidad del
agua es de 1 g/cm3 y se supone que el agua no rebota 51. El bloque de 8 kilogramos tiene una velocidad v y
hacia atrás, ¿cuál es la fuerza promedio que el chorro es detrás del bloque de 12 kilogramos que tiene una
de agua ejerce sobre la pared?. velocidad de 0,5 m/s. la superficie es de fricción
Respuesta. F = 1,5N despreciable. Los bloques chocan y se juntan.
Después de la colisión, los bloques tienen una
46. Se dispara horizontalmente una bala de 0,0045
velocidad común de 0,9 m/s.
kg de masa sobre un bloque de 1,8 kg de masa que
está en reposo sobre una superficie horizontal, luego
del impacto el bloque se desplaza 1,8 m y la bala se
detiene en él. Si el coeficiente de rozamiento cinético
entre el bloque y la superficie es de 0,2, ¿cuál era la
velocidad inicial de la bala?.
a) La pérdida de energía cinética de los bloques
Respuesta. v1i = 1073 m/s debido a la colisión está la más cercana a:

47. Se dispara una bala de 0,01 kg de masa contra un b) El impulso sobre el bloque de12 kg debido a la
péndulo balístico de 2 kg de masa, la bala se incrusta colisión es
en el péndulo y éste se eleva 0,12 m medidos Respuesta. a) 2,4 J, b) 4,8 N s
verticalmente, ¿cuál era la velocidad inicial de la
bala?. 52. Una bola de acero de 72 g se lanza desde el
reposo y cae verticalmente sobre una placa de acero.
Respuesta. v1i = 309,8 m/s
La bola golpes la placa y está en contacto con ella por
0,5 ms, la bola elásticamente, y vuelve a su altura
48. Una partícula A de masa mA se encuentra sujeta original. El intervalo de tiempo para el viaje es 0,30
por medio de un resorte comprimido a la partícula B s.
de masa 2mA, si la energía almacenada en el resorte a) La fuerza promedio ejercida sobre la bola durante
es de 60 J ¿qué energía cinética adquirirá cada el contacto es
partícula luego de liberarlas?. b) Asumiendo que la placa no se deforma durante el
Respuesta. Ec Bf = 20 J contacto. La energía elástica máxima almacenada por
la bola es:
Respuesta. a) 420 N, b) 0,08 J

37
Sistema de partículas Hugo Medina Guzmán

53. Una bala de la masa 0,01 kilogramos que se 56. En una demostración una bola de acero pequeña
mueve horizontalmente golpea un bloque de madera de la masa m se sostiene sobre una superbola de masa
de masa 1,5 kilogramos suspendida como péndulo. M (superbola es una bola de goma del coeficiente de
¿La bala se aloja en la madera, y juntos giran hacia restitución muy alto). La combinación junta se suelta
arriba una distancia de 0,40 m. cuál era la velocidad del reposo. Cuando el superbola golpea el piso rebota
de la bala momentos antes del impacto con el bloque casi elásticamente, golpeando a bola de acero que
de madera? La longitud de la cuerda es 2 metros. todavía está moviéndose hacia abajo. Esta colisión es
también bastante elástica, y consecuentemente bola
de acero se golpea y es lanzada derecho hasta una
altura H. Si h es la altura de la cual los objetos fueron
soltados, y M > > m, entonces bola de acero pequeña
se levantará a una altura:
Respuesta. 9 h

57. Una muchacha de masa 50 kilogramos lanza una


bola de la masa 0,1 kilogramos contra una pared. La
bola golpea la pared horizontalmente con una
velocidad de 20 m/s, y rebota con esta misma
velocidad. ¿La bola está en contacto con la pared
Respuesta. 66,7m/s 0,05 s, cuál es la fuerza media ejercida sobre la bola
por la pared?
54. Una bala de 10 g se dispara verticalmente en un Respuesta. 80N
bloque de 8 kilogramos. El bloque se levanta 3 mm.
La bala penetra en el bloque en un intervalo de 58. La bola A, de la masa 3,0 kilogramos, se une a
tiempo de 0,001 s. asume que la fuerza en la bala es una barra ligera de 0,4 m, que gira libremente en P.
constante durante la penetración. La bola B está suspendida de Q por una cuerda de 0,6
m y está en reposo. La bola A se levanta a cierto
nivel y se suelta. La bola A desciende, y tiene una
velocidad v1 = 3,6 m/s en el fondo, antes de chocar a
la bola B. Las velocidades de las bolas A y B después
del choque son: v2 = - 1,2m/s y v3 =2,2 m/s...

a) La energía cinética inicial de la bala es:


b) El impulse en el bloque debido a la captura de la
bala es:
c) La penetración de la bala en el bloque, es:
Respuesta. a) 190 J, b) 2,0 Ns, c)) 10 cm.

55. Una bala de 8 g se tira en un bloque de 4,0


a) La masa de la bola B es:
kilogramos, en reposo sobre una superficie horizontal
b) La magnitud del impulso sobre la bola A es:
sin fricción. La bala se aloja en el bloque. El bloque
c) La bola A rebota y gira un ángulo θ, donde la
se mueve hacia el resorte y lo comprime 3,0
centímetros. La constante de la fuerza del resorte es velocidad v4 es cero. El valor de θ es:
1500 N/m. d) La bola B se eleva hasta la altura h, donde la
velocidad v5 es cero. El valor de h es:
Respuesta. a) 6,6 kg, b) 14.4 N. s, c) 35º d) 0,25 m

59. Una pieza en forma de L se corta de una hoja


uniforme de metal. ¿Cuál de los puntos indicados es
a) La velocidad de la bala es: el más cercano al centro de la masa del objeto?
b) ) El impulso del bloque (con la bala), debido al
resorte, durante el tiempo en el cual el bloque y el
resorte están en contacto está es:
Respuesta. a) 290 m/s, b) 4,7 N.s

38
Sistema de partículas Hugo Medina Guzmán

Respuesta. Completamente inelástico

65. Las masas de los bloques, y las velocidades antes


y después del choque están dadas. ¿Qué clase de
choque es?

Respuesta. C
Respuesta. Parcialmente inelástico
60. Las masas de los bloques, y las velocidades antes
y después del choque están dadas. ¿Qué clase de
66. Las masas de los bloques, y las velocidades antes
choque es?
y después del choque están dadas. ¿Qué clase de
choque es?

Respuesta. Parcialmente inelástico.

Respuesta. Parcialmente inelástico


61. Las masas de los bloques, y las velocidades antes
y después del choque están dadas. ¿Qué clase de 67. Un resorte activa una bomba de juguete de 1,2 kg
choque es? sobre una superficie lisa a lo largo del eje x con una
velocidad de 0,50 m/s. en el origen O, la bomba
estalla en dos fragmentos. El fragmento 1 tiene una
masa de 0,4 kilogramos y una velocidad de 0,9 m/s a
lo largo del eje y negativo.

Respuesta. Perfectamente elástico.

62. Las masas de los bloques, y las velocidades antes a) La componente en x de la cantidad de movimiento
y después del choque están dadas. ¿Qué clase de del fragmento 2 debido a la explosión es:
choque es? b) El ángulo θ, formado por el vector velocidad del
Respuesta. no posible porque la cantidad de fragmento 2 y el eje x es:
movimiento no se conserva. c) La energía liberada por la explosión es:
Respuesta. a) 0., N. s, b) 31º, c) 0,32 J
63. Las masas de los bloques, y las velocidades antes
y después del choque están dadas. ¿Qué clase de 68. Un cono trunco homogéneo de metal tiene una
choque es? base circular mayor de radio 4 cm y la menor de radio
2 cm. Su altura es 6 cm. ¿A qué distancia de su
diámetro mayor está situado el centro de masa?

Respuesta. Caracterizado por un incremento en


energía cinética.

64. Las masas de los bloques, y las velocidades antes


y después del choque están dadas. ¿Qué clase de
choque es?
Respuesta. 2,36 cm

69. Cuatro masas puntuales se colocan como se


muestra en la figura: ¿Cuáles son las coordenadas del
centro de masa?

39
Sistema de partículas Hugo Medina Guzmán

72. Un carro de 19 kg está conectado por medio de un


resorte comprimido con un carro 38 kg. Los dos
carros se están moviendo a la derecha a una velocidad
de 25 m/s cuando el resorte se desenrolla y propulsa
repentinamente el carro de 19 kg hacia adelante con
una velocidad de 27 m/s. encontrar la velocidad del
segundo carro con respecto al centro de la masa del
sistema.
Respuesta. (23, 2,8)

70. Un alambre uniforme de longitud 60 cm y masa


60 g , está doblado en un triángulo rectángulo. Respuesta. 1 m/s
¿Cuáles son las coordenadas del centro de masa? 7
3. Una fuerza de 5,3 N es necesaria para sujetar a un
paraguas en un viento fuerte. Si las moléculas del
aire tienen una masa de 4,7 x 10-26 kilogramos, y cada
una golpea al paraguas (sin rebotar) con una
velocidad de 2,0 m/s en la misma dirección, ¿cuántos
átomos golpean al paraguas cada segundo? Asuma
que el viento sopla horizontalmente para no tomar en
cuenta la gravedad.
Respuesta. 5,6 x 1025 por Segundo
Respuesta. (10, 3)
74. Un cohete debe ser lanzado al espacio donde no
71. Una partícula de la masa 5,01 x 10-27 kilogramos, hay campo gravitacional. el 81% de la masa inicial
moviéndose a 1,88 x 105 m/s, choca con una del cohete es combustible y este combustible se
partícula idéntica que inicialmente está en el reposo. expulsa con una velocidad relativa de 2300 m/s. si se
Después de la interacción, las partículas (que no asume que todo el combustible será utilizado,
pueden ser distinguidas) se mueven con los ángulos encuentra la velocidad final de la última porción de
55,4º y 34,6º, ambos son medidos con respecto a la combustible expulsado relativo a un observador
dirección original del movimiento. ¿Qué velocidades estacionario.
finales tienen las partículas? Respuesta. 1500 m/ s
Respuesta. 1,55 x 105 m/s a 346º,
1,07 x 105 m/s a 55,4º

40
Cuerpo rígido Hugo Medina Guzmán

CAPÍTULO 7. Cuerpo rígido


INTRODUCCION La suma de las fuerzas que actúan sobre las n
En el capitulo anterior estudiamos el movimiento de partículas determinan la aceleración del centro de
un sistema de partículas. Un caso especial masa.
importante de estos sistemas es aquel en que la →
distancia entre dos partículas cualesquiera →
aCM =
∑ Fi
permanece constante en el tiempo, esto es un
M
CUERPO RIGIDO.
Tal como se mostró para un sistema de partículas,
A pesar que no existen cuerpos que sean
las fuerzas internas se anulan de pares, de forma
estrictamente rígidos, todos los cuerpos pueden ser
que solamente importarán las fuerzas externas tal
deformados, sin embargo el modelo del cuerpo
que
rígido es útil en muchos casos en que la → →
deformación es despreciable. M aCM = ∑ Fext
La descripción cinemática y dinámica de un cuerpo
extenso aunque este sea rígido en un movimiento en
tres dimensiones matemáticamente es muy “El movimiento de traslación del cuerpo rígido es
complejo y es tratado en libros avanzados de como si toda su masa estuviera concentrada en el
dinámica. Es complejo porque un cuerpo tiene seis centro de masa y las fuerzas externas actuaran sobre
grados de libertad; su movimiento involucra él”.
traslación a lo largo de tres ejes perpendiculares y Todo el estudio que hemos lecho anteriormente para
rotación alrededor de cada uno de estos ejes. No la partícula corresponde a la traslación de un cuerpo
llegaremos a hacer un tratamiento general directo, rígido. No importa ni la forma, ni el tamaño.
pero si desarrollaremos el movimiento del cuerpo
rígido en dos dimensiones. ROTACIÓN.
Es el movimiento en que uno de los puntos se
MOVIMIENTO DE UN CUERPO RÍGIDO considera fijo.
En esta parte expondremos algunos tipos de Sí se considera fijo un punto, el único movimiento
movimiento de los cuerpos rígidos. posible es aquel en el que cada uno de los otros
puntos se mueve en la superficie de una esfera cuyo
TRASLACION. radio es la distancia del punto móvil al punto fijo.
Por traslación entendemos al movimiento en el que Si se consideran dos puntos fijos, el único
lodos los puntos del cuerpo se mueven en la misma movimiento posible es aquel en que todos los
dirección, con la misma velocidad y la misma puntos con excepción de aquellos que se encuentran
aceleración en cada instante. sobre la línea que une los dos puntos fijos, conocida
como EJE, se mueven en circunferencias alrededor
de éste.

Por la definición de centro de masa, tenemos:


→ →

rCM =
∑ mi ri =
∑ mi ri
∑m i M
Donde M es la masa total del cuerpo rígido y Cualquier desplazamiento de un cuerpo rígido
→ → puede ser considerado como una combinación de
M rCM = ∑ mi ri traslación y rotación.

Diferenciando dos veces


d2 → d2 →
M 2 rCM = ∑ mi 2 ri
dt dt
→ → →
M a CM = ∑ mi ai = ∑ Fi En los capítulos anteriores ya hemos profundizado
bastante sobre movimiento de traslación

1
Cuerpo rígido Hugo Medina Guzmán

estudiaremos aquí el movimiento de rotación I) El teorema de Steiner o de los ejes paralelos.


alrededor de un eje y el movimiento de rotación “El momento de inercia del cuerpo respecto a un eje
traslación. es igual al momento de inercia del cuerpo respecto a
un eje paralelo al anterior y que pasa por el centro
CANT1DAD DE MOVIMIENTO ANGULAR de masa es el producto de la masa del cuerpo por el
DE UN CUERPO RÍGIDO cuadrado de la distancia entre los ejes”.
La cantidad de movimiento angular de una partícula I 0 = I CM + Md 2
respecto a un punto es
→ → → → → Demostración. La figura siguiente representa la
L = r× p = r× m v sección de un cuerpo en el plano del papel, CM es
En coordenadas polares: el eje normal al plano del papel a través del centro
de masa y O es un eje paralelo. Escogiendo un
→ dr→
r = rrˆ , v = rˆ + rω&t& elemento diferencial de masa dm , escribamos la
dt expresión para los momentos de inercia con

⎛ dr ⎞ respecto a los dos ejes.
L = rrˆ × m⎜ rˆ + rωtˆ ⎟
⎝ dt ⎠

L = mr 2 rˆω × tˆ

rˆ × tˆ tiene la dirección y sentido de ω
→ →
L = mr 2 ω I CM = ∫ rCM
2
dm I 0 = ∫ r 2 dm
M M
Si consideramos al cuerpo rígido como n partículas
que giran alrededor de un eje, la cantidad de usando la ley de los cosenos, obtenemos:
movimiento angular de éste será la suma de la r 2 = rCM
2
+ d 2 − 2rCM d cos θ
cantidad de movimiento angular de cada una de las reemplazando
partículas.
→ → → → I 0 = ∫ rCM
2
(
+ d 2 − 2rCM d cos θ dm )
L total = m r ω + m r ω + ........ + m r ω
2 2
n n
2 M

I 0 = ∫ rCM dm + d 2 ∫ dm − 2d ∫ rCM cosθdm


1 1 2 2 2

( )ω

M M M
= m r + m r + ........ + m r
2 2 2
1 1 2 2 n n El primer término
⎛ ⎞→ ∫
n
rCM
2
dm = I CM
=⎜

∑m r
i =1
i i
2
⎟ω

M
El segundo término
La cantidad entre paréntesis es el MOMENTO DE d 2 ∫ dm = Md 2
INERCIA DEL CUERPO RÍGIDO alrededor de un M
eje. El tercer término es cero porque es la suma en todo
n el cuerpo d los productos del elemento de masa y
I = ∑ mi ri 2 sus distancias al eje a través del centro de masa, de
i =1 aquí:
Es importante darse cuenta que el momento de I 0 = I CM + Md 2
inercia depende de la distribución de la masa del
cuerpo.
En el caso de un cuerpo rígido continuo, II. El teorema de la figura plana.
El momento de inercia de una figura plana con
los mi tienden a dm y respecto a un eje perpendicular a la misma es igual
∑ se transforma en ∫
M
, de aquí: a la suma de los momentos de inercia de la figura
plana con respecto a dos ejes rectangulares en el
I = ∫ r 2 dm plano de la figura los cuales se intersecan con el eje
M dado
Como m = ρV , donde ρ es la densidad y V el Demostración:
volumen del cuerpo: En la figura siguiente el eje z pasa por O
perpendicular al piano y. Elegimos un elemento
dm = ρdV diferencial de masa dm y escribimos los momentos
Tenemos: I = ∫ V
ρr 2 dV de inercia de la figura para cada uno de los tres ejes.
Para muchos cuerpos de forma geométrica simple
ésta integral puede evaluarse fácilmente.

Dos teoremas que simplifican los cálculos del


momento de inercia son:

2
Cuerpo rígido Hugo Medina Guzmán

= 7 mb + 5ma
2 2

Aquí comprobamos
Iz = Ix + Iy

b) Momento de inercia de una varilla delgada rígida


de longitud l y masa m, con respecto a un extremo
y con respecto al centro de masa.
Solución.

I x = ∫ y 2 dm , I y = ∫ x 2 dm , I z = ∫ r 2 dm
M M M

con r =x +y
2 2 2

∫ r dm = ∫ (x + y )dm
2 2 2
M M
Tomemos un elemento diferencial dx, cuya masa es:
= ∫ x dm + ∫ y dm
2 2
M M M
dm = dx
Iz = Ix + Iy l
El momento de Inercia de la varilla es:
Ejemplo 1. A continuación evaluaremos los l M
momentos de inercia algunos cuerpos simples. I O = ∫ x 2 dm = ∫ x 2 dx
M 0 l
a) Hallar el momento de inercia del sistema
mostrado en la figura, las masas son puntuales
unidas por varillas rígidas de masa despreciable.
=
M l 2

l 0
x dx =
M 3
3l
x [ ] l
0

1
= Ml
3

3
El momento de inercia de la varilla con respecto al
centro de masa

Solución.
Momento de inercia respecto al eje x.

[ ]
l l
M M 3
I x = ∑ y mi I CM = ∫ x dx = x
2 2 2 2
i l l

2
l 3l −
2

= m(0 ) + 2m(0 ) + 3m(b ) + 4m(b )


2 2 2 2 1
= Ml 3
12
Aquí comprobamos:
= 7 mb
2
2
Momento de inercia respecto al eje y. ⎛l⎞
I O = I CM + M⎜ ⎟
⎝2⎠
I y = ∑ xi2 mi c) Momento de inercia un anillo de masa M y radio
R, en el plano xy, Con respecto a los ejes x, y, z.
Solución.
= m(0 ) + 2m(a ) + 3m(a ) + 4m(0 )
2 2 2 2

2
= 5ma
Momento de inercia respecto al eje z.

I z = ∑ ri 2 mi

= m(0 ) + 2m(a ) + 3m a + b
2 2
( 2 2
) + 4m(b) 2 La masa del elemento diferencial ds = Rdθ es:

3
Cuerpo rígido Hugo Medina Guzmán

M M
dm = ds = dθ
2πR 2π
El momento de inercia del anillo con respecto al eje
z es:
2π M
I z = ∫ R 2 dm = ∫ R 2 dθ
M 0 2π
MR 2 2π
= [θ ]0 = mR2

Por el teorema de la figura plana
Iz = Ix + Iy Consideremos la esfera como una serie de discos.
Por simetría Tomemos un disco diferencial como se muestra en
Ix = Iy la figura, su radio es r = R 2 − z 2 , su espesor
Luego dz.
La masa del disco es:
Iz 1
Ix = Iy = = MR 2
2 2 dm =
M 2
V
M
(
πr dz = π R 2 − z 2 dz
V
)
d) El momento de inercia de un disco de radio R y
masa M con respecto al eje perpendicular que pasa 4 3
por su centro. M es la masa de la esfera y V = πR el
3
Solución. volumen de la esfera.
El momento de inercia del disco con respecto al eje
z es:

dI z =
1
2
dmr 2 =
1M
2V
(
π R 2 − z 2 dz
2
)
El momento de inercia de la esfera lo encontramos
integrando esta expresión desde z = - R a z = R.

I z = ∫ dI z = ∫
1M
R

2V
−R
( )
π R 2 − z 2 dz
2

8 πMR 5
Consideremos como elemento diferencial al anillo =
M
V
π∫
0
R
( )
2
R 2 − z 2 dz =
15 V
de radio r y ancho dr, su masa es:
M 2M 2
= MR
2
dm = 2πrdr = 2 rdr 5
πR 2
R
El momento de inercia de este anillo con respecto al
Para encontrar el momento de inercia con respecto a
eje perpendicular que pasa por O es
un eje arbitrario como se muestra en la figura
2M
dI O = r 2 dm = r 2 rdr siguiente aplicamos el teorema de Steiner.
R2
2M 3
= r dr
R2
El momento de inercia del disco es:
R
R 2M 3 2M ⎡ r 4 ⎤
I O = ∫ dI O = ∫ r dr
R 2 ⎢⎣ 4 ⎥⎦ 0
=
0 R2
1
= MR 2
2
e) El momento de inercia de una esfera con respecto 2
a un eje que pasa por su centro. I P = I O + Md 2 = MR 2 + Md 2
Solución. 5
⎡ 2 ⎛ R ⎞2 ⎤
I P = Md ⎢1 + ⎜ ⎟ ⎥
2

⎢⎣ 5 ⎝ d ⎠ ⎥⎦
En el caso en que R << d podemos considera como

4
Cuerpo rígido Hugo Medina Guzmán

si fuera una masa puntual y el momento de inercia los torques producidos por las fuerzas externas que
se reduce a: actúan sobre el sistema es igual al cambio de la
I O = Md 2 cantidad de movimiento angular.

→ dL
Ejemplo 2. Hallar el momento de inercia de un τ =
disco de masa M y radio R que gira alrededor de un dt
eje paralelo a un diámetro y que pasa por el borde Esto es válido también para el cuerpo rígido, donde
del disco. L es la cantidad de movimiento angular can
respecto al eje x de la figura anterior.

Solución. → d L d ⎛ →⎞

Como L = I ω ⇒ = ⎜Iω⎟
dt dt ⎝ ⎠
Siendo I el momento de inercia del cuerpo en torno
al eje dado, es constante en el tiempo y

→ dω
τ =I
dt

dω →
Como = α , aceleración angular del cuerpo
Por el teorema de las figuras planas dt
→ →
Iz = Ix + Iy ;
τ = Iα
Además por simetría Esta expresión tiene similitud a la ley de Newton
→ →
I x = I y,
F = ma
Por tanto
Ejemplo 3. Una barra uniforme de longitud L y
Ix = Iz/2 = ¼ MR2
masa M, que gira libremente alrededor de una
bisagra sin fricción, se suelta desde el reposo en su
Aplicando el teorema de Steiner
posición horizontal, como se muestra en la figura.
Calcular la aceleración angular de la barra y su
I = ¼ MR2 + MR2
aceleración lineal inicial de su extremo.
= 5/4 MR2

SEGUNDA LEY DE NEWTON PARA


ROTACION
En esta sección vamos a analizar el movimiento de
un cuerpo rígido que gira en torno a un eje fijo en el
espacio.

Solución.
Como el torque de la fuerza en la bisagra es cero,
se puede calcular el torque en torno a la bisagra
producido por la otra fuerza externa que actúa sobre
la barra, que es su peso, suponiendo que la barra es
homogénea y que el peso actúa en su centro
geométrico. Entonces:
1
El cuerpo gira en torno al eje x. Si θ = θ (t ) es el τ = rMg = LMg
desplazamiento angular del punto del cuerpo desde
2
la línea referencial, la velocidad angular del cuerpo Como τ = Iα , y el momento de inercia de la barra
es: 1
es I = ML .
2
dθ 3
ω=
dt 1
Como cada punto del cuerpo gira a la misma Se tiene: Iα = LMg
2
velocidad angular ω , el desplazamiento θ (t ) de
cualquier punto describe el desplazamiento del
cuerpo como un todo.
Para el sistema de partículas vimos que la suma de

5
Cuerpo rígido Hugo Medina Guzmán

1
LMg ( )
5 r 2 − b 2 gh 2 senθ
⇒ α= 2 =
3g =
(
7r 2 − 5b 2 )
1 2L
ML2
3 Ejemplo 5. Se tiene un disco de masa M y radio R,
Para calcular la aceleración lineal del extremo de la que pueda girar libremente alrededor de un eje que
barra, usamos la ecuación at = αL . pasa por su centro. Se enrolla una cuerda alrededor
del disco, se tira la cuerda con una fuerza F. Si el
Reemplazando α: disco está inicialmente en reposos ¿Cuál es su
3 velocidad al tiempo t?
a t = Lα = g
2
Ejemplo 4. Una esfera rueda sobre una barra, con
sección en forma de U, inclinada. Determinar la
aceleración.

Solución.
El momento de inercia del disco con respecto al eje
es:
1
I= MR 2
2
La dirección de la cuerda siempre es tangente al
disco por lo que el torque aplicado es:
Solución.
τ = FR
Las fuerzas que actúan sobre la esfera son el peso, Como τ = Iα
P, la reacción normal del plano, R, y la fuerza de τ
rozamiento Ff.
Tenemos α=
I
Reemplazando
Como la reacción R y el rozamiento Ff están FR 2F
aplicados en el eje instantáneo de rotación no α= =
1 MR
realizan ningún torque, sólo el peso: MR 2
2
τ = hmg senθ , siendo h = (r 2 − b 2 ) α ω = ω0 + α t
12 Siendo constante
2F
Como ω0 = 0 ⇒ ω = α t = t
El momento de inercia de la esfera con relación al MR
eje instantáneo de rotación es
Ejemplo 6. Se sujeta una masa M a una cuerda
2 2 ligera enrollada alrededor de una rueda de momento
I= mr + mh 2 de inercia I y radio R.
5 Hallar La tensión de la cuerda, la aceleración y su
velocidad después de haber descendido una
Aplicando la ecuación fundamental de la dinámica distancia h desde el reposo.
de rotación:

τ hmgsenθ hgsenθ
α= = =
I (2mr / 5 + mh ) (2r 2 / 5 + h 2 )
2 2

la aceleración lineal será: a = α h

Solución.
h 2 gsenθ gsenθ
a= = La figura siguiente muestra los diagramas de cuerpo
(2r / 5 + h ) (2r / 5h 2 + 1)
2 2 2 libre.

6
Cuerpo rígido Hugo Medina Guzmán

a) Por el teorema de las figuras planas, tenemos


que:
Iz = Ix + Iy ;
Además por simetría
Aplicando la segunda ley de Newton a la masa M I x = I y,
Mg − T = Ma (1) Por tanto
Iz 1
= ρLR = ρ (2πR )R = πρR
1
Aplicando la segunda ley de Newton para rotación Ix = 2 2 3
al disco 2 2 2
TR = Iα ,
a
( )
= π 1,6.10 (0,05) = 6,28x10-5 kg m2
−1 3

como a = Rα ⇒ α = b) Al comunicarle un momento angular


R L = 7,9 x10-4 kg m2/s,
a L 7,9 × 10 −4
TR = I o TR 2 = Ia (2) ω0 = =
R I 6,28 × 10 − 5
Resolviendo (1) y (2) obtenemos = 12,58 rad/s
M c) τ = 50 dina cm = 50x10-5 Nx10-2 m
a= g,
M + I R2 = 5x10-6 N m
Por lo tanto la ecuación del movimiento en términos
I R2 angulares será:
T= Mg
M + I R2 1
θ = θ 0 + ω0t + αt 2 = 12,6t − 0,0398t 2 , y
2
Siendo un movimiento con aceleración constante ω = 12,6 − 0,079t
v 2 = v02 + 2as Siendo ω = 0 para t = 158 s.
Conocemos: a , v0 = 0 , s = h :
Ejemplo 8. Maquina de atwood tomando en
2Mg cuenta la polea.
v2 = h
M + I R2
2 Mg
⇒ v= h
M + I R2

Ejemplo 7. Un anillo de 5 cm de radio, grosor


despreciable y densidad 1,6 g/cm, se pone en
rotación alrededor de un diámetro cuando se le
comunica un momento angular de 7900 g cm2/s.
a) Hallar la expresión analítica y el valor numérico
del momento de inercia respecto del eje de giro.
b) ¿Con qué velocidad angular empieza a girar?
c) Si el rozamiento con el aire y los pivotes origina
un par de fuerzas cuyo torque es de 50 dina cm, La polea es un disco de masa M y radio R. La figura
¿cuál será la ecuación del movimiento que efectúa muestra los diagramas de cuerpo libre de cada una
el anillo?, ¿cuánto tiempo tarda en pararse? de las partes de la máquina de atwood.
(Nota 1 N = 105 dinas)
Solución.

7
Cuerpo rígido Hugo Medina Guzmán

Planteando la segunda ley de Newton para cada


Aplicando la segunda ley de Newton a cada una de masa:
las partes.
Masa M1: m1 g − T1 = m1 a ,
T1 − M 1 g = M 1 a (1) T2 − m 2 g = m2 a
Masa M2: Para la polea:
M 2 g − T2 = M 2 a a
Polea:
(2)
∑τ = T R − T R = Iα = I R
1 2

T2 R − T1 R = Iα Como el hilo no desliza,


1 a 1 a = αR
= MR 2 = MRa (3) Por lo tanto tenemos tres ecuaciones:
2 R 2 m1 g − T1 = m1 a ,
Resolviendo (1), (2) y (3), obtenemos:
T1 = M 1 ( g + a ) , T2 − m 2 g = m2 a ,
T2 = M 2 ( g − a ) y T1 − T2 = I 2
a
(m2 − m1 ) R
a= g Que sumadas dan lugar a:
(m2 + m1 + M 2) (m1 – m2) g = a(m1 + m2 + I/R2).
Por lo tanto a vale:
Ejemplo 9. Una polea homogénea de radio R, masa m1 − m2 5
M y momento de inercia I, gira alrededor de su eje, a= g = 9,8
I 18
debido a la acción de dos masas m1 y m2. m1 + m2 + 2 25 +
R = 0,3 m, m1 =15 kg, m2 = 10 kg, M = 20 kg, I =18 R 0,3 2
kg m2. = 0,22 m / s2
Calcular: a 0,22
a) La aceleración angular de la polea. y α= = = 0,73 rad / s2
b) Las tensiones de las cuerdas. R 0,3
c) La tensión del soporte que fija el sistema al techo b) De las ecuaciones anteriores obtenemos:
T1 = m1 g − m1 a = 15( g − a ) = 143,7 N.
T2 = m 2 ( g + a ) = 100,2 N.
c) Considerando todas las fuerzas que actúan sobre
la polea, que debe estar en equilibrio:

Solución.
a) Vamos a suponer que el sistema acelera hacia el
lado de la masa mayor M. →
∑F = 0
S = P + T1 + T2 = 20 x 9,8 + 146,67 + 102,22
= 445 N

8
Cuerpo rígido Hugo Medina Guzmán

Ejemplo 10. La figura representa un cilindro kg y de la garganta de la polea pequeña pende otra
macizo y homogéneo de radio R = 20 cm y masa masa de 100 kg que tiende a hacer girar a las poleas
M = 20 kg. A su periferia va arrollado un hilo ideal en sentido contrario al anterior. El momento de
de cuyo extremo libre cuelga una masa m = 8 kg. inercia del sistema formado por las dos poleas es de
Por una hendidura muy fina se le arrolla otro hilo 10 kg m2. Al dejar el sistema en libertad, se pone en
ideal a una distancia del eje horizontal r = 10 cm, a movimiento espontáneamente. Se pide:
cuyo extremo libre se le aplica una fuerza constante a) ¿En qué sentido se mueven las poleas?
F = 200 N. Calcular: b) Valor de la aceleración con que se mueve cada
a) Momento de inercia del cilindro respecto a un eje una.
que coincida con una generatriz. c) Aceleración angular de las poleas.
b) Aceleración con que sube la masa m. d) Tensión de la cuerda que sostiene la masa de 100
c) Aceleración angular del cilindro. kg cuando el sistema está en movimiento.
d) Tensión del hilo que sostiene la masa. Solución.

Solución.
a) Aplicando el teorema de Steiner,
I = ½ MR2 +MR2 = 3/2 MR2
a) Cuando las poleas están inicialmente en reposo,
los pesos coinciden con las tensiones.
Por tanto T1 = 200 N, y T2 = 1000 N.
El momento que ejerce T1 valdrá
τ 1 = T1 R1 = 200 Nm
El que ejerce T2 valdrá
τ 2 = T2 R2 = 300 N m.
Por tanto, al ser el momento de la fuerza T2 mayor,
la polea girará de modo que la masa M1suba.
b) Podemos plantear dos ecuaciones: b) y c) Planteando la ecuación fundamental de la
T − mg = ma y dinámica a cada masa y a la polea, tendremos:
⎛1 ⎞⎛ a ⎞ 1 T1 − M 1 g = M 1 a1
Fr − TR = Iα = ⎜ MR 2 ⎟⎜ ⎟ = MRa
⎝2 ⎠⎝ R ⎠ 2 ⇒ T1 − M 1 g = M 1αR1 (1)
Que conducen a: M 2 g − T2 = M 2 a 2
⎛ 1 ⎞ ⇒ M 2 g − T2 = M 2αR2
Fr − mgR = a⎜ mR + MR ⎟ . (2)
⎝ 2 ⎠ τ 2 − τ 1 = Iα
Por lo tanto la aceleración a vale:
Fr − mgR 20 − 15,68 ⇒ T2 R2 − T1 R1 = Iα (3)
a= = De las tres ecuaciones obtenemos α :
1 1,6 + 2
mR + mR M 2 gR2 − M 1 gR1
2 α=
= 1,2 m / s2 M 2 R22 + M 1 R12 + I
a 1,2 30 − 20
c) α= = = g = 2,51 rad / s2.
R 0,2 20 + 9 + 10
= 6 rad/s2. La aceleración de cada masa será:
d) T = mg + ma = 8 (9,8 +1,2) a1 = αR1 = 2,51 m/s2,
= 88 N.
a 2 = αR2 = 0,75 m/s2
Ejemplo 11. Dos poleas cuyos radios son 1 m y d) T2 = M 2 g − M 2αR2 = 904,7 N
0,3 m, están acopladas pegada una a la otra en un
plano vertical, formando un bloque que gira Ejemplo 12. Un rollo de 16,0 kg de papel con
alrededor de su eje de rotación común. De la radio R = 18,0 cm descansa contra la pared
garganta de la polea grande pende una masa de 20

9
Cuerpo rígido Hugo Medina Guzmán

sostenido por un soporte unido a una varilla que τ (40,0 − 31,54)(18,0 × 10−2 )
pasa por el centro del rollo. La varilla gira sin α= = .
fricción en el soporte, y el momento de inercia del I (0,260)
papel y la varilla alrededor del eje es de 0,260 kg. = 4,71 rad/s 2
m2. El otro extremo del soporte está unido mediante
una bisagra sin fricción a la pared de modo que el
soporte forma un ángulo de 30,0° con la pared. El Ejemplo 13 Se debe aplicar una sola fuerza
peso del soporte es despreciable. El coeficiente de adicional a la barra de la figura para mantenerla en
fricción cinética entre el papel y la pared es equilibrio en la posición mostrada. Puede
despreciarse el peso de la barra.
μ k = 0,25 . Se aplica una fuerza vertical constante a) Calcule las componentes vertical y horizontal de
F = 40,0 N al papel, que se desenrolla. la fuerza requerida.
a) ¿Qué magnitud tiene la fuerza que la varilla b) ¿Qué ángulo debe formar ésta fuerza con la
ejerce sobre el rollo de papel al desenrollarse? barra?
b) ¿Que aceleración angular tiene el rollo? c) ¿Qué magnitud debe tener?
d) ¿Dónde debe aplicarse?

Solución.
Solución. a) La tensión en el resorte es W2 = 50 N, y la
En el punto de contacto, la pared ejerce una fuerza fuerza horizontal sobre la barra debe equilibrar la
F f de la fricción dirigida hacia abajo y una fuerza componente horizontal de la fuerza que el resorte
ejerce sobre la barra, y es igual a
normal N dirigida a la derecha. Esto es una
(50 N) sen 37° = 30 N, a la izquierda en la
situación donde es cero la fuerza neta en el rodillo,
pero el torque neto no es cero. figura.
La suma de fuerzas verticales La fuerza vertical debe ser
Fvar cosθ = F f + W + F , F f = μk N , 50 cos 37° + 10 = 50 N, arriba
b)
Las fuerzas horizontales
Fvarsenθ = N . ⎛ 50 N ⎞
arctan ⎜⎜ ⎟⎟ = 59°
De aquí tenemos: ⎝ 30 N ⎠
Fvar cos θ = μ k N + F + W c)
Fvarsenθ = N . (30 N) 2 + (50 N) 2 = 58 N.
a) Eliminando N y resolviendo para Fvar da d) Tomando torques alrededor (y midiendo la
distancia de) del extremo izquierdo
W +F 50 x = (40)(5,0 )
Fvar =
cos θ − μk sin θ ⇒ x = 4,0 m
40,0 + (16,0) (9,80) Donde solamente las componentes verticales de las
= = 266 N fuerzas ejercen torques.
cos 30° − (0,25)sen30°
b) Con respecto al centro del rodillo, la barra y la Ejemplo 14. Imagine que está tratando de subir
fuerza normal ejercen el torque cero. una rueda de bicicleta de masa m y radio R a una
La magnitud del torque neto es acera de altura h; para ello, aplica una fuerza
( F − F f ) R , y F f = μk N horizontal F. ¿Qué magnitud mínima de F logra
subir la rueda si la fuerza se aplica
Puede calcularse reemplazando el valor
a) al centro de la rueda?
encontrado para Fvar en cualquiera de las b) ¿En la parte superior de la rueda?
relaciones anteriores; c) ¿En cuál caso se requiere menos fuerza?
F f = μk Fvarsenθ = 33,2 N .
Luego,

10
Cuerpo rígido Hugo Medina Guzmán

Aplicando la segunda ley de Newton en la masa C:


m C g − T = mC a
8 g − T = 8 αR A
Aplicando la segunda ley de Newton de la rotación
en el conjunto giratorio:
TR B = Iα
Solución. Resolviendo el sistema formado:
a) Tome los torques respecto a la esquina superior 8 gR B − TR B = 8αR B2 ⎫
⎬ 8 gRB = 8aRB + Iα
2
de la acera.
→ TR B = Iα ⎭
La fuerza F actúa a una distancia perpendicular 8 gR B 35,28
R−h y ⇒α = = = 0,66 rad/s2
el peso actúa en una distancia perpendicular 8 R B + I 53,18
2

b) a 0 = αR0 = 0,6 m/s2


R 2 − (R − h ) = 2 Rh − h 2 .
2

Igualando los torques para encontrar la fuerza


necesaria mínima, c) ω (4 s ) = α t = 2,65 rad/s

2 Rh − h 2 aN = ω 2 RD = 6,34 m/s2
F = mg .
R−h
b) El torque debido a la gravedad es el mismo, pero EQUILIBRIO ESTÁTICO
→ En el capítulo 5 vimos que para que una partícula
la fuerza F actúa a una distancia perpendicular estuviera en equilibrio estático era suficiente que La
2 R − h, tal que la fuerza mínima es fuerza resultante fuese cero.

F = mg
2 Rh − h 2
.
∑F = 0
2R − h Esta condición también, es necesaria para que un
c) Se requiere menos fuerza que cuando la fuerza se cuerpo rígido este en equilibrio, pero no es
aplica en parte alta de la rueda. suficiente que solamente el centro de masa este en
reposo, el cuerpo puede girar Es necesario que el
Ejemplo 15. Un disco homogéneo A gira alrededor momento de: fuerzas o torque con respecto al centro
del eje y bajo la acción de la masa C unida a una de masa sea nulo.

∑τ
cuerda que pasa por una polea sin peso ni
rozamiento enrollada alrededor del tambor =0
cilíndrico macizo B, solidaria del disco A. A éste A continuación desarrollaremos algunos ejemplos
está unida una masa puntual D, como indica la de aplicación. En muchos de ellos la fuerza de la
figura. Las masas A, B, C y D son respectivamente gravedad ejercida sobre las diversas partes de un
65, 15, 8 y 4 kg. Se supone que la cuerda cuerpo puede sustituirse por una sola fuerza, el peso
permanece siempre horizontal. Calcular: total actuando en el centro de gravedad.
a) Aceleración angular del disco. Si la aceleración de la gravedad no varía a lo largo
b) Aceleración tangencial de D. del cuerpo, el centro de gravedad coincide con el
c) Aceleración normal de D, 4 s después de partir centro de masa.
del reposo.
Ejemplo 16. Demostrar que cuando un cuerpo está
en equilibrio y el torque con respecto al centro de
masa es cero, el torque con respecto a cualquier
punto también es cero.
Solución.

Solución.
a) Calculemos en primer lugar el momento de
inercia del sistema A-B-D. En la figura
1 1 1 →
I = m A R A2 + m B RB2 + m D RD2 rO es el vector del centro de masa a O
2 2 2
= 51,56 kg m2

11
Cuerpo rígido Hugo Medina Guzmán


ri es el vector del centro de masa al punto donde

actúa Fi .

rOi es el vector del punto O al punto donde actúa

Fi .
De la figura vemos:
→ → →
Solución.
ri = rO + rOi a) Sustituir la fuerza vertical dada por otra igual
El torque total alrededor de O es paralela cuya línea de acción pase por el centro de
masa.
→ → →
⎛→ → ⎞ →
τ O = ∑ r Oix × F i = ∑i ⎜⎝ ri − rO ⎟⎠ × F i =
i
→ → → → → → →
∑r× F − ∑r × F
i
i i
i
O i = τ CM − ∑ rO × F i
i

Como rO es constante
→ → → →
τ O = τ CM − ∑ rO × F i b) Hacer girar el plano del par, hasta desplazarlo
i hasta la línea A B.

Para un cuerpo en equilibrio ∑ Fi = 0
→ →
tal que τ O = τ CM

Si τ CM = 0 , el torque alrededor de cualquier
punto debe ser cero y viceversa.

Ejemplo 17. Par de fuerzas. Dos fuerzas iguales y


opuestas que actúan en la figura siguiente se c) Se cambian los módulos de las fuerzas a F’ de tal
denominan par de fuerzas, Según se indica modo que:
a
F ' b = Fa ⇒ F ' = F
b

F es el valor de cualquiera de las fuerzas y


d = ( x 2 − x1 ) es la distancia entre ellas.
El momento o torque producido por estas fuerzas
con respecto a O es:
τ O = Fx 2 − Fx1 = F ( x 2 − x1 ) = Fd
Este resultado no depende de la selección del punto Ejemplo 19. Sobre una placa sólida actúan cuatro
O, el momento producido por un par es el mismo fuerzas de módulos
respecto a cualquier punto del espacio. F1 = 28,3 N, F2 = 60 N, F3 = 20 N y F4 = 50 N.
Como se indican en la figura. Hallar la tuerza
Ejemplo 18. Una fuerza vertical F que actúa en A. resultante sobre la placa y determinar su línea de
en el sólido rectangular mostrado en la figura, acción.
queremos sustituirla por otra cuya línea de acción
pasa por el centro de masa más un par de fuerzas
que actúen horizontalmente aplicados en A y B.

12
Cuerpo rígido Hugo Medina Guzmán


r = xiˆ + yˆj
Tal que

( ) (30iˆ + 40 ˆj )
→ → →
τ = 6k̂ = r × F = xiˆ + yˆj ×
= (40 x − 30 y )kˆ
De aquí:
(40 x − 30 y ) = 6
⇒ 20 x − 15 y = 3
Solución. Esta es la ecuación de la línea de acción de la
Utilizando el cuadriculado obtenemos: fuerza; si esta tuerza a de situarse en algún punto

del borde inferior de la placa, y = - 0,2 m..
r1 = −0,2iˆ − 0,2 ˆj , Obtenemos
→ ⎛ 2⎞ ⎛ 2⎞ 3 + 15 y 3 + 15(− 0,2)
F1 = 28,3⎜⎜ ⎟iˆ + 28,3⎜

⎟ˆ
⎜ 2 ⎟ j = 20i + 20 j
ˆ ˆ x= = =0
⎝ 2 ⎠ ⎝ ⎠ 20 20
→ →
La figura siguiente muestra la fuerza resultante:
r2 = 0,1iˆ − 0,2 ˆj , F2 = 60 ˆj
→ →
r3 = 0,2iˆ + 0,1 ˆj , F3 =− 20iˆ
→ →
⎛3⎞ ⎛3⎞
r4 = −0,1iˆ + 0,2 ˆj , F4 = 50⎜ ⎟iˆ − 50⎜ ⎟ ˆj
⎝4⎠ ⎝4⎠
= 30iˆ − 40 ˆj
La fuerza resultante es
→ → → → →
F = F1 + F2 + F3 + F4
= (20 − 20 + 30 )&i& + (20 + 50 − 40 ) ˆj
Ejemplo 20. Se tiene una escalera dé masa M y
largo L apoyada contra la pared .No hay fricción en
= (30iˆ + 40 ˆj )N la pared y el coeficiente de fricción del piso es μ .
¿Cuál es el mínimo ángulo de inclinación para que
El torque resultante respecto al centro de masa es la no comience a resbalar?
suma de los torques individuales.
→ → → → →
τ = τ1+τ 2 +τ 3 +τ 4
Siendo:
→ → →
τ 1 = r1 × F1
( ) (
= − 0,2iˆ − 0,2 ˆj × 20iˆ + 30 ˆj = 0 )
→ → →
τ 2 = r2 × F2
= (0,1iˆ − 0,2 ˆj )× (60 ˆj ) = 6kˆ Solución.
La figura siguiente muestra el diagrama del cuerpo
→ → →
τ 3 = r3 × F3 libre de la escalera.

= (0,2iˆ + 0,1 ˆj )× (− 20 ˆj ) = 2kˆ


→ → →
τ 4 = r4 × F4
(
= − 0,1iˆ + 0,2 ˆj × ) (20iˆ − 40 ˆj ) = −2kˆ
Reemplazando:

τ = 6k̂ Nm

Para determinar la línea de acción de la tuerza,


consideremos que el punto de aplicación de la
fuerza resultante es: Condición para que el centro de masa no acelere:

13
Cuerpo rígido Hugo Medina Guzmán

∑F x = 0 = N x − μN y , R es a reacción de la pared.
Como el sistema está en equilibrio
∑F = 0 = Mg − N y →
y
De aquí obtenemos: ∑F = 0
N y = Mg , N x = μN y = μMg ⎧⎪∑ Fx = R cos α − T cos θ = 0

Condición de no rotación: ⎪⎩∑ Fy = Rsenα − Tsenθ − Mg − mg = 0
La suma de momentos de fuerza con respecto al →
centro de masa es cero.
L L L
Con ∑τ = 0 alrededor de cualquier punto.
N y cos θ − μN y senθ − N x senθ = 0 Tomamos momentos con respecto a O.
2 2 2 L
Reemplazando las fuerzas: TLsenθ − mg − Mgx = 0
L L L 2
Mg cos θ − μMg senθ − μMg senθ = 0 De esta última ecuación obtenemos
2 2 2
⇒ 2μsenθ = cos θ L
TLsenθ − mg
⎛ 1 ⎞ x= 2
⇒ θ = tan −1 ⎜⎜ ⎟⎟ Mg
⎝ 2μ ⎠ Si T = Tm obtenemos el valor máximo de x.
Otra forma: Si estuviéramos interesados en conocer R, sería
En lugar de tomar el centro de masa como origen mejor tomar momentos con respecto al otro
tomemos extremo inferior de la escalera. extremo.
Tomando momentos con respecto a este punto.
L Ejemplo 22. Un albañil de 75 kg camina sobre un
Mg cos θ − N x Lsenθ = 0 tablón de 3 m de largo y 80 kg apoyado sobre dos
2 vigas distantes 2 m, tal como indica la figura. ¿Cuál
Reemplazando el valor de Nx:
es la máxima distancia x que puede recorrer, sin
L
Mg cos θ − μMgLsenθ = 0 que caiga?
2
⎛ 1 ⎞
⇒ θ = tan −1 ⎜⎜ ⎟⎟
⎝ 2μ ⎠
Obtenemos la misma respuesta porque no importa
con respecto a que eje tomemos el torque.

Ejemplo 21. Una viga de masa m se empotra a la Solución.


pared como se muestra en la figura y se sujeta por Para que el tablón gire, el torque del peso del
medio de un alambre. Si la tensión en el alambre albañil respecto del punto O, más el torque del peso
excede Tm el alambre se rompe. ¿Para qué valor de de la parte de tablón que sobresale, debe ser mayor
o igual que el torque del peso de la parte de tablón
x, el alambre se romperá por una masa M colocada apoyada entre las vigas:
sobre la viga? Llamando λ a la densidad lineal del tablón:
M
λ= , haciendo d = 2 m, L = longitud del
L
tablón, M = masa tablón, m = masa albañil
tendremos:

mgx + λ (L − d )g
(L − d ) = λdg d
2 2
Solución.
La figura muestra el diagrama del cuerpo libre del
[ ]
⇒ mx + λ d 2 − (L − d )2 = M (2 Ld − L2 ) ,
2L
sistema viga-masa. M
⇒x= (2d − L ) = 0,53 m
2m
Ejemplo 23. Un baúl de masa M se empuja sobre
un suelo con coeficiente de rozamiento
a) Qué fuerza F se ejerce si el baúl se mueve con
aceleración constante a?
b) ¿Si el baúl se mueve con velocidad constante?

14
Cuerpo rígido Hugo Medina Guzmán

c) ¿Qué fuerza se necesita para inclinar el baúl? La componente horizontal de la fuerza ejercida en
la barra por la bisagra debe equilibrar la fuerza

F aplicada, y así tiene magnitud 120,0 N y es
hacia la izquierda.
Tomando torques alrededor del punto A
(120,0 N)(4,00 m) + FV (3,00 m)
Solución. La componente vertical es – 160 N, el signo menos
La figura siguiente muestra el diagrama del cuerpo indica una componente hacia abajo, ejerciendo un
libre del baúl. torque en una dirección opuesta a la de la
componente horizontal.
La fuerza ejercida por la barra en la bisagra es igual
en magnitud y contrario en la dirección a la fuerza
ejercida por la bisagra en la barra

Ejemplo 25. La caja es arrastrada sobre una


superficie horizontal con rapidez constante por una
fuerza. El coeficiente de fricción cinética es de 0,35.
a) Aplicando la segunda ley de Newton. a) Calcule la magnitud de F.
b) Determine el valor de h con el cual la caja
∑F x = F − μ k ( N 1 + N 2 )Ma , comience a volcarse.
∑F y = N 1 + N 2 − Mg = 0
Resolviendo las ecuaciones:
F = M (a + μ k g )
b) En el caso que el baúl va con velocidad constante
a = 0 y F = Mμ k g
c) Para analizar la inclinación del baúl tenemos que
Solución.
escribir la ecuación de momentos con respecto al
borde delantero, sin rotación α = 0, luego a) F = F f = μ k Ν = μ k mg
b
∑τ = −bN
2
− hF + Mg = 0 = (0,35)(30,0 kg)(9,80 m s )
1
2 = 103 N
Cuando el baúl empiece a inclinarse, empezará a b) Con respecto al borde delantero de la caja.
rotar en el sentido horario y N1 = 0, de aquí: El brazo de palanca del peso es
bMg 0,250
F= = 0,125 m
2h 2
y la aceleración: El brazo de palanca h de la fuerza aplicada es
F ⎛ b ⎞ entonces
a= − μk g = ⎜ − μk ⎟g mg 1
M ⎝ 2h ⎠ h = (0,125) = (0,125)
F μk
Ejemplo 24. El extremo A de la barra AB de la 0,125
figura descansa en una superficie horizontal sin = = 0,36 m.
fricción, y el extremo B tiene una articulación. Se
0,35
ejerce en A una fuerza horizontal F de magnitud
120 N. Desprecie el peso de la barra. Calcule las TRABAJO Y ENERGIA EN ROTACIÓN.
componentes horizontal y vertical de la fuerza Consideremos un cuerpo que gira alrededor de un
ejercida por la barra sobre la articulación en B. eje tal como se muestra en la figura

Solución. La energía cinética de un elemento de masa dm que


gira a una distancia r del eje de rotación es:

15
Cuerpo rígido Hugo Medina Guzmán

1 1 2 1 2
dK = dmv 2 , v = ωr = Iω2 − Iω1
2 2 2
1 = K 2 − K 1 = ΔK
⇒ dK = dmω 2 r 2
2 “E1 trabajo neto realizado por las fuerzas externas
Integrando. al hacer girar un cuerpo rígido alrededor de un eje
1 2 2 fijo es igual al cambio en la energía cinética de
K = ∫ dK = ∫ ω r dm rotación”.
M 2
como ω es constante. Por la analogía que existe entre las expresiones para
1 el movimiento lineal y el movimiento angular,
K = ∫ dK = ω 2 ∫ r 2 dm podemos decir que un torque será conservativo a
2 M
condición que exista una función potencial
El término integral es el momento de inercia del
cuerpo con respecto al eje de rotación U = U (θ ) de tal modo que el trabajo efectuado por

1 2
K= Iω τ , cuando el cuerpo sufre un desplazamiento
2
Para relacionar la energía cinética, al trabajo
(
angular (θ 2 − θ 1 ) es la diferencia U (θ1 ) − U (θ 2 ) .)
efectuado sobre el cuerpo por un torque τ . Así pues se deduce que:
Supongamos que se aplica una fuerza externa única U (θ1 ) − U (θ 2 ) = K 2 − K 1
F, que actúa en el punto P del cuerpo.
ó K 1 + U (θ1 ) = K 2 + U (θ 2 ) = constante
Cuando el sistema no es conservativo
( ) (
W NO CONSERVATI VO = K 1 + U (θ1 ) − K 2 + U (θ 2 ) )
POTENCIA
La rapidez con que se realiza este trabajo es:
dW dθ
=τ = τω
dt dt

Expresión que corresponde a la potencia
El trabajo realizado por F a medida que el cuerpo instantánea.
gira recorriendo una distancia infinitesimal P = τω
ds = rdθ en un tiempo dt es:
→ → Ejemplo 26. Para la barra giratoria, calcular su
dW = F ⋅ d s = Fsenφ rdθ rapidez angular y la rapidez lineal de su centro de
Como Fsenφ r es el torque de la fuerza F masa y del punto mas bajo de la barra cuando está
vertical.
alrededor del origen se puede escribir el trabajo
realizado para la rotación infinitesimal como:
dW = τ dθ
Cuando el cuerpo gira en torno a un eje fijo bajo la
acción de un torque. El cambio de su energía
cinética durante el intervalo dt se puede expresar
como:
dK d ⎛1 ⎞ Solución.
dK = dt = ⎜ Iω 2 ⎟dt
dt dt ⎝ 2 ⎠

= Iω dt = Iωα dt = Iα ω dt
dt
Como
τ = Iα y dθ = ωdt
Obtenemos:
dK = τ dθ = dW
Si se íntegra esta expresión se obtiene el trabajo
total
θ2 ω2
W1→2 = ∫ τ dθ = ∫ω Iωdω Usando el principio de conservación de la energía,
θ1 1
considerando que la energía potencial se calcula

16
Cuerpo rígido Hugo Medina Guzmán

respecto al centro de masa y la energía cinética es 1 1 1


de rotación: m1v 2 + m2 v 2 + Iω 2 + m1 gH
=
2 2 2
Ei = E f ⇒ K i + U gi = K f + U gf
1⎛ I ⎞
Cuando la barra esta inicialmente horizontal no ⇒ ⎜ m1 + m2 + 2 ⎟v 2 = (m2 − m1 )gH
tiene Ki y cuando esta vertical tiene solo Kf, 2⎝ R ⎠
entonces: Donde se ha usado la relación v = R ω, despejando
L 1 1⎛1 ⎞ v se obtiene:
Mg = Iω 2 = ⎜ ML2 ⎟ω 2 ⇒
2 2 2⎝3 ⎠ 2(m2 − m1 )gH
v=
I
3g m1 + m2 + 2
ω= R
L
Para calcular la rapidez del centro de masa, se usa:
Ejemplo 28. Sobre un cilindro homogéneo de
L 1 radio R y masa M. tiene El cual tiene libertad de
vcm = rω = ω = 3gL
2 2 girar sin fricción sobre un eje, como se muestra en
En el punto mas bajo la rapidez es la figura. Si se le aplica en su borde una fuerza
tangencial de magnitud F.
v = 2v cm = 3 gL a) ¿Cuál es la aceleración angular α del cilindro?
b) ¿Cual es la velocidad angular y la energía
Ejemplo 27. Para el sistema de la figura, las masas cinética del cilindro al tiempo t?
tiene momento de inercia I en torno a su eje de c) ¿qué cantidad de trabajo aplica la fuerza durante
rotación, la cuerda no resbala en la polea y el este intervalo t?.
sistema se suelta desde el reposo.
Calcular la rapidez lineal de las masas después que
una ha descendido H y la rapidez angular de la
polea.

Solución.
El momento de inercia del cilindro en torno a su eje
es:
1
I= MR 2
2
τ
a) Con τ = Iα ⇒ α= , τ = F0 R
I
F0 R 2F
tenemos α =
Solución.
= 0
MR 2 MR
Como no hay roce en la polea, se conserva la 1
energía, que aplicada a cada masa m1 y m2,
suponiendo que m2 se encuentra inicialmente en la 2
parte superior del sistema, es: b) Siendo α constante
Ei = E f ω = ω0 + α t
2 F0
Si ω 0 = 0 , ω = α t , ω = t
MR
La energía cinética:
2
1 1⎛1 ⎞⎛ 2 F ⎞
K = Iω 2 = ⎜ MR 2 ⎟⎜ 0 t ⎟
2 2⎝2 ⎠⎝ MR ⎠
F2 2
= 0 t
M
c) El trabajo realizado
F02 2
⇒ K 1i + K 2i + U 1i + U 2i W = ΔK = K 2 − K 1 = t −0
M
= K1 f + K 2 f + K p + U 1 f + U 2 f F02 2
= t
⇒ 0 + m2 gH M

17
Cuerpo rígido Hugo Medina Guzmán

ω2R 1
Otra forma de calcular es: Luego W = MgR = Mω 2 R 2
θ2 4g 4
W = ∫ τ dθ , τ = F0 R (constante) Otra forma de evaluar el trabajo es por la
θ1

W = F0 R(θ 2 − θ1 ) = F0 RΔθ
conservación de la energía.
1 2
2 F0 Iω − 0
W = ΔK = K 2 − K 1 =
Con α = , ω0 = 0 2
MR 1⎛1 2⎞ 2 1
= ⎜ MR ⎟ω = Mω R
2 2
1 1 ⎛ 2F ⎞ F
⇒ θ = αt 2 = ⎜ 0 ⎟t 2 = 0 t 2 2⎝2 ⎠ 4
2 2 ⎝ MR ⎠ MR
Finalmente Ejemplo 30. Una plataforma cilíndrica uniforme
⎛ F ⎞ F 2 de 180 kg de masa y 4,5 m de radio se frena de 3,2
2
W = F0 R⎜ 0 t 2 ⎟ = t 0
rev/s al reposo en 18 s cuando se desconecta el.
⎝ MR ⎠ M motor. Calcular la potencia de salida del motor (hp)
para mantener una velocidad constante de 3,2 rev/s.
Ejemplo 29. Un carrete de hilo delgado tiene radio Solución.
R y masa M. Si se jala el hilo de tal modo que el Como primer paso debemos conocer cuál es el
centro de masa del carrete permanezca suspendido torque de frenado que tenemos que vencer para
en el mismo lugar. mantener la velocidad constante, ese torque lo
a) ¿Qué fuerza se ejerce sobre el carrete? calcularemos de la siguiente manera:
b) ¿Cuánto trabajo se habrá realizado cuando el τ frenado = Iα frenado
carrete gira con velocidad angular ω ?
1
Solución. I= MR 2 .
La figura muestra al carrete suspendido. 2
Δω ω 2 − ω1 ω
α frenado = = =−
Δt t 2 − t1 t
1 ω MR 2ω
τ frenado = MR 2 =
2 t 2t
La potencia es:
MR 2ω 2
P =τ ω =
El carrete solo tiene movimiento circular ya que 2t
está en equilibrio vertical Siendo
Aplicando las leyes de Newton: M =180 kg, R = 4,5 m,
rev 2π rad
∑F = 0 y ⇒ T − Mg = 0 ω = 3,2
s rev
= 6,4π
rad
s
,

∑τ = Iα ⇒ TR = Iα t = 18 s.
1
Como I = MR , obtenemos:
2
P=
(180)(4,5) (6,4π )
2 2
= 40889,73 W
2 2(18)
1 Como 1 hp = 735,5 W
MgR = MR 2α
2
2g P = 55,6 hp
yα =
R Ejemplo 31. Se sujeta una masa M a una cuerda
a) La fuerza que se ejerce sobre el carrete es ligera enrollada alrededor de una rueda de momento
T = Mg de inercia I y radio R. Hallar La tensión de la
b) Como el trabajo realizado es: cuerda, la aceleración y su velocidad después de
haber descendido una distancia h desde el reposo.
W = τ Δθ , donde τ = TR = MgR 2 Resolver desde el punto de vista de energía.
α constante ⇒ ω = ω 0 + 2αΔθ
2 2
Siendo
Si ω 0 = 0 ⇒ ω 2 = 2αΔθ y
ω2 ω2R
Δθ = =
2α 4g

18
Cuerpo rígido Hugo Medina Guzmán

Solución.
Por el principio de conservación de la energía Etotal
= constante
Al inicio del movimiento toda la energía es
potencial, si consideramos como nivel cero el Las masas M1 y M2 inicialmente están en reposo en
indicado en la figura (a).
la posición y = 0 , después de soltarlas una sube y
Ei = Mgh la otra baja como muestra la figura.
Las masas estarán moviéndose con velocidad v la
Polea tendrá una velocidad angular ω .
Como no hay rozamiento por la conservación de la
energía
E1 = E 2
1 1
0 = + M 1v 2 + M 2 v 2 +
2 2
1 2
Iω + M 1 gy − M 2 gy
2
v 1
La energía final es pura energía cinética, de la nasa Siendo ω = , I = MR , tenemos:
2
M con velocidad v antes de chocar y el disco con R 2
momento de Inercia I con velocidad angular 1⎛ M⎞ 2
ω = v R , figura (b). ⎜ M 1 + M 2 + ⎟v = (M 1 − M 2 )gy
2⎝ 2 ⎠
2(M 1 − M 2 )
2
1 1 1 1 ⎛v⎞
Ef = Mv 2 + Iω 2 = Mv 2 + I ⎜ ⎟ ⇒ v2 = gy
2 2 2 2 ⎝R⎠ ⎛ M⎞
v ⎛
2
1 ⎞ ⎜ M1 + M 2 + ⎟
= ⎜M + 2 ⎟ ⎝ 2 ⎠
2 ⎝ R ⎠ Para un movimiento uniformemente acelerado
Como E i = E f v 2 = 2ay
Comparando:
v2 ⎛ 1 ⎞
Mgh = ⎜ M + 2 ⎟ (M 2 − M 1 )
2 ⎝ R ⎠ a= g
(M 2 + M 1 + M 2 )
2M
y v = gh
2

M +1 R2 Ejemplo 33. Una canica sólida uniforme de radio r


parte del reposo con su centro de masa a una altura
2M h sobre el punto más bajo de una pista con un rizo
⇒ v= gh
M +1 R2 de radio R. La canica rueda sin resbalar. La fricción
de rodamiento y la resistencia del aire son
despreciables.
Ejemplo 32. Resolver la máquina le Atwood
a) ¿Qué valor mínimo debe tener h para que la
utilizando Conceptos de trabajo y energía,
canica no se salga de la pista en la parte superior del
Solución.
rizo? (Nota: r no es despreciable en comparación
con R.)
b) ¿Qué valor debe tener h si la pista está bien
lubricada, haciendo despreciable la fricción?

19
Cuerpo rígido Hugo Medina Guzmán

g (h − 2 R + r ) =
1 2 1 2 7 2
v + v = v ⇒
2 5 10
g (h − 2 R + r ) = v 2
7
10
Reemplazando el valor de v :
2

g (h − 2 R + r ) = g (R − r ) ⇒
7
Solución. 10
a) De a a B, la distancia que la canica ha caído es
h − 2 R + r = (R − r ) ⇒
7
y = h − (2 R − r ) = h + r − 2 R. 10
h = 2 R − r + (R − r )
7
10
= 2R − r +
7
(R − r )
10

27 17
= R− r
10 10
El radio de la trayectoria del centro de masa de la
= 2,7 R − 1,7 r
canica es R − r , .
b) En ausencia de fricción no habrá rotación.
Luego:
1 2
mgy = mv
2
Sustituyendo las expresiones para y y v en
2

términos de los otros parámetros da

h − 2R + r =
1
(R − r )
2
La condición para que la canica permanezca en la Resolviendo obtenemos
pista es
v2 5 3
∑ Fr = mac ⇒ − mg = −m (R − r )
⇒ h=
2
R− r.
2
v 2 = g ( R − r ). Ejemplo 34. La figura muestra tres yoyos idénticos
La velocidad se determina del teorema del trabajo - que inicialmente están en reposo en una superficie
energía, horizontal. Se tira del cordel de cada uno en la
1 2 1 2 dirección indicada. Siempre hay suficiente fricción
mgy = mv + Iω para que el yoyo ruede sin resbalar. Dibuje un
2 2 diagrama de cuerpo libre para cada yoyo. ¿En qué
Se tiene: dirección girará cada uno? Explica tus respuestas
y = h − (2 R − r )
v
ω=
r
Se sabe que para una esfera
2 2
I= mr
5 →
Reemplazando estos valores en la ecución de la Solución. En el primer caso, F y la fuerza de la
energía: fricción actúan en direcciones opuestas, y la fuerza
2
1 2 1 ⎛ 2 2 ⎞⎛ v ⎞ de fricción tiene el torque mayor que hace rotar el
mg (h − 2 R + r ) = mv + ⎜ mr ⎟⎜ ⎟ yo-yo a la derecha. La fuerza neta a la derecha es la
2 2⎝5 ⎠⎝ r ⎠ diferencia F − F f , tal que la fuerza neta es a la

derecha mientras que el torque neto causa una
rotación a la derecha.

20
Cuerpo rígido Hugo Medina Guzmán

Para el segundo caso, el torque y la fuerza de 1 2 1 2


fricción tienden a dar vuelta al yoyo a la derecha, y mgh = mv + Iω
el yo-yo se mueve a la derecha. 2 2
En el tercer caso, la fricción tiende a mover al yo- Rodar sin resbalar significa
yo a la derecha, y puesto que la fuerza aplicada es v 2 2
vertical, el yoyo se mueve a la derecha.
ω= , I = mr
r 5
1 2 1 2
Tal que Iω = mv
2 5
7
mgh = mv 2
10
7v 2 7(17,82 m/s)
⇒h= =
10 g 10(9,80 m/s 2 )
Ejemplo 35.
= 23 m
Una canica uniforme baja rodando sin resbalar por
1 2 1 2
el trayecto de la figura, partiendo del reposo. b) Iω = mv , Independiente de r.
a) Calcule la altura mínima h que evita que la 2 5
canica caiga en el foso. c) Todo es igual, excepto que no hay el término de
b) El momento de inercia de la canica depende de energía rotacional cinética en K:
su radio. Explique por qué la respuesta a la parte (a) 1 2
no depende del radio de la canica. K= mv
c) Resuelva la parte (a) para un bloque que se 2
desliza sin fricción en vez de una canica que rueda. 1
Compare la h mínima en este caso con la respuesta mgh = mv 2
a la parte (a).
2
v 2
h= = 16 m .
2g
Comparado con la altura de la parte (a), 16 /23 =
0,7, es el 70 %.

Ejemplo 36. Una esfera sólida uniforme rueda sin


resbalar subiendo una colina, como se muestra en la
figura. En la cima, se está moviendo
horizontalmente y después se cae por un acantilado
vertical.
a) ¿A qué distancia del pie del acantilado cae la
Solución.
a) Encuentre la velocidad v que necesita la canica esfera y con qué rapidez se está moviendo justo
en el borde del hoyo para hacerlo llegar a la tierra antes de tocar el suelo?
plana en el otro lado. b) Observe que, al tocar tierra la esfera, tiene mayor
La canica debe viajar 36 m horizontalmente rapidez de traslación que cuando estaba en la base
mientras cae verticalmente 20 m. de la colina. ¿Implica esto que la esfera obtuvo
Use el movimiento vertical para encontrar el energía de algún lado? Explique.
tiempo.
Tome + y hacia abajo.
v 0 y = 0, a y = 9,80 m/s 2 , y − y0 = 20 m, t = ?
1
y − y 0 = v0 y t + a y t 2 ⇒ t = 2,02 s
2
Luego x − x 0 = v 0 x t ⇒ v 0 x = 17,82 m/s.
Solución.
a) Use la conservación de la energía para encontrar
Utilice la conservación de la energía, donde el la velocidad v 2 de la bola momentos antes que
punto 1 está en el punto de partida y el punto 2 está
salga de la parte alta del acantilado. Sea el punto 1
en el borde del hoyo, donde v = 17,82 m/s.
en la base de la colina y el punto 2 en la cima de la
Haga y = 0 en el punto 2, tal que colina.
y 2 = 0 e y1 = h Tome y = 0 en la base de la colina, tal que
K1 + U 1 = K 2 + U 2 y1 = 0 e y2 = 28,0 m.
K1 + U 1 = K 2 + U 2

21
Cuerpo rígido Hugo Medina Guzmán

1 2 1 2 1 1 en un eje sin fricción que pasa por su centro. Un


mv1 + Iω1 = mgy2 + mv22 + Iω22 bloque de masa M se suspende del extremo libre del
2 2 2 2 hilo. El hilo no resbala en la polea, y el cilindro rue-
Rodar sin resbalar significa ω = v r y da sin resbalar sobre la mesa. Si el sistema se libera
del reposo, ¿qué aceleración hacia abajo tendrá el
1 2 1⎛1 2 ⎞ 1
Iω = ⎜ mr ⎟ (v / r ) 2 = mv 2 bloque?
2 2⎝5 ⎠ 5
7 7
mv12 = mgy 2 + mv 22
10 10
10
v 2 = v12 − gy 2 = 15,26 m s
7
Considere el movimiento de proyectil de la bola,
después de salir de la cima del acantilado hasta Solución.
justo antes de tocar tierra. Tome + y hacia abajo. Hacer este problema usando la cinemática implica
Utilice el movimiento vertical para encontrar el cuatro incógnitas (seis, contando las dos
tiempo en el aire: aceleraciones angulares), mientras que usando
v0 y = 0 , a y = 9,80 m/s 2 consideraciones de la energía se simplifican los
cálculos.
y − y 0 = 28,0m , t = ? Si el bloque y el cilindro ambos tienen velocidad v,
la polea tiene velocidad angular v/R y el cilindro
1
y − y 0 = v 0 y t + a y t 2 ⇒ t = 2,39 s tiene velocidad angular v/2R, la energía cinética
2 total es
Durante este tiempo la bola viaja horizontalmente 1⎡ M ( 2 R) 2 MR2 ⎤
x − x 0 = v0 x t = (15,26 m s )(2,39 s ) = 36,5 m. K = ⎢Mv2 +
2⎣ 2
(v 2 R ) 2 +
2
(v R) 2 + Mv2 ⎥

Justo antes de tocar tierra,
v y = v0 y + gt = 23,4 m/s y 3
= Mv 2 . (1)
v x = v0 x = 15,26 m/s 2
Esta energía cinética debe ser el trabajo hecho por
v = v x2 + v y2 = 28,0 m s la gravedad; si la masa que cuelga desciende una
distancia y,
K = Mgy. (2)
De (1) y (2):
2
v2 = gy
3
Para aceleración constante
b) En la base de la colina, v 2 = 2ay,
v 25,0 m s Por comparación de las dos expresiones obtenemos:
ω= = g
r r a=
La razón de la rotación no cambia mientras la bola 3
está en el aire, después de dejar la parte alta del
acantilado, tal que momentos antes de tocar tierra Ejemplo 38. Una barra de largo 2L y masa M está
15,3 m/s articulada en un extremo a un punto fijo O,
ω= inicialmente en reposo y horizontal. Si ella se
r suelta, comienza a rotar respecto a la articulación
La energía cinética total es igual en la base de la
bajo el efecto del peso de la barra. Determine la
colina y momentos antes de tocar tierra, pero
reacción en la articulación y la velocidad angular de
momentos antes de tocar tierra poco de esta energía
la barra en función del ángulo que ella ha girado.
es energía cinética rotatoria, así que la energía
cinética de traslación es mayor.

Ejemplo 37. Un cilindro sólido uniforme de masa


M y radio 2R descansa en una mesa horizontal. Se
ata un hilo mediante un yugo a un eje sin fricción
que pasa por el centro del cilindro de modo que éste
puede girar sobre el eje. El hilo pasa por una polea Solución.
con forma de disco de masa M y radio R montada

22
Cuerpo rígido Hugo Medina Guzmán

a) Momento de inercia de la barra con respecto a un


Por conservación de energía tenemos que extremo
•2 1
M (2 L ) θ − Mgsenθ = 0
11 2
I A = ML2
23 3
Luego la velocidad angular de la barra es: Por conservación de energía.
•2 3g •3g L 1⎛1 2⎞ 2
θ = senθ ⇒ θ = senθ Mg = ⎜ ML ⎟ω ⇒
2L 2L 2 2⎝3 ⎠
d2 3g
Además − R H = M L cos θ , ω=
dt 2 L
d2 L 1
RV − Mg = M 2 (− Lsenθ ) vCM = ω = 3gL
dt 2 2
Entonces b) La aceleración angular en dicho instante.
L
Mg
d ⎛ ⎞ • 2 τA 2 = 3g
R H = ML
1
⎜ senθ θ ⎟ α= =
2 senθ dθ ⎝ ⎠ I A 1 ML2 2 L
3
1 d ⎛ 2 3 g ⎞
= ML ⎜ sen θ senθ ⎟ Ejemplo 40. Una barra de longitud 2L y masa M se
2 senθ dθ ⎝ 2L ⎠ coloca sobre un plano horizontal liso. Si la barra es
tirada por una fuerza constante F, inicialmente
9 perpendicular a la barra y aplicada en un extremo,
= MLsenθ cos θ la barra comienza a moverse sobre el plano. La
4 fuerza se mantiene aplicada a ese mismo extremo
d2 manteniendo su dirección original. Determine una
RV = Mg − M 2 (− Lsenθ ) ecuación para el ángulo que gira la barra en función
dt del tiempo.
1 d ⎛ 3g ⎞
= Mg − ML ⎜ cos θ senθ ⎟
2

2 cos θ dθ ⎝ 2L ⎠
5 9
= Mg − Mg cos 2 θ
2 4
Solución.
Ejemplo 39. Una barra de longitud L y masa M se
coloca verticalmente sobre un plano horizontal liso,
en reposo. Si ella es perturbada levemente comienza
a caer. Determine:
a) La velocidad del centro de masa de la barra justo
cuando ella se coloca horizontal.
b) La aceleración angular en dicho instante.

El torque respecto al centro de masa conduce a


1 •
FLsenθ = ML2 θ
3
• 3F
⇒ θ= senθ
L
Ejemplo 41. Una barra de longitud L y masa M
Solución. puede oscilar libremente en torno a uno de sus

23
Cuerpo rígido Hugo Medina Guzmán

extremos que se mantiene fijo, bajo la acción de su puramente energía cinética.


peso. Escriba la ecuación diferencial para el ángulo 1 2 1⎛1 ⎞ 1
que ella gira. K= Iω = ⎜ MR 2 ⎟ω 2 = MR 2ω 2
Solución. 2 2⎝2 ⎠ 4
Por conservación de energía.
1 2 1 2kθ 2
kθ = MR ω ⇒ ω =
2 2 2

2 4 MR 2
2kθ 2
Finalmente ω =
MR 2

TRASLACIONES Y ROTACIONES
COMBINADAS
Hasta ahora solo hemos tomado en consideración la
Por conservación de energía
rotación del cuerpo en torno a un eje fijo en el
11 •2 L espacio.
E= ML2 θ − Mg cos θ La finalidad de esta sección es estudiar el caso en
23 2
que el eje de rotación si acelera también vamos a
Derivando respecto al tiempo presentar tres métodos analíticos de resolver este
caso.
1 • •• L•
ML2 θ θ + Mg θ senθ = 0 Primer método
3 2 Aplicamos la segunda ley de Newton para
Finalmente traslación relativa ejes no rotantes a través del
••
3g centro de masa. Para ilustrar este método y los otros
θ+ senθ = 0 también, consideremos un cuerpo de radio R, masa
2L M y momento de inercia respecto a su entro masa I,
al que se le obliga a rodar sin deslizamiento a lo
Ejemplo 42. Un péndulo de torsión consiste en un largo de una superficie horizontal por medio de una
disco uniforme de masa M y radio R suspendido de fuerza F que actúa en su centro de masa, La tuerza
una barra delgada y vertical de masa despreciable y
de fricción F f y la reacción N actúan tal como se
que puede torcerse al dar vuelta al disco alrededor
de su eje, como se indica en la figura. La barra tiene muestra en la figura siguiente.
una Constante de elasticidad torsional k.
inicialmente se hace girar el disco un ángulo θ
respecto del equilibrio y luego se le suelta desde el
reposo. Determinar su velocidad de rotación cuando
llega nuevamente a la posición de equilibrio.

EL cuerpo se mueve con una aceleración horizontal


a que es la que corresponde a su centro de masa, y
a su vez rota con aceleración angular α .
Como rueda sin deslizamiento la relación entre el
desplazamiento lineal y el desplazamiento angular
es x = Rθ .
Solución.
Con la ley de Hooke para rotación,
La velocidad es
τ = −kθ
dx dθ
El trabajo para torcer un ángulo θ es: =R ⇒ v = Rω
θ θ dt dt
W = − ∫ τ dθ = − ∫ (− kθ )dθ =
1 2

0 0 2 La aceleración es
Este trabajo queda como energía potencial. dv dω
1 =R ⇒ a = Rα
U (θ ) = kθ 2 dt dt
2
Al liberarse esta se convierte en energía cinética. Aplicando la segunda ley de Newton para traslación
Al pasar por el punto de equilibrio la energía es

24
Cuerpo rígido Hugo Medina Guzmán

F − F f = Ma En este caso como la aceleración del centro masa es


a, la aceleración angular del cuerpo alrededor de O
Aplicando la segunda ley de Newton para rotación es α = a R .
alrededor del centro de masa
− RF f = − I CM α Aplicando la segunda ley de Newton para
traslación:
F − F f = Ma
Eliminando F f y α , obtenemos:
⎛ I ⎞ Aplicando la segunda ley de Newton para rotación a
⎜ M + CM2 ⎟a = F alrededor de O:
⎝ R ⎠
− FR = − I Oα
La aceleración
F Como α =a R y I O + I CM + MR :
2

a= con la segunda ecuación,


⎛ I ⎞
⎜ M + CM2 ⎟
⎝ R ⎠ (I CM + MR 2 ) Ra = FR ⇒ a=
F
⎛ I ⎞
⎜ M + CM2 ⎟
Si para t = 0: ⎝ R ⎠
x0 = 0 , v0 = 0 , Tercer método
Siendo a = constante Este método Consiste en usar las ecuaciones de la
energía directamente.
La velocidad es:
Es un Sistema Conservativo
v = v 0 + at
K + U = Constante
F Resolveremos por este método el ejemplo anterior.
v= t
(
M + I CM R 2 ) Puesto que no hay deslizamiento la tuerza de
fricción sobre el cuerpo no trabaja sobre el mientras
rueda. Siendo un sistema conservativo la fuerza F
El desplazamiento es: se puede deducir de una función Potencial U = - Fx
1 2 donde x es la coordenada horizontal del centro de
x = x0 + v0 t + at nasa.
2
La energía E del cuerpo es:
1 F E = K +U
x= t2
(
2 M + I CM R 2 ) 1 1
K = I CM ω 2 + Mv 2 , U = − Fx
2 2
Segundo método
1 1
En este método escribimos la ecuación para Luego: E = I CM ω + Mv − Fx
2 2
traslación igual que en el anterior método, pero para 2 2
la rotación se aplica la segunda ley de Newton con v
respecto al eje de rotación que pasa a través del Siendo ω = ⇒
punto de reposo instantáneo (punto de apoyo en el R
movimiento) si tal punto no existe no puede usarse 1 ⎛I ⎞
este método E = v 2 ⎜ CM2 + M ⎟ − Fx
Como ilustración veamos el ejemplo anterior. El 2 ⎝ R ⎠
punto contacto es el punto fijo instantáneo O, De aquí podemos evaluar la velocidad considerando
consideremos que este no desliza y todos los otros que para el instante inicial x = 0, y v = 0, por
puntos de eje momentáneamente rotan alrededor de consiguiente E = 0.
el. 1 2 ⎛ I CM ⎞
v ⎜ 2 + M ⎟ − Fx = 0 y
2 ⎝ R ⎠
1 2 ⎛ I CM ⎞
v ⎜ 2 + M ⎟ = Fx
2 ⎝ R ⎠
2 Fx
v=
I CM
+M
R2

25
Cuerpo rígido Hugo Medina Guzmán

Siendo un movimiento con aceleración constante α y F f obtenemos:


Eliminando
v = 2ax Mgsenβ
De esto a=
F M + I CM R 2
a= Considerando que para t = 0: s = 0, y v = 0.
I CM
+M ⎛ Mgsenβ ⎞
R2 v = ⎜⎜ ⎟t ,
2 ⎟
⎝ M + I CM R ⎠
Otra forma de calcular la aceleración.
1 ⎛ Mgsenβ ⎞ 2
s = ⎜⎜ ⎟t
Considerando que 2 ⎝ M + I CM R 2 ⎟⎠
dE
E = Constante ⇒ =0
dt Para un anillo:
dE ⎡ 1 2 ⎛ I CM ⎞ ⎤ 1
= ⎢ v ⎜ 2 + M ⎟ − Fx⎥ = 0 I CM = MR 2 , s = gsenβ t 2
dt ⎣ 2 ⎝ R ⎠ ⎦ 4
Para un disco:
dv ⎛ I ⎞ dx 1 1
⇒ v ⎜ CM2 + M ⎟ − F =0 I CM = MR 2 , s = gsenβ t 2
dt ⎝ R ⎠ dt 2 3
dv dx Para una esfera:
Como =a y =v 2 5
dt dt I CM = MR 2 , s = gsenβ t 2
⎛I ⎞ 5 14
va⎜ CM2 + M ⎟ − Fv = 0 ⇒ Para un plano sin fricción (sin rodadura)
⎝ R ⎠ 1
F s= gsenβ t 2
a= 2
⎛ I ⎞
⎜ M + CM2 ⎟ Por la ecuación de energía
⎝ R ⎠
Si para t = 0: K 0 = 0 y U 0 = 0
Ejemplo 43. Analizar el movimiento de un cuerpo E = K0 + U0 = 0
de radio R, momento de inercia respecto a su centro
de masa I que rueda sin deslizar hacia abajo en Llamando h a la caída del centro de masa desde la
plano inclinado de ángulo θ . posición de reposo, tenemos:
1 1
K= Mv 2 + I CM ω 2 ,
2 2
U = − Mgh = − Mgs senβ = 0 ,
ω=v R
1 2⎛ I ⎞
v ⎜ M + CM2 ⎟ − Mgs senβ
2 ⎝ R ⎠
2 Mgsenβ
Solución. ⇒v= s
Como se muestra en la figura hay dos fuerzas que M + I CM R 2
actúan sobre el cuerpo, Mg actúa en el centro de
gravedad y la fuerza de contacto que se Ejemplo 44. Usar la conservación de la energía
descompone en la reacción normal N y la fuerza de para describir el movimiento de rodadura de un
fricción Ff. cuerpo rígido de masa M que rueda por un plano
Vamos a resolver por el primer método.
inclinado θ y rugoso.
Traslación:
Mgsenβ − F f = Ma
Rotación:
RF f = I CM α
Por la condición de no deslizamiento:
α =a R

26
Cuerpo rígido Hugo Medina Guzmán

Estudiar el movimiento.

Solución.
Se supone que el cuerpo rígido parte del reposo
desde una altura h y que rueda por el plano sin
resbalar la conservación de energía da: Solución.
E = cte ⇒ K + U g = cte ⇒ Vamos a resolver primero por las ecuaciones del
movimiento de Newton.
K i + U gi = K f + U gf Traslación.:
Pero Mg − T = Ma
Ki = 0 y Ugf = 0, entonces Rotación.:
1 1
Mgh = I cmω 2 + Mvcm
2 RT = I CM α
2 2 Como:
Como 1 a
vcm= R ω ⇒ ω = vcm/R, se reemplaza en la ecuación I CM = MR 2 , α = :
2 R
anterior
1 v2 1 ⎛1 ⎞⎛ a ⎞ 1
I cm cm2 + Mvcm
2
= Mgh RT = ⎜ MR 2 ⎟⎜ ⎟ = MRa
2 R 2 ⎝2 ⎠⎝ R ⎠ 2
Despejando νcm se obtiene: De aquí se obtenemos:
2 gh 1 2
vcm = T= Ma y a = g
I + I cm MR 2 2 3
Por ejemplo, para una esfera sólida uniforme de El yo-yo funciona según este principio, está
proyectado para que a sea mucho menor que g.
2
momento de inercia I cm = MR 2 , se puede
5 Resolviendo por conservación de la energía
calcular su vcm en el punto más bajo del plano y su E = K +U =
aceleración lineal. 2
1 1⎛1 ⎞⎛ v ⎞
2 gh 2 gh 10 Mv 2 + ⎜ MR 2 ⎟⎜ ⎟ − Mgy
vcm
2
= = = gh 2⎝2 ⎠⎝ R ⎠
(2 5)MR 2 2
2 7
1+ 1+ dE
MR 2 5 Como E = constante ⇒ =0
dt
10 dy dv
⇒ vcm = gh También v = y a=
7 dt dt
La aceleración lineal se puede calcular con la Con esto encontramos que
ecuación
2
v2 a= g
v 2
cm =v 2
cmi + 2a cm x ⇒ a cm = cm 3
2x
De la geometría de la figura, se tiene: h = x sen θ, Ejemplo 46. Estudiar el movimiento de un disco
donde x es la longitud del plano, reemplazando en homogéneo de radio R y masa M, sobre el que actúa
acm: una fuerza horizontal F aplicada un punto variable a
5 lo largo de una línea vertical que pasa por el centro,
gxsenθ según se indica en la figura. Supóngase el
7 5
a cm = = gsenθ movimiento sobre un plano horizontal.
2x 7
Ejemplo 45. Un disco homogéneo de radio R y
masa M tiene una cuerda enrollada alrededor, según
vemos en a figura. Sujetando el extremo libre de la
cuerda a un soporte fijo, se deja caer el disco.

27
Cuerpo rígido Hugo Medina Guzmán

⎡ (0)⎤
3F f = F ⎢1 − 2 ⎥ = F ⇒ F f =
F
⎣ R⎦ 3
En la ecuación (3)
⎛F⎞
F (0 ) + ⎜ ⎟ R = I CM α = MR 2α
1
⎝3⎠ 2
F 1
R = MR 2α ⇒
3 2
2F
α=
Solución.
3MR
En la figura vemos que la fuerza F se aplica a una El cilindro rueda hacia la derecha.
distancia h sobre el centro.
d) Si F se hace muy grande tal que Ff tiende a
Suponiendo que F f actúa hacia la izquierda. aumentar, tan pronto como sobrepase el valor
Aplicando las leyes de Newton del movimiento: máximo posible de la tuerza de rozamiento (μN), el
Traslación disco deslizará.
F − F f = Ma (1) Se debe hacer una nueva hipótesis, esta vez se
tienen también las ecuaciones (1), (2) y (3) pero
N − Mg = 0 (2) α ≠ a R.
Rotación alrededor del centro de masa
1 Ejemplo 47. Un carrete de radio interior R1 y radio
Fh + F f R = I CM α = MR 2α (3) exterior R2 se halla sobre un suelo áspero. Se tira de
2
él con una tuerza F mediante un hilo arrollado en
a
Considerando α= torno a su cilindro interior. Se mantiene un ángulo
R θ con la horizontal. Se observa que hay un ángulo
h Crítico θ 0 , tal que θ < θ o , el carrete rueda sin
2F + 2 F f = Ma (3a)
R deslizar en el sentido del cual se tira de él, y para
Igualando (1) y (3a) θ > θ o el carrete rueda sin deslizar en sentido
h
F − F f = 2F + 2F f contrario, ¿Cuál es el valor del ángulo critico.
R
⎛ h⎞
3 F f = F ⎜1 − 2 ⎟
⎝ R⎠
Discusión:
h R
a) F f = 0 , cuando 1 − 2 =0 ⇒ h=
R 2
Esto quiere decir si F se aplica a R/2 del centro, la
fuerza de rozamiento es cero.
Solución.
b) Si h = R Aplicando las leyes de Newton del movimiento;
⎛ R⎞ F Traslación:
3F f = F ⎜1 − 2 ⎟ = − F ⇒ F f = − F cos θ − F f = Ma = MαR2 (1)
⎝ R⎠ 3
el rozamiento es en sentido contrario al indicado y Fsenθ − Mg + N = 0
la ecuación (3) se convierte en: Rotación:
⎛F⎞ − F f R2 + FR1 = I CM α ⇒
F (´R ) − ⎜ ⎟ R = MR 2α
1
⎝3⎠ 2 F f R2 = FR1 − I CM α ⇒
2 1
⇒ F = MRα R1 I
3 2 Ff = F R1 − CM α (2)
R2 R2
4F
⇒ α= Eliminando la fuerza F f ., reemplazando (2) en
3MR
Esto indica que el cilindro rueda hacia la derecha. (1):

c) Si disminuye h hasta que h = 0.

28
Cuerpo rígido Hugo Medina Guzmán

⎛ R I ⎞ Ff =
1
Ma
F cos θ − ⎜⎜ F 1 R1 − CM α ⎟⎟ = Ma
2
⎝ R2 R2 ⎠
Que sustituida en la primera da:
R I 2F
⇒ F cos θ − F 1 R1 + CM α = MαR2 a) a = ,
R2 R2 3M
⎛ R2 cos θ − R1 ⎞ ⎛ MR22 − I CM ⎞ a 2F
⇒ F ⎜⎜ ⎟⎟ = ⎜⎜ ⎟α b) α = = ,
R R2 ⎟ R 3MR
⎝ 2 ⎠ ⎝ ⎠

⇒ α=F
(R2 cos θ − R1 ) dω 1
c) F f = Ma =
F
=
(MR 2
2 − I CM ) dt 2 3
Ejemplo 49. Un disco de masa M y radio 2R se
La rotación hará que el movimiento del carrete será apoya sobre un plano horizontal áspero de modo
dω que puede rodar sin resbalar con su plano vertical.
hacia adelante cuando >0 El disco tiene un resalto de radio R como se indica
dt
en la figura, en el cual se enrolla una cuerda que se
R2 cos θ − R1 > 0 tira con una fuerza horizontal constante F,
R determine:
⇒ cos θ > 1 a) La aceleración del centro de masa del disco.
R2 b) La aceleración angular del disco.
dω c) La fuerza de roce.
El movimiento será hacia atrás cuando <0
dt
R2 cos θ − R1 < 0
R
⇒ cos θ < 1
R2
dω Solución.
El ángulo crítico es cuando =0
dt
R2 cos θ − R1 = 0
R
⇒ cos θ = 1
R2

Ejemplo 48. Un disco de masa M y radio R se


apoya sobre un plano horizontal áspero de modo
que puede rodar sin resbalar con su plano vertical.
Si se tira del centro del disco con una fuerza Ahora F − F f = Ma , N − Mg = 0
horizontal constante F, determine:
M (2 R ) α
1
F f 2 R + FR =
2
a) La aceleración del centro de masa del disco.
b) La aceleración angular del disco. 2
c) La fuerza de roce. 2⎛ a ⎞
Solución. = 2 MR ⎜ ⎟ = MRa
⎝ 2R ⎠
Simplificando:
2 F f + F = Ma = F − F f
⇒ Ff = 0
De donde resulta:
F
a) a =
m
F
Aquí b) α=
F − F f = Ma , N − Mg = 0 , 2MR
c) F f = 0
1 1
Ff R = MR 2α = MRa
2 2
Entonces

29
Cuerpo rígido Hugo Medina Guzmán

Ejemplo 50. Un disco de masa M y radio R tiene A partir del punto A en que el piso es áspero
enrollada una cuerda en su periferia y cae partiendo deslizará primeramente sobre el plano áspero, pero
del reposo mientras la cuerda que se sostiene de su acabará rodando sin deslizar.
extremo se desenrolla. Determine: En la parte intermedia habrá una aceleración a que
a) La aceleración de bajada del disco. disminuye a la velocidad de v1 a v 2 y una
b) La tensión de la cuerda.
aceleración angular α que disminuye a ω1 , la
hace igual a cero y cambia su rotación hasta que
llega la velocidad angular a un valor tal que
ω 2 = v2 R .
Aplicando las leyes de Newton en la figura
siguiente.

Solución.

Traslación: μN = Ma , N − Mg = 0
1
Rotación: − RμN = I CM α = MR 2α
2
2 μg
Aquí Mg − T = Ma , De esto obtenemos: a = − μg , α = −
R
TR =
1 1
MR 2α = MRa La velocidad es: v = v1 + at = v1 − μgt
2 2 La velocidad angular es:
1 v1 2 μg
De donde Mg − Ma = Ma ω = ω1 − α t = − t
2 R R
2 Parta encontrar el tiempo en que el disco deja de
a) a = g → → →
3 resbalar, debe cumplirse: v = ω× R
1 1
b) T = Ma = Mg viˆ = ωkˆ × Rˆj = −ωRiˆ
2 3
(v1 − μgt ) = −⎛⎜ v1 − 2μg t ⎞⎟ R
Ejemplo 51. Se da a un cilindro homogéneo de ⎝R R ⎠
radio R y masa M con una velocidad horizontal v1
2 v1
ω1 2v1 = 3μgt ⇒ t =
y una velocidad angular en sentido opuesto a
3 μg
las agujas del reloj ω1 = v1 R en la parte sin con este valor de t
rozamiento de la superficie horizontal. Más allá del ⎛ 2 v1 ⎞ v1
punto A, cambia la superficie de manera que a la v 2 = v1 − μg ⎜⎜ ⎟⎟ =
derecha de A el coeficiente de rozamiento es μ . ⎝ 3 μg ⎠ 3
La velocidad final es un tercio de la inicial

Solución.
En la parte lisa el cuerpo se mueve con velocidad
horizontal constante v1 hacia la derecha, rotando Ejemplo 52. Se lanza una bola de billar con una
con velocidad angular ω1 en el sentido antihorario. velocidad inicial v 0 sobre una mesa horizontal,

30
Cuerpo rígido Hugo Medina Guzmán

existiendo entre la bola y la mesa un coeficiente de 5μgt 2v


rozamiento μ. Calcular la distancia que recorrerá v0 − μgt = ⇒ t= 0
hasta que empiece a rodar sin deslizamiento. 2 7 μg
¿Qué velocidad tendrá en ese instante? la velocidad en ese instante es
Aplicar para el caso v 0 = 7 m/s, μ = 0,2. 5
v= v0 = 5 m/s, t = 1,02 s
7
La distancia recorrida
1
x = v0 t − μgt 2
2
Solución. 2
La fuerza de rozamiento µN = µmg se opone al 2v02 1 ⎛ 2v0 ⎞ 12v 02
= − μg ⎜⎜ ⎟⎟ =
movimiento, siendo además la fuerza resultante, por 7 μg 2 ⎝ 7 μg ⎠ 49μg
lo que:
− μmg = ma , a = − μg = 6,12 m.
La velocidad de la bola comenzará a disminuir de Ejemplo 53. Un tambor tiene un radio de 0,40 m y
tal modo que: un momento de la inercia de 5,0 kg m2. El torque
v = v 0 − at = v 0 − μgt . producido por la fuerza de fricción de los cojinetes
Al mismo tiempo, sobre la bola que inicialmente no de anillo del tambor es 3,0 Nm. Un anillo en un
rueda, (ω0 = 0) actúa un momento de fuerza: extremo de una cuerda se desliza en una clavija
corta en el borde del tambor, y una cuerda de 15 m
de longitud se enrolla sobre el tambor. El tambor
está inicialmente en reposo. Una fuerza constante
se aplica al extremo libre de la cuerda hasta que la
cuerda se desenrolla y se desliza totalmente de la
clavija. En ese instante, la velocidad angular del
τ = F f R = μmgR tambor es de 12 rad/s. El tambor después decelera y
τ se detiene.
que producirá una aceleración angular α=
I
τ μmgR 5μg
α= = =
I 2 2R
mR 2
5
Por lo que la velocidad angular irá aumentando:
5μgt
ω = αt = a) ¿Cuál es la fuerza constante aplicada a la cuerda?
2R b) ¿Cuál es la cantidad de movimiento angular del
tambor en el instante en que la cuerda deja el
La velocidad de un punto de la periferia de la esfera tambor?
vale v P = ωR , que irá aumentando con el tiempo, c) ¿Cuál es el trabajo negativo realizado por la
porque ω aumenta con el tiempo. fricción?
d) ¿Qué tiempo el tambor estuvo en movimiento?
Movimiento con la cuerda?
Solución.
a)
Trabajo de la fuerza F + trabajo de la fricción
= Energía cinética ganada al terminarse la cuerda
1
FΔs + τ f Δθ = I Oω 2
2
⎛ 15 ⎞ 1
⇒ F (15) − 3,0⎜ ⎟ = (5,0 )(12)
2

⎝ 0,4 ⎠ 2
Por tanto, observamos que la velocidad de la bola ⇒ F = 31,5 N
disminuye, y la velocidad de la periferia de la bola b)
aumenta. En el momento en que la velocidad de la L = I Oω = (5)(12)
periferia se iguale a la velocidad de traslación, se = 60 kg.m2/s
conseguirá la rodadura, es decir el no deslizamiento. c)
v = v P v = ωR Movimiento con la cuerda

31
Cuerpo rígido Hugo Medina Guzmán

⎛ 15 ⎞ m
W f 1 = −τ f Δθ = −3⎜ ⎟ = - 112,5 J = 0,75
⎝ 0,4 ⎠ s2
Movimiento sin la cuerda mg − T = ma
T 20
W f 2 = − I O ω O2 = − (5,0)(12) = −360
1 1 2
⇒ m= =
2 2 g − a 9,8 − 0,75
Trabajo total = 2,21 kg
W f = W f 1 + W f 2 = −482,5 J b)
a 0,75
d) α= =
∑τ O = I Oα R 0,4
FR − τ f I Oα ⇒ 31,5(0,4 ) − 3,0 = 5,0α rad
= 1,875 2
31,5(0,4) − 3,0
s
rad
⇒ α1 =
5,0
= 1,92
s ∑τ O = I Oα ⇒ TR = I Oα
TR 20(0,4 )
Por otra parte ⇒ IO = =
ω0 12 α 1,875
ω o = α 1t1 ⇒ t1 = = = 6,25 s
α1 1,92 = 4,27 kg m
2

Movimiento sin la cuerda


∑τ O = I Oα ⇒ − 3 = 5α 2 Ejemplo 55. El radio de una rueda de 3,0
kilogramos es 6,0 centímetros. La rueda se suelta
3 rad del reposo en el punto A en un plano inclinado 30°.
⇒ α 2 == − = −0,6 La rueda gira sin deslizar y se mueve 2,4 m al
5 s
punto B en 1,20s.
0 = ω0 + α 2t 2 a) ¿Cuál es el momento de inercia de la rueda?
− ω0 − 12 b) ¿Cuál es la aceleración angular de la rueda?
⇒ t2 = = = 20s
α2 − 0,6
El tiempo total es 26,25 s

Ejemplo 54. Una rueda tiene un radio de 0,40 m y


se monta en cojinetes sin fricción. Un bloque se
suspende de una cuerda que se enrolla en la rueda.
La rueda se libera de reposo y el bloque desciende Solución.

mR 2 = (3kg )(0,06m )
1,5 m en 2,00 segundos. La tensión en la cuerda 1 1
a) I O =
2
durante el descenso del bloque es 20 N. 2 2
a) ¿Cuál es la masa del bloque? = 0,0054 kg m2
b) ¿Cuál es el momento de inercia de la rueda? b)
mgsen30º − Ff = ma F f R = I Oα
⎛I ⎞
⇒ F f = ⎜ O ⎟α
⎝ R⎠
⎛I ⎞
mgsen30º −⎜ O ⎟α = mRα
⎝R⎠
mgsen30º 3(9,8)(0,5)
⇒ α= =
⎛ IO ⎞ 0,0054
+ 3(0,06)
Solución. ⎜ ⎟ + mR
⎝R⎠ 0,06
a) 14,7 rad
= = 54,4 2
1 2 0,27 s
h= at
2
2h 2(1,5) Ejemplo 56. Una masa de 20 kg se halla sobre un
⇒ a= 2 = plano inclinado 30º, con el que tiene un rozamiento
t (2)2 cuyo coeficiente vale 0,3, unida a una cuerda sin
masa e inextensible que pasa por una polea de MP =
160 kg, cuyo radio geométrico es de 20 cm y radio

32
Cuerpo rígido Hugo Medina Guzmán

de giro rg = 15 cm. De dicha cuerda pende una = 10 + 5,2 = 15,2 kg.


masa de 40 kg que es abandonada libremente.
Calcular:
a) Aceleración con que se mueve el sistema.
b) Tensiones en la cuerda.
c) ¿En qué rango de valores de la masa que pende,
el sistema estará en equilibrio?
Momento de inercia de la polea I P = Mrg .
2

Solución.
a) Partiendo de la suposición de que la masa
colgante acelera hacia abajo, plantearemos las tres
ecuaciones correspondientes al movimiento de las Si la masa m2 se hace aún menor, llegará un
tres masas: momento en que será arrastrada por m1. Esto
m2g - T2 = m2a produciría una inversión en el sentido de la fuerza
de rozamiento. El valor máximo de m2 deberá
cumplir ahora:
m2 = m1senθ + μm1 cos θ
= 10 – 5,2 = 4,8 kg.
Por tanto, entre 0 y 4,8 kg el sistema acelerará de
modo que m2 suba; entre 4,8 y 15,2 kg,
permanecerá en equilibrio; y para más de 15,2 kg
m2 acelerará hacia abajo.
T1 − m1gsenθ + μm1 g cos θ = m1 a ,
a Ejemplo 57. ¿Porqué una esfera que rueda se
T2 R − T1 R = Iα = M P rg2 detiene? En esta parte vamos a tratar de explicar la
R resistencia al rodamiento.
Sumando las tres ecuaciones siguientes La figura siguiente muestra una esfera de masa M y
m0 g − T2 = m2 a , radio R la cual está rodando con una velocidad
angular ω y avanza con una velocidad v = ωR .
T1 − m1gsenθ + μm1 g cos θ = m1 a
2
⎛ rg ⎞
T2 − T1 M P ⎜⎜ ⎟⎟ a
⎝R⎠
Obtenemos:
m2 g − m1gsenθ + μm1 g cos θ
⎡ ⎛ rg ⎞ ⎤
2

= a ⎢m1 + m2 + M P ⎜⎜ ⎟⎟ ⎥
⎢⎣ ⎝ R ⎠ ⎥⎦
m2 − m1senθ + μm1 cos θ
⇒a= 2
g Solución.
⎛ rg ⎞
m1 + m2 + M P ⎜⎜ ⎟⎟ La fuerzas que actúan sobre la esfera son el peso
Mg 1a reacción del piso N y la fuerza de fricción
⎝R⎠
40 − 10 − 5,2 F f . Si aplicamos la segunda ley de Newton a la
= 2
g traslación.
⎛ 15 ⎞ → →
60 + 160⎜ ⎟ Ff = M g
⎝ 20 ⎠
= 1,62 m/s2 → →

b) debe haber una aceleración a y v decrecería. Si


T2 = m2 ( g − a ) = 327 N, aplicamos segunda ley de Newton a la rotación.
2
RF f = I CM α
⎛ rg ⎞ la aceleración angular α depende de Ff. por
T1 = T2 ⎜⎜ ⎟⎟ a = 181 N.
⎝R⎠ consiguiente Ff actúa incrementando ω .
c) El valor mínimo que hace que la masa m2 acelere En resumen: en traslación Ff. acelera, en rotación
hacia abajo se produce cuando a = 0, es Ff. desacelera, esto aparentemente es una
decir: contradicción.
Por otra parte Mg y N están en la línea vertical que
m2 = m1senθ + μm1 cos θ por el centro de masa y no causan efecto en el

33
Cuerpo rígido Hugo Medina Guzmán

movimiento horizontal.
Si la esfera y el plano son rígidos, de modo que la
esfera esté en contacto solo en un punto, tampoco
originan alrededor del centro de masa. .porque
actúan a través de él
Para resolver la Contradicción suprimamos la
idealización de que todos los cuerpos son rígidos, la
esfera se aplana un poco y el nivel de La superficie
se hunde Ligeramente (ver la figura a continuación)

Sobre la varilla actúa el peso Mg y la reacción R.


La velocidad angular ω en este instante se puede
encontrar aplicando la ecuación de la energía.
L L 1
Mg = Mg cos β + I O ω 2
2 2 2
1
Como I O = ML
2

La reacción N actúa delante del centro de masa, 3


produciendo un torque τ N = dN de resistencia al L L 1⎛1 ⎞
Mg = Mg cos β + ⎜ ML2 ⎟ω 2
rodamiento. 2 2 2⎝3 ⎠
τ N − RF f = I CM α 3g
a
ω2 = (1 − cos β )
Como N = Mg , F f = Ma , α = :
L
R 6g β 3g ⎛ 2 β ⎞
a ω= sen = ⎜ 2sen ⎟
τ N − RMa = I CM L 2 L ⎝ 2⎠
R 6g β
⎛I ⎞ = sen 2
⇒ τ N = a⎜ CM + RM ⎟ L 2
⎝ R ⎠ Aplicando la segunda Ley de Newton para
traslación a lo largo de la varilla.
2
Para una esfera: I CM = MR
2
L
5 ∑ F = ma c ⇒ R − Mg cos β = − Mω 2
2
7
Luego: τ N = MRa , como N = Mg Cuando La varilla deja de presionar R = 0, y:
5 L
τ N 7R − Mg cos β = − Mω 2
d= = a 2
Mg 5 g reemplazando el valor de ω2 encontrado

Ejemplo 58. La figura muestra una varilla


⎛ 6g β⎞L
homogénea de masa M y longitud L en posición Mg cos β = M ⎜ sen 2 ⎟
vertical. La cual se deja caer desde el reposo. ⎝ L 2⎠2
a) ¿A que ángulo θ entre la varilla y la vertical, la Simplificando
varilla ya no presionará al piso? β β β
b) ¿Con qué coeficiente de fricción el extremo de cos β = 6 sen 2 ⇒ cos 2 = 5 sen 2
La varilla no resbalará hasta este momento? 2 2 2
5
⇒ tan β =
5
De aquí: β = 48,2º
b) Para que la varilla no resbale tenemos en la
figura siguiente.
Las componentes de R son:

R = Rsenβ iˆ + R cos βˆj
Solución.
a) La figura siguiente muestra .la varilla cuando
forma un ángulo θ con la vertical.

34
Cuerpo rígido Hugo Medina Guzmán

La condición para que la varilla no resbale es:


F f ≥ Rsenβ Por la conservación de la cantidad de movimiento
angular.
Con F f = μN y N = R cos β I1
μR cos β ≥ Rsenβ I 2ω 2 = I 1ω1 ⇒ ω 2 = ω1
I2
μ ≥ tan β Siendo I 2 < I 1 , resulta ω 2 > ω1
El coeficiente de rozamiento del piso debe ser
Su velocidad aumenta.
cuando menos igual a tan β para que llegue sin
β.
deslizar hasta el ángulo Ejemplo 60. Esta vez el mismo estudiante sentado
Para β = 48,2º ⇒ μ ≥ 1,12 sobre el mismo banco, sostiene en sus manos en
posición vertical al eje de rotación de una rueda de
bicicleta, la rueda gira alrededor de ese eje vertical
CONSERVACION DE LA CANTIDAD DE
MOVIMIENTO ANGULAR. con velocidad angular ω 0 , el estudiante y el banco
Anteriormente hemos visto que: están en reposo (a).
→ → El estudiante gira el eje de la rueda en ángulo θ
→ dp → dL
F= y también τ = con la vertical (b), como no hay torque respecto al
dt dt eje vertical, la cantidad de movimiento angular con
y mostramos que para un cuerpo rígido. respecto al eje vertical debe conservarse.

→ d L total
τ ext =
dt
Si no hay torque externo con respecto a algún eje la
cantidad de movimiento angular será constante con
respecto a ese eje.

L total = Constante
o expresado en función del momento de inercia
apropiado.

I ω = Constante
Esta relación nos va a ser muy útil como veremos a
continuación. Inicialmente se tiene

Ejemplo 59. Un estudiante está sentado sobre un L = I 0ω 0 kˆ
banco giratorio montado sobre cojinetes sin fricción
que puede girar libremente alrededor de un eje Cuando se inclina la rueda (respecto al eje vertical)
→ → →
vertical como se muestra en la figura (a). El
L' = L estudiante +banco + L rueda
estudiante sostiene en las manos extendidas dos →
pesas. Su momento de inercia en esta posición es I1 = I e ω e + I 0ω 0 cos θ kˆ
y su velocidad angular ω1 . No actúan sobre él
Siendo I e el momento de inercia del estudiante y
torques no equilibrados y en consecuencia su
cantidad de movimiento angular tiene que banco respecto al eje vertical, ωe su velocidad
conservarse. angular con respecto a ese eje.
Cuando el estudiante acerca las manos al cuerpo, su → →
momento de inercia varía, figura ( b) ahora es I2 y Como L = L'
su velocidad angular será ω 2 →
I e ω e + I 0ω 0 cos θ kˆ = I 0ω 0 kˆ

35
Cuerpo rígido Hugo Medina Guzmán

→ I de 2 rad/s2 durante 6 segundos, momento en el cual


ω e = 0 ω 0 (1 − cos θ ) kˆ encoge los brazos y acerca sus piernas al eje hasta
Ie tener un momento de inercia de 4 kg.m2 .
Es la velocidad angular del estudiante con el sentido Determinar su velocidad de giro final.
de giro inicial de la rueda.
Cuando la rueda se invierte se invierte totalmente
θ = π 2 , y:
→ 2I 0
ωe = ω 0 kˆ
Ie

Ejemplo 61. Una persona está sentada en una silla


giratoria manteniendo los brazos extendidos con
una pesa en cada mano. Gira con una frecuencia de
2 Hz. El momento de inercia de la persona con los Solución.
pesos es de 5 kg m2. Hallar:
a) la nueva frecuencia cuando encoja los brazos y Después de un tiempo t de iniciar el giro, su
disminuya el momento de inercia a 2 kg m2. velocidad angular será:
b) La variación de energía cinética del sistema.
c) ¿De dónde procede este incremento de energía
cinética? 1
ω (t ) = at 2 =
1
(2)(6)2 = 36 rad/s
2 2

al acercar brazos y piernas al eje, el torque de las


fuerzas sigue siendo nulo, por lo que se conserva la
cantidad de movimiento angular, Iω

(Iω )Antes = (Iω )Después ⇒


Solución. I Antes
a) Al encoger los brazos, están actuando fuerzas y ωDespués = ωAntes
torques de fuerzas internas, por lo que podemos I Después
admitir que se conserva la cantidad de movimiento
angular.
7
L1 = L2 ⇒ I 1ω1 = I 2ω 2 = 36 = 63 rad/s
4
I I
⇒ ω 2 = 1 ω 2 , ⇒ 2π f 2 = 1 2π f1 ,
I2 I2 Ejemplo 63. Un muchacho de 25 kg corre con
velocidad de 2,5 m/s hacia un tiovivo en reposo de
I 5
⇒ f 2 = 1 f1 = 2 = 5 Hz radio 2 m cuyo, momento de inercia vale 500 kg
I2 2 m2. Hallar la velocidad angular y frecuencia del
conjunto después de que el muchacho suba al
1 1 L2 L2
b) ΔK = I 2ω 22 − I 1ω12 = − tiovivo justo en el borde.
2 2 2I 2 2I1
L = I 1ω1 = 5(2π 2) = 20π kg m 2 s ;
⎛1 1⎞
ΔK = 200π 2 ⎜ − ⎟ = 60π 2 J .
⎝ 2 5⎠
El signo positivo nos indica que hay un aumento de
energía cinética.
c) Este incremento de energía cinética procede de la
energía química almacenada en los músculos del
brazo. Solución.
La cantidad de movimiento angular del muchacho
Ejemplo 62. Un patinador, con los brazos respecto al centro del tiovivo es:
extendidos y las piernas abiertas y con un momento
L1 = mvR = (25)(2,5)(2 ) = 125 kg m2/s
de inercia respecto a su eje vertical de 7 kg.m2 ,
inicia un giro sobre si mismo con una aceleración

36
Cuerpo rígido Hugo Medina Guzmán

El momento de inercia del conjunto tiovivo- ⎛ IO


⎞ ⎛ 2,0 ⎞
muchacho es ω ' = ⎜⎜ ⎟⎟ω = ⎜ ⎟1,5
I = Im + IT = 25x22 + 500 ⎝ I 'O
⎠ ⎝ 2,256 ⎠
= 600 kg m2
Planteando la igualdad entre la cantidad de
rad
= 1,33
movimiento angular inicial y final, tendremos: s
L1 = L2 , mvR = (I m + I T )ω Reemplazando:

I 'O ω ' 2 = (2,256)(1,33)


1 1
mvR 125 Energía después =
2
ω= =
(I m + I T ) 600 =2J
2 2
= 0,208 rad/s Se pierde 4,05 -2 = 2,05
ω 2,05
f = = 0,033 Hz fracción de energía = = 0,5
2π 4,05
= 1,99 r.p.m.
Ejemplo 65. Una barra rígida de masa M y largo L
Ejemplo 64. Una tornamesa con radio de 8,0 m y gira en un plano vertical alrededor de un eje sin
momento de inercia de 2,0 kg.m2. La placa fricción que pasa por su centro. En los extremos de
tornamesa rota con una velocidad angular de 1,5 la barra se unen dos cuerpos de masas m1 y m2.
rad/s sobre un eje vertical que pasa a través de su Calcular la magnitud del momento angular del
centro en cojinetes sin fricción. Una bola de 0,40 sistema cuando su rapidez angular es ω y la
kg se lanza horizontalmente hacia el eje de la aceleración angular cuando la barra forma un
tornamesa con una velocidad de 3,0 m/s. La bola es
ángulo φ con la horizontal.
cogida por un mecanismo con forma de tazón en el
borde de la tornamesa.
a) ¿Cuál es cantidad de movimiento angular de la
bola alrededor del eje de la tornamesa?
b) ¿Qué fracción de energía cinética se pierde
durante la captura de la bola?

Solución.
El momento de inercia por el eje de rotación del
sistema es igual a la suma de los momentos de
inercia de los tres componentes, con los valores de
la tabla se obtiene:
2 2
1 ⎛L⎞ ⎛L⎞
I = ML2 + m1 ⎜ ⎟ + m2 ⎜ ⎟
12 ⎝2⎠ ⎝2⎠
L2 ⎛ M ⎞
Solución. = ⎜ m1 + m2 + ⎟
a) La cantidad de movimiento angular de la bola 4 ⎝ 3 ⎠
alrededor del eje de la tornamesa es cero Como el sistema gira con rapidez angular ω, la
b) magnitud del momento angular es:
1 2 1 L2 ⎛ M ⎞
Energía antes = mv + I O ω 2 L = Iω = ⎜ m1 + m2 + ⎟ω
2 2 4 ⎝ 3 ⎠
= (0,4 )(3,0 ) + (2,0 )(1,5)
1 2 1 2 Para calcular la aceleración angular usamos la
relación
2 2
τt
= 4,05 J
τ t = Iα ⇒ α = , al calcular el torque total
1 I
Energía después = I ' O ω '
2

2 en torno el eje de rotación, se obtiene:


Para calcular esta energía necesitamos conocer I0 y L L
τ t = m1 g cos φ − m2 g cos φ
ω’. 2 2
I ' O = I O + mR 2 = 2,0 + (0,4)(0,8)
2

= 2,256 kg/ m 2 =
1
(m1 − m2 )gL cos φ
2
Lantes = Ldespués ⇒ I O ω = I 'O ω '

37
Cuerpo rígido Hugo Medina Guzmán

Reemplazando en α los valores de I y de τ t , se a) El momento de inercia será la suma del momento


de inercia de una varilla, más el de la esfera. Como
obtiene la aceleración angular: la esfera está a L+R del eje, aplicamos Steiner:
τt 2(m1 − m2 )g cos φ
α= = Ie =
2
me R 2 + me (L + R ) , I V = mV L2
2 1
I L(m1 + m2 + M 3) 5 3
I = I e + IV
Ejemplo 66. En la figura las masas m1 y m2 se
= me R + me (L + R ) + mV L
conectan por una cuerda ideal que pasa por una 2 2 2 1 2
polea de radio R y momento de inercia I alrededor 5 3
de su eje. La mesa no tiene roce, calcular la
I = (0,25)(0,1) + (0,25 )(0,85) + (0,5)(0,75)
2 2 2 1 2
aceleración del sistema.
5 3
Solución. Primero se calcula en momento angular = 0,27 kg.m2
del sistema de las dos masas más la polea:

b) L = Iω = 0,27 = 0,27( 2πf )
T
12
= 0,54π = 0,345 kgm2 / s
60
Ejemplo 68. Un cilindro de 50 kg y 20 cm de radio,
v gira respecto de un eje vertical que coincide con su
L = m1vR + m2 vR + I
R eje de simetría, debido a una fuerza constante,
Luego se calcula el torque externo sobre el sistema, aplicada a su periferia que, después de 40 s de
la única fuerza externa que contribuye al torque iniciado el movimiento, alcanza 200 r.p.m.
total es m1g, entonces el torque es Calcular:
El valor de la fuerza y el torque de la fuerza
τ = m1 gR . aplicada.
Entonces se tiene:
dL
τ= ⇒
dt
d ⎡ v⎤
m1 gR = ⎢(m1 + m2 )vR + I ⎥
dt ⎣ R⎦
dv I dv
m1 gR = (m1 + m2 )R +
dt R dt
⎛ I ⎞ Solución.
⇒ m1 gR = ⎜ m1 + m2 + 2 ⎟ Ra La frecuencia de rotación adquirida vale:
⎝ R ⎠
200
m1 g f = Hz
⇒a= 60
I
m1 + m2 + 2 La velocidad angular:
R 200 20 rad
ω = 2πf = 2π = π
60 3 s
Ejemplo 67. Una varilla de 500 g y 75 cm de La aceleración angular:
longitud, lleva soldada en un extremo una esfera de
Δω π rad
10 cm de radio y 250 g de masa. Calcular: α= =
a) El momento de inercia cuando gira, alrededor de Δt 6 s2
un eje perpendicular a la varilla que pasa por el Por otra parte el momento de inercia del cilindro
extremo libre. vale:
b) La cantidad de movimiento angular del conjunto
mR 2 = (50)(0,2 ) = 1 kgm2.
1 1
I=
2
si gira a 12 rpm.
2 2
Luego el torque de la fuerza aplicada
π
τ = FR = Iα = (1) = 0,52 Nm.
6
La fuerza tangencial:
τ 0,52
Solución. F= = = 2,6 N
R 0,2

38
Cuerpo rígido Hugo Medina Guzmán

Ejemplo 69. Un anillo de masa M y radio R (ICM = ⎛ 1 ⎞


MR2), cae en rodadura pura sobre un plano longitud l ⎜ I CM = Ml 2 ⎟ se sostiene de un
inclinado que forma un ángulo θ con la ⎝ 12 ⎠
horizontal. extremo mediante un pivote sin fricción. La barra se
a) Hacer el DCL. del anillo. encuentra inicialmente en reposo en forma vertical
b) Hallar la aceleración del centro de masa del cuando un proyectil de masa m impacta sobre ella y
anillo. queda incrustado instantáneamente. La velocidad
c) Encontrar el valor de la fricción entre el plano inicial del proyectil es v 0 . Hallar:
inclinado y el anillo. a) La cantidad de movimiento angular del sistema
d) ¿Cuál debe ser el mínimo valor del coeficiente de respecto del pivote justo antes de la colisión.
rozamiento estático entre el plano y el anillo para
que este se encuentre en rodadura pura? b) La velocidad angular de giro del sistema después
que el proyectil se incrusta en la barra.
c) La altura máxima que alcanzará el CM de la
barra.
d) El trabajo del proyectil cuando se incrusta contra
la barra.

Solución.
a) El DCL. del anillo.

Solución.
a) La cantidad de movimiento angular del sistema
b) Segunda ley de Newton para la traslación respecto del pivote justo antes de la colisión.
Mgsenθ − F f = Ma Lantes = mv0 d
b) La velocidad angular de giro del sistema después
Segunda ley de Newton para la rotación
que el proyectil se incrusta en la barra.
a
Iα = F f R ⇒ MR 2 = Ff R ⇒
R
F f = Ma
Reemplazando el valor de Ff en la primera
ecuación.
Mgsenθ − Ma = Ma ⇒ Mgsenθ = 2Ma
1
Finalmente a = gsenθ
2
c) El valor de la fuerza de fricción entre el plano
inclinado y el anillo.
1 Lantes = Ldespués
F f = Ma = Mgsenθ
2
Ml 2ω + (ωd )d
1
d) El mínimo valor del coeficiente de rozamiento mv0 d =
estático entre el plano y el anillo para que este se 3
encuentre en rodadura pura debe de cumplir
mv0 d
1
Ff = μk N =Mgsenθ ⇒ ω=
2 ⎛1 2⎞
⎜ Ml + md ⎟
2

Mgsenθ 1 ⎝3 ⎠
⇒ μk = = tan θ
2Mg cosθ 2 c) La altura máxima que alcanzará el CM de la
barra.
Energía justo después del choque
Ejemplo 70. Una barra uniforme AB de masa M y

39
Cuerpo rígido Hugo Medina Guzmán

1 ⎛ l ⎞ velocidad angular del bloque después del choque.


= I O ω 2 − ⎜ Mg + mgd ⎟
2 ⎝ 2 ⎠
⎛1 2⎞
= I O = ⎜ Ml + md ⎟
2

⎝3 ⎠
Energía cuando alcanza el punto más alto
⎛ l ⎞
= − ⎜ Mg + mgd ⎟(1 − cos θ )
⎝ 2 ⎠
Por conservación de energía: Solución.
Energía justo después del choque = energía cuando Cantidad de movimiento angular antes del choque
alcanza el punto más alto. con respecto al eje O.
→ → →
1 ⎛ l ⎞ L antes = r × p = − rmv0senθ kˆ = − mbv0 kˆ
I O ω 2 − ⎜ Mg + mgd ⎟
2 ⎝ 2 ⎠ Para encontrar la cantidad de movimiento angular
⎛ l ⎞ después del choque, según la figura siguiente.
= − ⎜ Mg + mgd ⎟(1 − cos θ )
⎝ 2 ⎠
1 ⎛ l ⎞
⇒ I O ω 2 = ⎜ Mg + mgd ⎟ cos θ
2 ⎝ 2 ⎠
1
I Oω 2
⇒ cos θ = 2
⎛ l ⎞
⎜ Mg + mgd ⎟
⎝ 2 ⎠
[ ]
→ →

(mv0 d ) 2 L después = I p + (m + M )b 2 ω
1⎛1 2⎞
⎜ Ml + md ⎟
2
Por conservación de la cantidad de movimiento
2⎝3 ⎠⎛1 2⎞
2

⎜ Ml + md ⎟ angular
2

= ⎝3 ⎠ → →

⎛ l ⎞ L antes = L después
⎜ Mg + mgd ⎟
[ ]

⎝ 2 ⎠ ⇒ − rmv0 senθ kˆ = I p + (m + M )b 2 ω
m 2 v 02 d 2
= → rmv0 senθ
⎛ l ⎞⎛ 1 ⎞ ⇒ω =− kˆ
2⎜ M + md ⎟⎜ Ml 2 + md 2 ⎟ g
⎝ 2 ⎠⎝ 3 ⎠ [
I p + (m + M )b 2 ]
⇒ hmáx = l(1− cos θ )
Ejemplo 72. Se tiene una plataforma circular que
puede rotar sin fricción alrededor de un eje
d) El trabajo del proyectil cuando se incrusta contra perpendicular al centro. E1 momento de inercia de
la barra.
la plataforma con respecto al eje es I p . Un insecto
⎛1 l ⎞
W = ΔE = ⎜ mv 02 − Mg − mgd ⎟ de masa m se coloca sobre la plataforma a una
⎝2 2 ⎠ distancia b del eje. El sistema se hace girar con una
⎡1 ⎛ l ⎞⎤ velocidad angular ω 0 en el sentido horario. El
- ⎢ I Oω 2 + ⎜ − Mg − mgd ⎟⎥
⎣2 ⎝ 2 ⎠⎦ insecto empieza a correr en una circunferencia de
radio b alrededor del eje con una velocidad de
1 2 1
= mv0 − I O ω
2
magnitud constante v 0 , medida relativa a tierra.
2 2
a) ¿Cual es la cantidad de movimiento angular total
Ejemplo 71. Un bloque de masa M se pega a una si el insecto corre con la plataforma?
plataforma circular, a una distancia b de su centro. b) ¿Cuál será si corre en oposición a la rotación de
La plataforma puede rotar, sin fricción, alrededor de la plataforma?
c) ¿Es posible que el pequeño insecto pueda detener
un eje vertical alrededor de su centro. Siendo I p su la gran plataforma? ¿Cómo?
momento de inercia con respecto a ésta. Si un Solución.
proyectil de masa m que se mueve con una La cantidad de movimiento angular del sistema
velocidad horizontal v 0 , como se muestra en la antes que el insecto comience a correr es:
figura, incide y queda en el bloque. Encontrar la

40
Cuerpo rígido Hugo Medina Guzmán

( ) ( )
→ → Resolver usando la conservación de la cantidad de
L = I p + mb 2 ω 0 = − I p + mb 2 ω 0 kˆ movimiento angular.

a) Cuando el insecto corre en el mismo sentido del Solución.


giro con módulo de velocidad v 0 su cantidad de En la parte lisa no hay fuerza de fricción, en la parte
áspera aparece la tuerza de fricción, cuya línea de
movimiento angular es: acción está en el plano. Por tanto, la cantidad de
L' = (I p + mb 2 )ω '− mbvo kˆ
→ →
movimiento angular del disco respecto a un punto
de referencia en el plano permanecerá Constante
Pero como la cantidad de movimiento angular es durante todo el movimiento (por ejemplo A).
constante. La cantidad de movimiento angular total La cantidad de movimiento antes de llegar a A.
es:

L' = L = −(I p + mb 2 )ω 0 kˆ
→ →

→ → → →
L = r × M v1 = I 0 ω1
→ →
Como r × v1 = − rv1senθ kˆ = − Rv1 kˆ ,
b) En este caso, como en el caso anterior 1 → v
→ → I0 = MR 2 , ω1 = ω1 kˆ = 1 kˆ
L' = L 2 R
( )
← → 1 1
L' = − I p + mb 2 ω 0 kˆ L = − MRv1 kˆ + MRv1 kˆ = − MRv1 kˆ
2 2
c) Si es posible, tomando el caso a) La cantidad de movimiento angular después de
( )
→ →
pasar A y haber 1legado a rodar sin deslizar. Se
L' = I p + mb 2 ω '− mbvo kˆ
v2
(
= − I + mb ω kˆ
2
p ) 0
traslada con velocidad v 2 tal que ω2 =
R
.

La plataforma se detiene cuando ω ' = 0 , es decir:


(
− mbv0 kˆ = − I p + mb 2 ω 0 kˆ )
Esto sucede cuando

v0 =
(I p + mb 2 )
ω0
mb
En el sentido indicado en el caso a).
→ → → →
Ejemplo 73. Se da a un cilindro homogéneo de L' = r × M v 2 = I 0 ω 2
radio R y masa M con una velocidad horizontal v1 → →
y una velocidad angular ω1 en sentido opuesto a Como r × v 2 = − rv 2 senθ kˆ = − Rv 2 kˆ ,
las agujas del reloj ω1 = v1 R en la parte sin 1 → v
I0 = MR 2 , ω 2 = −ω 2 kˆ = 2 kˆ
rozamiento de la superficie horizontal. Más allá del 2 R
punto A, cambia la superficie de manera que a la → 1 3
derecha de A el coeficiente de rozamiento es μ . L ' = − MRv 2 kˆ − MRv 2 kˆ = − MRv 2 kˆ
2 2

41
Cuerpo rígido Hugo Medina Guzmán

→ →
Igualando L' = L , tenemos: ⎡ 1 ⎛R⎞
2

= ⎢I f = MR 2 + m⎜ ⎟ ⎥ω f
3 1 v ⎢⎣ 2 ⎝4⎠ ⎥⎦
− MRv 2 kˆ = − MRv1 kˆ ⇒ v 2 = 1
2 2 3 1
MR 2 + mR 2
Ejemplo 74. Un proyectil de masa m y velocidad ωf = 2 2
ω0
v0 se dispara contra un cilindro sólido de masa M y 1 R
radio R. El cilindro está inicialmente en reposo MR + m
2

montado sobre un eje horizontal fijo que pasa por


2 16
su centro de masa. El proyectil se mueve ⎛ M ⎞
+m ⎟
perpendicular al eje y se encuentra a una distancia 1 ⎜⎜ 2 ⎟ω 0
D < R sobre el eje. Calcular la rapidez angular del =
2⎜ M m ⎟
sistema después que el proyectil golpea al cilindro y ⎜ + ⎟
queda adherido a su superficie. ⎝ 2 16 ⎠
b)
1 1⎛1 ⎞
Ki = I iω02 = ⎜ MR 2 + mR 2 ⎟ω02
2 2⎝2 ⎠

=
1
(M + 2m )R 2ω02
4
Solución.
1 ⎡1 ⎛R⎞ ⎤
2
El momento angular del sistema se conserva, 1
Kf = I f ω 2f = ⎢ MR 2 + m⎜ ⎟ ⎥ω 2f
entonces 2 2 ⎢⎣ 2 ⎝ 4 ⎠ ⎦⎥
Li = L f = 1 ⎡1 ⎛R⎞
2
⎤⎛ M 2 + m ⎞ 2
2

⎢ MR + m⎜ ⎟
2
⎥⎜⎜ ⎟⎟ ω 0
⎛1 ⎞ 2 ⎢⎣ 2 ⎝4⎠ ⎥⎦⎝ M 2 + m 16 ⎠
mv0 D = Iω = ⎜ MR 2 + mR 2 ⎟ω
⎝2 ⎠ = 1 ⎛⎜ M + m ⎞⎟⎛⎜ M + 2m ⎞⎟ R 2ω 2
2

8 ⎠⎝ M + m / 8 ⎠
0
4⎝
mv0 D
⇒ ω= = 1 ⎛⎜ M + 2m ⎞⎟(M + 2m )R 2ω 2
1 4 ⎝ M + m/8 ⎠
MR 2 + mR 2
0

2 Como ⎛⎜ M + 2m ⎞⎟ > 1
⎝ M + m/8 ⎠
Ejemplo 75. Un disco de masa M y radio R gira en La energía rotacional aumenta.
un plano horizontal en torno a un eje vertical sin
roce. Un gato de masa m camina desde el borde del Ejemplo 76. La barra horizontal de la figura tiene
disco hacia el centro. Si la rapidez angular del un momento de inercia respecto al eje de rotación
sistema es ω0 cuando el gato está en el borde del de 5x10-3 kg m2, y cada una de las bolas que pueden
disco, calcular: deslizar sobre ella pesan 50 g y se consideran de
a) la rapidez angular cuando el gato ha llegado a un dimensiones despreciables. El conjunto está girando
punto a R/4 del centro, libremente alrededor del eje O-O’ con las bolas
b) la energía rotacional inicial y final del sistema. dispuestas simétricamente respecto al eje y sujetas
Solución. por un hilo AB de 20 cm. Si se rompe el hilo
Llamando Id al momento de inercia del disco e Ig al cuando el conjunto gira a 20 rad/s, determinar la
momento de inercia del gato, el momento de inercia nueva velocidad angular cuando las bolas lleguen a
total inicial y final del sistema es: los topes del extremo de la barra.
1
Ii = Id + I g =
MR 2 + MR 2
2
2
1 ⎛R⎞
I f = MR + m⎜ ⎟
2

2 ⎝4⎠
a) Como no hay torques externos sobre el sistema
en torno al eje de rotación, se puede aplicar la
conservación de la cantidad de movimiento angular
I iωi = I f ω f
⎡ 1 2⎤
⎢⎣ I f = 2 MR + mR ⎥⎦ω 0
2

Solución.

42
Cuerpo rígido Hugo Medina Guzmán

Empecemos calculando el momento de inercia del 2


ωf = 180 = 120 rpm.
conjunto, cuando las bolas están separadas 20 cm.
(2 + 1)
I1 = Ibarra + Ibolas = Ibarra + 2 m r12 = 5x10-3 kg m2 +
0,1x0,12 = 6x10-3 kg m2 GIROSCOPOS Y TROMPOS - MOVIMIENTO
Cuando se alejen hasta los topes: DE PRECESION
I2 = Ibarra + Ibolas = Ibarra + 2 m r22 El giróscopo es una rueda montada en rodamientos
= 5x10-3 kg m2 + 0,1x0,252 sin fricción, en tal forma que la rueda tiene libertad
de rotar en cualquier dirección con respecto al
= 11,25x10-3 kg m2 marco que lo sujeta.
La rotura del hilo libera fuerzas exclusivamente Para lograr esto se necesitan tres gímbalos
internas, por lo que se conservará la cantidad de (correspondientes a los tres espacios
movimiento angular del sistema: dimensionales). Como los rodamientos no tienen
L1 = L2 ⇒ I 1ω1 = L2ω 2 ⇒ fricción no se ejercen torques sobre la rueda. Esto
significa que una vez iniciado el giro, el eje de
I 6
ω 2 = 1 ω1 = 20 = 10,67 rad / s rotación permanecerá fijo no importando que
I2 11,25 movimiento se de al mareo exterior. La dirección en
el espacio del eje no variará.
Hasta ahora vimos el movimiento rotacional en que
Ejemplo 77. Un disco de 2 kg de masa y 10 cm de el eje de rotación está fijo, o tiene movimiento de
radio gira alrededor de su eje a 180 r.p.m.. Encima, traslación sin cambio en su dirección. La mayoría
pero sin que exista contacto, se encuentra otro disco de los movimientos rotacionales quedan en estas
de 1 kg de masa, del mismo radio y en reposo. categorías, pero en el caso de un trompo o
Cuando el disco superior se deja caer, ambos se giróscopo en rotación no se cumple lo anterior. Si
mueven solidariamente. Calcular la velocidad se hace girar rápidamente el rotor de este aparato y
angular final. luego se coloca un extremo libre del eje de rotación
sobre un soporte fijo, como se muestra en la figura.
El giróscopo no caerá del soporte sino que se
mantiene en posición casi horizontal mientras que
el eje de su rotor gira lentamente en un plano
horizontal, esta rotación lenta del eje se conoce
como PRECESION.

Solución.

Cuando el disco superior se posa sobre el inferior,


el torque de las fuerzas sigue siendo nulo por lo que
se conserva la cantidad de movimiento angular,
Iω .
Veamos como se origina la precesión.
(Iω )Antes = (Iω )Después Consideremos un giróscopo simplificado mostrado
en la figura siguiente, un disco cilíndrico muy
macizo de masa M y radio a que tiene libertad para
I1 girar sin fricción en torno a una varilla muy ligera y
I1ωi = (I1 + I 2 )ω f ⇒ ω f = ωi delgada, a lo largo de su eje.
I1 + I 2

Como el Momento de inercia de un disco es


½.m.R2 se obtiene:

1
m1 R 2
2 m1
ωf = ωi = ω
⎛1 1 2⎞ (m1 + m2 ) i
⎜ m1 R + m2 R ⎟
2

⎝2 2 ⎠
Un extremo de la varilla se apoya en A. que está a
En este caso particular: una distancia l del disco. Si se mantiene la varilla

43
Cuerpo rígido Hugo Medina Guzmán

horizontal, y se hace girar al disco con una es de 2,7 kg m2. La velocidad angular inicial de la
velocidad angular ω en torno a su eje y luego, se rueda es de 55 rad/s en sentido antihorario. En un
suelta. momento dado la profesora gira 180º el eje de la
Como actúan dos únicas fuerzas el peso Mg y la rueda pasando a girar con -55 rad/s en sentido
reacción del apoyo R, podría pensarse que el disco contrario al anterior. Calcular:
→ a) La velocidad angular adquirida por la silla y el
caería. Si L0 fuera cero sucedería esto, pero el sentido de giro.
torque que produce Mg es: b) El trabajo realizado por la profesora.

( )
τ = (liˆ )× − Mgkˆ = Mglˆj

este torque produce un cambio en la cantidad de


movimiento angular

( )
→ →
d L = τ dt = Mglˆj dt
la magnitud. de este cambio es:
dL = Mgldt
Por otra parte: dL = L0 dθ
dθ Mgl
De aquí Mgldt = L0 dθ y =
dt L0 Solución.
a) Dado que no hay momentos externos sobre la
1
Como L0 = L0ω = Ma 2ω ; silla giratoria podemos considerar que el momento
2 angular no varía.
dθ Mgl 2 gl L1 = I RUEDA ω1 ,
= = 2
dt 1
Ma 2ω
a ω L2 = I RUEDA (− ω1 ) + I SILLAω 2
2 I RUEDA ω1 = I RUEDA (− ω1 ) + I SILLAω 2
Por consiguiente el disco no caerá, en lugar de ello
girará en el plano horizontal xy (ver la figura 2I
siguiente) en torno al eje vertical a través del punto
⇒ ω 2 = RUEDA ω1
I SILLA
de apoyo A.
2(0,02)
ω2 = 55 = 8,15 rad /s
2,7
(Positivo, por tanto en el sentido de rotación inicial
de la rueda)
b)
W = ΔE = E 2 − E1
I SILLAω 22 + I RUEDA (− ω1 ) − I RUEDA ω1
=
1 1 2 1 2

La velocidad angular de esta precesión es:


2 2 2
dθ τ 2 gl 1
Ω= = = = I SILLAω 22 = 89,6 J
dt Iω ωa 2 2
El trabajo es por tanto la energía adquirida por la
silla, ya que la energía de la rueda no varía.
Ejemplo 78. Una profesora de física se encuentra
sentada en una silla giratoria manteniendo en sus
Dicho trabajo, positivo, es producido por la fuerza
manos una rueda de bicicleta como se indica en la
muscular (interna) de la profesora.
figura. El momento de inercia de la rueda respecto a
su eje es de 0,2 kg m2, y el momento de inercia de
la profesora más la rueda respecto del eje de la silla

PREGUNTAS Y PROBLEMAS

1. El centro de masa de una pelota de radio R, se rotacional y la energía cinética de traslación.


mueve a una rapidez v. La pelota gira en torno a un Considere la pelota una esfera uniforme.
eje que pasa por su centro de masa con una rapidez
angular ω. Calcule la razón entre la energía 2. Un volante en la forma de un cilindro sólido de
radio R = 0,6 m y masa M = 15 kg puede llevarse

44
Cuerpo rígido Hugo Medina Guzmán

hasta una velocidad angular de 12 rad/s en 0,6 s por


medio de un motor que ejerce un torque constante.
Después de que el motor se apaga, el volante
efectúa 20 rev antes de detenerse por causa de la
fricción (supuesta constante). ¿Qué porcentaje de la
potencia generada por el motor se emplea para
vencer la fricción?
Respuesta. 2.8%. .
Respuesta. a) 2(Rg/3)½, b) 4(Rg/3)½, c) (Rg)½.
3. Un bloque de masa m1 y uno de masa m2 se
conectan por medio de una cuerda sin masa que 6. Un peso de 50 N se une al extremo libre de una
pasa por una polea en forma de disco de radio R, cuerda ligera enrollada alrededor de una pelota de
momento de inercia I y masa M. Así mismo, se deja 0,25 m de radio y 3 kg de masa. La polea puede
que los bloques se muevan sobre una superficie en girar libremente en un plano vertical en torno al eje
forma de cuña con un ángulo θ como muestra la horizontal que pasa por su centro. El peso se libera
6 m sobre el piso.
figura. El coeficiente de fricción cinético es μ para
a) calcular la tensión de la cuerda, la aceleración de
ambos bloques. Determine
la masa y la velocidad con la cual el peso golpea el
a) la aceleración de los dos bloques y
piso.
b) la tensión en cada cuerda.
b) Calcular la rapidez con el principio de la
conservación de la energía.
Respuesta. a) 11,4N, 7,6 m/s2, 9,5 m/s, b) 9,5 m/s.

7. Una ligera cuerda de nylon de 4 m está enrollada


en un carrete cilíndrico uniforme de 0,5 m de radio
y 1 kg de masa. El carrete está montado sobre un
eje sin fricción y se encuentra inicialmente en
Respuesta. reposo. La cuerda se tira del carrete con una
a) (m2sen θ - μ)(m1 + m2cos θ)g/(m1 + m2 + M), aceleración constante de 2,5 m/s2. a) ¿Cuánto
b) T1 = μm2g + m1a, T2 = T1 + ½Ma. trabajo se ha efectuado sobre el carrete cuando éste
alcanza una velocidad angular de 8 rad/s?
4. Una masa m1 y una masa m2 están suspendidas b) Suponiendo que no hay la suficiente cuerda sobre
por una polea que tiene un radio R y una masa m3. el carrete, ¿Cuánto tarda éste en alcanzar esta
La cuerda tiene un masa despreciable y hace que la velocidad angular?
polea gire sin deslizar y sin fricción. Las masas c) ¿Hay suficiente cuerda sobre el carrete?
empiezan a moverse desde el reposo cuando están Respuesta. a) 4 J, 1,6 s, c) sí.
separadas por una distancia D. Trate a la polea
como un disco uniforme, y determine las 8. Una barra uniforme de longitud L y masa M gira
velocidades de las dos masas cuando pasan una alrededor de un eje horizontal sin fricción que pasa
frente a la otra. por uno de sus extremos. La barra se suelta desde el
reposo en una posición vertical. En el instante en
que está horizontal, encuentre
a) su rapidez angular,
b) la magnitud de su aceleración angular,
c) las componentes x e y de la aceleración de su
centro de masa, y
d) las componentes de la fuerza de reacción en el
eje.
Respuesta. a) (3g/L)½, b) 3g/2L,
c) –(3/2î + ¾ĵ)g, d) (-3/2î + ¼ ĵ)Mg.
5. Un disco sólido uniforme de radio R y masa M
puede girar libremente sobre un pivote sin fricción
que pasa por un punto sobre su borde.
Si el disco se libera desde el reposo en la posición
mostrada por el círculo.
a) ¿Cuál es la rapidez de su centro de masa cuando
el disco alcanza la posición indicada en el círculo
punteado?
b) ¿Cuál es la rapidez del punto más bajo sobre el
9. Los bloques mostrados en la figura están unidos
disco en la posición de la circunferencia punteada?
entre si por una polea de radio R y momento de
c) Repetir para un aro uniforme
inercia I. El bloque sobre la pendiente sin fricción

45
Cuerpo rígido Hugo Medina Guzmán

se mueve hacia arriba con una aceleración constante


de magnitud a.
a) Determine las tensiones en las dos partes de la
cuerda,
b) encuentre el momento de inercia de polea.

L
Respuesta. x = (1 − μ s )
2
Respuesta. a) T1 = m1 (a + gsenθ ) , 12. Determinar la tensión en el cable AB que
T2 = m2 ( g − a )
Impide que el poste BC deslice. En la figura se ven
los datos esenciales. La masa del poste es de 18 kg.
2 g g Suponer que todas las superficies son lisas.
b) m2 R − m1R 2 − m2 R 2 − m1R 2 senθ
a a
10. Un cuerpo plano está sometido a cuatro fuerzas
como se indica en la figura.
a) Hallar el módulo y dirección del torque actuante
respecto a un eje perpendicular al plano y que pasa
por el punto A.
b) Respecto a un eje que pasa por el punto B.
e) Respecto a un eje que pasa por el punto C.
d) Determinar la fuerza equivalente y su línea de
acción.
e) Sustituir esta fuerza por otra que esté aplicada en
A y un par de fuerzas o cupla aplicadas en los
puntos B y C y hallar el valor mínimo de estas Respuesta. T = 46,2 N
fuerzas.
13. Un hombre de 70 kg, sostiene un objeto de 31,9
kg. Como se indica en la figura. La polea carece de
rozamiento. La plataforma sobre la que está situado
el hombre está colgada mediante dos cuerdas en A y
otras dos en B. ¿Cuál e tensión de una de las
cuerdas en A?

Respuesta.
a) τ = 23 Nm, b) τ = 23 Nm, c) τ = 24 Nm,
→ Respuesta. 124,5 N
d) F = iˆ + ˆj , y = x − 23 ,
→ →
(− 3iˆ + 4 ˆj ) = − FC
23 14. Reemplace la fuerza de 1000 N de la figura por
e) FB = una fuerza que pasa por A y una cupla cuyas
25 fuerzas actúan verticalmente a través de B C.
11. Un marco cuadrado de lado L. Se cuelga de un
clavo rugoso de coeficiente de rozamiento estático
μ s . ¿A qué distancia del vértice está clavado si el
marco está a punto de deslizar?

46
Cuerpo rígido Hugo Medina Guzmán

Respuesta. 3,82 s

18. Si se aplica La fuerza F a una cuerda ligera


→ →
atada a un bloque con el sistema de poleas mostrado
Respuesta. FA = 800iˆ + 600 ˆj , FB = 467 ˆj , en la figura. ¿Cuál es el máximo peso que puede
levantar?

FC = −467 ˆj

15. Un hombre de 60 kg que camina a 2 m/s


atraviesa un tabla de 30 kg y 10 m de largó
a) ¿Cuál es la fuerza sobre el soporte B en función d
tiempo?
b) Si la máxima fuerza que puede resistir B es 490
¿Cuándo y dónde caerá al río el hombre?
Considerar que el peso del hombre siempre actúa en
dirección de la vertical que pasa por su centro de
masa.
Respuesta. 3F

19. El rodillo que se ve en la figura tiene una masa


de 339 kg ¿Que fuerza F es necesaria para subir el
rodillo sobre el bloque?
Respuesta: a) FB = (12t + 15)9,8 N, b) t = 2,92
s, x = 5,83 m de A.

16. Un hombre de masa m quiere subir por una


escalera. La escalera tiene masa M, largo L y forma
un ángulo θ con e piso. El coeficiente de fricción
entre la escalera y e peso es μ , mientras que la
pared no tiene fricción.
a)¿A qué altura de la escalera puede llegar antes que Respuesta. F =3949,4 N
comience a resbalar?
b) ¿Si el ángulo θ es el mayor sin que la escalera 20. La línea de acción de una fuerza de 1N está en
el plano xz y corta el eje z en un punto que dista 0,6
sola puede estar sin resbalar, cuál es la altura a la
m del origen.
que puede llegar el hombre?
a) ¿Cuál es el torque respecto al eje y si el ángulo
Respuesta. a)
comprendido entre la dirección de la fuerza y el eje
⎡ ⎤
⎢⎣(m + M )μLsenθ − 2 ML cos θ ⎥⎦ μM cos θ
1 z es 60º?
b) ¿Si el ángulo e l80º?
c) ¿Si el ángu1o es 330º?
b) L 2 μ Respuesta. a) τ = 0,52 N m , b) τ = 0
c) τ = - 0,3 N m
17. El disco A tiene una masa de 2 kg y un radio de
7,5 cm, se coloca en contacto con una correa que se 21. Dos discos de masa 10 kg y radio R = 0,3 m
mueve con una velocidad v = 15 m/s. Sabiendo que cada uno están conectados mediante una cuerda. En
el coeficiente de rozamiento entre el disco y la el instante mostrado en la figura, la velocidad
correa es 0,2, calcular tiempo necesario para que el angular del disco B es de 20 rad/s en sentido
disco alcance una velocidad angular constante. horario. Calcular cuánto sube el disco A cuando la
velocidad angular del disco B sea de 4 rad/s.

47
Cuerpo rígido Hugo Medina Guzmán


Respuesta. μ≥
3mgR

Respuesta. 1,54 m 25. Una esfera de l00 kg de masa y 0,6 m de


diámetro baja rodando, partiendo del reposo, por un
22. Un cilindro de masa ni y radio r rueda sin plano inclinado 25º. recorriendo 30 m..
deslizar sobre la cara interior de una superficie a) ¿Cuál es su energía cinética al cabo de los 30 m?
cilíndrica de radio R. Sabiendo que la esfera parte b) ¿Cuál es la velocidad de su centro de masa?
del raposo en la posición indicada en la figura, Respuesta. a) 1268 kg m, b) 13,3 m/s
obtener:
a) La velocidad de la esfera al paso por B. 26. Un pasajero viaja de pie en un ómnibus. El
b) El módulo de la reacción normal en cada ómnibus se mueve con una velocidad de 50 km/h
instante. cuando el conductor aplica los frenos. El ómnibus
desacelera de modo uniforme durante una distancia
de 15 ni hasta detenerse. ¿Qué ángulo respecto a la
vertical deberá inclinarse el pasajero para evitar su
caída?
Respuesta. 33,27 hacia atrás.

27. a) ¿Cómo podría distinguirse una esfera de oro


de otra de plata si ambas tuviesen el mismo peso, el
mismo radio y las dos estuvieron pintadas del
mismo color?
g (R − r )(1 − cos θ ) ,
4 b) ¿Cómo podría distinguir un huevo duro de uno
Respuesta. a)
3 fresco si estuvieran juntos?
mg
b) (7 − 4 cos θ ) 28. Un carrete cilíndrico hueco y uniforme tiene
3 radio interior R/2, radio exterior R y masa M . Está
montado de manera que gira sobre un eje horizontal
23. ¿A que altura sobre la mesa debe golpearse una fijo. Una masa m se conecta al extremo de una
bola de billar con un taco mantenido cuerda enrollada alrededor del carrete. La masa m
horizontalmente para que la bola comience su desciende a partir del reposo una distancia y durante
movimiento sin rozamiento entre ella y la mesa? un tiempo t. Demuestre que el torque debido a la
fuerza de roce entre el carrete y el eje es:
⎡ ⎛ y⎞ 5 y⎤
τ = R ⎢m⎜ g − 2 2 ⎟ − M 2 ⎥
⎣ ⎝ t ⎠ 4 t ⎦

Respuesta. 7/5R

24. Un cilindro homogéneo de masa m y radio R


descansa sobre un plano horizontal. Se aplica un
torque, según se indica en la figura. Hallar el valor
del coeficiente de rozamiento entre la rueda y el
plano para que aparezca rodadura pura. 29. Un cilindro de 10 kg de masa rueda sin deslizar
sobre una superficie horizontal.
En el instante en que se su centro de masa tiene una
rapidez de 10 m/s, determine:
a) la energía cinética traslacional de su centro de
masa,

48
Cuerpo rígido Hugo Medina Guzmán

b) la energía rotacional de su centro de masa, y c) a) el origen,


su energía total. b) el punto más alto de su trayectoria,
Respuesta. a) 500 J, b) 250 J, c) 750 J. c) justo antes de chocar con el suelo.
mv03
30. Una esfera sólida tiene un radio de 0,2 m y una Respuesta. a) 0, b) − sen 2θ cos θ ,
masa de 150 kg. ¿Cuánto trabajo se necesita para 2g
lograr que la esfera ruede con una rapidez angular 2mv03
de 50 rad/s sobre una superficie horizontal? c) − sen 2θ cos θ
(Suponga que la esfera parte del reposo y rueda sin g
deslizar).
37. Un disco sólido uniforme de masa M y radio R
31. Un disco sólido uniforme y un aro uniforme se gira alrededor de un eje fijo perpendicular su cara.
colocan uno frente al otro en la parte superior de Si la rapidez angular es ω, calcular la cantidad de
una pendiente de altura h. Si se sueltan ambos desde movimiento angular del disco cuando el eje de
el reposo y ruedan sin deslizar, determine sus rotación
rapideces cuando alcanzan el pie de la pendiente a) pasa por su centro de masa, y
¿Qué objeto llega primero a la parte inferior? b) pasa por un punto a la mitad entre el centro y el
borde.
32. Una bola de boliche tiene una masa M, radio R
y un momento de inercia de (2/5)MR2. Si rueda por 38. Una partícula de 0,4 kg de masa se une a la
la pista sin deslizar a una rapidez lineal v, ¿Cuál es marca de 100 cm de una regla de 0,1 kg de masa.
su energía total de función de M y v? La regla gira sobre una mesa horizontal sin fricción
Respuesta. 0,7Mv2. con una velocidad angular de 4 rad/s. Calcular la
cantidad de movimiento angular del sistema cuando
33. Un anillo de 2,4 kg de masa de radio interior de la regla se articula en torno de un eje,
6 cm y radio exterior de 8 cm sube rodando (sin a) perpendicular a la mesa y que pasa por la marca
deslizar) por un plano inclinado que forma un de 50 cm,
ángulo de θ = 37° con la horizontal. En el momento b) perpendicular a la mesa y que pasa por la marca
en que el anillo ha recorrido una distancia de 2 m al de 0 cm.
ascender por el plano su rapidez es de 2,8 m/s. Respuesta. a) 0,43 kgm2/s, b) 1,7 kgm2/s.
El anillo continua ascendiendo por el plano cierta
distancia adicional y después rueda hacia abajo. 39. Una mujer de 60 kg que está parada en el borde
Suponiendo que el plano es lo suficientemente largo de una mesa giratoria horizontal que tiene un
de manera que el anillo no ruede fuera en la parte momento de inercia de 500 kg⋅m2 y un radio de 2
superior, ¿qué tan arriba puede llegar?
m. La mesa giratoria al principio está en reposo y
34. Una barra rígida ligera de longitud D gira en el tiene libertad de girar alrededor de un eje vertical
plano xy alrededor de un pivote que pasa por el sin fricción que pasa por su centro. La mujer
centro de la barra. Dos partículas de masas m1 y m2 empieza a caminar alrededor de la orilla en sentido
se conectan a sus extremos. Determine la cantidad horario (cuando se observa desde arriba del sistema)
de movimiento angular del sistema alrededor del a una rapidez constante de 1,5 m/s en relación con
centro de la barra en el instante en que la rapidez de la Tierra.
cada partícula es v. a) ¿En qué dirección y con qué rapidez angular gira
Respuesta. ½( m1 + m2)vD. la mesa giratoria
b) ¿Cuánto trabajo realiza la mujer para poner en
35. Un péndulo cónico consta de masa M que se movimiento la mesa giratoria?
mueve en una trayectoria circular en un plano Respuesta. a) 0,36 rad/s, antihorario.
horizontal. Durante el movimiento la cuerda de
longitud L mantiene un ángulo constante con la θ 40. Una barra uniforme de masa M y longitud d gira
vertical. Muestre que la magnitud de la cantidad de en un plano horizontal en torno de un eje vertical
movimiento angular de la masa respecto del punto fijo sin fricción que pasa por su centro. Dos
de soporte es: pequeñas cuentas, cada una de masa m, se montan
sobre la barra de manera tal que pueden deslizar sin
gM 2 L3 sen 4θ fricción a lo largo de su longitud. Al principio las
L=
cos θ cuentas se fijan por medio de retenes ubicados en
las posiciones x (donde x < d/2) a cada lado del
centro, tiempo durante el cual el sistema gira una
36. Una partícula de masa m se dispara con una
rapidez angular ω. Repentinamente, los retenes se
rapidez vo formando un ángulo θ con la horizontal.
quitan y las pequeñas cuentas se deslizan saliendo
Determine la cantidad de movimiento angular de la
de la barra. Encuentre,
partícula respecto del origen cuando ésta se
a) la rapidez angular del sistema en el instante en
encuentra en:
que las cuentas alcanzan los extremos de la barra, y

49
Cuerpo rígido Hugo Medina Guzmán

b) la rapidez angular de la barra después de que las


cuentan han salido de ella.

41. Un bloque de madera de masa M que descansa


sobre una superficie horizontal sin fricción está
unido a una barra rígida de longitud l y masa
despreciable. La barra gira alrededor de un pivote
en el otro extremo. Una bala de masa m que se
desplaza paralela a la superficie horizontal y normal
a la barra con rapidez v golpea el bloque y queda
incrustada en él.
45. A una bola de boliche se le da una rapidez
a) ¿Cuál es la cantidad de movimiento angular del
inicial vo en una canal de manera tal que
sistema bala-bloque?
inicialmente se desliza sin rodar. El coeficiente de
b) ¿Qué fracción de la energía cinética original se
pierde en la colisión? fricción entre la bola y la canal es μ. Demuestre que
durante el tiempo en que ocurre el movimiento de
Respuesta. a) mvl , b) M/(M+m). rodamiento puro,
a) la rapidez del centro de masa de la bola es 5vo/7,
42. Una cuerda se enrolla alrededor de un disco y
uniforme de radio R y masa M. El disco se suelta b) la distancia que recorre es 12 vo2/49 μg.
desde el reposo con la cuerda vertical y su extremo (Sugerencia: Cuando ocurre el movimiento de
superior amarrado a un soporte fijo. A medida que
rodamiento puro, vcm = Rω. Puesto que la fuerza de
el disco desciende, demuestre que
fricción proporciona la desaceleración, a partir de la
a) la tensión en la cuerda es un tercio del peso del
segunda ley de Newton se concluye que acm = μg.)
disco.
b) La magnitud de la aceleración del centro de masa
46. El alambre de un carrete de masa M y radio R se
es 2g/3, y
desenrolla con una fuerza constante F. Suponiendo
c) la rapidez del centro de masa es (4gh/3)½.
que el carrete es un cilindro sólido uniforme que no
Verifique su respuesta a la pregunta c) utilizando
desliza, muestre que, a) la aceleración del centro de
métodos de energía.
masa es 4F/3M, y
b) la fuerza de fricción es hacia la derecha y su
43. Una pequeña esfera sólida de masa m y de radio
magnitud es igual a F/3.
r rueda sin deslizar a lo largo de la pista mostrada
c) Si el cilindro parte del reposo y rueda sin
en la figura. Si parte del reposo en la parte superior
deslizar, ¿Cuál es la rapidez de su centro de masa
de la pista a una altura h, donde h es grande
después que ha rodado una distancia D?
comparada con r
Respuesta. c) (8FD/3M)½.
a) Cuál es el valor mínimo de h (en función de R)
de modo que la esfera complete la trayectoria?
b) ¿Cuáles son las componentes de fuerza de la
esfera en el punto P si h = 3R?

47. Suponga un disco sólido de radio R al cual se le


da una rapidez angular ωo alrededor de un eje que
pasa por su centro y después se baja hasta una
superficie horizontal y se suelta, como en la.
44. Un proyectil de masa m se mueve a la derecha Suponga también que el coeficiente de fricción
con rapidez v0. El proyectil golpea y queda fijo en entre el disco y la superficie es μ.
extremo de una barra estacionaria de masa M y a) Calcular la rapidez angular del disco una vez que
longitud D que está articulada alrededor de un eje ocurre el rodamiento puro.
sin fricción que pasa por su centro. b) Calcular la pérdida fraccionaria de energía
a) Encuentre la rapidez angular del sistema justo cinética desde el momento en que el disco se suelta
después de la colisión. hasta que ocurre el rodamiento puro
b) Determine la pérdida fraccionaria de energía c) Muestre que el tiempo que tarda en ocurrir el
mecánica debida a la colisión. movimiento de rodamiento puro es R ωo/3 μ g.
d) Muestre que el tiempo que recorre el disco antes
de que ocurra el rodamiento puro es R2 ωo 2/18 μ g.

50
Cuerpo rígido Hugo Medina Guzmán

1
Respuesta. β = sen -1 . Estará en
⎛ M r2 ⎞
⎜⎜ + 2 ⎟⎟
⎝m R ⎠
⎛ M r2 ⎞
reposo solo si ⎜⎜ + 2 ⎟⎟ ≥ 1
⎝m R ⎠
48. La figura muestra un carrete de alambre que
descansa sobre una superficie horizontal. Cuando se 51. Los discos A y B son del mismo material y
tira, no se desliza en el punto de contacto P. El tienen el mismo espesor, pudiendo girar 1ibemente
carrete se tira en las direcciones indicadas por alrededor de un eje vertical. El disco B se encuentra
medio de los vectores F1, F2, F3 y F4. Para cada en reposo cuando se deja caer sobre el disco A. el
fuerza determine la dirección en que rueda el está girando con una velocidad angular de 400 rpm.
carrete. Advierta que la línea de acción de F2 pasa Sabiendo que la masa del disco A es de 4 kg,
por P. calcular:
a) La velocidad angular final de los discos.
b) La variación de la energía cinética experimentada
por el sistema.
R A = 0,1 m, RB = 0,15 m,

49. El carrete mostrado en la figura tiene un radio


interior r y un radio externo R. El ángulo θ entre la
fuerza aplicada y la horizontal puede variar.
Demuestre que el ángulo crítico para el cual el Respuesta. a) 334 rpm, .b).- 6,5l J
carrete no rueda y permanece estacionario está dado
por cosθ = r/R. 52. Una bala de 3g se dispara, con una velocidad
(Sugerencia: En el ángulo crítico la línea de acción horizontal de 550 m/s, contra. Una varilla de
de la fuerza aplicada pasa por el punto de contacto.) madera AB de longitud L = 0,750 m. La varilla que
inicialmente está en reposo, se encuentra
suspendida de una cuerda de longitud L = 0,750 m.
Sabiendo que h = 0,150 m, calcular las velocidades
de cada uno de los extremos de la varilla
inmediatamente después de que la bala se haya
incrustado.

50. Se tiene un carrete sobre un plano inclinado, el


cual tiene enrollado un hilo delgado y su extremo
libre sujeta una masa m por medio de una polea sin
fricción y masa despreciables. Se asume que la
masa del carrete M está distribuida uniformemente
en un círculo de radio R. Determinar el ángulo de
inclinación β al cuál el centro de gravedad del
carrete estará en reposo.

→ →
Respuesta. v A = −0,566iˆ , v B = 6,22iˆ

53. Un tablón masa M se apoya sobre un pequeño


pivote D. Un gimnasta A de masa m está de pie
sobre el extremo C del tablón, un segundo gimnasta
B de la misma masa m salta desde la altura h y cae

51
Cuerpo rígido Hugo Medina Guzmán

sobre el tablón en E. Suponiendo que este choque es


perfectamente inelástico, determinar la altura que
alcanzará el gimnasta A. (El gimnasta A permanece
de pie completamente rígido).

Respuesta. Ω = 2,1 rad/s

55. Una rueda de bicicleta de 82 cm de diámetro


tiene una platina de acero enrollada en su parte
exterior de modo que la masa resultante del sistema
puede suponerse que está situada toda ella en la
m2h periferia de la rueda, siendo M = 7,3 kg
Respuesta.
(2m + M 3)2 sosteniendo los dos extremos del eje con las manos
en la posición horizontal. El eje sobresale 15,2 cm a
cada lado de la rueda. Mientras la rueda está
54. Un disco macizo de 1,2 kg de masa y 10 cm de girando con una velocidad angular de 25,12 rad/s se
diámetro está montado en un extremo de un eje de hace girar el eje con las manos en un plano
masa despreciable que está pivotado alrededor de horizontal alrededor de su centro. Calcular el valor
un punto a 6 cm del, centro del disco en el otro y dirección de la fuerza que deberá ejercer en cada
extremo del eje, a una distancia de 10 cm del mano para producir una velocidad angular de
pivote, se cuelga un objeto de 0,96 kg de masa. Si la precesión de 0,628 rad/s alrededor del centro.
velocidad angular de giro del disco es 37,37 rad/s. Respuesta. un par de fuerzas de 64,6 N aplicadas
¿Cuál es la velocidad de precesión? en cada extremo del eje.

52
FISICA 2

Autor: Hugo Medina Guzmán


Profesor de la Pontificia Universidad Católica del Perú
Agosto 2009
PRESENTACIÓN

Me agradó saber que Hugo Medina Guzmán estaba por publicar un texto sobre Física. Había
dos razones suficientes para este sentimiento. Por un lado, tenía curiosidad de saber lo que
podría aportar un texto más de Física sobre los otros ya disponibles. Por otro lado, conozco de
la larga carrera de Hugo Medina como cultor de la enseñanza de [a Física, y tenía curiosidad
de ver cómo este compromiso como docente y experiencia se manifestarían en su texto. Tuve
la suerte de conocer al Ing. José Castro Mendívil en su taller, donde desplegó una destacada
labor en el diseño y construcción de equipo de laboratorio para la enseñanza de la Física.
Considero que Hugo es un digno discípulo del Ing. Castro Mendívil e igualmente ha dedicado
una fracción considerable de su tiempo a la docencia, y al diseño y construcción de equipo de
laboratorio para resaltar los conceptos básicos de la Física.
He revisado el contenido de este texto y veo con gran satisfacción que su autor utiliza un
enfoque muy acertado. Toma como punto de partida una observación experimental y a partir
de allí desarrolla los conceptos físicos que permiten interpretar esta observación utilizando la
formulación matemática más sencilla. Todo esto lo hace con el detalle suficiente de manera
que el lector pueda seguir el argumento lógico con facilidad. Considero que éste es un gran
aporte de este texto. Este enfoque contrasta con textos que enfatizan la formulación
matemática y dejan al alumno huérfano de una orientación para aplicarla a una realidad física
concreta.
El contenido de temas de la Física General que son desarrollados en este texto se ajusta al
programa de estudios de la PUCP. El desarrollo de cada tema incluye ejemplos bien
seleccionados que son desarrollados con un detalle muy esmerado. Al final de cada capítulo
se incluye un conjunto de preguntas y problemas propuestos; se incluye las respuestas.
Algunos problemas plantean configuraciones complejas pero que contienen ciertas
propiedades de simetría que permiten su reducción a configuraciones sencillas. Al final del
texto encontramos un listado de referencias bibliográficas a un buen número de textos de
Física General que han servido de consulta al autor.
En general, considero que este texto constituye una representación gráfica de la obra cotidiana
que Hugo ha venido desarrollando durante su carrera docente y, por lo tanto, es un aporte muy
valioso para la comunidad académica y público en general.

Lima, julio de 2007


PRÓLOGO

Los estudiantes a menudo se preguntan por qué llevan un curso de Física. La mejor razón por la que se
estudia Física es porque proporciona un método coherente y lógico para comprender el mundo que nos
rodea; una persona que comprende lo que sucede a su alrededor, es capaz de convivir en su entorno de
manera racional y efectiva. Sin embargo, en ocasiones los estudiantes ignoran el potencial que tiene la
Física para explicar el entorno en términos fáciles de entender;
Este libro tiene por objeto brindar a los estudiantes de la Física General una ayuda para dominar los
principios físicos que son la base de la tecnología moderna. En éste libro se asume que los estudiantes
tienen una base de álgebra, geometría, y trigonometría. Es mucho más compacto que los libros de
texto tradicionales, proporciona muchos ejemplos trabajados y pide resolver problemas
Este libro será útil también como texto para una persona que repasa o que consolida su conocimiento
de la Física.
La discusión y las explicaciones narrativas son suficientemente claras y completas para poder utilizar
el libro o como texto, o como suplemento a un texto más amplio.
La forma de aprender la física es trabajar realmente con problemas. Al usar este libro, el estudiante
debe ser activo. Debe intentar trabajar cada uno de los problemas y los ejemplos. Debe mirar las
soluciones solamente si no logra dar con el camino a su solución.
Los ejemplos en este libro están trabajados exhaustivamente, de modo que puedan servir como
modelos para el propio trabajo de los estudiantes. En este sentido se considera que los estudiantes se
benefician al observar los cálculos realizados en más de una manera, por lo que se han incluido varios
métodos para efectuar los cálculos.
Además, se tuvo especial cuidado en incluir problemas y preguntas que combinan el material del
capítulo en cuestión, con material de capítulos anteriores. Tales problemas y preguntas destacan el
hecho importante de que diversas áreas de la Física se manifiestan de manera simultánea en el mundo
real. Además, este método de temas múltiples proporciona una manera para que los estudiantes
repasen lo estudiado y ayuda a mejorar la habilidad para resolver problemas.
El diseño gráfico es de gran importancia, y para mejorar su función se ha intentado enfocar solamente
una idea principal en cada figura en lo posible. Por consiguiente, las figuras del libro a menudo se
dividen en dos o más partes, para evitar la confusión de mezclar varias ideas en la misma figura.
Los profesores conocen la importancia de los diagramas de cuerpo libre cuando utilizan la segunda ley
de movimiento de Newton, y todos los estudiantes aprenden de ellos a medida que estudian Física.
Tales diagramas se utilizan en todo el libro, no solamente en los primeros capítulos en los que se
presenta y aplica la segunda ley de Newton. Por ejemplo, cuando se analiza la relación en las
oscilaciones, también entre la presión y profundidad en un fluido, el análisis se simplifica
considerablemente por medio de un diagrama de cuerpo libre. De manera semejante, cuando se deduce
la expresión para la rapidez de una onda transversal en una cuerda, un diagrama de cuerpo libre es
muy útil.
Cifras significativas. A lo largo de todo el libro se siguen los procedimientos normales para las cifras
significativas.
Se espera que el esfuerzo en la elaboración de este libro sea de utilidad tanto para los estudiantes como
para los profesores. Toda opinión al respecto será bienvenida.

Hugo Medina Guzmán


Lima Perú
AGRADECIMIENTOS

El autor agradece primeramente a los estudiantes, quienes han contribuido bastante en la


elaboración de este libro a través de su influencia en el establecimiento de las técnicas y
principios de enseñanza y a los profesores que con sus sugerencias y revisiones a las
separatas de los capítulos hicieron notar puntos que necesitaban una mayor aclaración.

Hugo Medina Guzmán


CONTENIDO
CAPÍTULO 1. Elasticidad
Esfuerzo y deformación. Régimen elástico y plástico, Módulos de elasticidad y tipos de
esfuerzo y deformación: Deformación por tracción o compresión longitudinal, Módulo de
Young, ley de Hooke, aplicaciones: deformación por peso, aceleración y área variable.
Deformación lateral, módulo de Poisson Deformación por cizalladura o corte módulo de
cizalladura Deformación volumétrica módulo de compresibilidad. Fuerza elástica y Energía
elástica.
CAPÍTULO 2. Movimiento oscilatorio
Movimiento oscilatorio: definición y características Ecuación y ley del movimiento
oscilatorio: Movimiento armónico simple lineal y angular. Movimiento armónico simple.
Movimiento armónico amortiguado: subamortiguado, sobreamortiguado y crítico.
Movimiento armónico amortiguado forzado. Resonancia, aplicaciones.
CAPÍTULO 3. Movimiento ondulatorio y ondas
Movimiento ondulatorio, definición, características, y tipos de ondas: por la naturaleza de la
perturbación, por la dirección de la perturbación, por la dirección de la propagación.
Descripción matemática de la propagación de una onda unidimensional. Función de onda:
onda viajera, ecuación de onda y velocidad de propagación: ondas en una cuerda, ondas
transversales y longitudinales en una barra, ondas sonoras en un tubo con aire. Fenómenos
ondulatorios: reflexión de ondas. Principio de superposición de ondas: 1.-ondas iguales
viajando en la misma dirección, superposición constructiva o destructiva. 2.-ondas iguales
viajando en sentidos opuestos, ondas estacionarias: en una cuerda finita y en un tubo o caja
acústica finita. Modos de vibración y armónicos. 3.-ondas de diferente frecuencia viajando en
el mismo espacio, pulsaciones. Interferencia de ondas (sonoras y electromagnéticas).
Interferencia entre dos fuentes separadas en el espacio con la misma fase, diferencia de
camino. Sonido: intensidad, efecto Doppler, ondas de choque.
CAPÍTULO 4. Mecánica de fluidos
Concepto, tipos de fluido, características. Densidad, peso específico y presión. Hidrostática:
Variación de la presión con la profundidad en un fluido en reposo. Principios de Pascal.
Empuje y flotación: Principio de Arquímedes. Barómetro y manómetro simple. Aplicaciones:
superficies planas y translación de fluidos. Tensión superficial. Dinámica de Fluidos: Flujo
de fluido ideal Ecuación de continuidad, caudal o gasto. Ecuación de Bernoulli. Aplicaciones:
medidor de Venturi y tubo de Pitot. Viscocidad y ley de Stokes.
CAPÍTULO 5. Termodinámica
Sistemas Termodinámicos: Variables termodinámica macroscópicas. Ley cero de la
Termodinámica y equilibrio Térmico. Temperatura y escalas Dilatación térmica: Dilatación
Lineal, superficial y volumétrica. Fatiga térmica. Calor y trabajo: Definición de Calor,
Equivalente mecánico del calor, calor específico. Fases de la materia: cambios de estado.
Procesos de Transferencia de calor: por conducción por convección, por radiación. Teoría
Cinética de gases Ideales: Definición de un gas Ideal. Ecuación de estado de un gas ideal,
curvas Isotérmicas. Energía Interna de un Gas Ideal: Trabajo realizado por un gas. Primera
Ley de La Termodinámica. Procesos Termodinámicos: isocórico, isobárico, isotérmico y
adiabático. Calor específico de un gas a volumen constante y a presión constante. Procesos
reversibles e irreversibles. Ciclos termodinámicos. Máquinas termodinámicas. Eficiencia y
segunda ley de la termodinámica. Ciclo de Carnot.
CAPÍTULO 1

Elasticidad

INTRODUCCIÓN 1
PROPIEDADES MECÁNICAS DE LOS MATERIALES 1
ENSAYO DE TENSIÓN Y DIAGRAMA DE ESFUERZO – DEFORMACIÓN 1
DEFORMACIÓN ELÁSTICA Y PLÁSTICA 1
DIFERENCIA ENTRE LOS CUERPOS ELASTICOS Y LOS INELASTICOS 2
LEY DE HOOKE 2
ESFUERZO Y DEFORMACIÓN UNITARIA 2
MODULO ELASTICO O DE ELASTICIDAD 2
Viga horizontal sostenida mediante un tirante. 5
Deformaciones no uniformes por peso propio. 6
Deformaciones por aceleración 7
Deformación debido a la rotación 11
Deformaciones no uniformes por peso propio y área variable 12
DEFORMACION LATERAL MODULO DE POISSON 18
DEFORMACIÓN POR CIZALLADURA O CORTE. 21
DEFORMACION VOLUMETRICA 24
RELACION ENTRE CONSTANTES ELASTICAS 25
FUERZA ELASTICA Y ENERGIA ELASTICA 28
PREGUNTAS Y PROBLEMAS 29
CAPÍTULO 2

MOVIMIENTO
OSCILATORIO

INTRODUCCION 1
MOVIMIENTO OSCILATORIO 1
Definición y características 1
Oscilaciones Sinusoidales 2
DESCRIPCIÓN DEL MOVIMIENTO ARMÓNICO SIMPLE 2
EL MOVIMIENTO ARMONICO SIMPLE Y EL MOVIMIENTO CIRCULAR 3
UNIFORME
ENERGIA EN EL MOVIMIENTO ARMONICO SIMPLE 7
PROBLEMA BASICO MASA – RESORTE 7
PENDULOS 11
Péndulo simple 11
Péndulo compuesto 12
Problema del sube y baja 14
SISTEMAS DE PENDULOS Y RESORTES 15
Problema del Metrónomo 15
PENDULO DE TORSIÓN 19
MOVIMIENTO ARMÓNICO EN DOS DIMENSIONES 19
Medida del desfase entre dos señales 20
Medida de la frecuencia 21
MOVIMIENTO ARMONICO AMORTIGUADO. 22
OSCILACIONES FORZADAS 26
PREGUNTAS Y PROBLEMAS 34
CAPÍTULO 3

Movimiento ondulatorio y ondas

INTRODUCCIÓN 1
DEFINICIÓN, CARACTERÍSTICAS 1
Pulso y tren de ondas – Onda viajera 1
TIPOS DE ONDAS: 1
Según el medio por el que se propaguen 2
Según el número de dimensiones que involucran 2
Según la relación entre la vibración y la dirección de propagación 2
EXPRESIÓN MATEMÁTICA PARA UNA ONDA VIAJERA 3
ONDAS ARMONICAS 4
VELOCIDAD DE PROPAGACIÓN EN FUNCIÓN DE LAS PROPIEDADES DEL 9
MEDIO.
ECUACION DE LA ONDA 11
ENERGÍA E INFORMACIÓN TRANSFERIDA MEDIANTE ONDAS 13
REFLEXION DE ONDAS 15
PRINCIPIO DE SUPERPOSICIÓN DE ONDAS – INTERFERENCIA 15
ONDAS QUE VIAJAN EN LA MISMA DIRECCION. 16
ONDAS IGUALES VIAJANDO EN SENTIDOS OPUESTOS. ONDAS 20
ESTACIONARIAS
LOS INSTRUMENTOS MUSICALES 27
OSCILACION DE VARILLAS. DIAPASÓN 28
ONDAS DE DIFERENTE FRECUENCIA VIAJANDO EN EL MISMO ESPACIO 29
PULSACIONES O BATIDOS. 29
INTERFERENCIA DE DOS ONDAS QUE VIAJAN EN DISTINTAS 30
DIRECCIONES
EFECTO DOPPLER 34
Observador en movimiento 34
Fuente en movimiento 34
FORMACION DE UNA ONDA DE CHOQUE 42
PREGUNTAS Y PROBLEMAS 43
CAPÍTULO 4

Mecánica de fluidos

INTRODUCCIÓN 1
DENSIDAD 1
Densidad relativa 1
Peso específico 1
LA PRESIÓN EN LOS FLUIDOS 1
Unidades de presión 1
HIDROSTÁTICA 2
PRESIÓN EN UN PUNTO DE UN FLUIDO 2
VARIACIÓN DE LA PRESIÓN CON LA PROFUNDIDAD EN UN LÍQUIDO 2
EL PRINCIPIO DE PASCAL. 4
MEDIDA DE LA PRESIÓN. 5
Barómetro 5
Manómetro simple 5
Presión relativa y la presión absoluta 6
EL PRINCIPIO DE ARQUÍMEDES 7
CENTRO DE EMPUJE 7
EQUILIBRIO ROTACIONAL DE OBJETOS FLOTANTES 8
FUERZAS SOBRE LAS PAREDES O COMPUERTAS 15
Centro de presión 16
Aplicación: Superficie rectangular 16
Aplicación: Fuerza sobre una superficie de forma rectangular inclinada 17
TRASLACIÓN DE FLUIDOS 19
Rotación uniforme alrededor de eje vertical 20
TENSION SUPERFICIAL - CAPILARIDAD 22
TENSIÓN SUPERFICIAL 22
ADHESIÓN Y COHESIÓN. 24
CAPILARIDAD 25
DINÁMICA DE FLUIDOS - MOVIMIENTO DE UN FLUIDO 30
CARACTERÍSTICAS GENERALES DEL FLUJO DE FLUIDOS 30
ECUACIÓN DE LA CONTINUIDAD. 30
ECUACIÓN DE BERNOULLI. 31
Fórmula de Torricelli 32
EFECTO MAGNUS 32
Velocidad de salida de un líquido 33
Tiempo de vaciado 34
El medidor de venturi 39
VISCOCIDAD 41
FLUJO VISCOSO EN UNA TUBERIA CIRCULAR 42
FÓRMULA DE STOKES 43
Medida del coeficiente de viscosidad 43
PREGUNTAS Y PROBLEMAS 44
CAPÍTULO 5

Termodinámica

INTRODUCCION 1
Sistemas Termodinámicos: Variables termodinámicas macroscópicas 1
LEY CERO DE LA TERMODINÁMICA Y EQUILIBRIO TÉRMICO 1
TEMPERATURA Y ESCALAS 2
DILATACION TERMICA 4
FATIGA DE ORIGEN TÉRMICO 9
CALOR Y TRABAJO 12
CAPACIDAD CALORIFICA. CALOR ESPECÍFICO 12
FASES DE LA MATERIA 15
CAMBIOS DE ESTADO. CALOR LATENTE 17
Dilatación térmica y equilibrio térmico 20
TRANSFERENCIA DE CALOR 20
CONDUCCION 20
CONVECCION. 24
RADIACION 27
DEFINICIÓN DE UN GAS IDEAL 28
LEY DE BOYLE 28
LEY DE GAY-LUSSAC 28
LEY DE CHARLES. 28
ECUACIÓN DE ESTADO DE UN GAS IDEAL 29
TEORÍA CINÉTICA DE LOS GASES IDEALES 35
ENERGÍA INTERNA DE UN GAS IDEAL 36
TRABAJO REALIZADO POR UN GAS 37
PRIMERA LEY DE LA TERMODINÁMICA 38
CALOR ESPECÍFICO DEL GAS IDEAL 38
PROCESOS TERMODINÁMICOS 39
Isocórico o a volumen constante 40
Isobárico o a presión constante 40
Isotérmico o a temperatura constante 40
PROCESO ADIABATICO 47
CICLOS REVERSIBLES E IRREVERSIBLES 50
CICLOS TERMODINÁMICOS. MÁQUINAS TERMODINÁMICAS 50
CICLO OTTO 52
CICLO DIESEL 52
SEGUNDA LEY DE LA TERMODINÁMICA. 55
EL CICLO CARNOT 56
Motor y Refrigerador 58
ENTROPIA 62
PREGUNTAS Y PROBLEMAS 63
BIBLIOGRAFÍA
THEORETICAL PHYSICS, Mechanics of particles, rigid and elastic bodies, fluids and heat flow. F:
Woobridge Constant. Trinity College. Addison – Wesley Publishing Company (1959)
THEORETICAL PHYSICS,Thermodinamics, electromagnetism,waves, and particles. F: Woobridge
Constant. Trinity College. Addison – Wesley Publishing Company (1959)
The Feynman LECTURES ON PHYSICS. Volumenes I, II y III. Richard P.Feynman, Robert B. Leighton.
California Institute of Technology, Matthew Sands, Stanford University. Addison – Wesley Publishing
Company (1964)
CORRIENTES, CAMPOS Y PARTÍCULAS. Francis Bitter. Massachussets Institute of Technology. Editorial
Reverté S. A. (1964).
INTRODUCCIÓN AL ESTUDIO DE LA MECÁNICA, MATERIA Y ONDAS. Uno Ingard, William L.
Kraushaar. Editorial Reverté. (1966).
FUNDAMENTOS DE ELECTRICIDAD Y MAGNETISMO. Arthur F. Kip. University of California. Mc
Graw – Hill Book Company (1967)
CIENCIA FÍSICA Orígenes y principios Robert T. Langeman, Universidad Vanderbilt. UTEHA, (1968)
PROBLEMS IN ELEMENTARY PHYSICS. B. Bukhotsev, V: Krivchenkov, G. Myakishev, V.Shalnov. Mir
Publishers. Moscow (1971)
PROBLEMES DE PHYSIQUE COMMENTES. Tomos I y II Hubert Lumbroso. Mason et Cie, París. (1971)
ELECTRICIDAD Y MAGNETISMO PARA ESTUDIANTES DE CIENCIAS E INGENIERÍA. Luis L.
Cantú. Instituto Tecnológico y de Estudios Superiores de Monterrey. Editorial Limusa Mexico (1973)
FÍSICA PARA LAS CIENCIAS DE LA VIDA Y LA SALUD. Simon G. G. MacDonald / Desmond M.
Burns University of Dundee. Fondo educativo interamericano. (1975)
MECÁNICA NEWTONIANA, MIT Physics course. A. P. French. Editorial Reverté. (1974).
FÍSICA I y II. Solomon Gartenhaus. Purdue University. INTERAMERICANA. (1977)
TEACHING TIPS. A guidebook for the beginning College Teacher. Wilbert J. McKeachie (University of
Michigan). Seventh edition D. C. Heath and Company (1978)
FÍSICA PARA LAS CIENCIAS DE LA VIDA. Alan H. Cromer. Northeastern University. Editorial Reverté.
(1978)
GENERAL PHYSICS WITH BIOSCIENCE ESSAYS. Jerry B. Marion. University of Maryland. John
Wiley & Sons Inc. (1979)
Física general II: Teoría Hugo Medina Guzmán, Miguel Piaggio H. QC 21 M19 (Biblioteca PUCP) (1979)
Física general II: Problemas resueltos Hugo Medina Guzmán, Miguel Piaggio H. FIS 111 M364 (Biblioteca
PUCP) (1979)
Física general I: problemas resueltos Hugo Medina Guzmán, Miguel Piaggio H. FIS 104 M364 (Biblioteca
PUCP) (1981)
FÍSICA PARA ESTUDIANTES DE CIENCIAS E INGENIERÍA. 1 y 2. John P. McKelvey, Clemson
University – Howard Grotch, Pennsilvania State University. HARLA. Mexico. (1981)
Física 3: electricidad y magnetismo para estudiantes de ciencias e ingeniería
Hugo Medina Guzmán, FIS 141 M36 (Biblioteca PUCP) (1982)
EXPLORING PHYSICS Concepts and applications. Roger W. Redding North Texas State University, Stuart
Kenter, Wadsworth Publishing Company (1984)
PROBLEMAS DE FISICA. J. Aguilar Peris, Universidad Complutense de Madrid - J. Casanova Colas,
Facultad de Ciencias de Valladolid. Alambra (1985)
PROBLEMAS DE FISICA. dirigido por S. Kósel. Editorial Mir Moscú. (1986)
PROBLEMAS DE FISICA Y COMO RESOLVERLOS. Clarence E. Benett Maine University. CECSA
(1986)
PHYSICS for Engineering and Science. Michael E. Browne, Ph. D. (professor of Physics University of Idaho.
Schaum’s outline series Mcgraw-Hill (1988)
FÍSICA: VOLUMEN 1. Mecánica, ondas y termodinámica. Duane E. Roller, Ronald Blum. Editorial
Reverté. (1990).
FÍSICA: VOLUMEN 2. Electricidad, magnetismo y óptica. Duane E. Roller, Ronald Blum. Editorial
Reverté. (1990).
PROBLEMAS DE FISICA. dirigido por O. Ya. Sávchenko. Editorial Mir Moscú. (1989)
MECÁNICA. Berkeley physics course – volumen 1. Charles Kittel, Walter D. Knight, Malvin A. Ruderman.
Editorial Reverté SA. (1992).
ELECTRICIDAD Y MAGNETISMO. Berkeley physics course – volumen 2. Edward M.Purcell. Editorial
Reverté SA. (1992).
FÍSICA. Tomos I y II Tercera edición revisada (Segunda edición en español), Raymond S: Serway, James
Madison University, Mcgraw-Hill, (1993)
PROBLEMAS DE FISICA Santiago Burbano de Ercilla, Enrique Burbano de Ercilla, Carlos Gracia Muñoz,
XXVI edición, Zaragoza, MIRA editores (1994)
ONDAS. Berkeley physics course – volumen 3. Frank S. Crawford, Jr. Editorial Reverté SA. (1994).
FÍSICA Para las ciencias de la vida, David Jou Mirabent Universidad autónoma de Barcelona, Joseph Enric
Llebot Rabagliati, Universidad de Girona, Carlos Pérez garcía, Universidad de Navarra. Mcgraw-Hill, (1994)
Física uno Hugo Medina Guzmán, FIS 104 M365 (Biblioteca PUCP) (1995)
APPLIED PHYSICS. Arthur Beiser, Ph. D. Schaum’s outline series Mcgraw-Hill (1995)
TEACHING INTRODUCTORY PHTSICS A Sourcebook. Clifford E: Swartz (State University of New York,
Stony Brook) and Thomas Miner (Associate Editor The Physics Teacher 1972 – 1988). ATP Press – Springer.
(1996)
TEACHING INTRODUCTORY PHYSICS Arnold Arons University of Washington JOHN WILEY &
SONS, INC. (1997)
FÍSICA John Cutnell / Kenneth W. Johnson. Southern Illinois University. LIMUSA (1998)
FÍSICA EN LA CIENCIA Y EN LA INDUSTRIA. A . Cromer. Northeastern University. Editorial Reverté.
(2000)
FÍSICA CONTEMPORANEA Edwin Jones.– Richard Childers, University of South Carolina. Mcgraw-
Hill, (2001)
PROBLEMAS Y CUESTIONES DE FISICA. Atanasio Lleó, Begoña Betete, Javier Galeano, Lourdes Lleó,
Ildefonso Ruiz – Tapiador. Universidad Politécnica de Madrid. Ediciones Mundi – prensa (2002)
The PHYSICS of every day phenomena. A conceptual introduction to Physics. W. Thomas Griffith, Pacific
University. Mcgraw-Hill, (2004)
FÍSICA UNIVERSITARIA. Francis W.Sears, Mark W. Zemansky, Hugh D. Young (Carnegie Mellon
University) y Roger A. Freedman (University of California. Santa Barbara) Volumen 1, Volumen 2. Undecima
edición. Pearson - Addison Wesley (2004)
FIVE EASY LESSONS Strategies for successful Physics teaching. Randall D. Knight California Polytechnic
State University, San Luis Obispo. Addison Wesley (2004)
FUNDAMENTALS OF PHYSICS. David Halliday (Univ. of Pittsburgh), Robert Resnick (Rensselaer
Polytechnic Institute), Jearl Walker (Cleveland State Univ.). 7th Edition (2005)
Elasticidad Hugo Medina Guzmán

CAPÍTULO 1. Elasticidad
INTRODUCCIÓN
Hasta ahora en nuestro estudio de mecánica hemos
asumido que los cuerpos son indeformables; esto no
es cierto, aunque se justifica cuando los efectos de
las deformaciones carecen de importancia.
En este capítulo trataremos sobre los cambios de
forma producidos en un cuerpo cuando está bajo la
acción de una fuerza, esto es, en el sentido del
comportamiento de los materiales bajo la acción de
diversos esfuerzos, iniciándonos en la técnica del
diseño.

PROPIEDADES MECÁNICAS DE LOS Muestra típica de sección circular para el ensayo de


MATERIALES tensión - deformación
Muchos materiales cuando están en servicio están Durante la tensión, la deformación se concentra en
sujetos a fuerzas o cargas. En tales condiciones es la región central más estrecha, la cual tiene una
necesario conocer las características del material sección transversal uniforme a lo largo de su
para diseñar el instrumento donde va a usarse de tal longitud. La muestra se sostiene por sus extremos en
forma que los esfuerzos a los que vaya a estar la máquina por medio de soportes o mordazas que a
sometido no sean excesivos y el material no se su vez someten la muestra a tensión a una velocidad
fracture. El comportamiento mecánico de un constante. La máquina al mismo tiempo mide la
material es el reflejo de la relación entre su respuesta carga aplicada instantáneamente y la elongación
o deformación ante una fuerza o carga aplicada. resultante (usando un extensómetro). Un ensayo de
Hay tres formas principales en las cuales podemos tensión normalmente dura pocos minutos y es un
aplicar cargas: Tensión, Compresión y Cizalladura. ensayo destructivo, ya que la muestra es deformada
permanentemente y usualmente fracturada.

Además en ingeniería muchas cargas son torsionales


en lugar de sólo cizalladura.
Ensayo tensión – deformación
Sobre un papel de registro, se consignan los datos de
la fuerza (carga) aplicada a la muestra que está
siendo ensayada así como la deformación que se
puede obtener a partir de la señal de un
extensómetro. Los datos de la fuerza pueden
convertirse en datos de esfuerzo y así construirse
una gráfica tensión – deformación.
ENSAYO DE TENSIÓN Y DIAGRAMA DE
ESFUERZO – DEFORMACIÓN. El ensayo de
tensión se utiliza para evaluar varias propiedades
mecánicas de los materiales que son importantes en
el diseño, dentro de las cuales se destaca la
resistencia, en particular, de metales y aleaciones.
En este ensayo la muestra se deforma usualmente
hasta la fractura incrementando gradualmente una
tensión que se aplica uniaxialmente a lo largo del eje
longitudinal de la muestra. Las muestras
normalmente tienen sección transversal circular, Gráfica típica tensión vs deformación
aunque también se usan especimenes rectangulares.
DEFORMACIÓN ELÁSTICA Y PLÁSTICA

1
Elasticidad Hugo Medina Guzmán

Cuando una pieza se somete a una fuerza de tensión Por definición, El esfuerzo S en la barra es igual al
uniaxial, se produce una deformación del material. cociente entre la fuerza de tensión uniaxial media F
Si el material vuelve a sus dimensiones originales y la sección transversal original A0 de la barra.
cuando la fuerza cesa se dice que el material ha
sufrido una DEFORMACIÓN ELASTICA. El F N
S= , sus unidades son .
número de deformaciones elásticas en un material es A0 m
limitado ya que aquí los átomos del material son
desplazados de su posición original, pero no hasta el Deformación unitaria: Por definición, la
extremo de que tomen nuevas posiciones fijas. Así deformación unitaria originada por la acción de una
cuando la fuerza cesa, los átomos vuelven a sus fuerza de tensión uniaxial sobre una muestra
posiciones originales y el material adquiere su forma metálica, es el cociente entre el cambio de longitud
original. de la muestra en la dirección de la fuerza y la
Si el material es deformado hasta el punto que los longitud original.
átomos no pueden recuperar sus posiciones l − l 0 Δl
originales, se dice que ha experimentado una
δ= = , la deformación unitaria es una
l l
DEFORMACIÓN PLASTICA. magnitud adimensional
En la práctica, es común convertir la deformación
DIFERENCIA ENTRE LOS CUERPOS unitaria en un porcentaje de deformación o
ELASTICOS Y LOS INELASTICOS. Los porcentaje de elongación
cuerpos elásticos son los cuerpos que después de % deformación = deformación x 100 % = %
aplicarles una fuerza vuelven a su forma normal elongación
mientras que los inelásticos tienen su grado de
elasticidad muy bajo y si los deforman no vuelven a MODULO ELASTICO O DE ELASTICIDAD.
su forma original. A la constante de proporcionalidad, podemos
escribir la ley de Hooke en su forma general.
LEY DE HOOKE.
esfuerzo
En la parte de comportamiento elástico se cumple la Módulo Elástico =
Ley de Hooke. Robert Hooke fue el primero en deformación
enunciar esta relación con su invento de un volante Para el caso de Deformación por tracción o
de resorte para un reloj. En términos generales, compresión longitudinal
encontró que una fuerza que actúa sobre un resorte F
produce un alargamiento o elongación que es El esfuerzo es S= , la deformación unitaria es
directamente proporcional a la magnitud de la A
fuerza. Δl
F = −kΔl δ=
l
El signo menos es porque la fuerza es en oposición a El módulo elástico es conocido como el MODULO
la deformación. DE YOUNG.
La constante de la proporcionalidad k varía mucho
F
de acuerdo al tipo de material y recibe el nombre de
Y= A =S
constante del resorte o coeficiente de rigidez.
Δl δ
F N l
k= , sus unidades son .
Δl m TABLA I
Módulo de elasticidad o módulo de Young.
ESFUERZO Y DEFORMACIÓN UNITARIA.
Esfuerzo. Consideremos una varilla cilíndrica de Módulo de
Nombre elasticidad Y
longitud l 0 y una sección transversal de área A0
1010 N/m2
sometida a una fuerza de tensión uniaxial F que Aluminio 6,8
alarga la barra de longitud l 0 a l , como se muestra Cobre 10,8
en la figura. Oro 7,6
Hierro, fundido 7,8
Plomo 1,7
Nickel 20,6
Platino 16,7
Plata 7,4
Latón 4,6
Acero 20,0

Ejemplo 1. Los ortodoncistas usan alambres de bajo


módulo de Young y alto límite elástico para corregir

2
Elasticidad Hugo Medina Guzmán

la posición de los dientes mediante arcos tensores.


¿Por qué?
Solución.
Bajo módulo de Young para que sea relativamente
fácil deformarlo elásticamente para montar los arcos
en los dientes. La tensión deberá ser menor que la
tensión de fluencia del material, de ahí que el límite
elástico tenga que ser alto, ya que si el arco se
deforma plásticamente, su deformación es
irreversible y por lo tanto, no estará tensionando los Suma de fuerzas verticales:
∑F
dientes para corregir su posición transversal se
convierte en un paralelogramo. y =0
2Tsenα − Mg = 0 ⇒
Ejemplo 2. De un alambre de cobre de 1,5 m de
Mg
longitud y 2 mm de diámetro se cuelga un peso de 8 T= .
kg. Se pregunta: 2senα
a) ¿Hemos rebasado el límite de elasticidad? Por la ley de Hooke deducimos
b) ¿Se romperá el alambre?
c) En caso de ser negativas las preguntas anteriores,
que
¿cuál es su alargamiento? ⎛ Δl ⎞
Módulo de Young = 12x1010 N/m2
T = ⎜ ⎟YA
⎝ l ⎠
Límite de elasticidad de 3x107 a 12x107 N/m2
Límite de ruptura de 20x107 a 50x107 N/m2 Igualando:
Solución. ⎛ Δl ⎞ Mg
⎜ ⎟YA =
a) y b) La sección del alambre es:
⎝ l ⎠ 2senα
A = πr2 = 3,14 mm2 = 3,14x10-6 m2
La fuerza que corresponde a cada m2 de sección es: De la figura siguiente:
F Mg 8 × 9,8
= =
A A 3,14 × 10 −6
N
= 2,49 × 107 2
m
Que no llega ni al límite inferior de elasticidad ni al
de ruptura. l
Fl 8 × 9,8 × 1,5 l' = y l' = l + Δl
c) Δl = = cos α
YA 12 × 1010 × 3,14 × 10− 6 De aquí:
= 0,0003 m l ⎛ 1 ⎞
= l + Δl ⇒ Δl = l⎜ − 1⎟ ⇒
= 0,3 mm cos α ⎝ cos α ⎠
Δl 1
Ejemplo 3. Entre dos columnas fue tendido un = −1
alambre de longitud 2 l . En el alambre, exactamente l cos α
en el centro, fue colgado un farol de masa M. El área Luego
de la sección transversal del alambre es A, el módulo ⎛ 1 ⎞ Mg
de elasticidad es Y. Determinar el Angulo α, de ⎜ − 1⎟YA =
pandeo del alambre, considerándolo pequeño. ⎝ cos α ⎠ 2senα

Para ángulos pequeños tenemos que


senα ≈ α y
cos α = 1 − 2sen 2 α( 2)≈ 1 − α 2

2
.
Reemplazando obtenemos
⎛ ⎞
⎜ ⎟
⎜ 1 2 − 1⎟YA = Mg
Solución. ⎜ α ⎟ 2α
Para encontrar la tensión del hilo. ⎜1− ⎟
Por condición de equilibrio: ⎝ 2 ⎠

3
Elasticidad Hugo Medina Guzmán

⎡⎛ α 2 ⎞ ⎤ Mg
⇒ ⎢⎜⎜1 + ⎟⎟ − 1⎥YA =
⎣⎝ 2 ⎠ ⎦ 2α
α2 Mg Mg
⇒ YA = ⇒ α3 =
2 2α YA
Finalmente
Mg
α =3
YA
Solución.
Ejemplo 4. Se cuelga una viga de 2000 kg de dos Partiendo de los conceptos de simetría, es evidente
cables de la misma sección, uno de aluminio y otro de que el alargamiento de los hilos será igual.
acero. Al suspenderla, ambos cables se estiran lo Designemos este alargamiento por Δl .
mismo. Calcular la tensión que soporta cada uno. De acuerdo con la ley de Hooke, la tensión del hilo
Módulos de Young: acero = 20x1010 N/m2, aluminio de acero es
=7x1010 N/m2 AYa
Fa = Δl y la del hilo de cobre, es
l
AYc
Fc = Δl
l
De donde concluimos que la relación de las
tensiones es igual a la relación de los módulos de
elasticidad correspondientes:
Fc Yc 1
= = .
Fa Ya 2
En equilibrio
2Fc + Fa = mg.
Por consiguiente,
mg
Fc = = 250 N y Fa = 2Fc = 500 N.
4
Solución. Ejemplo 6. Una columna de hormigón armado se
Si los cables inicialmente tienen igual longitud y la comprime con una fuerza P. Considerando que el
viga finalmente está horizontal, ambos cables han módulo do Young del hormigón Yha, es 1/10 del de
experimentado el mismo alargamiento: hierro Yh y que el área de la sección transversal del
Fl lT1 lT2 hierro es 1/20 de la del hormigón armado, encontrar
Como Δl = , = de aquí qué parte de la carga recae sobre el hormigón.
YA Y1 A Y2 A
T1 T2 Solución.
= Basándonos en la ley de Hooke, escribimos
7 20
⎛ Δl ⎞
Donde el subíndice 1 se refiere al aluminio y el 2 Fha = ⎜ ⎟ Aha Yha y
al acero. ⎝ l ⎠
Por estar el sistema en equilibrio:
⎛ Δl ⎞ ⎛ Δl ⎞ A
T1 + T2 = Mg = 2 000 x 9,8 N Fh = ⎜ ⎟ AhYh = = ⎜ ⎟ ha 10Yha
De ambas ⎝ l ⎠ ⎝ l ⎠ 20
T1 = 5 081,5 N T2 = 14 517,5 N
F
De allí deducimos que ha = 2 .
Ejemplo 5. Una barra homogénea, de masa m = 100 Fh
kg, está suspendida de tres alambres verticales de la De este modo, 2/3 del peso recae sobre el hormigón
misma longitud situados simétricamente. armado y 1/3, sobre el hierro.
Determinar la tensión de los alambres, si el alambre
del medio es de acero y los otros dos son de cobre.
El área de la sección transversal de todos los Ejemplo 7. Un peso W se encuentra sujeto entre dos
alambres es igual. barras de peso despreciable, de las mismas
El módulo de Young del acero es dos veces mayor
características pero de diferente longitud y como se
que el del cobre. muestra en la figura. Los extremos de las barras

4
Elasticidad Hugo Medina Guzmán

están ligados al peso y a los apoyos, los cuales son


indeformables. Solución.
Encontrar las reacciones que se producen en los Bajo la acción de la fuerza de compresión F, el tubo
apoyos. disminuye en Fl / AY . y bajo la acción de la
fuerza de extensión F, el perno se alarga en el valor
Fl / AaYa . La suma Fl / AaYa + Fl / AcYc es
igual al desplazamiento de la tuerca a lo largo del
perno:
Fl / AaYa + Fl / AcYc = h , de donde:
h ⎛ AaYa AcYc ⎞
F= ⎜ ⎟.
l ⎜⎝ AaYa + AcYc ⎟⎠
Solución.
Diagramas del cuerpo libre del conjunto y de las
partes: Ejemplo 9. Viga horizontal sostenida mediante un
tirante. En el sistema mostrado en la figura, ¿cuánto
bajará el peso W respecto a la posición en la cual el
tensor no estaba deformado?

Por equilibrio estático, ∑F y = 0:


R1 + R2 − W = 0 (1)
Geométricamente, tiene que cumplirse que los
alargamientos sean iguales: La barra es indeformable y de peso P.
Δl 1 = Δl 2 El tensor BC es de peso despreciable, área A y
Por elasticidad módulo de elasticidad Y.
Solución.
R1l 1 R2l 2
= ⇒
AY AY
R1l 1 = R2 l 2 (2)
Resolviendo las ecuaciones (1) y (2), obtenemos:
l2 l
R1 = W y R2 = 1 W
L L
Ejemplo 8. Un perno de acero se enrosca en un tubo
de cobre como muestra la figura. Encontrar las
fuerzas que surgen en el perno y en el tubo debido al
Por equilibrio estático, ∑τ o =0
hacer la tuerca una vuelta, si la longitud del tubo es Tl - Pl - W 2l = 0
l , el paso de rosca del perno es h y las áreas de la T - P - 2W = 0
sección transversal del perno y del tubo son iguales a T = P + 2W (1)
Aa, y Ac respectivamente

Geométricamente, considerando que el giro que se


produce es pequeño, podemos escribir:
x = 2 Δl
Por elasticidad, el estiramiento Δl del tensor es:
Tl
Δl =
AY

5
Elasticidad Hugo Medina Guzmán

Luego,

2Tl
x = (2)
AY
Reemplazando la expresión (1) en (2):

2(P + 2W )l
x =
AY Solución.

Ejemplo 10. Deformaciones no uniformes por


peso propio.
Determinar la deformación producida en una barra
debido a su peso propio de una barra del largo L,
sección A, módulo de elasticidad Y y densidad ρ .
Solución.
El elemento diferencial dy soporta el peso P ' de la
porción de barra de longitud y que está sobre él.
El elemento de columna dy es deformado por el peso
de la masa m.
mg dy
d (ΔL ) =
YA
Cálculo de m.
P' = m' g = ρV ' g = ρAyg dm = ρ l dy = κydy ⇒
L
Siendo la longitud de la barra L, su deformación será L y2
ΔL , la deformación del elemento diferencial dy m = ∫ κydy = κ
y 2
debido al pesoP' , será d (ΔL ) . y

κ
d (ΔL ) =
P' dy ρAg
= ydy
=
2
(L 2
− y2 )
YA YA Luego:
ρg κg
=
Y
ydy d (ΔL ) =
2YA
(L 2
)
− y 2 dy
Luego
Integrando
ρg
ΔL = ∫ d (ΔL ) =
L
κg
Y ∫
0
ydy ΔL = ∫ d (ΔL ) =
0
L

2YA ∫
(L
0
L
2
)
− y 2 dy
1 ρgL 2
1 (ρgAL )L L
= = κg ⎛ y3 ⎞
2 Y 2 AY ΔL = ⎜ L y − ⎟⎟ 2

1 (Peso Total ) × L 2YA ⎜⎝ 3 ⎠0


o ΔL =
2 AY κg ⎛ 3 L3 ⎞ κgL3
⎜ L − ⎟⎟ =
2YA ⎜⎝
=
Observamos que esta deformación es igual a la 3 ⎠ 3YA
mitad de la deformación que se produciría, como sí,
el peso estuviera concentrado en el extremo Como la masa total es
L
superior. L y2
L
M =∫ dm = ∫ κydy = κ
Ejemplo 11. Una barra de masa M, módulo Y,
0 0 2 0
sección A y altura L está sobre el piso. Determine la
L 2
deformación que sufre la atura de la barra por peso = κ
propio. Considere que la densidad lineal de la barra 2
varía según ρ l = κy , ( κ es constante e y la altura 2M κgL3 2MgL
ΔL = 2 =
medida desde el piso). κL 3YA 3YA
Datos: M, Y, A, L y κ .
Ejemplo 12. Hállese la longitud que ha de tener un
hilo de alambre, de densidad 8,93 y módulo de
rotura 1020,4 kg/cm2 para que se rompa por su
propio peso.

6
Elasticidad Hugo Medina Guzmán

Solución. F
1020,4 kg/cm2 = 1 020,4x9,8 N/cm2 =108 N/m2; d (ΔL) = xdx , y
ρ = 8930 kg/m3. YAL
x=L
Para que el hilo se rompa, su peso ha de ser por lo F
menos de 108A N, siendo A la sección. ΔL = ∫ d (ΔL) = ∫ YAL xdx
O sea: x =0

P = mg = Alρg = 10 A De donde ΔL = 1 FL
8

Es decir: 2 YA
10 8 A 10 8 Ejemplo 14. Se tiene una columna de largo L,
l= = =1143,6 m
Aρg 8930 x9,8 sección transversal A, densidad ρ, módulo de
elasticidad Y. Se jala cobre un piso liso de la manera
Ejemplo 13. Deformaciones por aceleración como se muestra en la figura. Calcule cuanto estira
Una barra uniforme de acero (Longitud L, área de el cuerpo.
sección recta A densidad ρ , módulo de young Y) se
halla sobre un plano horizontal exento de rozamiento
y se tira de ella con una fuerza constante F.
¿Cuál es el alargamiento total de la barra a
consecuencia de la aceleración?
Solución.
Primer método.
Aplicando la segunda ley de Newton:
Solución.
a) Sea m la masa total de la barra
∑ F = ma
2F 2F
m = ρAL 3F − F = ma ⇒ a = =
Tomemos un elemento diferencial dx, cuya masa es m ρAL
dm
dm = ρAdx

Haciendo el diagrama del cuerpo libre

Hagamos los diagramas del cuerpo libre de los tres El elemento diferencial es estirado por la fuerza R2.
sectores.
R2 dx
La fuerza sobre cada uno de los tres sectores se d (ΔL ) =
indica en las figura a continuación AY
Cálculo de R2:
R2 − F = m' a
2F
⇒ R2 = F + m' a = F + ρAx
ρAL
El elemento diferencial dm se mueve con
aceleración a debido a la fuerza (R1 –R2) x
= F + 2F
Y la fuerza que lo estira es R2. Por lo tanto su L
deformación será un diferencial de ΔL esto es
F ⎛ 2x ⎞
d (ΔL ) : d (ΔL ) = ⎜1 + ⎟dx
AY ⎝ L⎠
R2 dx L
d (ΔL) = y ΔL = ∫ d ( ΔL ) L
YA F L ⎛ 2x ⎞ F ⎛ x2 ⎞
ΔL = ∫ ⎜1 + ⎟dx = ⎜⎜ x + ⎟⎟
0

Como R2 = m' a , m' = ρAx y AY 0


⎝ L⎠ AY ⎝ L ⎠0
F F 2 FL
a= = , tenemos: =
m ρAL AY
⎛ F ⎞ x
R2 = (ρAx )⎜⎜ ⎟⎟ = F Segundo método.
⎝ ρAL ⎠ L El sistema de fuerzas puede ser desdoblado en dos
partes cuyas deformaciones parciales sumadas hacen

7
Elasticidad Hugo Medina Guzmán

el efecto total, tal como se muestra en la figura R 2 − m' g = m' a ⇒ R 2 = m' ( g + a ) ,


siguiente:
F − mg ⎛ F ⎞
m' = ρAy y a = = ⎜⎜ − g ⎟⎟ ,
m ⎝ ρAL ⎠
Tenemos:
⎛ F ⎞ y
R2 = (ρAy )⎜⎜ ⎟⎟ = F
⎝ ρAL ⎠ L
F
d (ΔL) = ydy , y
YAL
F L
ΔL = ∫ d (ΔL) =
YAL ∫0
ydy
La primera parte es la deformación de un cuerpo
De donde
jalado por la fuerza 2F:
1 FL
1 (2 F )L FL ΔL =
ΔL1 = = 2 YA
2 YA YA
La segunda parte es la deformación de un cuerpo Ejemplo 16. Para la barra compuesta mostrada
sujeto a la tensión F: determine:
FL a) Su aceleración.
ΔL2 = b) La deformación de cada una de sus tres partes y
YA su deformación total.
La deformación total es la suma de las
deformaciones parciales:
FL FL
ΔL = ΔL1 + ΔL2 = +
YA YA
2 FL Solución.
=
AY a) m1 = 2 ρLA , m2 = 4 ρLA y m3 = 2 ρLA
Aplicando la segunda ley de Newton:
∑ F = ma ⇒ 3F − 7 F = (m1 + m2 + m3 )a
Ejemplo 15. Si la barra se jala hacia arriba con una
fuerza F (F > mg). ¿Cuál es el alargamiento total de
la barra? ⇒ − 4 F = 10 ρLAa
Solución.
0,4 F
⇒ a=−
ρLA
El conjunto se mueve hacia la izquierda.
b) La figura siguiente muestra los diagramas del
cuerpo libre de cada uno de los elementos del
conjunto.

Tomando como positivo hacia la izquierda.


Cálculo de R2:
R2 − 3F = m3a ⇒
El elemento diferencial dm se mueve con R2 = 3F + m3a
aceleración a debido a la fuerza (R1 –R2) ⎛ 0,4 F ⎞
Y la fuerza que lo estira es R2. Por lo tanto su = 3F + (4 ρLA)⎜⎜ ⎟⎟
deformación será un diferencial de ΔL esto es ⎝ ρLA ⎠
d (ΔL ) : = 4,6 F
R2 dy L Cálculo de R1:
d (ΔL) = y ΔL = ∫ d ( ΔL) R1 − R2 = m2 a ⇒
YA 0

Como R1 = R2 + m2 a
⎛ 0,4 F ⎞
= 4,6 F + (4 ρLA)⎜⎜ ⎟⎟
⎝ ρLA ⎠
8
Elasticidad Hugo Medina Guzmán

= 5,2 F
Deformación de 3.
La deformación por fuerza es debido a 3F:
3F 4 L FL
ΔL3 = = 12
YA YA
La deformación por desplazamiento es debido a ser
jalado por la fuerza R2 – 3F = 1,6 F
1,6 F 4 L FL
ΔL'3 = = 3,2
2YA YA
Deformación total de 3:
FL FL FL Solución.
ΔL3Total = 12 + 3,2 = 15,2
YA YA YA Para calcular la aceleración de la barra aplicamos:
Deformación de 2.
La deformación por fuerza es debido a R2:
∑F y = ma y
3
R 2L FL 5Mg − Mg − Mg = 2Ma ⇒ a = g
ΔL2 = 2 = 9,2 2
YA YA
La deformación por desplazamiento es debido a ser
jalado por la fuerza
R1 - R2 = 5,2 F – 4,6 F = 0,6 F
0,6 F 2 L FL
ΔL' 2 = = 0,6
2YA YA
Deformación total de 2:
FL FL
ΔL2Total = 9,2 + 0,6
YA YA
FL
= 9,8
YA
Deformación de 1.
La deformación por fuerza es debido a R1: Tomemos un elemento diferencial de la barra dy
RL FL
ΔL1 = 1 = 2,6 Aplicando la segunda ley de Newton al elemento de
Y 2A YA longitud x:
La deformación por desplazamiento es debido a ser ⎛ y⎞ ⎛ y⎞
R 2 − R3 − ⎜ M ⎟ g = ⎜ M ⎟a
jalado por la fuerza 7F- R1 = 1,8 F ⎝ L⎠ ⎝ L⎠
1,8 FL FL y
ΔL'1 = = 0,45 R 2 − R3 = M ( g + a )
2Y 2 A YA L
Deformación total de 1: y⎛ 3 ⎞ 5Mg
FL FL R 2 − R3 = M ⎜ g + g ⎟ = y (1)
ΔL1Total = 2,6 + 0,45 L⎝ 2 ⎠ 2L
YA YA Aplicando la segunda ley de Newton a la masa
FL puntual:
= 3,05 3
YA R3 − Mg = Ma = M g⇒
Deformación total del conjunto. 2
FL FL FL 3 5
ΔLTotal = 15,2 + 9,8 `+3,05 R3 = Mg + M g = Mg (2)
YA YA YA 2 2
FL Reemplazando (2) en (1):
= 28,05 5Mg 5Mg
YA R2 − = y
2 2L
Ejemplo 17. Una barra vertical de longitud L, masa ⇒ R2 = 5 Mg ⎛⎜1 + y ⎞⎟
M, sección transversal A y módulo de Young Y, tiene 2 ⎝ L⎠
soldada en su extremo inferior una masa puntual M. Primer método.
Si la barra se eleva verticalmente mediante una Comenzando con la deformación del elemento
fuerza vertical 5Mg (g = gravedad), aplicada en el diferencial y luego integrar para toda la longitud.
extremo superior de la barra. Hallar la deformación
longitudinal de la barra.

9
Elasticidad Hugo Medina Guzmán

arrastrado sobre un plano liso, con una fuerza F =


2W.
a) Hallar la deformación longitudinal unitaria
cuando el plano es horizontal.
El elemento diferencial se deforma d (ΔL ) debido a
b) Hallar la deformación de la dimensión paralela al
plano, cuando el bloque sube sobre el plano que esta
la reacción R2 , (R1 − R2 ) le da la aceleración inclinado 37º.
3
a= g , luego:
2
5 ⎛ y⎞
Mg ⎜1 + ⎟dy
R dy 2 ⎝ L⎠
d (ΔL ) = 2 =
YA YA Solución.
5Mg ⎛ y⎞ a)
= ⎜1 + ⎟dy ΔL 1 2W W
2YA ⎝ L ⎠ = = 2
Integrando: L 2 YL2
YL
5Mg L ⎛ y⎞ 5Mg ⎛ L2 ⎞
2YA ∫0 ⎝ L ⎠
ΔL = ⎜ 1 + ⎟ dy ⎜ L + ⎟ b)
2YA ⎜⎝ 2 L ⎟⎠
=
Resuelto por integración.
Calculo de la aceleración.
15MgL
=
4YA ∑ F = ma ⇒
W W
Segundo método. 2W − Wsen37º = a ⇒ 2W − 0,6W = a
Comenzando con la deformación la los efectos de
g g
las fuerzas en los extremos de la barra. ⇒ a = 1,4 g
Nota: En R3 ya está considerado el peso de la masa
puntual M colocada en el extremo inferior de la
barra.

El diagrama del cuerpo libre

Cálculo de R2:
Deformación de la barra por 5Mg: x W x
1 5MgL 5MgL R2 − W sen37º = a⇒
ΔL1 = = L g L
2 YA 2YA 0,6 x W x x
Deformación de la barra por R3: R2 = W + 1,4 g = 2W
1 5MgL 5MgL L g L L
ΔL2 = = El elemento diferencial se deforma dΔL :
2 2YA 4YA
Deformación total: ΔL = ΔL1 + ΔL2
R dx 2W
dΔL = 2 2 = 3 xdx
YL YL
5MgL 5MgL
ΔL = + Para hallar ΔL integramos desde x = 0 hasta x = L.
2YA 4YA 2W L W
15MgL ΔL = ∫ dΔL = ∫ xdx =
= YL3 0 YL
4YA La deformación es:
Aquí no se considera el efecto del peso propio por
W
separado, porque en el cálculo de R2 ya está ΔL =
considerado. YL
Ejemplo 18. Un cubo como se muestra en la figura Resuelto directamente usando resultados
de peso “W” arista “L” módulo de Young “Y” es conocidos.

10
Elasticidad Hugo Medina Guzmán

Estiramiento debido a la aceleración:

Calculo de la aceleración.
∑ F = ma ⇒
W W
2W − Wsen37º = a ⇒ 2W − 0,6W = a
g g
⇒ a = 1,4 g Parte 1: Cálculo de la fuerza total sobre una sección
1 (2W − 0,6W )L 0,7W transversal a la distancia r del pivote.
ΔLa = =
2 YL2 YL
Estiramiento debido al peso:

1 0,6WL 0,3W
ΔL p = =
2 YL2 YL
Estiramiento total: Debido a la aceleración centrípeta se tiene una
fuerza:
0,7 0,3W W
ΔL = + = dF = (dm )a c = (dm )ω 2 r
YL YL YL
dm = ρAdr '
Ejemplo 19. Deformación debido a la rotación dF = (ρAdr ')ω 2 r ' = ρAω 2 r ' dr '
Una barra de longitud l , área A, densidad ρ y Integrando:
módulo de Young Y gira con velocidad angular ω l l
constante sobre una mesa horizontal sin fricción y F = ∫ ρAω 2 r ' dr ' = ρAω 2 ∫ rdr
r r
pivotado en uno de sus extremos. Determinar el
F = ρAω 2 (l 2 − r 2 )
alargamiento producido. ¿Cuál será el esfuerzo
1
máximo? 2
Parte 2: Cálculo del alargamiento
El alargamiento del elemento dr es:
Fdr
d (Δl ) =
YA
Y el alargamiento total será:
Fdr ρAω 2 l 2
Δl = ∫
r YA
l
=
2YA ∫r
(l − r 2 )dr

ρω 2
l3 1 ρω 2 l 3
Δl = (l - ) =
3

2Y 3 3 Y
Solución.
El elemento diferencial se alarga d (Δl ) , debido a
la fuerza centrípeta producida por la masa restante
hacia el extremo opuesto al pivote.

11
Elasticidad Hugo Medina Guzmán

Ejemplo 20. Una barra de hierro de 100 mm2 de l


sección y 50 cm de longitud gira alrededor de uno F = ∫ rω 2 dm
0
de sus extremos con una velocidad angular uniforme
de ω radianes por segundo. Se pide cuál debe ser Donde l es la longitud de]a barra, ω es la
esta velocidad para que la barra se rompa por la velocidad angular de la rotación; r, la distancia que
tracción que origina la fuerza centrífuga, sabiendo hay desde el elemento de masa dm hasta el eje de
que el material de que está hecha se rompe por rotación. Para una barra homogénea dm = ρAdr ,
tracción cuando se le carga con 30 kg por mm2. siendo ρ la densidad de la sustancia que forma la
Solución.
barra y A, su sección. Integrando, obtenemos
Se romperá cuando
Fc = (30x9,8) x100 = 29400 N. ρAω 2 l 2
Llamando dm a un elemento de masa situado a la F=
distancia x del eje de giro, será: 2
dFc = dmω x = ρdVω x = ρω Axdx
2 2 2 De donde el número límite de revoluciones por
segundo será
Integrando:
F ρω 2 l 2 2S r
0,5 1
Fc = ∫ ρω 2 Axdx = ρω 2 Ax 2 Sr = = ⇒ ω= ,
0 2 A 2 ρl 2

(
= (7800)ω 2 100 × 10− 6 0,52
1
)( ) reemplazando valores;
2
ω=
(
2 2,45.10 8 ) = 239 rad
Luego: (8600)(1) 2
s
1
2
( )( )
(7800)ω 2 100 × 10− 6 0,52 = 29400 o
239
= 38 rev/s

Por tanto:
Deformaciones no uniformes por área variable.
2 × 29400
ω =
2
= 301538 , o sea
1950 × 10− 4 Ejemplo 23. Calcular cuánto se comprime el bloque
mostrado en la figura, cuando se le aplica una fuerza
ω = 301538 = 549 rad/s . P. Módulo de elasticidad Y.

Ejemplo 21. Determinar el máximo valor admisible


de la velocidad lineal de rotación de un anillo fino
de plomo, si la resistencia del plomo tiene el límite
de rotura P =2000 N/cm2 y la densidad ρ = 11,3
g/cm3.
Solución.
Durante la rotación del anillo, en éste surge una
tensión T = mv2/2 π r .Para el anillo fino m =2πrSρ,
donde S es la sección transversal del anillo. Por lo Solución.
tanto, T/S = ρv2. Tomemos un elemento diferencial dy tal como se
De allí el valor de la velocidad máxima es muestra en la figura.

P
v= ≈ 41 m/s.
ρ

Ejemplo 22. Una barra homogénea de cobre de 1 m


de longitud gira uniformemente alrededor de un eje
vertical que pasa por uno de sus extremos.
¿A qué velocidad de rotación se romperá la barra? Según muestra el diagrama del cuerpo libre del
kg elemento diferencial, es comprimido por la fuerza P.
Densidad del cobre ρ = 8600 3 , Esfuerzo de Este elemento disminuye su longitud d(Δh), siendo
m
Δh la disminución de longitud de h debido a la
8 kg
rotura del cobre S r = 2,45 × 10 fuerza P.
m2 Pdy
Solución. d (Δh) =
La fuerza centrífuga que actúa sobre la barra en este YA
caso es

12
Elasticidad Hugo Medina Guzmán

Pdy
d (Δh) =
YA

Usando las figuras anteriores


a
A = a(a + 2 x) y x = y reemplazando
2h Usando las figuras anteriores
obtenemos; a
Pdy Phdy A = (a + 2 x) 2 y x = y reemplazando
d (Δh) = o d ( Δh) = 2h
a Ya (h + y )
2
obtenemos;
Ya(a + y )
h Ph 2 dy
Luego, como d (Δh) =
h h
Ya 2 (h + y ) 2
Phdy
Δh = ∫ d (Δh) = ∫ Luego, como
0 Ya ( h + y )
2 h h
Ph 2 dy
Δh = ∫ d (Δh) = ∫
0

0 Ya ( h + y )
Integrando 2 2

Ph Ph 0
Δh = 2 ln(h + y ) 0h = 2 ln 2 Integrando
Ya Ya Ph
Ph Δh =
El bloque se comprime Δh = 0,692 2Ya 2
Ya 2 1 Ph
El bloque se comprime Δh =
Ejemplo 24. Una pirámide truncada de bases 2 Ya 2
cuadradas de lados ”a” y “2a” respectivamente de
altura h y modulo elástico Y se somete en la Ejemplo 25. Determine la deformación debido a la
dirección axial a una fuerza de compresión P, fuerza F, sin considerar el peso. El sólido mostrado
Determine la deformación que sufre la altura por de modulo elástico Y tiene altura H y bases
acción de la fuerza P. circulares de radios R y 2R
Solución.

Solución.
Tomemos un elemento diferencial dy tal como se Fdy
muestra en la figura. d (ΔH ) = , r = R+x
Yπrr 2
En los triángulos ABC y ADE:

Según muestra el diagrama del cuerpo libre del


elemento diferencial, es comprimido por la fuerza P.
Este elemento disminuye su longitud d(Δh), siendo
y x R
Δh la disminución de longitud de h debido a la = ⇒ x= x
fuerza P. R H H

13
Elasticidad Hugo Medina Guzmán

Fdy F dy Este elemento sufre una acortamiento d(Δh), debido


d (ΔH ) = =
Yπ (R + x )
al peso de la porción de pirámide que soporta (de
2
πY ⎛ R ⎞
2
altura y, radio base de lado 2x).
⎜ R + x⎟
⎝ H ⎠
FH 2
= (H + x )−2 dy
πR Y
2

H
FH 2
ΔH = ∫ ΔH = 2 ∫ (H + x ) dy
−2

πR Y 0
FH 2 ⎡ (H + x ) ⎤
−1 H

= ⎢ ⎥
πR 2Y ⎣ − 1 ⎦ 0 El peso que soporta es: Peso = ρg ( 4 x y ) el
1 2

3
FH 2 ⎡ 1 ⎤ FH
ΔH = ⎢ ⎥ = área de su base es: Ax = 4 x
2
πR Y ⎣ 2 H ⎦ 2πR 2Y
2

ρg 4 x 2 ydy ρg
d (Δh) = = ydy
Deformaciones no uniformes por peso propio y 3Y 4 x 2
3Y
área variable. Integrando desde y = 0 hasta y = h
h
h
ρg ρg y 2 1 ρgh 2
Δh = ∫
Ejemplo 26. Determine la deformación que sufre la
altura de la Gran pirámide de Keops en Egipto ydy = =
0
3Y 3Y 2 0
2 3Y
debido a su propio peso, sabiendo que posee una
altura de 147 m, su base es cuadrada de lado 230 m ρgAh
y que fue construida con bloques de piedra caliza y Como el Peso total es , obtenemos:
3
granito con módulo de Young = 35 x 109 N/m2 y
densidad = 2400 kg / m3. 1 (Peso total)h
Δh =
Solución. 2 Y (Area base)

Ejemplo 27. Encontrar cuanto se comprime el cono


de altura h y base de área A debido a su propio peso.
El cono esta hecho de un material de densidad ρ y
módulo de elasticidad Y.

Tomemos un elemento diferencial dy, tal como de


indica en la figura

Solución.
Tomemos un elemento diferencial dy, tal como de
indica en la figura

Este elemento sufre una acortamiento d(Δh), debido


al peso de la porción de cono que soporta (de altura
y, radio de la base r).

14
Elasticidad Hugo Medina Guzmán

Cálculo del peso de la de la parte tronco de pirámide


que está sobre el elemento diferencial.
Para esto tomamos un elemento diferencial de altura
dy’ y lo integramos desde x = 0 hasta x = x’.

1
El peso que soporta es: peso = ρg ( πr 2 y ) el
3
área de su base es: A = πr
2
El peso del elemento diferencial es:
ρgπr 2 ydy ρg dP = ρgdV = ρg 4(a + x') dy '
2
d (Δh) = = ydy
3Yπr 2 3Y Del dibujo siguiente:
Integrando desde y = 0 hasta y = h
h
h
ρg ρg y 2 1 ρgh 2
Δh = ∫ ydy = =
0
3Y 3Y 2 0
2 3Y
Como el Peso total es ρgAh/3, obtenemos:
1 (Peso total)h Obtenemos:
Δh =
2 Y (Area base) y y
y' = x' y dy ' = dx' :
x x
Ejemplo 28. En la figura se muestra un tronco recto y
dP = 4 ρg (a + x') dx'
2
de pirámide regular de base cuadrada. Determinar
cuánto se comprime el sólido homogéneo debido a x
su peso propio. Integrando desde x = 0 hasta x = x’:
Datos: Densidad = ρ, gravedad = g, módulo de y x'
Young = Y
P = ∫ dP = 4 ρg ∫ (a + x') dx'
2

x 0
Lado de la base menor = 2a; lado de la base mayor =
y (a + x')
3 x
4a
Altura del tronco de pirámide regular = H = 4 ρg
x 3 0

=
4 ρgy
3x
[
(a + x )3 − a 3 ]
El elemento diferencial se comprime:
Pdy
d (ΔH ) = , A = (2a + 2 x ) = 4(a + x )
2 2

YA

Solución. Reemplazando:
Para determinar cuánto se comprime el sólido
tomamos un elemento diferencial dy y vemos cuanto
d (ΔH ) =
[
4 ρgy (a + x ) − a 3
3
dy
]
se comprime por efecto del peso de la parte tronco 3Yx 4(a + x )
2
de pirámide que está sobre él (la parte de altura y en
el dibujo). Del dibujo siguiente:

Obtenemos:

15
Elasticidad Hugo Medina Guzmán

H H
y= x , dy = dx :
a a

d (ΔH ) =
2
[
ρg H (a + x )3 − a 3
dx
]
3Y a 2 (a + x )2
=
ρg H 2
3Y a 2
[a + x − a (a + x ) ]dx
3 −2

El peso del elemento diferencial es:


Integrando desde x = 0 hasta x = a:
dP = ρgdV = ρgπ (R + x') dy '
2
ΔH = ∫ d (ΔH )
Del dibujo siguiente:

=
ρg H 2
3Y a 2 ∫
0
a
[a + x − a (a + x) ]dx
3 −2

a
ρg H 2 ⎡ x2 a3 ⎤
= ⎢ ax + + ⎥
3Y a 2 ⎣ 2 (a + x ) ⎦ 0
ρg H 2 ⎛ a2 a2 ⎞
= ⎜
2 ⎜
a 2
+ + − a 2 ⎟⎟ Obtenemos:
3Y a ⎝ 2 2 ⎠ y y
1 ρgH 2 y' = x' y dy ' = dx' :
= x x
3 Y y
dP = ρgπ (R + x') dx'
2

Ejemplo 29. Determine la deformación que sufre la x


altura debido al peso propio Integrando desde x = 0 hasta x = x’:
y x'
El sólido mostrado tiene peso F, modulo elástico Y,
altura H y bases circulares de radios R y 2R
P = ∫ dP = ρgπ ∫ (R + x')2 dx'
x 0

y ( R + x ')
3 x
= ρgπ
x 3 0

=
ρgπy
3x
[(R + x) 3
− R3 ]
Solución. El elemento diferencial se comprime:
Pdy
d (ΔH ) = , A = π (R + x )
2
Para determinar cuánto se comprime el sólido
tomamos un elemento diferencial dy y vemos cuanto
YA
se comprime por efecto del peso de la parte tronco
de cono que está sobre él (la parte de altura y en el
dibujo).

Reemplazando:

d (ΔH ) =
[
ρgπy (R + x )3 − R 3
dy
]
3Yx π (R + x )2
Del dibujo siguiente:

Cálculo del peso P de la de la parte tronco de cono


que está sobre el elemento diferencial.
Para esto tomamos un elemento diferencial de altura
dy’ y lo integramos desde x = 0 hasta x = x’.

Obtenemos:

16
Elasticidad Hugo Medina Guzmán

H H El elemento diferencial soporta el peso P de la parte


y= x , dy = dx :
R R de hemisferio que está sobre él.

d (ΔH ) =
ρg H (R + x )3 − R 3
2
dx
[ ] De tal manera que se deforma:
3Y R 2 ( R + x )2 P( y ) dy
d (ΔR ) =
ρg H
[R + x − R (R + x) ]dx
2
3 −2 YA
=
3Y R 2

Integrando desde x = 0 hasta x = R:


ΔH = ∫ d (ΔH )

=
ρg H 2
3Y R 2 ∫
0
R
[R + x − R (R + x) ]dx
3 −2

R
ρg H 2 ⎡ x2 R3 ⎤ Cálculo de P( y )
= ⎢ Rx + + ⎥
3Y R 2 ⎣ 2 (a + x ) ⎦ 0
ρg H 2 ⎛ R2 R2 ⎞
= ⎜
2 ⎜
R 2
+ + − R 2 ⎟⎟
3Y R ⎝ 2 2 ⎠
1 ρgH 2
=
3 Y
El peso del tronco de cono es: Peso del elemento diferencial

F = π (2 R ) (2 H )ρg − π (R ) (H )ρg
1
3
2 1
3
2
(
dP( y ) = ρπg R 2 − y ' 2 dy ' )
El peso P( y ) de la porción de hemisferio es:
= πR Hρg (8 − 1) = πR Hρg
1 2 7 2
3 3 R
Luego P( y ) = ρπg ∫ ( R 2 − y ' 2 )dy ' =
1 ρgH 2
y
F
ΔH =
7 2 3 Y ⎛ 2R 3 y
3

πR Hρg ρgπ ⎜⎜ − R2 y + ⎟
3 ⎟
⎝ 3 3 ⎠
FH
= Ahora la deformación total Integrando
7πR 2Y
P( y ) dy
Ejemplo 30. Un hemisferio (mitad de una esfera d (ΔR ) = :
sólida) de densidad ρ , radio R y modulo de Young YA
Y esta sobre el piso descansando sobre su base ⎛ 2R 3 y ⎞
3
circular determine cuanto se deforma por acción de ⎜
gπ ⎜ −R y+
2
⎟dy

su propio peso.
d (ΔR ) = ⎝ 3 3 ⎠
Sugerencia: Calcule la deformación de una porción
diferencial del hemisferio formada por un disco Yπ R − y
2 2
( )
delgado paralelo al piso.
1
R
⎛ 2R 3 y ⎞
3
dy
Δ R = ρg π ∫0 ⎜⎝ 3
⎜ − R y + ⎟ 2
2

Y ⎟
3 ⎠ (R − y 2 )

⎛2 3 2 2 ⎞ ⎛ 1 2 1 3⎞
⎜ R − R y⎟ + ⎜− R y + y ⎟
= ρg ⎝ 3
R
3 ⎠ ⎝ 3 3 ⎠

Y 0 R −y
2 2
dy
( )
Solución.
Vamos a considerar un elemento diferencial de
ρg R
2R 2
(R − y ) − y R 2 − y 2( )

= 3 3 dy
área A = π r , altura dy (R − y )(R + y )
2
Y 0

Donde r = ( R − y )
2 2 2

17
Elasticidad Hugo Medina Guzmán

ρg R ⎡ 2 R 2 ⎤ Cobre 0,35
= ∫ ⎢
3Y 0 ⎣ (R + y )
− y ⎥dy

Oro
Hierro, fundido
0,41
0,28
Plomo 0,33
R
ρg ⎡ y2 ⎤ Nickel 0,30
= ⎢ 2 R ln (2
R + y ) − ⎥ Platino 0,38
3Y ⎣ 2 ⎦0 Plata 0,37
Latón 0,33
ρgR 2 ⎛ 1 ⎞ 0,30 ρgR 2
= ⎜ 2 ln 2 − ⎟ =
3Y ⎝ 2⎠ Y Ejemplo 31. El paralelepípedo de la figura está
hecho de un material con módulo de Young Y, y
La altura del hemisferio disminuye constante poisson σ. ¿Cuál es el valor de ΔV/V?
0,30 ρgR 2
ΔR = Debido al peso propio
Y

DEFORMACION LATERAL MODULO DE


POISSON
Adicionalmente, cuando estiramos un bloque en una Solución.
dirección éste se contrae en las dimensiones Debido a la compresión ocasionada por la fuerza F:
perpendiculares al estiramiento, la contracción de las ΔL F Δa Δb ΔL
caras laterales es en la misma proporción para el =− y como = = −σ
ancho (a) y el alto (h). Por ejemplo, la contracción L YA a b L
Δa en el ancho es proporcional al ancho a y también Δa Δb F
Obtenemos: = =σ
Δl a b YA
a , lo que resumimos en la siguiente expresión:
l ΔV ΔL Δa Δb
Como = + +
Δa Δh Δl V L a b
= = -σ
a h l Reemplazando
Donde σ es otra constante del material conocida ΔV F F F
=− +σ +σ
como el módulo de Poisson. V YA YA YA
Finalmente:
ΔV F
= − (1 − 2σ )
V YA
Ejemplo 32. Al cubo de la figura de lado 50cm se
le aplica dos pares de fuerzas Fx=100 N y Fy=50 N
obteniendo como resultado que la longitud en el eje
x aumenta en 0,01% y la longitud en el eje y
disminuye en 0,006%.

a) Determine si el esfuerzo en x,y es de tracción o


compresión.
b) Determine el módulo de Young y la constante
de Poisson.
Como valores aproximados para algunos materiales
se puede tomar:
0,28 para hierro y acero, 0,5 para caucho y 0,25
para vidrio.
Las dos constantes Y y σ especifican
completamente las propiedades de un material
homogéneo isotrópico.

Módulo de Solución.
Nombre Poisson σ 100 50
a) S x = = 400 N/m2, S y = = 200
Aluminio
Sin dimensiones
0,34
(0,5)2
(0,5)2
Acero 0,28 N/m2

18
Elasticidad Hugo Medina Guzmán

Δax 0,01 Δh S
= = 1 × 10− 4 , a) Para la altura = , para el diámetro
a 100 h Y
Δa y 0,006 ΔD Δh S
=− = −6 × 10− 5 = −σ = −σ
a 100 D h Y
Haciendo un análisis de los cambios de longitudes: ΔV Δh ΔD
El esfuerzo en x es mayor y la longitud en x El cambio de volumen es = +2 =
aumenta mientras que en y disminuye, siendo el V h D
S S S
esfuerzo en y menor, se puede concluir que el
− 2σ = (1 − 2σ ) , por lo tanto
esfuerzo en x es de tracción y el esfuerzo en y es de Y Y Y
compresión.
S S πD 2 h
ΔV = (1 − 2σ )V = (1 − 2σ )
b) El paralelepípedo esta sujeto a esfuerzo por cuatro Y Y 4
caras, como se muestra en la figura siguiente: b) ΔV es igual a cero cuando (1 − 2σ ) = 0 ⇒
σ = 0,5
c) Para la mayoría de metales con un valor de σ
aproximado a 0,3:
ΔV S S
= [1 − 2(0,3)] = 0,4
V Y Y
Para el corcho, con un valor de σ aproximado a
0,0:
Sea S el esfuerzo sobre cada una de las caras
ΔV S S
laterales. = [1 − 2(0,0)] =
La deformación del lado horizontal ax es: V Y Y
Para el caucho, con un valor de σ aproximado a
Δax 400 200
= +σ = 1 × 10− 4 (1) 0,5:
a Y Y ΔV S
La deformación del lado horizontal a y es: = [1 − 2(0,5)] = 0,0
V Y
Δa y 200 400
=− −σ = −0,6 × 10− 4 (2) Ejemplo 34. El sólido de la figura está sometido a
a Y Y los esfuerzos de compresión y tracción mostrados en
Restando (1) + (2)/2, obtenemos: las direcciones x y z, respectivamente. Determine
400 100 300 cual será el esfuerzo (S’) en la dirección y, tal que la
− = 0,7 × 10− 4 ⇒ = 0,7 × 10− 4 deformación unitaria en esa dirección sea nula.
Y Y Y
Datos: S = esfuerzo, Y = módulo de Young, σ =
300
⇒ Y= = 4,28 x 106 N/m2 módulo de Poisson.
0,7 × 10− 4
Reemplazando el valor de Y en (1):
400 200
+σ = 1 × 10− 4 ⇒
4,28 × 10 6
4,28 × 10 6

4 + 2σ = 4,28
⇒ σ = 0,14

Ejemplo 33. a) Calcule la deformación volumétrica


durante la extensión elástica de una barra cilíndrica
Solución.
sometida a tracción axial. El material es isótropo y la
deformación se supone pequeña.
b) ¿Para qué valor del módulo de Poisson, el
alargamiento ocurre sin cambio de volumen?
c) El módulo de Poisson de la mayoría de metales es
aprox. 0,3. El del corcho, aprox. 0,0 y el del caucho
cercano a 0,5. ¿Cuáles son las deformaciones
volumétricas de esos materiales al someterlos a una
compresión elástica ε < 0 ?
Solución.
Para que la deformación unitaria en la dirección y
sea nula, se debe cumplir:

19
Elasticidad Hugo Medina Guzmán

ΔH S S'
1
(3σS − S ') = 0 ⇒ 3σS − S ' = 0 ⇒ = − + 2σ ⇒
Y H Y Y
S ' = 3σS ΔH S 2σ 2 S
=− + ⇒
Ejemplo 35. Se tiene el paralelepípedo mostrado en H Y (1 − σ ) Y
la figura que encaja perfectamente en una caja ΔH S ⎡ 2σ 2 ⎤
rígida. Luego de encajo el paralelepípedo se coloca
H
=−
Y ⎢ (1 − σ ) ⎥ ⇒
1 −
un peso P sobre éste, tal que lo aplasta ⎣ ⎦
uniformemente, la caja impide las expansiones
P ⎡ 2σ 2 ⎤
laterales. ΔH = − 2 ⎢1 − H
a) ¿Cuál es el esfuerzo sobre las paredes laterales? Ya ⎣ (1 − σ ) ⎥⎦
b) ¿Cuál es el cambio en la altura ΔH = H − H '
del paralelepípedo? Ejemplo 36. Hallar el valor del módulo de Poisson
para el cual el volumen de un alambre no varía al
alargarse.
Solución.
Δr Δl
=σ , de aquí el módulo de Poisson
r l
Δr
σ = r , siendo r el radio del alambre y l su
Solución. Δl
El paralelepípedo esta sujeto a esfuerzo por sus seis
caras, como se muestra en la figura siguiente: l
longitud. El volumen de dicho alambre antes de
estirarlo es V1 = πr
2
l y su volumen después de
estirado es V2 = π (r − Δr ) (l + Δl )
2

Si el volumen no varió con el alargamiento,


tendremos que πr l = π (r − Δr ) (l + Δl ) . Y
2 2

abriendo los paréntesis y despreciando las


magnitudes Δr y Δl al cuadrado, hallamos que
Δr
1
Sea S el esfuerzo sobre la cara superior e inferior y πr 2 l = 2πrΔrl , de donde r = = 0,5 , luego
S’ el esfuerzo sobre cada una de las caras laterales. Δl 2
La deformación del lado a es: l
Δa S' S' S σ = 0,5.
= − +σ +σ (1)
a Y Y Y
La deformación del lado H es: Ejemplo 37. Hallar la variación relativa de la
ΔH S S' densidad de una barra de cobre cilíndrica al ser
= − + 2σ (2) comprimida por una presión p = 9810 Pa. Para el
H Y Y cobre tómese un módulo de Poisson σ = 0,34.
Solución.
a) Como la longitud a no cambia, Δa = 0 .
De la ecuación (1):
La densidad de la barra antes de ser comprimida es
S' S' S σ
− + σ + σ = 0 ⇒ S'= S
Y Y Y (1 − σ ) ρ1 =
m
donde V1 = πr l . La densidad de la
2

P V1
Siendo S = 2
a
σP ρ2 =
m
⇒ S'= barra después de comprimida será ,
(1 − σ )a 2 V2

siendo V2 = π (r + Δr ) (l − Δl ) . Por
b) De la ecuación (2): 2

consiguiente la variación de la densidad será

20
Elasticidad Hugo Medina Guzmán

⎛ 1 1 ⎞ mΔV
Δρ = ρ 2 − ρ1 = m⎜⎜ − ⎟⎟ =
⎝ V2 V1 ⎠ V2V1
Como .la compresión no es muy grande,
aproximadamente se puede tomar V2V1 = V1
2

mΔV
Se puede considerar que Δρ = .
V12
Entonces la variación elativa de la densidad
Δρ ΔV
= . Hallemos pues la variación de
ρ1 V1
volumen ΔV = πr l − π (r + Δr ) (l − Δl ) .
2 2

Abriendo los paréntesis y despreciando los


cuadrados de las magnitudes Δr y Δl , obtenemos
⎛ Δl ⎞
que ΔV = V1 ⎜ ⎟(1 − 2σ ) , .donde σ es el
⎝ l ⎠
módulo de Poisson. Por lo tanto
Δρ ΔV Δl
= = (1 − 2σ ) . Pero como por la ley
ρ1 V1 l
Δl p n
de Hooke = , tendremos que en definitiva
l Y
Δρ p n
= (1 − 2σ ) .
ρ1 Y
3 N
En nuestro caso pn = 9,81 × 10 ,
m2
N
Y = 1,18 × 1011 2 y σ = 0,34. Poniendo estos
m
Δρ ΔV
datos obtenemos que = = 0,027 %. .
ρ1 V1

Ejemplo 38. El sólido de la figura (lados a, b y c) Consolidado


está sometido a los esfuerzos de compresión y
tensión mostrados.
Determine la deformación volumétrica unitaria,
ΔV / V .
Datos:
S = esfuerzo, Y = módulo de Young, σ = módulo
de Poisson.

ΔV ⎛ Δ a ⎞ ⎛ Δb ⎞ ⎛ Δc ⎞
=⎜ ⎟total + ⎜ ⎟total + ⎜ ⎟total
V ⎝ a ⎠ ⎝ b ⎠ ⎝ c ⎠
6S
= 3S (4σ ) − 6 S = (2σ − 1)
Y Y Y

DEFORMACIÓN POR CIZALLADURA O


CORTE. MODULO DE CIZALLADURA O
RIGIDEZ.

Deformación por cizalladura


Solución. Ya hemos estudiado el módulo de elasticidad Y de
Deformación de cada uno de los lados: un material, es decir, la respuesta del material

21
Elasticidad Hugo Medina Guzmán

cuando sobre él actúa una fuerza que cambia su b) ¿Cuál es la deformación de corte?
volumen (aumentando su longitud). Ahora, c) ¿Cuál es el módulo de corte?
examinaremos la deformación por cizalladura en el Solución.
que no hay cambio de volumen pero si de forma. F 1 N
Definimos el esfuerzo como F/A la razón entre la a) St == = 11,11 2
fuerza tangencial al área A de la cara sobre la que se A (0,30) 2
m
aplica. La deformación por cizalla, se define como la Δx 1
razón Δx/h, donde Δx es la distancia horizontal que b) δ = = = 0,033
se desplaza la cara sobre la que se aplica la fuerza y
h 30
h la altura del cuerpo, tal como vemos en la figura. S 11,11
c) G = t = = 333,33
δ 0,033
Ejemplo 40. Un cubo de acero de 5 cm de
arista se halla sometido a 4 fuerzas cortantes,
de 1200 kg, cada una, aplicadas en sentidos
opuestos sobre caras opuestas. Calcule la
deformación por cizalladura.
Cuando la fuerza F que actúa sobre el cuerpo es
paralela a una de las caras mientras que la otra cara
permanece fija, se presenta otro tipo de deformación
denominada de cizalladura en el que no hay cambio de
volumen pero si de forma. Si originalmente el cuerpo
tiene forma rectangular, bajo un esfuerzo cortante la
sección transversal se convierte en un paralelogramo. Solución.
G Acero al carbono = 8 x109 N/m2
F
El módulo de cizalladura o de rigidez G es una esfuerzo S
propiedad mecánica de cada material G= = A= t
deformación δ φ
h
F (1200(9,8))
Siendo pequeños los ángulos de desplazamiento
podemos escribir St = = = 4,704 x106 N/m2
Deformación =
δ
= tan φ ≈ φ
A (0,05) 2

h Consideremos solamente las fuerzas


F horizontales, estas producen una deformación
esfuerzo S φ , como se muestra en la figura
G= = A= t
deformación δ φ St 4,704 × 106
h φ= = = 0,588 x10-3
La ley de Hooke para la deformación por cizalladura G 8 × 109
se puede escribirla de modo siguiente: radianes
S t = Gφ
El módulo de cizalladura G es característico de cada
material
Módulo de
Nombre rigidez G
1010 N/m2
Aluminio 2,5
Cobre 4,3 La cara que se muestra queda como un rombo
Oro 3,5
⎛π ⎞ ⎛π ⎞
Hierro, fundido 3,2 con ángulos ⎜ −φ ⎟ y ⎜ +φ ⎟
Plomo 0,6 ⎝2 ⎠ ⎝2 ⎠
Nickel 7,4 Consideremos ahora solamente las fuerzas
Acero 7,5 verticales, estas producen una deformación
Latón 1,7 también φ , como se muestra en la figura
St 4,704 × 106
Ejemplo 39. Un cubo de gelatina de 30 cm φ= = = 0,588 x10-3
de arista tiene una cara sujeta mientras que a G 8 × 109
la cara opuesta se le aplica una fuerza radianes
tangencial de 1 N. La superficie a la que se
aplica la fuerza se desplaza 1 cm.
a) ¿Cuál es el esfuerzo de corte?

22
Elasticidad Hugo Medina Guzmán

= 2,65 x 105 N

Ejemplo 42. El acero promedio requiere,


típicamente, un esfuerzo de 3,45 x 108 N/m2 para la
ruptura por cizalladura. Determine la fuerza
requerida para perforar un agujero del diámetro 2,5
cm en una placa de acero de ¼ de pulgada (6,25
El cubo se deforma en el plano del papel y mm) de espesor.
toma la forma de un rombo con ángulos
⎛π ⎞ ⎛π ⎞
⎜ − 2φ ⎟ y ⎜ + 2φ ⎟
⎝2 ⎠ ⎝2 ⎠
Ejemplo 41. Una estatua se encuentra soldada a un
pedestal de latón, que se muestra en la figura. Al Solución.
producirse un movimiento sísmico se observa un La circunferencia de un círculo del diámetro D = 2,5
desplazamiento lateral de la cara superior del
cm es C = πD = 7,85 x10 m , El área del borde
−2
pedestal de 0,25mm.
del disco cortado AAAA es el producto de la
Calcular: circunferencia C por el espesor del material, esto es
a) El esfuerzo de corte. (6,25 × 10 )(7,85 × 10 ) = 49,06 × 10
−3 −2 −5
m2 .
b) La magnitud de la fuerza producida por el Una fuerza de la magnitud F se ejerce en el sacador,
movimiento sísmico. el esfuerzo de corte (fuerza por unidad de área) a
El pedestal de latón tiene una altura de 1m y una
F
sección cuadrada de 0,5m de lado. través del borde es S = ⇒
El módulo de Young del latón es 3,5x1010 Pa A
Módulo de rigidez G del latón es 1,7 x1010 N/m2 F = S . A = (3,45 × 10 8 )(49,06 × 10 −5 )
= 1,69 x 105 N. La hoja de acero se corta por
cizalladura cuando el esfuerzo llega a ser igual 3,45
x 108 N/m2, es decir, cuando F = 1,69 x 105 N.
Esta es la fuerza de 1,69 x 105 N, equivalente a 17,3
toneladas es requerida para perforar el agujero de
2,5 cm de diámetro El sacador y los dados son
operados por una máquina conocida como prensa;
Solución. en este caso uno tendría que utilizar una prensa con
Desplazamiento lateral de la cara superior del una capacidad de 20 toneladas o más.
pedestal de 0,25mm.
Ejemplo 43. Calcular el módulo de rigidez del
material en función a las características geométricas
de un alambre (longitud l y radio R) y del torque
aplicado.
Manteniendo el extremo superior fijo aplicamos un
torque τ que gira al extremo inferior un ánguloθ.
Consideremos una capa diferencial cilíndrica de
material concéntrica con el eje, de radio interior r y
a) El esfuerzo de corte. de espesor dr, como se muestra en la figura.
Δx 0,25 × 10 −3
δ= = = 0,25 × 10 − 3
h 1,00
S
G= t ⇒
δ
St = Gδ = (1,7 x 1010)(0,25 x10-3)
= 0,425 x 107 N/m2
b) La magnitud de la fuerza producida por el
movimiento sísmico.
F
St = ⇒
A La deformación es
F = St A = (0,425 x 107)(0,52)

23
Elasticidad Hugo Medina Guzmán

δ rθ
φ= =
l l
El esfuerzo cortante es
Grθ
S t = Gφ =
l
Como el esfuerzo cortante es la fuerza tangencial
por unidad de área, multiplicándolo por el área de la
sección transversal de la Capa, 2 π rdr, nos dará la
π R4 π D4
fuerza tangencial dF sobre la base de la Capa τ= G θ ⇒τ= G θ,
⎛ Grθ ⎞ θ 2 2 l 32 l
dF = S t dA = ⎜ ⎟(2πrdr ) = 2πG r dr π D4
⎝ l ⎠ l Como τ = FD ⇒ FD = G θ , de aquí
El torque sobre la base de la Capa cilíndrica es 32 l
⎛ θ ⎞ θ ⎛ 32 F ⎞⎛ l ⎞
dτ = rdF = r ⎜ 2πG r 2 dr ⎟ = 2πG r 3 dr θ =⎜ ⎟⎜ 3 ⎟
⎝ l ⎠ l ⎝ πG ⎠⎝ D ⎠
Integrando de 0 a R, el torque total sobre la base del Para la varilla de 100 cm y de 80 cm
cilindro es respectivamente son:
π R4 ⎛ 32 F ⎞⎛⎜ l 1 ⎞⎟ ⎛ 32 F ⎞⎛ l 2 ⎞
τ= G θ θ1 = ⎜ ⎟⎜ 3 ⎟ Y θ 2 = ⎜ ⎟⎜⎜ 3 ⎟⎟
2 l ⎝ πG ⎠⎝ D1 ⎠ ⎝ πG ⎠⎝ D2 ⎠
De aquí De estas últimas obtenemos:
2τl 3
G= ⎛l ⎞⎛ D1 ⎞
3
⎛ 80 ⎞⎛ 1 ⎞
πR 4θ θ 2 = ⎜⎜ 2 ⎟⎟⎜⎜ ⎟⎟ θ1 = ⎜ ⎟⎜ ⎟ 1º
⎝ l1 ⎠⎝ D2 ⎠ ⎝ 100 ⎠⎝ 2 ⎠
O sea, para determinar C bastará con medir el = 0,1º
ángulo θ que se produce al aplicar el torque M.
DEFORMACION VOLUMETRICA. MODULO
Ejemplo 44. Una varilla de cobre de 40 cm de DE ELASTICIDAD VOLUMETRICO.
longitud y de 1 cm de diámetro está fija en su base y Módulo de elasticidad volumétrico.
sometida a un par de 0,049 Nm en torno a su eje Consideramos ahora un volumen de material V
longitudinal. ¿Cuántos grados gira la cara superior sujeto a un esfuerzo unitario p 0 (por ejemplo la
respecto de la inferior? presión atmosférica) sobre toda la superficie.
Cuando el esfuerzo a presión se incrementa a
p = p 0 + Δp y el volumen sufre una disminución
ΔV , la deformación unitaria es δ = − ΔV V
F
Solución.
El esfuerzo es = Δp .
A
Cobre estirado en frío G = 48,0x109 N/m2 La razón del esfuerzo de compresión uniforme a la
π R4 2lτ deformación por compresión uniforme recibe es el
τ= G θ θ=
2 l πGR 4 módulo de elástico que en este caso se conoce como
2(0,4 )(0,049)
módulo de compresibilidad volumétrica o
θ= volumétrico (B).
π (48,0 × 10 )(0,5 × 10 )
−2
= 2,08 x10-4
Δp
9
B=−
radianes ΔV
V
Ejemplo 45. Una varilla que tiene 100 cm de Donde la constante de proporcionalidad B, depende
longitud y 1 cm de diámetro está sujeta solamente del material. El módulo volumétrico
rígidamente por un extremo y se le somete a tiene las dimensiones de la presión, esto es,
torsión por el otro hasta un ángulo de lº. Si se fuerza/área y es aplicable tanto para sólidos como
aplica la misma fuerza a la circunferencia de líquidos. Pero, los gases tienen un comportamiento
una varilla del mismo material pero que tiene diferente que será considerado posteriormente.
una longitud de 80 cm y un diámetro de 2 Módulo
cm, ¿cuál es el ángulo de torsión resultante? Nombre volumétrico
Solución. B 1010 N/m2
Aluminio 7,5
Cobre 14

24
Elasticidad Hugo Medina Guzmán

Hierro 16
Plomo 17 Ejemplo 48. Si con aluminio se fabrica un cubo de
Níckel 4,1 10 cm de lado, se quiere saber las deformaciones
Vidrio óptico 5,0 que experimentará en una compresión uniforme,
Latón 6,0 perpendicular a cada una de sus caras, de una
Acero 16 tonelada, y cuándo esta misma fuerza actúa
Agua 0,21 tangencialmente a la superficie de una de sus caras,
Mercurio 2,8 estando el cubo só1idamente sujeto por la cara
opuesta.
Solución.
Ejemplo 46. ¿Qué incremento de presión se requiere La presión que soporta, cada cara, en el primer caso,
para disminuir el volumen de un metro cúbico de será:
agua en un 0,005 por ciento? F (100)(9,8)
p= = = 9,8 × 10 Pa
Solución. A 0,12
Por elasticidad volumétrica tenemos:
Como el módulo volumétrico del aluminio es
ΔV B = 3,5x 1010 N/m2:
Δp = − B
V ΔV p 9,8 × 105
El módulo de compresibilidad del agua es =− =− = −2,8 × 10 − 5
9 2 V B 3,5 × 1010
2,1 x 10 N/m
De donde:
⎛ − 0,00005V ⎞ ΔV = - 2,8x 10-5 V = - 2,8x 10-5x 10-3 = - 2,8x 10-8
Δp = −2,1 × 10 ⎜ 9

⎝ V ⎠ m3.
2 En cuanto a la deformación, se obtiene a partir de la
= 1,05 x105 N/m expresión de la deformación de cizalla, que es:
1 F 1 (103 )(9,8)
Ejemplo 47. Calcule densidad del agua del océano a tan ϕ ≈ ϕ = =
una profundidad en que la presión es de 3430 N/cm2. G A 3 × 1011 x10−1 10− 2
La densidad en la superficie es 1024 kg/m3. = 3,27x10-5 rad
El módulo de compresibilidad del agua es 2,1 x
9 2
10 N/m
Solución. RELACION ENTRE CONSTANTES
p = 3430 N/cm2 = 3,430 x107 N/m2, ELASTICAS.
Δp = 3,430 x107– 1,013 x105 ≈ 3,430 x107 N/m2 Relación entre B, Y y σ
m kg
En la superficie ρ = = 1024 3 Muestra sometida a una presión uniforme.
V m
Cuando cambia el volumen a V ' = (V + ΔV ) , La figura siguiente muestra un bloque bajo presión
tenemos: uniforme en toda su superficie exterior
m m m
ρ'= = =
V ' V + ΔV ⎛ ΔV ⎞
V ⎜1 + ⎟
⎝ V ⎠
ρ
=
⎛ ΔV ⎞
⎜1 + ⎟
⎝ V ⎠
Δp ΔV Δp
Como B = − ⇒ =− Como la presión es uniforme, el esfuerzo unitario en
ΔV V B cada cara es el mismo.
V Y las deformaciones de cada una de las dimensiones
De aquí: son:
Dimensión l:
ρ ρ
ρ'= =
⎛ ΔV ⎞ ⎛ Δ p ⎞
⎜1 + ⎟ ⎜1 − ⎟
⎝ V ⎠ ⎝ B ⎠
1024
= = 1041 kg/m3
⎛ 3,430 × 107 ⎞ Δl p
⎜⎜1 − ⎟ =−
⎝ 2,1 × 109 ⎟⎠ l Y

25
Elasticidad Hugo Medina Guzmán

Dimensión a: - Propia:
Δb1 p
=−
b Y
- Debido a la deformación de a:
Δb2 Δa ⎛ p⎞ p
= −σ = −σ ⎜ − ⎟ = σ
b a ⎝ Y⎠ Y
- Debido a la deformación de l:
Δa p
=− Δb3 Δl ⎛ p⎞ p
a Y = −σ = −σ ⎜ − ⎟ = σ
Dimensión b:
b l ⎝ Y⎠ Y
Deformación total
Δb Δb1 Δb2 Δb3
= + +
b b b b
p
= − (1 − 2σ )
Y
Δb p El cambio de volumen es:
=−
b Y ΔV Δl Δa Δb
= + +
Pero, como la deformación de una dimensión lleva a V l a b
la deformación de las otras dimensiones, tenemos.
3p
Deformación de l: = − (1 − 2σ )
- Propia: Y
Δl 1 p Sabemos nosotros que el módulo de compresibilidad
=− es
l Y p
- Debido a la deformación de a: B=−
Δl 2 Δa ⎛ p⎞ p ΔV V
= −σ = −σ ⎜ − ⎟ = σ Luego:
l a ⎝ Y⎠ Y
Y
- Debido a la deformación de b: B=
Δl 3 Δb ⎛ p⎞ p 3(1 − 2σ )
= −σ = −σ ⎜ − ⎟ = σ Expresión que nos relaciona el módulo de
l b ⎝ Y⎠ Y Compresibilidad, el módulo de Young y la relación
Deformación total de Poisson
Δl Δl 1 Δl 2 Δl 3
= + + Ejemplo 49. Se somete a una muestra de cobre de
l l l l forma cúbica con 10 cm de arista a una compresión
p
= − (1 − 2σ ) uniforme, aplicando Un esfuerzo de 106 N/m2
Y perpendicularmente a cada una de sus caras. La
Deformación de a: variación relativa de volumen que se observa es de
- Propia: 7,25×10-6 .
a) Determinar el módulo de compresibilidad (B) del
Δa1 p
=− Cu en el sistema internacional.
a Y b) Determinar el módulo de Poisson sabiendo que el
- Debido a la deformación de l: módulo de Young del cobre es 120×109 Pa.
Solución.
Δa2 Δl ⎛ p⎞ p
= −σ = −σ ⎜ − ⎟ = σ a) Como:
a l ⎝ Y⎠ Y ΔV
- Debido a la deformación de b: Δp = 104 N/m 2 , = −7,25 × 10−6 y
V
Δa3 Δb ⎛ p⎞ p Δp
= −σ = −σ ⎜ − ⎟ = σ B=− ⇒
a b ⎝ Y⎠ Y ΔV
Deformación total
V
Δa Δa1 Δa2 Δa3
= + + 106
a a a a B=− −6
= 137,7 x 109 N/m2
p − 7,25 × 10
= − (1 − 2σ ) b)
Y
Deformación de b:

26
Elasticidad Hugo Medina Guzmán

Y Y
B= ⇒ (1 − 2σ ) =
3(1 − 2σ ) 3B
Y
1−
⇒σ = 3B
2
120 × 109
1−
⇒σ =
(
3 137,7 × 109 )
= 0,35
El esfuerzo de compresión sobre el plano B resulta
ser
2 2G G
SB = =
2A A
Relación entre G, Y y σ A e igualmente el esfuerzo de tracción sobre C
2G G
Muestra sometida a esfuerzo cortante. SC = =
Determinación de la relación entre el módulo de 2A A
rigidez, el módulo de Young y el módulo de Las deformaciones de las diagonales B y C se
Poisson. escriben entonces
Pretendemos analizar la relación entre los esfuerzos
ΔD B H
cortantes y los esfuerzos de compresión y de = (1 + σ )
tracción. Para ello consideremos primero el caso del D YA
bloque de la Figura que está sometido, por una parte, ΔDC H
a un esfuerzo de compresión y en la otra dirección a y = (1 + σ ) (1)
un esfuerzo de tracción. Sea 1 su longitud en la D YA
dirección horizontal y h su altura. Si expresamos el esfuerzo tangencial en términos del
ángulo φ, ya que suponemos que la deformación es
pequeña resulta
δ 2 Δ DC ΔDC
tan φ ≈ φ ⇒ φ = ≈ =2
h h D

La deformación en la dirección horizontal tiene dos


términos: el primero corresponde a la deformación
producido por el esfuerzo de tracción, mientras que
el segundo corresponde a la dilatación producida por
la compresión en la dirección vertical. Por tanto, nos
queda, Donde las dos últimas igualdades surgen a partir de
Δl F F F analizar la geometría esbozada en la Figura arriba.
= +σ = (1 + σ ) En efecto, si el ángulo entre δ y ΔD es de 45 grados
l YA YA YA
Por otra parte, la deformación en la dirección se cumple
vertical corresponde a las deformaciones causadas δ 1
= = 2
por un lado por la fuerza de compresión en la ΔDC sen 45o
dirección vertical y por otro por la tracción en la
dirección horizontal. Por tanto, Y por tanto
Δh F F F δ 2ΔDC 2ΔDC
=− −σ = −(1 + σ ) φ= = o
=
h YA YA YA h DC sen 45 DC
Ahora bien, en la Figura abajo representamos la En estas condiciones, sí sustituimos en (1) este
deformación de un bloque sometido a un esfuerzo último resultado nos queda
tangencial detallando lo que le ocurre a las H
diagonales de sus caras. Si observamos la figura, φ = 2(1 + σ )
vemos que los resultados de los esfuerzos YA
tangenciales equivalen a los producidos por las Esta ecuación, si tenemos en cuenta que φ es la
fuerzas H que producen, por una parte, un esfuerzo deformación tangencial y la comparamos con la
de tracción sobre el plano C y un esfuerzo de S H A
compresión sobre el plano B. ecuación G = = , nos permite obtener
φ φ

27
Elasticidad Hugo Medina Guzmán

Y Y = 2 x 1011 N/m2, A = área de la sección transversal


G= = 10 -6m2
2(1 + σ ) Solución.
Expresión que relaciona el módulo de rigidez con el l = 2 m , F1 = 5 × 9,8 N , F2 = 10 × 9,8 N
módulo de Young y con el módulo de Poisson
A = 10 -6 m 2 , Y = 2 × 10 2 N/m 2
FUERZA ELASTICA Y ENERGIA ELASTICA. W = trabajo realizado por la fuerza F = kx en alargar
Energía de deformación. el alambre una longitud x.
La energía necesaria para estirar una cantidad x una 1 2 F
muestra de material de constante de rigidez k es W= kx , con F = kx ⇒ x =
1 2 k
∫ fdx = ∫ kxdx = 2 kx
2
Energía = o en función 2
1 ⎛F⎞ 1 F2
W = k⎜ ⎟ =
de F 2 ⎝k⎠ 2 k
1 YA
Energía = Fx Para un alambre k =
2 l
Si la sección transversal de la muestra es A y su Reemplazando:
longitud l entonces podemos escribir la ecuación
1 F2
como W=
Energía 1 Fx Energía 1 ⎛ F ⎞⎛ x ⎞ 2 YA l
= o = ⎜ ⎟⎜ ⎟
Al 2 Al Al 2 ⎝ A ⎠⎝ l ⎠ F 2l
=
Energía por unidad de volumen = 2 AY
(5 × 9,8) (2)
1 2
(Esfuerzo)(Deformación unitaria) F12 l
a) W1 =
( )
2 = = 0,012 J
Esta es la energía necesaria para estirar o comprimir 2 AY 2 10 −6 2 × 1011
la muestra, teniendo en cuenta el módulo de Young
y la energía por unidad de volumen, puede
W2 =
F22 l (10 × 9,8)2 (2) = 0,048 J
2 AY 2(10 −6 )2 × 1011
b) =
expresarse como
Energía 1 (Esfuerzo) 2
= El incremento en energía almacenada es:
Volumen 2 Y ΔE = W2 − W1 = 0,048 – 0,012 = 0,036 J.
Ejemplo 50. Una carga de 100 kg está colgada de un Ejemplo 52. Demostrar que cuando se
alambre de acero de 1 m de longitud y 1 mm de somete un cuerpo elástico a una tensión de
radio. ¿A qué es igual el trabajo de tracción del corte pura que no supera el límite elástico de
alambre? corte para el material, la densidad de energía
Solución. elástica del cuerpo es igual a la mitad del
Por la ley de Hooke producto de la tensión de corte por la
Δl F YA deformación de corte.
= ⇒ F= Δl (1) Solución.
l YA l
Pero para las fuerzas elásticas F = kΔl (2)
Comparando (1) y (2) vemos que
AY
k= (3)
l
Entonces
AY (Δl )
2
W = k (Δl ) =
1 2
(4)
2 2l
Calculando la magnitud Δl por la fórmula (1) y
La fuerza que deforma por corte o cizalladura
poniendo todos los datos numéricos en la ecuación
(4) obtenemos definitivamente que W = 0,706 J. GA
es F= x
h
Ejemplo 51. Un alambre de acero de 2m de longitud El trabajo para deformar un dx es
cuelga de un soporte horizontal rígido. x = Δx GA
a) ¿Cuánta energía almacena cuando se suspende en W =∫ xdx
él una carga de 5 kg? x =0 h
b) ¿Si la carga se aumenta 10 kg, en cuanto aumenta
energía almacenada?

28
Elasticidad Hugo Medina Guzmán

1 GA Usando los diagramas del cuerpo libre mostrados en


W= (Δx )2 = 1 FΔx las figuras tenemos:
2 h 2 Para la parte de la liga L1: tenemos:
La densidad de energía es
PL0 / 2 PL0 / 2 P
W 1⎛F ⎞ 1 ΔL1 = = =
= ⎜ ⎟Δx = S t Δx YA FL0 2F
A 2⎝ A⎠ 2
Para la parte de la liga L2, tenemos:
Ejemplo 53. La elasticidad de una banda de goma
de longitud Lo es tal que una fuerza F aplicada a 2 PL0 / 2 2 PL0 / 2 P
ΔL 2 = = =
cada extremo produce una deformación longitudinal YA FL0 F
de una unidad. Se sujetan dos pesos del mismo
La mínima cantidad de trabajo que hará elevar
valor P, uno en un extremo y el otro en la mitad de ambos pesos del suelo es:
la banda y a continuación se levanta la banda con
Trabajo = Energía para estirar ΔL1 + Energía para
los pesos por su extremo libre. ¿Cuál es la mínima
cantidad de trabajo que hará elevar ambos pesos del estirar ΔL2 + Energía para elevar un peso P la altura
L1, el peso inferior no se levanta, solamente se
suelo?
despega del piso.
Energía para estirar una banda elástica es
1 2
U = kx
2

En este caso k = YA =
FL0
= 2 F , y x = ΔL1 ,
L0 / 2 Lo / 2
Solución.
Como cuando se aplicada a cada extremo una fuerza o ΔL2 , según corresponda
F se produce una deformación longitudinal de una
Trabajo = 2 F (ΔL1 ) + 2 F (ΔL2 ) + PL1
1 2 1 2
unidad: 2 2
FL0 Como conocemos ΔL1 , ΔL2 y
ΔL = 1 = , luego YA = FL0
YA L1 =
L0 L
+ ΔL1 = 0 +
P
2 2 2F
Tenemos
2 2
1 ⎛ P ⎞ 1 ⎛P⎞ ⎛L P ⎞
Trabajo = 2 F ⎜ ⎟ + 2 F ⎜ ⎟ + P⎜ 0 + ⎟
2 ⎝ 2F ⎠ 2 ⎝F⎠ ⎝ 2 2F ⎠
Finalmente
7 P2 1
Trabajo = + PL0
4 F 2

PREGUNTAS Y PROBLEMAS

1. ¿Cuál es el objeto del refuerzo de acero en una 3. ¿Qué clase de elasticidad se presenta en un puente
viga de concreto? colgante? ¿En un eje de dirección automotriz? ¿En
¿El concreto necesita mayor refuerzo bajo un resorte? ¿En tacos de caucho?
compresión o bajo tensión? ¿Por qué?
4. Una mujer distribuye su peso de 500 N
2. ¿Cuál es más elástico, caucho o acero? ¿Aire o igualmente sobre los tacones altos de sus zapatos.
agua? Cada tacón tiene 1,25 cm2 de área. a) ¿Qué presión
ejerce cada tacón sobre el suelo?

29
Elasticidad Hugo Medina Guzmán

b) Con la misma presión, ¿cuánto peso podrían 12. Una varilla metálica de 4 m de largo y sección
soportar 2 sandalias planas cada una con un área de 0,5 cm2 se estira 0,20 cm al someterse a una tensión
200 cm2? de 5000 N. ¿Qué módulo de Young tiene el metal?

5. ¿Cuál debe ser el diámetro mínimo de un cable 13. Una cuerda de Nylon se alarga 1,2 m sometida
de acero que se quiere emplear en una grúa diseñada al peso de 80 kg de un andinista.
para levantar un peso máximo de 10000 kg.?El Si la cuerda tiene 50 m de largo y 7 mm de diámetro,
esfuerzo de ruptura por tracción del acero es de ¿qué módulo de Young tiene el Nylon?
30×107 Pa. Igual pero si se quiere un coeficiente de
seguridad de 0,6. 14. Para construir un móvil, un artista cuelga una
esfera de aluminio de 5 kg de una alambre vertical
6. Dos alambres del mismo material, y misma de acero de 0,4 m de largo y sección 3×10-3 cm2. En
longitud l , cuyos diámetros guardan la relación la parte inferior de la esfera sujeta un alambre
similar del cual cuelga un cubo de latón de 10 kg.
n.
Para cada alambre calcular la deformación por
¿Qué diferencia de alargamientos tendrán bajo la
tensión y el alargamiento.
misma carga?
15. En el sistema mostrado en la figura, la barra OE
7. Un ascensor es suspendido por un cable de acero.
es indeformable y, de peso P; los tensores AC y DE
Si este cable es reemplazado por dos cables de acero
son de peso despreciable, área A y módulo de
cada uno con la misma longitud que el original pero
elasticidad Y.
con la mitad de su diámetro, compare el
Determinar cuánto bajará el peso W respecto a la
alargamiento de estos cables con el del cable
posición en la cual los tensores no estaban
original.
deformados.
8. Una cierta fuerza se requiere para romper un
alambre. ¿Que fuerza se requiere para romper un
alambre del mismo material el cual es
a) del doble de longitud?
b) el doble en diámetro y dé la misma longitud?

9. Un hilo de 80 cm de largo y 0,3 cm de diámetro se


estira 0,3 mm mediante una fuerza de 20 N. Si otro
hilo del mismo material, temperatura e historia previa
tiene una longitud de 180 cm y un diámetro de 0,25 l
cm. ¿qué fuerza se requerirá para alargarlo hasta una 16. Dos barras de longitud ( + Δl) cada una,
longitud de 180,1 cm?
2
Respuesta. áreas A 1 y A 2 y módulos de elasticidad Y 1 e Y 2
F = 211 N respectivamente, como se muestra en la figura, se
comprimen hasta introducirlas entre dos paredes
10. a) Calcule el cambio de dimensiones de una rígidas separadas una distancia l . ¿Cuál será la
columna de fundición gris (Y = 145 GPa) que tiene posición x de la unión de ambas barras?
dos tramos de 1,5 m cada uno y diámetros de 0,1 m
y 0,15 m, al soportar una carga de 500 kN. ¿Está
bien dimensionada la columna si el límite elástico de
la fundición gris es 260 MPa?
b) Si la columna fuera troncocónica de 3 m de altura,
y los diámetros de sus bases variaran entre 0,1 m y
0,15 m.
Respuesta. a) Lf = 3,001 m. Sí está bien 17. Una varilla de 1,05 m de largo y peso
dimensionada. despreciable está sostenida en sus extremos por
b) Lf = 3,0009 m alambres A y B de igual longitud. El área transversal
de A es de 1 mm2 y la de B 4 mm2. El módulo de
11. Un cable de acero de 2 m de largo tiene una Young de A es 2,4×1011Pa y de B 1,2×1011
sección transversal de 0,3 cm2. Se cuelga un torno de Pa. ¿En que punto de la varilla debe colgarse un
550 kg del cable. Determínese el esfuerzo, la peso P a fin de producir
deformación y el alargamiento del cable. Supóngase a) esfuerzos iguales en A y B? y b) ¿deformaciones
que el cable se comporta como una varilla con la iguales en A y B?
misma área transversal. El módulo de Young del
acero es 200×109 Pa. 18. Una barra de longitud L y masa m se encuentra
suspendida por un pivote B indeformable y por dos
barras en sus extremos como se muestra en la figura

30
Elasticidad Hugo Medina Guzmán

estas barras son iguales de área A, longitud l y


módulo de elasticidad Y. 22. Un alambre de cobre de 31 cm de largo y 0,5 mm
de diámetro está unido a un alambre de latón estirado
de 108 cm de largo y 1 mm de diámetro. Si una
determinada fuerza deformadora produce un
alargamiento de 0,5 mm al conjunto total y un valor
de Y = 12 x 1010 Pa, ¿cuál es el alargamiento de cada
parte?
Respuesta.
Δl = 0,27 mm para el latón.
19. En el sistema mostrado en la figura, calcular Δl = 0,23 mm para el cobre
cuánto desciende el extremo B de la barra
indeformable y de peso despreciable, cuando se le
23. Un alambre de acero dulce de 4 m de largo y 1
coloca un peso de 10 Ton. en ese extremo.
mm de diámetro se pasa sobre una polea ligera,
Los tirantes son de acero y de 2cm2 de área cada
uniendo a sus extremos unos pesos de 30 y 40 kg.
uno, suponga deformaciones pequeñas de tal manera
Los pesos se encuentran sujetos, de modo que el
que se puedan hacer las aproximaciones geométricas
conjunto se encuentra en equilibrio estático.
apropiadas.
Cuando se dejan en libertad, ¿en cuánto cambiará
la longitud del alambre?
Respuesta.
Δl = 1,0 mm
24. Un hilo está formado por un núcleo de acero
dulce de 1,3 cm de diámetro, al cual se le ha
fusionado una capa exterior de cobre (Y = 12 x
1010 Pa) de 0,26 cm de gruesa. En cada extremo del
hilo compuesto se aplica una fuerza de tracción de
Respuesta. Δy = 17,1 x 10-3 m 9000 N. Si la deformación resultante es la misma
en el acero y en el cobre, ¿cuál es la fuerza que
20. En el sistema mostrado en la figura, calcular soporta el núcleo de acero?
cuanto desciende el extremo B de la barra horizontal Respuesta.
rígida y de peso despreciable, cuando se le coloca F = 5812 N
una masa M en ese extremo.
Las barras inclinadas son iguales de área A y módulo 25. Un ascensor cargado con una masa total de
de elasticidad Y. 2000 kg esta de un cable de 3,5 cm2 de sección. El
Asuma pequeñas deformaciones, o sea, que se material del cable tiene un límite elástico de 2,5 x
pueden hacer las aproximaciones geométricas 108 Pa y para este material Y = 2 x 1010 Pa. Se
usuales. especifica que la tensión del cable nunca excederá
0,3 del límite elástico.
a) Hallar la tensión del cable cuando el ascensor
está en reposo.
b) ¿Cuál es la mayor aceleración permisible hacia
arriba?
c) ¿La distancia más corta de parada permisible
cuando la velocidad del ascensor es hacia abajo?
Respuesta.
F
a) = 5,6 x 107 Pa, b) a = 0,33 m/s2,
A
c) Δy = 33,8 m.

21. Un hilo delgado de longitud l , módulo de Young 26. Volver a resolver el Problema anterior,
Y y área de la sección recta A tiene unido a su teniendo en cuenta esta el peso del cable cuando
extremo una masa pesada m. Si la masa está girando tiene su longitud máxima de 150 m. La densidad
en una circunferencia horizontal de radio R con del material del cable es 7,8 x 103 kg /m3. Si se
velocidad angular ω, ¿cuál es la deformación del supera la carga máxima, ¿por dónde se romperá el
hilo? (Suponer que es despreciable la masa del hilo). cable: cerca de su punto más alto o próximo al
Respuesta. ascensor?
Δl mω 2 R Respuesta.
= F
l AY a) = 6,75 x 107 Pa, b) a = 1,32 m/s2,
A
31
Elasticidad Hugo Medina Guzmán

c) Δy = 85,3 m.

27. Un cable pesado de longitud inicial y área de


sección recta A tiene una densidad uniforme ρ y un
módulo de Young Y. El cable cuelga verticalmente
y sostiene a una carga Fg en su extremo inferior. La
fuerza tensora en un punto cualquiera del cable es
evidentemente suma de la carga Fg y del peso de la Respuesta. θ = 0,00422º
parte del cable que está debajo de dicho punto.
Suponiendo que la fuerza tensora media del cable 32. a) Desarrollar una expresión para la constante
actúa sobre la longitud total del cable l 0 , hallar el de torsión de un cilindro hueco en función de su
alargamiento resultante. diámetro interno Ro, su radio externo R1, su
Respuesta. longitud l y su módulo de corte G.
b) ¿Cuál deberá ser el radio de un cilindro macizo
⎛l ⎞⎛ Fg 1 ⎞
Δl = ⎜ 0 ⎟⎜⎜ + ρgl 0 ⎟⎟ de la misma longitud y material y que posee la
⎝Y ⎠⎝ A 2 ⎠ misma constante de torsión?
c) ¿Cuál deberá ser el ahorro de masa si se utilizase
28. Demostrar que cuando se somete un cuerpo el cilindro hueco en un eje de una máquina en lugar
elástico a una tensión de corte pura que no supera el de utilizar el cilindro macizo?
límite elástico de corte para el material, la densidad Respuesta.
⎛ πG ⎞ 4
( ) ( )
1
de energía elástica del cuerpo es igual a la mitad del
producto de la tensión de corte por la deformación
a) τ0 = ⎜ ⎟ 1R − R 4
0 , b) R = R1
4
− R 4 4
0
de corte.
⎝ 2l ⎠

c) Ahorro = 100 ⎢1 −
(
R12 − R02 ⎤ )
⎥%
29. El esfuerzo de la ruptura del cobre rolado para la
cizalladura es típicamente 1,5 x 108. ⎢⎣ (
R12 + R02 ⎥⎦)
¿Qué fuerzas F se deben aplicar a las cuchillas de
metal mostradas en la figura para cortar una tira de 33. A profundidades oceánicas de unos 10 km la
una hoja de cobre de 5 cm de ancho y 1,27 mm de presión se eleva a 1 kilobar, aproximadamente.
espesor? a) Si se hunde un trozo de acero dulce hasta esta
profundidad, ¿en cuánto variará su densidad?
b) ¿Cuál es la densidad del agua del mar a esta
profundidad si la densidad en la superficie vale 1,04
g/cm3?
B acero = 16 x 1010 N/m2 , B agua = 0,21 x 1010 N/m2,
1bar = 105 Pa
Respuesta.
a) 0,062 %, b) ρ = 1,105 g/cm3
Respuesta. 9525 N
34. Se somete a una muestra de cobre de forma
30. Una varilla que tiene 100 cm de longitud y 1 cúbica con 10 cm de arista a una compresión
cm de diámetro está sujeta rígidamente por un uniforme, aplicando una tensión equivalente a una
extremo y se le somete a torsión por el otro hasta tonelada perpendicularmente a cada una de sus
un ángulo de lº. Si se aplica la misma fuerza a la caras. La variación relativa de volumen que se
circunferencia de una varilla del mismo material observa es de 7,25×10-6 (∆V/Vo). Determinar el
pero que tiene una longitud de 80 cm y un diámetro módulo de compresibilidad del Cu en el sistema
de 2 cm, ¿cuál es el ángulo de torsión resultante? internacional, sabiendo que el módulo de Young del
Respuesta. θ = 0,1º cobre es 120×109 Pa. Obtener además el módulo de
Poisson.
31. La balanza de torsión de la figura se compone
35. Un depósito de acero de 60 litros de capacidad
de una barra de 40 cm con bolas de plomo de 2 cm
contiene oxígeno a una presión manométrica de 140
en cada extremo. La barra está colgada por un hilo
Pa. ¿Qué volumen ocupará el oxígeno si se le permite
de plata de 100 cm que tiene un diámetro de 0,5
que se expansione a temperatura constante hasta que
mm. Cuando se ponen muy de cerca de las bolas de
su presión manométrica es nula? (La presión
plomo, pero en lados opuestos, dos bolas mayores
manométrica es la diferencia entre la presión real en
de plomo de 30 cm de diámetro (ρ = 11,4 g/cm3),
el interior del depósito y la de la atmósfera exterior).
sus atracciones gravitatorias tienden a hacer girar la
Respuesta. V = 889 litros.
barra en el mismo sentido. ¿Cuál será la torsión del
hilo de plata?

32
Elasticidad Hugo Medina Guzmán

36. En cada extremo de una barra horizontal de 1,5 m fuerzas de compresión (valores negativos de F),
de larga, 1,6 cm de ancha y 1 cm de larga se aplica siempre disminuyen de volumen ¿Apoya esta
una fuerza de tracción de 2 800 N. El módulo de afirmación el hecho de que no existe ningún material
Young y el coeficiente de Poisson del material de la 1
barra son Y = 2 x 106 Pa y σ = 0,3. para el cual σ≥ ?
a) Hallar la deformación transversal barra. 2
b) ¿Cuáles son las variaciones relativas de la anchura
y altura? 38. Un manual de materiales relaciona estos datos
c) ¿Cuál es el aumento de volumen? para el aluminio en hoja laminada
d) ¿Cuál es la energía potencial adquirida por la Módulo de Young, 7 x 1010 Pa
barra? Límite elástico a la tracción, 7,2 x 107 Pa
Respuesta. Coeficiente de Poisson, 0,33
Tensión de tracción final, 14 x 107 Pa
Δd
a) == −2,625 × 10 − 4 , Tensión de tracción permisible, 0,4 de la tensión de
d0 tracción final
−4 La tensión de tracción permisible es la máxima
b) Δd = −4,2 × 10 cm tensión que se considera segura cuando este material
−4
c) Δh = −2,625 × 10 cm se utiliza en estructuras sometidas a de tracción
conocidas y constantes. Una tira de este aluminio de
37. a) Demostrar que el coeficiente de Poisson viene
76 cm de larga, 2,5 cm de ancha y 0,8 mm de gruesa
dado por
se estira gradualmente hasta que la tensión de
tracción alcanza su límite permisible. Calcular
3B. − 2 S
σ= a) su variación de longitud,
2(3B + S ) b) su variación de volumen,
b) Demostrar que a partir de esta ecuación se sigue c) el trabajo realizado y
que el coeficiente de Poisson debe estar comprendido d) la ganancia en la densidad de energía elástica.
Respuesta.
1
entre -1 y . a) Δl = 0,688 mm, b) ΔV = 0,0041 cm3,
2 c) W = 0,341 J, d) ΔU = 22400 J/m3
c) La experiencia demuestra que las barras sometidas
a fuerzas de tracción (valores positivos siempre
aumentan de volumen, mientras que si se someten a

33
Movimiento Oscilatorio Hugo Medina Guzmán

CAPÍTULO 2. Movimiento oscilatorio

INTRODUCCION. suelta se producen las oscilaciones.


Las vibraciones u oscilaciones de los sistemas Un carrito atado entre
mecánicos constituyen uno de los campos de estudio dos soportes en un
más importantes de toda la física. Virtualmente todo plano horizontal por
sistema posee una capacidad de vibración y la medio de resortes
mayoría de los sistemas pueden vibrar libremente de oscilará cuando el
muchas maneras diferentes. En general, las carrito se desplaza de su posición de reposo y
vibraciones naturales predominantes de objetos después se suelta.
pequeños suelen ser rápidas, mientras que las de
objetos más grandes suelen ser lentas. Las alas de un Una regla afianzada con
mosquito, por ejemplo, vibran centenares de veces abrazadera en un extremo a
por segundo y producen una nota audible. La Tierra un banco oscilará cuando se
completa, después de haber sido sacudida por un presiona y después se suelta
terremoto, puede continuar vibrando a un ritmo del el extremo libre.
una oscilación por hora aproximadamente. El mismo
cuerpo humano es un fabuloso recipiente de ¿Qué hacemos en éstos y otros ejemplos, para
fenómenos vibratorios; nuestros corazones laten, conseguir las oscilaciones? Las masas se sacan de su
nuestros pulmones oscilan, tiritamos cuando tenemos posición de reposo y después se sueltan. Una fuerza
frío, a veces roncamos, podemos oír y hablar gracias restauradora tira de ellas y parecen ir más allá de la
a que vibran nuestros tímpanos y laringes. Las ondas posición de reposo. Esta fuerza restauradora debe
luminosas que nos permiten ver son ocasionadas por existir de otra manera ellas no se moverían cuando
vibraciones. Nos movemos porque hacemos oscilar son soltadas. Porque hay una fuerza entonces
las piernas. Ni siquiera podremos decir correctamente debemos tener una aceleración. La fuerza de
"vibración" sin que oscile la punta de nuestra lengua.. restauración se dirige siempre hacia la posición de
Incluso los átomos que componen nuestro cuerpo equilibrio central -- la aceleración se dirige así
vibran. siempre hacia la posición de equilibrio central.
La traza de un electrocardiograma, mostrada en la
figura, registra la actividad eléctrica rítmica que
acompaña el latido de nuestros corazones.

Podemos determinar el gráfico distancia - tiempo


MOVIMIENTO OSCILATORIO para un objeto oscilante tomando una fotografía
Definición y características estroboscópica para un péndulo o usando el Sonic
¿Qué es un movimiento oscilatorio? ¡Es un Ranger del laboratorio. Se obtiene su desplazamiento
movimiento de vaivén! ¿Podemos hacer una máximo a un lado y otro de la posición de reposo.
descripción científica? Si estudiamos el movimiento La figura arriba muestra los gráficos distancia –
de un número de objetos podemos tiempo. Algunas oscilaciones parecen tener la misma
quizás contestar a la pregunta. Si una característica a la tomada al mismo tiempo para cada
masa se suspende a partir de un resorte, oscilación completa. Tales osciladores se conocen
se tira hacia abajo y después se suelta, se como isócronas, y mantienen esta característica
producen las oscilaciones constante del tiempo sin importar los cambios de la
amplitud debido al amortiguamiento.
El balanceo de una bolita Con un experimento simple como el mostrado en la
en una pista curvada, la figura a continuación, también se puede obtener el
bolita oscila hacia delante gráfico desplazamiento - tiempo para el movimiento
y atrás de su posición de oscilatorio de un sistema masa resorte, al que se le ha
reposo. atado un plumón que deja una traza en un rollo de
papel que se gira a velocidad constante. Esto produce
Una masa suspendida del una “hoja” que muestra que el movimiento de la
extremo de una cuerda (un masa tiene la forma sinusoidal.
péndulo simple), cuando la
masa se desplaza de su
posición de reposo y se la

1
Movimiento Oscilatorio Hugo Medina Guzmán

También ω 0 = 2πf , f es la frecuencia en


oscilaciones por segundo. Una oscilación por
segundo se llama 1 hertz (Hz).
Todos estos términos son muy importantes

Ejemplo 1. Demostrar que las ecuaciones


x(t ) = Asen (ω 0 t − ϕ ) , x(t ) = A cos(ω 0 t − φ )
Oscilaciones Sinusoidales ••
satisfacen la ecuación x + ω 0 x = 0 .
2
Concentraremos preferentemente nuestra atención
sobre las oscilaciones sinusoidales. La razón física Solución.
consiste en que realmente se presentan oscilaciones
x = Asen (ω 0 t − ϕ ) ,
puramente sinusoidales en una gran variedad de
dx •
= x = Aω 0 cos(ω 0 t − ϕ ) ,
sistemas mecánicos, siendo originadas por fuerzas
restauradoras que son proporcionales a los dt
desplazamientos respecto al equilibrio. Este tipo de
dx 2 ••
movimiento es posible casi siempre si el des-
plazamiento es suficientemente pequeño. Si, por 2
= x = − Aω 02 sen (ω 0 t − ϕ )
dt
ejemplo, tenemos un cuerpo sujeto a un resorte, la ••
fuerza ejercida sobre el mismo cuando el Reemplazando x y x en la ecuación:
− Aω 02 sen (ω 0 t − ϕ ) + ω 02 Asen (ω 0 t − ϕ ) = 0 ,
desplazamiento respecto al equilibrio es x puede
describirse en la forma
( )
F ( x ) = − k1 x + k 2 x 2 + k 3 x 3 + ... , donde k1, k2, con lo que queda demostrado.
De igual manera sucede con
k3, etc., son una serie de constantes, y siempre
podremos encontrar un margen de valores de x dentro
x(t ) = A cos(ω 0 t − φ ) .
del cual sea despreciable la suma de términos
correspondientes a x2, x3, etc., de acuerdo con cierto DESCRIPCIÓN DEL MOVIMIENTO
criterio previo (por ejemplo, hasta 1 en 103 o 1 en ARMÓNICO SIMPLE
106) en comparación con el término - k1x, a no ser Un movimiento del tipo descrito en la ecuación
que el mismo. k1 sea nulo. Si el cuerpo tiene masa m x(t ) = Asen (ω 0 t − ϕ ) , es conocido como
y la masa del resorte es despreciable, la ecuación del movimiento armónico simple (MAS), se representa
movimiento del cuerpo se reduce entonces a en un gráfico x - t de la forma indicada en la figura.
d 2x d 2 x k1 Destaquemos las características más importantes de
m 2 = − k1 x , o bien 2 + x = 0 esta perturbación sinusoidal:
dt dt m
k
Si por definición hacemos ω 0 = 1 , la ecuación
2

m
anterior se transforma en:
d 2x
2
+ ω 02 x = 0 , que en notación corta es
dt Movimiento armónico simple de período T y
•• amplitud A.
x + ω 02 x = 0 l. Está confinada dentro de los límites x = ± A . La
La solución a dicha ecuación diferencial puede magnitud positiva A se denomina amplitud del
expresarse en cualquiera de las formas: movimiento.
x(t ) = Asen (ω 0 t − ϕ ) , x(t ) = A cos(ω 0 t − φ ) , 2. El movimiento tiene un período T igual al tiempo
transcurrido entre máximos sucesivos o más
donde las fases iniciales , ϕ y φ difieren en π 2 . generalmente entre dos momentos sucesivos en se
Fácilmente se advierte que A representa el repitan tanto el desplazamiento x como la velocidad
desplazamiento máximo, esto es la amplitud. dx dt .
Las ecuaciones x(t ) = Asen (ω 0 t − ϕ ) , T es la inversa de la frecuencia f,
x(t ) = A cos(ω 0 t − φ ) , describen el movimiento 1
T= .
armónico simple. A es la amplitud, ω0 es la f
frecuencia angular, en radianes por segundo, ϕ es Dada la ecuación básica x = Asen (ωt + ϕ0 ) , el
la constante de fase. La cantidad en paréntesis período debe corresponder a un aumento de 2π en
(ωt + ϕ ) es la fase de la oscilación. A y ϕ se el argumento de la función sinusoidal. Así pues, se
determinan por las condiciones iniciales del tiene
problema.

2
Movimiento Oscilatorio Hugo Medina Guzmán

ω (t + T ) + ϕ 0 = (ωt + ϕ 0 ) + 2π , de aquí se tiene (


x = R cos θ + L2 − R 2 sen 2θ )12
,
2π ω Con θ = ωt da
T= y f = .
ω 2π
La situación en t = 0 (o en cualquier otro instante
(
x = R cos ωt + L2 − R 2 sen 2θ )
12

señalado) queda completamente especificada si se Esta expresión da el desplazamiento de P en función


del tiempo. Cuando comparamos esta ecuación con la
ecuación x = Asen (ωt + ϕ ) , vemos que el primer
establecen los valores de x y dx dt en dicho mo-
mento. En el instante particular t = 0, llamaremos a
término, Rcos ωt , corresponde al movimiento
estas magnitudes x 0 y v 0 , respectivamente.
armónico simple con ϕ = π 2 , pero el segundo no.
Entonces se tienen las identidades siguientes:
Así, aunque el movimiento de P es oscilatorio, no es
x0 = Asenϕ 0 armónico simple
Estas dos relaciones v 0 = ωA cos ϕ 0 pueden Un ingeniero mecánico al diseñar un mecanismo
como el de la figura tiene que pensar cómo aplicar la
utilizarse para calcular la amplitud A y el ángulo ϕ0 fuerza correcta en P de modo que el desplazamiento x
(ángulo de fase inicial del movimiento): esté dado por la ecuación expresada líneas arriba, de
modo que la rueda se mueve con movimiento circular
⎛ ωx ⎞
2
⎛v ⎞
A = x02 + ⎜ 0 ⎟ , ϕ 0 = tan −1 ⎜⎜ 0 ⎟⎟ uniforme. Cuando P está unido al pistón de una
⎝ω ⎠ ⎝ v0 ⎠
máquina de vapor, esto se lleva a cabo regulando la
admisión de vapor.
El valor de la frecuencia angular, ω del movimiento
se supone conocido por otros medios. EL MOVIMIENTO ARMONICO SIMPLE Y EL
MOVIMIENTO CIRCULAR UNIFORME.
Ejemplo 2. Determinar si P en el mecanismo La posición.
ilustrado en la figura se mueve con MAS. En este El movimiento armónico simple (MAS) se puede
mecanismo, QQ’ es una barra sobre la cual puede relacionar con el movimiento circular de la manera
deslizarse el cilindro P; está conectada por una varilla siguiente. Imagine una clavija P unida a una rueda
L al borde de una rueda de radio R que gira con orientada con su eje perpendicular al plano de la
velocidad angular constante (Este mecanismo, figura siguiente. La clavija está una distancia A del
encontrado en muchas máquinas de vapor, eje, y la rueda rota con velocidad angular constante
transforma el movimiento oscilatorio del pistón en el ω. Se proyecta la clavija sobre el eje horizontal (el
movimiento rotacional de la rueda). eje de x en la figura).

En t = 0, la clavija en toda la trayectoria está a la


El movimiento de P es oscilante pero no armónico derecha y la proyección está en x = A.
simple. La posición de la proyección es
Solución.
De la figura podemos ver fácilmente que P oscila
x = A cos θ = A cos ωt .
desde una posición a una distancia (L + R) a partir de
La velocidad.
O hasta una posición (L – R) a partir de O. Para
La velocidad tangencial de la clavija tiene una
determinar si el movimiento es armónico simple,
debemos encontrar si el desplazamiento de P magnitud Aω, y su proyección en el eje x es
satisface la ecuación x = Asen (ωt + ϕ ) . De la
v = − Aωsenωt como se muestra en la figura
siguiente.
geometría de la figura tenemos que
x = R cos θ + L cos φ y Lsenφ = Rsenθ , de
modo que
⎛R⎞
senφ = ⎜ ⎟senθ
⎝L⎠
(
cos φ = 1 − sen 2φ )
12
=
L
(
1 2
L − R 2 sen 2θ )
12
.

Por consiguiente
La aceleración.

3
Movimiento Oscilatorio Hugo Medina Guzmán

La aceleración de la clavija (centrípeta) es Aω A=2m


2

dirigida como se muestra en la figura siguiente. ω = π y como ω = 2πf ; π = 2πf; f = 0,5 s-1
T = 1/f = 1/0,5 = 2s.
b) La fase viene dada, en este caso por
φ = πt + π 4 ; φ = 2π + π 4 = 9π / 4rad ϕ
c) Derivando la ecuación de la elongación respecto a
la variable t tenemos la ecuación de la velocidad:
dy ⎛ π⎞
v= = −2πsen ⎜ πt + ⎟ (SI)
dt ⎝ 4⎠
d) Derivando de nuevo respecto a la variable t
La proyección de la aceleración en el eje de x es obtenemos la ecuación de la aceleración:
a = − Aω 2 cos ωt . Así vemos que la posición en dv ⎛ π⎞
a= = −2π 2 cos ⎜ πt + ⎟ (SI)
el eje x exhibe el movimiento armónico simple dt ⎝ 4⎠
desde que las ecuaciones para x, v, y a son iguales a Sustituyendo en las ecuaciones correspondientes:
lo obtenido arriba. Si en vez de fijar t = 0 cuando la y = -1,4142 m. ; v = 4,44 m/s. ; a = 13,96 m/s2
proyección estaba toda a la derecha, nosotros e) La velocidad máxima se adquiere cuando el seno
hubiésemos elegido otro punto de partida con del ángulo vale 1;
ωt = 0 , nuestras ecuaciones habrían incluido el v máx = ±6,29m/s
ángulo de la fase ϕ.
De la discusión anterior se puede ver porque se y la aceleración máxima cuando el coseno del ángulo
designa con la letra ω a la velocidad angular, así vale 1;
como también a la frecuencia angular. a máx = ±19,72m/s 2
f) El desplazamiento Δy viene dado por la
Ejemplo 3. Un punto material de 2,5 kg experimenta
un movimiento armónico simple de 3 Hz de diferencia entre y para t = 1 e y para t = 0.
frecuencia. Hallar: El valor de y para t = 1 es y1 = - 1,4142 m,
a) Su frecuencia. y para t = 0 es y0 = 2cos π/4 = 1,4142m ;
b) Su aceleración cuando la elongación es de 5 cm. Δy = -1,4142 -1,4142 = -2,83 m
c) El valor de la fuerza recuperadora para esa
elongación. Ejemplo 5. Sostengo con la palma de la mano
Solución. abierta una caja de fósforos. De repente comienzo a
La pulsación se relaciona con la frecuencia mediante mover la mano verticalmente con un movimiento
la expresión: armónico simple de 5 cm amplitud y frecuencia
a) ω = 2π f = 2π·3 = 6π rad. progresivamente creciente. ¿Para qué frecuencia
b) a = ω2·s = (6π)2·0,05 = 17,8 m/s. dejará la caja de fósforos de estar en contacto con la
c) F = m·s = 2,5 ·17,8 = 44,4 N. de prescinde del mano?
signo “-“ en la expresión de la aceleración pues tal
signo únicamente indica que el sentido de esta
magnitud es contrario al de la elongación.

Ejemplo 4. La amplitud de un móvil que describe un


MAS, viene dada, en función del tiempo, por la
⎛ π⎞
expresión: y = 2 cos⎜ π t + ⎟ (SI). Determinar: Solución.
⎝ 4⎠ Cuando baja la palma de la mano, la caja de
a) Amplitud, frecuencia y periodo del movimiento. fósforos, a partir de la posición de equilibrio, se
b) Fase del movimiento en t = 2s. encuentra sometida a la aceleración de la gravedad,
c) Velocidad y aceleración del móvil en función del g, constante en todo momento, y dirigida
tiempo. verticalmente hacia abajo, y a la aceleración
d) Posición, velocidad y aceleración del móvil en t = correspondiente al movimiento armónico simple:
1 s. ω 2 y = 4π 2 f 2 y , dirigida hacia arriba y que
e) Velocidad y aceleración máximas del móvil. alcanza el valor máximo en el extremo de la
f) Desplazamiento experimentado por el móvil entre
trayectoria: amáx = 4π f A amax .
2 2
t = 0 y t = 1 s.
Cuando esta última aceleración iguale o supere a la
Solución. de la gravedad la caja de fósforos dejará de estar en
a) Por comparación con la ecuación general contacto con la mano. Eso sucederá cuando:
y = A cos(ω t + ϕ 0 ) se deduce que: 4π 2 f 2 A = g ⇒ f = 2,23 s-1

4
Movimiento Oscilatorio Hugo Medina Guzmán

d 2x
Ejemplo 6. Un bloque descansa sobre una superficie a= 2
= −0,025ω 2 senω t
horizontal. dt
a) Si la superficie se encuentra en movimiento
armónico simple en dirección paralela al piso, ⇒ a máx = 0,025ω 2
realizando dos oscilaciones por segundo. El
El bloque dejará de tener contacto con la superficie
coeficiente estático de rozamiento entre el bloque y
la superficie es 0,5. ¿Qué magnitud debe tener la cuando la fuerza de fricción que la sostiene fija al
amplitud de cada oscilación para que no haya
piso sea mayor que el peso del objeto, eso sucede
deslizamiento entre el bloque y la superficie?
b) Si la plataforma horizontal vibra verticalmente con cuando ma máx = mg
movimiento armónico simple de amplitud 25 mm.
¿Cuál es la frecuencia mínima para que el bloque
deje de tener contacto con la plataforma?
Solución.

f = 2 c/s ⇒ ω = 2π f = 4π g
m0,025ω 2 = mg ⇒ ω 2 =
x = Asenω t x = Asen 4π t 0,025
Su aceleración es: g
2
⇒ω = = 19,8 rad/s
d x 0,025
a= 2
= − A16π 2 senω t ⇒ a máx = 16 Aπ 2
dt f = 3,15 c/s
El bloque dejará de tener contacto con la superficie
cuando la fuerza de fricción que la sostiene fija al Ejemplo 7. Si la Tierra fuese homogénea y se hiciese
un conducto recto de polo a polo, al dejar caer por él
piso sea menor que la fuerza de inercia, eso sucede un cuerpo desde uno de los polos.
a) Demostrar que adquirirla un movimiento armónico
cuando ma máx = F f
simple (MAS).
b) Calcular el período de este movimiento.

Solución.

μg
m16 Aπ 2 = μmg ⇒ A = =
16π 2

A=
(0,5)(9,8) = 0,031 m
16π 2 a) La ley de la gravitación universal nos dice:
→ M ′m
A = 31 mm F = −G rˆ
r2
b)
M ′m
En módulo: F = −G
x2

el signo menos nos indica que F va dirigida hacia O.


En nuestro problema M ′ la masa encerrada dentro
y = 0,025senω t del círculo de puntos de la figura. Si llamamos ρ a la
densidad de la Tierra, tendremos:
Su aceleración es:
4
M ′ = Vρ = πx 3 ρ
3

5
Movimiento Oscilatorio Hugo Medina Guzmán

Por la segunda ley de Newton: M 0m


→ → G = mg 0 ⇒ GM 0 = g 0 R02 ,
∑F = ma
2
R0
M ′m •• mg 0 r
F = −G = mx Obtenemos: F0 =
2
x R0
4πρGm La fuerza responsable del movimiento es:
Luego: F = − x = −kx
3
4πρGm mg 0 r x
De aquí k =
F =− senϕ , senϕ =
3 R0 R
El movimiento es, por tanto, vibratorio armónico De aquí
simple. mg 0
b) de período: F =− x = − kx
R0
m 3
T = 2π = 2π El signo menos nos indica que va dirigida hacia abajo.
k 4πρG El movimiento es oscilatorio armónico simple.
b) de período
Ejemplo 8. Si la Tierra fuese homogénea y se hiciese m R0
en conducto recto como se indica en la figura, al dejar T = 2π = 2π = 84 min
caer por él un cuerpo de masa m
k g0
a) demostrar que adquiriría un movimiento oscilatorio
armónico simple. Ejemplo 9. Una barra pesada uniforme de masa m
b) Calcular el período de ese movimiento. reposa sobre dos discos iguales que son girados
Suponer que no existen rozamientos entre el cuerpo y continuamente en sentidos opuestos, como se
las paredes del conducto. muestra. Los centros de los discos esta separados una
distancia d. El coeficiente fricción entre las barras y
la superficie de los discos es μ , constante
independiente de la velocidad relativa de las
superficies.
Inicialmente la barra se mantiene en reposo con su
centro a una distancia x0 del punto equidistante de los
discos. Al tiempo t = 0 se suelta. Encontrar el
movimiento subsiguiente de la barra.
Solución.
a) Llamando M a la masa de Tierra encerrada en la
esfera de radio r, obtenemos para valor del módulo
de la fuerza F0 que representamos en la figura:

Solución.
Aparato

Mm
F0 = G
r2
dm M M 0
Como: ρ = = =
dV V V0
r3
M= M0
R03
Sustituyendo, teniendo en cuenta que

6
Movimiento Oscilatorio Hugo Medina Guzmán

Diagrama de cuerpo libre de la barra 1


Las fuerzas actuantes sobre la viga se muestran en U = ∫ dU = ∫ kxdx = kx 2
dibujo siguiente. Los centros de los discos están 2
separados una distancia d. Las fuerzas de rozamiento La energía potencial del oscilador armónico simple
son en sentidos opuestos. 1 2
es U= kx
2
Como hemos visto es la energía de deformación
elástica del resorte.
Como
x = Asen (ωt − ϕ ) , y x max = A
Se tiene
1 2
U max = kA
2
Aplicando la segunda ley de Newton: Por otra parte la energía cinética del oscilador
Fy =0: N1 + N 2 − mg = 0 (1) armónico simple es
2
⎛d ⎞ ⎛d ⎞ 1 1 ⎛•⎞
τC = 0 : − N1 ⎜ + x ⎟ + N 2 ⎜ − x ⎟ = 0 K = mv 2 = m⎜ x ⎟
⎝2 ⎠ ⎝2 ⎠ 2 2 ⎝ ⎠
(2) Como

La ecuación de momentos (2) se escribe con respecto
al centro de gravedad C de la barra, Despejando N1 y
x = A cos(ωt − ϕ ) ,
N2 de (1) y (2), obtenemos Con

x max = Aω , se tiene
1 ⎛d ⎞ 1 ⎛d ⎞
N1 = mg ⎜ − x ⎟ , N 2 = mg ⎜ + x ⎟ 1 •2 1 1
2 ⎝2 ⎠ 2 ⎝2 ⎠ K max = m x max = mA 2ω 2 = kA 2
2 2 2
Como ∑ F = ma , para la barra, obtenemos: La Energía mecánica total es:
•• •• E = K +U
F f 1 − F f 2 = m x ⇒ μN1 − μN 2 = m x
mA 2ω 2 cos(ωt − ϕ ) + kA 2 cos(ωt − ϕ )
1 1
=
1 ⎡⎛ d ⎞ ⎛d ⎞⎤ •• 2 2
⇒ μmg ⎢⎜ − x ⎟ − ⎜ + x ⎟⎥ = m x Como
2 ⎣⎝ 2 ⎠ ⎝2 ⎠⎦ k
Simplificando: ω2 =
m
••
⎛ 2 μg ⎞
x+ ⎜ ⎟x = 0 . E= kA cos 2 (ωt − ϕ ) + kA 2 sen 2 (ωt − ϕ )
1 2 1
⎝ d ⎠ 2 2
Ecuación correspondiente al movimiento armónico 1 2
simple, cuya frecuencia natural es ωo es E= kA = Constante
2
2 μg
ωo = rad/s O sea que
d E = K + U = K max = U max
La ecuación del movimiento de la barra.
x = x0 cos ω 0 t PROBLEMA BASICO MASA – RESORTE
La barra se mantiene un moviendo oscilatorio Resorte horizontal.
armónico simple sobre los discos que giran en En nuestra primera referencia a este tipo de sistemas,
sentidos opuestos. considerábamos que estaba compuesto por un solo
objeto de masa m sujeto a un resorte de constante k y
ENERGIA EN EL MOVIMIENTO ARMONICO longitud l o a otro dispositivo equivalente, por
SIMPLE ejemplo, un alambre delgado, que proporciona una
fuerza restauradora igual al producto de cierta
Como oscilador armónico simple es un sistema
constante k por el desplazamiento respecto al
conservativo, la fuerza se puede derivar de la función
equilibrio.
energía potencial.
dU
F =−
dx
dU
Como F = − kx , tenemos: = kx Esto sirve para identificar, en función de un sistema
dx de un tipo particular sencillo, las dos características

7
Movimiento Oscilatorio Hugo Medina Guzmán

que son esenciales en el establecimiento de


movimientos oscilantes: Cuando se vea una ecuación análoga a éstas se puede
1. Una componente inercial, capaz de transportar llegar a la conclusión de que el desplazamiento x es
energía cinética. una función del tiempo de la forma
2. Una componente elástica, capaz de almacenar
x(t ) = Asen (ωt − ϕ ) , en donde ω =
k
energía potencial elástica. siendo
m
Admitiendo que la ley de Hooke es válida, se obtiene
una energía potencial proporcional al cuadrado del k la constante del resorte y m la masa.
desplazamiento del cuerpo respecto al equilibrio Esta solución seguirá siendo válida, aunque el
sistema no sea un objeto aislado sujeto a un resorte
1 2 carente de masa.
igual a kx . Admitiendo que toda la inercia del
2 La ecuación contiene otras dos constantes, la
sistema está localizada en la masa al final del resorte, amplitud A y la fase inicial ϕ , que proporcionan
se obtiene una energía cinética que es precisamente entre las dos una especificación completa del estado
1 2 de movimiento del sistema para t = 0 (u otro tiempo
igual mv , siendo v la velocidad del objeto. Debe señalado).
2
Resorte vertical.
señalarse que ambas hipótesis particularizaciones de
Hasta este punto hemos considerado solamente
las condiciones generales 1 y 2 y que habrá muchos
resortes en posición horizontal, los que se encuentran
sistemas oscilantes en que no se apliquen estas
sin estirar en su posición de equilibrio. En muchos
condiciones especiales. Sin embargo, si un sistema
casos, sin embargo, tenemos resortes en posición
puede considerarse compuesto efectivamente por una
vertical.
masa concentrada al final de un resorte lineal
(“lineal” se refiere a su propiedad y no a su forma
geométrica), entonces podemos escribir su ecuación
del movimiento mediante uno de estos dos
procedimientos:

1. Mediante la segunda ley de Newton (F = ma),


− kx = ma
2. Por conservación de la energía mecánica total (E),
1 2 1 2
mv + kx = E
2 2
La segunda expresión es, naturalmente, el resultado El resorte tiene una longitud
de integrar la primera respecto al desplazamiento x, original l , cuando se ata una masa
pero ambas son ecuaciones diferenciales del mo- m a un resorte en posición vertical,
vimiento del sistema. el sistema está en equilibrio cuando
Vamos a aplicar la segunda ley de Newton (F = ma), el resorte ejerce una fuerza hacia
en el instante en que el resorte se a estirado una arriba igual al peso de la masa. Esto
longitud x es, el resorte se estira una longitud
Δl dada por
mg
kΔl = mg ⇒ Δl =
k
Por consiguiente, una masa en
un resorte vertical oscila
alrededor de la posición de
La figura a continuación muestra el diagrama del equilibrio.
cuerpo libre de la masa. Aplicando la segunda ley de
Newton ( F = ma) ,
− k ( y + Δl ) + mg = ma
⇒ − ky − kΔl + mg = ma
Aplicando la segunda ley de Newton (F = ma), Como kΔl = mg :
d 2x ••
− ky = ma
− kx = ma ⇒ m + kx = 0 o m x + kx = 0
dt 2 d2y ••
•• k ⇒m 2
+ ky = 0 o m y + ky = 0
x+ x=0 dt
m
8
Movimiento Oscilatorio Hugo Medina Guzmán

•• k Para cada uno de los resortes:


y+ y=0
m ∑F x = ma x , F1 = k1 x , F2 = k 2 x
En todos los otros aspectos las oscilaciones son Visto en conjunto la masa oscila debido a un resorte
iguales que para el resorte horizontal. equivalente: F = k e x , ahora F = F1 + F2
y (t ) = Asen (ωt − ϕ ) Luego, podemos escribir.
El moviendo es armónico simple y la frecuencia está
k k e x = k1 x + k 2 x ⇒ k e = k1 + k 2
dada por ω= , siendo k la constante del
m ,
resorte y m la masa. (k1 + k 2 ) m
Con esto ω= y T = 2π
Ejemplo 10. Una masa m se conecta a dos resortes de
m (k1 + k 2 )
constantes fuerza k1 y k2 como en las figuras a , b y c.
En cada caso, la masa se mueve sobre una superficie c) Haciendo el diagrama de cuerpo libre.
sin fricción al desplazarse del equilibrio y soltarse.
Encuentre el periodo del movimiento en cada caso.

(a)
Para cada uno de los resortes:
∑F x = ma x , F1 = k1 x , F2 = k 2 x
Visto en conjunto la masa oscila debido a un resorte
(b)
equivalente: F = k e x , ahora F = F1 + F2
Luego, podemos escribir.
k e x = k1 x + k 2 x ⇒ k e = k1 + k 2 ,
(c)
Solución. (k1 + k 2 ) m
Con esto ω= y T = 2π
a) Haciendo el diagrama de cuerpo libre. m (k1 + k 2 )
¿Cuál sería el periodo para el caso de la figura
siguiente?
Para cada uno de los resortes:
∑F x = ma x , F = k1 x1 , F = k 2 x 2
Visto en conjunto la masa oscila debido a un resorte
equivalente: F = k e x , donde x = x1 + x 2
Luego, podemos escribir.
F F F 1 1 1
= + ⇒ = + , Ejemplo 11. Al suspender un cuerpo de masa m de un
k e k1 k 2 k e k1 k 2 resorte de constante k1, y separarlo ligeramente de su
posición de equilibrio, el sistema oscila con una
k1 k 2
y ke = , frecuencia f1. Si ahora este resorte se monta como
k1 + k 2 indica la figura, junto con otros dos, de constantes k2
= 2k1 y k3 = 4k1, utilizando una barra de peso
k1 k 2
Con esto ω= y despreciable, ¿cuál será la nueva frecuencia propia del
m(k1 + k 2 ) sistema con relación a la anterior? A es el punto
medio de la barra.
m(k1 + k 2 )
T = 2π
k1 k 2

b) Haciendo el diagrama de cuerpo libre.

Solución.

9
Movimiento Oscilatorio Hugo Medina Guzmán

Como k1 es diferente k2 , los estiramientos


f 8
de los resortes no son iguales, por lo tanto = = 1,26 .
no podemos considerar la suma de las f1 5
constantes como la constante equivalente
de la parte en paralelo. Ejemplo 12. Un pequeño proyectil de masa 10 g que
En este caso vamos hallar directamente la vuela horizontalmente a velocidad 20 m/s impacta
constante equivalente del conjunto. plásticamente contra un bloque de madera de masa
190 g unido a un resorte ideal de constante 500 N/m
que se halla en posición horizontal. Determine la
amplitud y frecuencia de las oscilaciones producidas.
Solución.

Por conservación de cantidad de movimiento:


mv0 = (m + M )v1
m
⇒ v1 = v
(m + M ) 0
El estiramiento del resorte 1 es: 10 m
= 20 = 1
x1 =
mg / 2 mg
=
(10 + 190) s
k1 2 k1 Por conservación de energía:
El estiramiento del resorte 2 es: 1
(m + M )v12 = 1 kA 2 ⇒
mg / 2 mg 2 2
x2 = =
k2 4k1 1
(0,2kg )⎛⎜1 m ⎞⎟ = 1 ⎛⎜ 500 N ⎞⎟ A 2
El estiramiento del resorte 3 es: 2 ⎝ s ⎠ 2⎝ m⎠
mg mg De aquí: A = 0,02m = 2cm
x3 = =
k3 4 k1 La frecuencia se obtiene de
Con el peso mg el resorte se estira k
ω = 2πf = ⇒
x= 2
x + x2
+ x3 (m + M )
2
1 k
mg f =
Siendo x =
k eq
2π (m + M )
1 500 25
Reemplazando x, x1, x2 y x3: f = = = 7,96 Hz
mg mg 2π 0,2 π
+
mg 2k1 4k1 mg 5mg
= + = ⇒ Ejemplo 13. En el diagrama de la figura el resorte
k eq 2 4 k1 8k1 tiene masa despreciable y una longitud de 20cm
8 cuando está sin deformar. Un cuerpo de 2kg. Unido
k eq = k1 al resorte puede moverse sobre una superficie plana
5 horizontal lisa. A dicho cuerpo se le ata un hilo que
La frecuencia del conjunto es: pasa por una polea sin rozamiento y del cual pende
k eq 8k1 un cuerpo de 4kg. El sistema se halla inicialmente en
ω= = = 2π f reposo en la posición representada y la longitud del
m 5m resorte comprimido es de 15cm. Se corta entonces el
1 8k1 hilo y el cuerpo de 2 kg empieza a oscilar con
⇒ f = movimiento armónico simple.
2π 5m
Como la frecuencia del resorte 1 es
1 k1
f1 =
2π m
Obtenemos:

10
Movimiento Oscilatorio Hugo Medina Guzmán

••
⇒ x + 392 x = 0
c) La amplitud del movimiento es A = 0,05 m, la
frecuencia angular es ω = 392 = 19,8 rad/s
ω 19,8
La frecuencia es: f = = = 3,15 c/s
2π 2π
d) Hallar la energía mecánica del sistema.
a) ¿Cuál es el valor de “k”?
kA = 784(0,05) = 0,98 J
b) Hallar la ecuación diferencial 1 2 1
E=
2
c) Hallar la amplitud de oscilación y la frecuencia
2 2
natural del MAS.
d) Hallar la energía mecánica del sistema.
Solución. PÉNDULOS
a) ¿Cuál es el valor de “k”? Péndulo simple
Un ejemplo de movimiento armónico simple es el
movimiento de un péndulo. Un péndulo simple se
define como una partícula de masa m suspendida del
punto O por una cuerda de longitud l y de masa
despreciable. Si la partícula se lleva a la posición B
de modo que la cuerda haga un ángulo θ con la
vertical OC, y luego se suelta, el péndulo oscilará
entre B y la posición simétrica B’.
Para determinar la naturaleza de las oscilaciones,
debemos escribir la ecuación de movimiento de la
Δx = 0,2 − 0,15 = 0,05 m partícula. La partícula se mueve en un arco de circulo
F = kΔx de radio l = OA. Las fuerzas que actúan sobre la
partícula son su peso mg y la tensión T a lo largo de
F mg 4(9,8) la cuerda. De la figura, se ve que la componente
⇒ k= = = = 784 N/m
Δx Δx 0,05 tangencial de la fuerza es

b) Hallar la ecuación diferencial


Ft = −mgsenθ , donde el signo menos se debe a
que se opone al desplazamiento s = CA. La ecuación
Al cortar la cuerda
del movimiento tangencial es Ft = mat y, como la
partícula se mueve a lo largo de un círculo de radio
l , podemos usar la ecuación
dv dω d 2θ
at = =R = R 2 = Rα (reemplazando
dt dt dt
R por l ) para expresar la aceleración tangencial.
d 2θ ••
Esto es at = l 2
= l θ . La ecuación del
dt
movimiento tangencial es por consiguiente
Vamos a aplicar la segunda ley de Newton (F = ma) •• •• g
al cuerpo de masa 2 kg en el instante en que el ml θ = −mgsenθ o θ + senθ = 0
resorte está comprimido una longitud x l

d 2x ••
− kx = ma ⇒ m + kx = 0 o m x + kx = 0
dt 2
•• k •• 784
x+ x = 0 ⇒ x+ x=0
m 2
Movimiento osci1atorio de un péndulo.

11
Movimiento Oscilatorio Hugo Medina Guzmán

Esta ecuación no es del mismo tipo que la ecuación Un péndulo compuesto (o físico) es cualquier cuerpo
•• rígido que puede oscilar libremente alrededor de un
x + ω 2 x = 0 debido a la presencia del senθ Sin eje horizontal bajo la acción de la gravedad. Sea ZZ’
embargo, si el ángulo θ es pequeño, lo cual es cierto el eje horizontal y C el centro de masa del cuerpo.
si la amplitud de las oscilaciones es pequeña, Cuando la línea OC hace un ángulo θ con la
podemos usar la aproximación senθ ≈ θ y escribir vertical, el torque alrededor del eje z actuante sobre
para el movimiento del péndulo el cuerpo es τ z = − Mgdsenθ , donde d es la
•• g
θ+ θ = 0 distancia OC entre el eje z y el centro de masa C. Si
l I es el momento de inercia del cuerpo alrededor del
Esta es la ecuación diferencial idéntica a la ecuación ••
•• eje z, y α = θ es la aceleración angular.
x + ω 2 x = 0 si reemplazamos x por θ , esta vez Aplicando la segunda ley de Newton para la rotación
refiriéndonos al movimiento angular y no al
movimiento lineal. Por ello podemos llegar a la
∑τ = Iα obtenemos:
••
conclusión que, dentro de nuestra aproximación, el − Mgdsenθ = I θ . Suponiendo que las
movimiento angular del péndulo es armónico simple
oscilaciones son de pequeña amplitud, podemos
g
con ω = . El ángulo θ puede así expresarse en
2
suponer que senθ ≈ θ , de modo que la ecuación
l del movimiento es
la forma θ = θ 0 cos(ωt + ϕ ) , el período de •• Mgd •• Mgd
θ =− θ o θ+ θ =0
oscilación está dado por la expresión T T
l
T = 2π
g
Nótese que el período es independiente de la masa
del péndulo. Para mayores amplitudes, la
aproximación senθ ≈ θ no es válida.

Ejemplo 14. Calcular la tensión en la cuerda de un


péndulo en función del ángulo que hace la cuerda con
la vertical.
Péndulo compuesto.
Solución.
Podemos comparar esta ecuación del movimiento
Para calcular la tensión T, primero obtenemos la ••
fuerza centrípeta sobre la partícula, comparar con la ecuación x + ω x = 0 ,
2

Fc = T − FN = T − mg cosθ , demostrando que el movimiento angular oscilatorio


ya que, de la figura del péndulo simple, FN está dada Mgd
por mg cosθ. Luego igualando esta expresión a la es armónico simple, con ω2 = . Por
I
masa multiplicada por la aceleración centrípeta
consiguiente, el período de las oscilaciones es
mv 2 / l (nótese que l es el radio), con esto
I
obtenemos T = 2π
v2 Mgd
T − mg cosθ = m
l Ejemplo 15. Un anillo de 0,10 m de radio está
Para conseguir la velocidad usamos la conservación suspendido de una varilla, como se ilustra en la
de la energía considerando como nivel 0, el punto de figura. Determinar su período de oscilación.
suspensión del péndulo:
1 2
mv − mgl cos θ = −mgl cos θ 0 ⇒
2
mv = mgl(cos θ − cos θ 0 )
1 2
2
Esto es, v = 2 gl(cosθ − cosθ 0 )
2

y por lo tanto Solución.


T = mg (3 cosθ − 2 cosθ 0 ) Designando el radio del anillo por R, su momento de
inercia con respecto a un eje que pasa a través de su
centro de masa C es I C = mR . Entonces, si
2
Péndulo compuesto
(
aplicamos el teorema de Steiner I O = I C + Md
2
),
12
Movimiento Oscilatorio Hugo Medina Guzmán

en este caso d = R, el momento de inercia con b) Obtenga una ecuación que dé la aceleración
respecto a un eje que pasa a través del punto de angular α de la barra como función de θ.
suspensión O es c) Determine el periodo para pequeñas amplitudes de
I = I C + MR 2 = MR 2 + MR 2 = 2MR 2 , oscilación respecto de la vertical.
Para un péndulo físico o compuesto
I
T = 2π , luego
Mgd
2 MR 2 2R
T = 2π ⇒ T = 2π
MgR g
Lo cual indica que es equivalente a un péndulo
simple de longitud 2R, o sea el diámetro del anillo.
Al reemplazar los valores de R = 0,10 m y g = 9,8
m/s2 obtenemos T = 0,88 s.
Solución.
Ejemplo 16. En una caminata normal, las piernas a)
del ser humano o del animal oscilan libremente más
r 2M
L 3L r 2 M
o menos como un péndulo físico. Esta observación I = ∫ r 2 dM = ∫ 4
dr = ∫ 4 dr
ha permitido a los científicos estimar la velocidad a 0 L 0 L
la cual las criaturas extintas tales como los
M ⎡⎛ L ⎞ ⎛ 3L ⎞ ⎤
3 3
dinosaurios viajaban. ¿Si una jirafa tiene una = +
⎢⎜ ⎟ ⎜ ⎟ ⎥
longitud de piernas de 1.8 m, y una longitud del paso 3L ⎢⎣⎝ 4 ⎠ ⎝ 4 ⎠ ⎥⎦
de 1 m, qué estimaría usted para el período de la
oscilación de la pierna? ¿Cuál sería su velocidad al M ⎛ 28 L3 ⎞ 7 ML2
⎜ ⎟=
3L ⎜⎝ 64 ⎟⎠
caminar? =
48
− MgL
b) r = senθ = I 0α
4
MgL 12 g
α= senθ = − θ
4I 0 7L
Para oscilaciones pequeñas,
12 g
Solución. α =− θ
Podemos modelar la pierna de la jirafa como un
7L
péndulo físico de longitud L que oscila alrededor de I0 7L
un extremo. Su momento de inercia alrededor del c) T = 2π = 2π
MgL 4 12 g
1 2
punto de oscilación es I = mL = 1,68 s
3
El periodo de un péndulo físico es Ejemplo 18. Un disco pequeño delgado de masa m y
1 2 radio r se sujeta firmemente a la cara de otro disco
I mL delgado de radio R y masa M, como se muestra en la
T = 2π = 2π 3 = 2π
2L
figura. El centro del disco pequeño se localiza en el
Mgd L 3g
mg borde del disco mayor. El disco mayor se monta por
2 su centro en un eje sin fricción. El dispositivo se gira
= 2,2 s un ángulo θ y se suelta
longitud del paso 1m m a) Demuestre que la rapidez del disco pequeño cuando
v= = = 0,46 pasa por la posición de equilibrio es
periodo 2,2 s s
v = 2 gR
(1 − cosθ )
Ejemplo 17. Considere una barra delgada con masa M ⎛ M r2 ⎞
⎜⎜ + 2 + 2 ⎟⎟
= 4 kg y de longitud L = 1,2 m pivotada en un eje ⎝m R ⎠
horizontal libre de fricción en el punto L/4 desde un b) Demuestre que el periodo del
extremo, como se muestra en la figura. movimiento es
a) Encuentre (a partir de la definición) la expresión
para el momento de inercia de la barra respecto del
T = 2π
(M + 2m )R 2 + mr 2
pivote. 2mgR

13
Movimiento Oscilatorio Hugo Medina Guzmán

uno, sentados en los extremos opuestos de la barra se


balancean. ¿Cuál es el periodo del movimiento de
este sube y baja modificado?

Solución.
a) E = K + U = constante, Luego
Karriba + Uarriba = Kabajo + Uabajo
Como Karriba = Uabajo = 0 Solución.
1 2 La ecuación del péndulo físico puede encontrarse
Obtenemos mgh = Iω , pero aplicando la segunda ley de Newton para la rotación:
2
∑τ O = I Oα
h = R − R cos θ = R(1 − cos θ ) , ω = e
v
••
R − mgdsenθ = I O θ ,
2 2
MR mr m es la masa total del sistema.,
I= + + mR 2 . Sustituyendo g la aceleración de la gravedad,
2 2
encontramos d la distancia del punto de apoyo al centro de masa
del sistema.
1 ⎛ MR 2 mR 2 ⎞ v2
mgR(1 − cosθ ) = ⎜⎜ + + mR 2 ⎟⎟ 2 I O es el momento de inercia del sistema con
2⎝ 2 2 ⎠R respecto al apoyo (centro de oscilación)
⎛ M mr 2 m ⎞ 2
mgR(1 − cos θ ) = ⎜⎜ +
y es el ángulo que forma la línea que pasa por el
2
+ ⎟⎟v y punto de apoyo y por el centro con la vertical cuando
⎝ 4 2R 2⎠
el sistema está oscilando.
v 2 = 4 gR
(1 − cos θ ) ••mgd
Luego θ+ senθ = 0 , para oscilaciones
⎛ M r2 ⎞ IO
⎜⎜ + 2 + 2 ⎟⎟
⎝m R ⎠ •• mgd
pequeñas θ + θ =0,
de aquí IO
v = 2 gR
(1 − cosθ ) mgd
⎛ M r2 ⎞ De aquí ω=
⎜⎜ + 2 + 2 ⎟⎟ IO
⎝m R ⎠
b) Para un péndulo físico Reemplazando valores:
I
T = 2π , aquí M = m + M ; m = 2(44) + 8,5 = 96,5 kg,
Mgd
mR + M (0 ) mR
d= = 2(44)(2,1)sen110 + 2((4,25)(1,05)sen110
m+M (m + M ) d=
96,5
Luego:
⎛ MR 2 mr 2 ⎞
⎜⎜ + + mR 2 ⎟⎟ = 0,38 m
T = 2π ⎝ ⎠
2 2
⎡ mR ⎤
(m + M )g ⎢ ⎥ 1
⎣ (m + M ) ⎦ I O = 2(44)(2,1) 2 + 2 (4,25)(2,1) 2
3
⇒ T = 2π
(M + 2m )R 2 + mr 2
2mgR = 400,56 kgm 2

Ejemplo 19. Problema del sube y baja. Una barra Luego:


de 4,2 m de longitud, 8,5 kg de masa tiene un doblez
de 202º en su centro de tal manera que queda como
96,5(9,8)0,38 rad
muestra la figura. El doblez de la barra reposa sobre ω= = 0,95
un apoyo agudo. Los gemelos de masa 44 kg cada 400,56 s

14
Movimiento Oscilatorio Hugo Medina Guzmán

SISTEMAS DE PENDULOS Y RESORTES

Ejemplo 20. El sistema mostrado en la figura


consiste de una barra de masa despreciable, pivotada
en O, Una masa m pequeña en el extremo opuesto a
O y un resorte de constante k en la mitad de la barra.
En la posición mostrada el sistema se encuentra en
equilibrio. Sí se jala la barra hacia abajo un ángulo
pequeño y se suelta, ¿cuál es el periodo de las
oscilaciones? La figura muestra un metrónomo y un modelo de
metrónomo.

Metrónomo vertical invertido La figura muestra un


metrónomo invertido, donde la masa M se puede
situar entre los extremos A y B. Despreciar el peso
de la barra rígida OAB. OA = l , OB = 10l , la
masa de la barra del péndulo se considera
Solución. despreciable.
Supongamos al sistema desviado un ángulo θ:
a) Encuentre la ecuación diferencial que gobierna el
movimiento cuando la masa M está situada a una
distancia h del punto O.

b) Cuál es la frecuencia natural de la oscilación


cuando M está primero localizada en A y luego en B

Aplicando la segunda ley de Newton para la rotación:


∑τ O = I Oα

El resorte es el único elemento que causa una fuerza


recuperativa, el efecto del peso de la masa está Solución.
compensado por el efecto del estiramiento previo del a)
reste para poner al sistema en posición horizontal.
⎛l⎞ ••
− kx⎜ ⎟ cos θ = ml 2 θ
⎝2⎠
⎛l⎞
Tenemos que x = ⎜ ⎟senθ
⎝2⎠
Para ángulos pequeños:
senθ ≈ θ y cos ϑ ≈ 1
kl 2 •• •• k
Así: − θ = ml 2 θ ⇒ θ + θ =0
4 4m
Ecuación de moviendo armónico simple con ∑τ O = −2kx.l cos θ − Mgh.senθ = I Oα
k 4m Como: x = lsenθ , I O = Mh :
2
ω= ⇒ T = 2π
4m k
d 2θ
− 2k .l senθ cos θ − Mgh.senθ = Mh
2 2

Ejemplo 21. Problema del Metrónomo. El dt 2


metrónomo es un aparato para medir el tiempo y Con senθ ≈ θ , cos θ = 1 y simplificando:
marcar el compás de la música k ••
− 2 l 2θ − ghθ = h 2 θ
M

15
Movimiento Oscilatorio Hugo Medina Guzmán

•• ⎛ 2k l 2 g ⎞ •• ⎛ 2k l 2 g ⎞
θ + ⎜⎜ 2
+ ⎟⎟θ = 0 θ + ⎜⎜ 2
− ⎟⎟θ = 0
⎝M h h⎠ ⎝M h h⎠
b) con M en A: h = l b) con M en A: h = l
••
⎛ 2k g ⎞ ••
⎛ 2k g ⎞
θ + ⎜ + ⎟θ = 0 θ + ⎜ − ⎟θ = 0 ⇒
⎝M l⎠ ⎝M l⎠
2k g 2k ⎛M ⎞g 2k g 2k ⎛M ⎞g
⇒ ω= + = 1+ ⎜ ⎟ ω= − = 1− ⎜ ⎟
M l M ⎝ 2k ⎠ l M l M ⎝ 2k ⎠ l
Con M en B: h = 10l Con M en B: h = 10l
••
⎛ 2k g ⎞
••
⎛ 2k g ⎞ θ+⎜ − ⎟θ = 0
θ+⎜ + ⎟θ = 0 ⇒ ⎝ 100M 10l ⎠
⎝ 100M 10l ⎠ 2k g
⇒ ω= −
2k g 1 2k ⎛ 100M ⎞g
ω= + = 1+ ⎜ ⎟ 100M 10l
100M l 10 M ⎝ 2k ⎠l 1 2k Mg
= 1− 5
Metrónomo vertical derecho La figura muestra un 10 M kl
metrónomo invertido, donde la masa M se puede
situar entre los extremos A y B. Despreciar el peso Metrónomo horizontal
La figura muestra un metrónomo invertido, donde la
de la barra rígida OAB. OA = l , OB = 10l .
masa M se puede situar entre los extremos A y B.
a) Encuentre la ecuación diferencial que gobierna el
movimiento cuando la masa M está situada a una Despreciar el peso de la barra rígida OAB. OA = l ,
distancia h del punto O OB = 10l .
b) Cuál es la frecuencia natural de la oscilación a) Encuentre la ecuación diferencial que gobierna el
cuando M está primero localizada en A y luego en B movimiento cuando la masa M está situada a una
distancia h del punto O
b) Cuál es la frecuencia natural de la oscilación
cuando M está primero localizada en A y luego en B

Solución.
a)
Solución.
a)

∑τ O = −2kx.l cos θ + Mgh.senθ = I Oα Equilibrio estático ∑τ O = −2kΔxl + Mgh = 0


El torque producido por los pesos de las masas es
Como: x = lsenθ , I O = Mh :
2
compensado por los torques producidos por las
reacciones a las deformaciones previas de los
d 2θ
− 2k .l 2 senθ cos θ + Mgh.senθ = Mh 2 resortes. Luego la ecuación dinámica es:
dt 2
Con senθ ≈ θ , cos θ = 1 y simplificando:
k 2 ••
−2 l θ + ghθ = h 2 θ
M

16
Movimiento Oscilatorio Hugo Medina Guzmán

••
∑ F = ma o m x = −kx − F f
Donde Ff es la fuerza de fricción,
Usando la segunda ley de Newton para la rotación,
••

∑τ O = −2kx.l cos θ = I Oα ∑τ = I o θ ,

Como: x = lsenθ , I O = Mh :
2 ⎛ •• ⎞
••
⎛1 2 ⎞⎜ x ⎟
I o θ = F f R o ⎜ mR ⎟⎜ ⎟ = F f R
d 2θ ⎝2 ⎠⎜ R ⎟
− 2k .l 2 senθ cos θ = Mh 2 ⎝ ⎠
dt 2 1 • •
Con senθ ≈ θ , cos θ = 1 y simplificando: De aquí F f = m x , sustituyendo esta expresión en la
k 2 •• 2
−2 l θ = h2 θ ecuación de la fuerza obtenemos
M •• 1 •• 3 ••
•• ⎛ 2 k l 2 ⎞ m x = −kx − m x o m x + kx = 0
θ + ⎜⎜ ⎟θ = 0
2 ⎟
2 2
⎝ M h ⎠ 2k
y ω0 = rad / s
b) Con M en A: h = l 3M
••
⎛ 2k ⎞ 2k
θ + ⎜ ⎟θ = 0 ⇒ ω = Por el método de la energía:
⎝M ⎠ M La energía total del sistema es la suma de la energía
Con M en B: h = 10l cinética (traslacional y rotacional) y la energía
potencial; y permanece igual para todo tiempo,
⎛ 2k
••
⎞ E = (Ktraslación +Krotación) +U
θ+⎜ ⎟θ = 0 1 •2 1 •2
⎝ 100M ⎠ K traslación = M x , K rotación = I o θ
1 2k 2 2
⇒ω = Donde el momento de inercia del cilindro es
10 M 1
Io = MR 2 ,
Ejemplo 22. Un cilindro de masa M y radio R se 2
• •
conecta por medio de un resorte de constante k como
de muestra en la figura. Si el cilindro tiene libertad de También Rθ = x y R θ = x
rodar sobre la superficie horizontal sin resbalar, La ecuación de la energía del sistema para cualquier
encontrar su frecuencia. tiempo es
⎡ .
2 ⎤
⎢1 • 2 1 1 ⎛•⎞ ⎥
⎛ ⎞⎜ x ⎟ 1
E = ⎢ M x + ⎜ MR 2 ⎟⎜ ⎟ + kx 2 ⎥
⎢2 2⎝2 ⎠⎜ R ⎟ 2 ⎥
⎢ ⎝ ⎠ ⎥
⎣ ⎦
3 •2 1 2
= M x + kx
4 2
Solución. dE
Por la ley de Newton Como E = constante, =0
Aplicando la segunda ley de Newton al cilindro, dt

dE ⎛ 3 •• ⎞•
= ⎜ M x + kx ⎟ x = 0
dt ⎝ 2 ⎠
.
Como x no siempre es cero, la ecuación del

17
Movimiento Oscilatorio Hugo Medina Guzmán

movimiento es
3 •• 2k
M x + kx = 0 o ω0 = rad / s
2 3M

Ejemplo 23. El disco homogéneo tiene un momento


de inercia alrededor de su centro Io = 0,5 kgm2 y
radio R = 0,5 m. En su posición de equilibrio ambos
resortes están estirados 5 cm. Encontrar la frecuencia
angular de oscilación natural del disco cuando se le
da un pequeño desplazamiento angular y se lo suelta.
k = 800 N/m
Para hacerlo oscilar hay que sacarlo del equilibrio
con un movimiento vertical de la masa m.

Solución aplicando la segunda ley de Newton:


Como el peso está compensado por el estiramiento
previo la única fuerza actuante es producida por el
estiramiento adicional del resorte.

Solución.
Usando la segunda ley de Newton para la rotación,
••
∑τ = I o θ ,
La tensión inicial en cada uno de los resortes es
⎛ 800 N ⎞(0,05m ) = 40 N
⎜ ⎟
⎝ m⎠
El cambio en tensión es 800(0,5θ) = 400θ, y
••
0,5θ = [(40 − 400θ ) − (40 + 400θ )]0,5
Aplicando la segunda ley de Newton:
••
ο θ + 400θ = 0 Para la masa m, ∑ F = ma
De la cual ••

ω0 = 400 = 20rad / s − T ' = ma = m x


•• ••
Como x = rθ , x = r θ
Ejemplo 24. Determinar la frecuencia natural del ••
sistema resorte-masa-polea mostrado en la figura. Luego T ' = − mr θ (1)
Para el disco de masa M, ∑τ = Iα
••
I 0α = I 0 θ = T ' r − (krθ )r (2)
1
Donde I 0 = Mr 2 es el momento de inercia de la
2
polea.
Reemplazando (1? En (2):
1 •• ••

Solución. Mr 2 θ = r (− mr θ ) − kr 2θ
2
Equilibrio estático:
El resorte tiene un estiramiento inicial igual a rθ o ⎛1 2⎞
••
y ⎜ Mr + mr ⎟θ + kr θ = 0 ,
2 2

que produce una fuerza krθ o que equilibra al peso


⎝2 ⎠
mg . k
Finalmente ω0 = rad / s
O sea krθ o = mg M 2+m

Solución por el método de la energía:


E = K + U = constante
K = Kmasa + Kpolea

18
Movimiento Oscilatorio Hugo Medina Guzmán

1 • 2 1 •. 2 1 2 ÷ 2 1 •. 2 κ
K= m x + I 0 θ = mr θ + I 0 θ armónico simple, con ω2 = ; el período de
2 2 2 2 I0
1 2 1 2 2 oscilación es
U = kx = kr θ
2 2 I0
T = 2π
Como la energía total de sistema permanece κ
constante, Este resultado es interesante debido a que podemos
d
(K + U ) = 0 o usarlo experimentalmente para determinar el
momento de inercia de un cuerpo suspendiéndolo de
dt
• •• • •• • un alambre cuyo coeficiente de torsión κ se conoce,
mr 2 θ θ + I 0 θ θ + kr 2θ θ = 0 y luego midiendo el período T de oscilación.

θ ⎛⎜ mr 2 θ + I 0 θ + kr 2θ ⎞⎟ = 0
• •• ••
⇒ MOVIMIENTO ARMONICO EN DOS
⎝ ⎠ DIMENSIONES.
• Hasta ahora no hemos limitado a estudiar el
Como θ no siempre es cero, movimiento armónico de la partícula o cuerpo
descrito por una sola variable, ahora permitiremos a
⎛ 2 •• ••
2 ⎞
⎜ mr θ + I 0 θ + kr θ ⎟ es igual a cero. Luego la partícula, movimiento en dos dimensiones.
⎝ ⎠ → →

•• 2 F = −k r
kr
θ+ θ =0 La fuerza se puede descomponer en dos
I 0 + mr 2 componentes
Fx = − kx , Fy = − ky
k
⇒ ω0 = rad / s Las ecuaciones del movimiento son:
M 2+m •• ••
x + ω 02 x = 0 , y + ω 02 y = 0
PENDULO DE TORSION.
Donde como antes ω 02 = k m . Las soluciones son:
x(t ) = A cos(ω 0 t − α ) , y (t ) = B cos(ω 0 t − β )
Luego el movimiento es armónico simple en cada
una de las dimensiones, ambas oscilaciones tienen la
misma frecuencia pero tienen que diferenciar
amplitudes y fases. Podemos obtener la ecuación de
Otro ejemplo de movimiento armónico simple es el la trayectoria de las partículas eliminando el tiempo t
péndulo de torsión, consistente en un cuerpo entre las dos ecuaciones. Para esto escribimos:
suspendido por un alambre o fibra de tal manera que y (t ) = B cos[ω 0 t − α − (α − β )]
= B cos(ω 0 t − α ) cos(α − β )
la línea OC pasa por el centro de masa del cuerpo.
Cuando el cuerpo se rota un ángulo θ a partir de su
posición de equilibrio, el alambre se tuerce, - Bsen (ω 0 t − α )sen (α − β )
ejerciendo sobre el cuerpo un torque τ alrededor de
Con
OC que se oponen al desplazamiento θ y de
cos(ω 0 t − α ) =
x
magnitud proporcional al ángulo, τ = −κθ , donde y
κ es el coeficiente de torsión del alambre. A
2
Aplicando la segunda ley del movimiento (para ⎛x⎞
variables angulares): sen (ω 0 t − α ) = 1 − ⎜ ⎟
⎝ A⎠
∑τ 0 = I 0α
Llamando δ = (α − β ) :
Si I 0 es el momento de inercia del cuerpo con 2
B ⎛x⎞
respecto al eje OC, la ecuación del movimiento y = x cos δ − B 1 − ⎜ ⎟ senδ
•• A ⎝ A⎠
− κθ = I 0α , con α = θ , es Elevada al cuadrado se transforma en:
•• •• κ A 2 y 2 − 2 ABxy cos δ + B 2 x 2 cos 2 δ
I 0 θ = −κθ o θ + θ =0
I0 = A B sen δ − B x sen δ
2 2 2 2 2 2

Nuevamente encontramos la ecuación diferencial del Que es:


MAs, de modo que el movimiento angular es B 2 x 2 − 2 AB cos δ + Ay 2 = A 2 B 2 sen 2δ

19
Movimiento Oscilatorio Hugo Medina Guzmán

π
Para δ =± , esta ecuación toma la forma de una
2
elipse:
x2 y2
+ =1
A2 B 2
En el caso particular de A = B y δ = ± π 2 ,
tendremos un movimiento circular:
x 2 + y 2 = A2
Otro caso particular es con δ = 0 , en que
tendremos:
Bx 2 − 2 ABxy + Ay 2 = 0 ⇒ (Bx − Ay ) = 0 ,
2

expresión de ecuación de una recta:


B
y= , para δ = 0 Curvas de Lissajous.
A
De forma similar para δ = ±π En el caso de que el cociente de las frecuencias no
B sea una fracción racional, la curva será abierta; es
y=− , para δ = ±π decir, la partícula no pasará dos veces por el mismo
A punto a la misma velocidad.
En la figura pueden observarse algunas de las curvas
correspondientes al caso A = B, cuando δ = 0 , Medida del desfase entre dos señales
δ =π 4 y δ =π 2 En un osciloscopio componemos dos MAS de
direcciones perpendiculares y de la misma frecuencia
ω, desfasados δ. Supondremos por simplicidad que
ambas señales tiene la misma amplitud A.
x =Asen(ω t)
y =Asen(ω t + δ)
La trayectoria como podemos comprobar es una
elipse.
La medida de la intersección de la elipse con los ejes
X e Y nos permite medir el desfase δ, entre dos
señales x e y.

En general las oscilaciones bidimensionales no tienen


por qué ser las mismas frecuencias en los mismos
movimientos según las direcciones x e y, de forma
que las ecuaciones se conviertan en
( )
a) Intersección con el eje Y
x(t ) = A cos(ω x t − α ) , y (t ) = B cos ω y t − β Cuando x = 0, entonces ω t = 0, ó π .
y la trayectoria no es ya una elipse, sino una de las y0 = Asenδ
llamadas curvas de Lissajous. Estas curvas serán y0 = Asen(π + δ ) = - Asenδ
cerradas cuando el movimiento se repita sobre sí Si medimos en la parte positiva del eje Y, tendremos
mismo a intervalos regulares de tiempo, lo cual sólo que sen δ = y0/A
será posible cuando las frecuencias ω x y ω y , sean En la pantalla del "osciloscopio" el eje X y el eje Y
está dividido en 20 partes, cada división es una
«conmensurables», o sea, cuando ωx ω y sea una unidad.
fracción racional. En la figura, A=10, e y0 =5, el desfase δ =30º, ó mejor
En la figura a continuación se representa uno de δ =π /6
estos casos, para el cual ω x ω y = 2/3 (y asimismo,
b) Intersección con el eje X
Cuando y = 0, entonces ω t = -δ , ó (π - δ) .
A=By α = β ). x0 = - Asenδ

20
Movimiento Oscilatorio Hugo Medina Guzmán

x0 =Asen(π - δ) = Asenδ 3
En la figura, A=10, e x0=5, el desfase δ =30º, ó mejor y=− x
2
δ = π /6
Corresponde a una recta de pendiente -3/2.
c) Intersección con x =A el borde derecho de la
pantalla del "osciloscopio"
A =Asen(ω t) por lo que ω t = π /2
y1 = Asen(π /2+δ ) = Acosδ

En la figura A = 10 y y1 = 8.75, el desfase δ » 30º, ó


mejor δ =π /6
Podemos comprobar que se obtiene la misma
trayectoria con el desfase 30º y 330º y también con
150º y 210º. Pero podemos distinguir el desfase 30º
de 150º, por la orientación de los ejes de la elipse.

Medida de la frecuencia
Componemos dos MAS de direcciones
perpendiculares y de distinta frecuencia ωx, y ωy
.Supondremos por simplicidad que ambas señales
tiene la misma amplitud A y el desfase δ puede ser Ejemplo 26. Encuentre la ecuación de la trayectoria
cualquier valor de un punto sometido a dos movimientos oscilatorios
x =Asen(ωxt) armónicos rectangulares dados por las ecuaciones
y = Asen(ωyt+δ ) ⎛ π⎞
La relación de frecuencias se puede obtener a partir x = 3senω t ; y = 5sen⎜ ω t − ⎟
del número de tangentes de la trayectoria en el lado ⎝ 6⎠
vertical y en el lado horizontal. Solución.
Número de tangentes lado vertical x
ωx número de tangentes lado vertical x = 3senω t ⇒ senω t =
= 3
ω y número de tangentes lado horizontal
2
⎛ x⎞
Luego: cos ω t = 1 − ⎜ ⎟
⎝3⎠
⎛ π⎞
y = 5sen⎜ ω t − ⎟ ⇒
⎝ 6⎠
⎛ π⎞ y
sen⎜ ω t − ⎟ = =
⎝ 6⎠ 5
π π
senω tcos + cos ω tsen =
6 6
Ejemplo: en la figura
⎛ x ⎞⎛⎜ 3 ⎞⎟ ⎛⎜ x 2 ⎞⎟⎛ 1 ⎞
ωx 3 ⎜ ⎟⎜ + 1 − ⎜ ⎟
= ⎝ 3 ⎠⎝ 2 ⎟⎠ ⎜⎝ 9 ⎟⎠⎝ 2 ⎠
ωy 2
y 3x 1 x2
Ejemplo 25. Dos movimientos vibratorios ⇒ − = −
5 6 4 36
perpendiculares de la misma frecuencia tienen sus
Elevando al cuadrado:
amplitudes en la relación 2/3 y una diferencia de
marcha de media longitud de onda. Hállese la forma y 2 2 3 y x 3x 2 1 x 2
del movimiento resultante. − + = −
25 30 36 4 36
Solución. Simplificando:
Las ecuaciones de estos movimientos son:
x = A1senω t ; y = A2 sen (ω t + π ) = y2

3y x x2 1
+ =
A2 senω t cos π + A2 cosω tsenπ = − A2 senω t 25 15 9 4
Corresponde a la ecuación de una elipse inclinada.
x A 2
= 1 =−
y − A2 3
El movimiento resultante es según la ecuación

21
Movimiento Oscilatorio Hugo Medina Guzmán

MOVIMIENTO ARMONICO AMORTIGUADO.


En el movimiento armónico simple la amplitud es
constante al igual que la energía del oscilador. Sin
embargo sabemos que la amplitud del cuerpo en
vibración, como un resorte, un péndulo, disminuye
gradualmente, lo que indica una pérdida paulatina de
energía por parte del oscilador. Decimos que el
Ejemplo 27. Dos oscilaciones perpendiculares entre movimiento oscilatorio está amortiguado.
si tienen el mismo periodo, la misma amplitud y una
diferencia de marcha igual a λ/6. ¿Qué oscilación El Amortiguamiento es causado por la fricción, para
resultante produce? una resistencia la viscosa tal como la fuerza
Solución. amortiguadora del aire, la fuerza amortiguadora
Una diferencia de marcha de λ equivale a 2π. puede tomarse como proporcional de la velocidad.
λ Sea la fuerza de un amortiguador Fb = -bv donde el
Una diferencia de marcha de equivale a signo menos indica que esta fuerza tiene sentido
6 opuesto al movimiento del cuerpo oscilante:
2π λ π
= .
λ 6 3
Luego, las ecuaciones de los movimientos
componentes son:
x = asenω t , y = asen (ω t − π / 3)
Trabajando con y:
⎛ π⎞
y = asen⎜ ω t − ⎟ =
⎝ 3⎠
π π
asenω t cos − a cos ω tsen Aplicando la segunda ley de Newton:
3 3
⎛1⎞ x2 ⎛ 3 ⎞
⎟ − a 1 − 2 ⎜⎜
= x⎜ ⎟
⎝2⎠ a ⎝ 2 ⎟⎠
x 3a
⇒ y− =− a2 − x2
2 2 ma = - ky - bv
Elevando al cuadrado y simplificando:
3 2 •• •
y 2 + x 2 − xy = a m y = −ky − b y
4
Corresponde a la ecuación de una elipse inclinada •• • •• b • k
m y + b y + ky = 0 , y + y+ y = 0
m m
•• •
y + 2 β y + ω o2 y = 0 (I)

22
Movimiento Oscilatorio Hugo Medina Guzmán

b k El desplazamiento decrece a su posición de equilibrio


β= , ωo = sin oscilar en el menor tiempo posible, a este
2m m movimiento se le conoce como CRITICAMENTE
Solución de la ecuación es de la forma y = e
rt AMORTIGUADO.
Reemplazando en la ecuación obtenemos:
r 2 e rt + 2β re rt + ω o2 e rt = 0
Simplificando
r 2 + 2βr + ω o2 = 0
Las raíces de esta ecuación son:
r1 = − β + β 2 − ω 02 y r2 = − β − β 2 − ω 02
Por consiguiente la solución general de la ecuación Pero para amortiguadores fuertes según lo mostrado
(I) es en la figura abajo, el período varía según la amplitud
y = e − βt ⎡ Be ⎤
β 2 − ω o2 t − β 2 − ω o2 t y el movimiento cambia considerablemente del
+ Ce
⎢⎣ ⎥⎦ modelo armónico simple. El amortiguamiento crítico
es la que lleva al oscilador al reposo en el menor
Discusión de la solución
tiempo. Esto encuentra aplicaciones en instrumentos
a) Cuando ω o2 > β 2 donde es una ventaja el poder tomar una lectura
rápida del indicador. Es también útil por resortes en
β 2 − ω 02 = iω1 s una cantidad imaginaria y asientos y amortiguadores de vehículos.
ω o2 < β 2
[Be ]
c) Cuando
− βt iω1t − iω1t
y=e + Ce
β 2 − ω 02 = ω1
A iδ A − iδ
Haciendo B = e y C = e en este caso la solución tiene la forma

Obtenemos
2 2
[
y = e − βt Be ω1t + Ce −ω1t ]
En este caso tampoco existe oscilación, pero se
− βt ⎡ ei (ω1t +δ ) + e − i (ω1t +δ ) ⎤
y = Ae ⎢ ⎥
acerca a la posición de equilibrio más lentamente que
⎣ 2 ⎦ el crítico, a este movimiento se le conoce como
SOBREAMORTIGUADO
Expresión que se puede escribir usando las relaciones
de Euler como
y = Ae − βt cos(ω1t + δ )
Donde ω1 es la frecuencia del oscilador
amortiguado, aunque hablando estrictamente no es
posible definir la frecuencia en el caso del
movimiento amortiguado desde que este no es un
movimiento periódico.
La amplitud máxima del movimiento disminuye
.βt
debido al factor e .

Ejemplo 28. Un péndulo se ajusta para tener un


período exacto 2 segundos, y se pone en
movimiento. Después de 20 minutos, su amplitud ha
disminuido a 1/4 de su valor inicial.
Si el movimiento del péndulo puede se representado
por θ = θ 0 e − βt cos(2πft ) , ¿cuál es el valor de β?
−1, 386 1
Nota: e =
4
Solución.
Este movimiento se conoce como
SUBAMORTIGUADO o poco amortiguado. a) θ = θ 0 e − βt cos(2πft )
b) Cuando ω o2 = β 2 Para t = 20 x 60 = 1200 s
θ0
β 2 − ω 02 = 0 cantidad real = θ 0 e −1200 β (1)
4
En este caso la solución tiene la forma
y = (B + Ct )e − βt

23
Movimiento Oscilatorio Hugo Medina Guzmán

e −1200 β =
1
= e −1,386 θ = De − βt cos(ωt − φ ) y
4 •
θ = − Dωe − βt sen (ωt − φ ) − Dβ e − βt cos(ωt − φ )
− 1200t = −1,386 Con D y φ constantes cuyos valores dependen de las
condiciones iniciales del movimiento (en este caso
1,386
⇒ β= = 0,001155 para t = 0, θ = 0,1rad y

θ = 0.
1200
=1,2 x 10-3 N.s/m ó kg/s.
y ω= β 2 − ω02 = 4 2 − 46,9 = 5,56 rad
Ejemplo 29. El cuerpo E de 32,7 N en la figura está Por las condiciones iniciales
0,1 = D cos(− φ ) = D cos φ
asegurado a la varilla DF cuyo peso puede ignorarse.
(1)
El resorte tiene un módulo k = 100 N/m y el
coeficiente del amortiguador es b = 26,7 N-s/m. El 0 = − Dωsen (− φ ) − Dβ cos(− φ )
sistema está en equilibrio cuando DF está horizontal.
La varilla se desplaza 0,10 rad en sentido horario y
= D(ωsenφ − β cos φ ) (2)
desde el reposo cuando t = 0. Determinar De (2) obtenemos
a) la ecuación del movimiento de la varilla, β 4
tan φ = =− = −0,72
b) la frecuencia del movimiento. ω 5,56
⇒ φ = -0,62 rad
De (1)
0,1 0,1
D= = = 0,12 rad
cos φ 0,81
La ecuación del movimiento es
Solución. θ = 0,12e −4t cos(5,56t − 0,62) rad
La figura muestra el diagrama del cuerpo libre de la Correspondiente a un movimiento oscilatorio
varilla DF, desplazada en la dirección positiva. subamortiguado cuyo gráfico se muestra a
• continuación, la vibración se amortigua rápidamente.
Cuando el cuerpo está en equilibrio, θ y θ ambas

valen cero y T = T0 =
1,2
(32,7 ) .
1,5

b) La frecuencia del movimiento es ω = 5,56 rad/s.


Aplicando la segunda ley de Newton para la rotación
con el desplazamiento y velocidad indicados en la
Ejemplo 30. El sistema mostrado en la figura se
figura es:
encuentra en el plano horizontal en equilibrio.
••
1,2(32,7 ) − 1,2(26,7 )Fb − 1,5T = I o θ Consiste en una barra rígida indeformable de masa
M, ligada a dos resortes de constante k, y con una
⎛ •⎞
1,2(32,7 ) − 1,2(26,7 )⎜1,2 θ ⎟ − 1,5[(1,5)32,7 + 100(1,5θ )] masa en el extremo libre de magnitud “m”, sobre la
⎝ ⎠ cual actúa una fuerza disipativa proporcional a su
••
=
32,7
(1,2)2 θ velocidad Fv = - b vm . Si se desplaza un ángulo 0,15
rad en sentido horario y luego se le suelta.
9,8
La ecuación se reduce a Determinar:
•• • a) La ecuación de movimiento del sistema para
4,8θ + 38.4θ + 225θ = 0 ángulos pequeños de deformación
o b) Encontrar la ley de movimiento para cuando
•• • k=1500 N/m , b =40 N s/m y M =3m= 3kg, además
θ + 8,00θ + 46,9θ = 0 . l =1,5m
De la forma
•• •
θ + 2β θ + ω02θ = 0
Donde: 2β = 8 y ω02 = 46.9
Cuya solución es

24
Movimiento Oscilatorio Hugo Medina Guzmán

Si para t = 0 se desplaza un ángulo 0,15 rad en


sentido horario y luego se le suelta.
0,15 = θ 0 cos(ϕ )
0 = −10θ 0 cos(ϕ ) − 16,58θ 0 sen (ϕ )
De estas ecuaciones obtenemos:
ϕ = −0,54rad y θ 0 = 0,175rad
θ = 0,175e −10t cos(16,58t − 0,54)
Solución.
a) Ejemplo 31. Un bloque de 5,0 kilogramos se une a
un resorte cuya constante es 125 N/m. El bloque se
jala de su posición del equilibrio en x = 0 m a una
posición en x = + 0,687 m y se libera del reposo. El
bloque entonces ejecuta oscilación amortiguada a lo
largo del eje x. La fuerza amortiguadora es
proporcional a la velocidad. Cuando el bloque
primero vuelve a x = 0 m, la componente x de la
velocidad es - 2,0 m/s y la componente x de la
aceleración es +5,6 m/s2.

a) Calcule la magnitud de la aceleración del bloque
∑τ O = −2kx1 .l cos θ − b x 2 2l cos θ = I Oα después de ser liberado en x = + 0,687 m?
b) ¿Calcule el coeficiente de amortiguamiento b?
Como: c) Calcule el trabajo realizado por la fuerza
x1 = lsenθ ≈ lθ , x2 = 2lsenθ ≈ 2lθ ⇒ amortiguadora durante el recorrido del bloque de
• • x = + 0,687 m a x = 0 m.
x 2 = 2l θ Solución.
a) Forma fácil
I 0 = M (2l ) + m(2l ) y cos θ ≈ 1 :
1 2 2
Como la ecuación del movimiento es
3 •• •
m x + b x + kx = 0


⎛M ⎞ ••
en x = + 0,687 m, x = 0
− 2k .l 2θ − 4bl 2 θ = ⎜ + m ⎟4l 2 θ
⎝ 3 ⎠ Luego:
•• • •• ••
⇒ θ+ 5 x + 125(0,687 ) = 0 ⇒ x = a = −17,18m / s 2
b k
θ+ θ =0
⎛ 3m ⎞ ⎛ 3m ⎞
⎜ + m⎟ 2⎜ + m⎟ Forma trabajosa
⎝ 3 ⎠ ⎝ 3 ⎠
•• •
x = Ae − βt cos(ωt − φ )
b k
⇒θ+ θ+ θ =0 •
2m 4m x = − Aβ e − βt cos(ωt − φ ) − Aωe − βt sen (ωt − φ )
••
x = Aβ 2 e − βt cos(ωt − φ ) + Aωβ e − βt sen (ωt − φ )
b) θ = θ 0 e − βt cos(ωt + ϕ ) + Aωβ e − βt sen (ωt − φ ) − Aω 2 e − βt cos(ωt − φ )
= A(β 2 − ω 2 )e − βt cos(ωt − φ ) + 2 Aωβe − βt sen(ωt − φ )
b k
β= ω0 = ω= β −ω2 2
0 Para t = 0
4m 4m e − βt = 1 , cos(ωt − φ ) = 1 y sen (ωt − φ ) = 0
( )
Cuando k =1500 N/m , b =40 N s/m y M =3m = 3kg,
Luego: a = A β − ω = Aω 0 ,
2 2 2
además l =1.5m y el ángulo inicial = 0,15 rad.

β=
(40) = 10 ω = (1500) = 375 k 125
4(1) ⇒ ω 02 = = = 25
0
4(1)
m 5
ω = 10 2 − 375 = 275 = 16,58
Reemplazando valores:
θ = θ 0 e −10t cos(16,58t + ϕ )
• a = 0,687(25) = 17,18m / s 2
θ = −10θ 0 e −10 t
cos(16,58t + ϕ )
− 16,58θ 0 e −10t sen (16,58t + ϕ ) b) Forma fácil

25
Movimiento Oscilatorio Hugo Medina Guzmán

Cuando el bloque primero vuelve a x = 0 m, la sucede. En este caso decimos que tenemos
componente x de la velocidad es - 2,0 m/s y la oscilaciones forzadas. Ahora hay dos frecuencias en
componente x de la aceleración es +5,6 m/s2. el problema: la frecuencia natural ω o de las
•• •
m x + b x + kx = 0 oscilaciones libres, y la frecuencia productora ω de
5(5,6 ) + b(− 2,0 ) + k (0 ) = 0 las oscilaciones forzadas

5(5,6)
5(5,6 ) + b(− 2,0 ) = 0 ⇒ b = = 14m / s
2,0
Forma trabajosa
x = Ae − βt cos(ωt − φ )

x = − Aβe − βt cos(ωt − φ ) − Aωe − βt sen (ωt − φ )
( )
••
x = A β 2 − ω 2 e − βt cos(ωt − φ ) + 2 Aωβe − βt sen(ωt − φ )
Descripción
2 Como observamos en un columpio, para mantener las
x = 0 m, v = - 2,0 m/s, a = +5,6 m/s .
oscilaciones hemos de aplicar una fuerza oscilante al
0 = Ae − βt cos(ωt − φ ) (1)
oscilador amortiguado.

− 2,0 = − Aωe − βt sen (ωt − φ ) (2)

5,6 = 2 Aωβ e − βt sen (ωt − φ ) (3)

(3) / (2)

5,6 2 Aωβ e − βt sen (ωt − φ )


=
2,0 Aωe − βt sen (ωt − φ )

5,6
⇒ β= = 1,4
4,0
Sea Fo senω ' t la fuerza oscilante aplicada,
b siendo ω’ su frecuencia angular. La ecuación del
Siendo β= movimiento será ahora
2m
∑ F = ma
⇒ b = 2mβ = 2(5)(1,4 ) = 14kg / s •
. − ky. − b y + Fo senω ' t = ma
c) En x = + 0,687 m •• •
m y + b y + ky = Fo senω ' t
kA = (125)(0,687 ) = 29,5 J
1 2 1
E=
2

2 2
En x = 0 m E 2 = mv = (5)(− 2,0 ) = 10 J
1 2 1 2

2 2
ΔE = E2 –E1 = 10 - 29,5= - 19,5 J
Trabajo realizado por la fuerza amortiguadora Expresamos la ecuación del movimiento en forma de
ecuación diferencial
OSCILACIONES FORZADAS •• • k
Las oscilaciones que hemos discutido hasta ahora son y + 2β y + ω o2 y = Fo senω ' t ω o2 =
las oscilaciones libres en las cuales el sistema se da m
una cierta energía, y dejado solo. Por ejemplo, usted b
podría empujar a un niño en un columpio hasta cierta 2β =
m
altura, después dejarlo y esperar que el movimiento La solución de esta ecuación diferencial es
termine. complicada, y se compone de la suma de dos
Pero ésta no es la única posibilidad; podríamos términos
y (t ) = Ae − βt cos(ωt + δ ) + Dsen (ω ' t + δ ' ) ,
también empujar en varias ocasiones el columpio a
cualquier frecuencia y que miramos a ver que

26
Movimiento Oscilatorio Hugo Medina Guzmán

donde D' y δ' son constantes arbitrarias que han de ω’que hace que D sea máxima, se le denomina
ajustarse a fin de satisfacer las condiciones iniciales y frecuencia de resonancia ωR.
ω' es la frecuencia del oscilador amortiguado no El valor de ω’ que hace máximo a D podemos
forzado. encontrarlo de la manera siguiente:
Pasado un tiempo suficientemente largo, tal que ∂D
bt = 0 , derivando D e igualando a cero, se
>> 1 , el primer término de la ecuación es ∂ω ' ω '=ωR
2m
prácticamente nulo y puede despreciarse frente al obtiene: ω ' = ω R = ω o2 − 2β 2
segundo término. Así, pues, la expresión:
Ae − βt cos(ω1t + δ ) se denomina solución
transitoria.
En cambio la expresión Dsen (ω ' t + δ ') se conoce
como solución estacionaria, y es la predominante
2m
siempre que se tenga t >> .
b
Para obtener las expresiones de A y δ ' , se sustituye
y = Dsen (ω ' t + δ ' ) en la ecuación diferencial, lo En la figura se muestra la respuesta en amplitud de la
que nos da: oscilación forzada, en el estado estacionario. Como
F0 / m podemos observar a partir de la fórmula o la gráfica,
D=
(ω )
la amplitud de la oscilación forzada en el estado
− ω '2 + 4 β 2ω ' 2
2 2
o estacionario disminuye rápidamente cuando la
frecuencia de la oscilación forzada ωf se hace mayor
2βω '
y tan δ ' = o menor que la frecuencia propia del oscilador ωo.
ωo2 − ω '2
En el caso ideal que no exista rozamiento, la
amplitud de la oscilación forzada se hace muy
grande, tiende a infinito, cuando la frecuencia de la
oscilación forzada ω’ se hace próxima a la frecuencia
propia del oscilador ωo.
En el caso de que exista rozamiento (β >0) la
amplitud se hace máxima cuando la frecuencia de la
oscilación forzada ω’ es próxima a la del oscilador
ωo
Los efectos de la resonancia igualmente pueden
resultar indeseables o incluso destructivos. EI
traqueteo en la carrocería de un automóvil o el
molesto zumbido en un alta voz estereof6nico se
deben casi siempre a la resonancia. Casi todos hemos
escuchado que una cantante de potente voz puede
romper el cristal al cantar a determinada frecuencia.
Igualmente conocida es la advertencia de que un
El comportamiento dependiente del tiempo real de un grupo de personas no debe marchar por un puente por
oscilador armónico amortiguado y forzado puede miedo a que la frecuencia de los pasos corresponda a
resultar muy complejo. La figura muestra la alguna frecuencia natural del mismo. Todos éstos son
respuesta de un oscilador amortiguado frente a la ejemplos de resonancia.
acción de una fuerza impulsora de frecuencia ω ′ =
½ ω o , suponiendo que el sistema está en reposo
Ejemplo 32. El extremo libre del resorte de
constante k2 empieza en t = 0 a oscilar
cuando la fuerza comienza a actuar. Obsérvese que
una vez eliminado el comportamiento transitorio, armónicamente con amplitud B y frecuencia ω
únicamente persiste el movimiento estacionario con alrededor de su posición de equilibrio “P”.
frecuencia ω ′ .

Resonancia, aplicaciones.
Resonancia
Como la amplitud D de depende de ω’, ésta puede
tomar diferentes valores, en particular, al valor de

27
Movimiento Oscilatorio Hugo Medina Guzmán

Haga el DCL del bloque y determine la ecuación aparatos sensibles. Una solución común al problema
de la vibración consiste en fijar la fuente de vibración
diferencial que gobierna el movimiento del bloque.
sobre un montaje elástico que amortigüe y absorba
Solución. los movimientos. Lo que quizás no sea tan obvio es
el hecho de que el problema puede agravarse con un
Movimiento del punto P
montaje elástico incorrecto. E1 aislamiento se
x' = Bsenω ' t consigue al disminuir la frecuencia natural del
sistema con relación a la frecuencia de la fuente
− k1 x − k 2 ( x − x') = ma vibratoria. La razón por la que esta técnica funciona
es la menor transferencia de energía cuando la
ma + (k1 + k 2 ) x = k 2 x' frecuencia de la fuerza impulsora es mucho mayor
que la frecuencia natural del sistema.
••
Hemos fundamentado completamente nuestro
m x + (k1 + k 2 ) x = k 2 Bsenω ' t análisis de la resonancia, así como de la respuesta de
un sistema al movimiento forzado, en el
•• ( k1 + k 2 ) k B
x+ x = 2 senω ' t comportamiento de una masa unida a un resorte que
m m cumple con la ley de Hooke. Sin embargo, se aplican
los mismos principios y resultados generales a otros
Ecuación que corresponde a un movimiento
sistemas oscilantes, sean mecánicos, eléctricos o de
armónico simple forzado. otro tipo.
•• F0
x + ω 02 x = senω ' t
m Ejemplo 33. Un equipo de ventilación del sistema
de calefacción y aire acondicionado de un edificio se
monta firmemente en el techo y opera en forma
CASO DEL PUENTE TACOMA continua. Las vibraciones se transmiten a la
estructura del edificio y generan niveles de vibración
inaceptables.

El puente Tacoma original era conocido como


"Galloping Gertie" debido a su balanceo,
comportamiento ondulado. Tenía una longitud de
1980 metros aproximadamente y fue abierto al tráfico Para reducir la vibración que se percibe abajo, se va a
el 1 de julio de 1940 uniendoTacoma y el puerto fijar el equipo a una placa montada sobre resortes. EI
Gig por carretera. eje del ventilador gira a 1800 rpm (revoluciones por
El puente era un diseño inusualmente ligero, los minuto) y la masa combinada de la unidad y la placa
ingenieros descubrieron, una peculiar sensibilidad a de montaje (véase la figura) es de 576 kg.
los fuertes vientos. En lugar de resistirlos, como lo
hacen la mayoría de los puentes modernos, El puente
Tacoma tendía a sacudirse y a vibrar. Esto empeoró
progresivamente debido a los fenómenos armónicos.
Cuatro meses después de la inauguración del puente,
hubo una tormenta con viento de 70 km/h en el área
alrededor del puente el 7 de noviembre de 1940. El
viento hizo sacudir puente violentamente de lado a
lado, y finalmente rompió el puente. ¿Cuál es la constante de rigidez apropiada para los
Este incidente sucedió debido a la estructura del resortes usados para soportar la placa? Suponga que
puente entró en resonancia con la vibración que se emplean cuatro resortes, uno en cada esquina.
producía el viento. Nadie murió, pues el puente había
sido cerrado debido a sacudidas anteriores. Éste es el Estrategia. El sistema de oscilación en este caso está
más conocido y estudiado de fallas por oscilación compuesto por el motor, el ventilador, la plataforma
forzada, gracias a la película y las fotografías que de montaje y los resortes. Una regla práctica a la que
registran el derrumbamiento. se recurre algunas veces establece que la frecuencia
impulsora, o perturbadora, debe ser por lo menos 3
Muchas veces necesitamos un sistema que no veces la frecuencia natural del sistema. Para muchos
transfiera eficientemente la energía. Un ejemplo es casos, resulta adecuado un factor de 5 y, en
un mecanismo para aislar de las vibraciones a condiciones críticas, resulta conveniente un factor de

28
Movimiento Oscilatorio Hugo Medina Guzmán

12 o superior. Podemos conseguir estos factores


reduciendo la frecuencia natural del sistema. Si
elegimos una proporción de 1 a 5, lo que corresponde
a una reducción en la fuerza de las vibraciones en el
edificio de más o menos 96%, la frecuencia natural
que se desea del sistema es
1 ⎛ 2π ⎞
(1800 rpm)⎜ ⎟ = 12π Hz
5 ⎝ 60s / min ⎠
Solución. Los resortes adecuados pueden elegirse
utilizando
1 1 k
f = =
T 2π m Solución.
AI resolver para la constante de resorte k, obtenemos
k = m(2πf)2 = (576 kg)(12π/s)2 = 8,18 x 105 N/m.
Esta sería la más grande constante de resorte
deseable si todas las masas se soportaran mediante un
resorte. Puesto que son cuatro en total, uno en cada
esquina de la placa de montaje, cada uno de estos
cuatro resortes tendrá una constante o rigidez de
1
4
( )
8,18 ×105 N/m = 2,05 ×105 N/m

Ejemplo 34. ¿Cuál debe ser la longitud del péndulo a) Aplicando la segunda ley de Newton al
en la figura para producir la amplitud máxima en el movimiento vertical.
∑F
carrito de 1,0 kg del carril neumático si la constante
de resorte es k = 120 N/m? y = ma y ,
− Fk − Fb + Fc senωt = ma y

⇒ − ⎛⎜ k + k ⎞⎟ y − b y + Fc senωt = m y
• ••

⎝2 2⎠
La ecuación del movimiento es
•• •
⎛k k⎞
m y + b y + ⎜ + ⎟ y = Fc senωt
⎝2 2⎠
•• •
Solución.
La amplitud máxima se alcanzará cuando el péndulo
⇒ m y + b y + ky = Fc senωt
oscile con la frecuencia de resonancia, en este caso •• • F
o y + 2 β y + ω 0 y = c senωt
2
no hay amortiguamiento, luego la frecuencia de de
m
resonancia es: ω R = ω 0
170
Donde m = = 17,3kg , b = 140 N.s/m
g k 9,8
= ⇒
L m b
mg (1,0 )(9,8) y β = =4
L= = = 0,0817 m = 8,2 cm 2m
k 120 k
k = 20000 N/m y ω0 = = 34 rad/s
Ejemplo 35. El motor en la figura está montado m
sobre dos resortes, cada uno con modulo k/2 = 10000 300 × 2π
N/m. El amortiguador tiene un coeficiente b = 140 ω'= = 31,4rad/s
N.s/m. El motor incluyendo la masa desbalanceada 60
4,5
B, pesa 170 N, y el cuerpo no balanceado B pesa 4,5 Fc = mB eω '2 = × 0,075 × (31,4) 2 = 34 N
N y está localizado a 7,5 cm. del centro del eje. 9,8
a) El motor gira a 300 rpm. Determine la amplitud y La solución de la ecuación es
el ángulo de fase (relativo a la posición de B) del y = Dsen (ω ' t + δ )
movimiento resultante.
Fc / m
Con D =
b) Determine la velocidad de resonancia y la
amplitud resultante del movimiento.
( )
ω o2 − ω ' 2 + 4ω ' 2 β 2
2

29
Movimiento Oscilatorio Hugo Medina Guzmán

2ω ' β a) Aplicando la segunda ley de Newton al


y tan δ ' = movimiento vertical.
ω o2 − ω ' 2
Reemplazando valores: ∑F y = ma y ,
34 / 17,3 − Fk − Fb = ma y
D=
(34 2
− 31,4 2 2
) + 4 × 31,4 2 × 4 2 − k ( y − y1 ) − b y = m y
• ••

= 7,8 × 10 −3 m = 7,8 mm Como y1 = e senω ' t


El ángulo de fase • ••
2 × 31,4 × 4 − k ( y − e senω ' t ) − b y = m y
tan δ = 2 = 1,48 , δ = 55,9º
34 − 31,4 2 La ecuación del movimiento es
•• •
b) La resonancia ocurre cuando m y + b y + ky = ke senωt
ω ' = ω R = ω o2 − 2 β 2 •• • ke
o y + 2 β y + ω0 y = senω ' t
2

ωR = 342 − 2 × 42 = 33,5rad / s m
Donde
33,5 × 60
ωR = = 320rpm m = 10kg , b = 100 N.s/m y β =
b
=5
2π 2m
La amplitud de resonancia es:
k
D=
Fc / m k = 1000 N/m y ω0 = = 10rad / s
(ω 2
o − ω R2 )
2
+ 4ω R2 β 2
40 × 2π
m

34 / 17,3 ω'= = 4,19rad / s


= 60
(34 2
− 33,5 2 )
2
+ 4 × 33,5 2 × 4 2 La solución de la ecuación es
= 7,3 × 10−3 m = 7,3 mm y = Dsen (ω ' t + δ )
ke / m
con D = y
Ejemplo 36. El cuerpo D de la figura tiene una masa
de 10 kg y está soportado por un resorte con una (
ωo2 − ω '2 + 4ω '2 β 2
2
)
constante de 1000 N/m. El cuerpo en la parte 2ω ' β
superior da al resorte un movimiento armónico tan δ ' =
vertical por medio de la manivela que tiene una ωo2 − ω '2
velocidad angular de 40 rpm. La longitud de la Reemplazando valores:
manivela es de 1,30 cm. 1000 × 1,3 × 10 −2 / 10
a) determine la amplitud y ángulo de fase del D=
movimiento de la masa D cuando el coeficiente de (10 2
− 4,19 2 )
2
+ 4 × 4,19 2 × 5 2
amortiguación es 100 N.s/m y cuando se desconecta
el amortiguador = 1,41× 10 −2 m = 1,41 cm
b) Determine el rango de valores de ω (si hay alguno) El ángulo de fase
que limitará al movimiento de la masa a 2 cm. 2 × 4,19 × 5
Cuando b = 0. tan δ ' = = 0,51 , δ '= 26,9º
10 2 − 4,19 2
Cuando se desconecta el amortiguador β = 0.
ke / m
Con D = y tan δ ' = 0
ωo2 − ω '2
Reemplazando valores:
1000 × 1,3 × 10 −2 / 10
D=
10 2 − 4,19 2
= 1,58 × 10 −2 m = 1,58 cm
El ángulo de fase
tan δ ' = 0 , ⇒ δ '= 0º
b) Determine el rango de valores de ω’ (si hay
alguno) que limitará al movimiento de la masa a 2
cm. Cuando b = 0.
Solución.
La resonancia ocurre cuando

30
Movimiento Oscilatorio Hugo Medina Guzmán

ω ' = ω R = 10rad / s •• •
x + 2 β x + ω 02 x = 0
La amplitud D es infinita.
El valor de D con un valor máximo de dos se Donde: 2β = 10,31 y ω02 = 137,2
encuentra con Cuya solución es
D=
ke / m
Para D = 2 cm x = Ae − βt cos(ωt − φ )
± (ωo2 − ω '2 ) Con A y φ constantes cuyos valores dependen de las
1,3 condiciones iniciales del movimiento y
⇒D= = 2 × 10− 2 m
± (10 − ω )
2 2 2
⎛ 10,31 ⎞
ω= β −ω
2 2
= ⎜ ⎟ − 137,2
se obtiene ω '1 = 5,9rad / s y ω '2 = 11,6rad / s
0
⎝ 2 ⎠
= 10,52 rad/s
Observamos que es un poco menor que la propia del
oscilador ω 0
ω 10,52
La frecuencia f = = = 1,674 Hz
2π 2π
c) Sí además actúa una fuerza sinusoidal de amplitud
10 N y frecuencia doble que la propia del oscilador
Aplicando la segunda ley de Newton al movimiento.
• ••

D tiene como valor máximo 2 cuando


∑ F = ma ⇒ − kx − b x+ F senω ' t = m x 0
La ecuación del movimiento es
5,9rad / s ≤ ω ' ≥ 11,6rad / s •• •
m x + b x + kx = F0 senω ' t
Ejemplo 37. La relación entre la fuerza aplicada a un •• • F0
resorte y el alargamiento producido (ley de Hooke) o x + 2β x + ω 02 x = senω ' t
es: F = 439 Δl (todo en SI): Si se suspende el resorte m
de un extremo y se cuelga en el otro una masa m = Donde además de los valores conocidos, tenemos
3,2 kg calcular: F0 = 10 N y ω ' = 2ω 0 = 2 (11,71) = 23,43 rad/s.
a) la frecuencia propia de las oscilaciones. La solución de la ecuación es
b) Si existe un amortiguamiento debido a una fuerza
x = Dsen (ω ' t + δ ) , la velocidad es
resistente F = -33v (velocidad) ¿cuál será la
= D ω ' cos (ω ' t + δ )
frecuencia y la ecuación diferencial del movimiento? dx
c) Sí además actúa una fuerza sinusoidal de amplitud dt
10 N y frecuencia doble que la propia del oscilador F'/ m
¿cuál es la velocidad máxima en las oscilaciones con D = y
forzadas permanentes?
Solución.
(ω 2
o − ω' )
2 2
+ 4ω ' β 2 2

a) La ley de Hooke es: F = k Δl ⇒ F = 439 Δl 2ωβ


tan δ =
Luego la constante del resorte es k = 439 N/m ω − ω '2
2
o
La frecuencia angular propia del resorte es:
Reemplazando valores:
k 439 10 / 3,2
ω0 = = = 11,7 rad/s D=
m 3,2 (11,71 2
− 23,43 2 )
2
+ 4 × 23,43 2 × 5,15 2
La frecuencia propia o natural es:
= 6,54 x 10-3 m = 6,54 mm
ω 11,7 Y la velocidad máxima es
f0 = 0 = = 1,864 Hz
2π 2π Dω ' = 6,54 × 10 −3 (23,43) = 0,153 m/s
b) La ecuación del movimiento con fuerza resistente
es: Ejemplo 38. Para estudiar el movimiento de un carro en un
• •• camino “encalaminado”, se puede usar el siguiente modelo:
− kx − b x = m x El camino se representa por una sinusoide de amplitud A y
Con k = 439 N/m, b = 33 N.s/m y m = 3,2 kg: separación entre crestas L.
• •• El carro se representa por una masa M apoyada sobre un
− 439 x − 33b x = 3,2 x resorte de constante de rigidez k y un amortiguador de
constante b (que representan a los 4 resortes y
•• •
x + 10,31 x + 137,2 x = 0 amortiguadores, realmente existentes, con el objeto de
considerar únicamente el efecto vertical). El carro avanza
Ecuación de la forma con una velocidad horizontal v constante.

31
Movimiento Oscilatorio Hugo Medina Guzmán

a) Encontrar la amplitud y frecuencia del movimiento Nota: En la solución consideramos para el amortiguador
vertical del carro. solo el efecto del movimiento de la masa, para el resorte
b) ¿A qué velocidad entrará en resonancia? consideramos el efecto del movimiento de la masa y el
producido por el calamonado. En el problema siguiente
consideraremos los dos efectos en los dos elementos.

Ejemplo 39. Para estudiar el movimiento de un carro


en un camino “encalaminado”, se puede usar el
siguiente modelo: El camino se representa por una
sinusoide de amplitud A y separación entre crestas L.
El carro se representa por una masa m apoyada sobre
un resorte de constante de rigidez k y un
amortiguador de constante b (que representan a los 4
resortes y amortiguadores, realmente existentes, con
Solución. el objeto de considerar únicamente el efecto vertical).
a) El carro tiene oscilación forzada debido al El carro avanza con una velocidad horizontal v
encalaminado de la carretera. El encalaminado le produce constante.
un movimiento vertical dado por la ecuación:
a) Encontrar la amplitud y frecuencia del movimiento
v v
y´= A sen ω´t , donde ω´= 2πf , con f = = .
vertical del carro.
λ L b) ¿A qué velocidad entrará en resonancia?

La ecuación del movimiento vertical de la masa M se Solución.


obtiene de ∑ Fy = Ma y a) El carro tiene oscilación forzada debido al
encalaminado de la carretera. El encalaminado le
• ••
produce un movimiento vertical dado por la
− k ( y − y´) − b y = M y ⇒
ecuación: y´= A sen ω´t , donde ω´= 2πf , con
•• •
M y + b y + ky = kA sen ω´t v v
f = = .
•• • kA b λ L
y + 2 β y + ω o2 y = sen ω´t , con 2β = y
M M
k
ω o2 =
M
La parte importante de la solución es la estacionaria
y = D sen(ω´t + δ ) , con
kA / M
D= y
(ω 2
o )
− ω ′2 + 4 β 2ω ′2
2

2βω ′ La ecuación del movimiento vertical de la masa M se


tan δ = 2
ωó − ω ′ 2 obtiene de ∑F y = Ma y
La amplitud del movimiento esta dada por D y la
frecuencia por ω´ d ( y − y´) d2y
− k ( y − y´) − b =M 2
v dt dt
b) Como ω R = ω o2 − 2 β 2 = 2π • • ••
L ⇒ − ky + ky '−b y + b y ' = M y
Entrará en resonancia con la velocidad
dy´ •
L ω o2 − 2 β 2 Con y´= A sen ω´t y = y ' = Aω´cosω´t :
v= dt
2π •• •
M y + b y + ky = kAsenω´t + bω ' cos ω ' t

32
Movimiento Oscilatorio Hugo Medina Guzmán

Haciendo kA = F0 senφ y bω ' = F0 cos φ , j) escribir la ecuación del movimiento completa y dar
la solución general, indicando el tiempo para el cual
Con φ = tan −1
kA
y F0 = (kA)2 + (bω ')2 la amplitud del tiempo transitorio se reduce ala
mitad.
b
•• • Solución.
M y + b y + ky = F0 senφsenω´t + F0 cos φ cos ω´t k 40
a) ω0 = = = 250 = 15,81 rad/s
•• •
m 0,16
M y + b y + ky = F0 sen (ω´t + φ ) ⇒ •• ••
b) x + ω 0 x = 0 ⇒ x + 250 x = 0
2
•• b • k F
y+ y+ y = 0 sen (ω´t + φ ) c) La energía para el oscilador si amortiguamiento
M M M para amplitud de 0,02 m.

kA = (40 )(0,02) = 0,008 N


b k 1 2 1
Con 2 β = y ωo = E=
2 2
:
M M 2 2
•• • F d) La ecuación diferencial del movimiento
y + 2 β y + ω 02 y = 0 sen (ω´t + φ ) amortiguado para es:
M • ••
La parte importante de la solución es la estacionaria − kx − b x = m x
y = Dsen (ω´t + φ + δ ) , con Con k = 40 N/m, b = 0,4 N.s/m y m = 0,16 kg:
• ••
F0 / M − 40 x − 0,4b x = 0,16 x
D= y
(ω 2
o )
− ω ′ 2 + 4 β 2ω ′ 2
2 •• •
x + 2,5 x + 250 x = 0
2βω ′
tan δ = 2 Ecuación de la forma
ωó − ω ′ 2 •• •
x + 2 β x + ω 02 x = 0
La amplitud del movimiento esta dada por D y la
frecuencia por ω´ Donde: 2β = 2,5 y ω 02 = 250
e) ¿Cuánto tendría que valer b para que el
v movimiento no fuese oscilatorio?
b) Como ω R = ω − 2 β = 2π
2 2
o El movimiento es oscilatorio cuando
L
Entrará en resonancia con la velocidad (β 2
)
− ω 02 < 0 y no es oscilatorio cuando
L ω o2 − 2 β 2 (β 2
− ω 02 )≥ 0 o β 2
≥ ω 02
v=
2π ⇒ β 2 ≥ 250 ⇒ β ≥ 15,81
De aquí b ≥ 15,81(2m ) b ≥ 5,06
Ejemplo 40. La constante elástica de un resorte es 40
N/m. Un extremo está fijo y en el otro hay una masa Como b = 0,4, realmente hay oscilación.
m = 0,16 kg. Calcular: f) Para b = 0,4 expresar la amplitud en función del
a) la frecuencia propia de la oscilación. tiempo.
b) la ecuación diferencial del movimiento sin La solución de la ecuación del movimiento es
amortiguamiento. x = Ae − βt cos(ωt − φ )
c) La energía para el oscilador si amortiguamiento
2,5
para amplitud de 0,02 m. La amplitud está dada por Ae − βt , donde β =
d) Si la masa se introduce en aceite se origina la 2
fuerza resistente viscosa F = - bv (siendo b el Ns
coeficiente de amortiguamiento y v la velocidad). = 1,25 y A depende de las condiciones iniciales.
Escribir la ecuación diferencial del movimiento m
amortiguado para b = 0,4 Ns/m. g) La frecuencia angular del movimiento es:
e) ¿Cuánto tendría que valer b para que el 2
⎛ 2,5 ⎞
movimiento no fuese oscilatorio? ω= β 2 − ω 02 = ⎜ ⎟ − 250
f) Para b = 0,4 expresar la amplitud en función del ⎝ 2 ⎠
tiempo.
= 15,76 rad/s
g) Dar la frecuencia del movimiento.
ω 15,76
h) Si b = 0,4 y además aparece una fuerza sinusoidal y la frecuencia f = = = 2,51 Hz
de amplitud 0,5 N y frecuencia doble que la propia, 2π 2π
calcular la amplitud de la oscilación y la diferencia h) Si b = 0,4 y además aparece una fuerza sinusoidal
de fase con la fuerza aplicada de amplitud 0,5 N y frecuencia
i) calcular la frecuencia de resonancia, y dar la
amplitud en este caso.
ω ' = 2ω 0 = 2(15,81) = 31,62 rad/s

33
Movimiento Oscilatorio Hugo Medina Guzmán

•• • La resonancia ocurre cuando


m x + b x + kx = F0 senω ' t ⇒
ω = ω R = ω o2 − 2β 2
•• •
0,16 x + +0,4b x + 40 x = 0,5sen31,62t ω R = 15,812 − 2 × 1,252 = 15,71rad / s
De donde: La amplitud de resonancia es:
•• •
x + 2,5 x + 250 x = 3,125sen31,62t D=
F0 / m

Ecuación de la forma x + 2β x + ω02 x =


•• • F0
senω ' t
( )
ω o2 − ω R2 + 4ω R2 β 2
2

m = 3,125
Cuya solución es
x = Dsen (ω ' t + δ )
(15,81 2
)
2 2
− 15,71 + 4 × 15,712 × 1,25 2
= 24,9 x 10-3 m
F0 / m j) escribir la ecuación del movimiento completa y dar
Con D = y
(ω 2
o − ω '2 )
2
+ 4ω ' 2 β 2
la solución general.
La ecuación completa del movimiento es.
2ω ' β x = xtransitoria + xparticular
tan δ = La solución particular es x = Dsen (ω ' t + δ )
ω o2 − ω ' 2
Y la solución transitoria es
Reemplazando valores:
3,125 x = Ae − βt cos(ωt − φ )
D=
(15,81 − 31,62 2 )2 + 4(31,62)2 (1,25)2
2 El tiempo para el cual la amplitud del tiempo
transitorio se reduce a la mitad es t '
= 4,14 x 10-3 m
A
D = 4,4 mm De tal modo que= Ae − βt '
El ángulo de fase 2
2 × 31,62 × 1,25 ln 2 0,692
tan δ = = −0,1054 , ⇒ t '= = = 0,554 s
15,812 − 31,62 2 β 1,25
δ = −6,07 0
i) calcular la frecuencia de resonancia, y dar la
amplitud en este caso.

PREGUNTAS Y PROBLEMAS

1. Un oscilador armónico simple de 5 g de masa a) Hallar la frecuencia natural angular ω0 .


tiene un período de 0,6 s y una amplitud de 18 cm.
Hallar el ángulo de fase, la velocidad y la fuerza b) Hallar la amplitud del movimiento subsiguiente si
aceleradora en el instante en que el desplazamiento se dejase de repente en libertad la masa.
del oscilador es -9 cm.
c) ¿Cuáles serán la posición y velocidad de la masa
Respuesta. Fase = 120° o 240°, v = 160 cm/s, 10 s después de haber quedado libre?
F = 0,05 N Respuesta. a) ω 0 = 115,47 rad s , b) A = 3 cm,
c) x = 0,492 cm a la izquierda de la posición de
2. Una nadadora de masa m está sobre una balanza equilibrio, d) v = - 341,66 cm.
situada en el extremo de una palanca de salto, que
ella ha puesto previamente en movimiento armónico 4. Un bloque descansa sobre una placa delgada que
simple con frecuencia angular ω y amplitud A = ym ejecuta un movimiento armónico simple vertical con
(a) ¿Cuál es la lectura de la balanza? (b) ¿En qué un periodo de 1.2 s. ¿Cuál es la máxima amplitud del
condiciones se verá lanzada la nadadora de la movimiento para el cual el bloque no se separa de la
palanca? placa?
Respuesta. Respuesta. A = 0,357
Fg = mg − mω 2 y m senωt
5. Una plataforma está realizando un movimiento
3. Una masa de 150 g situada en el extremo de un armónico simple en dirección vertical con una
resorte horizontal se ve desplazada 3 cm hacia la
amplitud de 5 cm y una frecuencia de 10 π
izquierda de la posición de equilibrio mediante una
fuerza de 60 N. vibraciones por segundo. En el punto más bajo de su
trayectoria se coloca un cuerpo sobre la plataforma.

34
Movimiento Oscilatorio Hugo Medina Guzmán

a) ¿En qué punto se separará el cuerpo de la referencia correspondiente a la oscilación horizontal.


plataforma?
b) Si el eje x representa el desplazamiento de un
b) ¿A qué altura ascenderá el cuerpo por encima del
oscilador armónico simple en unidades de la
punto más alto alcanzado por la plataforma?
amplitud A y el eje y representa su velocidad en
Respuesta. a) y = 2,5 cm, b) 1,25 cm
unidades de ωA , demostrar que el gráfico del mo-
vimiento en el plano xy es un círculo de radio
6. Un alambre de longitud l 0 se alarga en 10 −3 l 0 , unidad.

cuando se cuelga de su extremo inferior una cierta 10. Consideremos el oscilador armónico simple de 5
masa. Si se conecta este mismo alambre entre dos g de masa tiene un período de 0,6 s y una amplitud
puntos A y B, alejados l 0 y situados en el mismo de 18 cm.. a) Hallar la energía mecánica total del
plano horizontal y de su punto medio se cuelga la oscilador. b) ¿Cuál es su velocidad inicial v 0 si el
misma masa, como se ve en la figura, ¿cuál es la desplazamiento inicial es 6 cm?
depresión y en dicho punto y cuál es la tensión del Respuesta. a) E = 88,826x10-7 N
alambre?
b) v0 = 177,7 cm/s

11. En el instante t = 0 un oscilador armónico simple


con una frecuencia de 5 rad/s tiene un
desplazamiento de 25 cm y una celeridad de -10
cm/s.

l0 a) Hallar la amplitud A de la oscilación.


Respuesta. y = , tensión = 5 x peso del objeto.
20 b) ¿Cuál es su constante de fase?
c) Si existe un peso de 10 g en el oscilador, ¿cuál es
7. Una masa m se conecta a dos bandas de jebe de su energía mecánica total?
Respuesta. a) A = 25,08 cm, b) φ = 94,6º , c) E =
longitud L, cada una bajo una tensión T, como se
muestra la figura. La masa se desplaza una pequeña
distancia y en forma vertical. Suponiendo que la 78,625 x 10-7 N
tensión no cambia significativamente, demuestre que:
a) la fuerza de restitución es - (2T/L) y 12. Un oscilador armónico simple de masa 0,8 kg y
b) que el sistema presenta un frecuencia 10 3π Hz se pone en movimiento con
movimiento armónico simple con una una energía cinética inicial K 0 = 0,2 J y una
frecuencia dada por ω = 2T / mL energía potencial inicial U 0 = 0,8 J . Calcular
a) su posición inicial.
b) su velocidad inicial.
c) ¿Cuál es la amplitud de la oscilación?
Respuesta. a) x 0 = ±0,45m , b) v 0 = ±1,5m / s ,
c) A = 0,50 m,
8. Se observa que una fuerza de 0,1 N estira a una
determinada cuerda elástica en 50 mm. Se suspende 13. Se cuelga de un resorte un objeto de 1g de masa
de un extremo de la cuerda un objeto de 15 g Y se y se le deja oscilar. Para t = 0, el desplazamiento era
le hace adquirir una vibración vertical tirando hacia 43,785 cm y la aceleración -1,7514 cm/s2. ¿Cuál es
abajo de él y luego soltándolo. ¿Hasta qué punto la constante del resorte?
habrá que alargar la cuerda con el objeto colgado Respuesta. k = 0,025 N/m
para que al alcanzar el punto más alto de la vibra-
ción no exista tensión en la cuerda? 14. Una masa m cuelga de un resorte uniforme de
Respuesta. Δy = 7,5cm constante k.
a) ¿Cuál es el período de las oscilaciones del
sistema?
b) ¿Cuál sería el período si la masa m se colgase de
9. Una partícula gira con celeridad constante en una modo que:
circunferencia de radio R. (1) Estuviese sujeta a dos resortes idénticos
a) Demostrar que sus proyecciones sobre los ejes situados uno junto al otro?
horizontal y vertical (sus componentes x e y) (2) Estuviese sujeta al extremo inferior de dos
realizan movimientos armónicos simples con unas resortes idénticos conectados uno a
continuación del otro?
constantes de fase que se diferencian en π 2 . Esta
circunferencia se conoce como circunferencia de

35
Movimiento Oscilatorio Hugo Medina Guzmán

demostrar que cuando la masa suspendida se está


moviendo con una velocidad v la energía cinética del
sistema viene dada por

1⎛ M⎞ 2
K= ⎜ m + ⎟v
2⎝ 3 ⎠
Si el sistema masa-muelle realiza un movimiento
armónico simple, demostrar que tendrá un período
12 M
⎛m⎞ T m+
Respuesta. a) T0 = 2π ⎜ ⎟ , b) 0 , 2T0 3
⎝k⎠ 2 T = 2π
k
15. Una masa m descansa sobre una mesa horizontal
sin rozamiento y está unida a unos soportes rígidos 17. Si la masa de las poleas mostradas en la figura es
pequeña y la cuerda inextensible, encontrar la
mediante dos resortes idénticos de longitud l 0 sin
frecuencia natural del sistema.
deformar y constante k. Ambos resortes se estiran
hasta una longitud l considerablemente mayor que
l 0 . Los desplazamientos horizontales de m respecto
a su posición de equilibrio se denominarán x (sobre
AB) e y (perpendicular a AB).
a) Escribir la ecuación diferencial del movimiento
(es decir, la ley de Newton) que rige las oscilaciones
pequeñas en dirección x.
b) Escribir la ecuación diferencial del movimiento
que rige las oscilaciones pequeñas en dirección y
(admitir que y <<1).
c) Calcular el cociente entre los períodos de Respuesta.
oscilaciones sobre x e y en función de l y l 0 . k AkB rad
ω=
d) Si para t = 0 se deja libre la masa m desde el 4m(k A + k B ) s
punto x = y = A0 con velocidad nula, ¿cuáles son sus
coordenadas x e y en un instante posterior t?
e) Dibujar un gráfico de la trayectoria de m 18. Una placa plana P hace un movimiento armónico
resultante bajo las condiciones de la parte (d) si simple horizontal sobre una superficie sin fricción
con una frecuencia f = 1,5 Hz. Un bloque B descansa
9
l= l0. sobre la placa, como se muestra en la figura, y el
5 coeficiente de fricción estático entre el bloque y la
placa es μ = 0,60. ¿Cuál es la máxima amplitud de
oscilación que puede tener el sistema sin que resbale
el bloque sobre la placa?

12
Tx ⎛ l 0 ⎞
Respuesta. c) = ⎜1 − ⎟ , d)
Tx ⎝ l ⎠ Respuesta. A = 6,62 cm
12
⎛ 2k ⎞
x(t ) = A0 cos⎜ ⎟ t , 19. Se observó que el período de un determinado
⎝m⎠ péndulo era T = 1,002 s al nivel del mar. Cuando el

⎡ 2k (l − l 0 ) ⎤
12 péndulo se llevó a la cima de una montaña, el
período resultó ser T = 1,003 s.
y (t ) = A0 cos ⎢ ⎥ t
⎣ ml ⎦ a) ¿Qué altura tenía la montaña?
b) ¿Cómo se vería afectado por la altura un péndulo
de torsión?
⎛ h ⎞
16. Un resorte que tiene su masa M distribuida Para h << RT ⇒ g = g 0 ⎜⎜1 − 2 ⎟,
R ⎟
RT = 6
uniformemente en toda su longitud tiene colgada una ⎝ T ⎠
masa m en su extremo inferior. Si el resorte se 378,13 km
alarga uniformemente cuando el sistema oscila, Respuesta. a) h = 6,36 km, b) no, excepto en el

36
Movimiento Oscilatorio Hugo Medina Guzmán

caso de que la resistencia del aire sea más pequeña. 23. a) Una varilla homogénea delgada de longitud
l oscila alrededor de un eje horizontal que pasa por
uno de sus extremos. Hallar la longitud del péndulo
20. Un cohete que posee un empuje igual a cinco ideal equivalente y situar el centro de oscilación y el
veces su peso está equipado con un reloj de péndulo centro de percusión.
vertical. Se dispara el cohete en el instante t = 0 y se
eleva verticalmente. Después de 5 s se agota el b) Un disco macizo de radio R está oscilando con
combustible. ¿Cuál es el tiempo leído en dicho reloj una pequeña amplitud alrededor de un eje
de péndulo si un reloj semejante en el suelo marca perpendicular al plano del disco y situado a una
15 s? distancia r de su centro. ¿A qué distancia r' será
máxima la frecuencia?
Respuesta. t = 21,2 s
2 R
21. Un péndulo está constituido por una pequeña Respuesta. a) l 0 = l , b) r ' =
esfera, de dimensiones que consideramos
3 2
despreciables, cuya masa es M = 200 g, suspendida
en un hilo inextensible y sin peso apreciable, de 2 m 24. Se sujeta una masa M en el extremo de un barra
de largo. uniforme de masa M y longitud L, la cual se pivota en
a) Calcular el período para pequeñas amplitudes. la parte superior Determine las tensiones en la barra
b) Supongamos que en el momento de su máxima en el pivote y en el punto P, cuando la barra se
elongación la esfera se ha elevado 20 cm por encima encuentra en reposo. Calcule el periodo de oscilación
del plano horizontal que pasa por la posición de para pequeños desplazamientos del equilibrio y
equilibrio. Calcular su velocidad y su energía cinética determine el periodo para L = 2 m. (Sugerencia:
cuando pase por la vertical. Suponga que la masa en el extremo de la barra es una
c) Supongamos que al pasar por la vertical el hilo masa puntual.)
encuentra un clavo O' situado 1m debajo del punto de
suspensión O y normal al plano de oscilación.
Describir el movimiento posterior de la esfera.
Calcular la relación de las tensiones del hilo cuando
el péndulo alcanza sus posiciones extremas.
d) Calcular el período de este péndulo, tal como se
describe en b), para pequeñas amplitudes.

Respuesta.
4π 2(2)
T= = 2,68 s.
3 9,8

25. Un bloque cúbico de 20 cm de arista está


colgado por dos cuerdas de 15 cm de largo, como se
Respuesta. indica en la figura.
a) ¿Cuál es el período de oscilación cuando el
a) T = 2π L1 = 2π 2 = 2 2 s movimiento es paralelo al plano de la figura?
1
g 9,8
b) ¿Cuándo el movimiento es perpendicular al plano
b) v = 2 gh = 2 x9,8 x0,2 = 0,4m / s de la figura?
c) T1 = cos α = L1 − h = 2 − 0,2 = 1,12
T2 cos β L1 − 2h 2 − 2 x 0,2
2 2+2
d) T = = 2 + 1 = 2,4s
2

22. Un aro delgado y uniforme de diámetro d cuelga


de un clavo. Se desplaza un ángulo pequeño en su
propio plano y luego se le deja libre. Suponiendo
que el aro no desliza sobre el clavo, demostrar que Respuesta. a) T = 0,78s , b) T = 1,1s
su período de oscilación es el mismo que el de un
péndulo ideal de longitud d.
26. Un alambre delgado se dobla en forma de una
semicircunferencia de radio R. Se le hace oscilar en

37
Movimiento Oscilatorio Hugo Medina Guzmán

su propio plano alrededor de un eje perpendicular a


mgL + kL2
su plano y que pasa por el punto medio del alambre. Respuesta. ω=
Hallar la longitud del péndulo ideal equivalente. mL2
Respuesta. l 0 = 2 R 31. Un motor eléctrico está apoyado por 4 resortes,
cada uno de constante k como se muestra en la
27. Un semicírculo de radio R y masa m está figura. Si el momento de inercia del motor alrededor
pivotado alrededor de su centro como se muestra en del eje central de rotación es I0, encontrar la
la figura. Determinar su frecuencia natural de frecuencia natural de oscilación.
oscilación para pequeños desplazamientos.

8g
Respuesta: ω0 = rad/s
3Rπ
Respuesta.
28. Un arco circular de diámetro d se cuelga de un
k
clavo. ¿Cuál es el período de sus oscilaciones ω 0 = 2a rad/s
cuando las amplitudes son pequeñas? Io
12
⎛d ⎞
Respuesta. 2π ⎜⎜ ⎟⎟
32. a) Se cuelga una bola de acero maciza del
extremo de un alambre de acero de 2m de longitud y
⎝g⎠ radio 1 mm. La carga de rotura del acero es 1,1 x 109
N/m2. ¿Cuáles son el radio y la masa de la bola de
29. Una tabla horizontal de masa m y longitud L se mayor tamaño que puede soportar el alambre?
pivota en un extremo, y en el extremo opuesto se b) ¿Cuál es el período de las oscilaciones de torsión
sujeta a un resorte de constante de fuerza k . El de este sistema? (Módulo de cizalladucha del acero
momento de inercia de la tabla respecto del pivote es = 8 x 1010 N/m2. Momento de inercia de la esfera
1 2 2
mL . Si la tabla se desplaza un ángulo pequeño θ respecto a un eje que pasa por centro = 2 MR 5 .)
3 Respuesta. a) 22 cm radio, 360 kg b) 66 s.
de la horizontal y se suelta, demuestre que se moverá
con un movimiento armónico simple, con una
frecuencia angular dada por ω = 3k m . 33. La lenteja de un péndulo de torsión como el de la
figura es un disco de momento de inercia
desconocido I. Su período es T = 3 s. Cuando se
coloca sobre el disco un anillo delgado de 3 kg de
masa y un radio de 10 cm, de forma que el hilo de
suspensión pasa por el centro exacto del anillo, el
nuevo período de oscilación es T = 4 s. Hallar el
momento de inercia I.

30. Un péndulo de longitud L y masa M tiene


conectado un resorte de constante de fuerza k a una
distancia h por debajo del punto de suspensión.
Encuentre la frecuencia de vibración del sistema para
valores pequeños de la amplitud (θ pequeño).
(Suponga que el soporte vertical, de longitud L, es
rígido, pero de masa despreciable.) Respuesta. I = 0,0386 kg.m2

34. Un resorte de 20 cm de longitud cuelga de un


soporte fijo. Al colocarse una masa de 0,5 kg en el
extremo inferior la longitud aumenta a 25 cm. Al
poner en oscilación el sistema se observa que en el
tiempo π/0,65 segundos ejecuta 10 oscilaciones.
Analice y diga si el movimiento armónico es simple
o amortiguado. Justifique.
Respuesta.

38
Movimiento Oscilatorio Hugo Medina Guzmán

f) Si el oscilador se impulsa con una fuerza


k 0,5 × 9,8 N
ω0 = , k = = 98 , ⇒ mg cos ωt , siendo ω = 2 g / h ¿cuál es la
m 0,05 m
amplitud de la respuesta del estado estacionario?
98 rad
ω0 = = 14
12
⎛h⎞
12
⎛ 35 g ⎞
0,5 s Respuesta. b) ⎜ ⎟ , c) 3⎜⎜ ⎟⎟ , d) Q = 3,
⎝ 36h ⎠ ⎝g⎠
⎛ ⎞
La frecuencia medida es ω = 2π ⎜ 10 ⎟
= 13
rad π
⎜ ⎟ e) δ = , f) 0,90 h.
⎜ π 0,65 ⎟ s 2
⎝ ⎠
La diferencia se debe a que el movimiento es
37. Un objeto de masa 0,2 kg se cuelga de un resorte
amortiguado.
cuya constante es 80 N/m. El cuerpo se somete a una
fuerza resistente dada por - bv, siendo v su velocidad
35. Se cuelga un objeto de masa 0,2 kg de un resorte (m/s) y b = 4 N.m/s.
cuya constante es 80 N/m., Se somete el objeto a una a) Plantear la ecuación diferencial del movimiento
fuerza resistente dada por - bv, siendo v su velocidad en el caso de oscilaciones libres del sistema y hallar
en m/s. su período.
a) Plantear la ecuación diferencial del movimiento b) Se somete el objeto a una fuerza impulsora
en el caso de oscilaciones libres del sistema. sinusoidal dad F(t) = F0 sen ωt, siendo F0 = 2N y
ω = 30 rad/s. En estado estacionario, ¿Cuál es la
b) Si la frecuencia con amortiguamiento es 3 2 amplitud de la oscilación forzada?
de la frecuencia sin amortiguamiento, ¿cuál es el
valor de la constante b? π
Respuesta. b) 4 N.s/m, Respuesta. a) T = s , b) 1,3 cm
5 3

36. Se conecta un bloque de masa m él un resorte 38. Un Pontiac Grand Prix de 1550 kg se soporta
cuyo otro extremo se mantiene fijo. Existe también mediante cuatro resortes en espiral, cada uno con
un mecanismo de amortiguamiento viscoso. Sobre una constante de 7,00 x 104 N/m.
este sistema se han realizado las siguientes a) ¿Cuál es la frecuencia natural de este sistema?
observaciones: b) El automóvil vibra al rodar sobre los baches en
una autopista de concreto. Si los baches están
(1) Si se empuja horizontalmente el bloque con una
separados 18,5 m, ¿qué tan rápido se está moviendo
fuerza igual a compresión estática del muelle es
el automóvil cuando la frecuencia de los baches está
igual a h.
en resonancia con la frecuencia natural?
(2) La fuerza resistente viscosa es igual a mg si el
Respuesta. a) 2,14 Hz b) 39,6 m/s
bloque se mueve con una cierta velocidad conocida
u.
39. Un motor pequeño de velocidad variable tiene
a) Para este sistema completo (en el que se incluye
una masa de 9 kg se monta en una viga elástica tal
tanto el resorte el amortiguador) escribir la ecuación
como se muestra en la figura. El motor rota con una
diferencial que rige las oscilaciones horizontales de
masa excéntrica de 1 kg a 5 cm. del centro del eje.
la masa en función de m, g, h y u.
Cuando el motor no está funcionando, el motor y el
Responder a las siguientes preguntas en el caso de
peso excéntrico hacen desviar a la viga 1,25 cm.
que. u = 3 gh : Determine
b) ¿Cuál es la frecuencia angular de las oscilaciones a) la velocidad del sistema en la resonancia y
amortiguadas? b) la amplitud de las vibraciones forzadas cuando el
c) ¿Qué tiempo ha de transcurrir, expresado en motor está funcionando en 300 rpm.
c) ¿Sería posible reducir la amplitud de la vibración
forma de un múltiplo de h g , para que la energía forzada del motor en la parte
descienda en un factor 1 e ? b) sujetando un peso adicional al motor? ¿Si es así
qué peso se debe agregar para reducir la amplitud de
d) ¿Cuál es el valor Q de este oscilador?
la vibración a 1,25 cm?
e) Este oscilador, inicialmente en su posición de
reposo, se pone en movimiento repentinamente
cuando t = 0 mediante un proyectil de masa
despreciable, pero cantidad de movimiento no nula,
que se mueve en sentido positivo las x. Hallar el
valor del ángulo de fase δ en la ecuación
x = Ae − βt / 2 cos(ωt − δ ) que describe el
40. Un auto con amortiguadores en mal estado rebota
movimiento subsiguiente, y representar x en función hacia arriba y hacia abajo con un periodo de 1,5 s
de t para los primeros ciclos. después de pasar por un hoyo. El auto tiene una masa

39
Movimiento Oscilatorio Hugo Medina Guzmán

de 1500 kg y se soporta mediante cuatro resortes de b) La ecuación de Movimiento para cuando el


igual constante de fuerza k. Determine el valor de k. soporte A según la siguiente ley xA = xo cos(ωt).
Respuesta. k = 6580 N/m (Sugerencia: nótese que la deformación del resorte
puede expresarla como la diferencia de las
41. Un bloque de masa m está soportado por un deformaciones de sus extremos)
resorte de constante k el cual está montado sobre una c) La solución estable para el caso b.
base de peso despreciable sometida a un movimiento
armónico simple de arriba abajo A0 senωt como se
muestra en la figura. Determine el movimiento del
bloque.

Respuesta.
Respuesta. a) la frecuencia angular de las oscilaciones de la
A0ω 02 1 k
x = Asen (ω 0 t + φ ) + sen (ωt + δ ) masa m es: ω=
ω −ω2 2
0
2 m
•• k 1
k b) x + x= cos ωt
ω0 = . A, φ y δ dependen de las 4m 2m
c) x = D cos(ωt + δ )
m
condiciones iniciales.
1
k
42. En el sistema mostrado en la figura, si la masa D = 2 2m 2 , ω o =
de la polea mostrada en la figura es pequeña y la ωo − ω 4m
cuerda inextensible Encontrar:
a) La ecuación de movimiento para cuando el
soporte A no tiene movimiento alguno.

40
Movimiento ondulatorio y ondas Hugo Medina Guzmán

CAPÍTULO 3. Movimiento ondulatorio y ondas


INTRODUCCIÓN.
Existen en la naturaleza muchos fenómenos de los
cuales se dice “tienen naturaleza ondulatoria” pero
¿qué es exactamente una onda? ¿Qué propiedades
tienen? ¿Cómo se puede formalizar una expresión
matemática de un fenómeno ondulatorio? Estas y otras
cuestiones son el tema objeto de este capítulo.
No obstante, antes de entrar de lleno en lo que es una
onda y su formalismo, vamos a definir onda como:
Una onda es una perturbación física que transmite
energía, pero que no transmite materia.
En las ondas materiales las partículas concretas que
componen el material no se propagan, sino que se
limitan a oscilar alrededor de su posición de equilibrio.
No obstante cuando una onda se transmite por dicho La forma de la onda es la foto de la perturbación
material se produce una sincronización de oscilaciones propagándose, la instantánea que congela las
entre las distintas partículas componentes del medio posiciones de todas las partículas en ese instante.
que posibilita la propagación de energía. Curiosamente, la representación de las distancias de
La onda de choque de una explosión es un buen separación de la posición de equilibrio de las partículas
ejemplo. La creación súbita de calor en la explosión al vibrar frente al tiempo dan una función matemática
eleva a presión muy alta a la masa de gas de su seno que, una vez representada en el papel, tiene forma
vecindad inmediata. Esta presión se ejerce sobre el aire de onda.
que rodea el cual es comprimido e incrementado en Podemos predecir la posición que ocuparán dichas
presión. Esta presión a su vez es ejercida sobre el aire partículas más tarde, aplicando esta función
de más allá, o sea que hay una onda de presión que se matemática.
aleja de la explosión con una velocidad de 335 m/s esta El movimiento de cada partícula respecto a la posición
onda contiene la energía requerida para comprimir el de equilibrio en que estaba antes de llegarle la
aire. Esta energía rompe ventanas a grandes distancias perturbación es un movimiento oscilatorio armónico
de la explosión. Ningún material viaja, el movimiento simple.
de cualquier partícula de aire relativamente es Una onda transporta energía pero no transporta
pequeño, la perturbación es la que viaja rápidamente a materia: las partículas vibran alrededor de la posición
grandes distancias y transmite la energía de equilibrio pero no viajan con la perturbación.
Veamos un ejemplo: la onda que transmite un látigo
DEFINICIÓN - CARACTERÍSTICAS. lleva una energía que se descarga al golpear su punta.
Una onda es una perturbación que se propaga desde el Las partículas del látigo vibran, pero no se desplazan
punto en que se produjo hacia el medio que rodea ese con la onda.
punto.
Las ondas materiales (todas menos las Pulso y tren de ondas – Onda viajera
electromagnéticas) requieren un medio elástico para El movimiento de cualquier objeto material en un
propagarse. medio (aire, agua, etc.) puede ser considerado como
El medio elástico se deforma y se recupera vibrando al una fuente de ondas. Al moverse perturba el medio que
paso de la onda. lo rodea y esta perturbación, al propagarse, puede
La perturbación comunica una agitación a la primera originar un pulso o un tren de ondas.
partícula del medio en que impacta, este es el foco de Un impulso único, una vibración única en el extremo
las ondas y en esa partícula se inicia la onda. de una cuerda, al propagarse por ella origina un tipo de
La perturbación se transmite en todas las direcciones onda llamada pulso. Las partículas oscilan una sola
por las que se extiende el medio que rodea al foco con vez al paso del pulso, transmiten la energía y se
una velocidad constante en todas las direcciones, quedan como estaban inicialmente. El pulso sólo está
siempre que el medio sea isótropo (de iguales un tiempo en cada lugar del espacio. El sonido de un
características físico-químicas en todas las disparo es un pulso de onda sonora.
direcciones). Si las vibraciones que aplicamos al extremo de la
Todas las partículas del medio son alcanzadas con un cuerda se suceden de forma continuada se forma un
cierto retraso respecto a la primera y se ponen a vibrar, tren de ondas que se desplazará a lo largo de la
recuerda la ola de los espectadores en un estadio de cuerda, esto viene a ser una onda viajera.
fútbol.
TIPOS DE ONDAS:
Podemos establecer criterios de clasificación de las
ondas. Algunos serían:

1
Movimiento ondulatorio y ondas Hugo Medina Guzmán

Según el medio por el que se propaguen produce de arriba a abajo, es decir, perpendicularmente
Ondas mecánicas. Son las que requieren un medio a la propagación
material para propagarse. Ejemplo, el sonido
La onda de sonido ordinario es una forma de
transmisión de energía, perturbaciones en el aire entre
fuente vibrante que es la que produce el sonido y un
receptor tal como el oído. El sonido también puede
transmitirse en los líquidos y en los sólidos. Las ondas
en una cuerda, en un resorte y las ondas de agua son
otros ejemplos de ondas que necesitan de un medio
elástico para propagarse. A este tipo de ondas se los
denomina “ondas mecánicas”.
Ondas electromagnéticas. Son las que no requieren
un medio material. Ejemplo, la luz.
Existe otro tipo de ondas relacionada con la luz,
transmisión de radio y radiación de calor, esto es las
ondas electromagnéticas que no necesitan de un medio
para propagarse.

Según el número de dimensiones que involucran


Unidimensionales. Ejemplo, la propagación del
movimiento en una cuerda

Bidimensionales. Ejemplo, olas en la superficie de un


líquido.

Longitudinales. En este tipo la propagación es


paralela a la oscilación. Como ejemplo, si apretamos
un resorte las espiras oscilan de izquierda a derecha y
de derecha a izquierda, paralelas en cualquier caso a la
dirección de propagación.

Tridimensionales. Ejemplo, el sonido normal.

Según la relación entre la vibración y la dirección


de propagación
Transversales. Son aquellas ondas en las cuales la
oscilación es perpendicular a la dirección de
propagación de la onda. Por ejemplo en una cuerda
normal y tensa la onda se propaga de izquierda a
derecha (en cierto caso particular) pero, en cambio, la
oscilación de un punto concreto de la cuerda se

2
Movimiento ondulatorio y ondas Hugo Medina Guzmán

En la Figura arriba pueden apreciarse dos etapas del


EXPRESIÓN MATEMÁTICA PARA UNA ONDA movimiento de un pulso en una cuerda, a dos tiempos
VIAJERA. diferentes, cuando el pulso se propaga de izquierda a
En la Figura (Physical Science Study Committee, derecha con velocidad v. La figura está dibujada sobre
1965) se muestra una secuencia de fotografías de un un sistema de ejes coordenados de modo que el eje x
pulso propagándose de izquierda a derecha a lo largo muestra la dirección en que la cuerda no se distorsiona.
de un resorte. En esta sección haremos uso de estas Supongamos que la forma de la cuerda a t = 0 está
fotografías para descubrir la expresión matemática de dada por la expresión f ( x ) (Figura a). Después de un
una onda viajera y probar el significado de algunos de
tiempo t el pulso ha avanzado hacia la derecha una
los términos utilizados para describir las ondas.
distancia vt (Figura b). Debe notarse que la función
f ( x − a ) tiene la misma forma que la función
f ( x ) , sin embargo f ( x − a ) esta desplazada una
distancia a en la dirección +x. Si suponemos que el
pulso mantiene su forma mientras se propaga,
podemos expresar la forma del pulso en un instante de
tiempo t mediante
y ( x, t ) = f ( x − vt )
Una descripción similar a la anterior, nos proporciona
la expresión de un pulso que se mueve hacia la
izquierda con velocidad v
y ( x, t ) = f ( x + vt )

Se denomina función de onda a la función y ( x, t ) que


sirve para describir onda. Para el caso de una onda en
una cuerda, la función de onda representa la
coordenada y de un elemento de la cuerda. Por tanto, la
función de onda da el desplazamiento y de dicho
elemento desde su posición de equilibrio y = 0, pero es
una función que depende de x y de t.

Esto significa que el desplazamiento de un elemento de


cuerda depende de:
El intervalo de tiempo entre cada fotografía es el a) la coordenada x del elemento; y
mismo. Estas fotografías indican que la velocidad de b) el tiempo t de la observación.
Esto es, x y t deben aparecer combinados en y ( x, t )
un pulso es constante; y la forma del pulso
prácticamente no cambia durante el movimiento de
avance. Un examen más minucioso muestra que el como ( x − vt ) o ( x + vt ) . Para especificar una
pulso se va haciendo gradualmente más ancho función de onda debemos escribirla como una
conforme avanza; la altura del pulso se va haciendo determinada función. Así por ejemplo la función de
menor mientras el ancho del pulso crece. Este onda específica que vamos a discutir en la sección
siguiente es y ( x, t ) = Asen ( x − vt ) .
ensanchamiento del pulso es una consecuencia de la
dispersión. La dispersión no tiene un interés primordial
en las ondas que deseamos considerar, por lo que la
ignoraremos en nuestro estudio.
Ejemplo 1. De las funciones que se presentan a
continuación, sólo dos pueden representar ecuaciones
de onda, de ondas unidimensionales que se propagan
en el eje x:
5 × 10−2
y1 ( x, t ) =
[0,25 + ( x − 2t )
2
]
−2
5 × 10
y2 ( x, t ) =
[ (
0,25 + x 2 + 4t 2 − 2t )]
−2
5 × 10
y3 ( x, t ) =
[
0,25 + (2 x + t )
2
]

3
Movimiento ondulatorio y ondas Hugo Medina Guzmán

a) Decir cuales de las funciones: y1 , y 2 e y 3 son significado especial. Reemplazando el valor de x por
⎛ 2π ⎞
funciones de onda y justificar la respuesta.
⎜x + ⎟ , obtenemos para y ( x, t ) , el mismo
b) ¿Cuáles son las velocidades de propagación de ⎝ k ⎠
dichas ondas? valor; esto es,
c) En la figura se representan varias “fotografías” de
una cuerda tensa, en la cual se está propagando una ⎛ 2π ⎞ ⎡⎛ 2π ⎞ ⎤
y⎜ x + , t ⎟ = Asenk ⎢⎜ x + ⎟ − vt ⎥
onda que corresponde a una de las dos anteriores. Las ⎝ k ⎠ ⎣⎝ k ⎠ ⎦
“fotografías” corresponden a instantes separados 0,01
s. ¿A cuál de las ondas corresponden las “fotos”? = Asenk [( x − vt ) + 2π ]
= Asenk ( x − vt ) = y ( x, t )

Observamos que es el “periodo de espacio” de la
k

curva, repitiéndose cada , cantidad la llamaremos
k
longitud de onda y la designaremos por λ .

Entonces λ=
k
Para un determinado tiempo

Observamos que la ecuación (1) también puede ser


escrita en la forma
Solución y ( x, t ) = Asen (kx − kvt ) = Asen (kx − ωt )
a) Cualquier perturbación que obedece en todo instante ω
a la ecuación: y ( x, t ) = f ( x ± vt ) representa una Donde la frecuencia angular ω = kv y v=
k
La función y ( x, t ) es también periódica en el tiempo,
onda unidimensional que se propaga hacia la derecha
(signo negativo) o hacia la izquierda (signo positivo)
del eje x , con velocidad v. Así pues, las funciones yl e 2π
y3 son las únicas posibles representantes de ecuaciones con un periodo T =
ω
de onda.
ω
b) Para y1, el valor de la velocidad será v1 = 2m / s , Y por lo tanto, con una frecuencia f =
hacia la derecha del eje x . 2π
Para y3, la transformamos en: Para un determinado espacio x.

5 × 10 −2 1
y3 = ⇒ v3 = − m/s , hacia
2
⎛ 1 ⎞ 2
0,25 + 4⎜ x + t ⎟
⎝ 2 ⎠
la izquierda del eje x . .
c) Corresponde a y1 puesto que su propagación es
hacia la derecha del eje x , y además, es claro que su Podemos obtener una relación importante de las ondas.
velocidad es 2 m/s, lo que se deduce de las medidas λ
dadas en las fotografías sucesivas. v= = λf , expresión que concuerda con
T
ω 2πf
ONDAS ARMONICAS v= = = λf
Un caso especialmente interesante y frecuente es aquel k 2π λ
en que y es una función sinusoidal o armónica tal
como y ( x ) = Asenkx , de modo que También es frecuente escribir la ecuación de la onda
y ( x, t ) = Asenk ( x − vt ) (1) sinusoidal en la forma:
⎛x t ⎞
La cantidad k conocida como número de onda y = Asen 2π ⎜ − ⎟ ⇒ y = Asen (kx − ωt )
(diferente a la constante k del resorte) tiene un ⎝λ T ⎠
4
Movimiento ondulatorio y ondas Hugo Medina Guzmán

Onda que viaja a la izquierda. Similarmente para b) La separación entre las crestas de 2 olas
una onda que viaja a la izquierda se tendría
consecutivas es una longitud de onda:
⎛x t⎞
y = Asen 2π ⎜ + ⎟ ⇒ y = Asen (kx + ωt ) 450
⎝λ T ⎠ λ= = 7,50 m
60
Función sinusoidal desfasada con respecto al
origen. Adicionalmente, podemos tener una función Ejemplo 3. Una onda sinusoidal es enviada a lo largo
sinusoidal desfasada con respecto al origen de de una de un resorte, por medio de un vibrador fijo en
coordenadas, esto es, uno de sus extremos. La frecuencia del vibrador es 20
y ( x ) = Asen (kx − ϕ ) ciclos por segundo y la distancia entre puntos de
y la onda viajera será mínimo sucesivos en el resorte es 24 cm. Encontrar:
y ( x, t ) = Asen (kx − ωt − ϕ ) a) La velocidad de la onda
Similarmente para una onda que viaja hacia la b) La ecuación de la onda, sabiendo que el
izquierda se tendrá desplazamiento longitudinal máximo es de 4 cm. y que
se mueve en el sentido positivo de x.
y ( x, t ) = Asen (kx + ωt − ϕ ) Solución.
Nota. Una onda real no puede ser perfectamente a) Si f = 20 Hertz y λ = 24 cm.
armónica, puesto que unas ondas armónicas se la velocidad es
extienden hacia el infinito en ambos sentidos a lo largo v = λf = 24 x 20 = 490 cm/seg.
del eje x y no tienen ni principio ni fin en el tiempo.
Una onda real debe tener principio y fin en algún lugar b) La ecuación de la onda que se mueve en el sentido
del espacio y del tiempo. Las ondas existentes en la positivo es
naturaleza, como son las ondas de sonido o las ondas y = Asen (kx − ωt )
de luz, pueden frecuentemente aproximarse a ondas Siendo
armónicas, puesto que su extensi6n en el espacio es 2π π
mucho mayor que su longitud de onda, y el intervalo A = 4cm, k = = y
de tiempo que tardan en pasar por un punto es mucho λ 12
mayor que su período. Una onda de este tipo se 2π
denomina tren de ondas. Así que una onda armónica es ω= = 2πf = 40 π
T
una representación idealizada de un tren de ondas.
Luego la ecuación de la onda es
Ejemplo 2. Un veraneante que descansa en la playa ⎛ x ⎞
y ( x ,t ) = 4sen 2π ⎜ − 20t ⎟
observa que durante los últimos 30 minutos han ⎝ 24 ⎠
arribado 90 olas a la orilla. Luego se mete al mar y se y en cm x en cm y t en segundos.
dirige nadando hacia un bote anclado y ubicado a 450 Corno la variable x aparece en la expresión con signo
m mar adentro, tomándole un total de 5 minutos en opuesto a la variable t, la onda se propaga en la
llegar. En el trayecto el nadador sorteo 60 olas. dirección + x.
Determine
a) La velocidad con que las olas se acercan a la orilla Ejemplo 4. a) Una onda en una cuerda esta descrita
es:
b) La separación entre crestas de 2 olas consecutivas. por y = 0,002sen (0,5 x − 628t ) . Determine la
Solución. amplitud, la frecuencia, periodo, longitud de onda y
velocidad de la onda.
Si en 30 minutos llegan 90 olas a la orilla, la b) Una onda en una cuerda esta descrita
frecuencia de las olas es: [ ]
por y = 25sen 1,25π x − 0,40π t en el sistema
90 1 c cgs. Determine la amplitud, la frecuencia, periodo,
f = = longitud de onda, la velocidad de propagación y la
30 × 60 20 s velocidad transversal de la onda.
Si hay 60 olas en 450 metros la longitud de onda de las Solución.
a) La ecuación de la onda es
olas es: y ( x, t ) = Asen (kx − ωt )
450 A = 0,002 m ,
λ= = 7,50 m
60 2π
k= = 0,5 ⇒ λ = 12,6 m
a) La velocidad con que las olas se acercan a la orilla. λ
1 2π
v = λ f = 7,50 × = 0,375 m/s ω= = 628 ⇒ T = 0,001 s
20 T

5
Movimiento ondulatorio y ondas Hugo Medina Guzmán

f =
1
= 100 Hz ⎛ t + 1 t ⎞ 2π π
ϕ = 2π ⎜ − ⎟= = = 30°
T ⎝ 6 6⎠ 6 3
m c) En este caso, la diferencia de fase viene dada por
v = λf = 1260
s x 2 − x1 210 7 7π
ϕ = 2π = 2π = 2π = = 31°
b) La ecuación de una onda armónica, en general, es λ 240 8 4
t x d) Sabemos que
y = Asen (kx − ωt ) = Asen 2π ( + )
λ T ⎛t x ⎞
3 = 4 cos 2π ⎜ + ⎟
⎝ 6 240 ⎠
La ecuación dada en el problema se puede poner de la
⎛t x ⎞ 3
forma siguiente ⇒ cos 2π ⎜ + ⎟=
⎝ 6 240 ⎠ 4
El desplazamiento 2 segundos más tarde será
⎡ ⎤
⎢ x ⎛t +2 x ⎞
t ⎥ y = 4 cos 2π ⎜ + ⎟
y = 25sen 2π ⎢ − ⎥ ⎝ 6 240 ⎠
⎢ 2 1 ⎥ = 4 cos 2π ⎛⎜ t + x + 1 ⎞⎟
⎢⎣ 1,25 0,40 ⎥⎦ ⎝ 6 240 3 ⎠
= 4 cos ⎡2π ⎛⎜ t + x ⎞⎟ + 2π ⎤
⎢ ⎥
Identificando ambas ecuaciones tenemos: ⎣ ⎝ 6 240 ⎠ 3 ⎦
Amplitud A = 25 cm = 4⎡cos 2π ⎛ t + x ⎞ cos 2π − sen 2π ⎛ t + x ⎞sen 2π ⎤
⎢ ⎜ ⎟ ⎜ ⎟ ⎥
2 ⎣ ⎝6 240 ⎠ 3 ⎝6 240 ⎠ 3 ⎦
Longitud de onda λ = = 1,6 cm Pero
1,25
⎛t x ⎞ 3
1 cos 2π ⎜ + ⎟= y
Frecuencia f = = 0,40 Hz ⎝ 6 240 ⎠ 4
T
Velocidad de propagación ⎛t x ⎞ 9 7
sen 2π ⎜ + ⎟ = 1− =
λ ⎝ 6 240 ⎠ 16 4
v= = 0,64 cm/s
T Sustituyendo valores
La velocidad transversal será ⎡3 ⎛ 1 ⎞ 7 3⎤
dy y = 4⎢ ⎜ − ⎟ − ⎥ = -3,79 cm
vt = = 25 × 0,8π cos π (1,25 x − 0,80t ) ⎣4 ⎝ 2 ⎠ 4 2 ⎦
dt
= 20π (1,25 x − 0,80t ) cm/s Ejemplo 6. Una onda sinusoidal que viaja en la
dirección positiva x tiene una amplitud de 15 cm, una
Ejemplo 5. Un foco puntual realiza un movimiento longitud de onda de 40 cm y una frecuencia de 8 Hz.
periódico representado por la ecuación. El desplazamiento de la onda en t = 0 y x = 0 es 15
Las unidades están en el sistema cgs. cm
⎛t x ⎞ a) Determinar el número de onda, el período, la
y = 4 cos 2π ⎜ + ⎟ frecuencia angular y la rapidez de onda.
⎝ 6 240 ⎠ b) Determinar la constante de fase ϕ, y se escribirá una
Se pide determinar: expresión general para la función de onda.
a) La velocidad de la onda. Solución.
b) La diferencia de fase para dos posiciones de la a) Utilizando las ecuaciones estudiadas obtenemos:
misma partícula cuando el intervalo de tiempo 2π 2π
transcurrido es de 1 s k= = = 0,157 / cm
c) La diferencia de fase, en un instante dado, de dos λ 40
partículas separadas 210 cm. 1 1
d) Si el desplazamiento, y, de una determinada T = = = 0,125 s
partícula en un instante determinado es de 3 cm,
f 8
determinar cuál será su desplazamiento 2 s más tarde ω = 2πf = 2π (8) = 50,3 rad/s
Solución. v = λf = (40 )(8) = 320 cm/s
a) La velocidad de propagación de la onda es:
b) Puesto que la amplitud A = 15 cm, y como se tiene
λ
240 cm
v= = = 40 y = 15 cm en x = 0 y t = 0, obtenemos
T 6 s 15 = 15sen (− ϕ ) ⇒ sen (− ϕ ) = 1
La velocidad es de sentido contrario al positivo del eje
Esto puede comprobarse por simple observación
x. puesto que la función coseno está desplazada 90º
b) La diferencia de fase es

6
Movimiento ondulatorio y ondas Hugo Medina Guzmán

respecto de la función seno. Sustituyendo los valores a) Si a un período T le corresponde una diferencia de
de A, k y ω en esta expresión, se obtiene fase 2π:
y = 15 cos(0,157t − 50,3 x ) cm a Δt le corresponde una diferencia de fase Δϕ
2πΔt 2π 5 × 10 −4
Δϕ = = = π rad
Ejemplo 7. La ecuación de una onda armónica que se T 10 −3
propaga en una cuerda es b) Si a una longitud de onda λ le corresponde una
diferencia de fase 2π:
y = 25sen (1,25πx − 0,8πt ) a Δx le corresponde una diferencia de fase Δϕ
2πΔx
2π 2,75 × 10−2 π
Donde x se expresa en cm y t en segundos. Δϕ = = = rad
λ 33 × 10− 2 6
a) Determinar cual es el desfase para dos partículas de
la soga posicionadas en 2cm y 30cm λΔϕ 0,33 × π / 6
c) Δx = = = 0,11 m
b) Cual es la distancia mínima entre 2 partículas del 2π 2π
medio cuyo desfase es de π/3.
Solución. Ejemplo 9. Sometemos al extremo de una cuerda
a) y = 25sen (2,5π − 0,8πt ) = 25 cos 0,8πt tensa a vibraciones sinusoidales de 10Hz. La mínima
distancia entre dos puntos cuyas vibraciones tienen una
y = 25sen (37,5π − 0,8πt ) = − 25 cos 0,8πt
diferencia de fase π / 5 es de 20 cm, calcular:
El desfase es π rad a) La longitud de onda.
b) La velocidad de propagación.
Solución.
a) Si la diferencia de fase para dos puntos separados 20
cm es π / 5 , a diferencia de fase para una longitud de
onda λ es 2π.

Luego λ= 20 = 200 cm = 2 m
El desfase entre esos dos puntos en todo instante será π 5
igual a π rad.
b) La velocidad de propagación
π v = λf = 2m x 10s-1 = 20 m/s
b) 1,25πx 2 − 1,25πx1 = ⇒
3
1 1 Ejemplo 10. Una onda tiene por ecuación:
x 2 − x1 = = = 0,27 cm y ( x, t ) = 5senπ (4 x − 20t + 0,25) , expresada en el
3(1,25) 3,75
sistema CGS. Determinar la amplitud, la frecuencia, la
Otra forma longitud de onda, el número de onda, la frecuencia
Si 2π corresponde a 1,6 cm., cuando corresponde a angular, la fase inicial y la velocidad de propagación.
π Solución
: La ecuación general de la onda es:
3
π y ( x, t ) = yosen (kx − ωt + ϕ )
1,6 × ϕ ⎞
3 = 1,6 = 0,27 cm ⎛x t
d= = yosen 2π ⎜ − + ⎟
2π 6 ⎝ λ T 2π ⎠
que comparada con la dada:
Ejemplo 8. La velocidad de propagación de una onda ⎛ 1⎞
es de 330 m/s, y su frecuencia, y ( x, t ) = 5sen 2π ⎜ 2 x − 10t + ⎟
103 Hz. Calcúlese: ⎝ 8⎠
a) La diferencia de fase para dos posiciones de una 1
misma partícula que se presentan en intervalos de resulta: y0 = 5 cm , T = s
tiempo separados 5 x 10-4 s. f
b) La diferencia de fase en un determinado instante 1 1
entre dos partículas que distan entre sí ⇒ f = = 10 Hz , λ = cm ,
2,75 cm.
T 2
c) La distancia que existe entre dos partículas que se π
k = 4 cm −1 , ω = 20π r rad/s , ϕ = rad ,
encuentran desfasadas 120°. 4
Solución.
1
v 330 1 v = λf = × 10 = 5 cm/s
λ = = 3 = 0,33m , T = = 10− 3 s 2
f 10 f

7
Movimiento ondulatorio y ondas Hugo Medina Guzmán

Ejemplo 11. Sometemos al extremo de una cuerda a ω1 cm 8π


un vibrador que le produce una onda sinusoidal. Si la c) v1 = = , =8
ecuación de la vibración escrita en el sistema k1 π s
y = 5sen 0,2πt , propagándose en la cuerda con una ω 5π 5 cm
v2 = 2 = =
velocidad de 10 cm/s. Determine la ecuación de la k2 4π 4 s
onda producida.
d) Para x = 150 cm, obtenemos:
v y1 (150, t ) = 32π cos(8πt − 150π )
Solución.
La ecuación de la onda que se propaga el sentido
negativo del eje OX es: = 32π cos 8πt
⎛x t ⎞
y ( x, t ) = y0sen 2π ⎜ + + ϕ ⎟ si v y1 es máxima, entonces:
⎝λ T ⎠ n
⎛t ⎞ cos 8πt = ±1 ⇒ 8πt = nπ ⇒ t = s
⇒ y ( x, t ) = y0sen 2π ⎜ + ϕ ⎟ 8
⎝T ⎠ En v y 2 será:
Comparando con la dada: y (0, t ) = 5sen 0,2πt
v y 2 (150, t ) = −30π cos(600π − 5πt )

y0 = 5 cm , = 0,2π → T = 10 s , ϕ = 0 = − 30π cos 5πt
T En el máximo:
Además como n
λ = vT → λ = 10 × 10 = 100 cm cos 5πt = ±1 ⇒ 5πt = nπ ⇒ t = s
De aquí 5
e) Para t = 0, entonces: v y1 ( x,0 ) = 32π cos πx
⎛ x t ⎞
y ( x, t ) = 5sen 2π ⎜ + ⎟ y para que sea máxima:
⎝ 100 10 ⎠
cos πx = ±1 ⇒ πx = nπ ⇒ x = n
Ejemplo 12. Las ecuaciones de dos ondas escritas en Para v y 2 , será: v y 2 ( y,0 ) = −30π cos 4πx
el sistema CGS vienen dadas por: y para que sea máxima:
y1 (x, t ) = 4sen 2π (4t − 0,5 x ) e n
cos 4πx = ±1 ⇒ 4πx = nπ ⇒ x =
y2 (x, t ) = 6sen (4πx − 5πt ) 4
Calcular en cada caso:
a) Velocidad en función del tiempo, de un punto Ejemplo 13. Sometemos al extremo de una cuerda
situado a 10 cm del foco. tensa a un vibrador que le produce vibraciones
b) Velocidad máxima de ese punto. sinusoidales. Por este efecto se propaga por la cuerda
c) Velocidad de fase. una onda transversal que tiene por ecuación:
d) ¿En qué instante alcanza su velocidad máxima un y ( x, t ) = 10 senπ (1,6 x − 0,8t ) , expresada en el
punto situado a 1,5 m del foco? sistema CGS.
e) Posición de los puntos que tienen velocidad máxima a) ¿Qué condiciones iniciales nos determinan esta
en t = 0. ecuación de onda?
Solución. b) Determínese para esta onda su amplitud, velocidad
y1 (x, t ) = 4sen (8πt − πx ) , de propagación y longitud de onda.
y2 (x, t ) = 6sen (4πx − 5πt ) c) Tiempo que tarda en comenzar a vibrar una
partícula de la cuerda situada a 10 cm del extremo en
∂y
a) v y1 ( x, t ) = 1 = 32π cos(8πt − πx )
que se encuentra el vibrador y ecuaciones horarias del
∂t movimiento de el1a [ y (t ) , v(t ) , a (t ) ] una vez
∂y 2 transcurrido éste.
v y 2 ( x, t ) = = −30π cos(4πx − 5πt ) d) Dibujar la forma que tiene la cuerda [ y (t ) ] cuando
∂t
Cuando x = 10 cm, entonces: han transcurrido 5,625 s del comienzo de la vibración
v y1 (10, t ) = 32π cos(8πt − 10π ) = 32π cos 8πt (perfil de la onda).
Solución.
v y 2 (10, t ) = −30π cos(40π − 5πt ) = a) Si hacemos x = 0 y t = 0, tendremos:
− 30π cos 5πt y (0,0 ) = 10 sen 0 = 0
cm ∂y
b) En valor absoluto: v y1 max = 32π v ( x, t ) = = −8π cos π (1,6 x − 0,8t )
s ∂t
cm ⇒ v(0,0 ) = −8π < 0
v y 2 max = 30π La ecuación dada nos determina que en el extremo de
s
la cuerda en que se encuentra al vibrador x = 0 y para

8
Movimiento ondulatorio y ondas Hugo Medina Guzmán

t = 0 es cuando comienza a actuar el vibrador con lo que quiere decir es que a partir de esta distancia la
movimiento vibratorio armónico dirigido hacia abajo cuerda se encuentra en reposo, con lo que la gráfica
(en el sentido negativo del eje y. La onda se propaga en (forma de la cuerda en ese instante) será la de
la dirección positiva del eje x.
b) Como la ecuación general de una onda sin fase
inicial ( y = 0 ) es:
⎛x t⎞
y (x, t ) = y 0 sen 2π (kx − ωt ) = y 0 sen 2π ⎜ − ⎟
⎝λ T ⎠
Comparándola con la dada:
y ( x, t ) = 10 senπ (1,6 x − 0,8t ) La ecuación es
= 10 sen 2π (0,8 x − 0,4t ) y ( x ) = 0 ⇒ 2π (0,8 x − 2,25) = nπ ⇒
De aquí n + 4,5
x=
1 1,6
y0 = 10 cm , λ = = 1,25cm ,
0,8 Hay cinco valores de x para y ( x ) = 0 .
1 1 x0 corresponde a n = 0 ⇒
T= = 2,5s , f = = 0,4Hz ,
0,4 T 0 + 4,5
x0 = = 2,8125cm
v = λf = 1,25 × 0,4 = 0,5cm/s 1,6
c) La partícula comenzará a vibrar transcurrido un x-1 corresponde a n = -1 ⇒
tiempo t, tal que: − 1 + 4,5
x 10 x−1 = = 2,1875cm
x = vt ⇒ t= = =20 s 1,6
v 0,5
Pasado éste, la partícula comienza a vibrar con ⇒
x-2 corresponde a n = -2
movimiento armónico de ecuación: − 2 + 4,5
x = 10cm ⇒ y (t ) = 10 sen 2π (8 − 0,4t ) x− 2 = = 1,5625cm
1,6
Luego:

v(t ) =
dy
= −8π cos 2π (8 − 0,4t ) ⇒
x-3 corresponde a n = -3
dt − 3 + 4,5
x− 3 = = 0,9375cm
dv d 2 y 1,6
a(t ) = = 2 = −6,4π 2 sen 2π (8 − 0,4t )
dt dt
Obsérvese que el origen de las elongaciones para este x-4 corresponde a n = -4 ⇒
movimiento vibratorio armónico se encuentra a 20 s − 4 + 4,5
del comienzo de la actuación del vibrador. El signo x− 4 = = 0,3125cm
menos de la velocidad nos indica que comienza a
1,6
moverse hacia abajo (sentido negativo del eje y), y, por
tanto, la partícula se encuentra en fase con el vibrador. VELOCIDAD DE PROPAGACIÓN EN FUNCIÓN
(El tiempo 20 s = 8 T nos indica que han transcurrido 8 DE LAS PROPIEDADES DEL MEDIO.
períodos y, por tanto, la partícula se encuentra a Forma simple de calcular la velocidad de la onda en
8 λ = 10 cm de distancia del origen, y la forma de la una cuerda en función de las propiedades del
cuerda hasta esa partícula será 8 “bucles” hacia abajo medio. Supongamos que tenemos una cuerda de masa
del eje y y otros tantos hacia arriba). por unidad de longitud μ , que esta estirada por una
d) t = 5,625 s ⇒ y ( x ) = 1 sen 2π (0,8 x − 2,25) fuerza de tensión T. Un pulso se propaga en la cuerda.
Intersección con eje y: x = 0 ⇒
y (0) = −10 sen 4,5π = −10cm
lo que nos indica que el vibrador se encuentra en su
máxima elongación (amplitud) y por debajo del origen.

Intersección con eje x:


El trozo de cuerda que se ha puesto en movimiento en Tomamos un pequeño elemento Δl de la cuerda se
ese tiempo será: muestra en la figura.
x = vt = 0,5 x 5,625 = 2,8125 cm, correspondiente a
λ λ
2,8125 = 2,25λ = 2λ +
1,25 4

9
Movimiento ondulatorio y ondas Hugo Medina Guzmán

Ejemplo 15. Una onda y = Asen (k1 x − ω1t ) viaja


por una cuerda de densidad de masa lineal μ y tensión
T. Diga, para cada una de las ondas que se dan a
continuación, si pueden viajar por la misma cuerda
simultáneamente con la onda dada. ¿Por qué? ¿Bajo
qué condición?
Este elemento, de longitud, en la parte más elevada de y1 = Asen (k1 x + ω 2 t )
la onda, está sujeto a la tensión de la cuerda en los dos y 2 = Asen (k 2 x + ω1t )
sentidos de propagación de la onda. Podemos dibujar
una circunferencia de radio R, en que R es la amplitud y 3 = Asen (k 2 x + ω 2 t )
de la onda. Este elemento de la cuerda, considerado
y 4 = Asen (k1 x + ω1t )
bien pequeño, está en el lado de un triángulo cuyo
ángulo opuesto está dado por Δθ . Instantáneamente, Siendo ω1 ≠ ω2 y k1 ≠ k 2
es como si este elemento de cuerda estuviese en Solución.
movimiento en una trayectoria circular de radio R, con La velocidad de propagación es única;
velocidad v; la velocidad de la onda. T ω1 ω1
Aplicando la segunda ley de Newton al segmento de v = = , por lo tanto, la relación esta
cuerda Δl μ k1 k1
Δθ Δθ determinada o fija.
∑F x = ma y ⇒ T cos
2
− T cos
2
=0
y1 . No puede viajar, se requiere:
ω2
=
ω1
, lo que
Δθ k1 k1
∑ Fy = ma y ⇒ − 2Tsen 2 = −Δmac nos lleva a una falsedad, contra lo supuesto,
ω 2 = ω1
v2 Δθ
ac = . Como es pequeño, podemos y 2 . No puede viajar, por que similar al caso anterior:
R 2
Δθ Δθ ω1 ω1
considerar sen ≈ = también nos lleva a una falsedad contra lo
2 2 k2 k1
Reemplazando: supuesto, k 2 = k1
Δθ v2 T ω2 ω1
2T = μRΔθ ⇒ T = μv 2 y v = y 3 . Si puede viajar, bajo la condición: =
2 R μ k1 k2
y 4 . Si puede viajar, por que tienen igual ω1 y k1
Obtenemos la velocidad de la onda en la cuerda en
función de las propiedades de la cuerda: su tensión y es la misma onda que viaja en sentido contrario.
su densidad lineal.
Ejemplo 16. Una cuerda de masa M y longitud l
Ejemplo 14. La cuerda Si de un mandolina tiene 0,34 cuelga del techo de una habitación.
m de largo y tiene una densidad linear de 0,004 kg/m. a) Probar que la velocidad de pulso transversal en
El tornillo de ajuste manual unido a la cuerda se ajusta función de la posición cuando se propaga a lo largo de
para proporcionar una tensión de 71,1 N. ¿Cuál ella es v = gx , siendo x la distancia al extremo
entonces es la frecuencia fundamental de la cuerda?
Solución. libre.
b) Probar que un pulso transversal recorrerá la cuerda
v 1 T 1 71,1N
f1 = = = en un tiempo 2 l g .
2 L 2 L μ 2(0,34m ) 0,004 kg m
= 196 Hz

Un instrumento de cuerda tal como una guitarra es


templada ajustando la tensión en una cuerda por medio
de un tornillo de ajuste manual. La longitud de la
cuerda es fija, así que el ajuste de la tensión da la
frecuencia fundamental. Otras frecuencias
fundamentales pueden ser alcanzadas acortando la
longitud de la cuerda presionando en un traste.
Finalmente, varias cuerdas de diversas densidades se
utilizan para dar una gama de las velocidades de la
onda, de tal modo proporcionando el acceso a una Solución.
mayor gama de frecuencias fundamentales.

10
Movimiento ondulatorio y ondas Hugo Medina Guzmán

T ∂2y
a) La velocidad del punto P es v= , la tensión T denota la aceleración vertical del elemento de
μ ∂t 2
en ese punto es debido a la cuerda que cuelga de cuerda.
longitud x, cuya masa es μx y su peso T = μgx . ⎛ ∂y ⎞ ⎛ ∂y ⎞
Como tan α 1 = ⎜ ⎟ , tan α 2 = ⎜ ⎟
μgx ⎝ ∂x ⎠1 ⎝ ∂x ⎠ 2
Luego v = = gx
μ Δx
y, Δm = μΔl = μ ≈ μΔx
b) para encontrar el tiempo de recorrido del pulso cos θ
dx dx se tendrá
v= = gx ⇒ dt = ⎡⎛ ∂y ⎞ ⎛ ∂y ⎞ ⎤
dt ∂2 y
gx μ Δx = T ⎢⎜ ⎟ − ⎜ ⎟ ⎥
∂t ⎣⎝ ∂x ⎠ 2 ⎝ ∂x ⎠1 ⎦
2
l dx l
⇒ t=∫ =2 ó
0 gx g
⎡⎛ ∂y ⎞ ⎛ ∂y ⎞ ⎤
⎢⎜ ⎟ − ⎜ ⎟ ⎥
ECUACION DE LA ONDA. ∂ 2 y T ⎣⎝ ∂x ⎠ 2 ⎝ ∂x ⎠1 ⎦
Ondas transversales en una cuerda. En esta parte =
trataremos la ecuación de la onda y su solución,
∂t 2 μ Δx
considerando el caso particular de la onda transversal
en una cuerda, resultado que es general también para Llevando al límite cuando Δx → 0, obtenemos
los demás casos.
∂ y T ∂ y
2 2
=
∂t 2 μ ∂x 2
Ecuación diferencial del movimiento.
Como la velocidad de propagación de una onda en una
T
cuerda tensa es v = , por lo que la ecuación
μ
diferencial de la onda la escribimos como:
∂2 y 2 ∂ y
2
La cuerda tiene una masa uniforme μ por unidad de = v
longitud y está sometida a una tensión T. Sobre esta ∂t 2 ∂x 2
cuerda esta viajando una onda transversal. Cuya solución es la ecuación de la onda
Consideremos un elemento de longitud (de 1 a 2) y = Asen (kx − ωt )
como se muestra en la figura, sobre este elemento comprobación
actúan dos fuerzas externas a él, que la jalan en cada
∂2 y
extremo debido al resto de la cuerda. Estas fuerzas son = − Aω 2sen (kx − ωt ) ,
de igual magnitud que la tensón de la cuerda. ∂t 2

La fuerza horizontal sobre este elemento es: ∂2 y


= − Ak 2sen (kx − ωt )
∑F x = T1 cos α 2 − T2 cos α 1 = 0 ∂x 2
si la curvatura de la cuerda no es muy grande Reemplazando
cos α 1 ≅ cos α 2 − Aω 2sen (kx − ωt ) = −v 2 Ak 2sen (kx − ωt )
de aquí concluimos que T1 ≈ T2 ≈ T ω
La fuerza vertical sobre el elemento es:
⇒ v=
k
∑ Fy = T sen α 2 − T sen α1 Expresión válida para toda onda, ya que ω k
Si los desplazamientos transversales de la cuerda no corresponde a la velocidad de propagación de la onda.
son muy abruptos, podemos considerar que, Sen
α ≅ tanα De manera similar podemos encontrar la velocidad de
Luego, propagación de la onda para:
∑F = T (tan α 2 − tan α 1 ) a) Ondas longitudinales en una barra de metal de
densidad ρ módulo de elasticidad Y.
y
Que será la fuerza total neta que actúa sobre el
elemento Δx considerado.
Y
vL =
Aplicando la segunda ley de Newton, ρ
∂ y
2

∑F y = Δma y = Δm
∂t 2
11
Movimiento ondulatorio y ondas Hugo Medina Guzmán

b) Ondas transversales en una barra de metal de T Mg


densidad ρ módulo de elasticidad cortante o de vtransversal = = ⇒
cizalladura G. μ ρπR 2

vT =
G 150 × 9,8
vtransversal =
ρ 7,8 × 10 3 × π 25 × 10 −8
c) Ondas longitudinales en un gas de densidad ρ m
módulo de compresibilidad volumétrica B. = 490
s
B
v=
ρ Ejemplo 19. Se tiene un alambre de acero de 1,3 mm
de diámetro, sabiendo que 5 m de este alambre se
alarga 0,5 mm con una carga de 2,1 kg. (densidad del
Ejemplo 17. Para el cobre el modulo de elasticidad
acero, 7,8 g/cm3)
volumétrica es 14 x 1010 N/m2 y la densidad es 8920
a) Calcule el módulo de Young en el acero.
kg/m3. ¿Cuál es la velocidad del sonido en el cobre?
b) Calcule la velocidad de propagación de una onda
Solución.
Solución.
B 14 × 1010 Donde ρ, la densidad es un valor conocido igual a 7,8
v= = = 3960 m/s g/cm3.
ρ 8920
a) El módulo de Young Y puede calcularse de
F A Fl
Ejemplo 18. A un alambre de acero (Módulo de Y= = =
Young: Y = 2,0 x 1011 N/m2, densidad del acero: ρ = Δl l AΔl
7,8 g/cm3) que tiene un diámetro de 1 mm y 4 m de (2,1 × 9,8)(5)
longitud, lo colgamos del techo, calcular:
a) El alargamiento del alambre cuando de su extremo
libre colgamos un peso de 150 kg.
[π (1,3 ×10 ) 4](0,5 × 10 )
−3 2 −3
= 15,5 x 1010 N/m2

b) La velocidad de propagación del sonido en el acero


b) La velocidad de propagación de las ondas viene dada por
longitudinales y transversales a lo largo del alambre
cuando el cuerpo está suspendido. Y 15,5 × 1010
v= = = 4458 m/s
ρ 7,8 × 10 3

Ejemplo 20. Una cuerda de piano de longitud 40 cm,


sección 0,4 mm2 y densidad 7,8 g/cm3, emite un sonido
fundamental cuando se aproxima un diapasón de
frecuencia 218 Hz.
a) Determine la tensión a que está sometida.
b) Si la tensión se multiplica por 4, ¿cómo se modifica
la frecuencia de su sonido fundamental?
Solución.
λ
Solución. a) En este caso = L ⇒ λ = 2 L =0,8 m
2
Δl F La velocidad de las ondas es: v = λ f = 0,8 x 218 =
a) = ⇒
l YA 174,4 m/s
Mgl 150 × 9,8 × 4 La velocidad de las ondas transversales en la cuerda
Δl = = tensa está dada por:
YπR 2
2 × 1012 π 25 × 10−8
= 37,4 x 10-3 m T
b) La velocidad de propagación de las ondas
v= ⇒ T = μv2
μ
longitudinales lo largo del alambre
m mA
Y 2 × 1011 m La densidad lineal es: μ = = = ρA = (7,8 x
vlongitudinal = = = 5,06 × 103 L LA
ρ 7,8 × 10 3
s 103)(0,4 x 10-6) = 3,12 x 10-3 kg/m
La velocidad de propagación de las ondas transversales Finalmente T = μ v = (3,12 x 10-3)(174,4)2 = 94,9 N
2
a lo largo del alambre
b) En este caso la velocidad de las ondas transversales
es:
4T T
v' = =2 = 2v .
μ μ

12
Movimiento ondulatorio y ondas Hugo Medina Guzmán

La longitud de onda no cambia y la nueva frecuencia


será:
v' 2v
f '= = = 2 f = 2 x 218 = 436 Hz.
λ λ
Ejemplo 21. A un resorte cuya masa es 200 g y cuya
longitud natural cuando está colgado de un punto fijo
es 4 m, se le pone una masa de 100 g unida a su
extremo libre.
Cuando esta masa se encuentra en equilibrio, la
longitud del resorte es 4,05 m. Determinar la velocidad
de propagación de las ondas longitudinales en el
resorte. A un elemento de masa Δm en el punto P se le da una
Solución. energía cinética a medida que un pulso de onda pasa
con una velocidad v .

Para el tiempo t = 0, un pequeño segmento de la cuerda


alrededor del punto P de la figura anterior, con masa
Ondas longitudinales en un resorte.
Δm y longitud Δl , está en reposo y no tiene energía
Y kl o μ cinética. El movimiento hacia arriba y hacia abajo
v= , para un resorte Y = , ρ =
ρ A A proporciona la energía requerida para iniciar el pulso a
lo largo de la cuerda. A medida que el borde que
kl o encabeza el pulso alcanza P, el segmento Δl
luego para el resorte v =
μ comienza a moverse hacia arriba. A medida que la
cresta de la onda pasa el segmento Δl , el segmento se
l 0 = 4m , Δl = l − l o = 4,05 – 4 = 0,05 m
mueve a su posición más alta y empieza de nuevo a
Mg 0,1 × 9,8 N bajar, teniendo energía cinética mientras está en
F = kΔl ⇒ k = = = 19,6 movimiento. Cuando el pulso entero ha pasado P, el
Δl 0,05 m
segmento Δl regresa al reposo y de nuevo no tiene
m 0,2 kg
μ= = = 5 × 10− 2 energía cinética. El progreso del pulso a lo largo de la
lo 4 m cuerda corresponde al flujo de energía a lo largo de la
finalmente cuerda. Otro tipo de pulso, incluyendo un pulso que
viaja a través del aire, transferiría energía a lo largo de
kl o 19,6 × 4 m la dirección de la propagación de modo similar.
v= = = 39,6
μ 5 × 10 −2
s
¿Cuánta energía se ha transferido al pasar P durante un
tiempo t ? Para una onda armónica que viaja en una
ENERGÍA E INFORMACIÓN TRANSFERIDA
cuerda, cada punto se mueve con movimiento
MEDIANTE ONDAS
armónico simple en la dirección transversal (y).
Tenemos la experiencia de energía transferida por
Como vimos anteriormente, en ausencia de
ondas en muchas situaciones. Sentimos la fuerza de
amortiguamiento, la energía total de un oscilador
una ola en el océano, nuestra piel siente el calor de las
armónico es igual a su energía potencial en el
ondas luminosas del sol, escuchamos las ondas de
sonido. Además, la mayor parte de la información que 1 2
desplazamiento máximo A , es decir, kA .
recibimos nos llega mediante ondas. El habla y la 2
música se transmiten por ondas de sonido, la radio y la También vimos que la relación entre masa, constante
televisión por ondas electromagnéticas. La luz k del oscilador (no es el número de onda k ) y
reflejada por la cual usted lee esta página es una onda.
¿Cómo depende la energía (y en consecuencia la 1 k
frecuencia es f = . Si tratamos el segmento
información) transmitida por las ondas de las 2π m
propiedades de las ondas? Para responder esta de la cuerda como un oscilador armónico con masa
pregunta antes debemos considerar cómo es transferida
Δm que se mueve a la frecuencia f, podemos
la energía por un solo pulso. Luego, ampliaremos los
acomodar la ecuación para obtener una constante de
resultados con el fin de tener una expresión para la
salto efectiva k = (2πf ) Δm . La energía asociada
2
energía de una onda armónica.
con el movimiento de este segmento de la cuerda es
entonces
1 2 1
kA = (2πf ) ΔmA 2
2
ΔE =
2 2

13
Movimiento ondulatorio y ondas Hugo Medina Guzmán

ΔE = 2π 2 Δmf 2 A 2 W/m2. La sensación de sonido más o menos fuerte


depende de la frecuencia además de la intensidad del
mismo.
Ahora tenemos un resultado importante: la energía de
una onda depende del cuadrado de la amplitud de la Ejemplo 22. Una cuerda de densidad lineal 480 g/m
onda. Así, una onda con el doble de amplitud de otra está bajo una tensión de 48 N. Una onda de frecuencia
onda equivalente (con la misma frecuencia, el mismo
200 Hz y amplitud 4,0 mm recorre la cuerda. ¿A qué
medio) tendrá energía cuatro veces mayor. razón la onda transporta energía?
Solución.
Para encontrar la rapidez del flujo de energía, o
rad
potencia, observamos que Δm se puede escribir corno ω = 2πf = 2π (200 ) = 400π
ρSΔl , donde ρ es la densidad, S el área de la s
sección transversal y Δl la longitud del segmento de T 48 N m
v= = = 10
la cuerda. En un tiempo Δt , la onda con rapidez v μ 0,48 kg m 3
s
recorre una longitud Δl = vΔt , de manera que
1
podemos sustituir Δm = ρSvΔt dentro de la ecuación P= μvμ 2 A 2
2
para ΔE . Obtenemos una expresión para la energía
= (0,5)(0,48)(10 )(400π ) (0,004 ) = 61 W
2 2
transportada en el tiempo Δt .
ΔE = 2π 2 Sρvf 2 A 2 Δt
La rapidez a la cual se propaga la energía a lo largo de Ejemplo 23. La conversación normal se desarrolla a
la cuerda es la potencia P. cerca de 60 dB. ¿A qué nivel de intensidad
ΔE corresponde?
P= = 2π 2 Sρvf 2 A 2 Solución.
Δt
I I
El parámetro más útil generalmente es la intensidad 60 = 10 log10 ,⇒ = 10 6
I , que se define como la potencia que fluye a través I0 I0
de un área unidad. Para este caso, la intensidad en
watts por metro cuadrado (W/m2) es:
⇒ I = 10 6 I 0 = 10-6 W/m2
P
I= = 2π 2 ρvf 2 A 2 Ejemplo 24. Una fuente emite el sonido
S uniformemente en todas las direcciones en un nivel de
Aunque este resultado lo hemos derivado para el caso la energía de 60 W. ¿Cuál es la intensidad una
especifico de ondas en una cuerda, dan la dependencia distancia de 4 m de la fuente?
correcta de la densidad del medio, la velocidad de la
onda, la frecuencia y la amplitud apropiada para Solución.
cualquier onda armónica viajera. La potencia se distribuye sobre la superficie de una
esfera de área A = 4πr .
2
El oído humano puede acomodarse a un intervalo de
intensidades sonoras bastante grande, desde 10-12 P 60
I= = = 0,30W/m2
W/m2 aproximadamente (que normalmente se toma 4πr 4πr (4)
2 2

como umbral de audición), hasta 1 w/m2


aproximadamente que produce sensación dolorosa en
Ejemplo 25. A una distancia de 5 m de una fuente el
la mayoría de las personas. Debido a este gran
nivel de sonido es 90 dB. ¿A qué distancia el nivel ha
intervalo y a que la sensación fisiológica de fuerza
bajado a 50 dB?
sonora no varía directamente con la intensidad, se
Solución.
utiliza una escala logarítmica para describir el nivel de
intensidad de una onda sonora. P P I 2 r12
I1 = y I 2 = de aquí =
4πr12 4πr22 I 1 r22
Nivel de Intensidad.
El nivel de intensidad, β , se mide en decibelios (dB) I I
β 1 = 10 log10 1 = 90dB , ⇒ 1 = 10 9
y se define: I0 I0
I Similarmente,
β = log , donde I es la intensidad del sonido, e
I0 I2 I
β 2 = 10 log10 = 50dB , ⇒ 2 = 10 5
I 0 es un nivel de referencia cuyo valor es de 10-12 I0 I0
W/m2 que escogemos como la unidad de audición. I 2 10 5 r2
En esta escala, el intervalo de intensidad sonora para el Luego = 9 = 10 − 4 = 12
oído humano es de 0 dB a 120 dB, que corresponden a I 1 10 r2
intensidades a partir de 10-12 W/m2 hasta cerca de 1 ⇒ r2 = 10 2 r1 = 500 m
14
Movimiento ondulatorio y ondas Hugo Medina Guzmán

REFLEXION DE ONDAS
Ahora veremos que sucede con una onda al llegar a un
extremo que la confina; para este estudio
consideraremos una perturbación en una cuerda,
primero veremos cuando el extremo esta rígidamente
atado a la pared y la cuerda no tienen posibilidad de
desplazamiento en ese punto. Luego veremos el caso
en que la cuerda tiene posibilidad de desplazamiento
vertical en el punto de atadura. Esta propiedad de las
ondas que aquí introducimos se aplica a todas las
ondas.

Primer Caso.- Extremo fijo


PRINCIPIO DE SUPERPOSICIÓN DE ONDAS -
Cuando el pulso de una onda llega al extremo más
INTERFERENCIA
alejado de una cuerda que esta fija a una pared en ese
Tratamos en este punto el efecto combinado de dos o
extremo, la onda no se detiene repentinamente, sino
más ondas que viajan en el mismo medio. En un medio
que es reflejada. Si no se disipa energía en el extremo
lineal, esto es, en un medio en que la fuerza de
lejano de la cuerda, la onda reflejada tiene una
recuperación es proporcional al desplazamiento del
magnitud igual a la de la onda incidente; sin embargo,
mismo, se puede aplicar el principio de superposición
la dirección de desplazamiento se invertirá (vea
para obtener la perturbación resultante. Este principio
figura). Esta inversión sucede porque a medida que el
es aplicable a muchos tipos de ondas, incluyendo las
pulso encuentra la pared, la fuerza hacia arriba del
ondas en cuerdas, ondas sonoras, ondas superficiales
pulso en el extremo tira hacia arriba sobre la pared.
en el agua y ondas electromagnéticas. El término
Como resultado, de acuerdo con la tercera ley de
interferencia se empleó para describir el efecto
Newton, la pared tira hacia abajo sobre la cuerda. Esta
producido al combinar dos ondas que se desplazan
fuerza de reacción hace que la cuerda estalle hacia
simultáneamente a través de un medio.
abajo, iniciando un pulso reflejado que se aleja con una
amplitud invertida (o negativa).
Principio de superposición.
El principio de superposición establece que, cuando
dos o más ondas se mueven en el mismo medio lineal,
la onda resultante en cualquier punto es igual a la suma
algebraica de los desplazamientos de todas las ondas
componentes.

Segundo Caso.- Extremo Libre


Si la cuerda tiene libertad para moverse en su extremo Ejemplo 26. Entre dos barras paralelas se mantiene
lejano. De nuevo, un pulso de onda que viaja a lo largo tensa una cuerda mediante dos anillos, como se indica
de la cuerda se refleja cuando alcanza ese extremo (vea en la figura. Se perturba la cuerda partiendo de un
figura). Pero en este caso vemos que la onda reflejada desplazamiento inicial como el indicado en la figura
tiene la misma dirección de desplazamiento que la (muy exagerado en la misma). La longitud de la cuerda
onda incidente. A medida que el pulso alcanza el es d y la velocidad de propagación de las ondas
extremo de la cuerda, ésta se mueve en respuesta al transversales en dicha cuerda es v.
pulso. A medida que el extremo de la cuerda empieza a Cuánto tiempo transcurrirá hasta que la cuerda alcance
regresar a su posición, inicia un pulso inverso a lo un estado igual al representado si:
largo de la cuerda, justamente como si el movimiento a) Los anillos pueden moverse libremente a lo largo
final se debiera a alguna fuerza externa. El resultado es de las barras.
un pulso exactamente igual al pulso de onda incidente. b) Un anillo está fijo.
Pero viajando en el sentido contrario. c) Están fijos los dos anillos.

15
Movimiento ondulatorio y ondas Hugo Medina Guzmán

se produce en t = d / v , y el máximo en un tiempo:


2d
t3 =
v

Solución.
a) Si los anillos pueden moverse a lo largo de las
barras, cuando los pulsos de la figura llegan a los
extremos la reflexión se realiza sin cambio de fase. El
máximo central se produce en el instante t1 tal que:
d /2 d
t1 = 2 ⇒ t1 =
v v

b) En el anillo fijo se produce cambio de fase en la


reflexión. La propagación sigue los pasos de la figura.
ONDAS QUE VIAJAN EN LA MISMA
DIRECCION.
Se aplicará el principio de superposición a dos ondas
armónicas que viajan en la misma dirección en cierto
medio.

Ondas con la misma Amplitud y frecuencia.


Si el sentido de avance es el del semieje positivo de las
x, y tienen la misma frecuencia, longitud de onda y
amplitud, pero difieren en fase se pueden expresar sus
funciones de onda individuales como
y1 = Asen (ωt − kx ) e
y 2 = Asen (ωt − kx − ϕ )
La función de onda resultante y se obtiene haciendo
y total = y1 + y 2
= Asen (ωt − kx ) + Asen (ωt − kx − ϕ )
Empleando la identidad trigonométrica siguiente:

senA + senB = 2 cos


( A − B ) sen ( A + B )
2 2
Se produce un mínimo en el centro en el instante: Se obtiene
d / 2 d d / 2 2d ⎡ ϕ⎤ ⎛ ϕ⎞
t= + + = y el tiempo necesario y total = ⎢2 A cos ⎥sen⎜ kx − ωt − ⎟
v v v v ⎣ 2⎦ ⎝ 2⎠
para que se produzca el máximo en el centro es el Luego, observamos el movimiento resultante es
4d nuevamente ondulatorio, pues es de la forma
doble que el anterior, es decir: t 2 = f ( x − vt ) o bien f (kx − ωt ) .
v
c) Con los dos extremos fijos hay cambio de fase en
ambos. Como se aprecia en la figura el mínimo central

16
Movimiento ondulatorio y ondas Hugo Medina Guzmán

La onda resultante tiene igual frecuencia y longitud de Si 0 < ϕ < π la onda resultante tiene una amplitud
ϕ cuyo valor está entre 0 y 2A. La figura muestra un
onda que sus componentes, pero con desfase
2 π
desfase ϕ=−
ϕ 2
respecto a y1 y − respecto a y 2
2
La amplitud de este movimiento ondulatorio es
⎡ ϕ⎤
⎢⎣2 A cos 2 ⎥⎦ , vemos que es diferente al de sus
componentes y con la característica fundamental que
depende de ϕ .
ϕ
Si ϕ = 0, 2π, 4π..., entonces cos = ±1 y la
2
amplitud de la onda resultante es ± 2 A . En otras
palabras, la onda resultante es el doble de amplia que
las ondas individuales. En este caso se dice que las
ondas están en fase en todos los puntos, es decir, las
crestas y los valles de las ondas individuales ocurren
en las mismas posiciones. Este tipo de superposición APLICACIONES:
se denomina interferencia constructiva. El estetoscopio y la cancelación de ruidos.

El estetoscopio.
Este instrumento fue inventado en 1816 por el médico
francés R.T.H. Laennec. A este hombre, por pudor, no
le agradaba la idea de aplicar su oreja sobre el pecho
de las pacientes, por lo que se acostumbró a utilizar un
tubo de papel. Posteriormente perfeccionó la idea
aplicando el principio de interferencia constructiva.

Si ϕ = π (o cualquier múltiplo impar de veces ) π,


ϕ
entonces cos = 0 , y la onda resultante tiene
2 Cancelación de ruidos.
amplitud cero en cualquier parte. En este caso la cresta La interferencia destructiva puede ser muy útil. Es
de una onda coincide con el valle de la otra y sus muy importante que el piloto de un avión oiga lo que
desplazamientos se cancelan en cada punto. Este tipo sucede a su alrededor, pero el ruido del motor
de superposición se denomina interferencia representa un problema. Por eso, los pilotos pueden
destructiva. usar unos auriculares especiales conectados a un
micrófono que registra directamente el sonido del
motor. Un sistema en los auriculares crea una onda
inversa a la que llega a través del micrófono. Esta onda
es emitida, de forma que neutraliza la primera. En los
automóviles se está experimentando con un sistema
similar.

17
Movimiento ondulatorio y ondas Hugo Medina Guzmán

n = 0 ⇒ x2 = 0,67 m
Los mínimos están en valores de x igual 0,33 m; 0,67
m.
Los restantes máximos y mínimos se localizan fuera
del segmento F F'.

Ejemplo 28. Dos Fuentes separadas 20 m vibran de


acuerdo a las ecuaciones
y1 = 0,06senπ t m y 2 = 0,02senπ t m
Ellas envían ondas de velocidad 3 m/s a lo largo de
una varilla. ¿Cuál es la ecuación del movimiento de
una partícula a 12 m de la primera fuente y a 8 m de la
segunda?
Solución.
Ejemplo 27. Dos focos puntuales F y F', separados
entre si 1 m, emiten en fase sonidos de 500 Hz de
frecuencia con la misma intensidad.
a) Obtener la posición de los puntos, si los hay, en los
que no se registra sonido.
b) Obtener la posición de los máximos y mínimos de Referido a la figura. La fuente 1 envía ondas en el
intensidad que se registran a lo largo del segmento FF'. sentido +x, tal que
(v = 340 m/s). y1 = A1sen (kx1 − ωt ) .
x=D La fuente 2 envía ondas en el sentido -x, tal que
y 2 = A2 sen (kx 2 + ωt )
rad ω m
como ω = π ,y v= =3
s k s
ω π
Solución. ⇒ k= = m
a) Si consideramos que ambos sonidos se propagan v 3
con frentes de ondas esféricos y que por tanto la También A1 = 0,06 m y A2 = 0,02 m
amplitud disminuye con la distancia, para que se La perturbación resultante en el punto
produzca anulación total en un punto, éste deberá x1 = 12 m, x2 = -8 m es.
equidistar de F y F', con lo que los únicos puntos y = y1 + y 2
serian los de la mediatriz del segmento F F'; pero
⎛π ⎞ ⎛π ⎞
precisamente en esos puntos las dos amplitudes se = 0,06sen⎜ x1 − πt ⎟ + 0,02sen⎜ x 2 + πt ⎟
suman por estar los focos en fase. En consecuencia, no ⎝3 ⎠ ⎝3 ⎠
hay ningún punto a distancia finita en el que la
⎛ 12π ⎞ ⎛ 8π ⎞
intensidad resultante sea nula. = 0,06sen⎜ − πt ⎟ + 0,02sen⎜ − + πt ⎟
b) Desde un punto P del segmento F' a distancia x ⎝ 3 ⎠ ⎝ 3 ⎠
de F, la diferencia de caminos a los focos es: ⎛ 2π ⎞
Δx = x1 − x 2 = x − ( D − x) = 2 x − D = 0,06senπt + 0,02sen⎜ πt − ⎟
⎝ 3 ⎠
MÁXlMOS:
v D n v ⎡ 1 3 ⎤
Δx = n λ ⇒ 2 x − D = n ⇒x= + = 0,06senπt + 0,02 ⎢− senπt − cos πt ⎥
f 2 2 f ⎣ 2 2 ⎦
1 1 340 = 0,05senπt − 0,0173 cos πt
n = −1 ⇒ x1 = − = 0,16m
2 2 500
n = 0 ⇒ x2 = 0,50m Ejemplo 29. Dos fuentes F1 y F2, que vibran con la
misma fase producen en la superficie libre del agua
n = +1 ⇒ x3 = 0,84m ondas representada por las ecuaciones:
Los máximos están en valores de x igual a 0,16; 0,50; y1 = 8sen (20π t − 0,2π x ) (en cm)
y 2 = 4sen (40π t − 0,4π x ) (en cm)
0,84 m
MÍNIMOS:
λ D (2n + 1) v Determine la amplitud de la onda que se produce por
Δx = (2n + 1) ⇒ x=
+ interferencia en un punto P que dista 25 cm de F1 y 15
2 2 4 f cm de F2.
1 1 340 Solución.
n = −1 ⇒ x1 = − = 0,33m
2 4 500
18
Movimiento ondulatorio y ondas Hugo Medina Guzmán

Usando la relación
sen ( A − B ) = senA cos B − cos AsenB :
y1 = 8(sen 20π t cos 0,2π x - cos20π tsen 0,2π x )
y 2 = 4(sen 40π t cos 0,4π x - cos40π tsen 0,4π x )

En el punto P (x1 = 25 cm, x2= 15 cm):


y1 = 8(sen 20π t cos 5π - cos20π tsen5π )
y 2 = 4(sen 40π t cos 6π - cos40π tsen6π )
Con sen 5π = cos π = 0 , cos5π = cos π = −1
y sen 6π = cos 2π = 0 , cos6π = cos 2π = 1
Obtenemos:
y1 = 8(- sen 20π t ) = −8sen 2π t
Si se desea la onda resultante puede sumarse a una
y 2 = 4(sen 40π t ) = 4sen 2π t tercera onda y así sucesivamente. En general esta
La suma: superposición no es simple, puesto que tanto la
y = y1 + y 2 = −8sen 2π t + 4sen 2π t = amplitud como la fase resultante pueden ser funciones
del tiempo y de la posición.
− 4sen 2π t
La amplitud de la onda que se produce por Ejemplo 30. Dos ondas armónicas de amplitudes 2 y
interferencia en un punto P es 4 cm. 4 cm viajan en la misma dirección y tienen idéntica
frecuencia; si su diferencia de fase es π/4, calcúlese la
Ondas que difieren tanto en Frecuencia como en amplitud de la onda resultante.
Amplitud
Sean las ondas y1 e y2 que difieren tanto en frecuencia Solución.
como en amplitud
y1 = A1sen (ω1t ± k1 x ) = A1senθ1 e A una diferencia de fase δ =
π
, le corresponde una
y 2 = A2 sen (ω 2 t ± k 2 x ) = A2 senθ 2 4
Si las ondas componentes difieren tanto en frecuencia δ λδ π
distancia: Δx = = =
como en amplitud, existen varios modos de k 2π 8
combinarse, de modo que todos ellos exigen cierta
y como la amplitud de la onda resultante verifica:
habilidad en el cálculo trigonométrico. Si ponemos
θ 2 = θ 1 + δ y desarrollamos Ao2 = Ao21 + Ao22 + 2 Ao1 Ao 2 cos δ
sen (θ 1 + δ ) = senθ 1 cos δ + cos θ1senδ
Sustituyendo:

y = y1 + y 2 = A1senθ1 + A2 senθ 2 Ao = Ao21 + Ao22 + 2 Ao1 Ao 2 cos δ


= A1senθ 1 + A2 sen (θ 1 + δ ) π
= 4 + 16 + 16 cos = 5,6 cm
= ( A1 + A2 cos δ )senθ 1 + A2 senδ cos θ 1 (1) 4
Esta expresión puede recombinarse en la forma de una
sola onda Ejemplo 31. El aparato de Quincke consta de dos
y = Asen (θ1 + φ ) tubos en U, pudiéndose deslizar las ramas de uno de
ellos dentro de las ramas del otro. En las proximidades
= A cos φ senθ1 + Asenφ cos θ1 (2) de la ramificación
Igualando coeficientes de (1) y (2) obtenemos las A se produce un sonido que se escucha poniendo el
ecuaciones: oído en B. Deslizando el tubo 1 dentro del 2, se
A cos φ = A1 + A2 cos δ y Asenφ = A2 senδ encuentran posiciones en las que no se percibe sonido;
¿por qué? Si el desplazamiento lateral que hay que dar
Elevándolas al cuadrado y sumando obtenemos el al tubo 1, desde que no se percibe sonido hasta que, de
valor de A: nuevo, se deja de percibir, es de 25 cm, ¿cuáles son la
A= A12 + A22 + 2 A1 A2 cos δ longitud de onda, la frecuencia y el período de las
ondas sonoras? Velocidad de propagación del sonido
Y dividiéndolas obtenemos el valor de φ: en el aire, 340 m/s.
A2 senδ
tan φ =
A1 + A2 cos δ

19
Movimiento ondulatorio y ondas Hugo Medina Guzmán

y1 = Asen (kx − ωt ) e y 2 = Asen (kx + ωt ) , la


suma de estas ondas nos da:
y total = y1 + y 2
= Asen (kx − ωt ) + Asen (kx + ωt ) , haciendo uso
de la suma trigonométrica
y total = 2 Asenkx cos ωt
Solución. El movimiento resultante no es ondulatorio, pues no se
propaga al no ser de la forma f ( x − vt ) .
No se percibirá sonido cuando la diferencia de
recorridos A 1 B y A 2 B sea un número impar de semi
longitudes de onda. Si en tales condiciones se desplaza Una partícula en cualquier punto dado x ejecuta
el tubo 1 hasta dejar de nuevo de percibir sonido, el movimiento armónico simple conforme transcurre el
exceso de recorrido que hace el sonido, con respecto a tiempo. Nótese que todas las partículas vibran con la
la posición anterior, es una longitud de onda. misma frecuencia, pero con la particularidad que la
En la segunda posición el sonido ha recorrido en la amplitud no es la misma para cada partícula del medio,
rama A 1 B, 50 cm más que en la A 2 B (25 en la parte con la posición (en un movimiento ondulatorio al
superior y de 1 y 25 en la inferior). Por tanto: amplitud es igual para cualquier punto).
λ =50 cm La amplitud esta dada por 2A sen kx.
340
v
f = =
= 680 Hz Los puntos de mínima amplitud (nula) se llaman
λ 0,5 nodos. En ellos se debe cumplir:
1 1 senkx = 0 ⇒ kx = nπ
T= = s 2π
f 680 ⇒ x = nπ ⇒
λ
ONDAS IGUALES VIAJANDO EN SENTIDOS λ
OPUESTOS. ONDAS ESTACIONARIAS x=n
Un tipo de superposición de ondas especialmente 2
interesante es el que tiene lugar entre dos ondas de Para n = 0, 1, 2, 3, ……….
idénticas características pero propagándose en sentido
contrario. Las ondas resultantes reciben el nombre de Los puntos de máxima amplitud (± 2A) se llaman
ondas estacionarias, pues no implican un movimiento vientres o antinodos. En ellos se debe cumplir:
π
de avance de la perturbación
senkx = ±1 ⇒ kx = (2n + 1)
2
2π π
⇒ x = (2n + 1) ⇒
λ 2
λ
x = (2n + 1)
4
Para n = 0, 1, 2, 3,……….

Así pues, tanto los nodos como los vientres aparecen a


Este tipo de ondas están asociadas a reflexiones en los intervalos de longitud λ/2, mediando entre un nodo y
límites de separación de medios de propiedades un antinodo hay una distancia de λ/4.
diferentes. Dichos límites pueden ser básicamente de
dos tipos, libres y fijos. El nudo de unión de dos
cuerdas de diferente grosor sería un ejemplo de límite
libre; por el contrario, el extremo de la cuerda unido a
un punto fijo en una pared sería un límite fijo.
Vimos anteriormente que en un límite libre la onda
reflejada tiene las mismas características que la onda
incidente, tan sólo difieren en el sentido de avance de La figura muestra la envolvente de una onda
la perturbación. Por el contrario, en un límite fijo la estacionaria.
onda reflejada posee las mismas características que la Al no propagarse las ondas estacionarias, no
incidente, pero está desfasada π radianes respecto a la transportan energía.
onda incidente La energía se mantiene estacionaria, alternando entre
cinética vibratoria y potencial elástica. Por lo tanto el
Consideremos en primer lugar las ondas estacionarias movimiento repetimos no es ondulatorio, el nombre
(que se propagan en el eje x) por reflexión en un límite proviene del hecho que podemos analizarlo como
libre. La función de onda resultante será: superposición de ondas.

20
Movimiento ondulatorio y ondas Hugo Medina Guzmán

Ejemplo 32. Por un medio unidimensional (dirección


Condiciones de contorno del eje Ox) se propagan dos ondas transversales,
Las condiciones en los límites, llamadas condiciones vibrando en el plano xOy y dadas por:
de contorno, imponen restricciones a la hora de y1 = Asen (ωt + kx ) , y 2 = Asen (ωt − kx + ϕ ) .
formarse ondas estacionarias en el medio
a) Comprobar que la superposición de ambas da lugar
correspondiente. Así, si los límites son fijos, en ellos se
a una onda estacionaria.
tendrán que dar nodos necesariamente; si ambos
b) Si en x = 0 ha de haber un nodo de la onda
límites son libres se darán antinodos, y si uno es libre y
estacionaria, comprobar que el valor de ϕ debe ser π.
el otro es fijo se habrán de dar antinodo y nodo
c) Calcular la velocidad de un punto del medio cuya
respectivamente.
distancia al origen sea 1/4 de la longitud de onda.
Solución.
Límite fijo - Límite fijo: (como en los instrumentos
a)
musicales, violín, arpa, etc., la cuerda esta fija en sus
dos extremos) y = y1 + y 2
= Asen (ωt + kx ) + Asen (ωt − kx + ϕ )
⎛ ϕ⎞ ⎛ ϕ⎞
= 2 Asen⎜ ωt + ⎟ cos⎜ kx − ⎟ (1)
⎝ 2⎠ ⎝ 2⎠
⎛ ϕ⎞
Llamando: y o ( x ) = 2 A cos⎜ kx − ⎟
⎝ 2⎠
⎛ ϕ⎞
⇒ y = y o sen⎜ ωt + ⎟
⎝ 2⎠
Por lo tanto, la expresión (1) es la ecuación de la onda
estacionaria puesto que cualquier partícula en un punto
En este caso las condiciones a imponer son que, si la dado x efectúa un movimiento armónico simple al
longitud del medio es L, tanto en x=0 como x=L se transcurrir el tiempo, vibrando todas las partículas con
habrán de dar nodos. Aplicando la condición de nodo idéntico periodo; y cada partícula vibra siempre con la
en un límite fijo, resulta: misma amplitud, no siendo la misma para cada una
sino que varia con la posición (x) de cada partícula.
λn 2L
L=n ⇒ λn = ϕ
2 n b) y o (0 ) = 0 ⇒ cos =0 ϕ =π
o en términos de frecuencias, 2
v v ∂y ⎛ π⎞
fn = ⇒ fn = n c) v y = = y oω cos⎜ ωt + ⎟ = − y oωsenωt
λn 2L ∂t ⎝ 2⎠
Para n = 1, 2, 3, 4,………….. ⎛ 2π λ π ⎞
Por tanto, tanto la frecuencia como la longitud de onda y o ⎛⎜ λ ⎞⎟ = 2 A cos⎜ − ⎟ = 2A
sólo pueden tomar determinados valores, es decir,
⎝4⎠ ⎝ λ 4 2⎠
están cuantificadas. La frecuencia más baja de la serie Finalmente v y = −2 Aωsenωt
recibe el nombre de frecuencia fundamental, y las En tal punto existe un vientre.
restantes, que son múltiplos de la fundamental, reciben
Ejemplo 33. La onda y1 = Asen (kx − ωt ) viaja
el nombre de armónicos.

por una cuerda. Después de reflejarse se convierte en

sen (kx + ωt ) . Que es lo que se obtiene de


A
y2 = −
2
la combinación de estas dos ondas.
Solución.

Hagamos y = y1 + y 2 .

y = Asen (kx − ωt ) − sen (kx + ωt ) =


A
2
sen (kx − ωt ) + sen (kx − ωt ) − sen (kx + ωt )
A A A
Estas frecuencias posibles en la cavidad formada por 2 2 2
los límites fijos, se denominan modos de la cavidad

21
Movimiento ondulatorio y ondas Hugo Medina Guzmán

sen (kx − ωt ) − Asenωt cos kx


A
= Ejemplo 36. Una cuerda horizontal, de longitud l =
2
0,80 m, esta sometida en uno de sus extremos a
El primer término es una onda viajera y el Segundo
oscilaciones sinusoidales de frecuencia f = 120 Hz,
una onda estacionaria.
esta frecuencia corresponde a uno de los modos
resonantes de la cuerda y se observa que entre sus
Ejemplo 34. Calcular la frecuencia del sonido
extremos aparecen 4 antínodos ó vientres cuya
fundamental emitido por una cuerda de 1 m de
amplitud de oscilación es A = 2 cm. Calcular:
longitud y 1 mm de diámetro, cuya densidad es 2
a) La velocidad de propagación de las ondas.
g/cm3 y está tensa por un peso de 9231,6 g.
b) La velocidad y aceleración máxima que puede
alcanzar un punto de la cuerda.
c) La amplitud de oscilación de un punto de la cuerda
situado a 0,050 m de un extremo de la cuerda.
d) La amplitud de oscilación de un punto de la cuerda
situado a 0,125 m de un extremo de la cuerda.

Solución. Solución.
La frecuencia del sonido emitido por una cuerda es:
n T
fn =
2L μ
⎛ m⎞
T = (9,2316kg )⎜ 9,8 2 ⎟ = 90,47 N
⎝ s ⎠
⎛ kg ⎞⎛ 10 −6 π ⎞
μ = ρA = ⎜ 2000 3 ⎟⎜⎜ ⎟ λ = 0,40 m , f = 120 Hz,
⎝ m ⎠⎝ 4 ⎟⎠
a) La velocidad de propagación de las ondas.
−3 kg v = λ f = 0,40 × 120 = 48 m/s
= 1,57 × 10
m b) La velocidad y aceleración máxima que puede
1 90,47 alcanzar un punto de la cuerda.
f = = 38,8 Hz
2(1) 1,57 × 10 −3
dy
v= = Aωsenω t
dt
Ejemplo 35. Una cuerda está estirada por un peso de ⇒ v máx = Aω = 0,02 × 240π = 4,8π m/s
10 N. Calcular el peso que debe tensar a otra cuerda de
la misma sustancia, la misma longitud y doble radio
para que emita la octava aguda de la que produce la
d2y
primera. Se supone que ambas emiten el sonido a= 2
= − Aω 2 senω t
fundamental. dt
Solución.

μ = ρA = πr 2 ρ ,
a máx = Aω 2 = 0,02 × (240π ) = 1152π 2 m/s 2
2

m' ρπ (2r ) L
2
μ'= = = 4πr 2 ρ = 4μ
L L c) La amplitud de oscilación de un punto de la cuerda
1 T situado a 0,050 m de un extremo de la cuerda.
f = y Ecuación de una onda estacionaria:
2L μ
y = 2 Asenkx cos ω t
1 T' 1 T' La amplitud está dada por:
f '= 2 f = ⇒ f '= 2 f =
2L μ ' 2 L 4μ 2π
2 Asenkx = 2 Asen x
Relacionando f y f’: λ
1 T' Para 0,050 m

2 L 4μ T' ⎛ 2π ⎞ ⎛π ⎞
f' 2f 1 T' 0,04sen⎜ 0,050 ⎟ = 0,04sen⎜ ⎟ = 0,028 m
= = = ⇒ = 16 ⎝ 0,40 ⎠ ⎝4⎠
f f 1 T 2 T T
2L μ d) La amplitud de oscilación de un punto de la cuerda
Como T = 10 N, T’ = 160 N situado a 0,125 m de un extremo de la cuerda.
Para 0,125 m

22
Movimiento ondulatorio y ondas Hugo Medina Guzmán

⎛ 2π ⎞ ⎛π ⎞ n1 T n2 T n1 L1 ρ1
0,04sen⎜ 0,125 ⎟ = 0,04sen⎜ ⎟ = 0,037 m = ⇒ =
⎝ 0,40 ⎠ ⎝4⎠ L1 ρ1 L2 ρ2 n 2 L2 ρ2
n1
Ejemplo 37. Un alambre de aluminio de L1 = 60,0 Reemplazando valores, obtenemos: = 0,4
n2
cm y con una superficie transversal 1,00x10-2 cm2, está
conectado a un alambre de acero de la misma Como la menor es la frecuencia se obtiene con el
superficie. El alambre compuesto, cargado con un menor valor de n, tenemos que buscar los menores
bloque m de 10,0 kg de masa, está dispuesto como se valores de n1 y n2 que tengan la relación 0,4,
indica en la figura, de manera que la distancia L2 de n1 2
=
la unión con la polea de sostén es 86,6 cm. Se crean n2 5
ondas transversales en el alambre utilizando una fuente
Correspondiendo n1 = 2 y n 2 = 5 .
externa de frecuencia variable.
a) Determine la frecuencia más baja de excitación en a) Usando n1 = 2 , obtenemos la frecuencia que
que se observan las ondas estacionarias, de modo que produce un nodo en la unión
la unión en el alambre es un nodo.
b) ¿Cuál es el número total de nodos observados en n1 T 2 10(9,8)
f = =
esta frecuencia, excluyendo los dos en los extremos del
alambre?
2 L1 μ1 (
2(0,6 ) 2,6 × 10 3 10 −6 )
La densidad del aluminio es 2,60 g/cm3, y la del acero = 324 Hz
es 7,80 g/cm3. b) El número total de nodos observados en esta
frecuencia, excluyendo los dos en los extremos del
alambre, se pueden contar en el esquema de la figura,
son 6 (hay un nodo común para el aluminio y para el
acero).

Solución.
La frecuencia para ondas estacionarias en una cuerda
fija en los dos extremos es
Ejemplo 38. Dos ondas armónicas se escriben por
v T medio de:
fn = n , como para una cuerda tensa v = ,
2L μ ⎛x ⎞
y1 ( x, t ) = 0,015cos⎜ − 40t ⎟ ,
n T ⎝2 ⎠
obtenemos: fn =
2L μ ⎛x ⎞
y 2 ( x, t ) = 0,015cos⎜ + 40t ⎟
Como el punto de unión de los alambres tiene que ser ⎝2 ⎠
un nodo, tenemos n1 nodos para el aluminio y n 2 Donde x, y1, e y 2 están en metros y t en segundos.
nodos para el acero.
Dichas ondas se propagan en una cuerda tensa de gran
Siendo la frecuencia f , la tensión T y la sección de longitud e interfieren para producir una onda
alambre S común para los dos alambres, tenemos: estacionaria.
a) Determine la longitud de onda, frecuencia y rapidez
n1 T
Para el aluminio f = , para el acero de propagación de las ondas que interfieren.
2 L1 μ1 b) Determine la función de la onda estacionaria.
c) Determine la posición de los nodos y antinodos en la
n2 T onda estacionaria.
f = d) ¿Cuál es la amplitud de la onda en x = 0,4 m?
2 L2 μ2
Solución.
n1 T n2 T a) La onda viajera es de la forma
Luego = ⎛ 2π ⎞
2 L1 μ1 2 L2 μ2 y ( x, t ) = Acos⎜ x − 2π f t ⎟
La masa por unidad de longitud ⎝ λ ⎠
m mS ⎛ m ⎞ Luego comparando:
μ= = = ⎜ ⎟ S = ρS 2π 1
L LS ⎝ V ⎠ = ⇒ λ = 4π = 12,56 m
λ 2
Reemplazando las expresiones de μ1 y μ2 :
20
2π f = 40 ⇒ f = = 6,37 Hz
π

23
Movimiento ondulatorio y ondas Hugo Medina Guzmán

⎛ 20 ⎞ Solución.
v = λ f = (4π )⎜ ⎟ = 80 m/s a) ¿Cuáles son los armónicos producidos por estas
⎝π ⎠ masas?
b) y = y1 + y 2 Los armónicos se producen para m1 = 16 kg y para m2
= 25 kg.
⎛x ⎞ ⎛x ⎞
y (x, t ) = 0,015cos⎜ − 40t ⎟ + 0,015cos⎜ + 40t ⎟ λ 2L
⎝2 ⎠ ⎝2 ⎠ n =L ⇒ λ=
⎡ ⎛x ⎞ ⎛x ⎞⎤ 2 n
y ( x, t ) = 0,015⎢cos⎜ − 40t ⎟ + cos⎜ + 40t ⎟⎥
⎣ ⎝2 ⎠ ⎝2 ⎠⎦ 2L m1 g 2L m2 g
λ1 f = f = , λ2 f = f =
Siendo n1 μ n2 μ
cos(α − β ) + cos(α + β ) = 2 cos α cos β :
n2 m1 16 4
⎛ ⎞ Luego: = = =
y ( x, t ) = 0,015⎜ 2cos cos 40t ⎟
x
n1 m2 25 5
⎝ 2 ⎠ Los armónicos son el quinto y el cuarto.
x ¿Cuál es la relación entre las tensiones y el número
= 0,030cos cos 40t
2 armónico?
Función de la onda estacionaria T1 m1 16
c) Determine la posición de los nodos y antinodos en la T1 = m1 g , T2 = m2 g y = =
onda estacionaria.
T2 m2 25
x b) ¿Cuál es la frecuencia del generador?
Los nodos son para cos =0 n1 m1 g 5 16(9,8)
2 f = ⇒ f = = 350
x π π π π 2L μ 2(2) 0,002
= ,3 ,5 ,.....n Hz
2 2 2 2 2
c) ¿Cuál es el valor máximo de m que produce un
x = ±π ,± 3π ,±5π ,..... ± nπ armónico?
x 2L 4 L2 f 2 μ
Los antinodos son para cos = ±1 , mg
2 λf = f = ⇒ m=
n μ n2 g
x
= 0, 2π , 4π ....2nπ El mayor valor de m se produce con n = 1
2 4(2) (350) (0,002)
2 2
x = 0,±2π ,±4π ,±6π ,.... ± 2nπ m= = 400 kg
d) Amplitud de la onda en x = 0,4 m. (1)2 (9,8)
0,4
A = 0,030cos = 0,030(0,98) Ejemplo 40. El puente colgante de Tacoma, de
2 aproximadamente 1810m de longitud, fue abierto al
= 0,0294 m = 2,94 cm tráfico el 1 de julio de 1940, luego de 2 años de
construcción, uniendo Tacoma y Gig Harbor. 4 meses
Ejemplo 39. En el dispositivo de la figura, una masa después el puente colapsó durante una tormenta el 7 de
m es colgada de una cuerda que pasa sobre una polea. Noviembre de 1940. Durante la resonancia se observó
El otro extremo de una cuerda es conectada a un al puente oscilando en su segundo modo de vibración a
generador de frecuencia f fija. La cuerda tiene una razón de 60 oscilaciones cada minuto.
longitud L de 2 metros y una densidad lineal de 0,002
kg/m. Se observan armónicos únicamente cuando las
masas colgadas son 16 kg y 25 kg.
a) ¿Cuáles son los armónicos producidos por estas
masas?
¿Cuál es la relación entre las tensiones y el número
armónico?
b) ¿Cuál es la frecuencia del generador?
c) ¿Cuál es el valor máximo m para que se produzca un
armónico?
Determine:
a) la longitud de onda de la onda estacionaria formada.
b) la velocidad de propagación de la onda.
c) el módulo de corte del puente, asumiendo que la
densidad promedio del puente era de 5xl03kg/m3.
d) la ley de movimiento vertical de un carro que se
hallaba estacionado a un cuarto de la longitud del
puente desde uno de sus extremos.
24
Movimiento ondulatorio y ondas Hugo Medina Guzmán

Solución. Δθ Δθ
a) ∑F H = FB cos
2
− FA cos
2
=0
⇒ FA = FB = T
Δθ Δθ
∑F V = − FB sen
2
− FA sen
2
= −Δmac

λ = 1810 m De esta última ecuación:


b) v = λf = (1810 m )(1 / s ) = 1810 m/s
Con FA = FB = T , Δm = μΔl = μRΔθ y
v=
G
⇒ G = v2ρ = Δθ Δθ
c) Como sen ≈ :
ρ 2 2
(1810)2 (5 × 10 3 ) = 1,63805 × 1010 N Obtenemos:
m2
Δθ
d) Para ondas estacionarias y = 2 Asenkx cos ωt 2T = μRΔθ (ω 2 R )
λ 2
Para x= , senkx = 1 , luego
4 ⇒ T = μω 2 R 2
y = 2 A cos ωt ⇒ y = 2 A cos 2πft
b) En una cuerda con densidad lineal μ y tensión T una
onda viaja con velocidad
Ejemplo 41. Un lazo de cuerda se gira a una alta
velocidad angular ω , de modo que se forma un
T μω 2 R 2
círculo tenso del radio R. Se forma un pulso (como se v= = = ωR .
muestra en la figura) en la cuerda girante. μ μ
a) Demostrar que la tensión en la cuerda es Luego el pulso debe viajar con una velocidad
T = μω 2 R 2 , donde μ es la densidad lineal de la T
cuerda. v= relativa a la cuerda. Luego, si el pulso se
μ
b) Bajo que condiciones el pulso permanecería
estacionario relativo a un observador en tierra. mueve en sentido horario con respecto a la cuerda,
permanecería estacionario con respecto a tierra.

Límite libre. Límite libre: (un tubo abierto en ambos


extremos, como en los instrumentos musicales de
viento, ejemplo, la flauta).

Solución.
a) Según se muestra en figura tomemos ACB una
pequeña sección de la cuerda, que subtiende un ángulo
Δθ en O, el centro del lazo. Elegimos C en el punto
medio del arco.

En este caso las condiciones a imponer son que, si la


longitud del medio es L, tanto en x=0 como x=L se
habrán de dar antinodos. Aplicando la condición de
antinodo en un límite libre, resulta:
λn 2L
L=n ⇒ λn =
2 n
o en términos de frecuencias,
Aplicando la segunda ley de Newton:

25
Movimiento ondulatorio y ondas Hugo Medina Guzmán

λn v
fn = ⇒ fn = n
v 2L
Para n = 1, 2, 3, 4,…………..
Por tanto, igual que antes la frecuencia y la longitud de
onda sólo podrán tomar determinados valores, y
estarán cuantificadas. La frecuencia más baja de la
serie recibe el nombre de frecuencia fundamental, y
las restantes, que son múltiplos de la fundamental,
reciben el nombre de armónicos. Se representan a
continuación los cuatro primeros.

Ejemplo 42. Calcular la frecuencia de los sonidos


emitidos por un tubo abierto y otro cerrado de 1 m de
longitud, produciendo el sonido fundamental. Se
supone .que la velocidad del sonido en el aire es 340
m/s.
Solución.
Para tubo abierto (n = 1):
v 340
f = = = 170 Hz
2 L 2(1)
Para un tubo cerrado (n = 1):
Limite fijo. Límite libre: (una cuerda con un extremo v 340
f = = = 85 Hz
con libertad de movimiento y el tubo cerrado en un 4 L 4(1)
extremo).
Ejemplo 43. Calcular la longitud de un tubo abierto
que lleno de aire y a 0° C
(v = 330 m/s) emite como sonido fundamental el DO3.
Solución.
Frecuencia del DO 3 = 264 Hz:
v v 330
f = ⇒L= = = 0,625 m
2L 2f 2(264)

Ejemplo 44. Un tubo de 1 m de largo está cerrado por


En esta situación se tendrá un nodo en x=0 y un uno de sus extremos. Un alambre estirado se coloca
antinodo en x=L, lo que implica que en la longitud L cerca del extremo abierto. El alambre tiene 0,3 m de
de la cuerda han de caber un número impar de cuartos largo y una masa de 0,01 kg. Se sostiene fijo en sus
de onda. Aplicando la condición de antinodo reflexión dos extremos y vibra en su modo fundamental.
en un límite fijo resulta: Pone a vibrar a la columna de aire en el tubo con su
λn 4L frecuencia fundamental por resonancia.
L = (2n − 1) ⇒ λn = Encontrar:
4 2n − 1 a) La frecuencia de oscilación de la columna de aire.
o, en términos de frecuencias, b) La tensión del alambre.
⇒ f n = (2n − 1)
v v Velocidad del sonido en el aire 340 m/s.
fn =
λn 4L Solución.
a) La frecuencia fundamental (n =1) en el tubo sonoro
Para n = 1, 2, 3, 4,………….. cerrado valdrá:
que representan la serie de ondas permitidas por las
v 340
condiciones de contorno. Se representan a f = = = 85 Hz
continuación los cuatro primeros. 4 L 4(1)
b) Dicha frecuencia será la fundamental que se
produce en la cuerda, por lo que:
1 T m 0,01 1 kg
f = , μ= = =
2L μ L 0,3 30 m
2⎛ 1 ⎞
T = 4 L2 f 2 μ = 4(0,3) (85) ⎜ ⎟ = 86,7 N
2

⎝ 30 ⎠
26
Movimiento ondulatorio y ondas Hugo Medina Guzmán

x = 22,5 cm y cos ωt = 1
Ejemplo 45. El tubo de un órgano representado en la
y máx ( x =0, 225 ) = 2 Asenkx
figura tiene 45 cm de longitud y las onda estacionaria
que se produce por el silbato en espacio libre es de una =2 (0,5 ×10 )[sen(10,7 )(0,225)]
−8

longitud de onda de 60 cm. Dicho tubo se puede = 0,71 x 10-8 m


considerar abierto en el extremo izquierdo y cerrado en
el derecho. Ejemplo 46. Un ingeniero naval quiere averiguar si
un tubo que presenta externamente una boca circular
a) Muéstrese en un diagrama al interior del tubo la abierta está abierto o cerrado en el otro extremo que no
onda estacionaria que se produce ubicando la posición logra ver. Para esto decide usar una fuente sonora de
de las crestas nodos y vientres de amplitud. frecuencia variable. Tomó para ello dos frecuencias de
b).Si la máxima amplitud de oscilación de las resonancia consecutivas, que le dieron los siguientes
partículas de aire al interior del tubo es de 10-6 cm. valores: 125 y 175 Hz.
¿cuál será la máxima amplitud que podrán alcanzar a) ¿Cuál es el razonamiento en que se basó?
las partículas de aire en el centro del tubo? b) ¿A qué conclusión llegó?
c) ¿Para qué frecuencia resonaría en estado
fundamental?
d) Cree que también pudo averiguar la longitud del
tubo. Si se puede ¿cuál es?
Solución. Solución.
La frecuencia libre es a) Las frecuencias de resonancia están dadas:
340 n
f =
v
=
= 566,6 Hz , Para tubo abierto: v. f =
λ 0,6 2L
(2n − 1)
2π 2π Para tubo cerrado en un extremo: f = v
k= = = 10,47 rad 4L
λ 0,6 Para n = 1, 2, 3, 4, 5, ……………….
a) Diagrama al interior del tubo la onda estacionaria Y lo que tenemos que hacer es analizar para 125 y 175
para las tres primeras resonancias. Hz, con cuál de las condiciones concuerdan.
b) Para 125 correspondería (n – 1) y para 175
correspondería n.
n −1 n
De tal manera que = , se cumple para
125 175
n = 3,5 valor que no es entero, lo que descarta esta
posibilidad.
No cumple para tubo abierto
Haciendo lo mismo para el caso de tubo cerrado
2n − 3 2n − 1
= , se cumple para n = 4, valor entero
125 175
que si cumple
EL TUBO ES CERRADO .
b).Si la máxima amplitud de oscilación de las c) Como f = ( 2n − 1) v
partículas de aire al interior del tubo es de 10-6 cm. 4L
¿cuál será la máxima amplitud que podrán alcanzar La frecuencia fundamental es para n = 1
las partículas de aire en el centro del tubo? 1
y total = 2 Asenkx cos ωt f = v , f = 25 Hz
4L
La máxima amplitud de oscilación es en un vientre v v
d) Si, ya que = 25 , y L = ,
4L 100
senkx = 1 y cos ωt = 1 , luego: 10−8 = 2 A ⇒ Considerando la velocidad el sonido v = 340 m/s, se
−8 obtiene L = 3,40 m.
A = 0,5 × 10 m
La máxima amplitud que podrán alcanzar las LOS INSTRUMENTOS MUSICALES
La formación de ondas estacionarias está relacionada
partículas de aire en el centro del tubo con los instrumentos musicales tanto de cuerda como
de viento. Así, el sonido generado por un arpa es

27
Movimiento ondulatorio y ondas Hugo Medina Guzmán

consecuencia de la propagación por el aire de las ondas


estacionarias que se producen, entre dos límites fijos,
en las diferentes cuerdas, de modo que los graves
(frecuencias bajas) se producirán en las cuerdas más
largas y los agudos (frecuencias altas) en las cuerdas
más cortas. En los órganos, las ondas estacionarias que
se forman en los tubos se corresponden con las
formadas por reflexión en dos límites, uno fijo y otro
libre. Por tanto, cuanto mayor sea la longitud del Para aumentar la intensidad del sonido producido, se
órgano menor es la frecuencia: los tubos largos monta el diapasón sobre una caja. Si la caja está
corresponden a frecuencias bajas (sonidos graves) y cerrada en un extremo, su longitud es ¼ de la longitud
los cortos a frecuencias altas (sonidos agudos) de onda del sonido en el aire emitido por el diapasón.
Si la caja está abierta en los dos extremos la longitud
OSCILACION DE VARILLAS. DIAPASÓN de la caja es igual a la mitad de dicha longitud de onda.
Varilla fija por un extremo. Puesta en oscilación, al Al vibrar, las .dos ramas de un diapasón se mueven en
organizarse la onda estacionaria se debe tomar un nodo fases opuestas. Cuando las ramas se acercan, el punto
en el extremo fijo y un vientre en el opuesto. Los más bajo del pie del diapasón baja, y sube cuando las
razonamientos que se realizan para un tubo cerrado son ramas se alejan. Este pie se encuentra afectado de un
válidos para este caso; por lo tanto, una varilla que movimiento vibratorio de dirección vertical lo que
oscila fija por un extremo responde a la ley puede comprobarse apoyándolo en la mano. Es así
λ finalmente, como se transmite la vibración del
l = (2n + 1) diapasón a la columna de aire contenida en la caja.
4 Los diapasones se utilizan como patrones de registro
Varilla fija por un punto interior. Si se hace oscilar de frecuencia, pues pueden construirse de manera que
una varilla fija por un punto interior para que se or- no sean afectados por variaciones de temperatura. Es
ganice una onda estacionaria, se formará allí un nodo y posible lograr diapasones capaces de mantener una
vientres en los extremos. Todo esto depende exclusi- frecuencia de vibración con una precisión de 1 en
vamente del punto por el que se sostenga. Este punto 100000.
(siguiendo el razonamiento de tubos abiertos), deberá
estar situado en la mitad, a 1 4 a 1 6 , etcétera, de un Resonancia
extremo. Se ha visto que un sistema tal como una cuerda
Téngase presente que varillas de igual longitud, estirada es capaz de oscilar en uno o más modos
idénticamente .fijadas, pueden producir sonidos de naturales de vibración. Si se aplica una fuerza
distinta frecuencia si se varía la naturaleza de 1a periódica a este sistema, la amplitud resultante del
sustancia, las dimensiones o la forma de excitación. movimiento del sistema será mayor cuando la
frecuencia de la fuerza aplicada sea igual o
La frecuencia fundamental depende de la velocidad aproximadamente igual a una de las frecuencias
de propagación. Esta observación es válida para los naturales del sistema, que cuando la fuerza excitadora
tubos sonoros ya que, modificando la naturaleza y las se aplique en alguna otra frecuencia.
condiciones del gas, se modifica la velocidad de Las correspondientes frecuencias naturales de
propagación. oscilación de un sistema generalmente se conocen
como frecuencias resonantes
DIAPASÓN
Un aparato de aplicación en acústica es el diapasón Experimento de resonancia. En la figura se muestran
que consta de una barra metálica en forma de “U”, dos diapasones montados en sendas cajas de igual
soportada en su parte media. longitud, lo que indica que ambos tienen igual
frecuencia. A estas cajas se las llama de resonancia,
pues tienen 1a misma longitud que un tubo sonoro
capaz de emitir la misma nota que el diapasón.

Si se lo excita, entra en vibración formándose una


onda estacionaria; los nodos estarán ubicados a 2/3 de
su longitud al emitirse el sonido fundamental. La
frecuencia del diapasón depende de la elasticidad del Enfrentadas las aberturas de las cajas y excitado un
material y su densidad. diapasón, se comprueba que el otro entra
espontáneamente en vibración, En efecto, si se detiene
con la mano el diapasón excitado en un principio, se
percibe nítidamente el sonido producido por el otro y,

28
Movimiento ondulatorio y ondas Hugo Medina Guzmán

si se libera el diapasón detenido, éste vuelve a vibrar, de la misma amplitud A, pero de frecuencias diferentes
lo que podrá percibirse acercando levemente la mano a f1 y f2. El principio de superposici6n establece que la
las ramas del diapasón. Se ha producido un fenómeno amplitud combinada y es la suma algebraica de las
de resonancia acústica. amplitudes individuales.
Si existe un cristalero cerca podrá comprobar que
algunas copas mantienen la vibración por más tiempo
que otras y que durante algunos instantes la amplitud
de la vibración va en aumento. Lo que sucede es que
un cuerpo puede vibrar, entre otras razones, por la
recepción de ondas. Como cada cuerpo tiene una
frecuencia propia d vibración, si ésta coincide con la
de la onda recibida la vibración se mantiene

ONDAS DE DIFERENTE FRECUENCIA


VIAJANDO EN EL MISMO ESPACIO
La figura ilustra la suma de dos ondas sinusoidales de
frecuencia y amplitud diferentes. Esta onda resultante
mantiene la frecuencia del componente más grave,
pero con el timbre alterado
y = y1 + y 2

y total = A sen 2πf1t + Asen 2πf 2 t

Usando la identidad trigonométrica para la suma de los


senos de dos ángulos, tenemos

⎡ ⎛ f − f2 ⎞ ⎤ ⎛ f + f2 ⎞
y total = ⎢2 A cos 2π ⎜ 1 ⎟t ⎥sen 2π ⎜ 1 ⎟t
⎣ ⎝ 2 ⎠⎦ ⎝ 2 ⎠
Esta ecuación representa una onda que oscila con el
promedio de sus frecuencias. La amplitud resultante
también oscila, con una frecuencia de pulsación igual a
PULSACIONES O BATIDOS. la diferencia entre las frecuencias de la fuente.

Cuando dos fuentes de sonido que tienen casi la


misma frecuencia se hace sonar al mismo tiempo,
ocurre un efecto interesante. Puede oír un sonido con
una frecuencia que es el promedio de las dos. Sin
embargo, la sonoridad de este sonido crece
repetidamente y después decae, en lugar de
permanecer constante. Estas variaciones repetidas en
amplitud se denominan pulsaciones o batidos, y la
ocurrencia de pulsaciones es una característica general
de las ondas.

Si la frecuencia de una de las fuentes de ondas se


cambia, hay un cambio que corresponde en el grado en ⎛ f − f2 ⎞
que varía la amplitud. Este grado se llama frecuencia
La primera parte de ytotal es 2 A cos 2π ⎜ 1 ⎟t ,
de pulsación. A medida que las frecuencias se hacen
⎝ 2 ⎠
más cercanas, la frecuencia de pulsación se hace más esto da la amplitud de las pulsaciones que varían
lenta. Así, un músico puede afinar una guitarra a otra lentamente, como se indica en la figura anterior.
fuente de sonido escuchando las pulsaciones mientras
incrementa o disminuye la tensión en cada cuerda. A la Como el sonido alto se escucha siempre que el término
postre, las pulsaciones se hacen tan lentas que sea 2A o – 2A, la frecuencia de las pulsaciones es
efectivamente se desvanecen, y las dos fuentes están f p = f 1 − f 2 . Finalmente, las oscilaciones rápidas
en un tono.
con cada pulsación son debidas a la segunda parte de
⎛ f + f2 ⎞
Las pulsaciones se pueden explicar con facilidad y total , sen 2π ⎜ 1 ⎟t
considerando dos ondas sinusoidales y1 e y2 a partir ⎝ 2 ⎠
29
Movimiento ondulatorio y ondas Hugo Medina Guzmán

Estas oscilaciones tienen una frecuencia que es el Δf es debida a la diferencia de tensión ΔT .


promedio de las dos frecuencias de las fuentes. Las
pulsaciones se pueden escuchar hasta frecuencias de De la ecuación anterior obtenemos.
alrededor de 10 Hz. Más allá son difíciles de distinguir df 1 1 1 1 T 1 f
= = =
dT 2 L 2 Tμ 2 L μ 2T 2T
Ejemplo 47. Cuando se golpea un diapasón de 440 Hz
al mismo tiempo que se pulsa la cuerda de una guitarra df 1 dT
que debe dar la nota Sol, se escuchan 3 pulsaciones por
⇒ =
f 2 T
segundo. Después de que la cuerda de la guitarra se
tensa un poco más para aumentar su frecuencia las
Δf 1 ⎛ ΔT ⎞
pulsaciones aumentan a 6 por segundo. ¿Cuál es la Como Δf << f , tenemos = ⎜ ⎟.
frecuencia de la guitarra con la tensión final? f 2⎝ T ⎠

Solución. En este caso f = 100 Hz y Δf = 1 Hz .

En este fenómeno de interferencia de dos ondas ΔT Δf ⎛ 1 ⎞


sonoras de frecuencia parecida se producen Luego =2 = 2⎜ ⎟ = 2 por ciento.
f1 + f 2 T f ⎝ 100 ⎠
pulsaciones que el oído percibe con un tono
2 (si la cuerda desafina es “baja” su tensión debe ser
y una amplitud que oscila con f p = f 1 − f 2 . El aumentada; si la cuerda es “alta” su tensión se debe
bajar.)
oído responde a la intensidad de la onda sonora que
depende del cuadrado de la amplitud, es decir el INTERFERENCIA DE DOS ONDAS QUE
sonido será fuerte tanto para amplitud máxima como VIAJAN EN DISTINTAS DIRECCIONES
para amplitud mínima. Es decir para el oído la Una causa corriente que origina una diferencia de fase
frecuencia de batido es Δf . entre dos ondas sonoras, es la diferencia de longitudes
de los trayectos que deben recorrer las ondas desde su
En este caso y dado que la frecuencia de batido fuente o foco hasta el punto donde se produce la
percibida por el oído es 3 s-1, la frecuencia original interferencia. Supóngase que tenemos dos focos que
emitida por la cuerda es 437 Hz ó 443 Hz. están emitiendo ondas armónicas de la misma
frecuencia y longitud de onda. En la figura a
continuación, el espectador (a) recibe los dos sonidos
La frecuencia de oscilación de la cuerda es en fase, el observador (b) recibe los sonidos con
directamente proporcional a la velocidad de diferencia de fase.
transmisión de las ondas de la cuerda que a su vez
depende de la raíz cuadrada de la tensión de la cuerda.
Por tanto al aumentar la tensión de la cuerda,
aumentamos la frecuencia de oscilación.

En este caso, al aumentar la tensión, aumenta la


frecuenta de batidos a 6 s-1. Por tanto este hecho
implica que la frecuencia original de la cuerda era de
443 Hz y después de aumentar la tensión es de 446
Hz. Si hubiese sido 437 Hz se detectaría un
decremento en la frecuencia de batido. En el caso general, podemos escribir las funciones de
onda como:
Ejemplo 48. Algunas de las notas bajas del piano
tienen dos cuerdas. En una nota particular una de las y1 = Asen (kr1 − ωt − ϕ ) , y 2 = Asen (kr2 − ωt )
cuerdas se templa correctamente a 100 Hz. Cuando las La diferencia de fase para estas dos funciones de onda
dos cuerdas suenan juntas, se oye un batido por está dada por:
segundo. ¿En qué porcentaje debe un afinador de
piano cambiar la tensión de la cuerda desafinada para 2π
hacerla coincidir correctamente? (el batido es entre los δ = k (r2 − r1 ) − ϕ ⇒ δ = (r2 − r1 ) − ϕ
tonos fundamentales) λ
Solución. Este término se debe a:
1 T La diferencia de fase inicial entre y1 e y2;
La frecuencia fundamental es f = .
2L μ Si las ondas están oscilando en fase, en t = 0 y r = 0,
Asumimos que las dos cuerdas son de la misma entonces ϕ = 0.
longitud, composición y diámetro, tal que la diferencia

30
Movimiento ondulatorio y ondas Hugo Medina Guzmán

Realizando la composición de movimientos onda coincidirá con el mínimo de la otra y la


obtenemos para la amplitud de la onda resultante: interferencia será destructiva.

A= A12 + A22 + 2 A1 A2 cos δ , siendo el término


de interferencia: 2 A1 A2 cos δ .

Estudiamos ahora los máximos y mínimos a partir del


término de interferencia.
La diferencia para los caminos recorridos por las dos
ondas es

δ = k (r2 − r1 ) − ϕ ⇒ δ = (r2 − r1 ) − ϕ
λ Para cumplir con esta condición
1. Interferencia constructiva: Si la diferencia entre
los caminos recorridos por ambas ondas hasta un cierto δ = (2n + 1)π ⇒ cos δ = −1 n = 0 , ± 1, ± 2,
punto es un número entero de longitudes de onda, la .......
interferencia es constructiva

δ= (r2 − r1 ) − ϕ = (2n + 1)π ⇒
λ
(r2 − r1 ) = λ [(2n + 1)π + ϕ ]

En conclusión, para que ocurra la interferencia
constructiva la diferencia de caminos debe ser:

⎡ ϕ ⎤
Δr = r2 − r1 = ⎢(2n + 1) + λ
⎣ 2π ⎥⎦
Para cumplir con esta condición La amplitud de la onda resultante será:
δ = 2nπ ⇒ cos δ = 1 n = 0 , ± 1, ± 2, ....... A= A12 + A22 − 2 A1 A2

δ= (r2 − r1 ) − ϕ = 2nπ ⇒ A= ( A1 − A2 )2 = A1 − A2
λ
(r2 − r1 ) = λ (2nπ + ϕ ) Si A1 = A2 ⇒ A = 0

Resumiendo, si ϕ = 0 :
En conclusión, para que ocurra la interferencia
constructiva la diferencia de caminos debe ser: Condición de máximo: (r1 − r2 ) = nλ
⎛ ϕ ⎞
Δr = r2 − r1 = ⎜ n +

⎟λ
2π ⎠ Condición de mínimo: (r1 − r2 ) = (2n + 1) λ
2
La amplitud de la onda resultante será: APLICACIONES. Cuando se construye una sala de
conciertos hay que tener en cuenta la interferencia
A= A12 + A22 + 2 A1 A2 entre ondas de sonido, para que una interferencia
destructiva no haga que en algunas zonas de la sala no
A= ( A1 + A2 )2 = A1 + A2 puedan oírse los sonidos emitidos desde el escenario.

Si A1 = A2 ⇒ A = 2A1
Por tanto se puede afirmar que una diferencia en los
trayectos de una longitud de onda o de un número
entero cualquiera de longitudes de onda es equivalente
a que no haya ninguna diferencia en absoluto entre las
trayectorias.
2. Interferencia destructiva: Si la diferencia de
trayectos es una semilongitud de onda o un número
impar de semilongitudes de onda, el máximo de una
31
Movimiento ondulatorio y ondas Hugo Medina Guzmán

Arrojando objetos al agua estancada se puede observar


la interferencia de ondas de agua, que es constructiva
en algunos puntos y destructiva en otros. La
interferencia puede producirse con toda clase de ondas,
no sólo ondas mecánicas.

λ
x 2 − x1 = (2n + 1)
2
Según los valores dados:
x 2 − x1 = 101,2 − 100 = 1,2 m
λ 0,80
Las ondas de radio interfieren entre sí cuando se y = = 0,40 m
reflejan en los edificios de las ciudades, con lo que la 2 2
λ
Luego x − x1 = 1,2 = 3(0,40 ) = 3
señal se distorsiona. La luz visible está formada por
ondas electromagnéticas que pueden interferir entre sí. 2
La interferencia de ondas de luz causa, por ejemplo,
Por tanto, el aparato no registrará el sonido
las irisaciones que se ven a veces en las burbujas de
jabón. La luz blanca está compuesta por ondas de luz
Ejemplo 50. Dos parlantes S1 y S2 están separados
de distintas longitudes de onda. Las ondas de luz
una distancia de 5m, están conectados a un oscilador
reflejadas en la superficie interior de la burbuja
de audio. Un muchacho está en el punto P, a 12,0 m de
interfieren con las ondas de esa misma longitud
S1 y 13,0 m de S2: Formando un triángulo rectángulo
reflejadas en la superficie exterior. En algunas de las
S1, S2 y P. La onda de S2 llega al punto P, 2,00
longitudes de onda, la interferencia es constructiva y
periodos después que la onda de S1. La velocidad del
en otras destructiva. Como las distintas longitudes de
sonido es 350 m/s.
onda de la luz corresponden a diferentes colores, la luz
a) ¿Cuál es la frecuencia del oscilador?
reflejada por la burbuja de jabón aparece coloreada.
b) Si el muchacho camina alejándose de S1 por la línea
que pasa por P, hasta que una interferencia destructiva
ocurre. ¿En qué punto medido desde S1 la onda de S2
llega 1,50 periodos más tarde?

Solución.
1
a) 2λ = 13 − 12 = 1m ⇒ λ= = 0,5m
2
v 350
f = = = 700 Hz
λ 0,5
Ejemplo 49. Dos focos sonoros emiten b)
simultáneamente ondas de la misma frecuencia f = 425
Hz, siendo la velocidad del sonido en el aire v = 340
m/s. Si colocamos un aparato registrador de sonidos a
x1 = 100 m del primer foco y a x2 = 101,2 del segundo
¿Se registrará sonido en el aparato?
Solución.
La longitud de onda del sonido emitido por ambos
focos es
v 340
λ= =
f 425
= 0,80 m
(x + 1,5λ )2 − x 2 = 5 2 ⇒
Para que el aparato no registrara sonido sería preciso x 2 + 3λx + (1,5λ ) 2 − x 2 = 25
que en el punto donde está situado se produzca un
con λ = 0,5 m : 1,5 x + (0,75) = 25
2
mínimo de interferencia. De otra manera, R deberá
estar situado en un punto cuya diferencia de distancias finalmente x = 16,3 m
a S1 y S2 sea igual a un múltiplo impar de
semilongitudes de onda: Ejemplo 51. Dos altavoces se excitan mediante el
mismo oscilador a una frecuencia de 2000 Hz. La
separación entre los altavoces es de 3 m, como se
muestra en la figura. Un escucha está originalmente en

32
Movimiento ondulatorio y ondas Hugo Medina Guzmán

el punto O, situado a 8 m medidos sobre el eje axial Solución.


central. ¿Cuánto debe caminar el oyente a)
perpendicularmente a ese eje, antes de alcanzar el
primer mínimo en la intensidad sonora?

2
⎛ 16 ⎞ 4 2 + 32
r1 = ⎜ ⎟ + 4 = 16
2

⎝3⎠ 4 2 32 ( )( )
Solución. ⎛ 5 ⎞ 20
Puesto que la velocidad del sonido en el aire es 330 = 16⎜ ⎟ =
m/s y ya que f = 2000 Hz, la longitud de onda es ⎝ 12 ⎠ 3
v 330 ⎛ 16 7 ⎞
2
λ= = = 0,165 m r2 = ⎜ − ⎟ + 4 2 = 3 2 + 4 2 = 5
f 2000 ⎝ 3 3⎠
El primer mínimo ocurre cuando las ondas que
20 5 5 1
alcanzan el punto P están 180º fuera de fase, o cuando r1 − r2 = − 5 = = 10λ ⇒ λ = = m
la diferencia de trayectos, r2 − r1 , sea igual a λ 2  3 3 30 6
Por lo tanto, la diferencia de fase se obtiene de v 340
Luego la frecuencia f = s = = 2040 Hz
λ 0,165 λ 1/ 6
Δr = r2 − r1 = = = 0,0835 m
2 2 b) Si ahora el observador se ubica sobre el eje x
Del pequeño triángulo rectángulo de la figura del Como la separación entre las fuentes es 7/3 m y la
enunciado se observa que para una buena longitud de onda es 1/6, hay un número exacto de
aproximación, sen θ = Δr/3 para pequeños valores ⎛7/3 ⎞
de θ o sea
longitudes de onda entre las fuentes ⎜ = 14 ⎟ .
⎝ 1/ 6 ⎠
Δr 0,0825
senθ = = = 0,0275 - Entre los parlantes.
3 3 y1 = A sen (kx − ω t ) , y 2 = Asen (kx + ω t )
θ = 1,58º 2π 2π
Del triángulo rectángulo grande de la misma figura se k= = = 12π , ω = 2π f = 4080π
encuentra que tan θ = y/8, o sea
λ 1/ 6
y = 8 tan θ = 8 tan 1,58º = 0,22 m y = y1 + y 2 = Asen (kx − ω t ) + Asen (kx + ω t )
Es decir, el oyente escuchará mínimos en la intensidad = 2 Asenkx cos ω t
sonora resultante a 22 cm desde cualquier lado de la Se forman ondas estacionarias.
línea central. Si el escucha permanece en estas - A la izquierda de S1.
posiciones, ¿en qué otras frecuencias se escucharán y = y2 + y2 = 2 Asen (kx + ω t )
mínimos?
Hay interferencia constructiva.
- A la derecha de S2.
Ejemplo 52. Dos parlantes S1 y S2 son activados por
el mismo sistema de audio emitiendo simultáneamente y = y1 + y1 = 2 Asen (kx − ω t )
ondas sonoras armónicas idénticas de frecuencia “f” Hay interferencia constructiva.
que llegan a un observador en P.
Los parlantes S1 y S2 se encuentran en el origen (0, 0)
m y en (7/3, 0) m, respectivamente, mientras el
observador está en (16/3, 4) m, (vs = 340 m/s).
a) Si la onda emitida por S1 llega 10 periodos más
tarde que la emitida por S2. ¿Qué frecuencia emiten los
parlantes?
b) Si ahora el observador se ubica sobre el eje x, diga
justificando que fenómeno ondulatorio percibe entre
las regiones:
- Entre los parlantes.
- A la izquierda de S1.
- A la derecha de S2.

33
Movimiento ondulatorio y ondas Hugo Medina Guzmán

EFECTO DOPPLER λ . Sin embargo la velocidad se ha incrementado a


La mayoría de nosotros estamos familiarizados con la
v ′ = v + vo .
elevación y descenso posterior del tono de la sirena de
una ambulancia o la bocina de un automóvil cuando Entonces, v ′ = λf ′ , y de aquí
éste se aproxima y cuando ha pasado. Este cambio en
v ′ v + vo
el tono, debido al movimiento relativo entre una f′= =
fuente de sonido y el receptor se llama el efecto de λ λ
Doppler, en honor del físico austriaco Christian Como λ = v f , reemplazando, tenemos
Doppler (1803 -1853). Si usted escucha
cuidadosamente el efecto Doppler, usted notará que el v′ v + vo ⎛ v + vo ⎞ ⎛ v ⎞
f′= = = f⎜ ⎟ = f ⎜1 + o ⎟
tono aumenta cuando el observador y la fuente se λ v f ⎝ v ⎠ ⎝ v ⎠
acercan y disminuye cuando se alejan. Uno de los
Notamos que f’ es mayor que f.
aspectos más interesantes del efecto Doppler es el
En el caso de alejarse de la fuente sonora el
hecho que se aplica a todos los fenómenos
observador, para el observador el sonido parecerá tener
ondulatorios, no solamente al sonido. La frecuencia de
la luz también experimenta el efecto Doppler cuando una velocidad reducida v ′ = v − v o , repitiendo los
hay movimiento relativo entre la fuente y el receptor. cálculos encontramos que
Para la luz, este cambio en frecuencia significa un
v′ v − vo ⎛ v − vo ⎞ ⎛ v ⎞
cambio en color. En efecto, la mayoría de las galaxias f′= = = f⎜ ⎟ = f ⎜1 − o ⎟
distantes se observan cambiadas a rojo lo que significa λ v f ⎝ v ⎠ ⎝ v ⎠
que se están moviendo alejándose de la tierra. Algunas
galaxias, sin embargo, se están moviendo hacia En general el Efecto Doppler para un observador en
nosotros, y su luz muestra un cambio a azul. movimiento es
En el resto de esta sección, nos centramos en el efecto
⎛ v ⎞
Doppler en ondas acústicas. Demostramos que el f ′ = f ⎜1 ± o ⎟
efecto es diferente dependiendo de si el observador o ⎝ v ⎠
la fuente se está moviendo. Finalmente, el observador El signo mas ( + ) corresponde cuando el observador se
y la fuente pueden estar en el movimiento, y mueve hacia la fuente, y el signo menos ( – ) cuando
presentamos los resultados para tales casos también. el observador se aleja de la fuente.
Observador en movimiento Fuente en movimiento
Si tenemos una fuente sonora estacionaria en aire Con el observador estacionario y la fuente en
quieto. El sonido radiado es representado por frentes movimiento, el efecto Doppler no se debe a que el
de onda circunferenciales que se alejan de la fuente sonido parece tener una mayor o menor velocidad
con una velocidad v. La distancia entre los frentes de como en el caso del observador en movimiento. Por el
onda es la longitud de onda λ , y la frecuencia del contrario, el sonido una vez que la fuente emite una
sonido es f. Para las ondas estas cantidades están onda sonora, viaja a través del medio con su velocidad
relacionadas por v = λf . característica v, sin importar lo que la fuente haga.
Para un observador que se acerca con una velocidad Por analogía, consideremos una onda de agua. La
figura muestra una bolita oscilando hacia arriba y hacia
v o , como se muestra en la figura, el sonido parece
abajo sobre la superficie de con recipiente de agua. Su
tener una mayor velocidad v + v o (considerando que movimiento causa ondas de agua circulares que se
la velocidad del sonido relativa al aire es siempre la extienden alejándose del punto de contacto.
misma). Como resultado llegan al observador en un
determinado tiempo un mayor número de frentes de
onda que si hubiera estado en reposo. Para el
observador el sonido tiene una frecuencia, f’, que es
más alta que la frecuencia de la fuente, f.

Cuanto la fuente de ondas (la bolita) es movida hacia


la derecha (vea la figura), cada cresta de onda se
mueve alejándose como un círculo en expansión, pero
dado que la fuente se está moviendo, emite cada onda
a una ubicación diferente. Como resultado, las ondas
que se mueven en la misma dirección que la fuente se
juntan apretándose, en tanto que las que se mueven en
Podemos encontrar la frecuencia f’ notando que la la dirección opuesta se separan alejándose unas de
longitud de onda del sonido no cambia, sigue siendo otras. La velocidad de la onda es constante si la fuente
se mueve o no. Por tanto, donde la longitud de onda la

34
Movimiento ondulatorio y ondas Hugo Medina Guzmán

frecuencia es incrementada, y donde la longitud de


onda es alargada, la frecuencia es reducida. Esto es
válido también para las ondas de sonido.

Consideremos Una fuente que se mueve hacia el


observador con una velocidad v f tal como muestra la
f , emite un
figura. Si la frecuencia de la fuente es
frente de onda cada T segundos, donde T = 1 f .
Por consiguiente, durante un periodo un frente de onda
viaja una distancia vT mientras que la fuente emisora
viaja una distancia v f T . Como resultado el siguiente Finalmente
Si ambas se mueven:
frente de onda es emitido a una distancia vT − v f T (v + vo )
Acercándose uno a otro f ' = f
detrás del frente previo, como muestra la figura.
(v − v )
f

(v − vo )
Alejándose uno a otro f '= f
v + vf
La situación es más complicada para otras direcciones,
pero puede analizarse sencillamente si la distancia de
la fuente al punto de observación es grande comparada
Esto significa que la longitud de onda hacia delante es con la longitud de onda. En este caso se llega al
λ ′ = vT − v f T = (v − v f )T
resultado siguiente

Como mencionamos antes, la velocidad de la onda


sigue siendo v , de aquí
v = λ ′f ′
La nueva frecuencia es
v v
f′= =
λ ′ (v − v f )T
⎛ u cos θ ⎞
Considerando que T = 1 f , tenemos λ(θ ) = λ0 ⎜1 − ⎟
⎝ v ⎠
v v 1 o bien
f′= = f
(v − v )(1 f )
f (v − v f ) ⎛⎜1 − v f
= f

⎟⎟ f (θ ) = f 0
1
para la frecuencia.
⎜ ⎛ u cos θ ⎞
⎝ v ⎠ ⎜1 − ⎟
f’ es mayor que f, como esperábamos. ⎝ v ⎠
Estas últimas ecuaciones nos indican que el efecto
En general el Efecto Doppler para una fuente en Doppler depende de la componente de la velocidad de
movimiento y observador estacionario es la fuente en la dirección del observador
1
f′= f APLICACIÓN: Medidor de flujo doppler
⎛ vf ⎞
⎜⎜1 m ⎟⎟ Este dispositivo mide la velocidad del torrente
⎝ v ⎠ sanguíneo y constituye una aplicación interesante del
efecto Doppler. El dispositivo consta de elementos
El signo menos ( - ) corresponde cuando la fuente se
transmisor y receptor colocados directamente sobre la
mueve hacia el observador, y el signo menos ( – )
piel, como se observa en la figura. El transmisor emite
cuando la fuente se aleja del observador.
una onda sonora continua cuya frecuencia es de 5
MHz. Cuando los g1óbulos rojos reflejan el sonido su
frecuencia cambia como si se presentara el efecto
Doppler, debido a que se mueven las células. El

35
Movimiento ondulatorio y ondas Hugo Medina Guzmán

elemento receptor detecta el sonido reflejado y el


contador electrónico mide su frecuencia, que está
corrida por el efecto Doppler con respecto a la
frecuencia de transmisión. A partir del cambio en
frecuencia es posible determinar la rapidez con q fluye
el torrente sanguíneo. Por lo general, el cambio en
frecuencia es aproximadamente 6 000 Hz para
a) La velocidad de la fuente v f respecto al
rapideces de flujo aproximadamente iguales a 0,1 m/s.
El medidor de flujo Doppler puede usarse para observador será:
localizar regiones en las que los vasos capilares se
estrechan, ya que en tales regiones se producen
mayores rapideces de flujo, según la ecuación de
continuidad. Además, el medidor de flujo Doppler
puede utilizarse para detectar el movimiento cardiaco
de un feto de apenas 8 a 10 semanas de edad. V
EI efecto Doppler también se emplea en dispositivos v f = V cos 60 o =
de un radar para medir la velocidad de vehículos en 2
movimiento. Sin embargo, se utilizan las ondas ⎛ ⎞
electromagnéticas, en vez de las ondas sonoras, para ⎜ v ⎟
tales propósitos. Luego, f ' = f ⎜ s ⎟
⎜v −V ⎟
⎜ s ⎟
⎝ 2 ⎠
Vemos que aumenta la frecuencia.
b) En este caso, la velocidad de la fuente con respecto
al observador es cero.

Ejemplo 53. La sirena de un auto patrullero


estacionado emite un sonido de 1200 Hz. ¿Bajo
condiciones en que la velocidad del sonido en el aire es f '= f
340 m/s, qué frecuencia oirá un peatón parado si la
sirena se está acercando a 30 m/s? ¿Qué frecuencia
oirá cuando la sirena está alejándose en 30 m/s? c) Como en el caso a) la velocidad de la fuente
V
es: v f =
Solución.
, pero, se aleja del observador , o sea que
Acercándose 2
vf ⎛ 340 ⎞ la frecuencia disminuye :
f '= f =⎜ ⎟(1200) = 1316 Hz
vf −v ⎝ 340 − 30 ⎠
⎛ ⎞
Alejándose ⎜ v ⎟
vf ⎛ 340 ⎞ f '= f ⎜ s ⎟
f '= f =⎜ ⎟(1200) = 1103 Hz ⎜v +V ⎟
vf + v ⎝ 340 + 30 ⎠ ⎜ s ⎟
⎝ 2 ⎠
d) En el caso del niño, la velocidad de la fuente
Ejemplo 54. Una persona se encuentra a una distancia respecto a él es nula, por lo tanto, escucha la
3A 2 de línea férrea, por la cual viene un tren a frecuencia f.
velocidad constante V y tocando una sirena de
Ejemplo 55. Una sirena que emite un sonido de 1000
frecuencia f. La velocidad del sonido en el aire es v s . Hz se aleja de un observador y se aproxima a una
¿Qué frecuencia escucha la persona? Cuando: pared con una velocidad de 10m./seg. .
a) El tren esta acercándose a una distancia A de él. ¿Cuál es la frecuencia que escucha el observador?
b) Cuando se encuentra frente a él. Solución.
c) Cuando se ha alejado una distancia A de él. El observador escucha dos frecuencias:
d) ¿Qué frecuencia escucha un niño que se asoma por La frecuencia directa y la reflejada en la pared.
la ventanilla de uno de los vagones del tren? La frecuencia directa
Solución.
En el gráfico se ve fácilmente la situación geométrica
planteada en el problema.

36
Movimiento ondulatorio y ondas Hugo Medina Guzmán

1 1
f1 ' = f = 1000 = 971 Hz
⎛ vf ⎞ ⎛ 10 ⎞
⎜⎜1 + ⎟⎟ ⎜1 + ⎟
⎝ vs ⎠ ⎝ 330 ⎠
vs 330
Y la frecuencia reflejada en la pared. f3 = f 2 = 858,3 = 806,9 Hz
La frecuencia que llega a la pared es v s + v' 330 + 21
1 1 d) El receptor es el auto y el emisor el camión y la
f2 '= f = 1000 = 1 031 Hz frecuencia emitida f2:
⎛ vf ⎞ ⎛ 10 ⎞
⎜⎜1 − ⎟⎟ ⎜1 − ⎟
⎝ vs ⎠ ⎝ 330 ⎠
Las que sumadas producen pulsaciones de frecuencia
f b = f 2 '− f1 ' = 1031 − 971 = 60 Hz vs − v 330 − 30
Que es lo que escucha el observador. f4 = f2 = 858,3 = 733,6 Hz
v s + v' 330 + 21
Ejemplo 56. Un automóvil se mueve hacia la
izquierda con una velocidad v = 30 m/s. En dirección Ejemplo 57. Al nadar un pato patalea una vez cada 1,6
contraria (rebasado suficientemente el punto de cruce) s produciendo ondas superficiales en el agua, el pato
va un camión a una velocidad v' = 21 m/s, con una avanza con rapidez constante en un estanque en el que
gran superficie reflectora en su parte posterior. El las ondas superficiales viajan a 0,4 m/s. Las crestas de
automóvil emite un bocinazo (emisión instantánea) con las ondas están espaciadas adelante del pato 0,10m
una frecuencia de 1 000 Hz. Determinar: a) Calcule la rapidez del pato.
a) ¿Cuál es la frecuencia de las ondas percibidas por el b) ¿Cuál es el espaciamiento de las crestas detrás del
observador de la figura colocado a la derecha del auto? pato?
b) ¿Cuál es la frecuencia de las ondas que llegan a la Nota: En este problema las ondas son producidas por
superficie reflectora del camión? el pataleo del pato y las ondas se propagan en el agua
c) ¿Cuál es la frecuencia de las ondas que percibirá el (ondas acuáticas).
observador después que las ondas se han reflejado en Solución.
el camión?
d) ¿Cuál es la frecuencia de las ondas que percibiría el
conductor del auto, después de la reflexión en el
camión?
Velocidad del sonido: 330 m/s. Se supone el aire en
calma.

Perturbación producida por el pataleo del pato:


T = 1,6 s. ⇒ f = 1 / 1,6 Hz
Solución. Para la onda: vonda = 0,4 m/s.
a) vonda 0,4
a) Como λ1 ' = 0,1m ⇒ f1 ' = = = 4Hz
λ1 ' 0,1
Efecto doppler de fuente que se mueve, la frecuencia
hacia delante es:
vs 330 ⎛ ⎞
f1 = f 0 = 1000 = 916,7Hz ⎜ ⎟
vs + v 330 + 30 ⎜ 1 ⎟ v f
f1 ' = f ⎜ ⎟ ⇒ 1 − pato =
b) La superficie reflectora es ahora el receptor vpato vonda f1 '
auditivo. ⎜1− ⎟
⎝ vonda ⎠
⎛ f ⎞
⇒ vpato = vonda ⎜⎜1 − ⎟⎟
⎝ f1 ' ⎠
v s − v' 330 − 21 Reemplazando valores:
f2 = f0 = 1000 = 858,3 Hz
vs + v 330 + 30 ⎛ 1 / 1,6 ⎞
c) La superficie reflectora se vuelve foco emisor,
v pato = 0,4⎜1 − ⎟ = 0,3375 m/s.
⎝ 4 ⎠
emitirá con la frecuencia f2.
b) La frecuencia detrás del pato

37
Movimiento ondulatorio y ondas Hugo Medina Guzmán

⎛ ⎞ corto tiempo se capta un pulso de retorno de frecuencia


⎜ ⎟ 62 kHz proveniente de un automóvil.
⎜ 1 ⎟ Determine: la velocidad con que se mueve el carro e
f2 ' = f ⎜
vpato ⎟ indique si el carro se acerca o aleja del patrullero.
⎜1+ ⎟ (Velocidad del sonido = 340 m/s).
⎝ vonda ⎠ Solución.
⎛ ⎞ v s = 340 m/s
⎜ ⎟
1 ⎜ 1 ⎟ = 0,39 Hz
= v x es la velocidad del automóvil.
1,6 ⎜ 0,3375 ⎟
⎜ 1 + 0,4 ⎟
⎝ ⎠ El sonido se emite del patrullero, llega al automóvil y
v 0,4 regresa al patrullero.
λ2 ' = onda = = 1,18 m
f 2 ' 0,39
El espaciamiento de las crestas detrás del pato es de
1,18 m.

Ejemplo 58. El sonar de un patrullero estacionado al


borde de una pista emite hacia delante un corto pulso
f = 60 Hz y f ' ' = 62 Hz
de ultra sonido de frecuencia de 34 kHz .Luego de 1,5 Al automóvil llega
s de haberse emitido las ondas se capta un pulso de
retorno de frecuencia de 32 kHz proveniente de un ⎛ v ± vo ⎞
automóvil. f ' = f ⎜⎜ s ⎟⎟
(vsonido = 340 m/s) ⎝ vs ⎠
a) Determine la velocidad del automóvil
b) A que distancia del patrullero se encontraba el ⎛ 340 ± v x ⎞
automóvil al salir el pulso ⇒ f ' = 60⎜ ⎟ (1)
Solución.
⎝ 340 ⎠
a) Determine la velocidad del automóvil + o -, dependiendo si el automóvil se acerca o aleja del
La frecuencia que llega al automóvil
patrullero en reposo.
⎛ v ⎞ ⎛ v ⎞
f1 ' = f ⎜⎜1 − o ⎟⎟ = 34000⎜1 − 0 ⎟ Al patrullero llega de retorno
⎝ vs ⎠ ⎝ 340 ⎠
Esta frecuencia vuelve al patrullero ⎛ vs ⎞
f '' = f '⎜ ⎟
1 ⎛ v0 ⎞ ⎛ 340 − vo ⎞ ⎜v mv ⎟
f 2 ' = f1 ' ⎜1 − ⎟⎟ = 34000⎜⎜ ⎟⎟ ⎝ s f ⎠
⎛ vo ⎞ ⎜⎝ v s ⎠ ⎝ 340 + vo ⎠
⎜⎜1 + ⎟⎟ ⎛ 340 ⎞
⎝ vs ⎠ ⇒ 62 = f ' ⎜⎜ ⎟⎟ (2)
= 32000 Hz ⎝ 340 m v x ⎠
⎛ 340 − vo ⎞ 32 - o +, dependiendo si el automóvil se acerca o aleja del
⇒ ⎜⎜ ⎟⎟ = ⇒
⎝ 340 + vo ⎠ 34 patrullero en reposo.

34(340 − vo ) = 32(340 + vo ) Multiplicando (1) x (2):

⇒ 66vo = 340(34 − 32) 60(340 ± v x )


62 =
340(2 ) (340 m vx )
⇒ vo = = 10,3 m/s
33 La única posibilidad de tener solución es en el caso

b) A que distancia del patrullero se encontraba el que el automóvil se acerca al patrullero.:


automóvil al salir el pulso 60(340 + v x )
Cuando el pulso llega al automóvil han pasado 1,5/2 = 62 = ⇒ v x = 5,57 m/s
0,75 s (340 − v x )
La distancia a la que se encontraba el automóvil del
patrullero fue: 340(0,75) − 10,30(0,75) = 240,28 m. Luego el automóvil se está acercando al patrullero.
v x = 5,57 m/s.
Ejemplo 59. El sonar de un patrullero estacionado al
borde de una pista emite hacia delante un corto pulso
de ultrasonido de frecuencia 60 kHz. Luego de un

38
Movimiento ondulatorio y ondas Hugo Medina Guzmán

Ejemplo 60. Un altavoz es colocado sobre un bloque,


conectado a un resorte (k = 50 N/m), como se muestra
en la figura. La masa total (bloque y altavoz) es 0,5kg;
el sistema oscila con una amplitud de 5 cm. Si el
altavoz emite a 500 Hz, determinar:
a) La frecuencia máxima percibida por la persona.
b) La frecuencia mínima percibida por la persona.
Asumir c = 340m/s 1 1
f′= f = 500 = 499,27 Hz
⎛ vf ⎞ ⎛ 0,5 ⎞
⎜⎜1 + ⎟⎟ ⎜1 + ⎟
⎝ v ⎠ ⎝ 340 ⎠

Ejemplo 61. El carro de bomberos que se mueve con


una velocidad de 20 m/s, hace sonar su sirena que
Solución. tiene una frecuencia de 1000 Hz.
Oscilación de la masa: Un ciclista se acerca al carro alejándose del acantilado
con una velocidad de 10 m/s,
a) ¿Qué sonido escucha el conductor del carro de
bomberos?
b) ¿Qué sonido escucha el ciclista?

k 50 rad
ω= = = 10 Solución.
m 0,5 s a) El conductor escucha dos frecuencias f1 que es la
frecuencia f (1000 Hz) de la sirena y la frecuencia f’
x = Asenω t = 0,05sen10t ⇒ v = 0,5cos10t del eco producido por la presencia del acantilado.
⎧+ 0,5 La frecuencia que llega al acantilado se debe a una
v máx = ⎨ fuente que se acerca con vf = 20 m/s
⎩− 0,7 vs 340
En general el Efecto Doppler para una fuente en f '= f0 = 1000 = 1062,5 Hz
movimiento y observador estacionario es vs − v f 340 − 20
1 Esta frecuencia se refleja y llega a los oídos del
f′= f conductor que se acerca con velocidad vo = 20 m/s.
⎛ vf ⎞
⎜⎜1 m ⎟⎟ (v s + vo ) (340 + 20) = 1125 Hz
⎝ v ⎠ f ''= f ' = 1062,5
vs 340
El signo menos ( - ) corresponde cuando la fuente se
Como las dos frecuencias que escucha son muy
mueve hacia el observador, y el signo menos ( – )
cercanas escuchará batidos o pulsaciones
cuando la fuente se aleja del observador.
correspondientes a las frecuencia f = 1000 Hz y f’’ =
a) La frecuencia máxima percibida por la persona.
1125 Hz.
Es decir f b = 1125 − 1000 = 125 Hz (frecuencia de
los batidos).

1 1 b) Al oído del ciclista llega la frecuencia directa que es


f′= f = 500 = 500,74 Hz con observador y fuente acercándose
⎛ vf ⎞ ⎛ 0,5 ⎞
⎜⎜1 − ⎟⎟ ⎜1 − ⎟ v s + vo 340 + 10
⎝ 340 ⎠ f1 ' ' = f = 1000 = 1093,75 Hz
⎝ v ⎠ vs − v f 340 − 20
b) La frecuencia mínima percibida por la persona.
También llega el eco producido por el acantilado
(v s − vo ) (340 − 10) =
f2 ''= f ' = 1062,5
vs 340
1031,25 Hz

39
Movimiento ondulatorio y ondas Hugo Medina Guzmán

Como las dos frecuencias que escucha son muy rev × 2πrad × min rad
cercanas escuchará batidos ω = 300 = 10π
min× rev × 60s s
f b = 1093,75 − 1031,25 = 62,5 Hz (frecuencia de La velocidad de la fuente emisora es:
los batidos). v = ωl = 10π (2 ) = 20π m/s
Si el observador está lo suficientemente alejado,
tomamos la dirección de percepción en la misma
dirección que tiene la velocidad. Cuando la sirena se
encuentra en la posición A de la figura, tendremos:
Ejemplo 62. Las ondas sonoras emitidas por el carro v 340
f 1′ = f = = 515,2 Hz
de bomberos (A) se reflejan en el camión (B) y al v −vf 340 − 20π
regresar interfieren con las ondas emitidas.
Determinar el número de pulsaciones por segundo que En el punto B:
detecta un observador sobre el móvil A. v 340
fo = 400 Hz
f 2′ = f = = 354,2 Hz
v + vf 340 + 20π
El intervalo será el comprendido entre las dos
frecuencias calculadas

Ejemplo 64. Un carrusel de 5,0 m de radio, tiene un


Solución. par de altoparlantes de 600 Hz montados en postes en
Frecuencia que llega a la pared extremos opuestos de un diámetro. El carrusel gira con
f ' = f 0 Acercándose fuente y alejándose una velocidad angular de 0,80 rad/s. Un observador
estacionario está colocado a cierta distancia enfrente
“observador”
del carrusel. La velocidad del sonido es 350 m/s.
f ' = f0
(v − vB ) (1) a) Calcular la longitud de onda más larga que llega al
(v − vA ) escucha desde las sirenas.
b) Calcular la frecuencia de las sirenas más alta que
Frecuencia que recibe el pasajero
escucha.
Alejándose fuente y acercándose “observador”
c) Calcular la frecuencia de batido máxima en la
f ''= f '
(v + v A ) (2) posición del escucha.
(v + vB ) d) Un escucha montado en una bicicleta que se aleja
Reemplazando (2) en (1): directamente del carrusel con una rapidez de 4,5 m/s.
¿Cuál es la frecuencia de las sirenas más alta que
f ' ' = f0
(v − vB ) (v + v A ) = escucha?
(v − v A ) (v + vB ) Solución.
a)
⎛ c ⎞⎛ c ⎞
⎜c − ⎟ ⎜c + ⎟
15 ⎠ ⎝ 25 ⎠
400 ⎝
⎛ c ⎞⎛ c⎞
⎜c − ⎟ ⎜c + ⎟
⎝ 25 ⎠ ⎝ 15 ⎠
= 400
(15 − 1) (25 + 1) = 400 (14) (26)
(25 − 1) (15 + 1) (24) (16)
= 379,17 Hz.
El pasajero escucha fp = 400 – 379,17 = 20,83
pulsaciones/s
La longitud de onda más larga es con la menor
frecuencia
Ejemplo 63. Una sirena de 420 Hz gira atada al
Y eso sucede cuando la fuente se aleja
extremo de una cuerda de 2 m de longitud a razón de
300 r.p.m. ¿Qué intervalo de frecuencias percibe un vs 350
f1 ' = f = 600
observador situado en el plano de rotación de la sirena
y alejado de ésta? Tomar para velocidad del sonido en
(vs + v f ) (350 + 0,80 × 5)
el aire 340 m/s. 350
= 600 = 593,2 Hz
(354)
350
λ= = 0,59 m = 59 cm.
593,2
b) La frecuencia más alta sucede cuando la fuente se
Solución. acerca.

40
Movimiento ondulatorio y ondas Hugo Medina Guzmán

vs 350 1719,
f2 '= f = 600 tb =
(vs − v f ) (350 − 0,80 × 5,0)
350 d − vb t
= 600 = 606,9 Hz Como t b = , obtenemos:
(346) vb
1719,4 − 3,88(10)
c) f b = f 2 '− f 1 ' = 606,9 – 593,2 = 13,74 Hz tb = = 433,14 s = 7 min 13,14 s.
3,88
d) El tiempo que el buque tardará en alcanzar la costa es
7 min 13,14 s.

Ejemplo 66. Una sirena que emite con una frecuencia


f sube verticalmente hacia arriba, partiendo del suelo y
a una velocidad constante V. El punto de partida de la
sirena está a una distancia d de un observador.
a) Supuesto el observador parado, calcular en función
de los datos la frecuencia que percibiría el observador
después de transcurridos t segundos.
b) Supuesto que el observador se aleja del punto de
partida a una velocidad V', y que parte del punto a esa
distancia d, en el mismo instante que la sirena.
Calcular en función de los datos la frecuencia que
La fuente se acerca y el observador se aleja. percibiría el observador, después de transcurridos t
(v s − vo ) (350 − 4,5) = segundos.
f ''= f = 600
(v s − vf ) (350 − 4) (Velocidad del sonido: v)
Solución.
600
(345,5) a)
(346,0)
Ejemplo 65. Un buque se acerca a una costa acantilada
haciendo sonar una sirena de 600 Hz. El sonido se
refleja en la costa y se oye 10 s después, interfiriendo
con el propio de la sirena, lo que da lugar a 12
pulsaciones por segundo. Calcule con estos datos el
tiempo que el buque tardará en alcanzar la costa.
Solución. Frecuencia cuando la fuente se aleja del observador:
f b = f '− f ⇒ f ' = f + f b = 600 + 12 = 612 Hz v
f '= f ,
(v + vb ) (340 + vb ) (v + v F )
f '= f s = 600 = 612
(v s − vb ) (340 − vb ) v F = V cos ϕ1 = V
Vt
m V 2t 2 + d 2
⇒ vb = 3,88
s De aquí:
Sea d la distancia a la que se encuentra el buque v
cuando inicia el sonido de la sirena, 10 segundos f '= f
⎛ V 2t ⎞
después escucha pulsaciones, esto es el tiempo de viaje ⎜v + ⎟
de las ondas de ida y vuelta. ⎜ ⎟
⎝ V 2t 2 + d 2 ⎠
d
Tiempo de ida: t1 = v V 2t 2 + d 2
vs = f
d − vb t v V 2t 2 + d 2 + V 2t
Tiempo de vuelta: t 2 = b)
vs
Tiempo total:
d d − v b t 2d − v b t
t = t2 + t2 =
+ =
vs vs vs
(v + vb )t (340 + 3,88)10
⇒d= s = = 1719,4 m.
2 2

41
Movimiento ondulatorio y ondas Hugo Medina Guzmán

(v − v' a )
f '= f
[(v − v' a ) − v F ]
va d
v' a = v a cos ϕ = ,
h2 + d 2
Vd
v F = V cos ϕ =
h2 + d 2
Frecuencia cuando la fuente y el observador se alejan
⎛ va d ⎞
mutuamente: ⎜v − ⎟
(v − vO ) ⎜ 2 ⎟
⎝ h 2
+ d ⎠
f '= f f '= f
(v + v F ) ⎛ va d Vd ⎞
⎜v − − ⎟
vO = V ' cos(90º −ϕ 2 ) = V ' senϕ 2 ⎜
⎝ h2 + d 2 h2 + d 2


= V'
V 't + d
(v h 2 + d 2 − va d )
V t + (V ' t + d ) = f
(v )
2 2 2
h 2 + d 2 − v a d − Vd
Vt
vF = V cos ϕ 2 = V
V 2 t 2 + (V ' t + d )
2

⎛ V 't + d ⎞ FORMACION DE UNA ONDA DE CHOQUE


⎜v −V ' ⎟ Hemos visto en el efecto Doppler que los frentes de
⎜ 2 ⎟
V t + (V ' t + d ) ⎠
2 2 onda producidos por una fuente de sonido en
f '= f ⎝ movimiento están comprimidos en la dirección hacia la
⎛ V 2t ⎞ cual está viajando la fuente. A medida que aumenta la
⎜v + ⎟ velocidad de la fuente, la compresión se hace más
⎜ 2 ⎟
⎝ V t + (V ' t + d ) ⎠
2 2 pronunciada. ¿Qué sucede cuando la velocidad de la
fuente empieza a hacerse mayor que la velocidad de la
⎡v V 2 t 2 + (V ' t + d )2 − V ' (V ' t + d )⎤ onda? En este caso, la fuente se mueve más aprisa que
= f ⎣⎢ ⎥⎦ las ondas y los argumentos usados para describir el
⎡v V 2 t 2 + (V ' t + d )2 + V 2 t ⎤ efecto Doppler ya no son aplicables más. En su lugar,
⎢⎣ ⎥⎦ las ondas esf6ricas expandiéndose desde la fuente t
posiciones posteriores a lo largo de la trayectoria de la
Ejemplo 67. Un hombre se encuentra en lo alto de fuente, se combinan todas formando un frente de onda
una torre de altura h. A una distancia d del pie de ésta, único cónico que se conoce como onda de choque
un automóvil que se dirige hacia ella con una (véase la figura). Como la onda de choque está
velocidad V emite un bocinazo con una frecuencia f. compuesta por muchos frentes de onda actuando
El aire se mueve con una velocidad v a y en dirección juntos, tiene una gran amplitud.
contraria al automóvil. Calcular en función de estos
datos la frecuencia percibida por el hombre de la torre.
(Velocidad del sonido: v).
Solución.

Para el tiempo t = 0 la fuente emite una onda desde el


punto O En un tiempo posterior t, el frente de la onda
se ha expandido a un radio r = vt y la fuente ha viajado
a una distancia v,t para alcanzar al punto S. Frentes de
onda posteriores también se expanden como se indica
La velocidad del sonido en la dirección de percepción en la figura anterior, de manera que a ese tiempo t
está afectada por la velocidad del viento, de tal modo alcanzan justamente la línea tangente que se dibuja
que bajo estas condiciones v s = (v − v ' a ) desde S al frente de onda centrado en O. La envolvente
resultante de frentes de onda forma un cono con un
La frecuencia que percibe un observador en reposo con
la fuente acercándose, bajo estas condiciones es: semiángulo θ dado por

42
Movimiento ondulatorio y ondas Hugo Medina Guzmán

vt v
senθ = =
vft vf
La relación v f v , llamada número Mach, se usa
frecuentemente para dar la velocidad en términos de la
velocidad del sonido. Así una velocidad de 1,5 veces la
velocidad del sonido se denota como Mach 1,5.
Cuando la onda de choque es producida por un
aeroplano que se mueve a una velocidad mayor que la
velocidad del sonido, es decir, a velocidad supersónica, El gráfico muestra que la presión de aire se eleva
la onda de choque se conoce como explosión sónica. bruscamente a lo largo de la onda de choque formada
por la parte delantera de la nave. Luego la presión cae
En la figura siguiente se muestra la onda de choque por debajo de la presión atmosférica y nuevamente se
producida en el aire por un aeroplano supersónico que eleva bruscamente a lo largo de las ondas de choque
se mueve a Mach 1,1. Nótese que además de la onda formadas por la parte posterior de la nave. (Los frentes
de choque producida en el extremo frontal, en la parte de onda son curvos porque la velocidad del sonido
posterior del aeroplano aparecen ondas de choque depende de la temperatura del aire y la temperatura
menores. Una nave a alta velocidad produce dos o más varía con la altura.). La segunda elevación vuelve a la
ondas de choque, las cuales están asociadas con la normalidad a la presión.
nariz, la cola y otras proyecciones de la nave. El tiempo entre los dos cambios de presión es 1/30 de
Los aviones supersónicos producen ondas de choque segundo, de tal manera que se escucha un simple
que se escuchan como explosiones sónicas. “Bum” cuando las ondas de choque pasan.

PREGUNTAS Y PROBLEMAS

1. Un joven en un barco mira las ondas en un lago que amplitud de las oscilaciones es 0,1 mm y la frecuencia
pasan con una pausa de medio segundo entre cada es 10 oscilaciones por segundo. Hallar:
cresta, ¿Si a una onda le toma 1,5 s pasar los 4,5 m de a) ecuación de las ondas que se propagan a lo largo de
longitud de su barco de 4,5 m, cuál es la velocidad, la la barra.
frecuencia, el período, y la longitud de onda de las b) energía por unidad de volumen.
ondas? c) promedio del flujo de energía por unidad de tiempo
a través de una sección cualquiera de la barra.
2. Los delfines se comunican bajo el agua usando d) potencia requerida para operar el oscilador.
ondas de compresión de alta frecuencia. ¿Si la
velocidad del sonido en agua es 1,4 x 103 m/s y la 6. La ecuación de una onda transversal en una cuerda
longitud de onda promedio es 1,4 centímetros, cuál es es y = 1,8 mm sen[( 23,8rad/m) x + (317 rad/s)t ] . La
la frecuencia típica del sonido de un delfín? ¿Es esta cuerda se encuentra sometida bajo una tensión de 16,3
frecuencia audible a los seres humanos? N. Determinar la densidad lineal de masa.
3 La cavidad del pecho de un ser humano resuena 7. En una eco encefalografía se aplica una señal de
alrededor de 8 Hz. ¿qué longitud de onda causa tal ultrasonido para detecta la respuesta de un obstáculo
vibración? (hueso, tumor, etc.). Suponga la disposición de la
figura. Calcule el tiempo que emplea el ultrasonido
4. La ecuación de una onda transversal en una cuerda para obtener un eco en la segunda capa ósea (hueso 2).
es Se sabe que la velocidad de propagación de las ondas
y = 6,0 cm sen[(2,0π rad/m) x + (4,0π rad/s)t ] . sonoras en el hueso es de 3,370 m/s y en el tejido
Calcule: encefálico, de 1550 m/s
a) amplitud
b) longitud de onda
c) frecuencia
d) velocidad de propagación
e) dirección de propagación de la onda
f) La velocidad transversal máxima de una partícula de
la cuerda.

5. Una barra de acero transmite ondas longitudinales


por medio de un oscilador acoplado a uno de sus
extremos. La barra tiene un diámetro de 4 mm. La

43
Movimiento ondulatorio y ondas Hugo Medina Guzmán

Respuesta. 1,35 x 104 m/s 14. La cuerda de un violín de 30 cm de longitud emite


8. Calcule la distancia a la cual un nadador debe un sonido de 460 Hz. Al fijarla en un punto tal que su
encontrarse respecto del fondo (o de un obstáculo) para longitud disminuya a 25 cm, emite un nuevo sonido.
percibir el fenómeno del eco de un sonido producido Calcule su frecuencia.
por él mismo (velocidad del sonido en agua 1640 m/s). Respuesta. f = 552 Hz
Respuesta. 82 m
15. Un diapasón emite un sonido de frecuencia
9. La velocidad de propagación en un gas y en un constante. Este diapasón (vibrando) se coloca sobre un
líquido a la misma temperatura es de 330 m/s y 1500 tubo cilíndrico de vidrio que contiene agua. El nivel de
m/s respectivamente. Un dispositivo, por ejemplo, un agua puede variar observándose que, para ciertas
diapasón, produce ondas sonoras en ambos fluidos de alturas h de la columna de aire en el tubo, la intensidad
420 Hz. del sonido es mucho mayor que para otras. Las alturas
Halle la relación de longitudes de onda en el líquido para las -cuales existe resonancia son h1 = 12 cm; h2 =
respecto del gas y la longitud de onda del sonido en 36 cm; h3 = 60 cm. Calcule la longitud de la onda
cada medio. emitida por el diapasón.
Respuesta. Respuesta. λ = 48 cm
λlíquido
a) = 4,55 , b) λgas = 0,786m , 16. Una probeta tiene 80 cm de profundidad y recibe
λ gas
la mayor cantidad de agua para que el aire contenido
c) λllíquido
= 3,57m en el tubo entre en resonancia con un diapasón que
emite una onda sonora de período T = 32 x 10-1 s
10. Una ventana de 1,5 m2 se abre en una calle donde (valor de la velocidad del sonido en aire en las
el ruido propio produce un nivel sonoro en la ventana condiciones del problema, 340 m/s).
de 60 dB. Determine la potencia acústica que entra por Calcule la profundidad del agua.
la ventana mediante ondas sonoras. Respuesta. 0,52 m
Respuesta. 1,5 x 104 W
17. Un altoparlante se coloca en un punto A de una
11. Un alambre se doblado en un lazo circular del caja rectangular de sección C D E F. El sonido emitido
diámetro D. se asegura por medio de una abrazadera es de 120 Hz. Suponga que un micrófono A, puede
por los extremos opuestos. Se envía una onda desplazarse a lo largo de la línea AB. El micrófono es
transversal alrededor del lazo por medio de un vibrador conectado a un registrador de intensidad sonora.
pequeño que actúe cerca de la abrazadera. Encuentre Mediante este experimento se logra demostrar que en
las frecuencias de resonancia del lazo en los términos A y en puntos contados a partir de A hacia la derecha
de la velocidad v de la onda y el diámetro D. cada 1,20 m se registra un máximo e intensidad
Respuesta. Los soportes del lazo forman nodos en dos sonora.
D λ πD a) calcule la longitud de onda sonora emitida
puntos; para medio lazo π =n ⇒ λ= , b) calcule la velocidad de propagación
2 2 n c) ¿qué intensidad indicará el micrófono cuando llegue
v ⎛ v ⎞ a B?
con esto se encuentra f = = n⎜ ⎟
λ ⎝ πD ⎠
12. Una cuerda vibrante sometida a una fuerza de
tracción vibra con frecuencia de 220 Hz. Si la fuerza
de tracción se duplica y se mantienen las otras
condiciones constantes, determine la nueva frecuencia
de vibración.
Respuesta. f2 = 311,13 Hz
Respuesta. a) λ = 2,40 m, b) v = 288 m/s
13. Una cuerda de 80 cm de longitud y densidad lineal
de 1,69 x10-4 g/cm, está fija en sus dos extremos y 18. Sea un tubo de Kundt construido con un tubo de
emite un sonido fundamental cuando se la somete a vidrio, un émbolo que ajusta en un extremo y un
una fuerza de tracción de 1,92 kg. altoparlante colocado en el otro (fuente sonora en el
a) Determine la frecuencia fundamental del sonido cual se utiliza como “detector” polvo de corcho.
b) Calcule el factor por el cual debe multiplicarse la Ajustando, convenientemente la posición del émbolo
intensidad de la fuerza de tracción para que la se observa que el polvo de corcho en ciertos lugares se
frecuencia del nuevo sonido fundamental sea el tercer agita violentamente, mientras que en otros permanece
armónico del caso anterior en reposo. En estas condiciones se mide la distancia
Respuesta. a) 481Hz, b) 9 entre de puntos consecutivos en reposo que da 35 cm.
Entonces se retira el aire atmosférico y se lo sustituye

44
Movimiento ondulatorio y ondas Hugo Medina Guzmán

por otro gas, observándose ahora que distancia entre Respuesta.


los puntos citados precedentemente es de 45 cm, a) 13 cm b) 26 cm c) f =1,3 kHz d) f = 0.63 kHz
Sabiendo que la velocidad del sonido en el aire era de
340 m/s, calcule la velocidad del sonido en el gas 21. Dos altoparlantes están colocados con una
introducido. distancia de 0,5 m entre si emiten ambos una onda
Respuesta. 437,14 m/s sonora. Los altoparlantes son puestos a vibrar por un
amplificador que hace que emitan ondas en fase. A 10
19. Una fuente sonora F emite ondas de λ = 2 m. A m de distancia de los altoparlantes hay una fila de
100 m de la misma encuentra una pared con dos sillas y a una distancia de D = 1 m del punto central
ventanas separadas entre si. A 100 m otro lado de esta está situado el primer mínimo de intensidad.
última pared un observador detecta una posición
máxima intensidad de sonido. ; ¿A qué distancia
mínima debe colocarse observador para dejar de
percibir el sonido?

a) Diga si un oyente en la silla central escucha un


máximo o un mínimo de intensidad del sonido.
b) ¿Cuál es la longitud de onda y la frecuencia de las
ondas sonoras considerando que la velocidad del
sonido es 344 m/s (20ºC)?
Respuesta. c) ¿Cuál es la diferencia del camino del sonido emitido
por los dos altoparlantes correspondientes al máximo
1 Dλ
x= = 5,03m siguiente de intensidad?
2 λ2 d) Considere ahora que el amplificador hace que las
d2 − ondas producidas por los dos altoparlantes estén
4 desfasados 180º. Diga si tienen un máximo o mínimo
de intensidad en el punto central y a D = 1 m del
20. Dos altoparlantes están colocados como se muestra punto central.
en la figura con una separación de 3 m. Los Respuesta.
altoparlantes son puestos a vibrar sinusoidalmente por a) Máximo, b) λ =10 cm, f =3.5 kHz, c) 10 cm, d)
un amplificador que hace que emitan ondas sonoras mínimo; máximo
en fase. Un hombre se encuentra originalmente en el
punto O a 8 m de los altoparlantes en una línea 22. Una fuente S y un detector D de ondas de radio
perpendicular a ambos los altoparlantes y que pasa por están separados una distancia d, uno del otro. Las
el punto medio del segmento que une los altoparlantes ondas de radio de longitud de onda λ llegan a un
donde escucha un máximo de intensidad. El hombre detector después de un recorrido en línea recta o
después se desplaza a un punto P situado a 0,350 m del después de ser reflejados por alguna capa en la
punto O. En ese punto detecta el primer mínimo de la atmósfera. Cuando esa capa atmosférica está a una
intensidad sonora. altura H las ondas llegan en fase al detector. Si la
a) Calcule la diferencia de fase de las ondas sonoras altura de la capa reflectora aumenta gradualmente, la
producidas por los dos altoparlantes para el punto P. diferencia de fase entre las ondas en el detector cambia
b) Calcule la longitud de onda de las ondas de forma gradual hasta que las ondas están desfasadas
sinusoidales.
π cuando la capa reflectora está a la altura H + h.
c) Determine la frecuencia de la fuente sabiendo que
en estas condiciones (20ºC) la velocidad del sonido en
el aire es de 344 m/s.
d) Si la frecuencia de la fuente de ondas se ajusta de
modo que el hombre detecte el primer mínimo de
intensidad sonora a 0,75 m del punto O, ¿cuál será la
nueva frecuencia?

a) Encuentre la expresión de λ en función de: d, h, e H.


b) Calcule la longitud de onda sabiendo que la fuente y
el detector se encuentran apartados 100 km, y que H
= 6 km y h = 52 m.

45
Movimiento ondulatorio y ondas Hugo Medina Guzmán

Respuesta. Respuesta. La frecuencia del segundo diapasón debe


a) ser más alta que la del primer diapasón o al agregar la
⎡ ⎛ d ⎞2 2 ⎤ cinta habría aumentado el número de batidos. Por lo
⎛d ⎞
λ = 4 ⎢ ⎜ ⎟ + ( H + h )2 − ⎜ ⎟ + H 2 ⎥ tanto, v2 - 180 = 4 o v2 = 184 Hz.
⎢ ⎝2⎠ ⎝2⎠ ⎥
⎣ ⎦ 28. Un conductor viaja al norte en una carretera a una
b) λ =25 m velocidad de 25 m/s. Un auto patrullero, conduciendo
al sur a una velocidad de 40 m/s, se acerca con su
sirena que suena en una frecuencia de 2500 hertz.
23. Una fuente sonora emite un sonido de 540 Hz y se a) ¿Qué frecuencia escucha el conductor mientras que
aproxima a un observador detenido con velocidad de el patrullero se acerca?
valor 60 km/h. Supuesto que el valor de la velocidad b) ¿Qué frecuencia escucha el conductor después de
del sonido en el aire es de 340 m/s, calcule la pasar el patrullero?
frecuencia del sonido recibido por el observador. c) ¿si hubiera estado viajando el conductor al sur,
Respuesta. 567 Hz cuáles serían los resultados para (a) y (b)?
Respuesta. a) 3042 Hz b) 2072 Hz c) 2625Hz,
24. Una fuente sonora estacionaria emite un sonido de 2401 Hz
frecuencia 680 Hz. Si el valor de la velocidad del
sonido del aire es de 340 m/s, determine en qué sentido 29. Jorge se está dirigiendo hacia la isla con una
(con relación a la fuente) debe moverse un observador velocidad de 24 m/s cuando él ve a Betty que esta en
y con qué valor de velocidad debe hacerla si desea que orilla en la base de un acantilado. Jorge hace sonar la
la frecuencia del sonido percibido sea de 694 Hz. bocina de frecuencia 330 Hz.
Respuesta. a) hacia la fuente, b) 6,8 m/s a) ¿Qué frecuencia escucha Betty?
b) Jorge puede oír el eco de su bocina reflejado por el
25. Una fuente sonora emite sonido con frecuencia de acantilado. ¿La frecuencia de este eco mayor que o
840 Hz y se aproxima a un observador a velocidad de igual a la frecuencia es oída por Betty? Explique.
valor 40 m/s. Simultáneamente, el observador se c) Calcule la frecuencia que escucha Jorge del eco del
acerca a la fuente con velocidad de valor 10 m/s, Si se acantilado.
acepta como valor de la velocidad del sonido 340 m/s,
calcule el nuevo valor de la frecuencia percibida por el
observador.
Respuesta. 980 Hz

26. Una fuente sonora se aparta de un observador


partiendo del reposo y con aceleración constante de
valor 2 m/s2. La velocidad del sonido en este medio
vale 360 m/s. Determine el intervalo entre dos sonidos
recibidos a 20 s y 30 s.
Respuesta. 1,05
30. Dos naves en una niebla espesa están hacen sonar
27. Cuatro batidos o pulsaciones por segundo se oyen sus sirenas, que producen sonido con una frecuencia de
cuando dos diapasones suenan simultáneamente. 165 hertz. Una de las naves está en el reposo; la otra
Después de unir un pedazo pequeño de la cinta a una se mueve en una línea recta que pasa por la que está
rama del segundo diapasón, los dos diapasones se en el reposo. ¿Si la gente en la nave inmóvil oye una
hacen sonar otra vez y se oyen dos batidos por frecuencia de los batidos de 3,0 hertzios, cuáles son las
segundo. Si el primer diapasón tiene una frecuencia de dos velocidades y direcciones posibles del movimiento
180 hertzios, ¿cuál es la frecuencia original del de la nave móvil?
segundo diapasón? Respuesta. 6,13 m/s 6,35 m/s

46
Mecánica de fluidos Hugo Medina Guzmán

CAPÍTULO 4. Mecánica de fluidos


INTRODUCCIÓN ρ
La materia puede clasificarse por su forma física ρr = , cantidad adimensional.
como un sólido, un líquido o un gas. Las moléculas ρ agua
de los s6lidos a temperaturas y presiones ordinarias Densidad del agua a 4º C = 1g/cm3
tienen atracción fuerte entre ellas y permanecen en
posición fija relativa una a la otra. Luego un sólido Peso específico
tiene volumen y forma definida y sufre El peso específico denotado por γ se define como el
deformaciones finitas bajo la acción de una fuerza. peso por unidad de volumen del fluido, es decir
Las moléculas de los líquidos a temperaturas y γ = ρg , la unidad SI será N/m3.
presiones ordinarias tienen poca atracción entre ellas
y cambian de posición relativa una a otra. En
Ejemplo 1. Suponga que usted es capaz de llevar un
consecuencia los líquidos tienen volumen definido
peso de 400 N. ¿Cuál sería el tamaño del cubo hecho
tomando la forma del recipiente que los contiene,
de oro podría usted llevar? La densidad del oro es
pero no lo llenan necesariamente.
19300 kg/m3.
Las moléculas de los gases a. temperaturas y
Solución.
presiones ordinarias tienen muy poca atracción entre
ellas y tienen un movimiento al azar, o sea que los W = mg = ρVg = ρa 3 g ⇒
gases no tienen volumen ni forma definidas, adoptan
W 400
la forma del recipiente que los contiene y lo llenan a=3 =3 = 0,13
completamente. : . ρg (19300)(9,8)
A causa de que los líquidos y gases a temperaturas y Lado del cubo = a = 13 cm
presiones ordinarias no resisten la acción de un
esfuerzo cortante y continúan deformándose bajo su LA PRESIÓN EN LOS FLUIDOS. El concepto de
acción, son conocidos como fluidos. presión es muy general y por ello puede emplearse
La rama de la Física que estudia los efectos de las siempre que exista una fuerza actuando sobre una
fuerzas que actúan sobre 1os fluidos se denomina superficie. Sin embargo, su empleo resulta
Mecánica de Fluidos, tradicionalmente subdividida especialmente útil cuando el cuerpo o sistema sobre
en dos partes estática y dinámica. el que se ejercen las fuerzas es deformable. Los
Estática de los fluidos, estudia el equilibrio de los fluidos no tienen forma propia y constituyen el
fluidos bajo la acción de fuerzas estacionarias. principal ejemplo de aquellos casos en los que es más
Dinámica de los fluidos, estudia el movimiento de adecuado utilizar el concepto de presión que el de
los fluidos y las causas que la producen, sostienen o fuerza.
se oponen a este movimiento. Cuando un fluido está contenido en un recipiente,
ejerce una fuerza sobre sus paredes y, por tanto,
DENSIDAD, DENSIDAD RELATIVA Y PESO puede hablarse también de presión. Si el fluido está
ESPECÍFICO en equilibrio las fuerzas sobre las paredes son
perpendiculares a cada porción de superficie del
Densidad o masa especifica recipiente, ya que de no serlo existirían componentes
En un fluido, es importante la densidad o masa paralelas que provocarían el desplazamiento de la
específica ella permite calcular el peso del elemento masa de fluido en contra de la hipótesis de equilibrio.
de volumen que se considere, que es una posible La orientación de la superficie determina la dirección
fuerza exterior actuando sobre cada elemento de de la fuerza de presión, por lo que el cociente de
fluido. Para un elemento de volumen dV ubicado en ambas, que es precisamente la presión, resulta
algún punto del fluido y que contenga una masa dm , independiente de la dirección; se trata entonces de
la densidad ρ en ese punto se define mediante una magnitud escalar.
La presión se designa con la letra p , y se define
dm
ρ= como la fuerza de compresión por unidad de área
dV perpendicular a la fuerza.
La unidad de densidad en SI será kg/m3 pero se usa Fuerza normal sobre un área
generalmente densidades en g/cm3, p=
1 g/cm3 =1000 kg/m3. Area sobre la que se distribuye la fuerza
F
Densidad relativa =
A
Es posible utilizar una escala de densidades relativas
a la de alguna sustancia específica, por ejemplo ΔF dF
O bien p = lim =
existen las densidades de los fluidos respecto al agua, ΔA →0 ΔA dA
es decir
Unidades de presión. En el Sistema Internacional
(SI) la unidad de presión es el pascal, se representa

1
Mecánica de fluidos Hugo Medina Guzmán

por Pa y se define como la presión correspondiente a


una fuerza de un newton de intensidad actuando PRESIÓN EN UN PUNTO DE UN FLUIDO.
perpendicularmente sobre una superficie plana de un La presión sobre un punto totalmente sumergido en
metro cuadrado. un fluido en reposo es igual en todas las direcciones.
1 Pa = 1 N/m2. Para demostrar esto consideremos un pequeño prisma
Otras unidades: triangular como se muestra en la figura.
Atmósfera (atm) se define como la presión que a 0 ºC
ejercería el peso de una columna de mercurio de 76
cm de altura y 1 cm2 de sección sobre su base.
1 atm = 1,013x105 Pa.
Bar es realmente un múltiplo del pascal y equivale a
105 N/m2.
En meteorología se emplea con frecuencia el milibar
(mb) o milésima parte del bar
1 mb = 102 Pa ó 1 atm = 1013 mb.
También tenemos:
Milímetros de mercurio
1 mmHg = 133,322 Pa
Torr
1 torr = 133, 322 Pa Los valores de presiones promedio sobre cada una de
1 torr = 1 mmHg las tres superficies son p1, p2, y p3, en la dirección x
las fuerzas son iguales y opuestas y se cancelan
Ejemplo 2. En 1654, Otto Van Guericke, alcalde de mutuamente.
Magdeburgo e inventor de la bomba de aire, Haciendo la sumatoria de fuerzas obtenemos:
demostró que dos equipos de caballos no podrían
separar dos hemisferios de bronce evacuados. ¿Si los
∑F x = 0 ⇒ F2 − F3senθ = 0
diámetros de los hemisferios fueron 0,30 m, qué p2 (dydz ) − p3 (dsdz )senθ = 0
fuerza sería requerida para separarlos? Con dy = dssenθ :
p 2 (dydz ) − p3 (dydz ) = 0
Solución.

⇒ p 2 = p3
También
∑F y = 0 ⇒ F1 − F3 cos θ − dW = 0
⎛1 ⎞
p1 (dxdz ) − p3 (dsdz )cosθ − ρg ⎜ dxdydz ⎟ = 0
⎝2 ⎠
Con dx = ds cos θ :
⎛1 ⎞
p1 (dxdz ) − p3 (dxdz ) − ρg ⎜ dxdydz ⎟ = 0
⎝2 ⎠
Consideremos el hemisferio orientado con su eje a lo 1
largo del eje x. Tomemos una tira estrecha de la ⇒ p1 − p3 − ρgdy = 0
ancho ds que circunda el hemisferio. El componente 2
de x de la fuerza en esta tira es Cuando el prisma triangular se aproxima a un punto,
dFx = p a dA cosθ = pa (2πrsenθ )ds cosθ y
ds = rdθ
Así
π 2
Fx = ∫ 2πrpasenθ cosθrdθ
0
π 2 dy → 0 , y las presiones promedio se hacen
= 2πr pa ∫ senθ cosθdθ
2
0 uniformes, esto es la presión para un “punto”
⎡1
π 2 p1 = p3 .
2 ⎤
= 2πr pa ⎢ sen θ ⎥ = πr p a
2 2
Por lo tanto finalmente:
⎣2 ⎦0
p1 = p 2 = p3
Reemplazando valores:
(
Fx = π (0,15) 1,013 × 10 5 = 7160 N
2
) VARIACIÓN DE LA PRESIÓN CON LA
PROFUNDIDAD EN UN LÍQUIDO
HIDROSTÁTICA

2
Mecánica de fluidos Hugo Medina Guzmán

Para encontrar la variación de presión con la Cuando el punto 2 está en la superficie p 2 es la


profundidad, consideremos el estudio una porción de
fluido como se muestra en la figura, consistente en un presión atmosférica p a y se tendrá.
prisma de área A y altura dy , a una altura y un p1 = p a + ρgh
nivel de regencia arbitrario. Donde h representa la profundidad de un punto
cualquiera en el fluido y p su presión:

Ejemplo 3. Un dispositivo de exploración de las


profundidades del mar tiene una ventana de área 0,10
m2. ¿Qué fuerza se ejercida sobre ella por la agua de
mar (densidad 1030 kg/m3) a la profundidad de 5000
m?
Solución.
F = pA = ρghA = (1030)(9,8)(5000)(0,1)
= 5,05 x 106 N

PARADOJA HIDROSTÁTICA
La presión a la alturay es p y la presión en Una consecuencia de la ecuación p1 = p a + ρgh

( y + dy ) es ( p + dp ) . es el fenómeno que se ilustra en la figura, llamado


paradoja hidrostática. Podría parecer que el vaso
El peso del elemento es ρgAdy , donde ρ es la cónico ejerce una mayor presión en su base que el
densidad del fluido. que tiene la base más ancha, con lo cual el líquido
Como el elemento está en equilibrio: pasaría del cónico al otro, y alcanzaría una mayor
∑F = 0 ⇒ pA − ( p + dp )A − ρgAdy = 0 altura en este último. Sin embargo, ya hemos visto
que la ecuación p1 = p a + ρgh establece que la
y

− Adp − ρgAdy = 0
Simplificando:
presión depende únicamente de la profundidad, y no
dp de la forma de la vasija.
O dp = − ρgdy ⇒ = − ρg
dy
Esta ecuación nos da el cambio de presión con la
altura.

DIFERENCIA DE PRESIÓN ENTRE DOS


PUNTOS EN UN FLUIDO.

Ejemplo 4. Un experimentador desea determinar la


densidad de una muestra de aceite que ha extraído de
una planta. A un tubo de vidrio en U abierto en
ambos extremos llena un poco de agua con colorante
(para la visibilidad). Después vierte sobre el agua
una pequeña cantidad de la muestra del aceite en un
lado del tubo y mide las alturas h1 y h2, según como
se muestra en la figura. ¿Cuál es la densidad del
aceite en términos de la densidad del agua y de h1 y
de h2?

Diferencia de presión entre dos puntos cualquiera (1 y


2) en un fluido en reposo, será
p2 y2 y2
∫p1
dp = − ∫ ρgdy ⇒ p 2 − p1 = − ∫ ρgdy
y1 y1
Para fluidos que pueden considerarse incompresibles
(por lo general los líquidos), ρ es constante,
adicionalmente para diferencias de altura no muy Solución.
grandes g se puede considerar constante. La presión en el nivel x – x’ es igual en ambos lados
En este caso p 2 − p1 = − ρg ( y 2 − y1 ) , llamando a
del tubo.
h1
( y 2 − y1 ) = h ρ agua gh1 = ρ aceite gh2 ⇒ ρ aceite = ρ agua
h2
p 2 − p1 = − ρgh ⇒ p1 = p 2 + ρgh

3
Mecánica de fluidos Hugo Medina Guzmán

Ejemplo 5. Si la presión manométrica del agua en la de mayor área A2 sobre el que ejerce una fuerza F2
tubería a nivel del depósito de un edificio es de 500
mucho mayor:
kPa, ¿a qué altura se elevará el agua?
F1 F2 A
Solución. p= = ⇒ F2 = F1 2
p 5 × 10 5 A1 A2 A1
p = ρ a gh ⇒ h = = 3 = 51 m
ρ a g 10 × 9,8
Mientras mayor sea la relación entre las áreas de los
pistones, mayor es la fuerza ejercida sobre el pistón
Ejemplo 6. En unos vasos comunicantes hay agua y mayor.
mercurio. La diferencia de alturas de los niveles del
mercurio en los vasos es h = 1 cm. Calcular la altura
de aceite que se debe añadir por la rama de mercurio
para que el nivel de éste en los dos casos sea el
mismo.
Densidad del mercurio = 13,6 g/cm3.
Densidad del aceite = 0,9 g/cm3.

Ejemplo 7. Una gata hidráulica consiste en un


cilindro grande del área A conectado con un cilindro
pequeño del área a.. Ambos cilindros se llenan de
aceite. Cuando la fuerza f se aplica al cilindro
pequeño; la presión que resulta se transmite al
cilindro grande, que entonces ejerce una fuerza
ascendente F. Suponer que u auto pesa 12.000 N
sobre el cilindro grande de área 0,10 m2. ¿Qué fuerza
se debe aplicar al cilindro pequeño del área 0,002 m2
Solución. para soportar al auto?
La ley de los vasos comunicantes nos da para valor de F f
Solución. p=
= , tal que
la altura del agua: A a
hHg ρ agua 1 1 0,002
= ⇒ = ⇒
a
f = F = (12000) = 240 N
hagua ρ Hg hagua 13,6 A 0,10
hagua = 13,6 cm La gata tiene una ventaja mecánica de 50.
Una vez añadido el aceite los líquidos quedarán en la Ejemplo 8. Calcular la presión en los puntos 1, 2, 3 y
disposición de la figura segunda. Las presiones en las 4 en el sistema mostrado en la figura.
superficies de separación deben ser iguales y, por Densidad específica del aceite = 0,9
tanto:
ρ agua ghagua = ρ aciete ghaceite ⇒
ρ 13,6
hacite = hagua agua = = 15,11 cm
ρ aceite 0,9

EL PRINCIPIO DE PASCAL.
Si mediante algún método o sistema externo
aumentamos la presión en la superficie, la presión en
todos los puntos del fluido sufrirá igual aumento, es
decir, “el cambio de presión en alguna parte del Solución.
fluido confinado introduce el mismo cambio de Considerando la disposición y geometría mostrada en
presión en todas partes del fluido”. Enunciado que la figura:
corresponde al Principio de Pascal. Frecuentemente Presión en 1:
utilizado en la práctica de la ingeniería con la prensa p1 = patm – (0,25 + 0,25)ρagua g
hidráulica. = 1,033 x 105 – 4900
= 98400 Pa
La prensa hidráulica, representada en la figura a Presión en 2:
continuación. Mediante un pistón de sección p2 = patm + (0,50)ρagua g
transversal pequeña, A1 se ejerce una fuerza F1 = 1,033 x 105 + 4900
= 108200 Pa
sobre un líquido. La presión se trasmite a un cilindro
Presión en 3:

4
Mecánica de fluidos Hugo Medina Guzmán

p3 = p2 - (0,75)ρaire g p a A = ρ Hg ghA
Como la densidad del aire es 1000 veces manos que
la del agua podemos considerar La densidad del mercurio es ρ Hg = 13,6 g/cm3. Con
p3 = p2 = 108200 Pa
esto obtenemos para p a el valor
Presión en 4:
p4 = p3 + (1,25)ρaceite g p a ≈ 1,013 x 10 Pa = 1 atm .
5

= 108200 + 11025 La fuerza que eleva al mercurio al interior del tubo es


= 119225 Pa la presión atmosférica. El dispositivo que acabamos
de describir es un barómetro de mercurio. La altura
MEDIDA DE LA PRESIÓN. de la columna de mercurio mide la presión
atmosférica. La presión atmosférica promedio a nivel
Barómetro del mar corresponde a 760 mm de mercurio.
La presión en la superficie de un fluido que se AL repetir el mismo experimento, pero con una
encuentra en un recipiente abierto a la atmósfera no columna de agua, la altura será
es nula, sino igual a la presión atmosférica. Esta 13,6 veces mayor (recuerde que la densidad del
última se debe a que estamos inmersos en un fluido mercurio es 13,6 g/cm3 y la del agua
compresible constituido por el aire. La atmósfera de 1 g/cm3). Multiplicando los 76 cm por 13,6 se
la Tierra ejerce una presión sobre todos los objetos obtienen 10,34 m. Este dato es muy importante, ya
con los que está en contacto. La presión atmosférica que interviene en varias aplicaciones tecnológicas.
sobre la superficie terrestre la denotaremos por p a , y Por ejemplo, al intentar elevar agua de un pozo (cuya
es igual a la presión ejercida por el peso de toda la superficie está en contacto con el aire que nos rodea)
columna de aire que está por encima. La presión succionando por el extremo superior de un tubo largo,
sólo se tendrá éxito si el nivel de agua no está a más
atmosférica p a no es despreciable o insignificante
de 10,34 metros de profundidad (en la práctica esta
como algunas personas suelen creer. Por el contrario, altura es menor ya que el agua comienza a hervir
la presión atmosférica juega un papel importante en bastante antes de llegar a los 10,34 metros).
numerosos aparatos y máquinas de la vida diaria.
Considere un tubo de 1 m de largo y sección Barómetro de mercurio en U
transversal A, cerrado por uno de los extremos. Considere la figura donde se muestra un tubo cerrado
Llenemos el tubo con mercurio y coloquemos el tubo, en un extremo, doblado en forma de U, abierto por el
con el extremo abierto hacia abajo, en un recipiente otro extremo donde actúa la presión atmosférica que
con mercurio. Observaremos que el nivel de mercurio se desea medir. El mercurio alcanza una cierta
se situará aproximadamente 760 mm del nivel del posición de equilibrio, donde por el extremo cerrado
recipiente. por existir vacío, la presión es nula. Al nivel indicado,
la presión debe ser la misma, de modo que podemos
igualar
p a = h mmHg = h torr

El extremo superior del tubo queda al vacío.


Apliquemos la segunda ley de Newton a la columna
de mercurio (que sobresale de la superficie del
líquido en el recipiente). ¿Cuáles son las fuerzas que
actúan sobre ella? Manómetro simple.
Hay sólo dos: por una parte está la presión que el Otra aplicación práctica de la ecuación
fluido que está en el recipiente ejerce sobre el p1 = p 2 + ρgh son los instrumentos de medida de
mercurio que está en el tubo: tal fuerza es la presión:
F1 = p a A ; por otra, está el peso del mercurio al
interior de la columna Manómetro en U de líquido, para presiones relativas
de gases
Peso = ρ Hg gV = ρ Hg ghA . Como el fluido está La columna en U contiene un líquido (líquido
en reposo la fuerza neta debe ser nula, o sea: manométrico), por ejemplo agua, de modo que en la

5
Mecánica de fluidos Hugo Medina Guzmán

situación de equilibrio, cuando la presión p en el Solución.


recipiente que contiene un gas es mayor que la Podemos determinar sucesivamente las presiones de
atmosférica, la condición de equilibrio indicada en la los puntos indicados en la figura:
figura da p = p a + ρ L gh , de modo que si se mide p1 = p a + ρ ' gh
la altura h tenemos una medida de la presión relativa. p 2 = p1 − ρgh = p a + (ρ '− ρ )gh
p3 = p 2 + ρ ' gh = p a + (2 ρ '− ρ )gh
p 4 = p3 − ρgh = p a + 2(ρ '− ρ )gh
p5 = p 4 + ρ ' gh = p a + (3ρ '−2 ρ )gh
p = p5 = p a + (3ρ '−2 ρ )gh

Ejemplo 10. Los compartimientos B y C en la figura


están cerrados y llenos con aire, el barómetro lee 76
cm de mercurio cuando los manómetros leen x y 25
Presión relativa y la presión absoluta: cm. ¿Cuál será el valor de x?
La presión relativa (a la atmosférica) será Los tubos en U están llenos de mercurio de mercurio.
p − p a = ρ L gh .
La presión absoluta p puede también calcularse de
allí si se conoce o se mide la presión atmosférica
mediante un barómetro.
Si la presión en el recipiente que contiene el gas es
menor que la atmosférica, la situación de equilibrio
será como se indica en la figura siguiente de modo
que la condición de equilibrio será
p + ρ L gh = p a , dando para la presión relativa
Solución.
p − p a = − ρ L gh , un valor negativo que refleja Cálculo depC
que la presión en el interior del recipiente es menor
que la atmosférica. Igualmente se puede calcular la pC = p a − 0,25 ρ Hg g = 1,033 x 105 – 0,25 x
presión (absoluta) si la presión atmosférica es 13700 x 9,8 = 69735 Pa
conocida El valor de la presión en B se lee en el manómetro A:
p = p a − ρ L gh p B = 2 x105 Pa
La lectura del manómetro entre los tanques B y C es
la diferencia entre las presiones de dichos tanques:
p B − pC = ρ Hg g ( x )
200000 – 69735 = 13700 x 9,8 x
De aquí se obtiene: x = 0,97 m

Ejemplo 11. En una balanza de gran sensibilidad


fueron equilibrados dos barómetros de mercurio: uno
en forma de platillo (con un plato ancho) y el otro en
forma de U. Los barómetros están hechos del mismo
material, tienen el mismo diámetro de los tubos y
Ejemplo 9. Determinar la presión p de un gas, en el
contienen la misma cantidad de mercurio. Las
manómetro mostrado en la figura.
distancias entre las partes soldadas de los tubos y los
niveles superiores del mercurio en ellos son iguales.
¿Cómo variará el equilibrio de la balanza si aumenta
la presión atmosférica?

6
Mecánica de fluidos Hugo Medina Guzmán

altura Δh del tubo, para el barómetro en U solo


corresponde al volumen de una columna de altura
Δh / 2 , por lo tanto desaloja menor volumen y el
empuje es menor que en barómetro de cubeta.

EL PRINCIPIO DE ARQUÍMEDES.
Cuando un objeto se sumerge en un fluido (un líquido
o un gas), experimenta una fuerza ascendente de la
flotabilidad porque la presión en el fondo del objeto
es mayor que en la parte superior. El gran científico
Solución. griego Arquímedes (287-212 B.C.) hizo la
Como resultado de la variación de la presión observación cuidadosa siguiente, ahora llamada el
atmosférica, la fuerza de Arquímedes que actúa sobre principio de Arquímedes.
los barómetros por parte del aire se varía tanto por el Cualquier objeto totalmente o parcialmente
cambio de la densidad del aire, como por el cambio sumergido en un fluido es empujado para arriba por
del volumen de los barómetros, cuando se cambian una fuerza igual al peso del fluido desplazado.
los niveles del mercurio en sus secciones abiertas. Para ver que esto es verdad, considere una porción
Tomando en consideración todas las condiciones del pequeña de agua en un recipiente como se muestra en
problema, los barómetros tienen no sólo el mismo la figura. El agua sobre esta porción actúa hacia
peso, sino también el mismo volumen. Por eso, para abajo, al igual que su peso. El agua bajo la porción
cada uno de ellos la variación de la fuerza de empuje, empuja hacia arriba. Puesto que la porción de agua
debido a la primera causa, es la misma. La variación está en equilibrio, la fuerza hacia arriba equilibra las
de los volúmenes, como es evidente, será diferente. fuerzas hacia abajo.
En el barómetro en forma de U, para una variación de F1 + P = F2
la diferencia de niveles en un determinado valor, el
nivel del mercurio en cada caño acodado debe
cambiar sólo en la mitad de este valor. En el
barómetro de cubeta el nivel del mercurio en la
cubeta cambia muy poco y en el tubo cambia
prácticamente en todo el valor de variación de la
diferencia de niveles. Además, en la misma cantidad
en que cambia el volumen del mercurio dentro del
tubo variará el volumen en la cubeta. Por
consiguiente, para el barómetro de cubeta, la
variación del volumen será dos veces mayor que para
el barómetro en forma de U (a diámetros iguales de La fuerza neta hacia arriba debido al fluido se llama
los tubos). Al aumentar la presión, el volumen del la fuerza Empuje, así
barómetro de cubeta se hace menor que el volumen FE = F2 − F1 = P
del barómetro en forma de U, la fuerza de Aquí P es el peso del fluido desplazado por el objeto.
Arquímedes que actúa sobre el barómetro de cubeta Si la porción de agua de peso P es substituido por un
también será menor y por eso él pesa más. objeto de la misma forma y tamaño, este objeto
Explicación gráfica también sentiría la fuerza de empuje hacia arriba
F=P
O sea que la fuerza de empuje FE es FE = ρgV ¸
donde ρ es la densidad del fluido, y V es el
volumen del cuerpo sumergido.
Si el peso del objeto es mayor que P (el peso del
fluido desplazado), el objeto se hundirá (siempre
experimenta la fuerza de empuje, razón por la que un
objeto no se siente tan pesado cuando se sumerge que
cuando se saca del agua). Si el peso del objeto es
menor que el peso de agua desplazada cuando se
sumerge totalmente, experimentará una fuerza neta
hacia arriba y flotará a la superficie. Algo del objeto
La figura ilustra la variación en los barómetros con el resaltará sobre la superficie, de modo que la porción
aumento de presión. todavía sumergida desplace un peso de fluido igual al
Las alturas de las columnas de mercurio son iguales a peso del objeto.
(h + Δh ) . Mientras la variación en el volumen de la
cubeta corresponde al volumen de una columna de CENTRO DE EMPUJE

7
Mecánica de fluidos Hugo Medina Guzmán

Es el punto a través del cual actúan las fuerzas de Peso = mg = ρgV


empuje, y está en el centro de gravedad del volumen
de líquido desplazado. Si el cuerpo es homogéneo y ρgT1 ρ ρa
= = T
está totalmente sumergido, su centro de gravedad (ρ − ρ a )g [(ρ ρ a ) − 1] 1
coincide con el centro de empuje.
19,3
= (120 N ) = 127 N
EQUILIBRIO ROTACIONAL DE OBJETOS (19,3 − 1)
FLOTANTES.
Un cuerpo tiene estabilidad vertical cuando un Ejemplo 13. Un bloque de madera de la gravedad
pequeño desplazamiento vertical en cualquier sentido específica 0,8 flota en agua. ¿Qué fracción de
origina fuerzas restauradoras que tienden a volver al volumen del bloque se sumerge?
cuerpo a su posición original y tiene estabilidad Solución.
rotacional cuando al aplicar un pequeño Si V es el volumen del bloque y xV es el volumen
desplazamiento angular se origina un par restaurador. sumergido (x es la fracción de volumen sumergido),
En la figura se muestran los diversos casos de entonces
equilibrio que se presentan.
mg = FB o ρgV = ρ a xVg
ρ
⇒ x= = 0,8
ρa

Ejemplo 14. Consideremos el movimiento de un


objeto de volumen V y masa M que cae a través de un
fluido con viscosidad cero (sin rozamiento).
a) Dibuja las fuerzas que actúan sobre el cuerpo.
a) Estable. Ocurre cuando el centro de gravedad del
b) ¿La aceleración del objeto en caída es
cuerpo está por debajo del centro de empuje, para una
independiente de su masa?, ¿y de su volumen?
pequeña rotación el par de fuerzas hará retornar al
Solución.
cuerpo a su posición inicial.,
a)
b.) Inestable. Ocurre cuando el centro de gravedad del
cuerpo esta por encima del centro de empuje para una
pequeña rotación el par de fuerzas tenderá a hacer
rotar el cuerpo hacia una nueva posición de
equilibrio.
c) Indiferente. Ocurre para cilindro recto horizontal y
esfera, ya que su peso y fuerza de empuje son
siempre colineales al aplicarle cualquier rotación.

Ejemplo 12. Un hombre que está pescando en el b) Ecuación del movimiento


Mar Egeo pesca accidentalmente un artefacto antiguo ma = mg − Empuje
de oro. La densidad del oro es 19,3 x 103 kg/m3, y la
densidad del agua de mar es 1,03 x 103 kg/m3. ρc = densidad del cuerpo, ρf = densidad del fluido,
¿Mientras está levantando el tesoro, la tensión en su V = volumen del cuerpo
línea es 120 N. ¿Cuál será la tensión cuando saque el ρ cVa = ρ cVg − ρ f Vg
objeto del agua?
Nota: Si usted engancha un tesoro o un pez grande, ⎛ ρf ⎞
a = g ⎜⎜1 − ⎟ , a es independiente de la masa y el
no lo levante del agua. El cordel puede romperse.
⎝ ρ c ⎟⎠
Solución.
Si el objeto de peso mg se levanta lentamente, está en volumen, depende de las densidades del cuerpo y del
equilibrio y fluido.
mg = T1 + FE , donde m = ρV , ( ρ es la Ejemplo 15. Una pelota de plástico tiene 25 cm de
densidad del objeto, V es el volumen del objeto) radio y flota en agua con el 25% de su volumen
T1 es la tensión en la línea mientras está en el agua y sumergido.
a) ¿Qué fuerza deberemos aplicar a la pelota para
FE es la fuerza de empuje, sostenerla en reposo totalmente sumergida en agua?
FE = ρ a gV , ( ρ a es la densidad del agua) b) Si se suelta la pelota, ¿qué aceleración tendrá en el
instante en que se suelte?
T1
Así: ρVg = T1 + ρ aVg ⇒ V = Solución.
(ρ − ρ a )g Primero calcularemos la densidad de la pelota.
Cuando el objeto está en aire, la tensión es igual al Utilizando la primera condición:
peso mg.

8
Mecánica de fluidos Hugo Medina Guzmán

I) P1 es el peso de un recipiente con agua con un


Fuerza de empuje: E = ρ a 0,25Vg objeto sumergido en él.
II) P2 es el peso cuando el objeto está sumergido en el
Peso: P = ρ pVg agua, pero colgado de una cuerda sin que toque el
∑F v =0 fondo del recipiente.
III) P3 es el peso del recipiente con agua.
Peso = empuje Encuentre la densidad promedio del objeto.
ρ pVg = ρ a 0,25Vg ⇒ Solución.
Sean: m masa del objeto, V volumen del objeto, ρ
ρ p = 0,25ρ a = 0,25 g/cm3.
densidad del objeto.
a) Empuje cuando está totalmente sumergida en agua; Restando (III) de (I):
P1 − P3
P1 − P3 = mg ⇒ m = ,
g
m
Como V =
ρ
P1 − P3
⇒ V = (1)
ρg
De (II) y (III):
P2 = P3 + E ⇒ E = P2 − P3 ,
como E = ρ agua gV
Fuerza de empuje: E = ρ aVg
⇒ P2 − P3 = ρ agua gV
Peso: P = ρ pVg
P2 − P3
∑F v =0 y V =
ρ agua g
(2)
E−F−P=0 ⇒ F =E−P Igualando (1) y (2):
La fuerza que se necesaria para mantenerla en
equilibrio totalmente sumergida es:
(
Empuje – peso = ρ a − ρ p Vg ) P1 − P3 P2 − P3
=
P − P3
⇒ ρ= 1 ρ agua
ρg ρ agua g P2 − P3
⎛4 ⎞
= (1000 − 250 )⎜ π 0,25 3 ⎟9,8
⎝3 ⎠ Ejemplo 17. Disponemos de una plancha de corcho
= 481,06 N. de 10 cm de espesor. Calcular la superficie mínima
b) Sea a la aceleración de la pelota en el instante en que se debe emplear para que flote en agua,
que se suelte sosteniendo a un náufrago de 70 kg. La densidad del
F = ma ⇒ corcho es de 0,24 g/cm3.
Nota: entendemos por superficie mínima la que
F 481,06
a= = = 29,4 m/s2. permite mantener al hombre completamente fuera del
m
(250)⎛⎜ 4 π 0,253 ⎞⎟ agua aunque la tabla esté totalmente inmersa en ella.
Solución.
⎝3 ⎠

Ejemplo 16. Considere las tres mediciones mostradas


en la figura adjunta:

Peso = empuje

9
Mecánica de fluidos Hugo Medina Guzmán

[70 + 240(0,1A)]g = 1000(0,1A)g Ejemplo 20. En un vaso de agua flota un pedazo de


hielo,- ¿Cómo cambia el nivel del agua en el vaso
0,1 A es el volumen de la plancha de corcho.
cuando el hielo se derrite? Analizar los siguientes
casos:
70
70 = 100 A − 24 A ⇒ A = = 0,92m 2 a) el hielo es completamente homogéneo;
76 b) en el hielo se encuentra una piedra fuertemente
adherida;
Ejemplo 18. Un cable anclado en el fondo de un
lago sostiene una esfera hueca de plástico bajo su c) dentro del pedazo de hielo hay una burbuja de aire.
superficie. El volumen de la esfera es de 0,3 m3 y la Solución.
tensión del cable 900 N. a) Como el pedazo de hielo flota, el peso de toda el
a) ¿Qué masa tiene la agua desplazada por éste es igual al peso del propio
esfera? hielo o del agua recibida de éste. Por eso el agua que
b) El cable se rompe y la se forma después del deshielo ocupará un volumen
esfera sube a la superficie. igual al volumen de la parte hundida del pedazo de
Cuando está en equilibrio, hielo y por consiguiente el nivel del agua no
¿qué fracción del volumen cambiará.
de la esfera estará b) El volumen de la parte sumergida del pedazo de
sumergida? hielo con la piedra es mayor que la suma de los
Densidad del agua de mar volúmenes de la piedra y el agua que se obtiene
1,03 g/cm3 después del deshielo. Por lo tanto, el nivel del agua en
Solución. el vaso se descenderá.
a) E = mg + T E = Empuje, c) El peso del agua desplazada es igual al peso del
T = Tensión del cable. hielo (el peso del aire en la burbuja puede
1030 x 0,3 x 9,8 = m x 9,8 + 900 ⇒ m = 217,2 kg prescindirse). Por eso igualmente como en el caso a),
b) E = mg V = Volumen sumergido. el nivel del agua no cambia.
1030 x V x 9,8 = m x 9,8 ⇒ V = 0,21 m3
0,21 Ejemplo 21. Se tiene un cilindro vacío de radio 10
Fracción del cuerpo sumergido = = 0,7 cm, que flota en agua dejando fuera del nivel del
0,3 agua una altura de 10 cm cuando de el cuelga
externamente un bloque de hierro de peso 10 kg y
densidad 7,8 g/cm3 tal como lo muestra la figura (a).
Calcular la altura que quedara afuera del agua si el
bloque de hierro se introduce dentro del cilindro
como lo muestra la figura (b).

Ejemplo 19. Un pedazo de aluminio se suspende de


una cuerda y se sumerge completamente en un
recipiente con agua. La masa del trozo de aluminio es
de 1 kg. Calcule la tensión de la cuerda antes y
después de sumergir el trozo de aluminio. Solución.
Solución. Sea h la altura del cilindro.
La tensión antes es simplemente el peso del trozo de De la figura (a):
aluminio es decir 10
P = mg = 1 × 9,8 = 9,8 N 1000 g (h − 0,1)π 0,12 + 1000 g = 10 g
7800
Cuando se sumerge la fuerza de empuje es ⇒ h = 0,38 m
E = ρ aguaVal g , pero el volumen del aluminio es De la figura (b):
m 1000 g (h − x )π 0,12 = 10 g
V Al = ⇒ x = 0,06 m
ρ Al
de modo que la fuerza de empuje será: Ejemplo 22. Un cuerpo homogéneo y compacto,
m 1 colocado en un líquido con peso específico γ 1 , pesa
E = ρ agua g = 10 3 9,8 = 3,6 N.
ρAL 2,70 × 10 3 P1; y colocado en un líquido con peso específico γ 2 ,
y finalmente la tensión en la cuerda será la diferencia pesa P2 Determinar el peso específico ρ del cuerpo.
T = 9,8 - 3,6 = 6,2 N
Solución: El peso del cuerpo hundido en el líquido

10
Mecánica de fluidos Hugo Medina Guzmán

en el primer caso es igual a P1 = (γ − γ 1 )V ; en el Vs ρ Hielo


= = 0,917
segundo caso es igual a P2 = (γ − γ 2 )V . donde V es V ρ agua
el volumen del cuerpo; de allí resulta que O sea hay un 91,7% sumergido y por lo tanto 8,3 %

γ=
(P2γ 1 − P1γ 2 ) sobre el nivel del agua.

(P2 − P1 ) Ejemplo 28. Un tubo flota en el agua en posición


vertical. La altura del tubo que sobresale del agua es
Ejemplo 23. En el centro de un lago grande h = 5 cm. Dentro del tubo se vierte aceite de densidad
congelado han hecho Un claro. El grosor del hielo ρ’ = 0,9 g/cm3. ¿Cuál deberá ser la longitud del tubo
resultó igual a 1,0 m. ¿De qué longitud será necesaria para llenarlo totalmente de aceite manteniendo la
la cuerda para sacar un balde de agua? altura h?
Solución.
Solamente en los pequeños lagos el hielo puede
mantenerse suspenso gracias a la orilla. En el centro
de un lago grande éste obligatoriamente flotará. La
relación de las densidades del hielo y del agua es 0,9.
Por consiguiente, 0,9 de todo el espesor del hielo se
encuentra en el agua. La distancia entre la superficie
del hielo y el agua es 1 m.
Solución.
Ejemplo 24. En una taza con agua flota una cajita de La longitud del tubo x se halla de la condición
ρ ' gx = ρg (x − h ) que expresa la igualdad de las
fósforos dentro de la cual hay una piedra pequeña.
¿Variará el nivel del agua en la taza si la piedra se
saca de la cajita y se pone en el agua? presiones en la profundidad del extremo inferior del
tubo. Aquí ρ es la densidad del agua. Obtenemos,
Solución.
entonces, que
Al retirar la piedra de la caja se hizo más ligera en un
ρ
peso igual al de la piedra y. por lo tanto, el volumen x= h = 50 cm.
del agua desplazada por la caja disminuyó en V1 = ( ρ − ρ ')
P/ρ1g, donde P es el peso de la piedra y ρ1, la
densidad del agua. Al sumergirse en el agua, la piedra Ejemplo 29. La posición estable de un cilindro de
desalojará un volumen de agua igual a su propio longitud L, flotando en un liquido de densidad ρ , es
volumen, o sea, V2 = P/ρ2g, donde ρ2 es la densidad como se muestra en la figura (a). Cuando el bloque de
de la sustancia de la piedra. Como ρ2 > ρl. entonces concreto (densidad ρ ' ) se suspende del cilindro toma
V1 > V2 y por consiguiente el nivel del agua en la taza
disminuirá. la posición mostrada en la figura (b) Si se desprecia
el volumen y peso del cable. ¿Cuál es el volumen del
Ejemplo 25. Un cubo de Hielo flota en agua. bloque?
Determine la fracción del hielo que queda sobre la
superficie del agua.
Solución.
Sea m la masa de hielo. Su peso será
P = mg
Su volumen total será
m
V = ,
ρ Hielo
De modo que podemos escribir el peso en términos
del volumen como
P = ρ HieloVg Solución.
En la posición (a)
Cuando una fracción VS del volumen queda
πR 2 L
sumergida, la fuerza de empuje es peso = ρg
E = ρ aguaVs g . 2
En la posición (b.)
En la situación de equilibrio el peso iguala al empuje Tomando momentos con respecto al eje vertical por
de modo que el que pasa el empuje, tenemos:
ρ HieloVg = ρ aguaVs g , Peso(asen30º ) = (Vρ ' g − Vρg )R
De donde

11
Mecánica de fluidos Hugo Medina Guzmán

πR 2 L ⎛ a ⎞ Para la solución considere lo siguiente


⎜ ⎟ = VRg (ρ '− ρ ) El P tiene una sección A, la parte sumergida es z.
2 ⎝2⎠ La sección del depósito es A1
πRLρa
⇒ V =
4(ρ '− ρ )

Ejemplo 30. Un corcho cilíndrico de masa m1 y


sección transversal A1 flota en un líquido de densidad
ρ. El corcho está conectado por medio de una cuerda
sin masa, de largo L, a un cilindro de aluminio de
masa m2 y sección transversal A2.
El cilindro de aluminio puede deslizarse sin roce por
un orificio hermético en el fondo del recipiente. Solución.
Calcular la profundidad h a la que debe hallarse la El peso P tiene una sección A y está hundido una
base del corcho para que el sistema de los dos altura z, de tal manera que:
cilindros esté en equilibrio. La presión atmosférica, P = ρgAz
¿juega algún rol?

En el depósito: Peso P = empuje sobre P


La sección del depósito es A1.
P2 = ρg ( A1 y − Az ) = ρgA1 y − P ,
⇒ P2 + P = ρgA1 y (1)
Para el conjunto total:
Peso total = Empuje sobre P2
Solución. ⇒ P+ P1 + P2 = ρgA1 x (2)
Dividiendo (2) / (1):
ρgA1 x P + P1 + P2
=
ρgA1 y P + P2
x (P + P2 ) + P1 P1
⇒ = ⇒ n = 1+
y (P + P2 ) (P + P2 )
Finalmente:
P1
P= −P
E1 − F2 − (m1 + m2 )g = 0
(n − 1) 2
E1 = A1 hρg , F2 = ρg (h + L )A2 Ejemplo 30. En una tentativa de identificar un
A1 hρg − ρg (h + L )A2 − (m1 + m2 )g = 0 espécimen de roca, un geólogo pesa una muestra en
aire y también cuando que está sumergido en agua,
A1 hρg − A2 hρg − A2 Lρg = (m1 + m2 )g usando una balanza de brazos iguales improvisada...
( A1 − A2 )hρ = (m1 + m2 ) + A2 Lρ ¿Obtiene en su medición 120 g y 78 g. cuál es la
densidad de la muestra?
(m + m2 ) + A2 Lρ Solución.
h= 1
( A1 − A2 )ρ m = ρV = 120 y en agua
En aire
La diferencia de presión debido a la atmósfera para un
caso como este, en que las diferencias de altura son
120 − ρ aV = 78 ⇒ ρ aV = 42
pequeñas no juega un rol perceptible. De estas relaciones obtenemos:
ρ 120
Ejemplo 29. Un depósito de peso P1 flota en un = = 2,86
líquido y al mismo tiempo tiene una cantidad del
ρ a 42
mismo líquido, de peso P2, determinar el peso del La roca desconocida tiene una densidad 2,86 g/cm3
flotador P para que la relación de las profundidades
x y se igual a n .
Sugerencia.

12
Mecánica de fluidos Hugo Medina Guzmán

Ejemplo 31. Un cuerpo de forma rectangular de 10


cm de espesor está flotando en una laguna pequeña
con tres cuartos de su volumen sumergido
a) Si un camión cisterna derrama en la laguna aceite
de densidad 0,65 g/cm3, quedando la cara superior del
cuerpo justamente a nivel de la superficie del líquido.
¿Cuál es el espesor de la capa de aceite?
b) ¿Qué pasará si se para sobre el cuerpo un pajarito
de masa m y luego se sale? Determinar la ecuación
de movimiento considerando que el agua tiene una
constante de viscosidad bA (A es área del cuerpo
rectangular).

∑F y = ma y ⇒
Fuerza recuperadora por empuje extra (debido a y) +
Fuerza de oposición por viscosidad
= masa de palo moviéndose verticalmente.
Solución.
• ••
a) Antes que se derrame el aceite, el poste está ⇒ − ρgAy − bA y = ρ p Al y
flotando en el agua simétricamente. Sea su sección
transversal A y su densidad ρ p , si ¾ de su volumen l = 0,1 m , ρ p = densidad del palo
están sumergidos, sobresalen 2,5 cm y están •• b • ρg
sumergidos 7,5 cm. El peso es igual al empuje. La y+ y+ y = 0,
densidad del agua es ρ .
ρ pl ρ pl
•• • b
3 2
y + 2β y + ω 0 y = 0 , 2β =
ρ p gA(10) = ρgA(7,5) ⇒ ρ p = ρ ρ pl
,
4
Cuando el aceite de densidad ρa se derrama éste ρg
ω0 =
permanece sobre el agua y se extiende a una altura y ρ pl
sobre el agua, al agua le corresponde una altura (10 –
y). Como se ha alcanzado el equilibrio: y = y0e − βt cos(ωt + ϕ ) ,
m π
y0 = ,ϕ= yω= β 2 − ω 02 .
ρA 2

Ejemplo 32. Un recipiente se llena parcialmente de


agua. Aceite de densidad 750 kg/m3 se vierte sobre el
ρ p gA(10) = ρgA(10 − y ) + ρ a gAy ⇒ agua, y flota sin mezclarse. Un bloque de la madera
de densidad 820 kg/m3 se coloca en el recipiente, y
3 flota en la interfase de los dos líquidos. ¿Qué
ρgA(10) = ρgA(10 − y ) + ρ a gAy ⇒ fracción del volumen del bloque se sumerge en agua?
4
Solución.
3
ρgA(10) = ρgA(10 ) − ρgAy + ρ a gAy Sea el bloque de madera de sección A y altura h, la
4 parte sumergida en agua es x y la parte en aceite es (h
1 - x).
⇒ ρ (10) = (ρ − ρ a ) y El volumen en agua es Va = Ax , y el volumen en
4
10 ρ 10(1) aceite es Vo = A(h − x )
⇒ y= =
4(ρ − ρ a ) 4(1 − 0,65) El peso del bloque es equilibrado por los empujes
debidos al agua y al aceite.
ρ m gAh = ρ a gAx + ρ o gA(h − x )
= 7,14 cm
b) Si se para sobre el cuerpo un pajarito de masa m y
luego se sale, el cuerpo quedará en movimiento ⇒ ρ m h = ρ a x + ρ o (h − x )
armónico simple vertical, como lo demostraremos a
x x
continuación. ⇒ ρm = ρa + ρo − ρo
Vamos a considerar antes de derramado el aceite. h h
⇒ (ρ a − ρ o ) = ρ m − ρ o
x
h

13
Mecánica de fluidos Hugo Medina Guzmán

x (ρ m − ρ o ) El largo a de la parte de la varilla sumergida es


⇒ =
h (ρ a − ρ o )
h
a= .
senα
820 − 750
⇒ = 0,28 La fuerza de empuje viene dada por:
1000 − 750 h
E = ρ 0 Aag = ρ 0 A g
Ejemplo 33. Un gran bloque de hielo (densidad 917 senα
kg/m3) flota en la agua de mar (densidad 1030 La fuerza de gravedad es
kg/m3). ¿Si el área superficial del hielo es 20 m2 y El torque ejercido por ambas fuerzas respecto a O
tiene 0,20 m de espesor, cuál es la masa del oso polar debe ser nulo, o sea,
más pesado que puede estar parado en el hielo sin ⎛a ⎞ ⎛L ⎞
hacerlo ir debajo de la superficie del agua? E ⎜ cos α ⎟ = P⎜ cos α ⎟ ⇒ Ea = PL
Solución.
⎝2 ⎠ ⎝2 ⎠
Sustituyendo las expresiones para E y P se deduce
ma = ρ a Ah , m B = ρ B Ah que
m B g + mg = ma g ρ 0 Aa 2 g = ρ1 AL2 g ⇒ ρ 0 a 2 = ρ1 L2 ,
⇒ m = ma − m B = (ρ a − ρ B )Ah Reemplazando el valor de a .
= (1030 − 917 )(20 )(0,2 ) = 452 kg
2
⎛ h ⎞
ρ0 ⎜ ⎟ = ρ1 L
2

⎝ senα ⎠
Ejemplo 34. Una varilla de largo L y densidad ρ1 Despejando se encuentra finalmente que
flota en un líquido de densidad ρ 0 (ρ 0 > ρ1 ) . Un ρ0 h
extremo de la varilla se amarra a un hilo a una senα =
ρ1 L
profundidad h (ver figura adjunta).
b) Si el lado derecho de la última ecuación es mayor
a) Encuentre el ángulo α .
o igual a uno, la varilla se mantendrá en posición
b) ¿Cuál es el mínimo valor de h para el cual la
vertical. El mínimo valor de h para que la varilla
varilla se mantiene en posición vertical?
esté en posición vertical es
c) ¿Cuál es la tensión del hilo?
ρ1
hmin = L
ρ0
c) La tensión del hilo se obtiene exigiendo que la
fuerza total sea nula. De esta manera se obtiene que
h
T = E − P = ρ0 A g − ρ1 ALg
senα
⎛ ρ0 ⎞ ⎛ ρ0 ⎞
= ALgρ1 ⎜ −1 ⎟ = Mg ⎜ −1 ⎟,
⎜ ρ ⎟ ⎜ ρ ⎟
Solución. ⎝ 1 ⎠ ⎝ 1 ⎠
Donde M es la masa de la varilla.

Ejemplo 35. Una barra homogénea de peso P , área


de sección transversal A y longitud L flota en agua
con uno de sus extremos anclados a una profundidad
H , tal como se muestra en la figura. Considerando
el espesor de la barra pequeño, determinar el ángulo
θ de equilibrio.
Densidad del líquido = ρ .
a
a) La fuerza de empuje se aplica en el lugar y la
2
L
fuerza de gravedad en el lugar (medidos desde
2
O).
Sea A la sección transversal de la varilla
El volumen de la barra es: AL
El peso de la barra es P = ρ1 ALg Solución.
Geometría del problema

14
Mecánica de fluidos Hugo Medina Guzmán

Las únicas fuerzas que están actuando sobre el objeto


son el peso P y el empuje
E. Ya sabemos que ambas fuerzas tienen la misma
magnitud y apuntan en direcciones opuestas y, por lo
tanto, la fuerza neta sobre el objeto es nula. Pero para
que se encuentre en equilibrio también el torque neto
debe ser nulo. Esto se logra solo si ambas fuerzas son
colineales (actúan a lo largo de la misma recta).
Encontremos los puntos en que actúan las dos
fuerzas.
La gravedad actúa en el centro de masas.
H H L El centro de masas de los cubos A y B se encuentra en
y= , x1 = , x 2 = cos θ
senθ 2 tan θ 2 a y el centro de masas de C se encuentra en b. El
Determinación del empuje: centro de masas del objeto completo se encontrará
H sobre la recta que une a con b. Como el cubo C tiene
E = ρgVSumergido = ρgAy = ρgA el doble de masa de los dos cubos A + B juntos, el
senθ centro de masas del objeto completo se ubicará más
Estática, se tendrá equilibrio cuando: cerca de b que de a. En la figura más abajo hemos
∑τ O =0 designado el centro de masas del objeto completo con
el numero 1. Se tiene que
O sea, Px 2 = Ex1
ab
Sustituyendo valores: b1 =
3
⎛L ⎞ H ⎛ H ⎞
P⎜ cos θ ⎟ = ρgA ⎜ ⎟ La fuerza de empuje, por otra parte, actúa en el centro
⎝2 ⎠ senθ ⎝ 2 tan θ ⎠ de masas que se obtiene al sustituir los tres cubos por
agua (en la figura lo hemos designado con el numero
ρgAH 2 cos θ 2).
=
2sen 2θ
De aquí:
ρgAH 2
sen θ =
2

PL
ρgA
⇒ senθ = H
PL
Finalmente:
ρgA Nuevamente el centro de masas de los cubos A + B se
θ = arc sen H encuentra en a, mientras que el de C se encuentra en
PL
b. El centro de masas de los centros de masas
Ejemplo 36. Considere tres cubos del mismo tamaño, nuevamente se encontrará sobre la recta ab . Pero
adheridos tal como se muestra en la figura. La ahora los cubos A+B pesan el doble de lo que pesa C,
densidad del material del cual están hechos los dos luego el centro de masas ahora estará mías cerca de a
cubos A y B es ρ1 = 0,5 g/cm3, mientras que el cubo C que de b. De hecho, el centro de masas cuando los
está hecho de un material de densidad ρ2 = 2 g/cm3. tres cubos están hechos de agua debe estar sobre el
Observe que la densidad media de los tres cubos es plano de simetría indicado en la figura con una línea
igual a la del agua ρ = 1 g/cm3) y, por lo tanto, al punteada.
sumergirlo en agua, la fuerza de empuje exactamente En resumen, la fuerza de gravedad actúa en 1 y el
cancela el peso. ¿Cuál será la orientación de empuje actúa en 2. Para que no haya torque sobre el
equilibrio estable que el objeto adquirirla cuando está sistema la recta ab debe orientarse a lo largo de la
“flotando” rodeado de agua? vertical.
Concluimos que el ángulo β de la figura del
enunciado debe coincidir con el de la segunda figura.
Se deduce inmediatamente que tan β = 1/2.
Convénzase de que el equilibrio es estable cuando el
punto 2 está sobre el punto 1 e inestable cuando 1
está sobre 2.

FUERZAS SOBRE LAS PAREDES O


Solución. COMPUERTAS

15
Mecánica de fluidos Hugo Medina Guzmán

Ya hemos estudiado la variación de presión con la


profundidad de un fluido, el conjunto de fuerzas que Centro de presión. El centro de presión lo
resultan de la acción del fluido sobre la cara de una encontramos de la siguiente manera
superficie de área finita puede reemplazarse por una Torque de las fuerzas distribuidas = Torque de la
fuerza resultante. Luego, ahora nos ocuparemos de fuerza resultante
encontrar la magnitud de esta fuerza resultante y la y p F = ∫ ydF ⇒ y p ρgy G A = ∫ ρgy 2 dA
determinación de su línea de acción o punto de A A
aplicación.

2
y dA I
Las fuerzas horizontales causadas por la presión
⇒ yp = A
=
sobre superficies que encierran al fluido, aumentan yG A yG A
linealmente con la profundidad, de modo que se
tienen fuerzas distribuidas no uniformes actuando Donde I es el momento de inercia con respecto a un
sobre ellas. La resultante de ese sistema de fuerzas eje.
paralelas es en general una fuerza paralela aplicada en
un punto llamado centro de presión, respecto al cual APLICACIÓN: Superficie rectangular
el torque de las fuerzas distribuidas es equivalente al
torque de la fuerza resultante.

Para el caso de compuertas y situaciones similares, la


fuerza debido a la presión atmosférica actúa por El caso más simple es si la superficie es rectangular
ambos lados, y entonces la omitiremos del análisis como se indica en la figura que sigue donde se desea
por no contribuir en forma neta a la fuerza horizontal evaluar la fuerza resultante de las fuerzas distribuidas
actuando sobre la superficie. entre y1 e y 2 .
La figura siguiente ilustra una situación típica, donde
por el interior de una superficie hay un fluido y por el
exterior está la atmósfera.

F = ∫ pdA = ρg ∫ ydA
A A
y2
F = ρgL ∫ ydy
Para calcular la fuerza sobre superficie A en la pared y1

vertical. Tomemos un elemento de área dA de ancho ⎡ y2 ⎤


y2

L y altura dy que se encuentra a una profundidad = ρgL ⎢


2
1
⎥ = ρgL y 2 − y1
2
2 2
( )
y . La fuerza sobre este elemento diferencial es: ⎣ ⎦ y1
dF = pdA = ρgyLdy También podríamos calcularlo de otra forma
El centroide está en
La fuerza tota la encontramos integrando en toda la
1
superficie: F = ∫ pdA = ρg ∫ ydA yG = ( y 2 + y1 )
A A 2
El área A = L( y 2 − y1 )
Como ∫A
ydA = y G A
Y la fuerza es:
Donde y G es la posición del centroide del área de la
superficie sobre la que actúa la fuerza.
1
F = ρgy G A = ρgL y 22 − y12
2
( )
A es el área total de la superficie.
Para calcular el centro de presión:
Finalmente: F = ρgy G A

16
Mecánica de fluidos Hugo Medina Guzmán

yp =
∫ A
y 2 dA
=
I
yG A yG A
y2
I = ∫ y 2 dA = L ∫ y 2 dy
A y1

=
1
3
[ ]
y2 1
L y 3 y1 = L y 23 − y13
3
( )
Solución.
1
y G = ( y 2 + y1 ) , A = L( y 2 − y1 ) Elijamos el eje z a lo largo de la vertical, con el
2 origen al fondo de la caja sobre la tapa móvil. La
Reemplazando: presión a una altura z es p ( z ) = ρg (h − z ) .
1
3
(
L y 23 − y13 ) Dividamos la tapa en franjas horizontales de largo b y
ancho (altura) dz. La fuerza que ejerce el fluido sobre
yp = la franja que está a la altura z es
1
L( y 2 + y1 )( y 2 − y1 ) dF = p ( z )bdz .
2
⇒ yp =
(
2 y 22 + y 2 y1 + y12 ) Integrando la fuerza que el líquido ejerce sobre cada
una de las franjas se obtiene la fuerza total
3( y 2 + y1 ) 1
F = ∫ p ( z )bdz = ρgb ∫ (h − z )dz =
h h
En particular si la superficie está entre ρbgh 2 .
0 0 2
y1 = 0 e y 2 = h resultará Para encontrar a qué altura h p debemos aplicar esta
2
yp = h fuerza sobre la tapa, evaluemos el torque que ejerce el
3 fluido sobre la tapa respecto al origen.
APLICACIÓN: Fuerza sobre una superficie de El torque que el fluido ejerce sobre la franja que está
forma rectangular inclinada a la altura z es
dτ = zp ( z )bdz .
Integrando el torque que el líquido ejerce sobre cada
una de las franjas se obtiene el torque total
1
τ = ∫ zp ( z )bdz = ρgb ∫ z (h − z )dz
h h
= ρgbh 3 .
0 0 6
Para que la tapa esté en equilibrio el torque que ejerce
la fuerza total externa F debe ser igual en magnitud
con τ , es decir,
1 1 1
En una sección anterior se calculó la fuerza resultante Fh p = ρgbh 3 ⇒ ρgbh 2 h p = ρgbh 3
y centro de la fuerza para un área vertical de sección 6 2 6
rectangular. Para una sección rectangular inclinada un h
ángulo θ con la vertical, el cálculo es muy parecido, De esta ecuación se deduce finalmente que h p =
pero ahora, el eje Oy está inclinado luego resultarán 3

F=
1
2
(
ρgL y 22 − y12 cos θ ) Ejemplo 38. La figura nos representa el dique de un
embalse en el que el agua alcanza una profundidad h
y su punto de aplicación será = 60 m en la pared vertical, y tiene una longitud L =

yp =
(
2 y 22 + y 2 y1 + y12 ) 250 m. Calcular:
a) La fuerza resultante que actúa sobre el dique.
3( y 2 + y1 ) b) El torque o momento de la fuerza que tiende a
Note que la expresión para el centro de fuerza es la hacer girar el dique alrededor de OO'.
misma. c) Posición de la línea de acción de la resultante.

Ejemplo 37. Considere una caja de dimensiones a, b


y h, llena de agua. Todos los lados de la caja están
firmemente unidos entre sí, excepto uno de los lados
laterales (de dimensión b · h). Evalúe la magnitud de
la fuerza exterior mínima con que debe presionarse
ese lado contra el resto de la caja para que el agua no
escurra. Si la fuerza se aplica en un solo lugar,
encuentre la posición en la que debe aplicarla.
Solución.

17
Mecánica de fluidos Hugo Medina Guzmán

a)

Solución.
El valor de la fuerza sobre un elemento de área dA Cálculo de la fuerza resultante
será:
dF = pdA F = ∫ pdA = ρg ∫ ydA
A A

Con p = ρ a gh y dA = Ldy
y2
y2 ⎡ y2 ⎤
F = ρgL ∫ ydy = ρgL ⎢ ⎥
⇒ dF = ρ a gLydy y1
⎣ 2 ⎦ y1
Y la fuerza resultante es, por tanto:
H
F = ∫ dF = ρ a gL ∫
H 1
ydy = ρ a gLH 2
=
1
2
(
ρgL y 22 − y12 )
0 0 2
Expresión que podíamos haber obtenido aplicando
directamente:
1
= 1000(9,8)(1) 3,2 − 1,2
2
2 2
( )
F = ρghC A , sustituyendo valores: = 43120 N
Cálculo del centro de presión:
1
F= (1000)(9,8)(250)(60)2 = 4,42 x 109 N ∫ y 2 dA I
2 yp = A
=
b) El torque o momento de la fuerza dF respecto del yG A yG A
eje O O' es: y2
dτ = (H − y )dF = ρ a gLy (H − y )dy I = ∫ y 2 dA = L ∫ y 2 dy
A y1

[ ] ( )
y el torque resultante es: 1 y2 1
1 = L y 3 y1 = L y 23 − y13
y (H − y )dy = ρ a gLH 3
H H
τ = ∫ dτ =ρ a gL ∫ 3 3
0 0 6 1
Sustituyendo valores: y G = ( y 2 + y1 ) , A = L( y 2 − y1 )
1 2
τ= (1000)(9,8)(250)(60)3 = 8,82 x 1010 N Reemplazando:
6
c) Llamando h a la distancia por encima de O a la
1
3
(
L y 23 − y13 )
fuerza F para producir el torque τ calculado en (b), yp =
1
obtenemos: L( y 2 + y1 )( y 2 − y1 )
1 ⎛1 ⎞ 2
τ = hF ⇒
6
ρ a gLH 3 = h⎜ ρ a gLH 2 ⎟
⎝2 ⎠ =
(
2 y22 + y2 y1 + y12 )
H 3( y2 + y1 )
⇒ h= En particular si la superficie está entre
3
Sustituyendo valores: y1 = 1,2 e y 2 = 3,2 resultará:
h=
60
=20 m yp =
(
2 3,2 2 + 3,2 × 1,2 + 1,2 2 )
3 3(3,2 + 1,2 )
= 2,35 m.
Ejemplo 39. Determine la fuerza resultante y su
punto de aplicación debida a la acción del agua sobre Ejemplo 40. El agua se eleva hasta la altura c en el
la superficie plana rectangular de altura AB = 2 m y tubo soldado al tanque mostrado en la figura.
de ancho 1 m (hacia adentro del papel), donde el Despreciando el peso del tubo:
punto A está a profundidad de 1,2m. a) Determinar y localizar la fuerza resultante que
actúa sobre el área Lb.
b) Determinar la fuerza total en la base del tanque.
c) Comparar el peso total del agua con el resultado
obtenido en (b) y explicar la diferencia.

18
Mecánica de fluidos Hugo Medina Guzmán

a) El peso del agua en cada recipiente.


b) La fuerza sobre el fondo de cada uno.
c) La fuerza sobre las caras BC, EF y HK.
d) La fuerza sobre la cara vertical LMNO del cuarto
recipiente.

Solución.
a)
Solución. V1 = l 3 ⇒
a) La fuerza sobre el área A1 = Lb .
P1 = l 3 ρ a g
⎛ b⎞
F11 = ρghG1 A1 = ρg ⎜ c + ⎟ Lb ⎛ 1 ⎞
⎝ 2⎠ V2 = l 3 ⎜1 + cotan 60º ⎟ ⇒
(c + b ) ⎝ 2 ⎠
∫ y 2 dA L∫ y 2 dy ⎛ 1 ⎞
y p1 = A1
= c
P2 = l 3 ⎜1 + cotan 60º ⎟ ρ a g
y G1 A1 ⎛ b⎞ ⎝ 2 ⎠
⎜ c + ⎟ Lb
⎝ 2⎠ ⎛ 1 ⎞
V3 = l 3 ⎜1 − cotan 60º ⎟ ⇒
1
3
[ ]
(c + b )3 − c 3 2 (c + b )3 − c 3 [ ]
⎝ 2 ⎠
⎛ 1 ⎞
= =
3(2c + b )b P3 = l 3 ⎜1 − cotan 60º ⎟ ρ a g
⎛ b⎞ ⎝ 2 ⎠
⎜ c + ⎟b
⎝ 2⎠ V4 = l (1 + cotan 60º ) ⇒
3

b) La fuerza total en la base A2 = La del tanque.


P4 = l 3 (1 + cotan 60º )ρ a g
F2 = ρgh2 A2 = ρg (c + b )La Sustituyendo valores:
= ρg (Lac + Lab ) P1 = 10 N P2 = 12,89 N
c) El peso total del agua P3 = 7,11 N P4 = 15,77 N
b) La fuerza sobre el fondo de cada uno.
P = ρg (Lab − Sc )
F = ρgl(l 2 ) = 10 N
Resultado diferente al obtenido en (b)
Explicación: porque el peso es: c) La fuerza sobre las caras BC, EF y HK.
1 3
FBC = l ρ a g = 5N
2
FBF = FHK = 5,8 N
d) La fuerza sobre la cara vertical LMNO del cuarto
recipiente.
F = ρ a ghc A
l ⎛ l2 ⎞ l 2
2 ⎜⎜ ⎟+ l
3 ⎝ 2 3 ⎟⎠ 2
( )
hc = = 0,44l ,
P = F2 − F3 l2
+ l2
Donde: F2 = ρg (Lac + Lab ) y 3
F3 = ρgh3 A3 = ρgc(La − Sc ) l2
A= + l 2 = 1,58l 2
Luego: P = ρg (Lac + Lab ) − ρg (La − Sc ) 3
= ρg (Lab − Sc ) ( )
F = 1000 × 9,8(0,44l ) 1,58l 2 = 7 N

Ejemplo 41. Supongamos los recipientes de la forma TRASLACIÓN DE FLUIDOS.


indicada en la figura. El primer recipiente es cúbico, Un fluido puede estar sujeto a traslación o rotación
de 10 cm de arista; los otros tres recipientes tienen la con aceleración constante si movimiento relativo
misma base e igual altura y están llenos de agua. entre partículas. Esta condición de equilibrio relativo
Calcular: hace que el fluido este libre de esfuerzos cortantes y

19
Mecánica de fluidos Hugo Medina Guzmán

se aplican las leyes de la estática de fluidos teniendo


en cuenta los efectos de la aceleración.

Traslación horizontal.
Si el recipiente que contiene un líquido de densidad
ρ se traslada horizontalmente con aceleración
constante a x , la superficie inicialmente horizontal se
inclina con una pendiente que calcularemos a
continuación.

Para el prisma de líquido en la figura tenemos:


∑F y = ma y
p 2 A − p1 A − ρyAg = ρyAa x
⎛ ay ⎞
Simplificando: p 2 − p1 = ρgy⎜⎜1 + ⎟
⎝ g ⎟⎠
En la figura consideremos un prisma de líquido a lo
largo de una línea horizontal. La presión no varía Si el punto 1 estuviera en la superficie del líquido, la
igual que en un líquido en reposo, por lo tanto el presión en un punto cualquiera bajo la superficie a un
efecto de la aceleración a x será en la dirección x . a profundidad h sería:
Para el cuerpo libre se tiene:
⎛ ay ⎞
∑F x = ma x p = p a + ρgy⎜⎜1 + ⎟⎟
p1 A − p 2 A = ρLAa x , como p1 = ρgh1 y ⎝ g ⎠
p 2 = ρgh2 Rotación uniforme alrededor de eje vertical.
Podemos escribir: Si un recipiente abierto parcialmente lleno con un
ρgh1 A − ρgh2 A = ρLAa x líquido rota alrededor de un eje vertical con velocidad
angular constante, no hay movimiento relativo entre
Simplificando las partículas, la superficie que inicialmente era
(h1 − h2 ) ax
g (h1 − h2 ) = La x ⇒
horizontal toma una forma parabólica como lo
= demostraremos a continuación.
L g
Siendo
(h1 − h2 ) la pendiente de la superficie libre,
L
se tendrá finalmente:
ax
tan θ =
g
Como a x es constante, la superficie libre es un plano
inclinado.

Traslación vertical.
Si el recipiente que contiene un líquido de densidad
En la figura, consideremos una partícula de masa m
ρ se mueve con aceleración vertical a y , la en el punto A, aplicando la segunda ley de Newton se
superficie libre permanece horizontal. La presión es tiene:
constante en planos horizontales, pero es diferente a
cuando está en reposo, valor que calcularemos a
En el eje x : ∑F x = ma x
continuación. ⇒ Psenθ = mω 2 x (1)
En el eje y : ∑ Fy = 0
⇒ P cos θ − mg = 0 o P cos θ = mg (2)
Dividiendo (1) entre (2):
ω2x
tan θ =
g

20
Mecánica de fluidos Hugo Medina Guzmán

dy la fuerza producida por la resistencia del aire y la


Como la pendiente de la curva en A es tan θ = , fricción en ruedas es F f , ¿qué ángulo formaría la
dx
tenemos. superficie del agua con el fondo del tanque?
dy ω x 2
ω 2
= ⇒ dy = xdx
dx g g
ω 2 x2
Integrando: y = +C
2g
Para evaluar la constante, tenemos que para x = 0
→ y = 0 , por lo tanto C = 0.
Finalmente:
ω 2x2 Solución.
y= , ecuación de la parábola. Primeramente hallemos la aceleración a x del tanque
2g
que desciende por el plano.
La presión manométrica a una profundidad h del
vértice de la parábola será: ∑F x
= Ma x ⇒
⎛ ω 2 x2 ⎞ Mgsenα − F f = Max
p = ρg (h + y ) = ρg ⎜⎜ h + ⎟⎟
⎝ 2g ⎠ Ff
La aceleración será: a x = gsenα −
M
Ejemplo 42. Sobre un automóvil de carreras se La aceleración perpendicular al fondo del tanque es
instala un tubo en U lleno de agua. El conductor
a y = g cos α
acelera uniformemente desde el arranque, al cabo de
5 segundos el agua contenida en el tubo tiene la El ángulo θ que forma la superficie del agua con el
posición señalada en la figura. ¿Cuáles son la fondo del tanque (dirección x) se encuentra de la
aceleración y la velocidad del automóvil en ese siguiente manera:
instante? (No tome en cuenta los efectos viscosos a x = a y tan θ ⇒
transitorios del agua del tubo).
ax gsenα − F f M
tan θ = =
ay g cos α

Ejemplo 44. Un tanque sufre una caída libre.


Encuentre la diferencia de presión entre dos puntos
separados por una distancia vertical h .

Solución.
Observamos en el esquema que la gravedad efectiva
es normal a la línea trazada por los extremos de la
columna de agua. Sus extremos están a la presión
atmosférica y quedan en una línea de presión
constante. Podemos calcular fácilmente la magnitud
de a :
a = g tan θ = g tan 20º = 9,81 × 0,364
= 3,57 m/s2
La magnitud de la velocidad del automóvil se Solución.
determina de la siguiente ecuación: La diferencia de presiones entre dos puntos de un
fluido que se mueve verticalmente con aceleración a
dx m
a= = 3,57 2 ⎛ a⎞
dt s es ( p 2 − p1 ) = ρgh⎜⎜1 + ⎟
Integramos y para t = 5 s: ⎝ g ⎟⎠
m
v = 3,57t = (3,57 )(5) = 17,85 = 64
km Luego ( p 2 − p1 ) = 0 , consecuentemente
s h p 2 = p1
Ejemplo 43. Un tanque abierto, lleno de agua, rueda Ejemplo 45. Se tiene un tubo en U de área A y con
sobre un plano inclinado, que forma un ángulo α un fluido de densidad ρ , como se muestra en la
con la horizontal. Si el tanque tiene una masa M y

21
Mecánica de fluidos Hugo Medina Guzmán

figura. Determinar la diferencia de altura H que se L L2


producir entre las alturas que alcanza el líquido en F = ∫ dF = ω 2 Aρ ∫ rdr = ω 2 Aρ
cada una de las ramas cuando, 0 2
a) Se le imprime una aceleración lineal horizontal. Nuevamente, en equilibrio, la igualaremos:
b) Rote con una velocidad angular constante a al peso de la columna de líquido de altura H ,
alrededor de un eje vertical que coincide con una de L2
sus ramas. F = P ⇒ ω 2 Aρ = ρgHA
2
ω 2 L2
Finalmente: H =
2g

TENSION SUPERFICIAL - CAPILARIDAD

TENSIÓN SUPERFICIAL
Entre dos moléculas de un fluido actúan fuerzas.
Estas fuerzas, llamadas fuerzas de van der Waals o
Solución. fuerzas cohesivas son de origen eléctrico. Una de las
a) Solamente la masa de líquido que está en la parte características de estas fuerzas es que su alcance es
horizontal podrá desplazarse bajo la acción de la muy pequeño (rápidamente se desvanecen cuando la
aceleración, pues, la masa de líquido que esta en las distancia entre las moléculas es dos o tres veces su
ramas verticales tiene su movimiento restringido, por tamaño); otra característica es que mientras las
ser perpendiculares. moléculas no se traslapan, la fuerza es atractiva.
El efecto neto de las fuerzas de cohesión sobre una
molécula que está en el interior del líquido es nulo,
pero no así para una molécula que se encuentra en la
superficie.

Como todos los elementos diferenciales de masa en la


parte horizontal tienen la misma aceleración, la fuerza
total será:
F = ma = ρVa = ρALa
Esta fuerza, al alcanzarse el equilibrio, debe ser igual
Para poner una molécula en la superficie hay que
al peso de la columna de líquido de altura H, que es:
realizar un trabajo. O sea, la existencia de una
P = pA = ρgHA superficie en un fluido introduce una energía
Luego, igualando F = P ⇒ ρALa = ρgHA potencial. Esta energía es proporcional a la superficie
y se tiene que
a
De donde H = L dW = γ dA
g Aquí γ es una constante que depende del fluido y se
b) En este caso se tiene la acción de la aceleración
llama tensión superficial y dA es un elemento
centrípeta a c = ω r , al ser horizontal, como en el
2
(infinitesimal) de superficie. En realidad la tensión
caso anterior, solo actúan sobre la masa de líquido superficial depende de las dos sustancias que están en
que está en la parte horizontal del tubo, pero, como contacto.
es variable, función del radio r , la fuerza sobre cada
elemento diferencial de masa será: Medición de la tensión superficial.
dF = (dm )a = (ρAdr )ω r 2 Para medir la tensión superficial se puede usar el
dispositivo mostrado en la figura. Un alambre
movible, inicialmente sumergido, se tira lentamente,
extrayéndolo del líquido (con una película del líquido
adosada).

Integrando, tendremos la fuerza total F:

22
Mecánica de fluidos Hugo Medina Guzmán


Δp = .
R
Con γ = 0,025 N/m y R = 0,01 m se obtiene
Δp = 10 N/m2.
Si se deja de soplar por la pajita, la pompa se
desinfla.
Observe que la presión al interior de una pompa de
jabón es mayor tanto más pequeño es su radio. De
esta observación se deduce que al juntarse una pompa
de jabón grande con una pequeña, la pequeña inflará
a la más grande. De esta manera la pompa grande
aumentará su tamaño mientras que la pequeña
disminuirá: en otras palabras, la más grande
absorberá a la más pequeña.
La energía para desplazar la longitud d es Fd y el área Otra manera.
de la película se incrementa en 2dL, considerando que
existen dos superficies. La pompa es una película delgada sostenida por la
La relación entre la energía necesaria para realizar el tensión superficial de dos superficies (la superficie
desplazamiento y el área incrementada es la tensión externa y la superficie interna).
superficial
Energía
γ =
Area formada
En el caso del dispositivo empleado:
Fd F
γ = =
2 Ld 2 L
F es la fuerza paralela a la superficie de la película
necesaria para mantener la película extendida. Esta
Δp = p − p a
fuerza por unidad de longitud es la tensión superficial Fuerza debida a la presión dentro de la pompa.
γ.
Así la tensión superficial γ no sólo es igual a la F p = ( p − p a )πR 2 = ΔpπR 2
fuerza por unidad de longitud; sino también es igual
al trabajo hecho por unidad de incremento del área Fuerza debida a la tensión superficial de las dos caras
superficial. De ahí que y pueda especificarse en N/m de la pompa
o en J/m2. Fγ = γ 2(2πR ) = γ 4πR
Ejemplo 46. Deseamos encontrar la diferencia de Como están en equilibrio:
presión entre el interior y exterior de una pompa de
jabón de radio R = 1 cm. F p = Fγ
Solución.
Si, soplando con una pajita, aumentamos el radio de ΔpπR 2 = γ 4πR
la pompa de R a R + dR, entonces la superficie
aumenta en 4γ
[ ]
dA = 2 4π (R + dR ) − 4πR 2 = 16πRdR
2 ⇒ Δp =
R
El factor 2 nuevamente se debe a que hay que
considerar tanto la superficie interior como exterior La gota y la burbuja.
de la pompa. En el caso de la gota y la burbuja solamente hay una
El cambio de energía debido al aumento de la superficie que las encierra por lo tanto:
superficie es por lo tanto
dW = γ dA = 16γπRdR Fγ = γ 2πR
Por otra parte, podemos evaluar el trabajo
La diferencia de presión es:
directamente, multiplicando el desplazamiento
dR por la fuerza Δp (4πR ), es decir,
2
Δp =

dW = Δp.4πR 2 dR . R
Igualando las dos últimas expresiones se encuentra la Ejemplo 47. Un alambre con forma circular, 0,08 m
diferencia de presión de diámetro, con un alambre que puede deslizar en él,
está en un plano horizontal. Se forma una película

23
Mecánica de fluidos Hugo Medina Guzmán

líquida, limitada por los alambres, en el lado Siendo ΔA = 2(4πR 2 ) = 8πR 2


izquierdo, como se muestra en la figura. La tensión
superficial del líquido es 25 mN/m. una fuerza Obtenemos:
aplicada F, perpendicular al alambre deslizante ΔW = γ 8πR 2
mantiene a la película en equilibrio.
INSECTOS QUE CAMINAN SOBRE EL AGUA.

a) Cuando el alambre deslizante se encuentra a 0,06 Debido a la tensión superficial, los insectos pueden
m del punto P, la fuerza aplicada F, es: caminar sobre el agua y cuerpos más densos que ésta,
b) Cuando la fuerza F es 1,5 mN, la distancia del como una aguja de acero, pueden flotar realmente
alambre deslizante al centro del círculo es: sobre la superficie. La figura muestra cómo puede
c) Cuál es el valor máximo de la fuerza F: soportar el peso P de un objeto la tensión superficial.
Solución. En realidad, P es el “peso efectivo” del objeto (su
a) peso verdadero menos la fuerza de empuje) puesto
que el objeto se sumerge ligeramente en el fluido.

Si el objeto tiene forma esférica, que es


aproximadamente la forma que tienen las patas de los
insectos, la tensión superficial actúa en todos los
L = 2 0,04 2 − 0,02 2 = 0,069 m puntos a lo largo de un círculo de radio r . Sólo la
componente vertical, γ cos θ , actúa para equilibrar
mN
F = γ (2 L ) = 25 (2 × 0,069 m ) = 3,46 mN P. En consecuencia la fuerza neta ascendente debida
m a la tensión superficial es 2πrγ cos θ .
b)
F'
F ' = γ (2 L') ⇒ L' =

1,5
L' = = 0,03 m
2 × 25
Luego
Tensión superficial actuando sobre la pata de un
d = 0,04 2 − 0,03 2 = 0,026 m insecto.
c)
Fmax = γ (2 Lmax ) = 25(2 × 0,08) ADHESIÓN Y COHESIÓN.
En las superficies de un líquido algunas de sus
= 4,0 mN
moléculas dejan el líquido por evaporación, pero no
todas. Existe una fuerza de atracción entre las
Ejemplo 48. Cuál es el trabajo requerido para formar
moléculas de un líquido, por ejemplo una gota de
una pompa de jabón de radio R, usando una solución
mercurio tiene la tendencia a asumir la forma
jabonosa de tensión superficial γ .
esférica, esto es, una superficie de área mínima,
Solución. consistente con la fuerza atractiva entre moléculas,
Energía ΔW esta propiedad es conocida como cohesión.
Como γ = = La atracción que existe entre las moléculas de dos
Area formada ΔA
sustancias diferentes, como la atracción que hay entre
⇒ ΔW = γ ΔA el líquido y las paredes del recipiente que lo contiene,
es la propiedad conocida como adhesión.

24
Mecánica de fluidos Hugo Medina Guzmán

Consideremos una pequeña cantidad de líquido en La tensión superficial γ actúa en un ángulo θ


contacto con una superficie sólida plana y ambos en alrededor de un círculo de radio r . La magnitud de la
contacto con un gas.
fuerza vertical F debida a la tensión superficial es
Si la fuerza de adhesión (entre el líquido y el sólido)
es mucho mayor que la fuerza de cohesión (entre las F = (γ cos θ )(2πr ) . Esta fuerza está equilibrada
moléculas del líquido), entonces el líquido tenderá a por el peso del líquido de abajo que es
esparcirse sobre el sólido. En este caso se dice que el aproximadamente un cilindro de altura h y volumen
líquido moja al sólido,
V = πr 2 h . En consecuencia,
2πrγ cos θ = mg
Como mg = ρVg = ρπr hg , donde ρ es la
2

Si la fuerza de cohesión es mayor entonces el líquido densidad del líquido, tenemos:


tenderá a concentrarse, adquiriendo una forma 2πrγ cos θ = ρπr 2 hg
compacta tipo gota Resolviendo para h encontramos
2γ cos θ
h=
ρgr
Para mayor parte de los líquidos como el agua en un
Como resultado de esta competencia entre las vaso, θ , es casi cero y
distintas fuerzas de adhesión y cohesión, se forma un 2γ
ángulo de contacto θ bien característico entre el h=
líquido y el sólido. ρgr
Experimentalmente se determina que este ángulo de Esta ecuación también se cumple cuando desciende el
contacto para las substancias, en el caso θ < 90º el líquido, como en el caso del mercurio en un tubo de
fluido es humectante, o sea moja al sólido y cuando vidrio. En esta situación, el ángulo de contacto mayor
θ > 90º el fluido es no humectante. que 90° y cos θ será negativo; esto hace h negativa
lo que corresponde a un descenso de nivel.
Note que mientras más delgado sea el tubo mayor
CAPILARIDAD
será el ascenso (o descenso) del líquido.
En tubos que tienen diámetros muy pequeños se
observa que los líquidos se elevan o se hunden en
relación con el nivel del líquido de los alrededores. Ejemplo 49. Un cuadrado cuyas aristas miden 6 cm
hecho de una placa delgada de metal se suspende
Este fenómeno se conoce por capilaridad y dichos
tubos delgados se llaman capilares. El que un líquido verticalmente de una balanza tal que el borde inferior
suba o baje depende de los esfuerzos relativos de las de la hoja se moja en agua de tal forma que es
paralela a la superficie. Si la hoja está limpia, el
fuerzas adhesivas y cohesivas. Así, el agua sube en
un tubo de vidrio en tanto que mercurio baja. ángulo de contacto es 0º, y la hoja parece pesar 0,047
N. Si la hoja esta grasosa, el ángulo de contacto es
180º y el peso parece ser 0,030 N. ¿Cuál es la tensión
superficial del agua?
Solución.
Cuando la hoja está limpia

La cantidad real que sube (o que baja) depende de la


tensión superficial (puesto que es ésta la que
mantiene unida a la superficie del líquido), así como
del ángulo de contacto θ , y el radio r del tubo. Para
calcular h , la altura que nos referiremos a la figura
siguiente. La fuerza de tensión superficial en cada cara de la
placa es:
T = γL
No tomaremos en cuenta las partes del espesor, por
ser placa delgada.
Como hay equilibrio vertical
F1 = P + 2T , (1)
Cuando la hoja está grasosa

25
Mecánica de fluidos Hugo Medina Guzmán

Cuando el cuerpo cilíndrico de vidrio cerrado se


sumerge en agua aparecen las fuerzas de empuje y la
de tensión superficial
La tensión superficial actúa a lo largo del contacto del
líquido con el cilindro. La fuerza hacia abajo debida a
ella es:
F = 2πRγ
h
El empuje vale: E = Vs ρ a g = πR 2 ρa g
2

F2 = P − 2T (2)
Restando (2) de (1):
F1 − F2 F − F2
T= ⇒ γL= 1
4 4
F1 − F2
⇒ γ =
4L
Reemplazando valores:
0,047 − 0,030 0,017 Para volver al equilibrio (balanza horizontal) se
γ = =
4(0,06) 0,24 colocan las pesas en el platillo izquierdo de la balanza
(peso Mg), esto anula la acción del empuje E y a la
= 0,071 N/m fuerza de la tensión superficial F.
Ejemplo 50. Del platillo de una balanza se cuelga un
cuerpo cilíndrico de vidrio cerrado por su base
inferior, de 1 cm de radio y 4 cm de altura, y se pone
tara en el otro platillo hasta conseguir el equilibrio. Se
sumerge el cuerpo en agua destilada a 4º C hasta la
mitad de su altura exactamente. Para restablecer el
equilibrio hace falta poner en el platillo del cuerpo
pesas por valor de 5,8 g. Calcular el coeficiente de
tensión superficial del agua. El ángulo de contacto se
supone de cero grados, es decir, que el menisco es
tangente a la superficie lateral del cilindro. Por lo tanto
h
E = F + Mg ⇒ πR 2 ρ a g = 2πRγ + Mg
2
πR 2 hg − 2 Mg
γ =
4πR
(
= π 10
−2
) (4 × 10 )(10 )(9,8) − 2(5,8 × 10 )(9,8)
2 −2 3 −3

4π (10 )
−2

= 75,36 x 10-3 N/m


Solución. Ejemplo 51. Tenemos unas ampollitas de vidrio de
La tara T equilibra al sistema paredes muy estrechas en la forma indicada en la
figura. La ampollita va lastrada en su parte inferior
con mercurio para que se mantenga en la posición de
la figura (a) al dejarla sobre el nivel de un recipiente
con agua. Sumergimos el sistema hasta la posición de
la figura (b). Debería recobrar la posición a, pero se
queda en la b. ¿Por qué? El ángulo de contacto se
supone de cero grados

26
Mecánica de fluidos Hugo Medina Guzmán


2π rγ = π r 2 hρg ⇒ rh =
ρg
Ejemplo 53. Ocho gotas de mercurio de radio r se
unen para formar una sola. ¿Qué relación existe entre
las energías superficiales antes y después de la unión?
Solución.
Solución. El volumen de la gota formada, que tendrá por radio
Llamamos r y R los radios de la parte cilíndrica y de R, será ocho veces mayor que el volumen de una de
la esferita, respectivamente: R > r. las gotas pequeñas:
En la posición (a) el valor de la tensión superficial es:
4 3 4 R
F = 2π rγ πR = 8 π r 3 ⇒ R 3 = 8r 3 ⇒ =2
Y al estar en equilibrio, el empuje ha de ser igual al
3 3 r
La energía superficial de las ocho gotas será ocho
peso más la fuerza correspondiente a la tensión
veces la energía de una sola:
superficial:
E = P + 2π rγ W = 8γ 4π r 2 = 32γπ r 2
Y de la gota resultante: W ' = γ 4π R = 4γπR
2 2
Al sumergir la ampollita la fuerza debida a la tensión
superficial es: F ' = 2π Rγ
W r2 1
Y se habrá de verificar: Dividiendo: =8 2 =8 =2
E ' = P + 2π Rγ . W' R 4
La disminución que experimenta la superficie del
Y como el peso es el mismo, nos queda: mercurio (o de otro líquido cualquiera) al juntarse las
E − 2π rγ = E '−2π Rγ ⇒ gotas pequeñas para formar una grande, libera una
Vρg − 2π rγ = V ' ρg − 2π Rγ determinada energía que se emplea en calentar la
gota. Por el contrario cuando una gota grande se
V '−V 2π γ
⇒ = divide en otras más pequeñas, se produce un aumento
R−r ρg de energía en la película superficial y, como
Condición que se debe cumplir para que exista el consecuencia un determinado enfriamiento de las
segundo equilibrio. gotas.

Ejemplo 52. Los cilindros huecos y cerrados de la Ejemplo 54. El aceite de olivo tiene una tensión
figura son de vidrio y están unidos por la varilla superficial respecto del aire de 32 mN/m. Una gota
OO’; el inferior se ha lastrado con el mercurio esférica tiene un diámetro de 4 mm. Calcular:
necesario para que el sistema flote en un líquido, con a) La presión a que está sometida.
el cilindro inferior sumergido. Sumergimos el sistema b) La fuerza total a la que está sometida, debida a la
hasta que quede también flotando en la forma de la tensión superficial que actúa sobre su superficie.
figura (b), sin recobrar la primitiva posición (a). c) La energía potencial de superficie.
Demostrar que se debe cumplir: Solución.
rh = 2γ / ρg , 2γ 2(32 × 10 −3 )
a) p= =
γ es la tensión superficial y r 2 × 10 −3
ρ es la densidad del líquido respectivamente. = 32 Pa
F = pA = p 4πR 2 = 32 × 4π (2 × 10 −3 )
2
Se supone la varilla OO’ infinitamente delgada, que b)
el líquido moja al vidrio y que el ángulo de contacto
= 1,608 mN
es nulo.
W = γA = γ 4πR 2 = (32 × 10 −3 )4π (2 × 10 −3 )
2
Solución. c)
= 1,608 μJ

Ejemplo 55. Calcular la energía superficial de una


pompa de agua jabonosa de 1 cm de radio y la
presión debida a su curvatura. Consideramos el
espesor de la película líquida como despreciable.
Tensión superficial = 35 x 10-5 N/cm.
En el primer equilibrio: Mg = Vρg .
Solución.
W = 2γA = 2γ 4πR 2 = 2(35 × 10 −3 )4π (10 − 2 )
V = volumen del cilindro inferior. 2
En el segundo equilibrio:
Mg + 2π rγ = Vρg + π r 2 hρg = 87,96 μJ
Luego teniendo en cuenta la primera, nos queda:
p=2
( ) (
2γ 4 35 × 10 −3 4π 10 − 2
=
)2

r 10 − 2
27
Mecánica de fluidos Hugo Medina Guzmán

= 14 Pa paralelas separadas 0,05 mm es de 125°, calcular la


altura que desciende el mercurio al introducir tubo y
Ejemplo 56. En un dispositivo como el de la figura láminas en una cubeta con dicho líquido.
se han conseguido dos pompas de agua jabonosa en
los extremos D y E de los tubos. La llave A
incomunica el aire interior de las dos pompas. Abierta
tal llave, la pequeña se achica y la grande aumenta de
volumen. ¿Por qué?

Solución.
Hacemos este problema y los dos siguientes en el
sistema cgs.
a) En el tubo
2γ cos θ
h=
Solución. rρg
Las presiones del gas interior de las pompas pequeña 2(547 ) cos125º
y grande, respectivamente, exceden a la atmosférica = = -1 cm
2γ 2γ (0,05)(13,6)(980)
en: p1 = 2 p2 = 2 . b) En las láminas
r R
2γ cos θ
Al ser r < R, se ha de verificar que p1 > p2, y el aire h=
pasa de la pompa pequeña a la grande dρg
2(547 ) cos125º
Ejemplo 57. Sabiendo que la tensión superficial del = = - 10 cm
agua es 75 x 10-3 N/m. Calcular la altura a que (0,005)(13,6)(980)
asciende el agua en un tubo de 1 mm de diámetro y
en unas láminas cuadradas paralelas cuya distancia es El signo menos indica el descenso.
0,05 mm. Se supone el ángulo de contacto igual a
cero. Ejemplo 60. En un tubo en U cuyas ramas son de 0,6
Solución. mm y 0,6 cm de diámetro se introduce un líquido de
Como el líquido no moja: θ = 0 ⇒ cos θ = 1 , densidad 1,8 g/cm3 y de 32 dina/cm de tensión
luego: superficial. ¿Cuál será la diferencia de nivel del
2γ 2(75 × 10 −3 ) líquido en las dos ramas del tubo, si éste se encuentra
h1 = en posición vertical y el ángulo de contacto es 32°?
rρg (5 × 10 − 4 )(10 3 )(9,8)
=

= 0,031 m
La altura alcanzada entre dos láminas paralelas es:
2γ cos θ 2(75 × 10 −3 )
h2 =
dρg
=
(5 × 10 −5 )(103 )(9,8)
= 0,31 m

Ejemplo 58. El tubo de un barómetro de mercurio


(tensión superficial, 547 x 10-3 N/m; ángulo de
contacto, 125°) tiene 3 mm de diámetro. ¿Qué error Solución.
introduce en las medidas la tensión superficial? 2γ cos θ ⎡1 1 ⎤
Solución. Δh = ⎢ − ⎥
rρg
2γ cos θ 2(547 × 10 −3 )cos125º ⎣ r1 r2 ⎦
h= 2(32) cos 32º ⎡ 1
rρg
=
(1,5 × 10 −3 )(13600)(9,8) = −
1 ⎤
= 1 cm
= -0,003 m (1,8)(980) ⎣ 0,03 0,3 ⎥⎦

El signo menos nos indica que la medida es inferior a
la correcta. Ejemplo 61. En un experimento para calcular el
ángulo de contacto entre un líquido y el vidrio se han
Ejemplo 59. Sabiendo que la tensión superficial del obtenido los siguientes datos: densidad del líquido,
mercurio es 547 dina/cm y que el ángulo de contacto 0,8 g/cm3; radio del capilar, 0,5 mm; elevación en el
con un tubo de 1 mm de diámetro y con unas láminas

28
Mecánica de fluidos Hugo Medina Guzmán

tubo capilar, 1,2 cm; tensión superficial del líquido 28


dina/cm. Calcular dicho ángulo.
Solución.
2γ cos θ
h=
rρg
rρgh (0,05)(0,8)(9,8)(1,2)
⇒ cos θ = =
2γ 2(28)
= 0,84
⇒ θ = 32º 51' 36' '

Ejemplo 62. Demostrar que la línea de contacto de Sea γ la tensión superficial del líquido,
un líquido con dos láminas de vidrio verticales que
consideremos el ángulo de contacto cero.
forman entre sí un ángulo diedro muy pequeño es una
hipérbola equilátera. ∑F V =0
P − 2πRγ = 0 ⇒ P = Mg = 2πγR

Ejemplo 63. El estalagmómetro, aparato destinado a


la medida de tensiones superficiales, es una pipeta de
la que se vierte gota a gota, en una primera
experiencia, el líquido problema, contándose el
número de gotas n correspondientes a un determinado
volumen: se repite el recuento para el mismo
volumen de agua, obteniéndose n’ gotas. Determina
la tensión superficial del líquido (γ ) conocida la del
Solución. agua (γ ') y las densidades (ρ y ρ ') de ambos
líquidos.
Solución.
Las masas de una gota de líquido y de agua son:
Vρ Vρ '
M = M '=
n n'
Por división, y teniendo en cuenta la ley de Tate (ley
Tomaremos los ejes sobre una de las láminas: del cuentagotas):
2γ cos θ θ M ρ n' γ ρ n'
y= d = 2 xsen = = ⇒ γ =γ'
dρg 2 M ' ρ' n γ ' ρ' n
Luego
γ cos θ Ejemplo 64. En el platillo izquierdo de una balanza
y= ⇒ se coloca una tara; en el derecho un vasito y pesas de
θ
xρgsen masa M1 hasta equilibrarla. Se quitan las pesas y se
2 vierte en el vaso con un cuentagotas, n gotas de un
γ cos θ líquido; se vuelve a equilibrar la balanza (la misma
xy = = constante (l. q. d.)
θ tara) con pesas de masa M2. Se quitan éstas y se
gρsen vierten en el vasito, sobre el líquido, n gotas de agua.
2 Se consigue de nuevo el equilibrio con pesas de masa
M3. Conocida la constante de tensión superficial del
agua γ ' determinar la del líquido (γ ) .
Ley de Tate de la formación de gotas mediante un
cuentagotas.
Consideremos un gotero con agujero de salida de Solución.
radio R, Masa de n gotas de líquido:
El liquido ira saliendo formando la gota, la que se nM = M 1 − M 2
mantendrá unida al cuentagotas mientras la tensión Masa de n gotas de agua:
superficial la mantenga. Cuando el peso de la gota nM ' = M 2 − M 3
iguale a la tensión superficial, esta caerá como gota
suelta. Por división obtenemos:
M M − M2
= 1
M' M2 − M3

29
Mecánica de fluidos Hugo Medina Guzmán

Aplicando la fórmula de Tate al líquido y al agua, nos independientes del tiempo y la temperatura o sea que
da: no cambian en el punto (puede ser diferente de punto
P = Mg = 2π rγ ⎫ M γ a punto del espacio). Cuando ocurre lo contrario el
⎬ = flujo es no estacionario.
P' = M ' g = 2π rγ '⎭ M' γ' Un flujo en un campo es uniforme cuando el vector
Igualando: velocidades constante e igual n todos los puntos de
γ M1 − M 2 aquel campo y es no uniforme cuando el vector
= velocidad está variando.
γ ' M2 − M3 Un flujo es turbulento cuando las partículas del
M1 − M 2 fluido tienen un movimiento irregular, caótico
⇒ γ =γ' causando pérdidas de energía proporcionales al
M2 − M3 cuadrado de la velocidad, lo contrario ocurre cuando
el movimiento es suave, ordenado, sus pérdidas son
DINÁMICA DE FLUIDOS - MOVIMIENTO DE proporcionales a la velocidad y se conoce como flujo
UN FLUIDO laminar. (en cada punto no hay velocidad angular
El flujo describe el cambio en la posición de las respecto a ese punto).
partículas del fluido en el tiempo. Experimentalmente se ha encontrado que hay una
La descripción op1eta del movimiento de un fluido es combinación de cuatro factores que determinan si el
compleja por lo tanto, en el tratamiento que f1ujo por un tubo es laminar. Esta combinación es
utilizaremos será necesario suponer algunas conocida domo el Número de Reynolds, NRe y se
simplificaciones. define como
En particular, no analizaremos el comportamiento de ρ vD
cada una de las partículas con los conceptos de la N Re =
mecánica, sino más bien describiremos las
η
características del movimiento en cada punto del Donde:
espacio conforme transcurre el tiempo. ρ = densidad
v = velocidad promedio
LÍNEA DE FLUJO. Es una línea imaginaria
continua que denota en cada uno de sus puntos la
η = viscosidad .
dirección del vector velocidad del fluido. Las líneas D = diámetro de la tubería
de flujo de un sistema estable nunca se cruzan una a El número de Reynolds no tiene dimensiones, por lo
otra (pues una partícula podría seguir dos tanto, es independiente del sistema de unidades
direcciones) y representan utilizado.
Un patrón instantáneo de flujo el cual en otro instante Se observa que hasta el va1or de 2000 el flujo es
puede ser completamente diferente. laminar y para valores mayores de 3000 el flujo es
turbulento.

ECUACIÓN DE LA CONTINUIDAD.
De la conservación de la masa del líquido en un tubo
del flujo, resulta inmediatamente la ecuación de la
Si seleccionamos un número finito de líneas de continuidad.
corriente como se muestra en la figura, esta región
tubular se denomina tubo de flujo, las fronteras de
este son líneas de corriente y por lo tanto ninguna
partícula puede cruzar este tubo, comportándose
como una verdadera tubería.

Consideremos un tubo de flujo constante de un


líquido no viscoso; tal como el mostrado en la figura.
Sean 1 y 2 dos sectores cuyas secciones tienen áreas
normales al flujo A1 y A2, con velocidades v1 y v 2
respectivamente.
CARACTERÍSTICAS GENERALES DEL Considere las porciones sombreadas de los líquidos
FLUJO DE FLUIDOS: en 1 y 2. Luego, en un intervalo de tiempo Δt la
El flujo puede clasificarse como estacionario (o
estable) y no estacionario uniforme y no uniforme, masa de líquido Δm1 pasa por la sección 1 y la masa
laminar (o irrotacional) o turbulento (o rotacional), Δm2 que pasa por la sección 2 deben ser iguales,
compresible e incompresible y viscoso y no viscoso. porque las mismas partículas son las que se mueven
Un flujo es estacionario cuando los parámetros del
flujo (velocidad, densidad., presión) son

30
Mecánica de fluidos Hugo Medina Guzmán

en el tubo de flujo, sin haber ingresado o salido va disminuyendo con la distancia al caño y que al
partículas. Tal que Δm1 = Δm 2 . final, se rompe formando gotas.
Pero Δm1 = ρ1 ΔV1 = ρ1 A1v1 Δt y
¿Cuál es la velocidad del agua cuando a recorrido una
Δm2 = ρ 2 ΔV2 = ρ 2 A2 v 2 Δt distancia h.?
Donde ΔV1 y ΔV2 son los volúmenes del líquido
Solución. La ecuación de continuidad nos
en las secciones 1 y 2 respectivamente y ρ1 y ρ 2 proporciona la forma de la superficie del chorrito de
son las densidades del líquido en 1 y 2. agua que cae del grifo, tal como apreciamos en la
De tal manera que: ρ1 A1v1 Δt = ρ 2 A2 v 2 Δt ⇒ figura.
ρ1 A1v1 = ρ 2 A2 v 2 La sección trasversal del chorro de
Si consideramos el fluido incompresible o poco agua cuando sale del caño es A0, y
incompresible como los líquidos. la velocidad del agua es v0. Debido
ρ1 = ρ 2 , y ρ1 A1 = ρ 2 A2 ⇒ Av = Constante a la acción de la gravedad la
Ahora Av = Constante velocidad v del agua se incrementa.
A una distancia h del grifo la
distancia Volumen
Av = área × = = Gasto (G) velocidad es
tiempo tiempo
A esta razón de flujo de volumen G = Av = v 2 = v02 + 2 gh
constante, se le conoce con el nombre de GASTO o
CAUDAL y sus unidades son m3/s.
Aplicando la ecuación de
Ejemplo 65. El agua fluye en una manguera de continuidad
jardín de diámetro interior 2 centímetros a una
velocidad de 1,2 m/s. ¿Con qué velocidad emergerá A0 v0 = Av ⇒ π r02 v0 = π r 2 v
de un eyector del diámetro 0,5 centímetros?
Solución.
Despejamos el radio r del hilo de agua en función de
π (0,01)2
A
v 2 = 1 v1 = (1,2) = 4,8 m la distancia h al caño.
A2 π (0,025) 2
s
v02
Ejemplo 66. Calcule la velocidad media de la sangre r = r0 4
en la aorta (radio 1 centímetro) cuando el caudal es 5 v02 + 2gh
litros/min.
Solución. ECUACIÓN DE BERNOULLI.
5 litros 5000 cm 3 cm 3 Al aplicar las leyes de Newton a los fluidos en
Caudal = = = 83,33 movimiento se obtiene la ecuación de Bernoulli.
min 60 s s
Caudal = Av
Caudal 83,33
⇒ v= = = 26,54 cm/s
A π (1)2

Ejemplo 67. Una manguera de 2 cm. de diámetro por


Tomemos una partícula de fluido de forma prismática
la que fluye agua a una velocidad de 3m/s. termina en
un tubo cerrado que tiene 50 orificios pequeños de (sección A largo Δs ) que se mueve a lo largo de
0,2cm de diámetro. ¿Cuál es la velocidad de salida una línea de flujo en la dirección s . La partícula
del agua en cada agujero? prismática se muestra en detalle en la siguiente
Solución. figura.
Por la ecuación de la continuidad
A1v1 = 50 A2 v 2 ⇒ π (1) (3) = 50π (0,2 ) v 2
2 2

3 m
v2 = =6
50(0,01) s

Ejemplo 68. Cuando se abre poco a poco un caño de


agua, se forma un pequeño chorro, un hilo cuyo radio

31
Mecánica de fluidos Hugo Medina Guzmán

Considerando un fluido no viscoso, o sea, que no hay 1 2


pérdidas de energía, aplicamos la segunda ley de p a = p a − ρgh + ρv 2 ⇒ v 2 = 2 gh
Newton 2
que es la misma velocidad que tendría en caída libre
∑F s = ma s desde una altura h .
Las fuerzas que actúan son el peso y las fuerzas
debido a las presiones p y p + dp , la masa de la El atomizador.
partícula es Δm = ρAΔs La presión en el aire soplado a alta velocidad a través
de la parte superior del tubo vertical de atomizador.
Luego: Un atomizador de perfume o de un rociador de
pA − ( p + Δp )A − ρgAΔs cos θ = ρAΔs a s insecticida es menor que la presión normal del aire
Simplificando y dividiendo entre Δs : que actúa sobre la superficie del líquido en el frasco,
así el perfume es empujado hacia arriba del tubo
Δp
+ ρg cos θ + ρa s = 0 debido a la presión reducida en la parte superior.
Δs
En el límite Δs → 0
dp
+ ρg cos θ + ρa s = 0 (1)
ds
Como
dz dv ds dv dv EFECTO MAGNUS.
cos θ = y as = = =v Consideremos un cilindro (o una esfera) en un fluido
ds dt dt ds ds en movimiento. Si el cilindro rota en torno a un eje
Por consiguiente la ecuación (1) puede escribirse: perpendicular a la corriente del fluido, y además hay
dp dz dv roce viscoso entre le cilindro y el fluido, entonces el
+ ρg + ρv =0 cilindro arrastrará al fluido haciendo que las
ds ds ds velocidades del fluido a ambos lados del cilindro no
⇒ dp + ρgdz + ρvdv = 0 sean iguales. En el caso mostrado en la figura
Si ρ constante, integrando obtenemos: adjunta, la velocidad es mayor arriba que abajo.
1 2
p + ρgz + ρv = constante
2
Expresión que es la ecuación de Bernoulli. La misma
que puede ser obtenida por la conservación de la
energía, siendo por supuesto, equivalente.
Como la ecuación de Bernoulli es válida para De acuerdo a la ecuación de Bernoulli, la presión en
cualquier sección, entre dos puntos cualesquiera, se el lugar 1 serán inferior que en el lado 2 ( p1 < p 2 ) .
podrá escribir:
Esta diferencia de presión genera una fuerza neta
1 2 1 sobre el cilindro hacia arriba.
p1 + ρgz1 + ρv1 = p 2 + ρgz 2 + ρv 22
2 2 Es este efecto, llamado efecto Magnus, el responsable
Adicionalmente podemos decir que cuando existen de los así llamados “efectos” que pueden observarse
pérdidas por la presencia de fuerzas viscosas, ésta en numerosos juegos de pelota.
expresión de la ecuación de Bernoulli se modificará Suponga que una bola es pateada de tal manera que
escribiéndose. va rotando a la derecha sobre un perpendicular del
1 1 eje a su dirección móvil durante su movimiento a la
p1 + ρgz1 + ρv12 = p 2 + ρgz 2 + ρv 22 + pérdidas izquierda (véase la figura). Entonces la bola
2 2
experimentaría la fuerza de Magnus. Así la bola se
mueve con una trayectoria curvada hacia la derecha
APLICACIONES:
del arquero.
Fluido en reposo
v1 = v 2 = 0 → p1 + ρgy1 = p 2 + ρgy 2
p 2 − p1 = ρg ( y1 − y 2 )
Es decir, la presión disminuye con la altura (aumenta
con la profundidad).

Fórmula de Torricelli: Permite calcular la velocidad


v 2 con que sale un líquido de un recipiente con un
agujero a una distancia h de la superficie.
p1 = p 2 = p a , y1 = 0 , y 2 = − h y v1 ≈ 0

32
Mecánica de fluidos Hugo Medina Guzmán

la velocidad del viento tiende a incrementarse con la


Una bola en un chorro de aire. altura.
Una bola ligera se puede mantenido en un chorro de
aire como se muestra en la figura. Una pelota de La ventilación en una mina.
ping-pong puede hacerse flotar sobre un chorro de
aire (algunas aspiradoras pueden soplar aire), si la
pelota comienza a dejar el chorro de aire, la presión
más alta de afuera del chorro empuja la pelota de
nuevo hacia éste como se muestra en la figura La ventilación en una mina responde a tres propósitos
siguiente. principales: para proporcionar el aire fresco para la
respiración de los mineros, diluir los gases nocivos
que puedan ser formados subterráneamente.
En un túnel horizontal simple de minería
generalmente es suficiente la ventilación natural
utilizando la diferencia en la presión de aire asociada
a la diferencia en nivel entre dos aberturas, la entrada
de la mina y la parte superior de un eje de ventilación
(efecto chimenea).

Levantar una bola con un embudo. Empuje sobre las alas de un avión.
En el espacio entre la superficie del embudo y la Una superficie aerodinámica como el ala de un avión
superficie de la bola la presión es menor que la se diseña de tal modo que perturba las líneas de
presión atmosférica, y esta diferencia de presión corriente del fluido en una región de espacio, dejando
soporta la bola contra la acción de la gravedad. la otra no perturbada.

Las, líneas de corriente encima del ala son


comprimidas y las que se encuentran debajo del ala
permanecen no perturbadas, resultando el flujo mayor
en la parte superior.
1 2 1
p1 + ρv1 = p 2 + ρv 22
Una bola ligera apoyada por un jet del aire. La 2 2
presión sobre la bola es menos que debajo de ella.
Como v1 > v 2 , resulta p 2 > p1
Efecto chimenea. Produciendo una fuerza de empuje hacia arriba.
¿Por qué sube el humo por una chimenea? En parte se En realidad, el principio de Bernoulli es sólo un
debe a que el aire caliente se eleva (es decir, debido a aspecto de la sustentación de un ala. Las alas se
la densidad). Pero el principio de Bernoulli también inclinan un poco hacia arriba, de modo aire que choca
tiene un lugar importante. Debido a que el viento contra la superficie inferior se desvíe hacia abajo; el
sopla a través de la parte superior de la chimenea, la cambio en la cantidad de movimiento de las
presión es menor ahí que dentro de la casa. Por eso el moléculas de aire que rebotan deviene en una fuerza
aire y el humo son empujados hacia arriba de la ascendente adicional sobre el ala. De igual modo la
chimenea. Incluso en una noche calmada, existe el turbulencia desempeña una función de gran
flujo de aire suficiente en el ambiente en el extremo importancia.
superior de 1 chimenea para permitir el flujo
ascendente del humo. Ejemplo 69. Velocidad de salida de un líquido
Si las tuzas, perros de la pradera, conejos y otros Velocidad de salida de un líquido a través de un
animales que viven bajo el no se asfixian, el aire debe orificio
circular en sus madrigueras. Estas siempre tienen por
lo menos dos entradas. La velocidad del flujo del aire
a través de los diferentes hoyos por lo regular será un
poco distinta. Esto conduce a una pequeña diferencia
de presión que fuerza al flujo de aire a través de la
madriguera por el principio de Bernoulli. El flujo de
aire se intensifica si un hoyo está más arriba que el
otro (lo que a menudo hacen los animales) puesto que

33
Mecánica de fluidos Hugo Medina Guzmán

Solución. Aplicando la ecuación de Bernoulli a los 2


puntos 1 y 2 tenemos ⎛A ⎞
1 − ⎜⎜ 2 ⎟⎟
1 2 1 2 A1 ⎝ A1 ⎠ h1 2
p1 + ρgy1 + ρv1 = p 2 + ρgy 2 + ρv 22 =
2 2 A2 2g
Como en 1 y 2 la presión es la presión atmosférica,
la expresión se reduce a
1 2 1 2
ρgy1 − ρgy 2 = ρv 2 − ρv1 ⇒ El frasco de Mariotte.
2 2 La velocidad de salida de un líquido por un orificio
practicado en su fondo es la misma que la que
g ( y1 − y 2 ) =
2
(
1 2
v 2 − v12 ) (1) adquiere un cuerpo que cayese libremente en el vacío
desde una altura h, siendo h la altura de la columna de
Por la ecuación de la continuidad A1v1 = A2 v 2 fluido.
A2 v = 2 gh
v1 = v2 (2)
A medida que el fluido sale por el orificio, la altura h
A1
de fluido en el depósito va disminuyendo. Si A es la
Como ( y1 − y 2 ) = h (3) sección del orificio, el gasto G = Av , o volumen de
Reemplazando (2) y (3) en (1), obtenemos: fluido que sale por el orificio en la unidad de tiempo
1 ⎡ 2 ⎛ A2 ⎞ ⎤
2 no es constante. Si queremos producir un gasto
gh = ⎢v 2 − ⎜⎜ v 2 ⎟⎟ ⎥ constante podemos emplear el denominado frasco de
2⎢ ⎝ A1 ⎠ ⎥⎦ Mariotte.

1 ⎡ ⎛ A2 ⎞ ⎤ 2
2

⇒ gh = ⎢1 − ⎜⎜ ⎟⎟ ⎥ v 2
2 ⎢ ⎝ A1 ⎠ ⎥
⎣ ⎦
Finalmente:
2 gh
v2 = 2
⎛A ⎞
1 − ⎜⎜ 2 ⎟⎟
⎝ A1 ⎠
Si A1 >> A2 :
v 2 = 2 gh
Consiste en un frasco lleno de fluido hasta una altura
Resultado que es igual al caso de un orificio lateral. h0, que está cerrado por un tapón atravesado por un
tubo cuyo extremo inferior está sumergido en el
Tiempo de vaciado. Podemos calcular el tiempo de líquido. El fluido sale del frasco por un orificio
vaciado. practicado en el fondo del recipiente. En el extremo
Para este cálculo usaremos la velocidad con que baja inferior 2 del tubo, la presión es la atmosférica ya que
el fluido, es decir v1 está entrando aire por el tubo, a medida que sale el
líquido por el orificio.
dy A2
Como v1 = = v2 La velocidad de salida del fluido no corresponderá a
dt A1 la altura h0 desde el orificio a la superficie libre de
fluido en el frasco, sino a la altura h o distancia entre
dy A 2 gy el extremo inferior 2 del tubo y el orificio.
v1 = =− 2 2
dt A1 ⎛A ⎞ Dado que h permanece constante en tanto que el nivel
1 − ⎜⎜ 2 ⎟⎟ de líquido esté por encima del extremo inferior del
⎝ A1 ⎠ tubo, la velocidad del fluido y por tanto, el gasto se
mantendrán constantes. Cuando la altura de fluido en
2
⎛A ⎞ el frasco h0 es menor que h, la velocidad de salida v
1 − ⎜⎜ 2 ⎟⎟ del fluido deja de ser constante.
dt = −
A1 ⎝ A1 ⎠ dy La velocidad de salida v puede modificarse subiendo
A2 2g o bajando el extremo inferior 2 del tubo en el frasco.
y
2 Ejemplo 70. En la pared vertical de un depósito hay
⎛A ⎞
1 − ⎜⎜ 2 ⎟⎟ dos pequeños orificios, uno está a la distancia x de la
A1 ⎝ A1 ⎠ 0 dy superficie del líquido, y el otro está a una altura z
Integrando: t = −
A2 2g ∫
h y
sobre el fondo. Los chorros de líquido que salen

34
Mecánica de fluidos Hugo Medina Guzmán

encuentran el suelo en el mismo punto, en que volumen de agua que sale por segundo por este
relación está x y z. orificio. (Se considera que el área del orificio es muy
pequeña frente al área del depósito).
Considere la presión atmosférica como 105 Pa,
g ≈ 10 m s 2

Solución.
Si v1 es la velocidad por la salida superior y t1 el
tiempo que se tarda en alcanzar el punto B:

y = v1t1 ⎪ Solución.
⎪ 1 y2 y2 a) Presión en el fondo = p atmosférica + p ejercida por la placa
v1 = 2 gx ⎬ h − x = g 2 =
2 v1 4x + p columna de fluido
1 2⎪ 1200 × 10
h − x = gt1 ⎪ p = 10 5 + + 1000 × 10 × 3,5
2 ⎭ 12
Análogamente, para el orificio inferior: = 1,36 x 105 Pa
⎫ b) Ecuación de bernoulli
y = v2 t 2 ⎪
⎪ 1 y2 y2
v1 = 2 g (h − z ) ⎬ z = g 2 =
⎪ 2 v 2 4(h − z )
1 2
z = gt 2 ⎪
2 ⎭
Eliminando y se tiene:
x(h − x ) = z (h − z )
Por lo tanto: x = z 1 2 1
p1 + ρgy1 + ρv1 = p 2 + ρgy 2 + ρv 22
2 2
Ejemplo 71. Cuando el viento sopla entre dos
1200 × 10
edificios grandes, se puede crear una caída p1 = 10 5 + = 1,01 x 105 Pa
significativa de presión. La presión del aire 12
normalmente es una atmósfera dentro del edificio, así y1 = 3m, y2 = 0, v1 ≈ 0, ρ = 1000 kg/m3
que la caída de la presión en el exterior puede hacer 1
que una placa de vidrio de la ventana estalle hacia 31000 = 1000v 22 ⇒ v2 = 7,87 m/s
fuera del edificio y estrellarse en la calle abajo.¿Qué 2
diferencia de presión resultaría de un viento de 27 Gasto =A2v2 = π (0,05)2(7,87) = 0,062 m3/s
m/s? ¿Qué fuerza sería ejercida sobre la placa de
vidrio de 2 x 3 m de una ventana? La densidad del Ejemplo 73. Un tanque cilíndrico de radio 1 m y
aire es 1,29 kg/m3 a 27° C y 1 atmósfera. altura 4 m, lleno de agua, puede desaguar sobre un
Solución. recipiente, como se muestra en la figura. El recipiente
Alejado de los edificios la presión es 1 atmósfera, y la receptor se encuentra sobre una espuma de 10 cm de
velocidad del viento es aproximadamente cero. Así espesor y módulo de Young 0,79 x 10 N/m2. El
1 2 tanque posee 2 agujeros, el primero A de área 5 cm2
p+ ρv = p a + 0 ubicado a 3H/4 de su base y el segundo agujero B de
2 3 cm2 de área a H/2 de la base del tanque.
1 1
p − p a = ρv 2 = (1,29)(27 ) = 470 Pa
2

2 2
y F = pA = (470 )(2 × 3) = 2820 N

Ejemplo 72. Un depósito de agua está cerrado por


encima con una placa deslizante de 12 m2 y 1200 kg.
a) El nivel del agua en el depósito es de 3,5 m de
altura. Calcular la presión en el fondo.
b) Si se abre un orificio circular de 5 cm de radio a
medio metro por encima del fondo, calcúlese el

35
Mecánica de fluidos Hugo Medina Guzmán

a) Calcule la velocidad de salida del agua por cada v = 2 g (h + d )


uno de los agujeros suponiendo abierto solo uno a la
vez. b) Compare la superficie con el punto B.
1
b) Si se permite desaguar al tanque durante 3 minutos pa + 0 + ρg (h + d ) = pB + ρv 2 + ρg (h + d + H )
por sólo uno de los agujeros, determine en que caso el 2
esfuerzo de compresión sobre la espuma será mayor. 1
Justifique su respuesta ⇒ pB = pa − ρv 2 − ρgH
Solución. 2
Reemplazando el valor de v, hallado en (a).
a) La velocidad de salida está dada por: v = 2 gh 1
pB = pa − ρ [2 g (h + d )] − ρgH
m 2
v A = 2 g (1) = 4,43 ,
s = p a − ρg (h + d + H )
m c) Cuando H es un máximo, la velocidad y la presión
vB = 2 g (2) = 6,26 en ese punto se aproxima a cero, así que comparando
s la superficie y el punto B obtenemos:
b) El esfuerzo de compresión depende del peso al que
esté expuesto la espuma. p a + 0 + ρg (h + d ) = 0 + 0 + ρg (h + d + H )
G A = AA v A = (5 × 10 −4 )(4,43) ⇒ p a = ρgH , de donde obtenemos:
-4 3
= 22,15 x 10 m /s pa 1,013 × 10 5 N m 2
G B = AB v B = (3 × 10 −4
)(4,43) H= =
ρg (1000 kg m 3 )(9,8 m s 2 )
= 10,3 m
= 13,29 x 10-4 m3/s
Con estos valores obtenemos para un tiempo de 3 min
Ejemplo 75. Un tanque de almacenaje abierto grande
= 180 segundos:
se llena de agua. Se hace un agujero pequeño en un
VA = 0,3987 m3 y VB = 0,2392 m3
lado del tanque a una profundidad h debajo de la
Luego S A > S B superficie del agua. ¿Con qué velocidad el agua
fluirá del agujero?
Ejemplo 74. Un sifón es un dispositivo para sacar el Solución.
líquido de un envase que sea inaccesible o que no En la superficie p = p a y v ≈ 0 . En el agujero
pueda ser inclinado fácilmente. La salida C debe
estar más baja que la entrada A, y el tubo se debe p = p a y v = v , tal que
llenar inicialmente del líquido (esto generalmente se 1
logra aspirando el tubo en el punto C). La densidad p a + 0 + ρgh = p a + ρv 2 + 0
del líquido es ρ . 2
a) ¿Con qué velocidad el líquido fluye hacia fuera en ⇒ v = ρgh
el punto C?
b) ¿Cuál es la presión en el punto B? Ejemplo 76. Los bomberos utilizan una manguera
c) ¿Cuál es la altura máxima H que el sifón puede del diámetro interior 6,0 centímetros para entregar
levantar el agua? 1000 litros de agua por minuto. Un inyector se une a
la manguera, y se quiere lanzar el agua hasta una
ventana que está 30 m sobre el inyector.
a) ¿Con qué velocidad debe el agua dejar el inyector?
b) ¿Cuál es el diámetro interior del inyector?
c) ¿Qué presión en la manguera se requiere?
Solución.
litros 10 -3 m 3
G = 1000 = 1000 = 0,017
minuto 60 s
m3/s
Solución. a) Cuando el agua deja el inyector, p = p a y
a) Compare la superficie (donde la presión es la
v = v , en el punto más alto v = 0 , tal que aplicando
presión atmosférica p a y la velocidad es
la ecuación de Bernoulli:
aproximadamente cero) con el punto C. 1 2
Aplicando la ecuación de Bernoulli pa + ρv + o = p a + 0 + ρgh
1 2 2
p+ ρv + ρgh = Constante : ⇒ v = 2 gh = 2(9,8)(30 )
2
1 = 24,2 m/s
p a + 0 + ρg (h + d ) = p a + ρv 2 + 0
2

36
Mecánica de fluidos Hugo Medina Guzmán

2
⎛D⎞
b) El caudal G = Av = π ⎜ ⎟ v
⎝2⎠
4G 4(0,017 m 3 /s )
⇒D= =
πv π (24,2 m s )
= 0,03 m
c) La velocidad en la manguera es v m , Los flujos que circulan por la rama superior e inferior
Am v m = G ⇒ vienen dados por q1 = av1 y q2 = av2,
respectivamente. También se tiene que
G 4G 4(0,017 ) Q = q1 + q2. De las relaciones anteriores se deduce
vm = = =
Am πDm π (0,06)2
2 que
= 6,02 m/s Q 2 − 2a 2 gh Q 2 + 2a 2 gh
q1 = y q2 =
1 2 1 2Q 2Q
pm + ρv m + 0 = p a + ρv 2 + 0
2 2 b) Para que circule líquido por la rama superior se
debe tener que
⇒ p m − p a = ρ (v 2 − v m2 )
1
2 Q > a 2 gh .
1
2
2
(
= (1000 ) 24,2 − 6,02
2
) Ejemplo 78. El tanque cilíndrico presurizado de 5,0
5
= 2,75 x 10 Pa = 2,71 atm m de diámetro, contiene agua la que sale por el tubo
en el punto C, con una velocidad de 20 m/s. El punto
Ejemplo 77. Un tubo horizontal por el que fluye A está a 10 m sobre el punto B y el punto C está a 3
líquido de densidad ρ0 a razón de Q m3/s, se bifurca m sobre el punto B. El área del tubo en el punto B es
en dos ramas en el plano vertical, una superior y otra 0,03 m2 y el tubo se angosta a un área de 0,02 m2 en
inferior, de secciones transversales a1 = a2 = a, el punto C. Asuma que el agua es un líquido ideal en
abiertas a la atmósfera (ver figura). Si la distancia flujo laminar. La densidad del agua es 1000 kg/m3.
entre las ramas es h, determinar:
a) Las cantidades q1 y q2 de líquido (en m3/s) que
fluyen por ambas ramas.
b) La condición que debe cumplir Q para que haya
flujo en la rama superior.

a) ¿Cuál es el gasto o flujo en el tubo ?:


b) ¿A que razón está bajando el nivel de agua del
tanque?
c) ¿Cuál es la presión en B?
Solución. d) ¿Cuál es la presión absoluta del aire encerrado en
a) La relación de Bernoulli se puede aplicar entre los
el tanque?
puntos A y B1 y también entre
Solución.
A y B2. Por transitividad, la relación de Bernoulli
también es valida entre los puntos a) El gasto o flujo en el tubo:
B1 y B2. Se tiene m3
1 2 1 G = AC vC = πR 2 vC = (0,02)(20) = 0,4
p1 + ρgh1 + ρv1 = p 2 + ρgh2 + ρv 22 s
2 2 b) La razón a la que está bajando el nivel de agua del
Pero p1 = p 2 = p a (la presión atmosférica), tanque:
h1 = h y h2 = 0, luego m3
AA vA = G = 0,4
1 2 1 s
ρv1 + ρgh = ρv 22
2 2 ⎛5⎞
2

AA = π ⎜ ⎟ = 19,625m 2
⎝2⎠
G 0,4 m
vA = = = 0,02
AA 19,625 s

37
Mecánica de fluidos Hugo Medina Guzmán

c) La presión en B: a) Sea v la velocidad con que emerge el agua del


Por Bernoulli pitón.
1 2 1
p B + ρghB + ρvB = pC + ρghC + ρvC2
2 2
pB = ?
hB = 0
0,4 m
vB = = 13,33
0,03 s
pC = pa = 1,013 × 105 Pa
hC = 3m
La velocidad hacia arriba será:
m
vC = 20 2
s vv = vsen 45º = v
2
1
pB + (1000 )(13,33) El agua alcanza a subir una altura h = 10 m, luego su
2

2 velocidad es:

= 1,013 × 10 5 + (1000 )(9,8)(3) +


1
(1000 )(20)2 vv = 2 gh ⇒
2
2
1
2
[
⇒ pB = (1000)(20) − (13,33)
2 2
] Luego: v
2
= 2 gh ⇒

+ 1,013 × 105 + (1000)(9,8)(3) v


2
= 2(9,8)(10) ⇒ v = 19,8 m/s
⇒ pB = 2,418 x 105 Pa 2
El volumen de agua que emerge del pitón por minuto:
d) ¿Cuál es la presión absoluta del aire encerrado 2 2
⎛d ⎞ ⎛ 0,015 ⎞
en el tanque ( en atmósferas)? V = vtπ ⎜ ⎟ = (19,8)(60 )π ⎜ ⎟
1 1 ⎝2⎠ ⎝ 2 ⎠
p A + ρghA + ρvA2 = p B + ρghB + ρvB = 0,212 m3 =212 litros.
2 2
b) A la salida del pitón la presión es la atmosférica
⇒ pA = pB + ρg (hB B − hA ) + 1 ρ (vB2 − vA2 )
2
⇒ pA = 2,418 × 105 + (1000)(9,8)(− 10)
+
1
2
(
(1000) 13,332 − 0,022 ) Aplicando el principio de Bernoulli:
1 1
p1 + ρgv12 = p a + ρgv 2
⇒ pA = 2,32644 Pa 2 2
Aplicando la ecuación de la continuidad:
Ejemplo 79. Un bombero lanza agua con su 2
A ⎛ 1,5 ⎞ v
manguera hacia un incendio formando un ángulo de A1v1 = Av ⇒ v1 = v = v⎜ ⎟ =
45º con la horizontal. El agua que emerge del pitón A1 ⎝ 6 ⎠ 16
penetra horizontalmente por una ventana del tercer Luego tenemos:
piso que se encuentra a una altura h = 10 m. 2
La manguera que transporta el agua desde el carro 1 ⎛ v ⎞ 1
bomba tiene un diámetro D de 6 cm y concluye en un
p1 + ρg ⎜ ⎟ = pa + ρgv 2 ⇒
2 ⎝ 16 ⎠ 2
pitón cuya abertura tiene un diámetro d de 1,5 cm.
a) ¿Cuantos litros de agua emergen del pitón por 1 ⎛ 1 ⎞
p1 − pa = ρgv 2 ⎜1 − 2 ⎟
minuto? 2 ⎝ 16 ⎠
b) ¿Cuál es la presión p que debe soportar la
1
manguera (en atmósferas)? = (1000)(9,8)(19,8) (0,996 )
2

2
= 1913312,16 Pa
Como pa = 101325 Pa

p1 = 2014637,16 Pa
Aproximadamente 2 atm.

Solución.

38
Mecánica de fluidos Hugo Medina Guzmán

Ejemplo 80. El medidor de venturi, es un


manómetro colocado en el tubo para medir la
velocidad de flujo líquido
Un líquido de densidad ρ fluye por un tubo de
sección transversal A1. En el cuello el área se reduce a
A2 y se instala el tubo manométrico como se indica en
la figura.

a) ¿Cuál es la velocidad en A1 ?
b) El agua proviene de un gran depósito abierto. ¿A
qué altura se encuentra el nivel de agua?
c) ¿Cuál es la diferencia de presión entre 1 y 2?
d) ¿Cuál es la diferencia de alturas entre las columnas
de mercurio del tubo en U?
Solución. Solución.
Aplicando la ecuación de Bernoulli entre 1 y 2 litros dm 3
a) G = 4 =4 = 4 x 10-3 m3/s
1 2 1 s s
p1 + ρgy1 + ρv1 = p 2 + ρgy 2 + ρv 22
2 2 A1 = 8 cm = 8 × 10 m 2
2 −4

Como están a la misma altura


G = Av ⇒
1 2 1
p1 + ρv1 = p 2 + ρv 22 ⇒ G 4 × 10 −3
2 2 v1 = = = 5 m/s
A1 8 × 10 − 4
1
(
p1 − p 2 = ρ v 22 − v12
2
) b) El agua proviene de un gran depósito abierto. ¿A
qué altura se encuentra el nivel de agua?
Por la ecuación de la continuidad
A1
A1v1 = A2 v 2 ⇒ v 2 = v1
A2
Luego
1 2 ⎛ A12 − A22 ⎞
p1 − p 2 = ρv1 ⎜⎜ 2
⎟⎟ (1)
2 ⎝ A2 ⎠
1 2 1
Por otra parte, la presión en el, nivel 3 por la rama 1 p1' + ρgy1' + ρv1' = p 2 ' + ρgy 2 ' + ρv 22'
es 2 2
p3 = p1 + ρgH p1' = p 2 ' = p a , y '1 = h , y 2 ' = 0 . v'1 = 0 ,
y por la rama 2 es 4 × 10 −3
p3 = p 2 + ρg (H − h ) + ρ ' gh v 2' = v2 = =5
8 × 10 − 4
Luego Reemplazando:
p1 + ρgH = p 2 + ρg (H − h ) + ρ ' gh (2) 1 1
y p1 − p 2 = gh(ρ '− ρ )
p a + ρh +ρv(0 )2 = p a + ρg (0) + ρ (5)2
2 2
igualando las expresiones (1) y (2) 1 25
1 2 ⎛ A12 − A22 ⎞ ρgh = ρ (5)2 ⇒ h = = 1,28 m.
ρv1 ⎜⎜ 2
⎟⎟ = gh(ρ '− ρ ) 2 2g
2 ⎝ A2 ⎠ c)
Finalmente 1 2 1
p1 + ρgy1 + ρv1 = p 2 + ρgy 2 + ρv 22
2 gh(ρ '− ρ ) 2 2
v1 = A2
(
ρ A12 − A22 ) y1 = y 2 = 0 ,
A1 8
Ejemplo 81. La sección transversal del tubo de la
v1 = 5 m/s , v 2 = v1 = 5 = 10 m/s
A2 4
figura tiene 8 cm2 en las partes anchas y 4 cm2 en el
1 1
estrechamiento. Cada segundo salen del tubo 4 litros
de agua a la atmósfera.
p1 + ρg (0) + ρ (5)2 = p 2 + ρg (0 ) + ρ (10)2
2 2

39
Mecánica de fluidos Hugo Medina Guzmán

1 1000 Como AD = 2AC ⇒ vC = 2vD


p1 − p 2 = ρ (100 − 25) = (75)
2 2 Por la ecuación de Bernoulli:
= 37500 Pa 1 2 1
d) ¿Cuál es la diferencia de alturas entre las columnas pC + ρvC = p atm + ρv D2
de mercurio del tubo en U?
2 2
p1 − p 2 = ρ Hg gΔy 1
(
pC = patm − ρ 4vD2 − vC2 ⇒
2
)
p1 − p 2 37500
⇒ Δy = = = 0,28 m. 3
ρ Hg g (13600)(9,8) pC = patm − ρvD2
2
Finalmente pC = patm − 3ρgh1
Ejemplo 82. Dos depósitos abiertos muy grandes A y
F, véase la figura, contienen el mismo líquido. Un c) Aplicando la ecuación de Bernoulli entre 2 y C:
tubo horizontal BCD que tiene un estrechamiento en
C, descarga agua del fondo del depósito A, y un tubo
vertical E se abre en C en el estrechamiento y se
introduce en el líquido del depósito F. Si la sección
transversal en C es la mitad que en D, y si D se
encuentra a una distancia h1 por debajo del nivel del
líquido en A.
a) ¿Cuál es la velocidad de salida del líquido?
b) ¿Cuál es la presión en el estrechamiento (C)?
c) ¿A qué altura h2 alcanzará el líquido en el tubo E? Como patm = pC + ρgh2
Expresar la respuesta en función de h1.
Comparando con pC = patm − 3ρgh1
Obtenemos h2 = 3h1

Ejemplo 83. Una regadera de jardín tipo hongo de


las características mostradas en la figura, tiene la
velocidad de salida del agua de 1m/s. El diámetro del
hongo D es de 30cm, el diámetro de la tubería
horizontal d es de 5 cm, la altura H del tramo vertical
es de 20 cm, y el espacio entre los platos del hongo e
es igual a 2 cm.
a) Encontrar el caudal de agua en la tubería
Solución.
horizontal.
a) Aplicando la ecuación de Bernoulli entre 1 y D:
b) Calcular la velocidad en el tramo horizontal
c) Calcular la presión en la parte más alta del tubo
vertical
d) Calcular la presión en cualquier punto del tramo
horizontal.

1 2 1
p1 + ρgh1 + ρv1 = p 2 + ρgh2 + ρv 22
2 2
Con p1 = p 2 = p atm , h2 = 0, ⇒
1
v1 ≈ 0 patm + ρgh1 + 0 = patm + 0 + ρvD2
2
Solución.
vD = 2 gh1 El gráfico indica los puntos de interés del problema.
b) Por la ecuación de continuidad entre las secciones
CyD

:
AC vC = AD vD
a) El caudal de agua en la tubería horizontal.

40
Mecánica de fluidos Hugo Medina Guzmán

G = A1v1 = A2 v 2 = A3 v3 está MAL. Lo que pasa es justo al revés. El caudal


que manda el corazón es constante. Este caudal no se
A1v1 = πDe = (π 0,3)(0,02 ) frena por ningún motivo.
= 0,019 m3/s Para poder pasar por la obstrucción lo que hace la
b) La velocidad en el tramo horizontal sangre es aumentar su velocidad.
Como A1v1 = A2 v 2 ⇒ (La velocidad aumenta porque el diámetro de la
arteria disminuye).
A1 0,006π Al aumentar la velocidad dentro de la arteria, la
v2 = v1 = = 9,6 m/s
A2 0,025 2 π presión adentro tiene que disminuir. Pero afuera de la
arteria la presión sigue siendo la misma. Entonces la
Siendo A2 = A3 ⇒ presión de afuera le gana a la presión de adentro y la
v3 = v 2 = 9,6 m/s arteria se comprime.
¿Y qué pasa al comprimirse la arteria?
c) La presión en la parte más alta del tubo vertical
Aplicando la ecuación de Bernoulli a los puntos 1 y La obstrucción se cierra más. Esto provoca un
2: aumento de la velocidad dentro de la obstrucción, lo
1 2 1 que a su vez obliga a la arteria a cerrarse más todavía.
p1 + ρgy1 + ρv1 = p 2 + ρgy 2 + ρv 22 De esta manera, la arteria se va cerrando más y más
2 2
hasta que sobreviene el COLAPSO.
y1 = y 2 Esto significa que la arteria tiende a cerrarse del todo
1 1 e impide el pasaje de sangre.
p a + ρv12 = p 2 + ρv 22
2 2
1
(
⇒ p 2 = p a − ρ v 22 − v12
2
)
v1 = 1 m s , v 2 = 9,6 m/s , pa = 1,013 x 105 Pa
Reemplazando valores:
1
(
p 2 = 1,013 × 10 5 − 10 3 9,6 2 − 12
2
)
= 0,5572 x 105 Pa Esto es lo que ocurre cuando una persona tiene un
d) La presión en cualquier punto del tramo horizontal. ataque cardíaco. También pasa en el cerebro y en
otros lados. Los médicos lo llaman trombosis.
p3 = p 2 + ρgH = 0,5572 x 105 – 103 (9,8)(0,20) Dependiendo del tamaño y localización del trombo
= 0,5376 x 105 Pa pueden variar algunos de los síntomas, dolor,
isquemia, frialdad, ausencia de pulso, etc.
ARTERIA O VENA CON UNA OBSTRUCCIÓN
Parece que en la medicina es bastante común que las
arterias o las venas se taponen con cosas tipo
colesterol y demás.

Concretamente la situación es esta:

VISCOSIDAD
Viscosidad de un fluido es la resistencia de un fluido
a una fuerza cortante. Propiedad que se debe
fundamentalmente al tipo de interacción entre las
moléculas del fluido.
Para poder definirla, debemos considerar el estudio
Si se le pregunta a una persona que cree que va a ocurrir de la ley de Newton de la viscosidad. Consideremos
con la arteria cuando se obstruye, la respuesta más dos placas paralelas muy grandes como se muestra en
común es esta: La sangre se va a frenar al chocar con la la figura, el espacio entre las placas esta lleno con un
obstrucción, y va a empezar a presionar hacia fuera fluido
porque quiere pasar. Por lo tanto la arteria se va a
dilatar y se va a formar como un globo.
Este razonamiento es muy lindo y muy intuitivo pero

41
Mecánica de fluidos Hugo Medina Guzmán

Helio 20 1,94 x 10-5

De la tabla se observa que la viscosidad es mucho


mayor para los líquidos que para los gases. También
se observa una fuerte dependencia de la temperatura.
Para los líquidos la viscosidad disminuye al aumentar
la temperatura, mientras que para los gases aumenta.

FLUJO VISCOSO EN UNA TUBERIA


La placa superior bajo la acción de una fuerza CIRCULAR
constan te F se mueve con una velocidad constante Para poder encontrar la expresión para la caída de
v 0 . El fluido en contacto con la placa superior se presión en una tubería circular debido a la viscosidad
consideremos un elemento de fluido que se desplaza a
adherirá y se moverá con velocidad v 0 , y el fluido en velocidad constante como se muestra en la figura,
contacto con la placa fija tendrá velocidad cero si la corno el fluido no está acelerado, las fuerzas
distancia D y la velocidad v 0 no son muy grandes, la asociadas con la presión y la viscosidad se cancelan.
variación de velocidad será lineal.
Experimentalmente se ha demostrado que la fuerza F
varía directamente con la superficie A de la placa, con
la velocidad v 0 , e inversamente con la distancia D, o
Av0
sea: F ∝
D Aplicando la segunda ley de Newton al elemento, se
Más aún, en general, depende como varía v 0 con tiene;

v dv ∑F x =0
respecto a D, esto es: 0 ⇒ p1πr 2 − p 2πr 2 − Fv = 0
D dy
dv Donde Fv e s la fuerza viscosa (Tangencial).
Luego: F ∝ A
⎛F⎞
FV = ⎜ ⎟ × (área )
dy
Aquí introducimos una constante de proporcionalidad ⎝ A⎠
η , llamada la viscosidad absoluta (dinámica). F A F dv
dv Por viscosidad η = − ⇒ = −η
F = ηA dv dr A dr
dy El signo menos indica que la velocidad disminuye
O sea la fuerza de viscosidad es proporcional al área con un incremento del radio r .
A y al gradiente (derivada) de la velocidad. Los Siendo el área = 2πrL , tenemos:
fluidos que cumplen con esta relación se llaman
dv
fluidos newtonianos. Fv = −η 2πrL
F dr
N .s Reemplazando
Como η = A , sus unidades son: dv
dv m2 p1πr 2 − p 2πr 2 + 2πηLr =0
dy dr
Sim1ificando y agrupando términos
Otra unidad usada para medir la viscosidad es el
poise (p): 1 p = 0,1 Ns/m2 ( p1 − p 2 ) = −2πηLr dv
La siguiente tabla da la viscosidad para algunas dr
sustancias:

( p − p 2 ) rdr
− dv = 1
Fluido Temp. ºC η (N.s/m2) 2ηL
Agua 0 1,79 x 10-3 Integrando de r a R,
Agua 20 1,00 x 10-3 0 ( p1 − p 2 ) R
− ∫ dv =
2ηL ∫r
Agua 100 0,28 x 10-3
rdr
v
Alcohol 20 1,2 x 10-3
Glicerina 0 12,11 ( p − p2 ) R 2 − r 2
⇒ v= 1 ( )
Glicerina 20 1,49 4ηL
Aire -31,6 1,54 x 10-5 Esta ecuación corresponde a una parábola.
Aire 20 1,83 x 10-5 La velocidad máxima en la parte central (r = 0) es:
Aire 230 2,64 x 10-5

42
Mecánica de fluidos Hugo Medina Guzmán

( p1 − p 2 )R 2 petróleo son conducidos diariamente. ¿Cuál es la


v max = presión máxima permitida por el oleoducto?
4ηL

Para determinar el gasto Q, consideremos el fluido


que pasa por un elemento diferencial de sección como
se muestra en la figura siguiente:

El volumen que atraviesa el elemento en un tiempo


dt es Solución.
dV = vdAdt , donde dA = 2πrdr 10 6 kg/dia
G=
Luego ( )
850kg/m 3 (24hr/dia )(60min/hr )(66s/min )
dV
( p − p 2 ) (R 2 − r 2 )(2πrdr )dt
= 1 = 1,36 x 10-2 m3/s
4ηL La fórmula de Poiseuille:
y el gasto en la sección diferencial es π R 4 p1 − p 2
G=
dV ( p1 − p 2 ) 2
dG = =
4ηL
(
R − r 2 (2πrdr ) ) 8 η L
dt 8GηL
Por lo tanto el gasto total, será ⇒ p1 − p 2 =
πR 4
( p1 − p 2 )
∫ (R )
− r 2 (2πrdr )
R
G = ∫ dG = 2 Entre dos estaciones de bombeo la distancia es:
4ηL 0
7,2 × 10 5
π R ( p1 − p 2 )
4
6
= 1,2 x 105 m.
=
8 ηL 1 poise = 0,1 Ns/m2
Esta expresión podemos escribirla como Luego la diferencia de presión entre las estaciones de
bombeo es:
⎡ ( p1 − p 2 )R 2 ⎤
⎢ ⎥ p1 − p 2 =
( ) (
8 1,36 × 10 −2 m 3 /s (0,1Ns/m ) 1,2 × 10 5 m )
2 ⎣
4ηL ⎦ π (0,15m ) 4
G = πR
2 = 8,2 x 105N/m2 = 8,1 atm
La expresión entre corchetes corresponde a vmáx , Como el oleoducto finalmente da el petróleo a
presión atmosférica, la presión máxima permisible es
luego 9,1 atm.
⎛v ⎞
G = πR 2 ⎜ máx ⎟ FÓRMULA DE STOKES
⎝ 2 ⎠ Una burbuja de aire el agua, partículas de polvo
vmáx cayendo en el aire, objetos que caen en fluidos todos
Como la velocidad promedio es v =
2 ellos experimentan la oposición de fuerzas viscosas.
Finalmente George Stokes encontró la relación para esta fuerza
viscosa sobre un cuerpo en un fluido
G = πR 2 v = Área de la sección x velocidad
promedio
Fv = 6πRηv , donde r es el radio, v la velocidad
de la esfera y η el coeficiente de viscosidad.
Ejemplo 84. Un oleoducto de 30 cm de diámetro y Esta expresión se denomina fórmula de Stokes.
con seis estaciones de bombeo igualmente espaciadas
en sus 7,2 x 105 m, la primera estación está al inicio Medida del coeficiente de viscosidad
del oleoducto. El petróleo a presión atmosférica pasa La fórmula de Stokes permite determinar el
en cada una de las estaciones y es lanzado a la coeficiente de viscosidad de un líquido, midiendo la
siguiente estación a la a la máxima presión permitida, velocidad terminal de esferas cayendo en el fluido.
el petróleo finalmente llega al final a la presión
atmosférica. La densidad y la viscosidad del petróleo
son 850 kg/m3 1 poise respectivamente, y 106 kg de

43
Mecánica de fluidos Hugo Medina Guzmán

La ecuación del movimiento será, por tanto:


mg − E − Fv = ma .
La velocidad límite, se alcanza cuando la aceleración
sea cero, es decir, cuando la resultante de las fuerzas
que actúan sobre la esfera es cero.
La esfera se mueve bajo la acción de las siguientes mg − E = Fv
fuerzas: el peso, el empuje (se supone que el cuerpo 4 4
está completamente sumergido en el seno de un ρ ' πR 3 g − ρ πR 3 g = 6πRηvl
fluido), y una fuerza de viscosidad. 3 3
El peso es el producto de la masa por la aceleración Despejamos la velocidad límite vl
de la gravedad g. La masa es el producto de la
densidad del material ρ ' por el volumen de la esfera 2 g (ρ '− ρ )R 2
vl =
de radio R. 9η
4
mg = ρ ' πR 3 g 2 g (ρ '− ρ )R 2
De aquí: η = , ecuación que permite
3 9vl
De acuerdo con el principio de Arquímedes, el
empuje es igual al producto de la densidad del fluido determinar el coeficiente de viscosidad de un líquido
ρ , por el volumen del cuerpo sumergido, y por la de densidad ρ , midiendo la velocidad límite de una
aceleración de la gravedad. esfera de radio R y densidad ρ'
4
E = ρ πR 3 g
3

PREGUNTAS Y PROBLEMAS

1. Un tubo en U que está abierto en ambos extremos


se llena parcialmente con agua. Después se vierte
kerosén de densidad 0,82 g/cm3 en uno de los lados
que forma una columna de 6 cm de altura. Determine
la diferencia de altura h entre las superficies de los dos
líquidos.

3. Considere el sistema de la figura donde el tubo está


lleno de aceite de densidad ρ = 0,85gcm3. Uno de los
recipientes está abierto a la atmósfera y el otro está
cerrado y contiene aire. Determine la presión en los
2. Un tubo en U que está abierto en ambos extremos se puntos A y B.
llena parcialmente con mercurio. Después se vierte
agua en ambos lados obteniéndose una situación de
equilibrio ilustrada en la figura, donde h2 = 1 cm.
Determine la diferencia de altura h1 entre las
superficies de los dos niveles de agua.

Respuesta.
PA = 82475 Pa

44
Mecánica de fluidos Hugo Medina Guzmán

PB = 99135 Pa

4. Considere un vaso comunicante de 2 cm2 de sección


transversal que contiene mercurio ρHg = 13,6 g/cm3). A
un lado se echan 360 gramos de glicerina ρgl = 1,2
g/cm3) y en el otro 1/4 de litro de alcohol ρal = 0,8
g/cm3). Encuentre el desnivel d que existe entre los
niveles superiores de la glicerina y el alcohol. Haga un
grafico cualitativo de la presión “hidrostática” en
función de la profundidad para cada uno de los dos
“brazos” del vaso comunicante (grafique las dos 7. Un cuerpo de material desconocido pesa 4N en el
curvas en el mismo grafico). aire y 2,52 N sumergido en agua. Encuentre la
densidad del material.

8. Una balsa de área A, espesor h y masa 400kg flota


en aguas tranquilas con una inmersión de 5cm. Cuando
se le coloca una carga sobre ella, la inmersión es de 7,2
cm. Encuentre la masa de la carga.

9. Un cuerpo homogéneo prismático de 20cm de


espesor, 20 cm de ancho y 40 cm de longitud se
mantiene en reposo sumergido en agua a 50cm de
profundidad a aplicar sobre él una tensión de 50 N.
5. Considere un sistema de vasos comunicantes ¿Cuánto pesa en aire y cuál es su densidad relativa?
formado por dos tubos de sección transversal de 50
cm2 que están unidos por un tubito corto de sección 10. ¿Qué fracción del volumen de una pieza sólida de
transversal muy pequeña (o sea, para efectos de este metal de densidad relativa al agua 7,25 flotará sobre un
problema podemos despreciar la cantidad de fluido que mercurio de densidad relativa 13,57?
se encontrará en el tubito). Inicialmente en este sistema
de vasos comunicantes se encuentran dos litros de 11. Un tarro cilíndrico de 20cm de diámetro flota en
agua. agua con 10cm de su altura por encima del nivel del
agua cuando se suspende un bloque de hierro de 100 N
de peso de su fondo. Si el bloque se coloca ahora
dentro del cilindro ¿qué parte de la altura del cilindro
se encontrará por encima de la superficie del agua?
Considere la densidad del hierro 7,8gcm3.

12. Un bloque con una sección transversal de área A,


altura H y densidad ρ , está en equilibrio entre dos
a) Encuentre la altura en que se encontrarán las
interfases entre los líquidos y el aire en cada uno de los fluidos de densidades ρ1 y ρ 2 con ρ1 < ρ < ρ 2 .
tubos si en uno de los tubos se le agregan 2 litros de un Suponga que los fluidos no se mezclan. Determine la
líquido cuya densidad es 0,8 g/cm3. fuerza de empuje sobre el bloque y encuentre la
b) Para la situación descrita en la parte a), encuentre la densidad del bloque en función de ρ1 , ρ 2 , H y h.
presión en el fondo de los vasos comunicantes.
c) Encuentre la altura en que se encontrarán las
interfases entre los líquidos y el aire en cada uno de los
tubos si en uno de los tubos, en lugar de 2, se le
agregan 3 litros de un líquido cuya densidad es 0,8
g/cm3.

6. Considere una prensa hidráulica (ver figura adjunta).


Sean R1 = 25 cm y R2 = 150 cm los radios de los
émbolos de bombeo y de presión, respectivamente. 13, En una piscina se encuentra flotando una balsa que
Si de la palanca que actúa sobre el embolo de bombeo tiene forma de un paralelepípedo de densidad relativa
se tira con una fuerza F1 = 100 N, ¿qué fuerza ejercerá (al agua) de 0,3 y cuyas dimensiones son 120 cm de
el émbolo de presión sobre el objeto S? largo, 100 cm de ancho y 25 cm de alto. Determine
a) La fuerza de empuje.
b) La altura medida desde el fondo de la balsa a la que
se encuentra la línea de flotación.
c) El peso que debería colocarse sobre la balsa para
que esta se hundiera 6cm más.

45
Mecánica de fluidos Hugo Medina Guzmán

ρHe = 0,18 kg/m3, respectivamente.


14. El rey Hierón de Siracusa pidió a Arquímedes que
examinara una corona maciza que había ordenado 21. Se quiere confeccionar aluminio poroso (algo así
hacer de oro puro. La corona pesaba 10 kg en el aire y como queso suizo) que se mantenga en suspensión en
9,375 kg sumergida en agua. Arquímedes concluyó agua. Determine la razón entre el volumen de los poros
que la corona no era de puro oro. Asumiendo que en su y el volumen del aluminio poroso. (La densidad del
interior contenía plata, ¿cuánto oro tenia la corona de aluminio 2700 kg/m3).
Hierón? La densidad del oro es 19,3 g/cm3; la de la
plata, 10,5 g/cm3. 22. Dos globos esféricos inflados con aire, ambos de
radio R, se unen mediante una cuerda de longitud L.
15. Considere un vaso de agua lleno hasta el borde, Los dos globos se mantienen bajo el agua con el punto
con un trozo de hielo flotando en el. Por supuesto que medio de la cuerda fijo al fondo. Calcular la fuerza de
el hielo, al flotar, sobrepasará por encima del borde del contacto entre los globos.
vaso. A medida que el hielo se derrite. ¿Se derramará
el vaso?
Suponga ahora que en el mismo vaso flota un pequeño
barco de juguete hecho de latón. Suponga además que
el barquito tiene un pequeño orificio por el cual
penetra agua, haciendo que el barquito lentamente se
llene de agua. Durante este proceso, o sea mientras el
barco se llena de agua pero aún no se hunde, el nivel
del agua del vaso ¿baja, queda a igual altura o sube?
Cuando finalmente el barquito se hunde, que pasa con
el nivel del agua?
23. Una varilla yace en el fondo de un recipiente con
agua formando un ángulo de 60º con la vertical. La
16. Considere un cilindro de masa M, área A y altura h,
varilla es de sección uniforme y está formada por dos
que flota “parado” en un liquido de densidad ρ0.
pedazos iguales en longitud pero de distinta densidad.
a) ¿Hasta qué alguna estará sumergido el cilindro en el
La densidad de una de las porciones de la varilla es la
líquido?
mitad de la del agua. Determine la densidad de la otra
b) Si el recipiente que contiene le líquido es muy
porción.
grande (por ejemplo, un lago), ¿qué trabajo debe
realizarse para sacar el cilindro del líquido?
c) ¿Varía la respuesta si el recipiente que contiene el
líquido es un tambor cilíndrico de área A0?

24. Considere un bloque de hielo (densidad = 920


kg/m3) en forma de “L”, formado de tres cubos de 25
cm por lado. Mediante un peso se desea sumergir el
hielo en agua como se indica en la figura.
Determine la masa del peso y la ubicación en el hielo
17. Considere una varilla de donde debería adherirse de modo que el hielo se
madera muy liviana, de largo L, mantenga justo sumergido lo más estable posible.
sección transversal A y densidad ρ,
que se hace flotar en el agua
(designe la densidad del agua por
ρ0).
a) Convénzase de que no es posible
que la varilla flote “parada”.
b) Para lograr que la varilla flote
parada, agreguémosle una masa
puntual m en el extremo inferior.
¿Cual es la mínima masa m que debe agregarse para
lograr el objetivo?

18. ¿Qué volumen de helio se requiere si debe elevarse


un globo con una carga de 800 kg (incluido el peso del 25. Repita el problema anterior si la línea OAB forma
globo vacío)? Las densidades del aire y del helio, a la un ángulo de 30° respecto a la vertical.
presión de una atmósfera, son ρaire = 1,29 kg/m3 y

46
Mecánica de fluidos Hugo Medina Guzmán

30. Un tubo de largo L, lleno de agua, gira en el plano


horizontal en torno a un eje vertical que pasa por uno
de sus extremos. En el extremo junto al eje, el tubo
está abierto, coincidiendo por lo tanto la presión del
fluido con la presión atmosférica. El tubo gira con
velocidad angular constante ω. Si en el otro extremo,
en cierto instante, se abre un pequeño orificio, ¿con
qué velocidad emergerá el agua del tubo? (Especifique
la rapidez y dirección de la velocidad.)

26. Determine la ubicación “y “del pivote fijo A de


manera que justo se abra cuando el agua está como se
indica en la figura.

31. Para abastecer de agua a una casa de dos pisos se


recurre a un “hidropack”.
Este sistema consiste en un depósito subterráneo, una
bomba y un cilindro con agua y aire. La bomba inyecta
agua a presión al cilindro, que en su parte superior
queda con aire comprimido. Un medidor de presión
detiene la bomba cuando la presión del cilindro
alcanza el valor deseado (el mismo medidor vuelve a
encender la bomba cuando la presión baja de cierto
27. Una gotita de agua de 1 mm de radio se pulveriza nivel). Si el nivel del agua en el cilindro se sitúa 1
en gotitas de 10−4 mm de radio. ¿En qué factor metro por debajo del suelo, calcule la presión necesaria
aumenta la energía superficial (debido a la tensión en el aire comprimido para que una llave de 1 cm2 de
superficial)? sección, a una altura de 5 metros sobre el suelo,
entregue un caudal de 12 litros por minuto. (La sección
28. Considere dos placas planas de vidrio, separadas transversal del cilindro es grande respecto a la de la
por una distancia de 0,1 mm, con un extremo llave.)
sumergidas en agua en forma vertical. ¿Qué distancia También encuentre la presión del aire al interior del
se elevará el agua entre las placas debido a la cilindro.
capilaridad?

29. Un jardín es regado con un regador casero que


consiste en una botella plástica, con numerosos
agujeros de 1 mm de diámetro, acostada sobre el jardín
y conectada aun a manguera. Asuma que una bomba
de agua se encarga de generar un flujo de agua
constante de 0,2 litros por segundo. ¿Cuántos agujeros
debe tener la botella para que el agua llegue a mojar el
prado a 8 metros de distancia de la botella? ¿Cuál es la
presión al interior de la manguera si ésta tiene una
sección transversal de 4 cm2?

32. La fuerza de sustentación de un avión moderno es


del orden de 1000 N por metro cuadrado de ala.
Suponiendo que el aire es un fluido ideal y que la
velocidad del aire por debajo del ala es de 100 m/s,
¿cuál debe ser la velocidad requerida por sobre el ala
para tener la sustentación deseada? (La densidad del
aire es 1, kg/m3.)

47
Mecánica de fluidos Hugo Medina Guzmán

33. Considere la tubería que lleva el agua de una La parte superior del tubo se encuentra a h0 = 12 cm
represa hacia una turbina. Suponga que la bocatoma se sobre el nivel del agua del torrente y tiene un pequeño
encuentra a 10 metros bajo el nivel de las aguas y que agujero.
la turbina se encuentra 80 metros por debajo de ese ¿A qué altura h subirá el chorro de agua que sale por el
nivel. Al inicio, es decir a la salida de la represa, la agujero?
tubería tiene un diámetro de 40 cm. Suponga que el
fluido se comporta como un fluido ideal.
a) ¿Cuál es el diámetro máximo que puede tener la
tubería en su extremo inferior para que no se
produzcan cortes de la columna de agua al interior de
la tubería?
b) ¿Cual sería la cantidad de agua que pasaría en ese
caso por la tubería y cuál la velocidad del agua
emergente? Respuesta. Llegará a 20 cm.
c) Si el proceso de generación de energía eléctrica
usando la presente turbina fuese 100% eficiente, ¿cuál
sería la potencia de esta central? ¿Esto corresponde al 37. La figura muestra un tubo de Pitot, instrumento
consumo promedio de cuántas casas? que se usa para medir la velocidad del aire. Si el
d) Haga un grafico cualitativo de la presión al interior líquido que indica el nivel es agua y Δh = 12 cm,
de la tubería en función de la altura. ¿Cómo cambia encuentre la velocidad del aire. La densidad del aire es
esta presión si la sección de la tubería, en el punto 1,25 kg/m3.
emergente, se disminuye a la mitad? ¿A la centésima
parte?

Respuesta. 43, m/s = 156 km/h.

38. Considere un oleoducto de 5 km y 50 cm de


34. Considere una tubería de una calefacción. En el diámetro por el cual se desea bombear 1 m3 por
sótano su diámetro es de 4,0 cm y en el segundo piso, segundo. Si uno de los extremos está abierto a la
5 metros más arriba, la tubería tiene un diámetro de presión atmosférica, ¿qué presión p1 debe existir en el
sólo 2,6 cm. Si en el sótano una bomba se encarga de otro extremo? Suponga que la densidad del petróleo es
bombear el agua con una velocidad de 0,5 m/s bajo 950 kg/m3 y el coeficiente de viscosidad es 0,2 Pa s
una presión de 3,0 atmósferas, ¿cuál será la rapidez de aproximadamente. ¿Cual es la potencia dW/dt (energía
flujo y la presión en el segundo piso? por unidad de tiempo) disipada por la fricción interna
originada por la viscosidad?
35. Suponga que el nivel de un líquido (agua) en un Respuesta. p1 7,5 atm; potencia 650 kW.
tambor tiene una altura h. A una altura b se hace una
pequeña perforación lateral que permite que el agua
emerja horizontalmente. ¿A qué altura debe hacerse la 39. Un líquido viscoso, teniendo una viscosidad del
perforación para que el alcance d del agua se máximo? equilibrio 80 poises, está entre dos placas separadas
4,0 centímetros. Ambas placas están en el
movimiento, en direcciones opuestas, con velocidades
de 3,0 centímetros/s, y el líquido entre ellas está en
flujo laminar. El esfuerzo cortante aplicado al líquido,
en unidades SI, es:
Respuesta. 12

40. Encuentre la velocidad terminal que adquiere una


Respuesta. b = h/2. esfera de cobre de 0,5 cm de diámetro, cuando cae en
agua ( ρ Cu = 8,92 g/cm3). ¿En qué factor disminuye la
36. En un torrente de agua se sumerge un tubo
doblado, tal como se muestra en la figura adjunta. La velocidad terminal si el diámetro se achica en un factor
velocidad de la corriente con respecto al tubo es v = 10?
2,5 m/s.

48
Calor y Termodinámica Hugo Medina Guzmán

CAPÍTULO 5. Termodinámica

INTRODUCCION. consistentes con las leyes de Newton del


Sistemas Termodinámicos movimiento.
Variables termodinámicas macroscópicas. En la termodinámica la atención se dirige al exterior
Consideremos un gas encerrado en un tubo del sistema. Se determinan experimentalmente: las
cilíndrico cerrado a uno de sus extremos y provisto cantidades macroscópicas que son necesarias y
de una tapa deslizante (pistón) en el otro. Como se suficientes para describir el estado interno del
muestra en la figura. sistema, estas son llamadas coordenadas
termodinámicas.
El propósito de la termodinámica es encontrar las
relaciones entre las coordenadas termodinámicas
consistentes con las leyes fundamentales de la
termodinámica.
Finalmente, puntualizaremos que dentro de la física,
las leyes que relacionan las cantidades
macroscópicas, se denomina termodinámica clásica
El sistema descrito ocupa determinado volumen el o simplemente termodinámica y, las fórmulas
cuál puede conocerse en determinado momento por matemáticas que relacionan las cantidades
la posición del pistón, otra cantidad indispensable microscópicas, constituyen la Mecánica Estadística,
para la descripción del sistema es la presión del gas o Teoría atómica del calor, o bien, cuando se usan
en el cilindro, que también se puede conocer, técnicas simples estadístico-matemáticas se le llama
mediante un manómetro. Finalmente, para tener una teoría cinética.
idea completa de lo que sucede en el cilindro hay
que conocer la temperatura, la cual puede medirse LEY CERO DE LA TERMODINÁMICA Y
en forma simple al igual que las otras dos EQUILIBRIO TÉRMICO.
cantidades. Estas cantidades obtenidas por medición Supongamos que tenemos dos sistemas A y B,
directa, que describen al sistema, nos proporcionarán separados cada uno y definidos por las coordenadas
lo que se conoce como la Descripción microscópica (presión y temperatura) p, T y p’, T’
del sistema. respectivamente.
Otro punto de vista de describir el sistema es El estado de un sistema en el cual las velocidades
asumiendo que el gas esta formado por un gran macroscópicas tienen valores que permanecen
número de partículas, moléculas o átomos, todos de constantes mientras que las condiciones externas no
igual masa y cada uno moviéndose con una se cambien, se conoce como estado de equilibrio
velocidad independiente de las otras es imposible térmico.
aplicar las leyes de Newton del movimiento a cada
molécula por separado e incluso tabular las Equilibrio térmico. Los experimentos demuestran
coordenadas de cada molécula, en este caso es que la existencia de un estado de equilibrio depende
necesario usar métodos estadísticos las cantidades de la proximidad de otros sistemas y de la naturaleza
que lo especifican no están directamente asociadas, de la pared que los separa. Si cuando un sistema
con nuestro sentido de percepción, esta descripción está en un estado de equilibrio y este no cambia con
es conocida como Descripción microscópica del cualquier cambio en el ambiente, el sistema se dice
Sistema. que está “Aislado” o rodeado por una pared “Pared
La descripción macroscópica o sea las propiedades Adiabática”. Cuando las variables macroscópicas de
apreciadas por nuestros sentidos son el punto de dos sistemas que se encuentran conectadas por una
partida para todas las investigaciones y aplicaciones pared diatérmica no varían, se dice que se
prácticas. Por ejemplo, en la mecánica do un cuerpo encuentran equilibrios térmicos entre ellas.
rígido, considerando los aspectos, externos, Imaginemos a los sistemas A y B separados en
especificamos su centro de masa con referencia a un contacto, o separados por una pared diatérmica, con
eje de coordenadas en un tiempo particular. un sistema C.
La posición y e1 tiempo y la combinación de ambos,
tal como la. Velocidad, constituyen algunas de las
cantidades macroscópicas usadas en mecánica y son
llamadas coordenadas mecánicas y estas sirven para
determinar la energía potencial y cinética del cuerpo
rígido. Estos dos tipos de energía, constituyen la
energía mecánica o externa del cuerpo rígido. El
propósito de la mecánica es encontrar relaciones
entre las coordenadas de posición y el tiempo

1
Calor y Termodinámica Hugo Medina Guzmán

El sistema A estará en equilibrio con el sistema C y Donde las constantes a y b se evalúan de acuerdo a
el sistema B también estará en equilibrio con el un conjunto definido de reglas. Asignemos números
sistema C, luego los sistemas A y B estarán en arbitrarios a dos puntos fijos.
equilibrio térmico uno con el otro.
Esto se conoce como la Ley cero de la Escala Celsius o centígrada.
termodinámica, En la escala Celsius o centígrada uno de ellos el
"Si dos sistemas se encuentran en equilibrio térmico punto de congelación del agua, es decir el punto en
con un tercer sistema, los dos sistemas se encuentran que el agua y el hielo están en equilibrio a la presión
en equilibrio entre sí". atmosférica, a esta temperatura le damos el valor
Esta ley está de acuerdo a nuestra experiencia diaria cero grados Celsius o grados centígrados (0°C).
de nuestros sentidos, es sencilla pero no obvia, es un t = ayc + b = 0o C
hecho que sucede pero podría no haber sido así. Nos
expresa la idea fundamental de temperatura. Cuando El otro punto, el de ebullición del agua a presión
decimos que las variables macrosc6picas no varían, atmosférica, a este le llamamos Cien grados
nos hace falta definir una propiedad que asegure (100°C).
esto. t = aye + b = 100o C
Esta propiedad la llamaremos Temperatura. Al resolver las dos ecuaciones simultáneamente
Nosotros queremos asignar un número de cada encontramos los valores de a y b.
estado de equilibrio de un sistema que tenga la
propiedad que dos sistemas con el mismo número 100o C 100o C
a= y b=− yc
estén en equilibrio térmico entre ellos. ye − yc ye − yc
"La temperatura de un sistema es una propiedad que
Sustituyendo la expresión original
determina si un sistema está en equilibrio o no con
otros sistemas". t = 100o C
( y − yc )
( ye − yc )
TEMPERATURA Y ESCALAS Para un termómetro a gas a Volumen Constante la
La temperatura se determina por la medición de expresión sería
alguna cantidad mecánica, eléctrica u óptica cuyo
valor se correlaciona con la temperatura. t = 100o C
( p − pc )
Generalmente la temperatura de una sustancia, sino ( pe − pc )
en el termómetro el cual, se pone en contacto íntimo y para un termómetro a gas a presión constante la
con la instancia y adquiere la misma temperatura. expresión sería
Se llama TERMOMETRO, a un aparato que permite
medir la temperatura por medio de su propiedad t = 100o C
(V − Vc )
termométrica o variable macroscópica que es (Ve − Vc )
sensible al estado térmico de la sustancia. Los El termómetro a gas a volumen constante consiste en
principales termómetros y sus propiedades
un balón B 1 lleno de gas (hidrógeno por ejemplo)
termométricas se muestran en la tabla.
ligado a un tubo en forma de U lleno de mercurio, el
TERMOMETRO PROPIEDAD volumen de gas en el balón se mantiene constante
TERMOMETRICA subiendo o bajando B 3 hasta que el mercurio en B 2
Gas a volumen constante Presión se encuentra en la marca cero.
Gas a presión constante Volumen
Resistencia eléctrica Resistencia eléctrica
Termocupla Fuerza electromotriz
Columna líquida en un tubo Longitud
capilar

Construyamos una escala de temperatura, para esto


tomemos como termómetro una columna líquida de
mercurio en un tubo capilar de vidrio, observamos
que la columna de mercurio aumentará cuando
aumenta la temperatura, como la compresibilidad del
La presión p que equilibra la presión del gas es
mercurio es tan pequeña podemos considerar como
si fuera a presión constante. La relación más simple p = 76 cm + h
entre temperatura y longitud de la columna que La experiencia muestra que la dependencia de la
podemos elegir, es una relación lineal de y. presión con relación a la temperatura es lineal con
t ( y ) = ay + b esto se obtiene la escala de un termómetro
colocando el balón en un baño de hielo en fusión,
marcando pc y después repitiendo la operación con
vapor de agua, marcando pe.

2
Calor y Termodinámica Hugo Medina Guzmán

La distancia entre esos dos puntos se toma, por


convención igual a 100°.
Medidas usando el gas hidrógeno como sustancia
termométrica muestra que
pe
= 1,366
pc
o sea que la relación con la temperatura, sería:
⎛ p ⎞ Solución.
⎜⎜ − 1⎟⎟ Considerando el comportamiento del termómetro
p ⎠ = 100 C ⎛⎜ p − 1⎞⎟
o
t = 100 o C ⎝ c
con la linealidad mostrada en la figura.
⎛ pe ⎞ (1,366 − 1) ⎜⎝ p c ⎟

Para la presión del gas es 227 mm de Hg
⎜⎜ − 1⎟⎟ corresponde una temperatura 100 + 273,5 =373,15 K
⎝ pc ⎠ Para la presión 162 mm de Hg corresponde
373,15
⎛ p ⎞ x= 162 = 266,30 K o -6,85°C
t = 273,15⎜⎜ − 1⎟⎟o C 227
⎝ pc ⎠
En esta expresión se ve que cuando la temperatura es Ejemplo 2. En un lugar en que la presión
-273.15 la presión es Cero. Como no es posible para atmosférica es 760 mm de mercurio introducimos un
la presión tomar valores menores que cero, a este termómetro centígrado en hielo fundente y luego en
valor de la temperatura se le torna como origen de vapor de agua hirviendo. El termómetro, mal
una nueva escala de temperatura, escala graduado, marca 2° para el primero y 102,5° para el
ABSOLUTA de Temperaturas en grados KELVIN. segundo
T (K ) = t (o C) + 273,15o C a) ¿Qué fórmula de reducción deberemos emplear
para calcular la temperatura real en todos los
En realidad para calibrar el termómetro, no se toma
casos? Si el termómetro marca 50°,
como referencia el punto de fusión del hielo, sino
b) ¿cuál es la verdadera temperatura?
que se especifica corno "punto fijo patrón” al
c) ¿A qué temperatura sería correcta la lectura del
llamado "Punto triple de agua", único punto en el
termómetro?
que coexisten en equilibrio hielo, líquido y vapor de
Solución.
agua, dándose solamente a la presión de 4,58 mm
a) El cero de un termómetro correcto corresponde al
Hg.
2 del mal graduado, y el 100 corresponde 102,5°.
Obteniéndose:
El intervalo fundamental está, por tanto, dividido
t = 0,01 °C
en: 102,5 - 2 = 100,5
T = 273,16 K
Llamando A a la temperatura marcada por el
p incorrecto y C a la del centígrado perfecto, la
T = 273,16 K
pc fórmula será:
El termómetro de gas a volumen constante se toma C A−2
=
como standard porque es el que experimentalmente 100 100,5
mas nos conviene, pues es el que nos da las
variaciones más pequeñas y también porque cuando
C 50 − 2
el termómetro contiene gas a baja presión, la b) = ⇒
diferencia de lectura en temperatura usando 100 100,5
diferentes gases es reducida.
48 × 100
C= = 47,76o C
Ejemplo 1. Cuando el bulbo de un termómetro de 100,5
gas a volumen constante se coloca en un recipiente c) Si la indicación fuese correcta, se verificaría:
con agua a 100 oC, la presión del gas es 227 mm de C C−2
Hg. Cuando el bulbo se mueve a una mezcla de hielo = ⇒ 100,5C = 100C − 200
- sal la presión del gas cae a 162 mm de Hg. 100 100,5
Asumiendo el comportamiento ideal, como en la − 200
figura, ¿cuál es la temperatura Celsius de la mezcla ⇒C= = −400o C
de hielo – sal? 0,5
Lo cual es imposible, puesto que el cero absoluto
es - 273,16 °C, menor temperatura a la que puede
aproximar un sistema.

Ejemplo 3. Un termómetro centígrado mal graduado


marca 8° en el punto de fusión del hielo y 99° en el
de ebullición del agua, en un lugar en que la presión

3
Calor y Termodinámica Hugo Medina Guzmán

atmosférica es 760 mm. Resolver para este 5


termómetro las preguntas del problema anterior. a) Como TC = (TF − 32) y
Solución. 9
1) El intervalo fundamental será: 99 - 8 = 91 TC = TK − 273,15 , igualando ambas expresiones,
Luego la fórmula de reducción es: encontramos para la temperatura Fahrenheit:
C A−8 9
= TF = ⋅ (TK − 255,37 ) = 10340,33º F .
100 91 5
C 50 − 8 4200 5
2) = ⇒C= 46,15 o C b) TC = (TF − 32 ) = 37°C
100 91 91 9
C C −8 5
3) = ⇒ 91C − 800 = 100C c) TC = (TF − 32 ) = 73,89º C.
100 91 9
800 9
⇒C= = 88,9o C d) TF = TC + 32 = −297,4º C .
9 5
Otras escalas de temperatura. DILATACION TERMICA.
Así como la escala Celsius (Centígrado) y su Efectos frecuentes en los materiales al presentarse
correspondiente en la escala absoluta Kelvin, existen cambios de temperatura, son variaciones en sus
otras escalas en el sistema inglés. dimensiones y cambios de estado. En primer lugar
consideraremos aquí, las variaciones de dimensiones
que ocurren sin cambios de estado.
Cuando la temperatura de un cuerpo aumenta, este
por lo general se dilata. Una excepción es el agua
que se contrae entre 0ºC y 4°C, este comportamiento
es crítico en la manera como los lagos y los océanos
polares se congelan de la superficie hacia abajo, en
lugar de hacerlo del fondo hacia la superficie, ya que
el agua mas fría que 4°C se eleva en lugar de
hundirse y el agua a 0°C está en la superficie en
lugar de estar en el fondo. (La densidad del agua a
La escala FAHRENHEIT, al cero de la escala 4°C es máxima, = 1 g/cm3).
Celsius corresponde a 32° F y los 100°C
corresponden a 9 divisiones de °F, la relación de Expansión lineal.
equilibrio es: El cambio de una dimensión lineal de un sólido tal
9 como el largo, el ancho, alto o una distancia entre
t (°F) = t (°C) + 32°F dos marcas se conoce como la expansión lineal.
5
y
5
t (°C ) = t (°F) − 32°F
9 Experimentalmente se encuentra, para un amplio
La escala absoluta correspondiente a la escala rango de temperaturas, que el cambio de longitudes
Fahrenheit es la escala RANKINE.
Δl , es proporcional al cambio de temperatura Δt y
( )
T (R ) = t o F + 459,67(R ) a la longitud l, de tal manera que podemos escribir:
9
T (R ) = T (K ) Δl = α lΔt , donde α es el coeficiente de
5 expansión lineal. Este coeficiente tiene diferentes
valores para los diferentes materiales y tiene por
Ejemplo 4. a) La temperatura de la superficie del unidad l/grado.
Sol es de unos 600 ºC. Exprésese esa temperatura en O bien,
la escala Fahrenheit. Δl
b) Exprese la temperatura normal del cuerpo = α Δt
humano 98,6 ºF, en la escala Celsius. l
c) exprese la temperatura de pasteurización, 165 ºF, Para encontrar la longitud final después de un
en la escala Celsius. dl
cambio de temperatura Δt , escribimos = α dt ,
d) Exprese el punto normal de ebullición del l
Oxígeno –183 ºC, en la escala Fahrenheit.
e integramos considerando la longitud l para t = t1,
Solución.
y l' para t = t2, siendo t 2 − t1 = Δt

4
Calor y Termodinámica Hugo Medina Guzmán

dl
l' t2 Expansión de volumen.
∫ = α ∫ dt ⇒ ln l l = α t t2 ⇒
l' t
l l t1 1
Usando el mismo argumento se demuestra que el
cambio de volumen de un sólido de volumen V, al
l' l' elevarse la temperatura Δt es
ln = α (t 2 − t1 ) ⇒ ln = αΔt
l l ΔV = 3αVΔt = β VΔt
l' Donde β = 3α es el coeficiente de expansión de
= e αΔt ⇒ l' = le αΔt
l volumen.
αΔt
Desarrollando e en series de Taylor
Coeficiente de dilatación lineal de algunos de los
⎡ x x x2 x3 ⎤ materiales más usuales.
⎢e = 1 + + + + ......... − ∞ < x < ∞ ⎥
⎣ 1! 2! 3! ⎦
Sólidos α (° C-1)
Obtenemos:
Concreto 0,7 – 1,2 x 10-5
⎡ αΔt (αΔt )2 (αΔt )3 ⎤ Plata 2,0 x 10-5
l' = le αΔt = l ⎢1 + + + + ....⎥
1! 2! 3! Oro 1,5 x 10-5
⎣ ⎦
Invar 0,04 x 10-5
Como a es una cantidad muy pequeña podemos no
Plomo 3,0 x 10-5
considerar los términos con α2, α3, …..
Zinc 2,6 x 10-5
y finalmente
Hielo 5,1 x 10-5
l ´ = l ( 1 + αΔt) = l + Δl Aluminio 2,4 x 10-5
Latón 1,8 x 10-5
Expansión de superficie. Cobre 1,7 x 10-5
Consideremos ahora el área al elevar la temperatura Vidrio 0,4 – 0,9 x 10-5
Δt , para esto tomamos una superficie como se Hierro 1,2 x 10-5
muestra en la figura, antes de la expansión su área es Cuarzo 0,04 x 10-5
A = ab. Acero 1,2 x 10-5

Líquidos β (° C-1)
Glicerina 5,1 x 10-5
Alcohol etílico 7,5 x 10-5
Mercurio 1,8 x 10-5
Bisulfuro de 11,5 x 10-5
carbono
Agua (20 ° C ) 2,0 x 10-5

a se expande en Δa = α 1 aΔt
Ejemplo 5. En el comparador de la figura se mide la
b se expande en Δb = α 2 bΔt dilatación de una barra de hierro, de 1 m de longitud
Luego a ' = a + Δa = a(1 + α 1 Δt ) y a 0 °C, obteniéndose para los 50 °C una dilatación
de 0,06 cm.
b' = b + Δb = b(1 + α 2 Δt )
A' = a ' b' = a (1 + α 1 Δt )b(1 + α 2 Δt )
[
A' = a ' b' = ab 1 + (α 1 + α 2 )Δt + α 1α 2 Δt 2 ]
En esta expresión el último término se puede
despreciar ya que α 1 y α 2 son valores muy
pequeños, y A = ab tenemos
A' = A[1 + (α 1 + α 2 )Δt ]
En el caso de ser un cuerpo isotrópico, los
coeficientes de expansión lineal α 1 y α 2 son
iguales a α , luego Calcular:
a) El coeficiente de dilatación lineal del hierro.
A' = A(1 + 2αΔt ) b). Si tiene una sección de 10 cm2 a 0°C, ¿cuáles son
Como A' = A + ΔA , tenemos: su sección y su volumen a 100 °C?
ΔA = 2αAΔt = γAΔt Solución.
Donde γ = 2α es el coeficiente de expansión de
L′ − L0 0,060
a) α= =
área. L0 × ΔT 100 × 50
−6
= 12 × 10 °C −1
5
Calor y Termodinámica Hugo Medina Guzmán

b) γ = 2α = 24 × 10 °C
−6 −1
= − 83,2 C
o

Como T ′′ = T + ΔT ′ = 20 − 83,2 = −63,2°C


A′ = A0 (1 + γ ΔT ) = 10(1 + 24 ×10 × 100)
−6

Ejemplo 8. La varilla de un reloj de lenteja sin


= 10,024cm 2 compensar, que bate segundos a 0° C, es de latón.
Siendo β = 3α = 36 × 10 C
−6 o −1
Averiguar cuánto se retrasa el reloj en un día si se
Obtenemos: introduce en un ambiente a 200° C. Coeficiente de
dilatación del latón: α = 17 x 10-6 °C-1. (Considerar
V ′ = V0 (1 + β ΔT ) = 10 ×100(1 + 36 × 10 −6 × 100) el péndulo como simple, de longitud la misma que la
= 1003,6cm3 varilla.)
Solución.
Ejemplo 6. Un herrero ha de colocar una llanta l0
circular de 1 m de diámetro a una rueda de madera A 0° el semiperíodo (1 s) será: 1 = π
g
de igual diámetro. Con objeto de poder ajustarla,
calienta la llanta hasta conseguir que su radio supere l 0 (1 + αΔT )
en 2 mm al de la rueda. Sabiendo que la temperatura A 200°: τ = π
ambiente es de 20 °C y su coeficiente de dilatación g
lineal es 12,2 x 10- 6 °C-1, calcular la temperatura en Dividiendo:
grados centígrados a que debe calentarse la llanta
para cumplir las condiciones expuestas.
τ = 1 + αΔT = 1 + 17 × 10 −6 × 200
Solución. = 1,0034 s =1,0017 s
l ′ = l(1 + αΔT ) = 2πr ′(1 + αΔT ) Como un día dura 86400 segundos el péndulo dará
d ′ = d (1 + αΔT ) 86400
= 86253 semioscilaciones
Luego 1,0017
d′ − d 4 × 10 −3 El péndulo da en 1 día 86 400 - 86 253 = 147
ΔT = = = 327 o C semioscilaciones menos que en su marcha correcta:
αd 12,2 × 10 − 6 × 1
El reloj se retrasará en 147 s = 2 min 27 s
⇒ T = 20 + ΔT = 347o C
Ejemplo 9. Una varilla de cobre de densidad
Ejemplo 7. Un anillo de acero, de 75 mm de uniforme y de sección constante oscila como un
diámetro interior a 20 °C, ha de ser calentado e péndulo colgada de uno de sus extremos, con un
introducido en un eje de latón de 75,05 mm de periodo de 1,6 s cuando se encuentra a una
diámetro a 20 °C. determinada temperatura ambiente. Siendo el
a) ¿A qué temperatura ha de calentarse el anillo? coeficiente de dilatación lineal del cobre
b) ¿A qué temperatura tendríamos que enfriar el 19 x 10- 6 °C-1, determínese el incremento de
conjunto para que el anillo saliera él solo del eje? temperatura que habría que darle al ambiente para
(Coeficiente de dilatación del acero: 12 x 10-6 °C-1; que el período aumente en 3 milésimas de s.
coeficiente de dilatación del latón: 20 x 10-6 °C-1) Solución.
Solución. El período a la temperatura inicial T es:
a) D′ = D(1 + αΔT ) 1 2
Ml
⇒ 75,05 = 75(1 + 12 × 10−6 ΔT ) I 3 2l
τ = 2π = 2π = 2π
75,05 − 75 Mgd l 3g
⇒ ΔT = = 55o C Mg
75 × 12 × 10 −6 2

⇒ T = T + ΔT = 20 + 55 = 75o C y a la temperatura T + ΔT será:
b) Los diámetros a la temperatura que nos piden 2l(1 + αΔT )
deberán ser iguales:
T ′ = 2π
3g
D(1 + α a ΔT ′) = D ′′(1 + α l ΔT ′) dividiendo los dos:
D = diámetro del anillo a 20° C; T′
D’’= diámetro del eje a 20 °C; = (1 + αΔT ) ⇒
α a y α l , coeficiente de dilatación del acero y del T
2 2
latón, respectivamente). Luego: ⎛T′⎞ ⎛ 1,603 ⎞
⎜ ⎟ −1 ⎜ ⎟ −1
⎝ T ⎠ ⎝ 1,6 ⎠
D − D ′′ ΔT = = = 197ºC
ΔT ′ = α 19 × 10 −6
D ′′ × 20 × 10 −6 − 75 × 12 × 10 −6

6
Calor y Termodinámica Hugo Medina Guzmán

Ejemplo 10. La densidad del mercurio a 0°C es El volumen del mercurio que se derrama 100 °C es:
13,6 g/cm3; su coeficiente de dilatación, 182 x 10- 6 Vx = V ′ − VHg
′ = 5091 - 5043,5
°C-l. Calcular la densidad del mercurio a 100 °C.
Solución. = 47,5cm3
ρ 13,6
ρ′ = = Ejemplo 13. Dos barras de longitudes LA, LB
1 + β ΔT 1 + 182 × 10− 6100 coeficientes de dilatación lineal αA y αB
= 13,36 g/cm3 respectivamente se sujetan en un extremo, existiendo
en el extremo libre una diferencia de longitud ΔL.
Ejemplo 11. Una vasija de cinc (coeficiente de Qué relación debe existir entre sus coeficientes de
dilatación lineal: 29 x 10-6 °C-l) está llena de dilatación lineal tal que dicha diferencia de longitud
mercurio a 100 °C, teniendo entonces una capacidad se mantenga constante cuando el conjunto se somete
de 10 l . Se enfría hasta 0°C. Calcular la masa de a una variación de temperatura.
mercurio, medida a 0 °C, que hay que añadir para Solución.
que la vasija quede completamente llena.
Coeficiente de dilatación del mercurio, 182 x 10-6
°C-l.
Densidad del mercurio a 0 °C, 13,6 g/cm3.
Solución.
El volumen de la vasija a 0° quedará determinado Como ΔL = constante
por la ecuación:
LB − L A = L' B − L' A ,
V ′ = V (1 − βΔT )
LB − L A = LB (1 + α B ΔT ) − L A (1 + α A ΔT )
V'
⇒ V= , De aquí: LBα B ΔT = L Aα A ΔT
(1 − βΔT )
en la que: β = 3 x 29 x10-6°C-1 = 87 x10-6 °C-1
α L
Finalmente: B = A
V ′ = 1000 cm3 ΔT = (0 − 100) = - 100°C α A LB
1000
Por tanto: V = −6
= 991,38 cm3 Ejemplo 14. Un tubo de acero, cuyo coeficiente de
1 + 87 × 10 × 100 expansión lineal es α = 18 x 10-6, contiene mercurio,
cuyo coeficiente de expansión de volumen es β =
El volumen del mercurio a 0° quedará determinado 180 x 10-6 °C-1; el volumen de mercurio contenido
por la misma ecuación en la que en el tubo es 10-5 m3 a 0 °C, se desea que la columna
β Hg = 182 × 10 −6 o C −1 : de mercurio permanezca constante para un rango
normal de temperaturas. Esto se logra insertando en
V′ 1000 la columna de mercurio una varilla de silicio, cuyo
VHg = = =
1 + β Hg ΔT 1 + 182 × 10− 6 × 100 coeficiente de dilatación es despreciable.
Calcular el volumen de la varilla de silicio.
982,13 cm3

La diferencia es el volumen que queda por llenar:


V - VHg = 991,38 – 982,13 = 9,25 cm3

La masa del mercurio que hay que agregar es:


ΔM = ρ Hg ΔV = 13,6 x 9,25 = 125,8 g

Ejemplo 12. Una vasija de Zn está llena de


mercurio a 0°C, teniendo una capacidad de 5 l .
Calcular el volumen de mercurio que se derrama a Solución.
100 °C por efecto de la mayor dilatación de este A 0°C, sea Vo el volumen de la varilla de silicio y V
último. (Tomar los datos necesarios del problema el volumen de mercurio, a esta condición tenemos
anterior.) l 0 A0 = V + V0
Solución. A una temperatura t la sección Ao se incrementa a
β = 87 x10-6 °C-1 Ao (1 +2αt).
Vasija: V ′ = V (1 + β ΔT ) = 5000(1 + 87x 10-6 x Similarmente el volumen de mercurio cambia de V a
100) = 5043,5 cm3 V(1 +βt).
El volumen del mercurio a 100 °C es: Como se requiere que l o permanezca constante, se
VHg′ = 5000 (1 + 182 x 10-6 x 100) tiene
= 5091 cm3 l o Ao (1 +2αt) = (V + Vo) (1 + 2αt)

7
Calor y Termodinámica Hugo Medina Guzmán

Por otro lado este volumen es: V(1 +βt ) + Vo d = 4,99 mm (a 0 °C). ¿Hasta que temperatura hay
igualando ambas expresiones que calentar al disco para que por el orificio empiece
(V + Vo) (1 + 2αt) = V(1 + βt ) + Vo a pasar una bola de diámetro D = 5,00 mm? El
⇒ Vo (1 + 2αt-1) = V(1 + βt - 2αt) coeficiente de dilatación lineal del acero es α = 1,1 x
10-5 K-1.
⇒ V0 = V ( β - 2α )t = V( β - 2α ) Solución.
2αt 2α
-6
d (1 + αΔT ) = D , reemplazando valores:
V (180 - 36)10
= = 4V 4,99(1 + 1,1 × 10 −5 ΔT ) = 5,00
36 x10− 6
= 4 x 10-5m3 Resolviendo encontramos ΔT = 182 , como la
La varilla de silicio ocupa los 4/5 del volumen total temperatura inicial es 0°C, es necesario elevar la
a 0°C. temperatura hasta 182°C.

Ejemplo 15. Ejemplo 18. Una bola de vidrio de coeficiente de


dilatación cúbica es β, se pesa tres veces en el aire y
Una barra de acero, α ACERO = 11 × 10 /º C ,
−6
en un líquido a las temperaturas t1 y t2. Las
tiene un diámetro de 3 cm a la temperatura de 25 ºC. indicaciones de las balanzas para las tres pesadas
Un anillo de bronce, α BRONCE = 17,10 −6 /º C , son: P, P1 y P2. Determinar el coeficiente de
dilatación cúbica del líquido.
tiene un diámetro interior de 2,992 cm a la misma
Solución.
temperatura. ¿A qué temperatura común entrará
Supongamos que el volumen de la bola a la
justamente el anillo en la varilla?
temperatura t1 es igual a V, entonces a la temperatura
Solución.
t2 será igual a V (1 + βΔt), donde Δt = t2 – t1
Puesto que los diámetros son cantidades lineales,
Escribamos las indicaciones de las balanzas para las
éstas se dilatarán con la temperatura. Como la
tres pesadas:
temperatura inicial es de 25 ºC y la final T donde
los diámetros deben coincidir, se tiene:
P = ρVg ,
d A = d 0 A [1 + α ACERO (T − 25)] P1 = P − ρ1Vg ,
d B = d 0 B [1 + α BRONCE (T − 25)] (1 + βΔt )
P2 = P − ρ 1Vg .
Despejando T , encontramos: (1 + β 1 Δt )
d (1 − 25α A ) + d 0 B (25α B − 1) Donde ρ es la densidad del vidrio y ρ1 la densidad
T = 0A del líquido (ambas a la temperatura t1).
(d 0 Bα B − d 0 Aα A ) En la fórmula de P despreciamos la fuerza de
= 472,83 ºC. empuje por ser pequeña la densidad del aire. Por eso
no tiene importancia la temperatura a que hizo esta
Ejemplo 16. Un vaso de vidrio de 75 cm3 se llena pesada.
completamente de mercurio a la temperatura De las tres ecuaciones se obtiene β1 en función de P,
ambiente de 25 ºC. A la temperatura de 20 ºC, ¿Cuál P1 , P2, t1, t2 y β que son conocidos:
será el volumen de mercurio derramado? P2 − P1 + ( P − P1 ) β (t 2 − t1 )
β Hg = 18,21 x 10-5 / ºC, β1 =
( P − P2 )(t 2 − t1 )
αV = 9,6 x 10-6 / ºC . En la práctica se suele utilizar una bola de vidrio de
Solución. cuarzo cuyo coeficiente de dilatación cúbica es
El volumen derramado V D corresponde a la mucho menor que el coeficiente de dilatación cúbica
de la inmensa mayoría de los líquidos. En este caso
diferencia entre el volumen de mercurio VHg menos la respuesta se puede simplificar:
el volumen del vaso VV , es decir: ( P2 − P1 )
β1 =
VD = VHg − VV ( P − P2 )(t 2 − t1 )
= V0 (1 + β Hg ΔT ) − V0 (1 + 3α V ΔT ) Ejemplo 19. Dos láminas, una de acero y otra de
= V0 ΔT (β Hg − 3α V ) bronce, de igual espesor a = 0,2 mm, están
remachadas entre sí por sus extremos de manera que
= (75)(− 5)(18,21 − 2,88) × 10
−5
a la temperatura T1 = 293 K forman una lámina
= - 0,058 cm3 bimetálica plana. ¿Cuál será el radio de flexión de
Se derraman 0,058 cm3 de mercurio esta lámina a la temperatura T2 = 393 K?
El coeficiente de dilatación lineal:
Ejemplo 17. En el centro de un disco de acero hay Acero es α 1 = 1,1 × 10 −5 Κ −1 y del
un orificio de diámetro
Bronce es α 1 = 2 × 10 Κ .
−5 −1

8
Calor y Termodinámica Hugo Medina Guzmán

Solución.

Solución.
En el esquema se muestran las dilataciones que se
producirían en cada barra si no estuvieran soldadas
(a) y las deformaciones por estarlo (b).
Vamos a suponer que la línea medía de cada lámina
conserva la longitud que tendría en estado no
curvado. El radio r se determina por las condiciones
a
ϕ (r − ) = l + Δl 1 , Δl 1 = lα1ΔT ,
2
a
ϕ (r + ) = l + Δl 2 , Δl 2 = lα 2 ΔT ,
2
a a
(1 + α 1 ΔT )(r + ) = (1 + α 2 ΔT )(r − ) ,
2 2
Por consiguiente
a[2 + (α1 + α 2 )ΔT ]
r= = 22,5cm
2(α 2 − α1 )ΔT
También se tiene que la distribución de fuerzas
elásticas que igualan la longitud del sistema, por
FATIGA DE ORIGEN TÉRMICO. simetría se puede considerar de la siguiente forma
Consideremos una barra de sección A sujeta en siguiente:
ambos extremos
F2 = 2 F1

Al aumentar la temperatura Δt , debería producirse De este esquema tenemos las siguientes relaciones
un cambio de longitud geométricas entre las deformaciones:
Δl Dividiendo esta expresión entre L0 , tenemos una
= αΔt
l relación entre las deformaciones unitarias
pero como no se puede dilatar por estar sujeta, la ΔL2 ΔL' 2 ΔL1 ΔL'1
tensión debe aumentar hasta un valor suficiente para − = +
producir el mismo cambio pero de sentido inverso, L L L L
este esfuerzo es: Como:
F Δl ΔL1 ΔL'1 F
=Y , reemplazando obtenemos: = α 1 Δt y = 1
A l L L AY1
F ΔL2 ΔL' 2 F
= YαΔt = α 2 Δt y = 2
A L L AY2
Reemplazando se tiene:
Ejemplo 20. Una platina de cobre se suelda con dos
platinas de acero, como se muestra en la figura. Las F1 F2
tres platinas son iguales, teniendo exactamente la
α 1 Δt − = α 2 Δt +
AY1 AY2
misma longitud a temperatura ambiente. Calcular las
fatigas que se producirán al aumentar la temperatura Con F2 = 2 F1
en Δ t grados. F 2 F1
α 1 Δt − 1 = α 2 Δt +
AY1 AY2

9
Calor y Termodinámica Hugo Medina Guzmán

Despejando F1 A Resolviendo (1) y (2) obtenemos


(l Aα A + l Bα B ) 40
Δl A =
F1 (α 2 − α 1 )Δt l Y
(1 + B A )
=
A ⎛1 2⎞ l A YB
⎜⎜ + ⎟⎟ (l α + l Bα B ) 40
⎝ Y1 Y2 ⎠ Δl B = A A
l Y
Y las fatigas serán: (1 + A B )
F1 (α 2 − α 1 )Δt l B YA
S1 = = y Reemplazando valores tenemos:
A ⎛1 2⎞
⎜⎜ + ⎟⎟ Δl A = 2,1 x 10 -2 cm y
⎝ Y1 Y2 ⎠
Δl B = 2,1 x 10-2 cm.
F 2F 2(α 2 − α 1 )Δt
S2 = 2 = 1 = y el esfuerzo en cada varilla
A A ⎛1 2⎞ F YA Δl A YB Δl B
⎜⎜ + ⎟⎟ = =
⎝ Y1 Y2 ⎠ A lA lB
Nota: Por sencillez de exposición, se ha omitido
precisar que al determinar las deformaciones
dina 2,1 x 10 -2 cm
11
= 10 x 10 x
ΔL'1 ΔL' 2 cm 2 25 cm
unitarias y se han despreciado los dina
L L = 0,84 x 10
9

términos de segundo orden. cm 2


ΔL'1 ΔL'1 F
≈ = 1 y Ejemplo 22. Una barra de bronce se enfría en
L + ΔL1 L AY1 nitrógeno líquido hasta la temperatura T1 = 72 K.
ΔL' 2 ΔL' 2 F Así enfriada, esta barra se introduce ajustadamente
≈ = 2 en la abertura rectangular de una abrazadera rígida,
L + ΔL2 L AY2 que está a la temperatura T2 = 293 K, de manera que
Debido a L >> ΔL1 y L >> ΔL2 . la holgura entre los extremos de la barra y los planos
correspondientes de la abertura de la abrazadera
puede considerarse nula. ¿Qué presión ejercerá la
Ejemplo 21. Dos varillas del mismo diámetro, una barra sobre la abrazadera cuando se caliente hasta la
de bronce de 25 cm. de longitud, y la otra de acero temperatura T2 = 293 K? El coeficiente de dilatación
de 50 cm. De longitud se colocan extremo a extremo lineal del bronce es α = 1,75 x10-5 K-l y el módulo de
y aseguradas entre dos soportes rígidos. Young Y = 1,04 x 1011 Pa.
La temperatura de las varillas se eleva 40°C. Solución.
¿Cuál es el esfuerzo en cada varilla? Al enfriarse, la barra se contrae. Su longitud se hace
[ ]
11 −2
Módulo de Young del acero 20 x 10 dina cm igual a l = l 0 1 − α (T2 − T1 ) , de donde
11 −2
Módulo de Young del bronce: 10 x 10 dina cm
(l 0 − l )
Coeficiente de dilatación térmica acero 1,2x10
−5
= α (T2 − T1 ) , Después de calentar la
por °C l0
Coeficiente de dilatación térmica bronce 1,8x10
−5 barra, apretada en la abrazadera, su longitud sigue
por °C siendo l , y la compresión (l − l 0 ) estará ahora
Solución. motivada por las fuerzas elásticas.
Al elevarse la temperatura las varillas deberían
(l 0 − l ) p
expandirse si les fuera permitido, pero al no ser así Escribamos la ley de Hooke: = , donde
sufren esfuerzo de compresión, las fuerzas en las dos lo Y
varillas debe ser la misma. Por lo tanto, la unión p es la presión que ejerce 1a abrazadera sobre la
debe de desplazarse hasta alcanzar el equi1ibrio. barra en la dirección del eje de ésta.
Entonces los esfuerzos son iguales.
(l 0 − l )
F YA Δl A YB Δl B Comparando las expresiones de hallamos
= = (1) lo
A lA lB
que 1a presión que buscábamos:
Pero la longitud (Δl A + Δl B ) es igual a la p = Yα (T2 − T1 ) = 4 × 108 Pa .
cantidad que no se deje expandir por dilatación Conviene advertir que la presión no depende de la
Δl´A + Δl´B = l Aα AΔt + l Bα B Δt longitud de la barra.
Luego:
Δl A + Δl B = ( l Aα A + l Bα B ) 40 (2) Ejemplo 23. Entre dos paredes se encuentra una
barra, de sección A, compuesta de dos partes de

10
Calor y Termodinámica Hugo Medina Guzmán

igual longitud l/2 que tienen los coeficientes de


l 1 = l 2 [1 + α (T1 − T2 )] ,
(l1 − l 2 ) = α (T − T2 ) ,
dilatación lineal αl y α2 y los módulos de Young Yl y 1
l2
Y 2. A 1a temperatura T1 los extremos de la barra
apenas tocan las paredes. Donde l 1 y l 2 son las longitudes de la
¿Con qué fuerza presionará dicha barra sobre las circunferencia interna a las temperaturas T1 = 573 K
paredes si se calienta hasta la temperatura T2. y T2= 291 K. Despreciando la disminución del
Despréciese la deformación de las paredes. ¿Cuánto diámetro del cilindro de acero bajo la acción de los
se desplazará la junta de las partes de la barra? esfuerzos compresoras por parte del anillo,
Solución. consideraremos que, después de enfriarse el anillo,
Cuando la barra se calienta desde la temperatura T1 la longitud do su circunferencia interna sigue siendo
hasta la temperatura T2, sin paredes que la limiten, se igual a l1 y el anillo resulta estirado por las fuerzas
alarga en la magnitud elásticas. Como en nuestro caso el grosor del anillo
⎛l⎞ es pequeño en comparación con su diámetro se
Δl = Δl 1 + Δl 2 = ⎜ ⎟(α1 + α 2 )(T2 − T1 ) . puede suponer que el alargamiento relativo de todas
⎝2⎠
Con las paredes limitadoras la barra calentada (l 1 − l 2 )
sus capas es el mismo e igual a .
resulta comprimida en esta misma magnitud. Por la l2
ley de Hooke (la fuerza compresora F es la misma Entonces la extensión del anillo se puede relacionar
en ambas partos de la barra) con el esfuerzo de tracción por medio de la ley de
l1F l 2 F l ⎛ 1 1 ⎞ F (l 1 − l 2 ) F
Δl = + ≈ ⎜ + ⎟ =
Y1S Y2 S 2 ⎜⎝ Y1 Y2 ⎟⎠ A
Hooke: , donde F es el esfuerzo do
l2 YA
Esta relación, en términos generales, es aproximada, tracción; A, la sección del anillo, y Y, el módulo de
ya que las longitudes l1 y l2 de !as partes de la barra Young. En definitiva se obtiene que
a la temperatura T2 las hemos sustituido por su F = Yα (T1 - T2) = 3360 N.
longitud l/2 a la temperatura T1. No obstante, se Esta solución no es exacta totalmente debido o sólo
comprende fácilmente que el error relativo que se a que hemos sustituido la deformación no
comete al determinar Δl por esta f6rmula será del homogénea del anillo por su alargamiento uniforme,
orden do Δl/l y, por lo tanto, nuestra aproximación sino también a que las tensiones radiales provocan
en el anillo la variación de la longitud de su
es muy buena (Δl << l) De las relaciones antes
circunferencia. Cuanto menor sea el espesor del
escritas hallamos.
anillo en comparación con su diámetro, tanto
(α 1 + α 2 ) menores serán las correcciones a introducir por estas
F= Y Y A(T − T ) .
(Y1 + Y2 ) 1 2 2 1 circunstancias.
El desplazamiento Δl de la junta de las partes de la Ejemplo 25. Un tubo de acero de 28,0 m de
barra se puedo determinar tomando en consideración longitud, se instaló cuando la temperatura era de 15º
que éste se compone del desplazamiento debido a la C, se usa para transportar vapor sobrecalentado a la
dilatación (por ejemplo, de la primera parte de la temperatura e 110º C. El coeficiente de expansión
barra) y del desplazamiento inverso causado por lineal del acero es 1,2 x 10-5 K-1, el módulo de
compresión: Young es 2,0 x 1011 Pa, y el esfuerzo de ruptura es
l⎡ F ⎤ 5,0 x 108 Pa.
Δl = ⎢α1 (T2 − T1 ) −
Y1 A ⎥⎦
a) El tubo puede expandirse libremente cuando
2⎣ transporta vapor. ¿En cuánto incrementa su
l (α1Y1 − α 2Y2 ) longitud?
= (T2 − T1 ) b) A la temperatura de 15º C la tubería se aseguró al
2 (Y1 + Y2 ) piso de concreto tal que se impide la expansión
lineal. ¿Cuál es la relación entre el esfuerzo térmico
Ejemplo 24. Un anillo de latón de varios en el tubo y el esfuerzo de ruptura del acero, cuando
centímetros de diámetro se calienta hasta la se transporta el vapor?
temperatura T1 = 573 K y se encaja ajustadamente Solución.
sobre un cilindro de acero cuya temperatura es T2 = a)
291 K. ¿Qué esfuerzo de rotura experimentará el α =1,2 x 10-5 K-1, L = 28,0 m
anillo una vez enfriado hasta 291 K? .El coeficiente Δθ = 110 − 15 = 95º C .
de dilatación lineal del latón es α = 1,84 x 10-6 K-l y ΔL = αLΔθ
su módulo de Young Y = 6,47 x 1010 Pa. Las
dimensiones de la sección del anillo son 2 x 5 mm2. ( )
⇒ ΔL = 1,2 × 10 −5 (28)(95) = 3,192 x 10-2 m
Solución. ΔL F S
Al ser calentada, la longitud de la circunferencia b) = =
interna del anillo aumentó: L YA Y

11
Calor y Termodinámica Hugo Medina Guzmán

ΔL ⎛ αLΔθ ⎞ entre la cantidad de trabajo hecho contra la fricción


⇒ S =Y = Y⎜ ⎟ = YαΔθ y el calor producido.
L ⎝ L ⎠ En 1843 James Prescott Joule empleó un aparato en
Este es el esfuerzo térmico el cual el agua se agitaba por un conjunto de paletas
( )
S = 2,0 × 1011 1,2 × 10 −5 (95) = 2,28 x 108 Pa giratorias y la energía mecánica suministrada para
rotar las paletas podía medirse con aproximación. El
esfuerzo térmico 2,28 × 10 8 efecto térmico del trabajo mecánico hecho sobre el
= = 0,456
agua, era la elevación de la temperatura. El
esfuerzo de rúptura 5,0 × 10 8
experimento de Joule demostró que la elevación de
la temperatura era proporcional a la cantidad de
Ejemplo 26. Una esfera hueca del metal está trabajo hecho sobre el agua. Por consiguiente el
flotando en el agua a 0 ºC. Si la temperatura del trabajo realizado en agitar el agua es equivalente al
agua se eleva a θ ºC, la esfera se sumerge calor añadido al agua.
completamente en el agua sin hundirse. Desprecie la A pesar de que no necesitamos unidades especiales
expansión de la esfera. Encuentre la expresión para para el calor, una vez reconocido que es una forma
determinar coeficiente de dilatación cúbica del de energía medible en Joules, o cualquier otra
agua. unidad de energía, se sigue utilizando la unidad
Solución. histórica del calor, es decir la CALORIA. La caloría
Dados: se define cuantitativamente como la cantidad de
ρ e , la densidad de la esfera, energía necesaria para elevar la temperatura de un
gramo de agua desde 14,5°C a 15,5°C. La cantidad
ρ0 , la densidad del líquido
de energía para elevar la temperatura de un
β, Coeficiente de dilatación cúbica del líquido kilogramo de agua desde 14,5°C a 15,5°C es la
kilocaloría. La “caloría” utilizada para medir el
( ρ θ ) agua = ( ρ e ) esfera equivalente energético de los alimentos es realmente
la kilocaloría. En el sistema ingles la unidad es el
British thermal unit (BTU)
Como Vaθ = Va 0 (1 + βθ ) ⇒ 1 BTU = 252 calorías
El equivalente exacto entre el trabajo realizado y el
ma ma 1 1
= (1 + βθ ) ⇒ = (1 + βθ ) ⇒ calor añadido está dado por la relación experimental.
ρθ ρ0 ρθ ρ0 1 cal = 4,186 Joules
1 BTU = 778 libra pie
Esta relación es conocida como el EQUIVALENTE
ρθ = ρ 0 (1 − βθ ) MECANICO DE CALOR

ρ 0 (1 − βθ ) = ρ e
CAPACIDAD CALORIFICA. CALOR
Igualando ESPECÍFICO
La cantidad de calor necesario para producir un
aumento de temperatura en una cierta masa depende
ρ0 − ρe
Finalmente β= de la sustancia. Definamos primero:
θρ e La CAPACIDAD CALORIFICA. (C) de un cuerpo
es la cantidad de calor requerido para elevar la
temperatura de un cuerpo en un grado,
CALOR Y TRABAJO
Cuando dos sistemas a diferente temperatura se dQ
C=
hallan en contacto térmico, el calor fluye del sistema dT
mas caliente al más frío, hasta que alcanzan el Sus unidades son: Caloría/°C, BTU/°F.
equilibrio a una temperatura común, la cantidad de Luego, definamos:
calor que sale de un cuerpo es igual a la cantidad de El CALOR ESPECIFICO (c) es la capacidad
calor que entra en el otro. Inicialmente se elaboró la calorífica por unidad de masa:
teoría del calórico, para explicar este flujo, esta C dQ / dT dQ
sustancia no podía ser creada ni destruida, pero si c= = =
transferida de un cuerpo a otro. La teoría del m m mdt
calórico servía para describir la transferencia de Sus unidades son cal/gr x °C ó BTU/libra x °F
calor, pero se descartó al observar que el calórico se kcal cal
creaba por fricción y no habría una desaparición Observe que: 1 =1
correspondiente de ca1órico en ningún otro sitio.
kg°C g°C
En 1778 el Conde Rumford, como punto de sus Y que:
observaciones en el taladro de cañones propuso que 1 BTU 250 cal cal kcal
= =1 =1
él calor debe estar asociado con el movimiento. Pero 1 libra°F 453,6 g 5/9°C g°C kg°C
no se estableció sino hasta medio siglo después de O sea que el valor numérico del calor específico es
esta observación que había una relación definida el mismo en esas tres unidades.

12
Calor y Termodinámica Hugo Medina Guzmán

m1c1 (t − t1 ) = m2 c 2 (t − t 2 )
A pesar que el calor específico de la sustancias varía
o bien
ligeramente con la temperatura, será adecuado para
nuestra discusión, asumir que el calor específico es − m1c1 (t − t1 ) + m2 c 2 (t − t 2 ) = 0
constante independiente de la temperatura. Luego o sea: Calor perdido = calor ganado
podemos determinara el calor Q necesario para m1c1t1 − m1c1t = m2 c 2 t − m2 c 2 t 2
elevar la temperatura de la masa m de una sustancia
Δt grados, de la siguiente manera: m1c1t1 + m2 c 2 t 2 = (m1c1 + m2 c 2 )t
Despejando el valor de la temperatura final t:
Q = m ∫ cdt = mc(T f − Ti )mcΔT
Tf
m1c1t1 + m2 c 2 t 2
Ti
t=
m1c1 + m2 c 2
CALOR ESPECIFICO Determinación del calor específico de un sólido
Aluminio 0,212 Hielo 0,48 La experiencia se realiza en un calorímetro
Acero 0,11 Carbón 0,3 consistente en un vaso (Dewar) o en su defecto
Bronce 0,090 Concreto 0.16 convenientemente aislado. El vaso se cierra con una
Cobre 0,094 Vidrio 0,12 - 0,20 tapa hecha de material aislante, con dos orificios por
Oro 0,031 Parafina 0,69 los que salen un termómetro y el agitador.
Plata 0,056 Caucho 0,48
Platino 0,032 Madera 0,3 – 0,7
Plomo 0,031 Agua 1,00
Tungsteno 0,032 Alcohol 0,6
Zinc 0,094 Petróleo 0,51
Agua de mar 0,93

La capacidad calorífica depende del tipo de proceso


que se realiza durante la transferencia de calor. Se pesa una pieza de material sólido de calor
Tiene valores definidos solamente para procesos específico c desconocido, resultando m su masa. Se
definidos. pone la pieza en agua casi hirviendo a la temperatura
T.
En particular manteniendo la presión constante se Se ponen M gramos de agua en el calorímetro, se
denomina capacidad calorífica a presión constante agita, y después de un poco de tiempo, se mide su
Cp y si se mantiene el volumen constante se temperatura T0. A continuación, se deposita la pieza
denomina capacidad calorífica a volumen constante de sólido rápidamente en el calorímetro. Se agita, y
Cv. En general Cp y Cv son diferentes y se después de un cierto tiempo se alcanza la
analizarán con algún detalle más adelante. temperatura de equilibrio Te.
Ejemplo 27. Dos sustancias m1 y m2 de calores mc es la masa del vaso del calorímetro y c c su
específicos c1 y c2 están a temperatura t1 y t2 calor específico.
respectivamente (t1 > t2). mt la masa de la parte sumergida del termómetro y
Calcular la temperatura final que alcanzan al
ponerlos en contacto, sabiendo que no se presentan ct su calor específico
cambios de estado. ma la masa de la parte sumergida del agitador y c a
Solución.
su calor específico
M la masa de agua que contiene el vaso, su calor
específico es la unidad
Por otra parte:
Sean m y c las masa y el calor específico del
cuerpo problema a la temperatura inicial T.
En el equilibrio a la temperatura Te se tendrá la
siguiente relación.
(M + k )(Te − T0 ) + mc(Te − T ) = 0
La capacidad del calorímetro dada por
k = mc cc + mt ct + ma c a , se le denomina
Por conservación de energía:
equivalente en agua del calorímetro, y se expresa en
∑Q = 0 gramos de agua, y es una constante para cada
calorímetro.
Como: Q = mc (t f -tf)
El calor específico desconocido del será por tanto
Se tiene:

13
Calor y Termodinámica Hugo Medina Guzmán

c=
(M + k )(Te − T0 ) Solución.
Tomemos como calor específico del aluminio
m(T − Te ) c = 0,215 cal/g ºC, entonces
En esta fórmula tenemos una cantidad desconocida Q = mcΔt = 3000 x 0,215 x (50 - 20) = 1,935 x 104
k, que debemos determinar experimentalmente. cal

Determinación del equivalente en agua del Ejemplo 30. Un trozo de 300 g de cobre se calienta
calorímetro en un horno y en seguida se deja caer en un
Se ponen M gramos de agua en el calorímetro, se calorímetro de 500 g de aluminio que contiene 300 g
agita, y después de un poco de tiempo, se mide su de agua. Si la temperatura del agua se eleva de 15ºC
temperatura T0. A continuación se vierten m gramos a 30ºC ¿cuál era la temperatura inicial del cobre?
de agua a la temperatura T. Se agita la mezcla y (Suponga que no se pierde calor.) ¿Cuánto calor se
después de un poco de tiempo, se mide la debe agregar a 20 g de aluminio a 20ºC para fundirlo
temperatura de equilibrio Te. completamente?
Como el calorímetro es un sistema aislado Solución.
tendremos que cAl = 0,215 cal/g.ºC
(M + k )(Te − T0 ) + m(Te − T ) cH2O = 1 cal/g.ºC
cCu = 0,0924 cal/g.ºC
(T − Te ) Qabsorbido = 300 x 1 x (30 - 15) + 500 x 0,215 x (30 -
⇒ k= m−M
(Te − T0 ) 15)
Qcedido = 300 x 0,0924 x (ti - 30)
Entonces
Ejemplo 28. Calcule el calor específico de un metal
300 x 1 x (30 - 15) + 500 x 0,215 x (30 - 15) = 300 x
con los siguientes datos. Un recipiente
0,0924 x (ti - 30), de donde la temperatura inicial del
(“calorímetro”) hecho de metal cuya masa es 3,64
Cobre resulta ser ti = 250,51 ºC.
kg contiene 13,6 kg de agua. Un pedazo de metal de
Para saber las calorías necesarias para fundir 20
1,82 kg de masa, del mismo material del recipiente y
gramos de aluminio a 20 ºC, de las tablas obtenemos
con temperatura de 176,7 ºC se echa en el agua. El
para el calor de fusión:
agua y el recipiente tienen inicialmente una
Lf (Al) = 3,97x105 J/kg a t = 660 ºC, de modo que el
temperatura de 15,5 ºC y la temperatura final de todo
calor necesario será
el sistema llega a ser de 18,33 ºC.
Como 1 J = 0,24 cal de modo que
Solución.
Lf (Al) = 3,97 x 102 x 0,24 = 95,28 cal/g
Debido a que se trata de un problema de intercambio
de calor, el calor entregado por el metal = calor Entonces Q = mcΔt + mLf
Q = 20 x 0,215(660 - 20) + 20 x 95,28 = 4657,6 cal
recibido por el (agua y recipiente). Llamando Q1 al
calor liberado por el metal, Q 2 , Q3 a los recibidos Ejemplo 31. Una moneda de cobre de 3 g a 25ºC,
cae al piso desde una altura de 50 m.
por el agua y recipiente respectivamente:
a) Sí 60% de su energía potencial inicial se gasta en
Q1 + Q 2 + Q3 = 0. aumentar su energía interna, determine su
Considerando que el metal y recipiente tienen un temperatura final.
calor específico c m , reemplazando en la expresión b) ¿Depende el resultado de la masa del centavo?
Explique.
anterior:
Q1 = mmetal cm (T final − Tmetal ) ,
Solución.
cCu = 0,0924 cal/g ºC
Q2 = magua cagua (T final − Tagua ) y mCu = 3 g
a) La energía potencial será
Q3 = mrecipientecm (T final − Trecipiente ) U = mgh = 0,003 x 9,8 x 50 = 1,47 J = 0,35 cal
Entonces
mmcm (T f − Tm ) + ma ca (T f − Ta ) + mr cm (T f − Tr ) = 0 ,
Q 0,6 × 0,35
t f = ti + = 25 + = 25,76 ºC
Es decir: mcCu 3 × 0,0924
− ma ca (T f − Ta ) b) No depende de m: porque Q es proporcional m y
cm =
mm (T f − Tm ) + mr (T f − Tr )
el aumento de temperatura es inversamente
proporcional a m.

−2 ⎡ cal ⎤
= 1,38 × 10 ⎢ ⎥.
Ejemplo 32. Para medir el calor específico de un
⎣g º C⎦ líquido se emplea un calorímetro de flujo. Se añade
calor en una cantidad conocida a una corriente del
Ejemplo 29. ¿Cuántas calorías se requieren para líquido que pasa por el calorímetro con un volumen
elevar la temperatura de 3 kg de aluminio de 20ºC a conocido. Entonces, una medición de la diferencia
50ºC? de temperatura resultante entre los puntos de entrada
y salida de la corriente de líquido nos permite

14
Calor y Termodinámica Hugo Medina Guzmán

calcular el calor específico del líquido. Un líquido de Volumen definido.


0,85 g/cm3 de densidad fluye a través de un
calorímetro a razón de 8,2 cm3/s. Se añade calor por Puede ser orgánico o inorgánico
medio de un calentador eléctrico en espiral de 250
W, y se establece una diferencia de temperatura de
15oC en condiciones de estado estacionario entre los
puntos de entrada y salida del flujo. Halle el calor
específico (c) del líquido.
Solución.

El flujo de calor Q = 250 W que se pone produce
una elevación de temperatura ΔT = 15oC. LÍQUIDO. Incrementando la temperatura el sólido
se va "descomponiendo" hasta desaparecer la
El calor absorbido por una masa m es Q = mcΔT ,
estructura cristalina alcanzándose el estado líquido,
Como es masa que fluye y la entrada de calor es cuya característica principal es la capacidad de fluir
estacionariamente y adaptarse a la forma del recipiente que lo contiene.
En este caso, aún existe una cierta ligazón entre los
dQ • dm átomos del cuerpo, aunque de mucha menor
=Q= cΔT . intensidad que en el caso de los sólidos. El estado
dt dt
líquido presenta las siguientes características:
De aquí
• Fuerza de cohesión menor (regular)
Q
c= , como m = ρV ,
dm Movimiento-energía cinética.
ΔT
dt
Sin forma definida.
dm dV g
=ρ = 0,85 × 8,2 = 6,97
dt dt s Toma el volumen del envase que lo contiene.
Reemplazando valores, tenemos:
En frío se comprime.
250 J
c= o −3
= 2391 o
15 C × 6,97 × 10 kg C Posee fluidez.

FASES DE LA MATERIA Puede presentar fenómeno de difusión.


Otro de los efectos comunes de los cambios de
temperatura son los cambios de estado de los
materiales (sólido, líquido, gaseoso, plasma y CBE).

SÓLIDO. Manteniendo constante la presión, a baja


temperatura los cuerpos se presentan en forma sólida
tal que los átomos se encuentran entrelazados
formando generalmente estructuras cristalinas, lo
que confiere al cuerpo la capacidad de soportar
fuerzas sin deformación aparente. Son, por tanto, Gaseoso. Por último, incrementando aún más la
agregados generalmente rígidos, duros y resistentes. temperatura se alcanza el estado gaseoso. Los
El estado sólido presenta las siguientes átomos o moléculas del gas se encuentran
características: virtualmente libres de modo que son capaces de
ocupar todo el espacio del recipiente que lo contiene,
Fuerza de cohesión (atracción). aunque con mayor propiedad debería decirse que se
distribuye o reparte por todo el espacio disponible.
Vibración. El estado gaseoso presenta las siguientes
características:
Tiene forma propia.
Fuerza de cohesión casi nula.
Los sólidos no se pueden comprimir.
Sin forma definida.
Resistentes a fragmentarse.

15
Calor y Termodinámica Hugo Medina Guzmán

Sin volumen definido.

Se puede comprimir fácilmente.

Ejerce presión sobre las paredes del recipiente que


los contiene.

Los gases se mueven con libertad.


CONDENSADO DE BOSE-EINSTEIN (CBE).
Otro estado de la materia es el condensado de Bose-
Einstein (CBE), predicho en 1924 por Satyendra
Nath Bose y Albert Einstein, y obtenido en 1995 (los
físicos Eric A. Cornell, Carl E. Wieman y Wolfgang
Ketterle compartieron el Premio Nobel de Física de
2001 por este hecho). Este estado se consigue a
temperaturas cercanas al cero absoluto y se
PLASMA. Al plasma se le llama a veces "el cuarto caracteriza porque los átomos se encuentran todos en
estado de la materia", además de los tres "clásicos", el mismo lugar, formando un superátomo.
sólido, líquido y gas. Es un gas en el que los átomos
se han roto, que está formado por electrones La figura siguiente muestra la Condensación de
negativos y por iones positivos, átomos que han Bose-Einstein a 400, 200, y 50 nano-Kelvins
perdido electrones y han quedado con una carga
eléctrica positiva y que están moviéndose
libremente.
La lámpara fluorescente, muy usada en el hogar y
en el trabajo, contiene plasma (su componente
principal es el vapor de mercurio) que calienta y
agita la electricidad, mediante la línea de fuerza a la
que está conectada la lámpara.

El Condensado de Bose-Einstein se ve como una


La línea hace positivo eléctricamente a un extremo y pequeña masa en el fondo de una trampa magnética.
el otro negativo causa que los iones (+) se aceleren Esta masa de condensado es como una gota de agua
hacia el extremo (-), y que los electrones (-) vayan que se condensa del aire cuando éste es enfriado.
hacia el extremo (+). Las partículas aceleradas ganan Cuando se forma inicialmente, el condensado está
energía, colisionan con los átomos, expulsan rodeado todavía de átomos normales de gas, así que
electrones adicionales y así mantienen el plasma, parece la semilla dentro de una cereza.
incluso aunque se recombinen partículas. Las
colisiones también hacen que los átomos emitan luz
y, de hecho, esta forma de luz es más eficiente que
las lámparas tradicionales. Los letreros de neón y las
luces urbanas funcionan por un principio similar y
también se usan (o usaron) en electrónica.
La lámpara de plasma (también llamada "globo de ¿Para qué sirve la Condensación de Bose-Einstein?
plasma" o "esfera de plasma") es un objeto Es muy reciente y sabemos muy poco a cerca de ella
novedoso, que alcanzó su popularidad en los años para dar una respuesta. Es algo así como si
1980. Fue inventada por Nikola Tesla tras su viviéramos en una isla tropical hace 400 años y un
experimentación con corrientes de alta frecuencia en pedazo de iceberg llegara a la costa. Sin que nadie
un tubo de cristal vacío con el propósito de hubiera visto hielo antes, pasaría algún tiempo antes
investigar el fenómeno del alto voltaje. de que alguien se diera cuenta de que puede usarse
para hacer helados.

16
Calor y Termodinámica Hugo Medina Guzmán

Sublimación.
También bajo ciertas condiciones de temperatura y
presión se puede pasar directamente de sólido a gas
son pasar por líquido y se denomina sublimación, Ls
(calor de sublimación).

Ejemplo 33. Se añade calor a una sustancia pura en


un recipiente cerrado a una razón constante. El
gráfico muestra la temperatura de la sustancia como
una función del tiempo. Si Lf es el calor latente de
También hay ciertos problemas de ingeniería que fusión y Lv es el calor latente de vaporización. ¿Cuál
deben ser resueltos antes de que la CBE pueda es el valor de la relación Lv/Lf para esta sustancia?
usarse para mucho.
Sin embargo las similitudes entre CBE y la luz de
láser sugieren que probablemente lo sea.

Solución.
La relación de los tiempos empleados en absorber
calor para la vaporización y la fusión es 5/2, como se
trata de la misma masa en ambos casos, esta relación
CAMBIOS DE ESTADO - CALOR LATENTE será igual a la relación de los calores latentes; esto
Cuando la temperatura de un cuerpo aumenta por LV 5
causa de un calor suministrado, se origina un es: =
aumento de la energía cinética del movimiento de LF 2
las moléculas. Cuando un material pasa de la forma
líquida a la fase gaseosa, las moléculas, que, por Ejemplo 34. Determinar el calor necesario para
causa de sus atracciones naturales se mantenían vaporizar 200 gr. De hielo que se encuentra a la
originalmente en contacto, se alejan más de las otras. temperatura de –5°C.
Esto requiere se realice un trabajo en contra de las Solución.
fuerzas de atracción, es decir hace falta que se Como ocurren cambios de estado debemos calcular
suministre una energía a las moléculas para las calorías requeridas en cada proceso.
separarlas. De este modelo podemos deducir que un Utilicemos los siguientes valores:
cambio de fase de líquido a gas requiere calor aún Calor específico del hielo: 0,5 cal/g°C
cuando no se produzca elevación de la temperatura, Calor específico del agua: 1 cal/g°C
lo mismo sucede para sólido a líquido. Calor de fusión del agua: 80 cal/g
Para sustancias puras", los cambios de fase se Calor de vaporización del agua: 540 cal/g
producen a cualquier presión, pero a determinadas
temperaturas. Se requiere una determinada cantidad Calor para elevar la temperatura del hielo de –5°C a
de calor para cambios de fase de una cantidad de 0°C
sustancia dada. Q1 = m x c x Δ t = m x 0,5 x [0 - (-5)]
Esto es, el calor es proporcional a la masa de la = m x 2,5 cal
sustancia.
Q = mL Calor para pasar de hielo a agua (fusión)
Donde L es una constante característica de la Q2 = m x L = m x 80 cal
sustancia y de cambio de fase que se produce.
Calor para elevar la temperatura del Agua de 0°C a
Si el cambio es de sólido a líquido, será L f (calor
100°C
latente de fusión) y si el cambio el de líquido a gas, Q3 = m x c x Δ t = m x 1 x (100-0)
será Lv (calor latente de vaporización). = m x 100 cal
En el caso del agua a presión atmosférica la fusión
Calor para pasar de Agua a Vapor (vaporización)
se produce a 0°C y L f vale 79,7 cal/gr. Y la Q4 = m x 540 cal
vaporización se produce a 100°C y Lv vale 539.2
Finalmente,
cal/gr.
Similarmente ocurre para los procesos inversos de Q= ∑Q = Q 2 + Q2 + Q3 + Q4
solidificación y condensación. = m(2,5+80+100+540) = 200 x722,5
= 144500 cal.

17
Calor y Termodinámica Hugo Medina Guzmán

Ejemplo 35. Calcular la temperatura final cuando se Calor necesario para convertir el Hielo en Agua a
mezclan 2 kg. de hielo a -20°C con 10 kg. de agua a °C.
60°C. Q1 = mH x cH x Δ t = 0, 55 x 16 = 4,4 kcal
Solución. Q 2 = mH x L = 0,5 x 80 = 40,0 kcal
Como ocurren cambios de estados es preciso
primero, hacer un balance de energía para QH = ∑ Q = Q1 + Q 2 = 44,4 kcal (1)
determinar si el agua se convierte en hielo o el hielo Calor liberado para llevar el Agua a °C (incluyendo
en agua, u ocurre una conversión parcial. el recipiente)
Trabajemos en Kilocalorías utilizando los siguientes
Q’1 = ma x ca x Δ t = 1 x 1 x 20 = 20,0 kcal
valores:
Calor específico del hielo : 0,55 kcal/kg °C Q’2 = mc x cc x Δ t = 0,5 x 0,09 x 20 = 0,9 kcal
Calor específico del agua : 1
Calor de fusión del agua : 80
kcal/kg °C
kcal/kg
Qac = ∑ Q´ = Q’1 + Q’2 = 20,9 kcal (2)
Comparando (1) y (2), como Qac < Q H , nos indica
Calor necesario para convertir el hielo en que no se dispone de las calorías necesarias para
agua a 0 °C. convertir el hielo en agua a °C. Pero, como Qac >
Q1 = mH x cHx Δ t = 2 x 20 = 22 kcal Q 1 si se elevara la temperatura del hielo a 0°C y
Q2 = mH x L = 2 x 80 = 160 kcal solo parte del hielo se podrá convertir en agua.
QH = ∑Q = Q 1 + Q2 = 182 kcal (1) Luego, la temperatura final es 0°C, t = 0°C
¿Cuáles serán las masas finales de hielo y Agua?
La energía que resta después de elevar la
Calor liberado al llevar el agua de 60°C a 0ºC.
temperatura del hielo a 0°C es:
Q 1 = max cH Δ t =
´
10 x 1 x 60 = 600 kcal Qac - Q1 = 20,9 – 4,4 = 16,5 kcal.
Qa = ∑ Q' = Q' = 600 kcal
1 (2) Con estas calorías se convertirá en agua:
Q = MxL
⇒ 16,5 = M x 80 ⇒ M = 0,21 Kg.
Comparando (1) y (2), como Qa > QH, nos indica que y se quedarán como hielo a 0°C:
el agua dispone de las calorías necesarias para (0,50 – 0,21) = 0,29 kg.
convertir todo el hielo en agua y más aún elevar su Por lo tanto, se tendrá finalmente,
temperatura a más de 0°C. Esto es, la temperatura 1,21 kg. de Agua y 0,29 kg. de Hielo
final t estará entre, 0°C < t < 60°C y se determinará Por supuesto todo a 0°C, incluyendo el calorímetro.
igualando el calor ganado al calor perdido.
Ejemplo 37. Un trozo de hielo de 10 g y
Calor ganado temperatura –10 ºC se introducen en 1,5 kg de agua
Q1 = 22 (valor ya determinado) a 75 ºC. Determine la temperatura final de la mezcla.
Q2= 160 (valor ya determinado)
chielo = 0,45 cal g º C ,
Q3 = m c Δ t = 2 x 1 x (t-0) = 2t
QG = Q1 + Q2 + Q3 L fusión, hielo = 80 cal g
= 22 + 160 + 2t = 182 + 2t (3) Solución.
El calor cedido por el agua es igual al ganado por el
Calor perdido hielo. El hielo gana una porción calor desde la
QP = m c Δ t temperatura –10 ºC hasta 0 ºC, otra para cambiar de
= 10 x 1x (60-t) = 10(60 - t) (4) estado manteniendo la temperatura constante de 0 ºC
y otra cuando se ha convertido en agua al cambiar la
Finalmente, igualando (3) y (4) temperatura de 0 ºC hasta la temperatura de
QG = QP equilibrio Te . De este modo:
182 + 2t = 10(60 - t)
Despejando t, se obtiene la temperatura final de la mh ch [0 − (−10)] + mh L f
mezcla (agua)
T = 34,8°C + mh ca (Te − 0) + ma ca (Te − 75) = 0 .
Despejando Te encontramos:
Ejemplo 36. Determinar la temperatura final cuando
se mezclan 1/2 kg de hielo a -16°C con 1 kg de agua T e= 73,94º C
a 20°C que se encuentra contenida en un recipiente o
calorímetro de cobre de 1/2 kg. Ejemplo 38. Un recipiente de cobre de masa 0.5 kg
Solución. contiene 1 kg de agua a 20°C se le añade 0,5 kg de
Como en el ejemplo anterior es necesario hacer un hielo a –16°C
balance de energía. a) encontrar la temperatura de equilibrio
Nuevamente trabajando en kilocalorías y con b) Cuanto hielo y cuanta agua quedan.
Calor específico del cobre = 0,09 kcal/kg °C

18
Calor y Termodinámica Hugo Medina Guzmán

J J De aquí se concluye que no puede condensarse todo


ccobre = 390 , cagua = 4190 el vapor, pero sí fundirse todo el Hielo. De modo
kg K kg K que la temperatura final, en presencia de vapor debe
J 3 J ser tF = 100 ºC: Supongamos entonces que condensa
chielo = 2100 , L fusión hielo = 334 x10 m gramos de vapor
kg K kg Qcedido = 542,4 x m cal
Solución. Qabsorbido = 20 x 80 + 20 x 1 x 100 = 3600 cal
Calor cedido por el agua y el calorímetro al llevarlo
3600
de 20ºC a 0ºC 542,4 x m = 3600 ⇒ m= = 6,6 g
Q1 = (mc c c + ma c a )Δθ 542,4
Luego el estado final consiste en una mezcla a 100
= (0,5 × 390 + 1,0 × 4190 )20 = 87700 J ºC de 4,4 g de vapor y 26,6 g de agua líquida.
Calor para llevar el hielo -18ªC a 0ºC
Q2 = mh c h Δθ = 0,5x2100x16 = 16800 J Ejemplo 41. Un recipiente de cobre de 0,1 kg
contiene 0,16 kg de agua y 0,018 kg de hielo en
Calor para fundir el hielo
equilibrio térmico a presión atmosférica. Si se
Q3 = L f mh = 334x103x0,5 = 167x103 J introduce un trozo de plomo de 0,75 kg de masa a
Análisis: 255°C, ¿qué temperatura final de equilibrio se
Tenemos 87700 J , esa cantidad puede elevar la alcanza? (Considere que no hay intercambio de calor
temperatura del hielo hasta los 0ºC con el entorno)
Nos quedan 87700 -16800 = 70900 J cPb = 130 J kgK
Esto no puede fundir todo el hielo, solamente
cCu = 390 J kgK
70,900 × 103 J
alcanza para fundir = 0,212 kg cagua = 4190 J kgK
334 × 103 J kg
a) Temperatura de equilibrio 0ºC c fusión agua = 334 × 10 3 J kg
b) Finalmente quedan 1 + 0,212 = 1,212 kg de agua Solución.
y 0,5 – 0,212 = 0,288 kg de hielo

Ejemplo 39. Un recipiente metálico de masa 200 g,


aislado del exterior, contiene 100 g de agua en
equilibrio térmico a 22º C. Un cubo de hielo de 10 g,
en el punto de fusión, se suelta en el agua, cuando se
alcanza el equilibrio térmico la temperatura es 15º C.
Asumir que no hay intercambio de calor con el ⎧mcu = 0,1kg ⎧magua = 0,16kg
exterior. Cobre ⎨ , Agua ⎨ ,
Para el agua el calor específico es 4190 J/kg K y el ⎩t cu = 0º C ⎩t cu = 0º C
calor de fusión es 3,34 x 105 J/kg. ⎧mhielo = 0,018kg
¿Cuál es el calor específico del metal? Hielo ⎨
Solución. ⎩t cu = 0º C
Calor cedido = Calor ganado
c x (0,2 )(22 − 15) + 4190(0,1)(22 − 15) ⎧m Pb = 0,75kg
Plomo ⎨
(
= 0,01 3,34 × 10
5
) + 4190(0,01)(15 − 0) ⎩t Pb = 255º C
Para fundir el hielo = 334x103 (0,018) = 6012 J
J magua = 0,16 + 0,018 = 0,178 kg
⇒ cx = 739,64
kg K El plomo puesto a 0ºC nos proporciona = 130
(0,75)(255) = 24862,5 J
Ejemplo 40. Determine el estado final cuando se Nos quedarían 24862,5 – 6012 = 18850,5 J
mezclan 20 g de hielo a 0 ºC con 10 g de vapor a Los que se emplearía para elevar la temperatura del
100 ºC. sistema:
Solución. (mc + mc + mc )Δt = Qdisponible
Cagua = 1 cal/g. ºC
Lf = 3,33 x 105 J/kg = 80 cal/g
(0,178 × 4190 + 0,1 × 390 + 0,75 × 130)Δt = 18850,5
Lv = 2,26 x 106 J/kg = 542,4 cal/g
Mhielo = 20 g 18850,5
Δt =
Mvapor = 10 g (745,82 + 39 + 97,5)
Si se condensa todo el vapor cede 5424 cal.
Si se funde todo el Hielo absorbe 80x20 = 1600 cal 18850,5
= = 21,36º C
quedando agua que para ser llevada a 100 ºC 882,32
absorbería a lo más 20 x 100 = 2000 cal.

19
Calor y Termodinámica Hugo Medina Guzmán

Solución.
La temperatura final de equilibrio del sistema es t.
Calor cedido por el aluminio = Calor ganado por el
cobre
maluminio × c aluminio (100 − t ) = mcobre × ccobre (t − 0 )
Poniendo valores
maluminio × 0,212(100 − t ) = 21,6 × 0,094t
Diámetro final de la esfera de aluminio = diámetro
Ejemplo 42. Un trozo de hierro se deja caer en agua
final del anillo de cobre
tal como se muestra en la figura. Determine la
temperatura y fase del agua en el equilibrio. En caso Dalu min io [1 − α alu min io (100 − t )]
de coexistir 2 fases del agua determine la masa final = Dcobre [1 + α cobre (t − 0 )]
en cada fase.
chierro = 0,107 cal/g ºC, crecipiente ≈ 0 Poniendo valores
[
2,5433 1 − 24 × 10 −6 (100 − t ) ]
[
= 2,54 1 + 17 × 10 t
−6
]
2,5433
=
1 + 17 × 10 t[ −6
]

2,54 [
1 − 24 × 10 −6 (100 − t ) ]
El primer término por el binomio de Newton se
puede escribir como:
Solución.
Agua de 4ºC a 0ºC ⇒ 2,5433 2,54 0,0033
= + = 1 + 2,1×10 −3
Q1 = 200 × 1 × 4 = 800 calorías 2,54 2,54 2,54
Hierro de – 15ºC a 0ºC ⇒ El segundo término por el binomio de Newton se
puede escribir como:
Q2 = 600 × 0,107 × 15 = 963 calorías
En el balance 963 – 800 = 163 calorías, las que
[1 + 17 ×10 t ][1 + 24 ×10 (100 − t )]
−6 −6

convertirán en hielo a una parte del agua [1 + 17 ×10 t ][1 + 24 ×10 (100 − t )]
−6 −6

163 = 1 + 2,4 × 10 − 7 × 10 t
-3 −6
m= = 2,04 gramos
80 Luego:
La temperatura de equilibrio es 0ºC, 2,04 gramos de
hielo y 197,6 gramos de agua.
1 + 2,1× 10-3 = 1 + 2,4 × 10-3 − 7 × 10 −6 t
Resolviendo t:
Ejemplo 43. Dilatación térmica y equilibrio 0,3 × 10−3
térmico. t= −6
= 42,2o C
Un anillo de cobre de 21,6 g tiene un diámetro de 7 × 10
2,54000 cm a la temperatura de 0oC. Una esfera de Finalmente la masa de la esfera de aluminio será
aluminio tiene un diámetro de 2,54533 cm a la 21,6 × 0,094t
malu min io = = 7,17 gramos
temperatura de 100oC. La esfera se sitúa sobre el 0,212 × (100 − 42,8)
anillo, y se deja que ambos lleguen al equilibrio Es una esfera hueca.
térmico, sin que se disipe calor alguno al entorno. La
esfera pasa justamente a través del anillo a la TRANSFERENCIA DE CALOR
temperatura de equilibrio. Halle la masa de la esfera. En este capítulo veremos en forma breve las formas
en la cual la energía térmica fluye de u punto a otro
Calor específico del aluminio: 0,212 cal/gºC en un medio dado, existen tres modos de
Calor específico del cobre: 0,094 cal/gºC transferencia, conducción, convección y radiación.
Coeficiente de dilatación del aluminio: 24 x 10-6 °C-1
Coeficiente de dilatación del cobre: 17 x 10-6 °C-1 CONDUCCIÓN.
Cuando hay transporte de energía entre elementos de
volumen adyacentes en virtud a la diferencia de
temperatura entre ellas, se conoce como conducción
de calor.

20
Calor y Termodinámica Hugo Medina Guzmán

La expresión matemática fundamental de la •


conducción de calor es la generalización de los

(t -t ) l Q
Q = - k1 A 1 0 ⇒ t0 - t1 = 1
resultados de los experimentos en el flujo lineal de l1 k1 A
calor a través de una lámina de material de espesor
En la segunda capa
Δx y de área A, una de las caras se mantienen a •
temperatura θ + Δθ, los resultado muestran que Q es •
(t -t ) l Q
proporcional al tiempo Δt. Q = - k 2 A 2 1 ⇒ t1 - t2 = 2
l2 k2 A
Δθ
Q∝ A Δt En la Capa n
Δx •
Este resultado podemos generalizar, en el límite: •
(t -t ) l Q
dQ • dθ Q = - k n A n n −1 ⇒ tn −1 - tn = n
= Q = −kA ln kn A
dt dx Sumando miembro a miembro
Donde k es la CONDUCTIVIDAD TERMICA del •

material. l l l Q
to − tn = ( 1 + 2 + ..... n )
El signo menos se introduce dado que Q fluye en la k1 k2 kn A
dirección de la disminución de la temperatura (del
Luego
lado caliente al lado frío). • A(to - tn )
Q=
VALORES DE LA l1 l 2 l
CONDUCTIVIDAD TERMICA
+ + ... + n
k1 k 2 kn
Sustancias kilocal
k en •
A(t − t )
s m °C Q= n o n
⎛ li ⎞
Acero
Bronce
0,011
0,026
∑ ⎜⎜ ⎟⎟
i = 1 ⎝ ki ⎠
Aluminio 0,040
Ladrillo 1,7 x 10
−4
Ejemplo 45. Flujo estacionario a través de una
Concreto 4,1 x 10
−4 pared compuesta. Capas en “paralelo”

Madera 0,3 x 10
−4
Determinación de la cantidad de calor Q que fluye
Vidrio 1,4 x 10
−4
en la dirección normal a un medio múltiple formado
Hielo 5,3 x 10
−4 por placas paralelas como se muestra en la figura.
Lana de vidrio o 0,09 x 10
−4
mineral
Caucho 0,10 x 10
−4

Agua 1,43 x 10
−4

Aire 0,056 x 10
−4

Ejemplo 44. Flujo estacionario a través de una


pared compuesta. Capas en “serie”
Determinación de la cantidad de calor que fluye en Solución.
la dirección normal a través de un medio de capas • • •
múltiples entre las temperaturas externas t0 y tn El Flujo Q es la suma de los flujos Q1 , Q 2 ,
constantes, como se muestra en la figura. •
….. Q n a través de cada una de las placas, de tal
modo

Q=−
(k1 A1 + k2 A2 + ...kn An )(tb − ta )
l
n


(tb − ta )∑ ki Ai
Solución. Q=− i =1

Sea t1 la temperatura entre la capa 1 y 2, t2 la


l
temperatura entre las capas 2 y 3 y así
Ejemplo 46. Dos cuartos comparten una pared de
sucesivamente, luego tenemos:
ladrillos de 12 cm de grosor, pero están
En la primera capa
perfectamente aislados en las demás paredes. Cada
cuarto es un cubo de 4,0 m de arista. Si el aire de

21
Calor y Termodinámica Hugo Medina Guzmán

uno de los cuartos está a 10 ºC y el otro a 30 ºC. 9,672θ = 423,6


¿Cuántos focos de 100 W se necesitarán tener
encendidas en el cuarto más caliente para mantener
θ = 43,79º C
la misma diferencia de temperatura? El flujo es;

Solución.
Coeficiente de conductividad térmica del ladrillo Q = 5,436θ = 5,436 x 43,79 = 238,1 W
k = 1,0 W/(m K).

Q = − kA
Δθ
= (1)(4,0 × 4,0 )
(30 − 10) Ejemplo 48.- Un excursionista usa prendas de vestir
de 3,5 cm de grueso, cuya área superficial total es de
L 0,12 1,7 m2. La temperatura de la superficie de las
20 prendas es de –20 ºC y la de la piel de 34 ºC.
= (1)(4,0 × 4,0 ) = 2666,67 W Calcular el flujo de calor por conducción a través de
0,12 la ropa
Número de focos de 100 W que se necesitarán tener a) Suponiendo que ésta está seca y que la
encendidos en el cuarto más caliente para mantener conductividad térmica k es la del plumón igual a
la misma diferencia de temperatura 0,06x10-4 kcal/s m K
2666,67 b) Suponiendo que la ropa está mojada, de modo que
= 26,7 k es la del agua (1,4x10-4 kcal/s m K) y que la ropa
100 se ha comprimido hasta un espesor de 0,50 cm.
Se necesitan 27 focos de 100 W. Solución.
Ejemplo 47. Dos barras metálicas, cada una de

a) Q = − kA
Δθ
= 0,06 × 10 − 4 (1,7 )
(34 + 20)
longitud 5 cm y sección transversal rectangular de L 3,5 × 10 − 2
lados 2 y 3 cm, están encajadas entre dos paredes = 0,01,5737 W
una a 100 ºC y otra a 0 ºC. Las barras son de Pb y
Ag. Determinar:

b) Q = − kA
Δθ
= 1,4 × 10 − 4 (1,7 )
(34 + 20)
a) El flujo térmico total a través de las barras y L 0,50 × 10 − 2
b) La temperatura en la interfase. = 2,5704 W
DATOS: k(Pb) = 353 W/m K; k(Ag) = 430 W/m K.
Ejemplo 49. Flujo a través de un cilindro de radio
interior r1 y radio exterior r2, conductividad térmica
k, temperatura interior t1 y temperatura exterior t2.
Solución.
Tomemos una longitud L, y a una distancia r un
elemento diferencial dr como se muestra en la
Solución. figura,

Pb
A = 6 x 10-4 m, L = 5x10-2 m k = 353 W/m K;
El flujo a través del elemento diferencial es
Ag
A = 6 x10-4 m, L = 5x10-2 m k = 453 W/m K;

dt
Q = − kA
Flujo de calor en el plomo dr
• ⎛ 6 × 10−4 ⎞ •
Q = 353⎜⎜ ⎟(100 − θ )
−2 ⎟
Q es constante a través de cualquier sección
⎝ 5 × 10 ⎠ cilíndrica coaxial.
= 4,236(100 − θ ) A = 2 π rL
Flujo de calor en la plata. Luego
• dt
• ⎛ 6 × 10 −4 ⎞ Q = −k 2πrL
Q = 453⎜⎜ −2 ⎟
⎟(θ − 0 ) dr
⎝ 5 × 10 ⎠ Despejando dt
= 5,436θ •
Q dr
Igualando los flujos dt = −
4,236(100 − θ ) = 5,436θ 2πkL r
Integrando
423,6 − 4,236θ = 5,436θ

22
Calor y Termodinámica Hugo Medina Guzmán

• De la igualación de (2) y (3) tenemos:


t2 Q r2 dr
∫t1
dt = −
2πkL ∫r1 r
3
T1 + 3
• T2 = 2 . (5)
Q r 52
t1 − t2 = − ln 2
2πkL r1 Por otro lado, de la diferencia de las ecuaciones (4) y
(5), hallamos:
De aquí
T1 = 13,63º C y T2 = 13,63º C.

2πkL
Q= (t − t )
r 1 2
Reemplazando en ecuación (1):
ln 2 ΔQ
= kV A
(20 − T1 ) = 4,25 cal
r1
Δt 0,006 s
b) Si la ventana está formada por un solo vidrio:
Ejemplo 50. Una ventana de un metro de alto por 2
de ancho tiene un vidrio cuyo espesor es de 0,006 m,
conduce calor desde el interior a 20 ºC al exterior de
3 ºC. Encuentre la diferencia porcentual de la
conducción del calor, cuando se pone dos vidrios del
mismo espesor anterior, dejando una separación de
aire entre los vidrios de 0,012 m. Considere que:
kVidrio = kV = 2 × 10−6 kcal/smº C ,
k Aire = k A = 6 × 10−6 kcal/smº C .
Solución. ΔQ'
= kV A
(30 − 3) = 11,3 cal ,
a) Al poner los dos vidrios: Δt ΔX s
Es decir, la diferencia con respecto a
ΔQ Δt = 7,05 cal/s. De este modo hay una
diferencia de un 62,4%, con lo cuál, cuándo se
coloca aire entre los dos vidrios se pierde un 62,4%
menos de energía calórico que cuándo se usa un solo
vidrio.

Ejemplo 51. Una ventana de un metro de alto por


dos de ancho, está construida con láminas de vidrio
cuyo espesor es de 0,006 m. La ventana puede ser
ensamblada con un solo vidrio en ese caso el flujo
de calor es Q• 1 o puede construirse con dos vidrios
Sean T1 y T2 las temperaturas a la derecha del vidrio
dejando una separación de 0,012 m de aire
izquierdo e izquierda del vidrio derecho,
confinado entre las dos láminas de vidrio, en este
respectivamente:
caso el flujo de calor es Q• 2 . Encontrar la relación
ΔQ1 (20 − T1 )
= kV A , (1) entre los flujos de calor.
Δt 0,006 kvidrio = 2 × 10−6 kcal / s m°C ,
ΔQ 2 (T − T2 ) kaire confinado = 6 × 10−6 kcal / s m°C
= kAA 1 , (2)
Δt 0,012
ΔQ3 (T − 3)
= kV A 2 . (3)
Δt 0,006
En el estado de régimen estable, es decir, cuándo la
temperatura en cada punto es constante en el
transcurso del tiempo, por lo cuál ΔQ Δt es la
misma en todas las secciones transversales:
ΔQ ΔQ1 ΔQ 2 ΔQ3 Solución.
= = = . Al poner los dos vidrios:
Δt Δt Δt Δt
Igualando ecuaciones (1) y (2), encontramos:
• A
Q1 = − Δθ
⎛ 2 ⎞ 40 ⎛ L1 L2 ⎞
T2 = T1 ⎜1 + ⎟ − . (4) ⎜⎜ 2 + ⎟⎟
⎝ 3⎠ 3 ⎝ k1 k 2 ⎠

23
Calor y Termodinámica Hugo Medina Guzmán

Al poner un solo vidrio x y R


• A Por semejanza de triángulos: = ⇒x= y
Q2 = − Δθ R L L
⎛ L1 ⎞ 2
⎜⎜ ⎟⎟ •
⎛ R ⎞ dT
Luego: Q = − kπ ⎜ R + y⎟
⎝ k1 ⎠ ⎝ L ⎠ dy
La relación entre los flujos de calor es:
dy kπR 2
A ⇒ = dT
− Δθ ( y + L )2 Q• L2
⎛ L1 ⎞
• ⎜⎜ ⎟⎟
Q2 ⎝ k1 ⎠ L dy kπR 2 T2


=
A
Integrando ∫
0
=
( y + L )2 Q• L2 ∫
T1
dT
Q1 − Δθ
⎛ L1 L2 ⎞ L
⎜⎜ 2 + ⎟⎟ 1 kπR 2 T2
⎝ k1 k 2 ⎠ ⇒− =− • T
( y + L) 0 QL2
T1

⎛ L1 L2 ⎞
⎜2 +
• ⎟⎟ kπR 2
Q2 ⎜⎝ k1 k 2
1 1
⎠ = 2 + L 2 k1 ⇒ − + = • (T − T )

=
⎛ L1 ⎞ L1 k 2
(L + L ) (0 + L ) Q L2 1 2
Q1 ⎜⎜ ⎟⎟
⎝ k1 ⎠ 1 kπR 2
⇒ = • (T1 − T2 )
2 L Q L2
⎛ 12 ⎞⎛ 2 × 10 ⎞
−6
= 2+⎜ ⎟⎜⎜ ⎟
−6 ⎟
⎝ 6 ⎠⎝ 6 × 10 ⎠ • 2kπR 2
Finalmente: Q = (T1 − T2 )
2 8 L
= 2 + = = 2,66
3 3
CONVECCION.
Ejemplo 52. El sólido de la figura tiene bases Es el proceso de transferencia de calor de un lugar a
circulares de radio R y 2R, altura L y conductividad otro por el movimiento de la masa calentada.
térmica k. Si las bases se ponen en contacto con
reservorios de temperatura T1 y T2 .Determine la
corriente calorífica cuando el flujo es estacionario.
Considere las paredes laterales forradas con un
aislante térmico.

Las leyes que rigen el flujo de calor por convección


son muy complejas porque involucra fenómenos de
fluidos en movimiento y el cual todavía puede ser
forzado o natural por diferencia de densidades. Sin
Solución. embargo, se tiene una relación empírica dada por
Newton, para un cuerpo dado:
dQ •
= Q = hAΔθ
dt
Donde h es el coeficiente de convección, A es el
área de la pared, Δθ es la diferencia de temperatura
entre la superficie de la pared y el fluido.
El flujo a través de la porción de ancho dy y área

A = πr 2 = π (R + x ) , es también igual a Q
2 EL COEFICIENTE DE CONVECCION h depende
de la posición de la pared y de las características del
• dT 2 dT fluido y su movimiento.
Q = −kA = − kπ (R + x )
dy dy
COEFICIENTE DE CONVECCION EN
AIRE A PRESION ATMOSFERICA

24
Calor y Termodinámica Hugo Medina Guzmán

( ) kcal

kcal
DISPOSICION h( ) Q = 2,71 × 10 −3 (1)(80) = 0,217
s m 2 °C s
0,576x10-3 (Δt )
Pared horizontal Mirando 14 y el calor que se pierde en una hora será:
arriba Q = 0,217 x 3600 = 782 cal
Pared horizontal Mirando
abajo 0,314x10-3 (Δt )1 4
Ejemplo 54. El aire sobre la superficie de un lago
Pared vertical
0,424x10-3 (Δt )1 4 está a una temperatura θ A mientras que el agua está
Tubo horizontal o vertical
⎛ Δt ⎞
14
en su punto de congelación θ c (θ A < θ C ).
1,00x10 -3
⎜ ⎟ ¿Cuál es el tiempo T que ha de transcurrir para que
⎝D⎠
se forme una capa de hielo de espesor y
Asumir que el calor liberado cuando el agua se
Ejemplo 53. Una pared plana se mantiene a
congela fluye a través del hielo por conducción y de
temperatura constante de 100°C, y el aire sobre
la superficie al aire por convección natural.
ambas cara está a la presión atmosférica y a 20°C.
DATOS:
¿Cuánto calor se pierde por convección de un metro
h = coeficiente de convección del hielo
cuadrado de superficie en ambas caras en 1 hora?
a) Si la pared es vertical
ρ = densidad del hielo
b) Si la pared e horizontal L = calor de fusión del hielo
Solución. k = conductividad térmica del hielo
a) Si la pared es vertical. Solución.
El flujo de calor de ambas caras es

Q = −2hAΔt
Donde
kcal
h = 0,42 × 10 −3 (Δt )
14

s m2 º C
Δt = 80 y (Δt )
14
= 2,98
2
A=1m En la figura observamos como se va formando la
de aquí capa de hielo
h = 0,42 × 10−3 × 2,98 Calor de solidificación de la capa de hielo en
formación de área A y espesor dy.
−3 kcal dQ = dmL = ρAdyL
= 1,12 × 10 (1)
s m2 º C Éste calor se conduce a la superficie
• • dQ (θ − θ S )
Q = 2 × 1,12 × 10− 3 × 80 Q= = − kA C
dt y
kcal
= 0,179 (θ − θ C )
s dQ = kA S dt (2)
EL calor que se pierde en una hora será y
Q = 0,179 x 3600 = 645 kcal Igualando calores (1) y (2)
b) Si la pared es horizontal. (θ S − θ C )
En este caso tenemos los valores para h: ρAdyL = kA dt
Para la cara que mira arriba
y
k
h1 = 0,596 x 10 −3 (Δt )1/4 (θ S − θ C )∫0 dt
Y T
∫0
ydy =
ρL
−3 kcal
= 1,77 x 10
s m 2 °C Y2 k
= (θ S − θ C )T
Para la cara que mira abajo 2 ρL
h2 = 0,314 x 10 −3 (Δt )1/4 Y 2 ρL
= (θ S − θ C )T (3)
2k
−3 kcal
= 0,94 x 10
s m 2 °C El flujo de calor de la superficie al medio ambiente

se produce por convección, o sea
Luego: Q = - h1 A Δt − h2 A Δt
• dQ
Q= = − hA(θ S − θ A )
• dt
⇒ Q = − (h1 + h2 ) A Δt

25
Calor y Termodinámica Hugo Medina Guzmán

dQ = hA(θ A − θ S )dt c) Para encontrar el grosor de su vestido de la


persona en Puno para que tenga la misma pérdida
de calor que una persona en Lima, aplicamos la
Este es el mismo calor y por lo tanto
misma ecuación.
ρAdyL = hA(θ A − θ S )dt 1,5(37 + 20)
h 9,85 = ⇒
dy = (θ A − θ S )dt 0,03
+
e
+
1
ρL 0,01 0,0209 9
Integrando
⎡1,5(57 ) 0,03 1 ⎤
h e = 0,0209⎢ − −
∫0 dy = ρL (θ A − θ S )∫0 dt
Y T = 0,116 m
⎣ 9.85 0,01 9 ⎥⎦
h
Y= (θ A − θ S )T Ejemplo 56. Se construye un iglú en forma de
ρL hemisferio con un radio interno de 1,8 m y paredes
de nieve compactada de 0,5 m de espesor. En el
YρL
= (θ A − θ S )T (4) interior del iglú el coeficiente de transferencia de
h calor por convección es 6 W/m2·K; en el exterior, en
Sumando las expresiones (3) y (4) obtenemos condiciones normales de viento, es 15 W/m2K. La
conductividad térmica de la nieve compactada es
⎛Y 2 Y ⎞
⎜⎜ + ⎟⎟ ρL = (θ A − θ C )T 2,33 W/m K. La temperatura de la capa de hielo
⎝ 2k h ⎠ sobre la que se asienta el iglú es de -20 ºC y tiene la
Finalmente, misma conductividad térmica que la nieve
compactada.
ρL ⎛ Y 2 Y ⎞ a) Que calor debe proporcionar una fuente continua
T= ⎜ + ⎟
(θ A − θ C ) ⎜⎝ 2k h ⎟⎠ dentro del iglú, para que la temperatura del aire
interior sea 1º C cuando la del aire exterior es - 40
ºC. Considere las pérdidas de calor a través del
Ejemplo 55. El interior del ser humano se encuentra suelo.
a 37°C, el espesor efectivo de la piel puede b) ¿Cómo afecta el duplicar el espesor de las
considerarse como de 3cm. paredes?
a) Para una persona cubierta de pies a cabeza por un
vestido de lana de 0,5cm de espesor. Calcular el
flujo de calor que pierde en Lima (tamb = 15°C) y en
las madrugadas de Puno (tamb = -20°C).
b) ¿Cuál debería ser el grosor de su vestido de la
persona en Puno para tener la misma pérdida de
calor que una persona en Lima? Solución.
Datos: a)
kpiel = 0,01W/m°C
Área del cuerpo humano persona promedio = 1,5m2
klana = 0,0209 W/ºC
h (del cuerpo vestido) = 9 W/m2·K,
Solución.
a) El flujo de calor atraviesa la piel y el vestido por
conducción y de la superficie del vestido al ambiente
por convección.
Este flujo a través de este conjunto es:
• A(t piel − t ambiente ) Pérdida por convección en el piso
Q=
Q 2 = − hi A p (θ p − θ i ) , A p = πR12

Lpiel Llana 1
+ +
k piel k lana h
( )
Q 2 = − hi πR12 (θ p − θ i )

• 1,5(37 − 15)
En Lima: Q =
= −[6](π 1,8 )(− 20 − 1) = 1388,02W
= 9,85 W •
0,03 0,005 1 Q2 2
+ +
0,01 0,0209 9
• 1,5(37 + 20) Pérdida de calor por el domo
En Puno: Q = = 23,74 W
0,03 0,05 1 Por convección del aire interior a la pared interior
+ + •
0,01 0,0209 9
Q 1 = −hi A1 (θ1 − θ i )

26
Calor y Termodinámica Hugo Medina Guzmán

1 •
A1 = 4πR12 = 0,018 Q1
2 • 39
( )(θ − θ ) ⇒ Q1 = = 2166,67 W

Q 1 = −hi 2πR 1
2
1 i 0,029
= −6(2π 1,8 )(θ − 1) = − 122,08(θ

Q1 2
1 1 − 1) Salida total de calor
• 1388,02 + 2166.67 = 3554,69 W
Q1
⇒ (θ 1 − 1) = (1)
La fuente debe proporcionar 3,554 kW
122,08
b) Si se duplica el espesor de la pared del domo
Por conducción en la pared del iglú: •
1 ⎛ ⎞
A = 4π r 2 (θ1 − θ 2 ) = Q1 ⎜⎜ 1 − 1 ⎟⎟
2 2πk ⎝ R1 R2 ⎠


• dθ Q dr Q1 ⎛ 1 1 ⎞
Q 1 = −k 2π r 2 ⇒ dθ = − 1 2 ⇒ (θ1 − θ 2 ) = ⎜ − ⎟
dr 2πk r 2π (2,33)⎝ 1,8 2,8 ⎠

θ2 Q R2 dr •
⇒ ∫θ
1
dθ = −
2πk ∫R1 r 2 ⇒ (θ 1 − θ 2 ) =
Q1
(2a)
• 31,65
Q1 ⎛ 1 1⎞
⇒ θ 2 − θ1 = ⎜⎜ − ⎟⎟ Sumando (1), (2a) y (3):
2πk ⎝ R2 R1 ⎠
• • •
Q
(40 − 1) = 1 + Q1 + Q1

Q1 ⎛ 1 1 ⎞
⇒ (θ1 − θ 2 ) = ⎜⎜ − ⎟⎟ 122,08 31,65 498,32
2πk ⎝ R1 R2 ⎠

• • •

⇒ (θ1 − θ 2 ) =
Q1 ⎛ 1
⎜ −
1 ⎞
⎟ 39 = Q 1 (0,008 + 0,032 + 0,002 ) = 0,042 Q 1
2π (2,33)⎝ 1,8 2,3 ⎠ • 39

⇒ Q1 = = 928, 57 W
Q1 0,042
⇒ (θ 1 − θ 2 ) = (2)
120,93 Salida total de calor 1388,02 + 928,57 = 2316,59 W

Por convección de la pared exterior al aire exterior La fuente debe proporcionar 2,316 kW

Q 1 = −he A2 (θ e − θ 2 ) RADIACION.
1 Es el proceso de transferencia de calor por medio de
A2 = 4πR22 . ondas electromagnéticas durante el cual la masa del
2 medio no interviene puesto que no se refiere a la
( )
• convección, ni a la conducción, por ejemplo la
Q 1 = −he 2πR22 (θ e − θ 2 ) transferencia de energía del sol de la tierra.

( )

⇒ Q1 = −(15) 2π 2,3 2 (− 40 − θ 2 )
(
= (15) 2π 2,3 (θ 2 + 40 )
2
)
= 498,32(θ 2 + 40 )

Q1
⇒ (θ 2 + 40 ) = (3)
498,32 Una sustancia puede ser estimulada a emitir
radiación electromagnética en varias formas, como
Sumando (1), (2) y (3): por ejemplo un conductor eléctrico con corriente
• • • alterna de alta frecuencia emite ondas de radio, una
Q
(40 − 1) = 1 + Q1 + Q1 placa bombardeada por electrones con alta velocidad
emite rayos X, un líquido o sólido caliente emite
122,08 120,93 498,32
• radiación térmica, etc.
⇒ 39 = Q1 (0,008 + 0,008 + 0,002)

27
Calor y Termodinámica Hugo Medina Guzmán

En esta parte trataremos solamente la radiación = 56,8 W


térmica. b) 2,846 kcal/día.
Experimentalmente STEFAN y BOLTZMAN El gasto energético por día es:
encontraron la ley que rige la radiación, mostraron (56,8 + 81) J/s x 3600x24 s/día = 4907520 J
que la radiación emitida, energía por unidad de Como 1 kcal = 4186 J
tiempo y por unidad de área, por un cuerpo negro El gasto energético en kcal/día:
(Sustancia Capaz de absorber toda la energía que 4907520 J/día x 1 kcal /4186 J = 2,846 kcal/día.
llega a él) a una temperatura T (Temperatura
absoluta) θ es R = σT
4 Ejemplo 59. Calcular la pérdida neta de energía
Donde σ es la llamada constante de Boltzman. radiante de una persona desnuda en una habitación a
20 ºC, suponiendo que la persona se comporta como
kcal un cuerpo negro. El área del cuerpo es igual a 1,4 m2
σ = 4,88 x 10 -8
m hora K 4 2
y la temperatura de su superficie es de 33 ºC.
W Solución.
-8
= 5,67 x 10
( )

m K4
2
Q rad = σ eA TC4 − T A4 = (5,67x10-8
El calor transferido por radiación de un cuerpo a una W/m2.K4)(1)( 1,4 m2 )(3064K-2934K) = (5,67x10-8
temperatura T al medio que lo rodea a una W/m2.K4)(1)( 1,4 m2 )(13,98x108K) = 110, 97 W
temperatura T0 , es:
Ejemplo 60. Los cables de calefacción de una estufa
( )

Q = Aeσ T 4 − T04 eléctrica de 1kW se encuentran al rojo a una
temperatura de 900 K. Suponiendo que el 100% del
Donde e es el factor de emisividad del cuerpo a
calor emitido es debido a la radiación y que los
temperatura T , siendo igual a 1 para el cuerpo cables actúan como radiadores ideales. ¿Cuál es el
negro. área efectiva de la superficie radiante? Suponer la
temperatura ambiente de 20 ºC.
Ejemplo 57. La temperatura de trabajo del filamento Solución.
de tungsteno de una lámpara incandescente es 2450
( )

K, y su emisividad es 0,30. ¿Cuál es la superficie del Q rad = σ eA TC4 − T A4
filamento de una lámpara de 25 watts? 1000 = (5,67 x 10 )(1)( A )(11734-2934) ⇒
-8

Solución. 1000 = (5,67 x 10-8)(1)( A )(1885 x 108) ⇒



1000 = 10687,95 A ⇒
• Q
Como Q = AeσT ⇒ A=
4
1000
eσT 4 A= = 0,094 m2
• W 10687,95
Donde: Q = 25 W , σ = 5,67 × 10 −8 ,
m 2 .K Ejemplo 61. a) ¿Cuánta potencia irradia una esfera
e = 0,30 y T = 2450 K de tungsteno (emisividad = 0,35) de 18 cm de radio
Reemplazando valores obtenemos la superficie: a una temperatura de 25 º?
25 b) Si la esfera está encerrada en un recinto cuyas
A= = 0,408 x 10-4 m2 paredes se mantienen a –5 ºC ¿Cuál es el flujo neto
5,67 × 10 −8 (2450)
4
de la energía liberada de la esfera?
= 0,408 cm2 Solución.
a) A = πR = π (0,18) = 0,101736m
2 2 2
Ejemplo 58. Una persona desvestida tiene una

superficie de 1,5 m2 expuesta a un ambiente y a unos Q rad = σ eAT 4
alrededores de 27 ºC. La temperatura de su piel es de
33 ºC y se puede considerar un emisor de radiación = (5,67x10-8)(0,35)(0,10173)(2984)
perfecto. Si el coeficiente de transferencia de calor = 15,92 W
por convección es de 9 W/m2K, hállese: b)

( )

a) Las pérdidas de calor por convección y por
radiación. Q rad = σ eA TC4 − T A4
b) El gasto energético en kcal/día. = (5,67x10 )(0,35)( 0,10173)(2984K-2784)
-8

Solución. = 3,86 W

a) Q conv = − hAΔθ Ejemplo 62. La Tierra recibe aproximadamente 430
= (9)(1,5)(33-27) = 81 W. W/m2 del Sol, promediados sobre toda su superficie,
e irradia una cantidad igual de regreso al espacio (es
( )

Q rad = σ eA TC4 − T A4 decir la Tierra está en equilibrio). Suponiendo
nuestro planeta un emisor perfecto (e = 1,00), estime
= (5,67x10 )(1)( 1,5 )(3064-3004)
-8
su temperatura superficial promedio.
= (5,67x10-8 )(1)( 1,5)(6,68x108)

28
Calor y Termodinámica Hugo Medina Guzmán

Solución. Los gases reales no siguen exactamente este


A = πR = π (0,18) = 0,101736m comportamiento, pero es una buena forma para
2 2 2
comenzar.

Q rad
= σ eT 4 = (5,67x10-8)(1)(T4) = 430 El comportamiento de las masas encerradas de gases
A ideales se determina por las relaciones entre p, V o p,
430 T, o V, T cuando la tercera cantidad T o V o p
T =4 = 295K , t = 22,1ºC respectivamente, es mantenida constante; estas
5,67 × 10 −8 relaciones fueron obtenidas experimental por Boyle,
Gay-Lussac y Charles respectivamente.
Ejemplo 63. a) Encontrar la potencia total radiada
al espacio por el Sol. Suponiendo que éste es un LEY DE BOYLE. La presión (p) de un gas ideal
emisor perfecto con T = 5500 K. El radio del Sol es varía inversamente a su volumen (V) si la
7,0x108 m. temperatura (T) se mantiene constante.
b) A partir del resultado anterior, determinar la
potencia por unidad de área que llega a la Tierra, que
se encuentra a una distancia del Sol de 1,5x1011 m. 1
p∝ con T constante ⇒ pV = Constante
Solución. V
a)
(
A = πR 2 = π 7,0 × 10 8 )
2
= 153,86 × 1016 m 2 p1V1 = p 2V2

Q rad = σ eAT 4 LEY DE GAY-LUSSAC. La presión (p) de un gas
= (5,67x10-8)(1)(153,86x1016)(55004) ideal varía directamente a su temperatura (T) si el
= 79,83x1024 W volumen (V) se mantiene constante.
Potencia 79,83 × 10 24
b) =
Area (
4π 1,5 × 1011
2
) p ∝ T con V constante ⇒
p
T
= Constante
= 282,48 W/m2

DEFINICIÓN DE UN GAS IDEAL. p1 p 2


Los gases juegan un rol muy importante en muchos =
procesos termodinámicos, y antes de ir más allá, es T1 T2
importante considerar una forma ingeniosa de
comprender las propiedades de los gases. Esta idea Nota: Esta ley se deduce con el termómetro de gas a
es llamada la teoría cinética de los gases, trata de volumen constante
explicar las propiedades macroscópicas de un gas
examinando el comportamiento de los átomos y
moléculas que forman un gas. A simple vista esto ⎛ p ⎞ t p
t = 273,15⎜⎜ − 1⎟⎟ o C ⇒ +1 =
parece ser imposible porque el número de átomos 273,15 pC
involucrados es demasiado grande, alrededor de 1027
⎝ pC ⎠
átomos llenan una habitación. Sin embargo t + 273,15 p T p
⇒ = ⇒ =
utilizando la estadística, se puede predecir con 273,15 pC TC pC
mucha precisión las características de un gas. En lo
siguiente asumiremos que estamos trabajando con
un gas ideal con las propiedades siguientes: p1 p 2
o =
Un gas está formado por partículas llamadas T1 T2
moléculas.
Las moléculas se mueven irregularmente y obedecen LEY DE CHARLES. El volumen (V) de un gas
las leyes de Newton del movimiento. ideal varía directamente a su temperatura (T) si la
presión (p) se mantiene constante.
El número total de moléculas es grande.
El volumen de las moléculas mismas es una fracción V
inapreciablemente pequeña del volumen ocupado V ∝ T con p constante ⇒ = Constante
por el gas. T
Entre moléculas no obran fuerzas de consideración,
salvo durante los choques. V1 V2
Los choques son perfectamente elásticos y de =
duración insignificante. T1 T2

29
Calor y Termodinámica Hugo Medina Guzmán

Nota: Esta ley se deduce con el termómetro de gas a El número de moles se define como, el cociente de
presión constante la masa de gas M a su peso molecular (M0)

⎛V ⎞ t V M
t = 273,15⎜⎜ − 1⎟⎟º C ⇒ +1 = n=
⎝ VC ⎠ 273,15 VC M0
t + 273,15 V T V
⇒ = ⇒ = Si es m la masa de cada molécula de un gas y N es
273,15 VC TC VC el número de las moléculas que hacen la masa total
M.
V1 V2
o = N A . = número de Avogadro = número de moléculas
T1 T2
en 1 mol de gas (cualquier gas).

ECUACIÓN DE ESTADO DE UN GAS IDEAL. Entonces M = mN y M0 = mNA.


El comportamiento de gases ideales se caracteriza en
términos de p, V y T. Tal ecuación se llama la
ecuación del gas ideal. El comportamiento de N
Por lo tanto n =
cualquier estado de la materia se puede caracterizar Na
generalmente por una cierta relación entre la presión
(p) y la densidad (ρ) que por supuesto corresponde
al volumen (V). La ecuación de los gases ideales M N
Luego pV = nRT = RT = RT
puede obtenerse por la combinación de dos de las M0 Na
tres leyes de los gases indicadas anteriormente.
Sea el gas encerrado con condiciones iniciales p1 ,
M mN
V1 y T1 , llevado a un estado final p 2 , V2 y T2 Ahora, pV = RT ⇒ pV = RT
como sigue:
M0 mN A
p1V1 p 2V2 R
= ⇒ pV = N T
T1 T2 NA

pV El cociente entre las dos constantes R y N A es la


o = Constante constante que designamos por kB, la constante de
T
Boltzmann.
Nota: Se encontró que el valor de la constante es
dependiente en la masa del gas dado y también se 8,314J / mol K − 23 J
kB = = 1,38 × 10
encontró que no es igual para una unidad de masa de 6,022 × 10 / mol
23
K
diferentes gases. Sin embargo, se encuentra que si
lo es para 1 mol de masa (la masa numéricamente
equivalente en gramos al peso molecular, ejemplo, 2 Ejemplo 64. Un conductor inicia su viaje en una
g para H2, 32 g para el O2, 28 g para el N2, etc.) de mañana fría cuando la temperatura es 4ºC, y mide la
cualquier gas ideal entonces el valor de la constante presión de la llanta y ve que el manómetro lee 32 psi
es igual para todos los gases. Esta constante igual (2,2 x 105 Pa). Después de manejar todo el día, las
para todos los gases es denotada generalmente por llantas se han calentado, y por la tarde la
“R” y llamada la constante universal de los gases. temperatura de las llantas se ha elevado a 50ºC.
Asumiendo que el volumen es constante , ¿a que
presión se habrá elevado el aire en las llantas?
J cal
R = 8,314 = 1,986 1 atm = 1,013 Pa = 14,7 psi
mol K mol K
Solución.
Tomar en cuenta que un manómetro mide la presión
La ecuación del gas ideal por lo tanto se escribe
manométrica (pm = p – pa). Luego la presión inicial
normalmente como
es
p1 = p m + p a ⇒ p1 = 32 + 14,7 = 46,7 psi
pV = nRT
T1 = 4 + 273,15 = 277,15 K y
Donde n = número de moles. T2 = 50 + 273,15 = 323,15 K
p1V1 = nRT1 y p 2V2 = nRT2 , V1 = V2

30
Calor y Termodinámica Hugo Medina Guzmán

Dividiendo estas ecuaciones: p2V2 90 × 103 × 2,5


p1V1 nRT1 T2 = T1 = 400,15
= ⇒ p1V1 30 × 103 × 4
p 2V2 nRT2 = 750,28K = 477,13 ºC
T ⎛ 323,15 ⎞
p 2 = 2 p1 = ⎜ ⎟(46,7 ) = 54,5 psi Ejemplo 67. Se encuentra contenido un gas en una
T1 ⎝ 277,15 ⎠ vasija de 8 L, a una temperatura de
absoluta 20ºC y a una presión de 9 atmósferas:
o 54,5 – 14,7 = 39,8 psi, presión manométrica. a) Determine el número de moles en la vasija.
b) ¿Cuántas moléculas hay en la vasija?
Ejemplo 65. Un gas ideal ocupa un volumen de 100 Solución.
cm3 a 20 ºC y a una presión de 100 p = 9 atm, V = 8 litros, t = 20 ºC, T = 293,15K
Pa. Determine el número de moles de gas en el pV 9×8
recipiente. a) n = =
Solución. RT 0,082 × 293,15
p = 100 Pa = 9,8692 x 10-4 atm = 3,0 mol
V = 100 x 10-6m3 = 0,1 litros b) NA = 6,0221367 x 1023 / mol
t = 20 °C N = n NA = 3 x 6,0221367 x 1023
T = 293,15 K = 1,81 x 1024 moléculas
R = 0,082 litro atm/mol K = 8,31 J/mol K
Se puede hacer el cálculo en los dos sistemas de Ejemplo 68. Se infla la llanta de un automóvil con
unidades usando aire inicialmente a 10 ºC y a presión atmosférica
pV normal. Durante el proceso, el aire se comprime a
n= 28% de su volumen inicial y su temperatura aumenta
RT a 40 ºC. ¿Cuál es la presión del aire?
9,8692 × 10 −4 × 0,1 Después de manejar el automóvil a altas
n= = 4,11 x 10-6 mol velocidades, la temperatura del aire de las ruedas
00,082 × 293,15 aumenta a 85 ºC y el volumen interior de la rueda
100 × 100 × 10−6 aumenta 2 %. ¿Cuál es la nueva presión en la rueda?
n= = 4,11 x 10-6 mol Exprese su respuesta en Pa (absoluta) y en psi
8,31 × 293,15 (lb/pulg2) (manométrica).
(1 atm = 14,70 psi)
Ejemplo 66. Se mantiene un gas ideal en un Solución.
recipiente a volumen constante. Primera parte
Inicialmente, su temperatura es 10ºC y su presión es p1 = 1 atm, V1 = V , t1 = 10 ºC, T1 = 283,15K
2,5 atmósferas ¿Cuál será la presión cuando la V2 = 0,28V, t2 = 40 ºC, T2 = 313,15K
temperatura sea de 80ºC? De pV = nRT como la masa no varía
Solución. p1V1 p 2V2
p1 = 2,5 atm, t1 = 10 ºC, T1 = 283,15K, =
t2 = 80 ºC, T2 = 353,15 K T1 T2
p1V pV VT V × 313,15
n= = 2 ⇒ ⇒ p 2 = p1 1 2 = 1 ×
RT1 RT2 V2T1 0,28V × 283,15
pT = 3,95 atm = 4,0 x 105 Pa
p2 = 1 2 Nota la presión manométrica p ' 2 , es la presión
T1
relativa a la atmosférica, es decir
2,5 × 353,15
= = 3,118 atm p ' 2 = 3,95 – 1 = 2,95 atm
283,15 = 2,95 x 14,7 = 43,365 psi

Ejemplo 64. Un cilindro con un émbolo móvil Segunda parte


contiene un gas a una temperatura de t2 = 85 ºC, T2 = 358,15 K, V2 = 1,02 x 0,28V
127 ºC, una presión de 30 kPa y un volumen de 4 m3 V1T2 V × 358,15
¿Cuál será su temperatura final si el gas se p 2 = p1 = 1×
comprime a 2,5 m3 la presión aumenta a 90 kPa? V2T1 1,02 × 0,28V × 283,15
Solución. = 4,43 atm = 4,42884 x 105 Pa
p1 = 30 x 103 Pa, V1 = 4m3, t1 = 127 ºC, y la manométrica será
T1 = 400,15K p '2 = 4,43 -1 = 3,43 atm = 3,43 x 14,7 = 50,42 psi
p2 = 90 x 103 Pa, V2 = 2,5m3
p1V1 p 2V2 Ejemplo 69. Una caja cúbica metálica de 20 cm de
De n = =
RT1 RT2 lado, contiene aire a la presión de 1 atm y a 300 K

31
Calor y Termodinámica Hugo Medina Guzmán

de temperatura. Se cierra herméticamente, de forma p1 = pa + ρgh


que el volumen sea constante y se calienta hasta 400
K. Hallar la fuerza neta desarrollada sobre cada t1 = 5 ºC, T1 = 278,15 K
pared de la caja. 4
V1 = πr13
Solución. 3
p1 p 2 p p p2 = pa
= ⇒ a = 2 ⇒
T1 T2 300 400 t2 = 12 ºC, T2 = 285,15 K
4
p2 =
400
300
4
( )
p a = 1,013 x10 5 = 1,35 x 105 Pa
3
V2 = πr23
3
Fuerza neta desarrollada sobre cada pared de la caja p1V1 p 2V2 ( pa + ρgh )d12 bd 22
( p1 − p a )A = (1,35 x10 5
− 1,013 x10 5
)(0,2) 2
T1
=
T

T1
=
T
3
= 1,348x10 = 1348 N Supondremos que
Ejemplo 70. Una campana de buzo cilíndrica de 3 m
pa = 1 atm = 101325 Pa
de diámetro y 4 m de altura con el fondo abierto se ρ = 1025 kg/m3
sumerge a una profundidad de 220 m en el océano. g = 9,8m/s2
La temperatura en la superficie es de 25 ºC y en el Entonces
fondo, a los 220 m, es de 5 ºC. La densidad del agua (101325 + 1025 × 9,8 × 4,2)d12 101325d 22
de mar es de 1025 kg/m3. ¿Cuánto subirá el nivel del =
agua adentro de la campana cuando se sumerge? 278,15 285,15
Solución. d2
Sea h esa altura. o bien = 1,13
p1 = 1 atm = 101325 Pa, d1
V1 = πr 2 H , r = 1,5m, H = 4m
Ejemplo 72. Una campana de buzo en forma de
t1 = 25 ºC, T1 = 298,15 K cilindro con una altura de 2,50 m está cerrada en la
t2 = 5 ºC, T2 = 278,15 K parte superior y abierta en la parte inferior. La
V2 = πr 2 (H − h ) campana se baja desde el aire al agua de mar (ρ =
La campana está a una profundidad h’ = 220m 1,025 gm/cm3 ). El aire encerrado en la campana
El nivel del agua en la campana está a profundidad inicialmente está a 20ºC. La campana se baja a una
h’ - h profundidad (medida desde el nivel del agua dentro
La presión es p 2 = p1 + ρg (h'− h ) de la campana) de 82,3 m. A esta profundidad la
temperatura del agua es de 4ºC, y la campana está en
g = 9,8 m/s2
equilibrio térmico con el agua.( sugerencia: trate al
ρ = 1025 kg/m3 aire como un gas ideal y al mar como un líquido en
p1V1 p 2V2 reposo)
Donde tenemos = a) ¿Cuánto subirá el nivel del agua dentro de la
RT1 RT2
campana?
o sea b) ¿A qué presión se deberá someter el aire dentro
p1 H ( p1 + ρg )(h'− h )(H − h ) de la campana para sacar el agua que entró?
= Dato: la presión atmosférica es 1,013x105 Pa.
T1 T2
Solución.
Poniendo los valores: a)
101325 × 4 = [101325 + 1025 × 9,8(220 − h )(4 − h )]
298,15 278,15
Ecuación que tiene por solución
h = 3,834m

Ejemplo 71. Sube una burbuja de gas desde el


fondo en un lago con agua limpia a una profundidad
de 4,2 m y a una temperatura de 5 ºC hasta la
superficie donde la temperatura del agua es de 12ºC.
p1V1 p 2V2 pT
¿Cuál es el cociente de los diámetros de la burbuja = ⇒ V2 = 1 2 V1
en los dos puntos? T1 T2 p 2T1
(Suponga que la burbuja de gas está en equilibrio p1 = 1,013x105 Pa.
térmico con el agua en los dos puntos.) p2 = 1,013x105 + 1025x9,8x82,3 = 9,28x105 Pa
Solución. T1 = 20 + 273 = 293 K,
si pa indica la presión atmosférica T2 = 4 + 273 = 277 K
h = 4,2m Con los datos:

32
Calor y Termodinámica Hugo Medina Guzmán

V2 =
(1,013x10 )(277 )V
5
= 0,103V1
gas en el volumen izquierdo y el desplazamiento x

(9,28 x10 )(293)


5 1 del mercurio con respecto a la posición de equilibrio.

Como también V2 = Ay:


Ay = 0,103 A(2,5) ⇒ y = 0,258 m
El nivel del agua dentro de la campana subirá
(2,50 – 0,258) = 2,242 m Solución.
Como la cantidad de gas en los dos lados es igual,
b) podemos escribir, cuando la temperatura del lado
izquierdo sea T. La gota de mercurio se desplaza x,
hasta que las presiones en ambos depósitos sea igual
(po).
⎡ ⎛l ⎞⎤ ⎡ ⎛l ⎞⎤
p o ⎢Vo + A⎜ + x ⎟⎥ p o ⎢Vo + A⎜ − x ⎟⎥
⎣ ⎝2 ⎠⎦ ⎝2 ⎠⎦
= ⎣
T To
2Vo + S (l + 2 x )
Para que el volumen sea igual que en la superficie la ⇒ T = To
presión interior debe de igualar a la presión en esa 2Vo + S (l − 2 x )
profundidad
p = 1,013x105 + 1025x9,8x(82,3 + 2,242) Ejemplo 75. Un pez que se encuentra a 63,25m de
= 9,505x105 Pa profundidad en el mar donde la temperatura es 2° C
produce burbujas de aire de 1 cm de radio
Ejemplo 73. Un globo poroso tiene un volumen de 2 aproximadamente. Determine el radio de las
m3 a una temperatura de 10ºC y a una presión de 1,1 burbujas al llegar estas a la superficie del mar donde
atm. Cuando se calienta a 150ºC el volumen se la temperatura es de 27° C. Considere que la
expande a 2,3 m3 y se observa que se escapa el 5% densidad del agua de mar no varía con la
del gas. profundidad y tiene un valor de 1,035 g/cm3.
a) ¿Cuánto gas había en el globo a 10ºC? Solución.
b) ¿Cuál es la presión en el globo a 150ºC? h = 63,25 m
atmlitro p1 = pa + ρgh
R = 0,082 t1 = 2 ºC, T1 = 275,15 K
molK
4
Solución. V1 = πr13
p1 = 1,1atm , V1 = 2 m3, 3
p2 = pa
t1 = 10oC, T1 = 283,15K, n1 = ? t2 = 27 ºC, T2 = 300,15 K
p 2 = ? , V2 = 2,3 m3, 4
t2 = 150oC, T2 = 423,15 K, n2 = 0,95 n1. V2 = πr23
3
p1V1 1,1 × 2000 p1V1 p 2V2
= ⇒
(b + ρgh )r13 br23
a) n1 = = = 94,8 mol =
RT1 0,082 × 283,15 T1 T T1 T
Supondremos que
n RT2
b) p 2 = 2 pa = 1 atm = 101325 Pa
V2 ρ = 1035 kg/m3
0,95 × 94,8 × 0,082 × 423,15 g = 9,8m/s2
= Entonces
2300 (101325 + 1035 × 9,8 × 63,25)r13 101325r23
= 1,387 atm =
275,15 300,15
Ejemplo 74. El termómetro de gases consta de dos r2
recipientes idénticos con gas de volumen Vo cada o bien = 2 ⇒ r2 = 2 cm
uno, unidos por un tubo do longitud l y sección A. r1
Una gota de mercurio obstruye el tubo. Si las Ejemplo 76. Un depósito cerrado contiene agua
temperaturas do los gases en los volúmenes son hasta una altura h = 2,24 m, y por encima a = 1m,
iguales, el mercurio se encontrará en el centro del aire a la presión del exterior pa = 1 atm. Por un
tubo. El volumen derecho se coloca un termostato pequeño orificio de fondo se deja salir el agua.
con temperatura To. Gradúese el termómetro, Calcular el descenso de nivel, suponiendo invariable
buscando la dependencia entre la temperatura del la temperatura del agua.

33
Calor y Termodinámica Hugo Medina Guzmán

Solución.
Inicialmente

Solución.
Sea y la distancia desde la superficie de nivel al
fondo y p la presión del aire; se tiene:

Arriba : 3V0 , T0 , p 0
mg
Abajo: V0 , T0 , p 0 +
A
Como las masas son iguales
⎛ mg ⎞
⎜ p0 + ⎟V0
p 0 3V0 ⎝ A ⎠ mg
p a a = p(a + h − y ) T0
=
T0

A
= 2 p0
Transformación isotérmica
mg
Luego p 0 + = 3 p0
El equilibrio se establecerá cuando A
p + ρgy = p a ⇒ p = p a − ρgy Después de doblar la temperatura
De aquí resulta
p a a = ( p a − ρgy )(a + h − y )
Reemplazando valores:
(1,033 × 10 )(1) = (1,033 × 10
4 4
)
− 0,98 × 10 4 y (3,24 − y )

⎧0,64
y=⎨
⎩3,64
La respuesta posible es y = 0,64 m.
Arriba : Vs , 2T0 , p1
Ejemplo 77. En un recipiente cilíndrico se encuentra
Abajo: Vi , 2T0 , p1 + 2 p 0
en equilibrio un émbolo pesado. Por encima del
émbolo y por debajo de él se hallan masas iguales de El volumen total es el mismo
gas a temperatura idéntica. La relación entre el Vi + Vs = 3V0 + V0 = 4V0
volumen superior y el inferior es igual a 3. ¿Cuál En la parte superior
será la relación de los volúmenes si aumentamos la
temperatura del gas al doble? p 0 3V0 p1Vs 6 p 0V0
= ⇒ p1 =
T0 2T 0 Vs
En la parte inferior
3 p 0V0 ( p1 + 2 p 0 )Vi 6 p 0V0
= ⇒ Vi =
T0 2T 0 ( p1 + 2 p0 )

34
Calor y Termodinámica Hugo Medina Guzmán

6 p0V0 3 p 0V0Vs T3 = 0 ºC + 273,15 ºC = 273,15 K


Vi = = p 2 p3 T
⎛ 6 p 0V0 ⎞ (3V0 + Vs ) = ⇒ p3 = 3 p 2 =
⎜⎜ + 2 p0 ⎟⎟ T2 T3 T2
⎝ Vs ⎠
273,15
Como Vi = 4V0 − Vs 1,033 × 10 5 = 0,756 x 105 = 0,732 atm
373,15
Tenemos:
3 p0V0Vs
4V0 − Vs = TEORÍA CINÉTICA DE LOS GASES
(3V0 + Vs ) IDEALES.
⇒ (4V0 − Vs )(3V0 + Vs ) = 3 p 0V0Vs
El concepto de los átomos y de las moléculas que
⇒ 12V02 + 4V0Vs − 3V0Vs − Vs2 = 3 p 0V0Vs eran los últimos bloques de edificio de la materia fue
restablecido por Dalton para explicar las tres leyes
⇒ 12V02 − 2V0Vs − Vs2 = 0 de combinaciones químicas. En mediados del siglo
⇒ Vs2 + 2V0Vs − 12V02 = 0 diecinueve, estos conceptos, junto con ciertas
asunciones con respecto la naturaleza, el tamaño, la
Resolviendo:
distribución y a los movimientos de las moléculas y
Vs = −V0 ± V02 + 12V02 = de los átomos, fueron sintetizados con la mecánica
Newtoniana, para explicar el comportamiento de los
⎧− 4,6V0 gases ideales. Este trabajo realizado por Maxwell,
− V0 ± 3,6V0 = ⎨ Boltzman y otros, condujo al desarrollo de lo qué se
⎩2,6V0 conoce como la teoría cinética de gases.
La respuesta posible es VS = 2,6V0 , luego
Vi = 4V0 − 2,6V0 = 1,4V0 Las asunciones de la teoría cinética son:
Finalmente:
Cualquier gas se compone de un número muy
Vs 2,6V0 grande de moléculas.
= = 1,86
Vi 1,4V
Las moléculas de un gas son idénticas, con respecto
Ejemplo 78. Una esfera de 20 cm de diámetro a la forma, tamaño y masa.
contiene un gas ideal a una presión de 1 atm y a 20
ºC. A medida que se calienta la esfera hasta 100 ºC Las moléculas son esferas perfectamente rígidas del
se permite el escape de gas. Se cierra la válvula y se radio insignificante.
coloca la esfera en un baño de hielo a 0 ºC.
a) ¿cuántos moles de gas se escapan de la esfera al Las moléculas están en un estado incesante del
calentarse? movimiento caótico en todas las velocidades y
b) ¿Cuál es la presión en la esfera cuando está en el direcciones posibles.
hielo?
Constante de los gases R = 0,082 litro atm/mol K La distribución de moléculas es homogénea e
Respuesta. a) 0,04 moles; b) 0,695 atm
isotrópica en cualquier envase que encierre el gas.
Solución.
a) 0,04 moles
Las moléculas ejercen fuerzas una sobre otra
4
V = π (0,10) = 4,19 × 10 −3 m 3
3
solamente cuando chocan entre ellas o con las
3 paredes del envase.
p1 = 1 atm = 1,033 x 105 Pa
T1 = 20 ºC + 273,15 ºC = 293,15 K La colisión entre las moléculas o las moléculas y las
pV
n1 = 1 =
( )(
1,033 × 10 5 4,19 × 10 −3
= 0,178
) paredes del envase son colisiones perfectamente
elásticas, es decir, sólo tales colisiones pueden
RT1 (8,314)(293,15) cambiar las direcciones del movimiento pero no de
moles sus velocidades.
p2 = p1 = 1 atm = 1,033 x 105 Pa
T2 = 100 ºC + 273,15 ºC = 373,15 K Entre las colisiones sucesivas las moléculas viajan

n2 =
p1V
=
( )(
1,033 × 10 5 4,19 × 10 −3
=
) libremente con velocidades constantes; la distancia
RT2 (8,314)(373,15) viajada libremente se llama trayectoria libre. En
promedio, la trayectoria libre media de todas las
0,139 moles moléculas es igual.
Escapan 0,1788 – 0,139 = 0,04 moles.
b) 0,695 atm

35
Calor y Termodinámica Hugo Medina Guzmán

La energía cinética media de una molécula es Podemos promediar esta relación para todas las
proporcional a la temperatura absoluta del gas. moléculas:
v x = v x2 + v y2 + v z2
Expresión para la presión ejercida por un gas.
y como en nuestro modelo no hay una diferencia real
entre las direcciones x , y y z debido a que las
rapideces son muy altas en un gas típico, así que los
efectos de la gravedad son despreciables. Se sigue
que v x = v y = v z . Por lo tanto:
2 2 2

v 2 = 3v x2
Con esta relación obtenemos:
1⎛m⎞
Sea N el número de moléculas del gas ideal de masa p = ⎜ ⎟N v 2
3⎝V ⎠
M, encerrado en un cubo de lado L La molécula i
se mueve con velocidad vi , con v xi , v yi y v zi Ecuación del gas Ideal gas de la Teoría Cinética.
son sus componentes x , y y z respectivamente. 1 mN 2 2 ⎛ N ⎞⎛ 1 2 ⎞
Considerando p = v = ⎜ ⎟⎜ mv ⎟
3 V 3 ⎝ V ⎠⎝ 2 ⎠
Luego vi = v xi + v yi + v zi
2 2 2 2
Pero
1 2
mv = Energía Cinética promedio de una
Consideremos solamente la componente en x de la 2
molécula i . molécula ∝ T
1 2 3
La fuerza ejercida por esta molécula a causa de sus Por consiguiente mv = k B T
2 2
2L 3
colisiones periódicas con la pared cada Δt = ,y
v xi La elección de la constante como k B es
2
el cambio de cantidad de movimiento − 2mv xi es: mandataria para obtener la ecuación del gas ideal
similar a la ya encontrada.
2mv xi 2mv xi mv xi2
f xi = = = 2 ⎛ N ⎞⎛ 3 ⎞ Nk B T
Δt 2 L v xi L p= ⎜ ⎟⎜ k B T ⎟ =
3 ⎝ V ⎠⎝ 2 ⎠ V
⇒ pV = Nk B T
La fuerza sobre la pared debido a las N moléculas
es: Y también:
⎛ R ⎞
2 pV = Nk B T = N ⎜⎜ ⎟⎟T = nRT
N N
mv ⎝ Na ⎠
Fx = ∑ f xi = ∑ xi

i =1 i =1 L pV = nRT
1 2 3
La presión sobre la pared es: La asunción mv = k B T implica la
2 2
N
mv xi2
F ∑ L ⎛m⎞N 2 interpretación de la energía térmica como energía
mecánica de las moléculas, no obstante como
p x = 2x = i =1 2 = ⎜ 3 ⎟∑ v xi concepto estadístico solamente; es decir, la
L L ⎝ L ⎠ i =1 temperatura es la manifestación del movimiento
⎛m⎞ N medio de una gran cantidad de moléculas; es
p x = ⎜ ⎟∑ v xi2 , ( V = L3 = volumen del gas). 1 2 3
⎝ V ⎠ i =1 absurdo decir mvi = k B T para cualesquier i.
2 2
⎧V = L3
⎛m⎞ 2 ⎪
p x = ⎜ ⎟ N v x , con ⎨ 2 N ENERGÍA INTERNA DE UN GAS IDEAL
⎪ x ∑ v xi
= 2
V
⎝ ⎠ N v Cuando añadimos calor a un cuerpo poniéndolo en
⎩ i =1 contacto térmico con un cuerpo a mayor temperatura
Siendo vi = v xi + v yi + v zi
2 2 2 2 puede elevar su temperatura, fundirse o vaporizarse.
Se pueden efectuar estos mismos cambios realizando
trabajo que resulta en la disipación de energía
mecánica por fricción.

36
Calor y Termodinámica Hugo Medina Guzmán

Añadir calor y realizar trabajo sobre el cuerpo en tal presión p0. Los émbolos se dejan libres. Estímese
forma de disipar energía son equivalentes en lo que sus velocidades máximas. Menospréciese la masa
concierne a efectos térmicos. Ambos, involucran una del gas en comparación con las masas de los
transferencia de energía. émbolos.
La energía mecánica que se añade no desaparece,
permanece dentro del cuerpo en forma de energía
potencial y cinética asociada con los movimientos al
azar de los átomos del cuerpo.
A esta energía térmica se le conoce como ENERGÍA
INTERNA, a la que vamos a denotar con la letra U. Solución.
La energía interna del gas es
1 2 3
Como vimos anteriormente mv = k B T indica 3 3
2 2 U= nRT = p0V0
que la energía cinética traslacional media por 2 2
molécula depende solo de la temperatura; no de la Cuando se expande se convierte en energía cinética
presión, el volumen ni el tipo de molécula. Podemos de los émbolos
obtener la energía cinética por mol multiplicando la
ecuación por el número de Avogadro y usando la
relación M = NAm:

1 2 1 3 1 2 1
NA mv = M v 2 = RT (energía cinética K= mvm + MvM2 (1)
2 2 2 2 2
Cantidad de movimiento inicial: 0
media por mol de gas)
Cantidad de movimiento final: MvM − mvm
Esta ecuación ilustra un resultado general llamado el Cantidad de movimiento inicial = Cantidad de
teorema del equipartición de la energía que dice que movimiento final.
cada "grado de libertad" de un gas contribuye una 0 = MvM − mvm ⇒ MvM = mvm (2)
cantidad de ½ kBT a la energía interna total. Un De (1) y (2):
grado de libertad es un movimiento independiente
que puede contribuir a la energía total. Por ejemplo, 3 p0V0 m 3 p0V0 M
vM = , vm =
una molécula tal como O2 tiene, en principio, 7 M (M + m ) m (M + m )
grados de libertad. Tres se asocian a la traslación a
lo largo de los ejes x, y, y z, tres se asocian a
rotaciones sobre los ejes x, y, y z , y uno se asocia a TRABAJO REALIZADO POR UN GAS
las vibraciones de la molécula a lo largo del eje de Consideremos, por ejemplo, un gas dentro de un
O-O (como las masas que vibran en los extremos de cilindro. Las moléculas del gas chocan contra las
un resorte). Sin embargo, desde el momento de la paredes cambiando la dirección de su velocidad, o
inercia I para las rotaciones sobre el eje O-O es de su momento lineal. El efecto del gran número de
aproximadamente cero, las rotaciones sobre este eje colisiones que tienen lugar en la unidad de tiempo,
(
no agrega casi nada a la energía K = 1 2 Iω .
2
) se puede representar por una fuerza F que actúa
sobre toda la superficie de la pared
Además, la mecánica cuántica demuestra que los
modos vibratorios no están excitados
apreciablemente sino hasta que la temperatura del
gas es alta, así que para la mayoría de los propósitos
asumimos que una molécula diatómica tiene 5
grados de libertad. Un gas monatómico como el
helio tiene 3 grados de libertad.
La energía interna total de n moles de un gas
Si una de las paredes es un pistón móvil de área A, y
monoatómico (con tres grados de libertad) es:
éste se desplaza dx, el intercambio de energía del
3 sistema con el mundo exterior puede expresarse
U= nRT
2 como el trabajo realizado
La energía interna total de n moles de un gas dW = Fdx y F = pA
diatómico (con cinco grados de libertad) es: Se tiene:
U=
5
nRT dW = ( pA)dx = p ( Adx ) ⇒ dW = pdV
2
Siendo dV el cambio del volumen del gas.
Ejemplo 79. En un tubo termo aislado liso e infinito
se encuentran dos émbolos con masas M y m, entre
los cuales hay un gas monoatómico de volumen V0 a

37
Calor y Termodinámica Hugo Medina Guzmán

Expresión que nos permite al integrarla, calcular e energía termina se dice que el sistema ha
trabajo entre dos estados, conociendo la relación experimentado un cambio de energía interna.
entre la presión y el volumen. Supongamos un sistema al que se hace pasar del
B estado de equilibrio 1 al 2, mediante un determinado
W A→ B = ∫ pdV proceso termodinámico y durante el cual medimos el
A
calor absorbido Q y el trabajo realizado W.
Ejemplo 76. En cierto cilindro un émbolo móvil
encierra un volumen Vo con presión po. El émbolo se
deja libre. ¿Qué trabajo ejecutará el gas sobre el
émbolo?, si el volumen del gas, al desplazarse el
émbolo, aumenta al doble, en tanto que la presión
del gas en este caso:
a) permanece constante;
Estas cantidades dependen no solamente de las
b) crece linealmente hasta la presión 2po a medida
características de los estados inicial y final, sino
que aumenta el volumen.
también de los estados intermedios del camino en
particular seguido en el proceso. Sin embargo, si
calculamos la diferencia Q - W para ir del estado de
equilibrio y al 2 por diferentes caminos,
encontramos siempre el mismo valor.
Por consiguiente la diferencia Q - W representa la
variación de energía interna del sistema, si
Solución. asociamos un número con cada estado de equilibrio
a) p = constante de tal modo que sirva como medida de esta cantidad,
2Vo
W = ∫ pdV ⇒ W = p o ∫ dV = p o V
2V0 podemos escribir
Vo V0
U 2 − U1 = Q − W
= W = p 0 (2V0 − V0 ) = p 0V0 Expresión que constituye el primer principio de la
termodinámica.
b) El gráfico muestra la relación lineal de la presión o Q = (U 2 − U 1 ) + W
y la temperatura. Tenga en cuenta que Q y W deben expresarse en las
mismas unidades, ya sean de calor o trabajo.
También que Q es positivo cuando el sistema recibe
(entra) calor y W es positivo cuando el sistema
realiza (sale) trabajo.
Note que la convención de signos que estamos
utilizando aquí en este capítulo para el trabajo es
opuesta a la utilizada en la Mecánica., donde W es
positivo cuando es hecho sobre el sistema. Este
cambio obedece a la costumbre o tradición, dado que
el. Propósito de las máquinas as hacer trabajo y a
p − p0 2 p0 − p0 p0 p
= = ⇒ p = 0V este 1o llamamos en la vida diaria trabajo útil o
V − V0 2V0 − V0 V0 V0 positivo. Por otro lado la convención de signos de: Q
2V0 p
es consistente con este hecho, cuando una máquina
W =∫ 0
VdV disipa o pierde calor es indeseable o negativo.
V0 V La forma descrita se aplica cuando los valores de la
0
2V0 presión, volumen y temperatura correspondientes a
p V2
= 0
V0 2 V
=
p0
2V0
(
4V02 − V02 ) los estados 1 y 2 difieren en cantidades finitas. Si los
estados 1 y 2 varían infinitesimalmente, el primer
0 0 principio toma la forma
3 dQ = dU + dW
= p0V Si el sistema de tal naturaleza que el único trabajo se
2
realiza mediante una expansión o compresión
PRIMERA LEY DE LA TERMODINÁMICA. dQ = dU + pdV
Como ya hemos dicho la transferencia de calor y la Dado que: dW = pdV
realización de trabajo constituyen dos formas o
métodos de transferir, suministrar o quitar, energía a
CALOR ESFECÍFICO DEL GAS IDEAL
una sustancia, o sea, representa energía en tránsito y
Antes de ver las principales transformaciones de los
son los términos utilizados cuando la energía está en
gases veamos el calor específico de un gas ideal a
movimiento. Una vez que la transferencia de
volumen constante y a presión constante.

38
Calor y Termodinámica Hugo Medina Guzmán

Las capacidades caloríficas mas importantes son las c p = cV + R


que se determinan cuando los procesos se realizan a
volumen constante ( CV ) o a presión constante Calor específico del hidrógeno
(Cp ) El comportamiento del calor específico del
hidrógeno con el cambio de temperatura es
sumamente desconcertante a inicios del siglo XX.
Calor específico a volumen constante. En bajas temperaturas que se comporta como un gas
Sea (dQ )V una pequeña cantidad de calor que monoatómico, pero a temperaturas más altas su calor
absorbe un gas a volumen constante (dV = 0 ) . Por específico asume un valor similar a otras moléculas
diatómicas. Tomó el desarrollo de la teoría cuántica
lo tanto no se realiza trabajo (dW = 0 ) , aplicando para demostrar que el hidrógeno diatómico, con su
el primer principio de la termodinámica, pequeña inercia de rotación, requiere una gran
dQ = dU + dW , obtenemos: cantidad de energía para excitar su primera rotación
molecular de estado cuántico. Dado que no puede
(dQ )V = dU obtener esa cantidad de energía a bajas temperaturas,
(dQ )V actúa como un gas monoatómico
Como: CV =
dT
De aquí la capacidad calorífica a volumen constante,
(dQ )V dU
CV = =
dT dT
Para un gas ideal monoatómico:
3
U= nRT , luego,
2
dU 3
CV = = nR
dT 2
PROCESOS TERMODINÁMICOS.
Calor específico a presión constante. El estado de un gas cualquiera o una mezcla de
De igual modo si (dQ ) p es una pequeña cantidad gases está determinado por su temperatura, su
presión y su volumen. En el caso del gas ideal estas
de calor que absorbe un gas a presión constante, variables se unen por la relación para un mol de gas.
aplicando el primer principio de la termodinámica pV = RT
(dQ ) p = dU + (dW ) p La especificación del estado de un gas presupone:
Donde (dW ) p = pdV ⇒ a) Equilibrio térmico. La temperatura es uniforme en
todo el sistema e igual a la del recipiente;
(dQ ) p = dU + pdV b) Equilibrio mecánico. La fuerza ejercida por el
(dQ ) p sistema sobre el recipiente es uniforme en toda su
superficie y es contrabalanceada por tuerzas
Como Cp =
dT externas;
dU dV c) Equilibrio químico. La estructura interna del
De esto obtenemos: Cp = +p sistema y su composición química no varían de un
dT dT punto a otro.
dU dV Un estado que satisfaga estas condiciones se
y como CV = , C p = CV + p denomina estado de equilibrio termodinámico y sus
dT dT variables satisfacen la ecuación anterior. Si
para un gas ideal pV = nRT queremos usar la ecuación de estado durante una
A presión constante, dp = 0 , luego transformación, es necesario que el sistema no se
aleje mucho de las condiciones de equilibrio; esto se
dV
pdV = nRdT ⇒ p = nR consigue procurando que la transformación se
dT realice en una sucesión de estados de equilibrio poco
Luego, C p = CV + nR diferentes entre sí; este proceso se llama cuasi
estático; durante la transformación, el sistema está
Para un gas monoatómico: en todos los instantes en una proximidad infinita al
3 5 estado de equilibrio. Esto se consigue, en general,
Cp = nR + nR = nR haciendo los cambios en forma suficientemente lenta
2 2
para que el sistema entre en equilibrio después de
cada modificación (en rigor, una transformación
También como C p = CV + nR , exigiría un tiempo infinito para su realización).
La capacidad calorífica por mol

39
Calor y Termodinámica Hugo Medina Guzmán

La energía interna U del sistema depende


únicamente del estado del sistema, en un gas ideal
depende solamente de su temperatura. Mientras que
la transferencia de calor o el trabajo mecánico
dependen del tipo de transformación o camino
seguido para ir del estado inicial al final.

Isocórico o a volumen constante


No hay variación de volumen del gas, luego

W = 0, Q = ncV (TB − T A ) Ejemplo 80. Expansión libre de un gas.


Un recipiente de paredes rígidas y completamente
Donde cV es el calor específico a volumen constante aisladas está dividido en dos por medio de una
pared. Una parte contiene gas y la otra está
evacuada. Si la pared que los separa se rompe
súbitamente, mostrar que la energía interna final y la
inicial son iguales.
Solución.
Según el primer principio de la termodinámica:
Q = (U 2 − U 1 ) + W
Como el sistema está aislado Q es cero, o sea
(U 2 − U 1 ) + W =0
el trabajo W realizado sobre el sistema también es
cero. Note que el gas inicialmente tenía un volumen
Isobárico o a presión constante V y una presión p y finalmente un volumen V y una
W = p (V B − V A ) , Q = nc p (TB − T A ) presión p/2.
Luego:
Donde c p es el calor específico a presión constante (U 2 − U 1 ) = 0 ⇒ U 2 = U1

Ejemplo 81. Una cámara al vacío hecha de


materiales aislantes se conecta a través de una
válvula a la atmósfera, donde la presión es p o . Se
abre la válvula y el aire fluye a la cámara hasta que
la presión es p 0 . Probar que u f = u 0 + p oV0 ,
donde u 0 y V0 es la energía interna molar y
Isotérmico o a temperatura constante volumen molar de temperatura y presión de la
atmósfera.
u f es la energía interna molar del aire en la cámara.
pV = nRT
Solución.
Inicialmente la cámara tenía un volumen cero de
constante aire, al final se encuentra llena de aire y el trabajo
La curva p= , representa la
V por mol realizado sobre el sistema sería − p oV0 .
transformación en un diagrama p –V es una Como está aislado no ha habido pérdida ni ganancia
hipérbola cuyas asíntotas son los ejes coordenados de calor.
VB VB
nRT V Aplicando el primer principio de la termodinámica:
W= ∫ pdV = ∫
VA VA
V
dV = nRT ln B
VA Q = (U 2 − U 1 ) + W
Obtenemos por mol
0 = (u f − u 0 ) − p oV0
ΔU = 0 , Q = W Finalmente:
u f = u 0 + p oV0

Ejemplo 82. Un gas se expande desde I a F por tres


posibles trayectorias como se indica en la figura.

40
Calor y Termodinámica Hugo Medina Guzmán

Calcule el trabajo realizado por el gas a lo largo de pa ⎛ nRT nRT ⎞


las trayectorias IAF, IF y IBF. = nRT ln + pb ⎜⎜ − ⎟
pb ⎝ pa p b ⎟⎠
⎡ p ⎛p ⎞⎤
= nRT ⎢ ln a + ⎜⎜ b − 1⎟⎟ ⎥
⎣ pb ⎝ p a ⎠⎦
⎡ ⎛ 1 ⎞⎤
= RT ⎢ ln5 + ⎜ − 1⎟⎥
⎣ ⎝ 5 ⎠⎦
= 19,9 l atm = 2017,5 J
Solución.
f Ejemplo 84. La figura muestra un ciclo donde a es
a) W IAF = ∫ i
pdV = 2 x (4 - 2) el estado inicial del sistema.
Las energías internas de los estados son: Ua = 10 J,
= 4 litro atm = 4 x 101,33 J = 405,32 J
Ub = 35 J, Ud = 39 J.
1
(1 × 2) En el proceso b → c, el trabajo realizado por el gas
f
b) W IF = ∫
i
pdV = 2 × 1 +
2 es + 91 J.
= 3 litro atm = 3 x 101,33 J = 304 J
f
c) W IBF = ∫ i
pdV = 2 × 1
= 2 litro atm = 2 x 101,33 J = 202,7 J

Ejemplo 83. Una muestra de un gas ideal de 1 mol


se lleva a través de un proceso termodinámico
cíclico, como se muestra en la figura. El ciclo consta Encontrar:
de tres partes, una expansión isotérmica (a - b), una a) El calor añadido al sistema durante el proceso
compresión isobárica (b - c) y un aumento de la b → c.
presión a volumen constante (c -d). Si T = 300 K, pa b) El calor removido en el proceso d → a.
= 5 atm, pb = pc = 1 atm, determine el trabajo
Solución.
realizado por el gas durante el ciclo.
pV
Usando la ley del gas ideal = constante ,
T
podemos encontrar una relación entre las
temperaturas en a, b, c y d.
Si Ta = T, Tb = 2T, Tc = 4T y Td = 2T
a) Qbc = C p (Tc − Tb )
= C p ( 4T − 2T ) = 2C p T
Por la segunda ley de la termodinámica:
Solución.
U c − U b = Qbc − Wbc ⇒
W = Wab + Wbc + Wca
Para una expansión isotérmica ab U c − 35 = Qbc − 91
b b dV V Por otra parte en el proceso a → b :
Wab = ∫ pdV = ∫ nRT = nRT ln a
U b − U a = Qab − Wab
a a V Vb
pa ⇒ 35 − 10 = Qab − 0
= nRT ln
pb y Qab = 25 J y también
Para la compresión isobárica bc Qab = CV (Tb − Ta ) = CV (2T − T ) = CV T
Wbc = pb (Vc − Va ) luego CV T = 25 J
Para la compresión isocórica ca no hay trabajo.
En el proceso c → d :
Wca = 0
U d − U c = Qcd − Wcd ⇒ 39 − U c = Qcd − 0
De tal manera:
Como
p
W = nRT ln a + p c (Vc − Va ) Qcd = CV (Td − Tc ) ⇒
pb
Qcd = CV (2T − 4T ) = −2CV T
y Qcd = −2 × 25 = −50 J

41
Calor y Termodinámica Hugo Medina Guzmán

con lo que encontramos U d − U a = 10000 cal. Además, observe que al ir


U c = 39 − Qcd = 39 + 50 = 89 J por la trayectoria adb solo se hace trabajo en ad y no
Finalmente: en db, o sea, se tiene que:
Qbc = U c − 35 + 91 = 89 − 35 + 91 = 145 J Wad = Wadb = 2500 cal.
Qbc = 145 J Luego
Qad = (U d − U a ) + Wad
b) Qda = U a − U d + Wda
Qad = 10000 + 2500 = 12500 cal. (absorbido)
Qda = C p (Ta − Td ) = C p (T − 2T ) = −C p T
Como encontramos que
Como Qbc = 145 J = 2C p T Qadb = 15000 y Qadb = Qad + Qdb
145 Obtenemos
Luego Qda = −C pT = − = −72,5 J
2 Qdb = 15000 – 12500 = 2500 cal. (Absorbido)
Esta última cantidad también podría encontrarse
Ejemplo 85. En la figura se muestran diversas teniendo en cuenta que:
trayectorias entre los estados de equilibrio a, b, c y Wdb = 0
d, en un diagrama p-V.
Y como en (a) hemos determinado que
U b − U a = 12,500 cal.
Si U a = 0 , se tiene que U b = 12500, luego
U b − U d = 12500 – 10000 = 2500 cal.
Finalmente
Qdb = (U b − U d ) + Wdb = 2500 cal.
a) Cuando el sistema pasa de1 estado a al b a lo
largo de la trayectoria a, c, b recibe 20000 calorías y Ejemplo 86. Un mol de un gas ideal se encuentra en
realiza 7500 cal de trabajo. Calcular el cambio de un estado inicial p = 2 atm y V = 10 litros indicado
energía interna (U b − U a ) . por el punto a en el diagrama pV de la figura. El gas
b) ¿Cuánto calor recibe el sistema a lo largo de la se expande a presión constante hasta el punto b,
trayectoria adb, si el trabajo realizado es 2500 cal? cuyo volumen es 30 litros y luego se enfría a
c) Cuando el sistema vuelve de b hacia a, a lo largo volumen constante hasta que su presión es de 1 atm
de la trayectoria curva ba, el trabajo realizado es en el punto c.
5000 cal. ¿Cuánto calor absorbe o libera el sistema? Entonces se comprime a presión constante hasta
alcanza su volumen original en el punto d y
d) Si U a = 0 y U d = 10000 cal., hállese el calor finalmente se calienta a volumen constante hasta que
absorbido en los procesos ad y db. vuelve a su estado original.
Solución. a) Determinar la temperatura de cada estado a, b, c y
a) Por la trayectoria acb, se tiene: d.
Q = 20000 cal. b) Determinar el calor añadido a lo largo de cada
W = 7500 cal. una de las etapas del ciclo.
Luego, c) Calcular el trabajo realizado a lo largo de cada
Ub −Ua = Q −W ⇒ trayectoria.
d) Determinar la energía de cada estado a, b, c y d.
U b − U a = 20000 – 7500 = 12500 cal. e) ¿Cuál es el trabajo neto realizado por el gas en el
b) Por la trayectoria adb, W = 2500 cal. ciclo completo?
Q = (U b − U a ) + W
Q = 12500 + 2500
Qadb = 15000 cal. (absorbido)
c) Para la trayectoria ba,
W = + 5000 cal.
Luego,
Q = (U a − U b ) + W
Solución.
Q = - 12500 + 5000
a) Por la ley del gas ideal: pV = nRT ⇒
Q ba = - 7,500 cal. (libera)
pV
d) Si U a = 0 y U d = 10,000 cal, T=
nR

42
Calor y Termodinámica Hugo Medina Guzmán

litro.atm Como 1 litro-atm = 101,3 J = 24,2 cal:


n = 1 , R = 0,0821 W = 4052 J = 968 calorías (trabajo del sistema)
mol.K
2 × 10 De b → c (volumen constante)
En a ⎧⎨ p a = 2 atm Luego T = = 243,6 K
⎩Va = 10 litros 0,0821 El trabajo es W = 0 , (no hay trabajo).
2 × 30
En b ⎧⎨ pb = 2 atm Luego T = = 730,8 K De c → d (presión constante)
V
⎩ b = 30 litros 0,0821 El trabajo es W = p(Vd − Vc )
⎧ p = 1 atm Luego T = 1× 30 = 365,4 K
En c ⎨ c W = 1(10 − 30 ) = - 20 litro atm
⎩Vc = 30 litros 0,0821 W = - 2026 J = - 484 calorías (trabajo sobre el
sistema)
⎧ p = 1 atm Luego T = 1× 10 = 121,8 K
En d ⎨ d
⎩Vd = 10 litros 0,0821 De d → a (volumen constante)
b) El trabajo es W = 0 , (no hay trabajo).
De a → b (presión constante) d) Como
El calor suministrado es Q = C p ΔT 3
U= nRT
Siendo gas ideal (gas monoatómico) 2
5 3 ⎛ cal ⎞
Cp = nR = (1mol)⎜ 2 ⎟T
2 2 ⎝ mol K ⎠
cal cal = 3T
Como n = 1 , y R = 2 ⇒ Cp = 5
mol K K U a = 3Ta = 3(243,6 K ) = 730,8 cal
ΔT = 730,8 – 243,6 = 487,2 K U b = 3Tb = 3(730,8 K ) = 2192,4 K
Q = (5)(487,2 ) = 2436 calorías
U c = 3Tc = 3(365,4 K ) = 1096,2 K
De b → c (volumen constante) U d = 3Td = 3(121,8 K ) = 365,4 K
El calor suministrado es Q = CV ΔT e) Trabajo neto = Wab + Wbc + Wcd + Wda
Siendo gas ideal (gas monoatómico) = 4052 + 0 – 2026 + 0 = 2026 J
3 = 487 cal
C p = nR
2 Calor absorbido = Qab + Qbc + Qcd + Qda
cal = 2436 – 1096,2 – 1218 +365,4
Como n = 1 , y R = 2 = 487 cal
mol K Trabajo neto = calor absorbido
cal = Calor que entra – calor que sale.
⇒ Cp = 3
K
ΔT = 365,4 – 730,8 = -365,4 K Ejemplo 87. Considere el proceso cíclico descrito
en la figura. Si Q es negativo para el proceso BC y
Q = (3)(− 365,4 ) = -1096,2 calorías ΔU es negativo para el proceso CA:
a) determine los signos de Q asociados a cada
De c → d (presión constante) proceso.
El calor suministrado es Q = C p ΔT b) determine los signos de W asociados a cada
proceso.
ΔT = 121,8 – 365,4 = - 243,6 K
Q = (5)(− 243,6 ) = -1218 calorías

De d → a (volumen constante)
El calor suministrado es Q = CV ΔT
ΔT = 243,6 – 121,8 = 121,8 K
Q = (3)(121,8) = 365,4 calorías Solución.
c) a) QAB = positivo
De a → b (presión constante) QBC = negativo (Dato)
El trabajo es W = p(Vb − Va )
(UC –UB) = QBC -WBC = QCA = negativo
(UA –UB) = QCA - WCA ⇒ = QCA = (UA –UC) +
W = 2(30 − 10 ) = 40 litro atm WCA = (-) + (-) = negativo

43
Calor y Termodinámica Hugo Medina Guzmán

b) WAB = positivo temperatura original, como se esperaba. La salida de


WBC = 0 (A volumen constante) calor es por lo tanto:
WCA = 2 (6-10) = -8 = negativo Q’2 = ncp(T4- T1)
= 0,487 mol x 29,317 J/ mol K (325 – 250)K
Ejemplo 88. Un cilindro contiene un gas ideal a una = 1072 J.
presión de 2 atmósferas, el volumen es de 5 1itros a La salida de calor total durante el proceso de
una temperatura del gas de 250 K. El gas se calienta enfriamiento es.
a volumen constante hasta una presión de 4 Q' = H’1+ H’2= 4397 J.
atmósferas, y luego a presión constante hasta una La diferencia entre el calor de entrada y el de salida
temperatura de 650 K. Calcular el calor total es 304 J. Esto debe aparecer como trabajo hecho por
recibido durante estos procesos. Para el gas el cv es el gas, puesto que la energía interna del gas debe ser
21,0 J /mol K igual al principio y en el final de un proceso de
Luego el gas entonces es enfriado a vo1umen cíc1ico.
constante hasta su presión original y después a b) La cantidad 304 J debería estar de acuerdo con el
presión constante se lleva el gas hasta su volumen valor del área dentro de la curva del ciclo, que
original. representa el trabajo hecho por el gas. Es un
a) Encuentre la salida de calor total durante estos rectángulo de alto 2 atm y largo1,5 litros. El área
procesos y bajo ésta curva es:
b) el trabajo total hecho por el gas en el proceso W = 2 x1,013 x 106dinas/cm x 1,5 x 103 cm3
cíclico del conjunto. = 3,04 x 109 ergios = 304 J,
Solución. Lo que esta de acuerdo con el ingreso.

Ejemplo 89. Sobre un mol de gas se realiza un ciclo


cerrado que consta de dos isócoras y dos isóbaras.
Las temperaturas en los puntos a y c son Ta y Tc.
Determínese el trabajo que efectúa el gas durante
dicho ciclo, si se sabe que los puntos b y d yacen en
una isoterma
La ecuación del gas ideal permite el cálculo del
número de los moles originalmente presentes.
pV 2atm × 5litro
n= =
RT 0,0821 litro atm/mol.K
= 0,487 mol
También C p = CV + nR , la capacidad calorífica
por mol cp = cv + R.
a) cp = cv + R = (21,0+ 8,317)J/mol K Solución.
= 29,317 J/mol K
En el primer cambio p/T es constante y luego,
como p se duplica, T se duplica también a 500 K.
La entrada de calor por lo tanto es:
Q1 = ncv(T2 – T1)
= 0,487 mol x 21,0 J/ mol K x (500 – 250)K
= 2558 J.
En el Segundo cambio V/T es constante y, como T se W = Wab + Wbc + Wcd + Wda
incrementa en la razón 650/500, entonces V se hace
6,5 1itros. La entrada de calor por lo tanto es: Wab = 0 , Wbc = p 2 (V2 − V1 ) , Wcd = 0 ,
Q2 = ncp(T3 - T2)
= 0,487 mol x 29,317 J/mol K x(650 – 500)K
Wda = − p1 (V2 − V1 ) ,
= 2143 J. W = ( p 2 − p1 )(V2 − V1 )
La entrada de calor total durante estos dos procesos
es Q = Q1 + Q2 = 4701 J.
W = p 2V2 − p 2V1 − p1V2 + p1V1
Durante el primer proceso de enfriamiento p se hace Por la de los gases ideales p 2V2 = RTc .
la mitad, y T también se hace la mitad 325 K. La
salida de calor es p 2V1 = RTb , p1V2 = RTd , p1V1 = RTa
Q’1 = ncv (T3- T4) W = R(Tc − Tb − Td + Ta )
= 0,487 mol x 21,0 J /mol K x (650- 325)K
Como Tb = Td
= 3325 J.
En el Segundo proceso de enfriamiento V se reduce W = R(Tc + Ta − 2Tb )
en la razón de 5/6,5, y T se hace 250K, la De las relaciones

44
Calor y Termodinámica Hugo Medina Guzmán

p1 p 2 p T
= → 1 = a y
Ta Tb p 2 Tb
p1 p 2 p T
= → 1 = d
Td Tc p 2 Tc
Ta Td
= → Ta Tc = TbTd
Tb Tc
2
Con Tb = Td ⇒ Ta Tc = Tb Solución.
El trabajo es el área bajo la trayectoria de A a B en
Finalmente Ta Tc = Tb el gráfico pV . El volumen disminuye, tal que W <
Con lo que obtenemos 0.
(
W = R Tc + Ta − 2 Ta Tc ) W = − 12 (500 × 103 + 150 × 103 )(0,60 )
= R( ) 2 = - 1,95 x 105 J
Tc − Ta
ΔU = nCV ΔT
Ejemplo 90. Una cantidad de aire se lleva del p1V1 PV
estado a al b siguiendo una trayectoria recta en una Con T1 = , T2 = 2 2
nR nR
gráfica pV.
p2V2 − p1V1
a) En este proceso ¿la temperatura del gas: aumenta, ⇒ ΔT = T2 − T1 =
disminuye o no cambia? Explique. nR
b) Si V = 0,0700 m3, Vb = 0,1100 m3, pa = 1,00 x l05
⎛ C ⎞
Pa y pb = 1,40 x l05 Pa, ¿cuánto trabajo efectúa el ΔU = ⎜ V ⎟( p 2V2 − p1V1 )
gas en este proceso. Suponga que el gas tiene ⎝ R ⎠
comportamiento ideal.
⎛ 20,85 ⎞
ΔU = ⎜
8,315
[( ) ( )
⎟ 5 × 10 5 (0,20 ) − 1,5 × 10 5 (0,80 )]
⎝ ⎠
= -5,015 x 104 J
Luego ΔU = Q − W
⇒ Q = ΔU + W = - 0,5015 x 105 - 1,95 x 105
= - 2,45 x 105
Solución. Q es negativo, el calor fluye fuera del gas.
a) El producto pV se incrementa, y aun para un gas
Ejemplo 92. Sea 20,9 J el calor añadido a
no ideal, esto indica un incremento de temperatura. determinado gas ideal. Como resultado, su volumen
b) El trabajo es el área encerrada bajo la línea que cambia de 63,0 a 113 cm3 mientras que la presión
representa el proceso y las verticales en Va y Vb . El permanece constante a 1,00 atm.
área del trapezoide es: a) ¿En cuánto cambió la energía interna del gas?
b) Si la cantidad de gas presente es de 2,00 x 10-3
1
( pb + p a )(Vb − Va ) mol, halle la capacidad calorífica molar a presión
2 constante.
1 c) Halle la capacidad calorífica molar a volumen
= ( 2,40 × 10 ) (0,0400) = 400 J
5
constante.
2 Solución.
a) ΔU = Q − W
Ejemplo 91. Cuatro moles de O2 se llevan de A a
B con el proceso que muestra en una gráfica pV de la Q = 20,9 J,
figura. Suponga que el tiene comportamiento ideal. W = p (V2 − V1 ) ⇒
Calcule el flujo de calor Q durante este proceso. N
¿Entra calor en el gas o sale de él? W = 1,013 × 10 5 2 (113 − 63) × 10 −6 m 3
m
W = 5,06 J
b) Q = nC p (T2 − T1 )

p1V1 1,013 × 105 × 63 × 10−6


T1 = = = 384 K
nR 2,00 × 10.3 × 8,31

45
Calor y Termodinámica Hugo Medina Guzmán

p1V1 1,013 × 105 × 113 × 10−6 Para cada proceso, Q = ΔU + W . No se realiza


T1 = = = 689 K trabajo en los procesos ab y dc, también
nR 2,00 × 10.3 × 8,31
Wbc = Wabc y Wad = Wadc .
20,9 = 2,00 × 10 −3 C p (689 − 384)
El calor para cada proceso es,
20,9 × 103 J para ab Qab = 90 J ,
Cp = = 34,3
2 × 305 mol K para bc Qbc = 440 J + 450 J = 890 J ,
20.9 × 10 3
J para ad Qad = 180 J + 120 J = 300 J ,
Cp = = 34,3
2 × 305 mol K para dc Qdc = 350 J , el calor es absorbido en cada
c)
proceso. Las flechas representadas en los procesos
CV = C p − R ⇒ indican la dirección del incremento de la
J temperatura (incrementando U).
CV = 34,3 − 8,31 = 26
mol K Ejemplo 95. La figura muestra cuatro estados de un
sistema termodinámico: a, b, c y d. El volumen del
Ejemplo 93. Una mol de un gas ideal monoatómico
es llevado cuasiestáticamente desde el estado A sistema es Va tanto en el estado a como en el b, y es
recorriendo el ciclo ABCDA, tal como se muestra en Vc tanto en el estado c como en el d. La presión del
la figura.
Hallar: sistema es p a tanto en el estado a como en el d, y
a) La temperatura en A es p c tanto en el estado b como en el c. Las
b) El trabajo total.
energías internas de los cuatro estados son: Ua, Ub,
Uc y Ud. Para cada uno de los procesos: ab, bc, ad y
dc, calcule:
a) el trabajo efectuado por el sistema;
b) el flujo de calor al sistema durante el proceso;
c) El sistema se puede llevar del estado al c
siguiendo la trayectoria abc o bien la adc. Calcule el
Solución. flujo neto de calor al sistema y el trabajo neto
efectuado por el sistema en cada trayectoria. ¿Por
pV
a) pV = n RT, y T = , en el punto A: cuál trayectoria es mayor el flujo neto de calor? ¿Por
nR cuál es mayor el trabajo neto?
6 × 10 3 × 1 d) Un amigo le dice que las cantidades de flujo de
TA = = 722 K calor deben ser iguales para la trayectoria abc y la
1 × 8,31 trayectoria adc, porque el estado inicial (a) y el final
b) Trabajo total = Area ABCDA (c) del sistema son los mismos por ambas
= (3,5 + 3 – 2 – 1,5)2 = 6,0 kJ trayectorias. ¿Cómo respondería a esta afirmación?

Ejemplo 94. Un sistema termodinámico se lleva del


estado a al estado c de la figura siguiendo la
trayectoria abc o bien la trayectoria adc. Por la
trayectoria abc, el trabajo W efectuado por el sistema
es de 450 J. Por la trayectoria adc, W es de 120 J.
Las energías internas de los cuatro estados
mostrados en la figura son: Ua = 150 J, Ub = 240 J,
Uc = 680 J y Ud = 330 J. Calcule el flujo de calor Q Solución.
para cada uno de los cuatro procesos: ab, bc, ad y Vamos a usar las ecuaciones, W = p (V2 − V1 ) y
dc. En cada proceso, ¿el sistema absorbe o
desprende calor? ΔU = Q − W .
a) El trabajo hecho por el sistema durante el proceso:
A lo largo de ab o cd, W = 0. A lo largo de bc,
Wbc = pc (Vc − Va ) A lo largo de ad,
Wad = p a (Vc − Va ).
b) El calor que ingresa al sistema durante el proceso:
Q = ΔU + W .
ΔU ab = U b − U a , tal que,
Solución.
Qab = U b − U a + 0.

46
Calor y Termodinámica Hugo Medina Guzmán

ΔU bc = U c − U b , tal que V2 V
Wcd = nRT2 ln = p1V2 ln 2
Qbc = (U c − U b ) + pc (Vc − Va ) . Vc Vc
ΔU ad = U d − U a , tal que Wda = p1 (V1 − V2 )
Qad = (U d − U a ) + pa (Vc − Va ) . Wneto = Wab + Wbc + Wcd + Wda
ΔU dc = U c − U d , tal que Wbc se anula con Wcd
Qdc = (U c − U d ) + 0 . V V
Wneto = p1V1 ln b + p1V2 ln 2
c) Del estado a al estado c a lo largo de la trayectoria V1 Vc
abc.
pV
Wabc = pc (Vc − Va ). Qabc p1V1 = p 2Vb ⇒ Vb = 1 1 , p1V2 = p 2Vc
p2
= U b − U a + (U c − U b ) + pc (Vc − Va )
pV
= (U c − U a ) + pc (Vc − Va ) ⇒ Vc = 1 2
Del estado a al estado c a lo largo de la trayectoria
p2
adc. Reemplazando los valores de Vb y Vc
Wadc = pa (Vc − Va ) . respectivamente:
p1V1 p 2 V2
Qadc = (U c − U a ) + pa (Vc − Va ) Wneto = p1V1 ln + p1V2 ln
V1 p1V2 p 2
Asumiendo pc > pa , Qabc > Qadc y
p p
Wabc > Wadc . Wneto = p1V1 ln 1 + p1V2 ln 2
p2 p1
d) Para entender esta diferencia, comenzar por la
relación Q = W + ΔU . El cambio de la energía p p
= − p1V1 ln 2 + p1V2 ln 2
Interna ΔU es independiente de la trayectoria de tal p1 p1
manera que es igual para la trayectoria abc y para la p
trayectoria adc. El trabajo hecho por el sistema es el = p1 (V2 − V1 ) ln 2
área bajo los caminos en el diagrama pV- no es igual p1
para las dos trayectorias. De hecho, es más grande
para la trayectoria abc. Puesto que ΔU es igual y PROCESO ADIABATICO:
W es diferente, Q debe ser diferente para las dos Es un proceso termodinámico importante en el cual
trayectorias. El flujo del calor Q es dependiente de al cambiar, el sistema de estado de equilibrio no
la trayectoria. intercambia calor con el ambiente, Q = 0. En este
caso, de acuerdo al primer principio, se tiene:
Ejemplo 96. Un motor térmico funciona con un gas U 2 − U 1 = −W .
ideal que se somete a un ciclo termodinámico que Es importante hacer notar que este trabajo,
denominado TRABAJO ADIABATICO (Wad ) ,
consta de dos etapas isotérmicas y dos etapas
isobáricas de presiones p1 y p2 (p2 > p1) . Si las dos
isotermas cortan la isobárica de presión p1 en los hecho para cambiar el sistema desde un estado
volúmenes V1 y V2 (V2 > V1) inicial a un final, depende solo de los estados de
a) Grafique el proceso en los ejes pV . equilibrio dados. Conociendo Wad se puede
b) Determine el trabajo neto realizado en función de
determinar la trayectoria. Cuando se realiza un
p1 , p2 ,V1 Y V2
trabajo que no es adiabático, entre los dos estados
Solución.
dados, la cantidad en exceso o defecto comparado
a)
con el trabajo adiabático es calor y es lo que
realmente lo define como otra forma de trabajo.

Ecuación del proceso adiabático


Cuando un gas ideal va en un proceso adiabático, la
presión volumen y temperatura cambian de forma tal
que es descrito solamente por una relación entre p y
V, T y V, o p y T , en función de las capacidades
caloríficas. Esta relación puede calcularse aplicando
Vb V el primer principio de la termodinámica y utilizando
b) Wab = nRT1 ln = p1V1 ln b
V1 V1 la ecuación del gas ideal.
Según el primer principio tenemos:
Wbc = p 2 (Vc − Vb ) = p1 (V2 − V1 ) dQ = dU + dW = dU + pdV

47
Calor y Termodinámica Hugo Medina Guzmán

Como dU = CV dT (aunque este resultado se ⎡ γ −1



p1V1 ⎢ ⎛ p 2 ⎞ γ ⎥
obtuvo considerando un proceso a volumen c) W = 1− ⎜ ⎟
constante, relación solamente las variables U y T y γ − 1 ⎢ ⎜⎝ p1 ⎟⎠ ⎥
por lo tanto, es válido independientemente del ⎢⎣ ⎥⎦
proceso considerado), luego podemos escribir: Solución.
dQ = CV dT + pdV a) Por el principio de la termodinámica
Como dQ = 0 en un proceso adiabático, se tiene: dQ = dU + dW
CV dT + pdV = 0 Como el proceso es adiabático dQ = 0
Luego dW = − dU
pdV
dT = − (1) dU
CV Pero = CV ⇒ dU = CV dT
dT
De la ecuación del gas ideal
Y dW = −CV dT
pV = nRT
Integrando de 1 a 2:
pdV + Vdp = nRdT (2)
W1→2 = −CV ∫ dT = −CV (T2 − T1 )
T2
Reemplazando (1) en (2); para eliminar dT : T1

pdV + Vdp = − nR
p
dV W = CV (T1 − T2 )
CV b) Tenemos que dW = pdV
pCV dV + VCV dp = − nRpdV γ
Por ser proceso adiabático pV = C
(CV + nR ) pdV + CV Vdp = 0 C
⇒ p= γ
dp C p dV V
=−
p CV V dV
Luego dW = C γ
Cp V
Llamando a la relación = γ . Para gas ideal: V2
CV V2 dV V −γ +1 ⎤
5
Integrando: W = C ∫
V1 Vγ
=C ⎥ =
− γ + 1⎦ V
γ = = 1,67 1

3 CV1−γ +1 − CV2−γ +1
dp dV γ −1
= −γ
p V γ γ
Como p1V1 = p 2V2 = C
Integrando
Reemplazando C en la expresión de W en las formas
ln p = −γln V + ln const. arriba puestas, obtenemos finalmente:
pV γ = constante p1V1 − p 2V2
Utilizando la ecuación de los gases ideales W =
γ −1
pV = nRT se pueden encontrar las siguientes c) De la expresión anterior
relaciones:
γ −1 p1V1 ⎡ p 2V2 ⎤
W = 1−
p γ
γ − 1 ⎢⎣ ⎥
p1V1 ⎦
TV γ −1 = constante , = constante
T 1 1

La curva de un proceso adiabático, en un diagrama ⎛ C ⎞γ ⎛C ⎞γ


pV cae más rápidamente con el aumento de V que la pero V1 = ⎜⎜ ⎟⎟ y V2 = ⎜⎜ ⎟⎟
curva de un proceso isotérmico. ⎝ p1 ⎠ ⎝ p2 ⎠
de allí

Ejemplo 97. Demostrar que el trabajo realizado por ⎡ ⎛C ⎞ ⎤
un gas ideal, con capacidades caloríficas constantes, ⎢ p 2 ⎜⎜ ⎟⎟ ⎥
durante una expansión adiabática es igual a: p1V1 ⎢ p ⎥
a) W = CV (T1 − T2 ) W = ⎢1 − ⎝ 2 ⎠1 γ ⎥
γ −1 ⎛C⎞
⎢ p1 ⎜⎜ ⎟⎟ ⎥
p1V1 − p 2V2 ⎢ ⎥
b) W = ⎣ ⎝ p1 ⎠ ⎦
γ −1

48
Calor y Termodinámica Hugo Medina Guzmán

⎡ γ −1
⎤ pV γ = cte
p1V1 ⎢ ⎛ p 2 ⎞ γ ⎥
= 1− ⎜ ⎟ γ γ
Entonces: p iVi = p f V f
γ − 1 ⎢ ⎜⎝ p1 ⎟⎠ ⎥
⎢⎣ ⎥⎦ ⇒ 5(12) = p f (30 )
1, 4 1, 4

De donde
Ejemplo 98. Encontrar el módulo de
compresibilidad elástica en un proceso adiabático p f = 1,39 atm
(Badiabático). p iVi 5 × 12
Se conoce la relación de capacidades caloríficas b) Ti = = = 365,9 K
nR 2 × 0,082
⎛ c ⎞
⎜⎜ γ = p ⎟⎟ . p fVf 1,39 × 30
⎝ cV ⎠ Tf = = = 254,3 K
Solución. nR 2 × 0,082
Tenemos:
dp dV Ejemplo 100. Un mol de un gas ideal monoatómico
B=− ⇒ dp = − B (1) inicialmente a 300 K y a 1 atm se comprime
dV V cuasiestática y adiabáticamente a un cuarto de su
V volumen inicial. Encuentre la presión y temperatura
También, en un proceso adiabático: final. (γ = 1,67)
Solución.
pV γ = constante n = 1mol
derivando γ = 1,67
dpV γ + pγV γ −1 dV = 0 Ti = 300 K
de aquí pi = 1 atm
dV 1
dp = −γp (2) V f = Vi
V 4
γ
Igualando (1) y (2): pV = cte , pV = nRT
dV dV Bien
−B = −γp
V V piVi γ = p f V fγ ⎫
de aquí obtenemos: ⎪
Badiabático = γ p piVi p f V f ⎬ ⇒ Vi γ −1Ti = V fγ −1T f
=
Ti T f ⎪⎭
El sonido en el aire se propaga en un proceso De la última
adiabático γ −1
La velocidad de un gas está dada por ⎛V ⎞
Tf = ⎜ i ⎟ Ti = (4 )
0 , 67
300
B ⎜V ⎟
v= ⎝ f ⎠
ρ = 459,15 K
Para el aire: También
(
Badiabático = γ p = 1,4 1,013 × 10 5 ) ⎛V ⎞
γ

pf = ⎜ i ⎟ pi = (4)1,67 × 1 = 10,1 atm


ρ aire = 1,28 kg/m3 ⎜V ⎟
⎝ f ⎠
1,4(1,013 × 15)
v= = 333 m/s
Ejemplo 101. Durante el tiempo de compresión de
1,28
cierto motor de gasolina, la presión aumenta de 1 a
20 atm. Suponiendo que el proceso es adiabático y el
Ejemplo 99 . Dos moles de un gas ideal se expanden
gas es ideal con γ = 1,40.
cuasiestática y adiabáticamente desde una presión de
a) ¿en qué factor cambia el volumen? y
5 atm y un volumen de 12 litros a un volumen final
b) ¿en qué factor cambia la temperatura?
de 30 litros. (γ = 1,40) Solución.
(a) ¿Cuál es la presión final del gas? γ = 1,40, pi = 1 atm, pf = 20 atm
(b) ¿Cuáles son las temperaturas inicial y final?
γ γ
Solución. a) piVi = p f V f ⇒
n = 2 mol, γ = 1,4 , pi = 5 atm , Vi = 12 litros , 1 1

V f = 30 litros V f ⎛ p i ⎞ γ ⎛ 1 ⎞ 1, 4
=⎜ ⎟ = ⎜ ⎟ = 0,12
a) Para una expansión adiabática Vi ⎜⎝ p f ⎟⎠ ⎝ 20 ⎠

49
Calor y Termodinámica Hugo Medina Guzmán

γ −1
Tf ⎛V ⎞ 0, 4 Q 2 = calor liberado por el sistema al reservorio a
⎛ 1 ⎞
b) =⎜ i ⎟ =⎜ ⎟ = 2,33 θ 2 Donde θ 1 > θ 2 .
Ti ⎜⎝ V f ⎟
⎠ ⎝ 0,12 ⎠
W = Q1 – Q2 trabajo neto hecho por el sistema.
CICLOS REVERSIBLES E IRREVERSIBLES
Supongamos que ocurre un proceso en que el Eficiencia térmica.
sistema va de un estado inicial (i) a otro final (f) en Observe que el enunciado que hemos dado del
el que se realiza un trabajo W y se produce una segundo principio de la termodinámica establece que
transferencia de calor Q a una serie de reservorios de la máquina térmica perfecta en la que todo calor
calor. Si al final de este proceso, el sistema puede suministrado se convierte en trabajo sin perder calor,
ser restaurado a su estado inicial se dice que es no existe. Nos gustaría tenerla, pues no viola la
REVERSIBLE. Un proceso que no llena este primera ley, pero no se ha obtenido.
requisito se dice que es IRREVERSIBLE.
Las condiciones para un proceso reversible son: Dado que el trabajo neto en el ciclo es lo que
1) No debe existir trabajo realizado por fricción, obtenemos, y el calor absorbido por la sustancia de
fuerzas debidas a la viscosidad u otros efectos trabajo es lo que ponemos. Luego la eficiencia
disipativos. térmica de la máquina está definida por:
2) El proceso debe ser tal que el sistema se Trabajo obtenido
encuentre siempre en estado de equilibrio o Eficiencia térmica =
infinitamente próximo a él (cuasiestático - por calor puesto
ejemplo, si el pistón de un cilindro se mueve W
lentamente dando tiempo para que el sistema pueda e=
interactuar con el ambiente y alcanzar un estado de Q1
equilibrio en todo instante). Aplicando la primera ley a un ciclo completo.
Cualquier proceso que viole una de estas Como los estados inicial y final son los mismos la
condiciones es irreversible. La mayoría de los energía interna final debe ser igual a la inicial,
procesos en la naturaleza son irreversibles. Si obteniéndose
queremos conseguir un proceso reversible debemos Q1 − Q2 = W
eliminar las fuerzas disipativas y el proceso sea
de aquí
cuasiestático, en la práctica esto es imposible. Sin
embargo nos podemos aproximar mucho a un Q1 − Q2
e=
proceso reversible. Q1
CICLOS TERMODINÁMICOS. MÁQUINAS Q
e = 1− 2
TERMODINÁMICAS. Q1
Una máquina que realiza esta conversión, lo hace
mediante "PROCESOS" que llevan a la sustancia de Tenga en cuenta que en esta expresión Q1 y Q2
trabajo nuevamente a su estado original, al conjunto deben ser tomados en valor absoluto, como
de estos procesos se conoce como "CICLO" una vez positivos, dado que al haber aplicado la primera ley
completado el ciclo, los procesos se vuelven a
repetir.
(W = Q1 − Q2 ) ya se ha considerado su propio
Una máquina térmica se puede representar en forma signo.
idealizada como se muestra en la siguiente figura. Observe que la eficiencia sería 100% (e = 1) si
Q2 = 0 es decir sin ceder nada de calor, esto es
completamente imposible en la práctica y lo
establece el segundo principio que veremos más
adelante (e < 1). En cambio, si Q2 = Q1 se tendrá
e = 0 y W = Q1 − Q2 = 0 .

Ejemplo 102. Cierta máquina tiene una potencia de


salida de 5 kW y una eficiencia de 25%. Si la
máquina libera 8000 J de calor en cada ciclo,
Repitiendo el ciclo se puede obtener cualquier
encuentre:
cantidad de trabajo.
a) el calor absorbido en cada ciclo y
Damos la siguiente notación, refiriéndonos a un
b) el tiempo para cada ciclo.
ciclo completo.
Solución.
Q 1 = calor absorbido por el sistema del reservorio
a θ1 .

50
Calor y Termodinámica Hugo Medina Guzmán

W Q 8000 p 0V0
a) e = = 1 − 2 ⇒ 0,25 = 1 − ⇒ Cp
Q1 Q1 Q1 Q nR
e = 1− 2 = 1−
Q1 = 10666,67 J Q1 C
2 p 0V0 ln 2 + V p 0V0
b) W = eQ1 = 2666,67 J nR
Cp
W W 2666,67
P= ⇒ t= = = 0,53 s nR
t P 5000 = 1−
CV
2 ln 2 +
Ejemplo 103. En cierto proceso industrial se nR
somete un gas al siguiente ciclo termodinámico: Si es gas monoatómico
1-compresión isobárica hasta la mitad de su volumen
3 5
inicial, CV = nR y C p = nR
2-calentamiento isocórico, 2 2
3-expansión isotérmica hasta la presión y el volumen 5
inicial.
El control de calidad requiere que la eficiencia del e = 1− 2 = 0,1338 = 13,38%
proceso sea mayor al 11%. Determine la eficiencia 3
2 ln 2 +
del ciclo para un gas monoatómico y para un gas 2
diatómico, y en cada caso indique si aprueba o no el Si es gas diatómico
control de calidad. 5 7
Solución. CV = nR y C p = nR
2 2
7
e = 1− 2 = 0,09939 = 9,94%
5
2 ln 2 +
2
Se aprueba el control de calidad para gas
monoatómico.
1-compresión isobárica hasta la mitad de su volumen
Ejemplo 104.
inicial,
Un gas ideal monoatómico se somete a un ciclo
Cp
Q AB = C p (TB − T A ) = −
termodinámico que consta de 3 procesos:
p 0V0 A → B Compresión adiabática desde (V0, p0) hasta
nR
2-calentamiento isocórico, cuadriplicar la presión.
B → C Expansión isotérmica hasta la presión
QBC = CV (TC − TB ) = CV (T A − TB ) inicial.
Por la ly del gas ideal: C → A Compresión isobárica hasta el volumen
p AV A 2 p 0V0 inicial.
TA = = a) Presente un gráfico p versus V para el ciclo.
nR nR b) Determine las variables termodinámicas p, y, T
p BV B p 0V0 para cada estado A, B, C.
TB = =
nR nR c) Calcule la eficiencia del ciclo.
Luego Solución:
CV a)
QBC = p 0V0
nR
3-expansión isotérmica hasta la presión y el volumen
iniciales.
2V0
QCA = WCA = nRT A ln , como
V0
2 p 0V0
TA = ⇒ QCA = 2 p 0V0 ln 2
nR
De aquí deducimos que:
CV pV
Q1 = 2 p 0V0 ln 2 + p 0V0 y Q2 = C p 0 0 b)
nR nR
La eficiencia del ciclo es:

51
Calor y Termodinámica Hugo Medina Guzmán

p AV A p 0V0 compuesto por seis procesos simples mostrado en el


Estado A: p A = p 0 , T A = = diagrama p-V de la figura.
nR nR
Estado B: p B = 4 p 0 ,
γ −1
⎛p ⎞ γ γ −1 p 0V0 2 5 p 0V0
TB = ⎜⎜ B ⎟⎟ T A = (4 ) γ = 4
⎝ pA ⎠ nR nR
p 0V0
Estado C: pC = p 0 , = TC = TB = 4
25

nR
c)
Calor en A → B: Q AB = 0 e → a Entrada isobárica (presión constante), el
volumen varía de cero a V1 , a1 igual que el número
VC de moles de cero a n , de acuerdo a la ecuación
Calor en B → C: Q BC = nRTB ln p 0V = nRTa
VB
γ γ
Calculo de V B : p BV B = p AV A ⇒
1 3
⎛ p ⎞γ ⎛ 1 ⎞5
VB = ⎜⎜ A ⎟⎟ V A = ⎜ ⎟ V0
⎝ pB ⎠ ⎝4⎠
V V
Cálculo de VC : C = A ⇒
TC T A
⎛T ⎞
VC = ⎜⎜ C ⎟⎟V A = 4 2 5 V0 a → b Compresión adiabática, de acuerdo a la
⎝ TA ⎠ ecuación
Luego TaVaγ −1 = TbVbγ −1
25
4 V0
QBC = nRTB ln =
(1 4)3 5 V0
⎛ pV ⎞
(
nR⎜ 4 2 5 0 0 ⎟ ln 4 2 5 × 4 3 5 = 2,41 p 0V0
nR ⎠
)

Calor en C → A:
5 ⎛ p AV A pC VC ⎞
QCA = C p (T A − TC ) = nR⎜ − ⎟
2 ⎝ nR nR ⎠

=
5
2
( )
p0V0 − 4 2 5 p 0V0 = − 1,85 p 0V0 b → c Compresión isocórica (volumen constante)
la temperatura cambia de Tb a Tc . Este proceso es
aproximado a la explosión en el motor de gasolina.
La eficiencia es
Q2
e = 1− , Q1 = 2,41 p 0V0 y Q2 = 1,85 p 0V0
Q1
Luego:
1,85 p 0V0
e = 1− = 1 − 0,7676 = 0,2324
2,41 p 0V0
e = 23,23%

CICLO DE OTTO.
El funcionamiento de un motor a gasolina puede c → d Descompresión adiabática de acuerdo a la
idealizarse considerando que la sustancia de trabajo ecuación.
es aire, el cual se comporta como un gas ideal y que TcVcγ −1 = Td Vdγ −1
no hay fricción. En base a esto el ciclo de Otto está

52
Calor y Termodinámica Hugo Medina Guzmán

γ −1
⎛V ⎞
e = 1 − ⎜⎜ 2 ⎟⎟
⎝ V1 ⎠

Ejemplo 105. La figura representa un diagrama p-V


del ciclo Joule de gas ideal, C p es constante. ¿Cuál
es su eficiencia térmica?

d → a Descompresión a volumen constante, la


temperatura cambia de Td a Ta . Este proceso es
aproximado a la apertura de la válvula en el motor a
gasolina.

Solución.
En este ciclo, el ingreso de calor se produce en el
proceso adiabático b → c y la salida de calor en el
proceso isobárico d → a .

Q1 = ∫ C p dT = C p (Tc − Tb ) y
Tc
Luego
Tb

Q2 = ∫ C p dT = C p (Ta − Td )
Ta

a → e Proceso isobárico a presión atmosférica, el Td

volumen varía de V1 a cero, a temperatura Luego la eficiencia


constante. e =1−
Q2 (T − Ta )
=1 − d
Q1 (Tc − Tb )
Por la trayectoria adiabática a → b :
p2γ −1 γ pγ −1 γ
= 1
Ta Tb
γ −1 γ
ó Tb p2 = Ta p1γ −1 γ (1)
Por la trayectoria adiabática c → d :
p2γ −1 γ pγ −1 γ
= 1
Td Tc
Q1 = ∫ CV dT = CV (Tc − Tb )
Tc
γ −1 γ
Tb ó Tc p2 = Td p1γ −1 γ (2)
El calor liberado Q2 , a volumen constante Restando (1) de (2):

Q2 = ∫ CV dT = − CV (Td − Ta )
Ta (Tc − Tb ) p2γ −1 γ = (Td − Ta ) p1γ −1 γ
Td γ −1
La eficiencia es (Td − Ta ) = ⎛⎜ p2 ⎞⎟ γ
e =1−
Q2 (T − Ta )
=1 − d
De aquí:
(Tc − Tb ) ⎜⎝ p1 ⎟⎠
Q1 (Tc − Tb ) γ −1
De los procesos adiabáticos tenemos ⎛p ⎞ γ
Finalmente: e = 1 − ⎜⎜ 2 ⎟⎟
TdV1γ −1 = TcV2γ −1 y TaV1γ −1 = TbV2γ −1 ⎝ p1 ⎠
restando
(Td − Ta )V1γ −1 = (Tc − Tb )V2γ −1 CICLO DIESEL
γ −1
Este ciclo también se inicia con una compresión

o
(Td − Ta ) ⎛ V2 ⎞
=⎜ ⎟
adiabática, ocurre la explosión manteniéndose
(Tc − Tb ) ⎜⎝ V1 ⎟⎠ constante la presión, aunque no es necesario
introducir una chispa, ya que la combustión se
finalmente produce de manera espontánea. Nuevamente la etapa

53
Calor y Termodinámica Hugo Medina Guzmán

de trabajo se corresponde con una expansión Q2 = CV (Td − Tc ) = 149,58(250 − 432 ) =


adiabática y finalmente se realiza un enfriamiento
isócoro del fluido en el motor. 27223,56 J = 27 kJ
c) El cambio de energía interna del gas, en la
compresión adiabática
ΔU = U b − U a = CV (Tb − Ta )
= 149,58(375 − 250 )
= 18697,5 J = 19 kJ
d) El trabajo realizado por el motor, en la expansión
adiabática es igual al negativo del cambio de energía
interna en el proceso.
W = − ΔU = U d − U c = CV (Td − Tc )
= 149,58(432 − 540 )
= - 16154,64 J = - 16 kJ
Ejemplo 106. Un motor diesel opera en el ciclo e) La eficiencia térmica del motor.
reversible abcda, con 9,0 moles de un gas ideal. Los W Q1 − Q2 Q
procesos ab y cd son adiabáticos. Las temperaturas e= = = 1− 2
de los puntos a, b, c y d del ciclo son 250 K, 375 K,
Q1 Q1 Q1
540 K, 432 K, respectivamente. La constante 27223,56
adiabática del gas es 1,50. = 1− = 1 – 0,73 = 0,27
37469,79
La eficiencia es el 27 por ciento.

Ejemplo 107. 10 moles de un gas diatómico (Cv =


5R/2) se encuentran inicialmente a una presión de pA
= 5 x105 Pa y ocupando un volumen de
VA = 249 10-3 m3. Se expande adiabáticamente
(proceso AB) hasta ocupar un volumen VB = 479
x10-3 m3. A continuación el gas experimenta una
transformación isoterma (proceso BC) hasta una
presión pC = 1 x105 Pa. Posteriormente se comprime
a) Calcule el calor absorbido durante la expansión isobáricamente (proceso CD) hasta un volumen VD =
isobárica. VA = 249 10-3 m3. Por último, experimenta una
b) Calcule el calor rechazado en el proceso de transformación a volumen constante (proceso DA)
isocórico. que le devuelve al estado inicial.
c) Calcule el cambio de energía interna del gas, en la a) Representar gráficamente este ciclo en un
compresión adiabática. diagrama p-V.
d) Calcule el trabajo realizado por el motor, en la b) Calcular el valor de las variables termodinámicas
expansión adiabática. desconocidas en los vértices A, B, C y D.
e) Calcule la eficiencia térmica del motor, en c) Hallar el calor, el trabajo, la variación de energía
porcentaje. interna, en Joules, de forma directa y/o empleando el
Solución. Primer Principio, en cada etapa del ciclo.
a) Cálculo previo de las capacidades caloríficas d) Calcular el rendimiento.
nR R= 0,082 atm litro/mol K = 8,314 J/mol K ;
C p = CV + nR γ = 1 + 1 cal = 4,186 J; 1atm = 1,013 105 Pa
CV Solución.
9,0(8,31) 74,79 a) Representar gráficamente este ciclo en un
1,5 = 1 + CV = = 149,58 J/K diagrama p-V.
CV 0,5
C p = 149,58 + 74,79 = 224,37 J/K
C p = 149,58 + 74,79 = 224,37 J/K
El calor absorbido (Q1 ) durante la expansión
isobárica
Q1 = C p (Tc − Tb ) = 224,37(540 − 373)
= 37469,79 J = 37 kJ
b) El calor rechazado (Q2 ) en el proceso de b) Calcular el valor de las variables termodinámicas
isocórico desconocidas en los vértices A, B, C y D.

54
Calor y Termodinámica Hugo Medina Guzmán

5 7 ⎛7 ⎞
CV = R , C p = CV + R = R , = 10⎜ 8,314 ⎟(299,5 − 1132,7 )
2 2 ⎝2 ⎠
Cp 7 = - 248272,7 J
γ = = = 1,4 W = p(VD − VC )
CV 5
Vértice A p AV A = nRT A ⇒ (
= 10 249 × 10 − 958 × 10
5 −3 −3
)
= - 70930 J
T A = 1447,5 K
Comprobación, ΔU ≈ Q − W
A → B p AV Aγ = p BVBγ ⇒
p B = 2 × 10 5 Pa Proceso D → A (Isocórico)
Vértice B p BV B = nRTB ⇒ W = 0 no hay cambio de volumen
TB = 1152,7 K Q = nCV (TA − TD )
⎛5 ⎞
B → C pBVB = pCVC ⇒ = 10⎜ 8,314 ⎟(1447,5 − 299,5)
⎧VC = 958,3 × 10−3 ⎝2 ⎠
⎨ = 249004,3 J
⎩TC = 1152,7 K ΔU = Q = 249004,3 J
Vértice D pDVD = nRTD ⇒ En el ciclo completo

TD = 299,5 K ⎧ΔU = 0
⎪W = 67278,1 J
c) Hallar el calor, el trabajo, la variación de energía ⎪
interna, en Joules, de forma directa y/o empleando el ⎨
Primer Principio, en cada etapa del ciclo. ⎪Qabsorbido = 315462,4 J (+ )
Proceso A → B (adiabático) ⎪⎩Qcedido = 248272,7 J (−)
Q=0 Podemos ver que W ≈ Qabs + Qced
ΔU = nCV (TB − T A ) = ΔU (J) Q (J) W (J)
⎛5 ⎞ A→B - 71666,7 0 71750
10⎜ 8,314 ⎟(1152,7 − 1447,5) = 71166,7 J B→C 0 66438,1 66458,1
⎝2 ⎠
VB VB dV
C→D - 177337,6 -248272,7 -70930
W = ∫ pdV = cte ∫ D→A 249004,3 249004,3 0
VA VA V γ
0 67278,1
( p AV A − p BVB )
= d) Calcular el rendimiento.
γ −1
W
= (5 × 10 × 249 × 10 − 2 × 10 × 479 × 10 ) e= = 0,21 = 21%
5 −3 5 −3

1,4 − 1 Qabs
= 71750 J
Comprobación, ΔU ≈ Q − W SEGUNDA LEY DE LA TERMODINÁMICA.
La experiencia nos dice que a pesar de que es muy
Proceso B → C (Isotérmico) fácil convertir energía mecánica completamente en
Δ = 0 (no hay cambio de temperatura) energía térmica (como en la fricción), hay muchas
VC VC dV restricciones para efectuar La transformación
W = ∫ pdV = nRT ∫ inversa. La única forma en que somos capaces de
VB VB V efectuar la transformación continua de energía
VC 958 × 10 −3
= nR (1152,7 ) ln
térmica en energía mecánica es teniendo
= nRT ln “reservorios de calor” a dos temperaturas diferentes,
VB 479 × 10 −3
e interactuando entre ellas una máquina que
= 66458,1 J transforme una parte del calor que fluye del
Q = W = 66458,1 J reservorio caliente al frío en trabajo (máquina
Proceso C → D (Isobárico) térmica) . El segundo principio de la termodinámica:
ΔU = nCV (TD − TC ) se refiere a este hecho y se establece
cualitativamente como sigue:
⎛5 ⎞
= 10⎜ 8,314 ⎟(299,5 − 1132,7 )
⎝2 ⎠ "Es imposible construir una máquina de
funcionamiento continuo que produzca trabajo
= - 177337,6 J
mecánico derivado de la extracción de calor de un
Q = nC p (TD − TC )

55
Calor y Termodinámica Hugo Medina Guzmán

reservorio simple, sin dar calor, a un reservorio a p (N/m2) V (m3) T (K)


temperatura más baja” a 3324 1 400
b 1662 2 400
En resumen, la segunda ley establece los procesos c 1662 1,51 302
que sin violar la primera ley no ocurren en la
naturaleza. La primera Ley establece simplemente la b)
conservación de energía.
⎛V ⎞
Wab = nRT ln⎜⎜ b ⎟⎟
Reservorio de calor. Se define como un cuerpo de ⎝ Va ⎠
masa tal que es capaz de absorber o liberar calor en = (8,31)(400) ln 2 = 2304 J
cantidad ilimitada sin sufrir apreciable cambio de su
estado, temperatura u otra variable termodinámica. Wbc = p(Vc − Vb )
= (1662)(1,51 − 2) = - 814 J
Ejemplo 108. Una mol de un gas monoatómico se
lleva por un ciclo abca como se muestra en la figura. Wca = −ΔU = − nCV ΔT = - 1222 J
El proceso a → b es un proceso isotérmico a 400 WTotal = 268 J
K y el proceso c → a es un proceso adiabático. c)
a) Hallar la presión, el volumen y la temperatura
Qab = Wab = 2304 J
para los puntos a, b y c.
b) Hallar el trabajo total en el ciclo. Qbc = nC p ΔT = - 2036 J
c) Hallar los calores en cada uno de los procesos
(Qab, Qbc y Qca). Qca = 0
d) Hallar la eficiencia del ciclo. Q2 2036
e) e =1− =1− = 0,11
Q1 2304

Ejemplo 109. Una maquina tiene una potencia de


salida de 2 kW, si su eficiencia es del 40% y cede un
calor de 3000 calorías por ciclo.
a) Determine el trabajo realizado por ciclo.
b) El tiempo de duración de cada ciclo.
Solución.
Solución. a) Determine el trabajo realizado por ciclo.
a) Cálculo de las presiones: e = 40% , Q2 = 3000 calorías
paVa = pbVb = nRT = 1 x 8,31 x 400
Q2 W
3324 N e = 1− =
pa = = 3324 2 , Q1 Q1
1 m
Q 3000
3324 N Q1 = 2 = = 5000 calorías
pb = = 1662 2 , 1− e 0,6
2 m
Y el trabajo es:
N
pc = pa = 1662 2 W = Q1 − Q2 = 5000 – 3000 = 2000 calorías.
m b) 1 cal = 4,186 Joules
Cálculo de los volúmenes: Como la potencia es 2000 J/s
Va = 1m3 , Vb = 2m3 , 2000 J (1 caloría/4,186 J) = 477,78 calorías
γ γ El tiempo de duración de cada ciclo es:
Como p aVa = p bVb ,
2000
5 t= = 4,2 s
con γ = ⇒ 477,78
3
3324(1) = 1662(Vc )
53 53 EL CICLO CARNOT
Vamos a estudiar ahora una máquina térmica
∴Vc = (2 )
35
= 1,51m3 altamente idealizada conocida como la máquina de
Cálculo de las temperaturas: Carnot. Nos muestra como es posible obtener
trabajo por medio de una sustancia de trabajo que es
Ta = Tb , = 400 K, llevada a través de un proceso cíclico y también nos
Como p cVc = nRTc ⇒ permitirá establecer la escala absoluta
termodinámica de temperatura.
pcVc 1662 × 1,51
Tc = = = 302 K
nR 1 × 8,31

56
Calor y Termodinámica Hugo Medina Guzmán

Un ciclo de Carnot es un conjunto de procesos, la (Por ser un ciclo en que estado final = estado
sustancia de trabajo se imagina primero en equilibrio inicial)
térmico con un reservorio frío a la temperatura T2 . W = Q2 − Q2 = ΔQ (Calor total absorbido por el
Se realiza cuatro procesos, por ejemplo sobre un gas, sistema enunciado)
como se muestra en el diagrama p-V de la figura.. W = Trabajo neto entregado
Durante la expansión isotérmica b → c ingresa
calor Q1 .
Como la energía interna de un gas ideal depende
solo de su temperatura
Vc Vc dV V
Q1 = W1 = ∫ pdV = RT1 ∫ = RT1 ln c
Vb Vb V Vb
Del mismo modo durante la comprensión isotérmica
d → a en que se realiza calor Q2 .
a → b Compresión adiabática reversible hasta que Va Va dV V
Q2 = W2 = ∫ pdV = RT2 ∫ = RT2 ln a
1a temperatura se eleve a T1 . Vd Vd V Vd
Va
Siendo Vd > Va ln es una cantidad negativa,
Vd
como debemos de poner como cantidad positiva
Vd
escribimos Q2 = RT2 ln
Va

b → c Expansión isotérmica reversible hasta un En la expansión adiabática e → d


punto c. γ −1
γ −1 γ −1 ⎛V
T1 ⎞
T1Vc = T2Vd ⇒ ⎜⎜ d ⎟⎟ = (1)
⎝ Vc
T2 ⎠
En la comprensión adiabática a → b
γ −1
γ −1 γ −1 ⎛V ⎞ T1
T2Va = T1Vb ⇒ ⎜⎜ a ⎟⎟ = (2)
⎝ Vb ⎠ T2
de (1) y (2)
c → d Expansión adiabática reversible hasta que Va Vd V V
la temperatura baje a T2 . = ⇒ d = c (3)
Vb Vc Va Vb
V
T2 ln d
Q Va T
Entonces 2 = = 2
Q1 V T1
T1 ln c
Vb
La relación entre las temperaturas absolutas de
reservorios de calor en los que trabaja la máquina de
d → a Compresión isotérmica reversible hasta que Carnot tiene la misma relación que los calores
se alcanza el estado original. rechazado y absorbido.

La eficiencia térmica es
Q2
e = 1−
Q1
Q2
Reemplazando por su valor, obtenemos:
Q1
En este ciclo se tendrá: T2
ΔU = 0 e = 1−
T1

57
Calor y Termodinámica Hugo Medina Guzmán

gustaría tenerla, puesto viola la primera Ley, pero


Para que una máquina Carnot tenga una eficiencia tampoco se ha obtenido nunca.
de 100 por ciento es claro que e debería ser cero. Coeficiente de rendimiento de un refrigerador:
Como en la práctica no es posible tener e = 1, es Q2 Q
imposible obtener el cero absoluto de temperatura. η= = 2
Estos resultados que se han obtenido usando un gas Q2 − Q1 W
ideal como sustancia de trabajo, sin embargo, son
independientes de este hecho y en general la Ejemplo 110. La eficiencia de una máquina de
eficiencia de una máquina térmica reversible es Carnot es de 30%. La maquina absorbe 800 J de
independiente del material usado como sistema, calor por ciclo de una fuente caliente a 500 K.
dependiendo únicamente de las temperaturas de los Determine
reservorios. a) el calor liberado por ciclo y
b) la temperatura de la fuente fría.
MOTOR Y REFRIGERADOR Solución.
Un motor de Carnot es un dispositivo ideal que T2 = 500 K
describe un ciclo de Carnot. Trabaja entre dos focos, Q2 = 800 J
tomando calor Q1 del foco caliente a la temperatura e = 0,3
T1, produciendo un trabajo W, y cediendo un calor W Q
Q2 al foco frío a la temperatura T2. a) e = = 1− 2
En un motor real, el foco caliente está representado
Q1 Q1
por la caldera de vapor que suministra el calor, el Q
sistema cilindro-émbolo produce el trabajo, y se 0,3 = 1 − 2 ⇒ Q2 = 560 J
cede calor al foco frío que es la atmósfera.
800
T
b) e = 1 − 2
T1
T
0,3 = 1 − 2 ⇒ T2 = 350 K
500
Ejemplo 111. Una máquina de Carnot opera con 2
moles de un gas ideal. En el proceso cíclico, la
temperatura máxima que alcanza el gas es de 527°C
y la presión máxima es de 5 atm. En un ciclo, el
calor suministrado es de 400 J y el trabajo realizado
La máquina de Carnot también puede funcionar en por dicha máquina es de 300 J.
sentido inverso, denominándose entonces a) Calcular la temperatura del depósito frío y la
refrigerador o frigorífico. Se extraería calor Q2 del eficiencia porcentual.
foco frío aplicando un trabajo W, y cedería Q1 al b) Si empleando únicamente el calor expulsado por
foco caliente. la máquina se logra derretir totalmente un bloque de
En un refrigerador real, el motor conectado a la red hielo de 10 kg a 0°C, ¿Durante cuántos ciclos debe
eléctrica produce un trabajo que se emplea en operar esta máquina?
extraer un calor del foco frío (la cavidad del c fusión agua = 334 × 10 3 J kg
refrigerador) y se cede calor al foco caliente, que es c) ¿Cual debería ser la temperatura del depósito
la atmósfera caliente sin modificar la del depósito frío para elevar
la eficiencia hasta el 80%?
Solución.
a) T1 = 273+ 527 = 800 K

La segunda Ley establecería que no existe el


Refrigerador perfecto. No es posible transportar
calor de un cuerpo a otro de más alta temperatura,
sin efectuar trabajo sobre el sistema. También, nos
W = Q1 − Q2 ⇒

58
Calor y Termodinámica Hugo Medina Guzmán

Q2 = Q1 − W = 400 – 300 = 100 J Ejemplo 113. Un aparato de aire acondicionado


absorbe calor de su embobinado de enfriamiento a
Q2 T2 13 ºC y libera calor al exterior a 30 ºC.
= ⇒
Q1 T1 a) ¿Cuál es el máximo rendimiento del aparato?
b) Si el rendimiento real es de la tercera parte del
Q 100 valor máximo y si el aparato remueve 8 x 104 J de
T2 = T1 2 = 800 = 200 K
Q1 400 energía calórica cada segundo, ¿qué potencia debe
desarrollar su motor?
T2 = 200 - 273 = - 73 ºC
Solución.
La eficiencia es:
Q1 calor transferido a la fuente caliente
T2 200 Q2 calor absorbido de la fuente fría
e = 1− = 1− = 0,75 = 75 % W trabajo gastado por la bomba
T1 800
Q2
b) Para derretir los 10 kg de hielo se necesitan η=
10 (334x103) = 334x104 J W
Si en cada ciclo el calor expulsado por la máquina es a) Si el refrigerador es una máquina de Carnot
100 J funcionando a la inversa
Esta máquina debe operar Q2 T2 273,15 + 13
334 × 10 4 = = = 0,943922
= 33400 ciclos. Q1 T1 273,15 + 30
100
c) ¿Cual debería ser la temperatura del depósito Q1 T1 273,15 + 30
= = = 1,06
caliente sin modificar la del depósito frío para elevar Q2 T2 273,15 + 13
la eficiencia hasta el 80%? entonces
T Q2 Q2 1
e' = 1 − 2 ⇒ η= = = = 16,7
T '1 W Q1 − Q2 Q1
−1
T 200 200 Q2
T '1 = 2 = = = 1000 K
1 − e' 1 − 0,8 0,2 16,7
t’1 = 1000 – 273 = 727 ºC. b) Si η real = = 5,56 y P1 = 8 x 104 J / s,
3
entonces.
Ejemplo 112. Se ha propuesto una planta de
potencia que haga uso del gradiente de temperatura
en el océano. El sistema se diseñó para operar entre Q2 P2
20 ºC (temperatura de la superficie del agua) y 5 ºC
η real = = ⇒
W P1 − P2
(temperatura del agua a una profundidad de casi 1
km). P
P1 = 2 + P2
a) ¿Cuál es la máxima eficiencia de dicho sistema? η real
b) Si la potencia de salida de la planta es de 7,5
MW, ¿cuánta energía térmica se absorbe por hora? ⎛ 1 ⎞ ⎛ 5,56 + 1 ⎞
c) En vista de los resultados de la parte (a), ¿piensa P1 = P2 ⎜⎜ + 1⎟⎟ = 8 × 10 4 ⎜ ⎟
que se deba tomar en cuenta dicho sistema? ⎝ η real ⎠ ⎝ 5,56 ⎠
Solución. = 8x104 (1,18) = 9,44x104 W.
t1 = 5 ºC, T1 = 278,15K
t2 = 20 ºC, T2 = 293,15 K Ejemplo 114. Se dan dos máquinas de Carnot
P = 7,5 MW acopladas, la máquina A opera entre los reservorios
T2 278,15 T1 = 1000 K y T2 = 800 K y la máquina B entre
a) e = 1 − = 1− = 0,051 = 51%
T1 293,15 T2 = 800 K y T3 = 400 K. Sabiendo que el
W P P reservorio T1 suministra 1500 Joules de calor al
b) e = = ⇒ P1 =
Q1 P1 e sistema, calcular:
a) La eficiencia de cada máquina y del sistema.
o sea la potencia absorbida será b) El trabajo de cada máquina y el total del sistema.
7,5
P2 = = 147 MW Solución.
0,051
En una hora
Q2 = 147 x 3600 x 106 J = 5,292 x 1011 J
c) Se recomienda que no.

59
Calor y Termodinámica Hugo Medina Guzmán

Para éste problema,


Q2 − Q1 1 ⎛ T2 − T1 ⎞
e= = ⎜⎜ ⎟
Q1 5 ⎝ T1 ⎟⎠
El calor se toma del agua del lago mientras que se
enfría de 2°C a 0°C antes de la eyección. La
temperatura media del reservorio caliente es 274 K.
Si m es la masa del agua que fluye en el tiempo t, el
calor tomado adentro del reservorio caliente por
unidad de tiempo es Q2/t = (m/t)c x 2°C, donde c
está la capacidad específica de calor del agua.
El calor que sale al aire como reservorio frío a una
temperatura de -15°C = 258 K, por la cantidad
T2 800 infinita de aire disponible se asume que la
a) e A = 1 − = 1− = 20 %
T1 1000 temperatura permanece constante.
Además, el trabajo realizado (Q2 - Q1) es 10
T3 400 kilovatio = 104 J/ s. Así, de la primera ecuación,
eB = 1 − = 1− = 50 %
T2 800 tenemos
Eficiencia del sistema 104 J/s 1 (274 − 258)K
eS = 1 −
T3
=1−
400
= 60 % (m t )(4,18J/gº C)(2º C ) =
5 274K
T1 1000
m 5 × 274 × 104 g 3 g
b) Cálculo de WA ∴ = = 102,4 × 10
t 2 × 4,18 × 16 s s
Q2 T2 La razón del flujo de agua necesario es 102,4
= ⇒
Q1 T1 litros/s
T 800
Q2 = 2 Q1 = (1500) = 1200 J Ejemplo 116. Una máquina térmica realiza 200 J de
T1 1000 trabajo en cada ciclo y tiene una eficiencia de 30%.
Para cada ciclo de operación,
Luego WA = Q1 − Q2 = 1500 – 1200 = 300 J a) ¿cuánto calor se absorbe?, y
Cálculo de WB b) ¿cuánto calor se libera?
Solución.
Q3 T3
= ⇒ Q1 calor absorbido de la fuente caliente
Q2 T2 Q2 calor cedido a la fuente fría
W = 200 J
T 400
Q3 = 3 Q2 = (1200) = 600 J W
T2 800 e= = 0,3
Q1
Luego WB = Q2 − Q3 = 1200 – 600 = 600 J
entonces
y el trabajo total del sistema 200
WS = Q1 − Q3 = 1500 – 600 = 900 J a) Q1 = = 666,7 J
0.3
Nota: observe que:
b) Q2 = Q1 − W = 666,7 − 200 = 466,7 J
WS = WA + WB y eS ≠ eA + eB
Ejemplo 117. En un determinado refrigerador las
Ejemplo 115. Una casa cerca de un lago se serpentinas de baja temperatura están a -10°C y el
calefacciona mediante una motor térmico. En gas comprimido en el condensador tiene una
invierno, el agua debajo del hielo que cubre el lago temperatura de + 30°C. Considerando que trabaja
se bombea por medio del motor térmico. Se extrae con el ciclo Carnot. ¿Cuál es su rendimiento teórico?
el calor hasta que el agua está en el punto de Solución.
congelar cuando se expulsa. El aire exterior se Q2 Q2 1 1
utiliza como enfriador. Asuma que temperatura del η= = = =
aire es -15°C y la temperatura del agua del lago es W Q1 − Q2 Q1 T1
−1 −1
2°C. Calcule la razón en la cual el agua se debe Q2 T2
bombear al motor. La eficiencia del motor es un 1 263
quinto que el de un motor de Carnot y la casa = = = 6,58
requiere 10 kilovatios. 303 40
−1
Solución. 263
La eficiencia de un motor Carnot es
[1 - (T1/T2)] .

60
Calor y Termodinámica Hugo Medina Guzmán

Ejemplo 118. Una máquina térmica absorbe 360 J W W Pt


de calor y realiza un trabajo de 25 J en cada ciclo. e= = =
Encuentre: Q1 W + Q1 Pt + Q1
a) la eficiencia de la máquina y o bien
b) el calor liberado en cada ciclo. 5 × 10 2 t
Solución. 0,25 =
Q1 = 360 J 5 × 10 2 t + 8000
W = 25 J De donde se obtiene t = 0,53 s el tiempo para cada
W 25 ciclo.
a) e = = = 0,069 = 6,9% El calor absorbido en cada ciclo será
Q1 360 Q1 = 5 × 10 2 t + 8000
b) QLiberado = Q1 − W = 335 J
= 5 × 10 (0,53) + 8000 = 1,065 x 104 J
2

Ejemplo 119. Una máquina térmica realiza 200 J de


Ejemplo 122. El calor absorbido por una máquina es
trabajo en cada ciclo y tiene una eficiencia de 30%. el triple del trabajo que realiza.
Para cada ciclo de operación,
a) ¿Cuál es su eficiencia térmica?
a) ¿cuánto calor se absorbe?, y
b) ¿Qué fracción del calor absorbido se libera a la
b) ¿cuánto calor se libera? fuente fría?
Solución.
Solución.
Q1 calor absorbido de la fuente caliente
Q2 calor cedido a la fuente fría Q1 = 3W
W = 200 J W 1
W a) e = = = 0,33 = 33%
e= = 0,3 Q1 3
Q1 Q 2
Entonces b) Q2 = Q1 − W = Q1 − 1 = Q1
3 3
200
a) Q1 = = 666,7 J Fracción del calor absorbido que se libera:
0,3 Q2 2
= = 0,66
Q1 3
b) Q2 = Q1 − W = 666,7 − 200 = 466,7 J
Ejemplo 123. Dos máquinas frigoríficas de Carnot
Ejemplo 120. Un refrigerador tiene un coeficiente trabajan en serie la primera extrae calor de una
de operación igual a 5. Sí el refrigerador absorbe fuente a 0°C y consume 1000 J. La segunda maquina
120 J de calor de una fuente fría en cada ciclo, consume 500 J. y entrega calor a una fuente a 27ºC
encuentre: Considere que el calor que la primera cede a una
a) el trabajo hecho en cada ciclo y fuente intermedia es íntegramente absorbido por la
b) el calor liberado hacia la fuente caliente. segunda.
Solución. a) ¿Cuál es el calor que la primera maquina extrae?
η =5 b) ¿Cuál es la temperatura de la fuente intermedia?
Q1 = 120 J c) ¿Qué calor intercambian las máquinas con la
Q1 W + Q2 fuente de temperatura intermedia?
a) η= = Solución.
W W
W + 120
De donde 5 = ⇒ W = 30 J
W
b) Q2 = W + Q1 = 30 + 120 = 150 J

Ejemplo 121. Cierta máquina tiene una potencia de


salida de 5 kW y una eficiencia de 25%. Si la
máquina libera 8000 J de calor en cada ciclo,
encuentre: a) Para el conjunto
a) el calor absorbido en cada ciclo y
b) el tiempo para cada ciclo.
Solución.
P = potencia = 5 kW = 5 x 103W
e = 25 % = 0,25
Q1 = 8000 J
Si t es el tiempo de un ciclo

61
Calor y Termodinámica Hugo Medina Guzmán

η=−
Q3
Q3 − Q1
=−
Q3
W1 + W2
=−
1
Q
=−
1
T
reversible. Uno puede razonar que ∫ (dQ T ) > 0
1− 1 1− 1 para un ciclo irreversible. Además, es posible
Q3 T3 ampliar este razonamiento a cualquier proceso que
Q3 1 lleve un sistema del estado A al estado B, con el
⇒ − =−
− (1000 + 500)
1−
300 resultado que. ΔS = S ( B ) − S ( A ) .= ∫ (dQ T ) .
273 Para un sistema aislado, esto se convierte ΔS = 0
1500 × 273 para un ciclo reversible y ΔS > 0 para un ciclo
Q3 = = 15166,7 J irreversible.
27 Esto significa que la entropía de un sistema aislado
b) Para R – 1 sigue siendo constante o aumenta. Puesto que los
Q3 Q 1 1 procesos verdaderos son todos irreversibles, esto
η1 = − =− 3 =− =−
Q3 − Q2 W1 Q2 T significa que la entropía del universo aumenta
1− 1− 2
Q3 T3 siempre en cada proceso.
15166,7 1 Ejemplo 124. Calcular el cambio en la entropía
⇒ − =−
− 1000 T para un gas ideal siguiendo un proceso en el cual lo
1− 2
273 lleve de p1 , T1 , V1 a p 2 , T2 , V2 según se
T2 muestra en la figura.
⇒ − 1 = 0,066 ⇒ T2 = 290,1 K
273
c) Q3 = 15166,7 J,
Q2 = Q3 − W1 = 15166,7 – (-1000) = 16166,7 J

ENTROPIA
Recordemos para el ciclo reversible de Carnot,
Q1 T1 Q1 Q2
= o − =0
Q2 T2 T1 T2 Solución.
Es posible aproximar cualquier ciclo reversible por No importa qué trayectoria siga, el cambio de la
una serie de ciclos de Carnot, y éste nos conduce a la entropía será igual puesto que S es una función del
conclusión que estado. Para simplificar el cálculo, elegiremos la
dQ trayectoria reversible mostrada, primero viajando a
∫ T
= 0 para un ciclo reversible. lo largo de una trayectoria isotérmica, y luego a lo
largo de una trayectoria a volumen constante. A lo
Esto recuerda a las fuerzas conservativas, donde
→ →
largo de la isoterma la temperatura no cambia, por lo
∫ F ⋅ d s = 0 para una trayectoria cerrada. Que nos tanto no hay cambio en energía interna.
(U = nCV T )
llevó a definir la energía potencial U donde
B → → Así dQ = dW para este proceso, y
U B − U A = ∫ F .d s . En este caso un estado del B dQ V2 dW
A
sistema fue caracterizado por un valor definido de U,
S (B ) − S ( A ) = ∫
A T
= ∫
V1 T1
la energía potencial. De la misma manera,
V2 nRTdV1
definimos una nueva variable del estado, la entropía pV = nRT , tal que S ( B ) − S ( A ) = ∫
S, tal que V1 VT1
dQ B dQ
dS = y S (B ) − S ( A) = ∫ V1
T A T S ( B ) − S ( A ) = nR ln
V2
Note que aunque un valor definido de Q no
caracteriza un estado (es decir, un punto en un Para B → C, no se realiza trabajo, luego
diagrama p V), cada punto en el diagrama p V tiene dQ = dU = nCV dT :
un valor definido de S. Es curioso que aunque el
T2 dQ T
flujo del calor en un sistema depende de la
trayectoria seguida entre los dos estados, el cambio
S (C ) − S ( B ) = ∫
B
C dQ =
T
∫ T1
CV
T
= nCV ln 2
T1
en S es independiente de la trayectoria. Decimos El cambio total de la entropía es
que dQ es un diferencial inexacto, y dS es un
diferencial exacto. ΔS = S ( B ) − S ( A ) + S (C ) − S ( B ) :
La ecuación anterior es cierta para un ciclo ΔS = S ( p 2 , V2 , T2 ) − S ( p1 , V1 , T1 )

62
Calor y Termodinámica Hugo Medina Guzmán

T2 V T1 + T2
= nCV ln + nR ln 2 del Universo es 2mc p ln y pruebe que
T1 V1 2 T1T2
es necesariamente positivo.
Ejemplo 125. Un kilogramo de agua a temperatura Solución.
de 280 K se mezcla con 2 kilogramos de agua a 310 El cambio de entropía del Universo será el cambio
K en un recipiente aislado térmicamente. Determine de entropía de la mezcla, es decir
el cambio en la entropía del Universo. Tf Tf
Solución. ΔS = m1c1 ln + m2 c 2 ln
Aquí, un proceso de mezclado T1 T2
Tf Tf donde (calorimetría) se tiene que
ΔS = m1c1 ln + m2 c 2 ln m1c1T1 + m2 c 2T2
T1 T2 Tf =
donde (por calorimetría) se tiene que m1c1 + m2 c 2
pero m1 = m2 = m y c1 = c2 = c por lo cual resulta
m1c1T1 + m2 c 2T2 T1 + T2
Tf = Tf =
m1c1 + m2 c 2 2
siendo Y
m1 = 1000 g T f2 Tf
T1 = 280 + 273,15 = 553,15K ΔS = mc ln = 2mc ln =
m2 = 2000 g T1T2 T1T2
T2 = 310 + 273,15 = 583,15K
T1 + T2
entonces 2mc ln
553,15 + 2 × 583,15 2 T1T2
Tf = = 573,15 K
3 Para probar que es positivo, debemos demostrar que
573,15 573,15 en general
y ΔS = 1000 ln + 2000 ln
553,15 583,15 x+ y
>1
cal 2 xy
= 0,92
K y esto se deduce de

Ejemplo 122. Una masa m de líquido a temperatura


( x− y ) 2
> 0 ⇒ x + y − 2 xy > 0 ⇒
T1 se mezcla con una igual cantidad del mismo x + y > 2 xy
líquido a temperatura T2 en un recipiente aislado
térmicamente. Demuestre que el cambio de entropía x+ y
Finalmente: >1
2 xy

PREGUNTAS Y PROBLEMAS

1. Un termómetro de gas a volumen constante se pueden ser directamente relacionadas con la


calibra en hielo seco (dióxido de carbono en estado resistencia R, medida en ohms. Un cierto termómetro
sólido, temperatura de -80 ºC) y en alcohol etílico en de resistencia tiene una resistencia R = 90,35 cuando
ebullición (temperatura de 78 ºC). su bulbo se coloca en agua, a temperatura del punto
Los valores de las presiones son 0,9 atm y 1,635 atm, triple (273,16 K). Determine a temperatura indicada
respectivamente. por el termómetro cuando su bulbo se coloca en un
Determine: medio tal que a su resistencia sea igual a:
a) El valor del cero absoluto obtenido de la a) 105, b) 96,28 .
calibración;
b) El valor de la presión en el punto de congelación 3. Un recipiente de vidrio está lleno hasta el borde de
del agua; mercurio a la temperatura de 0º y masa 1 kg. El
c) El valor de la presión en el punto de ebullición del recipiente vacío tiene una masa de 0,1 kg. Calcular la
agua. cantidad de mercurio a 100 ºC que puede contener
este recipiente. El coeficiente de dilatación cúbica del
2. En un termómetro de resistencia la propiedad mercurio es 1,8x10-4 ºC-1 y el del vidrio 3x10-5 ºC-1.
usada para medir a temperatura es la resistencia ρHg = 13,6 g/cm3 a 0 ºC.
eléctrica de un conductor. Las temperaturas medidas Respuesta. 887 g de Hg.
por este termómetro (en Kelvin o en grados Celsius)

63
Calor y Termodinámica Hugo Medina Guzmán

4. Un vástago de latón AB tiene una longitud de 10. Se ha de introducir un remache de hierro en una
200,1 mm y ha de encajarse exactamente en el hueco placa también de hierro y para conseguir un ajuste lo
BC, de hierro que tiene la forma del esquema. Al más perfecto posible se introduce el remache, antes
intentarlo queda AB como se indica en la figura, de meterlo en la placa, en aire líquido (-187 ºC). El
siendo AC = 4 mm. Calcular el descenso de la diámetro del orificio es de 10 mm. ¿Que diámetro
temperatura para lograr el encaje. Los coeficientes de tendrá que tener el remache a la temperatura ambiente
dilatación del latón y del hierro valen (20 ºC) para que después de meterlo en aire líquido
respectivamente, entre justamente por el orificio de la placa?
α = 19,9 x10 ºC-1 y α' = 12,1x10-6 ºC-1. Coeficiente de dilatación lineal del hierro:
α = 12x10-6 ºC-1.
Respuesta. 10,025 mm.

11. Un recipiente a 0 ºC contiene la tercera parte de


su volumen de mercurio.
Respuesta. 25,6 ºC. Se calienta a una cierta temperatura y entonces el
mercurio ocupa el 34,37 por 100 del volumen del
5. Un anillo de latón de varios centímetros de vaso. ¿Cuál es dicha temperatura?
diámetro se calienta hasta la temperatura t1 = 300 ºC Coeficiente de dilatación del mercurio
y se encaja ajustadamente sobre un cilindro de acero γ = 18x10-5 ºC-1.
cuya temperatura es t2 = 18 ºC. ¿Qué esfuerzo de Coeficiente de dilatación del recipiente
rotura experimentará el anillo una vez enfriado hasta γ' = 25x10-6 ºC-1.
18 ºC? El coeficiente de dilatación lineal del latón es Respuesta. 202 ºC.
α = 1,84x10-5 ºC-1 y su módulo de Young Y =
6,47x1010 Nm-2. Las dimensiones de la sección del 12. ¿Que fuerzas hay que aplicar a los extremos de
anillo son 2x5 mm. una barra de acero, cuya sección transversal tiene el
Respuesta. 3,364 N. área S = 10 cm2, para impedir que se dilate cuando se
calienta desde t1= 0 ºC hasta t2 = 30 ºC?
6. Con una regla métrica de latón cuyas dimensiones Respuesta. 68,688 N.
son exactas a 0 ºC, se ha medido la longitud de una
barra de hierro, encontrándose l = 1,4996 m a 38 ºC. 13. De un alambre de 1 mm de radio cuelga una
Siendo α = 12,1x10-6 ºC-1 el coeficiente de dilatación carga. Esta carga hace que el alambre se alargue en la
lineal del hierro y β= 19,9x10-6 ºC-1 el del latón, misma magnitud que se alargaría sí se elevara 20 ºC
calcular la longitud a 0 ºC de la barra de hierro. su temperatura.
Respuesta. 1,500 m. Hallar la magnitud de la carga.
Respuesta. 148 N.
7. Si la temperatura del ambiente en que se encuentra α = 1,2x10-5 ºC-1
un reloj de péndulo que bate segundos se modifica en Y = 19,6x1010 N.m-2
20 ºC, ¿qué le pasará al reloj al cabo de 30 días si el
coeficiente de dilatación lineal del péndulo es 20x10-6 14. Un alambre de hierro se tendió entre dos paredes
ºC-1? fijas resistentes, estando la temperatura a 150 ºC ¿A
Respuesta. 8 min. 38 s. se atrasa. qué temperatura se romperá el alambre al enfriarse?
Suponer que la ley de Hooke se cumple hasta el
8. Una bola de acero de 6 cm de diámetro tiene 0,010 momento en que se produce la rotura.
milímetros más de diámetro que el correspondiente al α = 1,2x10-5 C-1
orificio de una plancha de latón donde se debe alojar Resistencia a la rotura F/S = 2,94x108 N/m2
cuando tanto la bola como la plancha están a una Módulo de Young Y = 19,6x1010 N/m2
temperatura de 30 ºC. A qué temperatura, tanto de la Respuesta. 25 ºC.
bola como de la plancha, podrá pasar la bola por el
orificio. 15. Unos carriles de acero de 18 m de longitud se
El coeficiente de dilatación lineal del acero vale colocan un día de invierno en que la temperatura es -6
12x10-6 ºC-1 y el del latón 19x10-6 ºC-1. ºC. ¿Qué espacio ha de dejarse entre ellos para que
Respuesta. 54 ºC. estén justamente en contacto un día de verano en que
la temperatura es 40 ºC. Coeficiente de dilatación del
9. Una vasija de vidrio está llena justamente con 1 acero α = 12x10-6 ºC-1?
litro de terpentina a 50 ºF. Hallar el volumen de Respuesta. 9,936x10-6 m.
líquido que se derrama si se calienta hasta 86 ºF.
El coeficiente de dilatación lineal del vidrio vale 16. La varilla de un reloj de péndulo sin compensar,
9x10-6 ºC-1 y el de dilatación cúbica de la terpentina que bate segundos a 0 ºC
97x10-5 ºC-1. es de latón. Averiguar cuanto se retrasa el reloj en un
Respuesta. 18,86 cm3. día si se introduce en un ambiente a 200 ºC.

64
Calor y Termodinámica Hugo Medina Guzmán

Coeficiente de dilatación del latón: 17x10-6 ºC-1 es 400 K. ¿El calor neto radiado, incluyendo la
(Considerar el péndulo como simple). dirección, en el espacio entre las esferas y la cáscara
Respuesta. 7 m. 12 s. es?

17. Un herrero ha de colocar una llanta circular de 22. Un proyectil de plomo choca contra un obstáculo.
hierro de 1 m de diámetro a una rueda de madera de ¿Cuál es la velocidad en el momento del choque sí su
igual diámetro. Con objeto de poder ajustarla, temperatura inicial era de 65 ºC y se funde la tercera
calienta la llanta hasta conseguir que su radio supere parte? Se supone el obstáculo inamovible e
en 2 mm al de la rueda. Sabiendo que la temperatura inalterable. Calor específico del plomo 0,031 cal/g
ambiente es de 20 ºC y su coeficiente de dilatación ºC.
lineal 12,2x10-6 ºC-1. Temperatura de fusión: 327,4 ºC; calor de fusión:
Calcular: 5,74 cal/g.
a) Temperatura en grados centígrados a que debe Respuesta. 289,93 m/s.
calentarse la llanta para cumplir las condiciones
expuestas. 23 Se lanza una esfera de plomo cuya temperatura
b) Expresar esta temperatura en grados Fahrenheit y inicial es de 36 ºC, verticalmente y hacia abajo con
en grados absolutos. una velocidad v0; 100 metros más abajo encuentra un
Respuesta. a) 347 ºC; b) 656,6 ºF, 620 K. plano absolutamente resistente de conductividad
calorífica nula. Calcular el valor de v 0 necesario para
18. Una vasija de cinc (coeficiente de dilatación
que la esfera se funda totalmente en el choque. Calor
lineal: 29x10-6 ºC-1), está llena de mercurio a 100 ºC;
teniendo entonces una capacidad de 10 litros. Se específico del plomo c = 0,031 cal/g ºC.
enfría hasta 0 ºC. Calcular la masa de mercurio a 0 ºC Temperatura de fusión del plomo t = 327,4 ºC.
Calor de fusión del plomo = 5,74 cal/g;
que hay que añadir para que la vasija quede
completamente llena 1 cal = 4,186 J; g = 9,8 m/s2.
(Coeficiente de dilatación cúbico del mercurio: Respuesta. 348,7 m/s.
182x10-6 ºC-1). Densidad del mercurio a 0 ºC 13,6
g/cm3. 24. Una masa de plomo igual a 10 g llega
Respuesta. 1,258 g. horizontalmente, con una velocidad de 250 m/s sobre
una esfera de plomo de 450 g, en la cual se incrusta.
19. La pared de concreto de un frigorífico mide 3,0 a) Estando, al principio, la esfera de plomo
m de alto, 5,0 m de ancho, y 20 cm de espesor. La inmovilizada, calcular el calentamiento que resultará
temperatura se mantiene en –10º C y la temperatura del choque.
exterior es 20º C. La pared interior está cubierta por b) Pudiéndose separar la esfera de plomo de la
vertical como un péndulo, se comprueba en una
una capa de lana para reducir el flujo de calor a través
de la pared por 90 %. Las conductividades térmicas segunda experiencia que se eleva 2 metros después
del concreto y de la lana son 0,8 y 0,04 W/m.K, del choque. Calcular el calentamiento resultante. CPb=
0,03 cal/g.
respectivamente.
a) ¿Cuál es la diferencia de temperaturas de la capa Respuesta. a) 5,4 ºC; b) 5,2 ºC.
de lana?
25. En un calorímetro sin pérdidas cuyo equivalente
b) ¿Cuál es el espesor de capa de lana requerido?
en agua es de 101 g y cuya temperatura inicial es de
20. Dos placas paralelas grandes están separadas por 20 ºC, se añaden 250 cm3 de agua a 40 ºC, 100 g de
hierro a 98 ºC (calor específico = 0,109 cal/g ºC) y 80
0,5 m. Un círculo de 1,5 m de radio se delinea sobre
la placa de la izquierda. Un segundo círculo, del g de hielo fundente. Calcular la temperatura de
mismo radio y opuesta a la primera, se delinea sobre equilibrio.
Respuesta. 15,1 ºC.
la placa de la derecha. La temperatura de la placa de
la izquierda es 700 K y la emisividad es 1,00. La
temperatura de la placa de la derecha es 600 K y la 26. Dentro de un calorímetro que contiene 1.000 g de
agua a 20 ºC se introducen 500 g de hielo a -16 ºC. El
emisividad es 0,80.
a) ¿El calor neto radiado entre los dos círculos es? vaso calorimétrico es de cobre y tiene una masa de
b) La temperatura de la placa izquierda se mantiene 278 g.
en 700 k. La temperatura de la placa derecha se Calcular la temperatura final del sistema, suponiendo
cambia, tal que ahora el flujo de calor neto radiado es que no haya pérdidas.
Calor específico del hielo: 0,55 cal/g ºC
cero, en el espacio entre los círculos. ¿Cuál es la
temperatura de la placa de la derecha? Calor específico del cobre: 0,093 cal/g ºC
Calor de fusión del hielo: 80 cal/g
Calor de vaporización del agua: 539 cal/g
21. Una esfera de 0,30 m de radio, tiene una
emisividad de 0,48 y su temperatura es de 600 K. La Respuesta. 0 ºC no se funde todo el hielo; 201 g.
esfera se rodea de una cáscara esférica concéntrica
27. En un calorímetro de latón sin pérdidas, de 240
cuya superficie interior tiene un radio de 0,90 m y
una emisividad de 1,00. La temperatura de la cáscara g, que contiene 750 cm3 de agua a 20,6 ºC se echa

65
Calor y Termodinámica Hugo Medina Guzmán

una moneda de oro de 100 g a 98 ºC y la temperatura Calor específico del metal 0,1 cal/g ºC.
sube a 21 ºC. Respuesta. 47,8 ºC.
Determinar la cantidad de oro y cobre que integra la
moneda. Calor específico del latón: 0,09 cal/g ºC; 34. Un calorímetro de latón de M1= 125 g contiene
calor específico del cobre: 0,0922 cal/g ºC; calor un bloque de hielo de M2 = 250 g todo ello a t1= -15
específico del oro: 0,031 cal/g ºC. ºC.
Respuesta. 85,16 g de oro; 14,84 g de cobre. Calcular la cantidad de vapor de agua a 100 ºC y a la
presión normal que es necesario para que todo el
28. En un calorímetro de cobre se queman sistema llegue a la temperatura de t = 15 ºC.
exactamente, 3 g de carbón produciéndose CO2. La Calor específico del latón: 0,09 cal/g ºC
masa del calorímetro es de 1,5 kg y la masa de agua Calor específico del hielo: 0,5 cal/g ºC
del aparato es 2 kg. Calor de fusión del hielo: 80 cal/g
La temperatura inicial de la experiencia fue de 20 ºC Calor de vaporización del agua: 540 cal/g
y la final de 31 ºC. Hallar el poder calorífico del Respuesta. 41,54 g.
carbón expresándolo en cal/g. El calor específico del
cobre vale 0,093 cal/g ºC. 35. En un recipiente de aluminio de 256 g que
Respuesta. 7,8x103 cal/gr. contiene 206 g de nieve a -11 ºC se introducen 100 g
de vapor de agua a 100 ºC. Calcular la temperatura
29. En un calorímetro cuyo equivalente en agua es final de la mezcla.
despreciable, hay 1 kg de hielo a -10 ºC. ¿Cuantos Calor específico del aluminio: 0,219 cal/g ºC
gramos de agua a 80 ºC hay que introducir en él para Calor específico del hielo: 0,5 cal/g ºC
que la temperatura final sea de 10 ºC? Sí en lugar de Calor de fusión del hielo: 80 cal/g
agua a 80 ºC, se introduce vapor de agua a 100 ºC, Calor de vaporización del agua: 539 cal/g
¿Cuántos gramos de éste habría que introducir para Respuesta: Solo se condensa parte del vapor y la
que la temperatura final sea de 40 ºC? ¿Que volumen temperatura final será de 100 ºC. Vapor condensado
ocupa el vapor de agua introducido, si la presión a 82,4 gramos.
que se mide es de 700 mm de mercurio? Peso
molecular del agua 18. 36. Una bala de plomo atraviesa una pared de
Calor específico del hielo (de -20 a 0 ºC): 0,5 cal/g ºC madera. Antes de chocar con la pared la velocidad de
Calor de vaporización del agua: 540 cal/g la bala era v 0 = 400 m/s y después de atravesarla v
Respuesta. 1,357 g; 208 g; 384 litros.
= 250 m/s. La temperatura de la bala antes del choque
30. Mezclamos 1 kg de agua a 95 ºC con un kg de era t 0 = 50 ºC. ¿Qué parte de la bala se fundirá?
hielo a -5 ºC. Calor de fusión del plomo: 5,74 cal/g
¿Dispondremos de suficiente calor para fundir todo el Temperatura de fusión del plomo: 327 ºC
hielo? Sí es así, ¿a qué temperatura queda la mezcla? Calor específico del plomo: 0,031 cal/g ºC
Calor específico del hielo: 0,5 cal/g ºC Suponer que todo el calor que se desprende lo recibe
Calor de fusión del hielo: 80 cal/g la bala.
Respuesta. Se funde todo el hielo, 6,25 ºC. Respuesta. 0,53.

31. Una bola de plomo (calor específico: 0,03 cal/g 37. En un calorímetro sin pérdidas cuyo equivalente
ºC) de 100 g está a una temperatura de 20 ºC. Se en agua es de 500 g, hay 4,500 g de agua a 50 ºC. Se
lanza verticalmente hacia arriba con una velocidad añaden 2 kg de hielo fundente y se introduce 1 kg de
inicial de 420 m/s vapor de agua a 100 ºC. El calor de fusión vale 80
y al regresar al punto de partida choca con un trozo cal/g y el de vaporización 540 cal/g. Calcular la
de hielo a 0 ºC. ¿Cuanto hielo se funde? temperatura de equilibrio.
Se supone que toda la energía del choque se convierte Respuesta. 91,25 ºC.
íntegramente en calor.
Respuesta. 27 g. 38. Un cubo de hielo de 20 g a 0 ºC se calienta hasta
que 15 g se han convertido en agua a 100 ºC y 5 g se
32. Un vaso cuya capacidad calorífica es despreciable han convertido en vapor. ¿Cuanto calor se necesitó
contiene 500 g de agua a temperatura de 80 ºC. para lograr esto?
¿Cuantos gramos de hielo a la temperatura de -25 ºC Respuesta. 6300 cal.
han de dejarse caer dentro del agua para que la
temperatura final sea de 50 ºC? 39. En un recipiente se almacenan 2 litros de agua a
Respuesta. 105 gramos de hielo. 20 ºC. Inmersas en el agua se encuentran dos barras:
una de latón de 5 cm de largo y 200 g y otra de hierro
33. Una bola, a una velocidad de 200 m/s, choca de idénticas dimensiones y 250 g.
contra un obstáculo. a) Hallar la cantidad de calor necesaria para calentar
Suponiendo que toda la energía cinética se transforma todo el conjunto (agua y barras) justo hasta que todo
en calor y que éste calienta tan solo la bola, calcular el agua se convierta en vapor a 100 ºC (calor
su elevación de temperatura.

66
Calor y Termodinámica Hugo Medina Guzmán

específico del latón y hierro: 0,09 cal/gºC y 0,11 p0 = 1,875ρgh


cal/gºC respectivamente).
b) Determinar las longitudes de ambas barras en esas = 1,875(13600 )(9,80 )(0,10 )
condiciones (coeficiente lineal de dilatación de latón = 24990 Pa
y hierro: 1,9x10-5 ºC-1 y 1,2x10-5 ºC-1
respectivamente). 42. Un tubo cilíndrico de medio metro de longitud se
c) ¿Cuál es más denso a 20 ºC, el latón o el acero? ¿Y introduce en mercurio hasta su mitad; después se tapa
a 100 ºC? el extremo superior y se retira. Calcular la longitud de
Respuesta. a) Q = 5,2x106 J; b) Llatón = 0,050076 m, mercurio que quedará en el tubo y la presión del aire
Lhierro = 0,050048 m. encerrado sobre él. La presión atmosférica es de 76
c) A 20 ºC y 100 ºC es más denso el hierro. cm de mercurio.
Respuesta. 58,45 cm Hg.
40. En un recipiente se mezclan 4,5 litros de agua a
20 ºC y 500 g de hielo 43. El peso de un metro cúbico de cierto gas a la
a 0 ºC. Se introduce en el recipiente una barra de temperatura de t = 67 ºC y presión p = 100 mm de
metal, de capacidad calorífica despreciable. mercurio es m = 282,32 g. Calcular la pérdida de peso
a) ¿Cuál es la temperatura en el equilibrio? que experimentaría un cuerpo sumergido en este gas a
b) El conjunto se calienta en un hornillo que una cierta presión y temperatura sabiendo que en
proporciona 5,000 cal/s, ¿cuál es la temperatura a los estas condiciones pierde en el aire 4,839 g.
100 s? La longitud de la barra a 0 ºC es de 10 cm y su ρ aire = 1,293 g/litro
coeficiente de dilatación lineal es de 2x10-5 ºC-1. Respuesta. 10,001 g.
c) Obtener una expresión de la longitud de la barra en
función del tiempo hasta t = 100 s. 44. Un depósito contiene 50 kg de oxígeno a la
Respuesta. a) t = 10 ºC, b) tfinal = 100 ºC. presión p1= 10 atm y a la temperatura t1= 27 ºC. Se
produce una fuga por donde escapa oxígeno y al cabo
41. Un tubo capilar de longitud 50 cm está cerrado en de cierto tiempo, localizada y tapada la fuga, la
ambos extremos. Contiene aire seco y cada extremo presión y la temperatura del depósito resultan ser
está separado por una columna de mercurio de 10 cm p2= 6 atm y t2= 20 ºC.
de largo. Con el tubo en posición horizontal, las ¿Que cantidad de oxígeno ha escapado?
columnas de aire son de 20 cm de largo, y con el tubo Respuesta. 19,3 kg.
en posición vertical son de 15 cm y 25 cm. ¿Cuál es
la presión en el tubo capilar cuando está horizontal? 45. Un frasco de 5 litros de volumen se tapa en un
recinto cuya presión es de 762 mm de Hg y cuya
temperatura es de 27 ºC. Luego se abre en un lugar
donde la presión es de 690 mm y la temperatura 9 ºC.
¿Entra o sale aire? Calcular el peso de aire que entra
o sale.
Respuesta. 0,1905 salen; 0,2165 g.

46. Calcular en gramos el peso del hidrógeno H2


Solución. contenido en un recipiente de 5 galones que está a la
Para el aire del aparte inferior presión de 14 psi y a la temperatura de 86 ºF.
p0V0 = p1V1 (1) Respuesta: 1,462 g.
Para el aire del aparte superior 47. Un recipiente cuyo volumen es igual a 5 litros,
p0V0 = p2V2 (2) contiene aire a 27 ºC de temperatura y a la presión de
En el tubo vertical 20 atm. ¿Que masa de aire hay que liberar del
p1 = p2 + ρgh (3) recipiente, para que la presión de éste caiga a 10 atm?
Respuesta. 59 g.
p1V1 = p2V2 (4)
De (1) y (2) 48. Calcular el trabajo que realiza un gas cuando se
( p2 + ρgh )A15 = p2 A25 ⇒ calienta isobáricamente desde los 20 ºC hasta 100 ºC,
si se encuentra dentro de un recipiente cerrado por
3( p2 + ρgh ) = 5 p2 ⇒ medio de un émbolo móvil, cuya sección es igual a
p2 = 1,5 ρgh 20 cm2 y su peso 5 kgf. Analizar dos casos:
a) cuando el recipiente se encuentra en posición
Reemplazando en (2) los valores de p2 horizontal y
V0 = A20 , V2 = A25 b) cuando el recipiente se encuentra en posición
vertical. El volumen inicial del gas es igual a 5 litros,
p0 A20 = 1,5ρghA25 y la presión atmosférica es la normal.
p0 A20 = 1,5ρghA25 Respuesta. a) 138 J; b) 172 J.

67
Calor y Termodinámica Hugo Medina Guzmán

49. Un tubo con su extremo superior cerrado es a) Si el motor tiene un rendimiento del 25%, ¿qué
sumergido completamente en un recipiente que trabajo total realiza el motor durante los 10 km del
contiene mercurio, después de lo cual, dentro del tubo recorrido?
queda una columna de aire de 10 cm de longitud. ¿A b) Si se supone que este trabajo se realiza contra una
que altura sobre el nivel del mercurio en el recipiente fuerza resistente constante F, hállese la magnitud de
hay que levantar el extremo superior del tubo para F.
que dentro de éste el nivel del mercurio quede igual al Respuesta. a) 8,05x106 J; b) 0,805 N.
nivel del mercurio en el recipiente. La presión
atmosférica es la normal. Calcular la masa de aire 56. En el ciclo que se muestra en la figura, un mol de
dentro del tubo, si su sección es igual a 1 cm2 y la un gas diatómico ideal (γ = 1,4) se encuentra
temperatura igual a 27 ºC. inicialmente a 1 atm y 0 ºC. El gas se calienta a
Respuesta. 11,3 cm; 13,3 mg. volumen constante hasta
t2 = 150 ºC y luego se expande adiabáticamente hasta
50. ¿Que cantidad de calor se desprenderá al que su presión vuelve a ser 1 atm.
comprimir por vía reversible e isoterma 100 litros de Luego se comprime a presión constante hasta su
un gas ideal a 27 ºC que se encuentran a 71 cm de estado original. Calcular:
mercurio de presión, hasta reducir su volumen a la a) La temperatura t3 después de la expansión
centésima parte? adiabática.
Respuesta. 10418 cal. b) El calor absorbido o cedido por el sistema durante
cada proceso.
51. Cien litros de oxígeno a 20 ºC y 69 cm de c) El rendimiento de este ciclo.
mercurio de presión se calientan d) El rendimiento de un ciclo de Carnot que operara
a volumen constante comunicando 2555 calorías. entre las temperaturas extremas del ciclo.
Calcular el incremento de la presión en cm de CV = 5 cal/molºC; Cp = 7 cal/molºC
mercurio.
Respuesta. 31,87 cm Hg.

53. Un tanque contiene 2,73 m3 de aire a una presión


de 24,6 kg/cm2. El aire se enfría hasta ser su presión
de 14 kg/cm2. ¿Cuál será la disminución de su energía
interna?
Considérese el aire como gas perfecto biatómico de
índice adiabático γ = 1,4.
Respuesta. 1,420x106 cal.
Respuesta. a) 373 ºK ; b) -2,93 kJ; c) 6,69 %; d) 35
53. Cinco moles de un gas perfecto diatómico a 27 %
ºC se calientan isobáricamente con el calor que se
desprende de un mol de otro gas perfecto que se 57. Un mol de gas N2 (CV = 5/2R; γ = 1,4) se
comprime isotérmicamente a 27 ºC hasta triplicar su mantiene a la temperatura ambiente (20 ºC) y a una
presión. Calcular la temperatura final del primer gas. presión de 5 atm. Se deja expandir adiabáticamente
Respuesta. 318,8 K = 45,8 ºC. hasta que su presión iguala a la ambiente de 1 atm.
Entonces se calienta a presión constante hasta que su
54. Se comprime adiabáticamente un mol de cierto temperatura es de nuevo de 20 ºC. Durante este
gas perfecto (índice adiabático γ = 1,15) que se calentamiento el gas se expansiona. Una vez que ha
encuentra a p1= 1 atm, t1 = 127 ºC hasta alcanzar una alcanzado la temperatura ambiente, se calienta a
presión p2. volumen constante hasta que su presión es de 5 atm.
Después se deja enfriar a volumen constante hasta Se comprime entonces a presión constante hasta
alcanzar las condiciones p3 = 10 atm y t3 = 27 ºC. volver a su estado original.
Calcular: a) Construir un diagrama pV exacto, mostrando cada
a) La presión p2 en atmósferas. etapa del ciclo.
b) El trabajo en la compresión adiabática. b) A partir de este gráfico determinar el trabajo
c) La cantidad de calor en calorías cedidas durante el realizado por el gas en todo el ciclo.
enfriamiento. c) ¿Cuánto calor fue absorbido o cedido por el gas en
Respuesta. a) 48,7 atm; b) 1,8x109 J; b) 4,621 cal. el ciclo completo?
R=0,082 litro.atm/mol K = 1,98 cal/mol K
55. Supóngase que 1 litro de gasolina propulsa un Respuesta. b) -65,1 litro.atm; c) -1.572,5 cal
automóvil una distancia de 10 km. La densidad de la
gasolina es aproximadamente 0,7 g/cm3, y su calor de 58. Una máquina de vapor con potencia de 14,7 kW
combustión es aproximadamente 4,6 x 104 J/g. consume durante 1 h de funcionamiento 8,1 kg de
carbón, cuyo calor específico de combustión es de
3,3x107 J/kg. La temperatura en la caldera es do 200
o
C, en la máquina frigorífica, 58oC. Hállese el

68
Calor y Termodinámica Hugo Medina Guzmán

rendimiento real de la máquina y compárese cl (hogar) y t2 = 20 ºC (refrigerante). El hogar comunica


resultado con el rendimiento de una máquina térmica al sistema 60 kcal por minuto. Calcúlese la potencia
ideal. del motor en caballos de vapor.
Respuesta. e ≈ 19,8% eo = 30% Respuesta. 2,16 C.V.

63. 8,1 kg de carbón de valor calorífico igual a


59. Un cuerpo calentado con temperatura inicial T1 se
3,3x107 J/kg. La temperatura de la caldera es de 200
aprovecha como calentador en una máquina térmica. ºC y la del condensador de 58 ºC. Hallar el
La capacidad calorífica del cuerpo no depende de la rendimiento real de la máquina e1y compararlo con el
temperatura y es igual a C. Un medio ilimitado, cuya
rendimiento e2 de la máquina térmica ideal que
temperatura es constante e igual a T0, sirve de funcione según el ciclo de Carnot entre las mismas
máquina frigorífica. Hállese el trabajo máximo que temperaturas.
puede obtenerse por cuenta del enfriamiento del Respuesta. 0,20; 0,30.
cuerpo. Realícese el cálculo para 1 kg de agua
hirviendo y de hielo que se derrite.
64. En una nevera de compresión se trata de fabricar
Respuesta. 5 kg de hielo cada hora, partiendo de agua a 0 ºC. El
⎡ ⎛ T ⎞⎤ ambiente exterior está a 27 ºC.
W = C ⎢T1 − T0 − T0 ln⎜⎜ 1 ⎟⎟⎥ ≈ 62 J Calcular:
⎣ ⎝ T0 ⎠⎦ a) La eficacia de la nevera.
b) La potencia teórica del motor.
60. Con ayuda do un hornillo e1éctrico de potencia. c) La potencia real si el rendimiento de la operación
de 1 kW en la habitación se mantiene la temperatura es el 75%.
de 17oC siendo la temperatura del aire circundante de d) El costo de la energía eléctrica necesaria para
–23oC. ¿Qué potencia se necesitaría para mantener en fabricar 100 kg de hielo a 5 soles el kW h.
la habitación la misma temperatura con ayuda de una Respuesta. a) 10; b) 46 W; c) 61 w; 4d) 6,10 soles.
bomba térmica ideal?
Respuesta. P = 138W 65. Una cierta máquina térmica ideal en la que se
realiza un ciclo de Carnot reversible en cada segundo,
61. Hállese el rendimiento de los ciclos mostrados en tiene el refrigerante a 27 ºC, una potencia de 4,18 kW
la figura, sí como agente propulsor se toma un gas y en cada ciclo se toman 3 kcal de la caldera. Calcular
monoatómico perfecto. la temperatura de ésta, el calor que se cede al
refrigerante y el rendimiento.
Respuesta. 2,000 cal; 177 ºC; 1/3.

66. En un ciclo de Carnot reversible, descrito por un


mol de un gas perfecto diatómico, la temperatura más
elevada es de 500 K y el trabajo en la expansión
adiabática 4,157 J. Calcular el rendimiento del ciclo.
Respuesta. 0,4.

67. Un refrigerador está impulsado por un pequeño


motor cuya potencia útil es de 150 W. Si suponemos
que este refrigerador trabaja como un refrigerador
ideal de Carnot, y que las temperaturas caliente y fría
de los recipientes térmicos son 20 y -5 ºC, ¿cuanto
hielo fabricará este refrigerador en 1 h si en el interior
se coloca agua a 10 ºC?
Respuesta. 15,4 kg.
Respuesta. 68. Tres kilogramos de agua a 18 ºC, se mezclan con
2
9 kg a 72 ºC. Una vez establecido el equilibrio, se
⎛V ⎞ 3
e = 1 − ⎜⎜ 1 ⎟⎟ restituyen las dos cantidades de agua a su estado
⎝ V2 ⎠ inicial colocando 3 kg en contacto con una fuente
térmica siempre a 18 ºC, y los 9 kg restantes en otra
2(T2 − T1 ) ln⎛⎜ 2 ⎞⎟
p siempre a 72 ºC.
e= ⎝ p1 ⎠ Calcular:
a) El incremento de la entropía del agua como
5(T2 − T1 ) + 2T2 ln⎛⎜ p 2 ⎞⎟ consecuencia del primer proceso y el incremento de
⎝ p1 ⎠ entropía del universo.
b) El incremento de entropía del agua producido por
62. Un motor térmico funciona mediante un ciclo de todas las operaciones y el del universo.
Carnot reversible entre las temperaturas t1 = 200 ºC

69
Calor y Termodinámica Hugo Medina Guzmán

c) El incremento de entropía del agua debido al Calor específico del acero 0,107 cal/g ºC
segundo proceso y el del universo. Respuesta. a) 26,14 ºC; b) –959 J/ K; c) 1,736 J/K;
Respuesta. a) 0,0315 kcal/ K que también es la del d) 777 J/ K; e) –1,736 J/ K; -18,6 J/ K; f) 1,754 J/ K
universo;
b) 0,0653 kcal/ K, la del agua 0; 71. Una máquina térmica trabaja con un gas perfecto
c) -0,0315 kcal/ K del agua, 0,0338 kcal/ K universo. (γ = 1,4) según el ciclo Otto, motores de explosión.
¿Cuánto vale el rendimiento térmico de este ciclo,
69. Un congelador fabrica cubos de hielo a razón de 5 para un estado inicial de p1 = 1 atm. T1 = 20 ºC y un
gramos por segundo, comenzando con agua en el grado de compresión V2/V1 = 1/4, si la combustión
punto de congelación. Cede calor a una habitación a aporta Q1 = 20 kcal/ciclo?
30 ºC. Si el sistema utiliza un frigorífico de Carnot ¿Cuánto vale el calor evacuado Q2?
ideal, ¿Cuánto valdrá la potencia de la máquina si realiza
a) ¿Qué potencia expresada en watios requiere el 300 ciclos por minuto?
motor?;
b) ¿Cuanto calor por unidad de tiempo cede a la
habitación?;
c) ¿Cual es la variación de entropía del agua?
Respuesta. a) 184 W; b) 444 cal/s; c) 6,15 J/ K.

70. Un herrero sumerge una herradura de acero


caliente con una masa de 2 kg en una cubeta que 72. Se dispone de botellas de 1,5 litros de agua a
contiene 20 kg de agua. La herradura al principio está temperatura ambiente (20 ºC);
a una temperatura de 600 ºC y el agua está a) calcular la temperatura final del conjunto si se
inicialmente a una temperatura de 20 ºC. Suponiendo mezcla una botella con 100 g de hielo a -5 ºC;
que no se evapora el agua, encuentre: b) calcular el calor necesario para evaporar toda el
a) la temperatura final del agua, agua de una botella; hallar el tiempo que requiere este
b) el cambio de entropía de la herradura, proceso si se usa un microondas de 100 W;
c) el cambio de entropía del agua c) hallar la eficiencia de una máquina de Carnot que
d) el cambio global en la entropía del agua y la utiliza el vapor a 100 ºC como foco caliente y agua a
herradura. 20 ºC como foco frío; dibujar un esquema de una
e) Después de cierto tiempo, que es bastante máquina de vapor en el que se explique cómo se
comparado con el tiempo que tarda la herradura en obtiene el trabajo mecánico.
enfriarse, la herradura y el agua se enfrían hasta la Respuesta. a) t = 13,6 ºC;
temperatura de los alrededores: 20 ºC. Durante este b) 930,000 cal = 3887,400 J, tiempo = 3.887,4 s;
proceso, encuentre los cambios en la entropía del c) Eficiencia = 21 %.
agua, la herradura y sus alrededores.
f) Usando los resultados del inciso d y e, encuentre el
cambio en la entropía del universo como resultado de
toda la consecuencia de eventos.

70
FISICA 3

Autor: Hugo Medina Guzmán


Profesor de la Pontificia Universidad Católica del Perú
PRESENTACIÓN

Me agradó saber que Hugo Medina Guzmán estaba por publicar un texto sobre Física.
Había dos razones suficientes para este sentimiento. Por un lado, tenía curiosidad de
saber lo que podría aportar un texto más de Física sobre los otros ya disponibles. Por
otro lado, conozco de la larga carrera de Hugo Medina como cultor de la enseñanza de
[a Física, y tenía curiosidad de ver cómo este compromiso como docente y experiencia
se manifestarían en su texto. Tuve la suerte de conocer al Ing. José Castro Mendívil en
su taller, donde desplegó una destacada labor en el diseño y construcción de equipo de
laboratorio para la enseñanza de la Física. Considero que Hugo es un digno discípulo
del Ing. Castro Mendívil e igualmente ha dedicado una fracción considerable de su
tiempo a la docencia, y al diseño y construcción de equipo de laboratorio para resaltar
los conceptos básicos de la Física.
He revisado el contenido de este texto y veo con gran satisfacción que su autor utiliza
un enfoque muy acertado. Toma como punto de partida una observación experimental y
a partir de allí desarrolla los conceptos físicos que permiten interpretar esta observación
utilizando la formulación matemática más sencilla. Todo esto lo hace con el detalle
suficiente de manera que el lector pueda seguir el argumento lógico con facilidad.
Considero que éste es un gran aporte de este texto. Este enfoque contrasta con textos
que enfatizan la formulación matemática y dejan al alumno huérfano de una orientación
para aplicarla a una realidad física concreta.
El contenido de temas de la Física General que son desarrollados en este texto se ajusta
al programa de estudios de la PUCP. El desarrollo de cada tema incluye ejemplos bien
seleccionados que son desarrollados con un detalle muy esmerado. Al final de cada
capítulo se incluye un conjunto de preguntas y problemas propuestos; se incluye las
respuestas. Algunos problemas plantean configuraciones complejas pero que contienen
ciertas propiedades de simetría que permiten su reducción a configuraciones sencillas.
Al final del texto encontramos un listado de referencias bibliográficas a un buen número
de textos de Física General que han servido de consulta al autor.
En general, considero que este texto constituye una representación gráfica de la obra
cotidiana que Hugo ha venido desarrollando durante su carrera docente y, por lo tanto,
es un aporte muy valioso para la comunidad académica y público en general.

Lima, julio de 2007


PRÓLOGO

Los estudiantes a menudo se preguntan por qué llevan un curso de Física. La mejor razón por la
que se estudia Física es porque proporciona un método coherente y lógico para comprender el
mundo que nos rodea; una persona que comprende lo que sucede a su alrededor, es capaz de
convivir en su entorno de manera racional y efectiva. Sin embargo, en ocasiones los estudiantes
ignoran el potencial que tiene la Física para explicar el entorno en términos fáciles de entender;
Este libro tiene por objeto brindar a los estudiantes de la Física General una ayuda para
dominar los principios físicos que son la base de la tecnología moderna. En éste libro se asume
que los estudiantes tienen una base de álgebra, geometría, y trigonometría. Es mucho más
compacto que los libros de texto tradicionales, proporciona muchos ejemplos trabajados y pide
resolver problemas
Este libro será útil también como texto para una persona que repasa o que consolida su
conocimiento de la Física.
La discusión y las explicaciones narrativas son suficientemente claras y completas para poder
utilizar el libro o como texto, o como suplemento a un texto más amplio.
La forma de aprender la física es trabajar realmente con problemas. Al usar este libro, el
estudiante debe ser activo. Debe intentar trabajar cada uno de los problemas y los ejemplos.
Debe mirar las soluciones solamente si no logra dar con el camino a su solución.
Los ejemplos en este libro están trabajados exhaustivamente, de modo que puedan servir como
modelos para el propio trabajo de los estudiantes. En este sentido se considera que los
estudiantes se benefician al observar los cálculos realizados en más de una manera, por lo que se
han incluido varios métodos para efectuar los cálculos.
Además, se tuvo especial cuidado en incluir problemas y preguntas que combinan el material
del capítulo en cuestión, con material de capítulos anteriores. Tales problemas y preguntas
destacan el hecho importante de que diversas áreas de la Física se manifiestan de manera
simultánea en el mundo real. Además, este método de temas múltiples proporciona una manera
para que los estudiantes repasen lo estudiado y ayuda a mejorar la habilidad para resolver
problemas.
El diseño gráfico es de gran importancia, y para mejorar su función se ha intentado enfocar
solamente una idea principal en cada figura en lo posible. Por consiguiente, las figuras del libro
a menudo se dividen en dos o más partes, para evitar la confusión de mezclar varias ideas en la
misma figura.
Los profesores conocen la importancia de los diagramas de cuerpo libre cuando utilizan la
segunda ley de movimiento de Newton, y todos los estudiantes aprenden de ellos a medida que
estudian Física. Tales diagramas se utilizan en todo el libro, no solamente en los primeros
capítulos en los que se presenta y aplica la segunda ley de Newton. Por ejemplo, cuando se
analiza la relación en las oscilaciones, también entre la presión y profundidad en un fluido, el
análisis se simplifica considerablemente por medio de un diagrama de cuerpo libre. De manera
semejante, cuando se deduce la expresión para la rapidez de una onda transversal en una cuerda,
un diagrama de cuerpo libre es muy útil.
Cifras significativas. A lo largo de todo el libro se siguen los procedimientos normales para las
cifras significativas.
Se espera que el esfuerzo en la elaboración de este libro sea de utilidad tanto para los
estudiantes como para los profesores. Toda opinión al respecto será bienvenida.

Hugo Medina Guzmán


Lima Perú
AGRADECIMIENTOS

El autor agradece primeramente a los estudiantes, quienes han contribuido bastante en la


elaboración de este libro a través de su influencia en el establecimiento de las técnicas y
principios de enseñanza y a los profesores que con sus sugerencias y revisiones a las
separatas de los capítulos hicieron notar puntos que necesitaban una mayor aclaración.

Hugo Medina Guzmán


CONTENIDO

CAPÍTULO 1. Electrostática
Introducción Carga eléctrica: formas de electrificación, conductores y
dieléctricos, Ley de Coulomb. Fuerza eléctrica y Campo eléctrico:
principio de superposición en distribución de carga discreta y continua.
Flujo de Campo eléctrico y ley de Gauss. Aplicaciones. Potencial eléctrico:
potencial de una distribución de cargas discretas y continuas, campo
eléctrico y potencial. Energía electrostática. Condensadores: definición y
capacidad, condensadores en serie y paralelo, condensadores de placas
paralelas con dieléctricos, aplicaciones
CAPÍTULO 2. Corriente continua
Corriente eléctrica, resistencia y ley de Ohm. Fuerza electromotriz.
Circuitos de corriente continua: resistencias en serie y paralelo, leyes de
Kirchhoff. Circuito RC: carga y descarga.
CAPÍTULO 3. Campo magnético
Campo magnético y fuerza de Lorentz. Fuerza magnética sobre un
conductor con corriente. Torque sobre una espira con corriente.
Aplicaciones: espectrómetro de masas, y motor eléctrico. La ley de Biot-
Savart. Ley de Ampere. Aplicaciones. Flujo de campo magnético. Ley de
Gauss en el magnetismo. Corriente de desplazamiento y la ley de Ampere
generalizada.
CAPÍTULO 4. Ley de Faraday e inductancia
Ley de inducción de Faraday. Ley de Lenz. Aplicaciones. Autoinductancia.
Circuito RL en corriente continua. Energía de una autoinductancia.
Densidad de energía magnética. Aplicaciones.
CAPÍTULO 5. Corriente alterna
Generador de corriente alterna. Circuitos de corriente alterna con una
resistencia, inductancia y capacitancia. Reactancia. Impedancia. Fasores.
Circuito RLC en serie y en paralelo. Resonancia. Valores medios y
eficaces. Potencia en corriente alterna. Transformador ideal.
CAPÍTULO 6. Ondas electromagnéticas
.Ondas electromagnéticas planas. Vector de Poynting. Presión de radiación.
El espectro electromagnético.
CAPÍTULO 1

Electrostática

CARGA ELECTRICA - INTRODUCCION. 1


El electroscopio 1
ELECTRIZACION 1
ELECTRICIDAD POSITIVA Y NEGATIVA 2
TEORIA DE LA ELECTRIZACION 3
LA LEY DE COULOMB 4
UNIDADES 4
PRINCIPIO DE SUPERPOSICION - DISTRIBUCION DE CARGAS 7
CARGA DISTRIBUIDA. 9
CAMPO ELECTRICO. INTRODUCCION. 10
DEFINICIÓN DE CAMPO ELÉCTRICO 10
Recta finita. 12
PLANO INFINITO 13
LINEAS DE FUERZA ELECTRICA 15
LA LEY DE GAUSS 17
MOVIMIENTO DE UNA PARTÍCULA CARGADA EN UN CAMPO 25
ELÉCTRICO UNIFORME
POTENCIAL ELECTRICO 26
POTENCIAL ELECTRICO PRODUCIDO POR UNA CARGA PUNTUAL. 27
POTENIAL ELECTRICO PRODUCIDO POR UNA DISTRIBUCION DE 27
CARGA
SUPERFIC1ES EQUIPOTENCIALES 35
CÁLCULO DEL CAMPO ELÉCTRICO A PARTIR DEL POTENCIAL. 37
CAPACIDAD, ENERGÍA ELECTROSTATICA 39
EL CONDENSADOR 40
COMBINACIONES DE CONDENSADORES 41
Condensadores en paralelo 41
Condensadores en serie 41
ENERGIA ELECTROSTATICA DE CARGAS 47
ENERGIA ALMACENADA POR UN CONDENSADOR 48
FUERZAS, TORQUES 49
CAMPO ELECTRICO EN UN DIELECTRICO 49
POLARIZACION. 50
CARGAS DE POLARIZACION. 50
LA LEY DE GAUSS PARA DIELECTRICOS – DESPLAZAMIENTO 51
ELECTRICO
LA CONSTANTE DIELECTRICA 52
PREGUNTAS Y PROBLEMAS 54
CAPÍTULO 2

Corriente continua

CORRIENTE ELECTRICA. INTRODUCCION. 1


CORRIENTE. DENSIDAD DE CORRIENTE 1
LA LEY DE OHM, RESISTIVIDAD Y RESISTENCIA 2
RESISTIVIDAD Y COEFICIENTE DE TEMPERATURA 3
FUERZA ELECTROMOTRIZ 4
ENERGIA Y POTENCIA EN LOS CIRCUITOS ELECTRICOS 5
CIRCUITOS DE CORRIENTE CONTINUA. INTRODUCCION 7
LEYES DE KIRCHHOFF 7
CONEXIONES DE RESISTENCIAS, EN SERIE Y EN PARALELO. 11
CAMBIO DE UN CIRCUITO TRIANGULO A ESTRELLA 16
CIRCUITO RC 17
INSTRUMENTOS Y DISPOSITIVOS DE MEDICION 20
Amperímetros y Voltímetros 20
Galvanómetro 20
MEDICION DE POTENCIAS 22
MEDICION DE RESISTENCIAS 23
Ohmímetro 23
Puente de Wheatstone 23
Potenciómetro 23
PREGUNTAS Y PROBLEMAS 24
CAPÍTULO 3

Campo magnético

INTRODUCCION 1
DEFINICION DE CAMPO MAGNETICO 1
Fuerza de Lorentz 2
EL FLUJO MAGNETICO 2
MOVIMIENTO DE UNA CARGA PUNTUAL EN UN CAMPO MAGNETICO 2
FUERZA SOBRE UN ALAMBRE CON CORRIENTE. 10
FUERZA Y TORQUE SOBRE UNA ESPIRA CON CORRIENTE 13
El Galvanómetro de D’ansorval 15
Motor de corriente Continua. 16
EFECTO HALL 16
LEY DE AMPERE, LEY DE BIOT Y SAVART 18
LEY DE GAUSS PARA EL MAGNETISMO 18
LEY DE AMPERE. 19
FUERZA ENTRE DOS CONDUCTORES PARALELOS 22
DEFINICIÓN DE LA UNIDAD DE CORRIENTE ELÉCTRICA (AMPERE) 25
LEY DE BIOT Y SAVART 25
PREGUNTAS Y PROBLEMAS 34
CAPÍTULO 4

Ley de Faraday e
inductancia

INTRODUCCION 1
LEY DE FARADAY 1
LEY DE LENZ 1
FUERZA ELECTROMOTRIZ DEL MOVIMIENTO 3
EL BETATRÓN. 10
CAMPOS ELECTRICOS INDUCIDOS POR CAMPOS MAGNETICOS 11
VARIABLES CON EL TIEMPO.
CORRIENTES DE FOUCAULT 11
GENERADOR DE CORRIENTE ALTERNA 12
INDUCTANCIA 12
DEFINICION DE INDUCTANCIA 12
Autoinductancia 12
Inductancia mutua 13
Autoinductancia de un toroide. 13
INDUCTANCIAS EN SERIE Y EN PARALELO 16
ENERGÍA ALMACENADA POR UNA INDUCTANCIA 17
MATERIALES MAGNETICOS 19
CIRCUITOS RL, LC y RLC 20
Circuito RL. 20
Circuito LC. 23
Circuito RLC. 24
PREGUNTAS Y PROBLEMAS 25
CAPÍTULO 5

Corriente alterna

GENERADOR DE CORRIENTE ALTERNA 1


Producción de una corriente alterna 1
Cálculo de la fem inducida 2
ANGULO DE FASE. FASORES 3
CIRCUITOS DE CORRIENTE ALTERNA CON UNA RESISTENCIA, 3
INDUCTANCIA Y CAPACITANCIA.
Un condensador conectado a un generador de corriente alterna 3
Una resistencia conectada a un generador de corriente alterna 4
Una inductancia conectada a un generador de corriente alterna 4
REACTANCIA 4
La reactancia inductiva 4
La reactancia capacitiva 4
CIRCUITO RLC EN SERIE 5
IMPEDANCIA EN SERIE 6
RESONANCIA EN SERIE 8
CIRCUITO RLC EN PARALELO 10
RESONANCIA EN UN CIRCUITO RLC EN PARALELO 10
POTENCIA EN CORRIENTE ALTERNA 12
Valores medios y eficaces 13
Valor cuadrático medio 13
Potencia media 13
Factor de potencia 14
Potencia en un circuito paralelo 14
TRANSFORMADOR IDEAL 21
PREGUNTAS Y PROBLEMAS 23
Capítulo 6

Las Ecuaciones de Maxwell y


ondas electromagnéticas

INTRODUCCIÓN 1
CORRIENTE DE DESPLAZAMIENTO DE MAXWELL 1
LAS ECUACIONES DE MAXWELL 2
ONDAS ELECTROMAGNÉTICAS 3
¿Qué es una onda electromagnética? 3
Vector poynting 5
Intensidad de la onda. 5
Potencia instantánea 6
Energía electromagnética 6
Presión de radiación 6
EL ESPECTRO ELECTROMAGNÉTICO 9
CARACTERÍSTICAS DE LAS DISTINTAS REGIONES DEL ESPECTO 9
Las microondas 9
La radiación infrarroja 9
La luz visible 9
Radiación ultravioleta 10
Rayos X 10
Rayos gamma 10
PREGUNTAS Y PROBLEMAS 14
BIBLIOGRAFÍA
THEORETICAL PHYSICS, Mechanics of particles, rigid and elastic bodies, fluids and heat flow.
F: Woobridge Constant. Trinity College. Addison – Wesley Publishing Company (1959)
THEORETICAL PHYSICS,Thermodinamics, electromagnetism, waves, and particles. F:
Woobridge Constant. Trinity College. Addison – Wesley Publishing Company (1959)
The Feynman LECTURES ON PHYSICS. Volumenes I, II y III. Richard P. Feynman, Robert B.
Leighton. California Institute of Technology, Matthew Sands, Stanford University. Addison – Wesley
Publishing Company (1964)
CORRIENTES, CAMPOS Y PARTÍCULAS. Francis Bitter. Massachusetts Institute of Technology.
Editorial Reverté S. A. (1964).
INTRODUCCIÓN AL ESTUDIO DE LA MECÁNICA, MATERIA Y ONDAS. Uno Ingard, William
L. Kraushaar. Editorial Reverté. (1966).
FUNDAMENTOS DE ELECTRICIDAD Y MAGNETISMO. Arthur F. Kip. University of California.
Mc Graw – Hill Book Company (1967)
CIENCIA FÍSICA Orígenes y principios Robert T. Langeman, Universidad Vanderbilt. UTEHA,
(1968)
PROBLEMS IN ELEMENTARY PHYSICS. B. Bukhotsev, V: Krivchenkov, G. Myakishev,
V.Shalnov. Mir Publishers. Moscow (1971)
PROBLEMES DE PHYSIQUE COMMENTES. Tomos I y II Hubert Lumbroso. Mason et Cie, París.
(1971)
ELECTRICIDAD Y MAGNETISMO PARA ESTUDIANTES DE CIENCIAS E INGENIERÍA.
Luís L. Cantú. Instituto Tecnológico y de Estudios Superiores de Monterrey. Editorial Limusa México
(1973)
FÍSICA PARA LAS CIENCIAS DE LA VIDA Y LA SALUD. Simon G. G. MacDonald / Desmond
M. Burns University of Dundee. Fondo educativo interamericano. (1975)
MECÁNICA NEWTONIANA, MIT Physics course. A. P. French. Editorial Reverté. (1974).
FÍSICA I y II. Solomon Gartenhaus. Purdue University. INTERAMERICANA. (1977)
TEACHING TIPS. A guidebook for the beginning College Teacher. Wilbert J. McKeachie
(University of Michigan). Seventh edition D. C. Heath and Company (1978)
FÍSICA PARA LAS CIENCIAS DE LA VIDA. Alan H. Cromer. Northeastern University. Editorial
Reverté. (1978)
GENERAL PHYSICS WITH BIOSCIENCE ESSAYS. Jerry B. Marion. University of Maryland.
John Wiley & Sons Inc. (1979)
Física general II: Teoría Hugo Medina Guzmán, Miguel Piaggio H. QC 21 M19 (Biblioteca PUCP)
(1979)
Física general II: Problemas resueltos Hugo Medina Guzmán, Miguel Piaggio H. FIS 111 M364
(Biblioteca PUCP) (1979)
Física general I: problemas resueltos Hugo Medina Guzmán, Miguel Piaggio H. FIS 104 M364
(Biblioteca PUCP) (1981)
FÍSICA PARA ESTUDIANTES DE CIENCIAS E INGENIERÍA. 1 y 2. John P. McKelvey,
Clemson University – Howard Grotch, Pennsilvania State University. HARLA. Mexico. (1981)
Física 3: electricidad y magnetismo para estudiantes de ciencias e ingeniería
Hugo Medina Guzmán, FIS 141 M36 (Biblioteca PUCP) (1982)
EXPLORING PHYSICS Concepts and applications. Roger W. Redding North Texas State University,
Stuart Kenter, Wadsworth Publishing Company (1984)
PROBLEMAS DE FISICA. J. Aguilar Peris, Universidad Complutense de Madrid - J. Casanova Colas,
Facultad de Ciencias de Valladolid. Alambra (1985)
PROBLEMAS DE FISICA. Dirigido por S. Kósel. Editorial Mir Moscú. (1986)
PROBLEMAS DE FISICA Y COMO RESOLVERLOS. Clarence E. Benett Maine University.
CECSA (1986)
PHYSICS for Engineering and Science. Michael E. Browne, Ph. D. (professor of Physics University of
Idaho. Schaum’s outline series Mcgraw-Hill (1988)
FÍSICA: VOLUMEN 1. Mecánica, ondas y termodinámica. Duane E. Roller, Ronald Blum. Editorial
Reverté. (1990).
FÍSICA: VOLUMEN 2. Electricidad, magnetismo y óptica. Duane E. Roller, Ronald Blum. Editorial
Reverté. (1990).
PROBLEMAS DE FISICA. Dirigido por O. Ya. Sávchenko. Editorial Mir Moscú. (1989)
MECÁNICA. Berkeley physics course – volumen 1. Charles Kittel, Walter D. Knight, Malvin A.
Ruderman. Editorial Reverté SA. (1992).
ELECTRICIDAD Y MAGNETISMO. Berkeley physics course – volumen 2. Edward M.Purcell.
Editorial Reverté SA. (1992).
FÍSICA. Tomos I y II Tercera edición revisada (Segunda edición en español), Raymond S: Serway,
James Madison University, Mcgraw-Hill, (1993)
PROBLEMAS DE FISICA Santiago Burbano de Ercilla, Enrique Burbano de Ercilla, Carlos Gracia
Muñoz, XXVI edición, Zaragoza, MIRA editores (1994)
ONDAS. Berkeley physics course – volumen 3. Frank S. Crawford, Jr. Editorial Reverté SA. (1994).
FÍSICA Para las ciencias de la vida, David Jou Mirabent Universidad autónoma de Barcelona, Joseph
Enric Llebot Rabagliati, Universidad de Girona, Carlos Pérez garcía, Universidad de Navarra. Mcgraw-
Hill, (1994)
Física uno Hugo Medina Guzmán, FIS 104 M365 (Biblioteca PUCP) (1995)
APPLIED PHYSICS. Arthur Beiser, Ph. D. Schaum’s outline series Mcgraw-Hill (1995)
TEACHING INTRODUCTORY PHTSICS A Sourcebook. Clifford E: Swartz (State University of
New York, Stony Brook) and Thomas Miner (Associate Editor The Physics Teacher 1972 – 1988). ATP
Press – Springer. (1996)
TEACHING INTRODUCTORY PHYSICS Arnold Arons University of Washington JOHN WILEY
& SONS, INC. (1997)
FÍSICA John Cutnell / Kenneth W. Johnson. Southern Illinois University. LIMUSA (1998)
FÍSICA EN LA CIENCIA Y EN LA INDUSTRIA. A. Cromer. Northeastern University. Editorial
Reverté. (2000)
FÍSICA CONTEMPORANEA Edwin Jones.– Richard Childers, University of South Carolina.
Mcgraw-Hill, (2001)
PROBLEMAS Y CUESTIONES DE FISICA. Atanasio Lleó, Begoña Betete, Javier Galeano,
Lourdes Lleó, Ildefonso Ruiz – Tapiador. Universidad Politécnica de Madrid. Ediciones Mundi – prensa
(2002)
The PHYSICS of every day phenomena. A conceptual introduction to Physics. W. Thomas Griffith,
Pacific University. Mcgraw-Hill, (2004)
FÍSICA UNIVERSITARIA. Francis W.Sears, Mark W. Zemansky, Hugh D. Young (Carnegie Mellon
University) y Roger A. Freedman (University of California. Santa Barbara) Volumen 1, Volumen 2.
Undecima edición. Pearson - Addison Wesley (2004)
FIVE EASY LESSONS Strategies for successful Physics teaching. Randall D. Knight California
Polytechnic State University, San Luis Obispo. Addison Wesley (2004)
FUNDAMENTALS OF PHYSICS. David Halliday (Univ. of Pittsburgh), Robert Resnick (Rensselaer
Polytechnic Institute), Jearl Walker (Cleveland State Univ.). 7th Edition (2005)
Electrostática Hugo Medina Guzmán

CAPÍTULO 1. Electrostática

INTRODUCCION.
La primera observación de la electrización se remonta
a la época de la Grecia antigua. A Tales de Mileto se
le atribuye haber observado la atracción que el ámbar
previamente frotado, ejerce sobre pequeños pedazos
de fibra y paja.
A pesar de que la electrización del ámbar por fricción
fue transmitida de un escrito a otro, nada nuevo se
descubrió hasta principios del siglo XVII en que Sir
William Gilbert anunció el descubrimiento de que
muchas sustancias podían ser electrizadas por
Si frotarnos una barra de plástico con una piel de
frotamiento y que el ámbar es uno de los muchos
gato, o si frotamos una barra de vidrio con seda. Las
materiales que manifiestan el efecto.
barras adquieren la propiedad de atraer cuerpos
ligeros corno pedacitos de papel a una pequeña bola
EL ELECTROSCOPIO.
hecha da material ligero como corcho o médula de
Dispositivo que sirve para detectar y medir la carga
saúco (Sambucus peruviana) suspendida por hilos de
eléctrica de un objeto. Los electroscopios han caído
seda. Se dice que estos cuerpos están electrizados.
en desuso debido al desarrollo de instrumentos
Si frotarnos una barra de cobre sostenida por la mano,
electrónicos mucho más precisos, pero todavía se
no habrá acción sobre los cuerpos ligeros, pero si
utilizan para hacer demostraciones. El electroscopio
frotamos la misma barra de cobre pero esta vez
más sencillo está compuesto por dos conductores
sostenida por un mango de vidrio, se electriza y
ligeros suspendidos en un contenedor de vidrio u otro
ejercerá acción sobre los cuerpos ligeros.
material aislante.
O sea que tenemos cuerpos de dos categorías, los
Se puede utilizar un electroscopio para determinar si
primeros como al vidrio, plexiglás, ebonita, resina
un objeto está cargado eléctricamente. Cuando un
que se electrizan agarrándolos con 1a mano y otros
objeto cargado se acerca al bulbo, las hojas divergen.
cuerpos que necesitan un mango de un material de la
a) El electroscopio neutro tiene las cargas distribuidas
primera categoría para poder ser electrizados
uniformemente, las hojas están juntas.
La experiencia demuestra que en los cuerpos de la
primera categoría la electricidad permanece
localizada en los puntos frotados, no se propaga, estos
cuerpos son malos conductores de la electricidad se
conocen como aislantes o dieléctricos.
Para los cuerpos de segunda categoría, las
propiedades de la atracción sobre cuerpos ligeros no
solo se manifiestan en los puntos frotados, sino en
todos los puntos, a sea la electrización producida se
transmite a todos los puntos del cuerpo, estos cuerpos
son conocidos como conductores.
El cuerpo humano y la tierra son conductores.
(b) Las fuerzas electrostáticas causan que las hojas Además de la electrización por frotamiento descrita,
diverjan. hay otras formas de electrización que indicamos a
continuación.

1° Electrización por contacto La carga es


transferida al electroscopio cuando la varilla cargada
toca el bulbo. Entonces, cuando una varilla con carga
opuesta se acerca al bulbo, las hojas se colapsan y se
juntan.
El electroscopio neutro se toca con una varilla
cargada negativamente; las cargas son transferidas al
bulbo.

ELECTRIZACION

1
Electrostática Hugo Medina Guzmán

3º Piezoelectricidad. Una lámina de cuarzo


convenientemente tallada, se electriza por compresión
o por tracción, el fenómeno es reversible. Estos
cuerpos son conocidos como piezoeléctricos.

4° Electrización por las fuentes de electricidad. Si


ponemos una pila o un acumulador, uno de sus polos
se conecta a tierra y el otro a un conductor aislado,
El electroscopio tiene una carga negativa neta. éste se electriza. Con una sola pila la electrización es
débil, pero si se dispone de muchos elementos se
obtiene electrización fácil de poner en evidencia.

ELECTRICIDAD POSITIVA Y NEGATIVA


Cuando frotamos entre sí dos sustancias diferentes y
luego las separamos, nos encontramos con dos tipos
de electricidad. Para ver mejor esto realicemos la
siguiente experiencia.
La varilla cargada positivamente atrae los electrones;
Dispongamos de dos barras de plástico y dos barras
las hojas se colapsan. de vidrio. Se carga por frotamiento con una piel una
de las barras de plástico y se la suspende mediante un
gancho y un hilo de nylon como se muestra en la
figura siguiente, pudiendo girar libremente.

2° Electrización por inducción Al tocar el bulbo se


proporciona una trayectoria para la transferencia de
carga, los electrones son transferidos a la tierra.
Cuando el dedo se retira, el electroscopio tiene una
carga neta, el electroscopio queda cargado Si acercamos a esta barra de plástico la otra frotada
positivamente. similarmente, observamos que gira alejándose, si
acercamos la piel a la barra de plástico suspendida
observamos que ésta gira acercándose. De igual modo
si acercamos la barra de vidrio electrizada por
frotación con seda observamos que el plástico gira
acercándose y si acercamos la seda el plástico gira
alejándose.

Tierra eléctrica se refiere a la tierra (o sea al “suelo”)


o a algún otro objeto que pueda recibir o suministrar
electrones sin cambiar significativamente su propia
condición eléctrica.
Como esto se debe a electrones que han sido
transferidos, usted puede preguntarse cómo se puede
cargar positivamente un electroscopio. Esto se hace
cargando por inducción. Al tocarse el bulbo con un
dedo, el electroscopio hace tierra, es decir, se da una
trayectoria para que los electrones puedan escapar del
bulbo. Entonces, cuando se acerca al bulbo una
varilla cargada negativamente los electrones son
repelidos del bulbo. Al retirar los dedos se deja al
electroscopio una carga positiva neta.
Puesto que la piel al igual que el vidrio, atraen al
plástico electrizado, ambos tienen la misma clase de
electrización. Se dice que estén cargados
2
Electrostática Hugo Medina Guzmán

positivamente. De modo similar el plástico y la seda “Cuerpos con cargas similares se repelen y con cargas
estarán carga dos negativamente. diferentes se repelen; para cargas puntuales (llamando
Las Cargas positivas las designamos por el signo (+) puntual cuando sus dimensiones espaciales son muy
y las Cargas negativas por el signo (-). pequeñas comparadas con cualquier longitud del
De esta experiencia también podemos describir que problema en cuestión) la fuerza de interacción es
“Las Cargas iguales se repelen y las Cargas contrarias proporcional al producto de lo cuerpos e
se atraen”. inversamente proporcional al cuadrado de la distancia
También es una observación experimental que la que los separa”.
carga no puede ser creada ni destruida, la carga total
de un sistema no puede ser cambiada. Del punto de
vista macroscópico las cargas pueden reagruparse y
cambiarse de diferentes maneras o sea que “La carga
neta se conserva en un sistema cerrado”.

TEORIA DE LA ELECTRIZACION
La Carga es una propiedad característica y La ley fue establecida en 1784 por Coulomb por
fundamental de las partículas elementales que forman medios experimentales utilizando una balanza de
luego materias. Las sustancias estén formadas por torsión.
moléculas y estas por átomos. Cada átomo contiene Podemos nosotros reproducir este experimento en una
un núcleo que tiene una cantidad conocida de carga forma sencilla, para esto dispongamos de dos esferas
positiva. Esta Carga positiva se debe a la presencia de pequeñas de médula de saúco (se puede usar esferas
un cierto número de protones. Todos los protones de tecknopor) suspendidas de hilo de nylon, como se
son semejantes y tienen la misma masa y la misma muestra en la figura (a) donde la distancia puede
carga positiva. Alrededor de cada núcleo atómico hay variarse a voluntad. Al ponérsele carga a las esferas
un número de partículas cargadas negativamente, estas toman la posición mostrada en la figura (b).
llamadas electrones.

Donde θ es el ángulo de deflexión, mg los pesos, FE.


la fuerza electrostática y T la tensión en las cuerdas.
Normalmente cada átomo de una sustancia es De las leyes de la mecánica encontramos la relación
eléctricamente neutro, o sea que tiene cantidades entre FE y θ .
iguales de carga positiva y negativa, la carga de un FE = mg tan θ
electrón es igual pero opuesta a la carga de un protón.
En cada núcleo hay tantos protones como electrones Variando la separación d entre los soportes podemos
hay rodeándolo. observar diferentes deflexiones.
Los átomos aislados o los grupos grandes de átomos De las medidas de FE en función de la separación de
y moléculas tienen una afinidad para adquirir equilibrio r de las cargas encontramos que
electrones adicionales sobre el número exacto que 1
neutralizan las cargas positivas del núcleo. Esta
FE ∝
r2
afinidad de los átomos para tener más electrones que esta evidencia experimental se presenta a
el número suficiente de ellos, varía continuación en forma de ecuación
considerablemente de una sustancie a otra. Por lo
tanto, cuando dos sustancias diferentes se ponen en q1 q 2
FE = k
contacto, la que tiene mayor afinidad toma las r2
electrones próximos de la otra y adquiere una carga Donde q1 y q 2 representan las magnitudes de las
negativa, mientras que la otra queda cargada positiva
mente, tal es el caso del Caucho cuando se frota con cargas. La dependencia de la carga no fue establecida
una piel. por Coulomb, puesto que no tenía medios
independientes para juzgar la magnitud de la carga, r
LA LEY DE COULOMB es la distancia entre los centros de las cargas.
Esta ley de la fuerza entre los cuerpos cargados puede Como la fuerza es un vector la ley de Coulomb la
enunciarse como sigue: podemos escribir de la siguiente forma, fuerza sobre
la carga

3
Electrostática Hugo Medina Guzmán

→ q1 q 2 → → q1 q 2 → 1 q1 q 2 →
F1 = k r O F1 = k r̂21 Fuerza sobre q1 : F1 = r21
4πε 0 r213
3 21
r21 r212

r21 = r1 − r2 = (2iˆ + ˆj + 3kˆ ) − (iˆ + 3 ˆj + kˆ )


→ → → →
r
Donde rˆ21 = 21 , vector unitario a lo largo de r21
r21 = iˆ − 2 ˆj + 2kˆ
Fuerza sobre la carga q
→ qq → →
[
r21 = 12 + (− 2) + 2 2
2
]12
= 9 =3
F2 = k 1 3 2 r12 = − F1 Luego
r12 →
F1 = 9 × 10 9
(2 × 10 )(3 × 10 ) (iˆ − 2 ˆj + 2kˆ )
−9 −9

32
F1 = 2 × 10 −9 (iˆ − 2 ˆj + 2kˆ ) N

Fuerza sobre q 2

F2 = − F1 = −2 × 10 −9 (iˆ − 2 ˆj + 2kˆ ) N
→ →

El modulo es
[
F1 = F2 = 2 × 10 −9 12 + (− 2 ) + 2 2
2
]
12
= 6 × 10 −9 N

Ejemplo 2. ¿Cuál de las dos situaciones siguientes


daría como resultado una mayor fuerza?
a) La fuerza de repulsión que ejerce una carga de
UNIDADES 100 C sobre una de 1 C.
Como la carga eléctrica es un concepto nuevo, no b) La fuerza de repulsión que ejerce una carga de 1
conocido en la mecánica es necesario introducir una C sobre una de 100 C.
nueva unidad fundamental.
Sistema CGS. En este sistema hacemos k = 1 y la Solución. Las dos opciones nos conducen a la misma
unidad de carga se llama el statcoulombio. situación, ya que tienen la misma distancia. Y el
Sistema M.KS. En esta sistema la unidad de carga se producto de la carga es la misma
define a partir de la corriente eléctrica, concepto que
veremos más adelante en detalle, la unidad
→ q1 q 2 →
F =k r
fundamental es el ampere y la carga está definida por r2
ampere - segundo, y a esto se le llama Coulombio
(C). Como FE está en Newton, q1 y q2 en Coulombios Lo único que cambia es la dirección de la fuerza.
y r en metros, la constante k se fija por estas
elecciones y toma el valor Ejemplo 3. ¿Si deseo trasladar una carga de 1C del
Nm 2 origen a el punto (100,100, 100) y luego del punto al
k = 8,9874 × 10 9 origen, en cual de los dos la fuerza de atracción debe
C2 ser mayor?
El cual puede aproximarse a 9x109 en la mayoría de
los cálculos numéricos. Solución. Al igual que en el problema anterior, la
Es útil expresar el valor de k en la forma misma fuerza, en magnitud, que se requiere para
1 trasladar la carga del origen al punto y del punto al
k= origen es la misma. Por lo tanto la misma fuerza que
4πε 0 se requiere para trasladar la carga del origen al punto
Donde ε0 es una constante que se conoce como la es la misma que la que se requiere para trasladar la
carga del punto al origen. Analicemos la siguiente
permitividad del espacio libre y su valor es
relación.
C2
ε 0 = 8,85415 × 10 −12

Nm 2 → q pQ →
F =k r
4πε 0 r 2
Ejemplo 1. Se tienen dos cargas puntuales.

q1 = 2 × 10 −9 C , r1 = 2iˆ + ˆj + 3kˆ en metros y Donde qp es una carga llamada carga de prueba, que
podemos utilizar para medir la fuerza necesaria del

q 2 = 3 × 10 −9 C , r2 = iˆ + 3 ˆj + kˆ en metros. para trasladar la carga del origen al punto P.
Analicemos ahora la fuerza para trasladar la carga del
¿Cuál es esfuerzo sobre cada una de ellas? punto P al origen
Solución.

4
Electrostática Hugo Medina Guzmán

→ q pQ → Q1Q2
F =k r F= Ya que la magnitud del vector
4πε 0 r 2 4πε 0 r 2
unitario es 1
Observemos que ambas relaciones son las mismas.
Ejemplo 5. Encuentra la fuerza de repulsión que
existe entre dos cargas de 2 C cada una. Una de
Ejemplo 4. Determine la fuerza eléctrica, de las cargas esta en el origen y la coordenada de la otra
atracción, ejercida por una carga Q1 = 2,0 μC que carga en (0, 0,0)
se encuentra en la posición (1, 1,1) sobre la carga
Q2 = −2,0 μC en el origen. Solución. Primeramente sabemos que las dos cargas
no pueden estar en el mismo punto. Para estar en el
mismo punto tendríamos r = 0, que en la ecuación de
la ley de Coulomb nos conduciría a una
indeterminación debido a la división entre cero.

q pQ
F =k
4πε 0 (0 )

Sabemos del hecho que la fuerza eléctrica entre dos


cargas es una ley

1
Solución. La fuerza, mediante la ley de Coulomb F∝
queda determinada por: r2
→ Q1Q2
F= rˆ La fuerza crece muy rápidamente a medida que r es
4πε 0 r 2 pequeña, como se puede observar en su gráfico
Determinemos para ello el vector que existe entre las

cargas: r 12 = (0 − 1)iˆ + (0 − 1) ˆj + (0 − 1)kˆ
= −iˆ − ˆj − kˆ

La magnitud del vector r 12 es:

r = r 12 = (− 1)2 + (− 1)2 + (− 1)2 = 3
Por lo tanto el vector unitario

r 12 − iˆ − ˆj − kˆ Ejemplo 6. Dos esferas idénticas pequeñas y
es: rˆ = = conductoras tienen cargas de 3x10-9 C y -1x10-9 C,

r 12 3 respectivamente. Cuando se colocan separadas 3 cm.
a) ¿Cuál es la fuerza entre ellas?
→ Q1Q2 b) Si se ponen en contacto y luego se separan 3 cm,
F= rˆ c) ¿cuál es la fuerza entre ellas?
4πε 0 r 2 Solución.

=
(2 × 10 )(− 2 × 10 ) ⎛⎜ − iˆ − ˆj − kˆ ⎞⎟
−6 −6 a) La fuerza entre las dos esferas cuando están
separadas 3 cm. es:
4πε 0 ( 3) 2 ⎜
⎝ 3 ⎟

=
36 × 10 −3
3
(− iˆ − ˆj − kˆ )
La magnitud de la fuerza es
36 × 10 −3

F=
1 (3 × 10 )× (− 1× 10 )
−9 −9

F=
3
3 = 36 x 10-3 N 4πε 0 (3 × 10 )
−2 2

−5
Es decir, el término: = − 3 × 10 N
La fuerza es de atracción e igual a 3x10-5 N.
b) Cuando se ponen en contacto se produce equilibrio
de las cargas diferentes o sea

5
Electrostática Hugo Medina Guzmán

Ejemplo 8. Un electrón tiene una masa de 9,1 x 10-31


kg y una carga eléctrica de 1,6 x 10-19 C. Suponga
que dos electrones están colocados cerca de uno de
3 × 10 −9 − 1 × 10 −9 = 2 × 10 −9 ⇒ Sería la carga otro. Compare las fuerzas gravitacionales y eléctricas
total, la que se distribuye por igual en cada una de las entre ellas.
esferas, por ser idénticas. Solución. La fuerza de atracción gravitacional entre
los electrones es:
2 × 10 −9
q1 = q 2 =
2
= 1 × 10 −9 C
FG = G 2 = (6,6 × 1011 )
m2 (9,1× 10−31 ) 2

r r2
54,6 × 10−72
= 2
N m2
c) La fuerza entre las dos esferas cuando se colocan a r
3 cm es: La fuerza de repulsión entre los electrones es:
9 (1,6 × 10 )
−19 2

Fe =
1q2
4πε 0 r 2
= (9,9 × 10 ) r 2

23,04 × 10−29

F=
1 (1× 10 )× (1× 10 ) = 1× 10
−9 −9
−5
N
=
r2
N m2
4πε 0 (3 × 10 ) −2 2 Luego
Fe 23,04 × 10−29
= = 4,2 × 1042
Ejemplo 7. Dos esferas conductoras iguales de FG 54,6 × 10− 72
tamaño insignificante se cargan con 16,0 x 10-14 C y La fuerza gravitacional entre los electrones es
-6,4 x 10-14 C, respectivamente, y se colocan despreciable en comparación con la fuerza eléctrica.
separadas 20 centímetros. Luego se mueven a una
distancia de 50 centímetros separación. Ejemplo 9. Dos esferas conductoras pequeñas, cada
a) Compare las fuerzas entre ellas en las dos uno de la masa 0,25 g, están colgando por medio de
posiciones. hilos aisladores de longitud 50 centímetros de modo
b) Las esferas se conectan por medio de un alambre que apenas se tocan. Se da una carga, que comparten
fino. ¿Qué fuerza ejerce la una sobre la otra? igualmente, y a cada una toma una posición tal que el
Solución. La ecuación que da la fuerza entre las hilo por el cual cuelga forma un ángulo de 45º con la
esferas, que se pueden considerar como cargas vertical. ¿Cuál es la carga en cada una?
puntuales, es
Solución.
1 q1q2
F=
4πε 0 r 2
Luego
1 q1q2 1 q1q2
F1 = y F2 =
4πε 0 (0,2 )2
4πε 0 (0,5)2
F (0,5)
2
∴ 1 = = 6,25
F2 (0,2)2
b) Si las esferas se unen por un alambre, las cargas,
que se atraen a una otra, pueden fluir por el alambre Hay tres fuerzas que actúan en cada esfera, el peso
bajo influencia de las fuerzas que actúan en ellas. mg que actúa hacia abajo, la fuerza repulsiva de
Las cargas neutralizarán lo más lejos posible y (16,0 Coulomb F que actúa horizontalmente, y la tensión T
x 10-14 – 6,4 x 10-14) = 9,6 x 10-14 C se distribuirán en el hilo de soporte. Puesto que la esfera está en
sobre el sistema. No tomar en cuenta el efecto del equilibrio, las componentes horizontales y verticales
alambre, por simetría 4,8 x 10-14 C quedará en cada deben equilibrarse por separado. Así
esfera. La fuerza entre las dos esferas ahora es: mg = Tcos 45º y F = Tsen 45º o F = mg tan 45º
Pero
q2
1
F= 1 q2 1 q2
4πε 0 r 2 F= =
4πε 0 r 2 4πε 0 (2hsen 45º )2
= (9,9 × 10 )
(4,8 × 10
9
−14
C )
2
Donde q es la carga sobre cada esfera.
(0,5)
2
∴ q 2 = 4πε 0 (2h sen 45º )mg
−17
= 8,29 × 10 N

6
Electrostática Hugo Medina Guzmán

2 = 8,40 x 10-16 N.
⎛ 1 ⎞⎡ ⎛ 1 ⎞⎤
= ⎜ ⎟ ⎢2 × 0,5⎜ ⎟⎥ (2,5 × 9,8) Del mismo modo
⎝ 9 × 10
9
⎠⎣ ⎝ 2 ⎠⎦ q4 ⎡ q1 q a ⎤
-14 2 Fy = ⎢ 2 + 2 2 2 32 ⎥
(a + b ) ⎥⎦
= 13,6 x 10 C .
y q = 3,7 x 10-7C. 4πε 0 ⎢⎣ a
= 13,58 x 10-16 N.
El vector F tiene componentes:
Ejemplo 10. Las cargas puntuales de magnitudes 2,5 (8,40x10-16 N y 13,58x10-16 N),
x 10-14 C, 1,0 x 10-14 C, -3,5 x 10-14 C, y 2.0 x 10-14 C Cuya magnitud es:
se ponen en las esquinas A, B, C, y D de un
rectángulo respectivamente en el cual el AB tiene una 8,40 2 + 13,58 2 = 15,97x10-16 N, a un ángulo de
de longitud 8 cm y BC tiene una longitud de 6 cm. tan-1 (13,58/8,40) = 58°16’ con el eje x.
¿Qué fuerza actúa sobre la carga en D debido a las
otras tres? PRINCIPIO DE SUPERPOSICION -
Solución. DISTRIBUCION DE CARGAS
Si más de dos cargas puntuales están presentes, las
fuerzas mutuas se determinan por la aplicación
repetida de la ley de Coulomb, esto viene a ser la ley
de adición o principio de superposición y se
demuestra experimentalmente.
Si son n las cargas esto es q1, q2, q3.
La fuerza sobre la carga es

→ n
qi r i1
F1 = q1 ∑
i ≠1 4πε 0 ri13
Donde la sumatoria so extiende sobre todas las cargas
con excepción de la Carga q1.

Ejemplo 11. Dos cargas puntuales se encuentran


Las cargas en A, B, C, y D son q1, q2, - q3, y q4, separadas una distancia d. ¿Dónde y qué carga se
respectivamente. Las fuerzas ejercidas sobre q4, por debe colocar para que el sistema quede en equilibrio?
las otras tres se muestran en el diagrama como los So1ución. La posición de la tercera carga Q debe
vectores F1, F2, y F3, respectivamente. Los ejes x e y estar en la recta formada por la línea que une las dos
se han seleccionado en las direcciones mostradas. cargas, de no ser así no habría equilibrio como
q1 q 4
1 1 q2 q4 muestra la figura la componente vertical no tiene
F1 = , F2 =
4πε 0 b 2
4πε 0 a 2 + b 2
,
( ) anulación posible.

1 q3 q 4
F3 =
4πε 0 a 2

Las tres fuerzas que actúan sobre q4 se combinan en


una sola fuerza F con componentes (Fx, Fy) a lo largo
de los ejes elegidos. F2 se descompone a lo largo de
los ejes x e y, y observamos que:
BD = (8cm )2 + (6cm )2 = 10 cm
Obtenemos Para determinar la posición de la carga Q llamaremos
Fx = F3 − F2 senθ , Fy = F1 − F2 cosθ x a esta posición.
De estas ecuaciones, se obtiene
q 4 ⎡ q3 q2 a ⎤
Fx = ⎢ 2 − 2 ×
4πε 0 ⎣ a a +b 2
( ⎥
a2 + b2 ⎦ )
q 4 ⎡ q3 q a ⎤
= ⎢ 2 − 2 2 2 32 ⎥
4πε 0 ⎢⎣ a a +b (
⎥⎦ ) Como Q está en equilibrio
1 qQ 1 3qQ
= (9 ×109 )(2,0 ×10 −14 )⎡ 3,5 ×10 −14 − (1,0 ×10 −14 )(8 ×10 − 2 )⎤ =

( −2 2
⎢⎣ 8 × 10 ) (10 ×10 )
−2 3 2

⎥⎦ 4πε 0 x 2
4πε 0 (d − x )2

7
Electrostática Hugo Medina Guzmán

⇒ (d − x ) = 3x 2
2
q2 q2
F = 2F1cos 30° = 2k cos 30 º = 3k
d2 L2 L2
⇒ x 2 + dx − =0
2
1 3
Resolviendo: x = − ±
2 2
hay dos posiciones posibles

x1 =
(
3 −1 )
d = 0,36d y
2

x2 = −
(
3 +1 )
d = − 1,36d ,
2 Ejemplo 14. Tres cargas puntuales, que inicialmente
x1 está entre ellas a la derecha de q. están infinitamente lejos unas de otras, se colocan en
x2 está fuera a la izquierda de q. los vértices de un triángulo equilátero de lados d. Dos
Ahora encontramos el valor Q para x1, para que haya de las cargas puntuales son idénticas y su carga es q.
equilibrio es necesario que las fuerzas sobre q se Si el trabajo neto que se requiere para colocar las tres
anulen. cargas en el triángulo es cero ¿cuál es el valor de la
tercera carga?
Solución.

1 3qq 1 qQ
+ =0
4πε 0 (d ) 2
4πε 0 (0,36d )2
⇒ Q = −1,71q
Similarmente pare x2
Trabajo para colocar la primera carga:
W1 = 0
Trabajo para colocar la segunda carga:
q2
W2 = qΔV 1=
4πε 0 d
1 qQ 1 3qq
+ =0 Trabajo para colocar la tercera carga:
4πε 0 (d ) 2
4πε 0 (1,36d )2 Q ⎛q q⎞ Qq
⇒ Q = −5,55q W3 = QΔV3 = ⎜ + ⎟=
4πε 0 ⎝ d d ⎠ 2πε 0 d
Trabajo neto para colocar las tres cargas:
Ejemplo 12. Las cargas se colocan en el eje x como
sigue: q1 = + 2 μ C en x = 0, q2 = - 3 μ C en x = 2 W = W1 + W2 + W3 = 0
m, q3 = - 4 μ C en x = 3m, y q4 = + μ C en x = 3,5 m. q2 2Qq q
+ =0 ⇒ Q=−
¿Cuál es la magnitud y la dirección de la fuerza en 4πε 0 d 4πε 0 d 2
q3?
Solución.
Ejemplo 15. La sal de mesa (cloruro de sodio) es un
⎛q q q ⎞ cristal con una estructura cúbica simple con iones de
F = kq3 ⎜⎜ 21 + 22 + 24 ⎟⎟ Na+ y de Cl- que se alternan en sitios adyacentes del
⎝ r13 r23 r43 ⎠ enrejado.
⎡ 2 μC 3μC 1μC ⎤ La distancia entre iones es 2,82 x 10-10m = 0,282 nm
= (9 × 10 )(4 μC )⎢− + 2 +
9

(1) (0,5)2 ⎥⎦
(1 nm = 10-9 m).
⎣ (3)
2
a) ¿Qué fuerza experimenta un ión del Na+ debido a
= 2,44 x 10-10N/C uno de sus vecinos Cl- más cercano?
b) ¿Qué fuerza experimenta un ión de Cl- debido a
Ejemplo 13. Tres cargas positivas idénticas q se un ión de Na+ vecino?
colocan en las esquinas de un triángulo equilátero de c) ¿En qué fuerza experimenta un ión de Na+ en el
lado L. ¿Qué fuerza experimenta una de las cargas? origen debido a los iones de Cl- en (a, 0, 0) y (0, a,
Solución. 0)?
d) ¿Cuál es el peso de un ión del Na+ de masa 3,82 x
1026kg?
8
Electrostática Hugo Medina Guzmán

Solución.
=
(0,03)(9,8)(tan 7º )(2)(0,3)(sen7º )
9 × 10 9
⇒ q = 0,146 x 10-6 C = 0,146 μC

CARGA DISTRIBUIDA. En el caso ya no de cargas


puntuales sino de una distribución continua, a pesar
que la carga eléctrica se encuentre en múltiplos de
una carga básica que es el electrón (e = 1,6x10-19 C),
la cual es extremadamente pequeña. Esto significa
que la carga macroscópica está compuesta de un
9 (1,6 × 10 )
−19 2
e2 número muy grande de electrones. Así podemos
a) F1 = k = 9 × 10
r 2
(0,282 × 10 )−9 2 describir una distribución de carga en términos de una
densidad de carga.
= 2,90 x 10-9 N Densidad de carga volumétrica definida por
b) Por la tercera ley de Newton: Δq dq
La fuerza sobre el Cl- debido al Na+ es igual que la ρ = lim =
ΔV →0 ΔV dV
fuerza en el Na+ debido al Cl-.
(
ˆ ˆ
c) F = F1 + F2 = 2,90 x 10-9 i + j N
→ → →
) Densidad de carga superficial definida por
Δq dq
σ = lim =
ΔS →0 ΔS dS
F = F12 + F22 = 4,10 x 10-9 N
Densidad de carga lineal definida por
d) W = mg = (3,82 x 1026)(9,8) = 3,7 x 105N
Como las fuerzas eléctricas que actúan en objetos Δq dq
λ = lim =
cargados pequeños son mucho más grandes que sus Δl →0 Δl dl
pesos, a menudo no se toman en cuenta.
Ejemplo 17. ¿Cuál sería la fuerza sobre une carga q,
Ejemplo 16. Dos esferas idénticas de tecknopor, cada debido a cargas distribuidas por volumen?
una de 0,030 kg, cada esfera atada a un hilo de 30 cm Solución.
de largo y suspendidas de un punto. A cada esfera se
le da una carga q (frotándola con un paño), las dos
esferas se repelen formando un ángulo de 7º con la
vertical. ¿Cuál es la carga en cada esfera?
Solución. Dibuje el diagrama de fuerzas para una de
las esferas. La esfera está en equilibrio, tal
que: ∑F H =0 y ∑F
V =0

Sea al volumen V con densidad de Carga ρ (r ' ) . La


fuerza sobre la Carga q es:
→ →
→ q r − r'
Fq =
4πε 0 ∫
V → →3
ρ (r ' ) dV '
r − r'
T cos θ − mg = 0 ⇒ T cos θ = mg
Del mismo para una distribución superficial, con
Tsenθ − F = 0 ⇒ Tsenθ = F
densidad de Carga σ ( r ' )
Tsenθ F
Dividiendo: = tan θ = , donde → →
T cos θ mg → q r − r'
q2
Fq =
4πε 0 ∫
S → →3
σ (r ' ) dS '
F =k r − r'
(2 Lsenθ )2
Resolviendo: y para una distribución lineal, con densidad de Carga

q2 =
(mg tan θ )(2 Lsenθ ) 2
λ(r ' )
k

9
Electrostática Hugo Medina Guzmán

→ →
ρ = A r , donde A es constante. Calcular la carga de
→ q r − r'
Fq =
4πε 0 ∫
l → →3
λ(r ' ) dl' la esfera.
Solución.
r − r'

Ejemplo 18. La figura muestra un alambre infinito


horizontal con densidad lineal de carga λ, y un
alambre finito de largo y densidad lineal de carga λ’ =
2.
λ (y + a), donde λ es una constante de unidad C/m
0 0 dq
Si ρ= ⇒ dq = ρdV
dV
r=R A
y q=∫ ρdV , con ρ = y dV = 4πr 2 dr :
r =0 r
R A R
q = ∫ 4πr 2 dr = 4πA∫ rdr = 2πAR 2
0 r 0

Ejemplo 20. Se tiene una esfera aisladora con


a) ¿Cuál es la fuerza que el alambre infinito ejerce densidad de carga variable de la forma
sobre el alambre de largo l ? ρ = (ρ 0 r 2 )e − r y radio R limitada exteriormente
b) ¿Cuál es la fuerza que el alambre de largo l ejerce por una esfera conductora de radio interior R y
sobre el alambre infinito?  exterior 2R. En la esfera conductora hay una carga
Solución. neta tal que el potencial exterior (r > 2R) es
a) La fuerza que el alambre infinito ejerce sobre el constante. Determine la carga total en la esfera
alambre de largo l : aisladora.

Solución. La carga total en una región del espacio


donde hay una densidad de carga ρ está dada por la
→ → → λ ˆj , integral:
d F = dq E , E =
2πε 0 (a + y ) q = ∫ ρdV
V
dq = λ ' dy = σ 0 (a + y )dy En este caso, donde la simetría es esférica, la
expresión toma la forma:
→ λ
Luego: d F = σ 0 (a + y )dy ˆj 2π π e− r 2
2πε 0 (a + y )
R
q=∫ ∫∫ ρ0 r senθ dr dθ dφ
0 0 0 r2
λσ 0 dy ˆ
=
2πε 0
j = 4πρ0 ∫
0
R
(
e − r dr = 4πρ0 1 − e − R )
→ → λσ 0 ˆ l λσ 0 l
F = ∫d F =
2πε 0 ∫0
j dy = ĵ CAMPO ELECTRICO - INTRODUCCION.
2πε 0 Nosotros conocemos la existencia del campo
b) La fuerza que el alambre de largo l ejerce sobre el gravitacional porque al dejar en un punto del espacio
(cerca de la tierra) una masa m esta sufre la acción de
alambre infinito por la tercera ley de Newton es: →
la fuerza gravitacional F , habiendo en ese punto
→ λσ 0 l ˆ →
−F =− j una intensidad de campo gravitacional g , donde
2πε 0

→ F
Ejemplo 19. Una esfera maciza, no conductora de g=
radio R, tiene una densidad de carga volumétrica m
Este valor en el punto no cambie con el tiempo,
depende de la masa de la tierra y de la distancia
10
Electrostática Hugo Medina Guzmán

→ mM T → GM Ejemplo 21. Un dipolo eléctrico consiste en las


F =G 2
ˆ
r y g = 2 T rˆ cargas + q y - q separadas por una distancia. 2a Si
r r las cargas se colocan en (0, 0, a) y (0, 0, - a) en el eje
La Ley de Coulomb establece la fuerza de interacción de z (el eje del dipolo), determine el campo eléctrico
entre dos cargas, pero cuando quitamos una de las del dipolo en un punto a una distancia z del origen en
cargas ¿qué hay en ese espacio? Similarmente al el eje de z, donde z >> 2 a. Exprese el resultado en
campo gravitacional podemos decir que el espacio los términos del momento dipolo eléctrico, definido
que rodea a la carga esta afectado por lo que como p = 2aq .
llamamos campo eléctrico.
Solución.
DEFINICIÓN DE CAMPO ELÉCTRICO
Sea una carga q1 fija en el espacio, la acción sobre la
carga q2 es
→ 1 q1 q 2 → 1 q1 q 2
F= r 12 = rˆ
4πε 0 r123
4πε 0 r122
Sí variamos la posición de q2 la fuerza en cada punto
dependerá de las coordenadas de su posición. Para
eliminar la dependencia de la fuerza a q2, se
especifica esta como carga unitaria y positiva. Así el
E=
1 q
+
1 (− q )
campo fuerza se define como la fuerza por unidad de
carga positiva en todos los puntos alrededor de q1, un
4πε 0 (z − a ) 2
4πε 0 (z + a )2
resultado equiva1ente se obtiene dividiendo la fuerza 1 ⎛ 1 1 ⎞
en cada punto por el valor de q2, esto es = q⎜ 2 − 2 2 ⎟

4πε 0 ⎝ z − 2az + a 2
z + 2az + a ⎠
→ F 1 q1 → 1 q1 Como a << 2az << z , podemos eliminar a
2 2 2
E= = r 12 = rˆ
q 2 4πε 0 r12
3
4πε 0 r122 1 1
=
z − 2a z
2
⎛ 2a ⎞
Para el caso del Campo debido a cargas distribuidas. z 2 ⎜1 − ⎟
- Cargas puntuales. Campo producido por las n cargas ⎝ z ⎠
puntuales ( q1 , q 2 , q3 , q2 ….. q 2 )en un punto 1
Usando ≈ 1 + δ , si δ << 1 :
→ 1−δ
determinado por r .
1 1 ⎛ 2a ⎞
⎛→ → ⎞ = 2 ⎜1 + ⎟
⎜r− ri ⎟
n 2⎛
z ⎜1 − ⎟
2a ⎞ Z ⎝ z ⎠
→ 1
E=
4πε 0
∑ qi ⎝
i =1

3 → →
⎝ z ⎠
r− ri 1 q ⎡ 2a ⎛ 2a ⎞ ⎤
E= 1+ − ⎜1 − ⎟ ⎥
- Distribución volumétrica
4πε 0 z 2 ⎢⎣ z ⎝ z ⎠⎦
⎛→ → ⎞ aq
⎜r− ri ⎟ =
πε 0 z 2
→ 1 ⎝ ⎠dV '
E=
4πε 0 ∫ ρ( r ')
→ → 3 p
V
r− ri o E=
2πε 0 z 2
- Distribución superficial La materia en la naturaleza es por general
eléctricamente neutra, no se encuentra tan a menudo
⎛→ →

⎜r− ri ⎟ situaciones donde las fuerzas se deben a la carga neta
→ 1 ⎝ ⎠dS ' en un objeto. Sin embargo, todo puede adquirir un
E=
4πε 0 ∫σ ( r ')
→ → 3 momento dipolar cuando está colocado en un campo
S
r− ri eléctrico porque las cargas negativas en los átomos
son jaladas en una forma tirada y las cargas positivas
- Distribución lineal en el núcleo son jaladas en la dirección opuesta. Así
los dipolos eléctricos desempeñan un papel muy
⎛→ → ⎞
⎜r− ri ⎟ importante en nuestra comprensión de la materia.
→ 1 ⎝ ⎠ dl '
E=
4πε 0 ∫ λ( r ')
→ → 3
Cuando un dipolo eléctrico se coloca en un campo
eléctrico, tiende a alinearse con su eje paralelo al
l
r− ri campo. Si no es paralelo al campo, experimenta un

11
Electrostática Hugo Medina Guzmán

→ π
torque τ . Asociado a este torque hay una energía y x = ∞.→ θ=
potencial U, donde 2
→ → → → → De aquí
τ = p× E y U = − p⋅ E = − pE cos θ π
π
1 senθ 2 2
Aquí θ es el ángulo entre el campo eléctrico y el eje I 1 = ∫ 2π cos θdθ = =
del dipolo y el p = 2aq es el momento del dipolo. r −2 r −
π r
2

Ejemplo 22. Campo eléctrico de una línea recta Integrando I2:


infinita con carga λ Coulombios/metro.
∞ xdx
I2 = ∫
Solución. Consideremos la línea como se muestra en
la figura el punto P situado a una distancia r de la
−∞
(r 2
+ x2 )32


recta, ∞ xdx 1
= ∫ (r
−∞ 2
+ x2 )
32
= −
(r 2
+ x2 )
12
=0
−∞
Valor que esperábamos para I2 ya que al haber
simetría los componentes horizontales se anulan entre
sí.
Finalmente, reemplazando los valores de I1 e I2.
→ 1
E= iˆ
La carga del elemento dx es dq = λ dx 2πε 0 r
El campo eléctrico en P producido por este elemento Como el campo sale radialmente, el resultado puede
es: escribirse.
→ 1 dq →
→ 1
dE= r 12 E= rˆ
4πε 0 r123 2πε 0 r
→ → → → →
Donde r 12 = r 2 − r 1 y r12 = r2 − r1 con Recta finita. Para el caso de una recta finita la
variación son los límites, veamos por ejemplo,
→ → encontrar el campo eléctrico en el punto P de la figura
r1 = xiˆ y r2 = rˆj
( )
→ 12
r 12 = rˆj − xiˆ ⇒ r12 = r 2 + x 2
λdx
1
( )

Luego: d E = rˆj − xiˆ
4πε 0 r + x 2 3 2
2
( )
→ λ ⎡ rdx ˆj − xdx ⎤
ˆ⎥
o dE= ⎢ 2 i
4πε 0 ⎣⎢ (r + x 2 )3 2 (r 2 + x 2 )3 2 ⎦⎥ aquí x varía de x = −l 1 a x = l 2
la integración
El campo eléctrico total lo encontraremos integrando θ2
desde x = −∞ hasta x = ∞ . 1 θ2 senθ
→ λ ⎡ ∞ rdx ∞ xdx ⎤
I1 = ∫
r θ1
cos θdθ =
r
⎢ ∫−∞ 2 ∫
ˆj − θ1
E= iˆ ⎥
4πε 0 ⎣⎢ (r + x ) 2 32 − ∞
(r + x 2 )3 2
2
⎦⎥ 1
= (senθ 2 − senθ1 )
λ r
= [I 1 − I 2 ]
4πε 0 1⎡ l2 l1 ⎤
= ⎢ + ⎥
Integrando I1:
(
r ⎢⎣ r 2 + l 22 1 2
)
r 2 + l 12 ( ) 12
⎥⎦
∞ rdx
I1 = ∫ (r
l2

−∞ 2
+x )
2 3 2
I2 = ∫
l2

−l1
(r 2
xdx
+ x2 )32
= −
(r 2
1
+ x2 )
12

Cambiando variable: x = r tan θ −l 1

⇒ dx = r sec 2 θdθ ⎡ l2 l1 ⎤
= −⎢ − ⎥
π
Los límites x = −∞ → θ =− ⎢⎣ (r 2 + l 2 12
2 ) (r 2
+l 2 12
1 ) ⎥⎦
2

12
Electrostática Hugo Medina Guzmán

El campo eléctrico es E =
→ λ
[I 1 − I 2 ]

dE =
λR
(
zdθkˆ − R cos θdθiˆ − Rsenθdθˆj )
4πε 0 4πε 0 z 2 + R 2(32
)
El campo eléctrico total lo encontramos integrando
→ λ ⎧⎪ 1 ⎡ l2 l1 ⎤ desde θ = 0 hasta θ = 2 π .
E= ⎨ ⎢ 2 + ⎥ ˆj
4πε 0⎪⎩ ⎢⎣ (r + l 2 ) (r + l 1 ) ⎥⎦
2 12 2 12 → 2π →
r 2
E=∫ dE =
0
⎡ l2 l1 ⎤ ⎫⎪ λR ⎛⎜ z 2π dθkˆ − R 2π cos θdθiˆ − R 2π senθdθˆj ⎞⎟
+ ⎢ − ⎥iˆ⎬ ( 2 32
) ⎝ ∫0 ∫0 ∫0 ⎠
( ) ( ) 4πε 0 z + R 2
2 12 2 12
⎢⎣ r + l 2
2
r + l 1 ⎥⎦ ⎪⎭
2
La primera integral es 2π , las dos últimas son 0,
como era de esperar ya que sobre el eje la resultante
Ejemplo 23. Campo eléctrico en el eje de un anillo del campo eléctrico por simetría debe ser vertical.
de radio R y carga λ Coulombios/metro Finalmente el Campo es
→ λRz
E= kˆ
(
2ε 0 z 2 + R 2 )
32

Si el anillo tiene una carga total Q:


→ Qz
Q = 2πRλ y E = kˆ
4πε 0 (z + R 2
)
2 32

Ejemplo 24. Un segmento de línea de carga positiva,


tiene una densidad lineal de carga uniforme λ . Este
Solución. Consideremos un elemento de anillo dl , se dobla en la forma indicada en la figura.
determinado por el ángulo θ y barrido por dθ de
tal manera que dl = Rdθ .
La carga del elemento dl es dq = λdl = λRdθ
El campo eléctrico producido por este elemento en P
Halle:
es
a) el campo eléctrico generado por la
→ dq →
1 semicircunferencia en el punto O.
dE= r 12
4πε 0 r123 b) el campo eléctrico generado por cada porción recta
en el punto O.
→ → → → →
Donde r 12 = r 2 − r 1 y r12 = r2 − r1 c) la fuerza eléctrica que ejercerá una carga puntual
positiva q ubicada en el punto O sobre la línea de

carga completa.
con r1 = xiˆ + yˆj = R cos θ iˆ + Rsenθ ˆj Solución.

a)
y r2 = zkˆ

→ dq
dE= (− cos θ iˆ − senθ ˆj )
→ 4πε 0 R 2
r 12 = zkˆ − R cos θ iˆ − Rsenθ ˆj , λdθ
(− cosθ iˆ − senθ ˆj )
r12 = (z + R )
2 12 =
2
4πε 0 R
λ
(− cosθ iˆ − senθ ˆj )dθ
Luego → π

4πε 0 R ∫0
E=
dE=
→ 1 λRdθ
(zkˆ − R cosθiˆ − Rsenθˆj )
(
4πε 0 z + R 2 2
)
32

λ ˆ
O = − j
2πε 0 R
b) Campo debido al lado izquierdo:
13
Electrostática Hugo Medina Guzmán

R
zσ ⎡ 1 ⎤
= ⎢− 2 ⎥ kˆ
2ε 0 ⎣⎢ (z + r 2 )1 2 ⎦⎥
0

→ σ ⎡ z ⎤
dqλdx ˆ E= ⎢1 − 2 ⎥ kˆ
( )

dE= iˆ = i 2ε 0 ⎣⎢ z + R 2 12
⎦⎥
4πε 0 x 2
4πε 0 x 2
Integrando: Si la carga total del disco es Q: Q = σ πR 2
→ λ ˆ − R dx λ ˆ ⎡ 1 ⎤
−R → Q ⎡ z ⎤
⎥ kˆ
4πε 0 ∫−3 R x 2 4πε 0 ⎢⎣ x ⎥⎦ − 3 R
E izquierda = i = i − y E= ⎢1 −
2πε 0 R 2 ⎣⎢ (z 2 + R 2 )1 2 ⎦⎥
λ ˆ⎡ 1 1 ⎤ λ ˆ
= i⎢ − = i
4πε 0 ⎣ R 3R ⎥⎦ 6πε 0 R Para el caso de un PLANO INFINITO la integración
se realiza desde r = 0 hasta r = ∞ y el campo
Similarmente encontramos el campo debido al lado eléctrico es
derecho: → σ ˆ
→ λ E= k
E derecha = − iˆ 2ε 0
6 πε 0 R
c) Si colocamos una carga q en el punto O, Ejemplo 26. Ahora veamos el Campo eléctrico en el
→ →
qλ ˆj
eje producido por un cilindro de radio R, largo L y
F = q E total = − carga ρ C/m3.
2πε 0 R
Aplicando la tercera ley de Newton, la línea será
empujada por una fuerza igual a:

ˆj
2πε 0 R

Ejemplo 25. Campo eléctrico en el eje de un disco de


radio R y carga σ C/m2.

Solución. Para evaluar el Campo eléctrico en el


punto P, tomemos un elemento diferencial con forma
de disco de radio R y espesor dz cuyo volumen es
dV = πR 2 dz y su carga dq = ρdV = ρπR 2 dz .
El Campo eléctrico producido por este elemento en el
punto P es: (utilizando el resultado del disco)
Solución. Consideremos un elemento anular de radio → ρπR 2 dz ⎧⎪ (z 0 − z ) ⎫⎪
dE= 2 ⎨
1− 12 ⎬

r y espesor dr cuya superficie es dA = 2πrdr , con
carga dq = σ dA = 2π σ rdr .
2πε 0 R ⎪
⎩ [
( z 0 − z )2 + R 2 ] ⎪⎭
El Campo eléctrico total lo encontramos integrando
El Campo eléctrico producido por este elemento en P
es: desde z = − L 2 hasta z = L 2 .
(utilizando el resultado obtenido para un anillo). → L2 → ρ ⎧⎪ L 2 L2 (z 0 − z )dz ⎫⎪
E=∫ dE= ⎨∫− L 2dz − ∫− L 2 ⎬kˆ
zdq [ ]

2ε 0 ( z 0 − z )2 + R 2
12
dE = kˆ
−L 2
⎪⎩ ⎪⎭
4πε 0 z + r ( 2
)
2 32
→ ρ
{ [ ] }
12 L 2
z + (z 0 − z ) + R 2 kˆ
2
E=
→ zσ rdr 2ε 0 −L 2
⇒ dE = kˆ
2ε 0 (z 2 + r 2 )
32
ρ ⎧ ⎡⎛ 2

12
⎡⎛ 2
⎤ ⎫⎪
12

⎪ L⎞ L⎞
E= + − + − + + ⎬kˆ
2 2
⎨ L ⎢ ⎜ z ⎟ R ⎥ ⎢ ⎜ z ⎟ R ⎥
2ε 0
0 0
El Campo eléctrico total lo encontramos integrando ⎪⎩ ⎢⎣⎝ 2⎠ ⎥⎦ ⎢⎣⎝ 2⎠ ⎥⎦ ⎪⎭
desde r = 0 hasta r = R .
→ R → zσ R rdr Ejemplo 27. Campo eléctrico producido por una
E=∫ dE= ∫ (z kˆ carga σ C/m2 distribuida sobre une superficie
0 2ε 0 0 2
+r )
2 32
esférica de radio R.
14
Electrostática Hugo Medina Guzmán

En el caso de entrar el punto P dentro de la esfera o


sea r < R , los límites son:
l = R−r y l = R+r
σR ⎡ (r 2 − R 2 )⎤ ˆ
R+r

De aquí: E= ⎢l − ⎥ k
4ε 0 r 2 ⎣ l ⎦ R−r
σR
= (0)kˆ = 0
4ε 0 r 2
⎧ → Q ˆ
⎪ Para r > R ⇒ E = k
Solución. Tomemos un anillo diferencial 4πε 0 r 2
determinado por el ángulo θ y barrido por dθ , su ⎪
área es: dS = ( 2πRsenθ ) Rdθ ,
⎪ → Q
En resumen ⎨Para r = R ⇒ E = kˆ
donde ⎪ 4πε 0 R 2

R = radio de la esfera ⎪ →
2πRsenθ = circunferencia del anillo ⎪Para r < R ⇒ E = 0
Rdθ ancho del anillo ⎩
su Carga es dq = σdS = σ (2πRsenθ ) Rdθ
LINEAS DE FUERZA ELECTRICA
El Campo producido por este elemento en el punto P →
es (utilizando la expresión del Campo eléctrico Para una mejor visualización del campo eléctrico E
obtenido para un anillo) Faraday ideó un modelo de campo eléctrico formado
dE=
→ [σ (2πRsenθ ) Rdθ ](l cosφ ) kˆ por líneas de fuerza. La relación entre las líneas de
fuerza y el vector campo eléctrico son:
4πε 0l3 a) La tangente a una línea de fuerza en un punto
→ σR 2senθ cos φ ˆ →
dE= dθk (1) cualquiera de la dirección de E en ese punto.
4πε 0l 2 b) Las líneas de fuerza se dibujan de tal manera que
Aplicando la ley de los cosenos al triángulo OPM el número de ellas por unidad de área que atraviesan

R 2 = l 2 + r 2 − 2lr cos φ y sea proporcional a la magnitud de E , es decir donde
l 2 = R 2 + r 2 − 2 Rr cosθ →
las líneas están cercanas, E es grande y donde están
l +r −R 2 2 2

De la primera obtenemos: cos φ = separadas, E es pequeño.
2lr
Si tenemos una carga puntual positiva las líneas de
Derivando la segunda: 2ldl = 2 Rrsenθdθ fuerza salen de ella. En los puntos cercanos las líneas
ldl están más juntas que en los alejados lo que nos
⇒ senθdθ = muestra que el campo eléctrico es más intenso cerca y
Rr
se va disipando a medida que el punto se aleja de la
Sustituyendo cos φ y senθdθ en la ecuación (1)
carga.
→ σR ⎛ r 2 − R2 ⎞ ˆ
dE= ⎜⎜1 + ⎟⎟dlk
4ε 0 r 2 ⎝ l2 ⎠
Para obtener el campo eléctrico total integrarnos entre
l=r−R y l=r+R
→ r+R → σR r+R ⎛ r 2 − R2 ⎞ ˆ
E=∫ dE= ∫r − R ⎜⎜⎝1 + l 2 ⎟⎟dlk
r−R 4ε 0 r 2 ⎠
σR ⎡

(
r 2 − R2 ⎤ ˆ )σR
r+R

(4 R )kˆ
= ⎢ l ⎥ k =
4ε 0 r 2 ⎣ l ⎦ r−R 4ε 0 r 2

σ R2 ˆ
= k
ε 0r 2 Las líneas de fuerza de una carga negativa se
representan como líneas que van hacia la carga.
= σ 4πR
Siendo la carga total Q
2

→ 1 Q ˆ
Tenemos para r > R : E = k
4πε 0 r 2
15
Electrostática Hugo Medina Guzmán

En el caso de dos Cargas positivas iguales la figura


siguiente nos muestra las líneas de fuerza.

Por medio del uso de aceite, semillas de grama y


empleando un pequeño electrodo cilíndrico que se
carga con el generador de Wimshurt, se obtienen las
líneas de campo para una carga puntual

Por medio del uso de aceite, semillas de grama y


empleando dos electrodos cilíndricos que se cargan

de igual signo con el generador de Wimshurt, se
Si dibujamos los vectores del campo eléctrico E en
obtienen las líneas de campo para dos cargas
los varios puntos alrededor de nuestra distribución de
eléctricas puntuales del mismo signo.
carga, una figura del campo eléctrico comienza a
emerger.

FLUJO DEL CAMPO ELÉCTRICO.


Vemos que las líneas salen o entran hacia la carga en
forma de un flujo de Campo eléctrico.
En el caso de dos cargas diferentes la figura a
Para poder encontrar una relación cuantitativa entre
continuación nos muestra las líneas de fuerza
1as líneas de fuerza y la intensidad del campo
eléctrico definimos que el número de líneas ΔΦ que
ocupa un elemento de superficie ΔS , perpendicular
a ésta, como el promedio de la intensidad de campo
(E n ) en ΔS , es decir:
ΔΦ = E n ΔS

Por medio del uso de aceite, semillas de grama y


empleando dos electrodos cilíndricos que se cargan
de distinto signo con el generador de Wimshurt, se
obtienen las líneas de campo de dos cargas eléctricas
puntuales de distinto signo. Para poder relacionar el número de líneas con el
campo eléctrico en cualquier punto, hacemos que
ΔS → 0 y dΦ = EdS .

16
Electrostática Hugo Medina Guzmán

Para quitar la restricción de perpendicularidad de la Φ q 50 × 10 −6


Φc = =
( )
superficie a las líneas de fuerza (es decir el campo =
eléctrico) podemos escribir vectorialmente 6 6ε 0 6 8,85415 × 10 −12
→ → 2
dΦ = E⋅ d S Nm
= 0,94 x 106
El número de líneas a través de una superficie S será C
→ →
Φ = ∫ dΦ = ∫ E⋅ d S Ejemplo 30. En la superficie cerrada de la figura a =
0,5 m, b = 0,4 m, c = 0,3 m e y0 = 0,2 m. El campo
A esto se llama propiamente FLUJO ELECTRICO.
electrostático en que está sumergida no es
homogéneo y viene dado en el SI por
Ejemplo 28. Encuentre el flujo total debido a una
( )

carga puntual q. E = 4 + 3 y 2 ˆj .
Solución. A una distancia r de la carga el campo
Determinar la carga neta encerrada en la superficie.
→ 1 q
eléctrico es: E = rˆ , ese campo
4 πε 0 r2

atraviesa una superficie d S = rˆdS .

El flujo eléctrico a través de d S es

→ → 1 q
Φ = ∫ dΦ = ∫ E ⋅ d S = ∫ rˆ ⋅ rˆdS
S S 4πε 0 r 2 Solución. Aplicando la definición de flujo a cada una
q1 de las áreas que forman el paralelepípedo las únicas
4πε 0 r 2 ∫S
Φ= dS en que éste no es cero son las paralelas al plano xz (en
las demás es nulo puesto que el vector campo

electrostático y los vectores área son
El valor de ∫ d S , siendo el lugar geométrico de un
S perpendiculares); llamando A1 la que dista y0 del
punto a una distancia r de la carga puntual es el área plano xz y A2 la que dista y0 + b, y teniendo en cuenta
→ →

∫ d S = 4πr que E y sus vectores área son paralelos:


2
de una esfera de radio r:
S → → → → → →
q 1 1 q Φ = ∫ E ⋅ d S = ∫ E1 ⋅ d S1 + ∫ E 2 ⋅ d S 2
2 ∫S
Luego: Φ = dS = 4πr 2 S S1 S2
4πε 0 r 4πε 0 r 2

( )
→ →

q E1 = 4 + 3 y02 ˆj , d S1 = − dS1 ˆj y
Φ= → →
ε0 E2 = [4 + 3( y0 + b)] ˆj , d S 2 = dS 2 ˆj
2

Con esto:
Ejemplo 29. En el centro geométrico de un cubo de 2
m de arista tenemos una carga de 50 μC . Calcular el
(
Φ = − 4 + 3 y 02 )∫ dS + [4 + 3( y + b)] ∫
S1
1 0
2
S2
dS 2

= − (4 + 3 y )ac + [4 + 3( y + b )] ac
flujo eléctrico que atravesará a cada una de las caras 2 2
del cubo. (El medio que se considera es el vacío). 0 0

= 3abc(b + 2 y 0 )
Como este flujo es debido a la carga encerrada en la
q
superficie tenemos: Φ =
ε0
Q
Igualando: 3abc(b + 2 y 0 ) =
ε0
⇒ Q = 3abc(b + 2 y 0 )ε 0
Solución. El flujo total debido a la carga es
q Ejemplo 31. Una carga puntual q se encuentra a una
Φ= distancia d en el eje de un disco de radio R. ¿Cuál es
ε0 el flujo a través de este disco?
Como el cubo tiene seis caras y la carga está en el Solución.
centro geométrico de éste, el flujo que atraviesa una
de ellas será:

17
Electrostática Hugo Medina Guzmán

Consideremos una carga puntual q y encontremos el


flujo del campo eléctrico a través de una superficie
esférica de radio r concéntrica con la carga.

La figura muestra una sección dS perteneciente al


disco de radio R.
El Campo eléctrico de la esfera es producido por la
→ →
El flujo a través de dS es: dΦ = E ⋅ d S carga en cada punto
Donde:
→ q
E= rˆ

E=
q
4πε 0 (r 2 + d 2 )
(
senβ cos θ iˆ + cos β ˆj + senβsenθ kˆ ) 4πε 0 r 2
→ →

, d S = rdrdθ ˆj :
El flujo a través de la esfera es Φ = ∫E⋅d S
S

q cos βrdrdθ donde d S = rˆdS
de aquí dΦ =
4πε 0 r 2 + d 2 (
, como
) q q
d
y Φ= ∫
4πε 0 r
S
rˆ ⋅ rˆdS =
4πε 0 r 2
2 ∫ dS
S
cos β = :
(r +d22
) 12
=
q
4πr 2 =
q
( )
qd rdrdθ 4πε 0 r 2
ε0
dΦ =
4πε 0 r 2 + d 2 ( ) 32 En resumen:
→ → q
E]. flujo a través del disco es: Φ = ∫E⋅d S =
S ε0
R 2π qd rdrdθ El flujo a través de la esfera es proporcional a la
Φ=∫ ∫ carga es independiente del radio de la superficie.
0 0
4πε 0 r 2 + d 2 ( )
32
Consideremos ahora el caso de una carga q encerrada
por una superficie arbitraria S como se muestra en la
2π figura siguiente.
R qd rdr
= ∫ 0
4πε 0 r 2 + d 2 ( )32
θ
0

R qd rdr
= ∫
0
(
2ε 0 r 2 + d 2 ) 32

R
qd
= −
( )
→ →

∫E⋅d S
12
2ε 0 r 2 + d 2 El flujo a través de le superficie S es Φ =
0 S

Como d S = nˆ dS
qd ⎡1 1 ⎤ →
= ⎢ − 2 12 ⎥ Φ = ∫ E ⋅ nˆ dS = ∫ E cos θ dS
2ε 0 ⎣⎢ d r + d
2
( ) ⎦⎥ S S
q
q ⎡ d ⎤
= ∫
4πε 0 r 2
S
cos θ dS
= ⎢1 − 2 12 ⎥
2ε 0 ⎢⎣ r +d2 ( ) ⎥⎦ =
q

cos θ dS
4πε 0 S r2
LA LEY DE GAUSS Pero

18
Electrostática Hugo Medina Guzmán

cos θ dS 1 q1
dS cos θ = r 2 dΩ ⇒ = dΩ , el ángulo E=
r2 4πε 0 ε 0
sólido subtendido por el elemento de superficie dS , → → q1
q Φ = ∫ E ⋅ d S = E 4πr 2 =
4πε 0 ∫
luego. Φ = dΩ S ε0
1 q1
como el ángulo sólido total es 4π : ⇒ E=
q 4πε 0 r 2
Φ= (4π ) = q Si colocamos la carga q2 en un punto en que el campo
4πε 0 ε0 sea E, la fuerza electrostática sobre esta carga será:
1 q1 q 2
Este resultado es el mismo que en el caso de una F = q2 E =
superficie esférica, por lo tanto válido para cualquier 4πε 0 r 2
superficie. Obsérvese que este ejemplo es lógicamente recíproco;
Si la carga está fuera de la superficie cerrada, el flujo sin embargo, demuestra la equivalencia entre las
a través de dS1 es igual pero opuesto al flujo a través leyes de Gauss y de Coulomb.
de dS2
Ejemplo 33. Se tiene una línea infinita con una carga
de densidad lineal uniforme λ C/m. ¿Cuál es el
campo eléctrico a una distancia r de la línea?
Solución. En este caso la superficie gausiana
conveniente es un cilindro de radio r y longitud l
como se muestra en la figura siguiente.

de aquí que el flujo neto es igual a cero.


Si hay varias cargas dentro de la superficie arbitraria,
el flujo total es igual a la suma de los flujos
producidos por cada carga. Aplicando la ley de Gauss.
→ →
ε0 ∫ E ⋅ d S = q
La superficie la podemos suponer en 3 partes: (1) y
(3) las tapas y (2) el cilindro propiamente dicho. La
carga encerrada por la superficie es q = λl

ε 0 ⎛⎜ ∫ E⋅ d S + ∫ E⋅ d S + ∫ E⋅ d S ⎞⎟ = λl
→ → → → → →

⎝ 1 2 3

Con todo esto podemos establecer la ley de Gauss. Como E es perpendicular a la línea.
El flujo eléctrico a través de una superficie cerrada → → → →
que encierra varias cargas es: ∫ E⋅ d S = 0 y ∫ E⋅ d S = 0
1 2
→ → qi
Φ = ∫E⋅d S = ∑
→ →

S ε0 Luego ε 0 ∫ E ⋅ d S = λl ⇒ ε 0 E (2πrl ) = λl
2

Esta ecuación derivada para cargas puntuales puede λ


aplicarse para cualquier configuración, notando que y E=
2πε 0 r
la integral en caso que no exista simetría geométrica
se complica en su solución. → λ
Vectorialmente E = rˆ
→ → dq 2πε 0 r
Φ = ∫E ⋅d S = ∫
S Q ε0
Ejemplo 34. Se tiene un plano infinito con una
Ejemplo 32. Deducir la ley de Coulomb para dos densidad de carga superficial. σ ¿Cuál es el Campo
cargas puntuales q1 y q2 partiendo de la ley de Gauss. eléctrico a una distancia r del plano?
Solución. Aplicando la ley de Gauss y el concepto de Solución. Aquí conviene como superficie gausiana
flujo a una superficie esférica en cuyo centro se un cilindro de sección A y longitud 2r, como se
encuentra la carga eléctrica q1, deducimos: muestra en la figura siguiente.

19
Electrostática Hugo Medina Guzmán

carga − σ . ¿Cuál será el valor del Campo eléctrico


entre ellos y fuera de ellos?

Aplicando la ley de Gauss


→ →
ε0 ∫ E ⋅ d S = q
Aquí nuevamente tenemos las tapas y el cilindro
propiamente dicho, como el campo eléctrico es
perpendicular al plano la integral en la superficie
lateral es igual a cero, quedando solo la parte de las
tapas, la carga encerrada es igual a q = σ A . Solución.
σ Cálculo por la ley de Gauss directamente.
ε 0 (EA + EA) = σ A ⇒ E =
2ε 0
→ σ
Vectorialmente E = rˆ
2ε 0
El Campo es independiente de r.

Ejemplo 35. Una carga se distribuye con densidad


uniforme ρ a través del volumen de una gran placa
aisladora fina de espesor 4a.
Calcule el campo en todo el espacio, suponga que la
placa es paralela al plano xy y que el origen esta en el
centro de la placa. Sugerencia, use la simetría del
problema y evalúe para toda altura z.
Solución.
a) Para − 2a ≤ z ≥ 2a El campo fuera de las placas:
Con la superficie gausiana (a).

ε 0 ⎛⎜ ∫ E⋅ d S + ∫ E ⋅ d S + ∫ E⋅ d S ⎞⎟ = σA − σA
→ → → → → →

⎝1 2 3

ε 0 (EA + 0 + EA) = 0 ⇒ E = 0
→ q ρ 4aA El campo eléctrico fuera de las superficies es cero.
∫ E ⋅ nˆdS = ε0
⇒ EA + EA =
ε0 El campo entre las placas:
ρ Primera manera: Usando la superficie gausiana (b).
⇒ E=2 a
ε 0 ⎛⎜ ∫ E⋅ d S + ∫ E ⋅ d S + ∫ E⋅ d S ⎞⎟ = −σA
→ → → → → →
ε0
Para − 2a < z < 2a ⎝ 1 2 3

En la superficie que mira hacia afuera
→ →
E = 0: ∫ E⋅ d S = 0
1
En la cara lateral E es perpendicular a la
→ →

→ q ρ 2 zA
superficie ∫ E⋅ d S = 0
2
∫ E ⋅ nˆdS = ε0
⇒ EA + EA =
ε0
En la superficie que mira hacia
→ →

⇒ E=
ρ
z
adentro: ∫ E⋅ d S = − EA
3
ε0 Luego: − ε 0 EA = −σA
Ejemplo 36. Se tiene dos planos conductores σ
infinitos paralelos separados una distancia d, uno con ⇒ E=
densidad de carga + σ y el otro con densidad de ε0
Segunda manera: Usando la superficie gausiana (c).
20
Electrostática Hugo Medina Guzmán

ε 0 ⎛⎜ ∫ E⋅ d S + ∫ E⋅ d S + ∫ E⋅ d S ⎞⎟ = σA
→ → → → → →

⎝ 1 2 3

En la superficie que mira hacia afuera
→ →
E = 0: ∫ E⋅ d S = 0
1
En la cara lateral E es perpendicular a
→ →
la superficie ∫ E⋅ d S = 0
2
En la superficie que mira hacia
→ →
adentro: ∫ E⋅ d S = EA
3
Cuando los planos están uno frente al otro, los
campos se superponen y obtenemos los campos tal
Luego: ε 0 EA = σA como se muestra en la figura siguiente.
σ
⇒ E=
ε0
Las cargas se colocan en las superficies interiores de
los planos, frente a frente a frente como se muestra en
la figura a continuación.

σ σ
Fuera de los planos: E = − =0
2ε 0 2ε 0
σ σ σ
Entre los planos E= + =
2ε 0 2ε 0 ε 0

Cálculo por distribución de las cargas.


Originariamente las cargas están distribuidas como se
muestra en las figuras siguientes:
Cálculo por superposición. El plano con carga positiva.
Este problema se puede resolver por el método de la
superposición.
El campo debido al plano con carga positiva se indica
en la figura siguiente.

El plano con carga negativa.

El campo debido al plano con carga negativa se


indica en la figura siguiente.

21
Electrostática Hugo Medina Guzmán

Cuando los planos están cercanos, El plano con carga


positiva influye sobre el otro y recíprocamente, como
se muestra en la figura Los campos eléctricos se pueden hallar haciendo
superficies gaussianas tal como se muestran en la
figura siguiente.

Aquí aplicamos la ley de Gauss a cualquiera de los


planos y obtenemos los resultados anteriores.
b) Cuando se conecta a tierra tenemos una fuente
inagotable y receptor de carga negativa y las cargas
Ejemplo 37. Encontrar la distribución de cargas y el
se acomodan tal como se muestra en la figura
campo eléctrico en los espacios A, B, C y D en el
siguiente.
dispositivo de la figura para:
a) la llave S abierta.
b) Después de cerrar la llave S.

Los campos eléctricos se pueden hallar haciendo


superficies gaussianas como las mostradas.

Solución. Ejemplo 38. Se tiene dos hojas infinitas ubicadas en


a) La carga + 2σ se distribuye + σ en cada cara y x = d y x = − d con cargas σ y − 2σ
la carga − σ se distribuye − σ 2 en cada cara e respectivamente, tal como se muestra en la figura.
inducen las cargas tal como se muestra en la figura
siguiente.

22
Electrostática Hugo Medina Guzmán

a) Determine el campo eléctrico en todo el espacio.


b) Si entre las placas colocamos una placa infinita
conductora descargada de espesor t. ¿Cuales son las
densidades de carga superficiales inducidas sobre la
placa?

c) Si se conecta con un alambre conductor muy


delgado a la placa conductora con la hoja que está
ubicada en x = d, la densidad de carga superficial de
esta hoja queda tal como se indica en la figura
c) Si con un alambre conductor muy delgado siguiente.
conectamos a la placa conductora con la hoja que
está ubicada en x = d, ¿cuál es la densidad de carga
superficial de esta hoja?
Solución.
Nota: la solución es considerando como si la carga se
distribuyera uniformemente en los planos infinitos, lo
que no es así, ya que están frente a un borde, motivo
es porque la carga debe estar concentrada en una
línea e ir haciéndose mas tenue hasta cero en el
infinito.
Las densidades de carga en los bordes también tienen
ese problema, aun más considerando que son de
espesor muy pequeño (el que debería de ser
conocido) Ejemplo 39. Se tiene una esfera no conductora de
a) Distribución de las cargas. radio R con densidad de Carga por volumen ρ .
¿Cuál es el valor del campo eléctrico para todo
punto?
Solución.

El campo eléctrico:
→ σ
Para x < − d : E = iˆ
2ε 0
→ σ Para r<R
Para x > −d : E = iˆ
ε 0 ∫ E ⋅ d S = q ⇒ ε 0 E (4πr 2 ) = ρ ⎜ πr 3 ⎟
2ε 0
→ →
⎛4 ⎞
→ 3σ ˆ ⎝3 ⎠
Para − d < x < d : E = − i ρ
2ε 0 ⇒ E= r
3ε 0
→ ρ →
Vectorialmente E = r
3ε 0
Para r=R
ε 0 ∫ E ⋅ d S = q ⇒ ε 0 E (4πR 2 ) = ρ ⎜ πR 3 ⎟
→ →
⎛4 ⎞
⎝3 ⎠
ρ
b) Las cargas se acumulan en los bordes tal como se ⇒ E= R
muestra en la figura 3ε 0

23
Electrostática Hugo Medina Guzmán

→ ρ → →
Vectorialmente E = Rrˆ ε0 ∫ E ⋅ d S = q
3ε 0
Para r > R ( )⎛4 4 ⎞
⇒ ε 0 E 4πr 2 = ρ ⎜ πr 3 − πa 3 ⎟
→ → ⎝3 3 ⎠
ε0 ∫ E ⋅ d S = q ⇒
⇒ E=
ρ r −a
3
( 3
)
( ) ⎛4 ⎞
ε 0 E 4πr = ρ ⎜ πR 3 ⎟ = Q ⇒ E =
2 Q 3ε 0 r2
4πε 0 r 2
⎝3 ⎠ → ρ (r 3 − a 3 )
→ Q Vectorialmente E= rˆ
Vectorialmente E = rˆ 3ε 0 r2
4πε 0 r 2
Para r > b
→ →
ε0 ∫ E ⋅ d S = q

( ) ⎛4
⇒ ε 0 E 4πr 2 = ρ ⎜ πb 3 − πa 3 ⎟
4 ⎞
⎝3 3 ⎠

⇒ E=
ρ b −a(
3 3
) =
Q
3ε 0 r 2
4πε 0 r 2
→ Q
Vectorialmente E = rˆ
4πε 0 r 2
Si la esfera fuera conductora toda la carga estaría En el ejemplo siguiente presentamos un caso de
concentrada en la superficie y el Campo eléctrico en asimetría geométrica que es posible resolver con la
el interior o sea para r < R seria nulo. ley de Gauss.

Ejemplo 41. Se tiene una esfera conductora con una


cavidad esférica en su interior como se muestra en la
figura, y densidad de carga ρ . ¿Cuál es el campo
eléctrico en la cavidad?

Ejemplo 40. Como una variante veamos el caso de


una esfera no conductora con una cavidad
concéntrica.
Solución.

Solución. Vamos a determinar el campo eléctrico en


el punto P de la figura siguiente.

Sean los radios de la esfera a y b.


Para r < a
Como no hay carga encerrada por la superficie
gausiana
E=0
Para a < r < b

24
Electrostática Hugo Medina Guzmán

→ → →
El Campo E en P es igual a: E = E1 − E 2
→ ρ
Donde E1 = rˆ
3ε 0

→ q
Cálculo de E 2 ⇒ ∫ E ⋅ nˆdS = ε0
4
ρ πa 3
3 ρ
⇒ E 2 4πa 2 = ⇒ E2 = a
ε0 3ε 0

E = E12 + E 22
→ ρ
y E2 = rˆ' 2 2
3ε 0 ⎛ρ ⎞ ⎛ ρ ⎞ ρ
= ⎜⎜ a ⎟⎟ + ⎜⎜ a ⎟⎟ = 1,05 a
⎝ ε0 ⎠ ⎝ 3ε 0 ⎠ ε0

Ejemplo 43. Sea una esfera sólida aislante de radio a


con una densidad volumétrica de carga igual a
r
ρ = ρ0 , donde ρ 0 es una constante de unidad
a
3
C/m . La esfera aislante está dentro de una esfera
conductora hueca concéntrica de radio interior b y
radio exterior c. La carga neta de la esfera hueca
conductora es igual a cero.
a) Halle la carga total Q en la esfera aislante.
b) ¿Cuál es la densidad superficial de carga en cada
→ ρ ρ ρ
De allí que E = rˆ − rˆ' = (rˆ − rˆ') superficie de la esfera conductora?
3ε 0 3ε 0 3ε 0 c) A partir de la ley de Gauss, deduzca las
expresiones de la magnitud del vector campo
Pero rˆ − rˆ' = 2aiˆ eléctrico en términos de la distancia radial r, para las
→ ρ regiones: r < a, a < r < b,
Finalmente E = 2aiˆ b < r < c y r > c.
3ε 0 d) (1 punto) Grafique la magnitud del campo eléctrico
Ejemplo 42. Una carga se distribuye con densidad en función de r desde r = 0 hasta r = 2c.
uniforme ρ a través del volumen de una gran placa Solución.
aisladora fina de espesor 4a. a) La carga total Q en la esfera aislante es:
Suponiendo que se quita algo de material, dejando r
una cavidad esférica de radio a colocada como se Q = ∫ ρdV , ρ = ρ 0 y dV = 4π r dr
2

muestra en la figura, evalúe el vector de campo


V a
a r a ρ0
eléctrico en el punto Q = ∫ ρ0 4π r 2 dr = ∫ 4π r 3dr
P. 0 a 0 a
a
ρ0
= π r4 = πρ0 a
3

a 0

Solución. b) La densidad superficial de carga en la superficie



interior de la esfera conductora (r = b).
E = E1 kˆ − E 2 ˆj πρ0 a 3 ρ0 a 3
σb = − = −
ρ 4π b 2 4b 2
Cálculo de E1 ⇒ E1 = a
ε0 La densidad superficial de carga en la superficie
exterior de la esfera conductora (r = c).
25
Electrostática Hugo Medina Guzmán

πρ0 a 3 ρ 0 a 3
σc = =
4π c 2 4c 2
c) El campo eléctrico:
r < a,
Para r < a
→ →
ε0 ∫ E ⋅ d S = q ⇒
ρ0 ρ0
ε 0 E (4πr 2 ) = ∫
r
4π r 3dr = π r4 ⇒
0 a a
ρ
E = 0 r2
4aε 0 b) Al conectarlo a tierra en la superficie exterior
desaparece la carga. El campo exterior es cero.
Para a < r < b, →
E=0
ε 0 ∫ E ⋅ d S = q ⇒ ε 0 E (4πr 2 ) = Q ⇒
→ →

Q πρ0 a 3 ρ0 a3
E= = E= =
4πε 0 r 2 4πε 0 r 2 4ε 0 r 2
Para b < r < c
E=0
Para r > c.

ε 0 ∫ E ⋅ d S = q ⇒ ε 0 E (4πr 2 ) = Q ⇒
→ →

Q πρ0 a 3 ρ0 a3
E= = E= =
4πε 0 r 2 4πε 0 r 2 4ε 0 r 2 Ejemplo 45. Una bola que conductora pequeña de la
masa 10-3 g suspendida de un hilo por medio de
d) Gráfico del campo eléctrico en función de r desde aislador a una placa conductora grande en posición
r = 0 hasta r = 2c vertical. Cuando la placa lleva una carga eléctrica de
densidad superficial 5 x 10-8 C.m-2, la bola es
rechazada de la superficie y el hilo queda formando
un ángulo de 30° con la vertical. ¿Cuál es la carga
que ha pasado a la bola?

Ejemplo 44. Cargas por inducción en cáscaras


esferas conductoras concéntricas. Si una carga +q
se encuentra en el centro de una cáscara esférica de
metal. Encontrar el campo eléctrico fuera de la esfera.
a) Cuando la cáscara está libre. Solución. Hay tres fuerzas que actúan sobre la bola,
b) Cuando la cáscara se conecta a tierra. el peso mg que actúa hacia abajo, la fuerza F ejercida
Solución. por la placa que actúa horizontalmente, y la tensión T
a) Se observan los campos eléctricos inducidos por q en el hilo. Puesto que la bola está en equilibrio, las
y los inducidos en las superficies de la cáscara. En el componentes horizontales y verticales de las fuerzas
centro de la cáscara el campo es cero, pues ambos deben balancearse por separado. Así:
campos se anulan. El campo fuera de la cáscara no es
cero.
mg = T cos θ y F = Tsenθ
→ 1 q Dividiendo: F = mg tan θ
E= rˆ Pero F = Eq, donde E es el campo eléctrico debido a
4πε 0 r 2 la placa y q es la carga en la bola. ¿Además,
F = mg tan θ = Eq = σ q ε 0 , dónde σ es la
densidad de la carga en la placa. Así

26
Electrostática Hugo Medina Guzmán

ε 0 mg tan θ F − eE
q= ax = = y v = v 0 + 2ax = 0 en
2 2

σ m m

=
(8,85 × 10 )(1× 10 )(9,8)(1 3 )
−12 −6
z=
d
5 × 10 −8 2
-9
= 1,00 x 10 C. ⎛ eE ⎞⎛ d ⎞ mvo2
Luego 0 = v 0 − 2⎜ ⎟⎜ ⎟ ⇒ E =
2

Ejemplo 46. Una gota de aceite cargada de masa 2,5 ⎝ m ⎠⎝ 2 ⎠ ed


x l0-4 g está en el espacio entre dos placas, cada uno Se puede también solucionar esto usando los
de área 175 cm2. Cuando la placa superior tiene una principios de la energía. Así:
carga de 4,5 x l0-7 C y la placa más baja una carga Pérdida en K = trabajo hecho contra el campo
negativa igual, la gota sigue estando inmóvil. ¿Qué eléctrico E:
carga lleva? 1 2 ⎛d ⎞ mvo2
Solución. El campo eléctrico entre placas paralelas mv0 = (eE )⎜ ⎟ ⇒ E =
con cargas iguales y opuestas está dado por la
2 ⎝2⎠ ed
Q
ecuación E = σ ε0 = , Ejemplo 48. Dos placas paralelas cargadas grandes
Aε 0 se utilizan a menudo para crear un campo eléctrico
donde Q y A son la carga y el área de la placa uniforme E. Una partícula cargada disparada entre
positiva. las placas será desviada por el campo eléctrico. Esta
La fuerza sobre la gota del aceite es técnica se utiliza para desviar electrones en un tubo
qQ catódico (como en un osciloscopio) o para desviar
F = qE = , gotitas de tinta en una impresora de inyección de
Aε 0 tinta. ¿Suponga una partícula de masa m, carga q, y
y como esto balancea el peso de la gota, velocidad inicial v 0 , lanzada paralela a las dos placas
qQ donde el campo eléctrico es E. La longitud de las
mg = ⇒
Aε 0 placas es L. ¿A través de qué ángulo se desviará la
partícula?
mgAε 0
q=
Q
=
(2,5 × 10 )(9,8)(175 × 10 )(8,85 × 10 )
−7 −4 −12

4,5 × 107
-13
= 8,43 x 10 C

MOVIMIENTO DE UNA PARTÍCULA


CARGADA EN UN CAMPO ELÉCTRICO Solución.
UNIFORME No hay ace1eration en la dirección x, tal que
Considere una carga q conforme a un campo eléctrico L = v 0 t donde t = tiempo entre las placas.
uniforme en la dirección z. No considere la fuerza de
la gravedad puesto que normalmente es mucho más v x = v 0 = constante:
pequeña que la fuerza eléctrica. La fuerza qE da
lugar a la aceleración a z = F m = qE m y
⎛ qE ⎞⎛ L ⎞
y v y = v 0 y + at = 0 + ⎜ ⎟⎜⎜ ⎟⎟
a x = 0 . Podemos determinar el movimiento usando ⎝ m ⎠⎝ v0 ⎠
las ecuaciones de la cinemática. vy ⎛ qEL ⎞ qEL
1
tan θ = ⎜⎜
= ⎟⎟ = 2
vx v0
⎝ mv0 ⎠ mv0
Ejemplo 47. Dos placas paralelas muy grandes de
metal separaron por una distancia pequeña d tienen qEL
cargas opuestas uniformes, creando en el espacio ⇒ θ = tan −1
entre ellas un campo eléctrico E uniforme. Un
mv02
electrón de carga - e se lanza con velocidad inicial
POTENCIAL ELECTRICO
v 0 a través de un agujero pequeño en la placa
INTRODUCCION
positiva. Viaja hasta medio camino en el espacio ¿Cuál es el trabajo que se realiza para llevar una
entre las placas antes de detenerse y tomar dirección carga pequeña de un lugar a otro?
contraria. ¿Cuál es el valor de E en términos de la El trabajo contra la fuerza eléctrica para transportar
velocidad inicial v 0 ? una carga a lo largo de una trayectoria con velocidad
Solución. constante es igual al negativo de la componente de la
fuerza eléctrica en la dirección del movimiento,

27
Electrostática Hugo Medina Guzmán

1 GeV ≡ 109 eV

POTENCIAL ELECTRICO PRODUCIDO POR


UNA CARGA PUNTUAL.
Consideremos el campo debido a una carga q.
Tenemos que encontrar la diferencia de potencial
entre los puntos 1 y 2 determinados por r1 y r2
Integrando en la trayectoria respectivamente.
2→ →
W12 = − ∫ F ⋅ d l
1

F es la fuerza eléctrica sobre la carga en cada punto.

d l es el vector desplazamiento diferencial a lo
largo de la trayectoria.
Como la definición de la energía potencial se hace en
términos de la diferencia de energía potencial,
La trayectoria más fácil está mostrada en la figura, no
podemos escribir: → →
2→ →
hay trabajo de 1 a l’ ya que E y d l .tienen
U 2 − U 1 = W12 = − ∫ F ⋅ d l
1 direcciones perpendiculares en cada punto, luego
Es conveniente para nuestros .propósitos considerar queda solo la trayectoria de 1’ a 2.
el trabajo por unidad de carga y en éste caso la 2→ →
energía potencial se denomina simplemente potencial V2 − V1 = V12 = − ∫ E ⋅ d l
1
eléctrico. → q →
Nuevamente aquí la definición la hacemos en Donde: E = rˆ , d l = rˆdr
términos de la diferencia de potencial eléctrico 4πε 0 r 2
V2 – V1. 2 q
W 2 F


Reemplazando V2 − V1 = − ∫1 4πε 0 r 2
rˆ ⋅ rˆdr
V2 − V1 = 12 = − ∫ ⋅d l
q 1 q 2
q 2 dr q 1
donde q es la carga positiva usada para evaluar el
trabajo
V2 − V1 = −
4πε 0 r 2 ∫1 r 2
=
4πε 0 r 1

F →
Como = E , podemos escribir q ⎛1 1⎞
q V2 − V1 = ⎜ − ⎟
2→ →
4πε 0 ⎜⎝ r2 r1 ⎟⎠
V2 − V1 = V12 = − ∫ E ⋅ d l Así como tomamos la trayectoria 1, 1’, 2 podríamos
1
haber tomado cualquier trayectoria de 1 a 2 a la que
La unidad de potencial eléctrico es igual a la unidad
se divide fácilmente en tramos de trayectoria circular
de trabajo por unidad de carga, en el sistema
y radial como se muestra en la figura siguiente.
internacional es Joule/Coulombio (J/C), a. la que se
le ha dado el nombre de Voltio (V).
A partir de potencial eléctrico, podemos definir una
nueva unidad de energía
W12 = qV12
Si un electrón se mueve a través de una diferencia de
potencial de un voltio, gana o pierde un electrón-
voltio (eV) de energía.
La conversión a las unidades SI la hacemos de la
siguiente manera:
V12 = 1 V y q = 1,6 x 10-19 C
1 eV = 1,6 x 10-19 VC = 1,6 x 10-19 J
En estudios que implican partículas atómicas tales
como electrones y protones, el electronvoltio es una
unidad conveniente y muy comúnmente usada. Si el q ⎛1 1⎞
electronvoltio es demasiado pequeño, podemos medir De la ecuación V2 − V1 = ⎜ − ⎟ =
la energía de la partícula en MeV (millones de 4πε 0 ⎜⎝ r2 r1 ⎟⎠
electrón voltios) o GeV (mil millones de electrón q q
voltios o Gigavoltios). − que nos da la diferencia de
1 M eV ≡ 106 eV 4πε 0 r2 4πε 0 r1
28
Electrostática Hugo Medina Guzmán

potencial entre 1 y 2, es conveniente identificar En este caso es conveniente tomar como origen el
arbitrariamente los términos como → → →
q q punto P de tal manera que r = 0 y r − ri
V2 = y V1 = .
4πε 0 r2 4πε 0 r1
a 2 2
Escribiéndose el caso general de la manera siguiente. para los cuatro casos es o sea metros.
q 2 2
V( r ) = 1 (q1 + q 2 + q3 + q 4 )
4πε 0 r Luego V =
Cantidad que algunas veces se le denomina potencial 4πε 0 2 2
absoluto. Como resultado de esta elección el Reemplazando valores
potencial en el infinito es igual a cero.
V = 9 × 10 9
(
1 × 10 −9 + 3 × 10 −9 − 2 × 10 −9 + 2 × 10 −9 )
POTENCIAL ELECTRICO PRODUCIDO POR 2 2
UNA DISTRIBUCION DE CARGA V = 32 2 = 45,12 voltios.
Para calcular el potencial de n cargas q1, q2, q3
…….qn situadas en r1, r2, r3 …….rn. El potencial Ejemplo 50. Dipolo eléctrico. El dipolo eléctrico
eléctrico en el punto situado en r es igual a la suma de consiste en dos cargas iguales y opuestas q y - q
los potenciales producidos por cada una de las cargas. separadas una distancia pequeña 2a. Queremos
n
qi
V( r ) = ∑
encontrar el potencial eléctrico para puntos situados a
→ → una distancia r, para el caso de r >> a.
i =1
4πε 0 r − ri Solución.
La figura siguiente muestra un dipolo
Cuando la distribución de carga es continua
tendremos:
- Distribución volumétrica
1 ρ (r ' )
V( r ) =
4πε 0 ∫ V → →
dV '
r − r'
- Distribución superficial
1 σ (r ' )
V(r ) =
4πε 0 ∫
S → →
dS '
r − r'
- Distribución lineal El potencial en el punto P es
1 λ(r ' ) q q q ⎛1 1⎞
V = − = ⎜ − ⎟
4πε 0 ∫l → →
V(r ) = dl'
4πε 0 r1 4πε 0 r2 4πε 0 ⎜⎝ r1 r2 ⎟⎠
r − r'
De la figura vemos

Ejemplo 49. En la figura siguiente calcular el


(
r1 = r 2 − 2ar cos θ + a 2 )
12

12
potencial eléctrico en el punto P producido por las ⎛ 2a cos θ a 2 ⎞
cargas. Donde: q1 = 1x10- 9 C, q2 = 3x10- 9 C, q3 = - r1 = r ⎜⎜1 − + 2 ⎟⎟
2x10- 9 C, q4 = 2x10- 9 C y a = 1 m. ⎝ r r ⎠
−1 2
1 1 ⎛ 2a cos θ a 2 ⎞
y = ⎜1 − + 2 ⎟⎟
r1 r ⎜⎝ r r ⎠
Usando la expresión binomial para n = - ½

(1 + x )−1 2 = 1 − 1 x + 3 x 2 + ...
2
8
2a cosθ a 2
En nuestro caso x = − + 2
Solución. r r
El potencial en el punto P es de allí
1 1 ⎡ 1 ⎛ 2a cosθ a 2 ⎞ 3 ⎛ 2a cosθ a 2 ⎞ ⎤
2
n
qi
V( r ) = ∑ → →
= ⎢1 − ⎜⎜ −
r1 r ⎢ 2 ⎝ r
+ 2 ⎟⎟ + ⎜⎜ −
r ⎠ 8⎝ r
+ 2 ⎟⎟ + .....⎥
r ⎠ ⎥⎦

i =1
4πε 0 r − ri Expandiendo términos y manteniendo sólo los
términos con r2 y menores en el denominador
29
Electrostática Hugo Medina Guzmán

ya que r >> a. a)
1 1 2a cos θ q q q
≈ + Va = + +
r1 r r2 ( )
4πε 0 L 2 4πε 0 L 2 4πε 0 L 2
=
( ) ( )
Procediendo de igual manera para r2
3 2q
(
r2 = r 2 + 2ar cos θ + a 2 )12

4πε 0 L
W = QV a → b = Q (V − V b )
1 1 2a cos θ
≈ − b)
r2 r r2
1 1 q q q 3q
Sustituyendo y en V tenemos: Vb = + + =
r1 r2 4πε 0 L 4πε 0 L 4πε 0 L 4πε 0 L
Luego:
q ⎡⎛ 1 2a cos θ ⎞ ⎛ 1 2a cos θ ⎞⎤
V = ⎜ + ⎟−⎜ − ⎟ W = Q(Vb − Va )
4πε 0 ⎢⎣⎝ r r2 ⎠ ⎝ r r 2 ⎠⎥⎦
2aq cos θ =
3Qq ⎛
⎜1 −
1 ⎞ 3Qq 2 − 2
⎟=
( )
=
4πε 0 r 2 4πε 0 L ⎝ 2⎠ 8πε 0 L
c)
En esta expresión al producto 2aq lo llamaremos
momento dipolo p.
Luego
p cos θ
V =
4πε 0 r 2
donde la orientación de p es de - q a q para
considerarlo vectorialmente, con esto el potencial
eléctrico puede expresarse como producto escalar.
→ → →
p⋅ rˆ p⋅ r →
q ⎛⎜ iˆ ˆj kˆ ⎞⎟
V = = Eb = − + +
4πε 0 r 4πε 0 r 3 4πε 0 ⎜⎝ L2 L2 L2 ⎟⎠
2

Ejemplo 51. Sean tres cargas puntuales de magnitud


q ubicadas en vértices de un cubo de lado L, tal como
=
q
4πε 0 L2
(
− iˆ + ˆj + kˆ )
lo muestra la figura. La fuerza sobre la carga Q en el punto b:
Determine:
Qq
(− iˆ + ˆj + kˆ ) , F = 3Qq2

a) El potencial eléctrico en el vértice a producido por F=
las 3 cargas. 4πε 0 L2
4πε 0 L
b) El trabajo externo necesario para desplazar una
d) El potencial del punto con respecto al infinito es:
carga Q del vértice a al vértice b.
c) La fuerza eléctrica que ejercen las cargas q sobre la q q q 3q
Vb = + + =
carga Q ubicada en el punto b. 4πε 0 L 4πε 0 L 4πε 0 L 4πε 0 L
d) Estando la carga Q en el vértice b se la deja libre.
La diferencia de potencial entre el punto b y el
Las tres cargas q permanecen fijas en los vértices del
infinito es:
cubo durante el desplazamiento de Q. ¿Cuál es la
energía cinética máxima que adquirirá Q por acción 3q
ΔV = Vb − V∞ =
de las tres cargas q? 4πε 0 L
Luego:
3q
K máx = QΔV = Q
4πε 0 L

Ejemplo 52. Se tiene una línea de longitud L y con


densidad de carga lineal λ . Calcular el Potencial
eléctrico a una distancia R sobre la línea
perpendicular bisectriz.
Solución.

30
Electrostática Hugo Medina Guzmán

El potencial eléctrico debido a esta carga en el punto


P es:
1 λRdθ
dV =
(
4πε 0 z + R 2 2
)
12

El potencial debido a todo el anillo es


λR 2π

) ∫
V = dθ
(
4πε 0 z 2 + R 2
12 0

Solución. λR
=
Considerar el elemento dx, el Potencial que produce
este elemento en el punto P es (
2ε 0 z 2 + R 2 )12

1 λdx En términos de la Carga total Q = 2πRλ .


dV =
(
4πε 0 R 2 + x 2 ) Q
1 2

V =
El potencial de toda la línea es (
4πε 0 z 2 + R 2 ) 12

λ L
dx
V=
4πε 0 ∫ (R

2
L
2
+ x2 )
1 2 Ejemplo 54. Dos anillos coaxiales uniformemente
2 cargados de radios iguales R está en planos paralelos
separados una distancia a, el trabajo que se realiza
2λ L
dx
o V =
4πε 0 ∫ (R0
2
2
+ x2 )
1 2
para traer una carga q del infinito al centro de cada
uno de los anillos es respectivamente A1 y A2.
Encontrar les Cargas q1 y q2 que tienen los anillos.

( )
L

2λ 2 Solución.
V= log x + R 2 + x 2
4πε 0 0

⎛L 2 ⎞
⎜ + R2 + L ⎟

V= log⎜⎜ 2 4 ⎟

4πε 0 R
⎜⎜ ⎟⎟
⎝ ⎠
Podemos notar en este resultado que para el caso de
una línea infinita, es decir para L = ∞ el potencial El potencial en 1:
es infinito, esto es porque nosotros para el potencial q1 q2
del elemento diferencial estamos considerando como V1 = +
referencia el infinito con potencial cero y esta línea 4πε 0 R (
4πε 0 a 2 + R 2 )
12

cargada va hasta el infinito. Luego pera una línea


infinita tenemos que tomar otro punto como 1 ⎡ q2 ⎤
= ⎢q1 + ⎥
referencia. 4πε 0 R ⎣⎢ (
1+ a2 / R2 )
12
⎦⎥
Ejemplo 53. Potencial eléctrico en el eje de un anillo El potencial en 2:
de radio R y Carga λ C/m. q2 q1
V2 = +
4πε 0 R 4πε 0 (a + R 2 )
Solución. 2 12

1 ⎡ q1 ⎤
= ⎢q 2 + ⎥
4πε 0 R ⎢⎣ (
1+ a2 / R2 ) 12
⎥⎦
El potencial en el infinito V∞ = 0
Trabajo para traer una carga q desde el infinito a 1:
A1 = q (V1 − V∞ )
q ⎡ q2 ⎤
= ⎢q1 + ⎥ (1)
Consideremos el elemento de anillo dl determinado
4πε 0 R ⎢⎣ (
1+ a2 / R2 )
12
⎥⎦
Trabajo para traer una carga q desde el infinito a 2:
por el ángulo θ y barrido por dθ de tal manera que
A2 = q (V2 − V∞ )
dl = Rdθ .

31
Electrostática Hugo Medina Guzmán

q ⎡ q1 ⎤ q Q
b) − = 0 ⇒ x = 2,7 m
= ⎢q 2 +
4πε 0 R ⎢⎣ (
1+ a2 / R2 ) 12 ⎥
⎥⎦
(2)
(
x x 2 + 2,4 2 )12

(1) y (2) son dos ecuaciones con incógnitas q1 y q2. c) 1 m v 2 = e ⎛⎜ ⎛⎜ q − Q ⎞⎞


⎟⎟
4πε 0 ⎜ ⎝ (5,0) (5,0 + 2,4 ) ⎟⎠ ⎟

e 0
Resolviendo (1) y (2) obtenemos: 2

2 2 1 2

⎡ 1 A1 ⎤ ⇒ v0 = 9 × 10 6 m/s
⎢ 12 − ⎥
q1 =
(
4πε 0 RA2 ⎢⎣ 1 + a R
2 2 A2 ⎥
⎦ ) d) WO →∞ = q(V∞ − VO )
q ⎡ 1 ⎤
⎢ 1 + a 2 R 2 − 1⎥
( ) q ⎡⎛ Q ⎞ ⎛⎜ Q ⎞⎤
⎣ ⎦ = ⎢⎜⎜ − ⎟⎟ − − ⎟⎥

4πε 0 ⎢⎝ (2,4) ⎠ ⎝ ∞ + 2,4 2

⎜ 2
( )
12 ⎟⎥
⎠⎦
1 A ⎤
⎢ − 2⎥ = + 1,8 x 10 J -3

( )
12

4πε 0 RA1 ⎢⎣ 1 + a R A1 ⎥
2 2

q2 = ⎦ Ejemplo 56. Potencial eléctrico en el eje de un disco


q ⎡ 1 ⎤ de radio R y carga σ C/m2.
− 1⎥

⎣ 1+ a R(2 2
⎦ ) Solución.

Ejemplo 55. Una carga Q = - 800 nC se distribuye


uniformemente en un anillo de radio de 2,4 m. Una
carga q = + 600 nC se pone en el centro del anillo.
Los puntos A y B están situados en el eje del anillo.

Consideremos el anillo diferencial de radio r y


superficie dS = 2πrdr .
El potencial debido a este anillo en el punto P es:
1 σ 2πrdr
dV =
4πε 0 z 2 + r 2 ( )
12

a) ¿Cuál es el trabajo hecho por una fuerza externa El potencial debido a todo el disco es
que transporta un electrón de B a A? σ 2π σ 2
rdr
(z + r2)
R 12 R
b) si el potencial eléctrico es igual a cero en un punto
en el eje del anillo. ¿Cuál es la distancia de este
V =
4πε 0 ∫ (z0 2
+ r2 )
12
=
2ε 0 0

[ ]
punto del centro del anillo?
σ
c) Si se un electrón se lanza a lo largo del eje del =
2ε 0
(z2 + r2 ) − z
12

anillo desde el infinito. El electrón llega y se detiene


momentáneamente en un punto en el eje que está a
En términos de le Carga total Q = πR σ
2
5,0 m del centro del anillo. ¿Cuál es la velocidad
inicial del electrón en el infinito?
d) Si una fuerza externa retira la carga q del centro V =
Q
2πR ε 0
2
z2 + r2 [( ) 12
−z ]
del anillo y la transporta al infinito. ¿Cuál es el
trabajo realizado por esta fuerza externa? En este caso al igual que para la línea infinita, para el
Solución. plano infinito hay que tomar otro punto de referencia
diferente al ∞ .
a) W B → A = e(V A − V B )

e ⎡⎛⎜ q Q ⎞

Ejemplo 57. ¿Cuál es el potencial en el eje producido
= ⎢ − por un cilindro de radio R, largo L y carga ρ C/m3.
4πε 0 ⎢⎜⎝ (1,8) 1,8 2 + 2,4 2
⎣ ( ) 12 ⎟
⎠ Solución.
⎛ ⎞⎤
-⎜ q − Q ⎟⎥
⎜ (3,2 )
⎝ (3,2 2
+ 2,4 )
2 12 ⎟⎥
⎠⎦
= -700 eV

32
Electrostática Hugo Medina Guzmán

ρR 3
V= + C1
3ε 0 r
Para r = ∞ ⇒ V = 0
de aquí
ρR 3 Q
yV = =
3ε 0 r 4πε 0 r
Para r = R
ρR 2 Q
⇒ V= =
Para evaluar el potencial eléctrico en el punto P, 3ε 0 4πε 0 R
tomaremos un disco de ancho diferencial dz, cuyo Para r < R
volumen es dV = πR dz .
2 →
ρ →
Por la ley de Gauss E = r
El potencial debido al disco en el punto P es 3ε 0
ρπR 2 dz ⎧
[ ] − ( z 0 − z )⎫⎬
12

⎨ (z 0 − z ) + R
dV =
2 2 El potencial es
2πε 0 R 2 ⎩ ⎭ → →
V = − ∫ E ⋅ d r + C2
El potencial debido al cilindro es
ρ L2 ⎧
[ ]
− ( z 0 − z )⎫⎬dz
12

∫ L ⎨ (z 0 − z ) + R
2
V = 2
ρ ρ 2
2ε 0 2 ⎩
− ⎭ = − ∫ 3ε rdr + C2 = −
6ε 0
r + C2
ρ ⎧ 1
[ ]
0
⎨− ( z 0 − z )( z 0 − z ) + R
2 2 12
V= + Como para r = R
2ε 0 ⎩ 2
ρR 2
L
2 2 ⇒ V=
R 2 log ⎡( z 0 − z ) + (z 0 − z )2 + R 2 ⎤⎥ ⎫⎬ − z 0 z + z 3ε 0
⎢⎣ ⎦⎭ 2 L

2 ρR 2 ρR 2
⎧ =− + C2
⎪ 3ε 0 6ε 0
12
L ⎞ ⎡⎛ ⎤
2
ρ ⎪⎛ L⎞ ρR 2
V= ⎨⎜ z 0 + ⎟ ⎢⎜ z 0 + ⎟ + R ⎥ − z 0 L
2
⇒ C2 =
4ε 0 ⎪⎝ 2 ⎠ ⎢⎣⎝ 2⎠ ⎥⎦ 2ε 0

⎩ ρr 2 ρR 2
12 y V =− +
L ⎞ ⎡⎛ ⎤ 6ε 0 2ε 0
2
⎛ L⎞
− ⎜ z0 − ⎟ ⎢⎜ z0 − ⎟ + R 2 ⎥
⎝ 2 ⎠ ⎣⎢⎝ 2⎠ ⎦⎥ ρ ⎛ 2 r2 ⎞
⇒ V = ⎜⎜ R − ⎟⎟
⎡ 2 ⎤⎫ 2ε 0 ⎝ 3 ⎠
⎢ z 0 + + ⎛⎜ z 0 + ⎞⎟ + R 2
L L
⎥⎪ La figura siguiente muestra la distribución de campo
+ R 2 log ⎢ 2 ⎝ 2⎠ ⎥⎪ eléctrico y potencial eléctrico en el caso visto.
⎢ 2 ⎥⎬
⎢ z − L + ⎛ z − L ⎞ + R2 ⎥⎪
⎢ 0 2 ⎜ 0 ⎟ ⎥⎪
⎣ ⎝ 2⎠ ⎦⎭

Ejemplo 58. Se tiene una esfera de radio R con carga


uniforme de densidad ρ C/m3. Encontrar el
potencial para todo punto. Considerar el potencial en
el infinito igual a cero.
Solución.
Para r > R
Por la ley de Gauss
→ ρR 3 Q
E= rˆ = rˆ
3ε 0 r 2
4πε 0
El potencial es
→ →
ρR 3 dr
V = − ∫ E ⋅ d r + C1 = − ∫ + C1
3ε 0 r 2 Ejemplo 59. Como una variante veamos el caso de
una esfera conductora.
33
Electrostática Hugo Medina Guzmán

La carga se encuentra en la superficie y el campo d) la densidad de carga en la superficie exterior de la


eléctrico en el interior es cero y por lo tanto el esfera conductora.
potencial es constante, igual al de la superficie. Lo
mismo sucede cuando la esfera es hueca estos dos
últimos casos se muestra en la figura siguiente

Solución.
a) La carga total en una región del espacio donde hay
una densidad de carga ρ está dada por la integral:
q = ∫ ρdV
V
En este caso, donde la simetría es esférica, la
expresión toma la forma:
2π π R e− r 2
q=∫ ∫∫ ρ 0 2 r senθ dr dθ dφ =
0 0 0 r
Ejemplo 60. Las superficies interior (r = a) y exterior
R
4πρ0 ∫ e − r dr = 4πρ0 1 − e − R
0
( )
(r = b) de un cascaron esférico no conductor tienen la
misma densidad de carga σ. La densidad de carga en b) Dado que el potencial en esa región es constante y
→ →
el resto del espacio es nula.
que E = −∇V , entonces E ext = 0 .
a) Encuentre el campo eléctrico en las zonas r < a, a
c) Obsérvese que las condiciones del problema dicen
< r < b, y r > b.
que para r > 2R , V = Cte. Igual cosa ocurre en el
b) Calcule el potencial electrostático en cada una de
conductor, donde el campo eléctrico es nulo. Si
las regiones mencionadas.
consideramos que φ = 0 en el infinito, entonces, en el
Solución.

resto del espacio, incluido el conductor, V = 0, de este
a) Para r < a E ( r ) = E (r ) rˆ , donde E ( r ) = 0 ¸ modo, la diferencia de potencial entre r = 3R/2 y el
centro de la esfera estará dado por:
σ a2 V = Vr = 0 − V = Vr = 0 − Vr = R = Vr = 0
Para a < r < b E (r ) = ; 3
r= R
ε 0r 2 2
0 0

Para r > b E (r ) =
σ
(a 2 + b 2 ) , es decir, V = − ∫ ∞
E(r )dr = − ∫ E(r )dr
R
ε 0r 2
Para evaluar esta integral es necesario determinar
σ 2 E(r) en la esfera aisladora, lo que se realiza utilizando
b) Para r > b V(r ) =
ε 0r
(a + b 2 ) ; la Ley de Gauss. Dado que E(r) es radial, podemos
escribir directamente:
σ 2 q(r )
Para a < r < b V(r ) =
ε 0r
(a + br ) y E= , donde q (r ) = 4πρ 0 1 − e ( −r
) . Así
4πε 0 r 2

σ ρ0 ⎛ 1 e −r ⎞
Para r < a V(r ) = (a + b ) . V =−
0
∫R ⎜⎜⎝ r 2 − r 2 ⎟⎟⎠dr = ∞
ε0 4πε 0
d) Dado que el campo eléctrico exterior a la esfera
Ejemplo 61. Se tiene una esfera aisladora con conductora es nulo, la carga total, comprendiendo la
densidad de carga variable de la forma esfera interior y la conductora debe ser nula. Además,
ρ = (ρ 0 r 2 )e − r y radio R limitada exteriormente para que el campo eléctrico en el conductor también
por una esfera conductora de radio interior R y sea nulo, la carga encerrada por cualquier superficie
exterior 2R. En la esfera conductora hay una carga gaussiana en su interior debe ser nula, es decir, en la
neta tal que el potencial exterior (r > 2R) es superficie interior del conductor debe haber una carga
constante. Determine: de igual magnitud a la de la esfera aisladora, con
a) La carga total en la esfera aisladora. distinto signo, las que sumadas se anulan. En
b) el campo eléctrico en el exterior (r > 2R). consecuencia, en la superficie exterior del conductor
c) a diferencia de potencial entre r = 3R/2 (esfera no hay carga, es decir, σ = 0.
conductora) y el centro de la esfera aisladora
(considere V = 0 en r = ∞).

34
Electrostática Hugo Medina Guzmán

Ejemplo 62. En la figura se ha representado parte de Eligiendo como trayectoria de integración una recta
2 cilindros muy largos, cada uno de radio ro, que paralela al eje y, que pasa por los centros de los
tienen en su superficie densidades de carga constantes cilindros, se tiene:
σ1 = σ y σ2 = -σ respectivamente. (No hay carga en el y2
interior de los cilindros). V = ∫ Edy =
y1

y2 σ r0 y2 σ r0
∫y1 ⎛ D⎞
dy + ∫
⎛D y1 ⎞
dy con
ε0⎜ y + ⎟ ε0⎜ − y⎟
⎝ 2⎠ ⎝2 ⎠
D ⎛D ⎞
y 2 = − r0 e y1 = −⎜ − r0 ⎟
2 ⎝2 ⎠
σ r0 σ r0
Donde y son las
⎛ D⎞ ⎛ D ⎞
ε0⎜ y + ⎟ ε 0 ⎜ − y ⎟
⎝ 2⎠ ⎝2 ⎠
a) Encuentre el campo eléctrico sobre la línea AB, magnitudes de los campos generados por los cilindros
que equidista de los cilindros en una distancia igual a con centros en y = - D/2 e y = +D/2 respectivamente.
la separación entre ellos (D). Resolviendo estas integrales se llega a
b) ¿Cuál es la diferencia de potencial entre los centros
de los cilindros? 2σ r0 ⎛ D − r0 ⎞
V = ln⎜⎜ ⎟⎟
c) Calcule el potencial en un punto sobre la línea AB. ε0 ⎝ r0 ⎠
Solución.
a) Utilizando una superficie gaussiana cilíndrica de c) El potencial eléctrico en general se puede calcular
radio r y largo L concéntrica con un cilindro cargado, sumando los potenciales originados desde diversas
de similares características que los propuestos en este fuentes. Utilizando esta idea, para responder a esta
problema, por la ley de gauss, se obtiene la expresión: pregunta, basta con calcular el potencial de una de las
distribuciones de carga, ya que la otra es similar,
2π r0 Lσ σ r0
E 2π rL = de donde E = , forma salvo el signo. (Además deben compatibilizarse los
ε0 ε0 orígenes del potencial).
general para el campo eléctrico para tales Considerando un cilindro con densidad de carga σ, el
distribuciones de carga. → σ r0
Si los cilindros se observan en corte y se dibujan allí campo eléctrico es E = rˆ , siendo r la distancia
ε0
los campos eléctricos generados por cada cilindro se
observaría lo siguiente: entre el centro de la distribución (cilindro) y el punto
en que calculamos el potencial.
Al calcular la integral de línea, eligiendo como
trayectoria de integración una recta radial y una curva
sobre el manto, como esta última no contribuye al
σ r0 ⎛ D ⎞
potencial, se tiene: V
+
ln⎜ ⎟ y, para la
=
ε 0 ⎜⎝ r0 ⎟⎠
σ r0 ⎛ D ⎞

otra distribución, V = − ln⎜ ⎟ , con lo cual
σ r0 ε 0 ⎜⎝ r0 ⎟⎠
donde E1 = E 2 = , con lo cual, la suma
ε0D V AB = 0 . En este caso se ha elegido el origen de
resulta ser un vector en la dirección horizontal cuyo V(rs ) en un punto equidistante de los cilindros (es
valor es la superposición de las proyecciones de E1 y arbitrario). Obsérvese que este resultado depende
E2: en esa dirección obviamente del origen de los potenciales y por lo
→ σ r0 tanto, su respuesta puede ser otra
E=2 cos 60°
ε0D Ejemplo 63. Tres trozos de hilo cargado con
b) Para contestar esta pregunta es necesario densidad de carga λ se disponen como se indica en la
considerar que el campo eléctrico en el interior de los figura.
cilindros es nulo, como se deduce directamente con la a) Determine el campo eléctrico total sobre la carga
ley de Gauss. Ello nos dice que allí V es constante. Q.
De este modo, la diferencia de potencial entre los b) Calcule la fuerza que ejerce Q sobre cada uno de
centros de los cilindros corresponde la diferencia de los trozos de hilo.
potencial entre sus mantos.

35
Electrostática Hugo Medina Guzmán

c) Determine la energía potencial de la carga Q Para el tercer segmento operamos en forma similar.
(origen en ∞). En este caso
→ →
⎛L⎞ L
r = 0 , r ' = ⎜ ⎟iˆ + y ' ˆj , dq = λdy ' , x' =
⎝2⎠ 2
con lo que
1 λ (L / 2)dy '
dEδ , x = −
[
4πε 0 (L / 2)2 + y '2 3 2 ]
1 λy ' dy '
dEδ , y =−
[
4πε 0 (L / 2)2 + y '2 3 2 ]
Integrando en y’ ambas expresiones entre 0 y - L
encontramos:
( )
Solución.
a) Para resolver este problema es conveniente 1 2λ 5 − 5 1 4 5λ
dividirlo en tres, uno para cada distribución. E3 x = y E3 y =
Calculamos primero el campo eléctrico de la 4πε 0 5L 4πε 0 5L
distribución lineal de la izquierda: Si ponemos un eje
De donde E3 =

1 ⎡ 2λ 5 − 5
iˆ +
(
4 5λ ) ⎤
ˆj ⎥
x conteniendo a este trozo, con el origen en Q se tiene ⎢
que 4πε 0 ⎣ 5L 5L ⎦
→ 1 −L / 2 λdx' → → → →
E1 = Ex1iˆ =
4πε 0 ∫
−3L / 2 ( x ')
2
iˆ Obviamente E = E1 + E2 + E3
b) Para responder a esta pregunta, basta con calcular
−L / 2
λ 1 λ ⎛ 2 2 ⎞ˆ la fuerza que realiza cada una de los trozos de hilo
= − iˆ = ⎜ − ⎟i sobre Q, así, la fuerza que ejerce Q sobre el hilo
4πε 0 x' − 3 L / 2 4πε 0 ⎝ L 3L ⎠ estará determinada por el principio de acción y
λ reacción, es decir, las respuestas a la pregunta son:
= iˆ Caso 1 F1 = - E1Q ; Caso 2 F2 = - E2 Q ; Caso 3 F3
3πε 0 L = E3 Q
c) Como son distribuciones finitas, podemos utilizar
Si llamamos E2 al campo generado por la la expresión deducida para ese fin a partir del
semicircunferencia, por simetría, éste debe apuntar potencial de una carga puntual respecto de infinito
hacia abajo (eje –y), luego, la única componente que, 1 dq
luego de integrar es diferente de cero, está dada por dV =
4πε 0 → →
r − r'
1 λdl
dE y = ( y − y ') En el primer caso tendremos
4πε 0 → →32
r − r' 1 −L/ 2 λdx' 1 −L/2 λdx'
V1 =
4πε 0 ∫−3 L / 2 x'
=−
4πε 0 ∫
−3 L / 2 x'
el
→ →32
en nuestro caso r − r ' = L/2, y = 0, cambio de signo es debido a que el dominio de
integración.
y ' = (L / 2 )senθ con θ medido desde el eje x. A su Comprende sólo valores de x < 0
1 ⎛1⎞
vez dl = (L / 2 )dθ . Reemplazando estos valores V1 = ln⎜ ⎟
en la integral se tiene.
4πε 0 ⎝ 3 ⎠
Del mismo modo,
⎛L⎞
λ ⎜ ⎟dθ ⎛L⎞
1 ⎝ 2 ⎠ ⎛ − L senθ ⎞ λ ⎜ ⎟dθ
dE y = ⎜ ⎟ 1 π
⎝2⎠ λ
4πε 0
⎜ ⎟
3
⎛L⎞ ⎝ 2 ⎠ V2 =
4πε 0 ∫0 L
=
4ε 0
⎝2⎠ 2
1 λ y finalmente
= − senθ dθ
4πε 0 ⎛ L ⎞ 1 −L λdy '
⎜ ⎟
⎝2⎠
V3 =
4πε 0 ∫0 2
⎛L⎞
⎜ ⎟ + ( y ')
2
Integrando entre 0 y π se tiene
⎝2⎠

1 4λ
E=− ˆj
4πε 0 L
36
Electrostática Hugo Medina Guzmán

ln (− 2 + 5 )
λ 432,8
= E máx = = 21640 V/m
4πε 0 20 × 10 −3
v2 q e Er
V=
λ
4πε 0
⎡ ⎛1⎞
( ⎤
⎢ln⎜ 3 ⎟ + π + ln − 2 + 5 ⎥ ) c) q e E = me
r
⇒ v=
me
⎣ ⎝ ⎠ ⎦
q e = 1,6 × 10 −19 C , me = 9,1 × 10 −31 kg ,
Ejemplo 64. Dos cáscaras cilíndricas conductoras
largas son coaxiales y tienen radios de 20 mm y de 80 432,8 432,8
r = 30mm , E = = = 14426,7 V/m
mm. El potencial eléctrico del conductor interno, con r 0,03
respecto al conductor externo, es + 600 V.
a) (1 punto) Se lanza un electrón desde el reposo en la
superficie del conductor externo. Encuentre la (1,6 × 10 )⎛⎜ 432,8
−19

0,03

⎟(0,03)
velocidad del electrón cuando alcanza al conductor v= ⎝ ⎠
− 31
interno. 9,1 × 10
b) (1 punto) ¿Cuál es a magnitud máxima del campo = 8,72 x 106m/s
eléctrico entre los cilindros? d) Se lanza un electrón radialmente hacia fuera
c) (1 punto) Si un electrón está en movimiento desde el punto 1 al punto 2
circular alrededor del cilindro interno en una órbita de El punto 1 esta a + 350 V, el punto 2 en la cáscara
radio 30 mm. ¿Cuál es a velocidad del electrón? exterior a 0 V.
d) (1 punto) Se lanza un electrón radialmente hacia
afuera de un punto entre las cáscaras en las cuales el
W1→2 = K 2 − K 1
potencial eléctrico es + 350 V con respecto al ⇒ − e(0 − 350V ) = K 2 − 150eV
conductor externo. La energía cinética inicial del
electrón es 150 eV. ¿Cuál es a energía cinética del ⇒ K 2 = 350eV + 150eV = 500eV
electrón, cuando alcanza cualquiera de las cáscaras?
Solución. Ejemplo 65. Un electrón e (carga - e) incide con
1 velocidad v a un pequeño agujero practicado en el
a) q e ΔV = me v 2f ⇒ centro de dos placas cuadradas planas de lado a. (En
2 sus cálculos utilice la aproximación de placas
2q e ΔV 2(1,6 × 10 −19 )(600 ) infinitas).
vf = = a) Encuentre la expresión que da cuenta de la
me 9,1 × 10 −31 densidad de carga en las placas cuando entre ellas hay
= 1,45 x 107m/s una diferencia de potencial V (inicialmente las placas
b) Si consideramos que el conductor externo tiene están descargadas).
b) ¿Cuál debe ser la diferencia de potencial V para
V R2 = 0 y el conductor interno V R1 = 600V El que el electrón llegue con velocidad v/2 al agujero
potencial entre los conductores es O’?
λ dr λ c) En las condiciones dadas en b). ¿Qué fuerza ejerce
V =− ∫
2πε 0 r
+C = −
2πε 0
ln r + C el electrón contra la placa positiva cuando ha
recorrido una distancia δ/2 entre las placas?
Cuando r = R2 , V R2 = 0 y
λ λ
0=− ln R1 + C ⇒ C = ln R1
2πε 0 2πε 0
Cuando r = R1 , V R1 = 600V y
λ
⇒ 600 = − (ln R1 − ln R2 )
2πε 0
λ R λ 1,39λ
= ln 2 = ln 4 =
2πε 0 R1 2πε 0 2πε 0
λ 600
⇒ = = 432,8
2πε 0 1,39
El campo entre los conductores es
λ 432,8
E= =
2πε 0 r r
El campo máximo se obtiene con el menor r.
Solución.
37
Electrostática Hugo Medina Guzmán

a) Si inicialmente las placas tienen carga nula, al c) Luego el átomo pasa por condensador C 0 de
conectarlas a la diferencia de potencial V, cada una de
ellas adquiere cargas de igual magnitud y distinto placas paralelas con cargas iguales pero de signo
opuesto entre E y D. Determine el valor las cargas en
signo. Dichas cargas deben ser tales que generen un
Placa 2 → → E y D cuando ΔV2 = V E − V D es responsable de la
campo eléctrico tal que V = ∫Placa1
E ⋅ d r integral desviación mostrada.
d) ¿Es posible que el átomo no sufra desviación
que, al ser E = cte, vale V = Eδ . Por otro lado, cada
alguna entre E y D? ¿Una desviación en sentido
σ opuesto?
placa produce un campo E = , (Gauss) y, por lo
2ε 0 Solución.
tanto, el campo total entre las placas resulta ser 1 2 2eΔV1
a) eΔV1 = mv0 ⇒ v0 =
σ ε0 2 m
E= ,con lo cual σ= V .
ε0 δ b) Como el átomo tiene carga positiva la placa A es
positiva y la placa B es negativa, por lo tanto A está a
b) La energía total del electrón antes de ingresar a O
mayor potencial que B.
1 v c) Q = C 0 ΔV2
es m0 v 2 y si llega a O’ con vO ' = , usando el
2 2 d) Si es posible que el átomo no sufra desviación
teorema de conservación de la energía podemos alguna entre E y D
2
1 ⎛v⎞ 1 mg = eE
escribir en O’, eV + m0 ⎜ ⎟ = m0 v . A
2

2 ⎝2⎠ 2 También es posible, invirtiendo las cargas de las


placas.
3
partir de esta expresión, V = mv 2 .
8e SUPERFICIES EQUIPOTENCIALES
c) La fuerza que ejercen las placas sobre el electrón Como hemos podido observar, el potencial es
→ → solamente función de las coordenadas de posición, de
en cualquier punto interior a las placas es F = e E tal manera que un potencial constante define una
eσ superficie equipotencial. En esta superficie no se
cuya magnitud es F = dirigida hacia la placa realiza trabajo al mover una carga de prueba sobre
ε0 ella, el campo eléctrico es perpendicular a estas
positiva. Por el principio de acción y reacción, la superficies.
fuerza que ejerce el electrón sobre las placas es igual, Los equipotenciales son perpendiculares a las líneas
pero con sentido contrario a la anterior. de fuerza, un campo eléctrico puede representarse
también por medio de las equipotenciales.
Ejemplo 66. Sea un gas de átomos neutros entre dos Las equipotenciales de una carga puntual están dadas
placas a ΔV1 = V A − V B , la separación es tal que el q
por r = , las que se muestran en la figura
campo eléctrico entre ellas es uniforme. Cuando un 4πε 0V
átomo de masa m choca con A queda inicialmente en siguiente.
reposo y pierde una carga − e (la carga de un
electrón).

A continuación se muestra las líneas equipotenciales


producidas por dos cargas iguales y opuestas.

a) Determine ΔV1 cuando el átomo que chocó con A


pasa con una velocidad en P igual a v 0 (suponga que
el agujero en P es pequeño y no afecta la forma del
campo)
b) ¿Cual placa, A ó B, estaría a mayor potencial? ,
¿Cuál placa cargada negativamente?

38
Electrostática Hugo Medina Guzmán

CÁLCULO DEL CAMPO ELÉCTRICO A


PARTIR DEL POTENCIAL.
Como las fuerzas están dadas por el campo eléctrico

E , ésta puede obtenerse inversamente a partir del
potencial V. Consideremos dos puntos orientados con
el eje x uno en x y el otro en x + Δx , la diferencia
de potencial entre estos dos puntos es ΔV .
ΔV = V (x. + Δx, y, z ) − V ( x., y, z )
Por definición, escribimos en función de la derivada
parcia1
Ejemplo 61. Se tiene dos cargas puntuales q y - 2q ∂V
situadas en los puntos (0,0,0) y (a,0,0), ΔV = Δx
respectivamente, encontrar la superficie ∂x
equipotencial. (Derivada parcial: Es la derivada de una función de
Solución. dos o más variables con respecto a una de ellas,
considerando en el desarrollo a las demás como
constantes).
Simultáneamente en la misma trayectoria
→ →
ΔV = − ∫ E ⋅ d l = − E x ΔX
Fácilmente podemos deducir que
∂V
Ex = −
∂x
El potencial en el punto P (x, y, z) es: Similarmente, podemos hacer a lo largo de los ejes z
1 q 1 2q e y.
V= −
4πε 0 x 2 + y 2 + z 2 4πε 0 ( x − a )2 + y 2 + z 2 ∂V ∂V
Ey = − y Ez = −
Cuando V = 0 ∂y ∂z
1 q 1 2q El Campo resultante es la suma vectorial de sus
− =0 componentes:
4πε 0 x 2 + y 2 + z 2 4πε 0 (x − a ) 2
+y +z
2 2

(x − a )2
+y +z =4x +y +z
2 2
( 2 2 2
) E = E x iˆ + E y ˆj + E z kˆ
2 a 2 ⎛ ∂V ˆ ∂V ˆ ∂V ˆ ⎞
x 2 + ax + y 2 + z 2 = = − ⎜⎜ i j− k⎟
3 3 ⎝ ∂x ∂y ∂z ⎟⎠
2
Sumando a ambos miembros a 9: → ⎛ ∂ ∂ ˆ ∂ ˆ⎞
o E = −⎜⎜ iˆ j − k ⎟⎟V
⎝ ∂x ∂y ∂z ⎠
2
⎛ a⎞ 4 2
⎜x+ ⎟ + y + z = a
2 2

3⎠ 9 En notación abreviada
⎝ →
Ecuación de la superficie equipotencial que E = −∇V
corresponde a una esfera con centro en − a 3 , 0 y donde el operador ∇ llamado nabla es
radio R = 2a 3 , la figura siguiente muestra la ∂ ˆ ∂ ˆ ∂ ˆ
∇= i+ j+ k
superficie equipotencial V = 0 . ∂x ∂y ∂z

Ejemplo 67. Responda los casos siguientes


justificando cada respuesta.
a) Un plano cargado con densidad de carga − σ se
coloca en el plano x y (z = 0). Halle el campo
eléctrico en todo el espacio por medio de la ley de
Gauss.
b) Usando el principio de superposición y el resultado
en a), halle el campo eléctrico en todo el espacio
cuando añadimos un segundo plano infinito con
densidad de carga + 2σ paralelo, en z = 2.
c) Usando la definición, evalué la diferencia de
potencial V (x1) - V (x2) entre x1 = -3 y x2 = 1, 5.

39
Electrostática Hugo Medina Guzmán

d) Se coloca un tercer plano infinito descargado en z → σ ˆ σ ˆ 3σ ˆ


= 3 y se une al plano en z = 0 con un alambre E=− k− k =− k
conductor muy delgado, puede suponer que lo hace 2ε 0 ε0 2ε 0
atravesando el plano z = 2 por un agujero muy Para z < 0:
pequeño, de manera de no afecta la geometría del → σ ˆ σ ˆ σ ˆ
sistema. E= k− k=− k
Calcule la diferencia de potencial pedida en c). 2ε 0 ε0 2ε 0
Solución.
a)

c) La diferencia de potencial V (x1) - V (x2) entre


x1 = -3 y x2 = 1, 5.

Sea la superficie gausiana de área A


⎛ σ
ΔV1 = V(−3) − V(0 ) = ⎜⎜ −

[ ( )]
kˆ ⎟⎟ ⋅ 0 − − 3kˆ
⎝ 2ε 0 ⎠
→ q
∫ E ⋅ dS = ε0 = −

2ε 0
− σA σ
⇒ 2 EA = ⇒ E=−
ε0 2ε 0
⎛ 3σ ˆ ⎞
ΔV2 = V(0 ) − V(1,5 ) = ⎜⎜ −
2 ε
[
k ⎟⎟ ⋅ 1,5kˆ − 0 ]
→ σ ˆ ⎝ 0 ⎠
Para z > 0 E = − k 4,5σ
2ε 0 = −
→ σ ˆ 2ε 0
Para z<0 E= k 3σ 4,5σ 7,5σ
2ε 0 ΔV = ΔV1 + ΔV2 = − − =−
2ε 0 2ε 0 2ε 0
b) El campo producido por el plano en z = 0: d) Con el tercer plano infinito descargado en z = 3, el
campo eléctrico queda distribuido como se muestra a
continuación.

El campo producido por el plano en z = 2:

La diferencia de potencial V (x1) - V (x2) entre


x1 = -3 y x2 = 1.
Tomando en cuenta la nueva disposición:
⎛ σ
ΔV1 = V(−3) − V(0 ) = ⎜⎜ −

[ ( )]
kˆ ⎟⎟ ⋅ 0 − − 3kˆ
⎝ 2ε 0 ⎠

= −
Para z > 2: 2ε 0
σ ˆ σ ˆ σ ˆ
[ ]

E=− k+ k= k ⎛ σ ⎞
2ε 0 ε0 2ε 0 ΔV2 = V(0 ) − V(1,5 ) = ⎜⎜ − kˆ ⎟⎟ ⋅ 1,5kˆ − 0
⎝ ε0 ⎠
Para 0 < z > 2:

40
Electrostática Hugo Medina Guzmán

1,5σ → 1 qz
= − Finalmente: E = kˆ
ε0 4πε 0 4πε 0 z 2 + R 2 ( )
12

3σ 1,5σ 3σ
ΔV = ΔV1 + ΔV2 = − − =− Ejemplo 70. Encontrar el Campo eléctrico en el eje
2ε 0 ε0 ε0
de un disco de radio R y con carga Q a partir del
potencial.
Ejemplo 68. Encontrar el campo eléctrico producido Solución.
por un dipolo eléctrico a partir del potencial. Para el disco

[(z ]
Solución.
El potencial del dipolo es V=
q
2πε R 2
2
+ R2 )
12
−1
p cos θ
1 0
V = →
4πε 0 r 2 El Campo eléctrico es E = −∇V
El Campo eléctrico es

Ex = 0 , E y = 0
E = −∇V
Para este caso es conveniente trabajar con Ez = −
∂ q
∂z 2πε 0 R 2
z2 + R2 [( ) 12
−1]
coordenadas esféricas,
∂ 1 ∂ ˆ 1 ∂ ˆ q ⎡1 2
( )−1 2 ⎤
∇= rˆ + θ+ φ = −
⎢ z + R2 2 z − 1⎥
∂r r ∂θ rsenθ ∂φ 2πε 0 R ⎣ 2 2

y q ⎡ z ⎤
→ ∂V 1 ∂V ˆ 1 ∂V ˆ = 2 ⎢
1− 2 12⎥
E=−
∂r
rˆ −
r ∂θ
θ−
rsenθ ∂φ
φ 2πε 0 R ⎣⎢ z + R2 ( ) ⎦⎥
→ q ⎡ z ⎤
y E= 2 ⎢
1− 12 ⎥

En esta ecuación el último término vale cero ya que
no existe variación de potencial respecto a φ ,
2πε 0 R ⎢⎣ z2 + R2 ( ) ⎥⎦
entonces:
Ejemplo 71. Perpendicular al eje x hay tres placas
∂ ⎛ 1 p cos θ ⎞ 1 p cos θ delgadas de metal, A en x = 0, B en x = 1 m, y C en z
E r = − ⎜⎜ ⎟⎟ =
∂r ⎝ 4πε 0 r ⎠ 2πε 0
2
r = 3 m. A se mantiene conectado a tierra, B tiene un
potencial de 200 voltios, y C tiene un potencial de
1 ∂ ⎛ 1 p cos θ ⎞ 1 psenθ 100 voltios.
Eθ = − ⎜⎜ ⎟⎟ = a) Encuentre el campo eléctrico entre A y B y entre
r ∂θ ⎝ 4πε 0 r ⎠ 4πε 0 r
2 2
B y C.
→ b) Encuentre la densidad de la carga en cada placa.
E = E r rˆ + Eθ θˆ

Ejemplo 69. Encontrar el campo eléctrico en el eje de


un anillo de radio R y con carga Q, a partir del
potencial.
Solución.
q
Para el anillo V =
4πε 0 (z + R 2 )
2 12


El Campo eléctrico es E = −∇V Solución.
Ex = 0 , E y = 0 y a) Los campos se asumen uniformes.
A la izquierda (entre A y B).
∂ q ΔV 200
Ez = −
(
∂z 4πε 0 z 2 + R 2 )
12 E=−
Δx
=−
1
= - 200 V/m.

q ⎛ 1⎞ 2z A la derecha (entre B y C).


= − ⎜− ⎟ (− 100) = + 50 V/m
(
4πε 0 ⎝ 2 ⎠ 4πε 0 z 2 + R 2 )
12
E' = −
2
q z b) Teniendo en cuenta que σ = ε 0 En , vemos que
( )
=
4πε 0 4πε 0 z 2 + R 2 12
σ A = -200 ε 0 C/m 2

σ C = - 50 ε 0 C/m2
41
Electrostática Hugo Medina Guzmán

La placa B tiene una densidad de carga de +200 ε 0 Q


d) Finalmente, C =
C/m2 a su izquierda y de - 50 ε 0 C/m2 a su derecha, V
Los condensadores se representan por medio de dos
o una densidad de carga neta σB = + 150 ε 0 C/m2. líneas paralelas iguales, como se ilustra n la figura

CAPACIDAD - ENERGÍA ELECTROSTÁTICA siguiente .


INTRODUCCIÓN. - La figura a continuación muestra diferentes
De lo visto anteriormente el potencial (tomado condensadores comerciales.
relativo a un potencial cero en el infinito) de un
cuerpo conductor con carga Q es proporcional al
valor de dicha Carga Q. O la carga total Q en el
conductor aislado es directamente proporcional al
potencial del conductor, es decir
Q = CV
Donde C es una constante que depende de la forma
del cuerpo, mas no de Q o V. La figura siguiente muestra como esta formado un
Esta constante C es una característica de un cuerpo al condensador moderno de placas paralelas
lado y se la denomina Capacidad.
Q
C=
V
En el sistema internacional su unidad es el
coulombio/voltio, al que se le denomina faradio,
cuya abreviatura es F, en la práctica la capacidad de
un faradio es muy grande, por eso se usan los
submúltiplos El condensador electrolítico.
microfaradio (μF ) = 10 F
−6

picofaradio ( pF ) = 10 −12 F
A manera de ejemplo calculamos la capacidad de una
esfera conductora aislada de radio R. El potencial de
1 Q
un conductor esférico con carga Q es V =
4πε 0 R
Q
y su capacidad C = = 4πε 0 R
V
EL CONDENSADOR Esquema de condensador electrolítico.
Los sistemas que se encuentran en le práctica no
están referidos al infinito, consisten en dos
conductores con cargas iguales y de signo opuesto,
esto se consigue llevando la carga de un conductor al
otro de tal manea que si este tiene una carga Q el
.primero habrá quedado con Carga -Q. A este sistema
se le conoce como condensador.
La relación que existe entre la carga Q y la diferencia
de potencial ΔV = V1 − V2 , es
Q = CΔV , donde a capacidad C depende solamente
de la geometría del Condensador.
Para calcular la capacidad C de un condensador es Como se carga un condensador
recomendable seguirlos siguientes pasos:
a) Dados los dos conductores situar una carga + Q en
uno de ellos y una carga - Q en el otro.
b) Determinar el campo eléctrico entre los
conductores.
c) Determinar la diferencia de potencial entre los
conductores

42
Electrostática Hugo Medina Guzmán

Solución.
Pongamos cargas Q y - Q a los cascarones de radios
Representación esquemática del circuito anterior. a y b respectivamente.
Aplicando la ley de Gauss encontrar que el Campo
eléctrico entre ellos es
λ Q
E= , donde λ =
2πε 0 r L
Q
De aquí E =
2πε 0 Lr
→ →

Ejemplo 72. El condensador de placas paralelas. Como dV = − E ⋅ d l


Este Condensador consiste en dos placas paralelas de b→ → b Q dr
área A y separadas una distancia d (d es muy pequeña
V1 − V2 = ∫ E ⋅ d l = ∫
a a 2πε 0 L r
comparada con
A). Q b
= ln
Solución. 2πε 0 L a
Finalmente la capacidad es
Q 2πε 0 L
C= =
V1 − V2 b
ln
a
Ejemplo 74. El condensador esférico. Este
Condensador consiste en dos cáscaras esféricas
conductoras y aisladas con radios a y b.
Solución.
Ponemos la Carga Q y - Q como se muestra en la
figura.
El campo eléctrico que se forma entre las placas es:
E = σ ε 0 , siendo σ = Q A , tenemos:
Q
E=
ε0A
→ →
Como dV = − E ⋅ d l Pongamos cargas Q y - Q a los cascarones esféricos
La diferencia de potencial entre las placas es: de radios a y b respectivamente.
2→ → 2 Q Aplicando la ley de Gauss encontramos el campo
V1 − V2 = ∫ E ⋅ d l = ∫ Edx = d eléctrico entre ellos
1 1 ε0 A
1 Q
Finalmente la capacidad, es E=
Q ε A 2πε 0 r 2
C= = 0 → →
V1 − V2 d Como dV = − E ⋅ d l
La diferencia de potencial entre los cascarones es
Ejemplo 73. El condensador cilíndrico. Este b→ → b Q dr
Condensador cilíndrico en dos conductores V1 − V2 = ∫ E ⋅ d l = ∫
cilíndricos concéntricos de radios a y b con longitud
a a 2πε 0 r 2
L, siendo esta longitud mucho mayor a fin de poder Q ⎛1 1⎞
despreciar las irregularidades de los extremos. = ⎜ − ⎟
4πε 0 ⎝ a b ⎠

43
Electrostática Hugo Medina Guzmán

V1 − V2 =
Q (b − a )
4πε 0 ab
Finalmente la capacidad es

Q ab
C= = 4πε 0
V1 − V2 (b − a )
Cuando b → ∞ , la capacidad tiende a
En este caso la diferencia de potencial es igual a ΔV
C = 4πε 0 a , que es la capacidad del conductor
para los dos condensadores.
esférico aislado. El Condensador C1 almacena una carga Ql
Ejemplo 75. Encontrar la capacidad entre dos esferas
Q1 = C1 ΔV
conductoras de radios a y b separadas una distancia c, El Condensador C2 almacena una carga Q2
siendo c mucho mayor que a y b. Q2 = C 2 ΔV
Solución. La carga total almacenada es
Q = Q1 + Q2 = C1 ΔV + C 2 ΔV
= (C1 + C 2 )ΔV
Q
Como le Capacidad es
Pongamos cargas Q y - Q a las esferas de radios a y b ΔV
respectivamente. La capacidad equivalente es
El potencial V1 sobre la esfera de radio a es el Q
C eq = = C1 + C 2
potencial debido a su carga Q mas el potencial ΔV
debido a la carga - Q, tomada como puntual ya que Si fueran n condensadores en paralelo la capacidad
c >> b equivalente sería
V1 =
Q
+
(− Q ) n

4πε 0 a 4πε 0 c C eq = ∑ C i
i =1
De1 mismo modo el potencial V2 sobre la esfera de
radio b CONDENSADORES EN SERIE. La figura
V2 =
(− Q ) + Q siguiente muestra una combinación en serie de los
4πε 0b 4πε 0c condensadores de capacidades C1 y C2.
La diferencia de potencial entre las esferas es
⎛ Q −Q ⎞ ⎛ −Q Q ⎞
V1 − V2 = ⎜⎜ + ⎟⎟ − ⎜⎜ + ⎟⎟
⎝ 4πε 0 a 4πε 0c ⎠ ⎝ 4πε 0b 4πε 0c ⎠
Q ⎛1 1 2⎞
V1 − V2 = ⎜ + − ⎟
4πε 0 ⎝ a b c ⎠
La capacidad del sistema es:
Q 4πε 0
C= =
V1 − V2 ⎛ 1 1 2 ⎞
⎜ + − ⎟ La diferencia de potencial entre los extremos es ΔV ,
⎝a b c⎠ de tal manera que la diferencia de potencial en el
condensador C1 es ΔV1 y la diferencia de potencial
COMBINACIONES DE CONDENSADORES
Generalmente los condensadores se encuentran en el Condensador C2 es ΔV2 y ΔV = ΔV1 + ΔV2 .
combinados en los circuitos eléctricos. Si los dos Condensadores se conectan y luego se
Los condensadores pueden combinarse de dos formas cargan, por la conservación de carga se requiere que
en paralelo o en serie. cada condensador tome la misma carga Q, de tal
A continuación encontraremos la capacidad manera que
equivalente de diferentes combinaciones. Q Q Q
ΔV1 = , ΔV2 = y ΔV =
CONDENSADORES EN PARALELO. La figura C1 C2 C
siguiente muestra una combinación en paralelo de los Q Q Q
condensadores de capacidades C1 y C2 Luego = +
C eq C1 C 2

44
Electrostática Hugo Medina Guzmán

1 1 1 Sea la capacidad Ceq entre los puntos a y b.


Simplificando Q: = +
C eq C1 C 2 Hagamos un corte en el circuito como se muestra en
la figura siguiente.
C1C 2
Finalmente C eq =
C1 + C 2
Si fueran n condensadores en serie
n
1 1
=∑
C eq i =1 C i
Observamos que el circuito partiendo del corte AA a
Ejemplo 76. En la figura siguiente encontrar la la derecha es igual al original luego la capacidad
capacidad equivalente entre a y b. Donde C1 = 2 equivalente entre a’ y b’ es Ceq también.
μF , C2 = l μF y C3 = 3 μF . De esta manera el circuito podemos dibujarlo como
se muestra a continuación.

Este circuito es equivalente al mostrado al inicio.


Luego

Solución.
Primero tenemos que encontrar el equivalente de los
condensadores C2 y C3
C eq1 = C 2 + C 3 = 1μF + 3μF = 4μF .
El Sistema se reduce a
(C + C )C
eq C 2 + C eq C
Finalmente: C eq =
(C + C ) + C
eq
=
2C + C eq
De donde resulta la ecuación
C eq2 + CC eq − C 2 = 0
Tomando la solución positiva

C eq =
( )
5 −1
C
2
Ejemplo 78. En el circuito mostrado, cada
C1 y Ceq1. Están en serie, el equivalente es condensador tiene un valor de 4 μ F. Si Va - Vb = 12
C1C eq1 2× 4 4 voltios, hallar la carga de cada condensador y además
C eq = = = μF Va - Vd.
C1 + C eq1 2+4 3

Ejemplo 77. Se tiene el sistema de Condensadores


mostrado en figura, todos los condensadores tienen
capacidad C y se extiende al infinito. ¿Cuál es la
capacidad equivalente entre los puntos a y b?

Solución.

Solución.

45
Electrostática Hugo Medina Guzmán

Solución.
El caso (a): Como la placa media y las dos adyacentes
están unidas al mismo potencial, de tal manera que no
hay campo entre ellas, y las placas extremas están al
mismo potencial, de tal manera que el sistema
consiste de dos condensadores en paralelo.

En la figura (4)

Q4 =
3C
5
3
( )
V = 4 x10 −6 (12 ) = 28.8x10-6 C
5
Esta misma carga es para los dos condensadores de la
figura (3), por estar en serie.
En la figura (2) 2ε 0 A
La Capacidad es: C eq = 2C =
Q3 Q'3 d
= ⇒ Q'3 = 2Q3 , además El caso (b): La figura a continuación muestra como se
C C/2 forman los campos y es equivalente a cuatro
Q'3 +Q3 = Q4 = 7,2x10-6 condensadores en paralelo.
Luego Q3 = 2,4x10-6 y Q ' 3 = 4,8x10-6
En la figura (1)
Q1 = Q2 = Q '3 = 4,8x10-6
En resumen Q1 = Q2 = 4,8x10-6 C, Q3 = 2,4x10-6 C,
Q4 = 7,2x10-6 C
Q3 2,4 x10 −6
Va – Vd = = = 0,6 V
C 4 x10 −6
Ejemplo 79. Un condensador de radio consiste de 4ε 0 A
cinco placas igualmente espaciadas cada una de área La Capacidad es: C eq = 4C =
A y separadas una distancia d. d
Ejemplo 80. Dos Condensadores de Capacidades
1 μF y 2 μF , cargados cada uno a 50 voltios se
conectan en las dos formas posibles. ¿Cuales son la
carga y diferencia de potencial de cada condensador
en cada caso?
Solución.
La Carga que toman los condensadores a 50 voltios,
es
Calcular la capacidad cuando las placas se conectan Q1 = C1V = 1 × 10 −4 × 50 = 0,5 x 10-6 C
en las formas mostradas en las figuras (a) y (b).

Q2 = C 2V = 2 × 10 −4 × 50 = 1,0 x 10-6 C

46
Electrostática Hugo Medina Guzmán

Finalmente el esquema queda como sigue

Primera posibilidad:
Conectar positivo con positivo y negativo con Ejemplo 81. Dos esferas aisladas conductoras, cada
negativo una con radio 3 cm, están muy separadas conectadas
por un alambre (no considerar la capacidad del
alambre). Las esferas están cargadas a un potencial de
100 Voltios. Una cáscara esférica de radio 4 cm
dividida en dos hemisferios se coloca
concentricamente alrededor de una de las esferas
conectándola a tierra, formando un condensador
Esta conexión nos da que la carga en cada esférico; el alambre que une a las esferas pasa a
condensador sigue siendo la misma través de un pequeño agujero en las cáscaras.
Calcular el potencial final de los dos Conductores.
Q1 = 0,5 x 10-6 C y Q2 = 1,0 x 10-6 C Solución.
La diferencia de potencial sigue siendo 50 voltios.

Segunda posibilidad:
Conectar positivo con negativo y negativo con
positivo.

En la posición antes: La capacidad de cada una de las


esferas es C1 = 4πε 0 R1 , y la carga en cada esfera
es Q = C1V1 = 4πε 0 R1V1
En la posición después: La capacidad de la esfera
En este caso la carga se redistribuirá
simple sigue siendo C1 = 4πε 0 R1 y su carga es
Q' 2 = C1V2 = 4πε 0 R1V1 . La capacidad de las
R1 R2
esferas concéntricas es C 2 = 4πε 0 y su
R2 − R1
carga es
Q' ' 2 = C 2 (V2 − VTierra )
De tal manera que
Q'1 +Q' 2 = Q1 + (− Q2 ) = Q1 − Q2 = 4πε 0
R1 R2
V2
Q'1 +Q' 2 = 0,5 × 10 −6 − 1,0 × 10 −6 = −0,5 × 10 −6 R2 − R1
Pero Q '1 = C1V ' y Q ' 2 = C 2V '
−6
De aquí C1V '+C 2V ' = −0,5 × 10
0,5 × 10 −6 0,5 × 10 −6
V '= − =−
C1 + C 2 1 × 10 −6 + 2 × 10 −6 Como la carga del sistema se conserva:
50 Carga antes = carga después.
V '= − V 2Q1 = Q'1 +Q ' ' 2
3
El valor negativo significa que el potencial es más R1 R2
negativo en la parte superior que la parte inferior.
24πε 0V1 R1 = 4πε 0V2 R2 + 4πε 0 V2
R2 − R1
Luego
2
⎛ 50 ⎞ De aquí: V2 = V1
Q'1 = 1 × 10 −6 ⎜ − ⎟ = - 1,66 x 10-5 C ⎛ R2 ⎞
⎝ 3⎠ ⎜⎜1 + ⎟⎟
⎛ 50 ⎞ ⎝ R2 − R1 ⎠
Q'2 = 2 × 10− 6 ⎜ − ⎟ = - 3,33 x 10-5 C Como V1 = 100 V, R1 = 3 cm y R2 = 4 cm,
⎝ 3⎠ remplazando obtenemos:
47
Electrostática Hugo Medina Guzmán

2
V2 = 100 = 40 V
⎛ 4 ⎞
⎜1 + ⎟
⎝ 4 −1⎠
El potencial final de los dos conductores es 40
voltios.

Ejemplo 82. Encontrar la capacidad equivalente entre


los puntos a y b, todos los condensadores tienen
capacidad C. y por último

Ejemplo 83. Encontrar la capacidad equivalente entre


los puntos a y b, todos los condensadores tienen
capacidad C.

Solución. Este caso al parecer muy complicado se


simplifica debido a su simetría.
El reparto de Cargas al ponerlo a una diferencia de
potencial sería como se muestra a continuación:
Solución. A este circuito aparentemente complejo,
porque no está ni en serie ni en paralelo podemos
hacerle algunas transformaciones sin cambiarlo.

Entre a y c hay un solo condensador


Entre a y d hay un solo condensador
Entre c y d hay un solo condensador
Entre d y b hay un solo condensador
Claramente se ve que este circuito es igual al que se
Entre c y b hay un solo condensador
muestra en la figura a continuación:
Esto nos permite hacer el dibujo equivalente siguiente

El que también se puede dibujar como


Y esto nos lleva a:

Continuando la simplificación Podríamos resolverlo en forma similar al hexágono


anterior, pero lo vamos a hacer en forma diferente.

48
Electrostática Hugo Medina Guzmán

Se puede ver: Vab = Vad y Vcb = Vdb


Con lo que podemos concluir:
Vc = Vd , esto hace que el condensador entre c y d
no se cargue y es como si no existiera, con lo que el
circuito queda reducido, a condensadores en paralelo
el que podemos ir simplificando hasta encontrar el
condensador equivalente.
De aquí Q = 2,0 × 10 (9 ) = 18,0 μ C
−6

Que es la carga que corresponde a cada condensador.


Q 18 × 10 6
b) Vcb = = =6V
3,0 × 10 −6 3,0 × 10 −6
Q 18 × 10 6
Vdb = = =3V
6,0 × 10 −6 6,0 × 10 −6
Vcd = Vc − Vd = Vcb − Vdb = 6 – 3 = 3 V
El punto a está a mayor potencial.
Ejemplo 84. Sean cuatro condensadores conectados c) La distribución de las cargas es como se indica a
como se indica en la figura anexa. Con el interruptor continuación
S abierto, se aplica una diferencia de potencial
Va − Vb = Vab = 9 V.

Esto se reduce a

a) Calcule el valor de la carga de cada condensador.


b) ¿Cuál es la diferencia de potencial
Vcd = Vc − Vd ? ¿Qué punto está a mayor potencial?
Y finalmente
c) Se encierra el interruptor S, sin desconectar la
diferencia de potencial Vab ¿Cuál es el valor de la
carga de cada condensador?
d) ¿Cuál es el cambio de energía ΔU = U i − U f al
De lo que obtenemos:
cerrar el interruptor S? comente el signo de su
resultado. Sugerencia use el concepto de condensador Q = 4,5 × 10 −6 (9) = 40,5x10-6 C = Q1 + Q2
equivalente para obtener el valor ΔU . Como Vac = Vad
Solución.
Q1 Q2 Q
a) El dispositivo se carga de la manera indicada en la ⇒ −6
= −6
⇒ 1 =2
figura siguiente. 6 × 10 3 × 10 Q2
De estas dos últimas relaciones:
Q1 = 27x10-6 C, Q2 = 13,5x10-6 C

d) U i =
1
2
1
(
C eq (i )Vab2 = 4 × 10 −6 (9 )
2
)2

= 1,62x10-4 J

Uf =
1
2
1
(
C eq ( f )Vab2 = 4,5 × 10 −6 (9 )
2
)2

1 1 1 1 = 1,82x10-4 J
= + = ⇒ C eq = 2,0 μ F
C eq 6,0 3,0 2,0
ΔU = U i − U f = 1,62x10-4 - 1,82x10-4
49
Electrostática Hugo Medina Guzmán

= - 0,20x10-4 J QX + QZ = Q ' X +Q 'Z = 1320 x 10-6C


El signo es negativo porque se ha realizado trabajo Q' X C X 9 × 10 −6
Como = = =3
sobre el sistema porque aumento su capacidad y había Q' Z C Z 3 × 10 −6
que aumentar la carga.

Ejemplo 85. El circuito mostrado inicialmente con


3
(
Q' X = 1320 × 10 −6 = 990 x 10-6C
4
)
los condensadores descargados X, Y, y Z, y los
interruptores abiertos, S1 y S2. Una diferencia Q' X 990 × 10 −6
Luego V ' X = = = 110 V.
potencial Vab = +120 V se aplica entre los puntos a y CX 9 × 10 −6
b. después de que el circuito esté montado, se cierra
el interruptor S1, pero el interruptor S2 se mantiene ENERGIA ELECTROSTATICA DE CARGAS
abierto. Encontrar Para colocar una carga q1 en el espacio no se realiza
a) La energía almacenada en el condensador X trabajo pero si querernos colocar otra carga q2
b) La carga del condensador Y. tenernos que realizar un trabajo, porque tenemos que
c) El voltaje a través del condensador Z. traer la carga q2 del infinito donde el potencial es cero
d) Si el interruptor S1 se abre, y se cierra S2. hasta el punto situado a r12 de q1 donde el potencial
Encontrar el voltaje final a través del condensador X, debido a esta última es
1 q1
V =
4πε 0 r12
El trabajo realizado es
1 q1 q 2
W = q 2V =
4πε 0 r12
Solución.
este trabajo realizado queda como energía en el
a) U X =
1
2
1
2
( 2
)
CV 2 = 9 × 10 −6 (120) = 0,065 J sistema
1 q1 q 2
1 1 1 1 1 1 U=
b) = + = + = 4πε 0 r12
C eq CY C Z 6 3 2 Ahora si tenernos tres cargas presentes la energía en
C eq = 2μF el sistema es

( )
QY = Q = CeqVab = 2 × 10 −6 (120 ) W =U =
1 ⎛ q1 q 2 q1 q3 q 2 q3 ⎞
⎜ + + ⎟
= 240 mC 4πε 0 ⎜⎝ r12 r13 r23 ⎟⎠
Q 0,240 Este resultado es independiente del radio en que se
c) VZ = = = 80 V lleva las cargas a sus posiciones finales.
C Z 3 × 10 −6 En el caso de ser o cargas la energía electrostática
d) Con S2 abierto. total es la suma de las energías de todos los pares
posibles.
1 qi q j
U=
4πε 0

todos rij
los
pares
Cuando queremos cargar un cuerpo con densidad de
carga ρ , consideramos que ya se ha juntado una
La carga del condensador X es: cantidad de carga q, el trabajo para traer una carga dq
Q X = C X V = (9 × 10 −6 )(120 ) = 1080 x 10-6C del infinito es la energía diferencial que se almacena.
La carga del condensador Z es: 1qdq
dU =
QZ = QY = 240 x 10-6C 4πε 0 r
Con S2 cerrado. Siendo dq = ρdV y dV un elemento diferencial del
volumen del cuerpo.
Nota: El concepto de “voltio” “potencial” en
electricidad es similar al concepto de “altura” en
gravedad y al concepto de “temperatura” en
termodinámica. En todos estos casos, se define un
nivel de referencia de el cual se mide
voltio/altura/temperatura.

50
Electrostática Hugo Medina Guzmán

• El cero para el voltaje se considera el voltaje uno de los anillos es respectivamente A1 y A2.
de la tierra del planeta, llamado en la Encontrar les Cargas q1 y q2 que tienen los anillos.
ingeniería eléctrica “tierra”. Solución.
• El cero para la gravedad se considera ser el
nivel del suelo (en el caso de la tierra).
• El cero para la temperatura se considera ser
el Kelvin cero supuesto.

El potencial en 1:
q1 q2
V1 = +
4πε 0 R (
4πε 0 a 2 + R 2 ) 12

1 ⎡ q2 ⎤
Ejemplo 86. Encontrar la energía almacenada al = ⎢q1 + 12 ⎥
cargar una esfera de radio R con carga uniformemente 4πε 0 R ⎣⎢ (
1+ a / R2
2
) ⎦⎥
distribuida de densidad volumétrica ρ . El potencial en 2:
Solución. Supongamos que ya hemos cargado una q2 q1
esfera de radio r, y queremos ponerle un diferencial V2 = +
de carga en el volumen inmediato de espesor dr, 4πε 0 R (
4πε 0 a 2 + R 2 )
12

como se muestra en la figura.


1 ⎡ q1 ⎤
= ⎢q 2 + 12 ⎥
4πε 0 R ⎣⎢ (
1+ a / R2
2
) ⎦⎥
El potencial en el infinito:
V∞ = 0
Trabajo para traer una carga q desde el infinito a 1:
A1 = q (V1 − V∞ )
q ⎡ q2 ⎤
La Carga q almacenada ya, es = ⎢q1 + ⎥ (1)
4 4πε 0 R ⎢⎣ (
1+ a2 / R2 )
12
⎥⎦
q = π r 3 drρ Trabajo para traer una carga q desde el infinito a2:
3
La Carga dq que vamos a traer del infinito es A2 = q (V2 − V∞ )
dq = 4π r drρ 2
q ⎡ q1 ⎤
= ⎢q 2 + 12 ⎥
(2)
El trabajo en traer la carga dq es
qdq 4π ρ 2 r 4 dr
4πε 0 R ⎢⎣ (
1+ a2 / R2 ) ⎥⎦
dU = = (1) y (2) son dos ecuaciones con incógnitas q1 y q2.
4πε 0 r 3πε 0 Resolviendo (1) y (2) obtenemos:
El trabajo para la carga toda la esfera se halla ⎡
integrando r desde 0 hasta R. 1 A1 ⎤
⎢ − ⎥
( )
12
R 1 R qdq 4πρ 2 R ⎢
4πε 0 RA2 ⎣ 1 + a R
2 2 A ⎥⎦
U = ∫ dU = ∫ ∫
2
= r 4 dr q1 =
0 4πε 0 0 r 3ε 0 0
q ⎡ 1 ⎤
4πρ 2 R 5 ⎢ 1 + a 2 R 2 − 1⎥
( )
U= ⎣ ⎦
15ε 0 ⎡ 1 A2 ⎤
4πR 3 3 Q2 ⎢ − ⎥
( )
12
Como Q = ρ ⇒U= ⎢
4πε 0 RA1 ⎣ 1 + a R
2 2 A 1 ⎥⎦
3 5 4πε 0 R q2 =
q ⎡ 1 ⎤
− 1
Ejemplo 87. Dos anillos coaxiales uniformemente
cargados de radios iguales R está en planos paralelos

⎣ 1+ a R
2
( 2 ⎥
⎦ )
separados una distancia a, el trabajo que se realiza
para traer una carga q del infinito al centro de cada

51
Electrostática Hugo Medina Guzmán

Ejemplo 88. Una carga Q = - 800 nC se distribuye colocado sobre la otra, la diferencia de potencial entre
uniformemente en un anillo de radio de 2,4 m. Una ellas es
carga q = + 600 nC se pone en el centro del anillo. q
Los puntos A y B están situados en el eje del anillo. V =
C
Para transferir en este instante una carga dq de una
placa a la otra. El trabajo requerido para transferir
esta carga es
dW = dU = Vdq
Poniendo V en función de q
q
dW = dq
C
Para obtener el trabajo total integramos desde q = 0
hasta la carga total q = Q0.
a) ¿Cuál es el trabajo hecho por una fuerza externa 1 Q0 1 Q02
C ∫0
que transporta un electrón de B a A? U= qdq =
b) si el potencial eléctrico es igual a cero en un punto 2 C
en el eje del anillo. ¿Cuál es la distancia de este 1
punto del centro del anillo? Como Q0 = CV0 ⇒ U = CV02
2
c) Si se un electrón se lanza a lo largo del eje del Esta expresión es general para todo tipo de
anillo desde el infinito. El electrón llega y se detiene condensador, ya que la geometría del condensador no
momentáneamente en un punto en el eje que está a interviene en el razonamiento.
5,0 m del centro del anillo. ¿Cuál es la velocidad
inicial del electrón en el infinito? Densidad de energía del campo eléctrico. Es
d) Si una fuerza externa retira la carga q del centro razonable considerar que esta energía se almacena en
del anillo y la transporta al infinito. ¿Cuál es el el campo eléctrico y por lo tanto es conveniente
trabajo realizado por esta fuerza externa? definir el concepto de densidad de energía del campo
Solución. eléctrico, para esto consideremos un condensador de
a) W B → A = e(V A − V B ) placas paralelas, despreciando las irregularidades de

e ⎡⎛⎜ q Q ⎞ ε9 A
⎟ 1os extremos tenemos C =
= ⎢ −

2
(
4πε 0 ⎢⎝ (1,8) 1,8 + 2,4 2
⎜ )12 ⎟

ε0 A
d
⎛V ⎞
⎛ ⎞⎤ Y Q0 = CV0 = V0 = ε 0 A⎜ 0 ⎟
-⎜ q − Q ⎟⎥ d ⎝d ⎠
⎝ (
⎜ (3,2 ) 3,2 2 + 2,4 2 )
12 ⎟
⎠⎥⎦ Q0 = ε 0 AE0
WB → A = -700 eV La densidad de energía es
q Q U
b) − =0 μE =
(
x x 2 + 2,4 2 ) 12 Volumen
el volumen es: Ad
⇒ x = 2,7 m
1 (ε 0 AE0 )
2
1 Q02
c) 1 m v 2 = e ⎡ q − Q ⎤
⎢ 12⎥ 2 (ε 0 A d )
2
e 0
(
4πε 0 ⎣⎢ (5,0) 5,02 + 2,42 ) ⎦⎥ μE = 2 C =
Volumen Ad
⇒ v0 = 9 × 10 6 m/s 1
d) WO →∞ = q(V∞ − VO ) μ E = ε 0 E 02
2
q ⎡⎛ Q ⎞ ⎛⎜ Q ⎞⎤
⎟⎥
Aunque para esta demostración particularizamos para
= ⎢⎜⎜ − ⎟⎟ − − el caso de placas paralelas, esta ecuación es aplicable
4πε 0 ⎢⎝ (2,4) ⎠ ⎝ ∞ + 2,4 2

⎜ 2
( )
12 ⎟
⎠⎥⎦ para cualquier caso.
-3
= + 1,8 x 10 J
Ejemplo 89. Se tiene un condensador esférico con
ENERGIA ALMACENADA POR UN radio interior a y radio exterior b, cuando la
CONDENSADOR diferencia de potencial entre las cáscaras esféricas es
La energía que se almacena en el condensador es V0, calcular la energía electrostática almacenada.
igual a la energía requerida para cargarlo. Solución.
Ahora encontraremos esa energía. La capacidad de este condensador es
Consideremos que se ha tomado una carga q de una
de las placas de un condensador de capacidad C
52
Electrostática Hugo Medina Guzmán

ab se aplica a través de la red. El interruptor S en la red


C = 4πε 0 se mantiene abierto. Calcule:
b−a a) la capacidad equivalente de la red.
La energía almacenada es b) la diferencia de potencial V – V .
1 1 ab c d
U= CV02 = 4πε 0 V02 = c) la carga y voltaje en el condensador de 2,0 μF.
2 2 (b − a ) Se cierra el interruptor S manteniendo la diferencia de
ab potencial V en 10,0 V.
2πε 0 V2
(b − a ) 0
ab
d) la carga y voltaje en el condensador de 2,0 μF.
Otra forma de cálculo es mediante la densidad de e) el cambio de energía electrostática ΔU = U - U ,
f i
energía. antes y después de cerrar el interruptor S. Comente su
1 resultado.
μE = ε 0 E 2
2
Por la ley de Gauss evaluamos E

Solución.
a) la capacidad equivalente de la red.
El Campo eléctrico entre las cáscaras es
Q
E=
4πε 9 r 2
ab
Donde Q = CV0 = 4πε 0 V
(b − a ) 0
ab V0
Luego: E =
(b − a ) r 2
1
-reemplazando el valor de E en μE = ε 0 E 2
2
ε 0 a 2b 2 V02
μE =
2(b − a ) r 4
2

Para obtener la energía total


dU
μE = ,
dV
Con dV = 4πr dr y dU = μ E dV
2

b ε 0 a 2b 2 V02
U = ∫ dU = ∫ μ E dV = ∫ 2(b − a ) 2
dr =
a r2
b) la diferencia de potencial V – V .
2πε 0 a 2b 2V02 dr 2πε 0 a 2b 2V02 ⎛ 1 1 ⎞
b c d

(b − a )2 ∫a r 2 = (b − a )2 ⎜⎝ a − b ⎟⎠ Vbc =
Q1
−6
, Vbd =
Q2
4 × 10 6 × 10− 6
ab
Finalmente: U = 2πε 0 V02
(b − a )
Resultado igual al evaluado directamente, lo cual
prueba la aseveración que hicimos sobre la
generalidad de la expresión de la densidad de energía.

Ejemplo 90. La red de condensadores mostrados


Q = (4,8 × 10−6 )(10,0) = 48 x 10-6 C.
está montada con los condensadores inicialmente
descargados. Una diferencia de potencial V = 10,0 V
ab

53
Electrostática Hugo Medina Guzmán

Consideremos un sistema formado por dos placas, si


permitimos el movimiento Δx de una de ellas,. El
trabajo mecánico realizado por una fuerza exterior
F para mover las placas es:
ΔW = F Δx
Siendo F la fuerza entre las placas, este trabajo debe
Q1 = Q2 = Q/2 = 24 x 10-6 C ser igual al cambio de energía electrostática en el
−6 condensador.
24 × 10
Luego: Vbc = = 6 V, Si el objeto en mención es obligado a un movimiento
4 × 10− 6 de rotación debido a un torque τ y a un
24 × 10−6 desplazamiento es Δθ , el trabajo realizado es
Vbd = = 4 V.
6 × 10− 6 ΔW = τ Δθ
Vc –Vd = Vbc – Vbd = 6 - 4 = 2 V.
Ejemplo 91. ¿Cuál es la fuerza entre las placas de un
c) la carga y voltaje en el condensador de 2,0 μF. condensador de placas paralelas, al separarlas una
distancia dx en, el condensador tiene una área A,
(
Vbd = 4 V, Q2 μF = 2 × 10
−6
)(4) = 8 x 10 -6
C. separación entre placas d y Carga Q?
Solución.
Se cierra el interruptor S manteniendo la diferencia de El trabajo es
potencial V en 10,0 V.
ab
ΔW = F Δx
que es igual al cambio de la energía electrostática
1 Q2
U=
2 C
el cambio es (por derivación con respecto a C)
1 2 ⎛1⎞
ΔU = Q Δ⎜ ⎟ .
2 ⎝C ⎠
d) la carga y voltaje en el condensador de 2,0 μF.
Como las capacidades equivalentes de la parte ⎛1⎞ ⎛ 1 ⎞ ⎛ d ⎞
superior y la inferior son iguales a 10 μF, las con Δ⎜ ⎟ = Δ⎜⎜ ⎟⎟ = Δ⎜⎜ ⎟⎟
diferencias de potencial de la parte superior y la parte
⎝C ⎠ ⎝ε0A d ⎠ ⎝ε0A⎠
inferior son iguales a 10/2 = 5 V. Como ε0 y A son constantes, el único variable es d y
(
Luego Q '2 μF = 2 × 10
−6
)(5) = 10 x 10
C. -6 Δd = Δx .
e) el cambio de energía electrostática ΔU = U - U , ⎛ 1 ⎞ Δx
f i Luego tenemos: Δ⎜ ⎟=
antes y después de cerrar el interruptor S. Comente su ⎝ C ⎠ ε0 A
resultado
Antes de cerrar Q 2 Δx
Y ΔU =
1 1
( )( )
U i = Ceq iV 2 = 4,8 × 10− 6 102 = 2,4 x 10-4
2ε 0 A
2 2 Siendo ΔU = ΔW = FΔx , podemos escribir:
J.
Q 2 Δx Q2
Después de cerrar FΔx = ⇒ F=
2ε 0 A 2ε 0 A
Uf =
1
2
1
2
( )( )
Ceq f V 2 = 5 × 10− 6 102 = 2,5 x 10-4
1 Q
Reacomodando: F = Q
J. 2 ε0 A
ΔU = 2,5 x 10-4 - 2,4 x 10-4. = 0,1 x 10-4 J = 10 x 10-6
Q σ Q
J. Como σ = y E= ⇒ E=
La capacidad equivalente en la nueva configuración A ε0 ε0A
es mayor que en la original, siendo la diferencia de
1
potencial igual, la energía almacenada en la nueva Finalmente: F = QE
configuración es mayor. 2
Este resultado que es diferente en el factor ½ al que
FUERZAS, TORQUES esperábamos por intuición se debe que el campo E no
Ahora mostraremos como la fuerza sobre uno de los es el campo en las cargas, ya que la carga en la
objetos en un sistema cargado puede calcularse a superficie tiene un espesor muy delgado y el campo
partir de la energía electrostática. va desde 0 en el interior hasta E en el espacio entre
las placas, es decir la distribución superficial de

54
Electrostática Hugo Medina Guzmán

cargas tiene un espesor. El campo que actúa sobre las Pero en presencia de un campo eléctrico se produce
Cargas es el promedio E / 2 , esto explica el factor una distorsión como se muestra en la figura siguiente.
½.

CAMPO ELECTRICO EN UN DIELECTRICO


Dieléctrico es un material no conductor como por
ejemplo el vidrio, el papel, el plástico.
Faraday experimentando con condensadores aplicó la
misma diferencia de potencial a dos condensadores
de placas paralelas iguales, pero uno de ellos con un
material dieléctrico entre las placas y observó que el
condensador con dieléctrico almacena mayor carga. El átomo se polariza, y esta configuración equivale en
La capacidad de un condensador de placas paralelas primera aproximación a un dipolo eléctrico..
es Es razonable considerar que si el campo no es
demasiado grande al momento sobre el dipolo será
ε0A proporcional a la intensidad de este campo.
C=
d →

y la carga con respectó a la diferencia de potencial es: Si consideramos que p sea el momento dipolar de
Q = CV cada átomo y que tenemos N átomos por unidad de
volumen, el momento dipolar por unidad de volumen
Con un material dieléctrico la carga es mayor, o sea, →
aumenta la capacidad. será N p , al que llamaremos Vector polarización
Manteniendo la carga constante, al poner el

dieléctrico, siendo las características de la capacidad
P.
las dimensiones, la diferencia de potencial → →
disminuirá. P=Np
Como V = Ed, el Campo eléctrico entre las placas →
disminuye, esto se explica porque las cargas del P = momento dipolar por unidad de volumen, que
dieléctrico en presencia del campo eléctrico se tiene el mismo valor en todos los puntos del
orientan como se muestra en la figura siguiente, a este dieléctrico.
efecto decimos que el dieléctrico está polarizado. → →
Como p = q δ , siendo δ el desplazamiento
promedio de las cargas positivas y negativas en el

sentido de p , podemos escribir
→ →
P = Nq δ
Consideremos un cuerpo tetraedral polarizado, tal
como se muestra en la figura siguiente

El dieléctrico polarizado a pesar de ser eléctricamente


neutro en promedio produce un Campo eléctrico tanto
exterior como interiormente, como resultado de esto
aparece un campo eléctrico opuesto al original,
disminuyéndolo.
En la superficie aparecen sólo cargas negativas. Sea A
la superficie total del tetraedro y el volumen
POLARIZACION.
CARGAS DE POLARIZACION. inmediato es Aδ (recordar que δ es la longitud del
La figura a continuación nos muestra un átomo en un dipolo).
dieléctrico en ausencia de campo eléctrico, la carga
positiva en el centro y la carga negativa distribuida
uniformemente.

55
Electrostática Hugo Medina Guzmán

→ Ejemplo 92. Una varilla delgada dieléctrica de


El momento dipolo de este volumen es (Sδ ) P , esto sección transversal A se extiende a lo largo del eje x
→ desde x = 0 a x = L la polarización es longitudinal y
lo podemos escribir corno SP δ . Si es la carga q la
( )

está dada por P = ax + b iˆ . Encontrar la
2
que debe desplazarse una distancia δ de - q para
→ → densidad de Carga de polarización y mostrar
producir el momento dipolo tenernos SP δ = q δ , explícitamente que la carga total es cero.
de donde obtenemos : q = SP . Solución.

q
Y =σP = P
S
q
Llamando a = σ P densidad de carga superficial
S
debido a la polarización.

En general σ P = P⋅ nˆ , donde n̂ es la dirección
normal a la superficie y q la carga total en la Primero encontremos la densidad superficial

superficie.

σ P = P⋅ nˆ
q = ∫ σ P dS = ∫ P ⋅ nˆ dS →
A A Para la cara en x = 0, nˆ = −iˆ y P = biˆ :
σ P ( x = 0 ) = biˆ ⋅ (− iˆ ) = −b
Ahora observamos lo que sucede en el interior del
volumen, le figure muestra el volumen inmediato

( )
interior a la superficie. →
Para la cara en x = L, nˆ = iˆ y P = aL + b iˆ :
2

σ P ( x = L ) = (aL + b )iˆ ⋅ iˆ = aL + b
2 2


Para la superficie cilíndrica lateral, como n̂ ⊥ P
σPS = 0

La densidad volumétrica
⎛ ∂ ˆ ∂ ˆ ∂ ˆ⎞
j + k ⎟⎟ ⋅ (ax 2 + b )iˆ

Por el principio de conservación de la carga, la carga
dejada en este volumen V debe ser igual a - q, ρP = ∇ ⋅ P = ⎜⎜ i+
⎝ dx dy dz ⎠
Donde − q = ∫ V
ρ P dV
=

(
ax 2 + b = 2ax)
Siendo ρ P la densidad de carga por volumen debido dx
a la polarización. La Carga total es:

Para evaluar ρ P en función de P



QT = σ P ( x =0 ) A + σ P ( x = L ) A + ∫ ρ P dV
V

− q = − ∫ P ⋅ nˆ dS = ∫ ρ P dV (
= − bA + aL + b A +
2
) ∫ L

0
2ax( Adx )
A V
Usando el teorema de la divergencia convertimos la QT = −bA + aL2 A + bA + aAL2 = 0
integral de superficie en integral de volumen. Valor que esperábamos encontrar.
→ →
− ∫ P ⋅ nˆ dS = ∫ ∇ ⋅ PdV LA LEY DE GAUSS PARA DIELECTRICOS -
A V
→ → DESPLAZAMIENTO ELECTRICO
Donde ∇ ⋅ P .es la divergencia de P , el operador Supongamos una región en que tenemos una carga q
→ en un medio dieléctrico, este medio se polariza y
∇ opera vectorialmente sobre P . aparecen cargas por polarización, aplicando la ley de
Luego Gauss.

∫V
ρ P dV = ∫ ∇ ⋅ PdV
V
De este último obtenemos finalmente

ρ P = −∇ ⋅ P

56
Electrostática Hugo Medina Guzmán

Llamando a (ε 0 + χ ) = ε , Permitividad del


→ →
material obtenemos una relación entre D y E .
El comportamiento de un material queda
completamente especificado ya sea por la
permitividad ε o por la susceptibilidad χ . Sin
embargo es más conveniente trabajar con una
cantidad sin dimensiones, La constante dieléctrica
K.
ε
→ 1 K=
∫ E ⋅ nˆdS = ε (q + Q )
S
0
P
ε0
Para el vacío K =1
Donde Q P es la carga de polarización.

CONSTANTE DIELECTRICA DE
QP = − ∫ ρ P dV = − ∫ ∇ ⋅ PdV ALGUNOS MATERIALES
V V MATERIAL K
Convirtiendo la integral de volumen a integral de Vidrio 5 - 10
superficie por el teorema de la divergencia. Mica 6,0
→ →
Nylon 3,5
∫ ∇ ⋅ PdV = ∫ P ⋅ nˆdS
V S Madera 2,5 - 3,0
→ Caucho 2 – 3,5
Obtenemos: Q P = ∫ P ⋅ nˆdS
S
Agua destilada (20 ºC) 60
De aquí: Aire (1 atm) 1,00059
→ 1 ⎛ →

∫ E ⋅ nˆdS =
S

ε0 ⎝
q + ∫S
P ⋅ nˆ dS ⎟

Ejemplo 93. Cuál es la capacidad de un condensador
de placas paralelas de área A y separación d, cuando
→ → se le introduce un dieléctrico de constante K que llena
⇒ ε 0 ∫ E ⋅ nˆ dS = q + ∫ P ⋅ nˆ dS completamente el espacio entre placas.
S S Solución.

ε 0 ∫ ⎛⎜ ε 0 E + P ⎞⎟ ⋅ nˆdS = q
→ → El Campo eléctrico entre las placas es:
Y
S
⎝ ⎠ σ
E=
Definamos el Campo vectorial Desplazamiento ε
⎛→⎞ Q
eléctrico ⎜ D ⎟ Donde σ= y ε = Kε 0
⎝ ⎠ A
→ → → Q
D = ε 0 E+ P ∴ E=
Kε 0 A
Y tenemos la ley de Gauss para dieléctricos
La diferencia de potencial entre las placas

∫ D ⋅ nˆdS = q Q
→ →
S V = ∫E ⋅d l = d
Kε 0 A
LA CONSTANTE DIELECTRICA Q ε A
Vimos que la polarización del medio ocurre como La Capacidad es C = =K 0
una respuesta al campo eléctrico en el medio, pera el V d
→ → La capacidad de1 condensador, con dieléctrico es K
caso en que P desaparece cuando se quita E y para veces a la capacidad sin dieléctrico.
materiales isotrópicos en los cuales se orienta según
→ → Si al dieléctrico cubriera parte del espesor del
E , P es proporcional a , pare Campos no muy espacio que separa a las placas, digamos un espesor t,
intensos como se muestra en la figura.
→ →
P=χE
donde χ es llamada Susceptibilidad eléctrica,
→ → →
como D = ε 0 E + P :
→ → → → En este caso tenemos el campo eléctrico en parte
D = ε 0 E + χ E = (ε 0 + χ ) E vacío y en parte con dieléctrico.
En el vacío:

57
Electrostática Hugo Medina Guzmán

σ Q
E vacío = = Ejemplo 95. Se tiene un condensador de placas
ε0 ε0A paralelas cargado con carga Q y aislado de manera
En el dieléctrico: que la carga en las placas se conserva. Calcular la
σ Q energía en el condensador antes y después de insertar
E dieléctrico = = un dieléctrico, llenando todo el espacio entre las
ε KεA placas. En base a lo calculado, ¿se realizó algún
La diferencia de potencial entre las placas trabajo para insertar el dieléctrico? ¿Quien lo realizó?
→ →
V = ∫ E ⋅ d l = E vac (d − t ) + E dilec t Solución.

Q ⎡
= ⎢ (d − t ) + t ⎤⎥
ε0A ⎣ K⎦
y la Capacidad
ε0A
C= 1 Q2
t
(d − t ) + Ui =
2 C
K
Ejemplo 94. Evaluar el Campo eléctrico debido a una
Carga puntual q dentro del dieléctrico de constante K.
Solución.
Aplicamos la ley de Gauss para una superficie
esférica de radio r, Colocando a la carga q en el
origen.
1 Q2
→ Uf =
∫ D ⋅ nˆdS = q
S
2 KC
q → q 1 Q2 ⎛ 1 ⎞
D 4π r = q ⇒ D =
2
y D = rˆ ΔU = U f − U i = ⎜ − 1⎟
4π r 2
4π r 2 2 C ⎝K ⎠
El Campo eléctrico es Si se realiza un trabajo, ese trabajo lo realiza el ente
→ → que ejerce la fuerza para introducir el dieléctrico.
→ D D q
E= = = rˆ
ε Kε 0 4πKε 0 r 2 Ejemplo 96. Cuál es la fuerza F necesaria para
introducir un dieléctrico de constante k entre les
En el caso en que la carga q esté entre dos medios placas de un condensador de placas paralelas, las
distintos de constantes K1 y K2 placas se mantienen a una diferencia de potencial
constante V. Siendo las dimensiones del condensador,
área La, separación d.
Solución.

Aplicamos la ley de Gauss para una superficie


esférica de radio y. La figura muestra el dieléctrico parcialmente
→ insertado, la energía potencial acumulada en esta
∫ D ⋅ nˆdS = q
S
⇒ D1 2π r 2 + D2 2π r 2 = q posición es igual a la energía de la parte con
dieléctrico (x) más la energía de la parte sin
Pero D1 = ε 1 E = K 1ε 0 E y dieléctrico (L – x).
D2 = ε 2 E = K 2 ε 0 E 1 Kε 0 a ( x ) ε a (L − x )
U= (ΔV )2 + 1 0 (ΔV )2
Luego: K 1ε 0 E 2π r + K 2 ε 0 E 2π r = q
2 2 2 d 2 d
∂U
E 2πε 0 r 2 (K 1 + K 2 ) = q La fuerza es F = Fx =
∂x
q
y E=
2πε 0 (K 1 + K 2 )r 2
→ q
Vectorialmente: E = rˆ
2πε 0 (K 1 + K 2 )r 2
58
Electrostática Hugo Medina Guzmán

De aquí 1 ε 0a
∂ ⎡ 1 Kε 0 a ( x ) ε a (L − x ) Finalmente F = (K − 1)(ΔV )2
F= (ΔV )2 + 1 0 (ΔV )2 ⎤⎥ = 2 d
∂x ⎢⎣ 2 d 2 d ⎦
1 Kε 0 a ε a
(ΔV )2 − 1 0 (ΔV )2
2 d 2 d

PREGUNTAS Y PROBLEMAS
1. Considere dos esferas iguales cargadas con 1C
separadas en una distancia r.
(a) Calcule la masa que debieran tener las esferas
para que se encuentren en equilibrio estático
considerando la fuerza gravitacional y la
electrostática.
(b) Considerando que la densidad de masa de las
partículas es de 5,5g/ cm3 (aproximadamente la
densidad del hierro), ¿Cuál es la distancia mínima a la Respuesta. 2,4061 x 10−7 C
cual se pueden poner dichas esferas?
Indicación: Aproxime la fuerza entre las esferas como 4. Dos globos iguales llenos de Helio, están cargados
cargas puntuales. La constante de gravitación con carga igual Q. Mediante dos hilos de longitud 1m
universal es G = 6,67 x 10−11Nm2/ kg2 y la constante amarrados a los globos se suspende una masa de 0,
en la Ley de Coulomb es k =9 x 109Nm2/C2. 005 kg quedando el sistema flotando en equilibrio
Respuesta. con los hilos formando un ángulo de 60° entre sí.
m = 1, 16 · 1010 kg; r = 159, 18m (entre centros) Determine el valor de la carga Q.

2. Tres cargas puntuales iguales a Q se encuentran


ubicadas en los vértices de un triángulo equilátero de
lado a. Determine la magnitud de la fuerza eléctrica
que experimenta cada una de ellas.

Respuesta. 1, 2537 x 10−6 C

5. Dos cargas iguales a Q y 5Q están en línea recta


separadas una distancia a. Determine los puntos en la
Respuesta.
línea que une las cargas donde el campo eléctrico es
1 Q2 cero.
F= cos 30°
2πε 0 a 2
6. Se tienen tres cargas como se indica en la figura.
3. Dos pequeñas esferas de masa m están suspendidas
de un punto común mediante cuerdas de longitud L.
Cuando cada una de las esferas tiene carga q, las
cuerdas forman un ángulo con la vertical como indica
la figura. Demuestre que la carga q viene dada por
q = 2L sen θ mg tan θ/k ,donde k es la constante de
Coulomb. Determine q si m = 10g, L = 50cm y θ =
10°.

a) Calcular el campo eléctrico en el origen del


sistema coordenado.

59
Electrostática Hugo Medina Guzmán

b) Determinar la fuerza que se ejerce sobre la carga en los cuales la carga de prueba esta en equilibrio.
en el eje x. Discuta si el equilibrio es estable o inestable.
Respuesta.
7. Cuatro cargas puntuales q, 2 q, - 4 q y 2 q están →
qQ ⎧⎪ (x − a )iˆ + yˆj (x + a )iˆ + yˆj ⎫⎪ ,
F= ⎨ + 4 32⎬
fijas en los vértices de un cuadrado de lado b. En el
centro del cuadrado se coloca una quinta carga q. [ ]
4πε 0 ⎪⎩ ( x − a )2 + y 2 3 2
[ ]
(x + a )2 + y 2 ⎪⎭
(a) Indique en que dirección apunta la fuerza que el punto (3a, 0) es un punto de equilibrio inestable.
actúa sobre la carga central q.
(b) Calcule explícitamente la fuerza (magnitud y 12. Deduzca una expresión para el campo eléctrico
dirección). producido por un trozo recto de hilo de longitud L
Respuesta. con carga Q distribuida uniformemente en su
Eligiendo el eje x como la diagonal que va desde - 4q longitud, en un punto de coordenadas (x; y), estando
a 2q y el eje y como la diagonal que va desde la otra el origen en el extremo izquierdo del hilo y el eje y
carga 2q a la carga q, las componentes de la fuerza perpendicular al hilo.
3q 2 q2
son: Fx = , F =
πε 0b 2 y
2πε 0b 2

8. Dos cargas Q1 y Q2 están a una distancia d:


(a) Determine el punto en la línea que une las cargas
donde el campo eléctrico es cero.
13. De una barra fina vertical que tiene densidad
(b) Si se trae desde el infinito una tercera carga
situándolo donde el campo eléctrico es cero, ¿La lineal uniforme de carga λ = 10- 4 C/m, se suspende
energía gastada en el proceso es también cero?. una carga puntual de magnitud Q = 10- 5 C de masa m
Calcúlela. = 0,1g, amarrándola con un hilo de longitud L = 1m a
un punto de la barra. Determine la tensión en el hilo y
9. Ocho cargas puntuales de magnitud q se el ángulo que forma con la vertical en la posición de
encuentran en los vértices de un cubo de arista a. equilibrio.

14. Una barra fina infinita, con densidad lineal de


carga λ, se dobla en forma de horquilla como se
a) Determine la fuerza eléctrica que actúa sobre la muestra en la figura. Determine el campo eléctrico en
carga en el origen, producida por las otras y el punto O.
b) la magnitud de la fuerza sobre cualquier carga.
Respuesta.
→ kq 2 ⎛
a) F = − 2 ⎜1 +
a ⎝
1
+
1 ⎞ˆ ˆ ˆ
⎟i + j+k
2 3 3⎠
( )
kq 2 ⎛ 1 1 ⎞
b) F = 2 3 ⎜1 + + ⎟ Respuesta. E = 0
a ⎝ 2 3 3⎠
15. Dos barras aisladoras delgadas se disponen como
10. En el problema anterior, calcule la energía que se se indica en la figura, una con densidad de carga ρ0 y
requiere para formar la mencionada distribución de la otra con ρ = 2ρ0.
cargas

11. Dos cargas puntuales están colocadas sobre el eje


x. Q1 = q en x = a y Q2 = −4q en x = - a . Encuentre
una expresión vectorial en coordenadas cartesianas
a) Calcular el campo eléctrico en el origen.
para la fuerza que actúa sobre una carga de prueba Q,
b) Determinar la fuerza que se ejercen las barras
ubicada en un punto cualquiera del plano XY.
sobre una carga q dispuesta sobre el eje x.
Encuentre las coordenadas (x, y) de todos los puntos

60
Electrostática Hugo Medina Guzmán

c) Encuentre el o los puntos en los cuales la fuerza


sobre q es nula.

16. En la figura la semicircunferencia yace en el


plano yz mientras la carga Q es una carga puntual
contenida en el eje z a la distancia a del origen. Tanto
Q como λ son positivos.
24. Un cilindro recto de radio R y altura L orientado
a lo largo del eje z, tiene una carga no uniforme
ρ ( z ) = ρ 0 + Az con referencia a un origen en el
centro del cilindro. ¿Cuál es la fuerza sobre una carga
q localizada en el centro del cilindro?

25. Un disco circular de radio R tiene una carga total


Q uniformemente distribuida en su superficie.
Calcule el campo eléctrico en un punto sobre el eje
del disco a una distancia z del plano de dicho disco.
a) Encontrar una expresión para el campo eléctrico Respuesta.
sobre el eje x debido a ambas cargas. ⎛z ⎞
→ σ ⎜ − z ⎟ zˆ
b) ¿Qué relación debe existir entre Q y la carga total E= ⎜z ⎟
de la semicircunferencia para que el campo eléctrico 2ε 0 ⎝ R + z2
2

en el origen sea nulo?
26. La figura muestra tres cargas que se mantienen
17. Un anillo metálico de radio a tiene una carga total
fijas en el plano xy.
Q distribuida uniformemente en su perímetro. Una
a) Halle, en cartesianas, la fuerza eléctrica neta sobre
carga puntual q se trae desde el infinito y se coloca en
q3 debida a las otras dos cargas.
un punto a distancia d sobre el eje perpendicular al
b) Evalúe el resultado anterior para el caso q1 = 25
plano del anillo y que pasa por su centro. Determine
mC, q2 = - 16 mC, q3 = 5mC, a = 3 m y b = 4 m.
el trabajo realizado por el campo eléctrico.

18. Un anillo aislador de radio a tiene una carga total


Q distribuida uniformemente en su perímetro.
a) Una carga puntual q se trae desde el infinito y se
coloca en un punto a distancia d sobre el eje
perpendicular al plano del anillo y que pasa por su
centro. Determine el trabajo realizado por el campo
eléctrico.
b) Si la carga puntual hubiese estado fija y el anillo se
trae desde infinito a la posición descrita antes, ¿Cuál
sería su respuesta?. Respuesta.
a)
19. Dos partículas, cada una con masa m y carga q, →
q3 ⎧⎪⎡ q1a ⎤
F3 = ⎨⎢ 2 ⎥iˆ +
están suspendidas por cuerdas de longitud l desde
un punto común. ¿Cuál es el ángulo θ que forman (
4πε 0 ⎪⎩⎢⎣ a + b 2 ⎥⎦
3 2
)
q ⎤ ⎫⎪
las cuerdas entre sí? ⎡ qb
⎢ 2 1 2 3 2 + 22 ⎥ ˆj ⎬
20. En los vértices de un cuadrado de 10 cm. de lado
se colocan cargas l x l0-9 coulombios. ¿Cuál es la
(
⎣⎢ a + b )
b ⎦⎥ ⎪⎭

fuerza sobre cada carga? (
b) F3 = 9 3iˆ − ˆj 10 N
3
)
21. Comparar la fuerza gravitacional y la fuerza
eléctrica entre dos electrones colocados muy cerca. 27. El sistema de la figura se encuentra en reposo.
(e = l,6 x l0-19 C, m = 9,1 x 10-31 kg) Las dos partículas tienen la misma carga q1 = q2 = q y
se encuentran a la misma altura. La carga q1 tiene
22. Se tienen tres cargas iguales a q en los vértices de masa m y cuelga de un hilo tenso que forma un
un triángulo equilátero, ¿qué carga y en qué posición ángulo θ con la vertical. La carga q2 se mantiene fija
debe colocarse para que el sistema esté en equilibrio? en su lugar por medio de un soporte unido a una
23. Se tiene una semicircunferencia de radio R con masa.
una distribución de carga lineal λ = λ0 cos θ . Halle la longitud L del hilo.
Calcular la carga total.

61
Electrostática Hugo Medina Guzmán

1 Nm 2
Aproxime ≈ 9 × 10 9 y halle la
4πε 0 C2
magnitud de la fuerza sobre la carga en el origen.
Respuesta.
⎛ 1 ⎞
F = ⎜⎜ ⎟(q1 q 2 + q1 q3 ) = 104 N

⎝ 4πε 0 r
2

Respuesta.
30. Un electrón y dos protones se colocan en los tres
q 2 cos θ diferentes arreglos mostrados en la figura. Llamemos
L= F al módulo de la fuerza eléctrica total que los
4πε 0 sen 3θ protones ejercen sobre el electrón. Compare F en los
tres casos y ordene de mayor a menor.
28. La figura muestra un sistema de tres partículas
cargadas en un plano xy horizontal. Las partículas 1 y
2 se mantienen fijas y la 3, de masa m, se está
moviendo a lo largo del eje x bajo la influencia de las
fuerzas eléctricas que le aplican las otras dos. Llame
x(t) a la posición de q3 respecto al origen.
a. Halle la fuerza neta sobre q3 y su ecuación de
movimiento.
b. ¿Es el punto x = 0 un punto de equilibrio? ¿Por
qué?
c. Halle el período del movimiento de q3 si Respuesta.
inicialmente se suelta desde el reposo en un punto En el caso B > que en el caso C > que en el caso A
x(0 ) << a .
31. En el sistema de la figura las tres partículas
poseen la misma carga, q1 = q2 = q3 = q. Las
partículas 1 y 2 se mantienen fijas y la 3, de masa m,
se está moviendo a lo largo del eje x bajo la
influencia de las fuerzas eléctricas que le aplican las
otras dos. Llamaremos x(t) a la posición de q3 respecto
al origen O.
a. Halle la fuerza neta sobre q3 y su ecuación de
movimiento.
b. ¿Es el punto x = 0 un punto de equilibrio? ¿Por
qué?
c. Suponga que x (t ) << L y halle el período de las
pequeñas oscilaciones de la partícula 3 en torno al
Respuesta. origen.
→ q 2 xiˆ
a) F3 = − ,
(
2πε 0 a 2 + x 2 )
32

•• q2x Respuesta.
x+ =0
2πε 0 m(a + x )
2 32 q2 ⎡ 1 1 ⎤
2 →
a) F3 = − ⎢ − iˆ ,
→ x =0 2πε 0 ⎣ (L + x ) 2
(L − x )2 ⎥⎦
b) Es un punto de equilibrio porque F3 ( x ) =0 •• q2 ⎡ 1 1 ⎤
x+ ⎢ − = 0,
2πε 0 ma 3 2πε 0 m ⎣ (L + x ) 2
(L − x )2 ⎥⎦
c) T = 2π → x =0
q2
b) Es un punto de equilibrio porque F3 ( x ) = 0.
29. Una carga puntual de 5 μC se coloca en el πε 0 mL3
origen, otra carga puntual de 8 μC se coloca a 3 m c) T = 2π
q2
del origen sobre el eje x, y una tercera carga de 12
μC se coloca a 3 m del origen sobre el eje y.
62
Electrostática Hugo Medina Guzmán

32. Dos pequeños cuerpos con cargas q1 y q2 del → qQR


a) F = rˆ , Signo(Q) = -
( )
mismo signo están en reposo suspendidos mediante 32
hilos de longitud L. 2πε 0 R 2 + a 2
Los hilos, como se muestra en la figura, forman un Signo(q).
ángulo θ con la vertical y sus puntos de sujeción al
techo están separados una distancia d.
qQ R 2
b) v =
a. Dibuje el diagrama de fuerzas de cada cuerpo.
b. Escriba en componentes (vertical y horizontal) la
(
2πε 0 M R 2 + a 2 )
32

segunda ley de Newton para cada carga. 2


c. Determine las masas de los dos pequeños cuerpos. c) R =
2

34. Un haz constituido por neutrones, electrones y


protones, todos con igual velocidad, penetra en un
campo vertical uniforme y se divide en otros tres
haces A. B y C como indica la figura. Desprecie el
efecto de la gravedad e indique a cuál tipo de
partículas corresponde cada haz. Indique también qué
se puede decir acerca del sentido del vector campo
eléctrico.
Respuesta.
b) T1 cos θ = mg ,
q1 q 2
T1senθ =
4πε 0 (d + aLsenθ )
2

para q2 se cambia T1 → T2 y m1 → m2.


q1 q 2
c) m1 = m 2 =
4πε 0 tan θ (d + aLsenθ )
2

33. Dos partículas, fijas y de carga q cada una se


encuentran separadas una distancia 2a. Una tercera
partícula de masa M y carga Q está sometida Respuesta.
solamente a la fuerza electrostática de las partículas Los neutrones forman el haz B los electrones el haz C
fijas, ella gira en una órbita circular de radio R; la y los protones el haz C.
órbita es perpendicular a la línea que une las dos El sentido del campo eléctrico es hacia arriba
partículas fijas y tiene su centro en el punto medio
entre ellas. Ver figura. 35. En los vértices de un cuadrado de lado 2L se fijan
a. Calcule la fuerza electrostática sobre Q. Indique 4 partículas cuyas cargas se señalan en el dibujo.
qué signo debe tener Q. a) Calcule el campo eléctrico del sistema en un punto
b. Halle la rapidez de Q. z sobre el eje perpendicular al cuadrado y que pasa
c. Determine para qué valor de R es máximo el por su centro.
módulo de la fuerza sobre Q. Ayuda: Calcule por separado la contribución de cada
par de cargas conectadas por una diagonal.

Respuesta. b) En el punto z se coloca una partícula de masa m y


Tomaremos el vector unitario r̂ como aquél que carga 2q, inicialmente en reposo. Suponga que la
apunta del centro O de la circunferencia hacia Q. gravedad no es relevante en este problema.
b1) Halle la ecuación de movimiento de la partícula
de masa m.

63
Electrostática Hugo Medina Guzmán

b2) Suponga que z < L y calcule el período de las


pequeñas oscilaciones que describe la partícula.
Respuesta.
Tomaremos el vector unitario k̂ paralelo al eje z y
apuntando hacia arriba.
→ qz
a) E = − kˆ .
πε 0 (z + 2 L2 )
2 32

•• 2q 2 z
b1) z + .
πε 0 m(z 2 + 2 L2 )
32

c) Para hallar el campo eléctrico producido por un


π 2ε 0 mL3 hilo recto de longitud infinita tomemos el límite
b2) T = 2π
q2 L → ∞ en b1 y en b2. Explique por qué son distintos
los dos límites. ¿Cuál se debe tomar?
Respuesta.
36. La figura muestra una barra delgada de longitud L
a)
y carga Q uniformemente distribuida. El punto P está
en la misma línea de la barra y a una distancia h del → Q ⎡⎛ L1 L2 ⎞
extremo de la misma. E= ⎢⎜ + ⎟iˆ
a) Halle el campo eléctrico producido por la barra en ⎜
4πε 0 lL ⎢ L1 + l 2 2
L2 + l 2
2 ⎟
⎣⎝ ⎠
el punto P y la fuerza eléctrica que le aplicaría a una
carga puntual q que se colocara allí. ⎛ l l ⎞ ⎤
+⎜ − ⎟kˆ ⎥
⎜ L2 + l 2 2 ⎟
L2 + l ⎠ ⎥⎦
2
⎝ 1
→ Q ⎡⎛ 1 ⎞
b1) E1 = ⎢⎜⎜ 2 ⎟iˆ
4πε 0 l ⎣⎢⎝ L + l ⎟⎠ 2

b) La figura muestra dos barras delgadas, colineales,


separadas una distancia D y de longitudes L1 y L2. Sus
→ λ ⎡⎛ L ⎞
b2) E 2 = ⎢⎜⎜ 2 ⎟iˆ
cargas Q1 y Q2 están uniformemente distribuidas. 2πε 0 l ⎣⎢⎝ L + 4l ⎟⎠ 2
Aproveche el resultado de la parte a) y halle la fuerza

eléctrica entre las dos barras.
c) lim E1 = 0 (corresponde a una carga Q finita
L →∞
diluida en un hilo infinito)

λ ˆ
lim E2 = i (Corresponde a un hilo infinito
L →∞ 2πε 0 l
con densidad de carga finita)
Respuesta.
→ Q⎛1 1 ⎞ 38. Un hilo circular de radio R y carga Q
a) E = ⎜ − ⎟iˆ ,
4πε 0 L ⎝ h h + L ⎠ uniformemente distribuida está en el plano xy y su
centro coincide con el origen.
→ qQ ⎛ 1 1 ⎞ a. Halle el campo eléctrico que produce en el punto
F= ⎜ − ⎟ iˆ de coordenadas cartesianas (0, 0, z).
4πε 0 L ⎝ h h + L ⎠
b. Estudie el comportamiento del campo encontrado
b) [Fuerza sobre Q2] = en la parte a cuando z > R.
→ Q1Q2 ⎡ (D + L1 )(D + L2 ) ⎤ Respuesta.
F 12 = ln ⎢ ⎥ iˆ
4πε 0 L1 L2 ⎣ D(D + L1 + L2 ) ⎦ Qz

a) E = kˆ
(
4πε 0 R 2 + z 2
32
)
37. El hilo recto de la figura tiene longitud L = L1 +L2 → Q
y carga Q uniformemente distribuida. b) E ≈ kˆ (desde lejos se ve como urna
a) Halle el campo eléctrico que produce el hilo en el 4πε 0 z 2
punto P. carga puntual).
b1) Halle el valor del campo eléctrico para puntos
tales que L1 = L2 = L/2. 39. La figura muestra un hilo cargado abc con
b2) Reescriba el resultado de b1 de forma tal que no densidad longitudinal de carga λ . El tramo bc es un
aparezca Q y aparezca λ (la densidad longitudinal cuarto de una circunferencia de radio R y centro en O.
de carga del hilo).

64
Electrostática Hugo Medina Guzmán

El tramo ab es recto, de longitud L = 4R/3 y b) Determinar el lugar donde pondría una segunda
perpendicular a la línea Ob. carga q = − Q/2 de modo que la fuerza neta sobre ella
sea nula.

42. Determine la fuerza entre un disco de radio R


cargado con densidad uniforme de carga σ y una
varilla de largo L y densidad lineal λ colocada en el
eje del disco, a una distancia b del mismo.
a) Calcule el campo eléctrico que producen en el Respuesta.
punto O cada uno de los dos tramos ab y bc. → σλ
b) Halle el campo eléctrico neto que produce todo el F= ⎡ L + R 2 + b 2 − R 2 + (b + L )2 ⎤ kˆ
hilo en el punto O. 2ε 0 ⎢⎣ ⎥⎦
Respuesta.
a) 43. Una esfera uniformemente cargada de radio R
λ
(iˆ + 2 ˆj ) , E bc = λ (− iˆ + ˆj )
→ → esta centrada en el origen con una carga Q. Determine
E ab la fuerza resultante que actúa sobre una línea
10πε 0 R 4πε 0 R uniformemente cargada, orientada radialmente y con
→ 3λ
b) E abc (− iˆ + 3 ˆj ) una carga total q con sus extremos en r = R y r = R +
d.
20πε 0 R
Respuesta.
→ Qλd
40. La figura muestra un hilo cargado con densidad F= rˆ
longitudinal de carga). El tramo be es la mitad de una 4πε 0 R(R + d )
circunferencia de radio R y centro en O. El tramo ab
es recto, de longitud L = 2R y paralelo a la línea bO 44. Un cilindro circular recto de radio R y altura L
esta orientado a lo largo del eje z y tiene una densidad
de carga volumétrica no uniforme dada por ρ(r) =
ρ0+βr, donde r se mide respecto del eje del cilindro.
Calcule el campo eléctrico producido por esta
distribución sobre el eje del cilindro.
a) Calcule el campo eléctrico que producen en el →
punto O cada uno de los dos tramos ab y bc. Respuesta. E = 0
b) Halle la magnitud del campo eléctrico neto que
produce todo el hilo en el punto O y el ángulo que 45. En la pregunta anterior suponga que la
forma con la dirección. distribución de carga es ρ(z) = ρo +βz donde z se
Respuesta. mide respecto de la base del cilindro. Calcule el
→ λ → λ campo eléctrico producido por esta distribución sobre
a) E ab = iˆ , E bc = ˆj el eje del cilindro.
6πε 0 R 2πε 0 R
λ 46. Una carga lineal de densidad λ con la forma de un
b) E ab = , ángulo arc tg (3). cuadrado de lado L se encuentra en el plano yz con su
6πε 0 R centro en el origen. Determine el campo eléctrico
sobre el eje x a una distancia arbitraria x, y compare
41. Dos discos de radio R se ubican como se muestra el resultado con el del campo que existe en el eje de
en la figura y una carga q = −Q/2 es puesta en el una anillo cargado de radio r = L/2, con un centro en
punto P. el origen y con la misma carga total.
El disco izquierdo tiene una carga Q (> 0) y el Respuesta.
derecho − Q, ambas uniformemente distribuidas. →
λL x 1
E cuadrado ( x ) = iˆ
πε 0 (x + L 4) x + L2 2
2 2
,
2


λL x
E anillo ( x ) = iˆ
4ε 0 (x + L2 4)3 2
2

47. Una carga puntual q está situada en el centro de


un cubo cuya arista tiene una longitud d.

⎛ → →⎞
a) Calcular la fuerza que la carga q = − Q/2, ejerce
a) ¿Cuál es el valor del flujo de E ⎜

∫ ∫ E⋅ d S ⎟⎠ en
sobre cada uno de los planos. una cara del cubo?.

65
Electrostática Hugo Medina Guzmán

b) La carga se traslada a un vértice del cubo. ¿Cuál es


el valor del flujo de a través de cada una de las caras
del cubo?.

48. Dos láminas planas, paralelas e infinitas, cargadas


con una densidad σ1 = 4 µC y σ2 = 6 µC, distan 2 cm.
Estudiar el campo eléctrico de este sistema.
Supongamos que dichos planos en vez de estar
paralelos se cortan perpendicularmente. Demostrar
que la magnitud del campo es la misma en las cuatro
regiones que ellos determinan en el espacio.

49. Calcule el campo eléctrico producido por una


superficie circular de radio R con distribución de
carga σ a lo largo del eje de simetría perpendicular 54. Una superficie gaussiana esférica (1) encierra y
al plano que la contiene y determine su valor en el se centra en una carga puntual + q. Una segunda
límite R >> z. Compare su resultado con el valor que superficie gaussiana esférica (2) del mismo tamaño
se obtiene utilizando la ley de Gauss en el caso de un también encierra la carga pero no se centra en ella.
plano infinito. ¿Cómo es comparado al flujo eléctrico a través de
superficie 1, el flujo a través de la superficie 2?
50. Repita el cálculo anterior para el caso en que la
superficie fuese un cuadrado de lado a y determine el
valor límite cuando a >> z

51. Un cilindro macizo, muy largo, de radio a, tiene


una carga distribuida con una densidad de carga
ρ = − Ar , donde A es una constante positiva.
Determine el valor del campo eléctrico en el interior
y el exterior cercano al cilindro, en puntos lejanos de
sus extremos.
Respuesta. iguales
52. La figura muestra una esfera aisladora de radio R
con densidad de carga ρ = constante, la cual tiene 55. Dos cargas del punto, + q y - q, se arregla según
lo demostrado. ¿A través de qué superficie(s) cerrada
una burbuja esférica vacía en su interior, de radio r, el flujo eléctrico neto es igual a cero?
situada a la distancia a del centro. Calcule el campo
eléctrico:

a) en el centro de la burbuja,
b) sobre una línea que contenga los centros de la Respuesta. La superficie C y la superficie D
esfera y la burbuja, dentro y fuera de la esfera y
c) sobre un eje perpendicular a la línea que une los
centros de la esfera y la burbuja, dentro y fuera de la 56. Se tiene una superficie cuadrada de lado a con
esfera. carga σ C/m2. ¿Cuál es el valor del Campo eléctrico
en puntos situados en el eje que pasa por el centro del
53. La figura representa un volumen aislante de cuadrado.
espesor d = 0,5m limitado por planos infinitos
(perpendiculares al eje x) (en corte). La densidad de 57. Se tienen dos alambres infinitos separados por
carga volumétrica es constante, ρ = 10 −6 C/m. una distancia 2d, con cargas iguales y opuestas λ y
a) Determine el campo eléctrico a ambos lados del − λ , ¿cuál es la fuerza entre ellos?
dieléctrico.
b) ¿porqué E = 0 en el centro del dieléctrico? 58. Una esfera no conductora de masa m y cargada,
c) Determine el campo eléctrico en el interior del cuelga por medio de un hilo de longitud l , se
dieléctrico como función de x.

66
Electrostática Hugo Medina Guzmán

→ 67. Una nube de carga con simetría esférica origina


encuentra bajo la acción de un campo eléctrico E . un campo eléctrico dirigido radialmente hacia afuera
Calcular la carga da la esfera si el ángulo entre la desde el centro de la nube y es el siguiente
vertical (perpendiculares al Campo) y el hilo es θ .
E (r ) =
A
2
(1 − e − ar )
59. Se tiene una recta infinita con densidad de carga r
lineal λ y una carga q situada a una distancia Hallar la densidad de carga ρ (r ) en la nube.
perpendicular d, ¿cuál es el campo eléctrico en un
punto entre ellos sobre la distancia d? 68. Dos cáscaras esféricas concéntricas de radios a y
b, aisladas una de la otra, la interior tiene una Carga
60. Un casquete esférico definido por el ángulo Q y la exterior está conectada a tierra. ¿Cuál es el
sólido π y radio R, tiene una carga superficial de campo eléctrico entre ellas?
densidad σ . ¿Cuál es el valor del Campo eléctrico ¿Qué pasa cuando la esfera exterior no está conectada
en el centro de curvatura del casquete? a tierra?

61. Se tiene una Carga q en el centro de un cilindro 69. Dos cáscaras esféricas concéntricas de radios a y
de longitud 2l y radio R, encontrar el flujo a través b tienen Cargas Q y − Q 2 respectivamente. ¿Cuál
de cada una de las caras y el flujo total. es el Campo eléctrico para todo punto?
→ 70. ¿Cuál es la densidad de carga de la superficie
62. Dado un campo eléctrico E = 2 xiˆ + yiˆ N/C. infinita que sostiene flotante una partícula con masa
Calcular el flujo eléctrico que atraviesa cada una de m y carga q.
las caras del cubo, de lado 1 m. y esté en el primer
octante 71. Hallar la fuerza por unidad de área sobre un
plano infinito con carga uniforme debido a otro plano
infinito con carga uniforme paralelo al primero.

72. Se tiene un cilindro infinito de radio b y un vacío


cilíndrico da radio a, tal como se muestra en la figura.
Si la carga por unidad de volumen es ρ ¿Cuánto
vale al Campo eléctrico en todos los puntos?

63. Calcular el flujo eléctrico producido por una


lámina con carga superficial de densidad σ que
atraviesa una superficie esférica de radio R en cada
una de las posiciones mostradas en la figura.

73. Bajo ciertas circunstancias la intensidad del


campo eléctrico en la atmósfera tiene un valor E1 en
64. Encontrar el Campo eléctrico de un cilindro la superficie y un valor E 2 a una altura h sobre la
infinito de radio R y con densidad de carga ρ superficie. En ambos casos el sentido del vector es
coulombios/m3 para puntos r > R, r = E y r < E. hacia la Tierra. Determinar el valor promedio de la
densidad de carga promedio en la atmósfera debajo
65. Se tiene una esfera de radio R y con densidad de de h.
ρ (r ) = c r . ¿A qué es igual el campo eléctrico Calcular para E1 = 300 V/m, E 2 = 20 V/m y h =
dentro de la esfera (r <R)? 1400 m.

66. Una lámina infinita de espesor a contiene une 74. Un haz de electrones que se mueven con una
densidad de carga uniforme ρ . Encontrar el campo velocidad de 2x106 m/s pasa entre dos placas
horizontales que tienen densidad de carga 1xl09 C/m2
eléctrico para todo punto. y -3xl09 C/m2. Suponer que el campo producido es el
mismo a que si fueran placas infinitas. El haz ingresa
paralelo a las placas y deja el campo después de pasar

67
Electrostática Hugo Medina Guzmán

1 cm., encontrar la deflexión resultante de los


electrones.

75. Se tienen dos hilos aisladores muy largos, uno en


la dirección del eje x con una densidad de carga λ1 y
el otro, en la dirección del eje y con una densidad de
carga λ 2 . Hallar el potencial V y el campo eléctrico
a) Determine el campo eléctrico total sobre la carga
en cualquier punto del plano xy y mostrar que

Q.
E = −∇V . b) Calcule la fuerza que ejerce Q sobre cada uno de
los trozos de hilo.
c) Determine la energía potencial de la carga Q.
76. Un globo esférico de radio R tiene una carga
superficial con densidad σ .
81. Un volumen esférico de radio a está lleno con
a) Calcule el campo eléctrico en el interior y el
carga de densidad uniforme ρ . Calcular la energía
exterior del globo.
b) Determine la energía eléctrica que se requiere para potencial U de esta distribución esférica de carga, es
cargar el globo trayendo las cargas desde el infinito. decir, el trabajo requerido para formarla.
c) Calcule el trabajo realizado por el campo eléctrico Respuesta.
generado por la carga en el globo al inflarlo entre R y 3Q 2
R + ∆R. U= , donde Q es la carga total de la esfera.
5a
77. Una esfera aisladora de radio a y densidad de
82. Un cilindro macizo, muy largo, de radio a, tiene
carga dada por ρ = ρ 0 e − r . Calcular el campo una carga distribuida ρ = − Ar , donde A es una
eléctrico en el interior y exterior de la esfera. constante positiva. Determine el valor del campo
eléctrico y el potencial en el interior y el exterior
78. Considere la misma esfera anterior, pero esta vez cercano al cilindro, en puntos lejanos a sus extremos.
rodeada por un casquete esférico conductor de radio
interior b > a y espesor d. El casquete exterior tiene 83. Un plano conductor tiene una carga + Q y a cada
carga nula. Calcule el campo eléctrico y el potencial
respecto de infinito, lado de éste, a las distancias x1 y x 2 , se colocan,
a) entre las esferas, paralelas, placas infinitas conductoras con carga total
b) en el interior de la esfera conductora y nula.
c) para un radio r > b + d. Encontrar la diferencia de potencial entre las caras
internas y entre las externas de las placas.
79. Se tiene una esfera aisladora con densidad de
84. En una región del espacio, el potencial eléctrico
carga variable de la forma ρ = ρ 0e −r r y radio R
está dado por V (x, y) = Axy siendo A una constante.
limitada exteriormente por una esfera conductora de Determine la fuerza ejercida sobre una carga puntual
radio interior q ubicada en un punto de coordenadas (x, y). Calcule
R y exterior 2R. En la esfera conductora hay una además el trabajo que realiza el campo eléctrico sobre
carga neta tal que el potencial exterior (r > 2R) es q al moverse la carga desde el punto (0, 0) al punto
constante. (x, y) en una línea recta.

85. Un disco circular de radio a tiene una densidad


superficial de carga uniforme σ . El disco se
encuentra en el plano xy con su centro coincidiendo
en el centro del sistema de coordenadas.
a) Obtenga una fórmula para el campo eléctrico en el
eje del disco en función de z.
Determine:
b) Aproxime el resultado anterior a primer orden para
a) La carga total en la esfera aisladora.
b) el campo eléctrico en el exterior (r < 2R). a
→ 0 y pruebe que el resultado coincide con el
c) la diferencia de potencial entre r = 3R/2 (esfera z
conductora) y el centro de la esfera aisladora campo de una carga puntual.
(considere el potencial cero ( V = 0) en r = ∞). c) Determine la posición de un punto sobre el eje z,
d) la densidad de carga en la superficie exterior de la mas allá del cual, el disco puede considerarse como
esfera conductora. →
una carga puntual con un error en el cálculo de E
80. Tres trozos de hilo cargado con densidad de carga menor al 1%.
λ se disponen como se indica en la figura.

68
Electrostática Hugo Medina Guzmán

86. Calcular el campo eléctrico de un cascarón superpuestas. Probar que el campo eléctrico en la
esférico de radio r con densidad de carga superficial región de intersección es constante y hallar su valor.
uniforme σ en todo punto del espacio.
a) Usando la ley de Coulomb. 94.Una distribución estática de carga produce un
b) Usando la ley de Gauss. → e − br
campo eléctrico radial dado por: E = A rˆ ,
87. Suponga que en lugar de la ley de fuerza de r
Coulomb, se encuentra experimentalmente que la donde A y b son constantes.
fuerza entre dos cargas puntuales q1 y q2 está dada a) Hallar la densidad volumétrica de carga de la

por, F12 =
(
q1 q 2 1 − r12
rˆ12
) distribución.
b) Halle la carga total de la distribución
4πε 0 r122
95. Considere un disco circular de radio a y densidad
a) Halle el campo eléctrico alrededor de una carga
de carga uniforme.
puntual correspondiente a esta fuerza.
a) Calcule el potencial en un punto cualquiera del eje
b) Pruebe que el campo es irrotacional.

y.
c) Halle el flujo de E a través de una superficie b) Determine la energía requerida para traer una carga
esférica centrada en la carga. desde el infinito hasta ese punto.
Compare con el caso de un campo coulombiano
96. Calcule el potencial respecto del infinito en el
88. Una carga q se encuentra en el vértice A del cubo centro de un cuadrado de lado b en el cual se tiene
ABCDEFGH. Calcular el flujo de campo eléctrico a una distribución uniforme de carga superficial σ .
través de la cara FECD.
89. Calcular el campo eléctrico de una esfera 97. En una región de espacio existe un campo
uniformemente cargada de carga total Q. eléctrico que se deriva del potencial V (x, y, z) = xyz -
a) Usando la ley de Coulomb. 3x - 2y - z.
b) Usando la ley de Gauss. Determine el trabajo que realiza el campo eléctrico al
llevarse una carga de 2 μC desde el punto (0, 0, 0)
90. Considere dada una distribución de carga con al punto (1, 1, 1) en forma cuasiestática (energía
simetría esférica. En este caso ρ es ρ ( r ' ) siendo r’ cinética despreciable).
la distancia al centro de la distribución.
98. Considere una esfera no conductora de radio R
a) Encuentre una expresión general para el campo
que tiene una carga total Q repartida uniformemente
eléctrico en función de ρ ( r ' ) , en un punto cualquiera en su volumen. Determine el potencial eléctrico en
de una esfera de radio r y centro en el centro de la todas partes.
distribución.
b) Aplique el resultado anterior al cálculo del campo 99. Determine el trabajo que realiza el campo
producido por una densidad de carga dada por eléctrico al traer una carga puntual Q desde una
ε 0 Ab distancia 2d hasta una distancia d de un hilo recto
ρ (r ') = − e −br ' + 4πε 0 Aδ (r ' ) . infinito que tiene una carga uniforme λ C/m.
r '2
c) Un cascarrón esférico tiene una densidad de carga 100. Se tienen dos esferas metálicas aisladas de radio
ρ = k r2 en la región a ≤ r ≤ b y cero en otro r1 = 0,10 m y r2 = 0, 20m, inicialmente descargadas y
lado. Hallar el campo eléctrico en todo el espacio. alejadas entre sí. Si a la esfera de radio r1 se le coloca
una carga de 6 x 10−8 C y luego se conectan ambas
91. Un cable coaxial tiene una densidad de carga mediante un hilo conductor muy fino, calcule:
volumétrica uniforme ρ en el cilindro interior (r < a) La carga final de cada esfera.
b) El potencial final de las esferas.
a) y una densidad de carga superficial uniforme σ c) La energía almacenada en el campo eléctrico antes
en la superficie del cilindro exterior (r = b). La carga y después de la conexión.
total del cable es cero. Hallar el campo eléctrico en
todo el espacio. 101. Calcule la diferencia de potencial entre dos
esferas concéntricas de radios a y b (a < b) que tienen
92. Una plancha plana infinita de espesor 2d tiene una cargas q y Q respectivamente.
densidad de carga uniforme ρ . Hallar el campo Respuesta.
eléctrico en todo punto como función de la distancia y q ⎛1 1⎞
al plano de simetría de la plancha. Graficar. ΔV = ⎜ − ⎟
4πε 0 ⎝ b a ⎠
93. Dos esferas de radio R con densidades de carga
+ ρ y - ρ cuyos centros están separados por una 102. Calcule el campo eléctrico producido por una
distancia d tal que d < 2R están parcialmente distribución de carga tal que el potencial que produce

69
Electrostática Hugo Medina Guzmán

e − λr
q 107. Un anillo de radio R está formado por dos
esta dado por: V(r ) = . Encuentre la semicircunferencias con cargas q y -q.
4πε 0 r Encontrar al potencial y el campo eléctrico a lo largo
distribución de carga ρ = ρ (r ) . del eje del anillo.

Respuesta. 108. En un plano infinito se ha hecho un agujero

E(r ) =q
(1 + λr ) e − λr , ρ = − qλ2 e
− λr
circular de centro O y radio R, el plano tiene una
(r ) densidad de carga - σ . Calcular el potencial y el
4πε 0 4πr campo en un punto en la recta perpendicular al plano
en O.
103. Las superficies interior (r = a) y exterior (r = b)
de un cascaron esférico no conductor tienen la misma 109. Se tiene un cilindro infinito de radios a y b como
densidad de carga σ. La densidad de carga en el resto se muestra en la figura, con densidad de carga
del espacio es nula. Encuentre el campo eléctrico en volumétrica ρ . ¿Cual es el valor del potencial para
las zonas r < a, a < r < b, y r > b. Calcule el potencial todo punto?
electrostático en cada una de las regiones
mencionadas.

Respuesta. E ( r ) = E (r ) rˆ , donde E (r ) = 0 si

σ a2
r < a ¸ E (r ) = si a < r < b ;
ε 0r 2
σ
E (r ) = (a 2 + b 2 ) si b < r ,
ε 0r 2

σ 2
V( r ) =
ε 0r
(a + b 2 ) si r > b ; 110. Encontrar le forma de las superficies
equipotenciales para un segmento de recta de
σ 2
V( r ) =
ε 0r
(a + br ) si b > r > a y
longitud 2a, colocado entre z = -a y z = a, con una
Carga total q.

σ
V( r ) = (a + b ) si a > r . 111. Se acomodan linealmente tres cargas, la Carga -
ε0 2q se coloca en el origen, y las otras dos cargas + q se
colocan en (0, 2, 5) y (3, 0, - a) respectivamente.
104. Una burbuja de forma esférica tiene una carga Encontrar el potencial V(r ) para un punto en el
Q. La energía asociada a la tensión superficial de la espacio r >> a.
burbuja es proporcional a su superficie, es decir Umec A partir de este valor encontrar al Campo eléctrico.
= Sτ, en que S es el área de la burbuja y τ es una
constante. Calcule la energía total de la burbuja 112. Un protón con una energía de 100 eV es
(eléctrica y mecánica) como función de su radio y disparado perpendicularmente hacia un plano
grafíquela. Finalmente calcule el radio de equilibrio metálico muy grande con una densidad de carga
de la burbuja. superficial de 10- 7 C/m2.
¿Desde dónde se disparará el protón si casi alcanza al
→ → plano?
105. Calcúlese ∫ E ⋅ d l desde el origen e un punto
113. Una gota de aceite tiene una carga neta de tres
( )

x0, y0 en el plano x,v para el Campo E = A yiˆ − xˆj veces la carga del electrón y un radio de l0cm. ¿Cuál
a lo largo de tras trayectorias rectilíneas diferentes. es su velocidad terminal cuando cae entre dos placas
a) desde (0, 0) a (0, y0) y a (x0, y0) horizontales con una diferencia de potencial de 1000
b) desde (0,0) a (x0, 0) y a (x0, y0) voltios y separadas 2cm, la positiva es la de arriba?
c) desde (0,0) directamente a (x0, y0) Densidad del aceite 839 kg/m3
Densidad del aire 1,29 kg/m3
106. Calcular el trabajo hecho contra las fuerzas Viscosidad del aire 1,80 N.s/m2
electrostáticas en mover una carga de -1x10-10 C
desde 10 cm. debajo de una carga da 1x10-5 C hasta 114. La presión dentro y fuera de una pompa de jabón
una posición da 1 m. debajo de ella. En la posición es la misma. ¿A qué potencial ha sido cargada la
final el cuerpo permanece suspendido, las fuerzas pompa de jabón de radio 2 cm? La tensión
electrostáticas y gravitacionales son iguales y superficial de la solución de agua jabonosa es
opuestas. ¿Cuál es la masa del cuerpo? 0,0265N/m.

70
Electrostática Hugo Medina Guzmán

115. Una esfera conductora de radio R1 = 3 cm. lleva 121. Determinar la capacidad de uno varilla delgada
una carga negativa Q1 Una segunda esfera conductora de radio R y longitud L, siendo L >> R.
le radio interior R2 = 4 cm. y radio exterior R3 = 5 cm,
neutra, se colocan concentricamente. 122. Determinar la capacidad por unidad de longitud
a) Encontrar le repartición de las cargas. Si se entre dos alambres paralelos de radios R y distancia
establece contacto entre las esferas ¿cuál es la nueva entre ellos d, siendo d >> R.
repartición de las cargas?
b) Considerando que la esfera interior está a un 123. Un condensador de l μF se carga 0V y otro de
potencial V1 = 0 lo esfera externa caté a potencial V2
2 μF se cargad a 200V; estos se conectan de tal
= 10000 V. ¿Cuál es le repartición de las cargas y el
valor de estas? manera que la placa positiva de uno se conecta al
negativa del otro. Encontrar la diferencia de potencial
116. Si una carga q se coloca a una distancia d de un y la carga de cada condensador, así mismo la pérdida
plano conductor infinito mantenido a potencial cero. de energía que hubo.
Se puede determinar que el campo eléctrico resultante
en frente del plano es el mismo que si se tuviera una 124. En que forma cinco condensadores, cada uno de
carga - q a una distancia - d. Esta segunda carga se capacidad l μF , deben de conectarse para producir
conoce como carga imagen. Encontrar la densidad de una Capacidad total de 4/9 μF .
carga en el plano y verificar que la carga total en el
plano es -q.
125. Se tiene un condensador de placas paralelas de
117. Encontrar las cargas imagen cuando una carga q área de placas A y separación d, se le introduce una
está a iguales distancias de dos planos conductores a placa metálica paralela de espesor, calcular la
capacidad resultante.
potencial cero que se encuentran formando un ángulo
de 60°. Encontrar la fuerza sobre la Carga q.
126. La Capacidad de un condensador variable
118. Un condensador coaxial está formado por dos cambia 1inealmente de 50 a 36 μF durante una
cilíndricos conductores concéntricos de radios a y b rotación de 0° a 180°. Cuando se coloca a 75° una
respectivamente y largo L. Suponiendo que el espacio diferencia de potencial de 00 voltios se mantiene a
entre los conductores es vacío y que el cilindro través del condensador. ¿Cuál es la dirección y
interior se encuentra a potencial V = Vo y el exterior a magnitud del torque electrostático experimentado por
potencial el condensador?
V = 0 y que tanto a como b son mucho menores que
L, encuentre la capacidad del condensador coaxial. 127. Un Condensador de 5 μF se carga a una
Respuesta. diferencia de potencial de 12 voltios, se conecta en
2πε 0 L paralelo este condensador a otro inicialmente
C=
b descargado, la diferenciada potencial disminuye a 3
ln voltios, ¿cuál es la capacidad del segundo
a condensador?
119. Considere un sistema formado por dos 128. Se tiene dos condensadores planos de
conductores cilíndricos paralelos, muy largos, de capacidades C1 y C2 con las armaduras de igual
radio a, separados una distancia d >> a, como superficie A y con una separación d y d/2
muestra la figura. Entre los cilindros hay una respectivamente. Se conectan en paralelo y se cargan
diferencia de potencial V. Encuentre la capacidad por
unidad de longitud para el sistema de conductores. a una diferencia de potencial V0 .
a) Calcular la carga y la energía en cada condensador.
b) Si es posible mover una de 1as armaduras del
condensador C2, ésta se mueve hasta que la
separación entre placas es d, ¿cuál es la carga en cada
condensador, cuál es la nueva diferencia de potencial
Respuesta. y cuál es la energía almacenada?
πε 0 c) ¿Cuál es la fuerza que se ejerce sobre la armadura
C=
(
⎡ d + d 2 − 4a 2 ⎤
ln ⎢ ⎥
) móvil de C2 en el estado intermedio x entre d y d/2
¿cuál es el trabajo a realizar para llevar la armadura
móvil de la posición inicial a la final?
⎣⎢ 2a ⎦⎥
129. El Conductor exterior dz un cable coaxial tiene
120- Encontrar la capacidad de un disco conductor un radio fijo b. Determinar el radio a del conductor
muy delgado de radio a. interior, tal que para determinada diferencia de
potencial entre los conductores, la intensidad del
campo eléctrico en su superficie sea mínima.
71
Electrostática Hugo Medina Guzmán

Determinar la capacidad por unidad de longitud de tal


cable.

130. Una esfera metálica de un metro de radio tiene


un exceso de carga eléctrica de 10-9 C. Se conecta
mediante un alambre a una esfera inicialmente
descargada de 30 cm. de radio que está lejos de la
primera esfera (no tomar en cuenta la capacidad del
alambre).
a) ¿Cuál es la carga de cada una de las esferas?
b) ¿Cuál es la energía antes de unirlas y después de
unirlas? ¿Y si es diferente, qué pasa con la
diferencia?
134. Encontrar la capacidad equivalente entre a y b
131. Las armaduras de un condensador forman un en la figura, todos los condensadores tienen
ángulo encontrar la capacidad en función de las capacidad C.
dimensiones.

132. Un Condensador esférico está constituido de


dos armaduras de radios R1 y R2. La armadura 135. En la figura se muestra un electrómetro
exterior está formada por dos hemisferios iguales, esquematizado, este se usa para determinar
unidos a tierra por un conductor. La armadura interna diferencias de potencial. Consiste en una balanza
tiene una carga Q y está aislada. Calcular la fuerza a cuyo platillo izquierdo es un disco de área A,
que esté sometido cada uno de los hemisferios colocado a una distancia a de un plano horizontal
exteriores. formando un condensador, cuando se aplica una
diferencia de potencial V entre el disco y el plano
aparece sobre el disco una fuerza hacia abajo. ¿Cuál
es el va1or de la masa que debe ponerse en el otro
platillo para retomar el equilibrio?

133. Se tiene el sistema de condensadores mostrado 136. Una manera de determinar la capacidad de un
en la figura, inicialmente cargado a la diferencia de condensador desconocido es utilizar el dispositivo
potencial de 100 V, Si se cierra la llave S ¿cual es la mostrado en la figura donde E es un electrómetro. El
diferencia de potencial en cada condensador y la procedimiento es tener los condensadores conocidos
carga en cada uno de ellos? C1 y C2; buscar un condensador C3 para el cual la
lectura en el electrómetro sea cero. Encontrar la
expresión para calcular Cx.

72
Electrostática Hugo Medina Guzmán

5Q 6Q
a) q1 = , q 2 = q3 = .
11 11
5Q 3Q 2Q
b) V1 = , V2 = , V3 =
11C 11C 11C
139. El condensador C1 de la figura está cargado
como se muestra y los otros dos condensadores están
descargados. Se cierra el interruptor S y se espera
hasta que el sistema alcance el equilibrio.
137. Dos condensadores idénticos de área A y lado a a. Determine las cargas en las placas de los tres
y separación entre placas d, inicialmente descargados, condensadores que están conectadas al punto a.
se conectan en paralelo. Mediante una batería se b. Halle la diferencia de potencial Va - Vb.
aplica al sistema una diferencia de potencial V0.
Posteriormente se desconecta la batería, con lo cual
los condensadores en paralelo quedan cargados y
aislados. Se introduce en uno de los condensadores
una placa conductora de igual área y de espesor t,
como se muestra en la figura.

Respuesta.
C1Q C2Q
a) q1 = , q2 = ,
C1 + C 2 + C 3 C1 + C 2 + C 3
a) Calcule la energía almacenada en el sistema C3Q
cuando la placa de espesor t ha penetrado una q3 =
distancia x en el condensador. C1 + C 2 + C 3
b) Calcule la cantidad de carga transferida de un Q
condensador a otro como función de x, e indique en b) Va − Vb =
que sentido es la transferencia. C1 + C 2 + C 3
Respuesta.
2ε 0 a 2 a (d − t ) 140. Responder a la pregunta anterior cuando la placa
a) E = V02 que se introduce en el condensador de la izquierda
d 2a (d − t ) + at está hecha con un dieléctrico cuya constante es ε .
at Q
b) ΔQ A = − , donde Q es la
2a(d − t ) + at 2 141. Ente la placas del condensador de la figura, de
lados a y b, existe una diferencia de potencial
carga total en el sistema. constante V0.
138. La figura muestra un circuito con tres
condensadores y dos interruptores unidos por hilos
conductores. El condensador C1 tiene carga Q y los
otros dos están descargados. En cierto momento se
cierran los interruptores y se espera hasta que el
sistema alcance el equilibrio.
a. Encuentre la carga que se almacena en cada placa
de cada condensador. a) Calcular la carga Q(x) en las placas en función de la
b. Halle la diferencia de potencial entre las placas de distancia x cuando se introduce un dieléctrico de
cada condensador. constante ε y ancho b, como se indica.
b) Determine la variación de energía en el
condensador en función de x.
c) Determine la fuerza sobre el dieléctrico en función
de x.
Respuesta.

Respuesta. Llamaremos q1, q2 y q3 a las cargas en


las placas izquierdas de los condensadores C1, C2 y
C3 respectivamente.
73
Electrostática Hugo Medina Guzmán

V0 b
a) Q( x ) = [ε x + ε 0 (a − x )] , b)
d
Q2d →
U(x) = , c) F = −∇U ( x )
2b[εx + ε 0 (a − x )]

142. En un condensador de placas cuadradas paralelas


146. El dieléctrico en un Condensador de placas
de área A, se introducen dos dieléctricos de
paralelas no es uniforme, varía linealmente de un
constantes ε 1 y ε 2 que llenan totalmente el interior valor en la superficie de una placa a en la superficie
del condensador como se muestra en la figura. de la otra placa. Le distancia entre placas es d y su
Calcule la capacidad del condensador. área A. Determinar la capacidad de este condensador.

147. ¿Cuál es la capacidad del condensador mostrado


en la figura?

2a 2
Respuesta. C = ε 0 (ε1 + ε 2 ) , A = 4a
2

d
143. Un cubo dieléctrico de lado L tiene una
→ →
polarización radial dada por P = A r , donde A es
→ 148. Considérese el mismo condensador del problema
constante y r = xiˆ + yˆj + zkˆ . El origen de anterior, con los dieléctricos dispuestos
coordenadas está en el centro del cubo. Encontrar diagonalmente como se indica, el cual se conecta a
todas las densidades de carga y mostrar que la carga una batería que proporciona una tensión V.
total se hace cero. a) ¿Se distribuye en forma uniforme la carga en las
placas? Explique.
144. Dos placas conductoras paraleles están b) ¿Es el campo eléctrico y el campo de
separadas una distancia d y mantenidas a la diferencie desplazamiento perpendiculares a las placas?
Explique.
de potencial ΔV . Un dieléctrico de constante k y
c) Calcule la capacidad de este condensador y la
espesor uniforme t se ensarta entre las placas. distribución de carga en las placas.
Determinar los campos vectoriales E y D en el
dieléctrico y en el vacío.

145. Se tiene un condensador de placas paralelas


separadas una distancia d y área A. El espacio entre
las placas se llena con dos dieléctricos como se
muestra en la figura, calcular la capacidad del
condensador.

74
Corriente continua Hugo Medina Guzmán

CAPÍTULO 2. Corriente Continua


CORRIENTE ELECTRICA. Para la dirección de la corriente vamos a utilizar la
Hasta ahora hemos considerado solamente cargas en convención que toman le dirección de los portadores
reposo; ahora consideraremos cargas en movimiento. de cargas positivas, en dirección del campo eléctrico
Esto implica que trabajaremos con conductores, externo como se muestra en la figura siguiente.
porque en un conductor como ya dijimos los
portadores de carga tienen movimiento libre. Aunque
esta definición no solo es para conductores
convencionales como los metales, sino también a los
semiconductores, electrolitos, gases ionizados,
dieléctricos imperfectos y aún en el vacío en vecindad
de un cátodo los electrones emitidos
termoiónicamente.
Los portadores de carga pueden ser positivos o
negativos.
DENSIDAD DE CORRIENTE
Las cargas en movimiento constituyen el flujo de
Consideremos un conductor con un solo tipo de
corriente o simplemente corriente, definimos como
conductores con carga q, el número de estos
corriente media (I m ) a través de una superficie conductores por unidad de volumen es N, suponiendo
limitada (S) como la cantidad de carga que atraviesa que la velocidad de desplazamiento de estos
por unidad de tiempo. conductores es v d cuando está sujeto a un campo
externo, en un tiempo Δt todos los elementos
contenidos en el volumen AΔL = Av d Δt son
NAv d Δt y su carga ΔQ = qNAv d Δt y pasa a
través de la sección A en P como se muestra en la
figura.

Donde
Δq
Im =
Δt
La corriente instantánea es la corriente media
calculada en el límite cuando Δt → 0
Δq dq
I = lím = La corriente en el punto P es
Δt →o Δt dt ΔQ qNAv d Δt
La unidad de corriente en el sistema MKS es el I= = = qNAv d
Ampere o Amperio (A). Δt Δt
C La corriente por unidad de área es la Densidad de
A= Corriente.
s I
Como habíamos visto anteriormente el coulombio se J= = qNv d
define a partir de la corriente, hasta este punto todavía A
no podemos hacer una definición de el Ampere, lo Esta cantidad representa la rapidez del transporte de
cual haremos cuando estudiemos campos magnéticos. carga a través de una unidad de área normal a la
En un metal los portadores de carga son los dirección del flujo, es una cantidad vectora1 orientada
electrones, mientras que los iones positivos están fijos con v d
a posiciones regula res en la estructura, solamente los → →
electrones de valencia son los que participan en el J = qN v d
proceso de la conducción.
La unidad de densidad de corriente en el sistema
En un electrolito los portadores de carga son iones
MKS es A/m2.
positivos y negativos, como algunos iones se mueven →
con mayor rapidez que otros, la conducción de uno de Siendo J constante en toda la superficie A. La
los tipos de iones es la que predomina.
corriente que atraviesa A es
En una descarga de gas, los portadores de carga son →
tanto electrones como iones positivos, pero como los I = J ⋅ nˆ A
electrones tienen mayor movilidad la corriente s
Siendo n el vector unitario perpendicular al plano A.
prácticamente es llevada en su totalidad por los
electrones Si la densidad de corrientes no es uniforme.

1
Corriente continua Hugo Medina Guzmán

→ energía se convierte en calor en el blanco. Si en un


I = ∫ J ⋅ nˆ dS segundo la elevación de la temperatura del blanco es
A
t, el calor ganado por la blanco es mct.
Ejemplo 1. Un conductor da cobre conduce una 1 1
Por lo tanto mct = × nm p v 2 o
corriente de densidad 1000 A/m2. Asumiendo que 3 2
cada átomo de cobre contribuye con un electrón como
portador de carga. Calcular la velocidad de nm p v 2
t=
desplazamiento correspondiente a esta densidad de 6mc
corriente
Solución. =
(3,00 × 10 )[1,67 × 10 ](2 × 10 )
13 −27 7 2

La densidad de corriente es J = qNv d 6(1)(4,18 × 0,334 )


La velocidad de desplazamiento es = 2,39°C
J
vd = LA LEY DE OHM RESISTIVIDAD Y
qN RESISTENCIA
El valor de N (portadores por unidad de volumen) lo Cuando un conductor conduce una corriente, existe
encontramos como sigue: un campo eléctrico E en su interior. Se ha encontrado
experimentalmente para muchos conductores a
N 0 × ρ × portadores/átomo
N= temperatura constante que la densidad de corriente J
M es directamente proporcional a este campo. Siendo
átomo esta expresión LA LEY DE OHM.
N 0 (número de Avogadro) = 6,02x1023 N → →
mol J = gE
gramo Donde la constante g es la conductividad del material,
M (peso atómico) = 63,5
mol si esta conductividad no depende del campo eléctrico,
se dice que el material obedece la ley de Ohm.
g g
ρ (densidad) = 8,92 3
= 8,92 x 106 La ley de Ohm no es una ley fundamental de la
cm m3 naturaleza, como las leves de Newton, sino es una
portadores descripción empírica que compara gran cantidad de
=1 sustancias,
átomo El recíproco de la conductividad es la resistividad η .
Finalmente
A 1
1000 η=
vd = m 2
g
⎛ 23 átomo ⎞⎛ g ⎞ En el caso de un conductor definido, digamos un
⎜ 6,02 × 10 ⎟⎜ 8,92 3 ⎟
( −19
1,6 × 10 C ⎝) mol ⎠⎝
g
m ⎠ alambre, podemos escribir la ley de Ohm en función
63,5 de la caída de potencial
mol
= 0,739 x 10-7 m/s

Ejemplo 2. Protones de masa 1,67 x 1027 kg y que se


mueven con una velocidad 2 x l07 m/s chocan con
un blanco de masa 1 g y de capacidad calorífica
específica 0,334 cal/g-°C. La corriente de protones
corresponde a una corriente de 4,8 A. ¿Con que razón
la temperatura del blanco se eleva inicialmente, si una
Escogemos un segmento corto de longitud L, la
mitad de la energía de los protones se convierte en
diferencia de potencial entre la sección 1 y 2 es
calor?
Solución. (V1 − V2 ) = ΔV . (El potencial en 1 es mayor)
Cada protón tiene una carga de 1,60 x 10-19 C. Si la → →
corriente que fluye es 4,8 A, el número de los Como ΔV = ∫ E ⋅ d l , es el igual en todos los
protones que chocan el blanco en 1 s debe ser n, puntos del alambre
donde
ΔV
n(1,6 × 10 −19 C ) ΔV = EL ⇒ E =
= 4,8 × 10 −6 A L
1s → → →
⇒ n = 3.00 x 1013 protones. Pero J = g E y la corriente I = ∫ J ⋅ nˆ dS = JA
A
En un segundo la energía cinética total perdida por
1 gA L
De aquí I = gEA = ΔV ⇒ ΔV = I
los protones es n × m p v 2 , y un tercio de esta L gA
2

2
Corriente continua Hugo Medina Guzmán

Esto nos da una relación lineal entre I y ΔV , a) ¿Cuál es la resistencia del bloque?
equia1ente a la ley de Ohm. b) ¿Cuál es la corriente?
ΔV V L o) ¿Cuál es la densidad de corriente?
A la cantidad ó = , se la denomina Solución.
I I gA L
resistencia R del segmento de alambre a) Tenemos que R = η
A
V L ηL
R= = = L
I gA A Como A = a
2
⇒ R =η
a2
La unidad de resistencia es Voltio/Ampare,
b) Por la ley de Ohm ΔV = RI
denominada Ohm o con el símbolo Ω y su
representación esquemática se muestra en la siguiente ΔV ΔV a 2 ΔV
figura.
I= = =
R
η 2
L ηL
a
c) La densidad de corriente es

Como podemos ver la resistencia de un conductor


a 2 ΔV
depende de la longitud, de su sección transversal y de I ηL ΔV
la resistividad que es una propiedad intrínseca de J= = =
cada material.
A a 2
εL
La unidad de la resistividad es el Ohm-m (Ω − m ) y Ejemplo 4. Para encontrar cuánto alambre aislado se
para cualquier metal depende de la temperatura. A ha colocado en una bobina un técnico mide la
temperaturas normales la resistividad varía casi resistencia total del alambre, encontrando 5,18 Ω .
linealmente con la temperatura, suele referirse los Después corta una longitud de 200 cm y encuentra
valores a temperatura de 20°C. La relación entre que la resistencia de este es 0,35 Ω . ¿Cuál era
resistividad y temperatura es la siguiente inicialmente la longitud del alambre en la bobina?
η = η 20 º [1 + α (t − 20º C )] Solución.
La tabla que se muestra e continuación nos da valores La resistencia del alambre en la bobina es relacionada
de η y a para algunos materiales a 20°C. ρl
con su longitud por la fórmula R = . La longitud
RESISTIVIDAD Y COEFICIENTE DE A
TEMPERATURA cortada tiene la misma resistencia y sección
Material ρ a 20ºC α a 20ºC 1/ºC ρl 0
(Ω − m) transversal. Luego su resistencia es R0 =
Plata 1,6 x 10-8 0,0038
A
Cobre 1,69 x 10-8 0,00393 l 0 R0 5,18
∴ = ⇒ l 0 = 200 = 2960 cm.
Oro 2,44 x 10-8 0,0084 l R 0,35
Hierro 10 x 10-8 0,0050
Níquel 7,24 x 10-8 0,006 Ejemplo 5. Se tiene un conductor de resistividad q
Aluminio 2,83 x 10-8 0,0039 en forma de anillo plano con radios a y b y espesor e
Mercurio 95,8 x 10-8 0,00089 como se muestra en la figura.
Tungsteno 5,51 x 10-8 0,0045
Constantan 44 x 10-8 0,00002
Nicrón 100 x 10-8 0,0004
Carbón 3,5 x 10-5 - 0,0005
Germanio 0,46 - 0,048
Silicio 640,0 - 0,075
Vidrio 1010 - 1014 a) ¿Cuál es su resistencia para una corriente
Mica 1011 - 1015 perpendicular al plano?
Cuarzo 7,5 x 1017 b) ¿Cuál es su resistencia para una corriente radial
Azufre 1015 hacia afuera, de la circunferencia de radio a hacia la
Jebe duro 1013 - 1016 circunferencia de radio b?
Solución.
Ambar 5 x 1014
Madera 108 - 1011 L
a) R = η
A
Donde L = e y A = π (b − a )
Ejemplo 3. Un trozo de carbón tiene una longitud L y 2 2
una sección cuadrada de lado a se mantiene una
diferencia de potencial V entre los extremos de la e
De aquí R = η
dimensión L. (
π b − a2
2
)
3
Corriente continua Hugo Medina Guzmán

b) Consideremos un elemento de radio r y ancho dr Para producir una corriente es necesario una
como se muestra en la figura. diferencia de potencial, así mismo para poder cargar
un condensador necesitamos una diferencia de
potencial, en ambos casos estamos poniendo cargas
en movimiento, O sea que se realiza trabajo, para esto
se necesitan fuentes de energía, dispositivos que
convierten le energía química o mecánica en energía
eléctrica, estas son las pilas y baterías y los
generadores.
Vamos a utilizarla abreviación fem por “fuerza
electromotriz” que es un término que se refiere a
L energía y no a fuerzas) como símbolo tomamos ε
en la expresión R =η
A y su representación esquemática es como se muestra
En este caso la resistencia solo es un diferencial de en la figura siguiente.
resistencia (dR), la longitud dr, la sección transversal
ηdr
A(2π r )e , de aquí: dR =
2πer
La resistencia al flujo radial es
bη dr η
R = ∫ dR = ∫
b
= ln r a
2πe r 2πe
a

η b
Finalmente R= ln Consideremos una fem, por ejemplo una pila seca. En
2πe a ella hay un terminal de bajo potencial (− ) y un

Ejemplo 6. Un alambre de cobre se encuentra e la terminal de alto potencial (+ ) . La diferencia de


temperatura de 20°C y tiene una longitud de 10 potencial entre los terminales (ΔV ) , cuando se
metros y una sección de 0,005m2 si le aplica una emplea la pila para establecer una corriente I en un
diferencie de potencial de 200 voltios, calcular: circuito como el de la figura siguiente, las cargas
a) La resistencia del alambre a 120°C positivas son movidas por acción de fuerzas no
b) El campo eléctrico en el alambre. electrostáticas contra las fuerzas electrostáticas (las
c) La densidad de corriente en el alambre. fuerzas de coulomb ejercidas por las cargas en
Solución. reposo) desde el terminal de bajo potencial hacia el
L terminal de alto potencial.
a) R =η
A
Donde L = 10 m, A = 0,005 m2,
η = η 20 [1 + α (t − 20º C )]
η = 1,7 × 10 −8 [1 + 0,0039(120 − 20 )] = 2,36 x
10- 8 Ω − m
10 Si analizamos los portadores de carga del circuito de
De allí R = 2,36 × 10− 8 = 4,72 x 10- 5 Ω
0,005 la figura vemos que al pasar de un potencial menor a
b) Tenemos que uno mayor adquieren una energía que es equivalente
al trabajo que hace la fuente para llevarlos del
V 200 V
V =EL y E = = = 20 terminal negativo al terminal positivo, esto es
L 10 m dW = ε dq
I Suponiendo que los conductores son ideales
c) Como J =
A (resistencia cero), la energía perdida por los
Tenemos que portadores de carga al pasar por la resistencia es igual
a le energía adquirida en la fem Podemos notar que la
V = IR ⇒ unidad de fem es también el Voltio.
V 200
I= = = 42,37 x 105 A ¿Como se produce el flujo de electrones?
R 4,72 × 10− 5
42,37 × 105 A
Luego J = = 8,5 x108 m 2
0,005

FUERZA ELECTROMOTRIZ

4
Corriente continua Hugo Medina Guzmán

manera que dq = Idt sufre un cambio de energía


Potencial dada por
− dW = dq (V1 − V2 ) = dqV
Donde V = dq (V1 − V2 ) es le caída de potencial,
luego
dW
− dW = IdtV ⇒ − = IV
dt
Para entender el flujo de electrones, que es la Expresión que nos da la razón en que se pierde la
corriente eléctrica, hay que recordar las reglas de las energía, que viene a ser la Potencia perdida en el
cargas positiva y negativa. conductor.
Las cargas desiguales (+ y -) se atraen. Cargas P = IV
iguales (+ y +), o (- y -) se repelen. Si V esta en voltios e I en amperes, que son unidades
Los electrones de un átomo tienen cargas negativas y MKS, obtendremos la potencia en Joule/s = Vatios o
son atraídos por las cargas positivas. Los electrones Watts.
se mueven con facilidad de un átomo a otro. Como en un circuito ohmico V = IR, podemos escribir
Para crear una corriente eléctrica en un alambre de la expresión disipada como
cobre, se necesita una carga positiva en un extremo y
una carga negativa en el otro. P = I 2R .
A este resultado se le conoce como Ley de Joule,
Para crear y mantener la corriente eléctrica
(movimiento de electrones), deben darse dos mientras que el caso de calentadores en que se desea
condiciones indispensables: transformar energía eléctrica en energía térmica este
efecto es deseable, en otros casos esta disipación
1. Que haya una fuente de electrones o dispositivo
para su generación (generador), pila, batería, térmica es indeseable, por ejemplo en los alambres de
fotocélula, etc. conducción.
2. Que exista un camino, sin interrupción, en el A esta pérdida (I2R) se le denomina pérdida por
exterior del generador, por el cual, circulen los calentamiento de Joule, a fin de reducir esta pérdida
electrones. A este Camino se le conoce como se utilizan conductores de baja resistencia (R) y mejor
conductor. aún se trata de transmitir la potencia con la corriente
más baja posible para lo cual hay que poner un
voltaje muy elevado, por otra parte para usar la
ENERGIA Y POTENCIA EN LOS CIRCUITOS
energía eléctrica con seguridad son recomendables
ELECTRICOS
Al pasar una corriente eléctrica por un conductor, la los voltajes relativamente bajos. Para esto es
necesario elevar y bajar los voltajes.
energía en realidad no se pierde sino se transforma
convirtiéndose en energía térmica. Cuando ponemos
un campo eléctrico en el conductor los electrones Ejemplo 7. Se diseña una unidad de calefacción que
disipe 1000 watts, alimentado con una fuente de 220
libres se aceleran
→ →
voltios. ¿En qué porcentaje se reducirá la producción
ma = qE de calor si el voltaje se reduce a 200 voltios?
Solución.
→ q→ Conectado a 220 Voltios
De donde a= E
m P = 1000 Watts
y su velocidad en el tiempo t es V2
→ → q→ Como P = IV = I 2 R =
v = at = Et R
m la resistencia de la unidad es
Por consiguiente adquirimos una energía cinética V 2 220 2
adicional que se transfiere continuamente al R= = = 48,4Ω
conductor mediante choques entre los electrones y los P 1000
iones de este. Es decir la energía se va transfiriendo Cuando la unidad se conecta a 200 Voltios la
inmediatamente manteniéndose la velocidad de disipación será
desplazamiento en un valor medio. V 2 200 2
P= = = 830 Watts
R 48,4
El porcentaje en que se reduce el calor es
1000 − 830
%= 100 = 17 por ciento.
1000
Ejemplo 8. Una tetera eléctrica contiene 2 litros de
En le figura anterior consideremos la carga dq que va agua que caliente desde 20°C al punto de ebullición
de (1) a (2) con la corriente I en un tiempo de tal

5
Corriente continua Hugo Medina Guzmán

en 5 minutos. El voltaje de la fuente es 200 V y la P 4,8 × 104


kW cuesta 2 centavos. Calcular × 100% = % = 15%
a) la energía consumida (asumir que las pérdidas de PC 3,2 × 105
calor son insignificantes),
b) el costo de usar la tetera bajo estas condiciones seis Ejemplo 10. Un alambre de diámetro 1 milímetro
veces, que lleva una corriente elevada tiene una temperatura
c) la resistencia del elemento de calefacción, y de 1200 K cuando ha alcanzado el equilibrio.
d) la corriente en el elemento. Asumimos que el calor perdido del alambre es
Solución. puramente por radiación. La temperatura de los
El calor ganado por el agua al ser llevado al punto de alrededores es 300 K, la resistencia del alambre a esta
ebullición está dado por la expresión temperatura es 5 x 10-8 Ω m, y el coeficiente de
Q = mc(θ 2 − θ1 ) . temperatura de la resistencia del alambre es 4 x 10-3
por ºC. ¿Cuál es la magnitud de la corriente en el
a) Con m = 2 × 10 3 cm 3 × 1g / cm 3 = 2 × 10 3 g alambre?
J Solución.
c = 4,18 Puesto que el calor está siendo perdido por radiación
gº C
solamente, la energía perdida por segundo por una 1
Tenemos: Q = (2 × 10 )(4,18)(100 − 20 ) = 6,69 x
( )
3 4

5
m de longitud de alambre es W = Aσ T 4 − T0 ,
10 J,
y puesto que no se toman en cuenta las pérdidas de donde A es el área superficial de la longitud del
calor, ésta es la energía eléctrica consumida por la alambre y σ es la constante de Stefan, se asume que
tetera. La energía es la energía consumida por el alambre irradia como cuerpo negro.
segundo, la que es Pero esta energía es provista por el flujo de corriente.
Así, si R es la resistencia de 1 m del alambre,
Q 6,69 × 10 5 J
P=
t
=
5 × 60 s
J
= 2,23 = 2,23 kW.
s
entonces I R = W = Aσ T − T0 . Pero
2
( 4 4
)
b) La tetera utiliza 2,23 kW por 5 minutos cada vez ρl ρ 0 [1 + α (T − T0 )]l
R= =
que se hierve el agua. Cuando se utiliza seis veces, A' A'
2,23 kW se usa por 30 min = ½ hora. Luego el costo Donde A’ es la sección transversal, l es la longitud,
es.
el ρ 0 es la resistencia a 300 K, y α es el coeficiente
2,23kW × ½ hr × 2 centavos.kW/hr = 2,23
centavos. de temperatura de la resistencia. Por lo tanto
c) La potencia consumida es 223 kW y el voltaje de I2 =
(
AA'σ T 4 _ T04 )
la fuente es 200 V. Pero P = V2/R o ρ 0 [1 + (T _ T0 )]l

R=
V2
=
(200V ) = 17,9 Ω
2
[ 2
]
= [1 × 2π (0,5 × 10 −3 )]π (0,5 × 10 −3 ) (5,67 × 10 −8 )(1200 4 − 300 4 )
(5 × 10 )[1 + (1200 − 300)](1)
−8

P 2,23 × 10 3 V
= 1258 A2.
d) Pero también podemos escribir la potencia como ⇒ I = 35,5 A.
P = IV.
P 2,23 × 103W Ejemplo 11. Un radiador eléctrico tiene una
⇒ I= =
V 200 A
= 11,2A
resistencia de (50 + aT )Ω en la temperatura T K
2

Ejemplo 9. Un dínamo conducido por un motor de


y emite βT 4 W , α y β son constantes. Su
vapor que utiliza 103 kg de carbón por día produce resistencia es 125 Ω cuando una diferencia potencial
una corriente de 200 A con una fuerza electromotriz de 50 V es conecta a través de ella. ¿Qué corriente
de 240 V. ¿Cuál es la eficiencia del sistema si el valor debe pasar a través del radiador para que emita 980
del carbón es 6,6 x 103 cal/g? W?
Solución. Solución.
La potencia provista por el carbón por segundo es Si el radiador tiene una resistencia de 125 Ω cuando
cal ⎛ J ⎞ hay una caída de 50 V a través de ella, la potencia
3 J
6,6 × 10 3 ⎜ 4,18 ⎟ = 27,6 × 10 consumida es
g ⎝ cal ⎠ g
V 2 (50 V )
2

27,6 × 10 × 10
3 6 = = 20 W .
PC = = 3,2 105W R 125Ω
24 × 60 × 60 Si se asume que todo se irradiada, entonces
La potencia eléctrica provista por el dínamo es
20 W = β T 4 W , Además,
P = IV = 200A x 240V=4,8 x104 W.
Luego la eficiencia del sistema es (
125Ω = 50 + αT 4 Ω o 75 = αT 4 .)

6
Corriente continua Hugo Medina Guzmán

α 2 75 2 calorímetro y la capacidad calorífica específica del


⇒ = líquido.
β 20 Solución.
Cuando el radiador emite 980 W, tenemos: El calor ganado por el calorímetro y el contenido
20α 2 4 debe ser igual al calor provisto por la energía
980 W = βT14 W = T1 W eléctrica. Así si c es la capacidad calorífica
75 2 específica del líquido y S la capacidad calorífica del
calorímetro, entonces
75 2 × 980
⇒ αT12 = = 525 . S (10º C ) + (400 g )c(10º C ) =
20
1
Así la resistencia del radiador ahora es (2A )2 (8,4Ω )(3 × 60s )
(50 + αT )Ω = 575Ω . Pero la potencia, la
1
2 4,18J/cal
resistencia, y la corriente se relacionan por si convertimos la energía eléctrica de julios a las
calorías. Similarmente,
P = I R.
2
S (10º C ) + (560 g )c(10º C )
1
Luego I =
980 W
= 1,3 A . = (2A )2 (8,4Ω )(4 × 60s )
575Ω 4,18J/cal
∴ (560 − 400 )g × c(10º C )
Ejemplo 12. Un aparato fabricado para funcionar con
1
115 V y para disipar 500W es utilizado donde el = (2A )2 (8,4Ω )60(4 − 3)s .
voltaje es 230 V ¿qué resistencia debe colocarse en 4,18J/cal
2 2 (8,4 )(60 ) cal
serie con el proyector antes de utilizarlo? ¿Qué
cal
energía se disipa en la resistencia agregada? ∴ c= = 0,3
Solución. 4,2(160 )(10 ) gº C gº C
V2 Por lo tanto, volviendo a la primera ecuación,
El aparato tiene una resistencia R dada por P = . tenemos
R
⎛ cal ⎞
(115V )2 S (10º C ) + 400g⎜⎜ 0,3 ⎟⎟(10º C )
Luego R = = 26,45Ω ⎝ gº C ⎠
500W
= 8 x 3 x 60 cal.
La corriente se obtiene de la ecuación
P 500W ∴ S=
(1440 − 1200) = 24 cal
I= = = 4,35 A 10º C ºC
V 115V
Cuando el voltaje de fuente es 230 V, una resistencia
CIRCUITOS DE CORRIENTE CONTINUA.
adicional X se inserta en serie para dar la misma
INTRODUCCION
corriente. Así
Los sistemas de corriente eléctrica estacionarios, o
230 V sea no cambiante que encontramos son
R+ X = = 52,9Ω ⇒ X = 26,45Ω
4,35A combinaciones de generadores y resistencias
La energía disipada en la resistencia agregada es interconectados por alambres a los cuales se les
considera conductores perfectos.
I 2 X = (4,35) (26,45) = 500 W .
2

Esto se ve más fácilmente de la manera siguiente. Si


la misma corriente se va a sacar de una fuente con un
voltaje dos veces el usado previamente, P = IV será
ahora el doble que antes.
Los 500 W extra serán disipados en la resistencia
agregada, que debe tener la misma resistencia que el
aparato, puesto que cada uno disipa la misma
potencia.

Ejemplo 13. Una corriente de 2 A se pasa a través de


La figura muestra un circuito eléctrico esquemático,
un calentador de la resistencia 8,4 Ω sumergido en
los puntos 1, 2, 3 son conocidos como nodos y el
400 g de un líquido contenido en un calorímetro y la
recorrido de un nodo a otro consecutivo se conocen
temperatura se eleva 10°C en 3 minutos. Cuando se
como ramas, por ejemplo entre 1-2 (hay dos ramas),
utilizan 560 g de líquido en el mismo calorímetro y se
entre 1-3, entre 2-3 (hay una rama). La malla es el
pasa la misma corriente, la temperatura se eleva 10°C
recorrido completo de un hondo hasta volver al
en 4 minutos. Despreciando cualquier pérdida de
mismo siguiendo las ramas, por ejemplo 1-2-3, otro
calor o cualquier cambio en la resistencia del
calentador, calcule la capacidad calorífica del

7
Corriente continua Hugo Medina Guzmán

ejemplo, 3-1, otro ejemplo 1-2-3-1 (hay tres mallas Le figura muestra esquemáticamente el circuito con
posibles). la pila y su resistencia
A continuación estudiaremos circuitos sencillos
compuestos de pilas o baterías, resistencias y
condensadores en diversas combinaciones, pero solo
con corriente continua que es la que no cambia de
sentido como iones con las corrientes alternas, que es
motivo de un estudio especial posterior.
Resolver un sistema significa que dados los valores
de la fuerza electromotriz y las resistencias debemos
determinar las intensidades de corriente en todas las Aplicando la segunda ley de Kírchhoff. Siendo I la
ramas o en general dados dos de ellos encontrar el corriente que circula por el circuito
tercero. ε − Ir − IR = 0
De aquí
LEYES DE KIRCHHOFF ε
Para resolver un circuito se necesitan dos reglas I=
r+R
denominadas Leyes o reglas de Kirchhoff.
La diferencia de potencial en la resistencia es:
εR
Primera ley de Kirchhoff Vab = IR =
La suma de corrientes que entran e un nodo es igual a r+R
la suma de corrientes que salen del mismo, esto se La potencia que se disipa a través de R es
deduce del principio de conservación de la carga. La
ε 2R
primera ley de Kirchhoff podemos expresarla como P = I 2R =
∑I = 0 (1) (r + R )2
En el nodo 2 de la figura anterior
Si queremos encontrar el valor de R para el cual la
I1 + I 2 − I 3 = 0 potencia disipada sea la mínima
Si tenemos N nodos en un circuito podemos obtener ∂P ε2 2ε 2 R
N - 1 ecuaciones independientes, la ecuación del nodo = − =0
N no es independiente ya que si la ecuación (1) ∂R (r + R ) (r + R )
cumple en (N-1) nodos, esta cumple automáticamente Resolviendo
en el nodo N. (R + r ) − 2 R = 0 ⇒ R=r
Segunda ley de Kirchhoff
Ejemplo 15. Un estudiante de física conecta una pila
La suma de las caídas de potencial a lo largo de
a un circuito y encuentra que la corriente de la pila es
cualquier malla o lazo debe ser igual a la suma de los
I1. Cuando conecta una segunda pila idéntica en serie
aumentos de potencial
con la primera, la corriente se convierte en I2.
∑ ε − ∑ RI = 0 (2) Cuando conecta las pilas en paralelo, la corriente a
Esta ley se deduce del principio de conservaci6n de la través del circuito es I3.
energía. Demuestre que la relación que él encuentra entre las
En el caso de la figura anterior, tenemos corrientes es 3I2I3 = 2 Ì1(I2 + I3).
V1 + ε 1 − I 1 R1 = V2 (Rama 1-2) Solución.
Sea ε la fuerza electromotriz de cualesquiera de las
V2 − I 5 R5 = V3 (Rama 2-3) pilas ser y r su resistencia interna. y el circuito
externo tiene una resistencia R.
V3 + ε 4 − I 4 R4 = V1 (Rama 1-2)
a) Cuando se usa una sola pila,
Sumando estas expresiones obtenemos la suma de las
caídas de potencial y aumento de potencial de la
malla 1-2-3-1 (un lazo cerrado)
ε 1 + ε 4 − I 1 R1 − I 5 R5 − I 4 R4 = 0 ⇒
ε 1 + ε 4 − (I 1 R1 + I 5 R5 + I 4 R4 ) = 0
Expresión que en general viene a ser
∑ ε − ∑ RI = 0
Aplicamos la ecuación del circuito,
Ejemplo 14. ¿Cual es la diferencia de potencial en ε
una resistencia R conectada entre los bornes de una I1 =
pila de fuerza electromotriz ε y con resistencia R+r
interna r? Si dos pilas idénticas se conectan en serie.
Solución.

8
Corriente continua Hugo Medina Guzmán

⎛1 1⎞
2⎜⎜ − ⎟⎟
I I2 ⎠
1= ⎝ 1 ,
⎛1 1⎞
⎜⎜ − ⎟⎟
⎝ I 3 I1 ⎠
∴ 2 I1I 2 − I 2 I 3 = 2 I 2 I 3 − 2 I1I 3
Sus fuerzas electromotrices actúan en igual sentido.
Por lo tanto ⇒ 3I 2 I 3 = 2 I1 (I 2 + I 3 )

I2 = Ejemplo 16. Una batería de 50 pilas se está cargando
R + 2r de una fuente de C.C. de 230 V y de resistencia
Cuando las pilas se conectan en paralelo, puesto que interna insignificante. ¿La fuerza electromotriz de
son idénticas, por la simetría del montaje, corrientes cada pila en carga es 2,3 V, su resistencia interna es
idénticas I 0 deben atravesar cada una de las pilas. 0,1 2 y la corriente de carga necesaria es 6 A. ¿qué
resistencia adicional debe ser insertada en el circuito?
Solución.
Sea R la resistencia adicional necesitada en el
circuito.

Además por la primera ley de Kichhoff Las 50 pilas tienen una fuerza electromotriz total de
I 3 = I 0 + I 0 = 2 I 0 (1) 115 V y una resistencia interna total de 5 Ω .
Considerando el paso de la corriente a través de Aplicamos la segunda ley de Kirchhoff para obtener
cualquiera de las pilas, tenemos: 230 − ε = I (R + r ) ⇒ 230 − 115 = 6(R + 5)
ε − I 0 r − I 3 R = 0 (2) 115
De aquí R = − 5 = 14,2Ω
I3 6
De (1) I 0 =
2
Reemplazando en (2): Ejemplo 17. Resolver el circuito mostrado en la
figura.
I3 ⎛ r⎞
ε− r − I3R = 0 ⇒ ε = I3⎜ R + ⎟
2 ⎝ 2⎠
ε
De aquí I 3 =
⎛ r⎞
⎜R + ⎟
⎝ 2⎠
De estas ecuaciones de I1, I2 e I3 encontramos que
ε 2ε r ε
R+r = , R + 2r = y R+ =
I1 I2 2 I3
Eliminando r entre las dos primeras ecuaciones da.
⎛1 1⎞
R = ε ⎜⎜ − ⎟⎟ , Solución.
Como primer paso fijemos el sentido de la corriente
⎝ I1 I 2 ⎠ en cada rama, las que finalmente pueden resultar con
y entre la primera y la tercera da signo negativo, lo que significaría que el sentido es
⎛2 2⎞ contrario al considerado.
R = ε ⎜⎜ − ⎟⎟ ,
⎝ I1 I 2 ⎠
Dividiendo estas dos últimas ecuaciones una por la
otra da

9
Corriente continua Hugo Medina Guzmán

Ejemplo 18. Se tienen dos baterías una de 6 voltios y


resistencia interna 2 ohms y otra de 12 voltios y
resistencia interna 4 ohms. Se conecta una resistencia
de 12 ohms. ¿Cuál es la diferencia de potencial en la
resistencia cuando se conecta tal como en
a) la figura (a)?
b) la figura (b)?

Aquí tenemos seis incógnitas por lo tanto


necesitamos seis ecuaciones
Por la primera ley de Kirchhoff. Nudo a:
I 3 + I 5 − I1 = 0 (1)
Nudo b: Solución.
I 4 + I 2 − I3 = 0 (2) a)
Nudo c:
I1 − I 4 − I 6 = 0 (3)
Por la segunda ley de Kirchhoff
Siendo cuatro nudos solo podemos obtener tres
ecuaciones.
Camino a-c-d-a
− I1R1 + ε1 − I 6 R6 − I 5 R5 = 0 (4)
Camino a-e-d-a
− I1R1 + ε1 − I 4 R4 − I 3 R3 = 0 (5)
Aplicando la segunda ley de Kirchhoff.
Camino a-d-b-a
ε 1 − Ir1 + ε 2 − Ir2 − IR = 0
+ I 5 R5 + ε 2 − I 2 R2 − I 3 R3 = 0 (6)
ε1 + ε 2
ya tenemos las 6 ecuaciones que nos resolverán el I=
circuito. r1 + r2 + R
Reordenando las ecuaciones:
6 + 12 18
0 = I 1 (− 1) + I 2 (0) + I 3 (1) + I 4 (0) + I 5 (0) + I 6 (0) I= = = 1 ampere
2 + 4 + 12 18
0 = I 1 (0) + I 2 (1) + I 3 (− 1) + I 4 (1) + I 5 (0) + I 6 (0) La diferencia de potencial en los puntos a y b es
0 = I 1 (1) + I 2 (0 ) + I 3 (0) + I 4 (− 1) + I 5 (0 ) + I 6 (− 1) Vab = IR =1 ampere x 12 ohms
0 = I 1 (R1 ) + I 2 (0) + I 3 (0 ) + I 4 (0) + I 5 (R5 ) + I 6 (R6 ) Vab = 12 voltios
b)
ε 1 = I 1 (R1 ) + I 2 (0) + I 3 (R3 ) + I 4 (R4 ) + I 5 (0) + I 6 (0)
ε 2 = I 1 (0 ) + I 2 (R 2 ) + I 3 (R3 ) + I 4 (0 ) + I 5 (− R5 ) + I 6 (0 )
Que podernos resolver por determinantes
0 0 1 0 0 0
0 1 -1 1 0 0
0 0 0 -1 0 -1
ε1 0 0 0 R 5 R 6
ε1 0 R 3 R 4 0 0
ε R2 R5 0 -R5 0
I1 = 2
−1 0 1 0 0 0 Aplicando la primera, ley de Kirchhoff en el nudo a
0 1 -1 1 0 0 I1 + I2 - I = 0 (1)
1 0 0 -1 0 -1 Aplicando la segunda ley de Kirchhoff de b, a, b por
R1 0 0 0 R 5 R 6 los dos caminos.
R1 0 R 3 R 4 0 0 ε 1 − I 1 r1 − IR = 0 (2)
0 R2 R5 0 -R5 0 ε 2 − I 2 r2 − IR = 0 (3)
De igual modo para I2, I3, I4, I5, I6. Reescribiendo las ecuaciones
0 = I 1 (1) + I 2 (1) + I (0 )

10
Corriente continua Hugo Medina Guzmán

ε 1 = I 1 (r2 ) + I 2 (0) + I (0) La potencia disipada en una resistencia Rr a través


ε 2 = I 1 (0) + I 2 (r2 ) + I (0) de la cual pasa la corriente I r es Pr = I r Rr .
2

Resolviendo por determinantes para I : Aplicando esto a los elementos en el diagrama,


1 1 0 tenemos

r1 0 ε1 1681ε 2 676ε 2
PAB = , PAC = ,
0 r2 ε2 (139)2 Ω (139)2 Ω
I =
1 1 0 450ε 2
PAE = ,
r1 0 R (139)2 Ω
0 r2 R 1728ε 2 8ε 2
PDB = , PGH = ,
Reemplazando valores (139)2 Ω (139)2 Ω
1 1 0
1156ε 2
2 0 6 PBF =
0 4 12 = − 24 − 24 = 0,5 A
(139)2 Ω
I= Está claro que la mayor potencia disipada es en la
1 1 0 − 48 − 24 resistencia entre los puntos B y D. Para satisfacer las
2 0 12 condiciones del problema, PDB es 1 W para el valor
0 4 12 máximo de la fuerza electromotriz ε . Así:
La diferencia de potencial entre los puntos a y b es 1728ε 2
= 1W ⇒ ε 2 =
(139)2 V 2
Vab = IR = 0,5 Amperes x 12 Ohms = 6 voltios
(139)2 Ω 1728
Ejemplo 19. Un circuito se conecta como en el 139
diagrama. ¿La disipación de la potencia no debe
⇒ ε= V = 3,34V
24 3
exceder a 1 W en ningún rama. ¿Cuál es el valor
máximo de la fuerza electromotriz de la batería?
CONEXIONES DE RESISTENCIAS, EN SERIE
Solución.
Y EN PARALELO.
Cuando se tienen varias resistencias en un circuito es
conveniente reducirlas a una resistencia equivalente
con el objeto de facilitar la resolución del circuito.
En serie.
Se dice que n resistencias están conectadas en serie
cuando están unidas extremo a extremo una a
continuación de otra, como se muestra en la figura
siguiente.

Todos los puntos en el diagrama se han etiquetado, y


las corrientes se han insertado en cada rama.
Aplicando la primera ley de Kirchhoff a los puntos A,
F, y D, tenemos
I1 = I 2 + I 3 , I 3 + I 5 = I 6 , I 2 = I 4 + I 5 Con una diferencia de potencial V fluye una corriente
I, aplicando la segunda ley de Kirchhoff
Aplicando la segunda ley de Kirchhoff a los circuitos
ACDB, BAEF, y DGHF. Tenemos V − IR1 − IR2 − IR3 − ..... − IRn = 0
ε = I1 (1Ω ) + I 2 (1Ω ) + I 4 (3Ω ) expresión de la cual se obtiene la resistencia
equivalente
ε = I1 (1Ω ) + I 3 (2Ω ) + I 6 (4Ω )
V
0 = I 5 (2Ω ) + I 6 (4Ω ) − I 4 (3Ω ) Re =
I
= R1 + R2 + R3 + ..... + Rn
Resolviendo estas seis ecuaciones simultáneamente n
encontramos las soluciones siguientes: Re = ∑ Ri
41ε 26ε 15ε i =1
I1 = , I2 = , I3 = , En paralelo.
139Ω 139Ω 139Ω Cuando n resistencias se conectan en la forma como
24ε 2ε 17ε muestra la figura siguiente, se dice que las
I4 = , I5 = y I6 =
139Ω 139Ω 139Ω resistencias están conectadas en paralelo.

11
Corriente continua Hugo Medina Guzmán

Q'1 V 2 / R1 R2 A1
= = =
Q'2 V 2 / R2 R1 A2
En este caso el calentamiento es mayor en el
conductor con sección transversal de mayor área.

Ejemplo 21. Cinco resistencias, cada uno de 10 Ω ,


se conectan para formar una letra H, una pila de 2 V y
con resistencia interna 1,86 Ω se conecta a través de
los extremos superiores y un amperímetro con
resistencia 5 Ω a través de los extremos inferiores.
¿Qué corriente pasa a través del amperímetro?
Solución.
El circuito mostrado en el diagrama (a) es equivalente
De la primera ley de Kirchhoff al circuito mostrado en el diagrama (a).
I = I 1 + I 2 + I 3 + ..... + I n
De la segunda ley de Kirchhoff
V = I 1 R1 + I 2 R2 + I 3 R3 + ..... + I n Rn
De este última encontramos que
V V V
I1 = , I2 = , I3 = , ……….,
R1 R2 R3
V
In =
Rn
Reemplazando en la primera expresión
I 1 1 1 1 1 Las resistencias de10 Ω y 25 Ω están en paralelo.
y = = + + + ..... + Por lo tanto la resistencia equivalente es R, donde
V Re R1 R2 R3 Rn
1 1 1 5+2 7
1 n
1 = + = = ⇒
De aquí =∑ R 10 25 50 0
Re i =1 Ri 50
R= = 7,14 Ω
7
Ejemplo 20. Dos conductores de la misma longitud y El circuito es por lo tanto equivalente al mostrado en
material pero con diferentes áreas de sección el diagrama (c).
transversal son: a) conectados en serie, y b) en
paralelo. ¿Cuándo una diferencia potencial se aplica
a través de las combinaciones, en cual de los
conductores el calentamiento será mayor?
Solución.
La resistencia de cada conductor tiene la forma
l
R=ρ . Como la resistividad y las longitudes son
A
R A
iguales en cada caso, 1 = 2 . Es posible ahora encontrar la corriente en el circuito.
R2 A1
ε 2 2
a) Cuando los conductores están en serie, la misma I0 = = = A
corriente pasa con cada uno. Por lo tanto el cociente R 10 + 10 + 7,14 + 1,86 29
del calentamiento producido en los alambres es: Esta corriente se divide en las corrientes I1 e I2 a
Q1 I 2 R1 R1 A2 través de las partes inferiores de los circuitos, según
= = = como se muestra en los diagramas (a) y (b), donde
Q2 I 2 R2 R2 A1
I 1 R2 10Ω I1 10
El calentamiento es mayor en el conductor con = = ⇒ =
sección transversal de menor área. I 2 R1 25Ω I 1 + I 2 35
b) Cuando los conductores están en paralelo, 10
diferentes corrientes pasan a través de ellos pero la ⇒ I 1 = (I 1 + I 2 )
diferencia potencial a través de cada uno es igual. Por 35
lo tanto Por la primera ley de Kirchhoff,
I1 + I 2 = I o

12
Corriente continua Hugo Medina Guzmán

10 10 2 La resistencia entre 4 y 3 es
Luego I 1 = I0 = × = 0,0197 A 1 1 1 2 1
35 35 29 = + = =
Es la corriente que atraviesa el amperímetro R43 4 R 4 R 4 R 2 R
Ejemplo 22. En la figura mostrada, calcular cada
R43 = 2 R
corriente en la dirección indicada para las resistencias El circuito queda reducido a
y calcular ε .

La resistencia entre 1, 2, 3 es
R123 = 3R + R = 4 R
La resistencia entre 1, 4, 3 es
R143 = 2 R + 2 R = 4 R
El circuito queda reducido a
Solución.
a) Cálculo de las corrientes:
En la malla aefda. I 1 = I 2
− 4,0 I 1 + 12 − 5,0 I 1 = 0 ⇒ 9,0 I 1 = 12 ⇒
12
I1 = I 2 = = 1,33A
9 La resistencia entre 1 y 3 es
En el nudo e. 1 1 1 2 1
4 7 = + = =
− − I 3 = 0 ⇒ I 3 = I 4 = 1A R13 4 R 4 R 4 R 2 R
3 3
Cálculo de ε :
R13 = 2 R
En la malla ebcfb. El circuito queda reducido a
− 3(1) − ε − 3(1) = 0 ⇒ ε = 6V

Ejemplo 23. Encuentre la resistencia equivalente Finalmente la resistencia entre a b es


entre los terminales a y b del circuito de la figura.
Rab = 3R + 2 R + R = 6 R

Ejemplo 24. Con un pedazo de alambre uniforme se


forman dos cuadrados con un lado común de longitud
10cm. Una corriente ingresa al sistema rectangular
por una de las esquinas y va diagonalmente para
Solución. salir por la esquina opuesta. Demuestre que la
Este circuito formado por partes en serie y partes en corriente en el lado común es un quinto de la
paralelo corriente que entra. ¿Qué longitud del alambre
conectado entre la entrada y la salida (A y F), tendría
un efecto resistente equivalente tendría un efecto
resistivo equivalente?
Solución.
Sea la R resistencia de cada lado del cuadrado, y el
flujo de corrientes tal como el mostrado en el
diagrama.
Le resistencia entre 2 y 3 es
1 1 1 1 3 1
= + + = =
R23 3R 3R 3R 3R R
R23 = R

13
Corriente continua Hugo Medina Guzmán

Aplicando la primera ley de Kirchhoff, ∑ I = 0, a Aplicando la primera ley de Kirchhoff


los puntos A, B, y E da:
I1 − I 2 − I 3 = 0 (1)
I1 + I 2 = I 3
Aplicando la segunda ley de Kirchhoff al circuito
I 2 − I 4 − I5 = 0 (2) cerrado que contiene ambas pilas y luego al circuito
I3 + I 4 − I6 = 0 (3) cerrado con la pila inferior y la resistencia externa,
tenemos
La aplicación de la segunda ley de Kirchhoff a los
circuitos ABED y BCFE da ε 1 − ε 2 = (2 − 1,2) = 0,1I 2 − 0,5I 1 y
I 2 R + I 4 R − I 3 2 R = 0 (4) ε 2 = 2 = 0,1I 2 + 5I 3
I5 2R − I6 R − I 4 R = 0 (5) De aquí I 2 − 5 I 1 = 8 y I 2 + 50(I 1 + I 2 ) = 20
Eliminando I 5 e I 6 de las ecuaciones, (2), (3), y (4) o 10 I 2 − 50 I 1 = 80 y 51I 2 + 50 I 1 = 20 .
obtenemos: 100
Luego I 2 = = 1,64 A,
2I 2 − I 3 − 4I 4 = 0 (6) 61
Eliminando I 2 las ecuaciones (1), (4), y (6): 388
I1 = − = -1,27 A,
Obtenemos: 355
(1) + (4) I1 − 3I 3 + I 4 = 0 (7) I 3 = I 1 + I 2 = 0,37 A
2(1) + (6) 2 I1 − 3I 3 − 4 I 4 = 0 (8)
Ejemplo 26. Un galvanómetro de resistencia 20 Ω
Eliminando I 3 de (7) y (8): da una desviación de toda la escala cuando una
1 corriente de 1 mA pasa a través de ella. ¿Qué
(8) – (7) I1 − 5 I 4 = 0 ⇒ I 4 = I1 (9) modificación se debe hacer al instrumento de modo
5 que dé la desviación de toda la escala para (a) una
Además, la caída de potencial de A a F por el camino corriente de 0,5 A, y (b) una diferencia potencial de
ADEF, empleando las ecuaciones. (3), (7), y (9), es: 500V?
VAF = I 3 2 R + I 6 R = R(2 I 3 + I 3 + I 4 ) Solución.
⎛ 2⎞ Si un galvanómetro tiene una resistencia de 20 Ω y
= R(I1 + 2I 4 ) = I1 R⎜1 + ⎟ da la desviación completa para una corriente de 1
⎝ 5⎠ mA, después la caída de voltaje a través de ella bajo
7 estas circunstancias es
=
5
RI1 V = IR = (10 −3 )(20) = 0,02 V
Empleando las ecuaciones. (3), (7), y (9). (a) Para permitir que el galvanómetro lea hasta 0,5
Por lo tanto el efecto equivalente se obtiene si un A, una resistencia de desviación debe ser agregada.
alambre 7 5 veces la longitud de cualquier lado del Esta resistencia debe tomar 499 mA, permitiendo
solamente 1 mA a través del galvanómetro. Pero la
cuadrado se conecta entre A y F, porque produce la diferencia potencial a través de cada una es igual.
misma caída de potencial que el cuadrado doble entre Así si r es la resistencia de la desviación, entonces
estos puntos.
1 mA x 20 Ω = 499 mA x r,
Ejemplo 25. Dos pilas, una de fuerza electromotriz 20
⇒ r= = 0,0401 Ω .
1,2V y resistencia interna 0,5 Ω , la otra de fuerza 499
electromotriz 2V y resistencia interna 0,1 Ω , están (b) Para cambiar la lectura del voltímetro hasta 500
conectadas en paralelo y la combinación se conectada V, uno debe agregar una resistencia en serie.
en serie con una resistencia externa de 5 Ω . ¿Qué Solamente 0,02 V caen a través del galvanómetro
corriente pasa con esta resistencia externa? para la corriente máxima de 1 mA. Así 499,98 V
Solución. deben caer a través de la resistencia R. La misma
El circuito es como el mostrado en la figura siguiente: corriente atraviesa la resistencia y el galvanómetro.
Por lo tanto

14
Corriente continua Hugo Medina Guzmán

499,98 V 1,9V 1
R= = 499,980Ω . = mA . Pero de la calibración
10 −3 A (1900 + 3000)Ω 3
del galvanómetro, cuando la corriente cae a un tercio
Ejemplo 27. Una bobina del alambre está conectada de su valor, la resistencia insertada debe tener un
a través de un puente de Wheatstone y de una valor
resistencia estándar de temperatura controlada de 1 [ ]
X = 2000(3 − 1) Ω = 4000Ω .
Ω a través del otro. Si la temperatura de la bobina
es 0°C, los otros brazos del puente tienen cociente de El error en la lectura es así 200 Ω , y el error del
0,923 entre las resistencias en el. Si la temperatura de porcentaje es
la bobina es 100°C el cociente es 1,338. ¿Cuál es el 200
coeficiente de temperatura de la resistencia del
× 100% = 5,3%
3800
alambre?
Solución. Ejemplo 29. Una longitud de 300 cm de alambre de
De la ecuación del puente de Wheatstone, las potenciómetro se requiere para balancear la fuerza
resistencias de la bobina, R0 a 0°C y R a 100°C, son electromotriz de una pila. Cuando una resistencia de
R0 = 0,923 Ω y Rt = 1,338 Ω . Pero 10 Ω se conecta a través de la pila, la longitud
Rt = R0 (1 + αt ) , donde α es el coeficiente de requerida para el balance es 250 cm. Calcule la
temperatura del alambre de la resistencia. Así resistencia interna de la pila.
(Rt R0 ) − 1 (1,338 0,923) − 1 Solución.
α= = El alambre del potenciómetro es uniforme y la caída
t 100º C de potencial a lo largo de el es regular. Por lo tanto la
= 0,0045 /ºC longitud a lo largo del alambre es directamente
proporcional a la caída potencial a través de el. Así
Ejemplo 28. Una resistencia variable en serie con ε = k x 300 cm, donde k es la constante de
una pila 2 V y un galvanómetro se ajusta para dar una proporcionalidad entre el potencial y la longitud,
desviación a escala completa, para una corriente de 1 teniendo unidades de V/cm.
mA. ¿Qué resistencia puesta en serie en el circuito Cuando una resistencia de 10 Ω se pone a través de
reducirá la lectura del galvanómetro por l/f? los terminales de la pila, el potencial a través del
El galvanómetro está calibrado para medir resistencia resistor es V = k x 250 cm.
sobre esta base, pero la fuerza electromotriz de la pila ε 300 6
cae el 5% y se reajusta la resistencia variable de Luego = = .
modo que la desviación a escala completa V 250 5
corresponda otra vez al cero de la resistencia variable. Pero cuando una resistencia se coloca a través de los
¿Qué error del porcentaje ahora se da en una terminales de la pila, una corriente fluirá en ese
resistencia que tenga un valor verdadero de 3800 Ω ? circuito, donde V = IR y ε = I(R + r).
Solución. ε 6 R + r 10Ω + r
La resistencia total en el circuito cuando el De aquí = = = ⇒
galvanómetro está dando la desviación a escala
V 5 R 10Ω
completa es r = 2Ω
ε 2V
R= = = 2000Ω Ejemplo 30. Se tiene el circuito mostrado en la
I 10 −3 A figura. Los valores de los diferentes elementos son:
Si una resistencia desconocida X se agrega al circuito R = 15,0 Ω, R = 5,0 Ω, R = 10,0 Ω, R = 20 Ω, R =
1 2 3 4 5
y produce una lectura de (l/f) mA en el 5,0 Ω, y ε = 80 V
galvanómetro, entonces
2
R+ X = = 2000 fΩ ⇒
(1 f ) × 10 −3
X = (2000 f − 2000 ) = 2000( f − 1)Ω
95
La fuerza electromotriz de la pila cae a de 2 V =
100
1,9 V.
Para la desviación a escala completa la resistencia en
el circuito será: a) Si el interruptor S permanece abierto, calcule la
1,9 resistencia equivalente del circuito y la corriente total
R' = = 1900Ω , y si otra resistencia de 3800 I . Luego calcular las corrientes I e I .
10 −3 1 2 3
b) Utilizando el resultado de la parte a), calcule la
Ω se inserta en el circuito, la corriente es potencia entregada por la fuente y la potencia
disipada por cada resistencia (si el interruptor “S”

15
Corriente continua Hugo Medina Guzmán

permanece abierto). Compare sus dos resultados y La suma de la potencia disipada por las resistencias
comente. es igual a la potencia entregada por la fuente porque
c) Suponga que el potencial eléctrico del punto e es la fuente es ideal sin resistencia interna.
cero (V ). Determine el potencial eléctrico de los
e c) Ve − 20 I1 + 80 = Va
puntos a y c. Con I1 = 2 A y Ve= 0:
d) Se adiciona 60 voltios al voltaje de la fuente, y a
0 − 20(2) + 80 = Va ⇒ Va = 40 V
continuación se cierra el interruptor “S”. Calcule la
corriente total en el circuito. Ve + 10 I 2 = Vc
Solución. Con I2 = 1 A y Ve= 0:
0 − 10(1) = Vc ⇒ Vc = −10 V
a) La corriente total I .
1

d) Se adiciona 60 voltios al voltaje de la fuente, y se


cierra el interruptor “S”.

La parte bcde equivale a un corto circuito.

La corriente del circuito es:


La resistencia equivalente es 40 . V 140
80 I= = = 4 A.
I1 = = 2 A. Req 35
40
Las corrientes I e I .
2 3 Ejemplo 31. Encontrar la resistencia equivalente
I1 = I 2 + I 3 = 2 , I 2 = I 3 ⇒ I 2 = I 3 = 1 A. entre los terminales a y b del circuito mostrado en la
figura.
b) La potencia entregada por la fuente:
( )
Pε = I12 40 = 22 (40) = 160 W.
La potencia disipada por cada resistencia con el
interruptor “S” abierto.
P1 = I12 R1 = (22 )(15) = 60 W.
P2 = I 22 R2 = (12 )(5) = 5 W. Solución.

( )
P3 = I 32 R3 = 12 (10 ) = 10 W.
P4 = I R4
2
4 = (2 )(20) = 80 W.
2

P5 = I R5
2
5 = (1 )(5) = 5 W.
2

16
Corriente continua Hugo Medina Guzmán

Consideremos que la resistencia entre a y b es Re Siguiendo las corrientes este circuito es equivalente a:
observemos ahora el corte AA en la figura,
considerado el lado izquierdo la resistencia entre a’ y
b’ es también igual a Re . Luego podemos dibujar el
circuito como en la figura siguiente.

Circuito que asta formado por partes en paralelo y en


Este circuito es equivalente a serie.
El circuito se reduce a:

Que es un circuito en serie cuya resistencia total es


RRe
R+ , que a su vez es igual a Re , de aquí
R + Re Reducidos los circuitos en paralelo se tiene:
RRe
Re = R + y Re − RRe − R = 0
2 2

R + Re
Resolviendo para Re obtenemos el valor positivo

Re =
( 5 +1)R
2
Ejemplo 32. Encontrar la resistencia equivalente Este a su vez se reduce a
entre los terminales a y b del circuito de la figura.

Finalmente

Solución. CAMBIO DE UN CIRCUITO TRIANGULO A


Como se trata de un circuito simétrico, la distribución ESTRELLA
de corrientes será simétricamente como mostramos a Se presentan algunos casos que no son en serie ni en
continuación. paralelo, cuya resolución es larga, pero que es posible
simplificar realizando ciertas transformaciones,
cambiar un circuito triángulo, a otro equivalente
estrella, mostrados en la figura siguiente.

17
Corriente continua Hugo Medina Guzmán

Por ser equivalentes los intensidades de corriente en 3 ×1 3 1


Rc = = = Ω
los nodos a, b, c en los dos circuitos deben ser iguales 1+ 3 + 2 6 2
respectivamente, las que de acuerdo a la primera ley 3× 2 6
de Kirchhoff se repartan en el circuito triangulo. Rb = = = 1Ω
Aplicando la segunda ley de Kirchhoff al lazo a, b, c, 1+ 3 + 2 6
a en el triángulo 1× 2 2 1
IR2 + (I 2 + I )R3 − (I 1 − I )R1 = 0 Rd = = = Ω
1+ 3 + 2 6 3
de donde E]. circuito se convierte en
I R − I 2 R3
I= 1 1 (1)
R1 + R2 + R3
Aplicando la segunda ley de Kirchhoff entre a y b de
los dos circuitos , el potencial Vab de ambos deben
de ser iguales.
Vab = IR2 = I 1 R12 − I 2 R23 (2)
Reemplazando el valor de I de (1) en (2)
I 1 R1 − I 2 R3 Reduciendo los partes en serie
R2 = I 1 R12 − I 2 R23
R1 + R2 + R3
R1 R2 R2 R3
I1 − I2
R1 + R2 + R3 R1 + R2 + R3
= I1 R12 − I 2 R23 (3) Reduciendo las partes en paralelo
Por observación de la expresión (3)
R1 R3 R2 R3
R12 , R23 = y finalmente
R1 + R2 + R3 R1 + R2 + R3
De igual manera se puede deducir
R1 R3 La resistencia equivalente es 2Ω
R13 =
R1 + R2 + R3 CIRCUITO RC
En esta parte estudiaremos un circuito en el que la
Ejemplo 33. Encontrar la resistencia equivalente corriente no es estacionaria, se trata del circuito con
entre los terminales a y b de la figura. resistencia y condensador en serie.

Solución.
Busquemos el circuito estrella equivalerte al circuito
triángulo cdb La figura muestra un condensador C, una resistencia
R que se conecta a une fuerza electromotriz ε por
medio de una llave S con tres posiciones.
Inicialmente el circuito está abierto, no hay carga en
el condensador. (Posición 0).

18
Corriente continua Hugo Medina Guzmán

Carga. Para t = ∞ , tenemos q = εC = Q0


En el instante t = 0 ponemos la llave S en la La variación de la corriente es
posición 1; empieza e fluir una corriente I.
dq ε − RC
t

I= = e
dt R
ε
Para t = 0 , I = y para t = ∞ , I = 0
R
La figura siguiente muestra los diagramas q versus t e
I versus t durante le carga

Aplicando la segunda ley da Kirchhoff en el circuito


de le figura anterior
ε − VR − VC = 0
V R , diferencia de potencial en le resistencia = IR Descarga.
Una vez que ha pasado un tiempo igual a varias veces
q
VC , diferencia de potencial en el condensador = , el valor del producto RC conocido como constante de
C tiempo del circuito se ruede considerar que el
llamando q a la carga del condensador e I a la condensador está con su carga total Q0 = εC .
corriente en cierto instante t .
Pasamos la llave a la posición 2 y obtenemos el
De aquí
circuito mostrado a continuación.
q
ε − IR − =0
C
dq
Como I = , podemos escribir
dt
dq q
ε −R − =0
dt C
dq 1 ε
o + q− =0
dt RC R
Resolviendo la ecuación para las condiciones
iniciales, para t = 0, q = 0 En este caso en el instante t = 0 , la carga en el
dq 1 condensador es q = Q0 .
=− (q − εC )
dt RC Aplicando la segunda ley de Kirchhoff
dq 1 q
=− dt V R + VC = 0 ⇒ IR + =0
(q − εC ) RC C
integrando dq dq 1
Como I = , escribimos + q=0
q dq 1 t
dt dt RC
∫ (q − εC ) = − RC ∫ dt
0 0 Resolviendo la ecuación para las condiciones
t iniciales
1
ln (q − εC ) 0 = −
q t = 0, q = Q0
t
RC 0 dq 1 dq 1
=− q=0 ⇒ =− dt
ln
(q − εC ) = − 1 t dt RC q RC
Integrando
− εC RC
2 dq 1 t
(q − εC ) = e − RC ∫ RC ∫0
t
=− dt
0 q
− εC t
Finalmente Q 1
ln q 0 0 = − t
⎛ −
t
⎞ RC 0

q = εC ⎜1 − e RC ⎟
⎟ q 1
⎝ ⎠ ln =− t
Q0 RC

19
Corriente continua Hugo Medina Guzmán

q dq
= e −t RC Como I = , podemos escribir
Q0 dt
Finalmente dq q
ε0 − R − = 0
q = Q0 e −t RC = εCe −t RC dt C
para t = 0, tenemos q = Q0 dq 1 ε
La variación de la corriente es o + q− 0 =0
dt RC R
dq ε
I= = − e −t RC Resolviendo la ecuación para las condiciones
dt R iniciales, para t = 0, q = 0
ε dq
=−
1
(q − ε 0 C )
Para t = 0 , I = − y para t = ∞ , I = 0
R dt RC
La corriente es en sentido contrario a la corriente dq 1
=− dt
durante la carga.
La figura a continuación muestra los diagramas q
(q − ε 0 C ) RC
versus t e I versus t durante la descarga. integrando
q dq 1 t
∫ (q − ε C ) = − RC ∫ dt
0
0
0

t
1
ln (q − ε 0 C ) 0
q
=− t
RC 0
(q − ε 0 C ) 1
ln =− t
− εC RC
(q − ε 0 C ) − RCt
=e
Ejemplo 34. Halle la ecuación para la carga de un − ε 0C
condensador conectado en serie con una resistencia R Finalmente
y una fuente continua ε 0 . ⎛ −
t

Solución.
q = ε 0 C ⎜⎜1 − e RC ⎟

En el instante t = 0 ponemos la llave S en la ⎝ ⎠
posición 1; empieza e fluir una corriente I.
Ejemplo 35. En el circuito de la figura, estando el
condensador descargado, se cierra la llave, calcular:
a) El tiempo para el cual el condensador almacene la
mitad de la carga.
b) La diferencia de potencial en el Condensador y en
la resistencia para ese tiempo.

Aplicando la segunda ley da Kirchhoff en el circuito


de le figura anterior
ε 0 − VR − VC = 0
V R , diferencia de potencial en le resistencia = IR
q Solución.
VC , diferencia de potencial en el condensador = ,
C a) La carga total del condensador sería Q0 , la mitad
llamando q a la carga del condensador e I a la
de la carga Q0 2 .
corriente en cierto instante t .
De aquí La expresión para la carga del condensador es
q
ε 0 − IR − = 0
(
q = Q0 1 − e −t RC )
C Q0
Si para el tiempo t1 2 , q = ⇒
2

20
Corriente continua Hugo Medina Guzmán

Q0
2
(
−t
= Q0 1 − e 1 2
RC 1
⇒ =1− e 12 ⇒
2
−t
) RC
y ε − I 2 R2 −
q
C
=0 (3)

t De (2) obtenemos
−t RC 1
e 12 = ⇒ 1 2 = ln 2 ⇒ ε
2 RC I1 =
R1
t1 2 = 0,692 RC = 0,692 x 20 x 5 x 106 = 0,692x10-
4 Trabajando con (3)
s.
q dq
b) La diferencia de potencial en el condensador es I 2 R2 + − ε = 0 , I2 =
VC =
q εC
C C
=
−t
1− e 12 (
RC −t
= ε 1− e 12 ) ( RC
)= C
dq 1 ε
dt
Luego + q− =0
dt R2 C R2
( ⎛ 1⎞
)
ε 1 − e − RC ln 2 RC = 6⎜1 − ⎟ = 3 V. Cuya solución es
⎝ 2⎠
La diferencia de potencial en la resistencia es (
q = εC 1 − e −t R2C )
ε −t1 2 RC −t1 2 RC
y la corriente es
V R = IR = e R = εe
R dq ε −t R2C
Para t1 2 = 0,692 × 10 -4 = RC ln 2 I2 = = e
dt R2
ε − t1 2 RC − t1 2 RC Reemplazando las expresiones de I1 e I2 en (1)
VR = IR = e R = εe
R ε ε
I= + e −t RC ⇒
ε R1 R2
= ε e − RC ln 2 RC = = 3 V.
2 ⎛ 1 1 −t RC ⎞
I = ε ⎜⎜ + e ⎟⎟
Ejemplo 36. En el circuito de la figura, estando el ⎝ R1 R2 ⎠
condensador descargado, se cierra la llave. Esta expresión corresponde a la corriente.
a) ¿Cuál es la corriente suministrada por le fem en el
En el instante en que se cierra la llave, t = 0 .
momento que se cierra la llave y cuál después de
largo tiempo? ⎛ 1 1 ⎞ (R + R2 )
b) Después de un tiempo largo t’ se abre la llave. I = ε ⎜⎜ + ⎟⎟ = ε 1
¿Cuánto tiempo tarda en disminuir la carga del ⎝ R1 R2 ⎠ R1 R2
condensador en un 90% con relación a la que tenía en Con los valores
t’?
I =6
(10 × 10 + 10 × 10 3
3
)
= 12 x 10-4 A
10 × 10 3 × 10 × 10 3
Mucho tiempo después, t = ∞ .
⎛1⎞ ε
I = ε ⎜⎜ ⎟⎟ =
⎝ R1 ⎠ R1
Con los valores
6
Solución. I= = 6 x 10-4 A
Cuando se cierra la llave circula la corriente tal como 10 × 103
se muestra a continuación. b) Después de un tiempo largo se abre la llave. En ese
instante la carga del condensador es Q0 , y el circuito
queda como se muestra a continuación.

Por le primera ley de Kirchhoff


I = I1 + I 2 (1)
Por le segunda ley de Kirchhoff Aplicando la segunda ley de Kirchhoff
ε − I 1 R1 = 0 (2)

21
Corriente continua Hugo Medina Guzmán

q q en una caja con solo los terminales y la escala visibles


IR1 + IR2 + = 0 ⇒ I (R1 + R2 ) + = 0 viene a ser un amperímetro o un voltímetro.
C C
1 Ejemplo 37. Con el galvanómetro, proyectar un
⇒ I+ q=0
(R1 + R2 ) amperímetro de 0 a 1 Ampere.
Solución.
dq Para tener un amperímetro de 0 a lA es necesario que
Con I = :
dt el galvanómetro marque una desviación máxima de 1
A. Como esto sucede para una corriente Ig = 1 mA,
dq 1
+ q=0 es preciso hacer un desvío a la corriente como se
dt (R1 + R2 ) muestra en la siguiente figura.
Cuya solución es
q = Q0 e −t ( R1 + R2 )C
Cuando la carga disminuye en un 90% queda el 10%
Q0
de la Carga o sea, q = .
10
Q0
= Q0 e −t ( R1 + R2 )C ⇒ t = (R1 + R2 )C ln 10
10
Poniendo valores Esto se logra conectando una resistencia en paralelo
( )
t = 10 × 10 3 + 10 × 10 3 10 −6 (2,3) llamado shunt (Rsh ) , cuyo valor se determina como
= 4,6 x 10-2 s. sigue:
La diferencia de potencial entre a y b, es
INSTRUMENTOS Y DISPOSITIVOS DE Vab = I g R g = (I − I g )Rsh ⇒
MEDICION
Amperímetros y Voltímetros. Ig
Rsh = Rg
Los dispositivos que miden, la corriente, la diferencia
de potencial en un circuito son el amperímetro y el (I − I ) g
voltímetro, respectivamente. con los datos
La parte principal de estos instrumentos es un 10 −3
Rsh = 20
(1 − 10 −3 ) = 0,020002Ω
Galvanómetro, que es un aparato que sirve para
detectar el paso de pequeñas corrientes. El tipo mas
común es el Galvanómetro de D’Ansorval, funciona Se debe de conectar en paralelo al galvanómetro una
basado en el principio de que una bobine por la cual resistencia de 0,020002Ω y se tendrá un
circula corriente y que está en el interior de un campo
magnético experimenta la acción de un torque amperímetro 0 - l A entre los terminales a y b.
proporcional al paso de la corriente de tal modo que Un amperímetro ideal debe tener una resistencia cero,
la lectura en la escala es proporcional a la corriente ya que se conecta en serie al circuito que se quiere
que pasa por él. medir.

Ejemplo 38. Con el mismo galvanómetro proyectar


un voltímetro de 0 a 6 Voltios.
Solución.
Para tener un voltímetro de 0 a 6V es necesario que el
galvanómetro marque a desviación máxima 6 Voltios,
como esto sucede cuando pasa una corriente 1 mA, es
preciso aumentar la resistencia de éste, esto se logra
mediante una resistencia en serie, como se muestra
en la figura a continuación.

Los parámetros mas importantes que se deben


conocer son la resistencia del galvanómetro (Rg) y la
corriente que produce le máxima desviación en la
aguja del galvanómetro (Ig), un ejemplo típico de
valores es una resistencia R g = 20Ω y una
La diferencia de potencial entre a y b, es
desviación máxima para una corriente de 1
Vab = (Rsh + R g )I g ⇒ Rsh =
miliampere (Ig = 1 mA). Vab
− Rg
Este instrumento conectado en la forma conveniente Ig
con una resistencia de determinado valor y montado
con los datos

22
Corriente continua Hugo Medina Guzmán

6 Substituyendo nuevamente dentro de las ecuaciones,


Rsh = − 20 = 5980Ω obtenemos
10 −3 5000 5000
Se debe conectar en serie al galvanómetro una R1 = = 1667Ω y R2 = = 2500Ω
resistencia d e 5980 Ω y se tendría un voltímetro 0 – 3 2
6 V entre los terminales a y b.
Un voltímetro ideal debe tener una resistencia infinita Ejemplo 40. El valor de una resistencia se mide
ya que se conecta en paralelo al circuito que se quiere usando un voltímetro y un amperímetro. Cuando el
medir. voltímetro se conecta directamente a través de la
resistencia, las lecturas obtenidas son 50 V y 0,55 A.
Ejemplo 39. Un banco de las pilas que tienen una Cuando el voltímetro se conectado a través del
fuerza electromotriz total de 12 V y una resistencia amperímetro y de la resistencia, las lecturas son 54,3
interna insignificante está conectado en serie con dos V y 0,54 A. La resistencia del voltímetro es 1000 Ω .
resistencias. Un voltímetro de resistencia 5000 Ω se Encuentre el valor de la resistencia y la resistencia del
conecta alternadamente a través de las resistencias, y amperímetro.
da las medidas 4 V y 6 V, respectivamente. ¿Cuáles Solución.
son los valores de las resistencias? Sea el valor de la resistencia R y la resistencia del
Solución. amperímetro r. La primera conexión se muestra en el
diagrama siguiente.

El voltímetro está conectado a través de R1 como en el


diagrama (a), y es equivalente al circuito mostrado en
Por la primera ley de Kirchhoff,
el diagrama (b), donde
I 1 + I 2 = 0,55A .
1 1 1 5000 R1
= + ⇒ R= Por la segunda ley de Kirchhoff,
R R1 5000 5000 + R1 Vab = I 1 R = 1000 I 2 = 50V ⇒
Puesto que 4 V es la caída a través de la resistencia R
y 8 V a través de la resistencia R2, tenemos: 50 1
I2 = = A
4V = IR y 8V = IR2 . 1000 20
5000 R1 R e I 1 = (0,55 − 0,05)A = 0,5A ⇒
Luego R = = 2 50V
5000 + R1 2 R= = 100Ω
20A
El segundo método de conexión se demuestra en

Similarmente, de los diagramas (c) y (d), muestran la


segunda conexión del voltímetro y del circuito diagrama siguiente.
equivalente, tenemos Aquí 54,3 V = 0,54 A x (R + r).
5000 R2 51,3V
R' = y 6V = IR1 = I ' R ' Luego r = − R = 100,56 − 100 = 0,56Ω
5000 + R2 o,54A
5000 R2
Luego R ' = = R1 , Ejemplo 41. Un galvanómetro cuya resistencia es 9,9
5000 + R2
Ω se le coloca una resistencia shunt de 0,1 Ω ,
Por lo tanto, de las dos ecuaciones obtenidas, tenemos cuando se utiliza como amperímetro con la
10000 R1 = 5000 R2 + R1 R2 y desviación a escala completa de 5 A. ¿Cuál es la
corriente del galvanómetro que lleva en la desviación
5000 R2 = 5000 R1 + R1 R2
máxima? ¿Qué resistencia se debe utilizar y cómo
Restando estas ecuaciones, obtenemos debe ser conectada si el galvanómetro va a ser
2 utilizado como voltímetro con la desviación a escala
15000 R1 = 10000 R2 ⇒ R1 = R2 completa de 50 V?
3
Solución.

23
Corriente continua Hugo Medina Guzmán

Cuando el galvanómetro se utiliza como amperímetro


debe conectarse tal como se muestra en el siguiente Ohmímetro. Es un instrumento que sirve para medir
diagrama. resistencias rápidamente, consta de una pila y una
resistencia en serie Rsh como se muestra en la figura
a continuación.

Por la primera ley de Kirchhoff,


I1 + I2 = 5A
Por la segunda ley de Kirchhoff,
I1 1
− 9,9 I 1 + 0,1I 2 = 0 ⇒ = ⇒
I 2 99
El valor de Rsh está dado de tal manera que el
I1 1
= galvanómetro marque desviación máxima al unirse a
I 1 + I 2 100 y b, lo que correspondería a una resistencia cero.
5A Sea R x la resistencia a medir, se conecta a los
Luego I 1 = = 50 mA .
100 terminales a y b y la ecuación del circuito es
Cuando el galvanómetro se utiliza como voltímetro ε − IRsh − IRx − IRg = 0
debe tener una resistencia en serie con él, como se
muestra en el diagrama siguiente. ε
I=
Rx + Rsh + Rg
Como el valor de I depende de R x y no tienen una
En la desviación a escala completa 50 mA afluyen a relación lineal y además depende de la constancia de
través del galvanómetro, según lo calculado en la ε , este instrumento no es de alta precisión pero es de
primera parte del problema. La caída de potencial a gran utilidad dada la rapidez de las lecturas.
través del galvanómetro debe por lo tanto ser
( )
V = IR = 50 × 10 −3 A (9,9Ω ) = 0,495V .
Ejemplo 42. Con el galvanómetro de ejemplos
anteriores proyectar un ohmímetro.
Pero 50 V caen a través de R y del galvanómetro. Así Solución.
49,505 V es la caída en la resistencia en serie. Por lo Usemos el galvanómetro con una pila común de
tanto tiene un valor. 1,5V. La deflexión máxima debe de producirse con
49,505V R x = 0 o sea
R= = 990,1Ω
50 × 10 -3 A ε
I=
MEDICION DE POTENCIAS Rx + Rg
Vab Con los datos
Como P = Vab I y R = , es necesario hacer la 1,5
I 10− 3 =
medición de Vab e I , para esto hay dos formas Rsh + 20
posibles de conectar el voltímetro y el amperímetro De donde
como se muestra en le figura siguiente. Rsh = 1480Ω
El galvanómetro hay que conectarlo en serie a una
pila de 1,5 Voltios y a una resistencia de 1480Ω ,
luego proceder a su calibración.

Puente de Wheatstone. Usando el circuito conocido


Forma a), en esta forma el voltímetro incluye la como Puente de Wheatstone se pueda medir
diferencia de potencial en el amperímetro, la que si es resistencias con exactitud. La figura (a) muestra un
pequeña (Resistencia de amperímetro muy baja) no esquema de este dispositivo.
necesitaría corrección.
Forma b) en esta forma el amperímetro incluye la
corriente que pasa por el voltímetro, si la resistencia
del voltímetro es muy alta la corriente debe ser muy
pequeña y no necesitaría corrección.

MEDICION DE RESISTENCIAS

24
Corriente continua Hugo Medina Guzmán

Consiste de un alambre AB de alta resistencia y Consiste de un alambre de alta resistencia AB, un


longitud 1 metro, un galvanómetro G con un terminal
de posición variable C, una resistencia conocida R, galvanómetro con resistencia interna R g , una fuerza
una pila ε y una resistencia RL limitadora de electromotriz ε , una resistencia limitadora R2 , una
corriente. La figura (b) muestra la distribuci6n de las
corrientes cuando se ha logrado que no haya paso de fuerza electromotriz patrón ε p y por supuesto la
corriente a través de l mediante la variación de la fuerza electromotriz por conocer εx con resistencia
posición C. Bajo estas condiciones tenemos:
I 1 R1 = I 2 R x y I 1 R2 = I 2 R4 interna ri .
Dividiendo miembro a miembro Se mueve el terminal variable hasta que el
R1 R x R galvanómetro marque cero (I 2 = 0 ) .
= ⇒ R x = R4 1 La diferencia de potencial entre C y B es
R2 R4 R2
VCB = IR1
Es aconsejable que el valor de sea del orden del valor
de la resistencia por conocer. También
Por otro lado, siendo uniforme el alambre que se usa VCB = I 2 (Rg + r ) − (− ε x ) = ε x
(mismo material e igual sección). De tal manera que
L L ε x = IR1
R1 = η 1 y R2 = η 2
A A Se repite la experiencia pero esta vez en lugar de la
Tenemos que fem desconocida ε x se pone la fem patrón ε p ,
R1 L1
= como I 2 es cero y el valor de (R1 + R2) es constante
R2 L2
el valor de I permanece igual, pero tenemos un
De aquí nuevo R1 que es R’1.
L1 ε p = IR'1
Rx = R4
L2 De estos resultados se ve que
Potenciómetro. Este dispositivo se usa para medir la R1
fuerza electromotriz de un generador sin que pase εx = ε p
corriente por él, La figura siguiente muestra un R'1
esquema de este dispositivo. Siendo R1 y R’1 el mismo alambre se puede decir que
L1
εx = ε p
L2

PREGUNTAS Y PROBLEMAS

1. Un alambre de cobre de sección transversal 3x10-6 2. La cantidad de carga (en C) que pasa a través de
m2 conduce una corriente de 10 A. Hallar la 2
una superficie de área 2cm varía con el tiempo
velocidad media de los electrones en el alambre.
como q = 4t + 5t + 6 , donde t está en s .
3
Datos: carga del electrón 1,6x10-19 C. Peso atómico
del cobre 63,5 g/mol, número de Avogadro 6,02 1023 a) ¿Cuál es la corriente instantánea a través de la
átomos/mol, se supone que cada átomo de cobre superficie en t = 1s ?
contribuye con un electrón libre a la conducción. b) ¿Cuál es el valor de la densidad de corriente?

3. La corriente I (en Amperes) en un conductor


depende del tiempo como I = 2t − 3t + 7 , donde
2

25
Corriente continua Hugo Medina Guzmán

t está en s ¿Qué cantidad de carga pasa a través de a) ¿Cuál es la velocidad de arrastre vd en nuestro
una sección del conductor durante el intervalo alambre cuando la corriente es de 1 A?
comprendido entre t = 2 s y t = 4 s ? Datos: densidad del cobre: 8,95 g/cm3; masa molar
del cobre: 63,5 g/mol; número promedio de
4. Corriente en la atmósfera: En la atmósfera inferior electrones móviles por átomo de cobre: 1,3.
de la Tierra existen iones negativos y positivos, b) ¿Cuál es la energía cinética (por encima de su
creados por elementos radioactivos en el suelo y en energía térmica) de los electrones en el alambre?
los rayos cósmicos del espacio. En cierta región, la c) Considerar los electrones que se encuentran en el
intensidad del campo eléctrico atmosférico es de 120 primer mm del alambre en un instante inicial.
V/m dirigido verticalmente hacia abajo. Debido a este ¿Cuánta energía potencial eléctrica pierden estos
campo, los iones con una sola carga e positiva, que electrones viajando hasta el final del alambre? La
son 620 por cm3, se dirigen hacia abajo con velocidad resistividad de cobre es ρ = 1,7 × 10-8 Ω m.
1,7 cm/s, y los iones con una sola carga negativa, -e,
550 por cm3, se dirigen hacia arriba con velocidad 1,7 8. En la «prospección eléctrica» los geólogos ponen
cm/s. dos electrodos en la superficie de la Tierra y miden la
a) ¿Cuál es la densidad de carga de los iones positivos resistencia entre ellos, la cuál depende de los
en el aire? materiales que hay entre los electrodos y por tanto
¿Cuál es la densidad de carga de los iones positivos brinda información sobre estos. En este problema
en el aire? vamos a tratar el caso más sencillo posible, en el cual
b) ¿Cuál es la densidad de corriente en el aire? que la Tierra consiste, hasta una profundidad debajo
c) ¿Cuál es la resistividad del aire según los datos de casi todas las corrientes, de una mezcla de arena
dados? con agua con conductividad de la mezcla σ 0 y
5. Un cable cilíndrico de Plata de 1 mm2 de sección y constante dieléctrica k = 1, y la superficie es un plano
5m de largo, conduce una corriente de 0,5A. infinito. Supongamos además que los electrodos son
Determinar: esferas de radio r separadas por una distancia R >> r.
a) La resistencia del conductor. Estas esferas están enterradas hasta sus ecuadores en
el suelo como muestra el diagrama.
b) La diferencia de potencial ΔV entre los extremos
del conductor.
c) El campo eléctrico E (uniforme) que determina
ΔV en el conductor. (Resistividad de la Plata
ρ = 1,59 × 10−8 Ωm ). a) Si los electrodos esféricos tienen carga Q y –Q
respectivamente y no hay acumulaciones de carga
6. Se tiene un cable de Nicromio de radio 0,321 mm. presente fuera de los electrodos ¿cuál es la diferencia
a) ¿Cuál es la longitud de este si tiene una resistencia de potencial V entre los electrodos?
de 28 Ω? b) Usando la Ley de Ohm microscópica y la Ley de
b) ¿Cuál es la ΔV entre los extremos de este cable si Gauss, ¿Cuál es la corriente total que entra en el
conduce una corriente de 4,3 A? electrodo negativo? (No olviden que I es el flujo de la

c) Calcular la densidad de corriente y el campo
eléctrico en el cable en el caso anterior.
densidad de corriente J: ∫ J ⋅ nˆdA = I
S
S es la

(Resistividad del Nicromio ρ = 1,5 × 10 Ωm ).


−6 corriente a través la superficie S en el sentido de la
normal n̂ .)
7. Al considerar la energía eléctrica en un circuito se c) ¿Cuál es la resistencia entre los electrodos? Si los
suele despreciar la energía cinética de las cargas electrodos están lejos uno del otro, la resistencia se
móviles. En este problema vamos a calcular esta acerca a un valor fijo independiente de la separación.
energía cinética para un alambre de cobre de longitud Es como si cada electrodo fuera conectado por un
1 m y diámetro 1 mm, que conduce una corriente de alambre de resistencia fija Rtierra a un conductor
1 A. perfecto (de resistencia cero) común. Esto es el
Los electrones móviles tienen un movimiento motivo por lo cual se puede usar la Tierra como un
aleatorio térmico de alta velocidad más una lenta cero de potencial: un conductor perfecto mantiene un
deriva debido a la diferencia de potencial entre los potencial uniforme bajo todas circunstancias. Como
extremos del alambre que da al electrón una se vio en c) esto funciona aun si la resistividad de la
velocidad promedio vd. Los electrones tienen una Tierra no es muy baja. Rtierra se llama la “resistencia
importante energía cinética media por el movimiento de conexión a Tierra”.
térmico. La deriva de un electrón debido a la d) ¿Cuál es la resistencia de conexión a Tierra de
diferencia de potencial agrega una cantidad ½ mevd2 a nuestros electrodos esféricos enterrados hasta su
su energía cinética por encima de su energía térmica, mitad? ¿Cómo se puede disminuir esta resistencia?
donde me es la masa del electrón.
9. Una resistencia real no tiene solo resistencia sino
también una capacidad. De hecho son las cargas

26
Corriente continua Hugo Medina Guzmán

acumuladas en la resistencia que producen el campo


eléctrico, y por tanto la diferencia de potencial, a 13. Un cable coaxial consiste en un cilindro metálico
través de esta. Supongamos que una cierta resistencia hueco de radios interior a y exterior b, y conduce la
consiste en un disco de carbón de grosor de radio 1 corriente en dirección radial desde el interior hacia el
mm y 0,5 mm de altura. Cada una de las caras esta exterior. Hallar la resistencia del cable.
unida con un alambre de radio 1mm de un metal de
resistividad despreciable (ver diagrama). 14. Si existe una caída IR de 1,5 V en un conductor
de cobre de 20 metros de longitud, hallar:
a) El campo eléctrico en el conductor.
b) La densidad de la corriente en el conductor.
c) La corriente en el conductor si la sección
transversal es igual a 2 mm2.

15. La región comprendida entre dos esferas


a) ¿Cuál es la resistencia del resistor? La resistividad concéntricas de radios a y b esta llena de un material
de carbón es de 3 × 10-5 Ωm. conductor de conductividad g. La esfera interior se
b) Supongamos que una corriente de 1A pasa por el mantiene a un potencial Va y la exterior a un potencial
conjunto, ¿cual es la diferencia de potencial entre los Vb de tal modo que existe una corriente radial hacia
bornes del resistor? Como la resistividad de los afuera, encontrar:
alambres es despreciable el campo eléctrico también a) La resistencia entre las esferas.
es despreciable en estos, y el potencial prácticamente b) La intensidad de corriente.
constante. Por lo tanto el potencial es constante sobre e) La densidad de corriente.
cada borne. d) La intensidad del campo eléctrico en un punto
c) ¿Cuál es el campo eléctrico en la resistencia? (El situado entre a y b.
campo eléctrico es uniforme).
d) Según la Ley de Gauss ¿cuál es la carga eléctrica 16. La región comprendida entre dos cilindros
en las caras del resistor? La constante dieléctrica del conducto res de radio interior a y exterior b esta llene
carbón es k = 2,7, entonces la cantidad de carga libre de una resistencia de resistividad η . El cilindro
(no de polarización) es mayor que la carga neta sobre interior esta a un potencial Va y el exterior a un
cada borne. ¿Cuánta carga libre hay sobre las caras de potencial Vb de modo que la corriente se dirige
la resistencia? radialmente hacia afuera, encontrar:
e) Se quiere modelar esta resistencia real con un a) La resistencia correspondiente a una longitud L.
dispositivo con dos bornes hecho de una resistencia b) La intensidad del campo eléctrico entre los
ideal (que tiene resistencia pero no acumula carga cilindros.
alguna en su interior) y un condensador ideal (que se
deja cargar pero que no deja pasar corriente alguna). 17. La resistividad de cierto material varía con la
Proponga un arreglo de estos elementos ideales que temperatura de acuerdo n la expresi6n:
η = η 20 (1 + 2t − t 2 ) .
modela (es decir, que se comporta de manera similar
a) el resistencia real. ¿A qué temperatura tendrá
f) En términos de la corriente y el voltaje a través del un alambre de este mate rial la resistencia mínima?
resistor real (las cantidades eléctricas más fácilmente
medíbles) ¿se les ocurre alguna forma en cómo se 18. Mientras una carga de 2 pasa por una cierta fem,
manifiesta el hecho de que el resistencia real funciona el trabajo realizado sobre la carga por fuerzas no
también como condensador? eléctricas es de 16 Joules. ¿Cuál es la fem de la
fuente?
10. Un alambre de cobre de resistividad ρ =1,7 10-8
19. Una batería de 6 voltios suministre 30 amperes
Ω m, tiene una longitud de 24 cm y una sección durante 3 segundos en el encendido de un motor de su
circular de diámetro 2 mm. Calcular la resistencia del automóvil. ¿Cuánta energía proporciona la batería?
alambre
20. Una refrigeradora conectada a 220 voltios
11. Un anillo de radio R tiene una carga por unidad de funciona durante 150 horas cada mes.
longitud λ . El anillo gira con una velocidad angular a) Si la corriente requerida pera el funcionamiento es
ω alrededor de su eje. Hallar la expresión que nos da de l.6A. ¿Cuánta energía consume?
le corriente en un punto del anillo. b) Comparar con el consumo de un receptor de
televisión que requiere 1,2A a 220 voltios y funciona
12. A una esfera metálica se le proporciona carga por durante 90 horas al mes.
medio de un alambre conductor de radio r, de acuerdo
− at 21. En el circuito simple de le figura. ¿Cuál es la
con la expresión q = q 0 e .
potencia que suministra cada fuente?
a) Encuentre la expresión para la corriente eléctrica. .
b) Calcule la densidad de corriente para el tiempo t.

27
Corriente continua Hugo Medina Guzmán

22. ¿Cuál es la diferencia de potencial entre los


puntos a y b de la figura.

27. En la figura. ¿Cuál es la diferencia de potencial


entre los puntos a y b cuando la llave S está abierta?
b) ¿Cuál es la diferencia de potencial entre a y b
cuando se cierra la llave, y cuánto cambia la carga de
cada condensador?

23. La resistencia larga entre a y b vale 32 Ω y está


dividida en cuatro partes iguales por tres tomas de
corriente.
a) ¿Cuál es la resistencia entre a y b?
b) Si la diferencia de potencial entre a y b es 220
voltios ¿cuál es la diferencia de potencial entre 2 y 3?

28. A un condensador de 0,l pF se le da una carga


Q0 . Después d 4 s se observa que su carga es Q0 .
¿Cuál es la resistencia efectiva a través de este
condensador?

29. Un conductor de capacidad 0,2 pF está aislado de


tierra por medio de una placa de silicio de 2,5 mm de
24. Hallar la resistencia equivalente entre los espesor y 5 cm2 de área. ¿Cuál es la resistividad
terminales a y b. mínima del silicio si la razón de disminución de
potencial no debe ser mayor que 0,1% por minuto?

30. Un tubo de neón se conecta e través de un


condensador de capacidad 25 μF el cual está siendo
cargado continuamente a través de una resistencia de
0,5 MΩ de una fuente de 2500V. Un flash de
duración despreciable descarga completamente al
condensador cuando el potencial a través del tubo de
neón alcanza 200V. ¿Cuántos flashes ocurren por
minuto y cuánta energía se disipa en cada descarga?
25. Encontrar la intensidad de corriente en cada una
de las ramas del circuito de la figura.
31. Un galvanómetro cuya resistencia es 9,9 Ω se
conecta con un-shunt de 0,1 Ω cuando se usa como
amperímetro de 0 a 5A. ¿Cuál es la corriente de
máxima deflexión?
¿Qué resistencia debe usarse pera tener un voltímetro
de 0 a 50V?

32. En el circuito de la figura. ¿Cuál es la lectura del


amperímetro?

26. ¿Cuál es la resistencia equivalente entre a y b?

28
Corriente continua Hugo Medina Guzmán

33. La figura muestra dos modos posibles para medir


una resistencia incógnita. 38. En el circuito de la figura, hallar la carga del
La resistencia del amperímetro es 100 Ω veces la del condensador C después de que el circuito ha estado
amperímetro. El valor calculado de R se considera conectado por mucho tiempo. Despreciar la
V resistencia interna de la batería.
que es Rc = , discutir en cuál de las formas se
I
obtiene un mejor resultado

39. Considere un manto cilíndrico de largo L, radio


34. Se tiene un hilo conductor de 1 m de longitud y medio R y espesor e (R >>e), con tapas en ambos
0,5 mm de radio, cuya extremos del mismo espesor e. El manto y las tapas
resistividad es 5×10-8 Ω m. Se aplica una diferencia están construidos de un material de conductividad g.
de potencial entre sus extremos de 5 V. Calcule: En el centro de ambas tapas se han soldado electrodos
a) el campo eléctrico en el interior del conductor; circulares de radios a (a < R) de un metal de muy alta
b) la densidad de corriente en el conductor; conductividad. Calcule la resistencia total entre
c) la velocidad efectiva de desplazamiento ambos electrodos.
de los portadores de carga.;

35. Hallar la resistencia entre los puntos a y b del


circuito de la figura. R1 = R5 = 1,00 Ω , R2 = R6= 2,00
Ω , R3 = R7 = 3,00 Ω  y R4= R8 = 4,00 Ω ..

40. En la figura se muestran dos tubos cilíndricos de


cobre de radios r1 y r2, el espacio entre ellos lleno de
grafito. ¿Cuál es la resistencia entre los terminales?
36. Para determinar el lugar de daño del aislamiento
Indicación: Como la conductividad a temperatura
entre los conductores de una línea bifilar telefónica de
ambiente del cobre es 105 mayor que la del grafito,
longitud L = 4,0 km a un extremo de esta se coloca
suponga que cada tubo de cobre es una equipotencial.
una fuente de fuerza electromotriz ε =15 V. Con
esto resulta que si los extremos opuestos están
separados, la corriente por la batería es I1 = 1,0 A, y
si se unen la corriente por la batería es I2 = 1,8 A. La
resistencia por unidad de longitud del conductor es
λ = 1,25 Ω /km. Hallar el punto en que se encuentra
el deterioro y la resistencia del aislamiento en dicho
punto.
La resistencia de la batería se desprecia.

37. Encontrar la condición que deben satisfacer las


resistencias R1, R2 y R3, para que el arreglo de la 41. En el circuito de la figura, calcule la corriente I
figura sea equivalente eléctricamente al arreglo de la que circula por la rama central.
figura derecha conformado por las resistencias r1, r2 y
r3.

29
Corriente continua Hugo Medina Guzmán

42. En el circuito de la figura la batería tiene una


resistencia interna de 1 Ω . El punto c está conectado
a tierra. Encuentre los potenciales en los puntos b y d.

45. Se ubican tres resistencias R1, R2, R3 cada una de


10 Ω en tres de los brazos de un puente de
Wheatstone y una cuarta resistencia R4 es ajustada en
el cuarto, de modo que el puente quede balanceado
(es decir, no circula corriente por el amperímetro
ideal de la figura). Luego se reemplaza la resistencia
R3 por una resistencia Rx y el balance se recupera
poniendo en paralelo con R4 una resistencia de
43. Considere un tren de juguete como se ilustra en la 10,123 Ω . ¿Cuál es el valor de Rx?. Discuta las
figura. Los rieles son circunferencias concéntricas de ventajas y desventajas de este método para medir
radios a y b respectivamente. Están conectados a una resistencias cuando se requiere gran precisión.
batería V0. Suponga que los rieles tienen una
resistencia por unidad de longitud λ . El tren a su vez
se puede modelar como una resistencia R.
La posición del tren queda descrita por el ángulo θ
de la figura.
a) Para un ángulo fijo encuentre el circuito
equivalente de este sistema.
b) Para un ángulo fijo encuentre la corriente I (θ ) que
circula por el tren (es decir, por la resistencia R).
c) ¿Para que posición angular la corriente es máxima?
¿Para que posición es mínima?
46. Una línea de transmisión consiste de un cable
sostenido por postes. Si la resistencia del cable entre
dos postes es r y la resistencia de fuga hacia tierra en
cada poste es R, calcule la resistencia de la línea de
transmisión.
Indicación: Considere que la tierra es un conductor
perfecto.
Note que como la línea es muy larga, la resistencia no
se altera al agregarle un poste y un segmento de cable
más.

44. En el circuito de la figura, muestre que una


condición de balance (es decir que no circule
corriente por el amperímetro ideal) que es
independiente del valor de r es:
R1 R3 R5
= = 47. Un cubo tiene una resistencia R en cada una de
R2 R4 R6 sus aristas.
Este es el doble puente de Kelvin que se utiliza para a) Calcule la resistencia entre dos vértices opuestos.
medir resistencias pequeñas, del orden de 0,01 Ω . b) Calcule la resistencia entre dos vértices opuestos
La resistencia r representa una resistencia de contacto de una cara del cubo.
entre las dos resistencias R1; R2, y su valor no afecta
el balance. 48. Demostrar que las corrientes en el circuito de la
figura se distribuyen de modo que la pérdida de
energía en forma de calor es mínima.

30
Corriente continua Hugo Medina Guzmán

de 3 Ω y una batería de 3 V cuya resistencia interna


es de 1/3 Ω . ¿Cuál deberá ser el valor de R si la
lámpara ha de funcionar al voltaje diseñado?

52. La carga en un condensador de placas paralelas ha


caído hasta el 95% de su valor original después de un
49. En el circuito de la figura, R1 = 3 Ω , R2 = 5 Ω , día debido a las pérdidas a través del dieléctrico. Si la
R3 = 4 Ω , cada batería tiene una resistencia interna resistividad y la permitividad del dieléctrico son η
de 0,5 Ω , V1 = 3 V, V2 = 5 V y V3 = 7 V. Calcular las y ε respectivamente, ¿cuál es la magnitud de ηε ?
corrientes en las diferentes resistencias y los
Respuesta. 1,68 × 10 ΩF
6
potenciales en las cuatro esquinas del cuadrado.

53. Una batería de fuerza electromotriz 4,5 V y


resistencia interna 0,3 Ω está conectada en paralelo
con una segunda batería de fuerza electromotriz 4,0V
y resistencia interna 0,7 Ω . El sistema común
proporciona la corriente para una resistencia externa
de 10 Ω .

¿Cuál es la diferencia potencial a través de la


resistencia externa y la corriente que atraviesa de
cada batería?
50. En el circuito de la figura, R3 = 8 Ω , R4 = 1 Ω  y Respuesta. 4,26 V; 0,978 A; - 0,372 A
R5 = 3 Ω . Encuentre las corrientes I1, I2 e I3.
54. Encuentre la resistencia equivalente de la red
mostrada en el dibujo.

51. Un foco de alumbrado de 0, 4 W se diseña para Respuesta. 2,27 Ω


que trabaje con 2 V entre sus terminales. Una
resistencia R se coloca en paralelo con la bombilla y
la combinación se coloca en serie con una resistencia

31
Campo magnético Hugo Medina Guzmán

CAPÍTULO 3. Campo magnético

INTRODUCCION fenómeno tiene dos partes, primero la generación


Otro campo que entra en el estudio de la del campo magnético y segundo la influencia del
electricidad y el magnetismo es el campo campo magnético sobre cargas móviles. Por ahora
magnético. Los efectos de estos campos son solo nos ocuparemos de esta segunda parte.
conocidos desde tiempos antiguos. En la Grecia →
antigua era conocido que ciertas piedras DEFINICIÓN DE CAMPO MAGNETICO B
procedentes de Magnesia (ahora denominadas El campo magnético se define por sus efectos
magnetitas) atraían trocitos de hierro. sobre una carga en movimiento. Supongamos una
región del espacio que contiene varias fuentes
magnéticas. Los experimentos que incluyen la
observación de las trayectorias de las partículas
cargadas que se desplazan por esa región
demuestran que la fuerza que actúa sobre ellos
tiene las características siguientes:

F es directamente proporcional a la carga (q)

F es directamente proporcional a la magnitud de
la velocidad de la partícula (v)
→ →
Magnetita
F es perpendicular a v en toda la trayectoria de
la partícula.
El descubrimiento de la propiedad de orientación
Debido a las propiedades experimentales
de este material en sentido norte-sur, influyó
mencionadas podemos definir el campo
profundamente en la navegación y exploración.
magnético que se asocia a las fuentes dadas
mediante la relación
Aparte de esta aplicación el magnetismo tuvo → → →
poco uso y no fue explicado hasta cuando se F = q v× B
inventó la pila voltaica. Le pila proporciona

corrientes continuas del orden da amperios, con A B se le conoce también como:
tales corrientes se descubrieron nuevos procesos Campo magnético vectorial
une detrás de otro en rápida sucesión que Inducción magnética
relacionaron el magnetismo con la electricidad. Densidad de flujo magnético
Como habíamos visto en la interacción de dos
cargas eléctricas estática la existencia del campo La magnitud de F está dada por qvBsenθ ,
eléctrico, cuando estas partículas cargadas están → →

en movimiento aparece un cambio en el siendo θ el ángulo entre v y B .


movimiento debido a una fuerza que no es El campo magnético B está dado por la relación
mecánica ni electrostática, es la fuerza de F
interacción magnética y depende de las B=
qv
velocidades relativas de las partículas cargadas, de
la carga de cada una, de la dirección relativa del En el sistema MKS, la unidad de B es
N N
movimiento y de la distancie entre las cargas.
=
m Am
C
s
A esta unidad se le conoce como tesla (T)
Otra denominación de esta unidad es
Weber Wb
= 2
m2 m
En la figura anterior mostramos dos cargas en Wb
movimiento, estas experimentan fuerzas T= 2

m
magnéticas F . Una carga eléctrica en También se usa el Gauss (G)
movimiento genera un campo magnético la otra G = 10 -4 T
carga eléctrica móvil sufre una fuerza debido a la Para tener una idea de la magnitud del Gauss
influencia de dicho campo sobre ella. Este daremos algunos ejemplos de campos magnéticos.

1
Campo magnético Hugo Medina Guzmán

De la Tierra es del orden de 0,5 G sobre la partícula cargada es que siempre es


De un imán pequeño 100 G perpendicular a la velocidad de la partícula.
De un imán grande 20000 G Cuando el campo magnético es uniforme y la
De un acelerador de partículas 60000 G →
Como la fuerza magnética sobre una partícula velocidad inicial es perpendicular a B , tanto la
carga da se presenta además de la fuerza eléctrica fuerza corno la veloci1ad quedan en un plano fijo
con la formulación obtenida es posible escribir →

una expresión para la fuerza total experimentada perpendicular a B . Como la fuerza es constante

por una partícula cargada. La fuerza
en magnitud y siempre perpendicular a v el
electromagnética total sobre la partícula cargada
es la suma vectorial de las fuerzas magnéticas y movimiento es circular uniforme como se muestra
las fuerzas eléctricas, es decir, a continuación.

⎛→ → →⎞
F = q⎜ E + v × B ⎟
⎝ ⎠
Esta ecuación es conocida como la ley de Fuerza
de Lorentz.

EL FLUJO MAGNETICO
De la misma manera que en la teoría del campo

eléctrico, asociaremos el campo magnético B a Por la segunda ley de Newton
un flujo magnético; A las líneas del campo
magnético se las llama líneas de inducción, una
∑F r = ma r = qvB
carga eléctrica moviéndose a lo largo de una línea v2
de inducción experimenta una fuerza magnética a r es la aceleración centrípeta
igual a cero. Las líneas así definidas y trazadas en
r
De aquí
el espacio constituyen una representación del
campo magnético. Cuando el campo tiene v2
intensidad uniforme se representa por líneas
qvB = m
r
rectas, uniformemente espaciadas, tal casa se El radio es
muestra a continuación.
mv
r=
qB
v
La velocidad angular de la partícu1a ω= es:
r
qB
Si se construye un área S normal a la superficie, ω=
el flujo se define como: m
Φ B = BS Ejemplo 1. Medida de la relación
q
para
Si el campo no es uniforme y la superficie no es m
uniforme, usaremos la expresión general electrones (experimento de Thomson)

dΦ B = B⋅ nˆ dS
el sentido de n̂ es hacia afuera para superficies
cerradas.
El flujo neto a través de cualquier superficie es La figura muestra el tubo de Thomson usado para

q
Φ B = ∫ B ⋅ nˆ dS la medición para la medición de .
S
m
El flujo magnético se mide en unidades de campo Los electrones proceden del cátodo C que c
magnético por unidad de área o sea Weber (Wb). encuentra a una diferencia de potencial V con el
Se hará uso de ésta porte cuando discutamos la ánodo A.
inducción electromagnética. Los electrones pasan a través de las rendijas A y
B con una velocidad que se calcula de la siguiente
MOVIMIENTO DE UNA CARGA PUNTUAL manera:
EN UN CAMPO MAGNETICO La carga q al moverse entra el cátodo y el ánodo
→ → →
que se encuentran a una diferencia de potencial
De la ecuación F = q v × B se observa que una
ΔV gana una cantidad de energía igual al
característica de la fuerza magnética que actúa

2
Campo magnético Hugo Medina Guzmán

producto qΔV , la que se convierte en energía


1 2
cinética E K = mv , siendo m la masa de la
2
carga y v la velocidad adquirida. De esto
obtenemos la relación
1 2
qΔV = mv
2
2 q ΔV
y de aquí v =
m En la figura se muestran los elementos de un
Cuando este haz de electrones ingresa en un espectrómetro, la sección entre A y C actúa como
campo magnético perpendicular forma una se-lector de velocidades y pasan por la ranura C
trayectoria circular cumpliéndose la relación solo las partículas con velocidad común v.
mv Al salir de C estas partículas entran en una región
= qB , de donde →
r en la que hay un campo magnético constante B 0
qBr
v= perpendicular al plano de la figura, las partículas
m forman una trayectoria circular hasta que chocan
Igualando ambas expresiones para la velocidad: con la pantalla o algún instrumento detector. La
distancia a la que se detecta la partícula está dada
2qΔV qBr
= por
m m 2mv
Finalmente
2R = , por debajo e C
qB0
q B2r 2
= E ⎛ E ⎞
m 2ΔV Como v = ⇒ R = m⎜⎜ ⎟⎟
Siendo las cantidades B, r y ΔV susceptibles de
B ⎝ qBB0 ⎠
→ →
q Sendo conocido q, y pudiendo conocerse E , B
medición se encuentra la relación para el
m →
electrón y B 0 , es posible determinar la nasa m con la
q C medición de R.
= 1,76 × 1011
m s Ejemplo 3. Fuerzas sobre cargas eléctricas. Un
También puede utilizarse los efectos de campos ión de masa m1 con carga eléctrica q se acelera
magnéticos y eléctricos en la misma región. Si los desde el reposo por medio de una diferencia de
campos y la velocidad son mutuamente potencial V. Luego el ión entra en una zona de
perpendiculares entre sí y además se ajustan las campo magnético uniforme B perpendicular a su
magnitudes de los campos de tal forma que velocidad, por lo cual es desviado en una
E trayectoria semicircular de radio R1.
qvB = qE , tendremos que v = . Esta Después de esta experiencia, un segundo ión e
B
carga 2q y masa m2 se acelera a través de la
medición puede combinarse con las otras a fin de
misma diferencia de potencial V y se le desvía
q mediante el mismo campo magnético B dando
obtener el valor de .
m Como resultado una trayectoria semicircular de
radio R2 = 2 R1. ¿Cuál es la relación de las masas
Ejemplo 2. El espectrómetro de masas. m1 y m2 de los iones?
El espectrómetro de masas es un aparato que Solución.
utiliza los principios anteriores para medir la masa
de los isótopos. Mide la razón q/m de los iones,
determinando la velocidad de estos y luego
midiendo el radio de su órbita circular en el
interior de un campo magnético uniforme

3
Campo magnético Hugo Medina Guzmán

Causa una fuerza sobre la partícula cargada


→ → →
F = q v× B

Ejemplo 5. Se lanza una partícula se lanza


horizontalmente con una velocidad de 104 m/s en
tal dirección que se mueve perpendicularmente a
un campo magnético horizontal, de magnitud 4,9
x 10-5 Wb.m2. La partícula, que lleva una sola
carga electrónica, permanece en el mismo plano
horizontal. ¿Cuál es su masa?
Solución.
Para el ión 1: Puesto que la partícula permanece en el mismo
1 plano horizontal durante su movimiento, la fuerza
qV = m1v12 (1), magnética en ella debe equilibrar su peso. Puesto
2 que el movimiento es perpendicular a la dirección
v2 de la inducción magnética, se deduce que
qv1 B = m1 1 (2)
R1

⎛→ →⎞
− m g = q⎜ v × B ⎟ , y así mg = qvB y
Para el ión 2: ⎝ ⎠
1 qvB
2qV = m2 v 22 (3), m= .
2 g
v 22 Reemplazando valores:
qv 2 B = m2
2 R1
(4)
m=
(1,6 × 10 )(10 )(4,9 × 10 )
−19 4 −5

Dividiendo (1): (3): 9,8


1 m1 v 2 = 8,0 x 10-21 kg.
= 1
2
(5)
2 m2 v 2 Ejemplo 6. Un haz de electrones acelerado por
Dividiendo (2): (4): una diferencia de potencial de 300 V, se introduce
v1 m 2v 2 en una región donde hay un campo magnético
= 1 21 uniforme perpendicular al plano del papel y hacia
2v 2 m2 v 2 el lector de intensidad 1,46 10-4 T. La anchura de
1 m v la región es de 2,5 cm. Si no hubiese campo
⇒ = 1 1 (6) magnético los electrones seguirían un camino
4 m2 v 2 rectilíneo.
Dividiendo (6)2: (5): a) ¿Qué camino seguirán cuando se establece el
campo magnético?
1 m12 v12 b) ¿Cuánto se desviarán verticalmente al salir de
2 2
16 = m2 v 2 la región? Razónese las respuestas
Datos: masa del electrón 9.1 10-31 kg, carga 1,6
1 m1 v12 10-19 C.
2 m2 v 22 Solución.
a)
m 1
⇒ 1 =
m2 8
La relación de las masas m1 y m2 es 1/8.

Ejemplo 4. ¿Es posible diseñar o plantear un


campo magnético capaz de modificar la
trayectoria y velocidad de una partícula cargada a
nuestra conveniencia? ¿Cómo? o ¿por qué?
Solución.
Con campos eléctricos y campos magnéticos. El haz de electrones acelerado por una diferencia
Campos eléctricos de potencial de 300 V adquiere una velocidad que
Causa una fuerza sobre la partícula cargada se obtiene por:
→ →
F = qE
Campos magnéticos.

4
Campo magnético Hugo Medina Guzmán

1 2 1 2 a través de un selector de la velocidad, ambos


q.V = mv f − mvi ⇒ iones del litio tienen la misma velocidad en el
2 2 campo. Además, tienen la misma carga y la
1 misma densidad magnética del flujo. Luego R6/m6
(1,6 × 10−19 )300 = 9,1 × 10− 31 v 2 ⇒ = R7/m7.
2
m R6 m 6 6
v = 1,03 × 107 ∴ = = = 0,857 .
s R7 m 7 7
Cuando se establece el campo magnético: Si los iones han pasado a través de un campo
→ → → eléctrico fuerte, ambos han adquirido la misma
F B = q v × B , FB = qVBsen 90º energía. Pero, de la ecuación anterior, tenemos
Por la segunda ley de Newton: 1 2 q 2 B2r 2
v 2 mv = .
ma c = FB ⇒ m = qVB ⇒ 2 2m
r r62 r72 r m6
mv ∴ = o 6 = = 0,926 .
r= = 0,4 m m6 m7 r7 m7
qB
Ejemplo 8. Cierto tipo de selector de velocidades
consiste en un par de las placas paralelas entre las
cuales se establece un campo eléctrico E. Un haz
de partículas de la masa m, carga q, y velocidad v
es dirigido paralelamente a las placas en la región
entre ellas. Se aplica un campo magnético B
→ → → perpendicular a E y a v. en la figura B se dirige
F B = q v × B , FB = qvBsen 90º hacia el papel, según lo indicado. Determine una
expresión para la velocidad de las partículas que
Por la segunda ley de Newton: man = FB ,
no son deflectectadas por este dispositivo.
v2 mv
m = qvB ⇒ r = = 0,4 m
r qB
b) Al salir de la región deja de actuar el campo
magnético y el haz de electrones continúa con su
última dirección.
Solución.
La fuerza magnética qvB balancea la fuerza del
campo eléctrico qE, tal que:
B
qvB = qE y v = .
E
0,025
senθ = , Ejemplo 9. Un electrón es acelerado por una
r diferencia de potencial de 300 V, entra en una
d = r − r cos θ = 7,82 x 10-4m. región donde hay un campo eléctrico producido
por las placas de un condensador de 40 cm de
longitud y separadas 4 cm a las cuales se les
Ejemplo 7. En un tipo de espectrómetro las
aplica una diferencia de potencial de 100 V.
partículas cargadas pasan a través de un selector
a) Calcular el punto de impacto o la desviación
de velocidades antes de ingresar al campo
del electrón a la salida de las placas.
magnético. En otras las partículas pasan a través
b) Ahora, aplicamos hay un campo magnético
de un campo eléctrico fuerte antes de ingresar al
perpendicular al plano. Determinar la intensidad y
campo magnético. Compare el cociente de los
el sentido (hacia dentro o hacia afuera) del campo
radios de iones simples de litio cargados de masas
magnético para que el electrón no se desvíe.
6 amu y 7 amu en los dos casos.
c) Se suprime el campo eléctrico, determinar el
Solución.
radio de la órbita del electrón. Dibujar su
En el campo magnético un ion se mueve en un
trayectoria. ¿Chocará contra las placas?
círculo, la fuerza centrípeta necesaria es
Razónese todas las respuestas haciendo los
proporcionado por la fuerza magnética sobre él.
esquemas correspondientes.
v2
Así qvB =m . Cuando los iones han pasado
R

5
Campo magnético Hugo Medina Guzmán

→ → → →
F B = q v × B , F B es de signo contrario a
→ → →
v × B , ya que la carga es negativa. Luego B
debe de ser perpendicular al plano del papel y
hacia adentro.

Datos: carga del electrón 1,6x10-19 C,


masa 9,1x10-31 kg.
Solución.
a) Para calcular la velocidad del electrón.
Por conservación de la energía
1 2 1 2 FE = FB , qE = qvBsen90º
qΔV = mv 2 − mv1 , ΔV = 300 V ,
2 2 E
⇒ B = = 2,43 × 10 − 4 T
v 2 = v0 , v
v1 = 0 1,6 × 10 −19 (300) =
1
9,1 × 10 −31 v02 c) Cuando se suprime el campo eléctrico.
2
m
⇒ v0 = 1,027 × 10 7
s
La fuerza debido al campo eléctrico constante
ΔV ' 100 N
FE = qE , E = = = 2500
d 0,04 C Por la segunda ley de Newton
El movimiento del electrón en presencia del FB = ma N
campo magnético es parabólico tal como el que se
muestra en la figura siguiente: v2
qE = qvBsen90º = m
r
mv
⇒ r= = 0,24 m
qB
Punto de impacto: 0,02 = 0,24 − 0,24 cos θ
⇒ 0,02 + 0,24 cos θ = 0,24 ⇒ θ = 23,6º
x = 0,24senθ ⇒ x = 0,096 m
Este movimiento esta dado por:
⎧a x = 0 Ejemplo 10. Se tiene un campo magnético

⎪ uniforme tal como se muestra en la figura, una
a ⎨ FE 2500 29 m
⎪a y = m = 9,1 × 10 −31 = 2,75 × 10 s 2
partícula de masa m y carga q ingresa
⎩ perpendicularmente con velocidad v. ¿Cuál es la
trayectoria cuando abandona el campo magnético?
→ ⎧v = v
⎧ x = v0 t
x 0


v ⎨ y r ⎨ 1
⎩v y = a y t ⎪⎩ y = 2 a y t
2

Para x = 0,4 m ⇒ y = 0,033 m , impacta


antes de salir
El punto de impacto es cuando
y = 0,02 m ⇒ x = 0,098 m
b) Para que el electrón no se desvíe.
Solución.
Al ingresar la partícula en el campo magnético
esta toma una trayectoria circular cuyo radio es
mv
R=
qB

6
Campo magnético Hugo Medina Guzmán

Si el valor de R es menor que L las partícula sale La frecuencia:


en sentido contrario al que ingreso tal como se
fc = =
(
1 qB 1,6 × 10 −19 (1,4) )
( )
nuestra en la figura a continuación. = 21 MHz
2π m 2π 1,67 × 10 − 27

Ejemplo 12. Un espectrómetro de masas es un


instrumento usado para separar los iones de masas
ligeramente diferentes. Éstos son a menudo
isótopos de un elemento, ellos tienen
características químicas muy similares. La
construcción de un espectrómetro de masas se
Si el valor de R es mayor que L muestra en la figura. Los iones de carga + q y
masa m se aceleran con una diferencia potencial
V0. Los iones luego se mueven en un campo
magnético perpendicular B, donde forman una
trayectoria semicircular. Se detectan a una
distancia d = 2r de la puerta de la entrada.
Determine la masa del ion en términos de los
parámetros conocidos.

mv
el radio sigue siendo R =
qB
la ecuación de la trayectoria de la partícula es
x 2 + ( y − R) = R 2
2

El punto de salida es cuando x = L y corresponde


a
L2 + ( y − R ) = R 2 ⇒
2

y − R = (R 2 − L2 )
12
Solución.
⇒ 1 2 mv 2
mv = qV0 y qvB =
y = R + (R 2 − L2 )
12
2 r
( )
2 12
Resolviendo:
la salida es en x = L , y = R + R − L
2
qr 2 2
La inclinación está dada por el ángulo θ . m= B
Derivando con respecto a x la ecuación de la
2V0
trayectoria: B puede ser variado para hacer que diversas
masas lleguen al detector.
2 x + 2( y − R )
dy dy x
=0 ⇒ =− Solamente la componente de la velocidad
dx dx y−x perpendicular al campo magnético es cambiada
Esta pendiente corresponde a tan θ por la fuerza magnética. Por lo tanto, una
partícula que se mueve en ángulo con excepción
dy x
tan θ = =− de 90° al campo magnético se moverá en una
dx y−x trayectoria helicoidal.
En el punto salida
L Ejemplo 13. Sea la región del espacio x ≥ 0 , en
tan θ = −
(R 2
− L2 )
12 la cual existe un campo magnético uniforme

B = B0 kˆ . Desde una posición x < 0 se dispara
Ejemplo 11. ¿Un protón (masa 1,67 x 10-27 kg) se una partícula de carga positiva q con una
mueve a lo largo de un arco de radio 32 cm →
cuando se mueve perpendicular a un campo velocidad v = v0 x iˆ + v0 y ˆj .
magnético de 1,4 T. ¿Cuál es la frecuencia del a) Calcule el vector Fuerza Magnética que actúa
ciclotrón y la cantidad de movimiento del protón? sobre la partícula en el punto de ingreso a la
Solución. región de campo magnético.
La cantidad de movimiento:
mv = qBr = (1,6 x 10-19C)(1,4T)(0,32m) b) Para el caso que las componentes iniciales v0 x ,
= 7,17 x 1020kg.m/s y v0 y , sean iguales y positivas. Trace

7
Campo magnético Hugo Medina Guzmán

(esquemáticamente) la trayectoria de la partícula mv


en la región de campo magnético. ¿Cuál es el r= ,
vector velocidad de la partícula al salir de la qB
región de campo magnético? 1 2 1 r 2e2 B 2
Solución. K = mv =
2 2 m
( )
→ → →
a) F = q v × B = qB0 kˆ × v0 xiˆ + v0 y ˆj = ( )
1 (0,012 ) 1,6 × 10 −19 (0,60 )
2 2 2

(
qB0 v0 x ˆj − v0 y iˆ ) =
2 (
1,67 × 10 − 27 )
b) Al ingresar la carga en la región del campo 3,97 × 10 −16 J
magnético tiene una trayectoria circular sobre el = = 2480 eV
1,6 × 10 -19 J eV
v02x + v02y
plano xy, el radio está dado por R=
qB Ejemplo 16.Un electrón se mueve con una de
velocidad 3,2 x 105 m/s en la dirección positiva de
El gráfico muestra el caso en que v0 x = v0 y x en presencia de un campo magnético

B = 0,1iˆ + 0,3 ˆj − 0,2kˆ (en teslas). ¿Qué
fuerza experimenta el electrón?
Solución.
→ → →
F = q v× B
( )[( ) (
= − 1,6 × 10 −19 2 × 10 5 iˆ × 0,1iˆ + 0,3 ˆj − 0,2kˆ )]
= 0,96 × 10 −14 kˆ − 0,64 × 10 −14 ˆj
F = Fx2 + Fy2 = 1,1 × 10 −14 N

Ejemplo 17. Un ión de litio cargado tiene una


Se puede observar que a en la salida del campo el masa de 1,16 x 10-26 kg. Se acelera con un voltaje
vector velocidad continuará con el sentido que de 600 V y después entra en un campo magnético
tenía en ese instante (tangente a la trayectoria). perpendicular a su velocidad de 0,60 T . ¿Cuál es

el radio de la trayectoria del ion en el campo
v salida = −v0 xiˆ + v0 y ˆj
magnético?
1 2
Ejemplo 14. Un selector de velocidades se Solución. mv , qV =
puede construir usando el principio siguiente: Los 2
iones de carga q y masa m mueven hacia arriba mv 1 2mV
comenzando en el origen con la velocidad v 0 a r= =
qB B q
un ángulo θ con el eje de z. Un campo
magnético B se establece a lo largo del eje de z. =
1 (
2 1,16 × 10 −26 (600 ) )
= 0,016 m
Determine el primer punto donde los iones 0,60 1,6 × 10 −19
regresan al eje de z.
Solución.
Ejemplo 18. Iones positivos, con carga simple se
Los iones se mueven en una trayectoria helicoidal
aceleran a través de una diferencia potencial y
y volverán al eje de z después de un período de la
entran en un campo magnético uniforme normal a
frecuencia del ciclotrón. Durante este tiempo
su línea de movimiento. ¿Si una diferencia
viajarán una distancia z = v0 cosθ T a lo largo potencial de 1000 voltios trae Li6 al detector, qué
del eje z. diferencia potencial haría que los iones Li7
1 qB 1 2π mv0 cos θ atraviesen la misma trayectoria?
fc = = y z= Solución.
2π m T qB En el campo magnético la fuerza que actúa en los
iones provee la fuerza centrípeta necesaria para
Ejemplo 15. En una emulsión fotográfica el trazo mantenerlos en una circunferencia. Por lo tanto
de un protón que se mueve perpendicular a un mv 2
campo magnético de 0,60 T se observa que es una = qvB .
circunferencia de radio 1,2 cm. ¿Cuál es la R
energía cinética del protón en electronvoltios?
Solución.

8
Campo magnético Hugo Medina Guzmán

qRB 1 2 q 2 R 2 B 2 mv 2 eB eB 2
∴ v= o mv = b) De = evB ⇒ m = R= R
m 2 2m R v E
Pero esta energía cinética es adquirida pasando a
través de una diferencia de potencial V. Ejemplo 20. En un horno a altas temperaturas una
1 2 q R B 2 2
qR B 2 2 2 muestra de sodio es calentada al punto de
∴ qV mv = oV = evaporarse, el gas de sodio resultante está
2 2m 2m formado por átomos que perdieron un electrón
Para ambos iones, q, R, y B son iguales. Por lo (carga del electrón, - e).
k k
tanto V1 =y V2 = ,
m1 m2
V m
∴ 2 = 1 ⇒
V1 m2
m ⎛6⎞
V2 = 1 V1 = ⎜ ⎟(1000 ) = 857 V. a) ¿Cuál debe ser la relación entre los módulos de
m2 ⎝7⎠ → →
E y B en la región I, de manera que sólo los
Ejemplo 19. La figura siguiente representa el átomos con velocidad exactamente igual a v0
dispositivo diseñado por Bainbridge para medir sigan sin desviarse? Suponga que la masa de los
exactamente masas de isótopos. S es la fuente de átomos es m0
iones cargados positivamente del elemento b) Si queremos que los átomos se detengan
investigado. Todos los iones tienen la misma exactamente cuando lleguen a la placa cargada
carga e pero tienen una gama de velocidades. A con σ , ¿cuál es el espesor de la región II?
través de la región un campo magnético uniforme c) Considerando que el campo magnético de la
→ →
B dirigida ingresando perpendicularmente a la tierra B H , está también presente en ambas

regiones, en la misma dirección en la que se
página. Además, un campo eléctrico E , dirigido
mueven las partículas cargadas. ¿Influirá en la
paralelo al plano de la página, se instala entre los
trayectoria de los átomos cargados? ¿Cómo?
electrodos P y N.
Solución.
a) Demuestre que solamente los iones que
E
velocidad y iguala E/B emergerán en C. a) qv0 B = qE ⇒ = v0
b) Demuestre que la masa m de un ion es B
proporcional al radio R de su trayectoria
1 σ m v 2ε
semicircular. b) m0 v02 = e ΔL ⇒ ΔL = 0 0 0
2 2ε 0 e
c) No, porque el campo magnético de la tierra

BH , está en la misma dirección en la que se
mueven las partículas cargadas

Ejemplo 21. .El elemento estaño se analiza en un


espectrómetro de Bainbridge. Están presentes los
Solución. isótopos de masas 116, 117, 118, 119, y 120 u.
a) De acuerdo a la figura anterior los iones que Los campos eléctricos y magnéticos E = 20 kV/m
viajan de S a C está sujetos a una fuerza eléctrica y B = 0,25 T. ¿Cuál es el espaciamiento entre las
→ → marcas producidas en la placa fotográfica por lo
F e = q E y una fuerza magnética iones de estaño 116 y los iones de estaño -120?
→ → → →
Solución.
F m = q v × B . Aquí E se dirige hacia la Del problema anterior, la distancia x del punto C a
→ →
la imagen de un isótopo se da por
derecha (de P a N) y v × B señala en la 2E
x = 2R = m por lo tanto,
dirección opuesta y tiene una magnitud vB . eB 2
Estas fuerzas se cancelan cuando qE = qvB y
E
v= .
B

9
Campo magnético Hugo Medina Guzmán

2E Del problema anterior el período del movimiento


Δx = Δm = 1 2π m
eB 2 circular uniforme es T= = . Para la
2(2,0 × 10 4 ) f qB
(4u )⎛⎜1,66 × 10 −27 kg ⎞⎟
(1,6 × 10 ) ⎝
−19
u ⎠ mitad de un círculo t =
T πm
= y es
= 2,66 x 10-2mm. 2 qB
independiente de la velocidad. Si el medio
Ejemplo 22. Una partícula con la carga q y masa período del campo eléctrico oscilante es igual a
m orbita alrededor perpendicular a un campo este tiempo, la partícula cargada será acelerada
→ otra vez cuando cruce el espacio entre las Des
magnético uniforme B . Demostrar que su nuevamente, debido a la dirección inversa del
BQ campo eléctrico. Así ganará energía. Esto hace
frecuencia del movimiento orbital es Hz. que el semicírculo siguiente tenga un radio más
2π m grande, según como se muestra en la figura. El
El hecho de que la frecuencia es independiente de aumento de energía se puede repetir muchas
la velocidad de la partícula es importante en los veces.
aceleradores de la partícula llamados ciclotrones;
esta frecuencia se llama la frecuencia del ciclotrón Ejemplo 24. Un ciclotrón tiene una frecuencia del
Solución. oscilador de 11,4 MHz y un radio de 60 cm.
2π r v a) ¿Qué intensidad de campo magnético se
El periodo es , tal que f = .
v 2π r requiere para acelerar los protones de la masa 1,67
x 10-27 kg y carga 1,6 x 10-19 C,
mv 2 qB
Usando qvB = , tenemos f = . b) ¿Cuál es la energía final que adquieren?
r 2π m c) ¿Qué error se comete si se asume que la masa
de los protones sigue siendo constante?
Ejemplo 23. Describa un ciclotrón y su Solución.
operación. a) La frecuencia angular de los protones en el
Bq Bq
ciclotrón es, ω= o f = .
m 2πm

B=
( )(
2π fm 2π 11,4 × 10 6 1,67 × 10 −27
=
)
q 1,6 × 10 −19
= 0,748Wb/m2.
b) La energía final de los protones es
1 2 q2B2R2
mv = =
(
1,6 × 10 −19 (0,748) (0,6 ) )
2 2 2

2 2m (
2 1,67 × 10 − 27 )
0,154 × 10 −11 J
Solución. = 0,154 x 10-11 J =
Un ciclotrón es un dispositivo para acelerar 1,6 × 10 -13 J/MeV
partículas nucleares. El corazón del aparato = 9,64 MeV.
consiste en una caja metálica partida fortín. La c) Desde E = mc2, esta energía es equivalente a un
figura muestra las vistas lateral y superior de las incremento de masa
mitades llamadas Des. Una diferencia potencial
0,154 × 10 −11 J
oscilante se aplica entre las Des. Esto produce un Δm = = 0,017 x 10-27 kg.
campo eléctrico oscilante en el espacio entre el 9 × 10 m /s
16 2

Des, la región dentro de cada D que esencialmente Luego el error es


está libre de campo eléctrico. Las Des se Δm 0,017
encierran en un envase evacuado, y la unidad × 100 = × 100 =1,02%.
entera se pone en un campo magnético uniforme m 1,67

B con dirección normal al plano de las Des. Una Ejemplo 25. Un ciclotrón está acelerando los
partícula cargada de la masa m y carga q en el deuterones los cuales son núcleos de hidrógeno
espacio entre las Des es acelerada por el campo pesado que llevan una carga + e y tienen una masa
eléctrico hacia uno de ellos. Dentro de las Des, se de 3,3 X 10-27 kg.
mueve con velocidad constante en un semicírculo. a) ¿Cuál es la frecuencia requerida del campo
eléctrico oscilante si B = 1,5 T.

10
Campo magnético Hugo Medina Guzmán

b) Si los deuterones deben adquirir el 15 meV de a) La energía cinética máxima se parte en dos:
energía cinética y la diferencia de potencial a ⎡⎛ 1 ⎞2 ⎤
través de la separación es 50 kV, cuántas veces el ⎢⎜ ⎟ ⎥
⎛1⎞
K max ( p ) = ⎢ ⎝ ⎠ ⎥ K max (d ) = ⎜ ⎟ K max (d )
deuterón experimenta la aceleración? 2
Solución. ⎢ ⎛1⎞ ⎥ ⎝2⎠
a) El período de la oscilación del campo eléctrico ⎢⎜ ⎟⎥
debe igualar al período orbital, así que la ⎢⎣ ⎝ 2 ⎠ ⎥⎦
frecuencia requerida de la oscilación es
(1,6 × 10 −19 )(1,5) = 11,6
(b) La energía cinética máxima se duplica:
1 qB ⎡ ⎤
f = =
2π (3,3 × 10 − 27 )
=
T 2π m0 ⎢ 1 ⎥
MHz. K max ( p ) = ⎢ ⎥ K max (d ) = 2 K max (d )
⎢⎛ 1 ⎞ ⎥
⎢ ⎜⎝ 2 ⎟⎠ ⎥
b) El deuterón al cruzar la separación, gana 50
keV = 5 x 104eV. Para ganar un total de 15 meV ⎣ ⎦
= 15 x 106 eV, el deuterón debe experimentar (15
x 106)/(5 x 104) = 300 travesías de la separación. Ejemplo 28. .En un experimento de resonancia
de ciclotrón, el campo magnético se dirige hacia
Ejemplo 26. Un ciclotrón se ha ajustado para arriba. Los resultados indican que las partículas
acelerar deuterones. Debe ahora ser ajustado para cargadas están circulando a la izquierda según la
acelerar, que tienen casi exactamente la mitad de vista de arriba. ¿Cuál es el signo de la carga en
la masa del deuterón. las partículas?
a) ¿Qué cambio debe ser realizado si no hay Solución.
cambio en la frecuencia, de la diferencia potencial Negativo (la fuerza debe estar dirigida al centro
oscilante aplicada entre el Des? del círculo).
b) ¿Qué cambio debe ser realizado si cambio en
campo magnético normal aplicado a las Des? FUERZA SOBRE UN ALAMBRE CON
Solución. CORRIENTE.
a) La frecuencia angular del ciclotrón es Cuando las cargas eléctricas se mueven en un
qB
ω= , así tenemos:
conductor que esté en un campo magnético, existe
una fuerza sobre el conductor que es la suma de
m
mω las fuerzas magnéticas sobre las partículas
B= cargadas en movimiento.
q
Desde el protón y el deuterón tienen la misma
1
carga q p = qd y m p = md , el campo
2
magnético debe ser disminuido a la mitad.
mω la figura muestra un conductor de sección A por el
b) Referente a la ecuación B= . Si B debe que pasa una corriente I y se encuentra en un
q →

permanecer invariable, la frecuencia de la campo magnético B , producido por fuentes


oscilación del campo debe ser duplicada. magnéticas diferentes. Tomemos un elemento
infinitesimal dl del alambre, consideremos que
Ejemplo 27. ¿Cómo cada uno de los cambios en el flujo de corriente se debe a N portadores de
el problema anterior altera la energía máxima que carga por unidad de volumen, cada uno de los
los protones pueden adquirir? cuales se desplaza con velocidad v d en la
Solución.
dirección de la corriente, por consiguiente la carga
Si se asume que la mecánica no relativista es
aplicable a través del movimiento, la energía total que participa es ΔQ = qNAdl siendo q la
cinética máxima carga de cada portador.

1 2 mv mnax
K max = mv max y qv nax B = , donde La fuerza d F sobre el elemento dl podemos
2 R expresarla como
R es el radio R del dispositivo. (la cantidad R es → → → → →
un límite superior para el radio orbital de una d F = (ΔQ ) v d × B = qNA v d × B
partícula acelerada). Resolviendo para K max , En esta expresión podemos hacer un intercambio

1 q2B2R2 entre dl y v d donde el sentido vectorial sin
encontramos que K max = .
2 m

11
Campo magnético Hugo Medina Guzmán

→ → → →
alterar la expresión o sea usar en cambio d l y F = F 1 + F 2 = IB(L + 2 R ) ˆj

v d ( d l con el sentido de I). Ejemplo 30. Una espira de alambre cuadrada de
→ → → 10 cm de lado yace en el plano xy tal como se
d F = qNAv d d l × B muestra en la figura. Se aplica un campo
La corriente es I = qNAv d , por consiguiente magnético paralelo al eje z, que varía a lo largo
del eje x de la forma B = 0,1 x T (donde x se
→ → →
expresa en metros).
d F = I d l× B a) Calcular el flujo del campo magnético que
Que es la expresión para calcular la fuerza atraviesa la espira.
magnética sobre un alambre con corriente. Para b) La fuerza (módulo, dirección y sentido) sobre
una longitud dada L, la fuerza es: cada uno de los lados de la espira.
→ → →
F = I ∫ d l× B
L

Ejemplo 29. ¿Cuál es la fuerza sobre el alambre


mostrado en la figura siguiente?

Solución.
a)
Solución.

La figura consta de dos partes, la parte recta y la


parte curva.
La fuerza sobre la parte recta es
→ L → →
F1 = I ∫ d l × B
0
→ →
Donde d l = dxiˆ , B = − Bkˆ

F 1 = I ∫ (dxiˆ ) × (− Bkˆ ) = I ∫ Bdxˆj = IBLˆj


→ L L

Φ = ∫ B ⋅ nˆdS = ∫ BdS cos 0º =
0 0
La fuerza sobre la parte curva es
→ π → → 0 ,1
F2 = I∫ d l × B 2
(0,1x )(0,1dx ) = 0,01 x
0 ,1
0

∫0 2
= 5 x 10-5Wb

d l = Rdφ (senφ iˆ − cos φ ˆj ) ,


0
Donde
→ b)
B = − Bkˆ .
F 2 = I ∫ Rdφ (senφ iˆ − cos φ ˆj ) × (− Bkˆ ) =
→ π

IRB ∫ (senφ ˆj + cos φ iˆ )dφ = IRB(2 ) ˆj =


π

2 IRBˆj
La fuerza total es

12
Campo magnético Hugo Medina Guzmán

Lado AB: B = 0 ⇒ F = 0
Lado CD: B1 = 0,1(0,1) = 0,01 T
⇒ F1 = (10)(0,1)(0,01)sen90º = 0,01 N , en
el sentido negativo de x

F 1 = −0,01iˆ
Para los lados BC y AD las fuerzas son iguales y
de sentidos opuestos, como B no es constante hay
que calcular la fuerza sobre un elemento dx y
luego por integración la fuerza total sobre el lado. Del primer diagrama, vemos que el campo
magnético debe ser perpendicular al plano de las
barras y debe actuar hacia abajo. La magnitud de
la fuerza experimentada por el alambre y los
accesorios es
F = IlB = (0,1)(10 )(0,049 ) = 0,049 N

Para el lado BC: dF = (10 )(dx )(0,1x )sen 90º =


xdx
0 ,1
0 ,1 x2
F2 = ∫ xdx = = 5 × 10 −3 N , fuerza
0 2 0
aplicada en el extremo C del lado BC.

Del segundo diagrama, vemos en que las fuerzas
−3
F 2 = −5 × 10 ˆj que actúan sobre el alambre y los accesorios son
tres: el peso que actúa verticalmente hacia abajo,
Para el lado AD: Es igual a F2 , pero de sentido la fuerza F, y N, la reacción normal. Puesto que
opuesto, aplicada en el extremo D de AD. el conjunto se mueve con velocidad uniforme,
→ N = mg cos θ y F = mgsenθ .
F 3 = 5 × 10 −3 ˆj
0,049
∴ senθ = = 0,02
Ejemplo 31. Un príncipe científico ha encontrado (0,25)(9,8)
un método de enviar mensajes secretos a una h
princesa hermosa que ese encuentra prisionera de Del diagrama, = 0,02
un ogro travieso en el piso superior de su castillo L
a 15 m del suelo. El príncipe coloca dos barras h 15
ligeras del metal (demasiado ligeras para ser Luego L = = = 750 m
usadas para subir) contra el travesaño de la
0,02 0,02
ventana, y entre las barras él monta un alambre 10 Llevar tales barras sería absolutamente una
cm de largo, en el cual pone el mensaje y un imán, hazaña. ¡El Príncipe mejor haría en recurrir a los
de tal manera que el alambre está servicios de una buena bruja!
permanentemente en un campo magnético de
fuerza 0,049 Wb/m2, perpendicularmente al plano Ejemplo 32. Un alambre que está a lo largo del
de las barras. Cuando él pasa una corriente de 10 eje de x lleva 2,0 A. La corriente fluye en la
A por una barra, a través del alambre que conecta dirección positiva de x. Un campo magnético de
y vuelve por la otra barra, el mensaje, el alambre 1,2 T paralelo al plano xy y que forma un ángulo
y el imán viajan con velocidad uniforme hacia de 30° con el eje de x (apuntando al primer
arriba de las barras. El conjunto móvil tiene una cuadrante). ¿Cuál es la fuerza sobre un segmento
masa de 0,25 kg. Despreciando la fricción, de alambre de 0,40 m de longitud?
calcule cual debe ser la longitud de las barras.
Solución.

Solución.

13
Campo magnético Hugo Medina Guzmán


F = BIL senθ kˆ F = (0,15T )(1,5A )⎛⎜ ˆj ∫ dx − iˆ ∫ dy ⎞⎟ =
3 4

= (1,2 T)(2 A)(0,40 m) sen 30º k̂ ⎝ 0 0 ⎠


= 0,48 k̂ N (
0,225 3 ˆj − 4iˆ )
La magnitud de la fuerza es
F = (0,225) 3 2 + (− 4 ) = 1,13 N
Ejemplo 33. Un riel electromagnético para lanzar 2
puede ser construido como sigue: Una barra
conductora de masa m sobre dos carriles Observe que no importa la trayectoria exacta del
conductores horizontales paralelos separados una alambre, desde que el alambre zigzaguea hacia
distancia L. Una fuente de poder hace circular una adelante y hacia atrás, las fuerzas se cancelan en
corriente I por los rieles y la barra A de la las partes que retroceden. La fuerza total es
distancia para atravesar los carriles y la barra. Se justamente la qué resultaría si el alambre fuera en
mantiene un campo magnético vertical uniforme línea recta de (0, 0) a (3, 4).
B. ¿Si la barra está inicialmente en reposo, cuál
será la velocidad después de que haya movido Ejemplo 36. Un lazo circular de alambre de radio
una distancia x? Se ha sugerido que este R lleva una corriente I. Un campo magnético
dispositivo se podría utilizar para proyectar cargas uniforme B actúa perpendicularmente al plano del
útiles en órbita alrededor de la tierra, o transportar lazo. ¿Cuál es la tensión en el alambre?
el mineral de la superficie de la luna a una fábrica Solución.
en el espacio, o inducir reacciones de fusión La fuerza en la mitad superior del lazo es
nuclear con choques de alta velocidad. equilibrada por la fuerza de la tensión en cada
Solución. extremo del semicírculo. Usando la regla
⎛F⎞ derecha, se ve que la fuerza magnética está
v 2 = v02 + 2ax = 0 + 2⎜ ⎟ x ⇒ dirigida radialmente hacia fuera. Por simetría la
⎝m⎠ fuerza resultante en el lazo está dirigida en la
2 Fx dirección z, donde Fz = F cos θ .
v=
m
Como F = BIL :
2 BILx
v=
m

Ejemplo 34. Un alambre recto que está a lo largo


del eje de x y que lleva una corriente de 2,0 A en
π
la dirección positiva de x. Un campo magnético
uniforme de 0,08 T en el plano xy hace un ángulo
Fz = ∫ BIdl cos θ , donde dl = Rdθ
0
π /2
BIR cos θ dθ = 2 BIR(senθ )0
de 60° con el alambre. Determine la magnitud y
Fz = 2 ∫
π /2
la dirección de la fuerza magnética en un 0
segmento del 1,5 m del alambre.
= 2 BIR = 2T
Solución.
De aquí T = BIR
F = BILsenθ
= (0,08)(2 )(1,5)sen60º Ejemplo 37. ¿Un alambre recto largo que lleva
= 0,21 N en la dirección positiva de z. una corriente I tiene una "torcedura semicircular"
en ella de radio R. ¿Cuál es el campo magnético
Ejemplo 35. Un alambre que está en una en el centro del semicírculo?
superficie horizontal en el plano xy lleva 1,5 A.
Un extremo del alambre está en el origen y el otro
está en (3 m, 4 m). El alambre sigue una
trayectoria errática a partir de un extremo al otro.
Un campo magnético de 0,15 T dirigido
verticalmente hacia abajo está presente. ¿Que Solución.
fuerza magnética actúa en el alambre? Las secciones rectas no contribuyen nada a B, ya
Solución. que para ellas θ = 0º y dxsenθ = 0 . La
Divida la trayectoria en pasos pequeños dx hacia contribución del semicírculo es justamente la que
la derecha y dy hacia arriba. La fuerza en cada corresponde a la mitad de un círculo completo,

( ) 1 μ0 I

segmento es d F = BIdxˆj + BIdy − iˆ . La B= . Luego
fuerza total luego es: 2 2R

14
Campo magnético Hugo Medina Guzmán

μ0 I Un lazo cuadrado da el torque máximo. (Un lazo


B= circular da aún más torque para una longitud dada
4R de alambre.)

FUERZA Y TORQUE SOBRE UNA ESPIRA


Ejemplo 38. En un altoparlante un imán CON CORRIENTE
permanente crea un campo magnético de 0,12 T Consideremos una espira rectangu1ar de lados a y
dirigido radialmente hacia fuera del eje de z. La b, situada en un campo magnético uniforme tal
bobina del altoparlante tiene 60 vueltas y un radio como se muestra en la figura siguiente.
de 0,013 m y se coloca en el plano xy. ¿Qué
fuerza actúa en la bobina cuando lleva una
corriente de 1,5 A?
Solución.
F = NBILsenθ
= (60 )(0,12 )(1,5)(2π )(0,013)senθ
= 0,88 N

Ejemplo 39. Un alambre con masa por unidad de


longitud 0,04 kg/m lleva 3 A horizontalmente al Sobre los miembros verticales (lados de longitud
este. ¿Qué campo magnético mínimo se requiere → →
para equilibrar este alambre contra la fuerza de la b) actúan las fuerzas F 1 y F 2 que son iguales y
gravedad? opuestas cuyo efecto es tratar da abrir la espira o
Solución. cerrarla en caso de invertir la corriente, nosotros
F = BILsenθ = mg , m = λL , consideramos una aspira rígida indeformable de

B=
λLq
=
(0,04)(9,8) = 1,13 T tal manera que no causan efecto alguno.
Sobre los miembros horizontales (lados de
IL 3 → →
longitud a) actúan las fuerzas F 3 y F 4 , tal que
Ejemplo 40. Un lazo cuadrado de lado L y de n F3 = F4 = IaB .
vueltas lleva una corriente I. Un lado del
cuadrado está a lo largo del eje de z, y la corriente Estas fuerzas son iguales en magnitud y opuestas
fluye hacia abajo en este lado. El resto del lazo en sentido, formando así un par de fuerzas de
está en el cuadrante xy positivo, y el plano del valor
lazo hace un ángulo φ < 90° con el eje x. Un τ = F3bsenθ = IaBbsenθ
campo magnético B se dirige a lo largo del eje con ab = A (área de lo espira) podemos escribir
positivo de x. ¿Qué torque experimenta el lazo? τ = IABsenθ
¿Cuándo es visto de arriba, en qué dirección el Este par puede escribirse como producto vectorial
lazo tenderá para rotar? →
Solución. de n̂ nˆ
(normal a la superficie A) y B .
m = nIA = nIL2 → →
τ = IAnˆ × B .
τ = mBsenθ = nIL2 B cos φ →
ya que θ + φ = 90º Si llamamos momento magnético m a IAnˆ .

El lazo rotará en el sentido antihorario.
m = IAnˆ
Ejemplo 41. Un alambre de longitud L en forma El par viene a estar dado por
→ → →
de un lazo rectangular. ¿Lleva una corriente I.
¿Cuáles deben ser sus dimensiones para
τ = m× B
Esta expresión deducida para una espira
maximizar el torque en él cuando es colocado en
rectangular es válida para una espira de forma
un campo magnético?
cualquiera.
Solución. τ = mBsenθ , τ es máximo cuando
m = IA es un máximo, es decir, cuando A es un Ejemplo 42. Una bobina circular de alambre lleva
máximo. Sea x = longitud de un lado 50 mA de corriente. La bobina tiene 50 vueltas y
⎛L ⎞ un área de 2,0 cm2. Un campo magnético de
Luego A = ( x )⎜
− x⎟ , 0,300 T orientado paralelo al plano de la bobina
⎝2 ⎠ está presente. ¿Qué torque actúa sobre la bobina?
dA ⎛L ⎞ L Solución.
= 0 = ⎜ − x⎟ − x ⇒ x =
dx ⎝2 ⎠ 4

15
Campo magnético Hugo Medina Guzmán

Cincuenta vueltas que llevan 50 mA son c) Φ = BA cos 25º


= (1,2 )(0,4 × 0,3) cos 25º = 0,13 Wb
equivalentes a una vuelta que lleva (50)(50 mA),
tal que
τ = NIABsen θ = 50(2 x 10-4)(50 x 10- Ejemplo 44. Una bobina circular radio 4 cm y
3
)(0,30)sen 90° = 1,5 x l04 N.m
100 vueltas lleva una corriente de 1,2 A. En
Si uno intenta rotar un dipolo magnético en un
presencia de un campo magnético de 0,80 T,
campo magnético, un torque − μBsenθ debe orientado perpendicularmente al plano de la
θ
∫θ τBsenθdθ =
bobina. ¿Cuánto trabajo es requerido para dar una
aplicarse. Así U − U 0 = − vuelta de 180° a la bobina?
0

− mB(cos θ − cos θ 0 ) . Elegimos U 0 tal que


Solución.
W = − μB (cos 180° − cos 0°)
U = 0 cuando θ = 90º , luego
→ →
= 2 μB = 2 NAIB
U = − μB cos θ = − m⋅ B , energía potencial (
= 2(100 ) π 0,04
2
)(1,2)(0,8) = 0,97 J
de un dipolo magnético.
Aplicación 1: El Galvanómetro de D’ansorval.
Ejemplo 43. Un lazo rectangular rígido, que mide Como ya vimos anteriormente el Galvanómetro
0,30 m por 0,40 m, lleva una corriente de 2,0 A, consiste de un campo magnético producido por un
como se muestra. Un campo magnético externo imán permanente y una bobine de n espiras, la
uniforme de la magnitud 1,2 T en la dirección cual tiene libertad de rotar contra un torque
negativa de x está presente. El segmento CD está restaurador de un espiral de suspensión. El torque
en el plano xz y forma un ángulo 25° con el eje de de rotación es causado por la corriente I que fluye
z. por la bobina y es justamente la que queremos
medir.

a) ¿Cuál es el valor de la componente y de la


fuerza magnética en el segmento AB?
b) Un torque externo se aplica al lazo y lo El torque producido por el movimiento es
mantiene en equilibrio estático. ¿Cuál es la τ = nIAB cos α , siendo α el ángulo rotado
magnitud del torque externo? desde la posición cero de equilibrio.
c) ¿Cuál es el flujo magnético a través del lazo La rotación se detendrá cuando el torque
debido al campo externo? restaurador producido por el resorte se iguala al
Solución. torque magnético, para obtener la posición de
→ → → equilibrio α con la corriente que pasa tenemos
a) F = I l × B κα = nIAB cos α
( )
= 2,0 − 0,3 cos 25º kˆ + 0,3sen 25º iˆ × (− 1,2iˆ ) Si el ángulo de deflexión es pequeño, cos α ≈ 1 ,
= + (7,2 )(0,91) ˆj = +0,65 ˆj la deflexión α directamente proporcional a la
corriente I .
F = + 0,65 N

( ) ( )
→ → →
b) F = I l × B = 2,0 0,4 ˆj × − 1,2iˆ Ejemplo 45. Una bobina de un galvanómetro
tiene 500 vueltas de alambre enrollado alrededor
= + 0,96k̂ de un marco de 2 cm de largo y 1 cm de ancho.
Esta fuerza es producida en los dos lados largos La bobina rota en un campo magnético de 0,05
produciendo un par de fuerzas o cupla, para Wb/m2, siempre paralelo a su plano y
mantener en equilibrio se debe aplicar un torque perpendicular a su largo. ¿Qué torque actúa en la
opuesto a este, cuya magnitud es: bobina cuando lleva una corriente de 10-8 A?
τ = (0,96 )(0,3sen 25º ) = (0,288)(0,42 ) Solución.
La magnitud del torque que actúa en una sola
= 0,12 N m vuelta de la bobina es,

16
Campo magnético Hugo Medina Guzmán

τ = AIB cos α .
Cuando la bobina tiene n vueltas,
τ = nAIB cos α
= (500 )(0,02 × 0,01)(10 )(0,05)(1)
−8

= 5 x 10-11 N.m.

Ejemplo 46. La bobina de un galvanómetro tiene


150 vueltas de área media 1 cm2 y el par
restaurador de la suspensión es 10-6 N.m por
radián. La magnitud del campo magnético radial
en la cual la bobina gira es 0,2 Wb/m2. ¿Qué El motor a corriente continua consiste en una
desviación será producida cuando una corriente de armadura formada por varias vueltas de alambre,
10 μ A atraviesa la bobina? la cual se encuentra en un campo magnético B
Las placas de un condensador de 1 μ F se cargan uniforme, los motores pequeños utilizan imanes
a una diferencia potencial de 1 V y después se pera producir este campo y los grandes tienen
descargan a través de la bobina del galvanómetro, electroimanes para este fin. La armadura esta
la desviación resultante que es 0,1 rad. ¿Cuál es conectada al conmutador el cual es un anillo
el momento de la inercia de la bobina? dividido. La finalidad del conmutador es invertir
Solución. la corriente en la apropiada fase de rotación, tal
La magnitud del torque que actúa en las 150 que el torque sobre la armadura actúa en la misma
vueltas de la bobina debido al campo magnético dirección. La corriente es proporcionada por la
batería a través de un par de resortes o escobillas
es, τ = 150 AIB , el campo es radial.
que están en contacto con el conmutador, El
La bobina gira hasta que este torsión es
torque del motor está dado por τ = NIABsenθ .
balanceado por el torque restaurador de la
suspensión κθ . Así en la posición del equilibrio Cuando θ = 0 , no fluye corriente por la
armadura y es el instante justo en que se invierte
κθ = 150 AIB o
150(10 −4 )(10 −5 )(0,2)
la corriente, en el proceso momentáneamente no
150 AIB interviene la batería. Pero como el motor ya esta
θ= =
κ 10 −6 funcionando, la inercia rotacional hace pasar la
armadura a través de la región de torque cero.
= 0,03 rad.
Cuando se descarga el condensador, la carga que
EFECTO HALL
atraviesa la bobina del galvanómetro es
Q = CV = (10 −6 )(1) = 10-6 V
Un caso de efecto que produce un campo
magnético sobre una corriente es el fenómeno
Pero la carga y la desviación máxima resultante descubierto por E.H. Hall en 1879, conocido
del galvanómetro están relacionadas por la como Efecto Hall. Este efecto nos permite
⎛ κI ' ⎞ determinar el signo de la carga situada en el
ecuación Q = ⎜ ⎟
⎜ nAB ⎟θ max , donde I’ es el
portador y el número de portadores de carga por
⎝ ⎠ unidad de volumen (n) del conductor.
Para describir este fenómeno consideremos una
momento de inercia de la bobina. Así
lámina delgada de material conductor con sección
n 2 A2 B 2Q 2 A, ancho a y largo l .
I'= =
κθ max
2 Conectamos una fuente de voltaje como se

150) (10 − 4 ) (0,2) (10 −6 )


muestra en la figura a continuación, aparece un
I
( 2 2 2 2

(10 )(10 )
−6 −1 2
campo E y una cierta corriente I asociada a él,
los electrones se desplazan con su velocidad v d
= 9 x 10-10kg.m2
en una dirección opuesta al campo.

Aplicación 2: Motor de corriente Continua.


Consideremos un motor a corriente continua muy
simple como el motor en la figura siguiente.

17
Campo magnético Hugo Medina Guzmán

Si se conecta un voltímetro transversalmente a la ⎛ 1 ⎞ IB


lámina, dará una lectura cero, ya que el campo Et = ⎜⎜ ⎟⎟
eléctrico no tiene componente a lo largo de la ⎝ nq ⎠ A
dirección vertical.
Ahora pongamos un campo magnético dirigido Ejemplo 47. Una cinta de metal de 2 cm. de
perpendicularmente fuera de la lámina como se ancho y 0,1 cm. de espesor lleva una corriente de
muestra en la figura siguiente, la fuerza magnética 20A y está situada en un campo magnético
sobre estas partículas estará en la dirección perpendicular al plano de la cinta de 2,0 T. La
→ → fuerza electromotriz Hall se mide y resulta 4,27
q v d × B (hacia abajo en la figura). μV . Calcular:
Los electrones se moverán hacia abajo con a) La velocidad de desplazamiento de los
trayectoria curva. electrones en la cinta.
b) El número de portadores de carga por unidad
de volumen de la cinta.
Solución.
a) Como E t = v d B
Et V a
tenemos que, v d = =
B B
−6 −2
Siendo V = 4,27 × 10 V , a = 2 × 10 m ,
Como consecuencia de este movimiento las
B = 2,0 T .
cargas negativas se apilan en el fondo y en
compensación aparecen cargas positivas en la Reemplazando valores
parte superior. Como se muestra en la figura 4,27 × 10 −6 −4
siguiente. vd = = 1,06 × 10 m/s
2 × 10 −2 × 2
b) Como
I
vd =
nqA
I
Tenemos que n =
qAv d
−19
Siendo I = 20A , q = 1,602 × 10 C,
La apilación de cargas continuará hasta que la −4
fuerza producida por el campo eléctrico
v d = 1,06 × 10 m/s y
transversal cancele la fuerza magnética, es decir A = (2 × 10 −2 )(0,1 × 10 −2 ) = 0,2 × 10 −4 m 2
→ → →
q Et + q v d × B = 0 Reemplazando valores
20
n=
(1,602 × 10 )( )( )
Tomando en cuenta los sentidos y cancelando q,
−19
obtenemos 0,2 × 10 −4 1,06 × 10 − 4
Et = v d B = 58,8 × 10
28
portadores / m3
La existencia de Et queda evidente con el hecho
Ejemplo 48. Los semiconductores tales como el

⎛→ ⎞
de que ente la presencia de B ⎜ B ≠ 0 ⎟ , el silicio se pueden dopar con impurezas de modo
⎝ ⎠ que los portadores de la carga sean negativos (los
voltímetro en la figura marca una lectura V . electrones) o positivos (agujeros). Esta es una
característica importante en la construcción de
El valor de E t está dado por dispositivos como los transistores. En la figura se
V bosqueja una disposición para medir el efecto
Et = Hall. Tal medida puede determinar el signo y la
a densidad de los portadores y, cuando está
Como la corriente está dada por la expresión calibrado, se puede utilizar para medir la fuerza de
I un campo magnético.
I = nqAv d , obtenemos: v d =
nqA
Reemplazando esta expresión de v d en E t :

18
Campo magnético Hugo Medina Guzmán

Determine una expresión en términos de los


parámetros dados para el voltaje Hall medido
entre los puntos X e Y en el arreglo mostrado. El número de electrones libres por unidad de
Solución. Naρ
La fuerza magnética desvía a los portadores hacia volumen del cobre será. N =
arriba de la muestra hasta que un campo eléctrico Mm
E aumenta lo suficiente hasta cancelarse por la La fuerza electromotriz Hall es: ε = Ed = vBa
fuerza magnética. Cuando sucede esto qE = qvB I I
y el voltaje entre X e Y es V H = Ed = v d Bd . Además J = = Nqv ⇒ v =
A NqA
La corriente es I = nAqv , donde A = ad y n es IaB
la densidad de del portadores. Luego: De lo anterior: ε =
NqA
VH I 1 IB
= B ⇒ VH = IBM m
d nAq nq a ε= =
1/nq es el coeficiente de Hall. Si los portadores N A ρeh
son negativos, y la corriente es hacia la derecha, (5)(1,5)(63,5 × 10 −3 )
como en el dibujo, la velocidad del portador se
dirige a la izquierda, y la fuerza magnética empuja (6,02 × 10 )(8,985 × 10 )(1,6 × 10 )(2 × 10 )
23 3 −19 −3

−6
otra vez los portadores hacia arriba. En este caso = 0,3 × 10 V
V H es negativo, mientras que V H es positivo
para los portadores positivos. Así la medida del LEY DE AMPERE, LEY DE BIOT Y
coeficiente de Hall determina el signo de los SAVART
portadores y su densidad ya que casi siempre Después de los experimentos de Oersted en 1820
q = e. que describen el movimiento de les agujas de una
brújula por la acción de un alambre por el que
Ejemplo 49. En un experimento de efecto Hall circula corriente eléctrica, muchos científicos
una muestra de 12 mm de espesor se utiliza con trabajaron sobre este fenómeno. Jean Baptiste
un campo magnético de 1,6 T. Cuando pasa una Biot y Félix Savart anunciaron los resultados de
corriente de 10 A a través de la muestra, se las mediciones de la fuerza que actúa la aguja
observa un voltaje Hall de 0,080 V. ¿Cuál es la magnética en la cercanía del alambre con
densidad del portador, asumiendo q = e? corriente. André Marie Ampere amplió estos
Solución. experimentos y demostró que los propios
1 IB IB elementos de corriente experimentan una fuerza
VH = ⇒ n= en presencia de un campo magnético, demostró
nq a eVH a que dos corrientes ejercen fuerzas entre sí.
Reemplazando valores:

n=
(10)(1,6) LEY DE GAUSS PARA EL MAGNETISMO
(1,6 × 10 )(0,08 × 10 )(12 × 10 )
−19 −6 −3
= Como vimos en nuestros estudios de la
electrostática, la ley de Gauss y la existencia de
1,04 x 1029 m-3 una función de potencial determinan, en gran
parte, todas las características esenciales del
Ejemplo 50. Determinar la fuerza electromotriz campo electrostático. Hay dos leyes análogas,
Hall que se produce en una cinta de cobre denominadas ley de Gauss para el magnetismo y
(suponiendo para éste metal un electrón libre por ley de Ampere que desempeñan el mismo papel
átomo) de 0,2 cm de espesor, por la que circula para el campo B. La finalidad de esta sección y la
una intensidad de corriente de 5 A, cuando se que sigue es analizar esas dos descripciones muy
aplica un campo magnético uniforme de 1,5 T, importantes del campo de inducción magnética.
perpendicular a la cinta. Densidad del cobre 8,95 Como se verá en el capítulo que sigue, la
g/cm3, masa atómica 63,5 g. importancia de estas dos leyes se debe, en general,
Solución. al hecho de que son básicas para las ecuaciones de
Maxwell. De hecho, la ley de Gauss para el
magnetismo es una de las cuatro relaciones
básicas. Además, para cualquier distribución dada
de corrientes, las leyes de Ampere y Gauss,

19
Campo magnético Hugo Medina Guzmán

cuando se toman juntas, constituyen una


especificación completa del campo B en todas
partes. Por tanto constituyen la generalización
necesaria de la ley más restringida de Biot Savart,
que se aplica sólo a los flujos de corriente en
alambres delgados.
Por analogía con la definición del flujo eléctrico, Al colocar varias brújulas en los alrededores del
definimos el flujo magnético a través de una alambre estas se orientan tangencialmente a la
superficie S mediante circunferencia formada por la distancia radial al
→ alambre, figura (a). Al invertir la corriente se
Φ B = ∫ B ⋅ nˆ dS orientan tangencialmente pero en sentido
S
en donde dS es un elemento de área normal a S, contrario, figura (b).

B es el valor del campo magnético en ese punto y
la integral es sobre la superficie de S. En función
de esta cantidad, la ley de Gauss para el
magnetismo establece que el flujo magnético de
todas las superficies cerradas desaparece. Por
ende

∫ B ⋅ nˆdS = 0 , en donde dS es un elemento


S
vectorial de área dirigido hacia el exterior a partir
de la superficie cerrada S. La comparación con la En la práctica se adopta la regla de la mano
→ Q derecha orientando el pulgar con la corriente y la
ley de Gauss para el campo E, ∫ E ⋅ nˆdS = ε
S
0
, punta de los dedos con el campo magnético como
muestra la figura siguiente.
nos lleva a la conclusión de que no hay análogo
magnético para la carga eléctrica. Con frecuencia
describimos esto, diciendo que no hay monopolos
magnéticos. La validez de (25-11) se estableció
mediante un gran número de experimentos y a
pesar de las investigaciones continuas, nadie ha
detectado todavía la presencia de un monopolo
magnético
Una de las consecuencias más importantes de la
→ Se observó experimentalmente que al alejarse del
ecuación ∫ B ⋅ nˆdS = 0 es la de que todas las
S
alambre el campo disminuía y al acercarse
líneas de campo B tienen que ser continuas. Por lo aumentaba. Asimismo, el campo aumentaba con
tanto, en general, es cierta la propiedad de las el aumento de la intensidad de la corriente, es
I I
decir B ∝ o B = K , donde K es una
líneas de campo B, como se ilustra en la figura
abajo, de que siempre se cierran sobre sí mismas. r r
Tesla.m
constante igual a 2x10-7 Ampere en el sistema

MKSC.
μ0
También K = , donde μ0 es la constante de

permeabilidad en el vacío tiene un valor de
LEY DE AMPERE. Tesla.m
Hans Oersted en 1820 descubrió
μ 0 = 4π 10 −7
Ampere
experimentalmente que una corriente que circula
en un alambre produce efectos magnéticos sobre μ0
una brújula situada a su alrededor. Finalmente B = I
2πr
Esta expresión es conocida como la ley de
Ampere.
Debido a la dependencia radial de B y a que r es
constante en la circunferencia, podemos
expresarla en la siguiente forma

20
Campo magnético Hugo Medina Guzmán

→ → radio conduce la misma corriente, uniformemente


∫ B ⋅ d l = μ0 I distribuida, pero en sentido contrario.
a) Determínese, aplicando la ley de Ampere, la
Esta expresión es válida en general para cualquier
arreglo de conductores con corriente, para expresión de campo magnético producido por
cualquier trayectoria cerrada de integración y para cada una de las corrientes rectilíneas infinitas a
cualquier campo B, siempre y cuando éste no una distancia r, de forma separada.
varíe con el tiempo. La corriente I es la corriente b) Hallar el campo magnético (módulo, dirección
encerrada por la integral de línea. y sentido), en los puntos (13 cm, 0), y en el punto
Se puede escribir en función de le densidad de (0 cm, 4 cm) producido por las dos corrientes.
corriente. c) Por último, hallar la fuerza, (módulo, dirección
→ → → → y sentido) que ejerce el cable sobre la unidad de
Como ∫ J ⋅ nˆdS ⇒
S ∫ B ⋅ d l = μ 0 ∫ J ⋅ nˆdS
S
longitud del hilo rectilíneo.

Donde la integral de superficie de la densidad de


corriente corresponde al área encerrada por la
integral de línea cerrada.
La ley de Ampere tiene aplicación muy limitada
ya que solo puede evaluar problemas que tienen
simetría. Solución.
a) El campo magnético producido por el hilo
Ejemplo 51. Se tiene un conductor cilíndrico rectilíneo
largo y recto de radio - R que lleva una corriente 1
uniformemente distribuida. Calcular el campo
magnético para puntos dentro y fuera del alambre.
Solución.
a) Campo magnético para r > R .
→ →

∫ B ⋅ d l = μ0 I
El campo es perpendicular al plano formado por
la corriente y el punto, su sentido está dado por la
μ0 I regla de la mano derecha.
B 2π r = μ 0 I ⇒ B = Se toma como camino cerrado una circunferencia
2π r concéntrica de radio r. Aplicando la ley de
b) Campo magnético: para r < R : Ampere:
→ → → → →
∫ B ⋅ d l = μ 0 ∫ J ⋅ nˆdS
S ∫ B ⋅ d l = μ0 I ,
Como la corriente I es uniforme B es constante en todos los puntos de la
I → I circunferencia
J= ; J = nˆ
πR 2
π R2 ∫ Bdl cosθ = B ∫ dl = B 2π r = μ 0 I

B 2π r = μ 0
I
π r2 μ 0 I 2μ 0
πR 2 B= =
2π r π r
μ 0 Ir El campo magnético producido por el cable
B=
2π R 2 cilíndrico.
c) Para r = R
μ I
B= 0
2π R
d) El gráfico de B versus r es

Para r < 0,03 m y la corriente está


π r2
uniformemente distribuida I = 4
π (0,03)2

r2 2μ 0 r
B (2π r ) = μ 0 4 ⇒ B=
Ejemplo 52. Un hilo rectilíneo conduce una 0,03 2
0,03 2 π
corriente de 4 A, un cable cilíndrico de 3 cm de Para r > 0,03 m I = 4 A

21
Campo magnético Hugo Medina Guzmán

μ 0 I 2μ 0
B= =
2π r π r
b) Campo magnético resultante en el punto (13
cm, 0)

2μ 0
B2 = = 0,67 × 10 −5 T
π 0,12
2μ 0
B1 = = 0,62 × 10 −5 T ,
π 0,13 → → →
→ F B = I L× B 2 , FB = (4 )(1)B2 sen90º =
B 1 = 0,62 × 10 −5 ˆj
2,7 × 10 −5 N en cada metro de hilo.
2 μ 0 (0,01)
B2 = = 0,89 × 10 −5 T ,
π (0,03) 2
Ejemplo 53. La figura muestra dos alambres
→ llevando corriente. La corriente en el anillo tiene
B 2 = −0,89 × 10 −5 ˆj un valor y dirección conocidos I 1 . Halle el valor
→ → →
B = B 1 + B 2 = −0,27 × 10 −5 ˆj T y dirección de la corriente I x en el alambre recto
Campo magnético en el punto (0 cm, 4 cm). e infinito de manera que el campo en el centro del
anillo sea nulo.

Solución.
El campo producido por el anillo es:
→ μ 0 I1
2μ 0 → B=
B1 = = 2 × 10 −5 T , B 1 = −2 × 10 −5 iˆ 2R
π 0,04 Para que el campo en el centro del anillo sea nulo,
el alambre debe producir un campo igual y de
2μ 0
B2 = = 6,32 × 10 −6 T , sentido opuesto
π 0,12 + 0,04
2 2 → μ 0 I1 ˆ μ0 I x
Ba = − k=− kˆ
→ 2R 2π (R + a )
B 2 = B2senθiˆ + B2 cosθˆj =
π (R + a )
2 × 10−6 iˆ + 6 × 10−6 ˆj Para esto el valor de I x = I 1 , con
R
sentido negativo de x.
→ → →
B = B1 + B 2 = −1,8 × 10− 5 iˆ + 6 × 10− 6 ˆj T Ejemplo 54. Se tiene un cable coaxial, tal como
se muestra en la figura. Calcular el campo
c) La fuerza, que ejerce el cable sobre la unidad magnético para todo punto.
de longitud del hilo rectilíneo.

Solución.
a) Para r < R1

22
Campo magnético Hugo Medina Guzmán

Resuelto en el problema anterior → → ⎛ r2 − a2 ⎞


μ 0 Ir b) ∫ B ⋅ d l = μ 0 I ⎜⎜1 − 2 ⎟
2 ⎟
B= ⎝ b −a ⎠
2π R12
⎛ r 2 − a2 ⎞
b) Para r = R1 ⇒ B 2πr = μ 0 I ⎜⎜1 − 2 ⎟ ⇒
2 ⎟
μ0 I ⎝ b −a ⎠
B=
2π R2 μ0 ⎛ r 2 − a 2 ⎞
B= I ⎜1 − ⎟ , en el sentido
c) Para R2 > r > R1 2πr ⎜⎝ b 2 − a 2 ⎟⎠
→ → μ0 I horario.
∫ B ⋅ d l = μ0 I ⇒ B = → → r r
2π r ∫B⋅d l = μ 0 I
a
⇒ B 2πr = μ 0 I ⇒
a
c) Para r = R2
μ0 I
μ I B= , en el sentido horario.
B= 0 2πa
2π R2
d) Para R3 > r > R2 Ejemplo 56. En el conductor cilíndrico hueco
mostrado en la figura circula una corriente I
→ →
⎛ →

∫ μ ∫
uniforme hacia afuera. Calcular el campo
B ⋅ d l = 0 ⎜ I − J ⋅ nˆ dS ⎟ ⇒
⎝ S
⎠ magnético en la parte hueca.

⎡ π (r − R2 ) ⎤
2 2
B 2π r = μ 0 ⎢ I − I⎥ ⇒
⎣ π (R3 − R2 ) ⎦
2 2

μ I ⎡ (r 2 − R22 ) ⎤
B = 0 ⎢1 − 2
2π r ⎣ (R3 − R22 )⎦

e) Para r > R3
Solución.
B=0 Estando la corriente distribuida uniformemente, la
densidad de corriente es
Ejemplo 55. La figura muestra una sección de un → I
J= kˆ
alambre coaxial muy largo. Esta formado por un
alambre interior de radio a y con densidad de (
π R − R' 2
2
)
→ I ˆ En la figura siguiente se muestra gráficamente el
corriente J = − k , y además por un campo magnético en un punto P en la parte hueca.
2πra
cascarón muy delgado de radio b llevando una
corriente I en su superficie. Hallar
a) La corriente neta en el alambre interior macizo.
b) El campo magnético en el espacio entre a y b.


Cálculo de B 1
→ → →

∫ B1 ⋅ d l = μ 0 ∫ J ⋅ nˆdS ⇒ S

B1 2π r = μ 0 Jπ r 2
Solución.
μ J
a) I ' =
→ →

∫ J ⋅d S =
S ∫r =0

r =a
⎜ −
I ˆ⎞
(
k ⎟ ⋅ 2πrdrkˆ
⎝ 2πra ⎠
) B1 = 9 r


I a La dirección del vector B 1 está en la dirección
= − ∫ dr = − I → → → →
a 0 del vector r × J como r y J son
perpendiculares podemos escribir.
→ μ0 → →
B1 = r× J

23
Campo magnético Hugo Medina Guzmán

→ Si juntamos varias espiras de un mismo radio,


Cálculo de B 2 conectadas entre si, colocadas una a continuación
de otras formando una bobina que tiene una
Procediendo de igual manera longitud grande comparada con el radio de las

μ0 → → espiras.
B2 = r '× J Cuando circula corriente por el solenoide se
2π produce la suma de los campos magnéticos de las

espiras, tal como se muestra en la figura.
Finalmente B es
→ → →
B = B1 − B 2
→ μ0 → → μ9 → →
B= r× J − r '× J
2π 2π
μ ⎛→ →⎞ → μ →
= 0 ⎜ r − r ' ⎟ × J = 0 aiˆ × J
2π ⎝ ⎠ 2π
μ μ
= 0 aiˆ × Jkˆ = 0 aJ ˆj El campo magnético se refuerza en el interior del
2π 2π solenoide y se anula en la parte exterior. Así que
podemos aplicar la ley de Ampere a lo largo de
Ejemplo 57. Un estudiante en un lugar donde la líneas cerradas a b c d mostrado a continuación.
componente horizontal del campo magnético de la
tierra es 1,7 x l0-7 Wb/m2, está realizando un
experimento usando una brújula en un laboratorio
que también tiene un experimento con un alambre
vertical largo que lleva una corriente de 50 A.
¿Qué distancia los experimentos deben estar
separados para que la aguja de la brújula sea
afectada insignificante por el campo del alambre?
Solución.
La componente horizontal del campo magnético Aplicando la ley de Ampere
→ →

∫ B ⋅ d l = μ 0 ( Nl I )
de la tierra es 1,7 x l0-5 Wb/m2. El efecto
magnético debido al alambre vertical debe ser
menor que 1/100 que esto para que su efecto sea → → b→ → c→ → d → → a→ →
insignificante a la exactitud de una aguja de
brújula. Así si r es la distancia mínima por la cual
∫B⋅d l = ∫ a
B⋅d l + ∫ B⋅d l + ∫ B⋅d l + ∫ B⋅d l
b c d
b→ →
los dos experimentos deben estar separados,
μ I
La única parte que tiene valor es ∫
a
B⋅d l = Bl
B = 0 = 1,7 × 10− 7 Wb/m 2 N es el número de espiras por unidad de longitud.
2π r La corriente total encerrada por la superficie es

⇒ r=
(2 × 10 )(50) = 58,8 m
−7
( N lI ) .
1,7 × 10 −7 De esta manera
Bl = μ 0 ( NlI )
Ejemplo 58. Encontrar el campo magnético de un Finalmente
solenoide. Solenoide es un conductor enrollado en
forma de hélice y se utiliza para producir un
B = μ 0 NI
campo magnético interno y uniforme en una
pequeña región. - Ejemplo 59. El toroide. Determine el campo
Solución. dentro de un toroide de N vueltas que llevan la
Si hacemos pasar corriente por una espira, ésta corriente I. Un toroide es como un solenoide
nos da un campo magnético como el mostrado en doblado en una forma de circunferencia.
la figura siguiente.

24
Campo magnético Hugo Medina Guzmán

Anteriormente vimos que cuando un alambre de


longitud l por el cual circula una corriente I se
encuentra en un campo magnético sufre la acción
→ → →
de una fuerza F = I l × B .
Si tenemos dos alambres rectos paralelos por los
cuales circulan corrientes I 1 e I 2
respectivamente como se muestra en la figura,
separados una distancia d .

Solución.
Aplique la ley del amperio a una trayectoria
circular dentro del toroide. Por simetría, B es
tangencial a esta trayectoria y constante en
magnitud a lo largo de la trayectoria, tal que Debido a la corriente I 1 a la distancia d se forma
→ → μ 0 I1
∫ B ⋅ d l = B ∫ dl = 2πRB = μ 0 NI un campo magnético B1 =
2πd
produciendo
μ 0 NI sobre el alambre por el que circula una fuerza
⇒ B=
2πR F2 = I 2 lB1 (dirigida hacia el alambre 1).
Si la trayectoria integral está fuera del toroide, la Reemplazando el valor de B1
corriente que atraviesa el plano encerrado por la
trayectoria es cero, así el campo fuera de un μ 0 I1 I 2 l
F2 =
toroide ideal es cero. 2πd
Ejemplo 60. Una hoja conductora infinita en el De igual modo encontramos el valor de F1 .
plano del xz lleva una densidad de corriente μ 0 I1 I 2 l
→ F1 = (dirigida hacia el alambre 2)
uniforme (por unidad de ancho) J l en la 2πd
dirección x. Determine el campo magnético fuera Por supuesto se cumple el principio de acción y
de la hoja. reacción.
→ →
F1 = − F 2
Si las corrientes son en sentidos opuestos la fuerza
entre los alambres es de repulsión.

Ejemplo 61. Sean dos alambres rectos, muy


largos, y paralelos entre sí, por los que pasa una
corriente I 1 e I 2 en el mismo sentido. Suponga
que el alambre I 1 coincide con el eje z, el

Solución. alambre I 2 pasa por el punto x = 2a, y que las


Por medio de la regla de la mano derecha se ve corrientes tienen el sentido positivo de z. La figura
→ anexa muestra un corte transversal de los
que B está dirigido según se muestra en la figura. alambres.
→ → a) Halle el vector de campo magnético en el punto
Por simetría, B es constante en magnitud. B y P1 = (x; y; 0) debido a los dos alambres.

b) Se coloca un alambre recto de longitud L,
d l son perpendiculares a lo largo de la CA y de paralelo a los alambres anteriores, cuyo centro
→ →
coincide con el punto P2 = (a; a; 0), y por el que
DE, tal que B⋅ d l = 0 allí. Aplicando la ley del
Ampere pasa una corriente I 3 en el mismo sentido de I 1 .
→ → J Suponga que I 1 = I 2 = I . Halle el vector de
∫ B ⋅ d l = 2BL = μ 0 J l L ⇒ B = μ 0 2 fuerza sobre el alambre I 3 .

FUERZA ENTRE DOS CONDUCTORES


PARALELOS

25
Campo magnético Hugo Medina Guzmán

c) Suponga que cambia el sentido de I 2 ; es decir,


I 2 = I . ¿Cómo cambia el vector de fuerza sobre
=
4πa
[
μ0 I ˆ ˆ
−i + j ]
μ0 I
[ ]

el alambre I 3 ? Justifique su respuesta.
B2 = − senθ iˆ − cos θ ˆj
2 2πa
μ I
[
= 0 − iˆ − ˆj
4πa
]
Solución, → → → → μ I
a) Halle el vector de campo magnético en el punto B = B1 + B2 ⇒ B = − 0 iˆ
P1 = (x; y; 0) debido a los dos alambres. 2πa

⎛ μ I ⎞ μ II
F = I 3 kˆ × ⎜ − 0 iˆ ⎟ = − 0 3 ˆj
⎝ 2πa ⎠ 2πa
c) Suponga que cambia el sentido de I 2 ; es decir,
I 2 = − I . ¿Cómo cambia el vector de fuerza
sobre el alambre I 3 ? Justifique su respuesta.
μ 0 I1
[− senθ iˆ + cos θ ˆj ]

B1 =
(
2π x 2 + y 2 )
12

μ0 I 2
[− cosθ iˆ − senθˆj ]

B2 =
[
2π ( x − 2a ) + y 2
2
] 12

y x
senθ = , cos θ =
(x 2
+y 2 12
) (x 2
+ y2 )
12
senθ = cos θ =
1
→ → → 2
B = B1 + B2 =
μ0 I
[ ]=

μ0
⎢ 1
(
⎡ I − yiˆ + xˆj
+
)
I 2 − xiˆ + yˆj ( ) ⎤
⎥ =
B1 =
2 2πa
− senθ iˆ + cos θ ˆj
[
2π x + y 2 2 12
] [ ⎢⎣ x + y
2 2 12
]
( x − 2 a )2 + y 2 [ ] 12
⎥⎦

μ0 ⎧⎪⎡ − yI
+
− xI 2 ⎤
4πa
[
μ0 I ˆ ˆ
−i + j ]
⎨⎢ 2 ⎥iˆ
[ ]
1

[
2π x + y
1 2
]
⎪⎩⎢⎣ x + y
2 12
[
(x − 2a ) + y ⎥⎦
2 12
]
[ ]=
2 2 2
→ μ0 I
⎡ ⎤ ⎫⎪ B2 = cos θ iˆ + senθ ˆj
+⎢
xI1
+
yI 2
⎥ ˆj ⎬ 2 2πa
[ ]
12
[(x − 2a) 1 2
]
[ ]
⎢⎣ x + y 2
2 2
+ y 2 ⎥⎦ ⎪⎭ μ0 I ˆ ˆ
i+ j
b) Se coloca un alambre recto de longitud L, 4πa
paralelo a los alambres anteriores, cuyo centro → → → → μ I
coincide con el punto P2 = (a; a; 0), y por el que B = B1 + B2 ⇒ B = 0 ˆj
pasa una corriente I 3 en el mismo sentido de I 1 . 2πa
Suponga que I 1 = I 2 =I . Halle el vector de →
⎛ μ I ⎞ μ II
F = I 3 kˆ × ⎜ 0 ˆj ⎟ = 0 3 iˆ
fuerza sobre el alambre I 3 . ⎝ 2πa ⎠ 2πa

Ejemplo 62. Tres alambres paralelos e infinitos se


colocan de manera que pasan por los vértices de
un triángulo equilátero. ¿Es posible que los tres se
repelan o los tres se atraigan simultáneamente?
¿Por qué?
1
senθ = cos θ =
2
μ0 I
[ ]

B1 = − senθ iˆ + cos θ ˆj
2 2πa Solución.
Corrientes en la misma dirección atraen y
corrientes en direcciones opuestas repelen. Si los

26
Campo magnético Hugo Medina Guzmán

tres alambres llevan corrientes en la misma → →


dirección ellos se atraen uno a otro. esta ecuación es simétrica, esto es F 2 = − F 1 ,
cumpliéndose así la tercera ley de Newton.
Sabemos que
→ → → → → →
d F = Id l × B , F = ∫ Id l × B
c

No hay forma de tener todos los pares con Esto implica que
corrientes opuestas, de tal manera no es posible →
⎛→ → ⎞
d l 1× ⎜ r 2 − r 1 ⎟
tener a los tres alambres repeliéndose uno a otro. → μ0 ⎝ ⎠
4π ∫1
B (r2 ) = I1 3
→ →
DEFINICIÓN DE LA UNIDAD DE
CORRIENTE ELÉCTRICA (AMPERE) r 2− r1
Anteriormente dejamos pendiente la definición de
y su forma diferencial
Ampere, la cual podemos hacerla ya en esta parte:
Si por dos conductores paralelos muy largos

⎛→ → ⎞
d l 1× ⎜ r 2 − r 1 ⎟
situados a la distancia de 1 metro entre sí, se → μ ⎝ ⎠
hacen circular corrientes iguales que causen una d B (r2 ) = 0 I1
fuerza por unidad de longitud sobre cada 4π → → 3

conductor de 2x10-7 N/m. La corriente en cada


r 2− r1
uno de ellos es igual a un Ampere.
Expresión conocida como ley de Biot y Savart,
En la práctica se escogen separaciones muy
análoga a la ley de Coulomb. La causa del campo
próximas y no es necesario que los conductores
sean tan largos, la fuerza resultante es lo magnético es el elemento de corriente dl , del
suficientemente grande como para medirse con mismo modo que la carga q es la causa del campo
exactitud. eléctrico, el campo magnético al igual que el
campo eléctrico disminuye proporcionalmente a la
LEY DE BIOT Y SAVART inversa del cuadrado de la distancia.
Hasta aquí solo hemos tratado con casos Mientras el campo electrostático señala en
simétricos. Pero en general no es así y se presenta dirección radial el campo magnético es
el problema de los casos carentes de simetría y perpendicular tanto a la dirección radial como a la
esto lo veremos a continuación. dirección del elemento de corriente dl .
En la figura siguiente se muestran dos circuitos
completos Otra demostración. Considere una corriente I
que fluye en un alambre. Rompa el alambre en
pequeños pedazos de la longitud ds. El campo
magnético debido a este pequeño pedazo de
corriente se encuentra experimentadme que es.

μ Id l × rˆ
→ μ 0 Idlsenθ
dB= 0 o dB =
4π r 2 4π r2
Ésta es la ley de Biot y Savart. Aquí r es la
distancia del elemento actual I ds en el punto P
del campo donde deseamos encontrar el campo
magnético B. r̂ es un vector unitario apuntando a

Ampere encontró experimentalmente la relación

lo largo de r . μ0 es una constante de la
que nos da la fuerza F 2 ejercida sobre el circuito naturaleza, es la permeabilidad del espacio libre.
2 debido a la influencia del circuito 1, esta →
relación expresada en un lenguaje de matemática Recuerde para determinar la dirección de d l × rˆ
moderna es usar la regla de la mano derecha, señale sus dedos


⎡ → ⎛ → → ⎞⎤ a lo largo de d l , y gírelos hacia
d l 2 × ⎢d l 1 × ⎜ r 2 − r 1 ⎟⎥ r̂ . EL pulgar
→ μ ⎣ ⎝ ⎠⎦ →
F 2 = 0 I1 I 2 ∫ ∫ derecho apuntará a lo largo de d B .
4π 1 2 → → 3

r 2− r1
A pesar de la aparente falta de simetría se puede
demostrar por medio del análisis vectorial que

27
Campo magnético Hugo Medina Guzmán

→ →
Aquí: I 1 = I , d l 1 = dyˆj , r 1 = yˆj ,
→ → →
r 2 = akˆ , r 2 − r 1 = akˆ − yˆj y

( )
→ → 12
r 2 − r 1 = a2 + y2

→ (
μ 0 b + L dyˆj × akˆ − yˆj )=
4π ∫b
B= I
Para encontrar el campo magnético total debido a
un conductor, sumamos las contribuciones de
(a 2 + y 2 )3 2
μ 0 b+ L ady
cada elemento de corriente integrando sobre el
I∫ iˆ
conductor entero. Así

4π b
(a + y 2 )3 2
2

→ μ I d l × rˆ Integrando
B= 0 ∫ b+ L
4π r2 → μ Ia ⎡ y ⎤
La ley de Biot y Savart fue descubierta B= 0 ⎢ ⎥ iˆ
4π ⎢ a a 2 + y 2
2
⎥⎦ b
experimentalmente, pero puede ser derivada de la ⎣
ley de Coulomb usando la teoría de la relatividad
especial. → μ0 I ⎡ b+L b ⎤
B= ⎢ − ⎥iˆ
Ejemplo 63. Campo magnético producido por un 4πa ⎢ a 2 + (b + L )2 a2 + b2 ⎥⎦
segmento de recta.

En el caso de tratarse de una recta infinita los
límites serían de − ∞ a ∞ .

→ μ Ia ⎡ y ⎤
B= 0 ⎢ ⎥ iˆ
4π ⎢ a 2 a 2 + y 2 ⎥⎦ −∞


μ0 I ⎡ y ⎤
= ⎢ ⎥ iˆ
2πa ⎢ a 2 + y 2 ⎥⎦ 0


Solución. ⎡ ⎤
Se quiere encontrar el campo magnético con el ⎢ ⎥
punto P, producido por el segmento mostrado en μ0 I ⎢ 1 ⎥ ˆ
la figura siguiente. =
2πa ⎢ a 2 ⎥ i
⎢ +1⎥
⎢⎣ y 2 ⎥⎦ −∞
μ I
= 0 iˆ
2πa
CONDUCTOR RECTILÍNEO INFINITO.
El campo magnético debido a un conductor
rectilíneo muy largo es tangente a una
circunferencia concéntrica con él mismo. El
En este caso →
sentido de B esta relacionado con el sentido de la

⎛ → →

d l 1× ⎜ r 2 − r 1 ⎟ corriente I por la regla de la mano derecha,
→ μ b+ L ⎝ ⎠
B = 0 I1 ∫
como se muestra en la figura a continuación.
4π b → → 3
r 2− r1

28
Campo magnético Hugo Medina Guzmán

→ → l l →
d l 1 = dl ˆj , r 2 = ˆj + kˆ , r 1 = 0
2 2
⎛→ → ⎞ l ˆ l ˆ → → l 2
⎜ r 2− r1⎟ = j + k , r 2− r1 =
⎝ ⎠ 2 2 2
⎛l l ⎞
dlˆj × ⎜ ˆj + kˆ ⎟

μ ⎝2 2 ⎠
d B (r2 ) = 0 I1
4π ⎛l 2 ⎞
3

⎜ ⎟
Ejemplo 64. La figura muestra un alambre recto ⎜ 2 ⎟
de corriente I que atraviesa un material no ⎝ ⎠
magnético en forma de un cubo de lado l . Una μ0 dl
= I1 iˆ

sección d l situada en el centro del cubo
2 2π l
→ →
produce un campo d B . Hallar d B cuando lo En e:
calculamos en los puntos a, b , c, d →
⎛→ → ⎞
d l 1× ⎜ r 2 − r 1 ⎟
y e. Los puntos a, b y c están en el centro de las → μ ⎝ ⎠
caras que forma el cubo, el punto d en el punto d B (r2 ) = 0 I1
medio de una arista y el punto e en el vértice. 4π → → 3

r 2− r1
→ → l l l →
d l 1 = dl ˆj , r 2 = iˆ + ˆj + kˆ , r 1 = 0
2 2 2
⎛ → →
⎞ l l l
⎜ r 2 − r 1 ⎟ = iˆ + ˆj + kˆ ,
⎝ ⎠ 2 2 2
→ → l 3
r 2− r1 =
Solución. 2
En a: cero
En b: ⎛l l l ⎞
dlˆj × ⎜ iˆ + ˆj + kˆ ⎟

⎛→ → ⎞ →
μ ⎝2 2 ⎠
d l 1× ⎜ r 2 − r 1 ⎟ d B (r2 ) = 0 I1
2
→ μ ⎝ ⎠ 4π 3
d B (r2 ) = 0 I1 ⎛l 3⎞
4π → → 3 ⎜ ⎟
r 2− r1 ⎜ 2 ⎟
⎝ ⎠

( )
→ → l →
d l 1 = dl ˆj , r 2 = kˆ , r 1 = 0 μ0 dl ˆ ˆ
= I1 −k +i
2 3 3π l
⎛ → →
⎞ l → → l
⎜ r 2 − r 1 ⎟ = kˆ , r 2 − r 1 =
⎝ ⎠ 2 2 Ejemplo 65. Se dispone de alambres conductores
delgados, infinitamente largos y revestidos de
ˆ l ˆ
μ 0 dlj × 2 k μ 0 dl ˆ
material aislante, los cuales se ubican uno al

costado del otro para formar una lámina infinita
d B (r2 ) = I1 = I1 i
4π ⎛l⎞
3
π l2 de corriente, con n conductores por unidad de
⎜ ⎟ longitud, y cada conductor portando una corriente
⎝2⎠ I.

En c: cero
En d: a) Calcule el campo magnético B producido por
este arreglo de conductores en puntos situados a

⎛→ → ⎞
d l 1× ⎜ r 2 − r 1 ⎟ una distancia h, a ambos lados de la lámina.
→ μ ⎝ ⎠
d B (r2 ) = 0 I1
4π → → 3 Ahora, suponga que se ubican dos de estas
r 2− r1 láminas infinitas de corriente en forma paralela,
separadas una distancia d, como lo muestra el
dibujo siguiente. Cada conductor de la lámina de

29
Campo magnético Hugo Medina Guzmán

arriba lleva una corriente I saliendo del plano del ∞


μ Inh ˆ ∞ dx μ In x
=− 0 i∫ 2 = − 0 iˆ tan −1
0 (x + h )
dibujo y cada conductor de la lámina de abajo
también lleva una corriente I, pero entrando al π 2
π h0
plano del dibujo.
μ0 In ˆ
El número de conductores por unidad de longitud = − i
de la lámina de arriba es n1 y el número de 2
conductores por unidad de longitud de la de abajo
es n2. Suponga que n1> n2 el eje de coordenadas x Otra manera de calcular es mediante la ley de
es horizontal. Ampere, como en el ejemplo 60.
Calcule la magnitud y dirección del campo Debido a que el campo magnético producido por
→ los infinitos alambres produce un campo
magnético B , producido por las dos láminas: uniforme, en la parte superior dirigido hacia la
b) En el punto P, situado arriba de la lámina derecha y en la parte inferior hacia la izquierda es
superior. factible utilizar la ley de Ampere.
c) En el punto S. situado debajo de la lámina
inferior.
d) En el punto R. equidistante de las láminas.
e) Cuál sería el campo en los mismos puntos P, R
y S si n1 = n2.

→ →

Solución.
∫ B ⋅ d l = μ 0 nlI ⇒ Bl + Bl = μ 0 nlI ⇒
a) μ 0 nI
2 Bl = μ 0 nlI ⇒ B =
2
μ In

A la distancia h: B = − 0 iˆ
2
b) En el punto P:

μ0 Indx
dB =
(
2π x 2 + h 2 )
12

Al integrar desde – infinito a + infinito las



μ0 In1 ˆ μ0 In2 ˆ μI
i = − 0 (n1 − n2 )iˆ
componentes verticales se anulan, de tal modo
que vamos a trabajar con la componente BP = − i+

2 2 2
horizontal de d B . c) En el punto R:
μ0 Indx

μ0 In1 ˆ μ0 In2 ˆ μ0 I
dB=−

cosθ iˆ = BR = i+ i= (n1 + n2 )iˆ
(
2π x 2 + h 2 ) 12 2 2 2
μ0 Indx h
− iˆ =
(
2π x + h 2
x +h ) (
2 12 2
)
2 12

μ0 Inhdx ˆ

(
2π x 2 + h 2
i
)
d) En el punto S:
→ μ 0 Inhdx ˆ

μ0 In1 ˆ μ0 In2 ˆ μ0 I
B = −∫

−∞ 2π (x 2 + h 2 )
i BS = i− i= (n1 − n2 )iˆ
2 2 2

30
Campo magnético Hugo Medina Guzmán

La integral de los términos en iˆ y ĵ da cero,


finalmente
→ μ0 R2
B= I kˆ
2 (R 2
+z )
2 32

Ejemplo 67. Una arandela delgada (o un disco


e) Si n1 = n2: con hueco concéntrico) de radio interno a y de
→ → → radio externo b, tiene una densidad de carga
B P = 0 , B R = μ0 Iniˆ , B S = 0 superficial σ . Suponga que la arandela gira en
torno a su eje con velocidad angular ω . Calcule
Ejemplo 66. Encontrar el campo magnético en el el campo magnético en el centro de la arandela.
punto P en el eje de una espira circular de
corriente.

Solución.

Solución.
En este caso

⎛→ → ⎞
d l 1× ⎜ r 2 − r 1 ⎟
→ μ0 ⎝ ⎠
4π ∫
B= I1 3
→ →
r 2− r1
2πrdrσ 2πrdrσω
i= = = σωrdr
T 2π
Esta corriente produce un campo magnético en el
centro:

μoi ˆ μoσω
dB= k = drkˆ
2r 2
El campo debido a toda la arandela lo
encontramos integrando de r = a a r = b.

μoσω ˆ b
k ∫ dr = μoσω (b − a )kˆ
1
B=
( )
→ 2 a 2
I 1 = I , d l 1 = Rdθ − senθ iˆ + cos θ ˆj , Otra manera, como la arandela tiene poco ancho
(b ≈ a )
( )
→ →
r 1 = R cos θ iˆ + senθ ˆj , r 2 = zkˆ
b+a
Rm =
( )
→ →
r 2 − r1 = zkˆ − R cosθ iˆ +senθ ˆj , 2
μo I ˆ μo I ˆ μo I ˆ
( )
→ → →
B= k=
12
r 2 − r 1 = R2 + z2 k= k
2 Rm ⎛b+ a⎞
2⎜ ⎟
(b + a)

B=
μ0
I∫
2π ( ) (
Rdθ − senθ iˆ + cosθ ˆj × − R cosθ iˆ − Rsenθ ˆj + zkˆ ) ⎝ 2 ⎠
(R ) Q σπ (b 2 − a 2 )ω σω (b 2 − a 2 )
4π 0 2
+z 2 32

I= = =
→ (
μ 0 2π zR cos θ iˆ + zRsenθ ˆj + R 2 kˆ ) T 2π 2
4π ∫0
B= I dθ Finalmente:
(R 2 + z 2 )3 2

31
Campo magnético Hugo Medina Guzmán

σω (b 2 − a 2 )
→ μo
kˆ = μoσω (b − a )kˆ
2 1
B=
(b + a ) 2

Ejemplo 68. Determine el campo magnético en el


centro de un cuadrado de lado 2a que lleva una
corriente I.
Solución.
El campo opuesto al punto medio de un segmento Solución.
recto de longitud 2a con corriente está dado por Los segmentos rectos no hacen ninguna
μ0 I a dxsenθ contribución al campo en el centro; los
la integral B =
4π ∫
−a r2
. segmentos curvos dan, por la Ley de Biot y
Savart,
μ 0 I ⎡ a(2π − θ ) b(2π − θ ) ⎤
Hay cuatro de tales segmentos en un cuadrado, tal
que: ΔB = −
μ 0 Ia ⎛ 2a ⎞ 2 μ0 I 4π ⎢⎣ a 2 b2 ⎥⎦
B=4 ⎜ ⎟=

4π ⎝ a 2 a 2 + a 2 ⎟ π a μ 0 I (b − a )(2π − θ )
⎠ =
4πab
Ejemplo 69. Un alambre recto largo que lleva
Ejemplo 72. Un disco fonográfico de radio R, con
una corriente I 1 se coloca en el plano de un lazo una carga uniformemente distribuida Q, está
rectangular que lleva una corriente I 2 , ¿cuál es la rotando con velocidad angular ω . Demostrar que
fuerza neta en el lazo? ¿Es atraída o rechazada el campo magnético en el centro del disco está
por el alambre? μ 0 ωQ
dado por B = .
2πR

Solución.
Las fuerzas en los extremos del rectángulo se
cancelan, tal que:
F = B1 I 2 L − B2 I 2 L
Solución.
μ I I L⎛1 1 ⎞ En la figura, se muestra el disco en rotación en
= 0 1 2 ⎜ − ⎟
2π ⎝ a a + b ⎠ sentido horario (visto de arriba) con una
frecuencia f c estándar de disco fonográfico. El
Ejemplo 70. Un solenoide largo de radio R y anillo de carga dq entre los radios r y r + dr
vueltas de N por el metro lleva una corriente I 0 . dq
En el eje del solenoide hay un alambre recto largo constituye una corriente di = , donde
T
con una corriente I . ¿Qué valor de I dará lugar a

un campo magnético en el punto r = 1/2 R que T= es el período de rotación del disco.
esté en 45º del eje del solenoide? ω
Solución. dq 2π rdr 2Q rdr
Si el campo resultante está a 45º del eje, el campo = , de modo que dq =
del alambre debe tener la misma magnitud que el Q πR 2
R2
campo del solenoide, puesto que son Usar el resultado del campo magnético en el
perpendiculares. Así μ0 I
μ0 I R centro de un anillo, B = , en este caso
= μ 0 NI 0 , r = , I = πRNI 0 2R
2π r 2 I = di , R = r , luego la contribución del anillo
diferencial es:
Ejemplo 71. Encontrar B en el punto central del
dispositivo de la figura.

32
Campo magnético Hugo Medina Guzmán

μ 0 di μ 0 ω 2Qrdr μ 0 ωQ μ0 I ⎛ ⎞

dB = = = dr ⎜ x ⎟
2 r 2 2π rR 2 2πR 2 =

4π ⎝ R x 2 + R 2 ⎟
⎠ −∞
2
El campo magnético en el centro en el centro del
disco lo hallamos por integración desde r = 0 a
r = R.
μ 0 ωQ R μ ωQ
2 ∫0
B= dr = 0
2πR 2πR
Para Q > 0, el campo tiene la dirección − k̂ .

Ejemplo 73. La corriente de una fuente de En el dibujo de la tapa de la derecha B sale del
corriente continua es conducida a un instrumento papel. Las líneas de B son círculos concéntricos,
por medio de dos alambres paralelos largos, con su espaciamiento aumentando a medida que
separados 10 cm. ¿Cuál es el campo magnético se aleja del alambre.
entre los alambres cuando la corriente es 100 A?
Ejemplo 75. Determine el campo magnético en el
centro de un lazo circular de radio R que lleva la
corriente I.
Solución.
μ 0 I dxsenθ μ 0 I 2π Rdθ
4π ∫ r 2 4π ∫0 R 2
B= =

μ I
= 0 (2π )

Donde dl = Rdθ y θ = 90º , luego
μ I
B= 0
Solución. 2π
El campo magnético debido a cada alambre en el El campo magnético de un lazo pequeño con
diagrama en el punto situado entre ellos son corriente es como el de un imán de barra pequeño,
perpendiculares e ingresando al papel. Los con las líneas de B que brotan fuera de un Polo
efectos debido a los alambres por lo tanto se Norte imaginario y que van al otro extremo a un
suman en ese punto y el efecto total es dos veces polo sur imaginario. Así el campo de un lazo
el efecto de cualquiera de ellos. Por lo tanto, en el pequeño con corriente es el de un dipolo
punto medio entre los alambres, magnético, con el mismo aspecto que el campo de
μ0 I
= 2(2 × 10 −7 )
100 un dipolo eléctrico.
B=2
2π r 0,05
= 8 x 10-4Wb/m2.

Ejemplo 74. Determine el campo magnético una


distancia R de un alambre recto largo que lleva
una corriente I.
μ0 I
B=
2πR
Solución. Ejemplo 76. ¿Un alambre recto largo que lleva
una corriente I está doblad 90º en un arco circular
μ0 I dxsenθ

B=
4π −∞ ∫ r2
del radio R. ¿Cuál es el campo magnético en el
centro del arco?
R
Donde senθ = y r= x +R
2 2

r
μ I ∞ Rdx
B= 0 ∫
4π − ∞
x + R2
2
(32
)

33
Campo magnético Hugo Medina Guzmán

Solución.
Cada sección recta es como una mitad de un
alambre recto infinitamente largo, así que la
Considere el alambre de longitud a en la figura
contribución de estas dos secciones es →
μ I arriba. Cualquier elemento d l tiene la
B= 0 . La contribución de la sección
2πR → →
dirección del flujo de la corriente y d l × r da un
curvada es el de un cuarto de un círculo →
1 μ0 I vector, para todo d l , dirigido hacia el papel.
completo, B = . Luego en el centro →
4 2R
Así la contribución de todos los d l está en la
μ I μ I μ I
B = 0 + 0 = 0,28 0 misma dirección y las magnitudes son sumadas
2πR 8 R R →
directamente. Para el di elemento d l mostrado,
Ejemplo 77. ¿Un alambre largo, horizontal, rígido la magnitud del campo magnético producido en O
apoyado lleva una corriente de 50 A. una distancia z del alambre es
Directamente sobre él y paralelo hay un alambre μ 0 Idl
dB = senθ .
fino, cuyo peso es 0,075 N por metro, que llevar
4π r 2
uno corriente de 25 A. ¿A qué distancia sobre el
z r
primer alambre debe estar el segundo alambre Pero = senθ , = -tanθ , por consiguiente
para ser sostenido por la repulsión magnética? r l
Solución.
Si el alambre superior va a ser soportado por la
dl = r cos ec θ dθ
2

repulsión magnética, la fuerza magnética por μ I z cos ec 2θ dθ .senθ


∴ dB = 0
longitud de unidad debe igualar el peso de una 4π z 2 / sen 2θ
longitud de unidad del alambre. Además, las
μ0 I
corrientes en los dos alambres deben estar en = senθ dθ
direcciones opuestas para que la fuerza entre los 4π z
alambres sea de repulsión. Por lo tanto Para todo el alambre,
mg F μ 0 II ' μ II ' μ0 I θ
= = ⇒ r= 0
4π z ∫θ
B= senθ dθ
2
=
l l 2π r 2π mg / l 1

(50)(25) = 0,33 x 10-2m =0,33 cm μ I


2 × 10 −7 = − 0 (cosθ 2 − cos θ 1 )
0,075 4π z
Por lo tanto los alambres deben ser muy finos para μ0 I a/2
permitir que sus centros estén muy cercanos. = .
4π z (a / 2)2 + z 2
Ejemplo 78. Determine el valor del campo
magnético en el centro de una bobina rectangular
de largo a y ancho b, que lleva una corriente I.
Solución.

Del dibujo arriba, es obvio que, puesto que las


corrientes entran en direcciones opuestas en los
dos alambres de la longitud a, el campo debido a
cada alambre en O es el mismo y z tiene el valor
b/2. Es también obvio que es el valor de B'

34
Campo magnético Hugo Medina Guzmán

debido a cada uno de los otros alambres de la


longitud b es
μ0 I a/2
B' =
4π (a / 2 ) (a / 2)2 + (b / 2)2
El valor total del campo magnético en O es:
BTotal = 2(B + B')
= μ0 I a/2
+
μ0 I a/2 Sea el solenoide de N vueltas por unidad de
2π a (a / 2) + (b / 2)
2 2 2π b (a / 2) 2
+ (b / 2 )
2 longitud, longitud L y radio R.
El número de espiras del elemento dz es
2μ 0 I ⎛a b⎞
= ⎜ + ⎟ NLdz
= Ndz .
π a2 + b2 ⎝ b a ⎠ L
2μ 0 I El campo magnético en un punto P debido a este
= a2 + b2 elemento es:
πab
→ μ 0 NdzI R2
dB= kˆ
Ejemplo 79. ¿Dado un alambre que lleva una
corriente, ¿Cuándo el campo magnético producido
2 [R 2
+ (z 0 − z ) ]
2 32

en el centro será mayor, doblando el alambre en El campo magnético del solenoide es


un círculo o en un cuadrado? → μ 0 NIR ˆ L Rdz
Solución. B=
2
k ∫0
[R 2
+ (z 0 − z )
2 32
]
El campo en el centro de una espira rectangular de
2μ 0 I Cambiando la variable
lados a y b es B = a 2 + b 2 , Luego en
πab z0 − z = Z , dz = − dZ
el centro de una espira cuadrada de lado a será: Cuando z = 0 ⇒ Z = z0
2 2μ 0 I y z = L ⇒ Z = z0 − L
B1 =
πa → μ 0 NIR 2 ˆ z0 − L dZ
Aquí la longitud del alambre con corriente es 4L,
luego a = L y así
de aquí B = −
2
k ∫ z0
[R 2
+Z2 ] 32

2 2μ 0 I Integrando
B1 = z0 − L
πL → μ 0 NIR 2 ˆ ⎡ ⎤ Z
B=− k⎢ ⎥
( )
El campo magnético en el centro de una bobina
2 12
2 ⎣⎢ R R + Z ⎦⎥ z0
2 2
del radio r es
μ0 I z0 − L
B2 = → μ 0 NIR 2 ˆ ⎡ Z ⎤
2 r B=− k⎢ ⎥
⎢⎣ R 2 (R 2 + Z 2 ) ⎥⎦ z0
1 2
=
2L 2
Aquí 2π r = 4 L , ⇒ r = o
π
μ 0πI
μ 0 NI ˆ ⎧⎪ (z 0 − L ) z0 ⎫⎪
− k⎨ 2 − ⎬
B2 =
4L
. Finalmente: 2 ⎪⎩ R + [(z 0 − L )]
12
R 2 + z 02 ( ) 12
⎪⎭

μ0 I ⎛ π
2 2⎞ Finalmente
B2 − B1 = ⎜ − ⎟
L ⎜⎝ 4 π ⎟⎠ → μ 0 NI ⎧⎪ z0 (z 0 − L ) ⎫⎪ ˆ
B= ⎨ 2 + 12 ⎬
k
μ I
= 0 (0,785 − 0,900 ) < 0
2 (
⎪⎩ R + z 02 )12
R 2 + [( z 0 − L )] ⎪⎭
L
Así el campo debido a una bobina cuadrada es En el caso de L >> R, la expresión entre llaves es
mayor que el de una bobina circular. igual a 2 y

Ejemplo 80. Campo magnético en el eje de un B = μ 0 NIkˆ


solenoide.
Solución. Ejemplo 81. Otra forma de solución.
Demuestre que el campo magnético en el punto P
en el eje de un solenoide de la longitud finita y N

35
Campo magnético Hugo Medina Guzmán

vueltas por unidad de longitud y radio R que lleva μ 0 NI z2 sen 2α dα


una corriente I es B=
2 ∫z1 senα
B = μ 0 NI (cos α 2 − cos α 1 ) , donde α 1 y
1
μ 0 NI α2 μ 0 NI
2 = ∫α senα dα = (− cos α ) αα 2

α2 2 2
1 1
se muestran en el dibujo

μ 0 NI
Finalmente B = (cos α 1 − cos α 2 )
2
Ejemplo 82. Dos bobinas circulares de Helmholtz
Solución. de 250 vueltas son paralelas una a otra y
Considere el solenoide como una serie de lazos separadas por una distancia igual a su radio
circulares de radio R y ancho dx , cada uno con común. Encuentre el valor del campo magnético
en un punto en el eje entre ellas cuando la
corriente di = NdxI . corriente atraviesa ambas bobinas en el mismo
El campo magnético a una distancia z en el eje de sentido, y demuestre que el campo es casi
μ 0 IR 2 uniforme sobre el punto medio.
un lazo circular es B =
( )
32
. Solución.
2 R2 + x2 El campo magnético debido a una sola bobina en
El campo magnético producido por un lazo un punto a lo largo del eje una distancia y del
circular de ancho diferencial es: plano de la bobina es
μ 0 R 2 di μ 0 NIR 2 dx μ 0 Iasenα
dB = B1 = =
2(R 2 + x )
= para
2 32 2r 3
2 r2
encontrar el campo resultante integramos desde μ 0 Ia 2 μ0 Ia 2
=
x = x1 a x = x 2 . 2 r3 2 a2 + y2( ) 32

μ 0 NI z2 R 2 dx
B=
2 ∫ z1 r3

De la figura anterior: r = R + x ( 2 2 12
) ,
R x r
senα = , cos α = y dx ≈
r r senα Similarmente, en el mismo punto el campo
magnético debido a una sola vuelta de la segunda
R R
cos α dα = − 2 dr = − 3 dx bobina es
r r μ0 Ia 2
B=
dr =
xdx x
= dx [
2 a 2 + (a − y )2 ]
32

(x + R ) r
2 2 12
Éstos actúan en la misma dirección, y el efecto

(
Con r = R + x
2 2 12
) y dx ≈
r
senα
:
total en O debido a las vueltas de n de ambas
bobinas es
B = n(B1 + B2 )
μ 0 NI R 2 dx μ 0 NI R 2 rdα ⎧⎪ ⎫⎪
= 250 μ 0 Ia
2
z2 z2 1 1
B=
2 ∫ z1 r3
=
2 ∫ z1 r 3 senα 2
⎨ 2
(
⎪⎩ a + y 2 )32
+
[a 2
+ (a − y ) ]
2 32

⎪⎭

R =0
Con senα = :
r

36
Campo magnético Hugo Medina Guzmán

a 8(250 μ 0 I ) Ejemplo 83. En un laboratorio de Física se


Si y = , entonces B = , mas requiere eliminar los efectos del campo magnético
2 53 2 a terrestre en un determinado punto P del mismo.
adelante, Para ello, se produce un campo magnético
contrario al campo magnético terrestre Btierra por
dB 250 μ 0 Ia 2 ⎧⎪ − 3 y 3(a − y ) ⎫⎪ medio de un par de espiras circulares que
= ⎨ 2 + ⎬
dy 2 (
⎪⎩ a + y 2 )
32
[a 2
+ (a − y ) ]
2 32
⎪⎭
comparten el mismo eje, ambas de radio R y
separadas una distancia 2R entre sus centros, Las
a corrientes de ambas espiras circulan en el mismo
, si y = , sentido y el punto P esta ubicado en el punto
2 medio del mencionado eje.
También a) Suponga que el campo magnético de la tierra
d 2 B 250 μ 0 Ia 2 ⎧⎪ −3 3 es paralelo a su superficie y se dirige hacia el
= ⎨ 2 −
dy 2 2 (
⎪⎩ a + y 2 )
32
[a 2
+ (a − y ) ]
2 32
norte, ¿Cuál es la posición en que tendrían que
ponerse las espiras para lograr el efecto deseado
15(a − y ) ⎫⎪
2 en el punto P?
15 y 2
+ + ⎬ b) ¿Cuál es la magnitud de la corriente en las
(a 2
+y 2 32
)[a + (a − y ) ] 2 2 32
⎪⎭ espiras que anulará el campo magnético terrestre
en el punto P?
250μ 0 Ia 2 ⎧⎪15 y − 3(a + y )
2 2 2
c) Obtenga la expresión de la magnitud del campo

⎪⎩ (a + y )
= magnético resultante de las espiras Bespiras en
2 2 2 32
cualquier punto sobre el eje que comparten,

15(a − y ) − 3 a 2 + (a − y ) ⎫⎪
2 2
[
⎬=0,
] tomando como origen el punto P.
d) A partir de su resultado en la parte anterior
[ ]
+
a 2 + (a − y )
2 32 esboce un gráfico del campo Bespiras a lo largo de
⎪⎭ dicho eje. Indique los valores críticos [máximo(s)
a o mínimo(s)] en su gráfico.
si y = , Solución.
2 a)
dB d 2 B
Así y son cada uno igual a cero en el
dy dy 2
a
punto y = , en el punto medio entre las
2
bobinas. Por lo tanto de B difícilmente es cero
alrededor de ese punto, dando una región grande
de campo uniforme en la región central entre las
bobinas.
Con este espaciamiento particular de las bobinas, Las bobinas deben estar orientadas de norte a sur,
al bajar el valor de B debido a una bobina cuando con las corriente como se indican en el dibujo de
nos alejamos de ella es compensado por el tal manera que el campo producido por estás esté
aumento de B debido a la otra bobina para buena en el mismo eje y opuesto al campo magnético
parte de la región entre ellas. terrestre.
b) El campo magnético debido a una sola bobina
en un punto P en su eje a una distancia R del
centro de la bobina es
→ μ0 IR 2 μI
B1 = − iˆ = − 10,5 iˆ
2 (R + R 2
)
2 32 2(2 )R
Lo mismo para segunda bobina
→ μ0 IR 2 μ0 I
B2 = − iˆ = − iˆ
La situación se ilustra en el diagrama. Las líneas 2 (R + R 2
)
2 32 2(21,5 )R
llenas dan la magnitud de B debido a cada bobina
El campo total producido por las dos bobinas es:
por separado a lo largo del eje. La línea → → →
μ0 I ˆ
discontinua muestra el efecto combinado de las B = B1 + B2 = −
dos bobinas, y la región del campo uniforme (2 )R i
1, 5

alrededor del punto medio del sistema se ve


claramente.

37
Campo magnético Hugo Medina Guzmán

El campo magnético terrestre es de la Tierra es


del orden de 0,5x10-4 T.
Para equiparar este campo debe de circular una
corriente:
4π × 10 −7 I
0,5 × 10 −4 = ⇒
2,83R

I=
(0,5 × 10 )(2,83R ) = 112,60 R
−4
d) En el gráfico puede verse que los máximos
están en x = ± R y el mínimo entre las espiras en
(4π × 10 ) −7
el centro x = 0.
(Amperes) El máximo en x = ± R es:
c) El campo magnético resultante de las espiras
Bespiras en cualquier punto sobre el eje que μ 0 IR 2 ⎧⎪ 1 1 ⎫⎪
Bmáx = ⎨ 2 + 32 ⎬
comparten, tomando como origen el punto P. 2 [
⎪⎩ R + (R + R )2 ] 32
[R 2
+ (R − R )
2
] ⎪⎭

μ0 I ⎛ 1 ⎞ μ I
Bmáx = ⎜ 3 2 + 1⎟ = 0,54 0
2R ⎝ 5 ⎠ R
El mínimo en x = 0. es:
μ 0 IR 2 ⎧⎪ 1 1 ⎫⎪
Bmin = ⎨ 2 + 32 ⎬
⎩ [
2 ⎪ R + (0 + R ) 2 32
] [
R 2 + (0 − R )
2
] ⎪⎭
→ μ0 IR 2
B1 = − iˆ
[
2 R 2 + ( x + R )2 ]
32

Bmin =
μ0 I ⎛ 1 1
⎜ 32 + 32
⎞ μ0 I μ I
⎟ = 3 2 = 0,35 0
2R ⎝ 2 2 ⎠ 2 R R

Ejemplo 84. Sobre la superficie de una esfera de


madera de radio R se enrolla en una sola capa un
número N de vueltas muy próximos entre si con
un alambre muy fino, cubriendo completamente la
superficie de le esfera. Como se muestra en la
figura. Si se hace circular una corriente I ¿cuál es
Lo mismo para segunda bobina el campo magnético en el centro de la esfera?
→ μ0 IR 2
B2 = − iˆ
[
2 R 2 + ( x − R )2 ]
32

El campo total producido por las dos bobinas es:


→ → → Solución.
B = B1 + B2 = El campo magnético formado en el centro de la
μ0 IR 2 ⎧⎪ 1 1 ⎫⎪esfera es la suma de los campos magnéticos de
− ⎨ 2 + 32⎬
iˆ las espiras, como no es posible calcular una
[ ] [R ]
todas
⎪⎩ R + ( x + R )2 + (x − R )
32
2 2 2
⎪⎭ una y sumar, encontraremos un elemento
por
diferencial e integraremos.

38
Campo magnético Hugo Medina Guzmán

μ 0 NIR 2 cos 2 θdθ ˆ



dB= j ⇒
2πR 3
→ μ NI ˆ 2
dB= 0 j cos θdθ
2πR
El campo magnético total es
→ μ 0 NI ˆ π
B= j ∫ cos 2 θdθ =
2πR 0
π
μ 0 NI ˆ 1
j (θ + senθ cos θ )
2πR 2
La espira formada por el ángulo dθ y 0

determinada por θ produce un campo en el


→ μ 0 NI ˆ
Finalmente B = j
centro de la esfera igual a 4R
→ μ 0 r 2 di
dB= ˆj
(
2r +y2
)
2 32

Donde
r = R cos θ , y = Rsenθ ,
NIdl NIRdθ NI
dI = = = dθ
l πR π
Reemplazando

PREGUNTAS Y PROBLEMAS

1. Estudiar el movimiento en el vacío de una dos trayectorias iguales y paralelas a través de la


partícula de masa m, carga q, que se encuentra en combinación, y se entrega a las escobillas una
un campo magnético uniforme que forma un corriente de 12 amperes, encontrar el par
ángulo θ con la velocidad inicial de la partícula. promedio desarrollado en la armadura. Tómese
No se considerará la influencia del peso. cada bobine como un cuadrado de 8 cm. de lado,
con los alambres paralelos al eje del motor a una
2. Una partícula de masa m y carga q en el vacío distancia de 5 cm. del eje.
está sometida a un campo eléctrico uniforme E
vertical a un campo magnético B uniforme 4. Un protón de 2MeV se desplaza en una región
horizontal. La partícula con una velocidad inicial del espacio donde hay un campo eléctrico
en el punto 0. uniforme de una intensidad 10 V/m y un campo
a) Estudiar el movimiento no tomando en cuenta magnético uniforme en ángulo recto con él. Si la
la acción del peso. Considerar B paralelo al eje y. dirección tanto del campo eléctrico como del
b) ¿Cómo se manifestaría la acción del peso? ¿Es campo magnético y el protón no se acelere,
despreciable? calcular la intensidad y el sentido del campo

3. La armadura de un motor de corriente continua, 5. Un electrón describe una trayectoria circular de


porta 24 bobinas espaciadas por igual, cada una 0,2m de radio en un campo magnético de 0,002
con 10 vueltas, conectada en serie con sus puntos Tesla. Calcular:
de conexión en los 24 segmentos del conmutador a) Su velocidad
rotatorio. Unas “escobillas” de carbón hacen b) Su periodo de revolución
contacto con el conmutador para poder admitir c) Su energía cinética en MeV
corriente en las bobinas; la disposición de las
escobillas y la geometría del campo magnético en 6. Una barra conductora de masa 50g en reposo, y
que gira la armadura son tales que cada alambre a ángulos rectos, dos carriles horizontales
paralelo el eje gira en un campo de 8500 Gauss en separados 10 cm. Una corriente de 20 A pasa a
promedio y los paras son todos del mismo sentido. través de la barra a partir de un carril al otro. El
Si, en cualquier instante, todas las bobinas están coeficiente de la fricción estática entre la barra y
conectadas a las escobillas de tal manera que haya los carriles es 0,30. ¿Cuál es el menor campo
magnético perpendicular al plano de la barra y de

39
Campo magnético Hugo Medina Guzmán

los carriles que moverá a la barra sobre los


carriles?
Respuesta. 0,0735 Wb/m2

7. El alambre de la figura tiene una longitud total


de 2l y el punto P está en la bisectriz
perpendicular con las coordenadas (0, 0, z).

Demuestren que B se dirige en el sentido del eje
x en la figura y tiene la magnitud b) Repitan el problema, calculando el campo en
μ0 I l
un punto general con las coordenadas (x, y). ¿Es
B= una restricción esencial la de que no
2πz (z 2 + l 2 )1 2
.
introduzcamos la coordenada z del punto de
campo?
c) Repitan el problema, suponiendo esta vez que
las corrientes en los dos alambres se dirigen
perpendicularmente hacia abajo.
Respuesta. b)
→ μ0 I ⎧⎪⎡ y − 2a y ⎤ˆ
B= ⎨⎢ 2 − 2 ⎥i
2π ⎪⎩⎣ x + ( y − 2a ) x + y2 ⎦
2

⎡ x x ⎤ ⎫⎪
+⎢ 2 − ˆj
2 ⎥ ⎬
8. Una corriente de 10 amperios fluye por un lazo ⎣ x + y 2
x 2
+ ( y − 2 a ) ⎦ ⎪⎭
de alambre con la forma de un triángulo
equilátero de 50cm de lado. Utilicen el resultado 11. ¿Qué corriente debe circular por un lazo
del problema anterior y calculen B en el centro del circular de alambre de radio de 50 cm para que el
lazo. campo en el centro sea de 5 x 10-3 tesla? ¿Qué
Respuesta. 3,6 x105T corriente debe de fluir para que el campo tenga
una intensidad de dos. teslas?
9. Un alambre de 2l de longitud lleva una Respuesta. 40A; 1,6 x 106A
corriente I y reposa en el eje z de cierto sistema de
coordenadas con su centro en el origen. 12. Una corriente I fluye en un segmento de un
→ lazo circular de radio a y un ángulo α como se
Demuestren que el campo B en un punto con las →
coordenadas (x, 0, z), tiene la magnitud muestra en la figura. Calcular B en el centro O
del lazo, desdeñando los alambres de alimentación
μ 0 I ⎧⎪ l−z de la corriente.
B= ⎨
[
4πx ⎪ (l − z )2 + x 2
⎩ ]
12

l+z ⎫⎪
+ 12 ⎬
[(l + z ) 2
]
+ x 2 ⎪⎭
y determinen su dirección.

10. Dos alambres paralelos e infinitos están


separados por una distancia 2a y llevan corrientes Respuesta.
I, en direcciones opuestas, como se muestra en la μ 0 Iα
figura. (abajo)
→ 4π a
a) Calculen B en un punto P que se encuentra u
una distancia b a lo largo de la bisectriz 13. Supongan que hubiera una corriente circular
perpendicular, de tal modo que, en función del en torno a la Tierra, en el ecuador. ¿Cuál sería la
sistema de coordenadas que se muestra, las intensidad de esta corriente para producir el
coordenadas de P son (b, a) ¿Cuál será su campo observado en los polos de
respuesta cuando b = 0? aproximadamente 7,5 x l0-5 tesla? ¿Iría la
corriente de este a oeste o en dirección opuesta?

40
Campo magnético Hugo Medina Guzmán

14. Calculen en el punto P la intensidad del que lleva una corriente de 2,0 A tenga un campo
campo magnético debido a la corriente I a través magnético sobre su eje de 1,5 x l0-2 T?
del alambre de la figura. (Indicación:
19. Una corriente de 0,5 A fluye en torno a un
solenoide de radio de 1,0cm y 40 cm de longitud.
Si el campo magnético uniforme cerca del centro
del solenoide es de l0-3 T, ¿qué cantidad de
vueltas por unidad de longitud tendrá el
solenoide?
Respuesta. 1,6 x 103 vueltas/m
15. Una línea de carga circular de radio a y carga
por unidad de longitud λ , gira a una velocidad 20. Supongan que el campo sobre el eje de un
angular ω en torno a su eje. solenoide muy largo con 1000 vueltas por metro
a) Demuestren que este movimiento corresponde es de 50 x l0-3T.
a una corriente I que fluye en un lazo circular de a) Cuál es la corriente?
radio a y está dada por I = λaω . b) Si se pone ahora un alambre que lleva una
→ corriente de 10 A con N vueltas por unidad de
b) Calculen B en un punto sobre el eje de la línea longitud en torno al solenoide original, de tal
de carga y a una distancia b de su centro. modo que el campo sobre el eje se reduzca a 2,5 x
l0-3T,calculen N. ¿Son los flujos de corriente
16. Un disco de radio a lleva una carga uniforme paralelos en las dos bobinas?
por unidad de área σ . y gira con una velocidad
angular ω en torno a su eje. 21. Un protón se desplaza a una velocidad de 5,0
x l05 m/s a lo largo del eje de un solenoide, que
tiene 1000 vueltas por metro y lleva una corriente
de 2,0 A. Calculen la aceleración del protón. ¿De
qué modo diferiría su respuesta, si se desplazan en
paralelo, pero no a lo largo del eje?
Respuesta. Cero

22. Un hombre camina hacia el norte por debajo y


en sentido paralelo a una línea de potencia en la
que fluye una corriente directa de 100 A. Si está a
10 m por debajo de la línea, ¿qué campo
a) Demuestren que el campo magnética dB en el magnético más allá del que se debe a la Tierra,
punto P sobre el eje, debida a un anillo de radio r medirá? ¿Tendrá esto una interferencia grave con
y espesor dr es una lectura de la brújula en este punto?
μ 0σω r 2 dr
dB = 23. ¿Cuál es el campo magnético en el eje de un
(
2 r 2 + b2 )
32
lazo circular de radio R con una corriente I a una
b) Demuestren que el campo total B en el punto P distancia z del centro del lazo?
es Respuesta.
μ 0 I 2π Rdφ
4π ∫0 r 2
B= cos θ ⇒
μ 0σω ⎡ a 2 + 2b 2 ⎤
B= ⎢ 2 − 2b ⎥ μ 0 IR 2
⎢⎣ (a + b 2 )
12
2 ⎥⎦ B=
(
2 R2 + z2 )
12

17. Un solenoide de 0,5 cm de radio y 20 cm de


longitud lleva una corriente de 10 A y tiene 1000
vueltas. Calculen el campo magnético sobre el eje
de la bobina en los puntos siguientes: a) el centro
de la bobina, b) el borde de la bobina, y c) a una
distancia de 100 cm al centro de la bobina, d)
tracen una gráfica de B en función de la posición
sobre el eje de la bobina.

18. ¿Qué cantidad de vueltas por unidad de


longitud se requieren para que un solenoide largo

41
Campo magnético Hugo Medina Guzmán

24 ¿Un alambre recto muy largo con una Determinar el carpo magnético para todo punto.
corriente I tiene un lazo circular de radio R. ¿Cuál
es el campo magnético en el centro del lazo? 29. Se tiene tres conductores paralelos como se
muestra en la figura. Calcular el campo magnético
en los puntos A, B, C y O.
I1 = 1 A , I 2 = 2 A , I 3 = 3 A .

Respuesta. Sobreponga el campo de un alambre


μ0 I
recto, B = , con el campo de un lazo,
2πR
μ0 I
B= .
2R
μ0 I μ0 I 30. Una espira lleva una corriente de 4 A y tiene
B= + un radio de l0 cm. Si colocamos un alambre recto
2πR 2 R y largo en el eje de la espira con una corriente de
1 A, determinar la fuerza que ejerce la espira
25. Tres alambres largos, rectos, paralelos, cada sobre el alambre por unidad de longitud.
uno con corriente I en la misma dirección. Son
equidistantes uno de otro con la misma separación
a . ¿Qué fuerza por unidad de longitud un
alambre experimenta debido a los otros dos?
Respuesta.
μ0 I 3 μ0 I 2
B=2 cos 30° , F = BIL =
2π a 2π a 31. Una aspira cuadrada de alambre de lado a a
lleva una corriente I , determinar el valor de B
26. ¿Un disco del radio R lleva una densidad en el centro de la espira.
uniforme σ de carga superficial. Rota sobre su
eje con velocidad angular ω . cuál es el campo
magnético en el centro del disco?
Respuesta.
Considere los anillos anulares de ancho dr . Cada
uno es como un lazo con corriente σ dAf ,
donde dA = 2πrdr y f = ω 2π .Luego el
campo en el centro de un lazo es: 32. Calcular el campo magnético en el eje de la
R μ 0 (rσωdr ) μ 0σωR espira radio R que está conectada en ambos lados
B=∫ = como se muestra en la figura y pasa una corriente
0 2r 2
I por ella. ¿Cual es el valor del campo en el
27. ¿Qué campo magnético es producido por un centro de la espira?
solenoide muy largo con 150 vueltas por metro
que lleva una corriente de 20 A?
Respuesta. B = μ 0 NI (
= 4π 10
−7
)(150)(20)
-3
= 3,77 x 10 T.

28. Un conductor recto, largo y de radio a , lleva


una corriente I 0 , se ha diseñado de tal manera
que la densidad de corriente dentro del conductor
varía de acuerdo a la expresión 33. Un disco de plástico de radio a tiene una
carga uniformemente distribuida en su superficie
J=
3
I0
r (σ C/m ). Si el disco gira con velocidad angular
2

2 π a3

42
Campo magnético Hugo Medina Guzmán

ω , encontrar el campo magnético en el centro del Las bobines están arrolladas densamente con N
disco. vueltas y pasa una corriente I.

34. Dada una tira delgada de metal de ancho a y


muy larga. La corriente es longitudinal y vale I .
Encontrar el campo magnético en el plano de la
tira a una distancia b del borde más cercano.

35. calcular es Campo magnético B creado por


una espira circular de radio R por la que circula
una corriente I , en un punto P sobre el plano del
anillo. Realizar un desarrollo en potencia de
limitarse el segundo orden.

36. Determinar el campo magnético en el punto O 39. Una bobina cuadrada de 1 cm de lado, con 15
en la figura. Los alambres rectos se consideran vueltas, está colgada en el centro de un solenoide
muy largos. largo de 200 vueltas por metro y conduce 300
mA. Si el plano de la bobina forma un ángu1o de
20º con el eje del solenoide y la bobina conduce 1
mA, encontrar el par sobre ella.

40. ¿Cual es el trabajo necesario para voltear la


bobina cuadrada del problema anterior de una
posición en que la normal positiva a la bobina sea

paralela a la dirección B en el solenoide a una
posición invertida en 180°?

41. Las bobinas de Helmholtz son dos bobinas


37. Determinar el flujo magnético a través del circulares planas (asimilarlo a dos espiras
contorno rectangular mostrado en la figura creado circulares de radio R), idénticas, con eje común,
por la corriente I que pasa por el alambre recto por las que pasa una misma corriente I en el
infinito. mismo sentido. La distancia entre las bobinas es
2d.

Calcular el campo magnético B en un punto P
Situado en el ejes la distancia x del centro O da
las dos bobinas

42. Dos placas delgadas infinitas, paralelas de


ancho a y la distancia entre ellas es b , llevan
corrientes igual a I pero opuestas. Encontrar la
fuerza por unidad de longitud de cada placa.

43. Determinar el campo magnético debido a dos


planos paralelos con iguales densidades de
corriente superficial i constante. Considere los dos
casos cuando las corrientes fluyen en el mismo
38. Determinar el flujo magnético a través de las sentido y en sentidos opuestos.
secciones de los toroides mostrados en la figura.

43
Campo magnético Hugo Medina Guzmán

44. Una esfera de radio a tiene carga uniforme es la razón de las energías cinéticas ale esos dos
sobre su superficie con una carga total q , rota isótopos?
alrededor de un diámetro con velocidad angular
constante ω . Encontrar el campo magnético
dentro y fuera de la esfera.

45. Un protón en la capa superior de la atmósfera


se desplaza a una velocidad de 10 m/s en ángulo
recto con el campo de la Tierra, que tiene una
intensidad de 5,0 x l0-5 T en este punto.
a) ¿Cuál es el radio de la órbita del protón?
b) ¿Cuánto tiempo necesita el protón para
completar una órbita?
c) ¿Cuál es su frecuencia de ciclotrón? Respuesta.
Respuesta. a) 210 m; b) 1,3x 10-3s; c) 4,8 x 103 K (Li 7 ) 7
=
K (Li 6 ) 6
rad/s 5, l2 cm bajo la ranura;

46. Un electrón se desplaza en ángulo recto con 50. Un ciclotrón que se utiliza para acelerar
un campo magnético uniforme de una intensidad protones tiene un radio de 0,5 m y un campo
de 3,0 x l0-2 T. Si su energía es de 50 keV (1 keV magnético de 0,75 T.
= l0-3 MeV), calculen: a) ¿Cuál es la energía de los protones que salen?
a) su velocidad, b) el radio de su órbita y c) la Exprese su repuesta en Joules y en Mev.
frecuencia del ciclotrón. b) ¿Cuál es la velocidad final de los protones
expulsados?
47. Un protón y una partícula alfa se desplazan en c) ¿Cuáles serían las energías de partículas alfa si
direcciones paralelas a la misma velocidad, se vieran aceleradas por este ciclotrón?.
cuando entran a una región del espacio en la que
→ 51. Con el ciclotrón del problema anterior.
hay un campo magnético uniforme B . Supongan a) ¿Cuál es la frecuencia del oscilador ω para
que se desplazan en ángulo recto con el campo. este ciclotrón, cuando acelera protones?
a) ¿Cuál es la razón de los radios de sus órbitas? b) Si los protones recogen 100 keV cada vez que
b) ¿Cuál es la razón de sus frecuencias de cruzan el espacio entre las Des, ¿cuántas órbitas
ciclotrón? semicirculares completarán los protones antes de
c) Cuál es la razón de sus energías? verse expulsados?
Respuesta. c) Calculen el tiempo necesario para acelerar los
Rα ω 1 K protones hasta sus velocidades finales.
a) = 2 , b) α = , c) α = 4 Respuesta.
Rp ωp 2 Kp a) 7,2 x 107 rad/s; b) 69 órbitas; c) 3,0 x10-6s

48. Un protón de 2 MeV se desplaza en una 52. Si se utilizara el mismo el ciclotrón para
región del espacio donde hay un campo eléctrico acelerar electrones, entonces, desdeñando los
uniforme de una intensidad de 105 V/m y un efectos relativistas, ¿cuáles serían las energías
→ finales de los electrones? ¿Tenemos razones para
campo magnético uniforme B en ángulo recto suponer que los efectos relativistas son
con él. Si la dirección de movimiento del protón desdeñables?
es perpendicular a la dirección tanto del campo Respuesta. 9,l.2 x 104 MeV; no.
eléctrico como del campo magnético y el protón
no se acelera, calculen la intensidad y el sentido 53. Un ciclotrón con un campo magnético de 2,0

T se usa para acelerar protones.
de dirección del campo B . a) ¿Cuál debe ser la frecuencia (en Hz) del campo
oscilador entre las Des?
49. Un haz que contiene una mezcla de los b) Si se utiliza este ciclotrón para acelerar
isótopos Li y 7Li entra en la región de campo deuterones, ¿a qué frecuencia se deberá ajustar la

frecuencia de este campo oscilador?
magnético uniforme B 0 por la ranura C del
espectrómetro de masas. Si los iones Li6 se
detectan a una distancia de l0 cm por debajo de la 54. Si E 0 es la energía de un ciclotrón, cuando
ranura C, ¿dónde aparecerán los iones Li7? ¿Cuál acelera protones, demuestren que puede acelerar

44
Campo magnético Hugo Medina Guzmán

iones, de masas atómicas A y con Z unidades de


carga, a la energía
⎛Z2 ⎞
E = ⎜⎜ ⎟⎟ E 0
⎝ A ⎠

55. Si existiera un monopolo magnético de


intensidad ε , entonces, el campo magnético

⎛ ε ⎞→
asociado a él sería de B = ⎜ 3 ⎟
r , en donde r
⎝r ⎠
es la posición en el espacio, medida a partir del
monopolo.
a) Escriban la ecuación del movimiento para una
partícula de carga q y masa m que se desplace en
el campo de un monopolo.
b) Demuestren que la energía cinética de la 59. Demuestren que la fuerza sobre una porción
partícula es una constante de movimiento. de alambre que lleva una corriente en un campo
→ → → magnético uniforme es la misma para todos los
dv v× r alambres que tengan los mismos puntos extremos.
Respuesta. a) m = qε 3 O sea, demuestren que la fuerza sobre los
dt r alambres 1 y 2 en la figura es la misma que si
pasara la misma corriente por cada uno de ellos.
56. Para el sistema físico del problema anterior,
demuestren que:
→ →
a) La cantidad de movimiento angular m r × v
(en relación al monopolo) de la partícula no es
constante en general.
⎛ → →⎞
d⎜m r⋅ v ⎟
b) La cantidad
⎝ ⎠ es constante y
dt
determinen su valor en función de la velocidad

inicial v 0 . 60. Una corriente de 5,0 A fluye en un lazo
cuadrado de lado de 10 cm. Calculen la fuerza
57. Se ha estimado que en la superficie de una total en dos de sus lados adyacentes, producida
estrella de neutrones, el campo B puede ser de por una inducción magnética externa
hasta 109 T. Para un protón de lo MeV que se perpendicular al plano del lazo y una intensidad
desplace en ese campo, determinen: de 0,1 T.
a) su frecuencia de ciclotrón y b) el radio de su
órbita. 61. Una corriente de una intensidad I fluye en una
Respuesta. a) 9,6 x 1016 rad/s b) 4,6x 10-10m bobina que tiene la forma de un pentágono regular
de lado a. Supongan que hay un campo magnético
uniforme B perpendicular al plano de la bobina.
58. Una bobina rectangular de alambre lleva una a) Calculen la fuerza producida por el campo
corriente I y tiene una anchura a. Si el extremo externo en cada segmento.
inferior de la bobina se mantiene entre los polos b) Al utilizar los resultados de (a) demuestren
de un electroimán con un campo magnético B y explícitamente que la fuerza total sobre el lazo es
dirigido como se muestra en la figura, calculen la cero.
→ Respuesta. a) IaB dirigido perpendicularmente
magnitud de la fuerza F necesaria para sostener al alambre en el plano de la bobina
la bobina, por encima de la gravedad.
62. Sean dos alambres paralelos, cada uno de ellos
con una longitud l , que tienen las mismas
corrientes I y están separados por una distancia a,
como en la figura.
a) Demuestren que el campo magnético B en el
punto (x, a) del alambre superior, debido a la
corriente en el interior, tiene la magnitud

45
Campo magnético Hugo Medina Guzmán

μ I 1 ⎧ (l 2) − x Respuesta. 9,7 x 10-5 N entre alambres más


B= 0 ⎨
[ ]
largos;
4π a ⎩ ( x − l 2)2 + a 2 2,4 x 10-6 N entre alambres más cortos

+
(l 2) + x ⎫ 66. Sean dos alambres de longitudes l y L en
[(x + l 2)2 + a 2 ]⎬⎭ ángulo recto entre sí, de la figura. Supongan que
las corrientes I1 e I2 se dirigen como se muestra.
¿Cuál es la dirección de B? →
b) Calculen la fuerza dF sobre el elemento de a) Demuestren que el campo magnético B en la
longitud dr, situado en el punto (x, a) del alambre posición del elemento de longitud dy situado en el
superior. punto (0, y, 0) es
c) Demuestren por integración que la fuerza total → μ I ⎧⎪ −b
F sobre el alambre superior es descendente y tiene B = kˆ 0 1 ⎨ 2
⎪⎩ (b + y 2 )
μ0 I 2 ⎛ 1 ⎞ 2
[ ] 4πy 12

⎜ ⎟ (l + a ) − a
2 12
la intensidad F=
2π ⎝ a ⎠ ⎫⎪
b+l
+ 12 ⎬
[(b + l) 2
+ y 2 ⎪⎭]
b) Demuestren que la fuerza magnética dF sobre
el elemento dy es
→ μ II ⎧⎪ −b
d F = iˆ 0 1 2 dy ⎨
4πy ⎪⎩ (b 2 + y 2 )
12

63. En la figura, si a = 3,0 cm, b = 5,0 cm,c = 3,0 b+l ⎫⎪


+ 12 ⎬
cm, I1 = 5,0 A e I2 = 2,0 amperios, calculen:
a) la fuerza en el segmento AC que se debe a la [(b + l) 2
+ y 2 ⎪⎭]
corriente en el alambre largo, y
b) la fuerza en el segmento AE que se debe a la →
corriente en el alambre largo. c) Calculen la fuerza total F sobre alambre de
longitud L.


67. Dos elementos de corrientes I 1 d l 1 e
64. Sea nuevamente el sistema del problema →
anterior. Esta vez, calculen la fuerza sobre el I 2 d l 2 se colocan uno en relación al otro, como
alambre recto y largo debida a la medición se muestra en la figura.
magnética producida por el lazo rectangular. a) Calculen la fuerza que ejerce el elemento
Respuesta. →
μ 0 I1 I 2 h ⎛ 1 1 ⎞ I 1 d l 1 sobre el otro.
⎜ − ⎟ , (izquierda)
2π ⎝ a b ⎠ b) Calculen la fuerza que ejerce el elemento

I 2 d l 2 sobre el otro.
65. Un lazo rectangular de alambre de lados de 10
c) Expliquen la razón por la que, cuando los
y 30 cm lleva una corriente de 15 A. Calcular las
resultados en (a) y (b) no sean iguales y opuestos,
fuerzas mutuas de repulsión entre los dos pares de
no haya contradicción esencial con la ley de
alambres opuestos.
Newton de la acción y la reacción.

46
Campo magnético Hugo Medina Guzmán

Respuesta. 9,4 x 10-3 N.m

73. Se suspende un lazo rectangular de lados a y b


de tal modo que tenga libertad para girar en torno
al eje horizontal AB. Si tiene una masa m y si la
corriente en torno es I, calculen el ángulo θ al
que estará en equilibrio en presencia de un campo

magnético vertical uniforme B .

68. En cierto sistema de coordenadas fluye una


corriente I2 a lo largo de un alambre infinitamente
largo que se encuentra a lo largo del eje x.
Demuestren que la fuerza magnética sobre el
segundo alambre de longitud l y que lleva una 74. Sea un lazo plano de alambre que lleva una
corriente I1 cuyos puntos extremos están en los corriente I en presencia de un campo magnético
puntos (0, 0, a) y (0, l , a) es →
uniforme B .
→ μ 0 I1 I 2 ⎛ l 2 ⎞ a) Demuestren que el torque τ en torno a un
F = iˆ ln⎜⎜1 + 2 ⎟⎟
4π ⎝ a ⎠ punto P dentro del lazo se puede expresar,

τ = iˆ ∫ r × ⎛⎜ d l × B ⎞⎟ ,
→ → → →
utilizando, en la forma
69. Sea la misma situación que en el problema ⎝ ⎠
anterior, suponiendo esta vez que los puntos →
extremos del segmento más corto están en los en donde r es el vector del punto P al elemento
puntos (0, − l 2 -, a) y (0, l 2 , a). →

a) Demuestren que esta vez no hay fuerza sobre el Id l .


segmento más corto. b) Utilizando el hecho de que la integral se debe
b) Calculen el torque en torno al punto (0,0, a) llevar en torno a un lazo cerrado, demuestren que
sobre el segmento más corto. otra fórmula equivalente para τ es

τ = iˆ⎛⎜ ∫ r ×d l ⎞⎟ × B
→ 1 → → →
70. Calculen los momentos dipolares magnéticos
asociados a cada uno de los lazos planos 2 ⎝ ⎠
siguientes (suponiendo que en cada caso, la c) Finalmente, demuestren que la integral
corriente es de 2,0 A y que hay 10 vueltas en cada 1 → →
2∫
lazo): r ×d l es el área vectorial del lazo y, en esa
a) Un lazo circular de radio de 10 cm.
forma, establezcan la validez general de
b) Un lazo rectangular de lados de 2 y 10 cm. → → →
c) Un lazo de forma elíptica de eje semi mayor de τ = m× B .
10cm y semi menor de 5cm.
Respuesta.
a) 0,63 A.m2, b) 4,0 x 10-2 A.m2, c) 0,31 A.m2

71. Al calcular el trabajo que se requiere para


hacer girar un lazo de corriente de momento

dipolar en un campo magnético B , demuestren
que la energía U asociada con él es → → →
→ → 75. Demuestren que τ = m× B es válido para un
U = − m⋅ B lazo de forma arbitraria, reemplazando el lazo
(Indicación: El trabajo dW que se re quiere para dado de corriente por un conjunto de lazos
hacer girar un dipolo en un pequeño ángulo dα rectangulares contiguos y muy delgados, cada uno
es rdα , donde τ es el torque que se tiene que de los cuales tenga la misma corriente I.
aplicar.)
72. Una bobina circular de alambre de radio de 76 Sea un alambre de longitud fija l y que lleve
10cm y 150 vueltas lleva una corriente una corriente I. Este alambre se puede formar en
de 10-2 A. Cuál es el torque máximo que se puede varios lazos, tales como el cuadrado del lado
ejercer sobre esta bobina mediante un campo l 4 , n lazos circulares cada uno de ellos de
magnético uniforme de una intensidad de 0,2 T.

47
Campo magnético Hugo Medina Guzmán

radio 1 2π n , etc. Demuestren que el torque área de corte transversal de la lámina metálica sea
máximo en cualquiera de ellos en un campo de 2 cm2 y que Et = l0 V/m, calculen:
→ a) La velocidad de deriva de los electrones.
magnético uniforme B se logra cuando el b) El valor del coeficiente de Hall.
alambre forma un circulo de radio 1 2π y e) El número de portadores de carga por unidad de
calculen la torque en este caso. volumen, comparando esto con el número de
Respuesta. átomos de sodio por unidad de volumen. La masa
atómica del sodio es de 23 y su densidad de l0
Il 2 B kg/m3.
4π Respuesta.
a) 1.3 x 10-5 m/s, b) 2,5x 10-10 m3/C, c) 2,5 x 1028
77. Un lazo circular de alambre de masa m y radio portadores de carga/m3
a lleva una corriente I y tiene libertad para girar
en torno a un diámetro horizontal AC en presencia 79. Supongan que el galvanómetro de bobina
→ →
de un campo B uniforme, dirigido verticalmente pivotante de la figura tiene un campo radial B de
hacia arriba. Si está distribuido en una cantidad una intensidad de 0,3 T, y que la bobina misma
ligera a partir de su posición de equilibrio, tenga 200 vueltas y un área de 3,0 cm2. Calculen
demuestren que oscilará en torno a AC con un la constante de resorte (torsión/desplazamiento
movimiento armónico simple de periodo angular), suponiendo que una corriente de 1,0 mA
produzca una deflexión angular de 15°.
⎛ 2π m ⎞
12

T =⎜ ⎟
⎝ IB ⎠
(Nota: El momento de inercia de este lazo en
1
torno a un diámetro es de I inercia = ma 2 .)
2
78. En un experimento para medir el efecto de
Hall en el sodio, supongan que se utiliza un
campo magnético de una intensidad de 0,8 T y se
mide una corriente de 10 A. Suponiendo que el

48
Ley de Faraday e inductancia Hugo Medina Guzmán

CAPÍTULO 4. Ley de Faraday e inductancia


INTRODUCCION LEY DE LENZ. Hasta este momento no hemos
Si tenemos una carga q, ésta ocasiona un campo explicado el signo menos, esto nos indica que la
→ fuerza electromotriz aparece en oposición al cambio
eléctrico E y si esta carga está moviéndose con una que se produce sobre él. Es decir, si incrementamos el
→ flujo magnético a través del circuito, la fuerza
velocidad v ocasionará también un campo electromotriz indicada tiende a causar corriente en tal
magnético. Es decir, tenemos asociados tanto un dirección que el flujo decrezca, si tratamos de
campo eléctrico en movimiento con un campo introducir un imán en una bobina, la corriente
magnético. En 1839 fue Michael Faraday quien inducida tendrá un sentido tal que forma un campo
presentó un informe de sus observaciones de un magnético que tiende a repeler el polo magnético.
campo eléctrico asociado con un campo magnético en Quien estableció el sentido de las corrientes
movimiento. A partir de estos experimentos se han Inducidas fue H.F. Lenz, contemporáneo de Faraday
desarrollado los generadores modernos, los con el siguiente enunciado conocido como Ley de
transformadores, etc. Lenz: “La corriente que es inducida tendrá una
En este Capítulo trataremos de la formulación de las dirección de tal forma que se oponga a la causa que la
leyes de la inducción y su aplicación en casos produce”; que es una consecuencia de la
simples. conservación de la energía, de no ser así la corriente
inducida podría seguir induciendo corriente y a la vez
LEY DE FARADAY aumentándola, lo que no es cierto.
Faraday observó experimentalmente la asociación de

Ejemplo 1. Un solenoide de longitud l y n2 vueltas,
un campo magnético variable en el tiempo B con un
campo eléctrico. diámetro D2 y resistencia R en su interior hay un
En la figura siguiente se muestra un imán en solenoide de igual longitud y n1 vueltas, diámetro D1
movimiento y el efecto que hace sobre una aspira de conectado a una pila por medio de una llave S.
alambre conectada a un galvanómetro. Determinar la corriente que circula en el solenoide
Se ve efecto solamente cuando el imán está en exterior al momento de cerrar le llave S en función de
movimiento no así cuando está estático. la corriente y en el solenoide Interior.

Solución.
El campo magnético producido por el solenoide
interior es
n1
B = μ 0 N1 I1 = μ 0 I1
l
El flujo magnético es
n1 ⎛ πD12 ⎞
El resultado de un gran número de experimentos Φ = BA = μ 0 I1 ⎜ ⎟
puede resumirse asociando a una fuerza electromotriz l ⎜⎝ 4 ⎟⎠
con un cambio de flujo magnético. Es fuerza electromotriz inducida en el solenoide
dΦ exterior es:
ε =−
dt d d μ 0 n1πD12
Esto se conoce como la Ley de Faraday.
ε = −n2 Φ = −n2 I1
dt dt 4l
→ →
Como ε = ∫E ⋅d l μ n n πD 2 dI
L ε =− 0 1 2 1 1
→ 4l dt
y Φ = ∫ B ⋅ nˆ dS La corriente es:
S
ε μ 0 n1 n2πD12 dI 1
Podemos escribir que
→ →
I2 = =
d → R 4 Rl dt
∫E ⋅d l = −
L dt ∫S
B ⋅ nˆ dS

1
Ley de Faraday e inductancia Hugo Medina Guzmán

De este resultado deducimos que tendremos corriente Ejemplo 3. El eje de una bobina de 250 vueltas y del
I 2 mientras la corriente I 1 este variando, o sea, área 0,002 m2 se alinea a 40° con un campo
magnético uniforme. ¿El campo magnético
mientras sube de su valor cero hasta que toma su
disminuye a partir de 0,08 a 0,02 T en 0,020 s. cuál
valor máximo.
es la fuerza electromotriz media generada en la
bobina?
Ejemplo 2. Tenemos dos circuitos como los
Solución.
mostrados en la figura. Usar la ley de Lenz para
determinar la dirección de la corriente inducida en la
resistencia R’, cuando.
a) Le bobina A se acerca a la bobina B.
b) Se disminuye la resistencia R.
c) Se abre la llave S.

ΔΦ B nAΔB
ε= = cos θ
Δt Δt
=
(250)(0,002)(0,06)(cos 40º )
0,02
= 1,15 V
Solución. Ejemplo 4. Acercando un imán a una bobina de 2000
a) Al acercarse B hacia A el flujo magnético aumenta espiras se incrementa el flujo magnético que corta a
en B, para contrarrestar esto debe aparecer un campo la bobina de 0 a 1,5 x 10-5 Wb en 1/10 de segundo. Si
la resistencia de la bobina es de 20 Ω , determine la
magnético en oposición. Este campo magnético sería
originado por una corriente como la indicada en la
corriente media que se induce en la misma.
figura.

Solución.
b) A1 disminuir 1a resistencia R aumenta la corriente ε ε
y al aumentar la corriente el flujo magnético se I= =
R 20
incrementa. En oposición a este cambio debe aparecer
dΦ 1,5 × 10 −5
un campo magnético originado por una corriente
ε =− = −2000
como la indicada en la figura. dt 10 −1
= - 0,3 V
0,3
I= = 0,015 V
20
Ejemplo 5. A través de un solenoide de 1000 vueltas
pasa un flujo magnético de 10-3 Wb. Si el flujo se
reduce en 10-3 s a 10-4 Wb. ¿Cuál será en voltios la
c) Al abrir la llave la corriente empieza a disminuir su fuerza electromotriz que se induce?
valor hasta cero, esto ocasiona una disminución de Solución.
flujo magnético. En oposición a este cambio debe La variación de flujo magnético es
aparecer un campo magnético originado por una ΔΦ = 10 −3 − 10 −4 = 9 × 10 −4 Wb
corriente como la indicada en la figura. y, por tanto, la fuerza electromotriz inducida es:
ΔΦ 9 × 10 −4
ε=n = 1000
Δt 10 −3
= 900 V.

Ejemplo 6. Un solenoide alargado transporta una


corriente que produce un campo magnético B en su
centro. Se introduce el solenoide dentro de una
bobina estrecha de 20 espiras y área 2 cm2 que tiene
2
Ley de Faraday e inductancia Hugo Medina Guzmán

una resistencia total de 1 Ω . Al conectar los ΔΦ BA


extremos de la bobina e invertir la corriente que ε =n =n
circula el solenoide se produce en la bobina una
Δt Δt
corriente inducida de 10 μA durante 0,1 s. Calcule el 0,5π (0,1) 40π
2
= 1× = = 0,4π
campo B. 1 80 100
Solución. = 1,26 V.
Al invertir la corriente la variación del flujo es:
ΔΦ = nBA − (− nBA) = 2nBA Ejemplo 9. Una bobina de n vueltas y de área A se
De aquí coloca en un campo magnético B y se la hace rotar
ΔΦ 2nBA con velocidad angular constante ω sobre un
ε= = diámetro perpendicular al campo magnético. Derive
Δt Δt una expresión para la fuerza electromotriz inducido
2 × 20 B (2 × 10 −4 ) en la bobina.
= = 8 × 10 − 2 B Solución.
0,1
Φ B = nAB cos θ = nAB cos ωt ,
La corriente inducida es:
ε 8 × 10 −2 B dΦ B
I = 10 × 10 −6
= = ε =− = nABsenωt
R 1 dt
De donde resulta: Ésta es la base para un generador de corriente alterna.
B = 1,25 x 10 T. Por medio de los contactos convenientes, puede
generarse corriente continua.
Ejemplo 7. Un campo magnético se puede medir de
la manera siguiente: Una bobina de 250 vueltas y de Ejemplo 10. Una bobina de 1 000 espiras y radio 5
área 1,80 cm2 se coloca en un imán permanente de cm conectada a un galvanómetro, y situada
modo que el flujo magnético máximo pase a través de perpendicularmente al campo de un electroimán se
la bobina. Se conecta con un galvanómetro que mide extrae bruscamente del mismo. El galvanómetro,
el flujo total de la carga. Cuando la bobina se mueve cuya resistencia es de 1 000 Ω , acusa en este
de un tirón rápidamente fuera del imán, se observa proceso una carga total inducida de 10 C.
que fluye una carga de 0,25 mC. El circuito del Determínese la inducción magnética del electroimán,
galvanómetro de bobina tiene una resistencia de 4 sabiendo que la bobina tiene una resistencia de 20
Ω . ¿Cuál es el campo magnético? Ω.
Solución. Solución.
Δq ε ΔΦ B nAB Φ
I= = = = ε =n = IR ,
Δt R RΔt RΔt t
RΔq (4) 0,25 × 10 −3( ) Φ ItR
B= = =
(
10 −3 (1020) )
⇒ B=
nA
=
(250) 1,8 × 10 −4 ( ) A nA (1000) π 0,05 2 ( )
= 0,022 T = 0,13 T

Ejemplo 8. Una espira circular de radio 10 cm está FUERZA ELECTROMOTRIZ DEL


situada perpendicularmente al campo magnético de MOVIMIENTO
0,5 T de un electroimán. ¿Cuál es la fuerza Un ejemplo interesante de una fuerza electromotriz
electromotriz que se induce en la espira cuando gira inducida ocurre cuando un conductor se mueve a
alrededor de su diámetro con una velocidad de 1 200 través de un campo magnético. En la figura a
rpm? continuación una barra del metal de longitud l
orientada perpendicularmente a un campo magnético
uniforme y movido con velocidad constante
perpendicular a su longitud y al campo. Un portador
→ →
positivo de carga experimentará una fuerza q v × B
dirigida hacia la parte superior de la barra.
Solución.
1200
ω = 1200 rpm = = 20 rps
60
En 1/4 de vuelta (equivalente a 1/80 segundos) la
espira pasa de una posición de máximo número de
líneas de flujo cortadas a otra posición de mínimo
(cero). Por tanto,
3
Ley de Faraday e inductancia Hugo Medina Guzmán

La carga positiva se acumulará en el extremo superior Φ = BA = Blx



Al moverse el conductor con velocidad v se produce
de la barra y formará un campo eléctrico E , tal que un cambio en este flujo
en el qE = qvB en la situación constante. Esto dΦ dx
significa que se desarrollará una fuerza electromotriz = Bl = Blv
entre los extremos de la barra, dt dt
Por la ley de Faraday esto produce una fuerza
ε = ∫ Edy = E l = B l v . Podríamos electromotriz.
también llegar este resultado usando la ley de dΦ
Faraday. ε '= − = − Blv
dt
Considere un lazo rectangular imaginario, indicado
Haciendo circular una corriente I’ en oposición a I
por la línea discontinua. La barra forma un lado del
(por la Ley de Lenz)
lazo. En el tiempo dt la barra se mueve una
ε' Blv
distancia vdt hacia la derecha, aumentando el área I'= =−
del lazo vdtl . Esto aumenta el flujo a través del R R
Siendo R la resistencia del circuito.
dΦ B
lazo en dΦ B = Bvldt , tal que ε= = Bvl .
dt Otra manera de encontrar esta fuerza electromotriz
Esta clase de fuerza electromotriz es conocida como inducida es mediante la fuerza de Lorentz y la
fuerza electromotriz de movimiento. conservación da la energía.
ε = Bvl La potencia suministrada es ε 0 I , se consume
I 2 R en la resistencia y Fv en mover al alambre con
Ejemplo 11. Se ha sugerido que Las aves podrían
una velocidad v .
utilizar la fuerza electromotriz inducida entre los
De tal modo que: ε 0 I = I R + Fv
2
extremos de sus alas por el campo magnético de la
tierra como un medio para ayudarles en su
navegación durante la migración. Qué fuerza
ε0
El valor de I es menor que , el valor en caso
electromotriz sería inducida para que un ganso de R
Canadá con una extensión de alas de 1,5 m que vuela estático.
con una velocidad de 10 m/s en una región donde la Para evitar esta dificultad sumamos una variable
componente vertical del campo de la tierra es 2 x 10-5 adicional con el fin de mantener a I constante.
T.
Solución.
(ε 0 + ε ')I = I 2 R + Fv
ε = Blv = (2 x 10-5)(1,5)(10) Aquí corresponde ε0I = I 2R
= 3 x 10-4 V = 0,3 mV.
Esto probablemente es demasiado pequeño para ser
Dejando ε ' I = Fv = (IlB )v ⇒ ε ' = Blv
detectado por las aves, puesto que los voltajes ε ' no es la fuerza electromotriz inducida, es el
celulares son típicamente 70 mV. Sin embargo, para negativo de ella, cuyo fin es mantener la corriente
los jets 747 con una extensión de alas de 60 m y una constante
velocidad de 900 km/h, el efecto es apreciable. dΦ
ε '= −
Fuerza electromotriz producida por un alambre dt
que se desliza en un campo magnético.
Supongamos dos rieles horizontales paralelos Ejemplo 12. Una barra horizontal de 5 m de longitud
orientada en la dirección Este-Oeste cae desde lo alto
separados una distancia l , como se muestra en la
de un precipicio formando un ángulo recto con la
figura. componente horizontal del campo magnético de valor
2,4 x 10-5 T. ¿el valor instantáneo de la fuerza
electromotriz inducida en la barra a los 5 s y a los 10
s de caída?
Solución.
La velocidad de la barra a los 5 s y 10 s de caída será,
respectivamente,
Sea un campo magnético perpendicular al plano. v5 = gt 5 = 9,81 × 5 = 49,05 m/s,
Debido a la corriente I, el conductor sufre una fuerza v10 = gt10 = 9,81 × 10 = 98,10 m/s.
de magnitud F = lIB F hacia la derecha. Esta fuerza
La fem inducida en la barra en cada caso será:
lo acelera hacia la derecha, de tal manera que en un
determinado lapso de tiempo se mueve con una ε 5 = Blv5 = 49,05 x 2,4 x 10-5
velocidad v . = 58,86 x 10-4 V
El flujo magnético a través de la aspira es: ε 10 = Blv10 = 98,10 x 2,4 x 10-5
4
Ley de Faraday e inductancia Hugo Medina Guzmán

= 117,72 x 10-4 V

Ejemplo 13. ¿Qué fuerza electromotriz se induce en


el eje frontal de un automóvil que se dirige hacia el
Norte a la velocidad uniforme de 100 km/h si el eje
tiene 1,5 m de longitud y la componente vertical del
campo magnético terrestre es de 5 x 10-5 T? ¿Qué
extremo del eje estará a mayor potencial?
Solución.
El eje se desplaza con una velocidad de: 100 km/h
=27,78 m/s. Al inducirse corriente aparecen fuerzas sobre la
Por tanto, la fuerza electromotriz inducida en un espira, F1 y F2 son iguales en magnitud y opuestas
campo de 5 x 10-5 T será:
ε = Blv = 27,78 x 5 x 10-5 x 1,5 en sentido, F3 = IlB en el sentido indicado, como
= 2,08 x 10-3 V. la espira se desliza a velocidad constante la fuerza
Las cargas positivas (regla de la mano derecha) se externa Fexterna es igual a F3 .
desplazan en estas condiciones de E a O. Por tanto,
estará a mayor potencial el extremo izquierdo (oeste) Fexterna = IlB
del eje. La potencia desarrollada por el agente externo
P = (Fexterna )v = IlBv
Ejemplo 14. Espira rectangular en presencia de un
campo magnético. Blv
Como I = , tenemos
R
B 2l 2v 2
P=
R
La potencia disipada en al circuito es
Blv
P = I 2 R , con I = :
R
Demostrar que la potencia entregada es igual a la
B 2l 2v 2
potencia disipada.
P=
Solución. R
Supongamos una espira como la mostrada en la figura Con lo que demostramos que la potencia entregada
de resistencia R y que sale con velocidad v de la es igual a la potencia disipada.
acción del campo B.
El flujo magnético encerrado por la espira en un Ejemplo 15. Una espira rectangular de dimensiones
momento dado es l y a con resistencia R se mueve con velocidad
Φ = Bl x constante v hacia la derecha como se muestra en la
Como x está variando figura, penetra en una región donde hay un campo
dΦ dx magnético uniforme perpendicular al plano del papel
= Bl = Blv y hacia dentro de módulo B. Calcular y hacer un
dt dt
La fuerza electromotriz inducida es: gráfico de:
a) El flujo, la fuerza electromotriz y la fuerza sobre la

ε =− = − Blv espira, en función de la posición de la espira, es decir,
dt cuando la espira se está introduciendo, está
La corriente producida es introducida, y está saliendo de la región que contiene
ε Blv el campo magnético.
I= = b) Explíquese el mecanismo (fuerza sobre los
R R portadores de carga) de establecimiento de la
El sentido en oposición a la disminución de líneas de corriente inducida en los tres casos citados.
flujo seria el indicado en la figura siguiente. c) Grafique el flujo, la fuerza electromotriz y la
fuerza neta en función de la posición x.

Solución.

5
Ley de Faraday e inductancia Hugo Medina Guzmán

a) El flujo, la fuerza electromotriz y la fuerza sobre la


espira, en función de la posición de la espira, es decir,
cuando la espira se está introduciendo, está
introducida, y está saliendo de la región que contiene
el campo magnético.

( )
→ →
F1 = I lˆj × B ( l lleva el sentido de la corriente I)
⎛ Blv ⎞ B 2l2v
F1 = IlB = ⎜ ⎟ lB =
⎝ R ⎠ R
Φ = B (lx ) Fuerza neta de F1 hacia la izquierda ( −iˆ) .

( )
→ →
dΦ dx F2 = I − .iˆx × B (x lleva el sentido de la corriente
ε =− = − Bl = − Blv
dt dt I)
ε Blv ⎛ Blv ⎞ B 2 lx
I = =− F2 = IxB = ⎜ ⎟ xB =
R R ⎝ R ⎠ R
El sentido de la corriente es antihorario.
Fuerza neta de F2 hacia abajo ( ˆj ) .

( )
Cuando la espira se encuentra dentro del campo → →
magnético. F3 = I iˆx × B (x lleva el sentido de la corriente I)
⎛ Blv ⎞ B 2 lx
F3 = IxB = ⎜ ⎟ xB =
⎝ R ⎠ R
Fuerza neta de F3 hacia arriba ( − ˆj ) .
→ →
F2 = −F3
Saliendo la espira.
Φ = Bl a

ε =− =0
dt
No hay corriente inducida, I = 0

Cuando la espira esta saliendo del campo magnético.


( )
→ →
F1 = I ( x − 2a ) − iˆ × B [(x – 2a) lleva el sentido
de la corriente I]
⎛ Blv ⎞ B 2l2v
F1 = IlB = ⎜ ⎟ lB =
⎝ R ⎠ R
Fuerza neta de F1 hacia la izquierda ( −iˆ) .
→ →
F2 = I ( x − 2a )iˆ × B [(x – 2a) lleva el sentido de la
Φ = B[a − (3a − x )]l = B( x − 2a )l corriente I].
dΦ ⎛ Blv ⎞
F2 = I ( x − 2a )B = ⎜ ⎟( x − 2a )B
dx
ε =− = Bl = Blv
dt dt ⎝ R ⎠
ε Blv
I= = B 2 l( x − 2a )
=
R R R
El sentido de la corriente es horario.
Fuerza neta de F2 hacia arriba ( − ˆj ) .
→ →
F3 = I (x − 2a )(− i ) × B [(x – 2a) lleva el sentido
Fuerza sobre las corrientes.
Ingresando la espira.
de la corriente I].

6
Ley de Faraday e inductancia Hugo Medina Guzmán

⎛ Blv ⎞
F3 = I (x − 2a )B = ⎜ ⎟( x − 2a )B
⎝ R ⎠
B 2 l( x − 2a )
=
R
Fuerza neta de F3 hacia abajo ( ˆj ) .
→ →
F2 = −F3
b) El mecanismo (fuerza sobre los portadores de
carga) de establecimiento de la corriente inducida en
los tres casos citados.
Ejemplo 16. Un lazo rectangular de dimensiones a x
Cuando se introduce la espira rectangular. b, resistencia R, y masa m está orientado
perpendicularmente a un campo magnético uniforme
horizontal. Se suelta del reposo y baja de modo que
parte del lazo esté fuera del campo, como se muestra
en la figura. ¿Qué velocidad terminal máxima
alcanza el lazo?

→ → →
F B = q v× B
( )

Fuerza neta F 1 hacia la izquierda − iˆ

Con toda la espira rectangular en el campo


magnético.
Solución.
Cuando la fuerza magnética en la corriente inducida
es igual al peso del lazo, la fuerza actuante será cero
y el lazo tendrá aceleración cero.
FB = mg
→ → → Las fuerzas exteriores en las dos ramas laterales se
F B = q v× B cancelan. La fuerza ascendente en la porción superior
No hay corriente, se compensan. es FB = IaB , donde
ε 1 dΦ B Badx Bav
Cuando se saca la espira rectangular. I= = = =
R R dt Rdt R
Así
⎛ Bav ⎞ B 2a 2v mgR
mg = B⎜ ⎟ a = y v= 2 2
⎝ R ⎠ R B a
→ → → Observe que la corriente inducida fluye a la derecha
F B = q v× B en un intento de mantener el flujo dentro del lazo
constante.
( )

Fuerza neta F 1 hacia la izquierda − iˆ
c) Gráficos del flujo, la fuerza electromotriz y la Ejemplo 17. Supongamos una espira como la
fuerza neta en función de la posición x. mostrada en la figura de resistencia R y que sale con

velocidad v = v0 iˆ de la acción del campo

B = B0 ˆj .

7
Ley de Faraday e inductancia Hugo Medina Guzmán

B 2l 2v 2
P=
R
La potencia disipada en al circuito es
Blv
P = I 2 R , con I = :
R
B 2l 2v 2
a) Calcule el flujo magnético y la corriente inducida P=
(valor y dirección) cuando un segmento x aún no sale R
la región con campo. Justifique. Con lo que demostramos que la potencia entregada
b) Calcule en es mismo instante las fuerzas es igual a la potencia disipada.
magnéticas sobre cada lado de la espira.
c) Si la velocidad de la espira y el campo magnético Ejemplo 18. Una espira cuadrada de lado l , masa m,
tienen valores constantes, calcule la fuerza externa y resistencia R. se encuentra en la frontera de una
necesaria para que este movimiento se realice. →
d) Calcule el trabajo por unidad de tiempo realizado zona donde actúa un campo magnético uniforme B 0 ,
por la fuerza externa y la potencia disipada por la perpendicular al plano del papel y saliendo de él,
corriente inducida. Comente los resultados. como lo muestra la figura.
Solución. En t = 0, se deja caer la espira la cual se introduce
a) El flujo magnético encerrado por la espira en un →
momento dado es progresivamente en el campo magnético B 0 . Se
Φ = Blx pide:
Como x está variando a) Hallar el flujo magnético en la espira en función de
la distancia vertical y.
dΦ dx
= Bl = Blv b) Encontrar la fem y la corriente inducida en la
dt dt espira. ¿Cuál es el sentido de dicha corriente?
La fuerza electromotriz inducida es: c) Mostrar en un diagrama de cuerpo libre las
dΦ diferentes fuerzas que actúan sobre la espira, e indicar
ε =− = − Blv los valores respectivos.
dt d) Plantear la ecuación diferencial del movimiento
La corriente producida es vertical en función de la velocidad de la espira.
ε Blv e) Resolver la ecuación diferencial planteada en (d), y
I= = obtener la velocidad en función del tiempo.
R R
El sentido en oposición a la disminución de líneas de
flujo seria el indicado en la figura siguiente.

Solución.
a)
b)
Al inducirse corriente aparecen fuerzas sobre la
espira, F1 y F2 son iguales en magnitud y opuestas
F3 = IlB en el sentido indicado.
en sentido,
F1 = F2 = IBx
c) Como la espira se desliza a velocidad constante la
fuerza externa Fexterna es igual a F3 .
Fexterna = IlB El flujo magnético en la espira en función de la
d) La potencia desarrollada por el agente externo distancia vertical y:
P = (Fexterna )v = IlBv Φ B = B0 A = B0ly
Blv b) La fem:
Como I = , tenemos
R
8
Ley de Faraday e inductancia Hugo Medina Guzmán

1 2 ⎛
v
y= gt mRg ⎞ B02 l 2 t
2 ln⎜⎜ v − 2 2 ⎟⎟ = − t ⇒
⎝ B0 l ⎠ 0 mR 0
dΦ dy
ε = − B = − B0l = − B0lv
dt dt mRg
v− 2 2
La corriente inducida en la espira B0 l B02 l 2
ε B0lv ln =− t⇒
I= = mRg
− 2 2
mR
R R B0 l
Como el campo durante la caída esta aumentando
hacia el papel, por la ley de lenz en oposición a ese mRg
aumento debe aparecer un campo magnético opuesto, v− 2 2 B 2l 2
B0 l − 0 t
este se debería a una corriente en la espira en sentido = e mR ⇒
horario. mRg
c)
− 2 2
B0 l
B 2l 2
mRg mRg − 0 t
v − 2 2 = − 2 2 e mR ⇒
B0 l B0 l
⎛ − 0 t ⎞
2 2
B l
mRg ⎜1 − e mR ⎟
v= 2 2
B0 l ⎜ ⎟
⎝ ⎠
Esta velocidad es mientras la espira está ingresando
en el campo, después de haber ingresado
completamente la velocidad llega a un valor v ' .
mRg
B 2l 2v v' < vlímite = (Cuando a = 0 ).
F = IlB0 = 0 B02 l 2
R Desde este instante en adelante la velocidad estará
∑ F = ma dada por:
1
mg − F = ma ⇒ mg − IlB0 = ma ⇒ v = v'+ gt '2 .
2
B02 l 2 v dv
mg − =m
R dt
La ecuación diferencial del movimiento vertical en
función de la velocidad de la espira. Es:
dv B02 l 2 v
+ −g=0
dt mR
d)
dv B 2l 2v
=− 0 +g ⇒
dt mR Ejemplo 19. Sabiendo que el módulo del campo
dv B02 l 2 ⎛ mRg ⎞ magnético producido por una corriente rectilínea
=− ⎜⎜ v − 2 2 ⎟⎟ ⇒ indefinida de intensidad I a una distancia r vale
dt mR ⎝ B0 l ⎠
μ0 I
dv B 2l 2 B=
= − 0 dt 2π r
⎛ mRg ⎞ mR a) Calcular la diferencia de potencial entre los
⎜v − 2 2 ⎟
⎜ B0 l ⎟⎠ extremos A y B de una varilla que se mueve
⎝ paralelamente a la corriente rectilínea con velocidad
Integrando: v.
b) ¿Cuál es el potencial más alto de los dos?. Razonar
las respuestas

9
Ley de Faraday e inductancia Hugo Medina Guzmán

¿Cuál punto a o b es el de mayor potencial?


Justifique.

Solución.
a) Un portador de carga positiva de la varilla, al b) Suponga que se sustituye la barra por una espira
moverse con velocidad v, en el campo magnético rectangular de alambre de resistencia R. Calcule el
producido por la corriente rectilínea indefinida flujo magnético y la corriente inducida en la espira.
experimente una fuerza FB = qvBsen 90º , dirigida ¿Cuál es el sentido de la corriente inducida?
a lo largo de la varilla hacia A.

c) Lo mismo que la parte anterior, pero para una


Las cargas positivas se acumulan en A y las negativas
corriente variable en el tiempo en el alambre igual a
en B.
I (t ) = I 0e −αt , donde α . es una constante positiva
de unidad s-1.
d) Para la situación de la parte c). ¿Cuál es la fuerza
magnética resultante sobre la espira? El resultado
puede ser expresado en función de la corriente
→ inducida I ind de la espira.
Surge un campo eléctrico E entre A y B, que se
opone a que se siga acumulando carga. Solución.
En el equilibrio a) Un portador de carga de la varilla, al moverse con
velocidad v, en el campo magnético producido por la
μ0 I
FB = FE ⇒ qvB = qE ⇒ E = vB = v corriente rectilínea experimente una fuerza
2π r → → →
FB = q v × B , dirigida a lo largo de la varilla hacia
b b μ0 I μ I b dr
V A − VB = ∫ Edr = ∫ v dr = v 0 ∫ b.
a a 2π r 2π a r
μ0 I b
= v ln
2π a
b) El potencial más alto esta en a (+), es un punto
mas cercano al alambre con corriente.

Las cargas positivas se acumulan en a y las negativas


en b.

Ejemplo 20. El alambre recto y largo conduce una


corriente constante I. Una barra metálica de longitud
L se desplaza a velocidad constante v, como se
muestra en la figura. El punto a está a una distancia d

del alambre. Calcule la fem inducida en la barra. Surge un campo eléctrico E entre a y b, que se opone
a que se siga acumulando carga.
10
Ley de Faraday e inductancia Hugo Medina Guzmán

En el equilibrioFB = FE ⇒ qvB = qE μ 0 aI 0α ⎛ d + L ⎞ −αt


= ln⎜ ⎟e
μ I 2π ⎝ d ⎠
⇒ E = vB = v 0
2π r
ε μ0 aI 0α ⎛ d + L ⎞ −αt
La fem inducida en la barra. I ind = = ⎜ ⎟e
R 2πR ⎝ d ⎠
d +L
Va − Vb = ∫ Edr
d El sentido de la corriente inducida de acuerdo con la
d +L μ0 I μ I d +L dr ley de Lenz, como la corriente va disminuyendo, el
= ∫
d
v
2π r
dr = v 0
2π ∫
d r
campo magnético también, luego para que aparezca
un campo en oposición la disminución del campo
μ 0 Iv d + L debe circular una corriente inducida en sentido
= ln horario.
2π d
El punto de mayor potenciales a. d) Para la situación de la parte c). ¿Cuál es la fuerza
magnética resultante sobre la espira? El resultado
puede ser expresado en función de la corriente
inducida I ind de la espira.

b) Suponga que se sustituye la barra por una espira


rectangular de alambre de resistencia R. Calcule el
flujo magnético y la corriente inducida en la espira.
¿Cuál es el sentido de la corriente inducida?
→ μ0 I
F 1 = I ind a ˆj
2πd
→ →
F2 = −F3
→ μ0 I
F 4 = − I ind a ˆj
2π (d + L )
La fuerza magnética resultante sobre la espira
→ → → μ0 I μ0 I
F = F 1 + F 4 = I ind a ˆj − I ind a ˆj
d +L μ0 I μ Ia ⎛ d + L ⎞ 2πd 2π (d + L )
ΦB = ∫ adr = 0 ln⎜ ⎟
d 2πr 2π ⎝ d ⎠
→ μ 0 II ind a ⎡ 1 1 ⎤
F= − ˆj
2π ⎣ d (d + L ) ⎥⎦

Como el flujo es constante no hay fem inducida, por
lo tanto tampoco corriente inducida.

No hay corriente inducida, puesto que el flujo En oposición al ingreso.


magnético es constante en el tiempo.
c) Si la corriente en el alambre igual a Ejemplo 21. Una varilla conductora de masa 10 g
I (t ) = I 0e −αt .
desliza sobre carriles paralelos distantes 20 cm y que
forman un ángulo de 30º con el plano horizontal. Los
carriles se cierran por la parte inferior, tal como se
dΦ B d +L μ I
ε =− , ΦB = ∫
0
adr indica en la figura. En la región existe un campo
dt d 2πr magnético uniforme y perpendicular al plano
horizontal de intensidad 1 T.
μ0 Ia ⎛ d + L ⎞ a) Calcular la fuerza electromotriz en función de la
= ln⎜ ⎟
2π ⎝ d ⎠ velocidad constante de la varilla. La intensidad de la
corriente inducida si la resistencia del circuito es de
10 Ω. La(s) fuerza(s) sobre la varilla.
dΦ B μ aI ⎛ d + L ⎞ −αt
ε =− = − 0 0 ln⎜ ⎟e
dt 2π ⎝ d ⎠ b) ¿Cuánto valdrá la velocidad de la varilla cuando
desliza con movimiento uniforme? (se desprecia el
rozamiento).

11
Ley de Faraday e inductancia Hugo Medina Guzmán

Razonar las respuestas dibujando los esquemas

mgsen30º = FB cos 30º


3
(0,01)(9,8)(0,5) = 0,002 3v
2
50 m
Solución. ⇒ v=
a) 3 s
Ejemplo 22. Faraday inventó un dispositivo
ingenioso llamado generador homopolar, o disco de
Faraday. Un disco de cobre del radio r se monta con
su eje paralelo a un campo magnético uniforme B. El
disco rota en la frecuencia angular ω . el contacto
eléctrico con las escobillas que conducen se hace en
los puntos A y C, en el perímetro y el eje del disco.
¿Qué fuerza electromotriz se genera entre los
terminales A y C?
El sentido de la corriente inducida es en oposición a
la disminución del flujo magnético.
El flujo es

Φ = B⋅ nˆ A = BA cos 30º
⎛ 3⎞
= (1)⎜ 0,2 x ⎟ = 0,1 3 x Wb
⎜ 2 ⎟
⎝ ⎠
b) La fuerza electromotriz es
dΦ dx Solución.
ε =− = −0,1 3 = 0,1 3 v V Imagine un lazo cerrado consistente en de las
dt dt conexiones mostradas, más un segmento que conecta
ε 0,1 3 A y C, más un pedazo radial del disco del centro a un
Y la corriente I = = v punto en la circunferencia, más un arco a lo largo de
R 10 la circunferencia.
En el tiempo dt este pedazo radio barre un triángulo
Fuerzas sobre la varilla
r (rdθ ) = r 2ωdt . Esto
1 1
pequeño de área
2 2
aumenta el flujo a través del lazo en
1 2
dΦ B = BdA = Br ωdt . Así la fuerza
2
electromotriz inducida entre A y C es:
dΦ B 1 2
ε= = Br ω
→ → →
dt 2
F B = I l× B ,
→ Ejemplo 23. Determinar la diferencia de potencial
( l lleva el sentido de la corriente I ) entre los extremos de una barra metálica que gira con
velocidad angular uniforme ω dentro de un campo
FB = I (0,2 )(1)sen90º = 0,02
0,1 3 magnético B como el mostrado en la figura.
v
10
= 0,02 3v N
Para que vaya a velocidad constante ambas fuerzas
deben ser iguales.

12
Ley de Faraday e inductancia Hugo Medina Guzmán

Solución.
Escogemos un elemento diferencial dr a una
distancia r del centro, tendrá una velocidad tangencial Solución.
v = ω r . El elemento de longitud dr se mueve con Consideremos una banda radial del disco. Como
une velocidad v en un campo B perpendicular el. De 2400
allí que la fuerza electromotriz entre sus extremos es: ω = 2400 rpm = = 40 rps , en una vuelta,
60
dε = B(dr )v = B(dr )(ω r ) = Bω rdr o sea en 1/40 segundos cada banda radial del disco
La fuerza electromotriz entre los extremos del cortará todas las líneas de flujo que cubre su área A.
alambre es: Por tanto,
Φ = BA = 1 × π (0,25) = 0,196 Wb,
L 2
L r2 1
ε = ∫ dε = ∫ Bω rdr = Bω = BωL2 ΔΦ 0,196
2 2 ε =− =
0
0 = 7,85 V.
Δt 1 40
Calculo por medio de la Fuerza de Lorentz. Otra
forma de llegar al mismo resultado es mediante la EL BETATRÓN. Un campo magnético que
Fuerza de Lorentz cambiante en el espacio crea un torbellino como
campos eléctricos, y esto es verdad sin o con un
conductor presente. Tales campos se utilizan en el
betatrón para acelerar electrones a alta velocidad.
Un “anillo” evacuado se coloca en el campo de un
electroimán grande, y los electrones viajan en órbitas
circulares. El campo magnético se varía
sinusoidalmente, y por un cuarto del ciclo los
electrones son acelerados por el campo eléctrico
inducido. Así el campo magnético sirve para hacer
girar a los electrones en órbitas circulares y
acelerarlos. Para lograr esto, el campo magnético
→ → → debe ser más débil en la órbita que en el interior de la
F = q v × B , F = qvB trayectoria circular, según lo indicado en la figura.
El trabajo realizado por la fuerza en la distancia dr es:
dW = Fdr = qvBdr = qBω rdr
el trabajo total es
L
L r2
W = ∫ dW = ∫ qBω rdr = qBω
0 2 0
1
= qBωL2
2 Ejemplo 25. Demuestre que el campo magnético en
El trabajo total por unidad de carga es la órbita en un betatrón debe ser la mitad del campo
W 1 magnético promedio sobre el área encerrada por la
= ΔV = ε = BωL2 órbita.
q 2
Solución.
La fuerza magnética proporciona la fuerza centrípeta
Ejemplo 24. Un disco de cobre de 25 cm gira con
mv 2
una velocidad angular de 400 rpm alrededor de su eje necesaria, tal que qvB = . La fuerza
y perpendicularmente a un campo magnético de 1 T. r
¿Qué diferencia de potencial se induce entre el borde electromotriz inducida es
del disco y su centro? → dΦ E
ε = ∫ E ⋅ nˆ dS = 2π rE = .
dt

13
Ley de Faraday e inductancia Hugo Medina Guzmán

Por la segunda ley de Newton, la fuerza sobre el ε →


b) i = , Φ B = B⋅ nˆ A = Bl cos ωt
2
electrón es
d (mv )
R
1 dΦ B
F= = qE = dΦ B d
dt 2π r dt Luego ε =− = − Bl 2 cos ωt
Desde que v = 0 en t = 0, dt dt
= Bl ω senωt
2
1
mv = Φ B = qrB
2π r Bl 2ω
Finalmente i = senωt
1 R
Pero Φ B = π r2 Bm , tal que B = Bm . c) La figura muestra la vista lateral y la vista frontal
2
sobre el plano xy.
Ejemplo 26. La espira conductora cuadrada en el
plano xy de la siguiente figura de lado l y resistencia
R gira con velocidad angular ω uniforme alrededor
del eje x. La espira se encuentra inmersa en un campo
magnético uniforme B que tiene la dirección del eje z.
a) Si el movimiento empieza cuando el flujo es
máximo, calcule el valor de flujo magnético inicial y
el sentido de la corriente inducida por el movimiento.
b) ¿Cuál es la corriente inducida en función del → →
tiempo? Actúan las fuerzas F 3 y F 4 , tal que
c) Calcule el momento o torque necesario para F3 = F4 = IlB .
mantener la espira girando a velocidad angular
Estas fuerzas son iguales en magnitud y opuestas en
constante.
sentido, formando así un par de fuerzas de valor
d) Explique si en este proceso se conserva la energía.
τ = F3 lsenθ = IlBlsenθ
con l = A (área de lo espira) podemos escribir
2

τ = IABsenθ
Este par puede escribirse como producto vectorial de

n̂ nˆ
(normal a la superficie A) y B .
→ →
τ = IAnˆ × B .
d) No se conserva la energía, se disipa en forma de
Solución. calor por la resistencia de la espira debido al efecto
→ Joule.
a) Φ B = B⋅ nˆ A
CAMPOS ELECTRICOS INDUCIDOS POR
El flujo magnético inicial Φ B = Bkˆ ⋅ kˆl = Bl
2 2
CAMPOS MAGNETICOS VARIABLES CON EL
Si consideramos que el movimiento se inicia en la TIEMPO.
posición mostrada en el dibujo (θ = 0) , para Tenemos que
θ = π 2 , el flujo es cero, para θ = π , el flujo es
→ → → → dΦ
ε = ∫E⋅d l y ∫E ⋅d l = − ,
máximo nuevamente pero en sentido contrario, como dt
la fuerza electromotriz inducida es alterna, la es decir un campo magnético que varia con el tiempo
corriente también es alterna. Como al inicio el flujo nos induce un campo eléctrico. El sentido del campo
es máximo y va disminuyendo, la fuerza eléctrico lo obtenemos mediante la ley de Lenz,
electromotriz es en oposición a ese cambio, la consideremos una espira circular en presencia de un
corriente inicial será en el sentido indicado en el campo magnético tal como se muestra en la figura a
dibujo. continuación, si el campo magnético se está
incrementando, se produce un campo eléctrico
tangente a la trayectoria circular con una dirección
contraria al sentido de las agujas del reloj

14
Ley de Faraday e inductancia Hugo Medina Guzmán

El campo en la barra es
Ebarra = E cos θ
si el campo va en disminución el sentido del campo
eléctrico será en el sentido horario. d
La diferencia entra el campo eléctrico producido por Como cos θ =
r
carga eléctrica y los inducidos es que los primeros
r dB ⎛ d ⎞ d dB
son conservativos. Ebarra = − ⎜ ⎟=−
→ →
2 dt ⎝ r ⎠ 2 dt
∫E⋅d l = 0 En esta expresión observamos que el campo eléctrico
Los segundos no lo son en la barra es constante en cualquier punto.
Los segundos no lo son
→ → dΦ CORRIENTES DE FOUCAULT
∫E ⋅d l = − dt
Acabamos de ver que por un flujo magmático
variable se establecen circuitos definidos.
Frecuentemente se establecen corrientes circulares
Ejemplo 27. Encontrar el campo eléctrico inducido denominadas Corrientes de Foucault o de Eddy en
en una barra metálica de longitud L que se encuentra un trozo de metal como en el núcleo de un
en una región cilíndrica de radio R en el espacio transformador. Por ejemplo, consideremos un bloque
donde el campo magnético cambia con una razón conductor situado entre los polos de un electroimán,
dB si el campo magnético varía con el tiempo se induce
.
dt una fuerza electromotriz en cualquier trayectoria
cerrada en el interior del metal, tal como la curva C.

Solución. La fuerza electromotriz inducida produce una


Para cualquier punto dentro da la región cilíndrica corriente en el circuito. Las corrientes de Foucault
con respecto al centro, el campo eléctrico se puede pueden reducirse construyendo el bloque mediante
obtener a partir de: láminas o tiras la resistencia del trayecto aumente
→ → dΦ debido al pegamento que se utiliza entre láminas, de
∫E ⋅d l = − dt
esta manera se rompen los grandes circuitos y se
reduce en gran manera la pérdida de potencia, una

∫ B ⋅ nˆdS ⇒
aplicación de esta corriente es el freno magnético
Como Φ =
debido a esta corriente y al campo magnético aparece
dΦ d → → → d → una fuerza magnética en oposición al movimiento del
= ∫ B ⋅ nˆ dS y ∫ E ⋅ d l = − ∫ B ⋅ nˆ dS bloque metálico.
dt dt dt
A lo largo de la circunferencia de radio r Ejemplo 28. Un disco de resistividad η , de radio a,
dB
E 2π r = −π r 2 y de espesor b se coloca con su plano perpendicular a
dt un campo magnético variable, B = Bo senω t . Se
r dB
⇒ E=− inducen corrientes circulares llamadas las corrientes
2 dt de Foucault, y la energía se disipa en el disco.
En la figura siguiente mostramos el detalle de la Calcule el índice de la disipación de la energía. Esta
región de radio r pérdida de energía termal presenta un problema en

15
Ley de Faraday e inductancia Hugo Medina Guzmán

dispositivos tales como transformadores. Este efecto Ejemplo 29. La espira conductora cuadrada en el
tiene aplicación en los detectores de metales usados plano xy de la siguiente figura de lado l y resistencia
en los aeropuertos, para encontrar monedas R gira con velocidad angular ω uniforme alrededor
enterradas y en hornos de inducción. del eje x. La espira se encuentra inmersa en un campo
Solución. magnético uniforme B que tiene la dirección del eje z.
Considere un anillo de ancho dr y radio r. el flujo a a) Si el movimiento empieza cuando el flujo es
través de este anillo es Φ = π r B0 senω t ,
2
máximo, calcule el valor de flujo magnético inicial y
La fuerza electromotriz inducida en el anillo es el sentido de la corriente inducida por el movimiento.
b) ¿Cuál es la corriente inducida en función del

ε =− = π r ωB0 cos ω t .
2 tiempo?
dt c) Calcule el momento o torque necesario para
La resistencia del anillo es mantener la espira girando a velocidad angular
⎛L⎞ ⎛ 2π r ⎞ constante.
R = η⎜ ⎟ = η⎜ ⎟, d) Explique si en este proceso se conserva la energía.
⎝ A⎠ ⎝ bdr ⎠
La energía total disipada en el disco es

P=∫
ε2
=∫
a (π r ωB
2
0 )
cos ω t bdr
2

R 0 2π rη
1
= π ba 4ω 2 B02 cos 2 ω t

La disipación de energía depende de la cuarta
potencia del radio del disco, y esta dependencia se
encuentra para otras formas también, así que los
núcleos de los transformadores con hojas laminadas
finas de metal ayudan a reducir las pérdidas Solución.
perceptiblemente. El promedio de cos
2
ωt en un →
a) Φ B = B⋅ nˆ A
período es 1/2, tal que:
El flujo magnético inicial Φ B = Bkˆ ⋅ kˆl = Bl
2 2
1
Pm = π ba 4ω 2 B02
16η Si consideramos que el movimiento se inicia en la
posición mostrada en el dibujo (θ = 0) , para
GENERADOR DE CORRIENTE ALTERNA θ = π 2 , el flujo es cero, para θ = π , el flujo es
La aplicación principal de la Ley da Faraday es el
máximo nuevamente pero en sentido contrario, como
generador de corriente alterna.
la fuerza electromotriz inducida es alterna, la
Un generador de corriente alterna básicamente
corriente también es alterna. Como al inicio el flujo
consiste una bobina cuadrada con n vueltas que gira
es máximo y va disminuyendo, la fuerza
con cierta frecuencia f (velocidad angular
electromotriz es en oposición a ese cambio, la
ω = 2π f ) en un campo magnético B uniforme, tal corriente inicial será en el sentido indicado en el
como se muestra en la figura. dibujo.

De acuerdo a la Ley de Faraday al venir el flujo


magnético se induce una diferencia de potencial en
ε →
los terminales de la bobina. El potencial que se b) I = , Φ B = B⋅ nˆ A = Bl cos ωt
2
obtiene es una función senoidal, habrá un voltaje R

máximo cuando B y la sección A = ab de la dΦ B d
Luego ε =− = − Bl 2 cos ωt =
bobina (el vector n̂ ) están en la misma dirección y dt dt
mínima cuando están en direcciones opuestas. Bl 2ω senωt

16
Ley de Faraday e inductancia Hugo Medina Guzmán

Bl 2ω
Finalmente I = senωt
R
c) La figura muestra la vista lateral y la vista frontal
sobre el plano xy.

El campo magnético n un punto P en el interior espira


1 puede calcularse con la ley de Biot y Savart y es la
suma de los campos producidos por la espira te 1
(corriente I 1 ) y por la espira 2 (corriente I 2 ). Por lo
tanto podernos escribir que el flujo a través de la
→ →
espira 1 es proporcional a I 1 y a I 2 , es decir, es
Actúan las fuerzas F 3 y F 4 , tal que
F3 = F4 = IlB . función de I 1 y de I 2 .
Estas fuerzas son iguales en magnitud y opuestas en Φ 1 = Φ 1 (I 1, I 2 )
sentido, formando así un par de fuerzas de valor La fuerza electromotriz inducida de la espira 1 es:
τ = F3 lsenθ = IlBlsenθ dΦ 1
= − Φ 1 (I 1 , I 2 ) =
d
ε1 = −
conl = A (área de lo espira) podemos escribir
2
dt dt
τ = IABsenθ dΦ1 dI1 dΦ1 dI 2 dI dI
Este par puede escribirse como producto vectorial de − − = − L1 1 − M 21 2

dI1 dt dI 2 dt dt dt
n̂ nˆ
(normal a la superficie A) y B .
→ → Definimos
τ = IAnˆ × B .
d) No se conserva la energía, se disipa en forma de AUTOINDUCTANCIA de la espira 1
calor por la resistencia de la espira debido al efecto dΦ 1
Joule. L1 = −
dI 1
INDUCTANCIA INDUCTANCIA MUTUA
Hemos estudiado dos elementos constituyentes de un (efecto de la espira 2 sobre 1)
circuito, resistencia y condensador. Mientras la dΦ 21
resistencia solo disipa energía, los condensadores y la M 21 = −
capacidad están relacionados con los campos dI 2
eléctricos, similarmente tenemos para con los campos L1 solo depende de la geometría del circuito 1
magnéticos un elemento que almacena la energía
magnética, este elemento es la inductancia. mientras que M 21 depende de la disposición de
ambos circuitos.
Es fácil demostrar que
M 21 = M 12
Asimismo, si en lugar de tener 2 circuitos tenemos n
circuitos la inductancia mutua de cualquiera de ellos
será
dΦ ij
M ij = − i≠ j
dI j
La unidad de inductancia es el HENRY (H) en honor
de Joseph Henry, científico norteamericano que
DEFINICIÓN DE INDUCTANCIA descubrió la ley de inducción electromagnética en la
Consideremos que tenemos dos circuitos como los misma época que Faraday.
mostrados en la figura y que en las cercanías no hay Comumente se usan los submúltiplos
ninguna fuente de campo magnético y todo el flujo milihenry (mH) 10-3H
magnético será debido a las dos espiras. microhenry μ H = 10-6H

Aplicación. El toroide
Autoinductancia de un toroide.

17
Ley de Faraday e inductancia Hugo Medina Guzmán

Observando los valores hallados, encontramos


M 12 = M 21 = L1 L2
En general
M = k L1 L2
donde k es el coeficiente de acople
Consideremos un toroide (bobina larga a la que se da
forma circular) mostrado en la figura siguiente, con k ≤1
n1 vueltas y corriente I1,
Ejemplo 30. Determine la autoinductancia de un
toroide con n vueltas de sección transversal
cuadrada a x a y radio interno R.
Solución.
De la ley de Ampere 2π rB = μ 0 nI :
⎛ μ 0 nI ⎞
R+a
Φ B = n ∫ BdS = n ∫ ⎜ ⎟(adr )
R
⎝ 2π r ⎠
μ n 2 Ia ⎡ 1 1 ⎤
= 0 ⎢ 2 −
El campo magnético en su interior se encuentra
aplicando la ley de Ampere
2π ⎣ R (R + a )2 ⎥⎦
→ → Φ B μ0 n 2 a ⎡ 1 1 ⎤
∫ B ⋅ d l = μ 0 I ⇒ B1 2π a = μ 0 n1 I1 ⇒ L=
I
=
2π ⎣ R
⎢ 2 −
(R + a )2 ⎥⎦
μ 0 n1 I 1
B1 =
2π a Ejemplo 31. Determine la autoinductancia por
El flujo magnético a través de A debido a las n1 unidad de longitud de un cable coaxial recto largo de
radio interno a y de radio externo b.
vueltas
Solución.
⎛ μ n I A⎞ μ n 2 AI De la ley de Ampere 2π rB = μ 0 I .
Φ 1 = (B1 A)n1 = ⎜ 0 1 1 ⎟n1 = 0 1 1
⎝ 2π a ⎠ 2π a Para la unidad de longitud.
La autoinductancia Φ B 1 b ⎛ μ0 I ⎞ μ0 ⎛ b ⎞
dΦ 1 μ n A 2 L= = ∫ ⎜ ⎟= ln⎜ ⎟
L1 = = 0 1 I I a ⎝ 2π r ⎠ 2π ⎝ a ⎠
dI 1 2π a
La inductancia mutua. En el mismo ejemplo Ejemplo 32. ¿Cuál es la autoinductancia de un
consideremos que tenemos un segundo arrollamiento solenoide largo del radio a con N vueltas por metro?
toroidal añadido al anterior, esta vez n 2 vueltas y Solución.
con corriente I2. El campo dentro de un solenoide es μ 0 NI , así que
El flujo el flujo con una vuelta es
⎛μ n I ⎞ π a 2 B = π a 2 μ 0 NI .
Φ 21 = (B2 A)n1 = ⎜ 0 2 2 A ⎟n1 =
⎝ 2π a ⎠ En 1 m hay N vueltas, así que el flujo con estas N
vueltas es
μ 0 n1 n2 A
I2 (
Φ B = N π a 2 μ 0 NI y )
2π a
ΦB
La inductancia mutua L= = π μ0 N 2a 2
dΦ 21 μ 0 n1 n 2 A I
M 21 = =
dI 2 2π a Ejemplo 33. Encontrar la inductancia de un solenoide
De igual manera podemos encontrar la de gran longitud l , n vueltas muy juntas y sección
autoinductancia del segundo arrollamiento A.
dΦ 2 μ 0 n22 A Solución.
L2 = = Aplicando la ley de ampere encontramos el campo
dI 2 2π a magnético en el interior del solenoide.
Y la inductancia mutua μ 0 nI
B=
dΦ 12 μ 0 n1 n 2 A l
M 12 = = Cuando nos piden encontrar la inductancia en
dI 1 2π a realidad están pidiendo la autoinductancia.
18
Ley de Faraday e inductancia Hugo Medina Guzmán

El flujo magnético es b) Calcúlese la fuerza electromotriz inducida en la


μ 0 nIA bobina B del ejercicio anterior si corriente que circula
Φ B = BA = por A se extingue en 0,1 segundos.
l
La inductancia
dΦ B μ 0 n 2 A
L=n =
dt l
Ejemplo 34. Dos bobinas circulares de los radios a y
b (a >> b) concéntricas y coplanares. La bobina más Solución.
grande tiene n1 vueltas, y la más pequeña tiene n 2
vueltas.¿Cuál es su inductancia mutua? a) L A = =
(
n A Φ A (1000) 3 × 10 −3 )
= 0,60 H.
Solución. IA 5
El campo magnético en el centro de la bobina grande
M AB = =
(
n B Φ B (2000) 1,3 × 10 −3 )
μ 0 n1 I 1 = 0,52 H.
es B1 = . IA 5
2a
ΔI A 5−0
Así el flujo a través de la bobina pequeña es b) ε B = M AB = 0,52 = 26,0 V.
aproximadamente Φ B 21 = n 2πb B1 ,
2 Δt 0,1
o también
Φ B 21 ⎛ b2 ⎞
tal que M = = μ 0 n1 n2π ⎜⎜ ⎟⎟ ΔΦ B 1,3 × 10 −3
I ⎝ 2a ⎠ ε B = nB = 2000 = 26,0 V.
Δt 0,1
Ejemplo 35. Determínense las dimensiones del
Ejemplo 37. Una corriente continua de 5 A en una
cociente L/R. ¿Cuánto vale 1 henrio/ohmio?
bobina de 1000 vueltas da lugar a un flujo de 10-3 Wb
Solución.
en el interior de la misma. Calcúlese:
Recordando las expresiones
a) La fuerza electromotriz de autoinducción inducida
dI
ε = −L y V = IR en la bobina si la corriente se interrumpe en 0,1s.
dt b) la autoinducción de la bobina.
Resulta Solución.

1V = 1H
1A
⇒H=
(1V )(1s ) a) ε =n
ΔΦ ⎛ 10 −3 − 0 ⎞
= 1000⎜⎜ ⎟⎟ = 10 V
1s 1A Δt ⎝ 0,1 ⎠
1V = (1A )(1Ω ) ⇒ (1Ω ) =
(1V ) ΔI
(1A ) b) ε =L
Δt
Por tanto, (5 − 0) = 50 L ⇒ L = 10 = 0,20 H.
(1 V )(1 s ) 10 = L
0,1 50
1H 1A
= = 1s nΦ 1000 × 10 −3
Ω 1V o también L = = = 0,20 H.
1A I 5
L
Dimensiones = tiempo Ejemplo 38. Una bobina de autoinducción L = 1
R dI
henrio se conecta a una batería de 24 V. Calcule :
1H dt
= 1s
Ω a) en el instante de conectar la bobina a la batería;
Por esta razón el cociente L/R suele llamarse b) en el momento en que la corriente alcanza el 90
constante de tiempo. por 100 del valor correspondiente a la ley de Ohm.
Solución.
Ejemplo 36. Dos bobinas A y B situadas una junto a a) En el instante inicial, I = 0 y, por tanto, IR = 0
la otra poseen 1000 y 2000 espiras respectivamente. dI dI V 24
Cuando por A circula una corriente de 3 A se produce V −L = IR = 0 ⇒ = = = 24 A/s.
d dt L 1
en a un flujo de 3 x 10-3 Wb, y en B un flujo de 1,3 x
b) Valor máximo de la corriente (ley de Ohm):
10-3 Wb.
a) Calcule la autoinducción de A y la inductancia
mutua de las bobinas A y B.

19
Ley de Faraday e inductancia Hugo Medina Guzmán

V 24 dI
I max = = ,V −L = IR ,
R R d
dI ⎛ 24 ⎞
24 − 1 = ⎜ 0,90 ⎟ R = 21,6
d ⎝ R⎠
dI
= 24 − 21,6 = 2,4 A/s.
dt
La figura muestra una sección de la línea. El campo
magnético en la sección diferencial es
→ → →
Ejemplo 39. Un cable coaxial está formado por un
conductor cilíndrico interno de radio a, y un cascarón B = B1 + B 2
cilíndrico exterior (coaxial al cilindro interno) de → μ I μ0 I
B= 0 +
radio b. Demuestre que su auto-inductancia por
2π r 2π (d − r )
μ0 ⎛ b ⎞
unidad de longitud es igual a L = ln⎜ ⎟ . El flujo magnético por unidad de longitud a través del
2π ⎝ a ⎠ área diferencial es
Solución. μ0 I ⎡1 1 ⎤
dΦ B = Bdr = ⎢⎣ r + d − r ⎥⎦ dr

El flujo por unidad de Longitud es
d −a μ0 I ⎡1 1 ⎤
Φ B = ∫ dΦ B = ∫ + dr
2π ⎣ r d − r ⎥⎦
⎢ =
a

μ0 I
[ln r − ln(d − r )]da −a =

μ 0 I ⎡ (d − a ) d − (d − a ) ⎤
⎢ − ln
(d − a ) ⎥⎦
ln =
2π ⎣ a
μ 0 I ⎡ (d − a ) a ⎤
⎢ − ln
(d − a )⎥⎦
El campo magnético de un conductor cilíndrico está ln =
dado por: 2π ⎣ a
Para r < a B=0 μ 0 I (d − a )
ln
μ0 I π a
Para a<r <b B=
2πr Como d >> a
Para r > b B=0 μ0 I d
ΦB = ln
El flujo magnético a través de la sección diferencial π a
dS = ldr es: La inductancia por unidad de longitud es
⎛μ I ⎞ μ Il dr dΦ B μ 0 I d
dΦ B = BdS = ⎜ 0 ⎟(ldr ) = 0 L= = ln
⎝ 2πr ⎠ 2π r dt π a
Integrando:
μ Il b dr μ 0 Il ⎛ b ⎞ Ejemplo 41. Encontrar la inductancia mutua entre un
Φ B = ∫ dΦ B 0 ∫ = ln⎜ ⎟ alambre delgado finito y una espira cuadrada.
2π a r 2π ⎝a⎠
La autoinductancia:
dΦ B μ l ⎛b⎞
L= == 0 ln⎜ ⎟
dI 2π ⎝ a ⎠
La auto-inductancia por unidad de longitud es igual a
μ0 ⎛ b ⎞
L= ln⎜ ⎟ .
2π ⎝ a ⎠

Ejemplo 40. Determinar la autoinductancia por Solución.


unidad de longitud de una línea de dos alambres Vamos a calcular la inductancia mutua entre 1 (el
paralelos. El radio de los alambres es a y la alambre) y 2 (la espira).
separación entre ellos d (d >> a). Tomemos un elemento diferencial tal como el
Solución. mostrado en la figura.

20
Ley de Faraday e inductancia Hugo Medina Guzmán

El flujo a través del área diferencial es μ 0 Ia 2


μ0 I 2 Bz ( z =d ) =
dΦ 21 = B2 dA1 =
2π x
cdx (
2 a2 + d 2 )
32

El flujo total es Luego el flujo enlazado por la espira de


μ cI a +b dx
Φ 21 = ∫ dΦ 21 = 0 2 ∫
radio b, debido a la otra espira es
=
2π a x
μ 0 cI 2 μ cI a+b μ 0πa 2 b 2 I
a +b Φ ab =
( )
ln x a = 0 2 ln 32
.
2π 2π a 2 a2 + d 2
La inductancia mutua es
dΦ 21 μ 0 c a + b Con esto, el coeficiente de inducción mutua
M 21 = = ln es
dI 2π a Φ ab μ 0πa 2 b 2
Como M 21 = M 12 = M M = =
La inductancia mutua M es
I (
2 a2 + d 2
32
)
μ c a+b
M = 0 ln INDUCTANCIAS EN SERIE Y EN PARALELO
2π a En los esquemas de circuitos 1as inductancias
aparecen bajo e! símbolo indicado en las figuras
Ejemplo 42. Una bobina de 10 vueltas se envuelve
siguientes.
apretadamente alrededor de un solenoide largo del 2
cm de radio y 200 vueltas por metro. ¿Cuál es la
inductancia mutua del sistema?
Solución.
Inductancias en serie
Φ B nμ 0 NI 1π r 2 Consideremos dos elementos en serie con
M = = autoinductancias L1 y L2 respectivamente e
I1 I1
( )(200)π (0,02)
inductancia mutua M.
= (10 ) 4π × 10
−7 2

= 3,2 x 10-6 H.

Ejemplo 43. Encontrar la inductancia mutua de dos


espiras, de radios a y b, dispuestas de manera que sus
centros están en el mismo eje (eje z), sus planos son
perpendiculares al eje z, y sus centros están a una
distancia d. Si una de las espiras es muy pequeña, d
>> a.

Solución.
Con la corriente I común
dI dI
ε 1 = − L1 −M
dt dt
dI dI
ε 2 = − L2 − M
dt dt
y ε = ε1 + ε 2

ε = −(L1 + L2 + 2 M )
dI
Tenemos:
El campo magnético, en el eje de la espira dt
mayor (de radio a) es El valor efectivo de la inductancia es
μ 0 Ia 2 Lef = L1 + L2 + 2M
Bz =
(
2 a2 + z2 )
32
En el caso que las inductancias estuvieran en
oposición, como se muestra en la figura a
Como la espira menor es muy pequeña, el continuación, las inductancias mutuas hacen un
campo en cualquier punto de ella debe ser efecto en sentido contrario.
constante, de valor

21
Ley de Faraday e inductancia Hugo Medina Guzmán

dI 1 dI
(1) × M → εM = − ML1 −M2 2
dt dt
dI 2 dI
(2) × L1 → εL1 = − L1 L2 − ML1 1
dt dt
Restando estas dos últimas obtenemos:

ε (L1 − M ) = −(L1 L2 − M 2 )
dI 2
(2) a
dt
Sumando (1)a y (2)a

ε (L1 L2 − 2M ) = −(L1 L2 − M 2 ) (I 1 + I 2 )
d
dI dI
ε 1 = − L1 + M dt
dt dt
Con I = I 1 + I 2
dI dI
ε 2 = − L2 + M
dt dt ε =−
(L L
1 − M 2 ) dI
2

y ε = ε1 + ε 2 (L1 + L2 − 2M ) dt
La inductancia efectiva es
ε = −(L1 + L2 − 2M ) (L L −M 2)
dI
Tenemos:
dt Lef = 1 2

El valor efectivo de la inductancia es (L1 + L2 − 2M )


Lef = L1 + L2 − 2M Si están alejados de tal manera que la inductancia
Si las inductancias están muy alejadas, de tal manera mutua sea despreciable
que la inductancia mutua sea despreciable L1 L2 1 1 1
Lef = o = +
Lef = L1 + L2 L1 + L2 Lef L1 L2
Inductancias en paralelo Es decir, la inductancia, inversa equivalente es igual a
Conectemos ahora las inductancias en paralelo la suma de las inversas de las inductancias
componentes en paralelo.

ENERGÍA ALMACENADA POR UNA


INDUCTANCIA
Cuando conectamos una bobina a una fuerza
electromotriz. ε 0 (por ejemplo una batería) se
produce un flujo magnético cambiante en aumento
hasta que se estabiliza la corriente. Durante ese
intervalo se induce una fuerza electromotriz ε .
Tenemos que
dI
dI dI ε = −L
ε = − L1 1 − M 2 (1) dt
dt dt Para producir esta fuerza electromotriz tiene que
dI dI realizarse un trabajo, que es desarrollado por la fuente
ε = − L2 2 − M 1 (2) que produce la corriente.
dt dt La potencia instantánea que nos proporciona la fuente
Tenemos que obtener dos ecuaciones una dependiente es
de I 1 solamente y otra dependiente de I 2 solamente. dW
Para eliminar I 2 .
Potencia = =ε I
dt
dI 1 dI ε
(1) × L2
Sustituyendo la magnitud de
→ εL2 = − L1 L2 − ML2 2 dW dI
dt dt = IL
dI 2 dI dt dt
(2) × M → εM = − ML2 −M2 1 Como los diferenciales de tiempo son iguales
dt dt
Restando estas dos últimas obtenemos:
dW = ILdI = LIdI
El trabajo realizado en llevar la corriente de I = 0 a
ε (L2 − M ) = −(L1 L2 − M 2 )
dI 1
(1) a su valor constante estacionario I
dt I 1 2
Para eliminar I 1 . W = ∫ dW = ∫ LIdI = LI
0 2

22
Ley de Faraday e inductancia Hugo Medina Guzmán

( )
Este trabajo es realizado por la batería y equivale a la 1 2 1
energía almacenada en forma de campo magnético UB = LI = πμ 0 N 2 a 2 I 2
por la bobina, esto es 2 2
Energía magnética, U B . El volumen por unidad de longitud es π a 2 , así que
la densidad de energía magnética es
1 2
UB = LI 1
2 uB = μ 0 N 2 I 2 (en J/m3).
Esta energía almacenada se puede recuperar como lo 2
veremos en la sección siguiente, esta ecuación vale En términos del campo magnético B = μ 0 NI , esto
para cualquier inductancia y es similar a la de la puede ser escrito:
energía eléctrica almacenada en un condensador que
1 B2
tiene una diferencia de potencial V. uB =
1 2 μ0
UE = CV 2
2
De igual manera a como hicimos para los Ejemplo 45. ¿Cuánta energía se almacena en un
condensadores podemos para la inductancia solenoide de longitud 10 cm y diámetro 1,2 cm si
encontrar una expresión de la energía almacenada en tiene 200 vueltas y lleva 1,2 A?
función del campo magnético. Consideremos una Solución.
Con las aproximaciones de un solenoide muy largo,
bobina larga de longitud l , sección A y N vueltas
por unidad de longitud (n = Nl ) , su inductancia es
tenemos L = μ 0πN 2 a 2 .
1
μ0 N 2 A UB = μ 0πN 2 a 2 I 2
L= = μ 0 n 2 lA 2
l 2
La energía almacenada
1 1
(
= (0,5) 4π × 10
−7
)(π )⎛⎜ 0200 ⎞
⎟ (0,006 ) (1,2 )
2 2

U B = LI 2 = μ 0 n 2 lAI 2 ⎝ ,10 ⎠
-4
2 2 = 4,1 x 10 J
Su volumen Vol = Al
Ejemplo 46. Determinar la energía almacenada por
Considerando que la energía almacenada esta un toroide de n vueltas, longitud media l , sección
distribuida uniformemente, definimos transversal A, la corriente que circula es I .
Densidad de energía u B : Solución.
La inductancia del toroide es
1
μ 0 n 2 lAI 2 μ0 n 2 A
uB =
UB
= 2 1
= μ0 n 2 I 2 L=
Vol Al 2 l
Recordando que para una bobina La energía almacenada es
B = μ 0 nI 1 2 1 ⎛ μ0n 2 A ⎞ 2
UB = LI = ⎜⎜ ⎟I
Reemplazando 2 2 ⎝ l ⎟⎠
1 B2
uB =
2 μ0 Ejemplo 47. Se propone el uso de grandes inductores
como dispositivos para almacenar energía. ¿Cuál es la
Esta expresión es aplicable para cualquier caso, inductancia que necesitamos para que al circular una
similar al caso de los condensadores en que corriente de 60 A, la energía sea la suficiente para
1 encender un foco de 100 vatios por una hora?
uE = ε0E2 Solución.
2
Ejemplo 44. ¿Determine la energía almacenada por U = PΔt = (100 )(3600 ) = 3,6x105 J
unidad de longitud y la energía almacenada por
unidad de volumen en un solenoide muy largo del
1 2 2U
U = LI ef ⇒ L = 2 =
2 3,6 x10 5
=2H
( )
radio a con N vueltas por metro y que lleva una 2 I 60 2
corriente I?
Solución. Ejemplo 48. Calcular para todo punto la densidad de
Tenemos que la inductancia de un solenoide muy energía magnética para un conductor largo, cilíndrico
largo de radio a con N vueltas por metro es de radio b con una cavidad cilíndrica concéntrica de
L = π μ0 N 2 a 2 . radio a y lleva una corriente I.
Solución.
La energía es:

23
Ley de Faraday e inductancia Hugo Medina Guzmán

Para r < a Solución.


No hay corriente encerrada, no hay campo magnético El campo magnético del alambre es
por consiguiente no hay energía almacenada. μ0 I
B=
Para a < r < b 2π r
Aplicando la ley de Ampere El flujo en el lazo es
→ → →
2a ⎛ μ I ⎞ μ Ia
∫ B ⋅ d l = μ ∫ J ⋅ nˆdS
0 Φ B = ∫ ⎜ 0 ⎟adr = 0 ln 2
I
a
⎝ 2π r ⎠ 2π
donde J =
(
π b − a22
) La fuerza electromotriz en el lazo es
dΦ B μ 0 a dI
μ0 I ε= = ln 2
B 2π r = π (r 2 − a 2 ) ⇒ dt 2π dt
π (b − a )
2 2
La potencia en la espira es:
μ I (r − a ) 1 ⎛ μ a ln 2 ⎞
ε2
2 2 2

B= 0 P= = ⎜ 0 ⎟ (I 0ω cos ωt )
2

2π r (b 2 − a 2 ) R R ⎝ 2π ⎠
La densidad de energía es Tenemos que:

uB =
1 B2 μ I 2 r2 − a
= 0
( )
2 2
(cos 2
ωt )m =
1 T
∫ cos 2 ωtdt ,
( ) T 0
2 μ 0 8π 2 r 2 b 2 − a 2 2
2π 1 − cos 2ωt
T= y cos ωt =
2
Para r > b
Aplicando la ley de Ampere
ω 2
→ →

∫B⋅d l = μ 0 I ⇒ B 2π r = μ 0 I ⇒ (
De aquí cos ωt m =
2
)1
2
μ0 I Finalmente:
B= μ 0 a ln 2(I 02ω 2 ) 1 ⎡ (μ 0 a ln 2)(I 0ω ) ⎤
2
2π r Pm = =
La densidad de energía es 8π 2 R 2 R ⎢⎣ 2π ⎥

1 B2 μ I2
uB = = 02 2
2 μ 0 8π r MATERIALES MAGNETICOS
Hasta esta parte hemos estudiado los campos
La energía almacenada podemos evaluarla por magnéticos producidos por distribuciones de
integración corrientes eléctricas específicas. Por ejemplo,
U = ∫ μ B dVol podemos calcular el campo magnético en el vacío
Vol producido por un alambre con la ayuda de la ley de
dVol = 2π rldr Biot y Savart, pero si rodeamos el alambre por un
b⎛ μ I ⎞ b μ I l dr medio material, el campo magnético se altera, ya que
2 2
U = ∫ ⎜ 2 2 ⎟2π rldr = ∫
⎜ 0
⎟ 0
la materia esté constituida por átomos, y cada átomo
a 8π r
⎝ ⎠
a 4π r consiste de electrones en movimiento, estos circuitos
de electrones, cada uno de los cuales confinado en un
μ I 2l b μ I 2l b átomo, son llamadas corrientes atómicas las cuales
= 0 ln r a = 0 ln
4π 4π a producen campos magnéticos que se suman al efecto
del campo magnético producido por la corriente
Ejemplo 49. Un alambre recto largo se coloca en el circulante por el alambre.
plano de una espira conductora cuadrada de lado a y
resistencia R. el alambre es paralelo al lado más EL ÁTOMO COMO UN IMÁN.
cercano del cuadrado y a una distancia a de él. ¿Cuál
es la energía media disipada en la espira cuando la
corriente en el alambre es I = I 0 senω ?

24
Ley de Faraday e inductancia Hugo Medina Guzmán

Hemos visto anteriormente que el campo magnético


en el eje de una espira con corriente I es Por lo tanto, en dispositivos tales como electroimanes
→ μ 0 Ir 2 o inductores, el hierro o algún otro material
B= kˆ “permeable” se inserta para realzar los efectos
(
2 z +r
2 2 32
) magnéticos. Una medida del grado de magnetización
Cuando z = 0 es el campo en el centro del anillo obtenido es la permeabilidad relativa μ r .
→ μ I En cálculos de los efectos magnéticos, por ejemplo,
B = 0 kˆ de la inductancia o el campo magnético resultante
2r generalmente es suficiente sustituir μ 0 en nuestras
Si ponemos en función del área A = π r
2
ecuaciones anteriores por μ = μr μ0 . μr , no tiene
→ μ 0 IA ˆ μ 0 IA ˆ
B= k= k dimensiones.
2r (π r 2 ) 2π r 3
El campo magnético es proporcional al producto IA Ejemplo 50. Sobre un núcleo de hierro de 1 m de
y con sentido perpendicular al área A. longitud y 2 cm de radio se arrollan dos solenoides.
Para pequeños círculos de corriente definimos Uno de ellos, que actúa de primario, posee 1000
→ espiras y por él circula una corriente de 5 A. ¿Qué
momento magnético μ = IAnˆ , tal que fuerza electromotriz se inducirá en el secundario, que
posee 5000 espiras, cuando la corriente del primario
→ μ0 →
B= μ se extingue en 1/10 de segundo? Para el hierro,
2π r 3 μ ' = 50 .
Solución.
EL FERROMAGNETISMO Y EL Flujo inicial
PARAMAGNETISMO. n1 I
En algunos materiales, tales como hierro, los átomos Φ = BA = μ ' μ 0 A
se comportan como pequeños lazos de corriente o l
− 7 (1000 )(5)
dipolos magnéticos, actúan como imanes de barra
pequeños, y obran recíprocamente de modo que se (
= (50 ) 4π × 10
1
) (
π 0,02 2 )
alinean espontáneamente y dan lugar algunas veces a
un material “magnetizado”. Tales sustancias son las = 3,94 x 10-4 Wb
ferromagnéticas, y se utilizan para hacer los imanes ΔΦ 3,94 × 10 −4
permanentes con los cuales estamos familiarizados. ε (sec undario ) = n2 = 5000
Si un elemento ferromagnético se calienta sobre
Δt 1 / 10
= 19,7 V.
cierta temperatura crítica (la temperatura de Curie), la
agitación termal hace que los pequeños imanes se
CIRCUITOS RL, LC y RLC
desordenen, y el material pierde su ferromagnetismo.
CIRCUITO RL.
Cuando los imanes no se alinean espontáneamente,
son los paramagnéticos. El paramagnetismo, así
como el ferromagnetismo, son de gran importancia
práctica. Por ejemplo, el campo en un solenoide de
base de aire debido a la corriente en las bobinas es
típicamente muy débil. Sin embargo, cuando un
solenoide se llena con hierro paramagnético, el
campo débil debido a la corriente es suficiente para
hacer que los dipolos del hierro se alineen, y La figura muestra una inductancia L, una resistencia
produzcan un campo magnético muy grande.
R que se conectan a una fuerza electromotriz ε 0 por
medio de una llave S de tres posiciones, la corriente
en el circuito inicialmente es cero, en el momento
t = 0 se pone la llave en posición 1.

25
Ley de Faraday e inductancia Hugo Medina Guzmán

d ε0 ⎛ − t ⎞
R R
− t
ε =L ⎜1 − e ⎟ = − ε 0 e L
L
dt R ⎜⎝ ⎟

Para t = 0 → ε = ε 0 , para t = ∞ → ε = 0 ,
La figura siguiente muestra los gráficos I − t y
ε −t.
Al ir aumentando la corriente existe una fuerza
electromotriz inducida ε en oposición, en la
resistencia R existe una caída de potencial IR.
Aplicando la segunda regla de Kirchoff, tenemos:
ε 0 + ε − IR = 0
dI
ε0 − L − IR = 0
dt
dI R ε Después de pasado un tiempo en el que consideramos
+ I− 0 =0 que la corriente ha alcanzado su valor estacionario
dt L L ε0
Resolviendo la ecuación para las condiciones I= pasamos la llave S a la posición 2.
iniciales, para t = 0 , y para un tiempo t genérico. R
dI R⎛ ε ⎞
= − ⎜I − 0 ⎟
dt L⎝ R⎠
dI R
= − dt
⎛ ε ⎞ L
⎜I − 0 ⎟
⎝ R⎠
Integrando En este circuito la inductancia actúa como fuente de
I dI R t voltaje hasta que se disipa en la resistencia la energía
∫o ⎛ ε ⎞
=−
L ∫0
dt que tenia almacenada.
⎜I − 0 ⎟ Aplicando la segunda ley de Kirchhoff al circuito.
⎝ R⎠
ε = − IR = 0 ⇒ − L
dI
− IR = 0 ⇒
ε0 ⎞
I dt
⎛ R t
= ln⎜ I − ⎟ =− t0 dI R
⎝ R ⎠0 L + I =0
dt L
ε0 ε0
I− Las condiciones son para t = 0 → I 0 = y
= ln L =−Rt R
ε L
− 0 para un t genérico tenemos una I genérica.
L I dI R t R t
∫ = − ∫ dt ⇒ ln I t ⇒
I
De aquí: I0
=−
I0 I L 0 L 0
ε0
I−
R
− t ε0 R
− t
I = I 0e =
R
− t L
e L
R =e L
ε0 R
− dI
R Como ε = − L
Finalmente dt
ε0 ⎛ R
− t ⎞
I = ⎜⎜1 − e L ⎟
⎟ d ⎛ − t⎞
R
− t
R

R⎝ ⎠ ε = − L ⎜⎜ e L ⎟⎟ = ε 0 e L
dt ⎝ ⎠
ε0
Para t = 0 → I = 0 , para t = ∞ → I = , Para t = 0 → ε = ε0 .
R
Para t =∞ → ε =0.
dI
Como ε = − L La figura siguiente muestra 1os gráficos I − t y
dt ε −t.

26
Ley de Faraday e inductancia Hugo Medina Guzmán

Ejemplo 51. En la figura suponga que el interruptor S


se cierra en t = 0. Determine:
a) La corriente que pasa por cada resistencia a t = 0. R2 y R3 están en paralelo, la resistencia equivalente
b) La corriente que pasa por cada resistencia para un 1 1 1 1 1 1
tiempo muy largo.
es: = + = + =
Req R2 R3 6 3 2
Luego de un tiempo muy largo, el interruptor S se
abre (suponga ahora este instante t = 0. Considere la ⇒ Req = 2Ω
corriente inicial I 0 como la corriente que pasaba por
L en la parte b).
c) Plantee y resuelva la ecuación diferencial, para
encontrar la ecuación de la corriente en la inductancia
L, como una función del tiempo.
d) Calcule el valor de L, sabiendo que el tiempo que
tarda la corriente en disminuir la mitad de su valor
inicial es 69 milisegundos.

ε 18
I1 = = =3A
R1 + Req 4+2
También tenemos: I 2 + I 3 = I 1 = 3 y
I 2 R3 3 1
I 2 R2 = I 3 R3 ⇒ = = =
I 3 R2 6 2
Luego I 2 = 1 A e I 3 = 2 A
Solución.
a) Para t = 0, La corriente por la inductancia L es cero Luego de un tiempo muy largo, el interruptor S se
y la corriente por R3 también es cero. Luego el abre (suponga ahora este instante t = 0). Considere la
circuito se reduce a: corriente inicial I 0 como la corriente que pasaba por
L en la parte b).

La corriente que pasa R1 y R2 es: c) Aplicando la segunda regla de Kirchhoff.


ε 18
IR2 + L
dI
+ IR3 = 0
I= = = 1,8 A
R1 + R2 4+6 dt
I 1 = I 2 = 1,8 A, I 3 = 0 dI (R + R3 ) 9
=− 2 I =− I
b) Para un tiempo muy largo la corriente en la dt L L
Inductancia tiene su valor máximo, el circuito es dI 9
equivalente al siguiente: ⇒ = − dt
I L
Integrando de t = 0 a t = t:

27
Ley de Faraday e inductancia Hugo Medina Guzmán

9 t I 9 d) Las ecuaciones de Kirchhoff que permitan


t 0 ⇒ ln = − ⇒
I
ln I I0
=− encontrar los valores de todas las corrientes del
L I0 Lt circuito.
9
− t
I = I 0e L

d) Para t = 0,069 s I = I 0 2
I0 − (0 , 069 )
9
9(0,069 )
= I 0e L ⇒ = ln 2
2 L
9(0,069) 9(0,069 )
⇒ L= = = 0,9 H Por la primera ley de Kirchhoff
ln 2 0,693
I = I1 + I 2 (1)
Ejemplo 52. Una bobina tiene un valor de auto- Por la segunda ley de Kirchhoff
inductancia L, y se conecta en paralelo a una
resistencia R. Los dos elementos se conectan en t = 0 dI1
a una fuente electromotriz “real” que esta formada
− Ir + V0 − L =0 (2)
dt
por una fuente ideal de potencial V0 constante; una
resistencia interna en serie r. Calcule los siguientes − Ir + V0 − I 2 R = 0 (3)
acápites, justificando físicamente los valores que e) La ecuación diferencial que permite obtener la
encuentre: ()
corriente instantánea I t en la resistencia interna r
a) El valor de la corriente en la fuente en t → 0 . de la fuente electromotriz.
b) El valor de la corriente en la fuente a un tiempo Derivando (3) y multiplicando por L:
muy largo. dI dI
c) Un esbozo del gráfico de la magnitud de la − rL − RL 2 = 0 (3a)
corriente en la resistencia R en función de t. dt dt
d) Las ecuaciones de Kirchhoff que permitan Multiplicando (2) por R:
encontrar los valores de todas las corrientes del dI1
circuito. − IRr + V0 R − RL = 0 (2a)
e) Plantee la ecuación diferencial que permita obtener
dt
()
Sumando (2a) y (3a):
la corriente instantánea I t en la resistencia interna
dI d (I1 + I 2 )
r de la fuente electromotriz. No resuelva la ecuación − IRr − rL + V0 R − RL =0
diferencial. dt dt
Solución. dI dI
⇒ − IRr − rL + V0 R − RL = 0
dt dt
⇒ − IRr + V0 R − (R + r )L = 0
dI
dt
⇒ (R + r )L + R (V0 − Ir ) = 0
dI
dt
Finalmente:
V0 dI R(V0 − Ir )
a)Para t = 0: I = + =0
R+r dt L(R + r )
V
b) Para t = ∞ : I = 0 Ejemplo 53. En el circuito mostrado, se conecta el
r interruptor S y después de un tiempo muy largo se
c) Esbozo del gráfico de la magnitud de la corriente
desconecta. Ese instante lo consideramos como el
en la resistencia R en función de t.
estado inicial.

a) Establezca la ecuación diferencial para hallar la


corriente.
28
Ley de Faraday e inductancia Hugo Medina Guzmán

b) Resuelva la ecuación, teniendo en cuenta que una L


d 2 f (t ) Como τ= ⇒
función f(t) que cumple que = −ω 2 f (t ) , R
( )
2
dt L
= 2,46t = 2,46 50 × 10 −6 = 1,23 x 10-4
tiene solución f (t ) = A cos(ωt + φ ) . Halle A , ω R
yφ. Finalmente L = 1,23 x 10-4R = 1,23 x 10-4(180)=
Solución. 0,022 H
L 0,022
a)
q
+L
dI
=0 ⇒
1 d 2I
I+L 2 =0 c) τ= = = 0,122 ms
C dt C R 180
dt
2
d I 1 CIRCUITO LC.
⇒ 2
+ I =0
dt LC
b) I = I 0 cos(ωt + φ ) , ω =
1
,
LC
ε
Para t = 0 , I = I 0 = , ϕ =0
R
Ejemplo 54. Un solenoide tiene una inductancia de
30 H y una resistencia de 20 Ω . Si se conecta a una La figura arriba muestra un circuito LC, que vamos a
batería de 12 V, ¿cuánto tiempo tarda la corriente en considerar ideal (sin resistencias propias). Si
alcanzar la tercera parte de su valor de equilibrio? ponemos la llave S en la posición 1 se carga al
Solución. condensador hasta tener un voltaje igual a la fuerza
El valor de equilibrio I = V/R = 12/20 = 0,6 A se electromotriz y almacena una energía igual a
alcanza para t → ∞ .
1 Q2
La tercera parte de este valor se alcanza cuando se UE =
cumple: 2 C
1V V
(
= 1 − e −t τ
3R R
) Donde Q = V0 C
Si pasamos el interruptor a la posición 2, el
es decir, condensador se descarga a través de la inductancia, a
2 2 t medida que el condensador se descarga la energía
= e −t τ , ln = pasa a la inductancia, una vez que el condensador
3 3 τ queda descargado, la corriente alcanza su valor
L máximo, como no hay fuente que siga manteniendo
y como τ = resulta:
R esta corriente comienza a decrecer llevando carga al
condensador hasta que la corriente es cero y el
L 2 30 2
t = ln = ln = 0,61 s. condensador queda cargado, pero esta vez con
R 3 20 3 polaridad invertida, como el circuito es ideal no hay
resistencia, se re pite el proceso y así siguen
Ejemplo 55. Un instrumento sensible de detección en intercambiando energía de tal manera que la energía
un vehículo espacial tiene una resistencia de 180 Ω total del circuito es constante.
y se diseña para funcionar con una corriente de 42 U B + U E = U = constante
mA. Sin embargo, es necesario que la subida de
1 2 1 q2
corriente sea no más del 10 por ciento de este valor o LI + =U
de funcionamiento dentro de los primeros 50 ms 2 2 C
después de aplicar el voltaje. Para alcanzar esto, se Derivando respecto al tiempo
conecta una inductancia en serie con el dispositivo. dI q dq
a) ¿Qué voltaje se requiere? LI + =0
b) ¿Cuál es la mínima inductancia requerida? dt C dt
c) ¿Cuál es la constante de tiempo del circuito? dq dI d 2 q
Solución. Como I = y =
a) ε = IR (
= 42 × 10
−3
)(180) = 7,56 V Tenemos
dt dt dt 2

(
b) I = I 0 1 − e
−t τ
) = 0,1I ⇒ e −t τ
= 0,9 d 2q q d 2q 1
+ = ⇒ + q=0
0
L 0
⎛ t ⎞ dt 2
C dt 2
LC
⇒ ln⎜ − ⎟ = ln 0,9 ⇒ τ = 2,46t
⎝ τ
La solución de esta ecuación es el caso de
⎠ movimiento armónico simple.

29
Ley de Faraday e inductancia Hugo Medina Guzmán

q = A cos(ωt + δ )
1
Donde ω= , A es el valor máximo de q y
LC
depende de las condiciones iniciales, en nuestro caso
A = Q y debe ser cero porque la carga es máxima
para t = 0, de aquí:
q = Q cos(ωt + δ )
y la corriente es Solución.
dq 1 1 1 1
I= = −ωQsenω t f =
(2,6 × 10 )(8 × 10 )
= =
dt 2π LC 2π −3 −2
La figura a continuación muestra los gráficos de
q−t e I −t. 1,1 MHz.

Ejemplo 57. Un receptor de radio de amplitud


modulada (AM) utiliza un circuito resonante LC
cuya frecuencia empareja la frecuencia de las ondas
de radio entrantes. ¿Si una inductancia de 16 μH se
utiliza, qué rango de capacidad variable se requiere
para sintonizar sobre la banda AM a partir de 500 a
1700 kHz?
Solución.
1 1
f = ⇒ C=
2π LC L(2π f )
2

Con 500 kHz → 6,3 x 10-9 F


Con 1700 kHz → 5,5 x 10-10 F
Podemos ver es un circuito oscilante, con frecuencias. De tal modo que C varía de 5,5 x 10-10 a 6,3 x 10-9 F.
ω 1 1
f = = periodo
2π 2π LC CIRCUITO RLC.


T= = 2π LC
ω
Con los valores de Q e I podemos demostrar que la
energía total es constante para cualquier tiempo t.
UB + UE = U
1 2 1 q2
LI + =U
2 2 C
La figura arriba muestra un circuito RLC, en la
1 1 Q2
Lω 2 Q 2 sen 2ω t + cos 2ω t = U posición 1 el condensador se carga. Al pasar a la
2 2 C posición 2, la energía almacenada en el condensador
1 se descarga, parte se disipa en la resistencia y parte se
Como ω =
2
almacena en la inductancia, entonces la inductancia
LC repite el proceso, parte se disipa en la resistencia y

( )
1Q 2
1 Q2 parte se almacena en el condensador y así
sen 2 ω t + cos 2ω t = =U periódicamente hasta que la energía que tenia
2 C 2 C inicialmente el condensador se disipa totalmente en la
Tal como esperábamos. resistencia.
Aplicando la segunda Ley de Kirchoff
Ejemplo 56. ¿Cuál es la frecuencia resonante del q dI
circuito de la figura si L = 260 mH y C = 8,0 pF? + L + IR = 0
C dt
dq dI d 2 q
Como I = y =
dt dt dt 2
Nos queda la ecuación
d 2 q R dq 1
2
+ + q=0
dt L dt LC
30
Ley de Faraday e inductancia Hugo Medina Guzmán

Ecuación que corresponde a la de las oscilaciones La expresión de la carga del condensador vimos que
amortiguadas, cuya solución es es:

q = e − β t ⎛⎜ Be
β 2 −ω02 t
+ De
− β 2 −ω02 t ⎞⎟ (
q = εC 1 − e − t RC )
⎝ ⎠ Cuando pasa un tiempo largo t
1R Q = εC
Donde β =
2L Cuando pasa a la posición 2 tenemos un circuito LC
con el condensador con carga inicial
ω0 =
1
( )
Q = (10 ) 4 × 10 −6 = 40 x 10-6 C.
LC a) La frecuencia de oscilación del circuito LC
B y D dependen de las condiciones iniciales. depende solamente de los valores de L y C.
La forma de la solución depende de los valores de R,
1 1
β 2 − ω 02 ω=
L y C, los cuales determinan si
igual o mayor que cero siendo para cada caso una
es menor,
LC
=
(2 × 10 )(4 × 10 ) =
−3 −6

oscilación subamortiguada, críticamente amortiguada 10


o sobreamortiguada respectivamente. × 10 8 = 1,12 x 104 rad/s.
En el caso de oscilación subamortiguada, 8
β 2 − ω 02 < 0 . ω
f = = 1,78 x 103 Hz.

β 2 − ω 02 = iω1 b) La energía almacenada en el condensador es
cos(ω1t + δ ) 1 Q 2 1 (40 × 10 −6 )
− βt − βt
q = Ae q = Ae cos ω1t
2

UE = = = 2 x10-4 J.
La figura a continuación muestra 2 C 2 4 × 10 −6
el grafico q − t para el caso en que δ = 0.
Como la energía pasa del condensador a la
inductancia y así sucesivamente, la energía máxima
almacenada por la inductancia es
U B = 2 x10-4 J.

Ejemplo 59. Se ha sugerido que la energía sobrante


de una planta de generación se podría almacenar en el
campo magnético de un toroide gigante. ¿Si el
campo magnético en el toroide fuera de 12 T (muy
grande), qué volumen sería necesario para almacenar
106 kWh de energía? ¿Si el toroide fuera en forma de
anillo, con el radio interno R y el radio externo 2R,
Ejemplo 58. En la figura tenemos un circuito RC cuán grande tendría que ser R?
cuando pone la llave en la posición 1 y un circuito LC Solución.
en la posición 2. Después de tener un tempo largo en
⎛ B2 ⎞
la posición 1 se pasa la llave a la posición 2. U = ⎜⎜ ⎟⎟(volumen )
Calcular: ⎝ 2μ 0 ⎠
a) La frecuencia de oscilación del circuito LC.
⎛ B2 ⎞ 3 ⎛ πR 2 ⎞
⎟⎟(2π )⎛⎜ R ⎞⎟⎜⎜
b) La energía máxima almacenada por la inductancia.
≈ ⎜⎜ ⎟⎟
⎝ 0⎠
2 μ ⎝ 2 ⎠ ⎝ 4 ⎠
3
U ≈ π 2R2B2
8
⎡ 8(10 6 × 3,6 × 10 6 )⎤
13 13
⎛ 8U ⎞
R=⎜ 2 2 ⎟ ≈⎢ ⎥
⎝ 3π B ⎠ 3π 2 (12 )
2
⎣ ⎦
= 1890 m ¡demasiado grande!
Solución.
Al estar en posición 1 la corriente circula hasta que se
carga el condensador completamente.

31
Ley de Faraday e inductancia Hugo Medina Guzmán

PREGUNTAS Y PROBLEMAS

1. Si una aspira rectangular conductora de lados a y b rota alrededor del eje vertical MN perpendicular a la
→ →
se aleja con una velocidad constante v de un velocidad angular ω . En al instante t = 0, es normal
alambre largo recto con una corriente I. al plano del conductor. Calcular en función del
Determinar la corriente inducida en la espira como tiempo, el flujo del campo magnético a. través del
función del tiempo, pera t = 0 , r = r0 . cuadro. Deducir le fuerza electromotriz de inducción
ε.

2. Una barra metálica de longitud a e inclinación α ,


separada b de un alambre recto y largo con corriente
6.- Una bobine de 100 vueltas y sección circular esté
I. La barra se desplaza una velocidad constante
arrollada compactamente de tal modo que las espiras
paralela al alambre. Calcular la fuerza electromotriz
están en un mismo plano. El radio promedio de la
inducida en la barra.
bobine es 3 cm. La bobina rota alrededor de un
diámetro a 900 rpm. Cuando el eje de rotación está
vertical, la fuerza electromotriz promedio inducida es
0,50 mV. ¿Qué se puede decir del campo magnético
terrestre en ese lugar?

7.- Un disco rota alrededor de su eje con velocidad


angular ω . El disco está hecho con metal de
conductividad g, y su espesor es t. El disco se coloca
entre los polos de un imán que produce un campo
3. Dos espiras circulares metálicas de radios a y b magnético uniforme B sobre una pequeña área
2
están conectadas por dos barras metálicas, como se cuadrada de tamaño a a la distancia promedio r
muestra en la figura. Si este conjunto rota con del eje, B es perpendicular al disco. Calcular el torque
→ aproximado sobre el disco.
velocidad angular ω perpendicular a B . ¿Cuál es la
fuerza electromotriz inducida entre las espiras? 8. Sea el circuito en serie constituido por una fuerza
electromotriz ε , un interruptor S, un condensador de
capacidad C, un sistema consistente en una barra que
desliza sin fricción sobre dos rieles horizontales y
paralelos separados una distancia l en presencia de

un campo magnético B uniforme y vertical. La
resistencia total del circuito es R.
La masa de la barra móvil es m.
4. Un circuito plano de resistencia R, compuesto de n
espiras, con una superficie A, se coloca perpendicular
a un campo magnético alterno uniforme tal que
B = B0 cos ωt .
Encontrar la expresión de la intensidad eficaz de En t = 0 se cierra el interruptor, inicialmente la
corriente I ef inducida en el circuito. barra está en reposo y el condensador descargado.
Escribir las expresiones de la velocidad y la carga del
5. Un conductor rectangular de lados a y b se coloca condensador.

en un campo magnético B uniforme y horizontal,

32
Ley de Faraday e inductancia Hugo Medina Guzmán

9. Una rueda de Barlow de radio a rota sin fricción en Determine la magnitud y dirección de la corriente que
→ registra el amperímetro para t = 4 s si R = 10 Ω .
un campo magnético B uniforme y normal al plano
de la rueda. Sobre su eje horizontal se fija un cilindro
de radio b sobre el que se enrolla una cuerda que
soporta un peso mg. El conjunto, rueda, eje, cilindro
tiene un momento de inercia IrB. El circuito eléctrico
de la rueda de Barlow está conectado a una 13. Dos solenoides (S1) y (S2) indeformables y
resistencia total R. Estudiar el movimiento a partir del coaxiales, de secciones A1 y A2, (A2 < A1) tienen
reposo, no tomar en cuenta las corrientes de Foucault. respectivamente N1 y N2 espiras por unidad de
longitud, son recorridos por corrientes iguales y del
mismo sentido, I1 e I2 constantes. El solenoide S2
penetra una longitud x en el interior de S1 donde el
campo magnético se supone uniforme.
a) Si consideramos que el flujo que atraviesa las
espiras de S2 exterior a S1 se escribe Φ 0 = KI ,
donde K es una constante, calcular el flujo total que
atraviesa S2 debido a S1.
10. Un conductor rectilíneo AOB puede rotar b) ¿Cuál es la fuerza magnética actuante sobre el
alrededor de un eje vertical que pasa por O. Está en solenoide S2? ¿Cuál es la acción que origina esta
contacto con un conductor C circular horizontal de fuerza?
radio a. El conjunto está colocado en un campo

14. Un alambre de cobre de 1 m de longitud se mueve
magnético uniforme B , vertical dirigido hacia arriba. con velocidad de 75 cm/s perpendicularmente a un
Se hace pasar una corriente por medio de una fuerza campo magnético de 1 T. ¿Qué fuerza electromotriz
electromotriz ε . La resistencia de C es despreciable se induce en la barra? Si sus extremos se conectan a
una resistencia de 10 Ω . ¿Cuál es la potencia
y la del circuito es constante e igual a R.
Determinar la ley de la variación de la velocidad
disipada en el movimiento del alambre?
después de cerrar la llave S, despreciar la fricción.

11. Una rueda de Barlow de radio a rota sin fricción



en un campo magnético B normal el plano de la Respuesta.
rueda, el circuito de la rueda está en serie con un ε = Blv = 1 x 1 x 0,75 = 0,75 V,
condensador C inicialmente descargado, una llave S y ε2 0,75 2
la resistencia total del circuito es R. La rueda rota con P= = = 0,056 W.
una velocidad angular ω 0 . Se cierra la llave S,
R 10
estudiar el movimiento de la rueda a partir de este 15. La hélice de un aeroplano mide2,0 m de un
instante. extremo al otro extremo, gira a 18000 RPM. ¿Si el
El momento de inercia de la rueda es I rB . Despreciar aeroplano está volando dirigido al norte debido en un
las corrientes de Foucault. punto donde la componente horizontal del campo
magnético de la tierra es 1,2 x 10-5 T, qué voltaje se
genera entre las extremos de la hélice?
Respuesta.
Cada mitad del propulsor barre un área π r2 en cada
revolución, tal que ε = 2 Bπ r f2
.

ε = 2(1,2 × 10 −5 )π (1,0)2 ⎜
⎛ 18000 ⎞
⎟ = 0,0036 V
⎝ 2π 60 ⎠
12. En el circuito mostrado en la figura tenemos dos
espiras circulares iguales de sección A = l m2 cada 16. Dos rieles conductores paralelos largos están
una, pero con diferentes campos magnéticos que va separados una distancia d. En un extremo están
unidos por medio de una resistencia R. Una barra
rían con el tiempo B1 = 2t + 4t y B2 = −5t .
2 2
conductora de longitud d se hace resbalar con

33
Ley de Faraday e inductancia Hugo Medina Guzmán

velocidad constante v a lo largo de los rieles. La gauss, ¿cuál es la diferencia de potencial inducida que
barra y los carriles tienen resistencia insignificante. se genera entre los extremos de un eje de las ruedas
a) ¿Qué corriente fluye en el circuito? del tren si su longitud es de 1,25 m?
b) ¿Qué potencia se requiere para mover la barra? Respuesta. 22,4 x 10-4 V.
c) ¿Cuál es la potencia disipada en la resistencia
compararla con la potencia requerida para mover la 20. Una barra metálica de 1 m de longitud se deja
barra? caer desde una altura de 20 m permaneciendo
Respuesta. horizontal con sus extremos apuntando en la
ε = Bdv , dirección Este-Oeste. ¿Qué diferencia de potencial se
ε Bdv induce entre sus extremos en el instante justo antes de
a) I = = . llegar al suelo? (Componente horizontal del campo
R R magnético terrestre: BH = 0,17 gauss.)
B 2 d 2v 2 Respuesta. 3,37 x 10-4V..
b) P1 = Fv = (IdB )v = ,y
R 21. Una bobina de 100 espiras es extraída en una
2 décima de segundo de su posición entre los polos de
⎛ Bdv ⎞
c) P2 = I R = ⎜ ⎟ = P1
2
un imán donde existe un flujo de 50 x 10-5 Wb.
⎝ R ⎠ ¿Cuál es la fuerza electromotriz media inducida en la
bobina?
17. Un alambre de resistencia insignificante se dobla Respuesta. 0,5 V.
formando un rectángulo de lados a y b. Hay un
campo magnético uniforme perpendicular al plano del 22. Una espira cuadrada de área 500 cm2 se sitúa
lazo. ¿Una barra de resistencia R se coloca entre los perpendicularmente a un campo de intensidad 1 T. La
puntos X e Y sobre el lazo y se mueve a la derecha la espira se voltea hasta que su plano es paralelo al
con velocidad constante v . ¿Qué corriente fluye en la campo en un tiempo de 0,1 s. ¿Cuál es la fuerza
barra? ¿En qué dirección va la corriente? electromotriz media inducida?
Respuesta. 0,5 V.

23. Encontrar la autojnductancia de un solenoide de


longitud finita h y radio a (h >> a) en términos de las
potencias de a/h.

24. ¿Cual es la inductancia mutua entre un alambre


delgado infinito y las espiras 1 y 2..

Respuesta.
La corriente fluye hacia abajo en la barra. Ambos
lazos contribuyen a la fuerza electromotriz inducida,
tal que:
2 Bbv
I=
R
18. Un solenoide largo, delgado tiene 750 vueltas por
el metro, y su corriente se aumenta con una razón de
60 A/s. ¿Cuál es el campo eléctrico inducido dentro
del solenoide en un punto a 5 mm del eje y a mitad de
la distancia entre los extremos?
25. ¿Cuál es la inductancia mutua entre la bobina
Respuesta.
toroidal con n vueltas densamente enrolladas y una
Considere el flujo a través de un círculo de radio r.
espira cerrada encerrando el toroide
dΦ B dB dI
ε= = π r2 = π r μ0 N = 2π rE
2

dt dt dt 26. ¿Cuál es la autoinductancia del toroide mostrado


en la figura?
1 dI
E = μ 0 rN
2 dt
(
= (0,5) 4π × 10
−7
)(
5 × 10 −3 (750 )(60) )
-3
= 1,4 x 10 V/m

19. Un tren se desplaza de Norte a Sur con una 27. Encontrar la autoinductancia de un toroide de
velocidad uniforme de 120 km/h. Si la componente radio b, n vueltas, y sección circular de radio a.
vertical del campo magnético terrestre es de 0,54

34
Ley de Faraday e inductancia Hugo Medina Guzmán

28. Determinar aproximadamente la inductancia 36. Para el Circuito de la figura. Encontrar;


mutua entre dos anillos delgados, coaxiales, de radios a) El valor inicial de la corriente a través de la batería.
a y b, si la distancia d entre sus planos es mucho b) La corriente final por R2.
mayo que a y b. c) La corriente final por R1.
d) La corriente final por la batería.
29. Determinar la inductancia mutua de dos
cuadrados idénticos de lado a colocados a una
distancia l y coincidentes con las caras opuestas de
un paralelepípedo. Encontrar la fuerza entre ellos.

30. Determinar la autoinductancia de un cuadrado de


lado b hecho de un alambre de radio a
(a << b).

31. Mostrar que cuando dos bobinas se conectan en


serie la autoinductancia de la combinación es
L = L1 + L2 ± 2 M . Explicar para que conexión es 37. A una batería de 2,73 V se conecta una resistencia
+ y para cual es -. de 1,73 Ω y dos autoinducciones en serie de 10 mh
cada una. Calcúlese la inductancia mutua entre ellas,
32. Encuentre la inductancia equivalente en el sabiendo que, a los 0,01 s de conectar, la corriente
circuito que se muestra en la figura. que circula es de 1 A.
Respuesta. M =1,35 mH.

38. Una bobina tiene una inductancia de 10 h y una


resistencia de 40 Ω . Si se aplica una fuerza
electromotriz de 200 V, ¿Cuál es la energía
almacenada en el campo magnético una vez que la
corriente alcanza su valor máximo?
Respuesta. 126 J...

33. Un alambre largo y recto de radio a lleva una 39. Calcúlese la autoinducción de una bobina de n
corriente I. ¿Cuál es la densidad de energía magnética espiras, sección A y radio medio R.
en la superficie del alambre? Respuesta.
Calcular la energía magnética almacenada por unidad μ 0 An 2
de longitud en el interior del alambre. L=
2πR
34. La máxima energía almacenada por un circuito 40. Determínese la energía por unidad de volumen
LC es 10-4 Joules y la capacidad es 4 μ f. Si la almacenada en la Inductancia formada por un
corriente máxima que circula por el circuito es 2A, solenoide toroidal de longitud l y sección A que el
calcular: paso de una corriente crea en su interior un campo
a) La inductancia del circuito. magnético B.
b) La frecuencia de oscilación. Respuesta.
W B2
35. Sea el circuito da la figura y supongan que el = .
interruptor S está cerrado en t = 0 ,
Al 2μ 0
a) Describir la trayectoria seguida por la corriente 41. Por un solenoide recto de 500 espiras, longitud 1
inmediatamente después de que se cierra el m y sección 10 cm2 circula una corriente de 10 A.
interruptor. Sobre este solenoide se arrolla una pequeña bobina de
b) ¿Cuál es la trayectoria de la corriente después de 50 espiras colocadas en su centro. ¿Cuál es la
mucho tiempo? inductancia mutua?
Respuesta. 3,14 x 10-4 h.

42. Un solenoide con 400 vueltas, 4 cm de diámetro


y 50 cm de largo. Lleva una corriente en el sentido
mostrado en la figura. La corriente produce un campo
magnético, de magnitud 1,5 mT, en el centro del
solenoide.

35
Ley de Faraday e inductancia Hugo Medina Guzmán

a) ¿Cuál es la corriente?
A) 1,1 A B) 1,3 A C) 1,5 A D) 1,7 A E) 1,9 A
b) ¿Cuál es el flujo magnético en el solenoide? a) La fuerza electromotriz media en la bobina
A) lxl06 Wb B) 2x106 Wb C) 4x106 Wb D) 6x 106 primaria, en el intervalo de los 0.20 s, es
Wb E) 8x106 Wb cercanamente igual a:
A) cero B) 0,25 μ V C) 0,45 μ V D) 0,65 μ V E)
43. Un alambre vertical largo lleva una corriente 0,85 μ V
constante de 10 A. Un par de carriles horizontales y b) La fuerza electromotriz media en la bobina
separados 0,20 m. Una resistencia de 20 ohmios secundaria, en el intervalo de los 0.20 s, es
conecta los puntos a y b, en el extremo de los carriles. cercanamente igual a:
Una barra está en contacto con los carriles, y es A) 3 μ V B) 4 μ V C) 5 μ V D) 6 μ V E) 7 μ V
movida por una fuerza externa con una velocidad c) Las polaridades de la fuerza electromotriz
constante de 0,40 m/s, como se muestra. La barra y inducidas en las bobinas primaria y secundaria son:
los carriles tienen resistencia insignificante. En un A) Q y R están al mismo potencial; S es positivo y T
instante t1 dado, la barra está a 0,20 m del alambre, es negativo
como se muestra en la figura. B) Q es positivo y R es negativo; S es positivo y T es
negativo
C) Q es positivo y R es negativo; T es positivo y S es
negativo
D) R es positivo y Q es negativo; S es positivo y T es
negativo
E) R es positivo y Q es negativo; T es positivo y S es
negativo

a) En el instante t1 , la corriente inducida y su 45. En la figura, un imán de barra se aleja del


solenoide. La corriente inducida a través de la
dirección a través de la resistencia es cercanamente
resistencia R es:
igual a:
A) 0,02 μ A, de a a b
B) 0,02 μ A, de b a a
C) 0,04 μ A, de a a b
D) 0,04 μ A, de b a a
E) 0,02 μ A, de b a a A) cero
b) En el instante t 2 , inmediato posterior, la B) de a a b
C) de b a a
diferencia potencial a través de la resistencia es 0,30
μ V. El intervalo del tiempo, Δt = t 2 − t1 es 46. En la figura, una barra está en contacto con un
cercanamente igual a: par de carriles paralelos. Un campo magnético
A) 0,5 s B) 1,0 s C) 1,5 s D) 2,0 s E) 2,5 s constante, uniforme, perpendicular al plano de los
carriles, está presente. La barra está en el movimiento
44. La figura (a) muestra la sección transversal de un con la velocidad v . La corriente inducida a través de
toroide. Una bobina primaria tiene 400 vueltas, que se la resistencia R es:
arrollan totalmente alrededor de la forma toroidal.
Los terminales de la bobina primaria son Q y R. Una
bobina circular secundaria de 3 vueltas se arrolla
firmemente alrededor de la forma toroidal, con los
terminales S y T. Las bobinas primarias y secundarias
se muestran en la figura (b). Se provee una corriente
constante de 300 mA a la bobina primaria por medio
de una batería que no se muestra. Un interruptor (no
mostrado) se abre, que desconecta los terminales Q y A) Cero B) de a a b C) de b a a
R de la batería. La corriente en la bobina primaria
baja a cero en un intervalo del tiempo de 0,20 S.
36
Ley de Faraday e inductancia Hugo Medina Guzmán

47. En la figura se muestra, un alambre recto largo polaridad de la fuerza electromotriz inducida entre los
que lleva una corriente constante I, y un cilindro terminales X y Y es:
coaxial largo por el que vuelve la misma corriente.
Un lazo rectangular abcd está fuera del cilindro. Se
puede observar una vista superior, y una vista de la
sección transversal, a través de la línea punteada. La
corriente I está aumentando. La corriente inducida a
través de la resistencia R es:

A) X e Y están al mismo potencial.


B) X es positivo e Y es negativo
C) Y es positivo y X es negativo

51. En la figura, hay un campo magnético uniforme


de la magnitud B = 1,5 T y dirigido hacia el plano del
papel en la región mostrada. Fuera de esta región el
campo magnético es cero. Un lazo rectangular de
0,20 m por 0,60 m y de resistencia 3 Ω está siendo
jalado hacia el campo magnético por una fuerza
A) Cero externa, como se muestra.
B) de a a b
C) de b a a

48. En la figura, dos solenoides están frente a frente.


El interruptor S, inicialmente abierto, es cerrado. La
corriente inducida a través de la resistencia R es:

a) ¿Cuál es la dirección (horario u antihorario) de la


corriente inducida en el lazo?
b) Calcular la magnitud de la fuerza externa Fext
requerida para mover el lazo con un de velocidad
constante de v = 3.9 m/s. La masa del lazo es 0,008
kilogramos.
Respuesta. a) antihorario, b) 1 x 10-1 N
A) cero
B) de a a b 52. En la figura, un lazo circular del área 0,050 m2
C) de b a a con 50 vueltas y resistencia 7,0 Ω se pone entre los
polos de un imán de herradura en forma de "U"
49. En la figura los dos solenoides están alineados. El grande, donde el campo magnético es 0,11 T. El lazo
interruptor S, cerrado inicialmente, se abre. La rota sobre un diámetro. ¿Cuánta carga atraviesa el
polaridad de los terminales inducidos de la fuerza alambre en 1/4 de revolución?
electromotriz en X e Y es:

A) X e Y están al mismo potencial.


B) X es positivo e Y es negativo
C) Y es positivo y X es negativo
Respuesta. 0,039 C
50. En la figura, un alambre recto lleva una corriente
constante I. Una barra está en contacto con un par de
carriles y está en el movimiento con velocidad v. La

37
Corriente alterna Hugo Medina Guzmán

CAPÍTULO 5. Corriente alterna


GENERADOR DE CORRIENTE ALTERNA. del imán en este punto, se entrega una corriente
Un generador eléctrico transforma energía máxima en la dirección mostrada.
mecánica en energía eléctrica. Se ha visto que se
induce una fem en un conductor cuando éste
experimenta un cambio en el flujo que lo
eslabona. Cuando el conductor forma un circuito
cerrado, puede detectarse una corriente inducida.
En un generador, una bobina de alambre gira en
un campo magnético, y la corriente inducida se
trasmite por alambres a gran distancia de su
origen.

En la figura (b) la espira está en posición vertical


con el lado M en la parte superior, en el punto no
se cortan las líneas de flujo y la corriente inducida
decae a 0.

En la figura anterior se muestra la construcción de


un generador simple. Básicamente hay tres
componentes: un inductor, una armadura y un
conmutador de anillos colectores de deslizamiento
con escobillas. El inductor puede ser un imán
permanente o un electroimán. La armadura del
generador de la figura consiste en una sola espira
de alambre (inducido) suspendida entre los polos Cuando la espira se encuentra otra vez horizontal
del inductor. Un par de anillos colectores de como en la figura (c), el lado M está ahora frente
deslizamiento se conectan a cada de la espira y al polo norte del imán. Por consiguiente, la
giran junto con ella en el campo magnético. La corriente suministrada al anillo colector M’ ha
corriente inducida se obtiene del sistema mediante cambiado de dirección. Una corriente fluye a
escobillas de grafito, las cuales están montadas través de la resistencia externa en una dirección
sobre cada anillo de deslizamiento. Se suministra opuesta a la que se experimentó anteriormente.
energía mecánica al generador al girar la
armadura en el campo magnético y se genera
energía eléctrica en corriente inducida.

Producción de una corriente alterna. A fin de


comprender la operación de un generador de
corriente alterna, es conveniente seguir los pasos
de la espira a través de una rotación completa, y
observación de la corriente en el proceso de
rotación. Supóngase que se hace girar la espira
mecánicamente en sentido contrario a las
manecillas del reloj. En la figura (d) la espira está en posición vertical
nuevamente, pero ahora el lado M está en la parte
inferior; no se cortan líneas del flujo, y otra vez la
corriente inducida decae a cero. A continuación la
espira regresa a su posición horizontal, como se
muestra en la figura (a), y el ciclo se vuelve a
repetir. Así pues, la corriente suministrada por tal
generador alterna en forma periódica, y la
dirección cambia dos veces en cada rotación.
La fem generada en cada segmento de la espira en
rotación debe cumplir la relación
En la figura (a) la espira está en posición
horizontal, con el lado M encarando al polo sur

1
Corriente alterna Hugo Medina Guzmán

ε = Blvsenθ , donde v es la velocidad de un La fuerza electromotriz varía desde un valor


segmento de alambre en movimiento de longitud máximo cuando θ = 90° a un valor de cero
l en un campo magnético B. La dirección de la cuando θ = 0º. La fuerza electromotriz
velocidad v con respecto al campo B en cualquier instantánea máxima es, por consiguiente,
instante se denota por medio del ángulo θ . ε máx = nBAω , ya que sen 90° = 1. Si se
Cálculo de la fem inducida. Considérese el establece la ecuación ε máx = nBAω senθ en
segmento M de la espira giratoria cuando alcanza términos de la fuerza electromotriz máxima,
la posición indicada en la figura a continuación. entonces
ε = ε máx senθ

Para ver la variación explícita de la fem generada


en función del tiempo necesario recordar que
θ = ω t = 2πf t
Donde f conocida como frecuencia es el número
La fem instantánea en esa posición queda de revoluciones por segundo que realiza la espira.
expresada por la ecuación ε = Blvsenθ . Si la
espira gira en un círculo de radio r, la velocidad La unidad en el SI para la frecuencia es el hertz
instantánea v puede encontrarse de v = ω r , (Hz), que se define como 1 ciclo por segundo.
donde ω es la velocidad angular en radianes por 1 Hz = 1 ciclo/s
segundo. La fuerza electromotriz instantánea se Por lo tanto, una corriente alterna de 60 ciclos por
encuentra al sustituir la velocidad instantánea en segundo tiene una frecuencia de 60 Hz.
la ecuación ε = Blvsenθ
ε = Blω rsenθ De este modo la ecuación ε = ε máx senθ puede
En el segmento de alambre opuesto a M se induce expresarse de la siguiente manera:
una fuerza electromotriz idéntica y no se genera ε = ε máx senω t = ε máx sen 2πf t
una fuerza electromotriz neta en los otros
segmentos. De aquí que la fem instantánea total
La unidad en el SI para la frecuencia es el hertz
sea el doble del valor expresado por la ecuación
(Hz), que se define como 1 ciclo por segundo.
ε = Blω rsenθ , o 1 Hz = 1 ciclo/s
ε máx = 2 Blω rsenθ Por lo tanto, una corriente alterna de 60 ciclos por
Pero el área A de la espira es segundo tiene una frecuencia de 60 Hz.
A = l × 2r Puesto que la corriente inducida es proporcional a
Y la ecuación ε máx = 2 Blω rsenθ puede la fuerza electromotriz inducida, de acuerdo con
escribirse como: la ley de Ohm la corriente inducida también
ε máx = nBAω senθ , donde n es el número de variaría sinusoidalmente según la expresión
vueltas de alambre. i = imáx sen 2πf t
Esta ecuación expresa un principio muy La corriente máxima ocurre cuando la fuerza
importante que está relacionado con el estudio de electromotriz inducida es máxima. La variación
las corrientes alternas: sinusoidal es análoga a la representada
gráficamente en la figura anterior.
Si la armadura gira con una velocidad angular
constante en un campo magnético constante, la Ejemplo 1. La armadura de un generador simple
magnitud de la fuerza electromotriz inducida varía de ca consta de 100 vueltas de alambre, cada una
sinusoidalmente en función del tiempo. de ellas con un área de 0,2 m2. La armadura gira
Este hecho se muestra en la figura siguiente. con una frecuencia de 60 rev/s en un campo
magnético constante de densidad de flujo l0-3 T.
¿Cuál es la fem máxima que se genera?
Solución.
Primero es necesario calcular la velocidad angular
de la bobina.
ω = 2π (60) = 377 rad/s.
Si este valor se sustituye y también los otros
parámetros conocidos en la ecuación
ε máx = nBAω , se obtiene

2
Corriente alterna Hugo Medina Guzmán

ε máx = nBAω (
= (100 ) 10
−3
)(0,2)(377) i R R = V0 senω t
= 7,54 V
V0
ANGULO DE FASE. FASORES. ⇒ iR = senω t
Una manera conveniente de representar una R
cantidad que varia sinusoidalmente (es decir,
senθ como varía con θ ) con el tiempo está en La diferencia de potencial en la resistencia es
términos de un vector que rota llamado fasor.
v R = V0 senω t
En el caso de un voltaje de corriente alterna, la

longitud del fasor V máx corresponde a Vmáx , y lo En una resistencia, la intensidad i R y la
imaginamos que rota f veces por segundo en
diferencia de potencial v R están en fase. La
sentido antihorario.
La componente vertical del fasor en todo relación entre sus amplitudes es
momento corresponde al voltaje instantáneo v
→ VR
desde que la componente vertical de V máx , es IR =
R
v = Vmáxsenθ = Vmáxsen2πf t = Vmáxsenω t

El resultado es mostrado en la figura siguiente. Con V R = V0 , la amplitud de la fuerza


electromotriz alterna

Como vemos en la representación vectorial de la


figura, al cabo de un cierto tiempo t , los vectores
rotatorios que representan a la intensidad en la
resistencia y a la diferencia de potencial entre sus
extremos, ha girado un ángulo ω t . Sus
proyecciones sobre el eje vertical son, los valores
en el instante t de la intensidad que circula por la
resistencia y de la diferencia de potencial entre
En una manera similar un fasor I máx se puede
sus extremos.
utilizar para representar una corriente alterna I .
Los fasores son útiles porque el voltaje y la Un condensador conectado a un generador de
corriente en una corriente alterna en un circuito de corriente alterna
corriente alterna o en un elemento del circuito
tiene siempre la misma frecuencia f pero los picos
en V e I pueden ocurrir en diferentes tiempos.

CIRCUITOS DE CORRIENTE ALTERNA


CON RESISTENCIA, INDUCTANCIA Y
CONDENSADOR.

Una resistencia conectada a un generador de


corriente alterna
vC = V0 senω t

En un condensador la carga q , la capacidad C y


diferencia de potencial v entre sus placas están
relacionadas entre sí

q = Cv
La ecuación de este circuito simple es:
Si se conecta las placas del condensador a un
generador de corriente alterna
(Intensidad por resistencia igual a la fem)

3
Corriente alterna Hugo Medina Guzmán

q = CV0 senω t V0 V ⎛ π⎞
iL = − cosω t = 0 sen⎜ ω t − ⎟
ωL ωL ⎝ 2⎠
La intensidad se obtiene derivando la carga
dq
respecto del tiempo, i= La intensidad i L de la en la bobina está
dt retrasada 90º respecto de la diferencia de
potencial entre sus extremos v L . La relación
⎛ π⎞ entre sus amplitudes es
iC = CωV0 cosω t = CωV0 sen⎜ ω t + ⎟
⎝ 2⎠
VL
IL =
Para un condensador, la intensidad iC está ωL
adelantada 90º respecto a la diferencia de
potencial vC . La relación ente sus amplitudes es Con V L = V0 , la amplitud de la fuerza
electromotriz alterna
I C = CωVC
REACTANCIA.

Con VC = V0 , la amplitud de la fuerza La reactancia inductiva de una inductancia es


una medida de su eficacia en resistir el flujo de
electromotriz alterna
una corriente alterna en virtud de la de la fuerza
electromotriz autoinducida por el cambio de la
Una inductancia conectada a un generador de corriente en ella. A diferencia con el caso de una
corriente alterna resistencia, no hay energía disipada en una
inductancia pura. La reactancia inductiva X L del
cualquier inductor cuya inductancia es L (en
henrios) cuando la frecuencia de la corriente es f
(en hertzios) la reactancia inductiva es
X L = 2πf L = ω L .
Cuando una diferencia potencial V de frecuencia f
se aplica a través de una inductancia cuya
reactancia es X L con la frecuencia f, fluirá la
V
corriente I = . La unidad de X L es el
v L = V0 senω t XL
ohmio.
Ya hemos estudiado la autoinducción y las
corrientes autoinducidas que se producen en una La reactancia capacitiva de un condensador es
bobina cuando circula por ella una corriente i una medida de su eficacia en resistir el flujo de
variable con el tiempo. una corriente alterna en virtud de la diferencia
potencial inversa a través del debido a la
La ecuación del circuito es (suma de fuerzas acumulación de carga en sus placas. No hay
electromotrices es igual a la intensidad por la pérdida de energía asociada a un condensador en
resistencia), como la resistencia es nula: un circuito de corriente alterna. La reactancia
capacitiva X C de un condensador cuya
capacitancia es C (en faradios) cuando la
frecuencia de la corriente es f (en hertzios) es
di 1 1
−L + V0 senω t = 0 XC = = .
dt 2π f C ω C
Cuando una diferencia potencial V de frecuencia f
se aplica a través de un condensador cuya
Integrando esta ecuación obtenemos i L en
reactancia es X C con la frecuencia f, fluirá la
función del tiempo

4
Corriente alterna Hugo Medina Guzmán

V Ejemplo 4. La reactancia de una inductancia es


corriente I = . La unidad de X C es el 80 Ω a 500 Hz. Encontrar la inductancia.
XC Solución.
ohmio. Como X L = ωL = 2π fL
XL 80
Ejemplo 2. a) Muestre gráficamente cómo X C y L= = = 0,0255 H
X L varían con la frecuencia. 2π f 2π (500)
= 25,5 mH
b) ¿Qué sucede a X C y X L en el límite f = 0?
c) ¿Cuál es el significado físico de la respuesta a CIRCUITO RLC EN SERIE
(b)? Ahora consideramos un circuito de corriente
Solución. alterna que contenga resistencia, inductancia, y
a) capacitancia en serie, como en figura siguiente.

La corriente es igual para todos los elementos:


b) Cuando f = 0, X L = 2π fL = 0 y i = isenω t
X C = 1 2π fC = ∞ . i = I Rsenω t = I Lsenω t = I C senω t
c) Una corriente con f = 0 es una corriente En todo momento el voltaje aplicado v es igual a
continua.
Cuando una corriente constante fluye en una la suma de las caídas de voltaje vR , vL , y vC :
inductancia, no hay una fuerza electromotriz v = vR + vL + vC
inducida para obstaculizar la corriente, y la
reactancia inductiva es por consiguiente cero. ⎛ π⎞
v = VRsenω t + VLsen⎜ ω t + ⎟
Una corriente directa no puede pasar a través de ⎝ 2⎠
un condensador porque sus placas se aíslan uno de
la otra, así que la reactancia capacitiva es infinita ⎛ π⎞
+ VC sen⎜ ω t - ⎟
e I = V X C = 0 cuando f = 0. ⎝ 2⎠
Una corriente alterna no pasa realmente a través Los voltajes instantáneos a través de los
de un condensador sino son oleadas hacia adelante elementos de circuito son
y hacia atrás en el circuito en ambos lados de él. VR = IR VL = IX L VC = IX C
Ejemplo 3. Un condensador de 10 μ F se conecta
a una fuerza electromotriz de 15 V, 5 kHz.
Encontrar
a) La reactancia del condensador.
b) El flujo de corriente. .
Solución.
1 1
a) X C = =
ωC 2π f C
1
= 3,18Ω
=
( )(
2π 5 × 10 10 × 10 −6
3
)
V 15
b) I = =
X C 3,18
= 4,72 A

5
Corriente alterna Hugo Medina Guzmán

Porque vR , vL , y vC están fuera de fase el uno


con el otro, esta fórmula es solamente para los
voltajes instantáneos, no para los voltajes
efectivos.
Puesto que deseamos trabajar con voltajes y
corrientes efectivos, no los instantáneos, debemos
Z = R 2 + (X L − X C )
2
consideramos considerar las diferencias de fase.
Para hacer esto, podemos utilizar los fasores para
representar las diferentes cantidades efectivas.
Cuando X L < XC :
Esto se hace en las figuras a continuación para los
voltajes. Para encontrar la magnitud V de la suma

V de los diferentes voltajes efectivos,
procedemos de esta manera:
→ →
Z = R 2 + (X L − X C )
2
1. Encuentre la diferencia V L − V C . Si
→ →
V L > VC , V L − V C será positivo y señalará Ejemplo 5. Una resistencia, un condensador y una
→ → inductancia están conectados en serie a una fuente
hacia arriba, si V L < VC , V L − V C será negativo de corriente alterna. Los voltajes efectivos a
y señalará hacia abajo. través de los componentes del circuito son:
→ → → → VR = 5V , VC = 10V , y V L = 7 V . Encontrar:
2. Sume V L − V C a V R para obtener V .
→ → →
a) El voltaje efectivo de la fuente y
Como V L − V C es perpendicular a V R , utilice b) El ángulo de fase en el circuito.
el teorema de Pitágoras para encontrar la Solución.
= VR2 + (VL − VC )
magnitud V: 2
a) V
V = VR2 + (VL − VC )
2

5 2 + (7 − 10 ) = 5 2 + (− 3)
2 2
=
Porque V R = IR V L = IX L VC = IX C
= 25 + 9 = 34 = 5,8 V
Podemos escribir la expresión para V en la forma
Se puede notar que los voltajes efectivos a través
V = I R 2 + (X L − X C )
2
de C y R son mayores que el voltaje
VL − VC 7 − 10 3
b) tan φ = = =−
Impedancia de un circuito en serie VR 5 5
= R 2 + (X L − X C )
V 2
La cantidad Z = = - 0,6
I ⇒ φ = −31º
Es conocida como la impedancia del circuito y El ángulo de fase negativo significa que el voltaje
corresponde a la resistencia de la corriente a través de la resistencia está adelantado al voltaje
continua. La unidad de Z es el ohm. Cuando un aplicado. Equivalentemente podemos decir que la
voltaje de corriente alterna cuya frecuencia es f se corriente en el circuito se adelanta al voltaje, (Los
aplica a un circuito cuya impedancia es Z a tal fasores rotan en el sentido antihorario).
frecuencia, el resultado es la corriente
V Ejemplo 6. Una fuerza electromotriz alterna de
I= tensión eficaz, 110 V y frecuencia 10 Hz está
Z
La figura siguiente muestra los diagramas de aplicada a un circuito formado por una resistencia
fasores de la impedancia. El ángulo de fase puede de 7 Ω en serie con un condensador.
ser calculado por cualquiera de las dos fórmulas Determínese la capacidad de éste, si la intensidad
siguientes. que circula es de 5 A.
Solución.
XL − XC R
tan φ = o cos φ = Aplicando la fórmula de la ley de Ohm de las
R Z corrientes alternas, tenemos:
Cuando X L > X C : V 110
I= =
2 2
⎛ 1 ⎞ ⎛ 1 ⎞
R +⎜ 2
⎟ 7 +⎜2

⎝ ωC ⎠ ⎝ ωC ⎠
= 5 A.

6
Corriente alterna Hugo Medina Guzmán

De donde:
Z = R 2 + X L2 = 20 2 + 12,6 2
1
= 20,85Ω , = 23,6Ω
ωC
Por lo tanto la corriente es
1 1
C= = V 28
20,85ω 20,85(2π 60) I= =
Z 23,6
= 127 x 10-6 F
= 1,18 A
Ejemplo 7. Un condensador de 5 μ F está en
serie con una resistencia de 300 Ω , y a la
combinación se aplica un voltaje de120 V y 60
Hz. Encuentre
a) la corriente en el circuito y
b) el ángulo de la fase.
Solución.
a) La reactancia del condensador a 60 hertzios es X L − X C 12,6 − 0
1 1 b) tan φ = =
XC = = = 531Ω R 20
2πfC 2π (60)(5 × 10 −6 ) = - 0,63
⇒ φ = 32º
El ángulo positivo de la fase significa que el
voltaje en el circuito adelanta a la corriente.

Ejemplo 9. Si al circuito anterior le conectamos


un generador que suministre tensión v alterna
dada por v = v A − vB = V0senωt , hallar la
intensidad I que circula y la diferencia de
potencial entre los dos extremos izquierdo y
Como X L = 0 , la impedancia del circuito es derecho de cada elemento cuando los transitorios
hayan desaparecido prácticamente.
Z = R 2 + (X L − X C ) = R 2 + (− X C )
2 2

2
= R2 + X C = 300 2 + 5312
= 610Ω
Luego la corriente es
V 120
I= = = 0,197 A
Z 610
b)
Solución.
XL − XC 0 − 531
tan φ = = V0 ⎛ π ⎞
R 300 I= cos⎜ ωt − − ϕ ⎟
= - 1,77 ⎛ 1 ⎞
2
⎝ 2 ⎠
⇒ φ = −61º R 2 + ⎜ ωL − ⎟
⎝ ωC ⎠
El ángulo negativo de la fase significa que la
corriente en el circuito adelanta al voltaje. ⎛ 1 ⎞
⎜ ωL − ⎟
Con ϕ = tan ⎜
−1 ωC ⎟
Ejemplo 8. Una inductancia de 5 mH, 20 Ω está
⎜ R ⎟
conectado a una fuente de energía de 28 V, 400 ⎜ ⎟
Hz. Encuentre ⎝ ⎠
a) La corriente en la inductancia y Entre los extremos de R:
b) el ángulo de la fase.
Solución. v = IR
a) La reactancia de la inductancia es:
(
X L = 2πfL = 2π (400) 5 × 10 −3 ) Entre los extremos de L:
= 12,6Ω
Y su impedancia es;

7
Corriente alterna Hugo Medina Guzmán

v=
V0ωL
cos(ωt − ϕ ) v = Vmsenωt = 1303 2sen (− 80º37')
= 1843(− 0,9866 )
2
⎛ 1 ⎞
R 2 + ⎜ ωL − ⎟
⎝ ωC ⎠ = -1818 V.

RESONANCIA EN SERIE
Entre los extremos de C: La impedancia en un circuito de corriente en serie
es un mínimo cuando X L = X C ; bajo estas
cos(ωt − ϕ − π )
V0
v= V
⎛ 1 ⎞
2
circunstancias Z=R e I= .
ωC R + ⎜ ωL −2
⎟ R
⎝ ωC ⎠ La frecuencia de la resonancia f R de un circuito

Ejemplo 10. Un circuito está formado por una es aquella frecuencia en la cual X L = X C :
autoinducción de 0,1 H, un condensador de 5 μ F 1 1
2π f R L = ⇒ fR =
y una resistencia de 100 Ω en serie. Calcúlese el 2π f RC 2π LC
valor del potencial instantáneo cuando la corriente
Cuando la diferencia potencial aplicada a un
es nula, si la intensidad eficaz del circuito es de
2,12 A y el generador es de 50 ciclos. circuito tiene la frecuencia f R , la corriente en el
circuito es un máximo. Esta condición se conoce
como resonancia. En la resonancia el ángulo de la
fase es cero ya que X L = X C .

Ejemplo 11. Un condensador de 10 μ F , una


inductancia de 0,10 H, y una resistencia de 60 Ω
se conectan en serie a través de una fuente de
poder de 120 V, 60Hz. Encontrar.
a) La corriente en el circuito y
b) El ángulo de fase.
Solución. Solución.
La impedancia del circuito vale a) Las reactancias son.
X L = 2πfL = 2π (60 )(0,1)
2
⎛ 1 ⎞
Z = R + ⎜ ωL −
2

⎝ ωC ⎠ = 38Ω
1 1
2
XC = =

= 100 + ⎜100π 0,1 −
2 1 ⎞
−6 ⎟
(
2πfC 2π (60) 10 × 10 −6 )
⎝ 100π 5 × 10 ⎠
= 265Ω
= 615Ω Luego la impedancia es
Con ello la fuerza electromotriz es:
Z = R 2 + (X L − X C )
2
V = IZ = (2,12 )(615)
=1303 V. 60 2 + (38 − 265)
2
=
El ángulo de fase es:
= 235Ω
⎛ 1 ⎞
⎜ ωL − ⎟ De aquí la corriente del circuito es.
⎝ ωC ⎠ (31,4 − 637 ) V 120
tan φ = = I= =
R 100 Z 235
= - 6,056, = 0,51 A
⇒ φ = −80º37' XL − XC
La intensidad instantánea resulta nula cuando se b) tan φ =
cumple: R
I m sen (ωt − φ ) = 0 =
38 − 265 − 227
= = -3,78
Es decir, cuando (ωt − φ ) = 0 , 60 60
⇒ φ = −75º
⇒ ωt = φ = −80º37'
El ángulo negativo de la fase significa que la
Con ello la fuerza electromotriz instantánea es: corriente en el circuito adelanta al voltaje.

8
Corriente alterna Hugo Medina Guzmán

Luego: i (t ) = 2cos(100 t − 36,87° )

ii) La amplitud (o valor máximo) del voltaje en la


resistencia, en la inductancia y en el condensador.
Vmáx 200
I máx = = = 2 A.
Z 100
Como La corriente es igual para todos los
elementos:
i(t ) = I máx cos(ω t + φ )
i (t ) = I R cos(ω t + φ ) = I L cos(ω t + φ ) = I C cos(ω t + φ )

I R = I L = I C = I máx = 2A.
El diagrama de fasores de la corriente y voltajes
del circuito,
Ejemplo 12. Corriente alterna. El voltaje i (t ) = 2cos(100 t − 36,87°)
instantáneo del generador de un circuito RLC en v(t ) = 200cos100 t
serie está dado por la relación: v(t) = 200 cos(l00t)
⎛ π⎞
en voltios. Los valores de la resistencia, la v(t ) = VR cos100 t + VL cos⎜100t + ⎟
inductancia; el condensador son: 80 Ω : 0,40 H y ⎝ 2⎠
100 μ F, respectivamente. Calcule:
⎛ π⎞
i) La corriente instantánea del circuito. + VC cos⎜100 t - ⎟
ii) La amplitud (o valor máximo) del voltaje en la ⎝ 2⎠
resistencia, en la inductancia y en el condensador. Con.
Dibuje el diagrama de fasores de la corriente y R = 80Ω , X L = ωL = 100(0,40 ) = 40Ω ,
voltajes del circuito, indicando sus valores
respectivos. 1 1
XC = = = 100Ω
Solución. (
ωC 100 100 × 10 −6 )
V R = I R R = 2(80) = 160 V.
V L = I L X L = 2(40) = 80 V.
VC = I C X C = 2(100) = 200 V.

i) La corriente instantánea del circuito.


La corriente es igual para todos los elementos:
i(t ) = I máx cos(ω t + φ )
R = 80Ω , X L = ωL = 100(0,40 ) = 40Ω ,
1 1
XC = = = 100Ω
ωC 100(100 × 10 −6 )
Z = R 2 + (X L − X C ) =
2

80 2 + (40 − 100) = 100 Ω


2

V 200
I máx = máx = = 2 A.
Z 100
X − X C 40 − 100 Ejemplo 13. En el circuito de la antena de un
tan φ = L = = −0,75 ⇒ receptor de radio sintonizado en una determinada
R 80 estación, R = 5 Ω , L = 5 mH, y C = 5 pF.
φ = -36,87° a) Encontrar la frecuencia de la estación.

9
Corriente alterna Hugo Medina Guzmán

b) Si la diferencia de potencial aplicada al circuito d) Siendo X L = 10Ω y X C = 25Ω ,


es 5x10-4 V, encontrar la corriente que fluye.
Cuando f = 400 Hz,
Solución.
1 XL 10
a) f R = L= =
2π LC 2π f 2π (400)
= 4 x 10-3 H.
1
= 1 1
2π (5 × 10 )(5 × 10 )
−3 −12 C= =
2π fX C 2π (400)(25)
= 1006 kHz = 1,6 x 10-5 F.
b) En resonancia, X L = X C y De aquí
Z = R. 1 1
fR = =
Luego
V 5 × 10 −4
2π LC 2π (4 ×10 )(1,6 ×10 )
−3 −5

I= = = 629 Hz,
R 5
= 0,1 mA. CIRCUITO RLC EN PARALELO.
Ahora consideramos un circuito de corriente
Ejemplo 14. En el circuito de la antena del alterna que contenga resistencia, inductancia, y
problema anterior, la inductancia es fija pero la condensador en paralelo, como en figura
capacidad del condensador es variable.. ¿Cuál siguiente.
debe ser la capacidad para recibir una señal de
radio de 800 kHz?
Solución.
1 1
C=
(2π f ) 2
L
=
(
2π 800 × 10 3 5 × 10 −3)( )
-12
= 7,9 x 10 F = 7,9 pF

Ejemplo 14. En un circuito de corriente alterna en


serie R = 20Ω , X L = 10Ω , y X C = 25Ω .
El voltaje es igual para todos los elementos:
Cuando la frecuencia es 400 Hz. v = Vsenω t
a) Encontrar la impedancia del circuito.
b) Encontrar el ángulo de fase. v = VRsenω t = VLsenω t = VC senω t
c) ¿Es la frecuencia de la resonancia del circuito Cuando una resistencia, una inductancia, y un
mayor o menor de 400 Hz? condensador se conectan en paralelo a través de
d) Encuentre la frecuencia de resonancia. una fuente de corriente alterna, el voltaje es igual
Solución. a través de cada elemento de circuito:
a) Z = R 2 + (X L − X C )
2 V = VR = VL = VC
En todo momento la corriente aplicada i es igual
20 2 + (10 − 25) = 25Ω
2
= a la suma de las caídas de voltaje iR , iL , y iC :
X L − X C 10 − 25 i = iR + iL + iC
b) tan φ = =
R 20 ⎛ π⎞
= - 0,75 i = I Rsenω t + I Lsen⎜ ω t − ⎟
⇒ φ = −37º ⎝ 2⎠
Un ángulo negativo de la fase significa que el ⎛ π⎞
+ I C sen⎜ ω t + ⎟
voltaje se retrasa de la corriente. ⎝ 2⎠
c) En la resonancia X L = X C . A 400 Hz,
i = Isen (ω t + φ )
X L < X C , así que la frecuencia se debe cambiar La corriente instantánea total es la suma de las
de tal manera en cuanto aumente X L , disminuya corrientes instantáneas en cada rama. Aunque el
I R la corriente en la resistencia está en fase con
1
X C . Como X L = 2π fL y X C = , V , la corriente I C en el condensador se adelanta
2π fC
está claro que el aumento de la frecuencia tendrá a V por 90° y la corriente I L en la inductancia se
este efecto. Por lo tanto la frecuencia de la retrasa a V por 90°. Para encontrar la corriente
resonancia debe ser mayor de 400 Hz.

10
Corriente alterna Hugo Medina Guzmán

total I , los fasores que representan a I R , a I C La corriente en la inductancia y en el condensador


está desfasados 180º.
y a I L se deben sumar vectorialmente, como en La corriente total I en el circuito es la
figura siguiente. diferencia entre corriente en L y en C.
I = IC − I L
La corriente que circula entre la inductancia y el
condensador sin contribuir a I es conocida como
corriente tanque y puede ser mayor de I .

En caso que X C = X L , las corrientes I C e I L


Las corrientes de las ramas en el circuito en
paralelo están dadas por: sean también iguales. Puesto que I C e I L están
V V V 180° fuera de fase, la corriente total I = 0. Las
I R = , IC = , IL =
R XC XC corrientes en la inductancia y el condensador se
cancelan. Esta situación se llama resonancia.
La suma de estas corrientes vectorialmente con la
ayuda del teorema de Pitágoras da: En un circuito en serie RLC, discutido
I = I + (I C − I L ) anteriormente, la impedancia tiene su valor
2 2

mínimo Z = R cuando X C = X L , una situación


R

El ángulo de fase φ entre la corriente y el voltaje


llamada también resonancia. La frecuencia para
está especificado por.
la cual X C = X L es
IR
cos φ = 1
I ω R = 2π f R =
Si I C es mayor que I L la corriente adelanta al LC
y se llama frecuencia de resonancia.
voltaje y el ángulo de la fase se considera
positivo; si I L es mayor que I C , la corriente se En un circuito paralelo RLC, la resonancia
retrasa al voltaje y el ángulo de la fase se corresponde a X C = X L , pero aquí la
considera negativo.
impedancia es un máximo con f R .
Impedancia del circuito paralelo RLC: Con f R , las corrientes en la inductancia y el
2
1 1 ⎛ 1 1 ⎞ condensador son iguales magnitud pero con 180°
= + ⎜ − ⎟ fuera de fase, así que ninguna corriente pasa con
Z R 2 ⎜⎝ X C X L ⎟⎠ la combinación. Así I = I R y Z = R. A
frecuencias más altas y menores que f R , I C no
RESONANCIA EN UN CIRCUITO EN
PARALELO. es igual al I L . y una cierta corriente puede pasar
La figura muestra una inductancia y un a través de la parte inductancia condensador del
condensador conectados en paralelo con una circuito, que reduce la impedancia Z a un valor
fuente de energía. menor que R. Por esto un circuito en serie puede
ser utilizado como selector para favorecer una
frecuencia particular, y un circuito paralelo con
los mismos L y C se puede utilizar como un
selector para discriminar contra la misma
frecuencia.

Ejemplo 15. Las reactancias de una bobina y de


un condensador conectados en paralelo y

11
Corriente alterna Hugo Medina Guzmán

alimentados por 15 V, 1000 Hz son, Luego:


respectivamente, X L = 20Ω y X C = 30Ω . V 10
IC = = = 0,5 A,
Encontrar X C 20
a) La corriente en cada componente,
V 10
b) La corriente total, IL = = = 0,8 A,
c) La impedancia del circuito, y X L 12,6
d) el ángulo de fase y la potencia total que se
V 10
disipa en el circuito. IR = = = 1,0 A,
Solución. R 10
V 15 b)
a) I L = = = 0,75 A,
X L 20
V 15
IC = = = 0,5 A
X C 30
b) I = I C − I L = 0,5 − 0,75
= - 0,25 A
El signo menos significa que la corriente total se
retrasa 90° detrás del voltaje.
V 15
c) Z = = = 60Ω
I 0,25 El diagrama de fasores muestra cómo las
corrientes deben ser Sumadas. Tenemos
La impedancia no es mayor que solamente
I = I R2 + (I C − I L )
2
X L o X C , también es mayor que su suma
aritmética.
1,0 2 + (0,5 − 0,8)
2
d) Porque el ángulo de fase aquí es 90°, =
cos φ = cos 90° = 0 y
1,0 2 + (− 0,3)
2
=
la potencia del circuito es
P = IV cos φ = 0 =1,09 = 1,04 A
Esto es una conclusión de la ausencia de V 10
resistencia en el circuito. c) Z = =
I 1,04
Ejemplo 16. Una resistencia de 10 Ω , un = 9,6Ω
condensador de 8 μ F, y una inductancia de 2 mH I 1,0
están conectados en paralelo a través de una d) cos φ = R = = 0,962
fuente de 10 V, 1000 Hz. Encuentre I 1,04
a) la corriente en cada componente, φ = −16º
b) la corriente total en el circuito, La corriente se retrasa del voltaje 16°.
c) la impedancia del circuito, y
P = IV cos φ = (10 )(1,04 )(0,962 )
d) el ángulo de fase y la disipación de potencia
total del circuito. = 10 W
Podemos también encontrar P de otra manera:
P = I R2 R = (1,0 ) (10)
2

= 10 W

Ejemplo 17. Un circuito se compone de dos


condensadores, una batería de 24V y una tensión
de corriente alterna conectados como se indica en
la figura. La tensión de corriente alterna viene
Solución. dada en V por V A − V B = 20 cos 120π t (t en
1 1
a) X C = segundos):
2π fC
=
( )(
2π 10 8 × 10 −6
3
) a) Hallar la carga en cada condensador en función
del tiempo t.
= 20Ω
( )( )
b) ¿Cuál es la intensidad I de corriente en el
X L = 2π fL = 2π 10 3 2 × 10 −3 instante t?
= 12,6Ω

12
Corriente alterna Hugo Medina Guzmán

c) ¿Cuál es la máxima energía U máx almacenada esto ocurra, la propia impedancia tiene que ser
mínima. Como ésta viene dada por:
en los condensadores?
2
d) ¿Cuál es la energía mínima U min almacenada ⎛ 1 ⎞
Z = R12 + ⎜ ωL − ⎟
en los condensadores? ⎝ ωC ⎠
El valor mínimo se obtiene para aquella
frecuencia que haga nula la reactancia:
1
ωL − =0
ωC
Con lo que Z = R1 .
Con ello el circuito queda reducido a dos
resistencias en paralelo.
Si llamamos I1 a la que circula por la impedancia
Solución.
(y, por tanto, por la bobina y por el condensador),
a) Condensador izquierdo
vale:
Q1 = 3 × 10 −6 (24 + 20 cos120π t ) C V
Condensador derecho I1 = .
I1
Q2 = 1,5 × 10 −6 (24 + 20 cos120π t ) C
VALORES MEDIOS Y EFICACES.
b) I = −4,5 × 10
−6
× 20 × 120π sen120π t Dado que la función seno oscila simétricamente
alrededor de cero, el valor promedio de una
⎛ π⎞
= 10 × 800π cos⎜120π t +
⎟ magnitud que varía sinusoidalmente, por ejemplo,
⎝ 2⎠ la corriente o el voltaje, es cero. Esto sucede así
π⎞ porque para cualquier instante en que la función

= 2,5 × 10 cos⎜120π t + ⎟ A
4
posee un valor positivo, existe otro instante dentro
⎝ 2⎠ del mismo ciclo para el que la función toma el
π⎞ mismo valor negativo.

c) Con cos⎜120π t + ⎟ = 1
⎝ 2⎠
1
U máx = 4,5 × 10 −6 × 44 2
2
−3
= 4,356 × 10 J
⎛ π⎞
Con cos⎜120π t + ⎟ = −1
⎝ 2⎠
1 Sin embargo, el valor promedio del cuadrado de
U min = 4,5 × 10 −6 × 4 2 una magnitud que varía sinusoidalmente no es
2
−5 cero, puesto que el cuadrado de la función seno es
= 3,6 × 10 J siempre positivo y oscila simétricamente
alrededor de + 1/2 .
Ejemplo 18. En el circuito de la figura, el
generador tiene una frecuencia tal que la corriente
que atraviesa la resistencia R es mínima. ¿Qué
corriente atraviesa entonces la bobina y el
condensador?

Para cualquier instante en que sen ω t tiene un


2

valor mayor de + 1/2, hay otro instante en el ciclo


para el que su valor es menor que + 1/2 en la
Solución. misma cantidad, de forma que el promedio de
Si la intensidad que circula por R tiene que ser sen 2ω t es + 1/2.
mínima, es porque la que circula por la
impedancia de la otra rama es máxima, y para que

13
Corriente alterna Hugo Medina Guzmán

Valor cuadrático medio. Para cualquier señal como para que la dependencia temporal de estos
periódica f (t ) , la señal al cuadrado es f (t ) y intercambios de energía sea de interés, por lo que
2
estaremos interesados principalmente en la
la señal “media al cuadrado” es la integral de
potencia promedio, donde este promedio se
f 2 (t ) en el período entero, dividido por el realiza sobre un gran número de ciclos. Por
período. simplicidad continuaremos suponiendo que toda
Para f (t ) = Vmáx senω t es. la capacidad del circuito está contenida en el
condensador, toda su inductancia en la
inductancia y toda su resistencia en la resistencia.
1 T Considerar los intercambios de energía entre los
Vrms = ∫ sen 2ωtdt
2
Vmáx cuatro componentes del circuito:
T 0
1. La fuente convierte otro tipo de energía en
energía electromagnética, y proporciona esta
energía al circuito.
1 1
∫ 2 (1 − cos 2ωt )dt
T
= Vmáx 2. La resistencia disipa la energía
T 0 electromagnética en forma de calor. La energía
abandona el circuito a través del calentamiento de
2
la resistencia, con una rapidez I R .
1 ⎛ T ⎞ Vmáx 3. En un instante dado, la energía puede estar
= Vmáx ⎜ ⎟=
T ⎝2⎠ 2 entrando o saliendo del condensador, dependiendo
de si en ese momento el condensador se carga o se
descarga. Puesto que la corriente oscila
Vmáx sinusoidalmente, la energía que entra en el
Vrms = condensador durante la parte del ciclo
2
correspondiente a la carga del condensador, es
igual a la energía que sale de él durante la parte de
De igual forma para la corriente: ciclo correspondiente a su descarga. En
consecuencia, la potencia promedio en el
I máx condensador es cero.
I rms = 4. Como ocurre con el condensador, la
2 inductancia es un dispositivo de almacenamiento
de energía. La potencia promedio en la
Generalmente, cuando se especifica un valor de inductancia es cero.
voltaje alterno o de corriente alterna, se trata del Considerando el circuito completo, la energía
valor cuadrático medio (rms), y los voltímetros y entra por la fuente y sale por la resistencia.
amperímetros de corriente alterna están calibrados
para indicar estos valores. Por ejemplo, el voltaje Potencia instantánea. Es la potencia en función
entre los dos terminales de un enchufe de su casa del tiempo la que nos permite obtener la potencia
tiene un valor nominal de 220 V, y como se en determinado momento.
muestra en la figura abajo, este es el valor
cuadrático medio del voltaje. La amplitud de este
P = vi = Vsenωt sen (ωt + φ )
voltaje es Vmáx = 311 V.

P = vi = VmáxsenωtI máxsen (ωt + φ )

P = Vmáx I máxsenωt (senωt cos φ + cos ωtsenφ )


(
P = Vmáx I máx sen 2ωt cos φ + cos ωtsenωtsenφ )

Potencia media. Para obtener la potencia media


POTENCIA EN CORRIENTE ALTERNA. realizamos la integración de la potencia
Estudiaremos ahora la rapidez con que se instantánea en un ciclo, es decir durante un
intercambian energía los diferentes componentes periodo (T):
de un circuito RLC conectado a una fuente de
corriente alterna, es decir, el ritmo con que la
1 T
T ∫0
energía entra y sale de cada uno de los Pm = Vmáx I máx sen 2ωt cos φdt
componentes del circuito. Generalmente, la
frecuencia de la fuente es suficientemente alta

14
Corriente alterna Hugo Medina Guzmán

1 T Z R
T ∫0
+ Vmáx I máx senωt cos ωt cos φdt ⇒ cos φ = = ,
4R Z
R 1
⇒ Z = 2R ⇒ =
1 T 1 Z 2

T 0
sen 2ωt cos φdt = cos φ
2 Por tanto
R 1
cos φ = = ⇒ φ = 60º
1 T Z 2
T ∫0
senωt cos ωt cos φdt = 0
Ejemplo 21. Mediante la red eléctrica ordinaria
de 220 V (eficaces) a 50 Hz, se alimenta un
Vmáx I máx circuito RLC con una R = 20 Ω , L= 0,02 H y C =
Pm = cos φ .
2 20 μ F. Calcular:
cos φ es el factor de potencia del circuito.
a) la potencia media disipada por el circuito
Esto también podemos escribir como:
b) deducir si se encuentra o no en resonancia.
Solución.
⎛ V ⎞⎛ I ⎞
Pm = ⎜ máx ⎟⎜ máx ⎟ cos φ = Vrms I rms cos φ a) X L = Lω = 2πfL = 2πΩ ;
⎝ 2 ⎠⎝ 2 ⎠
1 10 3
XC = = Ω
También se les llama voltaje y corriente efectiva Cω 2π
Z = R 2 + (X L − X C )
en vez de voltaje y corriente cuadrática media. 2

Ejemplo 19. Una corriente sinusoidal facilita 20 2


A eficaces bajo la tensión eficaz de 100 V a un ⎛ 103 ⎞
circuito; el ángulo de fase de la corriente sobre la
= 20 + ⎜ 2π −

2
⎟⎟ = 154,2Ω
⎝ 2π ⎠
fuerza electromotriz es de 30º. Determínese la
impedancia, la reactancia y la resistencia del 2
V R ⎛V ⎞
circuito. P = Ve Le cos ϕ = Ve ⋅ e ⋅ = ⎜ e ⎟ R
Solución. Z Z ⎝Z⎠
La impedancia vale 2
Vm 100 ⎛ 220 ⎞
=⎜ ⎟ 20 = 40,7 W
Z= = ⎝ 154´2 ⎠
Im 20
b) Si cuando X L = X C está en resonancia.
= 5Ω
La resistencia: Podemos ver que no son iguales, por lo tanto no
está en resonancia
R = Z cos φ = 5 cos 30º
= 4,3Ω Ejemplo 22. Los cinco parámetros que describen
La reactancia: un circuito RLC conectado a una fuente de
X = Zsenφ = 5sen30º corriente alterna son R, L C, Vmáx y ω . Obtener la
expresión de Pm en función de estos parámetros.
= 2,5Ω Solución.
Vrms
Ejemplo 20. Calcúlese el ángulo de fase de una Sustituyendo la expresión I rms = en la
corriente alterna cuya potencia viene dada por Z
Ecuación Pm = (I rms ) R obtenemos
2
V2
P=
4R ⎛V ⎞
2 2
Vrms R
Solución. Pm = ⎜ rms ⎟ R = 2
R + (X C − X L )
2
V2 V ⎝ Z ⎠
P= = VI cos φ , ⇒ cos φ = y sustituyendo ahora Vrms , X C y X L por sus
4R 4 RI
Teniendo en cuenta correspondientes expresiones se obtiene
V R
I= , y cos φ = :
R Z

15
Corriente alterna Hugo Medina Guzmán

1 2
Vmáx R
Pm = 2
2
⎛ 1 ⎞
R2 + ⎜ − ωL ⎟
⎝ ωC ⎠
Observar que cuando la fuente tiene la frecuencia
1
de resonancia, = ωL , la potencia promedio
ωC
1 2
es Vmáx R .
2 Solución.
a) X L = 2π fL = 2π (60 )(0,3)
Ejemplo 23. Una bobina de resistencia y de
inductancia desconocidas conduce 4 A cuando = 113Ω
está conectada con una fuente de 1 2 V de 1 1
XC =
2π (60)(50 × 10 −6 )
=
corriente continua y 3 A cuando está conectada a 2π fC
una fuente de 12 V de corriente alterna, 100 Hz.
a) Encuentre los valores de R y de L. = 53Ω
b) ¿Qué potencia se disipa cuando la bobina está
Z = R 2 + (X L − X C )
2
conectada con la fuente de corriente continua?
80 2 + (113 − 53)
c) ¿Cuándo está conectada con la fuente de 2
corriente alterna? =
Solución. = 100Ω
a) No hay reactancia inductiva cuando la corriente V 120
continua pasa a través de la bobina, así que su b) I = = = 1,2 A
resistencia es Z 100
V1 12 R 80
R= = = 3Ω c) cos φ = = = 0,8
I1 4 Z 100
= 80%
Con f = 100 Hz la impedancia del circuito es
d) La potencia real
V2 12 P = VI cos φ
Z= = = 4Ω
I2 3 = (120 )(1,2 )(0,8) = 115 W
R 2 + (X L − X C ) y Alternativamente,
2
Y como Z =
P = I 2 R = (1,2) (80) = 115 W
2
X C = 0 , tenemos:
e) La potencia aparente
X L = Z − R = 4 − 3 = 2,65Ω
2 2 2 2
= VI = (120 )(1,2 ) = 144 VA
Luego la inductancia de la bobina es
XL 2,65 Ejemplo 25. a) Encuentre las diferencias de
L= = = 4,22 mH potenciales a través de la resistencia, de la
2π f 2π (100) inductancia, y del condensador en el circuito
P1 = I 12 R = (4) (3) = 48 W anterior
2
b)
b) ¿Estos valores están de acuerdo con la
P2 = I 22 R = (3) (3) = 27 W
2
c) diferencia potencial aplicada de 120 V?
Solución.
Ejemplo 24. Un condensador de 50 μ F, una a) V R = IR = (1,2 )(80 )
inductancia de 0,3 H, y una resistencia de 80 Ω = 96 V.
están conectados en serie con una fuente de 120 V L = IX L = (1,2 )(113)
V, 60 Hz. = 136 V.
a) ¿Cuál es la impedancia del circuito?
b) ¿Cuánta corriente fluye en el?
VC = IX C = (1,2)(53)
c) ¿Cuál es el factor de potencia? = 64 V.
d) ¿Cuánta potencia es disipada por el circuito? c) La suma de estas diferencias potenciales es
e) ¿Cuál debe ser la mínima potencia en 296 V, más de dos veces los 120 V aplicados
voltamperios de la fuente de energía? al circuito. Sin embargo, esto es una manera
sin sentido de combinar las diferencias de
potenciales puesto que no están en fase el

16
Corriente alterna Hugo Medina Guzmán

uno con el otro: V L está 90° delante de V R b) ¿Están estos valores de acuerdo con la
diferencia potencial aplicada de 120 V?
y VC está 90º detrás de V R . La manera Solución.
correcta de encontrar la diferencia potencial a) A la frecuencia de la resonancia de f R = 41
total a través del circuito es como sigue: Hz, las reactancias inductiva y capacitiva son,
d)
respectivamente.
V = VR2 + (VL − VC ) X L = 2π f R L = 2π (41)(0,3)
2

= 77Ω
96 2 + (136 − 64)
2
=
1 1
XC =
2π f R C 2π (41)(50 × 10 −6 )
= 120 V. =
Este resultado está de acuerdo con la diferencia
potencial aplicada de 120 V. Notamos que el = 77Ω
voltaje a través de una inductancia o de un Las diferentes diferencias de potencial son:
condensador en un circuito de corriente alterna
VR = IR = (1,5)(80) = 120 V.
puede ser mayor que el voltaje aplicado al
circuito. V L = IX L = (1,5)(77 ) = 116 V.
VC = IX C = (1,5)(77 ) = 116 V.
Ejemplo 26. a) Encuentre la frecuencia de la
b) La diferencia potencial total a través del
resonancia f R del circuito anterior.
circuito es
b) ¿Qué corriente fluirá en el circuito si está
V = VR2 + (VL − VC )
2
conectado con una fuente de 20 V y frecuencia
fR ?
120 2 + (116 − 116 )
2
=
c) ¿Cuál será el factor de potencia en este caso?
d) ¿Cuánta potencia será disipada por el circuito? = 120 V.
e) ¿Qué debe ser la potencia mínima en La cuál es igual a la diferencia de potencial
voltamperios de la fuente ahora? aplicada.
Solución.
1 Ejemplo 28. Un motor eléctrico 5 hp tiene una
a) f R = eficiencia de 80 por ciento y un factor inductivo
2π f LC de potencia de 75 por ciento.
1 a) ¿Cuál es la potencia mínima en
= kilovoltamperes que su fuente debe tener?
2π (0,3)(50 × 10 −6 ) b) Un condensador está conectado en serie con el
= 41 Hz. motor para levantar el factor de potencia a 100 por
b) Con la frecuencia de resonancia ciento. ¿Qué potencia mínima en
kilovoltamperes debe tener la fuente ahora?
XL = XC y Z = R , Solución.
luego a) La potencia requerida por el motor es

I=
V 120
= = 1,5 A P=
(5)(0,746 kW hp )
R 80 0,8
R R = 4,66 kW
c) cos φ = = =1
Z R Desde que P = IV cos φ , la fuente de energía
= 100% debe tener la potencia mínima
d) La potencia real = P = VI cos φ P 4,66kW
IV = =
= (120 )(1,5)(1) cos φ 0,75
= 180 W = 6,22 kVA
e) La potencia aparente = VI b) Cuando cos φ = 1 ,
= (1290)(15) IV = P = 4,66 kVA.
= 180 VA
Ejemplo 29. ¿Una bobina conectada a 120 V, 25
Ejemplo 27. a) Encuentre la diferencia potencial Hz conduce una corriente de 0,5 A y disipa 50 W.
a través de la resistencia, de la inductancia, y del a) que cuál es su factor de potencia?
condensador en el circuito anterior cuando está b) ¿Qué condensador se debe conectar en serie
conectado con una fuente de la corriente alterna con la bobina para aumentar el factor de potencia
de 120 V con frecuencia igual a la frecuencia de 100 por ciento?
resonancia del circuito, 41 Hz. c) ¿Cuál sería la corriente en el circuito entonces?

17
Corriente alterna Hugo Medina Guzmán

d) ¿Qué potencia disiparía el circuito? V 220


Solución. Z= = = 3385Ω ,
a) Como P = IV cos φ ,
I 0,065
P 50 X C = Z 2 − R 2 = 3385 2 − 769 2
cos φ = =
IV (0,5)(120) = 3296Ω
= 0,833 = 83,3 % Luego el condensador necesario debe tener una
capacidad de:
b) El factor de potencia será 100 por ciento en la 1 1
C= = = 0,966 x 10-6 F
resonancia, cuando X L = X C . El primer paso ωX C (100π )(3296)
es encontrar X L , que se puede hacer de = 0,966 μF
(X L − X C )
tan φ = . Ejemplo 31. Se dispone de un generador de 44,4
R V y 50 Hz del que se quiere obtener una corriente
Aquí X C = 0 , y como cos φ = 0,833 , de 10 A y 313,9 W. Para ello se monta sobre sus
bornes una autoinductancia.
φ = 34º y tan φ = 0,663 . Halle sus características.
Siendo P = I R :
2
Solución.
P 50 El factor de potencia es:
R= 2 = = 200Ω P 313,9
I 0,5 2 cos φ = = = 0,707
VI (44,4)(10)
Luego X L = R tan φ + X C
⇒ φ = 45º ,
= (200 )(0,663) + 0
X
= 133Ω Como tan φ = , se tiene:
R
Éste también debe ser el valor de X
XC I = , de donde resulta que la autoinducción
cuando f = 25 Hz, luego R
1 1 necesaria ha de tener iguales su resistencia R y su
C= = reactancia X, es decir:
2π fX C 2π (25(133)) R = ωL , por lo tanto
= 4,8 x 10-5 f
= 48 μ F Z = R 2 + X 2 = 2R
De donde:
c) En la resonancia Z = R, luego
Z V I 44,4 10
V 120 R= = = = 3,14Ω
I= = = 0,6 A 2 2 2
Z 200
Por último, la autoinductancia vale:
d) P = I R = (0,6 ) (200)
2 2
R 3,14
= 72 W L= = = 0,01 H
ω 100π
Ejemplo 30. Mediante la red alterna de 220 V y
50 Hz se quiere encender normalmente un foco de Ejemplo 32. Un circuito serie RLC está formado
50 V, 3,25 W, conectándole un condensador en por una bobina de coeficiente de autoinducción
serie. ¿Qué capacidad debe tener para ello? L= 1 H y resistencia óhmica interna de 10 Ω , un
Solución. condensador de capacidad C = 5 μ F, y una
En las condiciones normales de funcionamiento el resistencia de 90 Ω . La frecuencia de la corriente
foco debe estar recorrido por una corriente: es de 100 Hz. Si el circuito se conecta a un
W 3,25 generador de corriente alterna de 220 V de tensión
I= = = 0,065 A máxima, calcular:
V 50
Y su resistencia es:
a) la potencia disipada por el circuito
V 50
R= = = 769Ω
I 0,065 b) la expresión de la intensidad instantánea
Por lo tanto, la impedancia total del circuito y su Solución.
reactancia capacitiva serán:

18
Corriente alterna Hugo Medina Guzmán

a) X L = Lω = 1(2π 100 ) = 628,3Ω


1 1
XC = = = 318,3Ω
Cω 5 × 10 (2π 100)
−6

Z = R 2 + (X L − X C )
2

90 2 + (628,3 − 318,3) = 225,7Ω


2
=
220
Ve = = 155,6 V ;
2
2
V R ⎛V ⎞
P = Ve I e cos ϕ = Ve e = ⎜ e ⎟ R cos φ = 0,8 ⇒ φ = 36,87º
Z Z ⎝Z⎠
2 IX 1 1
⎛ 155,6 ⎞ tan 36,87 º = 0,75 = C = =
=⎜ ⎟ 90 = 22,8 W IR ωCR 100πCR
⎝ 225,7 ⎠ 1
b) ⇒ CR =
75π
Por otra parte P = VI cos φ ⇒
40 1
40 = 220 I (0,8) ⇒ I = =
220(0,8) 4,4
V = I R 2 + X C2 ⇒
2
XL − XC 1 ⎛ 1 ⎞
tan ϕ = = 3,1 ; 220 = R2 + ⎜ ⎟
R 4,4 ⎝ 100πRC ⎠
2
ϕ = arctan 3,1 = 72º12’ = 1,26 rad. ⎛ 1 ⎞
R +⎜
2
⎟ = 968
2

⎝ 100πRC ⎠
⎡V (t ) = 220sen 200πt ⎤ ⎛ 75 ⎞
2

⎢ I (t ) = 0,68sen (200πt − 1,26)⎥ R +⎜


2
⎟ = 968 ⇒ R = 967
2

⎣ ⎦ ⎝ 100 ⎠
⎡V (t ) = 220sen (200πt + 1,26)⎤ El condensador es:
o ⎢ ⎥ 1 1
⎣ I (t ) = 220sen 200πt ⎦ C= = = 4,39 x 10-6 F
75πR 75π (967 )
Ejemplo 33. Un tubo fluorescente está instalado = 4,39 μF
en una línea de 220V y 50Hz y consume 40W con La potencia es:
un factor de potencia de 0,8. Determinar la
capacidad del condensador C que tenemos que ⎛ 1 ⎞
P = VI cos φ = 220⎜ ⎟0,8 = 40 W
conectar: ⎝ 4,4 ⎠
a) En serie.
b) En paralelo:
b) En paralelo.
c) Para que el factor de potencia sea 1. ¿Cuál
sería, en cada caso, la potencia media Pm que
consume el nuevo circuito?.
Solución.
a) En serie:

cos φ = 0,8 ⇒ φ = 36,87º

19
Corriente alterna Hugo Medina Guzmán

VC El flujo magnético que atraviesa la bobina vale:


tan 36,87º = 0,75 = Φ = nBAsenω t ,
VR
Y, por lo tanto, la fuerza electromotriz
IC 1 instantánea es:
= =
I RωCR 100πCR dΦ
ε =− = nBA cos ω t
1 1 dt
⇒ CR = ⇒ R= Con lo que la fuerza electromotriz eficaz es:
75π 75πC
2 2 V =
nBAω
=
( )
1700(0,18) 25 × 10 −4 (20π )
⎛ 220 ⎞ ⎛ 220 ⎞ 2 2
I = I R2 + I C2 = ⎜ ⎟ +⎜ ⎟
⎝ R ⎠ ⎜⎝ X C ⎟⎠ = 34 V.
La reactancia inductiva del circuito es:
= 220
1
+ (100πC )
2 X L = ωL = 20π (0,02 ) = 1,257Ω
2
R Y como la resistencia vale
Reemplazando el valor de R R = 0,7 + 2,5 = 3,2Ω , la impedancia es:
I = 220 (75πC ) + (100πC )
2 2
Z = R 2 + X L2 = 3,2 2 + 1,257 2
220(25π ) (3C ) + (4C ) = 3,44Ω
2 2
=
= 220(25π )(5C ) = 86350C
Y la intensidad:
V 34
Por otra parte Pm = VI cos φ I= = = 9,88 A.
Z 3,44
⇒ 40 = 220(86350C )(0,8)
El condensador es: El factor de potencia es:
40 R 3,2
C= = 2,63 x 10-6 F cos φ = = = 0,93
15197600 Z 3,44
= 2,63 μ F y la potencia:
c) Para que el factor de potencia sea 1. P = VI cos φ = (34 )(9,88)0,93 = 312 W.
En serie.
cos φ = 1 ⇒ φ = 0º Ejemplo 35. La figura muestra un circuito CA en
IX serie. La inductancia tiene una reactancia de 70
tan 0º = 0 = C ohmios y una inductancia de 200 mH. Una
IR resistencia de 50 ohmios y un condensador cuya
1 reactancia es 100 ohmios, están también en el
⇒ X C = 0 CR = circuito. La corriente rms en el circuito es 1,2 A.
75π
Solamente hay resistencia
V 220
I= =
R 967
= 0,297Ω
Por otra parte Pm = VI cos φ
⇒ Pm = 220(0,297 ) = 50,05 W
En paralelo.
El resultado es igual que en el caso anterior.
a) ¿Cuál es el voltaje rms de la fuente?
Ejemplo 34. Dentro de un campo magnético de b) ¿Cuál es la capacidad del condensador?
1800 gauss gira una bobina de 1700 espiras de 25 c) ¿Cuál es el ángulo de fase del circuito?
cm2 a razón de 600 rpm. Si la resistencia de su d) ¿Cuál es la amplitud del voltaje a través del
bobinado es de 0,7 Ω y sus bornes se conectan a condensador?
una autoinducción de 0,02 H y 2,5 Ω , ¿Cuál es la e) ¿Cuál es la energía magnética máxima en la
potencia de la corriente que la recorre? inductancia?
Solución. Solución.
= I rms R 2 + ( X C − X L )
2
⎛ 600 ⎞ a) Vrms
ω = 2π ⎜ ⎟ = 20π rad/s.
⎝ 60 ⎠

20
Corriente alterna Hugo Medina Guzmán

= 1,2 50 + (100 − 70 )
2 2
Ejemplo 37. Un circuito en serie tiene una fuente
= 1,2(58,31) = 69,87 V de CA de 50 hertzios, una resistencia de 30
ohmios, una inductancia de 0,20 H, y un
X L 70 rad condensador de 80 μ F, como se muestra en la
b) X L = ωL ⇒ω = = = 350
L 0,2 s figura. La corriente rms en el circuito es 2,5 A.
1
XC = ⇒
ωC
1 1 −6
C= = = 28 × 10 F
ωX C 350(100)
X − X C 70 − 100 − 30
c) tan φ = L = =
R 50 50
⇒ φ = −30,96º
( )
d) VC = IX C = 1,2 2 (100 ) = 169,2 V
a) ¿Cuál es la amplitud del voltaje de la fuente?
b) ¿Cuál es el factor de potencia del circuito?
e) U =
1 2
2
1
2
(
LI rms = 200 × 10 −3 (1,2)
2
) c) ¿Cuál es el voltaje rms a través de los puntos a
y b en el circuito?
d) La frecuencia de la fuente se cambia de modo
= 0,144 J
que la reactancia capacitiva sea igual a dos veces
la reactancia inductiva. Se conservan los
Ejemplo 36. Cuando se suspende una masa de un
elementos de circuito originales. ¿Cuál es la
resorte y se sumerge en líquido puede
nueva frecuencia de la fuente?
experimentar oscilaciones armónicas
e) Si se cambia la capacidad de modo que el
amortiguadas forzadas. El comportamiento es
circuito esté en resonancia. Se conservan la
análogo al de un circuito RLC en serie al cual se
resistencia y la inductancia originales. El voltaje y
aplica una fuerza electromotriz sinusoidal. ¿A
la frecuencia de la fuente se mantienen en los
cuál de las siguientes energías cinéticas es
valores originales. ¿Cuál es la nueva capacidad?
análoga la energía de la masa?
Solución.
A) La energía disipada como calor en la
= I R 2 + (X C − X L )
2
resistencia. a) V
B) La energía almacenada en el condensador.
C) La energía magnética almacenada en la I = 2,5 2 = 3,52 A, R = 30Ω
inductancia. 1 1
XC = =
( )
D) La energía total proporcionada por la fuente de
la fuerza electromotriz. ωC (100π ) 80 × 10 −6
E) La corriente en el circuito. = 39,8Ω ,
X L = ωL = (100π )(0,2 )
Solución.
La ecuación diferencial cuando se suspende una
masa de un resorte y se sumerge en líquido puede = 62,8Ω
experimentar oscilaciones armónicas
V = 3,52 30 2 + (39,8 − 62,8)
2
amortiguadas forzadas es
•• b • k F = 133,06 V
x+ x + x = 0 senω ' t
m m m XC − XL 39,8 − 62,8
b) tan φ = =
Para un circuito RLC en serie al cual se aplica R 30
una fuerza electromotriz sinusoidal es = -0,77
••R• 1 ε ⇒ φ = −37,5°
I+ I+ I = 0 senωt
L LC L cos φ = cos− 37,5° = 0,79
1 2
La energía almacenada en la masa es mv
2 c) V(ab )rms = I rms R 2 + X L2
La energía almacenada en la inductancia es
= (2,5) 30 + 62,8 2
2
1 2
LI
2 = 174 V
Por observación la energía cinética de la masa es
análoga a la energía magnética almacenada en la
inductancia (C).

21
Corriente alterna Hugo Medina Guzmán

1
d) X ' C = 2 X 'L ⇒ = 2ω ' L IR IR
ω'C d) cos φ = =
1 1 I I R2 + I L2
⇒ ω' =
2 LC
=
(
2(0,2) 80 × 10 −6 ) =
0,26
= 0,46
rad 0,26 2 + 0,5 2
= 177
s
ω ' 177 ⇒ φ = 62°
f '= = = 28,15 Hz
2π 2π
1
e) ω=
1 LC
e) Para resonancia: ω=
LC ' 1 1
⇒ C= 2 =
1 1 ω L (200π )2 (0,25)
⇒ C' = 2 =
ω L (100π )2 (0,20 ) −6
= 10,14 × 10 F
−6
= 50,7 × 10 F
Ejemplo 39. ¿Cuál de los diagramas de fasores
representa un circuito RLC en serie en
Ejemplo 38. Una resistencia de 300 ohmios y una resonancia?
inductancia de 250 mH están conectadas en
paralelo a través de una fuente de la CA de 100
hertzios, como se muestra en la figura. La
corriente rms a través de la inductancia es 0,50A.

Solución.
Es el diagrama 3 porque V L y VC tienen la
misma magnitud y son opuestos.

a) ¿Cuál es la corriente rms a través de la TRANSFORMADOR IDEAL.


resistencia? Con anterioridad se advirtió que al variar la
b) ¿Cuál es el voltaje rms de la fuente? corriente en una espira de a1a, inducirá una
c) ¿Cuál es la impedancia del circuito? corriente en una espira cercana. La corriente
d) ¿Cuál es el ángulo por cuál el voltaje se inducida se origina en la variación del campo
adelanta a la corriente (negativa si se retrasa)? magnético asociado con la variación en la
e) para que el circuito esté en resonancia, se corriente cuando es alterna, presenta una ventaja
conecta un condensador en paralelo a través de la distinta de la corriente continua debido al efecto
fuente de la CA. ¿Cuál es la capacidad de este inductivo de la corriente que en forma constante
condensador? varia en magnitud y dirección. La aplicación más
Solución. común de este principio la ofrece el
Vrms 78,5 transformador, un dispositivo que incrementa o
a) I Rrms = = = 0,26 A disminuye el voltaje en un circuito de corriente
R 300 alterna.

Vrms
b) I Lrms =
XL
⇒ Vrms = I Lrms X L = (0,5)(200π )(0,250 )
= 78,5 V
Un transformador sencillo se muestra en la figura
1 1 1 siguiente.
c) = 2
+ 2
Z R XL
1 1
= +
300 2
[(200π )0,25]2
= 139,12Ω Hay tres partes esenciales:

22
Corriente alterna Hugo Medina Guzmán

1) Una bobina primaria conectada a una fuente de Potencia de entrada ε S I S ,


Corriente alterna. E= =
Potencia de salida εPIP
2) Una secundaria y
3) Un núcleo laminado de hierro dulce. donde I P e I S son las corrientes en las bobinas
Conforme se aplica una corriente alterna en el del primario y secundario, respectivamente. La
devanado primario, las líneas del flujo magnético mayor parte de los transformadores eléctricos son
varían a través del núcleo de hierro de tal modo diseñados cuidadosamente para obtener
que se induce una corriente a1terna en el rendimientos muy altos, generalmente por arriba
devanado secundario. 90070.
El flujo magnético que varía constantemente se Es importante reconocer que no se obtiene una
establece en todo el núcleo del transformador y ganancia de potencia como resultado de la acción
pasa a través de las bobinas del primario y de un transformador. Cuando el voltaje se eleva,
secundario. La fuerza electromotriz ε P , inducida la corriente debe ser reducida para que el producto
en la bobina del primario es εI no se incremente. Para ver esto claramente
ΔΦ supóngase que cierto transformador tiene un
ε P = −n P rendimiento del l00 % , para este transformador
Δt
εS IS
donde n P = número de vueltas en el primario ideal, la ecuación E = se reduce a
εPIP
ΔΦ
= rapidez con que cambia el flujo IP εS
Δt εS IS = ε PIP o =
magnético. IS ε P
En forma análoga la fuerza electromotriz εS Esta ecuación demuestra claramente una relación
inducida en la bobina del secundario es inversa entre la corriente y el voltaje inducido.
ΔΦ
ε S = −n S Ejemplo 40. Un generador de corriente alterna
Δt que suministra 20 A a 6000 V se conecta aun
donde n S es el número de vueltas en el transformador. ¿Cuál es la corriente de salida a
120000 V si el rendimiento del transformador
secundario. Puesto que el mismo cambio de flujo 100%?
se produce con la misma rapidez a través de cada
bobina, de las dos ecuaciones anteriores
obtenemos:

El voltaje inducido está en proporción directa con


el número de vueltas. Si se varía la razón de las
vueltas del secundario N, respecto a las vueltas
del primario N, un voltaje de entrada (primario)
puede suministrar cualquier voltaje de salida
deseado (secundario). Por ejemplo, si hay 40
veces más vueltas en la bobina del secundario, un
voltaje de entrada de 120V incrementará el voltaje
de salida en la bobina del secundario a 40 x 120
V = 4800 V. El transformador que produce un
voltaje de salida mayor se llama transformador Solución.
elevador. Es posible construir un transformador
reductor si el número de vueltas en el primario es De
IP εS
=
ε I
⇒ IS = P P =
(6000)(20)
mayor que el número de vueltas en el secundario. IS ε P εS (120000)
Si se emplea un transformador reductor el =1A
resultado que se obtiene es un voltaje de salida Nótese que en el ejemplo precedente la corriente
menor. se redujo a 1 A en tanto que el voltaje aumentó 20
El rendimiento de un transformador se define veces. Ya que las pérdidas de calor en las líneas
como la razón de la potencia de salida respecto a de transmisión varían directamente con el
la potencia de entrada. Si se recuerda que la
potencia eléctrica es igual al producto del voltaje
( )2
cuadrado de la corriente I R , esto significa
y la corriente, el rendimiento E de un que la potencia eléctrica puede transmitirse a
transformador puede escribirse como grandes distancias sin que haya pérdidas

23
Corriente alterna Hugo Medina Guzmán

significativas. Cuando la potencia eléctrica llega a Utilicemos los subíndices 1 para el amplificador y
su destino se utilizan transformadores reductores 2 para la bobina de la voz. Para la transferencia
para suministrar la corriente deseada a voltajes máxima de potencia.
menores. n1 Z1 7350
= = = 1225 = 35
Ejemplo 41. En un autotransformador se utiliza n2 Z2 6
una sola bobina para el primario y secundario. En Así la relación de vueltas es 35:1, y la bobina del
la bobina mostrada aquí hay tres puntos unidos a transformador conectada con el amplificador debe
una bobina envuelta en un núcleo de hierro. Dos tener 35 veces tantas vueltas como la bobina
puntos cualesquiera se pueden utilizar como el conectada con la bobina de la voz.
primario, y dos cualesquiera se pueden utilizar
como el secundario. Entre A y B hay 300 vueltas, Ejemplo 44. Un transformador de baja recibe en
y entre B y C hay 900 vueltas. ¿Qué voltajes de su circuito primario una corriente alterna con una
salida son posibles para un voltaje de entrada intensidad eficaz de 10 A, bajo una tensión de 500
primario de 120 VAC? V. La relación de transformación es 5 y además
de él hay una pérdida de energía del 10 %.
Calcúlese la intensidad eficaz de corriente que se
produce en el secundario en régimen normal.
Solución.
La tensión del secundario será:
500
Solución. VS = = 100 V.
Los cocientes posibles de las vueltas son 300:900, 5
300:1200, 900:1200, y los recíprocos éstos. Teniendo en cuenta las pérdidas de potencia:
Por lo tanto los cocientes posibles de las vueltas PP (0,9) = PS ⇒ V P I P (0,9) = VS I S
V P I P (0,9 )
son 1:3, 1:4, 3:4, 3:1, 4:1, y 4:3. 1:1 también es
posible. ⇒ IS =
Por lo tanto los voltajes secundarios posibles son VS
30, 40, 90, 120, 160, 360, y 480 VAC.
n1 IS =
(500)(10)(0,9) = 45 A.
Ejemplo 42. Derive la relación de para la (100)
n2
transferencia máxima de potencia. Ejemplo 45. Un transformador reductor ha de
Solución. alimentar una instalación de 2500 focos de 125 V,
V1 n1 I n 0,5 A, tomando la corriente de una red de alta de
= y 2 = 1 12500 V. ¿Cuántas espiras han de tener las dos
V2 n 2 I1 n2 bobinas del transformador, si en el primario
V V precisa, por lo menos, de una espira por cada 0,3
Porque Z 1 = 1 y Z 2 = 2 , la relación de mA, y el rendimiento del transformador resulta
I1 I2 ser del 93,3 %?
impedancias es: Solución.
2 La potencia del secundario es:
Z 1 V1 I 2 ⎛ n1 ⎞
= =⎜ ⎟ PS = nVS I S = 2500(125)(0,5)
Z 2 V2 I 1 ⎜⎝ n2 ⎟⎠
= 1,563 x 105 W
Por lo tanto la relación de las vueltas necesarias y, por tanto, la del primario debe ser:
para emparejar las impedancias es
PS 1,563 × 10 5
n1 Z1 PP = =
= 0,933 0,933
n2 Z2 = 1,675 x 105 W
Así, la corriente en el primario será:
Ejemplo 43. Un altoparlante cuya bobina de la PS 1,675 × 10 5
voz tiene una impedancia de 6 Ω debe ser IP = = = 13,4 A.
utilizado con un amplificador cuya impedancia de VP 1,25 × 10 4
carga sea 7350 Ω . Encuentre la relación de y, por tanto, el número de espiras precisas es de:
vueltas en un transformador que empareje estas 13,4
impedancias. nP = = 44659 espiras.
0,3 × 10 −3
Solución.
Las del secundario son:

24
Corriente alterna Hugo Medina Guzmán

VS 125
nS = nP = 44659 = 447 espiras.
VP 12500

PREGUNTAS Y PROBLEMAS

1. La frecuencia de la fuerza electromotriz alterna 8. Un condensador tiene una reactancia de 200 Ω


aplicada a un circuito en serie que tiene a 1000 Hz. ¿Cuál es su capacidad?
resistencia, inductancia, y condensador es Respuesta. 0,796 μ F
duplicada. ¿Qué sucede a R, a X L , y a X C ?
Respuesta. 9. Un condensador de 5 μ F está conectado con
R no cambia, X L se duplica, y X C se hace la una fuerza electromotriz alterna de 6 kHz, y fluye
una corriente de 2 A. Encuentre la magnitud de la
mitad.
fuerza electromotriz eficaz.
Respuesta. 10,6 V
2. a) ¿Cuál es el valor mínimo el factor de
potencia que un circuito puede tener? ¿Bajo qué
10. Un condensador de 5 pF conduce 1 A cuando
circunstancias puede ocurrir esto?
está conectado con una fuente de corriente alterna
b) ¿Cuál es el valor máximo el factor de potencia
de 60 V. ¿Cuál es la frecuencia de la fuente?
que puede tener? ¿Bajo qué circunstancias puede
Respuesta. 531 Hz
ocurrir esto?
11. Un condensador de 25 μ f está conectado en
Respuesta.
a) El valor mínimo es 0, que puede ocurrir
solamente si R = O en un circuito en el cual el que serie con un resistencia de 50 Ω , y a una
X L no sea igual a X C . diferencia potencial de 1 2 V, 60 Hz. Encuentre
la corriente en el circuito y la potencia disipada en
b) El valor máximo es 1, que ocurre en la ella.
resonancia cuando X L = X C . Respuesta. 0,102 A; 0,524 W

3. Un voltímetro a través de un circuito de 12. Un condensador está conectado en serie con


corriente alterna lee 40 V, y un amperímetro en una resistencia de 8 Ω , y la combinación se pone
serie con el circuito lee 6 A. a través de una fuente de energía de 24 V, 1000
a) ¿Cuál es la diferencia potencial máxima a Hz. Fluye una corriente de 2 A. Encuentre
través del circuito? a) la capacidad del condensador,
b) ¿Cuál es la corriente máxima en el circuito? b) el ángulo de la fase, y
c) ¿qué otra información es necesaria para c) la potencia disipada por el circuito.
determinar la potencia en el circuito? Respuesta.
Respuesta. a) 17,8 μ F, b) - 48°, c) 32 W
a) 56,6, V, b) 8,5 A, c) Angulo de fase.
13. Una inductancia de 0,l H, 30 Ω está
4. Encuentre la reactancia de una bobina de 10 conectada con una fuente de 50V, 100Hz.
mH cuando está en un circuito de 500 Hz. Encuentre la corriente en la inductancia y la
Respuesta.31,4 Ω potencia disipada en ella.
Respuesta. 0,72 A; 15,6 W
5. La reactancia de una inductancia es 1000 Ωa
20 Hz. Encuentre su inductancia. 14. La corriente en un resistencia es 1 A cuando
Respuesta. 0,796 H está conectada con una fuente de 50 V, 100 Hz.
a) ¿Qué reactancia inductiva se requiere para
6. Una corriente de 0,20 A atraviesa pasa por una reducir la corriente a 0,5 A? ¿Qué valor de L
inductancia de 0,15 h de resistencia insignificante logrará esto?
que está conectada con una fuente 80 V de b) ¿Qué reactancia capacitiva se requiere para
corriente alterna. ¿Cuál es la frecuencia de la reducir la corriente a 0,5 A? ¿Qué valor de C
fuente? logrará esto?
Respuesta. 424 Hz c) ¿Cuál será la corriente si la inductancia y el
condensador antedichos se ponen en serie con la
7. Encuentre la reactancia de un condensador de 5 resistencia?
μ F a 10 Hz y a 10 kHz. Respuesta.
Respuesta. 3183 Ω ; 3,183 Ω . a) 86,6 Ω ; 137,8 mH b) 86,6 Ω ; 18,4 μ F e) 1
A

25
Corriente alterna Hugo Medina Guzmán

c) ¿Qué inductancia se debe conectar en serie en


15. Una resistencia pura, un condensador puro, y el circuito para aumentar el factor de la potencia a
una inductancia pura están conectados en serie a 100 por ciento?
través de una fuente de corriente alterna. ¿Un d) ¿Cuál sería la corriente?
voltímetro de corriente alterna colocado a través e) ¿Qué potencia disiparía el circuito?
de cada uno de estos elementos del circuito Respuesta.
alternadamente lee 10, 20, y 30 V. ¿Cuál es la a) 800 W b) 80 % c) 30 mH d) 5 A e) 1250 W
diferencia potencial de la fuente?
Respuesta. 14 V 21. Calcular la potencia W que consume el aparato
A en función de las tensiones eficaces V1, V2, V3 y
16. Un circuito inductivo con un factor de del valor de la resistencia R.
potencia de 80 por ciento consume 750 W de
energía. ¿Cuál es la mínima potencia en
voltamperios de la fuente?
Respuesta. 938 VA

17. Se conectan en serie a una tensión de corriente Respuesta.


alterna de 60Hz una resistencia R y una bobina de
V32 + V12 + V22
1,4 H. La tensión eficaz de la resistencia es 30 V y W =
en la bobina 40 V. 2R
a) ¿Cuánto vale la resistencia? 22. En un circuito en serie R = 100 Ω, XL =
b) ¿Cuál es la tensión eficaz V entre la entrada y la
salida del montaje? 120 Ω , y X C = 60 Ω cuando está conectado
Respuesta. con una fuente de la corriente alterna de 80 V.
a) R = 396Ω , b) V = 50 V Encuentre
a) la corriente en el circuito,
18. Dado el circuito de la figura: b) el ángulo de la fase, y
a) Hallar la perdida de potencia en la bobina. c) la corriente si la frecuencia de la fuente de
b) Hallar la resistencia r de la bobina. energía se cambiara a la frecuencia de resonancia
c) Hallar la inductancia L. del circuito.
Respuesta.
a) 0,69 A b) 31° c) 0,80 A

23. Un condensador de 10 μ f, una inductancia de


10 mH, y una resistencia de 10 Ω se conectan en
serie con una fuente de 45 V, 400 Hz. Hallar
a) La impedancia del circuito.
Respuesta. b) La corriente,
a) 15 W, b) r = 15Ω , c) L = 0,235 h c) La potencia que disipa.
d) La mínima potencia en voltamperios de la
fuente.
19. Una carga inductiva disipa 75 W de energía
cuando conduce una corriente de 1,0 A de una Respuesta. a) 18 Ω , b) 2,5 A, c) 62,5 W, d)
línea de energía de 120 V, 60 Hz. 112,5 VA
a) ¿Cuál es el factor de potencia?
b) ¿Qué condensador se debe conectar en serie en 24. a) Encuentre la frecuencia de resonancia f R
el circuito para aumentar el factor de potencia a del circuito de problema anterior.
100 por ciento? b) ¿Qué corriente fluirá en el circuito si está
c) ¿Cuál sería la corriente? conectada con una fuente de energía 45 V y que
d) ¿Qué potencia disiparía el circuito? frecuencia sea f R ?
Respuesta. a) 0,625 b) 28,6 μ F c) 1,6 A d)
c) ¿Qué potencia será disipada por el circuito?
192 W d) La mínima potencia en voltamperios de la
fuente.
20. Un circuito que consiste en un condensador en Respuesta.
serie con una resistencia de 50 Ω conduce 4 A a) 503 Hz b) 4,5 A. c) 202,5 W d) 202,5 VA
de una fuente de 250 V, 200 Hz.
a) ¿Qué potencia se disipa? 25. Un condensador de 60 μ F, una inductancia
b) ¿Cuál es el factor de potencia?
de 0,3 H, y una resistencia de 50 Ω se conectan

26
Corriente alterna Hugo Medina Guzmán

en serie con una fuente de 120 V, 60 Hz. 30. El circuito del problema anterior se conectan a
Encuentre una fuente de 24 V, 200 Hz. Conteste a las
a) La impedancia del circuito, mismas preguntas para este caso.
b) La corriente en ella, Respuesta.
c) La potencia que se disipa, y a) 0,960 A; 1,206 A; 0,477 A b) 1,205 A
d) La mínima potencia en voltamperios de la c) 19,91 Ω d) I se adelanta de V 37° e) 23 W
fuente.
Respuesta. 31. Un micrófono cuya impedancia es 20 Ω debe
a) 85 Ω b) 1,41 A c) 99,4 W d) 169 VA ser utilizado con un amplificador con impedancia
de entrada 50000 Ω . Encuentre el cociente de
26. a) Encuentre la frecuencia de resonancia f R vueltas en el transformador necesario para
del circuito del problema anterior. emparejar estas dos impedancias.
b) ¿Qué corriente fluirá en el circuito si está Respuesta.
conectada con una fuente de 20 V cuya frecuencia El cociente de vueltas es 50:1; la bobina con el
sea f R ? mayor número de vueltas se debe conectar con el
amplificador puesto que tiene la mayor
c) La potencia que se disipa, y impedancia.
d) La mínima potencia en voltamperios de la
fuente. 32. El circuito de la figura se denomina filtro de
Respuesta. pasa alta, porque las frecuencias de entrada
a) 37,5 Hz b) 2,4 A c) 288 W d) 288 VA elevadas se transmiten con una amplitud mayor
que las frecuencias bajas:
27. Un resistencia de 10 Ω y un condensador de 8 a) Si la tensión de entrada es Ve = V0 cos ωt ,
pF se conectan en paralelo a través de una fuente
de 10 V, 1000 Hz. Encuentre hallar el módulo de la tensión de salida Vs .
a) la corriente en cada componente, b) ¿A qué frecuencia angular la tensión de salida
b) la corriente total, es la mitad que la de entrada?
c) la impedancia del circuito, y
d) el ángulo de la fase y la potencia del circuito. c) Dibujar un gráfico de VS / V0 en función de
Respuesta. a) I C = 0,5 A, I R = 10 A b) 1,12 ω.
A c) 8.9 Ω d) 27°; 10 W

28. Una resistencia de 10 Ω y una inductancia de


2 mH se conectan en paralelo a través de una
fuente de 1 0 V, 1000 Hz. Encuentre
a) la corriente en cada componente,
b) la corriente total, Respuesta.
c) la impedancia del circuito, y
V0 1
d) el ángulo de la fase y la potencia del circuito. a) V S = , b) ω= ,
Respuesta. ⎛ 1 ⎞
2 RCω
a) I L = 0,8 A, I R = 1,0 A b) 1,3 A c) 7,7 Ω 1 + ⎜1 + ⎟
⎝ RCω ⎠
d) - 40°; 10 W
c)
29. Una resistencia 25 Ω , un condensador de 40
μ F, y una bobina de 40 mh se conectan en
paralelo con una fuente de 24 V, 100 Hz. Hallar.
a) la corriente en cada componente,
b) la corriente total en el circuito,
c) la impedancia del circuito,
d) el ángulo de fase entre la corriente y el voltaje,
y
e) la potencia total disipada por el circuito.
Respuesta. a) 0,960 A; 0,603 A; 0,955 A b)
33. En una línea de corriente alterna de 120 V se
1,022 A c) 23,48 Ω d) I se atrasa de V 20° e) derivan un calentador, que consume 1 kW, y un
23 W motor, que consume 2 kW, con un factor de
potencia 0,5. Calcúlese la intensidad total tomada
de la línea y el factor de potencia de la carga total.
Respuesta. I = 38,2 A; cos φ = 0,65 .

27
Corriente alterna Hugo Medina Guzmán

a) Lo que señala un voltímetro entre los extremos


34. Se conecta una impedancia Z a una línea de de la resistencia, si marca lo mismo entre los
corriente alterna de 100 V y 60 ciclos. Al conectar extremos de la autoinductancia.
un condensador de 265 μ F en paralelo con Z, la b) lo que indicará el voltímetro al modificar R y L
corriente de la línea se reduce a la mitad de su de modo que marque 50 V para la
primitivo valor, y está en fase con el voltaje de la autoinductancia.
línea. Calcúlese la potencia suministrada a la Respuesta. I 1 R1 = 70,7 V ; I 2 R2 = 86,5 V.
impedancia.
Respuesta. P = 580 W. 37. Un generador de corriente alterna alimenta un
circuito constituido por una resistencia óhmica R,
35. Un circuito se compone de una resistencia de en serie con un paralelo, una de cuyas ramas es
100 Ω en serie con un condensador. El circuito una autoinductancia pura L y la otra un
ha sido proyectado para tener a 100 Hz doble condensador C. Determínese la frecuencia que
impedancia que a 300 Hz. ¿Cuál será la capacidad haga que la intensidad que circula por el circuito
del condensador? sea mínima
Respuesta. C = 6,85 pF. Respuesta.
1 1
36. A una línea alterna de 100 V se conectan una f =
resistencia y una autoinductancia en serie. 2π LC
Determínese:

28
Las Ecuaciones de Maxwell y ondas electromagnéticas Hugo Medina Guzmán

Capítulo 6. Las Ecuaciones de Maxwell y Ondas Electromagnéticas


INTRODUCCIÓN de desplazamiento de Maxwell. La corriente de
¿Qué tienen en común la luz, las ondas de radio, las desplazamiento se define por
microondas, y los rayos X? Todos son formas de dΦ E
ondas electromagnéticas. Juntos representan una Id = ε0 , en donde Φ E es el flujo de campo
gama enorme de fenómenos que son muy importantes dt
en nuestro mundo tecnológico moderno. eléctrico. La forma generalizada de la ley de Ampere
La predicción de la existencia de ondas es entonces
electromagnéticas y una descripción de su naturaleza → → dΦ E
primero fue publicada por James Clerk Maxwell en ∫ B ⋅ d l = μ 0 (I + I d ) = μ 0 I + μ 0 ε 0 dt
1865. Maxwell era un físico teórico enormemente
Podemos comprender la generalización de la ley de
talentoso que hizo contribuciones importantes en
Ampere considerando nuevamente la figura anterior.
muchas áreas de la física incluyendo
electromagnetismo, termodinámica, la teoría cinética Denominemos a la suma I + I d corriente
de gases, la visión de color, y la astronomía. generalizada. De acuerdo con nuestros razonamientos
Él es el más conocido, sin embargo, por su tratado anteriores la misma corriente generalizada debe
sobre campos eléctricos y magnéticos, que hemos cruzar cualquier área limitada por la curva C. Así
mencionado ya. Su descripción de ondas pues no puede existir ninguna corriente generalizada
electromagnéticas, con una predicción de su que entre o que salga del volumen cerrado. Si existe
velocidad, es solamente una parte de su trabajo una verdadera corriente neta I que entra en el
volumen, deberá existir una corriente de
CORRIENTE DE DESPLAZAMIENTO DE desplazamiento igual I d que salga del volumen. En
MAXWELL
Cuando la corriente no es constante, la ley de Ampere el volumen de la figura existe una corriente verdadera
no es válida. Podemos ver por qué es así neta I que entra en el volumen la cual aumenta la
considerando la carga de un condensador. carga dentro del mismo:
dQ
I=
dt
El flujo de campo eléctrico fuera del volumen está
relacionado con la carga por la ley de Gauss:
→ Q
Φ E neto fuera = ∫ E ⋅ nˆ dS =
ε0
El aumento de carga por unidad de tiempo resulta ser
así proporcional al aumento del flujo neto por unidad
Consideremos la curva C. De acuerdo con la ley de de tiempo fuera del volumen:
Ampere, la integral curvilínea del campo magnético a dΦ E neto fuera dQ
lo largo de esta curva es igual a μ 0 multiplicado por ε0 =
dt dt
la corriente total que atraviesa una superficie
cualquiera limitada por dicha curva Esta superficie no
es necesario que sea plana. Se indican en la figura dos
superficies limitadas por la curva C. La corriente a
través de la superficie 1 es I. No existe ninguna
corriente a través de la superficie 2 puesto que la
carga está detenida en la placa del condensador. Así
pues existe una ambigüedad en la frase “corriente a
través de una superficie cualquiera limitada por la
curva”. Sin embargo, en el caso de corrientes La figura anterior muestra las superficies S1 y S2 que
constantes, la carga no se almacena en ningún punto y encierran una placa de un condensador. En este caso
se tiene siempre la misma corriente sin que importe la todo el flujo del campo eléctrico atraviesa S2. El
superficie que escojamos. campo eléctrico entre las placas está relacionado con
Maxwell se dio cuenta de esta falla de la ley de la carga por
Ampere y demostró que la ley podía generalizarse
σ Q
para incluir todos los casos si la corriente I de la E= =
ecuación se sustituía por la suma de la verdadera ε0 ε0 A
corriente I y otro término I d denominado corriente Siendo A el área de las placas. El flujo a través de S2
es

1
Las Ecuaciones de Maxwell y ondas electromagnéticas Hugo Medina Guzmán

Q LAS ECUACIONES DE MAXWELL


Φ E = EA = Lo que para la Mecánica Clásica significan las Leyes
ε0 de Newton lo son las Ecuaciones de Maxwell para los
La corriente de desplazamiento que atraviesa S2 es fenómenos Eléctricos y Magnéticos. Incluso, estas
dΦ E dQ ecuaciones superaron la dificultad de las Leyes de
Id = ε0 = Newton ya que son compatibles con la Teoría de la
dt dt Relatividad Especial como lo demostraría el propio
Que es igual a la verdadera corriente que atraviesa S1. A. Einstein en 1905.
Una característica significativa de la generalización Las leyes del electromagnetismo fueron enunciadas
de Maxwell consiste en que un campo magnético por Gauss, Coulomb, Ampere, Faraday, etc., de tal
resulta ser producido por un campo eléctrico variable forma que los fenómenos que describen afectan a una
lo mismo que por corrientes eléctricas verdaderas. No región del espacio de dimensiones finitas. Estas leyes
hay ninguna duda de que Maxwell fue inducido a fueron recopiladas por James Clerk Maxwell quien
obtener esta generalización por el resultado recíproco elaboró una completa teoría Electromagnética
de que se produce un campo eléctrico mediante un basándose en sus famosas ecuaciones, las que a partir
flujo magnético variable lo mismo que por cargas de ese momento se denominaron las ecuaciones de
eléctricas. Este último resultado, conocido como ley Maxwell-
de Faraday, precedió a la generalización de Maxwell.

Ejemplo 1. En una región cilíndrica del espacio de 1. Ley de gauss para campo eléctrico ( E )
radio R un campo eléctrico está variando en función → → q
del tiempo, determine la dirección y magnitud del
campo magnético inducido para r < R.
∫E⋅d l = ε0
Solución. donde q es la carga neta encerrada por la superficie
De la ecuación sobre la que se integra (superficie gaussiana), esta

ecuación se puede expresar en su forma diferencial de
→ → dΦ E dE
∫ B ⋅ d l = μ 0ε 0 dt = μ 0ε 0 ∫ dt ⋅ nˆdS la siguiente manera:
→ ρ
tenemos que: ∇E =
ε0
→ → dΦ E
∫ B ⋅ d l = μ 0ε 0 dt
donde ρ es densidad volumétrica de carga eléctrica
de la superficie gaussiana.
De donde: Estas ecuaciones establecen que el flujo eléctrico

→ → dE total a través de cualquier superficie cerrada es igual
∫ B ⋅ d l = μ 0ε 0 ∫ dt ⋅ nˆdS a la carga neta encerrada por la superficie dividida
por . Lo que implica con un análisis posterior que
Integrando para r <R ambos lados de la ecuación se las líneas de campo eléctrico se originan en cargas
tiene que: positivas y terminan en cargas negativas (Existen
dE fuentes y sumideros).
B 2π r = μ 0 ε 0π r 2
dt

Despejando el campo magnético:
2. Ley de gauss para campo magnético ( B )
r dE
B = μ 0ε 0 → →
2 dt ∫B⋅d l = 0
La dirección del campo magnético inducido tiene la lo que corresponde a:
dirección del movimiento de las manecillas del reloj, →
ya que el campo eléctrico está entrando al plano y ∇B = 0
creciendo, como se muestra en la figura siguiente. Esto implica que el flujo magnético a través de una
superficie cerrada es cero, es decir, el número de
líneas de campo magnético que entran a la superficie
es igual al número de ellas que salen. Esto significa
que en la naturaleza NO existen monopolos
magnéticos, sólo existen dipolos magnéticos.

3. Ley de Faraday (Inducción Electromagnética)


La Ley de Faraday explica cómo un flujo campo
magnético variable en el tiempo puede inducir en un
circuito una corriente eléctrica cuya fuerza
electromotriz viene dada por:

2
Las Ecuaciones de Maxwell y ondas electromagnéticas Hugo Medina Guzmán

dΦ B
ε =−
dt
En su forma integral se puede expresar como:
→ → dΦ B
∫E ⋅d l = − dt
y en su forma diferencial:

→ dB
∇× E = −
dt
establece que el rotacional del campo eléctrico
inducido por un campo magnético variable es igual a
menos la derivada parcial del campo magnético con Si la dirección de la corriente cambia lo
respecto al tiempo. suficientemente rápido, una corriente alterna fluye en
este dispositivo aun cuando parece ser un circuito
4. Ley generalizada de Ampere - Maxwell abierto. La carga de un signo comenzará a
Fue Maxwell, quien modificó la Ley de Ampere acumularse en los alambres, pero antes de que la
introduciéndole la corriente de desplazamiento que carga acumulada sea demasiado grande, la corriente
fue una de sus mayores contribuciones a la se invierte, la carga regresa, y carga de signo opuesto
electricidad y magnetismo, la corriente de comienza a acumularse. Tenemos así una cantidad de
desplazamiento se debe precisamente a la variación carga cambiante y una corriente eléctrica cambiante
del campo eléctrico en función del tiempo, es decir, en los alambres.
que la ecuación de Ampere generalizada se escribe de Los campos magnéticos generados por este
la siguiente forma: dispositivo se pueden representar por líneas circulares
→ → dΦ E de campo con centro en los alambres. Este campo, no
∫B⋅d l = μ 0 I + μ 0 I d , donde I d = ε 0
dt
obstante está cambiando constantemente en magnitud
y dirección como los cambios de la corriente. Por la
Por lo tanto, la ecuación anterior la podemos expresar ley de Faraday, Maxwell sabía que un campo
como: magnético que cambia generaría un voltaje en un
→ → dΦ E circuito cuyo plano es perpendicular a las líneas del
∫ B ⋅ d l = μ 0 I + μ 0ε 0 dt
, que se conoce campo magnético. Un voltaje implica un campo
eléctrico, aun en ausencia de un circuito, un campo
como Ley de Ampere-Maxwell.
magnético cambiante generará un campo eléctrico en
Luego de algunas reducciones, se puede expresar esta
cualquier momento en el espacio en el cual el campo
ley en su forma diferencial:

magnético está cambiando.
→ → ∂E Así, esperamos que un campo eléctrico cambiante sea
∇ × B = μ 0 J + μ 0ε 0 generado por el campo magnético cambiante, según
∂t la ley de Faraday. Maxwell consideró simetría en el
comportamiento de los campos eléctricos y
ONDAS ELECTROMAGNÉTICAS magnéticos: un campo eléctrico cambiante también
¿Qué es una onda electromagnética? genera un campo magnético. Maxwell predijo este
Para entender las ondas electromagnéticas, fenómeno en sus ecuaciones que describían el
necesitamos repasar los conceptos de campo eléctrico comportamiento de campos eléctricos y magnéticos.
y de campo magnético. Ambos campos pueden ser Las mediciones experimentales confirmaron su
producidos por partículas cargadas. El movimiento de existencia.
la carga genera un campo magnético, pero un campo Maxwell se dio cuenta que una onda que implicaba
eléctrico está presente aun sin movimiento de la estos campos se podría propagar a través del espacio.
carga. Estos campos son una característica del Un campo magnético cambiante produce un campo
espacio alrededor de las cargas y son útiles para eléctrico cambiante, que, a su vez produce un campo
predecir las fuerzas en otras cargas, esto ya fue magnético cambiante, y así sucesivamente. En el
discutido anteriormente. vacío, el proceso puede producirse indefinidamente y
Suponga que la carga está fluyendo hacia arriba y afectar partículas cargadas a distancias mucho
hacia abajo en dos alambres conectados a una fuente mayores de la fuente que podría estar con campos
de corriente alterna, como en la figura siguiente. estáticos generados por corrientes o cargas no
cambiantes. Así es cómo se produce una onda
electromagnética. Los alambres en la figura anterior
sirven como antena transmisora para las ondas. Se
puede utilizar una segunda antena para detectar las
ondas.

3
Las Ecuaciones de Maxwell y ondas electromagnéticas Hugo Medina Guzmán

Aunque Maxwell predijo la existencia de tales ondas 1


en 1865, el primer experimento para producirlas y v=
para detectarlas con circuitos eléctricos fue realizado μ 0ε 0
por Heinrich Hertz (1857 - 1894) en 1888. Las Tomando los valores más acertados de μ0 y de ε0
antenas originales de Hertz eran lazos circulares del
alambre en vez de los alambres rectos, pero él se encuentra que esta es precisamente la velocidad de
también utilizó alambres rectos trabajos posteriores. la luz en el espacio libre.
Él podía detectar una onda producida por el circuito Así pues, hemos demostrado que tanto el campo
fuente con otro circuito a una distancia considerable eléctrico E como el magnético B obedecen a una
de la fuente. Hertz descubrió las ondas radiales ecuación de onda para las ondas que se mueven con
haciendo estos experimentos. velocidad que es la velocidad de la luz.
En cualquier caso, el campo eléctrico y el campo
magnético son perpendiculares al eje x, dirección de
propagación. Como hemos visto, las ondas
electromagnéticas son transversales. Para mayor
sencillez, hemos supuesto que el campo eléctrico
tiene dirección y. Esto equivale a seleccionar una
polarización particular para la onda transversal. Una
vez hecha esta selección ya no podemos escoger
La figura anterior muestra la naturaleza de ondas libremente la dirección del campo magnético.
electromagnéticas simples. Si el campo magnético
está en plano horizontal, como en la figura, el campo Ejemplo 2. a) ¿Cuánto tiempo tarda la luz en viajar
eléctrico generado por el campo magnético cambiante de la Luna a la Tierra, una distancia de 384000 km?
está en la dirección vertical. Estos dos campos son b) La luz de la estrella Sirio tarda 8,61 años luz en
perpendiculares entre si, y son también llegar a la Tierra. ¿Cuál es la distancia a Sirio?
perpendiculares a la dirección del recorrido alejados Solución.
de la antena fuente. Las ondas electromagnéticas son
por lo tanto ondas transversales. Las magnitudes de d 3,84 × 10 8
a) t = = = 1,28 s
los campos eléctrico y magnético se representan aquí c 3,00 × 10 8
sinusoidalmente y en fase el uno con el otro, pero
b) El tiempo de viaje de la luz:
patrones son también posibles más complejos.
8,61 años
Como los otros tipos de ondas que hemos estudiado
el patrón sinusoidal de la onda se mueve. La figura = (8,61 años) (365 dias) (24 horas) (3600 s)
(1 año) (1 dia ) (1 hora )
muestra las magnitudes y las direcciones en un 8
instante de tiempo y a lo largo de solo una línea en = 2,72 x 10 s
espacio. La misma clase de variación ocurre en todas d = ct = (3,0 × 10 8 )(2,72 × 10 8 )
las direcciones perpendiculares a la antena. Como el = 8,16x1016 m = 8,16x1013 km
patrón sinusoidal se mueve, los valores del campo en
todo momento en del espacio aumentan y la Ejemplo 3. ¿Cuál del siguiente es una afirmación
disminuyen alternativamente. Mientras los campos verdadera?
pasan por cero, cambian la dirección y comienzan a A) Todas las ondas electromagnéticas viajan a la
aumentar en la dirección opuesta. Estos cambios misma velocidad en vacío.
coordinados de los campos eléctrico y magnético B) La luz aumenta su velocidad cuando se mueve
componen la onda electromagnética. desde el aire al agua.
Maxwell demostró que las ondas electromagnéticas C) Algunas ondas electromagnéticas tienen carga
(propagándose a lo largo de una dirección x) en el eléctrica.
espacio libre se describirían por medio de las D) En el vacío las ondas electromagnéticas de alta
siguientes ecuaciones: frecuencia viajan a una velocidad más alta que las
→ → → →
ondas electromagnéticas de baja frecuencia.
∂2 E ∂2 E ∂2 B ∂2 B
= μ ε
0 0 , = μ ε
0 0
E) Los electrones son una clase de onda
∂x 2 ∂t 2 ∂x 2 ∂t 2 electromagnética.
Ecuación de onda para E y Solución.
(A) Todas las ondas electromagnéticas viajan a la
∂2Ey ∂⎛ ∂E y ⎞ ∂2Ey misma velocidad en vacío.
=− ⎜⎜ − μ 0 ε 0 ⎟⎟ = + μ 0 ε 0 m
∂x 2 ∂t ⎝ ∂t ⎠ ∂t 2 c = 3 × 10 8 =λ f
Comparando esta ecuación con la ecuación de la onda s
Ejemplo 4. Fantasmas en el televisor. En una
∂ 2 y ( x ,t ) 1 ∂ y ( x ,t )
2
imagen de televisión, se ven imágenes fantasma
= 2 , vemos que E y obedece a
cuando la señal de la transmisora viaja el receptor
∂x 2 v ∂t 2
una ecuación de onda para unas ondas con velocidad tanto directa como indirectamente, luego de reflejarse
en un edificio o alguna otra masa metálica grande. En

4
Las Ecuaciones de Maxwell y ondas electromagnéticas Hugo Medina Guzmán

un televisor de 25 pulgadas, el fantasma aparece


aproximadamente 1,0 cm a la derecha de la imagen
principal si la señal reflejada llega 0.60 s después de
la señal principal. En este caso, ¿cuál es la diferencia
de longitud entre las trayectorias de las dos señales?
Solución.
d = cΔt = (3,0x108)(6,0x10-7) = 180 m
A partir de esta figura puede observarse que la
→ →
Ejemplo 5. Una onda electromagnética sinusoidal
con una frecuencia de 6,10x1014 Hz viaja en un vacío dirección de es paralela al vector E× B propagación
→ →

en la dirección + z. El campo B es paralelo al eje de E× B
en dirección x en este caso. El vector se
las y y tiene una amplitud de 5,80x10-4 T. Escriba las μ0
→ →
conoce como vector Poynting (en honor de John H.
ecuaciones vectoriales de E ( z ,t ) y B ( z ,t ) .
Poynting):
Solución. → →
→ → E× B
B( z , t ) = Bmáx cos(kz − ωt)ˆj S=
μ0
⎛ ⎛ z ⎞⎞
= Bmáx cos⎜ 2πf ⎜ − t ⎟ ⎟ ˆj ⇒ El vector Poynting en el sistema MKS, está dado en
⎝ ⎝ c ⎠⎠ unidades de W/m2.

B(z, t ) = INTENSIDAD DE LA ONDA.
A continuación demostraremos que su valor medio es
⎛ z ⎞ igual a la intensidad de la onda electromagnética. En
(5,80 × 10 −4 ) cos 2π (6,10 × 1014 )⎜⎜ − t ⎟⎟ ˆj
⎝ (3.00 × 10 8
) ⎠ nuestro estudio del transporte de energía por ondas
→ cualquier tipo, veíamos que la intensidad (energía
E ( z,t ) = By(z,t)ˆj × (ckˆ) ⇒ media por unidad de área y unidad de tiempo) era, en
→ general, igual al producto de la densidad de energía
E (z,t ) = (energía por unidad de volumen) por la velocidad de

(1,74 × 10 )cos(1,28 × 10 z − 3,83 × 10 t )iˆ


5 7 15
la onda. La energía por unidad de volumen asociada
con un campo eléctrico E es
1
Ejemplo 6. Una onda electromagnética con una uE = ε0E2
frecuencia de 38,0 MHz y una longitud de onda de
2
En el caso de la onda armónica dada por la ecuación
6,15 m viaja en un material aislante con k m muy
E y = E y 0 sen (kx − ωt ) tenemos
cercana a la unidad.
1
a) ¿Cuál es la rapidez de propagación de la onda?
b) ¿Cuál es la constante dieléctrica del material
uE = ε 0 E 02 sen (kx − ωt )
2
aislante a esta frecuencia? Por sencillez en la notación se han suprimido los
Solución. subíndices de las componentes y y z
a) v = f λ = (3,80 × 10 )(6,15 m)
7
La energía por unidad de volumen asociada con un
campo magnético B es
= 2,34 × 10 m s
8

B2
b) k E =
c 2
=
(
3,0 × 10 8 2
) = 1,64
uB =
2μ0
v 2
(
2,34 × 10 8 ) 2
k
Según las ecuaciones B z = E z 0 sen (kx − ωt ) =
VECTOR POYNTING.
ω
El vector que nos da la dirección y la magnitud de la Bz 0sen (kx − ωt ) tenemos
rapidez del flujo de energía electromagnética por
unidad de área en un punto del espacio, es llamado
B02 E02
vector Poynting y se define por la siguiente ecuación: uB = sen (kx − ωt ) = sen 2 (kx − ωt )
→ 1 → → 2μ0 2μ0c 2

S= E× B
μ0 1
Pero como c =
2

ε 0 μ0
1
u B = ε 0 E02sen 2 (kx − ωt )
2

5
Las Ecuaciones de Maxwell y ondas electromagnéticas Hugo Medina Guzmán

y vemos que son iguales las densidades de energía ENERGÍA ELECTROMAGNÉTICA


eléctrica y magnética. La energía electromagnética La densidad de energía eléctrica dada por la ecuación:
total por unidad de volumen es 1 1 → →
E2 uE = ε 0 E 2 = ε 0 E⋅ E
u = u E + u B = ε 0 E02sen 2 (kx − ωt ) = 0 2 sen 2 (kx − ωt ) 2 2
μ0c La densidad de energía magnética que está expresada
La energía por unidad de tiempo y por unidad de área por:
es el producto de esta energía por la velocidad c: 1 B2 1 → →
E 2 uB = = B⋅ B
uc = 0
sen 2 (kx − ωt ) 2 μ0 2μ 0
μ0c Para una región del espacio libre donde existe campo
⎡ E0 ⎤ eléctrico y campo magnético la ecuación de la energía
[
= E 0 sen (kx − ωt ) ⎢ ] sen (kx − ωt )⎥ electromagnética es expresada por:
⎣ μ0c ⎦ U = ∫ (u E + u B )dV =
V
EB
= =S 1 ⎛ → → → →

μ0 ∫
2μ 0 ⎝
V
⎜ ε μ
0 0 E ⋅ E + B ⋅ B ⎟dV

La intensidad en un punto cualquiera x es el valor
medio temporal de la energía por unidad de tiempo y En esta ecuación hemos combinado campos eléctricos
por unidad de área. A partir de la ecuación y magnéticos.
E 02
u= sen 2 (kx − ωt ) vemos que la densidad PRESIÓN DE RADIACIÓN.
μ0c 2
A partir de la observación de que se requiere energía
para establecer campos eléctricos y magnéticos,
de energía es proporcional a sen (kx − ωt ) . El
2
hemos demostrado que las ondas electromagnéticas
valor medio de esta cantidad en uno o más ciclos es transportan energía. Además se puede demostrar que
1/2. Luego, la densidad de energía media vale las ondas electromagnéticas transportan L cantidad de
E 02 movimiento p, con una densidad de cantidad de
uB = , y la intensidad es movimiento correspondiente (cantidad de
2μ 0 c 2 movimiento dp por volumen dV ) de magnitud
E 02 E B0 E B dp EB S
I = cu B = = 0 = rms rms = = 2
2μ 0 c 2 μ0 2 2 dV μ 0 c 2
c
= SB Esta cantidad de movimiento es una propiedad del
Donde campo; no está asociada con la masa de una partícula
en movimiento en el sentido habitual.
E0 Existe además una rapidez de flujo de cantidad de
E rms = es el valor medio cuadrático de E y
2 movimiento correspondiente. El volumen dV
B ocupado por una onda electromagnética (rapidez c)
Brms = 0 es el valor medio cuadrático de B. que pasa a través de un área A en un tiempo dt es
2 dV = Acdt . Cuando se sustituye esto en la
dp EB S
POTENCIA INSTANTÁNEA. ecuación = = 2 y se reordena, se
El módulo o valor del vector Poynting es la potencia dV μ 0 c 2
c
instantánea por unidad de área. La velocidad con la encuentra que la rapidez de flujo de cantidad de
cual fluye energía electromagnética a través de una movimiento por unidad de área es
superficie cualquiera es el flujo del vector de 1 dp S EB
Poynting a través de la misma: = = (rapidez de flujo de cantidad de
A dt c μ 0 c
⎛→ →⎞
E× B ⎟
P = ∫ S ⋅ nˆ dA = ∫ ⎜⎜

movimiento)
⋅ nˆ dA
⎜ μ0 ⎟⎟ Esto representa la cantidad de movimiento que se
⎝ ⎠ transfiere por unidad de área y por unidad de tiempo.
En donde P es la potencia que atraviesa el área y Se obtiene la rapidez promedio de transferencia de
→ → cantidad de movimiento por unidad de área
→ E× B sustituyendo S por S m = I len la ecuación
S= es el vector Poynting. La potencia neta
μ0 1 dp S EB
radiada por cualquier volumen es el flujo neto del = = .
vector de Poynting a través de la superficie que
A dt c μ 0 c
encierra dicho volumen. A esa cantidad de movimiento se debe el fenómeno
de la presión de radiación. Cuando una onda
electromagnética es absorbida en su totalidad por una

6
Las Ecuaciones de Maxwell y ondas electromagnéticas Hugo Medina Guzmán

superficie, se transfiere a ésta la cantidad de NASA que efectúa su primer vuelo en el trasbordador
movimiento de la onda. Para simplificar espacial. En virtud de su extensa capacitación en
consideraremos una superficie perpendicular a la física, se le ha asignado la tarea de evaluar el
dirección de propagación. Con base en las ideas comportamiento de un nuevo transmisor de radio a
expuestas en la sección 8.1, vemos que la rapidez bordo de la Estación Espacial Internacional (EEI).
dp/dt con la que se transfiere cantidad de movimiento Encaramado en el brazo móvil del trasbordador, usted
a la superficie absorbente es igual a la fuerza ejercida apunta un detector sensible hacia la EEI, que está a
sobre la superficie. La fuerza promedio por unidad de 2,5 km de distancia, y encuentra que la amplitud de
área debida a la onda, o presión de radiación p rad , campo eléctrico de las ondas de radio provenientes
del transmisor es de 0,090 V/m, y que la frecuencia
es el cociente del valor promedio de dp/dt entre el de las ondas es de 244 MHz. Halle lo siguiente:
área absorbente A. (Utilizamos el subíndice “rad” a) la intensidad de la onda de radio donde usted se
para distinguir la presión de la cantidad de encuentra;
movimiento, que también se representa mediante el b) la amplitud de campo magnético de la onda donde
símbolo p). De acuerdo con la ecuación
usted se encuentra;
dp EB S c) la potencia de salida total del transmisor de radio
= = 2 , la presión de radiación es
dV μ 0 c 2
c de la EEI.
d) ¿Qué suposiciones, en su caso, hizo usted para
S I efectuar sus cálculos?
p rad = m = (presión de radiación, absorción
c c Solución.
total de la onda) 1
Si la onda se refleja en su totalidad, el cambio de
a) I = ε 0 cE máx
2
, E máx = 0,090 V m , luego
2
cantidad de movimiento es dos veces más grande, y la
presión es I = 1,1 × 10 −5 W m 2
Sm I 2S 2I b) E máx = cBmáx Luego
p rad = = p rad = m = (presión de
c c c c E
radiación, reflexión total de la onda) Bmáx = máx = 3,0 x10-10 T
c
Por ejemplo, el valor de I (o S m ) correspondiente a
c) Pav = I ( 4πr ) =
2
la luz solar directa, antes de atravesar la atmósfera
terrestre, es de aproximadamente 1,4 kW/m2. De (1,075 × 10 )(4π ) (2,5 × 10 )
−5 3 2
= 840 W
Sm I d) El cálculo en la parte (c) asume que el transmisor
acuerdo ecuación p rad = = la presión emite uniformemente en todas las direcciones.
c c
promedio correspondiente sobre una superficie
Ejemplo 8. Una sonda espacial situada a 2,0 x 1010 m
absorbente es
de una estrella mide la intensidad total de la radiación
I 1,4 × 10 −3 electromagnética proveniente de la estrella, la cual
p rad = = = 4,7x10-6 Pa
resulta ser de 5,0 x 103 W/m2. Si la estrella irradia de
c 3,0 × 10 8
modo uniforme en todas direcciones, ¿cuál es la
S I potencia de salida promedio total?
Según la ecuación p rad = m = , la presión
c c Solución.
promedio sobre una superficie totalmente reflectante ( )
Pm = I (4πr 2 ) = 5,0 × 103 (4π ) (2,0 × 1010 ) 2
2I 25
= 2,5x10 J
es el doble de esto: o 9,4xl06 Pa. Estas presiones
c
son muy pequeñas del orden de 10-10 atm, pero se Ejemplo 9. La intensidad de un rayo láser cilíndrico
pueden medir con instrumentos sensibles. es de 0,800 W/m2. El área de sección transversal del
La presión de radiación de la luz solar es mucho haz es de 3,0 x l0 m2, y la intensidad es uniforme en
mayor en el interior del Sol que en la Tierra. En el toda la sección transversal del haz.
interior de las estrellas de masa mucho mayor que la a) ¿Cuál es la potencia de salida promedio del láser?
del Sol y más luminosas que éste, la presión de b) ¿Cuál es el valor rms (eficaz) del campo eléctrico
radiación es tan grande que aumenta en el haz?
considerablemente la presión gaseosa en el interior de Solución.
la estrella y de este modo contribuye a evitar que la a) La potencia media del haz es
estrella se colapse bajo el efecto de su propia P = IA = (0,800) (3,0 ×10 −4 )
gravedad. En ciertos casos la presión de radiación = 2,4x10-4 W
proyecta efectivamente parte del material de la 1
estrella hacia el espacio. b) Tenemos que, I = ε 0 cE máx
2
= ε 0 cE rms
2
.
2
Ejemplo 7. Prueba de un transmisor espacial de Luego,
radio. Usted es un especialista en misiones de la

7
Las Ecuaciones de Maxwell y ondas electromagnéticas Hugo Medina Guzmán

a) la densidad de cantidad de movimiento (cantidad


I 0,800
E rms = = de movimiento por unidad de volumen) promedio de
ε 0c (8,85 × 10 −12 )(3,00 × 10 8 ) la luz solar;
= 17,4 V/m b) la rapidez de flujo de cantidad de movimiento
promedio de la luz solar.
Ejemplo 10. Una fuente intensa de luz irradia Solución.
uniformemente en todas direcciones. A una distancia a) La densidad de cantidad de movimiento
de 5,0 m de la fuente de radiación la presión sobre dp S m 780 −15 kg
= 2 = = 8,7 × 10
una superficie perfectamente absorbente es de 9,0 x
10-6 Pa. ¿Cuál es la potencia media de salida de la
dV c 3,0 ×108 ( )2
m 2s
fuente? b) La razón de flujo de la cantidad de movimiento
Solución. 1 dp S av 780
p rad = I c de aquí I = cp rad = = = 2,6x10-6Pa
A dt c 3,0 ×10 8

= 2,70x103 W/m2
Luego Ejemplo 14. En las instalaciones del Simulador
( )
Pm = I (4πr 2 ) = 2.70 × 10 3 (4π ) (5,0) 2 Espacial de 25 pies del Jet Propulsion Laboratory de
= 8,5x105 W la NASA, una serie de lámparas de arco elevadas
produce luz con una intensidad de 2500 W/m2 en el
Ejemplo 11. Una onda electromagnética sinusoidal piso de las instalaciones. (Esto simula la intensidad
emitida por un teléfono celular tiene una longitud de de la luz solar cerca del planeta Venus). Halle la
onda de 35,4 cm y una amplitud de campo eléctrico presión de radiación promedio (en pascal y en
de 5,40 x 10-2V/m a una distancia de 250 m de la atmósferas) sobre
antena. Calcule a) una sección totalmente absorbente del piso.
a) la frecuencia de la onda; b) una sección totalmente reflectante del piso.
b) la amplitud del campo magnético; c) Halle la densidad de cantidad de movimiento
c) la intensidad de la onda. promedio (cantidad de movimiento por unidad de
Solución. volumen) de la luz en el piso.
Solución.
c 3,00 ×108 a) Luz absorbida:
a) f = = = 8,47 Hz
λ 0,354 1 dp S m 2500
p rad = = =
E 0,0540 A dt c 3,0 ×108
b) Bmax = max = =1,80x10-10T
c 3,00 × 10 8
= 8,33x10-6 Pa ⇒

EB (0,0540)(1,80 ×10 −10 ) p rad =


8,33 × 10 −6
= 8,33x10-11 atm
c) I = S av = =
2μ 0 2μ0 1,013 × 10 5

= 3,87x10-6W/m2 b) Luz reflejada:


1 dp 2 S m
Ejemplo 12. Una fuente de luz monocromática con p rad = =
A dt c
una potencia de salida de 60,0 W irradia luz
uniformemente en todas direcciones con una longitud 2(2500 )
= = 1,67x10-5 Pa ⇒
de onda de 700 nm. Calcule E máx y Bmáx de la luz 3.0 ×108
de 700 nm a una distancia de 5,00 m de la fuente. 1,67 × 10 −5
Solución. p rad = = 1,65x10-10atm.
1,013 × 10 5
E 2
Pcμ 0
P = Sm A = max
(4πr 2 ) ⇒ Emax = El factor 2 se presenta porque la dirección del vector
2cμ 0 2πr 2 cantidad de movimiento se invierte con la reflexión.

⇒ E máx =
(
(60,0) 3,00 × 10 8 μ 0 )
= 12,0 V/m
Así el cambio en cantidad de movimiento es dos
veces la cantidad de movimiento original
2π (5,00) 2 c) La densidad de cantidad de movimiento
⇒ dp S m 2500
= 2 =
Bmáx
E
= máx =
12,0
= 4,00x10-8T
dV c (
3,0 ×108 )2

c 3,00 × 10 8 −14 kg
= 2,78 × 10
m 2s
Ejemplo 13. Si la intensidad de la luz solar directa en
un punto de la superficie terrestre es de 0,78 kW/m2, Ejemplo 15. Una onda electromagnética sinusoidal
halle se propaga en un vacío en la dirección + z. Si en un

8
Las Ecuaciones de Maxwell y ondas electromagnéticas Hugo Medina Guzmán

instante en particular y en cierto punto del espacio el


campo eléctrico tiene la dirección + x y una magnitud
de 4,00 V/m, ¿cuáles son la magnitud y dirección del
campo magnético de la onda en el mismo punto del
espacio y en el mismo instante?
Solución.
Emáx
Bmáx = = 1,33x10-8 T
c
→ → →
E× B está en la dirección de propagación. Para E
→ →
en la dirección + x , E× B está en la dirección

+ z cuando B está en la dirección + y .

Ejemplo 16. Se establece una onda electromagnética


estacionaria en el aire con una frecuencia de 750
MHz entre dos planos conductores separados por una
distancia de 80,0 cm. ¿En qué posiciones entre los
planos se podría colocar una carga puntual en reposo
de modo que permaneciera en reposo? Explique su
respuesta.
Solución.
λ c 3,00 × 10 8
Δx nodos = = = Las ondas electromagnéticas cubren una amplia gama
2 2f 2(7,50 × 10 8 ) de frecuencias o de longitudes de ondas y pueden
= 0,200 m = 20,0 cm clasificarse según su principal fuente de producción.
Debe haber nodos en los planos, separados 80,0 cm, y La clasificación no tiene límites precisos.
hay dos nodos entre los planos, cada uno a 20,0 cm
de un plano. Es a 20 cm, 40 cm, y 60 cm que una Región del espectro Intervalo de
carga puntual permanece en reposo, puesto que el frecuencias (Hz)
campo eléctrico en esos puntos es cero. Radio-microondas 0 - 3,0x1012
Infrarrojo 3,0x10 - 4,6x1014
12
Ejemplo 17. Con respecto a una onda
Luz visible 4,6x1014 -7,5x1014
electromagnética que se en aire, determine la
frecuencia de una onda cuya longitud de onda es de a) Ultravioleta 7,5x1014 - 6,0x1016
5,0 km; b) 5,0 m; c) 5,0 μ m; d) 5,0 nm. Rayos X 6,0x1016-1,0x1020
Radiación gamma 1,0x1020 -….
Solución.
c 3,0 × 10 8
a) f = = = 6,0x104Hz Fuente: Leonberger. Revealing the small range of
λ 5000 radio-microwave frequencies. Phys. Educ. Vol. 37,
September 2002, pp. 425-427
c 3,0 × 10 8
b) f = = = 6,0x107Hz
λ 5,0 CARACTERÍSTICAS DE LAS DISTINTAS
REGIONES DEL ESPECTO
c 3,0 × 10 8
c) f = = = 6,0x1013Hz
λ 5,0 × 10 -6
Las ondas de radiofrecuencia
c 3,0 × 10 8
d) f = = = 6,0x1016Hz Sus frecuencias van de 0 a 109 Hz, se usan en los
λ 5,0 × 10 -9
sistemas de radio y televisión y se generan mediante
circuitos oscilantes.
EL ESPECTRO ELECTROMAGNÉTICO.
Las ondas de radiofrecuencia y las microondas son
especialmente útiles por que en esta pequeña región
del espectro las señales producidas pueden penetrar
las nubes, la niebla y las paredes. Estas son las
frecuencias que se usan para las comunicaciones vía
satélite y entre teléfonos móviles. Organizaciones
internacionales y los gobiernos elaboran normas para
decidir que intervalos de frecuencias se usan para

9
Las Ecuaciones de Maxwell y ondas electromagnéticas Hugo Medina Guzmán

distintas actividades: entretenimiento, servicios Estas dos reacciones absorben prácticamente toda
públicos, defensa, etc. La región denominada AM radiación ultravioleta que viene del Sol por lo que
comprende el intervalo de 530 kHz a 1600 kHz, y la solamente llega una pequeña fracción a la superficie
región denominada FM de 88 MHz a 108 MHz. La de la Tierra. Si desapareciese de la capa de ozono, la
región FM permite a las emisoras proporcionar una radiación ultravioleta destruiría muchos organismos a
excelente calidad de sonido debido a la naturaleza de causa de las reacciones fotoquímicas.
la modulación en frecuencia.
La radiación ultravioleta y rayos X producidos por el
Las microondas Sol interactúa con los átomos y moléculas presentes
en la alta atmósfera produciendo gran cantidad de
Se usan en el radar y otros sistemas de comunicación, iones y electrones libres (alrededor de 1011 por m3).
así como en el análisis de detalles muy finos de la La región de la atmósfera situada a unos 80 km de
estructura atómica y molecular. Se generan mediante altura se denomina por este motivo ionosfera.
dispositivos electrónicos.
Algunas de las reacciones que ocurren más
La radiación infrarroja frecuentemente son:

Se subdivide en tres regiones, infrarrojo lejano, NO + fotón → NO+ + e (5,3 eV)


medio y cercano. Los cuerpos calientes producen
radiación infrarroja y tienen muchas aplicaciones en N2 + fotón → N2+ + e (7,4 eV)
la industria, medicina, astronomía, etc.
O2 + fotón → O2+ + e (5,1 eV)
La luz visible
He + fotón → He+ + e (24,6 eV)
Es una región muy estrecha pero la más importante,
ya que nuestra retina es sensible a las radiaciones de Entre paréntesis se indica la energía de ionización.
estas frecuencias. A su vez, se subdivide en seis Como resultado de esta ionización tienen lugar
intervalos que definen los colores básicos (rojo, muchas reacciones secundarias.
naranja, amarillo, verde, azul y violeta).
Rayos X
Radiación ultravioleta
Si se aceleran electrones y luego, se hacen chocar con
Los átomos y moléculas sometidos a descargas una placa metálica, la radiación de frenado produce
eléctricas producen este tipo de radiación. No rayos X. Los rayos X se han utilizado en medicina
debemos de olvidar que la radiación ultravioleta es la desde el mismo momento en que los descubrió
componente principal de la radiación solar. Roentgen debido a que los huesos absorben mucho
más radiación que los tejidos blandos. Debido a la
La energía de los fotones de la radiación ultravioleta gran energía de los fotones de los rayos X son muy
es del orden de la energía de activación de muchas peligrosos para los organismos vivos.
reacciones químicas lo que explica muchos de sus
efectos. Rayos gamma

El oxígeno se disocia en la ozonósfera por la acción Se producen en los procesos nucleares, por ejemplo,
de la radiación ultravioleta. Una molécula de oxígeno cuando se desintegran las sustancias radioactivas. Es
absorbe radiación de longitudes de onda en el también un componente de la radiación cósmica y
intervalo entre 1600 Å y 2400 Å (o fotones de tienen especial interés en astrofísica. La enorme
energía comprendida entre 7.8 eV y 5.2 eV) y se energía de los fotones gamma los hace especialmente
disocia en dos átomos de oxígeno. útiles para destruir células cancerosas. Pero son
también peligrosos para los tejidos sanos por lo que la
O2 + fotón → O + O manipulación de rayos gamma requiere de un buen
blindaje de protección.
El oxígeno atómico producido se combina con el
oxígeno molecular para formar ozono, O3, que a su Ejemplo 18. Un espejo pequeño con un área de 5,00
vez se disocia fotoquímicamente por absorción de la cm2 frente a una fuente de luz monocromática situada
radiación ultravioleta de longitud de onda a 3,20 m de distancia. En el espejo la amplitud de
comprendida entre 2400 Å y 3600 Å (o fotones de campo eléctrico de la luz de la fuente es de 0,0280
energía entre 5,2 eV y 3,4 eV). V/m.
a) ¿Cuánta energía incide en el espejo?
O3 + fotón → O + O2

10
Las Ecuaciones de Maxwell y ondas electromagnéticas Hugo Medina Guzmán

b) ¿Cuál es la presión de radiación promedio que la = 1,09x10-6 J/m3


luz ejerce sobre el espejo?
c) ¿Cuál es la salida total de energía radiante te si se Observar el factor adicional ½ puesto que estamos
supone que irradia uniformemente en todas haciendo un promedio.
direcciones?
Solución. c) En un metro de haz láser, la energía total es:
a) La energía incidente en el espejo es
2
⎛D⎞
1 E tot = u tot Vol = 2u E ( AL) = 2u E π ⎜ ⎟ L
Energía = Pt = IAt = ε 0 cE 2 At ⎝2⎠
2
⇒ E tot = 2(1,09 × 10 )π (2,50 × 10 ) (1,00)
−6 −3 2

⇒ 4
E = 1 ε 0 (3,00 ×108 )(0,028)2 (5,00 ×10−4 )(1,00)
2 = 1,07x10-11 J
-10
= 5,0x10 J

b) La presión de radiación Ejemplo 20. Un conductor cilíndrico de sección


transversal circular tiene un radio a y una
2I conductividad g , y transporta una corriente
= ε 0 E 2 = ε 0 (0,0280 )
2
p rad = constante I.
c a) ¿Cuáles son la magnitud y dirección del vector de
campo eléctrico E en un punto situado dentro del
= 6,94x10-15 Pa alambre, a una distancia a del eje?
b) ¿Cuáles son la magnitud y dirección del vector de
c) La potencia P = I 4πR = cprad 2πR
2 2
campo magnético B en el mismo punto?
c) ¿Cuáles son la magnitud y dirección del vector de
⇒ P = 2π (3,00 × 108 )(6,94 × 10 −15 )(3,20 ) 2
Poynting S en el mismo punto? (La dirección de S es
aquélla en la que fluye energía electromagnética
hacia adentro o hacia afuera del conductor).
= 1,34x10-4 Pa d) Con base en el resultado del inciso (c), halle la
rapidez de flujo de energía hacia el volumen que
Ejemplo 19. Un láser pequeño de helio y neón emite ocupa un tramo de longitud l del conductor.
luz visible roja con una potencia de 3,20 mW en un (Sugerencia: Integre S sobre la superficie de este
haz de 2,50 mm de diámetro. volumen). Compare su resultado con la rapidez de
a) ¿Cuál es la amplitud de los campos eléctrico y generación de energía térmica en el mismo volumen.
magnético de la luz? Comente por qué se puede pensar que la energía
b) ¿Cuál es la densidad de energía promedio asociada disipada en un conductor portador de corriente,
con el campo eléctrico y con el campo magnético? debido a su resistencia, entra a través de los lados
c) ¿Cuál es la energía contenida en un tramo del haz cilíndricos del conductor.
de 1,00 m de largo? Solución.
Solución. gl gl
a) La intensidad del laser a) E = gJ = = 2 , en dirección de la
P 4P 4(3,20 ×10 −3 ) A πa
I= = = = 652 W/m2 corriente.
A πD 2 −3 2
π (2,50 × 10 ) → → μ0 I
b) ∫ B ⋅ d l = μ0 I ⇒ B = 2πa
en sentido
1 2I antihorario mirando a la corriente.
Pero I = ε 0 cE 2 ⇒ E = =
2 ε 0c c) La dirección del vector Poynting Sˆ = Eˆ × Bˆ , con
la corriente en la dirección z.
2(652 Su magnitud es
= 701 V/m
ε 0 (3,00 ×108 ) EB 1 gI μ 0 I gI 2
S= = =
μ0 μ 0 πa 2 2πa 2π 2 a 3
E 701
Y B= = = 2,34x10-6T d) Sobre una longitud l , la razón con la que ingresa
c 3.00 × 108 la energía es
ρ l2 ρ l l2
1 1 SA = 2πal =
b) u Bm = u Em = ε 0 Emáx
2
= ε 0 (701)
2
2π 2 a 3 πa 2
4 4

11
Las Ecuaciones de Maxwell y ondas electromagnéticas Hugo Medina Guzmán

La pérdida de energía térmica es dB


g l g lI 2 = ωBmax = 2πfBmax
I 2R = I 2 = , que es exactamente igual dt
A πa 2 = 2π (95,0 × 10 ) (2,42 × 10 )
6 −9

al flujo de la energía electromagnética.


= 1,44 T s ⇒
Ejemplo 21. Cierto condensador consiste en dos dΦ dB πD 2 dB
placas circulares de radio r separadas por una ε =− = −A =
distancia d. Sin tener en cuenta el pestañeo, dt dt 4 dt
demuestre que, durante el proceso de carga del π (0,180 ) (1,44 )
2

condensador. La rapidez de flujo de energía hacia el = = 0,0366 V


espacio entre las placas es igual a la rapidez con la 4
que aumenta la energía electrostática almacenada en
→ Ejemplo 23. Se ha propuesto colocar en órbita
el condensador. (Sugerencia: Integre S sobre la terrestre un satélite recolector de energía solar. La
superficie del volumen cilíndrico vacío comprendido potencia recogida se enviaría a la Tierra en forma de
entre las placas). un haz de radiación de microondas. En el caso de un
Solución. haz de microondas con un área de sección transversal
de 36,0 m2 y una potencia total de 2,80 kW en la
μ 0i → q
B=
2πr
,y ∫ S E⋅ nˆ dA = EA =
ε0
⇒ superficie terrestre, ¿cuál es la amplitud del campo
eléctrico del haz en la superficie del planeta?
Solución.
q
E= tal que la magnitud del vector Poynting P 1
πε 0 r 2 I= = ε 0 cE 2
A 2
EB qi q dq
es S = = = . 2I 2(2,80 × 10 3 / 36)
μ 0 2ε 0π r 2 3
2ε 0π r dt
2 3
⇒ E= =
Ahora, la razón del flujo de energía en la región entre
ε 0c ε 0 (3,00 × 10 8 )
las placas es: = 242 V m.
→ lq dq
∫ S ⋅ nˆdA = S (2πrl) = ε 0πr 2 dt Ejemplo 24. Linterna al rescate. Usted es el único
tripulante de la nave espacial interplanetaria T:1339
1 l d (q 2 ) d ⎛ 1 l 2 ⎞ Vorga, que lleva a cabo misiones de carga entre la
= = ⎜ q ⎟
2 ε 0πr 2 dt dt ⎜⎝ 2 ε 0 A ⎟⎠ Tierra y las colonias mineras del cinturón de
asteroides. Cierto día usted está trabajando afuera de
d ⎛ q 2 ⎞ dU la nave, a una distancia de 2,0 UA del Sol. [1 UA
= ⎜⎜ ⎟= (unidad astronómica) es la distancia promedio de la
dt ⎝ 2C ⎟⎠ dt Tierra al Sol: 149 600 000 km.). Por desgracia, usted
Éste es la razón de incremento en la energía pierde contacto con el casco de la nave y comienza a
electrostática U almacenada en el condensador. alejarse hacia el espacio. Entonces intenta regresar
hacia la nave con ayuda de los cohetes de su traje
Ejemplo 22. Se puede usar una espira circular de espacial, pero el combustible se agota y los cohetes
alambre como antena de radio. Si una antena de 18,0 dejan de funcionar antes que usted consiga regresar a
cm de diámetro está situada a 2,0 km de una fuente de la nave. Se halla usted en una situación difícil,
95,0 MHz con una potencia total de 55,0 kW, ¿cuál flotando a 16,0 m de la nave con velocidad cero
es la fuerza electromotriz máxima que se induce en respecto a ella. Por fortuna, usted tiene una lámpara
la espira? (Suponga que el plano de la espira de la de 200 W, la cual enciende para utilizar su haz como
antena es perpendicular a la dirección del campo un “cohete de luz” que lo impulse de regreso a la
magnético de la radiación y que la fuente irradia nave,
uniformemente en todas direcciones). a) Si usted, su traje espacial y la lámpara tienen en
Solución. conjunto una masa de 150 kg, ¿Cuánto tiempo tardará
2 en regresar a la nave?
cBmax
La potencia de la antena es P = IA = 4πr 2 b) ¿Existe alguna otra forma de utilizar la lámpara
2μ 0 para conseguir el mismo objetivo de regresar a la
Tal que nave?
Solución.
2μ 0 P 2 μ 0 (5,50 × 10 4 ) a) Encontrar la fuerza sobre la persona debido a la
Bmax = =
4πr 2 c 4π ( 2500 ) 2 (3,00 × 10 8 ) cantidad de movimiento por la luz:

= 2,42 × 10 −9 T ⇒ p rad = I c y F = prad A da


F = I A c = Pav / c

12
Las Ecuaciones de Maxwell y ondas electromagnéticas Hugo Medina Guzmán

F Pm 200 ejerce sobre la partícula (totalmente absorbente)


ax = = = debido a la presión de radiación solar. El área
m mc (150 )(3,00 × 108 ) pertinente es el área de sección transversal de la
= 4,44x10-9 m/s2 partícula, no su área total. Como parte de su
Luego x − x 0 = v0 x t + 12 a x t 2 da respuesta, explique por qué es así.
c) La densidad de masa de una partícula
2( x − x 0 ) 2(16,0 ) representativa de polvo interplanetario es de
t= = alrededor de 3000 kg/m3. Halle el radio de partícula R
ax 4,44 × 10 −9 con el que las fuerzas gravitatoria y de radiación que
= 8,49 x 104 s = 23,6 h actúan sobre la partícula son de igual magnitud. La
luminosidad del Sol es de 3,0 x1026 W. ¿Depende su
La fuerza de la radiación es muy pequeña. En el respuesta de la distancia entre la partícula y el Sol?
cálculo hemos no se ha tomado en cuenta cualquier ¿Por qué?
otra fuerza obre la persona. d) Explique por qué es poco probable hallar en el
b) Podrías lanzar la linterna en la dirección fuera de Sistema Solar partículas de polvo con un radio menor
la nave. Por la conservación de la cantidad de que el calculado en el inciso (c). [Sugerencia:
movimiento lineal te moverías hacia la nave con la Construya la relación de las dos expresiones de fuerza
misma magnitud de la cantidad de movimiento que halladas en los incisos (a) y (b)].
diste a linterna. Solución.
GM S m GM S 4πR 3 ρ
Ejemplo 25. El inventor Nikola Tesla, del siglo XIX, a) FG = =
propuso transmitir energía eléctrica por medio de r2 r2 3
ondas electromagnéticas sinusoidales. Suponga que 4πGM S R ρ 3
se pretende transmitir energía eléctrica en un haz con =
un área de sección transversal de 100 m2. ¿Qué 3r 2
amplitudes de campo eléctrico y magnético se b) Si se asume que la radiación del sol es
requieren para transmitir una cantidad de potencia interceptada por la sección transversal de la partícula,
equivalente a la que manejan las líneas de transmisión podemos escribir la fuerza sobre la partícula como:
modernas, las cuales tienen voltajes y corrientes del IA L πR 2 LR 2
orden de 500 kV y 1000A? F= = =
c 4πr 2 c 4cr 2
Intensidad I , corriente i .
Solución. c) Si la fuerza de la gravedad y la fuerza de la
P 1 presión de la radiación en una partícula del sol son
P = IA ⇒ I = = ε 0 cE 2
A 2 iguales, podemos resolver para el radio de la
partícula:
2P 2Vi
⇒ E= = ⇒ 4πGM S R 3 ρ LR 2
Aε 0 c Aε 0 c FG = F ⇒ =
3r 2 4cr 2
2Vi 2(5,00 × 10 5 )(1000) 3L
E= ⇒ R= ⇒
(100)ε 0 (3,00 × 108 )
=
Aε 0 c 16πGM S ρc
= 6,14 × 10 4 V m 3(3,9 × 1026 )
R= −11
16π (6,7 × 10 ) (2,0 × 1030 ) (3000 kg m3 ) (3,0 × 108 )

E 6,14 × 10 4 ⇒ R = 1.9 × 10 −7 m
B= = = 2,05 × 10 4 T d) Si la partícula tiene un radio más pequeño que lo
c 3,00 × 10 8
encontrado en la parte (c), entonces la presión de la
radiación supera la fuerza gravitacional y tiene como
Ejemplo 26. El espacio interplanetario contiene resultado una aceleración alejándose del sol, de esta
muchas partículas pequeñas conocidas como polvo manera quita tales partículas del Sistema Solar.
interplanetario. La presión de radiación proveniente
del Sol fija un límite inferior al tamaño de estas Ejemplo 27. La NASA está considerando seriamente
partículas de polvo. Para comprender el origen de el concepto de navegación solar a vela. Un velero
este límite, considere una partícula esférica de polvo solar utiliza una vela grande, de poca masa, y la
de radio R y densidad de masa p. energía y la cantidad de movimiento de la luz solar
a) Escriba una expresión de la fuerza gravitatoria que para impulsarse.
el Sol (masa M) ejerce sobre la partícula cuando ésta a) ¿Cómo debe ser la vela: absorbente o reflectante?
se halla a una distancia r del Sol. ¿Por qué?
b) Sea L la luminosidad del Sol, equivalente a la b) La producción total de potencia del Sol es de 3,9 x
rapidez con la que emite energía en forma de 1026W. ¿De qué tamaño debe ser una vela para
radiación electromagnética. Halle la fuerza que se impulsar un vehículo espacial de 10000kg contra la

13
Las Ecuaciones de Maxwell y ondas electromagnéticas Hugo Medina Guzmán

fuerza gravitatoria del Sol? Exprese su resultado en 2 LA


kilómetros cuadrados. Frad = Así: FG = Frad ⇒
c) Explique por qué su respuesta al inciso (b) es 4πr 2 c
independiente de la distancia respecto al Sol. GM S m 2 LA 4πGM S mc
= ⇒ A= ⇒
Solución.
r 2
4πr c2
2L
a) La transferencia de la cantidad de movimiento es
4π (6,7 × 10 −11 ) (2,0 × 10 30 ) (10000 ) (3,0 × 10 8 )
siempre mayor cuando se utilizan superficies A=
reflectoras (considere una bola que choca con una (2)3.9 × 10 26
pared, la pared ejerce una mayor fuerza si la bola ⇒ A = 648 × 10 6 m 2 = 6,48 km 2
rebota en lugar de pegarse). Tal que en navegación
solar es mejor utilizar una vela reflectora.
c) Esta respuesta es independiente de la distancia del
b) La ecuación para la repulsión viene de equilibrar
sol desde que la fuerza gravitacional y la presión de la
la fuerza gravitacional y la fuerza de la presión de la
radiación disminuyen con el cuadrado de la
radiación. Según lo visto en el problema anterior, lo
distancia, y la distancia se cancela del problema.
último es:

PREGUNTAS Y PROBLEMAS

1. a) Demostrar que la ecuación a) B0 ,


E y = E y 0 sen (kx − ωt ) puede escribirse en la b) la densidad de energía electromagnética total
ω media y
forma E y = E y 0 senk ( x − ct ) , siendo c = . ¿En c) la intensidad.
k Respuesta.
qué sentido se mueve esta onda? a) 13,3 mG, b) 7,08x10-7 J/m3, c) 212 W/m2.
b) Sustituir esta función en la ecuación
∂2Ey ∂2Ey 6. Un haz de láser tiene un diámetro de 1,0 mm y una
= μ 0ε 0 y demostrar que se satisface potencia media de 1,5mW. Hallar la intensidad del
∂x 2 ∂t 2 haz y E rms y Brms .
1
esta ecuación si c= . Respuesta.
μ 0ε 0 I = 1,91 kW/m2, E rms = 849 V/m, Brms = 28 mG
c) Utilizar los valores conocidos de μ0 y ε0 en
1 ∂B z ∂E y
unidades SI para demostrar que vale 7. a) Deducir la ecuación = −μ 0ε 0 . (b)
μ 0ε 0 ∂x ∂t
aproximadamente 3x10 m/s. 8 ∂E y ∂B
Eliminar E de las ecuaciones = − μ 0ε 0 z y
Respuesta. ∂x ∂t
a) La onda se está moviendo en la dirección x. ∂E y
∂B z
= −μ 0ε 0 para obtener la ecuación de
2. a) Demostrar que el valor medio cuadrático del ∂x ∂t
campo eléctrico en una onda a relacionado con la onda correspondiente a B z .
intensidad I por E rms = μ 0 cI
8. Una fuente o foco puntual emite radiación
E rms
b) Demostrar que Brms = . uniformemente en todas direcciones.
c a) Demostrar que si m es su potencia media, la
Pm
3. Una onda electromagnética tiene una intensidad I intensidad a una distancia r es I = .
4πr 2
=100 W/m2. Hallar E rms y Brms .
b) Si la intensidad de la luz solar que incide sobre las
Respuesta. capas superiores de la atmósfera terrestre es 1400
E rms = 194 V/m, Brms = 6,47 mG W/m2, hallar la emisión de potencia del Sol
→ → c) Hallar E rms y Brms debido al Sol en las capas
→ E× B superiores de la atmósfera terrestre.
4. Demostrar que el vector de Poynting S =
μ0 Respuesta.
tiene unidades de W/m2. b) 3,9x1026 W
c) E rms = 726 V/m, Brms = 24,2 mG
5. La amplitud de una onda electromagnética es E0
= 400 V/m. Hallar

14
Las Ecuaciones de Maxwell y ondas electromagnéticas Hugo Medina Guzmán

9. Suponer que un foco incandescente de 100 W es un →


foco puntual que radia uniformemente en todas b) Hallar el campo magnético B en el exterior y
direcciones. junto al conductor.
a) Hallar la intensidad del foco a 1,0 m. c) En r = a (superficie del conductor) utilizar los
b) Suponiendo que la onda es plana, hallar E rms y resultados de las partes (a) y (b) para calcular el
→ →
Brms a esta distancia. → E× B →
vector Poynting S = . ¿Qué sentido tiene S?
μ0
10. Por un conductor cilíndrico largo de radio a y →
resistividad ρ circula una corriente constante I d) Hallar el flujo de S a través de la superficie del
distribuida uniformemente en toda su sección recta. conductor de longitud L y área 2πaL y demostrar
a) Utilizar la ley de Ohm para relacionar el campo que la potencia que entra en el conductor es igual a

eléctrico E en el conductor con I 2 R , siendo R la resistencia.
I, ρ y a.

15

También podría gustarte